Sei sulla pagina 1di 1367

@iitjeehelps

THE #1 BEST SELLING

@iitjeehelps
@iitjeehelps
THE #1 BEST SELLING

ARIHANT PUBLICATIONS (INDIA) LIMITED

@iitjeehelps
THE #1 BEST SELLING

Arihant Publications (India) Ltd.


All Rights Reserved

© Publisher
No part of this publication may be re-produced, stored in a retrieval system or distributed
in any form or by any means, electronic, mechanical, photocopying, recording, scanning,
web or otherwise without the written permission of the publisher. Arihant has obtained
all the information in this book from the sources believed to be reliable and true. However,
Arihant or its editors or authors or illustrators don’t take any responsibility for the absolute
accuracy of any information published and the damages or loss suffered there upon.

All disputes subject to Meerut (UP) jurisdiction only.

Administrative & Production Offices


Regd. Office
‘Ramchhaya’ 4577/15, Agarwal Road, Darya Ganj, New Delhi -110002
Tele: 011- 47630600, 43518550; Fax: 011- 23280316
Head Office
Kalindi, TP Nagar, Meerut (UP) - 250002
Tele: 0121-2401479, 2512970, 4004199; Fax: 0121-2401648

Sales & Support Offices


Agra, Ahmedabad, Bengaluru, Bareilly, Chennai, Delhi, Guwahati,
Hyderabad, Jaipur, Jhansi, Kolkata, Lucknow, Meerut, Nagpur & Pune

ISBN : 978-93-13196-77-8

Published by Arihant Publications (India) Ltd.


For further information about the books published by Arihant
log on to www.arihantbooks.com or email to info@arihantbooks.com
/arihantpub /@arihantpub Arihant Publications /arihantpub

@iitjeehelps
THE #1 BEST SELLING

CONTENTS

PHYSICS
1. Units, Measurements and Dimensions 3-14
2. Scalars and Vectors 15-23
3. Motion in 1, 2 & 3 Dimensions and Projectile Motion 24-44
4. Newton's Laws of Motion and Friction 45-62
5. Circular Motion 63-72
6. Work, Energy and Power 73-83
7. Centre of Mass, Momentum and Collision 84-94
8. Rotational Motion of Rigid Body 95-106
9. Gravitation 107-118
10. Simple Harmonic Motion 119-131
11. Fluid Mechanics 132-146
12. Elasticity 147-155
13. Waves Motion 156-164
14. Sound Wave 165-174
15. Heat, Temperature and Calorimetry 175-184
16. Physics for Gaseous State 185-194
17. Laws of Thermodynamics 195-205
18. Transmission 206-214
19. Ray Optics 215-233
20. Waves Optics 234-242
21. Electric Charge 243-251
22. Gauss's Law and Electric Potential Theory 252-262
23. Electric Capacitor 263-270
24. Current Electricity 271-288
25. Magnetic Field 289-300

@iitjeehelps
THE #1 BEST SELLING

26. Magnetostatics 301-310


27. Electromagnetic Induction (EMI) 311-318
28. Alternating Current and EM Wave 319-330
29. Cathode Rays, Photoelectric Effect of Light and X-Rays 331-340
30. Atomic Structure 341-348
31. Nucleus 349-357
32. Semiconductor Devices and Logic Gates 358-375
33. Universe 376-380

CHEMISTRY
1. Some Basic Concepts of Chemistry 383-394
2. Atomic Structure 395-408
3. Nuclear Chemistry 409-416
4. Chemical Bonding 417-431
5. Periodic Properties 432-441
6. States of Matter 442-454
7. Chemical Thermodynamics 455-466
8. Chemical and lonic Equilibria 467-480
9. Chemical Kinetics 481-491
10. Solution 492-501
11. Adsorption and Colloidal System 502-510
12. Redox Reactions 511-517
13. Electro-chemistry 518-529
14. Hydrogen 530-537
15. s-Block Elements 538-549
16. Metallurgy 550-558
17. p-Block Elements - I (Group 13 & 14) 559-567
18. p-Block Elements - II 568-586
19. d - and f-Block Elements 587-597

@iitjeehelps
THE #1 BEST SELLING

20. Coordination Compounds and Organometallics 598-607


21. General Organic Chemistry 608-625
22. Purification and Estimation of Organic Compounds 626-631
23. Hydrocarbons 632-650
24. Halogen Derivatives of Hydrocarbons 651-663
25. Alcohol, Phenol and Ether 664-678
26. Aldehyde and Ketones 679-695
27. Carboxylic Acid and Its Derivatives 696-706
28. Nitrogen Containing Compounds 707-720
29. Polymers, Biomolecules and Chemistry in Action 721-738
30. Qualitative Analysis 739-743
31. Stereochemistry 744-756

MATHEMATICS
1. Complex Number 759-769
2. Quadratic Equation 770-779
3. Sequences and Series 780-792
4. Exponential and Logarithmic Series 793-800
5. Permutations and Combinations 801-811
6. Binomial Theorem and Mathematical Induction 812-823
7. Matrices 824-832
8. Determinant 833-846
9. Sets, Relations and Functions 847-860
10. Linear Inequality 861-866
11. Trigonometry 867-897
12. Rectangular Coordinates and Straight Line 898-913
13. The Circle 914-928
14. Conic Sections 929-952
15. Three Dimensional Geometry 953-967

@iitjeehelps
THE #1 BEST SELLING

16. Limits, Continuity and Differentiability 968-986


17. Differential Coefficients 987-998
18. Application of Derivatives 999-1013
19. Indefinite Integral 1014-1024
20. Definite Integral and Its Applications 1025-1039
21. Differential Equations 1040-1053
22. Probability 1054-1069
23. Vector Algebra 1070-1085
24. Statistics 1086-1096
25. Linear Programming 1097-1102

Ÿ English Proficiency 1105-1143


Ÿ Logical Reasoning 1147-1191
Ÿ Practice Sets (1-5) 1195-1251
Ÿ Solved Paper 2017 1-29
Ÿ Solved Paper 2018 1-34
Ÿ Solved Paper 2019 1-31

@iitjeehelps
THE #1 BEST SELLING

SYLLABUS
PART I PHYSICS
1. Units & Measurement 6. Rotational Motion
1.1 Units (Different systems of units, SI 6.1 Description of rotation (angular
units, fundamental and derived units) displacement, angular velocity and
1.2 Dimensional Analysis angular acceleration)
1.3 Precision and significant figures 6.2 Rotational motion with constant
1.4 Fundamental measurements in Physics angular acceleration
(Vernier calipers, screw gauge, 6.3 Moment of inertia, Parallel and
Physical balance etc.) perpendicular axes theorems,
rotational kinetic energy
2. Kinematics 6.4 Torque and angular momentum
2.1 Properties of vectors 6.5 Conservation of angular momentum
2.2 Position, velocity and acceleration 6.6 Rolling motion
vectors
2.3 Motion with constant acceleration 7. Gravitation
2.4 Projectile motion 7.1 Newton’s law of gravitation
2.5 Uniform circular motion 7.2 Gravitational potential energy, Escape
2.6 Relative motion velocity
7.3 Motion of planets – Kepler’s laws,
3. Newton’s Laws of Motion satellite motion
3.1 Newton’s laws (free body diagram,
resolution
8. Mechanics of Solids and Fluids
of forces) 8.1 Elasticity
3.2 Motion on an inclined plane 8.2 Pressure, density and Archimedes’
3.3 Motion of blocks with pulley systems principle
3.4 Circular motion – centripetal force 8.3 Viscosity and Surface Tension
3.5 Inertial and non-inertial frames 8.4 Bernoulli’s theorem

4. Impulse and Momentum 9. Oscillations


4.1 Definition of impulse and momentum 9.1 Kinematics of simple harmonic motion
4.2 Conservation of momentum 9.2 Spring mass system, simple and
compound pendulum
4.3 Collisions
9.3 Forced & damped oscillations,
4.4 Momentum of a system of particles resonance
4.5 Center of mass
10. Waves
5. Work and Energy 10.1 Progressive sinusoidal waves
5.1 Work done by a force 10.2 Standing waves in strings and pipes
5.2 Kinetic energy and work-energy 10.3 Superposition of waves, beats
theorem
10.4 Doppler Effect
5.3 Power
5.4 Conservative forces and potential 11. Heat and Thermodynamics
energy 11.1 Kinetic theory of gases
5.5 Conservation of mechanical energy 11.2 Thermal equilibrium and temperature

@iitjeehelps
THE #1 BEST SELLING

11.3 Specific heat, Heat Transfer - 15.4 Alternating current (peak and rms
Conduction, convection and radiation, value)
thermal conductivity, Newton’s law of 15.5 AC circuits, LCR circuits
cooling
11.4 Work, heat and first law of 16. Optics
thermodynamics 16.1 Laws of reflection and refraction
11.5 2nd law of thermodynamics, Carnot 16.2 Lenses and mirrors
engine Efficiency and Coefficient of 16.3 Optical instruments – telescope and
performance. microscope
16.4 Interference – Huygen’s principle,
12. Electrostatics Young’s double slit experiment
12.1 Coulomb’s law
16.5 Interference in thin films
12.2 Electric field (discrete and continuous 16.6 Diffraction due to a single slit
charge distributions)
16.7 Electromagnetic waves and their
12.3 Electrostatic potential and characteristics (only qualitative ideas),
Electrostatic potential energy
Electromagnetic spectrum
12.4 Gauss’ law and its applications 16.8 Polarization – states of polarization,
12.5 Electric dipole Malus’ law, Brewster’s law
12.6 Capacitance and dielectrics (parallel
plate capacitor, capacitors in series 17. Modern Physics
and parallel) 17.1 Dual nature of light and matter –
Photoelectric effect, De-Broglie
13. Current Electricity wavelength
13.1 Ohm’s law, Joule heating 17.2 Atomic models – Rutherford’s
13.2 D.C circuits – Resistors and cells in experiment, Bohr’s atomic model
series and parallel, Kirchoff’s laws, 17.3 Hydrogen atom spectrum
potentiometer and Wheatstone bridge,
17.4 Radioactivity
13.3 Electrical Resistance (Resistivity,
origin and temperature dependence of 17.5 Nuclear reactions Fission and fusion,
resistivity). binding energy

14. Magnetic Effect of Current 18. Electronic Devices


14.1 Biot-Savart’s law and its applications 18.1 Energy bands in solids (qualitative
14.2 Ampere’s law and its applications ideas only), conductors, insulators and
semiconductors;
14.3 Lorentz force, force on current
carrying conductors in a magnetic 18.2 Semiconductor diode – I-V
field characteristics in forward and reverse
14.4 Magnetic moment of a current loop, bias, diode as a rectifier;
torque on a current loop, I-V characteristics of LED, photodiode,
Galvanometer and its conversion to solar cell, and Zener diode; Zener
voltmeter and ammeter diode as a voltage regulator.
18.3 Junction transistor, transistor action,
15. Electromagnetic Induction characteristics of a transistor;
15.1 Faraday’s law, Lenz’s law, eddy transistor as an amplifier (common
currents emitter configuration) and oscillator
15.2 Self and mutual inductance 18.4 Logic gates (OR, AND, NOT, NAND
15.3 Transformers and generators and NOR). Transistor as a switch.

@iitjeehelps
THE #1 BEST SELLING

PART II CHEMISTRY
1. States of Matter 2.2 Quantum Mechanics Wave-particle
1.1 Measurement Physical quantities and SI duality de-Broglie relation, Uncertainty
units, Dimensional analysis, Precision, principle; Hydrogen atom: Quantum
Significant figures. numbers and wavefunctions, atomic
orbitals and their shapes (s, p, and d),
1.2 Chemical Reactions Laws of chemical Spin quantum number.
combination, Dalton’s atomic theory;
2.3 Many Electron Atoms Pauli exclusion
Mole concept; Atomic, molecular and
principle; Aufbau principle and the
molar masses; Percentage composition
electronic configuration of atoms, Hund’s
empirical & molecular formula; Balanced
rule.
chemical equations & stoichiometry
2.4 Periodicity Brief history of the
1.3 Three states of matter, intermolecular
development of periodic tables Periodic
interactions, types of bonding, melting
law and the modern periodic table; Types
and boiling points Gaseous state: Gas
of elements: s, p, d, and f blocks; Periodic
Laws, ideal behavior, ideal gas equation,
trends: ionization energy, atomic, and
empirical derivation of gas equation,
ionic radii, inter gas radii, electron
Avogadro number, Kinetic theory –
affinity, electro negativity and valency.
Maxwell distribution of velocities,
Nomenclature of elements with atomic
Average, root mean square and most
number greater than 100.
probable velocities and relation to
temperature, Diffusion; Deviation from 3. Chemical Bonding &
ideal behaviour – Critical temperature, Molecular Structure
Liquefaction of gases, van der Waals’
3.1 Valence Electrons, Ionic Bond Lattice
equation.
Energy and Born-Haber cycle; Covalent
1.4 Liquid State Vapour pressure, surface character of ionic bonds and polar
tension, viscosity. character of covalent bond, bond
1.5 Solid State Classification; Space lattices parameters
& crystal systems; Unit cell in two 3.2 Molecular Structure Lewis picture &
dimensional and three dimensional resonance structures, VSEPR model &
lattices, calculation of density of unit cell molecular shapes
– Cubic & hexagonal systems; Close 3.3 Covalent Bond Valence Bond Theory-
packing; Orbital overlap, Directionality of bonds &
hybridization (s, p & d orbitals only),
Crystal Structures Simple AB and AB2
Resonance; Molecular orbital theory-
type ionic crystals, covalent crystals –
Methodology, Orbital energy level
diamond & graphite, metals. Voids,
diagram, Bond order, Magnetic properties
number of atoms per unit cell in a cubic
for homonuclear diatomic species
unit cell, Imperfections- Point defects,
(qualitative idea only).
non-stoichiometric crystals; Electrical,
magnetic and dielectric properties; 3.4 Metallic Bond Qualitative description.
Amorphous solids qualitative 3.5 Intermolecular Forces Polarity; Dipole
description. Band theory of metals, moments; Hydrogen Bond.
conductors, semiconductors and
insulators, and n- and p- type 4. Thermodynamics
semiconductors. 4.1 Basic Concepts Systems and
surroundings; State functions; Intensive
2. Atomic Structure & Extensive Properties; Zeroth Law and
2.1 Introduction Radioactivity, Subatomic Temperature
particles; Atomic number, isotopes and 4.2 First Law of Thermodynamics Work,
isobars, Thompson’s model and its internal energy, heat, enthalpy, heat
limitations, Rutherford’s picture of atom capacities andspecific heats,
and its limitations; Hydrogen atom measurements of ΔU and ΔH, Enthalpies
spectrum and Bohr model and its of formation, phase transformation,
limitations. ionization, electron gain;

@iitjeehelps
THE #1 BEST SELLING

Thermochemistry; Hess’s Law, cell reactions; Standard electrode


Enthalpy of bond dissociation, potentials; EMF of Galvanic cells;
combustion, atomization, sublimation, Nernst equation; Factors affecting the
solution and dilution electrode potential; Gibbs energy
4.3 Second Law Spontaneous and change and cell potential; Secondary
reversible processes; entropy; Gibbs cells; dry cells, Fuel cells; Corrosion
free energy related to spontaneity and and its prevention.
non-spontaneity, non-mechanical work; 6.2 Electrolytic Conduction Electrolytic
Standard free energies of formation, Conductance; Specific and molar
free energy change and chemical conductivities; variations of
equilibrium conductivity with concentration ,
4.4 Third Law Introduction Kolhrausch’s Law and its application,
Electrolysis, Faraday’s laws of
5. Physical and Chemical Equilibria electrolysis; Coulometer; Electrode
5.1 Concentration Units Mole Fraction, potential and electrolysis, Commercial
Molarity, and Molality production of the chemicals, NaOH,
Na, Al, Cl2 & F2.
5.2 Solutions Solubility of solids and gases
in liquids, Vapour Pressure, Raoult’s 7. Chemical Kinetics
law, Relative lowering of vapour 7.1 Aspects of Kinetics Rate and Rate
pressure, depression in freezing point; expression of a reaction; Rate constant;
elevation in boiling point; osmotic Order and molecularity of the reaction;
pressure, determination of molecular Integrated rate expressions and half life
mass; solid solutions, abnormal for zero and first order reactions.
molecular mass, van’t Hoff factor.
Equilibrium: Dynamic nature of 7.2 Factor Affecting the Rate of the
equilibrium, law of mass action Reactions Concentration of the
reactants, catalyst; size of particles,
5.3 Physical Equilibrium Equilibria Temperature dependence of rate
involving physical changes (solid- constant concept of collision theory
liquid, liquid-gas, solid-gas), Surface (elementary idea, no mathematical
chemistry, Adsorption, Physical and treatment); Activation energy;
Chemical adsorption, Langmuir Catalysis, Surface catalysis, enzymes,
Isotherm, Colloids and emulsion, zeolites; Factors affecting rate of
classification, preparation, uses. collisions between molecules.
5.4 Chemical Equilibria Equilibrium 7.3 Mechanism of Reaction Elementary
constants (KP, KC), Factors affecting reactions; Complex reactions;
equilibrium, Le- Chatelier’s principle. Reactions involving two/three steps
5.5 Ionic Equilibria Strong and Weak only.
electrolytes, Acids and Bases 7.4 Surface Chemistry Adsorption
(Arrhenius, Lewis, Lowry and Bronsted) Physisorption and chemisorption;
and their dissociation; degree of factors affecting adsorption of gasses
ionization, Ionization of Water; on solids; catalysis: homogeneous and
ionization of polybasic acids, pH; Buffer heterogeneous, activity and selectivity:
solutions; Henderson equation, Acid- enzyme catalysis, colloidal state:
base titrations; Hydrolysis; Solubility distinction between true solutions,
Product of Sparingly Soluble Salts; colloids and suspensions; lyophillic,
Common Ion Effect. lyophobic multi molecular and
5.6 Factors Affecting Equilibria macromolecular colloids; properties of
Concentration, Temperature, Pressure, colloids; Tyndall effect, Brownian
Catalysts, Significance of DG and DG0 movement, electrophoresis,
in Chemical Equilibria. coagulations; emulsions – types of
emulsions.
6. Electrochemistry
6.1 Redox Reactions Oxidation-reduction 8. Hydrogen and s-block Elements
reactions (electron transfer concept); 8.1 Hydrogen Element Unique position in
Oxidation number; Balancing of redox periodic table, occurrence, isotopes;
reactions; Electrochemical cells and Dihydrogen: preparation, properties,

@iitjeehelps
THE #1 BEST SELLING

reactions, and uses; Molecular, saline, nitrogen fixation; Compound of nitrogen;


ionic, covalent, interstitial hydrides; Ammonia: Haber’s process, properties
Water: Properties; Structure and and reactions; Oxides of nitrogen and
aggregation of water molecules; Heavy their structures; Properties and Ostwald’s
water; Hydrogen peroxide: preparation, process of nitric acid production;
reaction, structure & use, Hydrogen as a Fertilizers – NPK type; Production of
fuel. phosphorus; Allotropes of phosphorus;
8.2 s-block Elements Abundance and Preparation, structure and properties of
occurrence; Anomalous properties of the hydrides, oxides, oxoacids (elementary
first elements in each group; diagonal idea only) and halides of phosphorus,
relationships; trends in the variation of phosphine.
properties (ionization energy, atomic & 9.5 Group 16 Elements Isolation and
ionic radii). chemical reactivity of dioxygen; Acidic,
8.3 Alkali Metals Lithium, sodium and basic and amphoteric oxides;
potassium: occurrence, extraction, Preparation, structure and properties of
reactivity, and electrode potentials; ozone; Allotropes of sulphur;
Biological importance; Reactions with Preparation/production properties and
oxygen, hydrogen, halogens water and uses of sulphur dioxide and sulphuric
liquid ammonia; Basic nature of oxides acid; Structure and properties of oxides,
and hydroxides; Halides; Properties and oxoacids (structures only), hydrides and
uses of compounds such as NaCl, halides of sulphur.
Na2CO3, NaHCO3, NaOH, KCl 9.6 Group 17 and group 18 Elements
and KOH. Structure and properties of hydrides,
8.4 Alkaline Earth Metals Magnesium and oxides, oxoacids of halogens (structures
calcium: Occurrence, extraction, only); preparation, properties & uses of
reactivity and electrode potentials; chlorine & HCl; Inter halogen
Reactions with O2, H2O, H2 and halogens; compounds; Bleaching Powder; Uses of
Solubility and thermal stability of oxo Group 18 elements, Preparation,
salts; Biological importance of Ca and structure and reactions of xenon
Mg; Preparation, properties and uses of fluorides, oxides, and oxoacids.
important compounds such as CaO,
Ca(OH)2, plaster of Paris, MgSO4, MgCl2, 9.7 d - Block Elements General trends in the
CaCO3, and CaSO4; Lime and limestone, chemistry of first row transition elements;
cement. Metallic character; Oxidation state;
ionization enthalpy; Ionic radii; Color;
9. p - , d - and f - block Elements Catalytic properties; Magnetic properties;
9.1 General Abundance, distribution, Interstitial compounds; Occurrence and
physical and chemical properties, extraction of iron, copper, silver, zinc, and
isolation and uses of elements; Trends in mercury; Alloy formation; Steel and some
chemical reactivity of elements of a important alloys; preparation and
group; electronic configuration, oxidation properties of CuSO4, K2Cr2O7, KMnO4,
states; anomalous properties of first Mercury halides; Silver nitrate and silver
element of each group. halides; Photography.
9.2 Group 13 Elements Boron, Properties 9.8 f - Block Elements Lanthanoids and
and uses of borax, boric acid, boron actinoids;Oxidation states and chemical
hydrides & halides. Reaction of reactivity of lanthanoids compounds;
aluminium with acids and alkalis; Lanthanide contraction and its
consequences, Comparison of actinoids
9.3 Group 14 Elements Carbon, carbon
and lanthanoids.
catenation, physical & chemical
properties, uses, allotropes (graphite, 9.9 Coordination Compounds Coordination
diamond, fullerenes), oxides, halides and number; Ligands; Werner’s coordination
sulphides, carbides; Silicon: Silica, theory; IUPAC nomenclature; Application
silicates, silicone, silicon tetrachloride, and importance of coordination
Zeolites, and their uses compounds (in qualitative analysis,
9.4 Group 15 Elements Dinitrogen; extraction of metals and biological
Preparation, reactivity and uses of systems e.g. chlorophyll, vitamin B12, and
nitrogen; Industrial and biological hemoglobin); Bonding: Valence-bond
approach, Crystal field theory

@iitjeehelps
THE #1 BEST SELLING

(qualitative); Stability constants; 11.2 Conformations Ethane conformations;


Shapes, color and magnetic properties; Newman and Sawhorse projections.
Isomerism including stereoisomerisms; 11.3 Geometrical isomerism in alkenes
Organometallic compounds.
12. Organic Compounds with
10. Principles of Organic Chemistry Functional Groups Containing
and Hydrocarbons Oxygen and Nitrogen
10.1 Classification General Introduction,
12.1 General Nomenclature, electronic
classification based on functional
structure, important methods of
groups, trivial and IUPAC
preparation, identification, important
nomenclature. Methods of purification:
reactions, physical and chemical
qualitative and quantitative.
properties, uses of alcohols, phenols,
10.2 Electronic Displacement in a ethers, aldehydes, ketones, carboxylic
Covalent Bond Inductive, resonance acids, nitro compounds, amines,
effects, and hyperconjugation; free diazonium salts, cyanides and
radicals; carbocations, carbanions, isocyanides.
nucleophiles and electrophiles; types
12.2 Specific Reactivity of a-hydrogen in
of organic reactions, free radial
carbonyl compounds, effect of
halogenations.
substituents on alphacarbon on acid
10.3 Alkanes and Cycloalkanes Structural strength, comparative reactivity of acid
isomerism, general properties and derivatives, mechanism of nucleophilic
chemical reactions, free redical addition and dehydration, basic
helogenation, combustion and character of amines, methods of
pyrolysis. preparation, and their separation,
10.4 Alkenes and Alkynes General importance of diazonium salts in
methods of preparation and reactions, synthetic organic chemistry.
physical properties, electrophilic and
free radical additions, acidic character 13. Biological, Industrial and
of alkynes and (1,2 and 1,4) addition Environmental Chemistry
to dienes. 13.1 The Cell Concept of cell and energy
10.5 Aromatic Hydrocarbons Sources; cycle.
properties; isomerism; resonance 13.2 Carbohydrates Classification;
delocalization; aromaticity; Monosaccharides; Structures of
polynuclear hydrocarbons; IUPAC pentoses and hexoses; Anomeric
nomenclature; mechanism of carbon; Mutarotation; Simple chemical
electrophilic substitution reaction, reactions of glucose, Disaccharides:
directive influence and effect of reducing and nonreducing sugars –
substituents on reactivity; sucrose, maltose and lactose;
carcinogenicity and toxicity. Polysaccharides: elementary idea of
10.6 Haloalkanes and Haloarenes structures of starch, cellulose and
Physical properties, nomenclature, glycogen.
optical rotation, chemical reactions 13.3 Proteins Amino acids; Peptide bond;
and mechanism of substitution Polypeptides; Primary structure of
reaction. Uses and environmental proteins; Simple idea of secondary ,
effects; di, tri, tetrachloromethanes, tertiary and quarternary structures of
iodoform, freon and DDT. proteins; Denaturation of proteins and
enzymes.
10.7 Petroleum Composition and refining,
uses of petrochemicals. 13.4 Nucleic Acids Types of nucleic acids;
Primary building blocks of nucleic
11. Stereochemistry acids (chemical composition of DNA &
11.1 Introduction Chiral molecules; optical RNA); Primary structure of DNA and
activity; polarimetry; R,S and D,L its double helix; Replication;
configurations; Fischer projections; Transcription and protein synthesis;
enantiomerism; racemates; Genetic code.
diastereomerism and meso structures. 13.5 Vitamins Classification, structure,
functions in biosystems; Hormones

@iitjeehelps
THE #1 BEST SELLING

13.6 Polymers Classification of polymers; bases; pH measurements of some


General methods of polymerization; solutions obtained from fruit juices,
Molecular mass of polymers; solutions of known and varied
Biopolymers and biodegradable concentrations of acids, bases and
polymers; methods of polymerization salts using pH paper or universal
(free radical, cationic and anionic indicator; Lyophilic and lyophobic sols;
addition polymerizations); Dialysis; Role of emulsifying agents in
Copolymerization: Natural rubber; emulsification.
Vulcanization of rubber; Synthetic Equilibrium studies involving ferric
rubbers. Condensation polymers. and thiocyanate ions (ii) [Co(H2O)6]2+
13.7 Pollution Environmental pollutants; and chloride ions; Enthalpy
soil, water and air pollution; Chemical determination for strong acid vs.
reactions in atmosphere; Smog; Major strong base neutralization reaction
atmospheric pollutants; Acid rain; (ii) hydrogen bonding interaction
Ozone and its reactions; Depletion of between acetone and chloroform;
ozone layer and its effects; Industrial Rates of the reaction between (i)
air pollution; Green house effect and sodium thiosulphate and hydrochloric
global warming; Green Chemistry, acid, (ii) potassium iodate and sodium
study for control of environmental sulphite (iii) iodide vs. hydrogen
pollution. peroxide, concentration and
13.8 Chemicals in medicine, health-care temperature effects in these reactions.
and food: Analgesics, Tranquilizers, 14.4 Purification Methods Filtration,
antiseptics, disinfectants, anti- crystallization, sublimation,
microbials, anti-fertility drugs, distillation, differential extraction, and
antihistamines, antibiotics, antacids; chromatography. Principles of melting
Preservatives, artificial sweetening point and boiling point determination;
agents, antioxidants, soaps and principles of paper chromatographic
detergents. separation – Rf values.
14.5 Qualitative Analysis of Organic
14. Theoretical Principles of Compounds Detection of nitrogen,
Experimental Chemistry sulphur, phosphorous and halogens;
14.1 Volumetric Analysis Principles; Detection of carbohydrates, fats and
Standard solutions of sodium proteins in foodstuff; Detection of
carbonate and oxalic acid; Acidbase alcoholic, phenolic, aldehydic, ketonic,
titrations; Redox reactions involving carboxylic, amino groups and
KI, H2SO4, Na2SO3, Na2S2O3 and H2S; unsaturation.
Potassium permanganate in acidic,
14.6 Quantitative Analysis of Organic
basic and neutral media; Titrations of
Compounds Basic principles for the
oxalic acid, ferrous ammonium
quantitative estimation of carbon,
sulphate with KMnO4, K2
hydrogen, nitrogen, halogen, sulphur
Cr2O7/Na2S2O3, Cu(II)/Na2S2O3.
and phosphorous; Molecular mass
14.2 Qualitative Analysis of Inorganic determination by silver salt and
Salts Principles in the determination of chloroplatinate salt methods;
the cations Pb2+, Cu2+, As3+, Mn2+, Al3+, Calculations of empirical and
Zn2+, Co2+, Ca2+, Sr2+, Ba2+, Mg2+, molecular formulae.
NH4 +, Fe3+, Ni2+ and the anions CO32–,
14.7 Principles of Organic Chemistry
S2–, SO42–, SO32–, NO2–, NO3–, Cl–, Br–, I–,
Experiments Preparation of iodoform,
PO43–, CH3COO–, C2O42–.
acetanilide, p-nitro acetanilide, di-
14.3 Physical Chemistry Experiments benzayl acetone, aniline yellow,
Preparation and crystallization of b-naphthol; Preparation of acetylene
alum, copper sulphate. Benzoic acid and study of its acidic character.
ferrous sulphate, double salt of alum 14.8 Basic Laboratory Technique Cutting
and ferrous sulphate, potassium ferric glass tube and glass rod, bending a
sulphate; Temperature vs. solubility; glass tube, drawing out a glass jet,
Study of pH charges by common ion boring of cork.
effect in case of weak acids and weak

@iitjeehelps
THE #1 BEST SELLING

PART III
a. English Proficiency, b. Logical Reasoning
a. English Proficiency 5. Verbal Reasoning
This test is designed to assess the test takers’ 5.1 Analogy Analogy means
general proficiency in the use of English correspondence.
language as a means of self-expression in real In the questions based on
life situations and specifically to test the test analogy, a particular relationship
takers’ knowledge of basic grammar, their is given and another similar
vocabulary, their ability to read fast and relationship has to be identified
comprehend, and also their ability to apply the from the alternatives provided.
elements of effective writing.
5.2 Classification Classification
1. Grammar means to assort the items of a
1.1 Agreement, Time and Tense, Parallel given group on the basis of
construction, Relative pronouns certain common quality they
1.2 Determiners, Prepositions, Modals, possess and then spot the odd
Adjectives option out.
1.3 Voice, Transformation 5.3 Series Completion Here series
1.4 Question tags, Phrasal verbs of numbers or letters are given
2. Vocabulary and one is asked to either
complete the series or find out
2.1 Synonyms, Antonyms, Odd Word, One
the wrong part in the series.
Word, Jumbled letters, Homophones,
Spelling 5.4 Logical Deduction –
2.2 Contextual meaning. Reading Passage Here a brief
2.3 Analogy passage is given and based on
the passage the candidate is
3. Reading Comprehension required to identify the correct or
3.1 Content/ideas incorrect logical conclusions.
3.2 Vocabulary
5.5 Chart Logic Here a chart or a
3.3 Referents table is given that is partially
3.4 Idioms/Phrases filled in and asks to complete it in
3.5 Reconstruction (rewording) accordance with the information
given either in the chart / table or
4. Composition in the question.
4.1 Rearrangement
4.2 Paragraph Unity 6. Non-verbal Reasoning
4.3 Linkers/Connectives 6.1 Pattern Perception Here a
certain pattern is given and
generally a quarter is left blank.
b. Logical Reasoning The candidate is required to
The test is given to the candidates to judge their identify the correct quarter from
power of reasoning spread in verbal and the given four alternatives.
nonverbal areas. The candidates should be able
to think logically so that they perceive the data 6.2 Figure Formation and
accurately, understand the relationships Analysis The candidate is
correctly, figure out the missing numbers or required to analyze and form a
words, and to apply rules to new and different figure from various given parts.
contexts. These indicators are measured 6.3 Paper Cutting It involves the
through performance on such tasks as detecting analysis of a pattern that is
missing links, following directions, classifying formed when a folded piece of
words, establishing sequences, and completing paper is cut into a definite
analogies. design.

@iitjeehelps
THE #1 BEST SELLING

6.4 Figure Matrix In this more than one 6.5 Rule Detection Here a particular
set of figures is given in the form of a rule is given and it is required to select
matrix, all of them following the same from the given sets of figures, a set of
rule. The candidate is required to figures, which obeys the rule and
follow the rule and identify the forms the correct series.
missing figure.

PART IV MATHEMATICS
1. Algebra mappings, binary operation, inverse of
function, functions of real variables
1.1 Complex numbers, addition,
like polynomial, modulus, signum and
multiplication, conjugation, polar
greatest integer.
representation, properties of modulus
and principal argument, triangle 1.10 Mathematical Induction
inequality, roots of complex numbers, 1.11 Linear Inequalities, solution of linear
geometric interpretations; inequalities in one and two variables.
Fundamental theorem of algebra.
1.2 Theory of Quadratic equations,
2. Trigonometry
quadratic equations in real and 2.1 Measurement of angles in radians and
complex number system and their degrees, positive and negative angles,
solutions, relation between roots and trigonometric ratios, functions and
coefficients, nature of roots, equations identities.
reducible to quadratic equations. 2.2 Solution of trigonometric equations.
1.3 Arithmetic, geometric and harmonic 2.3 Properties of triangles and solutions of
progressions, arithmetic, geometric triangles
and harmonic means, arithmetico- 2.4 Inverse trigonometric functions
geometric series, sums of finite
2.5 Heights and distances
arithmetic and geometric progressions,
infinite geometric series, sums of 3. Two-dimensional
squares and cubes of the first n natural
numbers.
Coordinate Geometry
3.1 Cartesian coordinates, distance
1.4 Logarithms and their properties. between two points, section formulae,
1.5 Exponential series. shift of origin.
1.6 Permutations and combinations, 3.2 Straight lines and pair of straight lines:
Permutations as an arrangement and Equation of straight lines in various
combination as selection, simple forms, angle between two lines,
applications. distance of a point from a line, lines
1.7 Binomial theorem for a positive through the point of intersection of two
integral index, properties of binomial given lines, equation of the bisector of
coefficients, Pascal’s triangle the angle between two lines,
concurrent lines.
1.8 Matrices and determinants of order
two or three, properties and evaluation 3.3 Circles and family of circles : Equation
of determinants, addition and of circle in various form, equation of
multiplication of matrices, adjoint and tangent, normal & chords, parametric
inverse of matrices, Solutions of equations of a circle , intersection of a
simultaneous linear equations in two circle with a straight line or a circle,
or three variables, elementary row and equation of circle through point of
column operations of matrices, intersection of two circles, conditions
for two intersecting circles to be
1.9 Sets, Relations and Functions, algebra
orthogonal.
of sets applications, equivalence
relations, mappings, one-one, into and 3.4 Conic sections : parabola, ellipse and
onto mappings, composition of hyperbola their eccentricity, directrices

@iitjeehelps
THE #1 BEST SELLING

& foci, parametric forms, equations of 6.4 Application of definite integrals to the
tangent & normal, conditions for y = determination of areas of regions
mx + c to be a tangent and point of bounded by simple curves.
tangency.
7. Ordinary Differential Equations
4. Three Dimensional 7.1 Order and degree of a differential
Coordinate Geometry equation, formulation of a differential
4.1 Co-ordinate axes and co-ordinate equation whole general solution is
planes, distance between two points, given, variables separable method.
section formula, direction cosines and 7.2 Solution of homogeneous differential
direction ratios, equation of a straight equations of first order and first degree
line in space and skew lines. 7.3 Linear first order differential equations
4.2 Angle between two lines whose
direction ratios are given, shortest 8. Probability
distance between two lines. 8.1 Various terminology in probability,
4.3 Equation of a plane, distance of a point axiomatic and other approaches of
from a plane, condition for coplanarity probability, addition and multiplication
of three lines, angles between two rules of probability.
planes, angle between a line and a 8.2 Conditional probability, total
plane. probability and Baye’s theorem
8.3 Independent events
5. Differential Calculus
8.4 Discrete random variables and
5.1 Domain and range of a real valued distributions with mean and variance.
function, Limits and Continuity of the
sum, difference, product and quotient 9. Vectors
of two functions, Differentiability.
9.1 Direction ratio/cosines of vectors,
5.2 Derivative of different types of addition of vectors, scalar
functions (polynomial, rational, multiplication, position vector of a
trigonometric, inverse trigonometric, point dividing a line segment in a
exponential, logarithmic, implicit given ratio.
functions), derivative of the sum,
difference, product and quotient of two 9.2 Dot and cross products of two vectors,
functions, chain rule. projection of a vector on a line.
9.3 Scalar triple products and their
5.3 Geometric interpretation of derivative,
Tangents and Normals. geometrical interpretations.
5.4 Increasing and decreasing functions, 10. Statistics
Maxima and minima of a function. 10.1 Measures of dispersion
5.5 Rolle’s Theorem, Mean Value Theorem 10.2 Measures of skewness and Central
and Intermediate Value Theorem. Tendency, Analysis of frequency
distributions with equal means but
6. Integral Calculus different variances
6.1 Integration as the inverse process of
differentiation, indefinite integrals of 11. Linear Programming
standard functions. 11.1 Various terminology and formulation of
6.2 Methods of integration: Integration by linear Programming
substitution, Integration by parts, 11.2 Solution of linear Programming using
integration by partial fractions, and graphical method, feasible and
integration by trigonometric identities. infeasible regions, feasible and
6.3 Definite integrals and their properties, infeasible solutions, optimal feasible
Fundamental Theorem of Integral solutions (upto three nonitrivial
Calculus, applications in finding areas constraints)
under simple curves.

@iitjeehelps
@iitjeehelps
@iitjeehelps
1
Units, Measurement
and Dimensions

Introduction
Science is a systematic attempt to understand natural phenomena in as much detail and depth as
possible and use the knowledge, so gained to predict, modify and control the phenomena.
Every natural occurrence around us like the Sun, the wind, the planets, atmosphere, human body etc.,
follows some basic laws. To understand these laws, by observing natural occurrence is called Physics.
These laws of physics are related and applicable to every aspect of life, thus understanding them leads
to their applications in several fields for further development of society, which is also known as
technology.

Physical Quantities
All those quantities which can be measured directly or indirectly and in terms of which the laws of
Physics can be expressed, are called physical quantities. For example, length, mass, temperature,
speed and force, electric current, etc.

Units of Physical Quantities


Unit of any physical quantity is its measurement compared to certain basic, arbitrarily chosen,
internationally accepted reference standard. There are several systems of units like CGS (Centimetre,
Gram and Second), FPS (Foot, Pound and Second) and MKS (Metre, Kilogram and Second).

Fundamental and Derived Units


The number of physical quantities is quite large. Thus, we may define a set of fundamental quantities
and all other quantities may be expressed in terms of these fundamental quantities. These all other
quantities are known as derived quantities. Units of fundamental and derived quantities are known as
the fundamental units and derived units, respectively. A complete set of these units, both
fundamental and derived units is known as the system of units.

@iitjeehelps
4 SELF STUDY GUIDE BITSAT

System of Units Least Count (LC)


The least count of a measuring instrument is the least
There are some systems used in units, can be defined as value, that can be measured using the instrument. It is
1. CGS System (Centimetre, Gram, Second) is often used denoted as LC.
in scientific work. This system measures, length in
centimetre (cm), mass in gram (g) and time in Least Count of Certain Measuring
second (s). Instruments
2. FPS System (Foot, Pound, Second) It is also called the 1 mm
● Vernier calliper, Least count = = 0.1 mm
British Unit System. This unit measures, length in foot 10 divisions
(foot), mass in gram (pound) and time in second (s). ● Screw gauge, Least count
3. MKS System (Metre, Kilogram, Second) This system Value of 1 pitch scale reading
measures length in metre(m), mass in kilogram (kg)and =
Total number of head scale divisions
time in second (s).
1 mm
4. SI Units (International System of Units) A variety of Least count =
100 divisions
system of units (CGS, FPS and MKS) leads to the need
of a unique system of units which is accepted = 0.01 mm
world-wide. So, in 1971, a system of units named SI ● Travelling microscope,
(System International in French) was developed and Value of 1 main scale division
recommended by general conference on weights and Least count =
Total number of vernier scale divisions
measures. It is an extended version of the MKS system.
SI system has seven fundamental units and two 0.5 mm
= = 0.01 mm
supplementary units, which are as follows 50 divisions
0.5 degree
The two supplementary units of SI system are ● Spectrometer, Least count =
30 divisions
(i) Radian for Plane Angle Angle subtended by an arc
30°
at the centre of the circle having length equal to = = 1°
radius of circle has unit radians. It is denoted by rad. 30 divisions
(ii) Steradian for Solid Angle It is the solid angle which 1 degree (angle) = 60′
has the vertex at the centre of the sphere and cut-off and 1′ = 60′′
an area of the surface of sphere equal to that of
square with sides of length equal to radius of sphere.
It is expressed in unit steradian and denoted by sr. Errors in Measurement
The uncertainty in results of every measurement by any
measuring instruments, is called error in measurement.
Precision of Measuring There can be several causes of errors like instrumental
Instruments errors, imperfection in experimental techniques
procedures, error caused by random changes in
or

Measurement is the foundation of all experimental science temperature, pressure, humidity etc. In systematic errors,
and technology. The instruments used for measurement in mean of many separate measurement differs significantly.
any experiment is called measuring instruments.

Accuracy, Precision and Resolution of Calculation of Magnitude of


an Instrument Errors
(i) Accuracy An instrument is said to be the
accurate, if the physical quantity measured by a (i) True Value
measuring instrument resembles very close to its
It is the mean of observed values.
true value.
a + a2 + a3 + K+ an
(ii) Precision An instrument is said to have high degree ∴ atrue = amean = 1
n
of precision, if the value measured by it remains n
unchanged, however large number of times it may =
1
Σ ai
have been repeated. n i=1
(iii) Resolution It stands for the minimum reading, where, a1 , a2 , K, an are observed values and n is the number
which an instrument can read. of observations.

@iitjeehelps
UNITS, MEASUREMENT AND DIMENSIONS 5

(ii) Absolute Error Significant Figures


Absolute error of a particular measurement is the Significant figure in the measured value of a physical
difference between mean of observed value and true value. quantity tells the number of digits in which we have
Absolute error, confidence. All accurately known digits in a measurement
∆a1 = a mean − a1 , plus the first (only one uncertain digit together in a
∆a2 = a mean − a2 , measured value form significant figures). Larger the
number of significant figures obtained in a measurement,
M M M greater is the accuracy of the measurement.
and ∆an = a mean − an
Rules for Counting Significant Figures
(iii) Mean Absolute Error
(i) All the non-zero digits are significant. In 2.738, the
The arithmetic mean of the magnitudes of different values number of significant figures is 4.
of absolute errors, is known as the mean absolute error.
(ii) All the zeroes between two non-zero digits are
∴ Mean absolute error, significant, no matter where the decimal point is, if at
| ∆ a1 | + | ∆ a2 | + K + | ∆ an | all. As examples, 209 and 3.002 have 3 and
∆ amean =
n 4 significant figures respectively.
The final result of measurement can be written as (iii) If the measurement of number is less than 1, the
a = a mean ± ∆a mean . This implies that value of a is likely to zero (es) on the right of decimal point and to the left
lie as a mean + ∆a mean and a mean − ∆a mean . of the first non-zero digit are non-significant.
In 000807,
. first three underlined zeroes are
non-significant and the number of significant figures
(iv) Relative or Fractional Error is only 3.
The ratio of the mean value of absolute error and the true (iv) The terminal or trailing zero (es) in a number
value, is known as the mean relative error. without a decimal point are not significant. Thus,
Mean absolute error 12.3 = 1230cm = 12300 mm has only 3 significant
Mean relative error =
Mean value of measurement figures.
∆amean (v) The trailing zero (es) in number with a decimal point
=
amean are significant. Thus, 3.800 kg has 4 significant
figures.
(v) Percentage Error (vi) A choice of change of units does not change the
number of significant digits or figures in a
When relative error is expressed in terms of percentage,
measurement.
then relative error is called the percentage error.
Hence,
Percentage error =
∆amean
× 100%
Rules for Arithmetic
amean
Operations with Significant
Combination of Errors Figures
(i) In addition or subtraction, the final results should
(i) Sum of errors (Z) of two physical quantities A and B,
retain as many decimal places as there are in the
where ∆A and ∆B are their absolute errors, is number with the least decimal place. As an example
∆Z = ± ( ∆A + ∆B ) sum of 423.5 g, 164.92 g and 24.381 g is 612.801 g, but
(ii) Difference of errors (Z) of two physical quantities A it should be expressed as 612.8 g only because the
and B, where ∆A and ∆B are their absolute errors, is least precise measurement (423.5 g) is correct to only
one decimal place.
∆Z = ± ( ∆A + ∆B )
(ii) In multiplication or division, the final result should
(iii) Errors of a product If Z = AB , then retain as many significant figures, as are there in the
∆Z  ∆A ∆B  original number with the least significant figures.
= ± + 
Z  A B  For example, suppose an expression is performed like
A 24.3 × 1243
(iv) Errors of a quotient If Z = , then = 676.481522
B 44. 65
∆Z  ∆A   ∆B  Rounding the above result upto three significant
=  + 
Z  A  B  figures, the result would become 676.

@iitjeehelps
6 SELF STUDY GUIDE BITSAT

Rules for Rounding off the i.e. dimensions of all the terms in a physical expression
must be same.
Uncertain Digits LHS = RHS
Result of arithmetic computation, we get a number having
Dimensional analysis can be used in conversion of units, to
more digits than the appropriate number of significant
figures, then these uncertain digits are rounded off as per check the dimensional correctness of physical relation and
the rules given ahead. to establish relation among various physical quantities.
(i) The preceding digit is raised by 1, if the insignificant
digit to be dropped is more than 5 and is left Dimensional Analysis and
unchanged, if the latter is less than 5. Its Applications
e.g.18.764 will be rounded off to 18.8 and 18.74 to 18.7.
Dimensional analysis help us in deducing certain relations
(ii) If the insignificant figure is 5 and the preceding digit among different quantities. Main applications of
is even, then the insignificant digit is simply dropped. dimensional analysis are as follows:
However, if the preceding digit is odd, then it is
raised by one, so as to make it even. e.g. 17.845 will be To check the correctness of a given
rounded off to 17.84 and 17.875 to 17.88.
physical equation
If both sides of a physical relation have same dimensions,
Dimensions of Physical then the relation is dimensionally correct. Dimensional
analysis is also used to deduce relation among the physical
Quantities quantities, i.e. if the dimensions of physical quantities on
The dimensions of a physical quantity are the power to both sides is known, then we can deduce relations
which the base quantities are raised to represent that correlating the quantities with these dimensions.
quantity. The expression which shows how and which base
quantities represent the dimensions of a physical quantity, To convert a physical quantity from
is called the dimensional formula. e.g. for volume, one system to another
dimensional formula is [M0L3T0 ]. An equation, where a
Let dimensional formula of a given physical quantity be
physical quantity is equated with its dimensional formula is
called dimensional equation. e.g. dimensional equation for [M aLbT c ]. If a physical quantity is known in one system of
force is unit (n 1 ). Then, we can relate it with another system of unit
[ F ] = [MLT −2 ] (n 2 ) as below
a b c
n 1  M 1   L 1   T1 
=
Principle of Homogeneity of n 2  M 2  L 2  T2 
Dimensions NOTE Here, a system having base units [M1, L1, T1 ] the numerical
According to this principle, a correct dimensional equation value of the given quantity be n1, and the numerical value n2 in
must be homogeneous, another unit system having the base units M2 ,L 2 , T2 .

@iitjeehelps
Practice Exercise
1. Which one is not a unit of time? 11. The nearest star to our solar system is 4.29 light year
a. Leap year b. Year c. Shake d. Light year away. How much is this distance in terms of parsecs?
2. The height of the building is 50 ft. The same in a. 1.32 b. 3.21 c. 2.31 d. 3.12
millimetre is 12. The concorde is the fastest airlines used for
a. 560 mm b. 285 mm commercial service. It can cruise at 1450 mile per
c. 1786.8 mm d. 15240 mm hour (about two times the speed of sound or in other
words, mach 2). What is it in m/s?
3. Which of the following is the most precise device for
measuring length? a. 644.4 m/s b. 80 m/s
c. 40 m/s d. None of these
a. A vernier calliper with 20 divisions of the sliding
scale 13. One light year is defined as the distance travelled by
b. An optical instrument that can measure length within light in one year. The speed of light is 3 × 108 m/s.
wavelength of light Find the same in metre.
c. A screw gauge of pitch 1 mm and 100 divisions on a. 3 × 1012 m b. 9.461 × 1015 m
the circular scale
c. 3 × 1015 m d. None of these
d. None of the above
4. The radius of hydrogen atom in ground state is 14. The acceleration of a car is 10 mile per hour per
5 × 10 −11
m. Find the radius of hydrogen atom in ft
second. The same in is
fermimetre. (1 fm = 10−15 m). s2
ft ft
a. 5 × 104 fm b. 2 × 104 fm a. 1467
. b. 14.67
s2 s2
c. 5 × 102 fm d. 5 × 106 fm c. 40 ft / s2 d. None of these
5. One nautical mile is 6080 ft. The same in kilometre is 15. The speed of light in vacuum is 3 × 108 m/s. How
a. 0.9 km b. 0.8 km many nanosecond does it take to travel one metre in a
c. 1.85 km d. None of these vacuum?
6. The area of a room is 10 m2. The same in ft 2 is a. 8 ns b.
10
ns
a. 107.6 ft 2 b. 77 ft 2 3
c. 77.6 ft 2 d. None of these c. 3.34 ns d. None of these
3
7. The density of iron is 7.87 g/cm . If the atoms are 16. The time taken by an electron to go from ground state
spherical and closely packed. The mass of iron atom to excited state is one shake (one shake = 10−8 s).
is 9 .27 × 10−26kg. What is the volume of an iron atom? Find this time in nanosecond.
a. 1.18 × 10−29 m3 b. 2.63 × 10−29 m3 a. 10 ns b. 4 ns c. 2 ns d. 25 ns
. × 10−28 m3
c. 173 d. 0.53 × 10−29 m3 17. The time between human heart beat is 8 × 10−1 s.
8. In the previous question, what is the distance between How many heart beats are measured in one minute?
the centres of adjacent atoms? a. 75 b. 60 c. 82 d. 64
a. 2.82 × 10−9 m b. 0.282 × 10−9 m 17
18. The age of the universe is 5 × 10 s. Find the age of
c. 0.63 × 10−9 m d. 6.33 × 10−9 m universe in year.
9. The world’s largest cut diamond is the first start of a. 158 × 106 year b. 158 × 109 year
Africa (mounted in the British Royal Sceptre and kept c. 158 × 108 year d. 158 × 1011 year
in the tower of London). Its volume is 1.84 cubic inch. 19. Assuming the length of the day uniformly increases
What is its volume in cubic metre? by 0.001 second per century. Calculate the net
a. 30.2 × 10−6 m3 b. 33.28 m2 effect on the measure of time over 20 centuries.
c. 4.8 m3 d. None of these
a. 3.2 hour b. 2.1 hour c. 2.4 hour d. 5 hour
10. Crane is British unit of volume. 20. Find the number of molecules of H2O in 90 g of water.
(One crane = 170.474 litre). Convert crane into SI unit.
a. 35.6 × 1023 molecules . × 1023 molecules
b. 4122
a. 0.170474 m3 b. 17.0474 m3
c. 0.0017474 m3 d. 1704.74 m3 c. 27.2 × 1023 molecules d. 30.11 × 1023molecules

@iitjeehelps
8 SELF STUDY GUIDE BITSAT

21. The mass of Earth is 5.98 × 1024 kg. The average 32. 1 revolution is equivalent to 360°. The value of
atomic weight of atoms that make up Earth is 40 u. 1 revolution per minute is
How many atoms are there in Earth? a. 2π rad/s b. 0.1047 rad/s
a. 9 × 1051 b. 9 × 1049 c. 9 × 1046 d. 9 × 1055 c. 3.14 rad/s d. None of these
22. One amu is equivalent to 931 MeV energy. The rest 33. The height of a man is 5.87532 ft. But measurement is
mass of electron is 9.1 × 10−31 kg. The mass equivalent correct upto three significant figures. The correct
energy is (Here, 1 amu = 1.67 × 10−27 kg) height is
a. 0.5073 MeV b. 0.693 MeV a. 5.86 ft b. 5.87 ft
c. 4.0093 MeV d. None of these c. 5.88 ft d. 5.80 ft
. × 10−27 kg. The
23. One atomic mass unit in amu = 166 34. 4.32 × 2.0 = .........
atomic weight of oxygen is 16. Find the mass of one a. 8.64 b. 8.6
atom of oxygen. c. 8.60 d. 8.640
a. 26.56 × 10−27 kg b. 10.53 × 10−27 kg 35. 4.338 + 4.835 × 3.88 ÷ 3.0 is equal to
c. 74 × 10−27 kg d. 2.73 × 10−27 kg
a. 10.6 b. 10.59
24. One horse power is equal to c. 10.5912 d. 10.591267
a. 746 W b. 756 W c. 736 W d. 766 W . × 2.88 is equal to
36. 10
2
25. If E = mc a. 2.88 b. 2.880
where, m = mass of the body, c = speed of light c. 2.9 d. None of these
Guess the name of physical quantity E . . × 2.88 is equal to
37. 100
a. Energy b. Power a. 2.88 b. 2.880
c. Momentum d. None of these c. 2.9 d. None of these
26. One calorie of heat is equivalent to 4.2 J. BTU (British 38. If v = velocity of a body, c = speed of light.
Thermal Unit) is equivalent to 1055 J. The value of v
one BTU in calorie is Then, the dimension of is
c
a. 251.2 cal b. 200 cal
c. 263 cal d. None of these a. [M0L0T 0] b. [MLT −1]
c. [ML2T −2] d. None of these
27. It is claimed that the two cesium clocks, if allowed to
run for 100 yr, free from any disturbance, may differ by 39. The expression for centripetal force depends upon
only about 0.02s. Which of the following is the correct mass of body, speed of the body and the radius of
fractional error? circular path. Find the expression for centripetal force.
a. 10− 9 b. 10− 5 c. 10− 13 d. 10− 11 mv 2 mv 2
a. F = b. F =
28. Which of the following is the average mass density of 2r 3 r
sodium atom assuming, its size to be about 2.5 Å mv 2 m 2v 2
c. F = 2 d. F =
(Use the known values of Avogadro's number and the r 2r
atomic mass of sodium). 40. The maximum static friction on a body is F = µN .
a. 0.64 × 103 kg / m3 b. 8.0 × 102 kg / m3 Here, N = normal reaction force on the body,
c. 8.6 × 103 kg / m3 d. 6.4 × 105 kg / m3 µ = coefficient of static friction. The dimensions of µ is
29. Electron volt is the unit of energy (1 eV = 1.6 × 10−19 J). a. [MLT −2] b. [M0L0T 0θ −1]
In H-atom, the binding energy of electron in first orbit c. dimensionless d. None of these
is 13.6 eV. The same in joule (J) is 41. What are dimensions of Young’s modulus of
a. 10 × 10−19 J b. 21.76 × 10−19 J elasticity?
c. 13.6 × 10−19 J d. None of these a. [ML−1T −2] b. [MLT −2]
30. 1 mm of Hg pressure is equivalent to one torr and one c. [MLT −1] d. None of these
torr is equivalent to 133.3 N/m 2. The atmospheric 42. The surface tension is T =
F
, then the
pressure in mm of Hg pressure is l
a. 70 mm b. 760 mm dimensions of surface tension is
c. 3.76 mm d. None of these a. [MLT −2] b. [MT −2]
5 2
31. One bar is equivalent to 10 N/m . The atmosphere c. [M0L0T 0] d. None of these
pressure is 1.013 × 105 N/m 2 The same in bar is 43. The dimension of heat capacity is
a. 1.88 bar b. 1.013 bar a. [L2T −2θ −1] b. [ML2T −2θ −1]
c. 2.013 bar d. None of these c. [M−1L2T −2θ −1] d. None of these

@iitjeehelps
UNITS, MEASUREMENT AND DIMENSIONS 9
44. If ∆H = mL, where m is mass of body. 52. The work done by a battery is W = ε ∆q , where
∆H = total thermal energy supplied to the body ∆q = charge transferred by battery ε = emf of the
L = latent heat of fusion. battery. What are dimensions of emf of battery?
Find the dimensions of latent heat of fusion. a. [A−2M0L0T −2] b. [A−2ML2T −3]
a. [ML T ] 2 −2 2 −2
b. [L T ] 0 0 −2
c. [M L T ] d. [ML T ] 0 −1 c. [A0M2T −3] d. [A−1ML2T −3]

45. Solar constant is defined as energy received by Earth J


53. The expression for drift speed is v d = .
per cm2 per minute. Find the dimensions of solar ne
constant. Here, J = current density,
a. [ML2T −3] b. [M2L0T −1] c. [MT −3] d. [MLT −2] n = number of electrons per unit volume,
C2 . × 10−19unit
e = 16
46. The unit of electric permittivity is . Find the
Nm 2 The unit and dimensions of e are
dimensions of electric permittivity. a. coulomb and [AT]
2 −1 −3 4
a. [A M L T ] −1 −3 4
b. [AM L T ] b. ampere per second and [AT −1]
c. no sufficient information
c. [A2M−1L−3T 0] d. [A2M0L−3T 4]
d. None of the above
47. A physical relation is ε = ε 0εr 54. The unit of current element is ampere-metre. Find the
where, ε = electric permittivity of a medium dimensions of current element.
ε0 = electric permittivity of vacuum a. [AML] b. [AML2T]
εr = relative permittivity of medium c. [MLT 2] d. [AL]
What are dimensions of relative permittivity?
55. The magnetic force on a point moving charge is
a. [ML2T −2] b. [M0L2T −3] F = q ( v × B).
c. [M0L0T 0] d. [M0L0T −1] Here, q = electric charge
1 2 v = velocity of the point charge
48. The dimensions of εE are same as
2 B = magnetic field
a. energy density (energy per unit volume) The dimensions of B is
b. energy a. [AMLT −1]
c. power b. [A−1MLT −2]
d. None of the above
c. [A−1MT −2]
49. The electric flux is given by scalar product of electric d. None of these
field strength and area. What are the dimensions of E
electric flux? 56. What are dimensions of ?
B
a. [A−2ML3T −2] b. [A−1ML3T −2]
a. [LT −1] b. [LT −2]
c. [A−1ML3T −3] d. [A0M2LT −1]
c. [MLT −1] d. [ML2T −1]
50. Electric displacement is given by D = εE
57. What are the dimensions of µ 0ε 0?
Here,
ε = electric permittivity Here, µ 0 = magnetic permeability in vacuum,
E = electric field strength. ε 0 = electric permittivity in vacuum
Find the dimensions of electric displacement. a. [ML−2T −2] b. [L−2T −2]
−2 −2 −1
a. [AML T] b. [AL T ] c. [L−2T 2] d. None of these
c. [AL−2T] d. None of these
58. In the formula, a = 3bc 2 ‘a’ and ‘c’ have dimensions
51. The energy stored in an electric device known as of electric capacitance and magnetic induction,
q2 respectively. What are dimensions of ‘b’ in MKS
capacitor, is given by U = system?
2C
where, U = energy stored in capacitor a. [M−3L−2T 4Q4] b. [M−3T 4Q4]
C = capacity of capacitor c. [M−3T 3Q] d. [M−3L2T 4Q−4]
q = charge on capacitor R
59. Find the dimensions of .
Find the dimensions of capacity of the capacitor L
a. [A2M−1L−2T 4] Here, R = electric resistance
b. [AM−1L−2T 4] L = self-inductance
c. [A2M−2L−2T 4] a. [T −2] b. [T −1]
d. [A0M0L−2T 4] c. [ML−1] d. [T]

@iitjeehelps
10 SELF STUDY GUIDE BITSAT

60. The magnetic energy stored in an inductor is given by 67. The dimensions of frequency is
1 a. [T −1] b. [M0L0T]
E = La I b . Find the value of ‘a’ and ‘b’.
2 c. [M0L0T −2] d. None of these
Here, L = self-inductance, I = electric current.
68. The dimensions of wavelength is
a. a = 3, b = 0 b. a = 2, b = 1
a. [M0L0T 0] b. [M0LT 0]
c. a = 0, b = 2 d. a = 1, b = 2
c. [M0L−1T 0] d. None of these
61. In L-R circuit, I = I 0 [1 − e −t / λ ] 2π
69. The optical path difference is defined as ∆x = .
Here, I = electric current in the circuit. Then, λ
a. the dimensions of I 0 and λ are same What are dimensions of optical path difference?
b. the dimensions of t and λ are same a. [M0L−1T 0] b. [M1L1T 0]
c. the dimensions of I and I 0 are not same c. [ML0T1] d. [ML−2T]
d. All of the above
W
70. The unit of intensity of a wave is ? What are
62. A physical quantity u is given by the m2
2
B dimensions of intensity of wave?
relation u = .
2µ 0 a. [MT −3] b. [AML0T −2]
Here, B = magnetic field strength c. [M0L−1T −2] d. None of these
µ 0 = magnetic permeability of vacuum a sin θ + b cos θ
71. If x = , then
The name of physical quantity u is a +b
a. energy b. energy density a. the dimensions of x and a are same
c. pressure d. None of these b. the dimensions of a and b are not same
63. The energy of a photon depends upon Planck’s c. x is dimensionless
constant and frequency of light. Find the expression d. None of the above
for photon energy. dv x 
a. E = hν b. E =
h 72. ∫ 2nv − v 2
= a n sin−1  − 1 on
 a 
the basis of
ν
ν dimensional analysis, the value of n is
c. E = d. E = hν 2
h a. 0 b. − 2
64. If energy of photon is E ∝ h c λ . a b d c. 3 d. None of these

Here, h = Planck’s constant, c = speed of light and Find the value of following on the basis of significant
λ = wavelength of photon figure rule
Then, the value of a , b and d are  ∆x 
 , if x = a .
n
a. 1, 1, 1 b. 1,− 1,1 73. Calculate the fractional error 
 x 
c. 1, 1, − 1 d. None of these n
 ∆a   ∆a 
65. The radius of nucleus is r = r 0 A 1/ 3
, where A is mass a. ±   b. ± n  
 a   a 
number. The dimensions of r 0 is ∆a ∆a
c. ± n loge d. ± n log
a. [MLT −2] b. [M0L0T −1] a a
c. [M0LT 0] d. None of these a 3b 3
74. The relation gives the value of ‘x’ x = .
1 c d
66. The power of lens is P = where f is focal length of
f′ Find the percentage error in ‘x’, if the percentage
the lens. The dimensions of power of lens is error in a , b , c and d are 2%, 1%, 3%, and 4%,
a. [LT −2] b. [M0L−1T 0] respectively.
c. [M0L0T 0] d. None of these a. ± 8% b. ± 10% c. ± 12% d. ± 14%

@iitjeehelps
BITSAT Archives
1. For the equation F ∝ Aav bd c , where F is the force, A is 8. The energy (E ), angular momentum (L ) and universal
the area, v is the velocity and d is the density, the gravitational constant (G ) are chosen as fundamental
values of a , b and c are, respectively [2014] quantities. The dimensions of universal gravitational
a. 1, 2, 1 b. 2, 1, 1 c. 1, 1, 2 d. 0, 1, 1 constant in the dimensional formula of Planck's
constant (h ) is [2008]
2. If edge lengths of a cuboid are measured to be
a. zero
1.2 cm, 1.5 cm and 1.8 cm, then volume of the cuboid
b. −1
is [2014]
c. 5/3
a. 3.240 cm3 b. 3.24 cm3 c. 3.2 cm3 d. 3.0 cm3
d. 1
3. If the force is given by F = at + bt 2 with t as time. The α − az
dimensions of a and b are [2012]
9. In the relation p = e , where p is the pressure, z
β kθ
a. [MLT − 4] and [MLT − 2] b. [MLT − 3] and [MLT − 4] the distance, k is Boltzmann constant and θ is the
2 −3 2 −2
c. [ML T ] and [ML T ] d. [ML2T − 3] and [ML3T − 4] temperature, the dimensional formula of β will be
4. The dimensional formula for inductance is [2012] a. [M0L2T 0] b. [ML2T] [2007]
2 −2 −2
a. [ML T A ] 2
b. [ML TA ] −2 c. [ML0T − 1] d. [ML2T − 1]
c. [ML2T − 1A− 2] d. [ML2T − 2A− 1] 10. A physical quantity is given by X = [Ma Lb Tc ]. The
5. A cube has a side of length 1.2 × 10 −2
m. Calculate its percentage error in measurement of M, L and T are
α, β and γ, respectively. Then, the maximum % error in
volume. [2011] the quantity X is [2006]
a. 1.7 × 10− 6 m3 b. 1.73 × 10− 6 m3 a. aα + bβ + cγ
c. 1.70 × 10− 6 m3 d. 1.732 × 10− 6 m3 b. aα + bβ − cγ
 h a b c
+ +
6. The dimensions of the quantity hc  where, h =  is c.
 2π  α β γ
d. None of the above
a. [ML2T − 1] b. [MLT − 1] [2010]
c. [ML3T − 2] d. [ML2T − 1] 11. Which one of the following is not a unit of Young's
modulus? [2006]
7. A resistor of 10 kΩ has a tolerance of 10% and a. Nm− 1
another resistor of 20 k Ω has a tolerance of 20%. The b. Nm− 2
tolerance of the series combination is nearly [2009] c. dyne cm− 2
a. 10% b. 20% c. 15% d. 17% d. mega pascal

Answer with Solutions


Practice Exercise or 6080 × 12 × 2.54 cm = n 2 × 103 × 100 cm
6080 × 12 × 2.54
1. (d) leap year, year and shake are units of time and light ∴ n2 = = 1.85 km
year is the unit of distance. 105
6. (a) n1u1 = n 2u 2
2. (d) n1u1 = n 2u 2
nu 50 ft 50 × 12 × 2.54 cm n1u1 10 m2 10 × (100 cm)2
∴ n2 = 1 1 ⇒ = = 15240 mm ∴ n2 = = = = 107.6 ft 2
u2 mm 0.1 cm u2 ft 2 (12 × 2.54 cm)2
∴ 50 ft = 15240 mm mass 9.27 × 10−26 × 10−3 g
7. (a) Volume = =
3. (b) An optical instrument gives most precise density 7.87 g / cm3
measurement. . × 10−29 m3
= 118
−15
4. (a) ∴ 10 m = 1 fm 4
∴ 1 m = 1015 fm 8. (b) Volume = . × 10− 29
π r 3 = 118
3
∴ 5 × 10−11 m = 5 × 10−11 × 1015 fm = 5 × 104 fm ⇒ r = 1409
. × 10− 10
5. (c) ∴ n1u1 = n 2u 2 So, distance between centre of adjacent atoms is
or 6080 ft = n 2 km 0.282 × 10− 9 m.

@iitjeehelps
12 SELF STUDY GUIDE BITSAT

9. (a) n1u1 = n 2u 2 20. (d)Q 18 g of H2O = 6.022 × 1023 molecules


n u 1.84 (inch)3 1.84 (2.54)3 cm3 6.022 × 1023
∴ n2 = 1 1 = = ∴ 1g of H2O =
u2 m3 (100 cm)3 18
= 30.2 × 10−6 m3 6.022 × 1023
∴ 90 g of H2O = × 90
10. (a) 1 crane = 170.474 litre 18
= 30.110 × 1023 molecules
SI unit of volume is m2.
Using, 1 litre = 103 cm3 m 5.98 × 1024
21. (b) n = = = 9 × 1049
or 1 litre = 10− 3 m3 . × 10−27
M 40 × 166

170.474 litre = 170.474 × 10− 3m3 = 0.170474 m3 9.1 × 10−31


22. (a) The mass of electron = (amu)
1.67 × 10−27
11. (a) Distance = 4.29 light year.
9.1 × 10−31
= 4.29 × 9.46 × 1015 m ∴ E= × 931 MeV = 0.5073 MeV
. × 10−27
167
[Q 1 light year = 9.46 × 1015 m]
23. (a) The mass of one atom of oxygen,
4.29 × 9.46 × 1015
= parsec . × 10−27 kg = 26.56 × 10−27 kg
m = 16 amu = 16 × 166
3.08 × 1016
[Q1 parsec = 3.08 × 1016 m] 24. (a) 25. (a)
= 132
. parsec 26. (a) Q 4.2 J = 1 cal
1 1
12. (a) n1u1 = n 2u 2 ∴ 1J= cal ∴ 1055 J = × 1055 cal = 2512
. cal
4.2 4.2
n1u1 1450 mile 1450 s mile
∴ n2 = = = 27. (d) Time interval = 100 years
u2 m/ s hour m hour
= 100 × 365 × 24 × 60 × 60 s
1450 s × 1.6 km
= −3 = 644.4 m/s = 3.155 × 109 s
10 km 60 × 60 s
Difference in time = 0.2 s
13. (b)
Difference in time (s)
14. (a) n1u1 = n 2u 2 ∴ Fractional error =
Time interval (s)
10 mile ft 0.2
or = n2 2 = = 6.34 × 10− 12 = 10 × 10− 12 ≈ 10− 11
hour second s 3.155 × 109
10 × 1760 × 3 ft ft
or 2
= n2 2 28. (a) Average radius of sodium atom,
60 × 60 s s
2 r = 2.5 Å = 2.5 × 10−10 m
∴ n 2 = 1467
. ft/s
4
s 1 1 ∴ Volume of sodium atom = πr 3
15. (b) t = = = × 10−8 s 3
v 3 × 108 3 4
= × 3.14 × ( 2.5 × 10 ) = 65.42 × 10− 30 m3
− 10 3
1 10−8 10 3
= × ns = ns
3 10−9 3 Mass of a mole of sodium = 23 g = 23 × 10− 3 kg
16. (a)∴ n1u1 = n 2u 2 One mole contains 6.023 × 1023 atoms, hence the mass of
1 × shake = n 2s sodium atom,
or 10−8 s = n 2s 23 × 10− 3
M = kg = 3.82 × 10− 26 kg
∴ n 2 = 10 ns 6.023 × 1023
t 1 minute 60 s 600 ∴ Average mass density of sodium atom
17. (a) n = −1
= −1
= −1
= = 75
8 × 10 s 8 × 10 s 8 × 10 s 8 M 3.82 × 10− 26
ρ= = kgm− 3 = 0.64 × 103 kgm− 3
18. (c) n1u1 = n 2u 2 V 65.42 × 10− 30
or 5 × 1017 s = n 2 year 29. (b)Q . × 10−19 J
1 eV = 16
5 × 1017 s 5 × 1017 s ∴ . × 10−19 J = 2176
13.6 eV = 13.6 × 16 . × 10−19 J
∴ n2 = = = 158 × 108 year
year 365 × 24 × 60 × 60 s 30. (b) 1 atm = 1.013 × 105 N/m2
19. (b) The length of day after 20 century is 24 hour and Q 133.3 N / m2 = 1mm of Hg
0.21 s 1
∴1 N / m2 = mm of Hg = 1.013 × 105 N / m2
0.21 × 365 × 100 hour 133.3
∴The change in time, ∆t =
60 × 60 1
= × 1.013 × 105 mm of Hg = 760 mm of Hg
= 2.1hour 133.3

@iitjeehelps
UNITS, MEASUREMENT AND DIMENSIONS 13
31. (b) 1 bar = 105 N / m2 q2
51. (a)U =
1.013 × 105 2C
= 1.013 × 105 N/m2 = bar = 1.013 bar
105 q2 [AT] 2
∴ C= ∴ [C] = = [A2M−1L−2T 4]
rev 1 × 2π rad π 2U [ML2T −2]
32. (b) 1 = = rad/s = 0.1047 rad/s
min 60 s 30 52. (d)W = ε∆q
33. (a) 34. (b) 35. (a) 36. (c) 37. (a) 38. (a) W [ML2T − 2]
⇒ ε= = 0 0 ⇒ [ε] = [ML2A− 1T − 3]
39. (b) F = m v r a b c
q [M L AT]
[MLT − 2] = [ML0T 0] a [M0LT − 1]b [M0LT 0]c J
53. (a) Q vd =
[MLT − 2] = [Ma Lb + c T − b ] ne
⇒ a = 1, b + c = 1 and b = 2  J  [AL−2]
[e] =   = = [AT] = amp-second = coulomb
2 + c = 1 and c = − 1 nvd   1  [LT −1]
mv 2  L3 
F = mv 2r − 1 ⇒ F =
r
F 54. (d) Dimensions of current element are [ampere-metre]
40. (c)Q F = µN ∴ µ= = [AL]
N
F  [ MLT ]
−2 55. (c)Q F = qv × B or F = qvB sin θ
∴ [µ] = = = dimensionless
N  [ MLT −2]  F  [MLT −2]
∴ [B] =   = −1
= [A−1MT −2]
stress qv  [ATLT ]
41. (a) Young’s modulus of elasticity,Y =
strain E N / C N Am Am Am m
56. (a) The unit of = = = = =
  B N / Am C N C As s
 F   Fl  [MLT −2L]
∴ [Y ] =   = = = [ML−1T −2] E m  L 
∴ = = = [LT −1]
 A ∆l   A ∆l  [L2L] B  s   T 
 l 
1
F [MLT − 2] 57. (c) As we know the speed of light, c =
42. (b)T = = = [ML0T − 2] µ 0ε 0
l [M0LT 0] 1 1 1
or c 2 = ∴ µ 0ε 0 = 2 = = [L−2T 2]
43. (b) ∆H = c ∆θ µ 0ε 0 c [LT −1] 2
∆H [ML2T −2] 58. (a)
∴ c = = = [ML2T −2θ −1]
∆θ [θ] L
59. (b)Q = time constant
2 −2 R
∆H [ML T ]
44. (b) ∆H = mL ∴ [L] = = = [L2T −2] [L] [R]
m [M] ∴ = [ T] ⇒ = [T −1]
[R] [L]
energy [ML2T −2] 1
45. (c) Unit = = 60. (d) E = La I b
cm2 minute [L2T] 2
∴The dimensions of solar constant = [MT −3] [E] = [ ML2T −2], [L] = [ ML2 T −2 A−2], [I ] = [M0L0T 0A]
C2 Using principle of Homogeneity,
46. (a) Unit of ε =
Nm2 [ML2 T −2] = [M1L2T −2A−2] a [M0L0 T 0A] b
[(AT)2] [MLT −2] = [ Ma L2a T −2a A' −2a + b ]
Dimensions of ε = = [A2M−1L−3T 4]
[MLT −2L2]
⇒ a = 1 and − 2a + b = 0 ⇒ b = 2
ε
47. (c) ε = ε 0εr ⇒ εr = 61. (b) Exponent has no dimensions.
ε0
∴ [e − t / λ ] = [M0L0T 0]
Relative permittivity is the ratio of ε and ε 0, hence it is
 t
dimensionless. or − = [M0L0T 0] or [t ] = [ λ ] ∴ [ λ ] = [T ]
i.e. [M0L0T 0]  λ 
Hence, (b) is correct
48. (a)
B2
49. (c) Electric flux is φ = E ⋅ S = ES cos θ 62. (b) u =

N 2 MLT −2 × L2 (N/Am)2 N2A2 N Nm J
∴ [ φ] = m = = [A−1ML3T −3] Unit of u = = = = =
C AT N/A2
NA2m2 m2 m3 m3
50. (c) D = εE = energy per unit volume = energy density
C2 N  C  AT 63. (a) E = h a . νb …(i)
Unit of D = ∴ [D] =  2  = 2 = [AL−2T]
Nm2 C m  L where, h = Planck’s constant and ν = frequency.

@iitjeehelps
14 SELF STUDY GUIDE BITSAT

[E] = [ML2T −2], [ ν] = [M0L0 T −1] and [h] = [ML2 T −1] a sin θ  b cos θ 
∴ [x ] =  =  or [x ] = dimensionless
2 −2 2 −1 a
From Eq. (i), [ML T ] = [ML T ] [M L T ] 0 0 −1 b
a +b   a +b 
[ML2T −2] = [Ma L2a T − a − b ] 72. (a) [LHS] = [RHS]
⇒ a = 1 and − a − b = − 2 ⇒ a + b = 2 ⇒ b = 1  dv   x  [dv ]
⇒ E = hν or   = an sin−1 − 1  or = [an]
2
 2av − v   a  [ v 2]
64. (c) 65. (b) 66. (b) 67. (a) 68. (a) 69. (a)
[dv ]
W J or = [an] or [M0L0T 0] = [an] ∴ n = 0
70. (a) Unit = 2 = 2 [v ]
m sm
[ML2T − 2] ∆x ∆a
∴ [Intensity of wave] = = [MT −3] 73. (b) x = an ∴ =±n
[TL2] x a
a sin θ + b cos θ 74. (d)
71. (c) x =
a+b

BITSAT Archives
1. (a) [MLT −2] = [L2a ] × [Lb T −b ] [Mc L−3c ] = [Mc L2a + b – 3c T −b ] 6. (c) hc = Eλ = [ML2T −2] [L] = [ML3T −2]
Comparing powers of M, L and T, on both sides, we get 10
7. (d) r1 = 10 kΩ, ∆r1 = × 10 = 1 kΩ
c = 1, 2a + b − 3c = 1, − b = − 2 or b = 2 100
Also, 2a + 2 − 3(1) = 1 ⇒ 2a = 2 or a = 1 20
r2 = 20 kΩ, ∆r2 = × 20 = 4 kΩ
∴ This is 1, 2, 1 100
2. (a) Volume of cuboid = l × b × h = 1.8 × 1.5 × 1.2 cm3 Maximum tolerance = ∆r1 + ∆r2 = 1 + 4 = 5
= 2.70 × 12. = 3.240 cm3 r1 + r2 = 10 + 20 = 30 kΩ
5
Using concept of significant figures, product is reported in % age of tolerance = × 100 = 16.67% = 17%
30
number of significant figures present in measurement
which has least number of significant figures, here all 8. (a) h ∝ G x Ly E z
measurement have 2 significant figures. Write the dimensions on both sides
So, volume = 3.2 cm3 [ML2T −1] ∝ [M− 1L3T −2] x [ML2T −1] y [ML2T −2]z
(Keeping 2 significant figures only) [ML2T −1] = k [M− 1L3T −2] x [ML2T −1] y [ML2T −2] z
3. (b) Dimension of at = Dimension of F
Comparing the powers, we get
F 
[at ] = [F ] ⇒ [a] = 1= − x + y + z …(i)
 t 

2 = 3x + 2y + 2z …(ii)
 MLT  2
−3
[b] =   ⇒ [a] = [MLT ] − 1 = − 2x − y − 2z …(iii)
 T  On solving Eqs. (i), (ii) and (iii), we get
Dimension of bt 2 = Dimension of F x =0
F  αz
[bt 2] = [F ] ⇒ [b] = 2 9. (a) In given equation, should be dimensionless.
t  kθ
 MLT −2  kθ
[b] =  −4 ∴ α=
2  ⇒ [b] = [MLT ] z
 T 
[ML2T −2K −1 × K]
4. (a) EMF induced in an electrical circuit ⇒ [a] = = [MLT −2]
[L]
dI
e =L (numerically) α α   MLT −2 
dt and p = ⇒ [ p] =   =  −1 −2  = [M0L2T 0]
dt W dt  W β  β   ML T 
or L =e = . Q e = V = 
dI Q dI  Q 10. (a) X = [ Ma Lb Tc ]
2 −2
Wdt [ML T ] [T] Maximum percentage error = a α + b β + cγ
= = = [ML2T −2A−2]
It . dI [A][T][A] stress
11. (a) Y = = N/ m2 or pascal [in SI system]
5. (a) Volume,V = l 3 = (1.2 × 10−2 m)3 = 1.728 × 10−6m3 strain
Since length (l) has two significant figure, the volume (V ) dyne
and y = [in CGS system]
will also have two significant figure. cm2
Therefore, the correct answer isV = 1.7 × 10−6 m3 Thus, Nm−1 is not the unit of Young’s modulus.

@iitjeehelps
2
Scalar and Vectors

Physical Quantity
Physical quantity is that which can be measured by available apparatus.

Scalar and Vector Quantities


A scalar quantity is one whose specification is completed with its magnitude only. Two or more than
two similar scalar quantities can be added according to the ordinary rules of algebra. e.g., mass,
distance, speed, energy etc.
A vector quantity is a quantity that has magnitude as well as direction. Not all physical quantities have
a direction. Temperature, energy, mass, and time, for example, do not ‘‘point’’ in the spatial sense. We
call such quantities scalars, and we deal with them by the rules of ordinary algebra.
Vector quantities can be added according to the law of parallelogram or triangle law.
A vector quantity can be represented by an arrow. The front end (arrow head) represents the direction
and length of the arrow gives its magnitude.
NOTE Orthogonal vectors If two or more vectors are perpendicular to each other, then they are known as
orthogonal vectors.
Unit vector A vector of unit magnitude and whose direction is same as the given vector is called unit vector.
Basically, unit vector represents the direction of the given vector.
Consider a vector A. This vector is represented as
Vector = (Magnitude of the vector) × (Direction of the vector)
i.e. A = | A | A$
A
So, A$ =
|A |
where, A$ is a unit vector drawn in the direction of A.
Unit vector is a dimensionless physical quantity. Unit vectors along X, Y and Z-axes are $i , $j and k$ respectively.

@iitjeehelps
16 SELF STUDY GUIDE BITSAT

Laws of Vector Addition Polygon Law of Vector Addition


If a number of non-zero D
There are three laws for the addition of vectors vectors are represented by
E C
(i) Triangle law of vector addition the (n − 1) sides of an n sided
(ii) Parallelogram law of vector addition polygon taken in same order
(iii) Polygon law of vector addition then the resultant is given by
R B
the closing side or the nth
side of the polygon taken in
Triangle Law of Vector Addition opposite order. So,
A

If two vectors are represented R= A+ B+ C+ D+ E


both in magnitude and direction
by the two sides of a triangle R B sin θ
taken in the same order, then
the resultant of these vectors is
B Subtraction of Vectors
α θ Negative of a vector (– v) is a vector of
represented both in magnitude B
and direction by the third side of O A B cos θ
the same magnitude as vector v but v
the triangle taken in reverse Vector A and B along sides of pointing in a direction opposite to that O u A
order as shown below. a triangle of v. u–v
–v
i.e. R = A+B = B+A Therefore,
The resultant R can be calculated as u − v = u + (− v ) C Q

R = A + B + 2 AB cosθ
2 2
NOTE If the vectors form a closed n sided polygon with all the sides
in the same order, then the resultant is 0.
If θ is the angle between A and B, then
| A + B | = A2 + B 2 + 2 AB cosθ
C C
B sin θ
If R makes an angle α with A, then tan α = B B
A + B cosθ
A A
Parallelogram Law of Vector Addition Vector in three dimension
According to parallelogram law of vector addition, if two
If r = xi$ + y$j + zk$
vectors acting on a particle are represented in magnitude
and direction by two adjacent sides of a parallelogram, then (a) | r | = x 2 + y 2 + z 2
the diagonal of the parallelogram represents the magnitude
and direction of the resultant of the two vectors acting on (b) Let r makes α , β and γ angles with x-axis, y-axis and
the particle. z-axis respectively, then
B C
x y
cos α = , cos β =
B R x + y2 + z2
2
x + y2 + z2
2
β Q
α z
θ and cos γ =
O A A x + y2 + z2
2

Vectors A and B along


sides of a parallelogram (c) cos2 α + cos2 β + cos2 γ = 1
Q Lami’s theorem
i.e. OA + AC = OC; A + B = R
a b c
Magnitude of the resultant R is given by = =
sin α sin β sin γ
R = A2 + B 2 + 2 AB cosθ y

Here, θ = Angle between A and B. So, the direction of R can a


be found by angle α or β of R with A and B.
B sin θ
Here, tan α = γ β
A + B cosθ
b α c
Asin θ
⇒ tan β = x
B + Acosθ z

@iitjeehelps
SCALAR AND VECTORS 17

The Scalar Product or Dot ●


Important Points
The dot product of force F and displacement s gives work
Product (scalar quantity), i.e. F ⋅ s = W.
● The dot product of force (F ) and velocity (v ) is equal to power
The scalar product of two vectors a and b in Fig. (a) is (scalar quantity), i.e. F ⋅ v = P.
written as a ⋅ b and is defined to be ● The dot product of magnetic induction (B ) and area vector
a ⋅ b = abcos φ …(i) (A ) is equal to the magnetic flux (φ) linked with the surface
(scalar quantity) B ⋅ A = φB
where, φ is the angle between the vectors a and b.
Component of b
along direction of
a is b cos φ
The Vector Product or
φ
a
Cross Prduct
a b
φ The vector product of a and b, written
Component of a c =a ×b
b along direction of
as a × b, produces a third vector c
b is a cos φ whose magnitude is c = absin φ
(a) (b) where, φ is the smaller of the two
Because of the notation, a ⋅ b is also known as the dot angles between a and b. b
product and is spelled as ‘‘a dot b.’’ Because of the notation, a × b is φ
(i) Dot product of the vectors with itself is equal to the also known as the cross product, and
a
square of the magnitude of the vector. it is spelled as ‘‘a cross b’’.
a ⋅ a = a ⋅ a cos0° ⇒ a ⋅ a = a 2 (cos 0° = 1)
(i) If two vectors are perpendicular to each other, we
If θ = 180°, i.e. vectors are anti-parallel. have θ = 90° and therefore, sinθ = 1. So that,
Then, a ⋅b = ab( −1) [Qcos 180° = − 1] a × b = ab n.
$
a ⋅ b = − ab (ii) The vectors a, b and a × b thus form a right handed
i.e. If two vectors are anti-parallel then their dot system of mutually perpendicular vectors. It follows
product equals the negative product of the at once from the
magnitudes of vectors. i i
above that in case
If θ = 90°, i.e. vectors are perpendicular. of the orthogonal Plus Minus
a ⋅ b = ab cos 90° = ab (0) = 0 triad of unit vectors
^ ^ ^ k j k j
Vectors are perpendicular ⇔ Dot product = 0 i , j and k (each
^ ^ ^ ^ ^ ^ perpendicular to
(ii) If a = ax i + a y j + az k and b = bx i + b y j + bz k and θ
each other).
a⋅ b
is the angle between a and b, then cosθ = where, ^ ^ ^ ^ ^
ab i× j =−j×i=k
a ⋅ b = ab cosθ ^ ^ ^ ^ ^
j×k=−k× j=i
The component of a parallel to b in the vector form is
^ ^ ^ ^ ^
(a ⋅ b ) b and k×i=−i×k= j
c=
|b|2
The component of a B
Cross Product of Two Vectors in
perpendicular to b in Determinant Form
vector form is 180°– θ
 a ⋅ b θ For two vectors a and b, their cross product is given by
–A A
d= a−  2 b θ $i $j k$ B
|b|  180°– θ
a × b = ax a y az ,
If angle between A and
–B bx b y bz γ
B is θ, then α
A
(a) Angle between −A and B is (180° − θ ) where, a = a i$ + a $j + a k$ β
x y z
(b) Angle between A and −B is (180° − θ )
and b = bx $i + b y $j + bz k$ . C
(c) Angle between −A and −B is θ

@iitjeehelps
Practice Exercise
1. An insect moves on a circular path of radius 7 m. Find 10. ABCD is a parallelogram, and a, b, c and d are the
the maximum magnitude of displacement of the position vector of vertices A, B, C and D of a
insect. parallelogram, choose the correct option.
a. 7 m b. 14π m c. 7π m d. 14 m a. c + b = d − a b. c − b = d − a
2. In previous problem, if the insect moves with constant c. c − c = d − a d. None of these
speed 10 m/s. Find the minimum time to achieve
maximum magnitude of displacement.
11. A man walks 4 km due West, 500 m due South finally
750 m in South-West direction. Find the distance and
a. 10 s b. 2 s c. 1.4 s d. 2.2 s magnitude of displacement travelled by the man.
3. Two forces of magnitudes 3 N and 4 N are acted on a a. 4646.016 m and 5250 m b. 5250 m and 4646.016 m
body. The ratio of magnitude of minimum and c. 4550.016 m and 2300 m d. None of these
maximum resultant force on the body, is
12. Calculate the resultant force, when four force of 30 N
a. 3/4 b. 4/3 due East, 20 N due North, 50 N due West and 40 N
c. 1/7 d. None of these due South, are acted upon a body.
4. A vector a makes 30° and b makes 120° angle with the a. 20 2 N, 60°, South of West
x-axis. The magnitude of these vectors are 3 unit and b. 20 2 N, 45°, South of West
4 unit, respectively. The magnitude of resultant vector c. 20 2 N, 45°, South of East
is d. 20 2 N, 45°, South of East
a. 3 unit b. 4 unit c. 5 unit d. 1 unit
13. A block of 150 kg is placed on an inclined plane with
5. If two forces of equal magnitude 4 units acting at a an angle of 60°. Calculate of the weight parallel to the
point and the angle between them is 120°, then find inclined plane.
the magnitude and direction of the sum of the two
vectors 0
15kg
a. 4, θ = tan−1(173
. ) b. 4, θ = tan−1( 0.73)
c. 2, θ = tan−1(173
. ) d. 6, θ = tan−1( 0.73)
60°
| a + b|
6. If = 1, then the angle between a and b is a. 1300 N b. 1400 N c. 1100 N d. 750 N
| a − b|
a. 0° b. 45° c. 90° d. 60° 14. A cat is situated at a point A (0, 3, 4) and rat is situated
at point B (5, 0, −8). The cat is free to move but the rat
7. The angle between A and the resultant of ( A + B) and is always at rest. Find the minimum distance travelled
( A − B) will be
by cat to catch the rat.
 A
a. 0° b. tan−1   a. 5 unit b. 12 unit c. 13 unit d. 17 unit
B
B  A − B 15. An insect fly start from one corner of a cubical room
c. tan−1   d. tan−1   and reaches at diagonally opposite corner. The
 A  A + B
magnitude or displacement of the insect is 40 3 ft.
8. Three forces are acted on a body. Their magnitudes Find the volume of cube.
are 3 N, 4 N and 5 N. Then, a. 64 3 ft 3 b. 1600 ft 3
a. the acceleration of body must be zero c. 64000 ft 3 d. None of these
b. the acceleration of body may be zero
c. the acceleration of the body must not be zero 16. In above problem, if the insect does not fly but crawls.
d. None of the above What is the minimum distance travelled by the insect?
a. 89.44 ft b. 95.44 ft
9. In the given figure, O is the centre A E
c. 40 ft d. 80 ft
of regular pentagon ABCDE . Five F0 F 0
forces each of magnitude F0 are 17. If a particle is moving on an elliptical path given by
F0
acted as shown in figure. The B D r = b cos ωt i$ + a sin ωt $j, then find its radial
resultant force is F0
F0 acceleration along r.
a. 5F0 b. 5F0 cos 72° a. ωr b. ω 2r
C
c. 5F0 sin 72° d. zero c. −ω 2r d. None of these

@iitjeehelps
SCALAR AND VECTORS 19
18. Obtain the magnitude and direction cosines of vector 26. The unit vector perpendicular to vectors a = 3 i$ + $j and
( A − B), if A = 2i$ + 3 $j + k$ , B = 2i$ + 2$j + 3k$ b = 2$i − $j − 5 k$ is
1 −2 2 1
a. 0, , b. 0, , ( $i − 3$j + k$ ) 3$i + $j
5 5 5 5 a. ± b. ±
1 11 11
c. 0, 0, d. None of these
5 ( 2$i − $j − 5 k$ )
c. ± d. None of these
30
19. The vertices of a quadrilateral are A (1, 2, −1),
B ( −4, 2, − 2), C (4, 1, −5) and D (2, −1, 3). Forces of 27. If three vectors along coordinate axes represent the
magnitude 2, 3, 2 N are acting at point A along the adjacent sides of a cube of length b, then the unit
lines AB, AC, AD, respectively. Find their resultant. vector along its diagonal passing through the origin
10$i − 9 $j + 6 k$  $i − 9 $j − 6 k$  will be
a. b.   $i + $j + k$ $i + $j + k$ $i + $j + k$
 
26  26  a. b. c. $i + $j + k$ d.
$i − 9 $j + 16 k$ $i − 19 $j + 6 k$ 2 3b 3
c. d.
26 26 28. Choose the correct option A × B = C.
20. A force F = a $i + b $j + c k$ is acted upon a body of (i) C is perpendicular to A
mass m. If the body starts from rest and was at the (ii) C is perpendicular to B
origin initially, find its new coordinate after time t. (iii) C is perpendicular to ( A + B)
at 2 bt 2 ct 2 at 2 2bt 2 ct 2 (iv) C is perpendicular to ( A × B)
a. , , b. , ,
2m 2m 2m 2m m 2m a. Only (i) and (ii) are correct
at bt ct 2
2 2
b. Only (ii) and (iv) are correct
c. , , d. None of these c. (i), (ii) and (iii) are correct
m m 2m
d. All of the above
21. The angle between vector a = 2i$ + $j − 2 k$ and
29. Find the vector area of a triangle whose vertices are
b = 3 $i − 4 $j is equal to a , b and c .
−1  3  1
−1 1 1
a. cos   b. cos   a. (b × c + c × a + a × b ) b. (b × c + c × a + a × b )
 15  15 2 3
−1 2 1
c. zero d. cos c. (b × c + a × c + b × a ) d. None of these
15 3
22. The resultant of two vectors P and Q is R. If the vector 30. If three vectors xa − 2b + 3c , −2a + yb − 4c and
Q is reversed, then the resultant becomes S, then −zb + 3c are coplanar, where a , b and c are unit
choose the correct option. (or any) vectors, then
a. R 2 + S 2 = 2(P 2 − Q 2 ) b. R 2 + S 2 = 2(P 2 + Q 2 ) a. xy + 3zx − 3z = 4 b. 2xy − 3zx − 3z − 4 = 0
c. R 2 + S 2 = (P 2 − Q 2 ) d. R 2 − S 2 = 2(P 2 + Q 2 ) c. 4xy − 3zx − 3z = 4 d. xy − 2zx − 3z − 4 = 0
23. For what value of x, will the two vectors 31. A force F = ( 2i$ + 3 $j − k$ ) N is acting on a body at a
A = 2i$ + 2$j − x k$ and B = 2i$ − $j − 3 k$ are position r = (6 $i + 3 $j − 2 k$ ). Calculate the torque about
perpendicular to each other? the origin.
a. x = − 2 / 3 b. x = 3 / 2 c. x = − 4 / 3 d. x = 2 / 3
a. ( 3$i + 2$j + 12 k$ ) Nm b. (9 $i + 2$j + 7 k$ ) Nm
24. Calculate the work done by a force F = ( $i + 2$j + 3 k$ ) N c. ( $i + 2$j + 12 k$ ) Nm d. ( 3$i + 12$j + k$ ) Nm
to displace a body from position A to position. B. The
32. Find the values of x and y for which vectors
position vector of A is r1 = ( i$ + 3 $j + k$ ) m and the
position vector of B is r2 = ( 2i$ + 2$j + 3k$ ) m. A = (6 $i + x$j − 2 k$ ) and B = (5 $i − 6 $j − y k$ ) are be
a. 5 J b. 3 J c. 2 J d. 10 J parallel.
2 36 5
25. If c = a × b, then a. x = 0, y = b. x = − ,y =
3 5 3
a. the direction of c changes, when the angle between 15 23 36 15
c. x = − ,y = d. x = ,y =
a × b increases up to θ (θ < 180° ) 3 5 5 4
b. the direction of c changes, when the angle between
a and b decreases up to θ (θ > 0° ) 33. Find the area of the parallelogram determined by
c. the direction of c does not change, when the angle A = 2i$ + $j − 3 k$ and B = 12$j − 2 k$
between a and b increases a. 42 b. 56
d. None of the above c. 38 d. 74

@iitjeehelps
20 SELF STUDY GUIDE BITSAT

34. Choose the correct option. 38. One day in still air, a motor-cyclist riding north at
a. a × (b × c) + b × ( c × a) + c × ( a × b) = 0 30 m/s, suddenly the wind starts blowing Westward
b. a × ( c × b) + b × ( c × a) + c × ( a × b) = 0 with a velocity 50 m/s, then calculate the apparent
c. a × ( c × b) + b × ( c × a) − c × ( a × b) = 0 velocity with which the motor-cyclist will move.
d. None of the above a. 58.3 m/s b. 65.4 m/s c. 73.2 m/s d. 53.8 m/s

35. The three conterminous edges of a parallelopiped are 39. Calculate the distance travelled by the car, if a car
travels 4 km towards north at an angle of 45° to the
a = 2$i − 6$j + 3k$ , b = 5$j, c = − 2$i + k$
east and then travels a distance of 2 km towards north
Calculate the volume of parallelopiped.
at an angle of 135° to the east.
a. 36 cubic units b. 45 cubic units
c. 40 cubic units d. 54 cubic units a. 6 km b. 8 km
c. 5 km d. 2 km
36. If the three vectors are coplanar, then find x.
40. On one rainy day a car starts moving with a constant
A = i$ − 2$j + 3k$ , B = x$j + 3k$ , C = 7i$ + 3 $j − 11k$ acceleration of 1.2 m/s 2. If a toy monkey is suspended
a. 36 / 21 b. −51/ 32 c. 51/ 32 d. −36 / 21 from the ceiling of the car by a string, then find the
angle with the vertical with the string be now inclined.
37. A particle is moving along a circular path with a a. tan−1 ( 0.25)
constant speed 30 m/s. What is change in velocity of a
b. tan−1 ( 0.63)
particle, when it describe an angle of 90° at the centre
of the circle. c. tan−1( 0.12)
d. tan−1 ( 3)
a. zero b. 30 2 m/s c. 60 2 m/s d. 30 2 m/s

BITSAT Archives
1. If A and B denote the sides of a parallelogram and its 3. The component of vector A = a x $i + a y $j + a z k$ along
1 the direction of $i − $j is [2008]
area is AB (A and B are magnitude of A and B
2 a. a x − a y + a z b. a x − a y
respectively), the angle between A and B is [2014]
c. (a x − a y ) / 2 d. (a x + a y + a z )
a. 30° b. 45°
c. 60° d. 90° 4. If A ⋅ B = A × B, then angle between A and B is [2006]
2. A vector F1 acts along positive x-axis. If its vector a. 45° b. 30° c. 60° d. 90°
product with another F2 is zero, then F2 could be [2009] 5. If a = $i + 2$j − 3 k$ and b = 3 i$ − $j + 2 k$ , then the angle
a. 4 $j b. (k$ + $j ) between the vectors a + b and a – b is [2005]
c. ( $j + k$ ) d. −4 $i a. 60° b. 90° c. 45° d. 55°

Answer with Solutions


Practice Exercise ⇒
Fmin 1
=
Fmax 7
1. (d) The maximum magnitude of displacement
4. (c) The figure shown as described in the question.

B A
b

= | AB| = 2r = 14 m a
Distance πr 22 7 90°
2. (d) Time = = = × = 2.2 s
Speed 10 7 10 30° 120°

3. (c) F1 = 3 N and F2 = 4 N
Fmax = F1 + F2 = ( 3 + 4) = 7 N
Fmin = F2 − F1 = 4 − 3 = 1 N

@iitjeehelps
SCALAR AND VECTORS 21
According to parallelogram law of vectors, ∴ | s| = x 2 + y 2
R = a + b + 2ab cos α
2 2
= 20523889.9 + 1061580.08 = 4646.016 m
= 32 + 42 + 2 × 3 × 4 cos 90° = 5 units 12. (b) Effective force along West (x ) = (50 − 30) = 20 N
5. (a) | R| = | a + b | = 4 + 4 + 2( 4)( 4) cos 120°
2 2 Effective force along South, (y ) = 40 − 20 = 20 N
| R| = 4 Resultant = R = x 2 + y 2 = 202 + 202 = 20 2 N
Let θ = angle between a + b with x-axis y π
tan θ = = 1 = tan 45° ⇒ θ= = 45°
 4 sin(120° )  ( 3.464) x 4
θ = tan−1   = tan−1 = tan−1 (173
. )
 4 + 4 cos (120° ) 2 13. (a) Component parallel to inclined plane, is 150 g sin 60°.
| a + b|
6. (c) =1
| a − b|
| a + b | = | a − b|

15
or

0
15
or a 2 + b 2 + 2ab cos α = a 2 + b 2 − 2ab cos α

0
g

15
co
or a 2 + b 2 + 2ab cos α = a 2 + b 2 − 2ab cos α

0
s

g
60

s in
4ab cos α = 0 or cos α = 0

°
or

60
α = 90°

°
or
7. (a) R = ( A + B) + ( A − B) 150 g

R = 2A 1500 3
The angle between A and 2A is zero, because they are = = 1299 ≈ 1300 N
2
parallel vectors.
14. (c) The minimum distance
8. (b) The magnitude of three forces
5N = The magnitude of displacement of cat = | rB − rA |
3N, 4N and 5 N will be zero, if these
vectors from a close polygon will all
3N Here, r = 5$i − 8 k$ , r = 3$j + 4 k$
B A
the sides in the same order as 4N
∴ s = rB − rA = 5$i − 3$j − 12 k$
shown in figure.
Hence, option (b) is correct . ∴ | s | = (5)2 + ( −3)2 + ( −12)2
9. (d) According to polygon law, resultant force will be zero. = 25 + 9 + 144
10. (b) As shown in figure, D C = 13.34 ≈ 13 unit
according to condition given in
15. (c) Here, 3a = 40 3 ft
question. c
BC = c − b ∴ a = 40 ft
d
AD = d − a A ∴ Volume = a 3 = ( 40 ft )3 = 64000 ft 3
B
but BC = AD a 16. (a) Distance = ( 40)2 + ( 40 + 40)2
b
∴ c− b= d− a
= 1600 + 6400 = 89.44 ft
O
11. (b) The given figure shows the dr
17. (a) = − bω sin ωt $i + aω cos ωt $j
direction of motion of man. dt
N d 2r
= − bω 2 sin ωt $i + aω 2 cos ωt $j
dt 2
d 2r
= − ω 2 [b cos ωt $i + a sin ωt $j] = − ω 2r
dt 2
4000 m
W O E 18. (a) ( A − B) = 2$i + 3$j + k$ − 2$i − 2$j − 3 k$ = $j − 2 k$
A 45°
| A − B | = 1+ 4 = 5
m

0 1 −2
0

500 m
75

Direction cosines = , ,
G 5 5 5
C
S ( −4 − 1) $i + ( 2 − 2)$j + ( −2 + 1) k$ 
19. (b) F1 = 2  
 ( −5)2 + ( 0)2 + ( −1)2 
Here, distance = 4000 + 750 + 500 = 5250 m
 −5$i − k$ 
Here, x = − 4000 − 750 cos 45° = − 4530.33 F1 = 2  
 26 
y = − 500 − 750 sin 45° = − 1030.33  

@iitjeehelps
22 SELF STUDY GUIDE BITSAT

y B(–4, 2, –2)
F1 26. (a) a = 3$i + $j and b = 2$i − $j − 5 k$
–1)
, 2, $i $j k$
A(1 F2
C(4, 1, –5)
F3 a × b = 3 1 0 = ( −5) $i − ( −15 − 0) $j + ( −3 − 2) k$
2 −1 −5
D(2, –1, 3)
x
= − 5$i + 15$j − 5 k$
R = a × b = − 5 ( $i − 3$j + k$ )
$i − 3$j + k$ $i − 3$j + k$
$ =
R =
z
(1)2 + ( −3)2 + (1)2 11
( 4 − 1) $i + (1 − 2)$j + ( −5 + 1)k$ 
F2 = 3   27. (d) Diagonal vector A = b $i + b $j + b k$
 (1)2 + ( −3)2 + ( −4)2 
 3$i − $j − 4 k$  or A = b2 + b2 + b2 = 3 b
F2 = 3   $ $ $
 26  ∴ $ =A = i+ j+k
A
A 3
( 2 − 1) i + ( −1 − 2) $j + ( 3 + 1) k$ 
$
F3 = 2   28. (c) A × B = C
 (1)2 + ( −3)2 + ( 4)2  It is clear from the figure that C is perpendicular to A and
 $i − 3$j + 4k$  B, and C is parallel to A × B.
F3 = 2  
 26  C
FR = F1 + F2 + F3
1
= [ −10$i − 2k$ + 9 $i − 3$j − 12 k$ + 2$i − 6$j + 8 k$ ] B
26 θ
1 $ A
= [ i − 9 $j − 6 k$ ]
26
1
1 29. (a) Vector area of triangle = ( BC × BA)
20. (a) F = mA ⇒ A = (a$i + b$j + c k$ ) 2
m 1 1
Distance travelled, = [(c − b ) × (a − b )] = [c × a − b × a + b × b − c × b]
2 2
1
s = ut + At 2 1
2 = [c × a + b × c + a × b]
2
1 1 $
s = 0 ×t + (a i + b $j + c k$ ) t 2 x −2 3
2m
30. (d) Condition of coplanarity, −2 y −4 = 0
t2
s = (a $i + b $j + c k$ ) 0 −z 3
2m
21. (d)Q a ⋅ b = | a| | b| cos θ ⇒ x ( 2y − 4z ) + 2( −4 − 0) + 3( 2z − 0) = 0
⇒ 2xy − 4zx − 8 + 6z = 0
a ⋅b ( 2$i + $j − 2k$ ) ⋅ ( 3$i − 4$j )
∴ cosθ = = ⇒ xy − 2zx − 4 + 3z = 0
| a ||b | 22 + 12 + ( −2)2 ( 3)2 + ( −4)2 $i $j k$
6−4 2  2
= = ⇒ θ = cos−1  31. (a) τ = r × F = 6 3 −2
3 × 5 15  15
2 3 −1
22. (b) 23. (a) 24. (a)
25. (c) c = a × b = $i ( −3 + 6) − $j ( −6 + 4) + k$ (18 − 6)
c is perpendicular to a and b as show in figure. The τ = ( 3$i + 2$j + 12 k$ ) Nm
direction of c does not change when the angle between a 32. (b) Condition for parallel vectors, A × B = 0
and b is increases or decreases.
c $i $j k$
= 6 x −2 = 0
5 −6 −y
b
θ = $i ( −xy − 12) − $j ( −6y + 10) + k$ ( −36 − 5x ) = 0
a 36 5
or xy = − 12, +6y = 10 −36 = 5x ⇒ x = − ,y =
5 3

@iitjeehelps
SCALAR AND VECTORS 23
$i $j k$ |v1| = |v 2| = v
33. (a) A × B = 2 1 −3 | ∆v | = v + v 2 − 2v cos 90°
0 12 −2 = 2 v = 30 2 m/s
= $i ( −2 + 36) − $j ( −4 − 0) + k$ ( 24 − 0) 38. (a) Here, θ = 90°
= 34$i + 4$j + 24 k$ N

| A × B | = 1156 + 16 + 576 = 4181


. ≈ 42
30 m/s
34. (a) From vector triple product,
a × (b × c) = (a ⋅ c) b − (a ⋅ b) c …(i) W E
50 m/s
b × (c × a) = (b ⋅ a) c − (b ⋅ c) a …(ii)
c × (a × b) = (c ⋅ b) a − (c ⋅ a) b …(iii)
Now adding Eqs. (i), (ii) and (iii), we get
S
⇒ a × (b × c) + b × (c × a) + c × (a × b) = 0
2 −6 3 Let v′ = apparent velocity
35. (c) Volume,V = a ⋅ (b × c) = 0 5 0 v′ = 302 + 502 + 2(50)( 30) cos 90°
−2 0 1
= 900 + 2500
= 2 (5 − 0) + 6( 0) + 3 ( +10) = 10 + 30 = 40 cubic units
v′ = 58.3 m/s
36. (b) The three vectors are coplanar, if their scalar triple
product is zero. 39. (a) As the distance is a scalar quantity. So, total distance
travelled = 4 + 2 = 6 km
1 −2 3
i.e. A ⋅ ( B × C) = 0 = 0 x 3 =0 40. (c) Let mass of toy = m
7 3 −11 H = M(1.2)
A B
(1) [ −x (11) − 9] + 2 ( 0 − 21) + 3 ( 0 − 7x ) = 0
51 θ
− 11x − 9 − 42 − 21x = 0 ⇒ − 32x − 51 = 0 ⇒ x = − V = mg
32
37. (b) ∆v = | v 2 − v1|
v2

v1 D I
H m(1.2)
tan θ = =
V m(10)
θ = tan−1 ( 0.12)

BITSAT Archives
1. (a) Area of parallelogram = | A × B | 4. (a) A ⋅ B = A × B ⇒ AB cos θ = AB sin θ
1 1 tan θ = 1 ⇒ tan θ = tan 45° ⇒ θ = 45°
AB sin θ = AB ⇒ sin θ = ⇒ θ = 30°
2 2 5. (b) Given that, a = $i + 2$j − 3 k$
2. (d) Let F = x$i
1 and b = 3$i − $j + 2 k$
As, F1 × F2 = 0 and only $i × $i = 0 Now, a + b = $i + 2$j − 3k$ + 3$i − $j + 2k$ = 4$i + $j − k$
∴ F = − 4 $i
2 and a − b = ( $i + 2$j − 3 k$ ) − ( 3$i − $j + 2 k$ ) = − 2$i + 3$j − 5 k$
3. (c) Let B = $i − $j Let θ be the angle between a + b and a – b.
A⋅ B ( a + b) ⋅ ( a − b) ( 4$i + $j − k$ ) ⋅ ( − 2$i + 3$j − 5 k$ )
Then, component of vector A along B = ∴ cos θ = =
| B| | a + b| | a − b| |4$i + $j − k$|⋅ |− 2$i + 3$j − 5 k$ |
(a x $i + a y $j + a z k$ ) ⋅ ( $i − $j ) a x − a y −8 + 3 + 5
= = = = 0 ⇒ θ = 90°
| $i − $j | 2 16 + 1 + 1 4 + 9 + 25

@iitjeehelps
3
Motion in 1, 2 & 3
Dimensions and
Projectile Motion
Motion
If the position of an object is continuously changing with respect to its surroundings with time, then it
is said to be in the state of motion. A bird flying in air, a train moving on rails, a ship sailing on water,
are some of the examples of motion.

Motion in a Straight Line


The motion of a point object in a straight line is one-dimensional motion. During such motion, the
point object occupies definite position on the path at each instant of time t.

Distance and Displacement


Distance It is the total length of the path travelled by a particle in a given interval of time. It is a scalar
quantity having unit metre or centimetre.
Displacement It is the shortest distance between initial and final Y
positions with the direction from initial to final position. It is a vector Distance
quantity having unit metre or centimetre. B
A C
Mathematically, it is equal to difference of position vectors, i.e. ∆r
∆r =| r2 − r1 | , while distance is actual path length, i.e. ABC as shown r1 ∆r = r2 – r1
r2 Displacement
in figure.
X
The displacement of motion may be zero or negative but path length or
distance can never be negative.
NOTE Magnitude of displacement is less than or equal to distance.
Thus, |Displacement | ≤ Distance.

@iitjeehelps
MOTION IN 1, 2 & 3 DIMENSIONS AND PROJECTILE MOTION 25

Speed Velocity
It is defined as the total path length (i.e. actual It is defined as the ratio of displacement and the
distance covered) divided by total time taken by the object corresponding time interval taken by the object, i.e.
Total path length Displacement
i.e. Speed = Velocity =
Total time taken Time interval
So, speed is a scalar quantity. It gives no idea about the Different types of velocity are given below
direction of motion of the object. Hence, speed of the object
(i) Variable Velocity An object is said to have variable
can be zero or positive but never negative.
velocity, if either its speed or its direction or both
change w.r.t. time.
Average Speed (ii) Average Velocity Average velocity is defined as
Average speed of an object is defined as the ratio of ratio of change in position (displacement) to the time
the total distance travelled by the object to the total time interval ( ∆t ).
taken, i.e. Total displacement
Average velocity, v =
Total distance travelled Total time taken
Average speed =
Total time taken x 2 − x 1 ∆x
= =
Average speed of particles in different cases is discussed t2 − t1 ∆t
below
(iii) Instantaneous Velocity Instantaneous velocity is
Case I If a particle travels distances s1 , s2 , s3 etc., with defined as the velocity at a particular instant of time.
speeds v1 , v2 , v3 etc., in same direction, then Instantaneous velocity at particular instant of time t
distance travelled = s1 + s 2 + s3 + K is
∆ x dx
s s s vi = lim =
Total time taken = 1 + 2 + 3 + K ∆t → 0 ∆ t dt
v1 v2 v3
s1 + s2 + ...
Average speed, vav = Acceleration
 s1 s2 
 + + ... Acceleration of an object is defined as the rate of change of
 v1 v2 
velocity. It is a vector quantity having unit m/s 2 or ms −2 .
If s1 = s2 = s , i.e. the body covers equal distance Acceleration can be positive, zero or negative. Positive
with different speeds, then acceleration means velocity increasing with time, zero
2s acceleration means velocity is uniform, while negative
vav = acceleration (retardation) means velocity is decreasing
1 1 with time.
s + 
v
 1 v 2
2v1v2 Average and Instantaneous Acceleration
= Average acceleration is defined as the change in velocity
v1 + v2
( ∆v ) divided by the time interval ( ∆t ).
It means average speed is equal to harmonic mean
Let us consider the motion of a particle. Suppose that, the
of individual speeds.
particle has velocity v 1 at t = t 1 and at a later time t = t 2 it has
Case II If a particle travels with speeds v1 , v2 , v3 etc., velocity v 2 . Thus, the average acceleration during the time
during time intervals t 1 , t 2 , t3 etc., then total interval ∆t = t 2 − t 1 , is
distance travelled, s = v1 t 1 + v2 t 2 + K v − v 1 ∆v
a av = 2 = .
Total time taken = t 1 + t 2 + t3 + K t2 − t1 ∆t

So, average speed, On a plot of velocity versus time, the average acceleration is
v t + v2 t 2 + v3 t3 + K the slope of the straight line connecting the points (v2 , t 2 )
vav = 1 1 and (v1 , t 1 ).
t 1 + t 2 + t3 + K
If the time interval approaches zero, average acceleration is
Case III If t 1 = t 2 = t3 = K = t n , then we have known as instantaneous acceleration. Mathematically,
(v1 + v2 + K + vn ) t v1 + v2 + K + vn ∆ v dv
vav = = a = lim =
nt n ∆t → 0 ∆t dt
It means average speed is equal to arithmetic When a body is moving with a constant acceleration, then
mean of individual speeds. acceleration time graph is a straight line.

@iitjeehelps
26 SELF STUDY GUIDE BITSAT

Negative acceleration is known as retardation. It indicates


When particle covers one-third distance at speed v1, next
that velocity is decreasing with respect to time. one-third at speed v 2 and last one-third at speed v 3 , then
3v1v 2 v 3
Uniformly Accelerated Motion vav =
v1v 2 + v 2 v 3 + v 3 v1
A motion in which change in velocity in each unit of time is
constant, is called as uniformly accelerated motion. For a ● If a particle is accelerated for time t1 with acceleration a 1and
uniformly accelerated motion (a = constant), equations of for time t 2 with acceleration a 2 , then average acceleration is
motion are as follows a t + a 2 t2
a av = 1 1
Kinematic equations of motion t1 + t 2
v = u + at …(i) ● If an object is moving with uniform acceleration, then
1 Instantaneous acceleration = Uniform acceleration
s = ut + at 2 …(ii) = Average acceleration
2
If a particle is moving with constant acceleration from point A
and v ⋅v = u ⋅ u + 2 a ⋅ s …(iii) ●

to B in a straight line. If u and v are corresponding velocities


v 2 = u 2 + 2as at A and B respectively, then velocity at the mid-point of A
where, u = initial velocity, v = velocity at time t, u2 + v 2
and B is v 0 = .
2
s = displacement of particle at time t.
If motion is described in one dimension, so vector sign (→ )
need not be used. Graphs
Sign convention for Fig. (a) motion
in the vertical direction, and Fig. Graphs in One-Dimensional Motion
(b) motion in the horizontal +ve –ve
(a) (b) Graphs can be generalised on the basis of positon-time and
direction is shown in figure.
velocity-time in one-dimensional motion with uniform
Non-uniformly Accelerated Motion velocity and uniform acceleration.
When motion of a particle is not uniform, i.e. acceleration of Displacement-Time Graph
particle is not constant or acceleration is a function of time,
Different cases s-t Graph Characteristics
then following relations hold for one-dimensional motion.
ds dv dv At rest  ds 
(i) v = (ii) a = =v Slope of s-t graph   is
 dt 
dt dt ds
s zero, i. e. zero velocity.
(iii) ds = vdt and (iv) dv = adt or v dv = ads
v=0
Such problems can be solved either by differentiation or
integration by applying some boundary conditions.
t

Free Fall Acceleration Uniform motion Slope of s-t graph is


a =0 constant, i.e.
An important example of a straight line motion with s
v = constant,
constant acceleration, is that of an object rising or falling
s = vt a =0
freely near the earth’s surface.
The constant acceleration equation describes this motion t
but we make two changes in notation, (1) we refer the
motion to the vertical y-axis with + y vertically up, and (2) Uniformly Slope of s-t graph is
accelerated increasing, i. e. positive
we replace a with g, where g is the magnitude of the free fall motion with s value.
acceleration. Near earth’s surface, g = 9.8ms – 2 (32 fts –2 ) u = 0, s = 0, s = 1 at2
at t = 0 2

Some Special Cases t


● If an object starts from rest and moves with a uniform
acceleration, then displacement covered by it in time t is Uniformly Slope of s-t graph is
directly proportional to t 2 , i.e. s ∝ t 2 and if a body starts from retarded motion decreasing, i. e. negative
s value.
rest and moves with a uniform acceleration, then
displacement covered by it in successive seconds will be in
the ratio of odd natural number, i.e.
s 1st : s 2nd : s 3rd = 1 : 3 : 5 K t

@iitjeehelps
MOTION IN 1, 2 & 3 DIMENSIONS AND PROJECTILE MOTION 27
Velocity-Time Graph Relative Motion in Two Dimensions
Different cases v-t Graph Characteristics The description of the motion of an object in two or three
Uniform motion (i) θ = 0 ° (ii) v = constant dimensions depends on the choice of the coordinate system.
v  dv  Figure shows two reference frames in two dimensions. The
Slope of v-t graph   is
v=constant  dt 
vectors r PA and r PB are the position vectors of the object P in
zero, i.e. acceleration is zero.
reference frame A and in reference frame B, respectively.
t
Vector r BA is the position of observer B (located at the
v–u v origin of reference frame B ) w.r.t. frame A. The position
Uniformly a = t = t = constant Slope of v-t graph is
accelerated v constant.
vector of the object P in reference frame B can be obtained
motion with
u = 0, s = 0 from the position vector in reference frame A.
at t = 0
t Frame B

Uniformly Slope of v-t graph is negative


Frame A P
decelerated v (retarded)
motion
v = u – at
rPB
rPA
t0 t
Non-uniformly Slope of v-t graph increases
accelerated v with time. v is increasing, so rBA
motion acceleration is increasing.

r PA = r BA + r PB
t
Non-uniformly v v is decreasing so
The velocity and acceleration of the object P are therefore,
decelerating acceleration is decreasing. v PA = v BA + v PB and a PA = a BA + a PB
motion
For crossing the river in the shortest B vr C
t time, the boat should sail
perpendicular to the flow. If the v vr
NOTE Slope of s-t graph = ds = tanθ width of the river is d and v is the
dt velocity of the boat in still water,
where, θ is the angle made by the tangent at any point (with then O
time axis) on the graph taken anti-clockwise.
2
d  d
t= and OC = d 2 + vr × 
v  v
Relative Motion vr
For crossing the river by the shortest B
If the velocity of two bodies are known w.r.t. a common A
frame of reference, then the velocity of one of the two distance, the boat moves such that
bodies can be measured w.r.t. the second body. the horizontal component of the v vr
θ
Let us consider, if the velocities of the two bodies A and B velocity balances the speed of flow.
w.r.t. earth are v A and v B , then the relative velocity of A Time of crossing, O
w.r.t. B is v AB = v A − v B d
t=
Similarly, we see that v AB = − v BA v − vr2
2

Also, acceleration of A w.r.t. B is where, vr = velocity of river flow


a AB = a A − a B
and a AB = − a BA Applications of the Concept of
If a car moves from west to east (the direction of motion of Relative Motion
the earth) vC = vCE + vE
(i) Rain Problems
and if the car moves from east to west (opposite to the
motion of the earth) vC = vCE − vE In this case, the three velocities with which we have to deal
are
where, vC and vE are the velocities of car and the earth
respectively with respect to an inertial frame of reference. vRG → velocity of rain w.r.t. ground
vMG → velocity of man w.r.t. ground
Also, note that the earth is assumed as the rotating frame
v RM → velocity of rain w.r.t. man
which is a non-inertial one.

@iitjeehelps
28 SELF STUDY GUIDE BITSAT

From the equation of relative motion, v AG → velocity of air w.r.t. ground.


vRM = vRG − vMG or vRM + vMG = vRG v PG → velocity of plane w.r.t. ground.
From this equation we can construct a vector diagram, The relative motion equation would be as shown in figure.
which would be very helpful in solving the questions. v PG = v PA + v AG
The vector diagram for a particular situation is shown in
figure.
v MG Projectile Motion
β α We considered objects moving along straight line paths,
v RM
such as the x-axis. Now, let us look at some cases in which
v RG an object moves in a plane. By this we mean that the object
γ
has a motion in both the x and the y-directions
simultaneously.
From sine law, Which is another way of saying, it moves in two
| v MG | | v RM | | v RG | dimensions. The motion of a particle thrown in a vertical
= = plane, making an angle with the horizontal (≠ 90°), is an
sin γ sin β sin( π − α )
example of two-dimensional motion. This is called
Remember | v MG |, | v RM | or | v RG | are not representing the projectile motion.
length of triangles.

(ii) River Boat Problems Time of Flight


In this case, we have to deal with the following velocities. It is defined as the total time for which the projectile
v SR → velocity of swimmer/steamer/boat w.r.t. river 2u sin θ
remains in air, T =
/water/stream g
B Q Shore
Maximum Height
v RG It is defined as the maximum vertical distance covered by
d P v RG u 2 sin 2 θ
projectile, H =
θ 2g
v SR v SG

A Shore
Horizontal Range
v RG → Velocity of river/water/stream w.r.t. ground. It is defined as the maximum distance covered in horizontal
v SG → Velocity of swimmer steamer/boat w.r.t. ground. u 2 sin 2 θ
From relative motion equation v SR + v RG = v SG distance, R =
g
For the situation shown in the figure, a boat is crossing the
river and it is heading (moving) along AP as shown in the ● Horizontal range is maximum when it is thrown at an angle
figure then the time taken by the boat to cross the river is u2
d of 45° from the horizontal Rmax =
t= g
| v SR cosθ| ● For angle of projection θ and (90° – θ) the horizontal range is
and it will reach the point Q due to river flow. The distance same.
BQ = (| v RG | − | v SR | sin θ )t is termed as drift.

(iii) Aeroplane Problems


Here the following velocities are to be analysed
A Projectile Fired Horizontally
Y from a Certain Height
vAG Let a particle be projected O u
x
PG

horizontally with a velocity u from a


v

vPA height h above the ground. Then, h


taking the point of projectionO as the
x
O X origin, horizontal direction as x-axis A B β0
and vertically upward direction as y v0
v PA → velocity of plane w.r.t. air. y-axis, we find that

@iitjeehelps
MOTION IN 1, 2 & 3 DIMENSIONS AND PROJECTILE MOTION 29

(i) Equation of trajectory is y =


g
x2 u 2 sin 2θ
2 (iv) Horizontal range , R = OB = u x T =
2u g
which represents the equation of a parabola. For a given range R, there are two different angles of
(ii) Time taken by the projectile to reach the ground projection, one of them being the compliment of the
other (15° or 75°, 30° or 60°) as shown in figure.
2h
T = However, the vertical height and the time of flight
g will be different for the two values of θ.
The time is the same as that taken by an object falling y
freely from the same height. 75°
2h
(iii) Horizontal range, AB = R = uT = u 60°
g
45°
(iv) Velocity of projectile at any instant of time t is
30°
^ gt
v = (u i + gt $j ) ⇒ v = u 2 + g 2t 2 and tanβ =
u 15°

O x
A Projectile Fired at an Angle of
Elevation θ from the Ground (v) Maximum horizontal range
u2
Let a particle be projected at an angle of elevationθ from the Rmax = , when θ = 45°
ground level at point O with an initial velocity u. The g
velocity may be resolved into (i) a horizontal component For a given speed of projection, R depends on θ, the
u x = u cosθ and (ii) a vertical component u y = u sinθ. angle of projection. Range will be maximum for a
Moreover, here, ax = 0 and a y = − g . From these values of u given speed of projection, when sin 2 θ = 1 , i.e.2 θ = 90°
and a, we get the following results. or θ = 45°. Hence, for maximum range, the angle of
y projection must be 45°.
(vi) Velocity at any instant t Let the instantaneous
A velocity be v, inclined at an angle β with the
u cos θ
u horizontal. Then,
uy = u sin θ

| v | = vx2 + v 2y
h

θ = (u cos θ )2 + (u sin θ − g t )2
O
ux = u cos θ B x u sin θ − g t
R and tan β =
u cosθ
g If K is the kinetic energy at the point of launch, then
(i) Equation of the trajectory, y = x tan θ − x2
2 u cos θ 2 2
kinetic energy at the highest point is
which is the equation of a parabola, symmetric about 1 1
K ′ = mv 2x = mu 2 cos2 θ
the y-axis (i.e. vertical direction) 2 2
or K ′ = K cos2 θ
u 2 sin 2 θ
(ii) Vertical height covered, h =
2g (vii) Relation between R and h It is found that
R
2 u sinθ = 4cotθ
(iii) Time of flight, T = h
g R
For complementary angles φ and (90° − φ ), if T φ and and if θ = 45°, then max = 4
hmax
T( 90° − φ ) are the time of flight and R is the range, then
or Rmax = 4hmax
2 Rφ 2 R90 ° − φ 2 R
T φ T 90 ° − φ = = = (viii) Two angles of projection for the same range It is
g g g
found that the range of a projectile is the same when
2 R 1° 2 R89 ° it is projected either at an angle of elevation θ° or at
e.g., T 1 °T89 ° = =
g g an angle of (90° − θ ).

@iitjeehelps
Practice Exercise
1. A cat wants to catch a rat. The cat follows the path 8. A sheet of wood moves over a v v
whose equation is x + y = 0. But rat follows the path smooth surface (shown in the θ θ
B
whose equation is x 2 + y 2 = 4. The coordinates of figure). The magnitude of C
possible points of catching the rat are velocity of C is
a. v
a. ( 2, 2 ) b. ( − 2, 2 ) c. ( 2, 3 ) d. (0, 0)
b. 2v cos θ
2. Speed is too velocity as c. 2v sin θ
a. centimetre is too metre d. 2v
b. force is too torque
c. velocity is too acceleration 9. In the arrangement shown in
d. distance is too displacement figure, the ends P and Q of an A B
inextensible string move θ θ
3. A person travelling on a straight line moves with a downwards with uniform speed u.
uniform velocity v 1 for some time and with uniform Pulleys A and B are fixed. The P Q
velocity v 2 for the next the equal time. The average u M u
mass m moves upwards with a
velocity v is given by
speed
v1 + v 2 2 1 1
a. v = b. = + u 2u
2 v v1 v 2 a. 2u cos θ b. c. d. u cos θ
cos θ cos θ
1 1 1
c. v = v1v 2 d. = +
v v1 v 2 10. In the given figure, find the speed of pulley P .
4. A car moves at 80 km in the first half of total time of
motion and at 40 km in the later half. Its average
speed is
a. 60 km/h b. 30 km/h θ θ
c. 120 km/h d. None of these
5. A particle moves with constant speed v along a P
regular hexagon ABCDEF in same order. (i.e. A to B,
B to C, C to D, D to E , E to F and F to A). The
v v
magnitude of average velocity for its motion from A to
C is v 2v v
a. b. 2v cos θ c. − d.
v 2 cos θ 2 sin θ
a. v b.
2
3v 11. A link AB is moving in a vB
c. d. None of these vertical plane. At a certain
2
instant when the link is
6. During the shooting of a superhit film ‘MARD’ Amitabh inclined 60° to the
Bachchan was waiting for his beloved Amrita Singh horizontal the point A is
with his dog. When he saw her approaching, the dog moving horizontally at
was excited and dashed to her than back to master 3 m/s, while B is moving in 60°
and so on, never stopping. How far would you the vertical direction. What A vA
estimate the dog ran if his speed is 30 km/h and each is the velocity of B?
of them walked at 4 km/h, starting 400 m apart? 1
a. 400 m b. 880 m c. 1500 m d. 30 km a. m/s b. 2 3 m/s
3
7. Two particles start from the same point with different 3
c. 3 m/s d. m/s
speeds but one moves along y = a sin ωx and other 2
moves along curve y = a cos ωx
12. Two intersecting straight lines move parallel to
a. they must collide after some time themselves with speeds 3 m/s and 4 m/s,
b. they never collide with each other respectively. The speed of the point of intersection of
 π a  the lines, if the angle between them is 90° will be
c. they may collide at the point P  , 
 4ω 2  a. 5 m/s b. 3 m/s
d. they must collide at the point P c. 4 m/s d. None of these

@iitjeehelps
MOTION IN 1, 2 & 3 DIMENSIONS AND PROJECTILE MOTION 31
13. The displacement-time graph is shown in figure. The 18. A particle starts with a velocity of 2 m/s and moves in
instantaneous velocity is negative at the point a straight line with a retardation of 0.1 m / s 2 . The time
that it takes to describe 15 m is
a. 10 s in its backward journey
Displacement

b. 30 s in its forward journey


c. 10 s in its forward journey
D d. 30 s in its backward journey
C E F
e. Both (b) and (c) are correct
Time 19. A particle starts from rest with acceleration 2 m / s 2 .
a. D b. F c. C d. E The acceleration of the particle decreases down to
zero uniformly during time-interval of 4 s. The velocity
14. In the given x -t curve, of particle after 2 s is
B a. 3 m/s b. 4 m/s c. zero d. 8 m/s
x 20. An aeroplane moves 400 m towards north, 300 m
towards west and then 1200 m vertically upwards.
C Then, its displacement from the initial position is
A
a. 1300 m b. 1400 m
t
c. 1500 m d. 1600 m
a. the velocity at A is zero but at B is non-zero
b. the velocity at A and B are zero 21. The displacement of a particle is given by x = (t − 2)2
c. the velocity at A and B are non-zero where, x is in metres and t in seconds. The distance
d. the directions of velocity at A and B are definite covered by the particle in first 4 s is
15. Which of the following speed-time graph exists in the a.4 m b. 8 m
nature? c. 12 m d. 16 m

v 22. A person walks up a stalled escalator in 90 s when


a. standing on the same escalator, now moving, he is
t
carried in 60 s. The time it would take him to walk up
the moving escalator will be
v a. 27 s b. 72 s c. 18 s d. 36 s
b.
t
23. A body starts from rest and moves with a constant
acceleration. The ratio of distance covered in the nth
second to the distance covered in n second is
v 2 1 1 1 2 1 2 1
a. − b. − c. − d. +
c. n n2 n2 n n2 n n n2

t
24. A particle moving with a uniform acceleration along a
straight line covers distances a and b in successive
d. All of the above
intervals of p and q second. The acceleration of the
16. Two particles describe the same circle of radius a in particle is
the same sense with the same speed v . What is their pq( p + q ) 2(aq − bp ) bp − aq 2(bp − aq )
relative angular velocity? a. b. c. d.
2(bp − aq ) pq( p − q ) pq( p − q ) pq( p + q )
a. v /a b. 2v /a c. v /2a d. va
25. A body moves along x-axis with velocity v x at position
17. A particle of mass m is initially situated at the point P
x. If the plot v x -x is an ellipse with major axis 2A and
inside a hemispherical surface of radius r as shown in
minor axis 2v 0, the maximum acceleration has a
figure. A horizontal acceleration of magnitude a 0 is
modulus
suddenly produced on the O
particle in the horizontal v 02 A
a. b.
direction. If gravitational A v 02
P
acceleration is neglected, the c. v 0A d. None of these
time taken by particle to
touch the sphere again is 26. The distance time graph of a particle at time t makes
angle 45° with respect to time axis. After 1s, it makes
4r sin α 4r tan α angle 60° with respect to time axis. What is the
a. b.
a0 a0 acceleration of the particle?
4r cos α a. ( 3 − 1) unit b. ( 3 + 1) unit
c. d. None of these
a0 c. 3 unit d. 1 unit

@iitjeehelps
32 SELF STUDY GUIDE BITSAT

27. The velocity-time plot for a particle moving on a 31. A bee flies in a line from a point A to another point B in
straight line is shown in the figure, then 4 s with a velocity of | t − 2| m/s. The distance between
A and B in metre is
10 a. 2 b. 4 c. 6 d. 8
v(m/s)

32. The velocity of a particle is v = v 0 + gt + ft 2. If its


10 20 30 position is x = 0 at t = 0, then its displacement after
unit time (t = 1) is
t(s)
g f
a. v 0 + 2g + 3f b. v 0 + +
2 3
–10 g
c. v 0 + g + f d. v 0 + + f
2
33. A particle located at x = 0 at time t = 0, starts moving
a. the particle has a constant acceleration along the positive x-direction with a velocity v that
b. the particle has never turned around varies as v = α x . The displacement of the particle
c. the average speed in the interval 0 to 10 s is the varies with time as
same as the average speed in the interval 10 s to 20 s a. t 2 b. t c. t 1/ 2 d. t 3
d. Both (a) and (c) are correct
34. A particle moves as such whose acceleration is given
28. The acceleration of a train between two stations 2 km
by a = 3 sin 4t , then
apart is shown in the figure. The maximum speed of
the train is a. the initial velocity of the particle must be zero
b. the acceleration of the particle becomes zero after
+5 π
each interval of second
m/s2

4
12 16 c. the particle does not come at its initial position after
4 8 some time
d. the particle must move on a circular path
–5 35. A particle moves along a straight line such that its
t(s)
position x at any time t is x = 3t 2 − t 3 , where x is in
a. 60 m/s b. 30 m/s c. 120 m/s d. 90 m/s metre and t in second, then
29. A body is at rest at x = 0, it starts moving in the a. at t = 0 acceleration is 6 m/s2
positive x-direction with a constant acceleration. At the b. x -t curve has maximum at 8 m
same instant another body passes through x = 0 c. x -t curve has maximum at 2 s
moving in the positive x-direction with a constant d. Both (a) and (c) are correct
speed. The position of the first body is given by x1(t ) 36. The motion of a body falling from rest in a resting
after time t and that of the second body by x 2(t ) after dv
the same time interval. Which of the following graphs medium is described by the equation = a − bv ,
dt
correctly describes (x1 − x 2 ) as a function of time t ? where a and b are constant. The velocity at any time t is
(x1 – x2) (x1 – x2) a
a. a(1 − b 2t ) b. (1 − e −bt )
b
b. c. abe −t d. ab 2(1 − t )
a.
37. A rectangular box is sliding B
O t O t
on a smooth inclined plane A
(x1 – x2) (x1 – x2) l
of inclination θ. At t = 0, the C
box starts to move on the E
c. d. inclined plane. A bolt starts
D
O
to fall from point A. Find
O t t
the time after which bolt α
30. A particle moves in a straight line, so that after t strikes the bottom surface
second, the distance x from a fixed point O on the line of the box.
is given by x = (t − 2)2(t − 5 ). Then,  2l   2l 
a. after 2 s, velocity of particle is zero a.   b.  
 g cos α   g sin α 
b. after 2 s, the particle reaches at O
c. the acceleration is negative, when t < 3 s  2l l
c.   d.  
d. All of the above g g

@iitjeehelps
MOTION IN 1, 2 & 3 DIMENSIONS AND PROJECTILE MOTION 33
38. A car starting from rest, accelerates at the rate f 45. A ball is projected vertically upwards. If resistance due
through a distance s, then continues at constant to air is ignored, then which of the following graphs
f represent the velocity-time graph of the ball during its
speed for time t and then decelerates at the rate to
2 flight?
come to rest. If the total distance travelled is 15 s, then
1 2 1 2 1
a. s = ft b. s = ft c. s = ft d. s = ft 2 a. b. c. d.
6 72 4
39. An object moves, starting from rest through a resistive
medium, such that its acceleration is related to 46. An object is thrown upward with a velocity u, then
velocity as a = 3 − 2v . Then, displacement-time graph is
a. the terminal velocity is 1.5 unit
b. the terminal velocity is 3 unit s
s
c. the slope of a − v graph is not constant
d. initial acceleration is 2 unit a. u2 b. u2
2g 2g
40. If the velocity of a moving particle, v ∝ x , where x is
n

the displacement, then t t


a. when x = 0, the velocity and acceleration are zero
1 s
b. n > s
2
1 c. d.
c. n <
2
d. Both (a) and (b) are correct t t
41. A particle is projected vertically upward in vacuum
with a speed 40 m/s, then velocity of particle when it
47. A balloon going upward with a velocity of 12 m/s is at
a height of 65 m from the earth surface at any instant.
reaches at maximum height 2 s before, is
Exactly at this instant a packet drops from it. How
(Take, g = 10 m/s) much time will the packet take in reaching the surface
a. 20 m/s b. 4.2 m/s of earth? ( Take g = 10 m/s 2)
c. 9.8 m/s d. None of these
a. 7.5 s b. 10 s
42. A juggler keeps on moving four balls in the air throws c. 5 s d. None of these
the balls in regular interval of time. When one ball 48. A stone is released from a balloon moving upward
leaves his hand (speed = 20 m/s), the position of other with velocity v 0 at height h at t = 0. Which of the
balls will be (Take, g = 10 m/s 2) following graph is best representation of velocity-time
a. 10 m, 20 m, 10 m b. 15 m, 20 m, 15 m graph for the motion of stone?
c. 5 m, 15 m, 20 m d. 5 m, 10 m, 20 m
43. Balls are thrown vertically upward in such a way that
v v
the next ball is thrown when the previous one is at the a. b.
maximum height. If the maximum height is 5 m, the
number of balls thrown per minute will be (Take, t t
g = 10 m/s 2)
a. 60 b. 40 c. 50 d. 120
v v
44. A ball is dropped vertically from a height d above the c. d.
ground. It hits the ground and bounces up vertically to
a height d / 2. Neglecting subsequent motion and air t t
resistance, its speed v varies with the height h above
the ground as 49. For Q. 48, draw speed-time graph.

a. b. a. v b. v

t t

v d. None of these
c. d. c.

@iitjeehelps
34 SELF STUDY GUIDE BITSAT

50. A particle P is at the origin starts with velocity mvh


a. zero b.
u = ( 2$i − 4 $j ) m/s with constant acceleration 2
mvh 2
(3 $i + 5 $j) m/s 2. After travelling for 2 s, its distance from c. d. None of these
2
the origin is
a. 10 m b. 10.2 m c. 9.8 m d. 11.7 m 59. Two particles are projected vertically upwards with the
same velocity on two different planets with
51. At an instant t, the coordinates of a particles are accelerations due to gravities g1 and g 2 respectively. If
x = at 2, y = bt 2 and z = 0. The magnitude of velocity of they fall back to their initial points of projection after
particle at an instant t is lapse of time t1 and t 2 respectively. Then,
v v
a. t a 2 + b 2 b. c. d. 2t a 2 + b 2 a. t1t 2 = g1g 2 b. t1g1 = t 2g 2
2 3 tg
c. 1 2 = 2 d. t12 + t 22 = g1 + g 2
52. If x = a (cos θ + θ sin θ ) and y = a (sin θ − θ cos θ ) and θ t 2g1
increases at uniform rate ω. The velocity of particle is 60. A point P moves in counter-clockwise direction on a
a 2θ aθ circular path as shown in the figure. The movement of
a. aω b. c. d. aθω
ω ω P is such that it sweeps out a length s = t 3 + 5, where
53. A particle is moving with velocity v = k (Y $i + X $j ), s is in metre and t is in second. The radius of the path
is 20 m. The acceleration of P when t = 2 s is nearly.
where k is a constant. The general equation for its
path is Y
a. Y = X 2 + constant b. Y 2 = X + constant
P (x, y)
c. XY = constant d. Y 2 = X 2 + constant 20 m
54. A light rigid rod is placed on a smooth horizontal
X
surface. Initially the end A begins to move vertically O A
upward with constant velocity v 0 and centre of the rod
upward with a velocity v 0 / 2 having downward a. 13 ms−2 b. 12 ms−2 c. 7.2 ms−2 d. 14 ms−2
acceleration a 0/ 2, the other end moves downward with
61. A number of particles are projected from a given point
a. zero initial velocity having zero acceleration with equal velocities in different directions in the same
b. zero initial velocity having a 0 downward acceleration
vertical plane. At any instant, they will lie on
c. non-zero initial velocity and zero acceleration
a. parabola b. circle c. hyperbola d. rectangle
d. None of the above
62. Rain water is falling vertically downward with velocity
55. At the top of the trajectory of a projectile, the
v . When velocity of wind is u in horizontal direction,
directions of its velocity and acceleration are
water is collected at the rate of R m 3 / s . When
a. parallel to each other velocity of wind becomes 2u in horizontal direction.
b. inclined to each other at an angle of 45°
The rate of collection of water in vessel is
c. anti-parallel to each other
R
d. perpendicular to each other a. R b.
2
56. A projectile is thrown at an angle of θ = 45 ° to the R 4u 2 + v 2
horizontal, reaches a maximum height of 16 m, then c. 2R d.
u2 + v 2
a. its velocity at the highest point is zero
b. its range is 64 m 63. A ball projected from ground at an angle of 45° just
c. it is thrown at an angle of 30°, its range will decrease clears a wall in front. If point of projection is 4 m from
d. (b) and (c) both are correct the foot of wall and ball strikes the ground at a
57. A heavy stone is thrown from a cliff of height h in a distance of 6 m on the other side of the wall, the
given direction. The speed with which it hits the height of the wall is
ground (air resistance may be neglected) a. 4.4 m b. 2.4 m c. 3.6 m d. 1.6 m
a. must depend on the speed of projection
b. must be larger than the speed of projection
64. The maximum range of a bullet fired from a toy pistol
c. must be independent of the speed of projection mounted on a car at rest is R 0 = 40 m. What will be the
d. (a) and (b) both are correct acute angle of inclination of the pistol for maximum
range when the car is moving in the direction of firing
58. A particle of mass m is projected with a velocity v
with uniform velocity v = 20m / s , on a horizontal
moving at an angle of 45° with the horizontal. The
magnitude of angular momentum of projectile about surface? ( g = 10 m / s 2 )
the point of projection when the particle is at its a. 30° b. 60°
maximum height h is c. 75° d. 45°

@iitjeehelps
MOTION IN 1, 2 & 3 DIMENSIONS AND PROJECTILE MOTION 35
65. Figure shows four paths for a kicked football. Ignoring 71. For a man who wants to cross the river with the
the effect of air on the flight, rank the paths according shortest path AB, find the angle θ (see figure).
to initial horizontal velocity component highest first. A
D
Y
β

β
1 2 3 4
X C O B
 3
a. 1, 2, 3, 4 b. 2, 3, 4, 1 c. 3, 4, 1, 2 d. 4, 3, 2, 1 a. sin−1 
 4
66. A body is projected at an angle θ with the horizontal. −1  4 
When it is at the highest point, the ratio of the potential b. sin  
 3
and kinetic energies of the body is
c. 30°
a. tan θ b. tan2 θ d. Given situation is not possible
c. cot θ d. cot 2 θ
72. To a person going towards East in a car with a velocity
67. A particle is projected at an angle 60° with the
of 25 km/h, a train appears to move towards North
horizontal with a speed 10 m/s. Then, latus rectum is
with a velocity of 25 3 km/h. The actual velocity of
(Take, g = 10 m/s 2)
y the train will be
10 m/s A
D
β

β
60°
O x
a. 5 m b. 15 m C O B
c. 10 m d. 0
a. 25 km/h b. 50 km/h
68. A projectile is thrown at an angle θ such that it is just c. 5 km/h d. 53 km/h
able to cross a vertical wall at its highest point of
journey as shown in the figure. The angle θ at which 73. A man can swim with a speed of 4 km/h in still water.
the projectile is thrown is given by How long does he take to cross a river 1 km wide, if
the river flows steadily 3 km/h and he makes his
v0 strokes normal to the river current. How far down the
H
river does he go, when he reaches the other bank?
a. 800 m b. 900 m
θ
c. 400 m d. 750 m
√3H 74. Rain drops fall vertically at a speed of 20 m/s. At what
−1  1 −1
angle do they fall on the wind screen of a car moving
a. tan   b. tan ( 3) with a velocity of 15 m/s, if the wind screen velocity
 3
inclined at an angle of 23° to the vertical?
 2  3
c. tan−1   d. tan−1    −1 4 
 3  2  cot ≈ 37°
 3 
69. Two cars move in the same direction along parallel a. 60° b. 30°
roads. One of them is a 200 m long travelling with a c. 45° d. 90°
velocity of 20 m/s. The second one is 800 m long
travelling with a velocity of 7.5 m/s. How long will it 75. A bus moves over a straight level road with a constant
take for the first car to overtake the second car? acceleration a. A boy in the bus drops a ball out side.
a. 20 s b. 40 s The acceleration of the ball with respect to the bus
c. 60 s d. 80 s and the earth are respectively
a. a and g
70. A motorboat covers the distance between two spots
on the river banks in t1 = 8 h and t 2 = 12 h in down b. a + g and g − a
stream and upstream respectively. The time required c. a 2 + g 2 and g
for the boat to cover this distance in still water will be
a. 6.9 h b. 9.6 h c. 69 s d. 96 s d. a 2 + g 2 and a

@iitjeehelps
BITSAT Archives
1. A train accelerating uniformly from rest attains a 9. Do we observe the time variation of position is nature
maximum speed of 40 ms −1 in 20 s. It travels at this as shown in the graph? [2009]
speed for 20 s and is brought to rest with uniform
retardation in further 40 s. What is the average velocity

Time
during this period? [2014]
a. 80 m/s b. 25 m/s c. 40 m/s d. 30 m/s
Position
2. A ball is projected upwards from the top of tower with
a velocity 50 m/s making an angle 30° with the a. Yes b. No c. Often d. Rarely
horizontal. The height of tower is 70 m. After how 10. If a body is projected with an angle θ to the horizontal,
many seconds from the instant of throwing will the ball then [2008]
reach the ground? [2014]
a. its velocity is always perpendicular to its acceleration
a. 2 s b. 5 s c. 7 s d. 9 s b. its velocity becomes zero at its maximum height
3. A ball thrown upward from the top of a tower with c. its velocity makes zero angle with the horizontal at its
speed v reaches the ground in t1 sec. If this ball is maximum height
thrown downward from the top of the same tower with d. the body just before hitting the ground, the direction
speed v it reaches the ground in t 2 sec. In what time, of velocity coincides with the acceleration
the ball shall reach the ground, if it is allowed to fall 11. Velocity-time (v-t) graph for a moving object is shown
freely under gravity from the top of the tower? [2014] in the figure. Total displacement of the object during
t1 + t 2 t1 − t 2 the time interval when there is non-zero acceleration
a. b. c. t1 t 2 d. t1 + t 2
2 2 and retardation is [2007]
4. A man runs at a speed of 4 m/s to overtake a standing
bus. When he is 6 m behind the door at t = 0, the bus
moves forward and continuous with a constant 4
acceleration of 1.2 m / s 2. The man reaches the door in 3
time t. Then, [2013] 2
v(m/s)
a. 4 t = 6 + 0.6 t 2 b. 1.2 t 2 = 4 t 1
c. 4 t 2 = 1.2 t d. 6 + 4 t = 0.2 t 2

5. For a given velocity, a projectile has the same range R 0


10 20 30 40 50 60
for two angles of projection if t1 and t 2 are the time of t(s)
flight in the two cases, then [2012]
1 1 a. 60 m b. 50 m c. 30 m d. 40 m
a. t1t 2 ∝ R b. t1t 2 ∝ R 2 c. t1t 2 ∝ d. t1t 2 ∝
R2 R 12. At the top of the trajectory of a projectile, the direction
of its velocity and acceleration are [2007]
6. A body moves with uniform acceleration, then which
a. perpendicular to each other
of the following graph is correct? [2012]
b. parallel to each other
a a a a c. inclined to each other at an angle of 45°
a. b. c. d. d. anti-parallel to each other
t t t t 13. At what point of a projectile motion, acceleration and
velocity are perpendicular to each other? [2006]
7. One second after the projection, a stone moves at an a. At the point of projection
angle of 45° with the horizontal. Two seconds from the b. At the point of drop
start, it is travelling horizontally, its angle of projection c. At the top most point
with the horizon is (given, g = 10 ms −2) [2010] d. Anywhere in between the point of projection and top
a. 60° b. tan−1 ( 4) c. tan−1 ( 3) d. tan−1 ( 2) most point
8. A body covers a total distance of 3s . The first s is 14. From the top of a tower of two stones, whose masses
covered with a velocity u the second s with v and the are in the ratio 1 : 2 are thrown on straight up with an
last s with ω. Then, the average velocity during the initial speed u and the second straight down with the
whole journey is [2010] same speed u. Then, neglecting air resistance [2005]
u +v + ω 3 uvω a. the heavier stone hits the ground with a higher speed
a. b. b. the lighter stone hits the ground with a higher speed
3 u +v + ω
c. both the stone will have the same speed when they
3 uvω
c. d. zero hit the ground
uv + vω + uω d. the speed cannot be determined with the given data

@iitjeehelps
Answer with Solutions
Practice Exercise 5. (c) Average velocity =
Total displacement
Total time
1. (b) The paths of rat and cat are shown in figure. The AC AC
possible points of catching the rat by the cat are A (x1, y1) v av = ∴ | v av | =
t t
and B (x 2, y 2 ). From figure,
y
Path
of cat
C
A
(x1, y1) 30°
45° 45°
x′ x a
O 120°
30°
B(x2, y2) A a B

The time taken by particle to reach from A to C is


Path AB + BC 2a
y′ of rat t= =
v v
OA = OB = radius of circle = 2 unit a 2 + a 2 − AC 2 1 2a 2 AC 2
In ∆ABC, cos 120° = 2
or − = −
2 2a 2 2a 2 2a 2
∴ x1 = − OA sin 45° = − =− 2
2 3a 3v
∴ AC = 3a ∴ | v av | = =
and y1 = OA sin 45° = 2 2a 2
v
Similarly for point B, x 2 = 2, y 2 = − 2
6. (c) 7. (c)
Alternate 8. (c) From figure of v sin θ v sin θ
Let the catching point of rat by cat is P (x1, y1). problem, horizontal
v v
The coordinates of point P satisfy the both equations. components of
∴ x1 + y1 = 0 velocities vanish.
Hence, net velocity θ θ
∴ y1 = − x1 …(i)
is v ′ = 2v sin θ v cos θ v cos θ
For rate, x12 + y12 = 4 dx
9. (b) Here, =u
or x12 + ( − x1)2 = 4 dt
∴ 2x12 = 4 From figure, l 2 + y 2 = x 2
∴ x1 = ± 2 l

The corresponding values of y1 = m 2


x y
∴ The possible coordinates of point P are ( 2, − 2 ) and
u
( − 2, 2 ). u θ
m/s
2. (d) Speed is too velocity = = No unit
m/s
dy dx dy x u  y 
m 2y = 2x or = u= Q = cos θ
Similarly, distance is too displacement = = No unit dt dt dt y cos θ  x 
m
10. (c) The total length of string is
3. (a)
Total distance d 2 
4. (a) Average speed = l = 2x + 2  + y 2 + 2x ...(i)
Total time  4 
t t
Here, s 1 = 80 × and s 2 = 40 × d
2 2
s 1 = 40 t and s 2 = 20 t y

Total distance = s 1 + s 2 = 40t + 20t θ θ


4 +y2
d +

x x
2

x
d2
4

t t
∴ Total time = + = t
y
2

2 2
40t + 20t
∴ Average speed = = 60 km/h
t v v

@iitjeehelps
38 SELF STUDY GUIDE BITSAT

Differentiating of Eq. (i) on both sides with respect to t, PB O


In ∆OPB, cos α = α α
dx 2y dy r
0=4 +
dt d 2 dt ∴ PB = r cos α r r
+ y2 l
4 or = r cos α α α
dy dy 2v 2
P B A
or 4v + 2 cos θ =0 or =− ∴ l = 2r cos α
dt dt cos θ
1
11. (c) The situation of problem is shown vB But, l = a 0t 2
2
in figure. Let the length of link AB is l.
From figure,  2l   2 × 2r cos α 
∴ t=   =  
x A2 + xB2 = l 2  a0   a0 

dx A dx  4r cos α 
or 2x A + 2xB B = 0 l xB =  
dt dt  a0 
 dx A  18. (e) Let the particle returns back after time t = t 0.
or − x Av A + xBvB = 0 Q = − v A
 dt 
60° For this, v = 0, at t = t 0
because x A decreases with respect to
xA Q v = u + at or 0 = 2 − 0.1t 0
time.
2  1 2

x v
vB = A A = v A cot 60° ∴ t0 = = 20 s Qs = ut + at 
0.1  2 
xB
1
v 3 or 15 = 2t − 0.1t 2 or 30 = 4t − 0.1t 2
= A = = 3 m/s 2
3 3
or 300 = 40t − t 2 or t 2 − 40t + 300 = 0
12. (a) The velocity of intersection point is
40 ± 1600 − 1200 40 ± 20
4 m/s ∴ t= =
2 ×1 2
3 m/s ∴ t = 30s or 10 s
19. (a)
20. (a) Given, x = 400 m, y = 300 m, z = 1200 m

v = v x2 + v y2 = 32 + 42 = 5 m/s ∴ r = x 2 + y 2 + z 2 = magnitude of displacement

13. (d) The slope of displacement-time graph gives velocity. = ( 400)2 + ( 300)2 + (1200)2
At the point E in the given graph slope is negative. Hence, = 102 16 + 9 + 144 = 1300 m
instantaneous velocity will be negative corresponding to
point E. 21. (b) Here, x = (t − 2)2
14. (b) The slope of x-t graph gives velocity. The tangents at dx
Velocityv = = 2 (t − 2) m/s
points A and B on the given graph are parallel to X-axis. dt
So, the slopes of the graph at points A and B are zero. dv
Hence, velocities at A and B are zero. Acceleration, a = = 2 ms−2 (i.e. uniform)
dt
15. (b) Speed never be negative. Hence, (b) is correct.
4 m/s C
16. (a) The situation is shown in figure.
v
v
B D
O t
2r 2r=2a 2s 4s

v
v –4 m/s
The relative angular velocity is
A
The relative velocity of component 
of velocity in perpendicular direction  When t = 0, v = − 4 m/s
  t = 2 s, v = 0
 of line joining the particles 
ω rel = 
2v v
= = t = 4 s, v = 4 m/s
The shortest distance between particles 2a a
Velocity (v ) -time (t) graph of this motion is as shown in
17. (c) Let the particle touches the sphere at the point A. figure.
Let PA = l Distance travelled = area AOB + area BCD
l 4×2 4×2
∴ PB = = + = 8m
2 2 2

@iitjeehelps
MOTION IN 1, 2 & 3 DIMENSIONS AND PROJECTILE MOTION 39
22. (d) Let the walking distance is l. 26. (a) The slope of distance-time graph gives speed.
l The change in velocity in one second
∴ v1 = velocity of person =
90 = tan 60° − tan 45° = 3 − 1
l
v 2 = the velocity of escalator = ∆v 3 −1
60 ∴ Acceleration = = = ( 3 − 1) unit
∆t 1
∴The resulting velocity of person on moving escalator is
27. (d) The slope of velocity-time graph gives acceleration.
v = v1 + v 2
Since, the given graph is straight line slope of graph is
l l
∴ t= = constant. Hence, acceleration is constant.
v1 + v 2 l
+
l
The area of given v-t graph between 0 to 10 s is same as
60 90 between 10 s to 20 s.
1 180
t= = = 36 s 28. (b) The area of a-t graph gives change in velocity. The
3+ 2 5
area of the graph from 0 to 8 s
180
1
1 2 = v − u = × 4 × 5 + 4 × 5 = 30
23. (a) Here,sn = an 2
2
But, u = 0 ∴ v = 30 m/s
snth = distance travelled in n second − distance travelled in 1
(n − 1) second 29. (b) As, x1(t ) = at 2 and x 2(t ) = vt
2
 2n − 1 s 2n − 1 2 1
snth =  a ∴ nth = = − 2 1
 2  sn n2 n n ∴ x1 − x 2 = at 2 − vt [parabola]
2
24. (b) According to problem, whens = a, t = p Clearly, graph (b ) represents it correctly.
1
∴ s = ut + ft 2 [Here, f = acceleration] 30. (d)Qx = (t − 2)2(t − 5)
2
dx d d
or a = up + fp 2 v = = (t − 5) (t − 22 ) + (t − 2)2 (t − 5)
dt dt dt
fp 2 or v = (t − 5)2(t − 2) + (t − 2)2= 2(t − 5)(t − 2) + (t − 2)2
or a = up + …(i)
2
At t = 2 s, v = 0
For s = b, t = q
At t = 2 s, x = 0
fq 2 dv d
b = uq + …(ii) a= = { 2(t − 5) (t − 2) + (t − 2)2}
2 dt dt
2(aq − bp )
After solving Eqs. (i) and (ii), we get f = = 2(t − 5) + 2(t − 2) + 2(t − 2)
pq( p − q ) = 2t − 10 + 2t − 4 + 2t − 4 = 6t − 18
x2 v2 a + 18
25. (a) The corresponding equation is + =1 When t < 3, then < 3 or a < 0
A 2 v 02 6
Hence, at t < 3, acceleration a will be negative.
31. (b)Q v = | t − 2| m/s
v0
v = t − 2 when t > 2 s
A
v = 2 − t, when t < 2 s
dv
∴ a= = 1m/s2 t > 2 s
2 2
dt
v x
or = 1− 2 …(i) a = − 1m/s2 t < 2
v 02 A
a1=1 m/s2 a2=1 m/s2
Differentiating of Eq. (i) on both sides with respect to t, C
2v dv 2x dx
2
=− 2 A t=2 s B
v 0 dt A dt
2v dv 2x v 02 In the direction of motion from A to C, bee decelerates but
or = − v or a = − x from C to B, bee accelerates.
v 02 dt A2 A2
Let AC = s 1 CB = s 2
v 02
Q a max = − x max u A = 2 m/s at t = 0, uC = 0 at t = 2 s, uB = 2 m/s at t = 4 s
A2
v2  u + uC   uC + uB 
= − 02 A [Qx max = A] Q s1 =  A  t1, s 2 =   t2
Q a max  2   2 
A
v2  2 + 0  0 + 2
∴ s =s1 + s 2 =   2+   2 = 4m
= 0 [In the sense of magnitude]  2   2 
A

@iitjeehelps
40 SELF STUDY GUIDE BITSAT

dx  dx  a2 = The actual acceleration of box = − g sin αi$


32. (b) As, v = v 0 + gt + ft 2 or = v 0 + gt + gt 2 Qv = 
dt  dt 
∴ s = − l $j, u = 0
rel rel
⇒ dx = (v 0 + gt + ft 2 )dt 1
x 1 g f s rel = urelt + arelt 2
So, ∫ dx = ∫ (v 0 + gt + ft )dt 2
⇒ x =v0 + + 2
0 0 2 3 1 gt 2 cos α
or − l $j = 0 + ( a1 − a2 ) t 2 ∴ l=
33. (a) Given,v = α x 2 2
dx dx  2l 
or =α x or = α dt ∴ t= 
dt x 
 g cos α 
x dx t
On integrating ∫ = ∫ α dt
0 x 0 38. (c)
[∴ at t = 0, x = 0 and let at any time t, particle is at x] 39. (a) The meaning of terminal velocity is constant velocity.
1/ 2 x For constant velocity, acceleration should be zero
x α α 2
⇒ = α t or x 1/ 2 = t or x = × t 2 or x ∝ t 2 (i.e. a = 0)
1/ 2 0 2 4
Q a = 3 − 2v or 0 = 3 − 2v
34. (b) a = 3 sin 4 t 3
∴ v = = 1. 5 unit
As the acceleration is a sin function, hence it is a periodic 2
function and the acceleration of the particle becomes zero 40. (d) Here, v ∝ x n
π   π 
after each interval of seconds a = 3 sin  4 ×  = 0 dx
4   4   or = kx n
dt
35. (d) The given expression is x = 3t 2 − t 3
d 2x dx
dx dv ∴ = knx n − 1
v = = 6t − 3t 2, a = = 6 − 6t dt 2 dt
dt dt
d 2x n −1 n d 2x
At t = 0, a = 6 m/s2 or = knx kx or = k 2nx 2n − 1
dt 2 dt 2
(c) For maximum of x-t curve, Since, at a = 0, v = 0
dx
=0 It means for moving the body, acceleration should be
dt positive. For this
∴ 6t − 3t 2 = 0 ⇒ t = 2 s 1
∴ 2n − 1 > 0 ⇒ n >
Hence, (c) is correct. 2
Hence, finally (d) is correct. 1
If n < , particle remains in rest.
dv 2
36. (b)Q = a − bv
dt 41. (a) At the maximum height,v = 0
v t
dv 1 ∴ v = u − gt ⇒ 0 = u − gt 0
or ∫ = ∫ dt or − [log (a − bv )]v0 = t
a − bv b u 40
0 0
∴ t0 = = =4s
 a − bv  − bt b g 10
or   =e or 1 − v = e −bt
 a  a Q v = u − gt = 40 − 10( 4 − 2) = 20 m/s
a −bt
∴ v = (1 − e ) 42. (b) v=0
b
Tricky approach According to homogeneity principle, the
dimensions of left hand side should be equal to
dimensions of right hand side. Options (a), (c) and (d) are
u 20 m
dimensionally wrong. But, (b) is dimensionally correct. t=
g
37. (a) The actual acceleration of bolt =2s
15 m
= a = − g sin α$i − g cos α$j
1
y
x Ground
Position of balls
1 2 1
s rel

h1 = gt = × 10 × 12 = 5 m
2 2
a0=g sinα 1 2 1
y′ h2 = gt = × 10 × 22 = 20 m
α 2 2
From ground, 15 m, 20 m, 15 m (shown in figure).
x′

@iitjeehelps
MOTION IN 1, 2 & 3 DIMENSIONS AND PROJECTILE MOTION 41
43. (a) Let the time taken by ball to reach at maximum height 54. (b)
is t 0. 55. (d) The trajectory is shown in figure. From figure, at the
At maximum height, v = 0 top, velocity and gravitational acceleration are mutually
(u + v ) u
Here, h = t or 5 = t 0 perpendicular to each other.
2 2
10 P
∴ t0 = …(i) v
u
g
But for upward motion, v 2 = u 2 − 2gh
or ( 0)2 = u 2 − 2 × 10 × 5
∴ u = 2 × 10 × 5 = 10 m/s u 2 sin2 θ
56. (d) The maximum height is H =
2g
10
∴ t0 = = 1s u 2 sin2 45°
10 or 16 =
t 1 min 60 s 19.6
∴ Number of balls per minute = = = = 60
t0 1s 1s ∴ u = 16 × 2 × 19.6

44. (b) u 2 sin 2θ


But, R=
45. (a) For the given condition, initial height h = d and velocity g
16 × 2 × 19.6 sin 90°
of the ball is zero. When the ball moves downwards its = = 64 m
velocity increases and it will be maximum when the ball 9.8
hits the ground and just after the collision it becomes half When θ = 45°, sin 2θ = 1
and in opposite direction. As the ball moves upward, its But, when θ = 30°,
d
velocity again decreases and becomes zero at height . 3
2 sin 2θ = = 0.866
This explanation match with graph (a). 2
So, at θ = 30°, range is lesser than that of at θ = 45°
46. (a)
57. (d) When the stone reaches at ground. The potential
47. (c) In this case, the motion of balloon is undergravity. So, energy of stone decreases. This decrease in potential
acceleration is constant. Hence, kinematic equation energy of stone. Hence, speed increases. Hence, (d) is
1 correct.
s = u t + at 2
2 58. (b) The angular momentum of a particle is given by
is applicable. y L = r × mv
Here, s = − 65$j ∴ L = mvr sin θ
a = g = − 10$j u r
θ
u = 12$j x′ x v cos45°
Initial O r
1
∴ − 65$j = 12t − 10t 2$j position θ
h
2 of balloon
O
or − 65 = 12t − 5t 2
y′ Ground mv
∴ t =5s From figure, L = rm(v cos 45° ) sin θ = (r sin θ )
2
Remarks In this case,
mvh  h
magnitude of displacement is not equal to distance. = Q sin θ = 
2  r
48. (a) According to kinematic equation,
v = v 0 − gt 59. (b) In the absence of air,
u 2u
Upward direction is taken as positive and downward t = time of flight = 2 × time of ascent = 2 × =
direction is taken as negative. Hence, v-t graph is straight g g
line having negative slope. 2u
∴ t1 = …(i)
49. (b) 50. (b) 51. (d) 52. (d) g1
$ $ 2u
53. (d) Given, velocityv = kY i + kX j and t2 = …(ii)
g2
dX dY
= kY , = kX Now, from Eqs. (i) and (ii), we get
dt dt
dY dY dt kX ∴ t1g1 = t 2g 2
= × =
dX dt dX kY 60. (d) Given,s = t 3 + 5
YdY = X dX ds
∴ Speed, v = = 3t 2
⇒ Y2 =X2 + C (where, C = constant) dt

@iitjeehelps
42 SELF STUDY GUIDE BITSAT

dv 66. (b) Potential energy PE


and rate of change of speed at = = 6t
dt mg(u 2 sin2 θ) mu 2 sin2 θ
= mghmax = =
∴ Tangential acceleration at t = 2 s 2g 2
at = 6 × 2 = 12 ms−2 1
KE = m(u cos θ ) 2

and at t = 2 s, v = 3( 2)2 = 12 ms−1 2


PE
v 2 144 Hence, = tan2 θ
∴ Centripetal acceleration, ac = = ms−2 KE
R 20
67. (a) From review of concepts of the chapter
Net acceleration = a 2c + a 2t = 14 ms−2
2a 2 cos2 α 2 × 102 × cos2 60°
Latusrectum = = = 5m
61. (b) Let a particle is projected at an angle θ with horizontal g 10
(shown in figure)
R/2 3H
y 68. (c) We can write = = 3
H H
(v 20 sin θ cos θ)/g
or = 3
(v 20 sin2 θ )/ 2g
P(x, y)
g u 2  2
2 cot θ = 3 ⇒ tan θ = , θ = tan−1  
θ 3  3
x
69. (d) From the figure, the relative displacement is
At an instant t, the particle reaches at point P (x , y ).
∴ x = (u cos θ )t 800 m
∴ ut cos θ = x …(i) 7.5 m/s
1
and y = (uyt ) − gt 2 20 m/s
2
200 m
1
∴ y = (u sin θ )t − gt 2
2
1 s rel = ( 800 + 200) m = 1000 m
∴ ut sin θ = y + gt 2 …(ii) v rel = v1 − v 2 = ( 20 − 7.5) m/s = 12.5 m/s
2
s 1000
Squaring Eqs. (i) and (ii) and then adding, we get ∴ t = rel = = 80 s
2 v rel 12.5
 1 
(ut sin θ )2 + (ut cos θ )2 = x 2 + y + gt 2
 2  70. (b) Let the distance between spots A and B is l.
2 4
g t u = speed of river stream
∴ u 2t 2 = x 2 + y 2 + + gt 2y v = speed of motorboat in still water
4
During downstream,
g 2t 4
x2 + y2 + + gt 2y − u 2t 2 = 0 l
4 t1 = …(i)
v +u
This is an equation of circle.
Similarly in upstream
62. (a) The rate of storing water in vessel depends upon l
vertical component of velocity. In this problem, vertical t2 = …(ii)
component of velocity in both cases are same. v −u
l
u 2 sin 2θ In still water, t= …(iii)
63. (c) As range = 10 = ⇒ u 2 = 10g v
g
After solving Eqs. (i), (ii) and (iii), we get t = 9.6 h
v
71. (d) For the man to cross the river from the shortest path,
45º
Wall Vm sin θ = vr
v
4m 6m sin θ = r
vm
∴ u = 10 m/ s ( as g 10 m/s ) 2
4
sin θ =
1 gx 2 1 g × 16 3
Y = x tan θ − = 4 tan 45° −
2 2 v 0 cos θ
2 2
2 2 v 0 cos2 45°
2
 4
θ = sin−1 
1 10 × 16  3
= 4 × 1− = 4 − 0.8 = 3.2 m
2 2 × 10 × 10 × 1 But it is not possible, because sin θ could not be greater
2 than 1. Hence, the man could not cross the river from the
64. (b) 65. (d) shortest path AB.

@iitjeehelps
MOTION IN 1, 2 & 3 DIMENSIONS AND PROJECTILE MOTION 43
72. (b) Let vt = actual velocity of train Time taken by the man to cross the river,
Width of the river 1km 1 1
Y N t= = = h = × 60 = 15 min
Speed of the man 4 km / h 4 4
X′ X Distance travelled along the river = vr × t
W E 1 3 3000
= 3 × = km = = 750 m
4 4 4
Y′ S
20
vc = actual velocity of car 74. (a) tan (90° − θ ) =
15
∴ vtc = relative velocity of train with respect to car 20 4
∴ vc = 25$i and vtc = 25 3 $j ∴ cot θ = =
15 3
Q vtc = vt − vc ⇒ θ = 37°
∴ vt = vtc + vc = 25 3 $j + 25$j ∴ α = 37° + 23° = 60°
Q vt = ( 25 3 )2 + ( 25)2 = 25 4 = 50 km/h 75. (c) Let arel = acceleration of ball with respect to ground −
acceleration of bus with respect to ground
73. (d) Given, speed of manvm = 4 km/h
y
B vr C
a
1 km v Motion of bus
vm β g

A
= − g$j − a$i
Speed of river vr = 3 km/h
Width of the river, d = 1 km ∴ | arel| = g 2 + a 2

BITSAT Archives
1. (b) (i)v = u + at1 So, 70 = − 25 × t +
1
× 10 × t 2
2
40 = 0 + a × 20
or 5t 2 − 25t − 70 = 0 or t 2 − 5t − 14 = 0
a = 2 m / s2
On solving = 7 s
v 2 − u 2 = 2 as
1 h 1
402 − 0 = 2 × 2s 1 3. (c) h = − vt1 + gt12 or = − v + gt1 …(i)
2 t1 2
∴ s 1 = 400 m 1 −h 1
(ii) s 2 = v × t 2 = 40 × 20 = 800 m h = vt 2 + gt 22 or = − v + gt 2 …(ii)
2 t2 2
(iii) v = u + at h h 1
0 = 40 + a × 40 ∴ + = g (t1 + t 2 )
t1 t 2 2
∴ a = − 1m/s2 1
or h = gt1t 2
02 − 402 = 2( − 1)s 3 2
∴ s 3 = 800 m For faling under gravity from the top of the tower
Total distance travelled = s 1 + s 2 + s 3 1
h = gt 2
= 400 + 800 + 800 = 2000 m 2
1 1
Total time taken = 20 + 20 + 40 = 80 s ∴ gt1t 2 = gt 2 ⇒ t = t1t 2
2000 2 2
∴ Average velocity =
80 4. (a) Let us draw the figure for given situation,
= 25 m/s 6
1 at2
2
2. (c) Taking vertical downward motion of projectile from B M/B
point of projection to ground, we have M
u = − 50 sin 30° = − 25 m/s 4t
a = + 10 m/s2, s = 70 m, t = ? 1
⇒ 4t = 6 + × 12. × t2
1 2
s = ut + at 2
2 ⇒ 4t = 6 + 0.6 t 2

@iitjeehelps
44 SELF STUDY GUIDE BITSAT

2 u sin α 9. (b) As x increases, time first increases and then


5. (a) t1 =
g decreases. This is not possible.
2 u sin(90° − α )
t2 = 10. (c) Direction of velocity is always tangent to the path so at
g the top of trajectory, it is in horizontal direction.
u2 11. (b) Total displacement of the object = Area (DABCE)
So, t1 × t 2 = 2 2 sin 2α
g = Area of ∆ ABC + Area of rectangle (ACED)
2R  u 2 sin 2α 
or t1 × t 2 = Q R = 
g  g 
t1 × t 2 ∝ R 4
B

6. (c) An object is said to be moving with a uniform 3


acceleration, if its velocity changes by equal amount in 2
v(m/s) C
equal intervals of time. The velocity-time graph of 1
uniformly accelerated motion is a straight line inclined to A
time axis. Acceleration of an object in a uniformly
accelerated motion in one dimension is equal to the slope 0
10 20 30 40 50 60
of the velocity-time graph with time axis. D E
t(s)
7. (d) When stone travels horizontally, then it must be at the
maximum height.
u sin θ 1 
=  × ( 40 − 20) × ( 4 − 1) + [( 40 − 20) × 1]
So, 2= 2 
g
1
⇒ u sin θ = 20 …(i) = × 20 × 3 + 20
u sin θ − g (1) 2
Also, tan 45° = = 30 + 20
u cos θ
= 50 m
⇒ u cos θ = 20 − 10
12. (a) Direction of velocity is always tangent to the path, so
⇒ u cos θ = 10 …(ii)
at the top of trajectory it is in horizontal direction and
Squaring of Eqs. (i) and (ii) and adding, we get acceleration due to gravity is always in vertically
u = 10 5 ms−1 and tan θ = 2 downward direction.
⇒ θ = tan−1( 2) Hence, v and g are perpendicular to each other.
Total distance 13. (c) At the top most point of the projectile there is only
8. (c) Avery velocityv av =
Total time horizontal component of velocity and acceleration due to
s +s +s 3 uv ω gravity is vertically downward, so velocity and
⇒ v av = ⇒ v av = acceleration are perpendicular to each other.
s s s
+ + uv + vω + uω
u v ω 14. (c)

@iitjeehelps
4
Newton’s Laws of
Motion and Friction
Force
Force is an external effort in the form of push or pull which
(i) generates or tends to generate motion in a body at rest,
(ii) stops or tends to stop a body in motion,
(iii) increases or decreases the magnitude of velocity of the moving body,
(iv) changes the direction of motion of the body.

Classification of Forces
Based on the nature of interaction between two bodies, forces may be broadly classified as under
(i) Field forces are those forces that act between two bodies separated by a distance without any
actual contact. Gravitational force between two bodies and electrostatic force between two
charges are the examples of field forces.
(ii) Contact forces are those forces that act between two bodies in contact.
e. g. tension, normal reaction, friction, etc.

Inertia
It is an inherent property of all bodies, by virtue of which they cannot change by themselves their state
of rest or of uniform motion along a straight line.
As inertia of a body is measured by the mass of the body. So, heavier the body, greater is the force
required to change its state.

@iitjeehelps
46 SELF STUDY GUIDE BITSAT

Impulse of a force is a measure of the total effect of force.


Newton’s Laws of Motion

Dt
Mathematically, impulse J = Fav dt = ò F dt
0
There are three laws of motion stated by Newton. These
laws describe the relationship between a body and the ● For a force-time (F-t) graph, the area under the graph
forces acting upon it and its motion in response to said dp
gives the value of the impulse. As, F = .
forces. Newton’s laws are not valid in the non-inertial dt
Dt
frames, they have to be modified by introducing the
concept of pseudo force.
● Hence, impulse J= ò0 F dt
Dt dp p
= ò0dt
dt = ò f dp = p f - pi . This relation is known as
pi
First Law or Law of Inertia the impulse-momentum theorem.
If the net force SF exerted on an object is zero, then the
object continues in its original state of motion (or rest). That Third Law
is, if SF = 0, an object at rest remains at rest and or object
If two objects interact, the force F12 which the object 1 exerts
moving with constant velocity. This is Newton’s first law. on object 2 is equal in magnitude but opposite in direction
to the force F21 which the object 2 exerts on object 1. This is
Linear Momentum Newton’s third law, i.e. F12 = - F21
The linear momentum of a body is defined as a product of
mass and velocity of the body, i.e.
p = mv
Conservation of Linear
Momentum is a vector quantity. Momentum
A body at rest cannot possess linear momentum and a According to the principle of conservation of linear
moving body always possesses linear momentum. momentum, if there is no external force acting on a system,
then total momentum of the system remains constant.
Second Law According to second law of motion,
dp
The acceleration of an object is directly proportional to the F=
net force acting on it and is inversely proportional to its dt
mass. If no force is acting, then F = 0.
SF dp
a= So, = 0 Þ p = constant
m dt
This is the Newton’s second law. m1v1 = m2v2 = constant
The above equation in vector form is SF = ma,
where a is the acceleration of the F3 Connected Motion
object, m is its mass and SF represents
The connected motion describes the relative motion
the vector sum of all the forces acting F1
between two objects whether it is between two blocks and a
on the object. plane or it is between a block and an inclined plane.
It also states that the time rate of 1. If two blocks of masses m1 and m2 are placed on a
change of momentum of body is equal F2 perfectly smooth surface and are in contact, then
to the net external force exerted on that B
Concurrent Forces A
body
f f
dp F
i.e. Fnet = m1 m2
dt
If F1 , F2 , F3 ... are the concurrent forces acting at the same F
Acceleration of the blocks, a =
point, then the point will be in equilibrium, if m1 + m2
F1 + F2 + F3 K = 0 and the contact force (acting normally) between the
F m2
two blocks is f = m2 a =
Impulse (m1 + m2 )
● When a force of large magnitude acts on an object for a 2. For a block of mass m placed on a fixed, perfectly
small time interval, the force is called impulsive force. In smooth inclined plane of angle q , the forces acting on
such cases, we measure the total effect of force. the block are shown in the figure.

@iitjeehelps
NEWTON’S LAWS OF MOTION AND FRICTION 47
Obviously, here T
N
F
m
a T2
θ θ T2
in
m gs mg cos θ
θ mg a
m2 B
C m3
Þ a = g sinq
T1 a
3. If a block of mass m is placed on a smooth movable
wedge of mass M , which in turn is placed on smooth
m1 A
surface, then a force F is applied on the wedge,
horizontally.
2m1m3 g
The acceleration of the wedge and the block is Tension, T 1 =
(m1 + m2 + m3 )
N cos θ
2m3 (m1 + m2 ) g
N Tension, T 2 = and
θ (m1 + m2 + m3 )
m
N sin θ
4m3 (m1 + m2 ) g
Tension, T = 2 T 2 =
ma F (m1 + m2 + m3 )
7. For the pulley and block arrangement shown in the
θ mg M figure, we have
F T2 T1
a= M
(M + m)
Force on the block, F = ( M + m )a
T2
= ( M + m ) g tan q T1
a
4. For a block system shown in the figure, acceleration of a
the system m2 (m1 > m2)
F m1
a=
m1 + m2 + m3
Net acceleration
A B C
m2 m3 Net accelerating force
F m1 T1 T2 a=
Total mass
Tension in the string (m1 - m2 ) g
=
F (m3 + m2 ) (m1 + m2 + M )
T 1 = (m2 + m3 )a =
(m1 + m2 + m3 ) ( M + 2m2 )m1 g
Tension, T 1 = m1 ( g - a ) =
F m3 ( M + m1 + m2 )
and tension T 2 = m3a =
m1 + m2 + m3 ( M + 2m1 )m2 g
and Tension, T 2 = m2 ( g + a ) =
5. For a block system suspended freely ( M + m1 + m2 )
from a rigid support as shown in the T1 8. For the system of block and pulley, with a smooth
figure, the acceleration of the system
inclined plane shown in the figure, we have
a = 0. m1 A

Tension in the string, N a


T2 T
T 1 = (m1 + m2 + m3 ) g
T 2 = (m2 + m3 ) g m2 B m2 T
inθ
gs
and T3 = m3 g T3 m2
θ a
6. For a block system and a pulley shown in m2 g cos θ m1
θ m2 g
the figure, value of the acceleration of the m3 C
system
(m1 + m2 - m3 ) g (m1 - m2 sin q ) g
a= Net acceleration, a = ,
(m1 + m2 + m3 ) (m1 + m2 )

@iitjeehelps
48 SELF STUDY GUIDE BITSAT

If m1 g > m2 g sinq
(m2 sin q - m1 ) g Types of Frictions
and a= ,
m1 + m2 Force of friction can be classified into following types
If m1 g < m2 g sin q
and tension in the string Laws of Friction
m1m2 (1 + sin q ) 1. Limiting friction is directly proportional to the normal
T = m1 ( g - a ) =
(m1 + m2 ) reaction ( R), i.e. f s µ R.
2. It acts opposite to the direction of body which is at the
9. For a pulley and block system on a smooth double
verge of moving over the other.
inclined plane as shown in the figure, we have
3. It depends upon the nature of the material and nature
N of the surface in contact.
a
N
m1 m2 Coefficient of Friction (m)
in θ1 a It is defined as the division of the force of limiting friction
gs
m2

m1 ( f s ) and normal reaction (R) between them.


g

m1g cos θ1 m2g cos θ2 θ2


s in

θ1 f
i.e. m= s
θ2

R
Net acceleration So, value of m depends on (a) nature of the surface in contact
(m1 sin q 1 - m2 sin q 2 ) g i.e. dry or wet; rough or smooth, (b) material of the surface
a= ,
(m1 + m2 ) in contact.
for q 1 > q 2 , m1 > m2
and tension in the string Angle of Friction (q)
It is defined as the angle A R
T = m1 ( g sin q 1 - a ) B
m m (sin q 1 + sin q 2 ) g between the normal reaction
= 1 2 (R) and the resultant of the θ
(m1 + m2 )
frictional force ( f s ) and C F
reaction R, as shown in fs O
figure.
Lami’s Theorem In DAOB ,
mg

For three concurrent forces in equilibrium position. AB f m R


If three forces acting at a point be in equilibrium, then each tanq = = s = s or tanq = m s Þ q = tan -1 (m s )
OB R R
force is proportional to the sine of the angle between the i.e. coefficient of limiting friction between any two surfaces
other. Thus, if the forces are P, Q and R; a , b , g be the angles in contact is equal to tangent of the angle of friction between
between Q and R, R and P, P and Q respectively, also the them.
forces are in equilibrium, we have
P
β
R
Angle of Repose (a )
The minimum angle of inclination R
γ α
of a plane with the horizontal, F
such that a body placed on the
α
plane just begins to slide down the
α
Q inclined plane is known as the mg sin mg cos α
mg
P Q R angle of repose or angle of sliding.
= = α
From figure, we have
sin a sin b sin g
mmg sin a = F = Force of limiting friction
mg cosa = R = Force of normal reaction
Friction m mg sin a F
So, =
The opposing force which comes into play when a body mg cosa R
moves or tries to move over the surface of another body, is
F mR
known as friction. Force of friction acts tangentially to the Þ tana = = = m Þ a = tan -1 (m ) or a = q
surface in contact. e. g. A ball rolling over the floor stops R R
after sometime. i.e. Angle of repose = Angle of friction

@iitjeehelps
NEWTON’S LAWS OF MOTION AND FRICTION 49
m 2m2 g = m2a
Some Important Points \ a = m2 g
(i) When F increases, normal reaction shifts from centre (xi) If force is applied on upper block, f 2 = limiting
of mass to right. At the time of toppling, normal friction between m1 and m2 = m 2 m2 g
reaction acts through point A. Also, the net force on f 1 = limiting friction between the surface and
the body gives acceleration of the centre of mass. m1 = m 1 (m1 + m2 ) g
F µ2
m2 F

µ1 m1

(ii) If the body is in rest with respect to the surface, then


fr < ms N . If F > f 1 , then both blocks move with different
(iii) If the body is just in motion, then f r = m s N acceleration and the maximum friction acts between
the blocks. F - f 2 = m2a2
(iv) If the body is in motion, then f r = m k N
(v) If some bodies have same accelerations, then they N2
are taken as a system. If they do not move together,
bodies are not taken as a system.
(vi) The force of friction during pushing is greater than f2 m2 F
that of pulling in the same manner.
(vii) It is misconception to say that friction always
opposes the motion of the body. It only opposes the m2g
relative motion between surfaces.
F - m 2 m2 g = m2a2
mg - T = ma
N 2 = m2 g
(viii) Gravitational force : Electromagnetic force : Strong
force : Weak force = 1 : 1036 : 1038 : 1025 N 1 = N 2 + m1 g = (m1 + m2 ) g

(ix) For the equilibrium of a body on an inclined plane, N1


N2
mg sin q = m smg cos q f2
Þ m s = tan q m1
In this case, 0 < m < 1 f1

m 1g
m

\ f 1 = m 1 N 1 = m 1 (m1 + m2 ) g
f 2 = m 2 m2 g
θ
If f 2 < f 1 , then m1 remains in rest.
(x) The force acting on m2 is f 2 = m 2m2 g .
If f 2 > f 1 , then m1 moves in the direction of f 2 .
If the system moves with the common acceleration,
then f 2 - f 1 = m1a1
If F < f 2 , then no relation is found between m1 and m2 .
µ2
m2 i.e. m1 and m2 move together.
µ1 If F < f 1 , then the system is in rest.
m1 F
If F > f 1 , the system moves with the common
acceleration a. In this case,
F - m 1 (m1 + m2 ) g = (m1 + m2 )a F - f 1 = (m1 + m2 )a
f 2 = m2a or F - m 1 (m1 + m2 ) g = (m1 + m2 )a

@iitjeehelps
Practice Exercise
1. Two bodies have same mass and speed, then a. a cannot remain positive for all t in the interval
a. their momentums are same 0 £t £1
b. the ratio of momentums is not determined b. | a | cannot exceed 2 at any point in its path
c. the ratio of their magnitudes of momentum is one c. | a | be ³ 4 at point or some points in its path
d. Both a and b are correct d. Both a and c are correct
2. In the superhit film ‘Raja Hindustani’ Amir khan greets 8. A 0.1 kg body moves at a constant speed of 10 m/s. It
his beloved by shaking hand, What kind of force do is pushed by applying a constant force for 2 s. Due to
they exert? this force, it starts moving exactly in the opposite
a. Nuclear direction with a speed of 4 m/s. Then,
b. Gravitational a. the deceleration of the body is 7 m/s2
c. Weak
b. the magnitude of change in momentum is 1.4 kg-m/s
d. Electromagnetic
c. impulse of the force is 1.4 N-s
3. A heavy block of mass m is supported by a C d. the force which acts on the ball is 0.7 N
cord C attached to the ceiling, and another e. All of the above
cord D is attached to the bottom of the 9. Water jet issues water from a nozzle of 2 cm 2
block. If a sudden jerk is given to D, then m cross-section with velocity 30 cm/s and strikes a plane
a. cord C breaks surface placed at right angles to the jet. The force
b. cord D breaks exerted on the plane is
c. cord C and D both break
D
a. 200 dyne
d. None of the cords breaks
b. 400 dyne
4. At time t second, a particle of mass 3 kg has position c. 1800 dyne
vector r metre, where r = 3t $i - 4 cos t $j. Find the d. None of the above
p 10. The action and reaction forces referred to Newton’s
impulse of the force during the time interval 0 £ t £
2 third law of motion
a. 12$j N- s b. 9 $j N- s c. 4$j N- s d. 14$j N- s a. must act upon the same bodies
b. must act upon different bodies
5. Three blocks of masses m1 = 1kg, m 2 = 2 kg and c. need not to be equal in magnitude but must have the
m 3 = 3 kg are placed in contact on a horizontal same line of action
frictionless plane as shown m3
d. must be equal in magnitude but need not have the
in figure. A force of 12 N is m2
same line of action
applied on m1. Acceleration m1 11. A man is pulling a rope attached to a block on a
of the system is F smooth horizontal table. The tension in the rope will
be the same at all points
a. 12 m/s 2 b. 2 m/s 2 c. 6 m/s 2 d. 4 m/s 2 a. if and only if the rope is not accelerated
b. if and only if the rope is massless
6. A particle is acted upon by a force of constant c. if either the rope is not accelerated or is massless
magnitude which is always perpendicular to the d. always
velocity of the particle. The motion of the particle takes
place in a plane. It follows that
12. A particle of mass m moves on the x-axis under the
influence of a force of attraction towards the origin
a. its velocity is constant k
b. its kinetic energy is constant O given by F = - 2 $i. If the particle starts from rest at
c. it moves in a circular path x
d. Both b and c are correct x = a . The speed of it will attain to reach at distance x
from the origin O will be
7. A particle of mass m moves on the x-axis as follows. It 1/ 2 1/ 2
starts from rest at t = 0 from the point x = 0 and comes 2k é x - a ù 2k é a + x ù
a. b.
to rest at t = 1at the point x = 1. No other information is m êë ax úû m êë ax úû
available about its motion at intermediate time 1/ 2
( 0 < t < 1). If a denotes the instantaneous acceleration k é ax ù m éa - x ù
c. ê ú d.
of the particle, then m ëx - a û 2k êë ax úû

@iitjeehelps
NEWTON’S LAWS OF MOTION AND FRICTION 51
13. A particle is on a smooth horizontal plane. A force F is Then, at the equilibrium of the system, the value of q is
applied whose F-t graph is given. Then,

F
F θ
t1 t2 A m1
t m B
a. at t1, acceleration is constant
b. initially body must be in rest a. 30° b. 60°
c. at t 2, acceleration is constant c. 90° d. 0°
d. initially acceleration is zero 20. A body of mass 10 kg is to be raised by a massless
e. both c and d are correct
string from rest to rest, through a height 9.8 m. The
14. A force F is applied to the initially, stationary greatest tension which the string can safely bear is
cart. The variation of force with' time is shown in the 20 kg-wt. The least time of ascent is
figure. The speed of cart at t = 5 s is a. 1 s
b. 3 s
c. 4 s
Parabolic d. None of the above
10kg F 21. Which of the following expression correctly
5 t(s) represents T1 and T2 if the system is given an
upward acceleration by a pulling up mass A?
a. 10 m/s b. 8.33 m/s c. 2 m/s d. zero
m T1
15. The mass m is placed on a body of
mass M . There is no friction. The a
M F
force F is applied on M and it moves A MA
with acceleration a. Then, the force
on the top body is
T2
a. F b. ma
c. F - ma d. None of these
B MB
16. Three identical blocks each of mass M are along
a frictionless table and a force F is acting as
shown. Which of the following a. T1 = M A (a - g ) + MB (a - g ), T2 = MB (a - g )
F A B C
statements is false? b. T1 = M A (g - a ) + MB (g - a ), T2 = MB (g - a )
a. The net vertical force on block c. T1 = M A (g + a ) + MB (g + a ), T2 = MB (g + a )
A is zero d. T1 = M A (g + a ),T2 = MB (g + a )
F
b. The net force on block A is
3 22. A chain consisting of 5 links
F 1
c. The acceleration of block C is each of mass 0.1 kg is lifted F
3M vertically with a constant
2F 2
d. The force of interaction between A and B is acceleration of 2.5 m/ s 2 as
3 shown in the figure. The 3
17. A 40 N block is supported by two ropes. One rope is force of interaction between
4
horizontal and the other makes an angle of 30° with the top link and the link
the ceiling. The tension in the rope attached to the immediately below it, will be 5
ceiling is approximately a. 6.15 N
a. 80 N b. 40 N c. 34.6 N d. 46.2 N b. 4.92 N
c. 3.69 N
18. A weight w is suspended from the midpoint of a rope, d. 2046 N
whose ends are at the same level. In order to make
the rope perfectly horizontal, the force applied to each 23. In the given figure,
of its ends must be a. acceleration of m1 and m2 are same
a. less than w b. equal to w b. the magnitude of relative acceleration
c. equal to 2w d. infinitely large of m1 with respect to m2 is twice the
magnitude of acceleration of m1 m2
19. A ring of mass 5 kg sliding on a frictionless vertical rod
c. the velocities of m1 and m2 are same m1
connected by a block B of mass 10 kg by the help of a
massless string. d. the speed of m1 and m2 are not same

@iitjeehelps
52 SELF STUDY GUIDE BITSAT

24. The actual acceleration of body A is a. Then, 28. In the given figure,
B

A
a
v m m
a. the acceleration of B is a B A
b. the acceleration of B is 2 a a. both masses always remain in same level
c. the magnitude of relative acceleration of B with b. after sometime, A is lower than B
respect to A is 2a c. after sometime, B is lower than A
d. the momentum of A may be equal to that of B d. no sufficient information
25. In the given ideal pulley system, 29. Observer O1 is in a lift a0
going upwards and O2
is on the ground. Both
B apply Newton's law
and measure normal
reaction on the body O1 O2
A a. both measure the
same value
C b. both measure zero
c. both measure different value
m2 d. no sufficient data
m1
30. A particle is found to be at rest when seen from frame
a. tension in string is zero S1 and moving with a constant velocity when seen
b. pulleys B and C rotate counter anti-clockwise and the from another frame S 2.
pulley A clockwise Mark the possible points from the following.
c. A and B are same and is equal to g
a. Both the frames are inertial
d. All of the above b. Both the frames are non-inertial
26. Pulleys and strings are massless. The horizontal surface c. S1 is non-inertial and S 2 is inertial
is smooth. The acceleration of the block A is d. Both a and b are correct
A
31. A block of mass 10 kg is suspended through two light
m springs which are balanced as shown in the figure.
Then,

F
F F F
a. b. c. d. zero
m 2m 4m
27. For the system shown in the figure, the pulleys are
light and frictionless. The tension in the string will be
10kg

a. both the scales will read 10 kg


b. both the scales will read 5 kg
m c. the upper scale will read 10 kg and the lower zero
d. the readings may be of any value but their sum will
θ m be 10 kg

2 3 32. The normal reaction on a body placed in a lift moving


a. mg sin q b. mg sin q up with constant acceleration 2 m/ s 2 is 120 N. Mass
3 2
1 of body is (Take, g = 10 m/ s 2)
c. mg sin q d. 2mg sin q
2 a. 10 kg b. 15 kg c. 12 kg d. 5 kg

@iitjeehelps
NEWTON’S LAWS OF MOTION AND FRICTION 53
33. A block of mass m is moving on a wedge with the b. in the forward direction on the front wheel and in the
acceleration a 0. The wedge is moving with the backward direction on rear wheel
acceleration a1. The magnitude of pseudo force on the c. in the backward direction on both the front and the
block is rear wheels
æ a + a0 ö d. in the forward direction on both the front and the rear
a. ma 0 b. ma1 c. m a 02 + a12 d. m ç 1 ÷ wheels
è 2 ø
40. If a body of mass m is moving on a rough horizontal
34. A point mass m is moving surface of coefficient of kinetic friction m, the net
m
along inclined plane with electromagnetic force exerted by surface on the
acceleration a with respect to body is
smooth triangular block. The
triangular block is moving θ a0 a. mg 1 + m 2 b. mmg c. mg d. mg 1 - m 2
horizontally with acceleration 41. A block is placed on a rough floor and a horizontal
a 0. The value of a is force F is applied on it. The force of friction by the floor
a. g sin q + a 0 cos q b. g sin q - a 0 cos q on the block is measured for different values of F and
c. g cos q - a 0 sin q d. g cos q - a 0 tan q
a graph is plotted between them
a. the graph is a straight line of slope 45°
35. A uniform fine chain of length l is suspended with b. the graph is a straight line parallel to the F -axis
lower end just touching a horizontal table. Find the c. the graph is a straight line of slope 45° for small F
pressure on the table, when a length x has reached and a straight line parallel to the F -axis for large F
the table. d. there is a small kink on the graph
mgx 42. When a body is in rest in the condition of a horizontal
a. mgx b. 2mgx c. 3mgx d.
2 applied force. Then, the slope of force and friction
36. A uniform chain is coiled up on a horizontal plane and from graph is
one end passes over a small light pulley at a height ‘a a. 1 b. m c. 0 d. -1
’ above the plane. Initially, a length ‘b’ hangs freely on
the other side. If b = 2a , then 43. Look at the situation, when the body is in air and is
moving with pure translation. This situation is shown
a. the end descends with a constant accelerationg /3
in the figure. What happens, when the body hits the
b. the end descends with acceleration depends
upon hanging position
surface?
c. acceleration cannot be determined m
d. acceleration is variable v
m A
37. A mass m is placed over a spring of
spring constant k, the acceleration of
mass at the lowest position is Frictional surface
a. g in rest
b. zero a. Sliding friction will act in the backward direction
æ kx ö
c. ç - g ÷ , where x is compression in spring b. The velocity of the point of contact gradually
èm ø
decreases
d. None of the above
c. The sliding friction acts in such a way so as to try to
38. In the figure, the ball A is released make the point of contact velocity of the body same
from rest when the spring is at its as that of the surface
natural length. For the block B of d. Both a and b are correct
mass M to leave contact with the
44. Let F, FN and f denote the magnitudes of the contact
ground at some stage, the minimum m A
mass of A must be force, normal force and the frictional force exerted by
one surface on the other kept in contact. If one of
a. 2M
these is zero, then
b. M B M
M a. F > FN b. F > f
c.
2 c. FN - f < FN + f d. All of these
d. a function of M and the force constant of the spring
45. A car starts from rest to cover a distance s. The
39. A bicycle is in motion. When it is not pedaled, the coefficient of friction between the road and the tyres is
force of friction exerted by the ground on the two m. The minimum time in which the car can cover the
wheels is such that it acts distance is proportional to
a. in the backward direction on the front wheel and in 1 1
the forward direction on the rear wheel a. m b. m c. d.
m m

@iitjeehelps
54 SELF STUDY GUIDE BITSAT

46. A block of mass 1 kg is placed on a wedge shown in 51. A piece of ice slides down a rough inclined plane at
figure. Find out minimum coefficient of friction 45° inclination in twice the time that it takes to slide
between wedge and block to stop the block on it. down an identical but frictionless inclined plane. What
is the coefficient of friction between ice and incline?
3 4 3 7
Block a. b. c. d.
7 cot q 7 cot q 4 cot q 9 cot q
µ Fixed
wedge 52. Consider the situation shown in the figure. The wall is
θ = 45º
smooth but the surfaces of A and B in contact are
rough. Then,
a. 0.6 b. 0.9 c. 1 d. 0.2
47. A body of mass 2 kg is placed on rough horizontal
plane. The coefficient of friction between body and A B F
plane is 0.2. Then,

F
a. system may remain in equilibrium
b. both bodies must move together
µ=0.2 c. the system cannot remain in equilibrium
d. None of the above
a. body will move in forward direction if F = 5 N 53. The coefficient of static friction between the two
b. body will be move in backward direction blocks is 0.363. What is the acceleration of block 1 so
with acceleration 0.5 m/s 2 if force F = 3 N that block 2 does not fall?
c. if F = 3 N, then body will be in rest condition a
d. Both a and c are correct
M m
48. Two blocks of masses M = 3 kg and m = 2 kg
are in contact on a horizontal table. A constant
horizontal force F = 5 N is applied to block M as a. 6 ms-2 b. 12 ms-2
shown. There is a constant frictional force of 2 N c. 18 ms-2 d. 27 ms-2
between the table and the block m but no frictional 54. Two fixed frictionless inclined plane making angles
force between the table 30° and 60° with the vertical are shown in the figure.
F M
and the first block M , m
Two blocks A and B are placed on the two planes.
then acceleration of the What is the relative vertical acceleration of A with
two blocks is respect to B?
a. 0.4 ms-2 b. 0.6 ms -2 c. 0.8 ms-2 d. 1 ms -2

49. The coefficient of static friction between the bodies A A


and B is 0.30. Determine minimum stopping distance
B
that the body A can have from a speed of 70 km/h
with constant deceleration if the body B is not to slip
forward
60° 30°
B
a. 4.9 ms-2 in horizontal direction
b. 9.8 ms-2 in vertical direction
A v0
c. Zero
d. 4.9 ms-2 in vertical direction
a. 3 m b. 30.3 m c. 70 km d. 63 m
55. A body is in equilibrium on a rough inclined plane
50. In the given figure, force of friction on body is under its own weight. If the angle of inclination of the
inclined plane is a and the angle of friction is l, then
A l
a. a > l b. a > c. a = l d. a ³ l
2
B
Smooth 56. For the equilibrium of a body on an inclined plane of
inclination 45°, the coefficient of static friction will be
a. towards left b. towards right a. greater than one b. less than one
c. either left or right d. no sufficient data c. zero d. less than zero

@iitjeehelps
BITSAT Archives
1. A block of mass 0.18 kg is attached to a spring of 6. Pseudo force is [2013]
force constant 2 N/m. The coefficient of friction a. electromagnetic in nature
between the block and the floor is 0.1. Initially, the b. a nuclear force
block is at rest and the spring is unstretched. An c. a gravitational force
impulse is given to the block. d. None of the above
The block slides a distance of 0.06 m and comes to 7. A body is moved in straight line by constant power of
rest for the first time. The initial velocity of the block in machine. What will be the relation between the
m/s is v = N /10. Then, N is [2014] travelling distance and time? [2012]
a. 2 b. 3 c. 4 d. 6 a. s 2 µ t 3 b. s 4 µ t 3
2. The sum of the magnitudes of two forces acting at a c. s 3 µ t 2 d. s µ t 3
point is 16 N. The resultant of these forces is 8. A 10 kg stone is suspended with a rope of breaking
perpendicular to the smaller force which has a strength 30 kg-wt. The minimum time in which the
magnitude of 8 N. If the magnitude of smaller force is stone can be raised through a height 10 m starting
x, then the value of x is [2014] from rest is (Take, g = 10 N kg -1) [2012]
a. 2 N b. 4 N
c. 6 N d. 7 N a. 0.5 s b. 1.0 s
2
3. A block of mass m = 1 kg is placed over a plank Q of c. s d. 2.0 s
3
mass M = 6 kg, placed over a smooth horizontal
surface as shown in figure. Block m
9. The minimum force required to move a body up an
inclined plane is three times the minimum force
P is given a velocity v = 2 m/ s 2 P n
required to prevent it from sliding down the plane. If
to the right. If the coefficient of M Q
the coefficient of friction between the body and the
friction between P and Q is 1
m = 0.3. Find the acceleration of inclined plane is , the angle of the inclined
2 3
Q relative to P . [2013]
plane is [2012]
a. 4 m/s 2 b. 3.5 m/s 2
c. 2m/s 2 d. 10.0 m/s 2 a. 60° b. 45°
c. 30° d. 15°
4. A bomb at rest explodes into three parts of the same
10. Natural length of a spring is 60 cm and its spring
mass. The linear momentum of two parts are - 2p $i
constant is 4000 N/m. A mass of 20 kg is hung from it.
and p $j. The magnitude of momentum of third part is The extension produced in the spring is
p x . Find x . [2013] (Take, g = 9.8 m/s 2 ) [2011]
a. p b. 5p a. 4.9 cm b. 0.49 cm
c. 2 p d. 10 p c. 9.4 cm d. 0.94 cm
5. A block is placed on an inclined plane. The block is 11. The linear momentum p of a body moving in one
moving towards right horizontally with an acceleration dimension varies with time t according to the equation
a 0 = g . The length of the inclined plane ( AC ) is equal p = a + bt 2, where a and b are positive constant. The
to 1 m. Whole the situation are shown in the figure. net force acting on the body is [2011]
Assume that all the surfaces are frictionless. The time a. a constant
taken by the block to reach from C to A is b. proportional to t 2
( Take, g = 10 m/ s 2 ) [2013] c. inversely proportional to t
d. proportional to t
A
12. A block of mass 5 kg is placed on a rough inclined
plane. The inclination of the plane is gradually
increased till the block just begins to slide down. The
a0=g
inclination of the plane is than 3 in 5. The coefficient of
friction between the block and the plane is
( Take, g = 10 m/ s 2 ) [2010]
30° C 3 3
a. b.
5 4
a. 0.74 s b. 0.9 s 4 2
c. 0.52 s d. 1.24 s c. d.
5 3

@iitjeehelps
56 SELF STUDY GUIDE BITSAT

13. A block weighing w is held against a vertical wall by a. m1g b. m2 g


pressing horizontally with a force F. Then, F needed m1m2
c. (m1 + m2 )g d. g
to hold the block is [2010] m1 + m2

18. Starting from rest, the time taken by a body sliding


down on a rough inclined plane at 45° with the
F horizontal is twice the time taken to travel on a smooth
plane of same inclination and same distance. Then,
the coefficient of kinetic friction is [2008]
a. 0.25 b. 0.33 c. 0.50 d. 0.75
W
19. Three weights w, 2w and 3w are connected to
w identical spring suspended from a rigid horizontal rod.
a. equal to w b. equal to only
m The assembly of the rod and the weights fall freely.
w w The positions of the weight from the rod are such that
c. greater than only d. greater than or equal to
m m a. 3w will be farthest [2007]
14. A body of mass 2 kg is placed on a horizontal surface b. w will be farthest
having coefficient of kinetic friction 0.4 and limiting c. all will be at the same distance
coefficient of static friction 0.5. If a horizontal force d. 2w will be farthest
2.5 N is applied on the body, the frictional force acting 20. Consider the following statement. When jumping from
on the body will be (Take, g = 10 ms -2 ) [2010] some height, you should bend your knees as you
a. 8 N b. 10 N c. 20 N d. 2.5 N come to rest instead of keeping your legs stiff. Which
of the following relations can be useful in explaining
15. In figure, block A is released from rest, when spring is the statement? [2007]
at its natural unstretched length. For block B of mass
a. Dp1 = - Dp2
M to leave contact with the ground at some stage, the
minimum mass of A must be [2009] b. DE = - D( PE + KE) = 0
c. FDt = mDv
d. Dx µ DF
where symbols have their usual meaning.
21. A body of mass 5 kg is suspended by a spring balance
on an inclined plane as shown in figure. The spring
A
balance measure [2006]

B
M
a. 3M b. c. M d. cannot say
2 m
16. A 600 kg rocket is set for a vertical firing. If the
30°
exhaust speed is 1000 m/s, the mass of the gas
ejected per second to supply the thrust needed to
a. 50 N b. 25 N
overcome the weight of rocket is [2009]
c. 500 N d. 10 N
-1 -1 -1 -1
a. 117.6 kgs b. 58.6 kgs c. 6 kgs d. 46.4 kgs
22. Under the action of a force F = Cx , the position of a
17. A system consists of two cubes of masses m1, and m 2
body changes from 0 to x. The work done is [2006]
respectively connected by a spring of force constant k.
1
force (F) that should be applied to the upper cube for a. Cx 2 b. Cx 2

which the lower one just lifts after the force is 2


1
removed, is [2009] c. Cx d. Cx
2
F
23. A student unable to answer a question on Newton’s
m1 laws of motion attempts to pull himself up by tugging
on his hair. He will not succeed [2005]
a. as the force exerted small
b. the frictional force while gripping is small
c. Newton’s law of inertia is not applicable to living
m2 beings
d. as the force applied in internal to the system

@iitjeehelps
Answer with Solutions
Practice Exercise 12. (a)QF = -
k
x2
1. (d) (a) The magnitudes of momentum are same to each k
other. \ Acceleration, f = -
mx 2
(b) Momentum is a vector quantity and vector does not When x decreases, v increases.
obey division law. So, the ratio of momentum is not dv
determined. \ f = -v
dx
2. (d) Contact force is an electromagnetic force. dv k
\ -v =-
3. (b) Cord D will breaks due to inertia offered by the block of dx mx 2
mass m. v k x 1
4. (a)Q r = 3t $i - 4 cos t $j
or ò0vdv = m òa x 2dx
1/ 2
dr 2k æ x - a ö
\ v= = 3$i + 4 sin t $j \ v = ç ÷
dt m è ax ø
d 2r
\ a = 2 = 4 cos t $j 13. (e) At t 2, force is constant.
dt So, acceleration is constant.
\ F = ma = 12 cos t $j At t = 0 , force is zero. Hence, acceleration is zero.
p/2
\ Impulse = ò F dt 14. (b) The equation of parabola is
0
p/ 2 x 2 = 4ay
= 12ò cos t dt
0 Here, t 2 = 4aF
= 12 [sin t ] 0p / 2 = 12 N-s When t = 5 s, F = 50 N (see graph)
The direction of impulse will be in the direction of force. \ 52 = 4a ´ 50
\ Impulse = 12 $j N-s 25 1
\ a= =
5. (b) The acceleration of the system is 200 8
1 F
a=
Net force
=
12
=
12
= 2 m/s2 \ t2 = 4 ´ F = Þ F = 2t 2
Total mass 1 + 2 + 3 6 8 2
F 2t2 2t2 t2
6. (d) A force of constant magnitude which is always Acceleration = = = =
m m 10 5
perpendicular to the velocity of the particle does not do
work. Hence, KE of the particle is constant. As force is dv t 2 v 5t 2
\ = or ò dv = ò dt
perpendicular to the velocity, the particle moves in a dt 5 0 0 5
circular path. 5
1 ét3 ù 125
7. (d) Since, at t = 0 and t = 1s, particle is in rest. This is \ v = ê ú = = 8.33 m/s
5 ë 3 û 0 15
possible only when the particle accelerates for sometime
and then decelerates to come in rest. 15. (d) Since, no force of friction is present. So, no horizontal
v - u -4 - 10 force is present on body of mass m. In vertical direction
8. (e) a = = = - 7 m/s 2 normal force balances weight of the body. Hence, net
t 2
force of top body must be zero. Lesser contact force
The magnitude of change in momentum corresponds more comfortableness.
= | m(v - u )| = 14. kg-m/s = Impulse 16. (b,c,d) The acceleration of system in rightward direction is
Now, F = ma = 0.7 N F F NA
Dp Dm Dx a= =
9. (c)\F = =v = rAv mA + mB + mC 3M
Dt Dt Dt
For A, F A F1
æ vD t ö 2
= rAv ç ÷ = 1 ´ 2 ´ ( 30) = 1800 dyne From force diagram of A,
è Dt ø
Net force on A = F - F1
10. (b) The action and reaction forces must act upon different MF F mg
bodies. = m1a = =
3M 3
11. (c) The tension in the rope will be same at all points if it is The force interaction between A and B is
massless or if the string has mass, the tension will be the F 2F
same in the string when it is not accelerating. F1 = F - =
3 3

@iitjeehelps
58 SELF STUDY GUIDE BITSAT

17. (a) From force diagram shown in figure, If the force of interaction between top (first) 1
40 N = mg link and second isT . Then, F
ma = F - mg - T 2
T1 sin 30° 30°
T1 T = F - mg - ma 3
= 5mg + 5ma - mg - ma
30° T1 cos 30° = 4mg + 4ma = 4m(g + a ) 4
T2 T = 4 x 0.1(9.8 + 2.5) = 4.92 N
5
40 N = mg 23. (b) x1 + x 2 = l (length of string)
Þ T1 sin 30° = 40 dx1 dx 2
+ =0
\ T1 = 80 N dt dt
2T cosθ v1 + v 2 ¢ = 0
18. (d) For equilibrium of body,
x2
mg = 2T cos q Þ v1 = - v 2
x1
mg and a1 = - a2
\ T =
2 cos q am1m 2 = am1 - am 2 m2
θ θ
For the string to be horizontal, = a - ( -a ) = 2a
q ® 90° 24. (c) aBA = aB - aA m1
mg a
\ T = m
2 cos 90° B
T ®¥ a
mg
19. (b) For ring,
From force diagram of ring, T cosθ A
T cos q = m1g …(i) T

For block B, N θ T sinθ = aB + ( - aA )


mg = T
T cos q = m1g a
–a √2
mg cos q = m1g
m 5 1 m 1g
\ cos q = 1 = = a=aB
m 10 2
Þ q = 60°
T - mg ( 20 - 10)
20. (a) a max = = =g aA=a
m 10
During half time, body is accelerated from rest and during 25. (d) Since, the mass of the pulleys and the string are
next half time decelerates with same magnitude to come negligible, so tension is same everywhere.
to rest.
h 1 Considering free body diagrams of m1, m2 and the pulley
\ = a maxt12 A, we have
2 2
m1g - T = m1a1
h
\ t1 = = 1s 2T - m2 g = m2 a 2
a max
2T - T = 0
21. (c) The force diagram of system ( A + B ) is shown in figure. 2T = T
But this is possible only whenT = 0
a
\ a1 = a 2 = g
A+B
Thus, weights falls freely and pulleys B and C rotate
(MA+MB)g anti-clockwise but pulley A clockwise.

Here, T1 - (M A + MB )g = (M A + MB )a
T2 26. (b)
N T
\ T1 = (M A + MB )g + (M A + MB )a a
B ⇒ F = 2T
= (M A + MB )(g + a )
T ⇒ T = ma
The force diagram of body B, MB g ⇒ F = ma
T2 - MBg = MBa 2
mg ∴ a= F
\ T2 = MB (g + a ) 2m
F
22. (b)Q F = 5mg = 5ma
F = 5mg + 5ma 27. (c) From force diagram in Fig. (a),
= 5m(g + a ) …(i) mg sin q - T = ma …(i)

@iitjeehelps
NEWTON’S LAWS OF MOTION AND FRICTION 59
N
a T 34. (b) ma0cosθ
N
a
θ θ m
sin ma0
mg m
θ mg cosθ T
mg sinθ
(a) (b) θ
a0
In force diagram (b), T = ma …(ii) mg cosθ+mg sinθ mg
From Eqs. (i) and (ii), we get From force diagram,
g
a = sin q mg sin q - ma 0 cos q = ma
2
\ a = g sin q - a 0 cos q
mg sin q
\ T = 35. (c)
2
28. (c) After sometime, right load moves on straight line 36. (a) Let the mass of hanging part of chain in left is m.
(mg - T ), but left load will oscillate in addition to the
motion in straight line (mg - T cos a ).

T cosα
a
T T
T
B α b=2a
m
2m
T mg
a1 a2
A 2mg
m
mg
mg m m
mg 2mg
But, mg - T cos a > mg - T
\ a1 > a 2 [in downward direction] mg 3m 2 mg
So, at any instant, the left load is lower than the right one.
29. (c) The value measured by O1 is N1 = mg because Since, b = 2a, so the mass of hanging part of chain in right
acceleration of body with respect to O1 is zero. will be 2m.
\ 2mg - mg = 3mf
The value measured by O 2 is
g
N2 - mg = ma 0 \ f =
3
\ N2 = m(g + a 0 )
37. (c) The force diagram of the body at a
So, N1 ¹ N2
lowest position is shown in the figure m
30. (d) Let the velocities of S1, v1 v2 v3 force diagram of body.
S 2 and the particle in the kx - mg = ma kx
frame of ground arev1,v 2 S1 S2 S3
kx
andv 3, respectively. \ a= -g mg
m
The velocity of the particle in the frame of S1 is
38. (c) For minimum mass of m, mass M breaks off contact
v 31 = v 3 - v1 = 0 …(i) when elongation in spring is maximum. At the time of
From this expression, it is obvious that at every instant, break off, block A is at lowest position and its speed is
velocity of S1 and the particle are same with respect to zero. At an instant t1
ground. The velocity of the particle in the frame of S 2 is T=kx
v 32 = v 3 - v 2 = constant …(ii) mg - kx = ma
dv mg - kx
If S1 and S 2 are inertial, then accelerations of S1 and S 2 in v =
the frame of ground are zero. If the velocity of the particle dx m
0
mg . kx
is constant in the frame of ground, then the statements of x0 æ k ö = mg
Eqs. (i) and (ii) are easily satisfied. If S1 and S 2 are ò vdv = ò0 çg - x ÷ dx
è m ø
0
non-inertial frames of acceleration, then the statements of a
Eqs. (i) and (ii) are also satisfied. where, x 0 is maximum elongation in spring.
mg
31. (a) 32. (a) kx 02
0 = gx 0 -
33. (b) 2m

@iitjeehelps
60 SELF STUDY GUIDE BITSAT

2mg N=0 kx0 F -f


x0 = \ a=
k m+M
At the time of break off of block B, 5-2 3
= = = 0.6 m/s 2
Mg = kx 0 5 5
Mg = 2mg 49. (d) The maximum deceleration is
M Mg a = mg = 3 m/s2 B
\ m=
2 v0
\ v 2 = u 2 - 2as A
39. (c) When the bicycle is being pedaled then during
pedaling, the force is acting in the backward direction on 0 = v 02 - 2mgs min
the rear wheel and thus frictional force will act in the æ 5ö
forward direction. The choice (c) is correct, when the ç 70 ´ ÷
v 02 è 18 ø
bicycle is not pedaled. \ s min = = = 63 m
2 mg 6
2 2
40. (a) The net electromagnetic force = N + f
50. (b) From force diagram shown in Fig. (b) and (c), the
But, N = mg, f = mmg direction of force of friction on B is rightward.
Þ Force = mg 1 + m 2
A F
41. (d)
42. (a) When body remains in rest. B
For equilibrium, static friction f is
equal to applied force F. F (a)
Friction
\ F =f
The corresponding graph is A F B
45° Friction
shown in figure. f
From graph, slope = tan 45° = 1 (b) (c)
43. (d) Since, motion of body is in forward direction. So, 51. (c)
friction acts in backward direction. Due to this, velocity of
52. (c) From the diagram shown in figure, the resultant force
body starts to decrease.
of B in vertical direction is not zero. Hence, the system
44. (d) never be in equilibrium.
45. (d) Fmax = mmg f
Maximum acceleration to the car is provided by the limiting
frictional force. B
A
F N2 F
Þ a= = mg N1 N2
m
1 1
Using, s = ut + at 2 Þ s = 0 + a maxt min 2
2 2 f mBg
mA g
2s 1
Þ =t Þ t µ 53. (d) In the frame of block M, m is in rest.
mg m
\ N = ma
46. (c) For minimum value of m, we fL=µN
m
can write
ax

mg sin ( 45° ) = (fs )max


s )m

a
(f

mg sin 45° = (m )min mg cos 45° mg sin 45°


θ = 45° ma
(m )min = 1 m N M m
47. (d) For moving the body, applied force should be greater
or equal to limiting friction (i.e. mN).
The limiting friction is f2 = mN = mmg
= 0.2 ´ 2 ´ 9.8 = 3.92 N mg
(a) Since, applied force F = 5 N, so body starts to move For not falling, mg < fL
in forward direction. or mg < mN
(b) Here, applied force is lesser than limiting friction. So, or mg < mma
body does not move. g
or a>
(c) The reason is same as option (b). m
48. (b) If both blocks move together g 9.8
\ a min = = = 27 m/s2
F - f = (m + M )a m 0.368

@iitjeehelps
NEWTON’S LAWS OF MOTION AND FRICTION 61
54. (d) We can write, mg sin q = ma inα
gs
N µm
\ a = g sin q
where, a is along the inclined plane. α
sin
Q Vertical component of acceleration is g sin2 q. mg
α
\ Relative vertical acceleration of A with respect to B is
mg cosα
g
g (sin2 60° - sin2 30° ) = = 4.9 ms-2 [in vertical direction] mg sin a = mmg cos a
2 Þ tan a = m = tan l Þ a = l
55. (c) For equilibrium, 56. (a)

BITSAT Archives
1. (d) Here, m = 0.18 kg, k= 2 N/m, m = 0.1,x = 0.06 m. p3 = - p ¢ = - p 5 = 5 p
According to conservation of mechanical energy principle, p3 = 5 p
we know
5. (a) The forces on smaller block is given as
Decrease in mechanical energy
mg cos 30°
= Work done against friction N
1 2 1 2
mv - kx = mmgx
2 2
a0=g ma0
2 mmgx + kx 2
Þ v =
m mg cos θ mg sin θ
Substituting the values of m, m, g, x and k, we get mg
2 ´ 0.1 ´ 0.18 ´ 9.8 ´ 0.06 + 2 ´ 0.06 30°
v =
0.18
v = ( 4 /10) m/s For the motion of the block along the incline plane in
upward direction.
So, N = 4
Net force on the block = mass ´ acceleration of the block
2. (c) x + y = 16,
Þ mg cos 30° - mg sin 30° = ma [Q a 0 = g]
Also, y 2 = 82 + x 2 æ 3 - 1ö
y Þ a=ç ÷ g = 3.66 m/s 2
or y 2 = 64 + (16 - y )2 [Q x = 16 - y ] è 2 ø
8N
or y 2 = 64 + 256 + y 2 - 32y 1 2
Now, from equation of motions = at
or 32y = 320 2
x
y = 10 N 2s 2 ´1
Þ t= = = 0.74 s
\ x + 10 = 16 Þ x = 6 N a 3.66
3. (b) Frictional force between P and Q is f = mmg which will 6. (d) Pseudo force is not a real force.
retard P and accelerate Q. 7. (a) Power = [ML2T -3] = constant
f - mmg
Retardation of P is aP = - = = - mg é ML2 ù
m m \ ê 3 ú = constant
+ f mmg ë T û
Acceleration of Q is aQ = =
M M \ [ L ] µ [ T 3]
2
or s 2 µ t 3
Acceleration of Q relative to P is 8. (b) Tension in the stringT = mg
mmg
aQP = aQ - aP = - ( - mg ) = 30 ´ 10 = 300 N T a
M
é mù é 1ù T - Mg = Ma
= mg ê1 + ú = 0.3 ´ 10 ê1 + ú = 3.5 m/s 2
ë Mû ë 6û From the figure, 300 - 10 ´ 10 = 10a
M = 10 kg
$ \ a = 20 ms -2
4. (b) Given, p = - 2p i = 2 p along negative x-axis.
1
Thus, the maximum acceleration with
p 2 = p $i = p along y-axis. Mg
which the stone can be raised is
The resultant momentum of two parts 20 ms -2.
p ¢ = p12 + p 22 = ( 2p )2 + p 2 = p 5 Given, s = 10 m and u = 0
As the bomb was initially at rest final momentum of all the 1
\ 10 = ( 20)t 2
three parts must be zero 2
p3 + p ¢ = 0 Þ t = 1s

@iitjeehelps
62 SELF STUDY GUIDE BITSAT

9. (c) Minimum force required to move a body up a rough 15. (b) The spring will exert maximum force when A is at
inclined plane is lowest position, say at distance x from the present
F1 = mg (sin q + m cos q) position.
Minimum force required to prevent the body form sliding 1
\ mgx = kx 2
down the rough inclined plane, is 2
F2 = m mg cos q where, k is force constant of the spring, for the block B to
According to the question, leave contact, kx = Mg
1
F1 = 3F2 \ mgx = Mgx
2
\ mg(sin q + m cos q) = 3 (m mg cos q)
M
Þ sin q + m cos q = 3 m cos q \ m=
2
Þ sin q = 2 m cos q
dm
1 1 16. (c) Thrust = u = Mg
Þ tan q = 2 m = 2 ´ = = tan 30° dt
2 3 3
dm Mg 600 ´ 9.8
q = 30° or = = = 6 kgs-1
dt u 1000
10. (a) Given, l = 60 cm= 60 ´ 10-2 m
17. (c) Let x 0 be the maximum compression, then
m = 20 kg, k = 400 N/m
m1g + F = kx 0 …(i)
The weight hung from the spring 1 2
= mg = 20 ´ 9.8 = 196 N and kx 0 = m1gx 0 …(ii)
2
Suppose x is the extension produced in spring If x is the maximum elongation, then
Now, force applied by the spring kx = m2g …(iii)
= downward force on the spring 1 2
mg and - kx = m1g(x + x 0 ) …(iv)
\ kx = mg Þ x = 2
k or k (x 0 - x ) = 2m1g
20 ´ 9.8 From Eqs. (i) and (iii), we get
Þ x = = 0.049 m = 4.9 cm
4000 m1g + F - m2g = k (x 0 - x ) = 2m1g
11. (d) Given, p = a + bt 2 or F = 2m1g + m2g - m1g = (m1 + m2 )g
dp é 1ù
= 2bt 18. (d) m = tan q ê1 - 2 ú
dt ë n û
dp
Q F = Here, q = 45° and n = 2
dt é 1ù 1 3
\ F = 2bt or F µ t \ m = tan 45° ê1 - 2 ú = 1 - = = 0.75
ë 2 û 4 4
3
12. (b) sin q = 19. (c) Forw , 2w , 3w , apparent weight will be zero because
5
the system is falling freely. So, the distances of the
weights from the rod will be same.
5 3
20. (c) FDt = mDv
θ mDv
4 Þ F=
3 t
m = tan q Þ m =
4 By doing so time of change in momentum increases and
impulsive force on knees decreases.
13. (b) F - N + 0
21. (b) Acceleration of the body down the rough inclined
Þ F =N
plane. = g sin q
Further, f = mN = w
Force applied on spring balance = mg sin q
w
Þ mF = w Þ F = 1
m = 5 ´ 10 ´ sin 30° = 5 ´ 10 ´ = 25 N
2
14. (d) fstatic = (m static ) mg x
x x æx 2 ö 1
Þ fstatic = ( 0.5)( 2)(10) Þ fstatic = 10 N 22. (a) Work done,W = ò F × dx = ò Cx dx = C ç ÷ = Cx 2
0 0 è 2 ø0 2
Q Applied force is just 2.5 N < 10 N. So, the block will not
move. As, we know that in static region, friction is a self 23. (d) In this case, the internal force is applied on the
adjusting force and has a value equal to that of applied system, so he will not succeed. According to Newton’s
force. law, the state of a body can only be changed if some
So, f = Fapplied = 2.5 N external force is applied on it.

@iitjeehelps
5
Circular Motion

Concept of Circular Motion


The circular motion differs from the linear motion in one very important aspect that in a circular
motion particles move along circular track such that direction of motion changes continuously unlike
in a linear motion. Therefore, circular motion is described in terms of angular displacement i.e. angle
turned by the rotating body in an unit time.

Terms Related to Circular Motion


Some important terms used in circular motion are given as

1. Angular Displacement
It is defined as the angle turned by the particle from some reference line. Angular displacement Dq is
usually measured in radian.
Finite angular displacement Dq is a scalar but an infinitesimally small displacement is a vector.

2. Angular Velocity
It is defined as the rate of change of the angular displacement of the body.
v2

B
r2

O ∆θ v1
r1

æ D q ö dq
\ Angular velocity, w = lim ç ÷ =
Dt ®0 è Dt ø dt
2p
w=
T
It is an axial vector whose direction is given by the right hand rule. Its unit is rad/s.

@iitjeehelps
64 SELF STUDY GUIDE BITSAT

3. Angular Acceleration Dynamics of Uniform


It is the rate of change of angular velocity.
dw d 2q
Circular Motion
Thus, a= =
dt dt 2 If a particle in circular motion moves with a uniform speed,
Its unit is rad/s 2 . then motion of the particle is called uniform circular
motion. In such a case,
dv
4. Velocity = 0 and a = w2r
dt
A particle in a circular motion has two types of velocities
and corresponding two speeds. v2
or a = 2r
(i) Linear velocity (v) or speed (v) r
ds ds v2
v= and v = | v | = a=
dt dt r

(ii) Angular velocity (w) or speed ( w) Thus, if a particle moves in a circle of radius r with a
v2
dq dq dq uniform speed v, then its acceleration is towards the
w= and w = | w| = = r
dt dt dt
centre. This acceleration is termed as centripetal
Relation between linear speed (v ) and angular speed ( w) is acceleration.
v=rw
In vector form, v = w ´ r , which is a relation between linear
velocity (v) and angular velocity ( w). Forces in Circular Motion
Here, r is the position vector of particle with respect to the In circular motion of an object, two kinds of forces occur
centre of the circle. which are described below

5. Acceleration (i) Centripetal Force


Acceleration of a particle in circular motion has two A particle performing circular motion is v
components acted upon by a force directed along the
(i) Tangential acceleration (at ), Y′
Y ey radius towards the centre of the circle. This Fc
A er m
which is the component of a in force is called the centripetal force. O
the direction of velocity. P θ
r X′ If m is the mass of the particle, then the
at = component of a along v θ
o X magnitude of centripetal force is given by
dv d | v | A particle
= = Centripetal force = Mass ´ Centripetal performing
dt dt acceleration uniform
(ii) Radial acceleration (ar ), which is the component of a æ v2 ö circular motion
towards the centre of the circular motion. This is Þ F =mç ÷
responsible for a change in the direction of velocity. èr ø
v2 or F = mr w2
ar = = r w2
r In vector form, centripetal force is given by
(iii) Tangential acceleration If we differentiate above mv 2
F=- r$
equation, we get, r
dv dw or F = -(mw2r ) r$
= ´r
dt dt where, r$ is a unit vector directed away from the centre
i.e. at = a ´ r along the radius.
where, at is the tangential component of acceleration
which is responsible for changing the magnitude of
velocity.
(ii) Centrifugal Force
v
when speed of the particle increases at is positive or It can be defined as the
in the direction of the velocity. radially directed outward
m
Thus, net acceleration in this case will be force acting on a body in O T Centrifugal
2
a = at + ar
2 circular motion, as observed force on body
by a person moving with the
dv d |v| mg
where, at = or = body.
dt dt

@iitjeehelps
CIRCULAR MOTION 65
Centrifugal force = Mass ´ Centrifugal acceleration 3. Bending of a Cyclist
mv 2
Þ F = = mrw2 or F = mvw [Qv = wr ] If a curved road is not banked, then a cyclist/scooterist
r bends away from the vertical, while negotiating a turn on a
mv 2 curved road. The angle q at which the cyclist bends from the
This can be written in vector form as F = r$
r v2
vertical, is given by tanq =
rg
Some Important Points
If a tube filled an incompressible fluid of mass m and closed
at both ends is rotated with constant angular velocity w 4. Motion of a Cyclist in a Death Well
about an axis passing through one end. Then, the force For equilibrium of cyclist in a death well,
1
exerted by liquid at the other end is mLw2 . as shown in the figure, the normal reaction
2 f
ω N provides the centripetal force needed Cyclist N
and the force of friction balances his mg
weight mg.
L
mv 2 r
Thus, N =
r
rg
Applications of Centripetal and f = mN = mg Þ vmax =
m
and Centrifugal Forces Vertical Circular Motion
1. Motion of a Vehicle on a Level It is an example of non-uniform circular B
vH
Circular Road motion in which speed of object
TH
decreases due to effect of gravity as the mg
When a vehicle negotiates a circular path, it requires a
object goes from its lowest position A to v
centripetal force. In such cases the lateral force of friction O
highest position B. θ
may provide the required centripetal force. Thus, for TL C
maintaining its circular path required centripetal force (i) At the lowest point A, the tension
mg
T L and the weight mg are in A vL
æ mv 2 ö
ç ÷ £ frictional force (mmg ) mutually opposite directions and mg
è r ø their resultant provides the
Þ vmax = mrg necessary centripetal force,
where, m = coefficient of friction between road and mvL2
i.e. T L - mg =
r
vehicle tyres and r = radius of circular path.
mvL2
or T L = mg +
2. Banking of a Curved Road r

For the safe journey of a vehicle on a (ii) At the highest point B , tension T H and the weight mg
curved (circular) road, without any N cos θ are in the same direction and hence,
N
risk of skidding, the road is slightly mvH2 mvH2
T H + mg = or T H = - mg
raised towards its outer end. Let the r r
road be banked at an angle q from the N sin θ Moreover, vL and vH are correlated as vH2 = vL2 - 2 gr
horizontal, as shown in the figure.
θ (iii) In general, if the revolving particle, at any instant of
vmax = rg × tan q time, is at position C, inclined at an angle q from the
If b = width of the road and h = height vertical, then
of the outer edge of the road as mg v 2 = vL2 - 2 gr (1 - cosq )
compared to the inner edge, then mv 2
and T = mg cosq +
v2 h r
tanq = =
rg b (iv) In the critical condition of just looping the vertical
loop, (i.e. when the tension just becomes zero at the
In case of friction is present between road and tyre
highest point B), we obtain the following results
r g (m s + tan q )
then vmax = T H = 0, T L = 6 mg , vL = 5rg andvH = rg
1 - m s tan q
In general, T L - T H = 6 mg
where, m s = coefficient of static friction.

@iitjeehelps
66 SELF STUDY GUIDE BITSAT

Some Important Points Expression for the Radius of Curvature


● When a vehicle is moving over a convex bridge, the
maximum velocity v = rg, where r is the radius of the road. for a Particle at the Highest Point in the
● When the vehicle is at the maximum height, the reaction of Case of Projectile Motion
mv 2
the road, is N1 = mg -
r N1
● When the vehicle is moving in a u cosα
dip B, then
A α mg
mv 2
N2 = mg + N2
r
● The weight that we feel is the
normal force and not the actual weight. mv 2
B mg = (But, v = u cosa)
● When body losses the contact, normal r
force reduces to zero.
The normal on tangent at a point on the curve gives the v2
r =

direction of radius. g
é
3/ 2 or
dy ö ù
2
æ
ê1 + ç ÷ ú u 2 cos2 a
ê è dx ø ú =
i.e. r= ë 2 û g
d y / dx 2
● where, r is radius of curvature. where, r is the radius of curvature.

Practice Exercise
1. In circular motion 5. Two moving particles P and Q are 10 m apart at a
a. radial acceleration is non-zero certain instant. The velocity of P is 8 m/s making an
b. radial velocity is zero angle of 30° with the line joining P and Q and that of Q
c. body is in equilibrium is 6 m/s making an angle 30° with PQ as shown in
d. All of the above figure.
2. A particle is acted upon by a force of constant 6 m/s
magnitude which is always perpendicular to the
30°
velocity of the particle. The motion of the particle takes
place in a plane. It follows that 30° Q
a. its velocity is constant
b. its acceleration is constant 8 m/s
c. its kinetic energy is constant
d. it does not move on a circular path Then, angular velocity of P with respect to Q is
3. A stone of mass m tied to a string of length l is rotated a. zero b. 0.1 rad/s c. 0.4 rad/s d. 0.7 rad/s
in a circle with the other end of the string as the
centre. The speed of the stone is v . If the string 6. A solid body rotates about a stationary axis, so that its
breaks, the stone will angular velocity depends on the rotational angle f as
w = w0 - kf, where w0 and k are positive constants. At
a. move towards the centre
b. move away from the centre
the moment t = 0, f = 0. Find the time dependence of
c. move along a tangent rotation angle.
w0 - kt
d. stop a. k w0e - kt b. e
k
4. Particles are released from rest at A and slide down w0 k
the smooth surface of height h to a conveyor B. The c. (1 - e - kt ) d. (e - kt - 1)
k w0
correct angular velocity w A
of the conveyor pulley of 7. The position of a point P is r = a cos q $i + b sin q$j,
radius r to prevent any h
where a and b are constants and q is angle between r
sliding on the belt as the
and x-axis. If the rate of increasing of q is w. Find the
particles transfer to the B
ω equation of path of particle.
conveyor is
a. Circle b. Parabola
2gh 2gh 2gh 2
a. 2gh b. c. d. c. Ellipse d. Straight line
r r r2

@iitjeehelps
CIRCULAR MOTION 67
8. The angular displacement A 16. A cyclist is travelling on a circular section of highway
of the rod is defined as B of radius 2500 ft at the speed of 60 mile/h. The cyclist
3 2 suddenly applies the brakes causing the bicycle to
q= t , where q is in
20 slow down at constant rate. Knowing that after
r 8 second, the speed has been reduced to 45 mile/h.
radian and t is in second.
The collar B slides along The acceleration of the bicycle immediately after the
θ
the rod in such a way that O brakes have been applied is
its distance from O is a. 2 ft/s2 b. 4.14 ft/s2 c. 3.10 ft/s2 d. 2.75 ft 2/s
r = 0.9 - 0.12 t 2, where r is
17. A road of width 20 m forms an arc of radius 15 m, its
in metre and t is in second. outer edge is 2 m higher than its inner edge. Calculate
The velocity of collar at for what velocity the road is banked?
q = 30° is a. 10 m/s b. 14.7 m/s
a. 0.45 m/s b. 0.48 m/s c. 0.52 m/s d. 0.27 m/s c. 9.8 m/s d. None of these
9. Two buses A and B are moving around concentric 18. Three identical cars A, B and C are moving at the
circular paths of radii r A and rB . If the two buses same speed on three bridges. The car A goes on a
complete the circular paths in the same time. The ratio plane bridge, B on a bridge convex upwards and C
of their linear speeds is goes on a bridge concave upwards. Let FA , FB and FC
a. 1 b. rA / rB be the normal forces exerted by the cars on the
c. rB / rA d. None of these bridges when they are at the middle of the bridges.
Then,
10. A stone of mass 0.3 kg attached to a 1.5 m long string a. FA is maximum of the three forces
is whirled around in a horizontal circle at a speed of
b. FB is maximum of the three forces
6 m/s. The tension in the string is
c. FC is maximum of the three forces
a. 10 N b. 20 N c. 7.2 N d. None
d. FA = FB = FC
11. A cyclist goes round a circular path of length 400 m in
20 second. Calculate the angle through which he 19. A car runs from east to west and another car B of the
bends from vertical in order to maintain the balance same mass runs from west to east at the same path
a. sin- 1 (0.64) b.tan- 1 (0.64)
along the equator. A presses the track with a force N1
and B presses the track with a force N 2. Then,
c. cos- 1 (0.64) d. None of these
a. N1 > N2 b. N1 < N2
12. Find the maximum speed with which an automobile c. N1 = N2 d. None of these
can round a curve of radius 8 m without slipping if the
road is unbanked and the coefficient of friction 20. A small body of mass m slides down from the top of a
between the road and the tyres is 0.8. (g = 10 m/ s 2) hemisphere of radius R. The surface of block and
a. 8 m/s b. 10 m/s hemisphere are frictionless. The height at which the
c. 20 m/s d. None of these body loss contact with the surface of the sphere is
3 2 1 1
13. A tube of length L is filled completely with an a. R b. R c. R d. R
2 3 2 3
incompressible liquid of mass M and closed at both
ends. The tube is then rotated in a horizontal plane 21. A person wants to drive on the vertical surface of a
about one of its ends with a uniform angular velocity w. large cylindrical wooden well commonly known as
The force exerted by the liquid at the other end is death well in a circus. The radius of well is R and the
coefficient of friction between the tyres of the
ML w2 ML w2 ML2 w2
a. b. ML w2 c. d. motorcycle and the wall of the well is ms . The minimum
2 4 2 speed, the motorcyclist must have in order to prevent
14. A point on the periphery of a rotating disc has its slipping should be
acceleration vector making an angle of 30° with the æ Rg ö æm ö
velocity vector. The ratio ac / at (ac is centripetal a. ç ÷ b. ç s ÷
è ms ø è Rg ø
acceleration and at is tangential acceleration) equals
a. sin 30° b. cos 30° c. tan 30° d. None æm g ö æ R ö
c. ç s ÷ d. ç ÷
è R ø è msg ø
15. A car of 1400 kg is moving on a circular path of radius
30 m with a speed of 40 km/h. When the driver applies 22. A car is moving in a circular horizontal track of radius
the brakes and the car continues to move along the 10 m with a constant speed of 10 m/s. A plumb bob is
circular path, what is the maximum deceleration suspended from the roof of the car by a light rigid rod
possible if the tyres are limited to a total horizontal of length 1 m. The angle made by the rod with the
friction of 10.6 kN? track is
a. 10 m/s2 b. 6.36 m/s2 c. 4 m/s2 d. None a. zero b. 30° c. 45° d. 60°

@iitjeehelps
68 SELF STUDY GUIDE BITSAT

23. A heavy sphere of mass m is block is observed to slip at q = 50 °, the coefficient of


suspended by a string of length l. static friction between the block and the rod is
θ
The sphere is made to revolve (Given that, sin 50° = 0.766, cos 50° = 064
. )
about a vertical line passing T A
through the point of suspension, in
a horizontal circle such that the
string always remains inclined to r
the vertical making an angle q. mg
What is the period of revolution?
l l cos q O θ
a. T = 2p b. T = 2p
g g
l sin q l tan q
c. T = 2p d. T = 2p
g g a. 0.2 b. 0.55 c. 0.8 d. 1
24. A rod of length L is hinged from one end. It is brought 29. A particle of mass m is moving in a horizontal circle of
to a horizontal position and released. The angular æ- kö
velocity of the rod when it is in vertical position is radius r under a centripetal force given by ç 2 ÷,
èr ø
æ 2g ö æ 3g ö æg ö æg ö where k is a constant, then
a. ç ÷ b. ç ÷ c. ç ÷ d. ç ÷
è L ø è L ø è 2L ø èLø æ-kö
a. the total energy of the particle is ç ÷
25. Two wires AC and BC are tied at C A è 2r ø
of small sphere of mass 5 kg, æk ö
b. the kinetic energy of the particle is ç ÷
30

which revolves at a constant èr ø


°

B
speed v in the horizontal circle of æk ö
45°

c. the potential energy of the particle is ç ÷


radius 1.6 m. The minimum value è 2r ø
of v is æ kö
C d. the kinetic energy of the particle is ç - ÷
a. 3.01 m/s è rø
b. 4.01 m/s
c. 8.2 m/s 1.6 m 30. Kinetic energy of a particle moving along a circle of
d. 3.96 m/s radius R depends on the distance covered as K = as 2,
where a is a constant. Find the force acting on the
26. The small spherical balls are free to move on the inner
particle as a function of s .
surface of the rotating spherical chamber of radius
2 2
R = 0.2 m. If the balls reach a steady state at angular 2a æs ö æR ö
a. 1+ ç ÷ b. 2as 1+ ç ÷
position q = 45°, the angular speed w of device is s èR ø ès ø
ω 2 2
3r æs ö 2s æR ö
c. 2as 1+ ç ÷ d. 1+ ç ÷
èR ø a ès ø

31. A projectile is projected at a angle 60° with horizontal


θ with speed 10 m/s. The minimum radius of curvature
θ
of the trajectory described by the projectile is
a. 8 rad/s b. 2 rad/s c. 3.64 rad/s d. 9.34 rad/s a. 2.55 m b. 2 m
27. A frictionless track ABCD c. 10 m d. None of these
A
ends in a semicircular 32. The skate board negotiates the circular surface of
loop of radius R. A body D radius 4.5 m. At q = 45°, its speed of centre of mass is
slides down the track from h
6 m/s. The combined mass of skate board and the
point A which is at a 2R person is 70 kg and his centre of mass is 0.75 m from
C
height h = 5 cm. Maximum the surface. The normal reaction between the surface
value of R for the body to B and the skate board wheel is
successfully complete the
loop is
15 10 θ
a. 5 cm b. cm c. cm d. 2 cm v
4 3
28. A rod OA rotates about a horizontal axis through O
with a constant anti-clockwise velocity w = 3 rad/s. As
it passes the position q = 0° a small block of mass m is
placed on it at a radial distance r = 450 mm. If the a. 500 N b. 2040 N c. 1045 N d. zero

@iitjeehelps
CIRCULAR MOTION 69

BITSAT Archives
1. A body of mass m = 20 g is attached to an elastic 3. An inclined track ends in a circular loop of radius r .
spring of length L = 50 cm and spring constant From what height on the track a particle should be
k = 2 Nm -1. The system is revolved in a horizontal released so that it completes the loop, assuming there
plane with a frequency n = 30 rev/min. Find the radius is no friction? [2010]
of the circular motion and the tension in the spring. r 3r 5r
a. 0.25 m, 0.1 N [2014] a. b. c. 2 r d.
2 2 2
b. 0.5 m, 0.52 N
c. 0.55 m, 0.1 N 4. A small ball describes a horizontal circle on the
d. 0.9 m, 0.2 N smooth inner surface of a conical funnel. If the height
of the plane of the circle above the vertex be 10cm,
2. A car is moving on a circular road of diameter 50 m what is the speed of the particle? [2009]
with a speed of 5 m/s. It is suddenly accelerated at a
a. 2 m/s b. 4 m/s c. 16 m/s d. 1 m/s
rate of 1 m/s 2. If the mass of the car is 500 kg, then
the net force acting on the car is [2011] 5. The angle turned by a body undergoing circular
a. 5 N b. 1000 N motion depends on time as q = q 0 + q 1t + q 2t 2. Then,
500 the angular acceleration of the body is [2006]
c. 500 2 N d. N
2 a. q1 b. q2 c. 2q1 d. 2q2

Answer with Solutions


Practice Exercise \ wrel =
8 sin 30° + 6 sin 30°
10
v2
1. (d) In circular motion radial acceleration ar = and the = 0.7 rad /s
r
particle does not move along the radius of the circular 6. (c)Q w = w0 - kf
path hence radial velocity is zero, radial acceleration only
df
changes the direction of motion of the particle. or = w0 - kf
dt
2. (c) Perpendicular force does not changes the magnitude
of velocity hence the kinetic energy remain constant. f df t
or ò0 w0 - kf = ò0dt
3. (c) When the string break, there is no tension in the string,
hence there is no radial acceleration to change the 1
direction of the stone the required (tension provide or - [ln (w0 - kf)]f0 = t
k
centripetal force), hence the stone will move along the
tangent. æ w - kf ö
or ln ç 0 ÷ = - kt
4. (c) First of all, we have to calculate the velocity of è w0 ø
particles at the point B. w0
\ f= (1 - e - kt )
1 k
Q Loss in PE = gain in KE or mgh = mv 2
2 7. (c) Q r = x$i + y $j
r =xi + yj
\ v = 2gh
But, r = a cos q$i + b sin q$j
To prevent sliding,v = rw
v 2gh \ x = a cos q r
\ w= = θ
r r y = b sin q
x2 y2
5. (d) 6 sin 30° \ + = cos2 q + sin2 q = 1
a2 b2
6 m/s x2 y2
30° \ + =1
P a2 b2
30° Q
This is an equation of ellipse.
dr
8 m/s 8. (c)Q vr = = - 0.24t
8 sin 30° dt

@iitjeehelps
70 SELF STUDY GUIDE BITSAT

dq æ 6 ö æ 6t ö 15. (b) The net acceleration to car is provided by force of


v t = rw = r =rç t ÷ = ( 0.9 - 0.12 t 2 ) ç ÷
dt è 20 ø è 20 ø friction.
Q v = vr2 + vt2 \ a = an2 + at2
2
3 2 p 3 2 æv 2 ö
\ q= t or = t \ F = ma = m ç ÷ + at2
20 6 20 è r ø
æ 20p ö
\ t= ç ÷ v4
è 18 ø or 10.6 ´ 103 = m + at2
r2
Putting the value of q, v = 0.52 m/s
After solving, at = 6.36 m/ s2
9. (b)
10. (c) The force of tension in the string provides required 16. (b) Tangential acceleration
vf - vi æ 45 - 60 ö æ 22 ö 11
centripetal force to keep the stone in circular motion. = at = =ç ÷ç ÷=- ft/s2
mv 2 0.3 ´ (6)2 t è 8 ø è 15 ø 4
\ T = = = 7.2 N
R 1.5 The radial acceleration is
2
11. (b) 400 = 2pr æ 22 ö
ç 60 ´ ÷
æ 400 ö
2 v 2
è 15 ø
ç ÷ an = = = 3.1
v è 20 ø
2 r 2500
Q tan q = =
rg æ 400 ö \ a = an2 + at2 = 4.14 ft/s2
ç ÷g
è 2p ø
17. (b)
400 ´ 2p 6.28
tan q = = = 0.64 18. (c) FB
400g 9.8 FA FC
-1
Q q = tan ( 0.64)
12. (a) Here centripetal force is provided by force of friction.
Q f = msN
mv 2 mg mg mg
\ f =
r For car A, FA = mg ... (i)
mv 2 mv 2
or msN = For car B, mg - FB = ... (ii)
r r
mv 2 mv 2
or msmg = For car C, FC - mg = ... (iii)
r r
\ v = mgr = 0.8 ´ 10 ´ 8 = 8 m/s
\ FC > FA > FB
M 19. (a) Earth rotates from west to east about its axis
13. (a) dm = dx
L Þ wA > wB …(i)
dF = dm xw2 Using, N - mg = mw2R …(ii)
dx
F F+dF
x
F M 2 L W E
ò0dF =
L
w ò xdx
0
Car B Car A
2
M 2L MLw2
F = w =
L 2 2
14.(c) The tangential acceleration is along the tangent and ac
acts radially inwards. The resultant acceleration From Eqs. (i) and (ii), we get NA > NB
or N1 > N2
a= ac2 + at2 (from figure)
at 20. (b) Suppose body slips at point B
a A
B
30
°

mv 2
ac θ R
c os
g
h θm mg mg sinθ

ac
\ tan 30° = O R
at

@iitjeehelps
CIRCULAR MOTION 71
mv 2 In the frame of rotating spherical chamber (non-inertial)
mg cos q = [v = 2g (R - h )]
R N cos 45° = mr w2 ... (i)
2g (R - h ) N sin 45° = mg ... (ii)
g cos q =
R Dividing Eq. (ii) by Eq. (i), we get
2 (R - h ) mg g
cos q = \ tan 45° = =
R mr w2 rw2
h 2 (R - h ) é hù æ ö
= êë cos q = R úû ç ÷
R R g g
\ w= = ç ÷ = 3.64 rad/s
2 r ç 3R + R ÷
\ h= R
3 è 2ø
mv 2 27. (d) Velocity at the bottom is 2gh
21. (a) N =
R For completing the loop,
msN > mg
2gh = 5 gR
msmv 2 æ Rg ö
< mg or v > ç ÷ Hence, R = 2h /5 = ( 2 ´ 5) / 5 = 2 cm
R è ms ø
28. (b) At the time of slipping, maximum friction acts on the
æ Rg ö body.
\ v min = ç ÷ N
è ms ø
mrω2+µN
22. (c) 23. (b) 24. (b)
25. (d) From the figure mg sinθ
T1 cos 30° mg
+T2 cos 45° mg cosθ
θ
In the frame of rod,
T1 sin 30° mg sin q = mN + mr w2
+T2 sin 45° Also, N = mg cos q
mg \ mg sin q = mmg cos q + mr w2
T1 cos 30° + T2 cos 45° = mg …(i) Here, r = 0.45 m, q = 50°,
mv 2 sin 50° = 0.766 and cos 50° = 0.64
and T1 sin 30° + T2 sin 45° = …(ii) mg sin q - mr w2 g sin q - r w2
r Q m= = = 0.55
mv 2 mg cos q g cos q
mg -
From Eq. (i) and Eq. (ii), we get T1 = r 29. (a)
æ 3 - 1ö
ç ÷ 30. (c) According to given problem,
è 2 ø 1
Mv 2 = as 2 [Here, M = mass]
But, T1 ³ 0 2
mv 2 Þ v =s
2a
mg - …(i)
r ³0 M
Þ T1 =
æ 3 - 1ö v 2 2as 2
ç ÷ So, ag = =
è 2 ø R MR
dv dv ds dv
mv 2 Further more as at = = × =v (By chain rule)
mg ³ Þ v £ rg dt ds dt ds
r
from Eq. (i), we get
v max = rg = 16
. ´ 9.8 = 3.96 m/s
2a
i.e. v =s yields
26. (c) Here, R = 0.2 m M
ω
é 2a ù é 2a ù 2as æ v 2a ö
\ at = ês úê ú= çQ = ÷
N sin45° ë M ûë M û M è s m ø

N So that, a net = aR2 + at2


2
mrω 45°
2as æç
2 2 2ö
N cos45° æ 2as 2 ö æ 2as ö æs ö
R sin45° = ç ÷ +ç ÷ = 1+ ç ÷ ÷
3R è MR ø è M ø M çè èR ø ÷
ø
mg
From figure, r = 3R + R sin 45° \ F = Ma net = 2as 1+ (s / R )2

@iitjeehelps
72 SELF STUDY GUIDE BITSAT

31. (a) The radius of curvature is given by 32. (c) From the diagram,
mv 3
mv mv 3 3
mv 2
r= = = [QF = mg] N - mg cos q =
F ´ v Fv sin q mg v sin q r
v2 where, r = CO = 4.5 - 0.75 = 3.75 m
or r= mv 2
g sin q \ N= + mg cos q
u v=u cosα r
O

α N
θ
2 C
v
\ r min =
g(sin q)max
θ
v2
\ r min = [Q qmax = 90°] mg cos θ
g
mg
q = 90° is only possible when the projectile is at the
From the figure, q = 45° , m = 70 kg, g = 9.8 m/s 2,
maximum height. At the maximum height,
v = u cos a = 10 cos 60° = 5 m/s v ¢ = 6 m/s, r = 4.5 m
5´ 5 Putting these values in above equation, we get
\ r min = = 2.55 m
9.8 N = 1045 N

BITSAT Archives
30 4. (d) As it is clear from figure
1. (c) Angular velocity w = 2pr = 2p ´ = p rad/s
60 R sinθ R
For an elastic spring force F = kx , where x is the
extension θ
Radius of circular motion r = L + x R cosθ

Centripetal force = mrw2 = F


mg
Þ m(L + x )w2 = kx
h
mLw2 0.02 ´ 0.5 ´ ( 3.14)2
Þ x = = θ
k - mw2 2 - 0.02 ´ ( 3.14)2
Radius of the circular motion (r )
R sin q = mg …(i)
= L + x = 0.5 + 0.05 = 0.55 M mv 2
R cos q = …(ii)
Tension in the springT = kx r
= 2 ´ 0.05 = 0.1N Dividing Eq. (i) by Eq. (ii), we get
rg
2. (c) Given, r = 25 m, v = 5 m/s, m = 500 kg \ tan q = 2
v
v2 5 ´5 r rg
at = 1m/s2, ar = = = 1m/s2 or = or v = gh
r 25 h v2
a net = at2 + ar2 1
v = 10 ´ = 1m/s
10
= 12 + 12 = 2 m/s2
5. (d) Angle turned by the body q = q0 + q1t + q2t 2
F = ma net = 500 2 N dq d
Angular velocity, w = = ( q0 + q1t + q2t 2 )
dt dt
3. (d) For looping the loop,v = 5gr
w = q1 + 2q2t
By law of conservation of energy. dw
Loss in potential energy = Gain in kinetic energy Angular acceleration, a =
dt
1
Þ mgh = mv 2 d
a = ( q1 + 2q2t )
2 dt
5r
Þ h= a = 2q2
2

@iitjeehelps
6
Work, Energy and
Power
Work
Work is said to be done when a force applied on a body displaces the body through a certain distance,
in the direction of force.
The work done by the force F in displacing the body through a distance s is given by
W = ( F cos θ )s = Fs cos θ = F ⋅ s
where, F cos θ is the component of the force acting along the direction of the displacement produced.
The SI unit of work is joule (J).
1J = 1 N-m
Work is a scalar quantity. Work can be of three types
(i) Positive work Work is said to be positive, if value of the angle θ between the directions of
F and s is either zero or an acute angle.
(ii) Negative work Work is said to be negative, if value of angle θ between the directions of F and s
is either 180° or an obtuse angle.
(iii) Zero work As work done W = F ⋅ s = F s cos θ , hence work done can be zero if both F and s are
finite but the angle θ between the directions of force and displacement is 90°. In such a case,
W = F ⋅s
= Fs cos θ
= Fs cos 90° = 0

Work Done in Different Conditions


● Work done by a variable force is given by
W = ∫ F ⋅ ds
It is equal to the area under the force-displacement graph, along with proper sign.

@iitjeehelps
74 SELF STUDY GUIDE BITSAT

B Kinetic energy is correlated with momentum as


Force p2
A K= or p = 2 mK
2m
Kinetic energy for a system of particle will be
1
D C K = Σ mivi2
2i
Displacement
Kinetic energy depends on the frame of reference. Kinetic
Work done = Area ABCDA energy of a passenger sitting in a running train is zero in the
● The force (or field) is said to be conservative, if the work frame of reference of the train but is finite in the frame of
done by the force, also referred to as the line integral of reference of the earth.
the force, i.e. ∫ F ⋅ dI is independent of the path followed
between any two points. In such cases, the work done Potential Energy
simply depends upon the initial and the final positions. Potential Energy (PE) is energy of the body by virtue of its
Moreover, work done by a conservative force for a closed position, configuration or state of strain. The relation
path is always zero. Gravitational force, force of gravity, between potential energy and work done is
electrostatic force, elastic force are some examples of W = − ∆U
conservative forces (fields). where, ∆U is change in potential energy.
● The force is said to be non-conservative, if the work Change in potential energy of a body between any two
done by the force depends on the actual path followed by points is equal to the negative of work done by the
the body for a displacement between any two points. conservative force in displacing the body between these
Friction, viscous force, damping force, etc., are some two points, without there being any change in kinetic
examples of non-conservative forces. energy. Thus,
dU = − dW = − F ⋅ dr
and U 2 − U 1 = −W
Energy r2
= −∫ F ⋅ dr
Energy is defined as the capacity or ability of a body to do r1
work. Energy is scalar quantity and its units and Value of the potential energy in a given position can be
dimensions are the same as that of work. Thus, SI unit of defined only by assigning some arbitrary value to the
energy is joule. There are so many types of energy e.g. reference point. Generally, reference point is taken at
kinetic, potential, electrostatic, magnetic, geothermal,
infinity and potential energy at infinity is taken as zero. In
elastic, solar etc. Some of them are described below.
that case,
Some other commonly used units of energy are r
U = − W = − ∫ F ⋅ dr
−7 ∞
1 erg = 10 J
1 cal = 4.186 J ~
= 4.2 J Potential energy is a scalar quantity but has a sign. It may be
1 kcal = 4186 J, 1 kWh = 3.6 × 10 J
6 positive as well as negative.

and 1 electron volt = 1 eV = 1.60 × 10−19 J Generally potential energy is of two types

Gravitational Potential Energy


Kinetic Energy It is the energy associated with the state of separation
Kinetic Energy (KE) is the capacity of a body to do work by between two bodies which interact via the gravitational
virtue of its motion. Motion may be either translational or force. The gravitational potential energy of two particles of
rotational. masses m1 and m2 separated by a distance r is
A body of mass m, moving with a velocity v, has a kinetic −Gm1m2
energy U =
1 r
K = mv 2
2 Generally, one of the two bodies is our earth of mass M and
Thus, K ∝m radius R. If m is the mass of the other body, situated at a
distance r (r ≥ R) from the centre of earth, the potential
K ∝ v2
energy of the body
Kinetic energy of a body is always positive irrespective of GMm
U (r ) = −
the sign of velocityv. Negative kinetic energy is impossible. r

@iitjeehelps
WORK, ENERGY AND POWER 75

Some Important Points


kx PE = mgh
● If a body of mass m is raised to a height h from the surface Gravitational
of earth, the change in potential energy of the system
h potential energy

Force
(earth+body) comes out to be v = mgy
mgh A = 1/2 kx2
∆U = or ∆U ≈ mgh, if h << R
1 + h  y
 
 R Distance x
Thus, the potential energy of a body at height h, dU − d
i.e. mgh is really the change in potential energy of the Fy = − = (mg y )
system for h << R.
dy dy
dU d  1 2
● For the gravitational potential energy, the zero of the Fx = − =−  kx 
potential energy is chosen to be the ground. dx dx  2 
F y = − mg , Fx = − kx
Elastic Potential Energy
Whenever an elastic body (say a spring) is either stretched Electric Potential Energy
or compressed, work is being done against the elastic The electric potential energy of two point charges q1 and q2
1
spring force. The work done is W = kx 2 separated by a distance r in vaccum is given by
2
1 q1q2
where, k is spring constant and x is the displacement. U =
4 π ε0 r
and elastic potential energy
1 1 N -m 2
U = kx 2 where, = 9.1 × 109 = Constant
2 4 π ε0 C2
Elastic potential energy is always positive.

Potential Energy of a Spring


Work-Energy Theorem
According to this theorem, work done by all the forces
Elastic potential energy is Unstretched
PE = 0 (conservative or non-conservative, external or internal)
equal to work done to spring
Elastic acting on a particle or an object is equal to the change in
stretch the spring, which potential kinectic energy of it.
depends upon the spring energy
constant k as well as the PE = 1/2 kx2 i.e. W net =W c + W nc + W ext = ∆KE = K f − Ki
distance stretched. From If work is being done by the force on the body, then its
Hooke’s law, the force kinetic energy increases, i.e. ∆K = + ve. On the other hand,
required to stretch the x if work is being done by the body on the force, then its
spring will be directly
kinetic energy decreases i.e. ∆K = − ve.
proportional to the amount
of stretch. This theorem is true for all type of forces
F = − kx [force by spring] (i) The work-energy theorem is not independent of the
Newton’s second law. It may be viewed as scalar
Then, the work done to stretch the spring by a distance x is
form of second law.
1
W = PE = kx 2 [work done by stretcher] (ii) When a body moves along a circular path with
2 uniform speed, there is no change in its kinetic
Since, the change in potential energy of an object between energy. By the work-energy theorem, the work done
two positions is equal to the work that must be done to by the centripetal force is zero.
move the object from one point to the other, the calculation
of potential energy is equivalent to calculating work. Since,
the force required to stretch a spring changes with distance, Law of Conservation of
the calculation of the work involves an integral.
x x2
Energy
W = ∫ kx dx = k Total mechanical energy of a system remains constant if
0 2
the internal forces are conservative and external forces do
Work can also be visualized as the area under the force
not work. We define the change in potential energy of a
curve. Here, object is displaced slowly.
system corresponding to a conservative internal force as
If the potential energy function U is known, the force at any
f
point can be obtained by taking the derivative of the U f − U i = −W = − ∫ F ⋅ dr
i
potential.

@iitjeehelps
76 SELF STUDY GUIDE BITSAT

W
where, W is work done by internal force on the system as Pav =
system passes from the position i to f . t
Power is a scalar quantity and its SI unit is watt, where
U f − U i = −W = −( K f − Ki ) ⇒ U i + Ki = U f + K f
1W = 1 J /s
If non-conservative internal forces operate within the dW
system or external forces do work on the system, then apply Instantaneous power, Pinst =
dt
work-energy theorem. F ⋅ ds
⇒ W c + W nc + Wext = K f − Ki [Q W c = −(U f − U i )] = = F ⋅v
dt
⇒ W nc + Wext = E f − Ei The power of a system is defined as the rate of change of
kinetic energy per unit time. Mathematically,
d
Power Power, P = (KE )
dt
Power is defined as the rate of doing work. If an agent does dK
work W in time t , then its average power is given by =
dt

Practice Exercise
1. A body of mass 10 kg is moving on a horizontal 7. A force of 0.5 N is applied µ=0.1
surface by applying a force of 10 N in forward on upper block as shown 1kg F=0.5 N
direction. Find the work done by force of friction. in figure. Find the work
a. − 20 J b. 10 J done by lower block on 2 kg
c. 20 J d. − 5 J upper block for a Smooth
displacement 3 m of the
2. In Q. (1), the work done by normal reaction is upper block. (Take, g = 10 m/ s 2)
a. 20 J b. 196 J
c. zero d. None of these a. 1 J b. −1 J
c. 2 J d. − 2 J
3. A body of mass 10 kg is moving on an inclined plane of
inclination 30° with an acceleration 2 m / s 2. The body 8. A force F = − k ( y $i + x $j ), acts on a particle moving in
starts from rest. Find the work done by force of gravity the xy -plane. Starting from the origin, the particle is
in 2 s. taken along the positive x-axis to the point (a , 0) and is
then taken parallel to the y -axis to the point (a , a ). The
a. 10 J b. zero
total work done by the force is
c. 98 J d. 196 J
a. −2 ka 2 b. 2 ka 2
4. In Q. (3), find the work done by force of friction. c. − ka 2 d. ka 2
a. −58 J b. 58 J
c. 98 J d. −116 J 9. A particle moves along a curve of unknown shape but
5. A body of mass 1 kg moves from point A( 2 m, 3 m, 4 m ) magnitude of force F is constant and always acts
B (3 m, 2 m, 5 m). During motion of body, a along tangent to the curve. Then,
force F = ( 2N) $i − ( 4 N) $j acts on it. The work done by a. F may be conservative
the force on the particle during displacement is b. F must be conservative
c. F may be non-conservative
a. 2 $i − 4$j b. 2 J d. F must be non-conservative
c. − 2 J d. None of these
10. If a man of mass M jumps to the ground from a height
6. During swinging of simple pendulum, h and his centre of mass moves a distance x in the
a. the work done by gravitational force is zero time taken by him to hit the ground, the average force
b. the work done by tension force is always acting on him is
zero Mgh Mgx
c. the mechanical energy of bob does not remain a. b.
x h
constant in the absence of air 2 2
h x 
d. the mechanical energy remains constant in the c. Mg   d. Mg  
presence of air resistance x  h

@iitjeehelps
WORK, ENERGY AND POWER 77
11. A particle is moving in a region where the potential U a. 2 mg/x b. 2 mg h/x
is given by U = k (x 2 + y 2 + z 2 ). The force acting on c. 2 mg (h + x ) / x 2 d. 2 mg (h ) + 2mg hx /x 2
the particle is
19. The kinetic energy of a particle moving on a curved
a. k ( $i x + $j y + k$ z ) b. − k ( $i x + $j y + k$ z ) path continuously increases with time. Then,
c. −2k ( $i x + $j y + k$ z ) d. zero a. resultant force on the particle must be parallel to the
velocity at all instants
12. The potential energy is a function for the force b. the resultant force on the particle must be at an angle
between two atoms in a diatomic molecule is given by less than 90° all the time
a b
U (x ) = 12 − 6 , where a and b are positive constants c. its height above the ground level must continuously
x x decrease
and x is the distance between the atoms. The position d. the magnitude of its linear momentum is increasing
of stable equilibrium for the system of the two atoms is continuously
given by e. both b and d are correct
a a 3a  2a  20. Force F acts on a body by mass 1 kg moving with an
a. x = b. x = c. x = d. x = 6  
b b b b initial velocity v 0 for 1 s. Then,
F
13. The potential energy of a particle of mass 5 kg moving a. distance covered by the body isv 0 +
2
in the xy -plane is given by U = ( −7x + 24 y ) J. x and y
b. final velocity of body is (v 0 + F )
being in metre. If the particle starts from rest from
origin then speed of particle at t = 2 s is c. momentum of body increased by F
d. All of the above
a. 5 m/s b. 14 m/s c. 17.5 m/s d. 10 m/s
21. When a bullet of mass 10 g and speed 100 m/s
14. The potential energy of a particle of mass 5 kg moving penetrates upto distance 1 cm in a human body in
in the xy -plane is given by U = − 7x + 24 y J, x and y rest. The resistance offered by human body is
being in metre. Initially at t = 0, the particle is at the a. 2000 N b. 1500 N c. 5000 N d. 1000 N
origin ( 0 , 0) moving with a velocity of 6[ 2.4 $i + 0.7 $j]
22. A 60 g bullet is fired through a stack of fibre board
m/s. The magnitude of force on the particle is
sheet, 200 mm thick. If the bullet approaches the
a. 25 units b. 24 units stack with a velocity of 600 m/s. The average
c. 7 units d. None of these retarding force offered to the bullet is
15. If a compressed spring is dissolved in acid, then a. 54 kN b. 2 kN c. 20.25 kN d. 10 kN
a. the energy of the spring increases 23. A bucket tied to a string is lowered at a constant
b. the energy of acid decreases g
c. the potential energy and kinetic energy of molecules acceleration of . If the mass of the bucket is m and is
of acid increases 4
d. the temperature of acid decreases lowered by a distance d, the work done by the string
will be
16. A long spring, when stretched by x cm has a potential mgd 3
energy U. On increasing the length of spring by a. b. − mgd
4 4
stretching to nx cm, the potential energy stored in the 4 4
spring will be c. − mgd d. mgd
3 3
U U
a. b. nU c. n 2U d. 24. A uniform chain of length l and weight w is hanging
n2 n2
from its ends A and B which are close together. At a
17. Mark the correct option. given instant end B is released. The tension at A when
a. The negative change in potential energy is equal to l
B has fallen a distance x < is
work done 2
b. Mechanical energy of a system remains constant w  3x  w  3x 
a. −2 b. 3−
 l   4 
c. If internal forces are non-conservative, then the net 2 2
work done by internal forces must be zero
w  3x  w  3x 
d. None of the above c. 1+ d. +4
2  l  2  l 
18. A ball of mass m is dropped m n
from a height h on a massless 25. A small sphere of mass m is suspended
platform fixed at the top of a by a thread of length l. It is raised upto the l
vertical spring as shown below. height of suspension with thread fully
The platform is depressed by a stretched and released. Then, the
distance x. What will be the maximum tension in thread will be
value of the spring constant? a. mg b. 2mg c. 3mg d. 6mg

@iitjeehelps
78 SELF STUDY GUIDE BITSAT

26. An object of mass m is tied to a 30. A man weighing 60 kg climbs a staircase carrying a
string of length L and a variable θ 20 kg load on his hand. The staircase has 20 steps
horizontal force is applied on it and each step has a height of 20 cm. If he takes 20 s
which starts at zero and to climb, his power is
F
gradually increases until the m a. 160 W b. 230 W c. 320 W d. 80 W
string makes an angle θ with the
vertical. Work done by the force F is 31. The average human heart forces 4 L of blood per
minute through arteries a pressure of 125 mm. If
a. mgL (1 − sin θ ) b. mgL
the density of blood is 1.03 × 103 kg/m 3 , then the
c. mgL (1 − cos θ ) d. mgL (1 + cos θ )
power of heart is
27. An elastic string of unstretched length L and force a. 112.76 × 10−6 HP b. 112.76 HP
constant k is stretched by a small length x. It is further c. 1.03 × 103 HP d. 1.03 × 10−6 HP
stretched by another small length y. The work done in
the second stretching is 32. A bullet fired into a fixed target losses half of its
1 1 velocity after penetrating distance of 3 cm. How much
a. ky 2 b. k (x 2 + y 2 ) further it will penetrate before coming to rest,
2 2
1 1 assuming that it faces constant resistance to its
c. k (x + y )2 d. ky ( 2x + y ) motion?
2 2
a. 3.0 cm b. 2.0 cm c. 1.5 cm d. 1.0 cm
28. A body of mass m is accelerated uniformly from rest to
a speed v in a time interval T. The instantaneous 33. The power supplied by a force acting on a particle
power delivered to the body as a function of time, is moving in a straight line is constant. The velocity of
given by the particle varies with the displacement x as
mv 2 mv 2 2 1 mv 2 1 mv 2 2 a. x b. x c. x 2 d. x 1/ 3
a. t b. t c. t d. t
T2 T2 2 T2 2 T2 34. A particle of mass m is moving in a circular path of
29. Two small balls of equal masses are joined by a light constant radius r such that its centripetal acceleration
rigid rod. If they are released from rest in the position ac is varying with time t as ac = k 2rt 2. The power is
shown and slide on the smooth track in the vertical mk 4r 2t 5
plane. The speed of balls when A reaches B’s position a. 2πmk 2r 2t b. mk 2r 2t c. d. zero
3
and B is at B ′ is
A 35. A uniform chain of length 2 m is kept on a table such
that a length of 60 cm hangs freely from the edge of
m
0.
5 the table. The total mass of the chain is 4 kg. What is
r= the work done in pulling the entire chain on the table?
B v B′ a. 7.2 J b. 3.6 J c. 120 J d. 1200 J
36. A particle move with a velocity (5 $i − 3 $j + 6k$ ) m/s
under the influence of a constant force
a. 4 m/s F = 10i$ + 10$j + 20k$ . The instantaneous power applied
b. 4.21 m/s
c. 2.21 m/s to the particle is
d. None of the above a. 200 J/s b. 40 J/ s c. 140 J/s d. 170 J/s

BITSAT Archives
1. A light is extensible string that goes over a smooth 2. How much work must be done by a force on 50 kg
fixed pulley as shown in the figure, body in order to accelerate it from rest to 20 m/s in
connect two blocks of masses 0.36 10 s? [2012]
kg and 0.72 kg. Taking g = 10 m/ s 2. a. 103 J b. 104 J c. 2 × 103 J d. 4 × 104 J
Find the work done by string on the
block of mass 0.36 kg during the first 3. An object placed on a ground is in stable equilibrium.
second after the system is released If the objects is given a slight push, then initially the
from rest. [2013] position of centre of gravity [2012]
a. 4J a. moves nearer to ground
b. 2J b. rises higher above the ground
c. 8J c. remains as such
d. 10 J d. may remain at same level

@iitjeehelps
WORK, ENERGY AND POWER 79
4. A shell of mass 20 kg at rest explodes into two a. g (r − H ) b. 2gH
fragments whose masses are in the ratio 2 : 3. The c. g (r − 2H ) d. Zero
smaller fragment moves with a velocity of 6 ms −1. The
kinetic energy of the larger fragment is [2012] 7. A river of salty water is flowing with a velocity 2 m/s. If
a. 96 J b. 216 J c. 144 J d. 360 J the density of the water is 1.2 g/cc, then the kinetic
energy of each cubic metre of water is [2008]
5. The force constant of a spring gun is 50 N/m. If a ball a. 2.4 J b. 24 J c. 2.4 kJ d. 4.8 kJ
of 20 g be shoot by the gun, so that its spring is
compressed by 10 cm, the velocity of the ball is [2011] 8. A ball is released from the top of a tower. The ratio of
a. 5 m/s b. 15 m/s work done by force of gravity in first, second and third
c. 25 m/s d. 20 m/s seconds of the motion of the ball is [2007]
a. 1 : 2 : 3 b. 1 : 4 : 9 c. 1 : 3 : 5 d. 1 : 5 : 3
6. A stone of mass m is pushed down a frictionless
hemispherical bowl of radius r from a point O. The 9. Two springs have their force constant as k1, and k 2
stone just rises up to the
P
r (k1 > k 2 ). When they are stretched by the same force
half of other side' of the [2007]
bowl as shown in the figure, Om a. no work is done in case of both the springs
what is the initial speed of H rl2
b. equal work is done in case of both the springs
stone with which it was c. more work is done in case of second spring
pushed? [2009] d. more work is done in case of first spring

Answer with Solutions


N
Practice Exercise = 10 × 9.8 ×
1
− 10 × 2
f
2
1. (a) Since, body moves with constant velocity. So, net = 49 − 20 = 29 N ° m
30
force on the body is zero. in
∴ W = f ⋅ s = fs cos 180° m gs
N
s = − fs = − 29 × 4 mg cos 30°
30°
= − 116 J
5. (c) The displacement of body is
f F
AB = rB − rA
= ( 3$i + 2$j + 5k$ ) − ( 2$i + 3$j + 4k$ )
= $i − $j + k$
mg
Here, N = mg ∴ W = F ⋅ AB
F =f = ( 2$i − 4$j ) ⋅ ( $i − $j + k$ )
∴ W = f ⋅ s = fs cos 180° =2−4=−2J
= − fs = − 10 × 2 = − 20 J 6. (b) Since, the tension force is in
2. (c)QW = F ⋅ s = N ⋅ s perpendicular direction of instantaneous
= Ns cos 90° = 0 displacement of bob. Hence, work done T
1 2 1 is zero. [QW = ∫ Fds cos 90° = 0]
3. (d)s = at = × 2 × 4 = 4 m 90°
2 2 7. (b) From force diagram shown in figure.
∴ Work done by force of N
ds
N
gravity
=W = m g⋅ s m

= mg s cos 60° s in3
1 mg 60° m=1 kg F=0.5 N
= 10 × 9.8 × 4 × = 196 J mg cos30° fmax=µN
2 s 30° mg
4. (d) From force diagram,
mg sin 30° − f = ma mg
∴ f = mg sin 30° − ma ∴ fmax = µN = µmg = 0.1 × 1 × 10 = 1 N

@iitjeehelps
80 SELF STUDY GUIDE BITSAT

Since, F < f 15. (c) The spring loses elastic potential energy which
So, both blocks move together. The f appears as energy of molecules of acid.
acceleration of system is 1
2 kg 16. (c) Initially potential energy = kx 2
F 0.5 5 1 2
a= = = = m/s2
1+ 2 3 30 6 1 2U
⇒ U = kx 2 or 2U = kx 2 ⇒ k = 2
For lower block, 2 x
1 1 When it is stretched to nx cm.
f = 2a = 2 × = N
6 3 1 1 2U
∴ PE = kx12 = × 2 × n 2x 2 = n 2U
So, the force of friction due to lower block on upper block is 2 2 x
1
N in the opposite direction of displacement of the block. ∴ Potential energy stored in the spring = n 2U
3
∴ Work done by lower block on upper block is 17. (d) (a) This statement is correct only when forces are of
1 conservative nature.
W = − fs = − × 3 = − 1J (b) In the absence of non-conservative force, mechanical
3
a a energy and non-mechanical energy are separately
8. (c)W1 = ∫ F dx = ∫ −k (y $i + x$j )$i dx constant.
0 0
a (c) The work done by internal forces may not be zero.
= ∫ −k ( 0$i + x$j )$i dx = zero
0 1
a a
18. (c) Here, mg(h + x ) = kx 2
W2 = ∫ F ⋅ dy = ∫ −k (y $i + x$j )$j dy 2
0 0 2mg(h + x)
a a ⇒ kx 2 = 2mg(h + x ) ⇒ k =
=∫ −k (a $i + a$j )$j dy = − ka ∫ dy = − ka 2 x2
0 0
19. (e) If particle moves on a curved path, then there are two
Total work done,
components of force
W = W1 + W2 = 0 − ka 2 = − ka 2
(i) Tangential Component This component
9. (d) Since, work done by this force depends upon path provides tangential acceleration.
followed by the particle between two points. Hence, the Due to this, speed increases.
force must be non-conservative. (ii) Radial Force This component provides centripetal
∆U ( −Mgh ) Mgh acceleration.
10. (a) Here, F = − =− =
∆x x x v
Ft
11. (c)U = k (x + y + z ) = kr
2 2 2 2
θ<
90
dU − d °
F =− = (kr 2 )
dr dr
^ ^ ^
F
−2 kr = − kr = − 2k ( i x + j y + k z )
Fr
a b
12. (d)Q U (x ) = −
x 12 x 6 The resultant force is
At the stable equilibrium, F = Fr 2 + Ft 2
1/ 6
dU 12a 6b  2a  Fr
= 0 ⇒ − 13 + 7 = 0 ⇒ x =   tan θ =
dx x x b Ft
∂U $ ∂U $
13. (d) F = − i− j = 7$i − 24$j But Ft is not zero.
∂x ∂y ∴ tan θ < ∞ ⇒ θ < 90°
Fy 7 When speed increases, p = mv increases.
∴ ax = = = 1.4 m/s2
m 5 F F
Fy 24 20. (d) a = = = F m/s2
ay = =− m 1
m 5 v = u + at
= 4.8 m/s2 along negative y -axis v = v 0 + (F × 1) ⇒ v = v 0 + F
∴ v x = a x t = 14
. ×2 where,v = final velocity
= 2.8 m/s along negative y -axis 1
s = ut + at 2
and v y = 4.8 × 2 = 9.6 m/s 2
F
∴ v = v x2 + v y2 = 10 m/s s =v0 +
2
∂U $ ∂U $ pi (initial momentum) = mv 0 − v 0
14. (a) F = − i− j = 7$i − 24 $j
∂x ∂y pf (final momentum) = mv = v 0 + F
∴ | F | = ( 7)2 + ( −24)2 = 25 units Change in momentum, ∆p = pf − pi = F

@iitjeehelps
WORK, ENERGY AND POWER 81
21. (c) Here, u = 100 m/s,v = 0,s = 1cm = 10−2 m Tension at A = Weight of half the chain
x
∴ v = u − 2as
2 2
+ Weight of length + F
2
u2
100 × 100 w wx wx w  3x 
∴ a= = = 5 × 105 m/s2 = + + = 1+
2s 2 × 1 × 10−2 2 2l l 2  l 
∴ F = ma = 10 × 10−3 × 5 × 105 = 5000 N 25 (c) The tension in string is l
O u=o
Alternate method maximum at the lowest position
1 1 of the ball.
W = ∆K or − Fs = mv 2 − mu 2 Tmax
2 2 mv 2
∴ Tmax − mg =
1
= 0 − × 10 × 10−3 × 10000 l
2 mv 2 v
1 ∴ Tmax = mg + …(i)
or −F × 10−2 = − × 10 × 10−3 × 104 l mg
2
The loss in potential energy =
∴ F = 5000 N
gain in kinetic energy
60
22. (a) m = 60 g = = 0.06 kg or
1
mgl = mv 2
1000 2
s = 200 mm = 0.2 m ∴ v = 2gl …(ii)
u = 600 m/s 2
v =0 mv m
Q = 2gl = 2mg …(iii)
Using v 2 = u 2 + 2as l l
From Eqs. (i), (ii) and (iii),
( 0)2 = ( 600)2 + 2 × a × 0.2
Tmax = mg + 2mg = 3mg
−600 × 600 × 10
a= 26. (c)
0.4
L
Average retarding force L cosθ
0.06 × 600 × 600 × 10
F = ma =
4 L
F = 54 kN h
23. (b) From force diagram as shown in T
figure, mg − T = ma
mg 3mg
∴ T = mg − ma = mg − = a = g/4
4 4 W = ∆K or WT + Wg + WF = 0
QWT = work done by tension s [Since, change in kinetic energy is zero]
= T ⋅ s = Ts cos 180° = −
3mgd
mg Here,WT = work done by tension = 0
4 Wg = work done by force of gravity
m w = − mgh = − mgL (1 − cos θ )
24. (c) Mass per unit length, λ = =
l lg ∴ WF = − Wg = mgL (1 − cos θ )
x 
Velocity,v = 2g   orv = gx
2
27. (d) Elastic force in string is conservative in nature.
 2
∴ W = − ∆U
Change in momentum when an element dx falls is
w where,W = work done by elastic force of string
gx dx ∆U = change in elastic potential energy
lg
dp w dx w  dx  Q W = − (Uf − Ui ) = Ui − Uf
F = = gx = x Q = gx
dt lg dt l  dt  1 1
or W = kx 2 − k (x + y )2
A B A 2 2
1 2 1
or W = kx − k (x 2 + y 2 + 2xy )
2 2
1 2 1 2 1 2 1
l/2 x = kx − kx − ky − k ( 2xy )
2 2 2 2
1 2 1
= − kxy − ky = ky ( −2x − y )
B 2 2
The work done against elastic force is
x/2 ky
Wexternal = − W = ( 2x + y )
2

@iitjeehelps
82 SELF STUDY GUIDE BITSAT

mv  v − 0  d 2x   dx 
28. (a) F = ma = Qa = 33. (d )QP = Fv = (ma ) v = m  2   
T  T   dt   dt 
mv
Instantaneous power = Fv = mav = at Since, power is constant.
T
mv v mv 2  d 2x   dx 
= t= 2 t ∴  2   =k
T T T  dt   dt 
2 2
29. (c) The final speeds of each ball are same. d  dx   dx 
or   = k or   = k1t
∴ Loss in PE by ball A = gain in KE by the system dt  dt   dt 
1 1 dx
or mgr = mv 2 + mv 2 or = k1t = k 2t 1/ 2 or x = k 3t 3/ 2
2 2 dt
or mgr = mv 2 dx
Hence, ∝ t 1/ 2 ∝ x 1/ 3
∴ v = gr = 9.8 × 0.5 = 4.9 dt
v2
 49  34. (b) ac = = k 2rt 2
=   = 2.21 m/s r
 10 
Q v = krt
( 60 + 20) × (10) × ( 20 × 0.2)
30. (a) Power = = 160 W The tangential acceleration is
20 dv d (krt )
a1 = = = kr
31. (a) Hight of liquid column, h = 125 mm = 0.125 m dt dt
Density of blood, ρ = 103
. × 103 kg/m 3. The work done by centripetal force will be zero.
Accleraction due to gravity, g = 9.8 m/s2 So, power is delivered to the particle by only tangential
force which acts in the same direction of instantaneous
Volume of blood,V = 4 litre = 4 × 10− 3 m3 velocity.
Time, t = 1minute = 60 second. ∴ Power = Fv t
Work done,W = force × distance = mat krt
= pressure × area × distance = m(kr )(krt ) = mk 2r 2t
= pressure × volume M 4
35. (b) Mass per unit length = = = 2 kg m–1
= hρgV L 2
W hρgV
Power, P = = 1.4 m
t t
. × 103 × 9.8 × 4 × 10− 3
0.125 × 103
=
60

0.6 m
= 112.76 × 10− 6 HP
32. (d) According to the work-energy theorem,
W = ∆K
1 v 0 
2
1 The mass of 0.6 m of chain = 0.6 × 2 = 12
. kg
Case I −F × 3 = m   − mv 02
2  2  2 The height of the centre of mass of the hanging part
where, F is the resistive force andv 0 is the initial speed. 0.6 + 0
h= = 0.3 m
Case II Let the further distance travelled by the bullet 2
before coming to rest iss . Hence, work done in pulling the chain on the table =
1 work done against gravity the force of gravity
∴ −F ( 3 + s ) = Kf − Ki = − mv 02
2 i.e. W = mgh
1 1
⇒ − mv 02( 3 + s ) = − mv 02 = 1.2 × 10 × 0.3 = 3.6 J
8 2
1 36. (c) Pinst = F ⋅ v
or (3 + s ) = 1
4 = (10$i + 10$j + 20k$ ) ⋅ (5$i − 3$j + 6k$ )
3 s
or + =1 = 50 − 30 + 120
4 4
or s = 1 cm = 140 J/s

@iitjeehelps
WORK, ENERGY AND POWER 83

BITSAT Archives
1. (c) According to figure v (velocity of the larger fragment) = 4 m/s
1
Kinetic energy = mv 2
2
1
= × 12 × ( 4)2 = 96 J
a 2
1 1 2
a T 5. (a) mv = kx
2
T 2 2
0.36 kg R
0.72 kg v = x
m
0.72 g
50
So, acceleration =
Net pulling force = (10 × 10−2 )
Total mass 20 × 10−3
0.72g − 0.36g g = 50 × 10−1
= =
0.72 + 0.36 3
= 5 m/s
1 2
Distance,s = ut + at 6. (c) Let the velocity with which stone is pushed = u.
2
1 g g We have,
= 0 + × × (1)2 = 1 r
2 3 6 mu 2 + mgH = mg
So, T − mg = ma 2 2
T − 0.36 g = 0.36 a r 
u = 2g  − H
T = 0.48 g 2 
Now, work done by string on the block or u = g (r − 2H )
WT = Ts cos 0° (on 0.36 kg of mass)
7. (c) Given, velocity of river, (v ) = 2 m/s
g
= ( 0.48 g )   (1) = 0.08 g 2 Density of water ρ = 12
 6 . g/cc
= 0.08 × (10) = 8 J 2 Mass of each cubic metre
1.2 × 10−3
1 2 m=
2. (b) Now, s = ut + at (10−2 )3
2
1 = 1.2 × 103 kg
s = 0 + × 2 × 10 × 10
2 1
∴ Kinetic energy = mv 2
s = 100 m 2
Hence, work done 1
= × 1.2 × 103 × ( 2)2 = 2.4 kJ
W = F ×s 2
W = ma × s 8. (c) When the ball is released from the top of tower, then
∴ W = 50 × 2 × 100 ratio of distance covered by the ball in first, second and
W = 10000 = 104 J third seconds is
hI : hII : hIII = 1: 3 : 5 [because hn ∝ ( 2n − 1)]
3. (b) In stable equilibrium, the centre of gravity of object,
∴ Ratio of work done
lies at minimum height from ground. As the object is given
a slight push, its centre of gravity rises because it comes mghI : mghII : mghIII = 1: 3 : 5
2
in unstable equilibrium. F
9. (c)W =
4. (a) Total mass of the shell = 20 kg 2k
1
Ratio of the masses of the fragments = 2 : 3 If both springs are stretched by same force, thenW ∝
K
∴ Masses of the fragments are 8 kg and 12 kg. Now,
As K1 > K 2
according to the conservation of momentum
Therefore,W1 > W2
m1v1 = m2v 2
∴ 8 × 6 = 12 × v i.e. more work is done in case of second spring.

@iitjeehelps
7
Centre of Mass,
Momentum and
Collision
Centre of Mass
The centre of mass of a body is a point where the whole mass of the body is supposed to be
concentrated for describing its translatory motion. In a uniform gravitational field, the centre of mass
and the centre of gravity of a system are coincident.

Centre of Mass of Two Particles System


Consider the two particles system in which two masses m1 and m2 are P R Q
m1 m2
separated by a distance d. CM
d
The Centre of Mass (CM) divides internally the line joining two particles Centre of mass of two
(or masses) in inverse ratio of their masses. particles system
m2 d m1 d
PR = and QR =
m1 + m2 m1 + m2

Centre of Mass of N-Particles System


If there are N-particles distributed in space whose masses are m1 , m2 , m3 ,…,mN and position vectors
r1 , r2 , r3 , K, rN respectively, then the coordinates of the centre of mass are given as below :
m1 x 1 + m2 x 2 + m3 x3 +…+ mN x N m y + m2 y 2 + m3 y3 +…+ mN y N
XCM = , YCM = 1 1
m1 + m2 + m3 +…+ mN m1 + m2 + m3 +…+ mN

m1 z 1 + m2 z 2 + m3 z3 +…+ mN z N
ZCM =
m1 + m2 + m3 +…+ mN

@iitjeehelps
CENTRE OF MASS, MOMENTUM AND COLLISION 85
We can also write as, Coordinates of centre of mass of a rigid body are
1 1
M∫ M∫
XCM =
1
Σ mi xi ; YCM =
1
Σ mi yi ; ZCM =
1
Σ mi zi XCM = x dm, YCM = y dm
M M M
1
M∫
Y
mi (xi, yi, zi) and ZCM = z dm
N-particles

CM (xCM, yCM, zCM) where, M is total mass of the rigid body and x , y and z are
X
the coordinates of elementary mass dm.

Centre of Mass of Some Rigid Bodies


Z
1. Centre of mass of a uniform rectangular, square or
Centre of mass of -particles system
circular plate lies at its centre.
where, M = m1 + m2 + m3 +…+ mN
The position vector of centre of mass of N-particles system
is given by
CM
m r + m2 r2 + m3 r3 +…+ mN rN CM CM
rCM = 1 1
m1 + m2 + m3 +…+ mN
2. Centre of mass of a uniform semi-circular ring lies at a
2R
Important Formulae of Centre of Mass distance of h = from its centre on the axis of
π
● Velocity of centre of mass of N-particles system is given by
n
symmetry where R is the radius of the ring.
Σ mi vi
i =1
vCM =
M
● Similarly, acceleration of centre of mass is given by R CM 2R
n π
Σ mi a i
i =1
a CM = O
M
● In accordance with Newton’s second law of motion 3. Centre of mass of a uniform semi-circular disc of
FCM = F1 + F2 + K + Fn radius R lies at a
n 4R
or FCM = Σ Fi distance of h =
i =1 3π
R CM 4R
● This is called as equation of motion of centre of mass. from the centre on 3π
In pure translation, every particle of the body moves with the the axis of symmetry
same velocity at any instant of time. as shown in figure. O

4. Centre of mass of a hemispherical shell of radius R lies


R
at a distance of h = from
Rigid Body 2
its centre on the axis of R CM R
It is a continuous solid body of finite size in which
symmetry as shown in 2
deformation is not possible, i.e. the relative distance
between the two given points within the rigid body is figure. O
always constant whatever the force is applied on it. 5. Centre of mass of a solid hemisphere of radius R lies at
Practically, there is no existence of it. In order to assume a 3R
a distance of h = from its centre on the axis of
body to be completely rigid, deformation would be 8
neglected. symmetry. If some mass or
area is removed from a
For a rigid body which is not under Y Rigid body rigid body, then the R CM 3R
any external force, centre of mass is dm position of centre of mass 8
always a fixed point while in case of (x,y,z) of the remaining portion is
body which is not rigid, the centre of O
obtained from the
mass can vary. O
X following formula
m1 r 1 − m2 r 2 A r − A2 r 2
r CM = or r CM = 1 1
Z m1 − m2 A1 − A2

@iitjeehelps
86 SELF STUDY GUIDE BITSAT

In rotation about a fixed axis, every particle of the rigid Perfectly Elastic Collision in a Plane
body moves in a circle which is in a plane
perpendicular to the axis and has its centre on the axis. In a two-dimensional (or oblique) collision between two
bodies, momentum remains conserved.
v1
Collision m1
It is defined as an isolated event in which two or more A
colliding bodies exert relatively strong forces on each other A B θ
for a relatively short time, as a result of which the kinetic m1 u1
energy and momentum of the colliding particles change. u2 φ
m2
Before collision
B
Types of Collision m2
There are two basic types of collision in one dimension as v2
given below : After collision
(i) Elastic collision The collision is said to be elastic, if ∴ Along the x-axis
both the bodies regain their original shape and size
completely, after they are separated. Also, the total m1u 1 + m2u 2 = m1v1 cos θ + m2v2 cos φ … (i)
linear momentum as well as kinetic energy of bodies and along the y-axis
remain conserved. 0 = m1v1 sin θ − m2v2 sin φ … (ii)
m2 m1 m2 m1 As the total kinetic energy remains unchanged.
v2 v1 v2′ v1 ′
1 1 1 1
Hence, m1u 12 + m2u 22 = m1v12 + m2v22 …(iii)
(a) Before collision (b) After collision 2 2 2 2
We can solve these equations provided that either the
Let the two balls of mass m1 and m2 collide each value of θ or φ is known to us.
other elastically with their respective velocities v1
and v2 in the same direction as shown in figure. Their
Rebounding of a Ball on Collision with
velocities become v1 ′ and v2 ′ after the collision along
the same line. Applying conservation of linear the Floor
momentum, we get 1. Speed of the ball after the nth rebound
m1v1 + m2v2 = m1v1 ′+m2v2 ′ …(i) vn = e nv0 = e n 2 gh0
In an elastic collision, kinetic energy before and after 2. Height covered by the ball after the n th rebound
collision is also conserved. Hence,
hn = e 2 nh0
1 1 1 1
m1v12 + m2v22 = m1v1 ′ 2 + m2v2 ′ 2 …(ii)
2 2 2 2 3. Total distance (vertical) covered by the
ball before it stops bouncing
Solving Eqs. (i) and (ii) for v1 ′ and v2 ′, we get
1 + e2 
 m − m2   2m2  H = h0 + 2h 1 + 2h 2 + 2h 3 + . . . = h0  
v1 ′ =  1  v1 +   v2 …(iii) 1 − e 
2
 1
m + m 2  m1 + m2 
 m − m1   2m1  4. Total time taken by the ball before it stops bouncing
and v2 ′ =  2  v2 +   v1 …(iv)
 m1 + m2   m1 + m2  T = t0 + t 1 + t 2 + t3 + . . .
2 h0 2 h1 2 h2 2 h0  1 + e 
(ii) Inelastic collision The collision is said to be = +2 +2 +... =  
inelastic, if the bodies do not return to their original g g g g 1 − e
form after the collision. In this type of collision, the
total linear momentum of the system of bodies will
remain conserved but kinetic energy of the system is
not conserved.
Coefficient of Restitution (e)
i.e. p initial = p final For a collision, it is defined as the ratio of relative velocity of
separation to the relative velocity of approach.
⇒ ∆p = 0
v − v1
But, K initial ≠ K final Thus, coefficient of restitution, e = 2
u1 − u2
⇒ ∆K ≠ 0
For a perfectly elastic collision, e = 1.

@iitjeehelps
CENTRE OF MASS, MOMENTUM AND COLLISION 87
If 0 < e < 1, the collision is said to be partially elastic. But if, (b) Two bodies A and B having masses m1 and m2 have
e = 0, the collision is said to be perfectly inelastic. equal kinetic energies. If they have velocities v1 and v2 ,
● In a perfectly inelastic collision, e = 0 which means that then
v2 − v1 = 0 or v2 = v1 v1 m2 p m 
= and 1 =  1 
● It can be shown that for an inelastic collision, the final v2 m1 p2  m2 
velocities of the colliding bodies are given by
where, p1 and p2 are their momenta.
 m − em2  (1 + e )m2
v1 =  1  u1 + u2 (c) If a bullet of mass m1 moving with a velocity u, strikes a
 m1 + m2  (m1 + m2 )
mass m2 which is free to move in the direction of the
(1 + e )m1  m − em1  bullet and is embedded in it, then the loss of kinetic
and v2 = u1 +  2  u2
(m1 + m2 )  m1 + m2  1  m m u2 
energy is  1 2 .
● If a particle of mass m, moving with velocity u , hits an 2  m1 + m2 
identical stationary target inelastically, then final
(d) A shell of mass m1 is ejected from a gun of mass m2 by
velocities of projectile and target are correlated as
an explosion which generates kinetic energy equals to
i.e. m1 = m2 = m
v 1−e E. Then, the initial velocity of the shell is
and u2 = 0 ; 1 =
v2 1 + e  2m2 E 
m (m + m )
 1 1 2 

Important Concepts Regarding (e) A gun of mass m2 fires a shell of mass m1 horizontally
the Collision and the energy of explosion is such as would be
sufficient to project the shell vertically to a height h.
(a) Two bodies of mass m1 (heavy) and mass m2 (light) are
Then, the velocity or recoil of the gun is
moving with same kinetic energy. If they are stopped
by the same retarding force, then  2m12 gh 
 
(i) The bodies cover the same distance before coming m2 (m1 + m2 )
to rest.
(f) A bullet of mass m1 penetrates a thickness of a fixed
(ii) The time taken to come to rest is lesser for m2 as it plate of mass m2 . If m2 is free to move and the
p
has less momentum, i.e. t = . resistance is supposed to be uniform, then the
F m2s
thickness penetrated is .
(iii) The time taken to come to rest is more for m1 as it m1 + m2
has greater momentum.

Practice Exercise
1. Two homogeneous spheres A and B of masses m and 3. A non-uniform thin rod of length L is placed along
2m having radii 2a and a respectively, are placed in X -axis as such its one of end is at the origin. The
touch. The distance of centre of mass from first linear mass density of rod is λ = λ 0 x. Find the
sphere is distance of centre of mass of rod from the origin.
a. a b. 2a a. L / 2
c. 3a d. None of these b. 2L / 3
c. L / 4
2. A circular hole of radius 1 cm is cut-off from a disc of
radius 6 cm. The centre of hole is 3 m from the centre d. L / 5
of the disc. Find the position of centre of mass of the 4. Which of the following has centre of mass not situated
remaining disc from the centre of disc. in the material of body?
3 1
a. − cm b. cm a. A rod bent in the form of a circle
35 35 b. Football
3 c. Hand ring
c. cm d. None of these
10 d. All of the above

@iitjeehelps
88 SELF STUDY GUIDE BITSAT

5. A ball kept in a closed box moves in the box making 11. In the given figure, the mass m 2
m2
collisions with the walls. The box is kept on a smooth starts with velocity v 0 and moves
surface. The centre of mass with constant velocity on the
a. of the box remains constant surface. During motion, the m1
b. of the box plus the ball system remains constant normal reaction between the
c. of the ball remains constant horizontal surface and fixed θ A
d. of the ball relative to the box remains constant triangular block m1 is N. Then,
6. A man of mass M stands at one end of a plank of during motion
length L which lies at rest on a frictionless surface. a. N = (m1 + m2 ) g b. N = m1g
The man walks to the other end of the plank. If the c. N < (m1 + m2 ) g d. N > (m1 + m2 ) g
mass of plank is M / 3, then the distance that the mass
moves relative to the ground is 12. Four particles of masses 1 kg, 2 kg, 3 kg and 4 kg are
placed at the corners A, B , C and D respectively of a
a. 3L / 4 b. L / 4 c. 4L / 5 d. L / 3
square ABCD of edge 1 m. If point A is taken as
7. The centre of mass of a system cannot change its origin, edge AB is taken along X -axis and edge AD is
state of motion, unless there is an external force taken along Y -axis. Find the coordinates of centre of
acting on it. Yet the internal force of the brakes can mass in SI
bring a car to rest. Then a. (1, 1) b. (5, 7)
a. the brakes stop the wheels c. (0.5, 0.7) d. None of these
b. the friction between the brake pads and the wheel
stops the car
13. Three bodies having masses 5 kg, 4 kg and 2 kg are
c. the car is stopped by the road moving at the speed of 5 m/ s, 4 m/s and 2 m/s,
d. the car is stopped by the driver pressing the pedal respectively along X-axis. The magnitude of velocity
of CM is
8. Two blocks A and B are connected by a massless a. 1.0 m/s b. 4 m/s
string (shown in figure) A force of 30 N is applied on
c. 0.9 m/s d. 1.3 m/s
block B. Find the
B
distance travelled by 10 kg 20 kg 14. Figure shows the situation involving two blocks of
F=30 N
centre of mass in 2 masses m1 and m 2. The
Smooth m2
s starting from rest. heavier block of mass m1 is
a. 1 m b. 2 m pulled by a force F. The m1 F
c. 3 m d. None of these acceleration of CM is (all the
9. A loaded spring gun of mass M fires a shot of mass m surfaces are smooth)
with a velocity v at an angle of elevation θ. The gun is F F F 2F
a. b. c. d.
initially at rest on a horizontal frictionless surface. m2 m1 m1 + m2 m1m2
After firing, the centre of mass of the gun-shot system
15. A particle is projected such that its horizontal range
m
a. moves with a velocityv would be R. At the highest point, the particle breaks
M into two identical parts P and Q. If P comes to rest, the
vm
b. moves with a velocity cos θ in the horizontal horizontal distance of point from the point of projection
M (i.e. origin) where the particle Q lands on the ground is
direction
c. remains at rest a. 1.5 R b. R
v (M − m ) c. 2.5 R d. 2R
d. moves with a velocity in the horizontal
(M + m ) 16. A string of negligible thickness is wrapped several
direction times around a cylinder kept on a rough horizontal
surface. A boy standing at a
10. Two bodies A and B of masses m1 and m 2 respectively
distance l from the cylinder holds
are connected by a massless spring of force constant one end of the string and pulls the
k. A constant force F k cylinder towards him. Assuming
starts acting on the m2 m1 F
no slipping, the length of the
body A at t = 0. Then, A B thread passed through the hands
a. at every instant, the of the man is
F
acceleration of centre of mass is l 3l
m1 + m2 a. b. l c. 2l d.
2 2
b. at t = 0, acceleration of B is zero but that of A is
maximum 17. If momentum of a body remains constant. Then
c. the acceleration of A decreases continuously and mass-speed graph of body is
finally becomes equal a. circle b. straight line
d. All of the above c. rectangular hyperbola d. parabola

@iitjeehelps
CENTRE OF MASS, MOMENTUM AND COLLISION 89
18. Two bodies of masses m and 4m are moving with a. they reach at ground with same magnitude of
equal linear momentum. The ratio of their kinetic momenta if bodies have same masses
energies is b. they reach at ground with same kinetic energy
a. 1 : 4 b. 4 : 1 c. they reach at ground with same speed
c. 1 : 1 d. 1 : 2 d. both (a) and (c) are correct

19. A man is sitting in a moving train, then 26. If a ball is dropped from rest. It bounces from the floor
respectively. The coefficient of restitution is 0.5 and
a. his momentum must not be zero
the speed just before the first bounce is 5 m/s. The
b. his kinetic energy is zero
c. his kinetic energy is not zero
total time taken by the ball to come to the rest is
d. his kinetic energy may be zero a. 2 s b. 1 s c. 0.5 s d. 0.25 s

20. When a meteorite burns in the atmosphere. Then, 27. Three identical blocks A, B and C are placed on
a. the momentum conservation principle is applicable to horizontal frictionless surface. The blocks B and C are
the meteorite system in rest. But A is approaching towards B with a speed of
b. the energy of meteorite remains constant 10 m/s. The coefficient A B C
c. the conservation principle of momentum is applicable of restitution for all
to a system consists of meteorites, earth and air collision is 0.5. The
molecules speed of the block C
d. the meteorite momentum remains constant just after collision is
21. If a bullet is fired from a gun, then a. 5.6 m/s b. 6 m/s c. 8 m/s d. 10 m/s
a. the mechanical energy of bullet-gun system remains 28. A thin uniform bar lies on a frictionless horizontal
constant surface and is free to move in any way on the surface.
b. the mechanical energy is converted into Its mass is 0.16 kg and length is 1.7 m. Two particles
non-mechanical energy each of mass 0.08 kg are moving on the same surface
c. the mechanical energy may be conserved and towards the bar in the direction perpendicular to
d. the non-mechanical energy is converted into the bar, one with a velocity of 10 m/s and other with
mechanical energy velocity of 6 m/s. If collision between particles and bar
22. A nucleus moving with a velocity n emits an α-particle. is completely inelastic. Both particles strike with the
Let the velocities of the α-particle and the remaining bar simultaneously. The velocity of centre of mass
nucleus be n1 and n2 and their masses be m1 and m 2, after collision is
then a. 2 m/s b. 4 m/s c. 10 m/s d. 16 m/s
a. n, n1and n2 must be parallel to each other 29. When two bodies collide elastically. The force of
b. none of the two of n, n1, n2 should be parallel to each interaction between them is
other.
a. conservative
c. n1 + n2 must be parallel to n
b. non-conservative
d. m1 n1 + m2 n2 must be parallel to n
c. either conservative or non-conservative
23. Two blocks of mass m1 and m 2 are connected by a d. zero
massless spring and placed at smooth surface. The
spring initially stretched and released. Then, 30. A body of mass M moving with a speed u has a
head-on collision with a body of mass m originally at
a. the momentum of each particle remains constant
separately rest. If M > > m , then the speed of the body of mass m
b. the momentum of both bodies are same to each other after collision will be nearly
c. the magnitude of momentum of both bodies are same um uM u
a. b. c. d. 2u
to each other M m 2
d. the mechanical energy of system remains constant 31. A ball moving with a certain velocity hits another
e. Both (c) and (d) are correct identical ball at rest. If the plane is frictionless and
24. When two blocks A and B coupled by a spring on a collision is elastic, then the angle between the
frictionless table are stretched and then released, then directions in which the balls move after collision,
will be
a. kinetic energy of body at any instant after releasing is
inversely proportional to their masses a. 30° b. 60° c. 90° d. 120°
b. kinetic energy of body at any instant may or may not 32. A shell is fired from a cannon with a velocity v at an
be inversely proportional to their masses angle θ with the horizontal direction. At the highest
KE of A mass of B point in its path, it explodes into two pieces, one
c. = , when spring is massless
KE of B mass of A retraces its path to the cannon and the speed of the
d. Both (b) and (c) are correct
other pieces immediately after the explosion is
a. 3v cos θ b. 2v cos θ
25. Two bodies are projected from roof with same speed
 3 3
in different directions. If air resistance is not taken into c.   v cos θ d. v cos θ
account. Then,  2 2

@iitjeehelps
90 SELF STUDY GUIDE BITSAT

33. A smooth steel ball strikes a fixed smooth steel plate 37. A ball collides with another e=1/2
at an angle θ with the vertical. If the coefficient of identical ball which was v
restitution is e, then the angle at which the rebounce initially at rest. The speed I m II m rest
will take place is
of the second ball after u=0
 tan θ 
a. θ b. tan−1 collision is
 e  v 3v v v
 e  a. b. c. d.
c. e tan θ d. tan−1 
4 5 5 2
 tan θ  38. A 2 kg body moving
34. A machine gun fires 120 shots per minute. If the mass at a speed of 6 m/s
of each bullet is 10 g and the muzzle velocity is collides with another µ=0
800 m/s, then the average recoil force on the machine body of 9 kg at rest
and sticks together. v
gun is m⇒ M
The loss of kinetic
a.120 N b. 8 N µ=0
energy would be
c. 16 N d. 12 N
a. 30 J b. 40 J c. 33 J d. 32 J
35. A machine gun fires a steady stream of bullets at the
rate of n per minute into a stationary target in which 39. A bullet collides with a block which was initially at rest
the bullets get embedded. If each bullet has a mass m and embedded in it. Upto what height from the
horizontal, the block bullet system will rise?
and arrive at the target with a velocity v , the average
M 2v 2 m 2v 2
force on the target is a. b.
60v 2 (m + M )2g 2 (m + M )2g
a. 60mnv b. mv Mv
mn c. d.
mnv mv 2 (m + M ) 2 (m + M )
c. d.
60 60n
40. A heavy sheet of 2v
36. Two boys of masses 10 kg and 8 kg are moving along wood and a light ball
v M>>>m
a vertical rope, the former climbing up with is moving towards
M
acceleration of 2 m/s 2 while later coming down with each other as shown m
e = 1/3 Wall
uniform velocity of 2 m/s. Then, tension in rope at in the figure. What
fixed support will be (Take g = 10 m/s 2) will be the speed of
the ball after collision?
a. 200 N b. 120 N
a. 3v b. 2v c. v d. v /2
c. 180 N d. 160 N

BITSAT Archives
1. In completely inelastic collision, [2013] a. A bullet fired into a block, if bullet gets embedded into
a. the complete KE of the medium must lost block
b. the linear momentum of the system must remain b. Capture of an electron by an atom
conserved during collision c. A man jumping onto a moving boat
c. Both (a) and (b) are correct d. A ball bearing striking another ball bearing
d. Both (a) and (b) are incorrect 5. A sphere of mass m moving with a constant velocity u
2. A shell of mass 20 kg at rest explodes into two hits another staionary sphere of the same mass. If e is
fragments whose masses are in the ratio 2 : 3. The the coefficient of restitution, then the ratio of the
smaller fragment moves with a velocity of 6 ms −1. The velocity of two spheres after collision will be [2010]
kinetic energy of the larger fragment is [2012] 1− e 1+ e
a. b.
a. 96 J b. 216 J 1+ e 1− e
c. 144 J d. 360 J e+1 e −1 2
c. d. t
e −1 e +1
3. In non-elastic collision, [2012]
a. momentum is conserved 6. A 600 kg rocket is set for a vertical firing. If the
b. energy is conserved exhaust speed is 1000 m/s, the mass of the gas
c. momentum and energy are conserved ejected per second to supply the thrust needed to
d. momentum and energy are non-conserved overcome the weight of rocket is [2009]
4. Which of the following is not an example of perfectly a. 117.6 kg s−1 b. 58.6 kg s−1
inelastic collision? [2011] c. 6 kg s−1 d. 76.4 kg s−1

@iitjeehelps
CENTRE OF MASS, MOMENTUM AND COLLISION 91
7. If gas molecules undergo inelastic collision with the instance when the speed of A is v and speed of B is
walls of the container, [2009] 2v , the speed of centre of mass (CM) is [2008]
a. temperature of gas will increase a. zero b. v c. 2.5v d. 4v
b. temperature of gas will fall
c. pressure of the gas will increase 10. Consider the following statement. When jumping from
d. neither the temperature nor the pressure will change some height, you should bend your knees as you
come to rest instead of keeping your legs stiff. Which
8. A ball is dropped from a height h on a floor of of the following relations can be useful in explaining
coefficient of restitution e. The total distance covered the statement? [2007]
by the ball just before second hit is [2008]
a. ∆ p1 = − ∆ p2 b. ∆E = − ∆( PE + KE) = 0
a. h(1 − 2e 2 ) b. h(1 + 2e 2 ) c. h(1 + e 2 ) d. he 2
c. F ∆t = m∆ v d. ∆ x ∝ ∆ F
9. Two particles A and B initially at rest, move towards
where, symbols have their usual meanings.
each other, under mutual force of attraction. At an

Answer with Solutions


Practice Exercise L
x 3 
L3
λ0   λ0
1. (b) We assume that origin is 3a  3 0 3 = 2L
A B = =
situated at the O1. x 2 
L
L2 3
λ0   λ0
∴ x1 = 0, m1 = m 2 2
 0
x 2 = 3a, m2 = 2m O1 O2
m x + m2x 2 4. (d) (a) If a rod is bent in the form of a circle. Its centre of
∴ x CM = 1 1 mass is at the centre of curvature.
m1 + m2 m C 2m
(b) Football is in the shape of hollow sphere. So, its
m × 0 + 2m × 3a O1 O2
= centre of mass is at the centre of sphere.
m + 2m (c) The reason is same as (a).
6ma In all the above geometrical shapes, no material is
= = 2a
3m present at the centre of mass.
2. (a) For the calculation of the position of 5. (b) Since, no external force is present on the system. So,
centre of mass, cut-off mass is taken as the centre of mass of system will not be changed.
negative. The mass of disc is O 6. (b) Since, external force on system is zero. So, no change
m1 = πr12σ = π 62 σ = 36πσ O takes place in the centre of mass.
where, σ = Surface mass density m x + m2x 2
x CM = 1 1
The mass of cutting portion is m1 + m2
m2 = π (1)2 σ = πσ m1 ∆x1 + m2 ∆x 2
or ∆x CM =
m x − m2x 2 m1 + m2
x CM = 1 1
m1 + m2 Here, ∆x CM = 0
M
Taking origin at the centre of disc, ∴ m1 ∆x1 + m2 ∆x 2 = 0 or M ∆x1 + ∆x 2 = 0
3
x1 = 0, x 2 = 3 cm
M ∆x 2
36πσ × 0 − πσ × 3 ∴ ∆x 1 = − ∆x 2 = − … (i)
x CM = 3M 3
36πσ − πσ
But, x rel = ∆x1 − ∆x 2 or L = ∆x1 − ∆x 2
−3πσ 3 ∆x 2 4
= =− cm or L=− − ∆x 2 = − ∆x 2 [from Eq. (i)]
35πσ 35 3 3
3. (b) The mass of considered 3
Q ∆x 2 = − L
element is 4
dm = λdx = λ 0xdx
dm ∆x 2 −3L L
O ∴ ∆x 1 = − =− =−
L L dx 3 4×3 4
∴ x CM = 0
∫ xdm ∫
= 0 L
x ( λ 0 xdx ) x
Negative sign indicates that both move in opposite
∫ dm
∫ λ 0 xdx
0
direction.

@iitjeehelps
92 SELF STUDY GUIDE BITSAT

7. (c) The car is stopped by the contact force exerted due to 14. (c) FBD of two blocks N2
road. F
For m1, a1 =
8. (b) The acceleration of centre of mass is m1
F 30 m m1 F
a CM = = = 1m/s2 and N2 + m1g = N1 2
mA + mB 10 + 20 For m2, a 2 = 0,N2 = m2g

1 1
s = a CM t 2 = × 1 × 22 = 2 m m a + m2 a2
∴ aCM = 1 1 N2 m2g N1 m1g
2 2 m1 + m2
9. (c) If we consider a system consists of gun plus mass m. F
Then, no external force is present on the system. So, the m1 × +0
m1 F
acceleration of centre of mass of system will be zero. = =
m1 + m2 m1 + m2
Since, initially the system is in rest. So, centre of mass will
always remains at rest. 15. (a) Here, m1 = m2 = m m1 m 2Q
V
10. (d) (a) Since, external force on the system is F . (suppose), when P comes
F to rest, it falls vertically
∴ a CM =
m1 + m2 downward.
X1
R
(b) At t = 0, elongation in spring is zero. So, force on m2 is X1 = O R
zero but force on m1 is only in forward direction, i.e. F . 2 X2
F Centre of mass,
∴ a1 = , at t = 0 m X + m2X 2
m1 X CM = 1 1
m1 + m2
This is maximum acceleration of m1.
R
11. (c) When mass m2 moves m + mX 2
R= 2 [Q X CM = R]
downwards, the centre of mass
2m
of system (m1 + m2 ) moves
downwards. It means the ⇒ X 2 = 1.5 R
acceleration is found in centre of m2 g 16. (c) If velocity of centre of mass isv, then velocity of contact
N
mass in downward direction is 0 and that of the top is 2 v, hence when centre of mass
during motion of m2. This is covers a distance l, thread covers a distance 2l.
possible only when net m 1g 2v
downward force is greater than
that of upward force.
Mathematically, v
m1g + m2g > N
∴ N < m1g + m2g
m x + mBxB + mCxC + mDxD
12. (c) x CM = A A 17. (c) p = mv
mA + mB + mC + mD
p
1× 0 + 2 × 1+ 3 × 1+ 4 × 0 ∴ m=
= y v
1+ 2 + 3 + 4 Hence, m-v graph will be rectangular hyperbola.
2+ 3 1
= = = 0.5 m (0, 1) C (1, 1) p12
10 2 D
Ek1 2m m 4m
Similarly, 18. (b) = 21 = 2 = = 4 :1 [Q p1 = p 2]
m y + mByB + mCyC + mDyD Ek2 p2 m1 m
y CM = A A 2m2
mA + mB + mC + mD B
x
1× 0 + 2 × 0 + 3 × 1+ 4 × 1 A (0, 0) (1, 0) 19. (d) The momentum and kinetic energy depend upon
= reference frame. In the frame of train, the man is in rest.
1+ 2 + 3 + 4
So, the momentum and kinetic energy of the man in the
7
= = 0.7 m frame of train are zero. But in the frame of ground, velocity
10 of the man is non-zero. So, in the frame of ground,
m v + m2v 2 + m3v 3 momentum and kinetic energy are not zero.
13. (b) As, v CM = 1 1
m1 + m2 + m3 20. (c) Since, gravitational force is present. So, conservation
5 ×5 + 4 × 4 + 2 × 2 principle is only applicable to a system consists of air
=
5+ 4+ 2 molecules + earth + meteorite.
25 + 16 + 4 21. (d) (a) During firing of the bullet from gun, the chemical
=
11 energy of powder of bullet is converted into the kinetic
45 energy of the system. So, final mechanical energy of
∴ v CM = ≈ 4.09 ≈ 4 m/s system is greater than initial kinetic energy of the system.
11

@iitjeehelps
CENTRE OF MASS, MOMENTUM AND COLLISION 93
(b) In this case, chemical energy (non-mechanical) is vC − vB = 7.5e = 7.5 × 0.5 = 3.75
converted into mechanical energy of the system. ∴ vC − vB = 3.75 m/s …(iii)
(c) The same reason as the above. and vC + vB = 7.5 … (iv)
(d) The same reason as (a) and (b). Adding Eqs. (iii) and (iv), we get
22. (d) Since, no external force is present. So, momentum 2vC = 1125
.
conservation principle is completely applicable. 1125
.
∴ vC = = 5.6 m/s
∴ m n = m1n1 + m2n2 2
or (m1 + m2 ) n = m1n1 + m2n2 28. (b) Here, m = 0.08 kg v1
m
23. (e) Since, no external force is present on the system. So, m0 = 0.16 kg
m0
conservation principle of momentum is applicable. According to conservation
∴ pi = pf = p1 + p2 principle of momentum
v CM
∴ p1 = − p2 [∴ pi = 0] mv1 + mv 2 = ( 2m + m0 ) v CM
∴ | p1| = | − p2| mv1 + mv 2
∴ v CM =
∴ p1 = p2 2m + m0
v2
From this point of view, it is clear that momenta of both 0.08 × 16 m
particles are equal in magnitude but opposite in direction. =
0.16 + 0.16
Also, friction is absent. So, total mechanical energy of 128
. 128
system remains conserved. = = = 4 m/s
0.32 32
24. (d) When spring is massless. Then according to
29. (a) 30. (d) 31. (c)
momentum conservation principle,
pi = pf or 0 = m1v1 + m2v 2 32. (a) As we know that at the highest point, the shell has only
the horizontal component of velocity which isv cosθ. If u be
∴ m1v1 = − m2v 2
the velocity of second exploded piece, then applying
∴ m1v1 = m2v 2 or p1 = p 2 conservation of linear momentum along X-axis.
p2 p2 ∴ 2mv cos θ = − mv cos θ + m
Q K1 = 1 ⇒ K 2 = 2
2m1 2m2 or u = 3v cosθ
K1 m2
∴ = [Q p1 = p 2] 33. (b) Since, no force is present v1
K 2 m1 along the surface of plate. So,
25. (d) In the case of projectile motion, if bodies are projected momentum conservation θ1
θ2
with same speed, they reaches at ground with same principle for ball is applicable n
θ1
speeds. So, if bodies have same masses, then momentum along the surface of plate.
of body will be different for different directions of projection mv sin θ1 = mv1 sin θ 2
but magnitude of momenta must be same. v
or v sin θ1 = v1 sin θ 2 ...(i) Plate
v −u v −v0
26. (c)Q Acceleration, a = or a = v1 cos θ 2 v1 cos θ 2
t t e= = [Let θ1 = θ]
v −v0 v cos θ1 v cos θ
or g= [considered average force]
t ∴ v1 cos θ 2 = ev cos θ ...(ii)
v =0 v1 sin θ 2 v sin θ tan θ
Q ∴ = =
Speed before first bounce v1 cos θ 2 ev cos θ e
v 0 = 5 m/s ( ↑ ) tan θ
∴ tan θ 2 =
v − v 0 0 − ( −5 ) 5 e
∴ t= = = = 0.5 s
g 10 10  tan θ 
∴ θ 2 = tan− 1 
27. (a) For collision between blocks A and B,  e 
v − v A vB − v A vB − v A ∆m
e= B = = 34. (c) F = v
u A − uB 10 − 0 10 ∆t
∴ vB − v A = 10e = 10 × 0.5 = 5 … (i) ∆m nm 120 × 10 × 10− 3
Here, = =
From momentum conservation principle, ∆t t 60
mAu A + mBuB = mAv A + mB vB = 20 × 10− 3 kg/s
or m × 10 + 0 = mv A + mvB ∴ = 800 × 20 × 10− 3 N = 16 N
∴ v A + vB = 10 …(ii) ∆p ∆m nm
Adding Eqs. (i) and (ii), we get 35. (c) F = =v =v
∆t ∆t t
vB = 7.5 m/s n mvn
F = vm =
Similarly for collision between B and C, 60 60

@iitjeehelps
94 SELF STUDY GUIDE BITSAT

36. (a) Since, m2 moves with constant velocity. 37. (a) 38. (a)
∴ f2 = m2g 39. (b) Initially from conservation of momentum theorem, for
f2 = 8 × 10 = 80 N bullet and block system
mv
Since, block m1 moves with acceleration a = 2 m/ s2 in mv ′ = (m + M ) v ⇒ v ′ =
(m + M )
upward direction
∴ f1 − m1g = m1a Let the bullet-block system rises to height h, so from
conservation of mechanical energy,
∴ f1 = m1g + m1a = 10 × 10 + 10 × 2 = 120 N 1
(m + M ) v ′ 2 = (m + M ) gh
a=2 m/s2 m1=10 kg, m2=8 kg 2
v′2
m1 h=
2g
m 2v 2 1
m2 T ⇒ h= ×
(m + M )2 2g
v=2 m/s 40. (a) From definition, we can write coefficient of restitutions,
f1 f2 v ′ − 2v 2v
e=
v + 2v
v
m1 m2 1 v ′ − 2v
⇒ =
3 3v
m 1g m2 g f1 f2 ⇒ 3v ′ = 9 v After collision
⇒ v ′ = 3v
For equilibrium of rope, As wood sheet is heavier than ball, so its speed will not
T = f1 + f2 = (120 + 80) N = 200 N change.

BITSAT Archives
dm dm Mg 600 × 9.8
1. (b) In any type of collision, the linear momentum of the 6. (c) Thrust = u = Mg or = = = 6 kgs−1
system remain conserved even during collision. dt dt u 1000
2. (a) Total mass of the shell = 20 kg 7. (d) Usually, temperature of the gas in a container is same
Ratio of the masses of the fragments = 2 : 3 as the temperature of the walls of the container. Therefore,
∴ Masses of the fragments are 8 kg and 12 kg. Now, during collision, there is no exchange of energy. Hence,
according to the conservation of momentum whether collision is elastic or inelastic, molecules
rebounce with same average speed. Therefore, neither the
m1v1 = m2v 2 ∴ 8 × 6 = 12 × v temperature not the pressure will change.
v (velocity of the larger fragment) = 4 m/s
8. (b) Total distance travelled by the ball before its second hit
1 1
Kinetic energy = mv 2 = × 12 × ( 4)2 = 96 J is
2 2
3. (a) Momentum is conserved in non-elastic collision but
kinetic energy is not conserved.
4. (d) A ball bearing striking another ball bearing is not an
example of perfectly inelastic collision. h
5. (b) Since, if momentum is m m m h1
conserved, then A B A B
mu = mv1 + mv 2 u u = 0 v1 v2
⇒ u = v1 + v 2 …(i) Before collision After collision H = h + 2h1 = h[1 + 2e 2] [Q h1 = he 2]
As we know that coefficient of restitution can be given as 9. (a) As initially both the particles were at rest, therefore
v −v2
e= 1 …(ii) velocity of centre of mass was zero and there is no external
u force on the system. So, speed of centre of mass remains
From Eqs. (i) and (ii), we get constant, i.e. it should be equal to zero.
v −v2
e= 1 …(iii) m∆v
v1 + v 2 10. (c) F ∆t = m∆v ⇒ F =
∆t
v 1+ e
On solving Eq. (iii), we get 1 = By doing so, time of change in momentum increases and
v2 1– e
impulsive force on knees decreases.

@iitjeehelps
8
Rotational Motion

Concept of Rotational Motion


If a force is exerted on some body that does not pass through its centre of mass, then it can produce a
turning effect in the body about its centre of mass. If the body is fixed, then there will not be its linear
motion. Thus, it will go under pure rotation. But if the body is not fixed, then there will be a linear
motion along with the rotational motion.

Terms Related to Rotational Motion of the Body


ω
ω Rω = v
P
P
Q R
R r

(a) (b)
( R and r are two radii)

(i) Linear speed


v = ωR [Its SI unit is m/s]

(ii) Angular speed



ω= [ItsSI unit is rad/s]
T
∆ θ dθ
Also, ω = lim = [where, ∆θ = angular displacement]
∆t→ 0 ∆ t dt

(iii) Angular acceleration


∆ω dω Change in angular speed
α = lim = =
∆t→ 0 ∆t dt Time taken
α is same for all the points of the rotational body. Its SI unit is rad/s2 .

(iv) Linear acceleration


a = Rα [ItsSl unit ism /s 2 ]

@iitjeehelps
96 SELF STUDY GUIDE BITSAT

Equations of Rotational Motion (iii) Total effect of a torque applied on a rotating body in
a given time is called angular impulse. Angular
If angular acceleration α is uniform, then equations of impulse is equal to total change in angular
rotational motion may be written as momentum of the system in given time. Thus,
1
(i) ω = ω0 + α t (ii) θ = ω0 t + α t 2 angular impulse
2 ∆t

(iii) ω 2 − ω02 = 2 αθ
α
(iv) θ nth = ω0 + (2n − 1)
J= ∫0 τ dt = ∆L = L f − Li
2 (iv) The angular momentum of a system of particles
about the origin is
n
Torque or Moment of a Force L= ∑ri × pi
Torque is a quantity which measures the capability of a
force to rotate a body. It is a vector quantity which is
directly perpendicular to both position vector (r ) and
force ( F ). Its SI unit is Nm. Law of Conservation of
y
Angular Momentum
F
According to the law of conservation of angular
momentum, if no external torque is acting on a system, then
r
x total vector sum of angular momentum of different
O particles of the system remains constant.
θ
P dL
z
We know that, = τext
F r dt
Representation of torque on axes dL
Hence, if τext = 0, then = 0 ⇒ L = constant
dt
Torque, τ = r × F = r F sin θ n$
Therefore, in the absence of any external torque, total
where, θ = angle between r and F .
angular momentum of a system must remain conserved.
and n$ is unit vector which shows the direction of τ.
Torque due to a force is also known as the moment of a Relation between Angular
force. It is defined as the cross product of the position
vector of point ( P ) from the origin (O ) on which the force is Momentum and Torque
acting, with the force (see figure). If the external force exerted on a body rotates it, then there
is an external torque on the body. This external torque will
Angular Momentum be equal to the time rate of change of angular momentum.
dL
The moment of linear momentum of a given body about an τ ext =
axis of rotation is called as its angular momentum. If p is the dt
linear momentum of a particle and r is its position vector Moment of Inertia of Different Bodies
from the point of rotation, then Body Axis Moment of Inertia
Angular momentum, L = r × p = rpsin θ n$ 1. Rectangular Passing through its CG M(l 2 + b 2 )
where, n$ is a unit vector in the direction of rotation. Lamina and perpendicular to its 12
plane of length of breadth.
Angular momentum is an axial vector and its SI unit is
kgm 2s−1 or J-s. 2. Uniform thin (i) Through centre of gravity Ml 2
bar and perpendicular to 12
(i) For rotational motion of a rigid body, P length.
angular momentum is equal to the (ii) Through one end and Ml 2
product of angular velocity and perpendicular to length. 3
r
moment of inertia of the body about 3. Disc (i) Passing through its centre MR2
the axis of rotation. Mathematically, and perpendicular to its 2
plane.
L = Iω
(ii) About a diameter. MR2
(ii) According to the second law of rotational motion, 4
the rate of change of angular momentum of a body is 4. Ring or Hoop (i) Passing through its centre MR2
equal to the external torque applied on it and takes and perpendicular to its
place in the direction of torque. Thus, plane.
dL d dω (ii) About a diameter. MR2
τ= = ( Iω ) = I = Iα 2
dt dt dt

@iitjeehelps
ROTATIONAL MOTION 97
Body Axis Moment of Inertia −1 / 2
I r 2 + r22 + r32 +... +rn2 
5. Solid cylinder (i) About its own geometric MR2 K= or K =  1 
axis.
M  n 
2
(ii) Passing through CG and  l 2 R2  SI unit of radius of gyration is metre.
M + 
perpendicular to its  12 4 
geometric axis. NOTE Radius of gyration depends upon shape and size of the body,
6. Hollow disc of Passing through its centre M( R12 + R22 ) position and configuration of the axis of rotation and also on
radii R1 and R2 and perpendicular to its 2 distribution of mass of body w.r.t. axis of rotation.
plane.
7. Hollow (i) About its own geometric MR2
cylinder axis.
(ii) Passing through CG and  l 2 R2 
M + 
Theorems on Moment
perpendicular to length.

2
 12 2 
of Inertia
8. Thin spherical (i) About a diameter. MR2
shell 3 There are two theorems of moment of inertia, i.e. theorem
(ii) About a tangent. 5 of parallel axes and theorem of perpendicular axes. Both
MR2
3 the theorems are described below
9. Diatomic Passing through centre of m1 m2 2
r
molecule gravity and perpendicular m1 + m2
to bond length. 1. Theorem of Parallel Axes
10. Solid sphere (i) About a diameter. 2 Moment of inertia of a body about a given I ICM
MR2
5 axis I is equal to the sum of moment of
(ii) About a tangent. 7 inertia of the body about a parallel axis
MR2 A
5 CM
passing through its centre of mass I CM
and the product of mass of body ( M ) and
square of normal distance d between the
Moment of Inertia two axes.
d

The property by virtue of which a z (Axis of rotation) Mathematically, I = I CM + Md 2


body opposses its change in its state
of motion is called its inertia. Body of 2. Theorem of Perpendicular Axes
In similar manner, a body have the R mass m
The sum of moment of inertia of a plane laminar body
tendency to oppose its rotational x about two mutually perpendicular axes lying in its plane is
motion or change in rotational
equal to its moment of inertia about an axis passing
motion about its axis is known as
through the point of intersection of these two axes and
moment of inertia.
y perpendicular to the plane of z
Mathematically, it is given as the A particle is rotating laminar body. If I x and I y be Planar body
product of mass and square of the about z-axis in a circle moment of inertia of the body
perpendicular distance of a particle of radius R. about two perpendicular axes in
from the axis of rotation. its own plane and I z be the O N
y
I = mR2 moment of inertia about an axis M
P(x, y)
where, m = mass of the particle passing through point O and
perpendicular to the plane of x
and R = perpendicular distance from axis of rotation.
lamina, then
Iz = Ix + I y
Radius of Gyration
Radius of gyration of a given
body about a given axis of
Rigid Body Rotation
rotation is the normal distance of m3 If a rigid body of moment of inertia I about a given axis is
a point from the axis, where if
m1
r1
r3 rotating with an angular speed ω, then its rotational kinetic
m4 M 1
whole mass of the body is placed, m2
r4 energy is given by, K R = Iω 2 . Rotational kinetic energy is a
then its moment of inertia will be r2
K
P 2
exactly same as it has with its scalar having SI unit joule (J). Rotational kinetic energy is
actual distribution of mass. Axis of rotation related to angular momentum as per relation,
L2
Thus, radius of gyration KR = or L = 2 IK R
2I

@iitjeehelps
98 SELF STUDY GUIDE BITSAT

 K2
Rolling on an Inclined Plane (ii) Acceleration in motion a = g sin θ / 1 + 2 
 R 
For a body of mass M, rolling on an inclined plane having
inclination θ, slant height s and height h. (iii) Time of descent,
 K2
R 2h 1 + 2 
1  R 
t=
T sinθ g
h
s
(iv) The velocity of CM at the bottom of the incline,
θ 2 gh
v=
1 + K 2 / R2
v2  K 2 
(i) Height of an inclined plane, h = 1 + 2  Here, K = Radius of gyration.
2g  R 

Practice Exercise
1. The angular displacement at any time t is given by 7. A car is moving in a circular horizontal track of radius
θ (t ) = 2 t 3 − 6t 2. The torque on the wheel will be zero 10 m with a constant speed of 10 m/s. A plumb bob is
at suspended from the roof of the car by a light rigid rod
a. 1 s b. 0.1 s c. 2 s d. 0.2 s of length 1 m. The angle made by the rod with the
track is (Take g = 10 m/s 2)
2. Four 2 kg masses are connected by 1/ 4 m spokes to
a. zero b. 30°
an axle. A force of 24 N acts on a lever 1/2 m long to
c. 45° d. 60°
produce angular acceleration α . The magnitude of α
(in rad s–2) is (the angle between r and F is 30°) 8. If a raw egg and a boiled egg are spinned on the table
a. 24 b. 12 c. 6 d. 3 by applying same torque. Then
a. boiled egg will spin faster
3. A flywheel of moment of inertia 0.4 kg m2 and radius b. raw egg will spin faster
0.2 m is free to rotate about a central axis. If a string is c. moment of inertia of boiled egg will be lesser than that
wrapped around it and it is pulled with a force of 10 N, of the raw egg
then its angular velocity after 4 s will be d. Both a and c are correct
a. 10 rad s−1 b. 5 rad s−1 c. 20 rad s−1 d. None
9. The torque τ on a body about a given point is found to
4. The centre of a wheel rolling on a plane surface be equal to A × L, where A is constant vector and L is
moves with a speed v 0. A particle on the rim of the
the angular momentum of the body about that point.
wheel at the same level as the centre will be moving at
From this, it follows that
speed
dL
a. zero b. v 0 c. 2 v 0 d. 2v 0 a. is perpendicular to L at all instant of time
dt
5. A meter stick is hold vertically with one end on the b. the component of L in the direction of A does not
floor and then is allowed to fall. Assuming that the end change with time
on the floor of the sticks does not move. The velocity c. the magnitude of L does not change with time
of the other end when it hits the floor, will be d. All of the above
a. 9.8 m/s b. 6.2 m/s c. 8.9 m/s d. 5.4 m/s 10. The tricycle weighing 20 kg has a small wheel
6. The instantaneous velocity symmetrically placed 1 m behind the two large
of point B of the given rod 60° wheels, which are also 1 m apart. If the centre of
of length 0.5 is 3 m/s in the gravity of machine is at a horizontal distance of 25 cm
represented direction. A B behind the front wheels and the rider whose weight is
The angular velocity of the 40 kg is 10 cm behind the front wheels. Then,
rod for minimum velocity of the thrust on each front wheel is
end A is a. 255 N b. 90 N
a. 1.5 rad/s b. 5.2 rad/s c. 2.5 rad/s d. 3.6 rad/s c. 200 N d. 400 N

@iitjeehelps
ROTATIONAL MOTION 99
11. A rectangular plate of mass 20 A B 17. The moment of inertia of a system of four rods each of
kg is suspended from points A length l and mass m about the axis shown is
and B as shown. If the pin B is
suddenly removed, then 0.15 m
I1 I4
determine the angular
acceleration (in rad/sec 2) of the 0.2 m
plate.
I2 I3
a. 48 b. 19.6 c. 29.4 d. 23.6
12. A uniform rod of length l and mass m is suspended by
2 2 8 2
two vertical inextensible strings as shown in figure. a. ml b. 2 ml 2 c. 3 ml 2 d. ml
Then, tension in the left string when right string snaps, 3 3
is 18. The surface density of a circular disc of radius a
depends on the distance as ρ(r ) = A + Br . The
moment of inertia about the line perpendicular to the
String String plane of the disc is
 A 2a   A 2B 
a. πa 4  + B b. πa 4  + 
2 5  2 5
 A Ba 
Rod c. 2πa 3  +  d. None of these
2 5
3mg mg
a. b.
4 4 19. A hoop of radius 2 m, weight 100 kg. It rolls along
mg mg horizontal floor so that its centre of mass has a speed
c. d.
2 8 of 20 cm/s. How much work has to be done to stop it?
a. 4.8 J b. 4.0 J c. 6.2 J d. 7.2 J
13. A rod of length L is hinged from one end. It is brought
to a horizontal position and released. The angular 20. Mass of bigger disc having radius 2R is M. A disc of
velocity of the rod, when it is in vertical position, is radius R is cut from bigger disc. Moment of inertia of
2g 3g g g disc about an axis passing through periphery and
a. b. c. d. perpendicular to plane is
L L 2L L
27 MR 2 29 MR 2
14. A light rod carries three equal masses A, B and C as a. b. c. 3.5 MR d. 2MR 2
8 8
shown in figure. Find the velocity of B in vertical
position of rod, if it is released from horizontal position 21. Three thin rods each of length L and mass M are
as shown in figure. placed along x , y and z-axes such that one of each rod
is at origin. The moment of inertia of this system about
I/3 I/3 I/3
z-axis is
A B C 2 2 4ML2 5 ML2 ML2
a. ML b. c. d.
3 3 3 3
22. If r is the distance between the Earth and the Sun.
Then, angular momentum of the Earth around the Sun
is proportional to
a. r −1/ 2 b. r 1/ 2 c. r 2 d. r −2
23. Two wheels A and B are mounted on 6 kg m 2 the
18gl 4gl 8gl same shaft. One of them having their moment of
a. 2gl b. c. d. inertia 8 kg m 2 with angular speed 600 rpm and other
7 3 7
is at rest. The value of moment of inertia of another
15. The kinetic energy of a lamina moving in its plane is wheel in order to achieve the combined angular speed
1
2
a. M (v CM + K 2ω 2 ) b. 2
M (v CM + K 2ω 2 ) 300 rpm is
2 a. 4 kg m 2 b. 3 kg m 2 c. 6 kg m 2 d. 9 kg m 2
1
c. Iω 2 d. None of these
2 24. Choose the correct option.
a. Friction is necessary for rolling motion
16. A wheel has mass of the rim 1 kg, having 50 spokes
b. Friction is necessary for pure accelerated rolling
each of mass 5 g. The radius of the wheel is 40 cm. motion
The moment of inertia is c. Friction is necessary for pure accelerated rolling on
a. 0.273 kg m2 b. 1.73 kg m2 an inclined plane
c. 0.173 kg m2 d. 2.73 kg m2 d. None of the above

@iitjeehelps
100 SELF STUDY GUIDE BITSAT

25. An arm making an angle of 120° at the 32. Two cubes A and B of same shape, size and mass are
centre of ring of mass m and radius r is 120° placed on a rough surface in the same manner. Equal
cut from the ring. The arc is made to forces are applied on the both cubes. But at the cube
rotate about z-axis perpendicular to its A, the force is applied at the top in horizontal direction.
plane and passing through the centre of But at the cube B just above the centre of mass of the
the ring. The moment of inertia of the z cube in the same manner. Then,
arc about the z-axis is a. A will topple first
mr 2 mr 2 mr 2 b. B will topple first
a. mr 2 b. c. d. c. Both will topple at the same time
3 2 4 d. None of the above
26. A particle of mass m rotates in a circle of radius a with 33. A regular polygon of n sides is placed on a rough
a uniform angular speed ω 0. It is viewed from a frame surface vertically as such one of the side of regular
rotating about the z-axis with a uniform angular speed polygon touches the surface. A force is applied
ω. The centrifugal force on the particles is horizontally at the top. The chosen value of n are 3, 5
a. mω 2a b. mω 20 a and 8. For which value of n, the polygon first is likely to
2
 ω + ω0  topple?
c. m   a d. mωω 0
 2  a. 3 b. 5
c. 8 d. All of these
27. At any instant, a rolling body may be considered to be
in pure rotation about an axis through the point of 34. A particle performs uniform circular motion with an
contact. This axis is translating forward with speed angular momentum L. If the frequency of particles
motion is doubled and its KE is halved, then the
a. equal to centre of mass b. zero
angular momentum becomes
c. twice of centre of mass d. No sufficient data
L L
a. 2L b. 4L c. d.
28. In the given figure, the sphere rolls without slipping 2 4
on the plank which is
moving with constant 35. A mass m is moving with a constant velocity along a
vCM
velocity v 0. The radius and line parallel to the x-axis, away from the origin. Its
angular velocity of the angular momentum with respect to the origin
sphere is r and ω, v0 a. is zero
respectively. The velocity of b. remains constant
centre of mass of the c. goes on increasing
sphere is d. goes on decreasing

a. v 0 + rω b. v 0 − rω c. rω d. v 0 36. A particle of mass m is projected with velocity v


moving at an angle of 45° with horizontal. The
29. A uniform cube of mass m and edge a moves on a magnitude of angular momentum of projectile about
horizontal surface along the positive x-axis, with initial point of projection when particle is at maximum
velocity v 0. Then, height, is
a. during motion, N > mg mv 3
a. zero b.
b. during motion, normal reaction acts on the centre of 4 2g
mass mv 3
c. d. m 2gh 3
c. during motion, the normal reaction shifts towards 2g
positive x-axis from the centre of mass
d. during motion, normal reaction shifts in the direction 37. A uniform rod of length 2a is held with one end
of the forces of friction resulting on a smooth horizontal table making an
F angle α with the vertical. When the rod is released,
30. In the case of toppling of the v1
body about the point A (shown a. its centre of mass moves vertically downwards on a
v2 straight line
in the figure),
b. its centre of mass remains in rest
C vC
a. vC > v 2 > v1 > v A c. the rod rotates about a vertical axis
vA d. Both a and c are correct
b. v1 > v 2 > vC > v A A
c. v A ≥ 0 38. A 70 kg man standing on ice throws a 3 kg body
d. vC < v1 < v 2 < v A horizontally at 8 m/s. The friction coefficient between
the ice and his feet is 0.02. The distance, the man
31. In Q. 30, acceleration of the point A is slips is
a. ≥ 0 b. > 0 a. 0.3 m b. 2 m
c. < 0 d. = 0 c. 1 m d. ∞

@iitjeehelps
ROTATIONAL MOTION 101
39. A weightless rod of length I carries two equal masses 44. When a wheel moves a distance shorter than 2πR
m one fixed at the end and other in the middle of the while making one rotation. Then,
rod. The rod can revolve in a vertical plane about A. a. v CM < Rω b. v CM ≤ Rω
Then, horizontal velocity which must be imparted to c. v CM > Rω d. v CM ≥ Rω
end C of rod to deflect it to horizontal position is
A
45. If a body moves through a distance greater than 2πR
in one full rotation. Then,
a. v CM > Rω
m B b. v CM < Rω
c. v CM ≥ Rω
d. v CM ≤ Rω
m C
46. A uniform sphere of radius a rotating with an angular
12 16 velocity ω about an axis perpendicular to the plane of
a. gl b. 3gl c. gl d. 2gl motion and its centre impinges on a horizontal plane,
5 5
let u and v are horizontal and vertical component of
40. Two balls A and B of angular velocities ω A and ωB velocity before impact. Then
collide with each other. Then, after collision
a. if u = aω, then u and ω are unaltered
a. both have same angular velocities
b. if u = aω, then surface is frictionless
b. ω A > ωB
c. if u > aω, then angular velocity increases
c. ω A = ωB , when balls are smooth
d. All of the above
d. ω A > ωB , when balls are smooth
47. In the given figure, a solid sphere is placed on a plank
41. A uniform rod OA of mass M and length 2a rests on a having acceleration a 0 (shown in the figure). Then,
smooth table and is free to turn about a smooth pivot
at its end O, in contact with it at a distance b from O is
an inelastic particle of mass m, a horizontal blow of
C
impulse p is given to rod at a distance x from O in a p v0
direction perpendicular to the rod. The resultant
instantaneous angular velocity of the rod is a0
px px
a. b.
4Ma 2 M a. if a p = a 0, then pure rolling takes place
+ mb 2
3 b. if v p = v 0, then pure rolling takes place
px c. if a p = a 0, v p ≠ v 0, then pure rolling takes place
c. d. None of these
ma + mb 2
2
d. if a p = a 0, v p = v 0, then pure rolling takes place
42. A uniform rod AB of mass m and length l is at rest on a 48. The cylinder of mass M is suspended through two
smooth horizontal surface. An impulse p is applied to strings as shown in figure. The speed of cylinder after
the end B. The time taken by the rod to turn through a descending through a depth h, is
right angle is
mI p πml πp
a. 2π b. 2π c. d.
p ml 12p ml T T

43. A sphere of radius R is rolling on a A


2r
rough horizontal surface. The
θ
magnitude of velocity of A with vCM
respect to ground will be Mg
a. 2 v CM 3gh 4gh
b. 2v CM sin θ a. b.
2 3
c. 2 v CM 1 + sin θ 2gh 8gh
c. d.
d. No sufficient information 3 3

@iitjeehelps
BITSAT Archives
1. Two rings of radius R and nR made of same material 6. The radius of a wheel is R and its radius of gyration
have the ratio of moment of inertia about an axis about its axis passing through its centre and
passing through centre is 1:8. The value of n is perpendicular to its plane is K . If the wheel is rolling
[2013, 07] without slipping, then the ratio of its rotational kinetic
a. 2 b. 2 2 c. 4 d. 1/2 energy to its translational kinetic energy is [2010]
K2 R2
2. A mass m is moving with a constant velocity along a a. b.
line parallel to x-axis. Its angular momentum with R2 K2
respect to origin on z-axis is [2013, 07] R2 K2
c. 2 d. 2
a. zero b. remain constant R + K2 R + K2
c. goes on increasing d. goes on decreasing
7. The moment of inertia of a thin circular disc about an
3. The moment of inertia of the body about an axis is axis passing through its centre and perpendicular to
1.2 kg m 2. Initially, the body is at rest. In order to its plane, is I. Then, the moment of inertia of the disc
produce a rotational kinetic energy of 1500J, an about an axis parallel to its diameter and touching the
angular acceleration of 25 rad/s 2 must be applied edge of the rim is [2008]
about the axis for the duration of [2011] 3 5
a. I b. 2I c. I d. I
a. 2 s b. 4 s c. 8 s d. 10 s 2 2
4. When the mass is rotated in a plane about a fixed 8. The moment of inertia of a circular disc about an axis
point, then angular momentum is directed along the passing through the circumference perpendicular to
a. radius [2010] the plane of the disc is [2006]
3
b. tangent to orbit a. MR 2 b. MR 2
c. axis of rotation 2
d. line at an angle of 45° to plane of rotation MR 2 5
c. d. MR 2
2 4
5. A solid cylinder of mass 2 kg rolls down (pure rolling)
an inclined plane from a height of 4 m. Its rotational 9. Angular momentum is conserved [2006]
kinetic energy, when its reaches the foot of the plane a. always
is ( Take g = 10 ms −2 ) [2010] b. never
80 c. when external force is absent
a. 20 J b. 40 J c. J d. 80 J d. when external torque is absent
3

Answer with Solutions


Practice Exercise
dθ 3. (c) As, τ = F × r = Iα
1. (a) Given, θ (t ) = 2t 3 − 6t 2 ⇒ = 6t 2 − 12 t (ω 2 − ω1)
dt =I
t
d 2θ
= 12 t − 12 F ×r ×t
dt 2 ∴ ω 2 − ω1 =
I
For torque, τ = Iα = 0 10 × 0.2 × 4
d 2θ =
= 0 ⇒ 12 t 2 − 12 = 0 0.4
dt 2 = 20 rad s–1
∴ t =1 s
4. (c)vp = r ω = ( 2R ) ω = 2 v 0
2. (b) As, force × distance = τ = Iα
1   1 
2
⇒ F sin 30° × = 4 2 ×    α P0
2   4  ω
 v0 ⇒
r
1 1 α
or 24 × × =
2 2 2
∴ α = 12 rad s–2 v0=Rω ω

@iitjeehelps
ROTATIONAL MOTION 103
5. (d) From energy conservation, A In the given figure, the tricycle with two front wheels A
M 1 2 and B with one rear wheel C with centre of gravity of
g l = Iω …(i) machine G1 and that of rider G2.
2 2 Stick
Moment of inertia of the rod, Now, force at G1 = 20 × 10 = 200 N
1 Force at G2 = 400 N
I = Ml 2 …(ii) M B Let FC = Force on ground under rear wheel C
3
From Eqs. (i) and (ii), we get Force at D = FD = 2FA acts of mid-point of AB.
M 1 Ml 2 2 3g Taking torque about D, for equilibrium,
gl = ω ⇒ ω=
2 2 3 l FC × 100 = 200 × 25 + 400 × 10 = 9000
Given, l = 1m, ω = 3g ∴ FC = 90 N
∴Thrust on each front wheel is N.
Also, v = rω = 1× 3 × 9 . 8 = 5.4 m/ s
∴ 2N + FC = Total weight = 600 N
6. (b) If rod is rotated about end A, then vertical component of 600 − FC 600 − 90
∴ N= = = 255 N
velocity,v ⊥ of end A will be zero. 2 2
v sin 60° 3v 3×3 11. (a) When plate starts to rotate l/2
∴ ω= = = = 5.2 rad/s
l 2l 2 × 0.5 about point A, then b/2
v⊥=0 v sin 60° I  I
τ = mg Q τ = mg 
2  2
v cos 60° l
or Iα = mg
2
7. (c) From figure, Here, l = 0.2 m, b = 0.15 m mg
T cos θ 2 2 2 2
T cos θ = mg …(i) θ l b ( 0.2) ( 0.15) 0.0625
∴ d2 = + = + =
mv 2 θ
T 4 4 4 4 4
or T sin θ = …(ii)
r mv2 ∴The moment of inertia about vertical axis passing
T sin θ
Dividing Eq. (ii) by Eq. (i) r through point A is
m 2
v2 102 I = IC + md 2 = (I + b 2 ) + md 2
∴ tan θ = = =1 12
rg 10 × 10 mg 20 0.0625
⇒ θ = 45° = {( 0.2)2 + ( 0.15)2} + 20 ×
12 4
τ 1
8. (d) τ = Iα , α = , α ∝ On putting the values,
I I α = 48 rad/s2
It means, greater moment of inertia, lesser the spin. The MI
of boiled egg is less than that of the raw egg. So, options 12. (b) The torque on rod about point
O is
(a) and (c) are correct. I I T
dL τ = mg or Iα = mg
9. (c)Q τ = A × L or =A×L …(i) 2 2
dt mI 2 I 3 O
 d L or α = mg or lα = g α
i.e.   is always perpendicular to L L 3 2 2 mg
 dt  θ For centre of rod,
(and also A) A mg − T = ma CM
 dL l
From Eq. (i),   = AL sin θ …(ii) ∴ T = mg − ma CM = mg − m α
 dt  2
3 mg
The component of L in the direction of A (L cos θ) will not = mg − mg =
change with time. And also torque is perpendicular to L, the 4 4
magnitude of L will not change with time. 13. (b) The loss in potential energy = G
10. (a) 1m
Gain in angular kinetic energy
L 1 L
A B
∴ mg = Iω 2 2
2 2
10 cm L 1 mL2 2
G2 25 cm or mg = ω G′
2 2 3
100 cm
Lω 2
G1 or g=
3
 3g 
∴ ω=  
 L
C

@iitjeehelps
104 SELF STUDY GUIDE BITSAT

14. (d) Loss in PE = Gain in angular kinetic energy 20. (b) Surface density of motional disc is
I  2l  1
σ=
M
=
M
or mg + mg   + mgI = Iω 2 …(i)
3  3 2 π ( 2R )2 4πR 2
Here, I = Moment of inertia about fixed point Mass of cutting portion is
I
2
 2I 
2 M
= m   + m   + mI 2 =
14 2
mI m1 = σ × πR 2 =
 3  3 9 4
I = I1 − I 2
From Eq. (i),
where,
1  14 2 2  36g 
2mgI =  mI  ω ∴ ω =   I1 = Moment of inertia of disc about given axis without

2 9   14I 
cutting portion
2I  36g   8gI  I 2 = Moment of inertia due to cutting portion
∴ v B = rω =   =  
3  14I   7  M ( 2R )2 m R 2 
I= + M ( 2R )2 −  1 + m1( 3R )2 
15. (b) If a lamina is moving withv CM having angular velocity ω. 2  2 
2 2
Then, 19 MR 29 MR
= 6MR 2 − =
KE = Angular kinetic energy + Translatory kinetic energy 8 8
1 1
= I CM ω 2 + mv CM 2 21. (a) Moment of inertia of the rod lying along z-axis will be
2 2 2
1 1 1 zero. MI of the rods along x and y-axes will be ML each.
= MK 2ω 2 + Mv CM 2
= M (K 2ω 2 + v CM
2
) 3
2 2 2 2 2
Hence, total moment of inertia is ML .
ml 2 3
16. (c) I = mr 2 + 50
3 22. (b) By the Kepler’s rule,
50 (5 × 10−3 ) ( 0.4)2 GM
= 1 × ( 0.4)2 + L = mvr = m r
3 r
= 0.16 (1083
. ) = 0.173 kg m2 ⇒ L∝ r
17. (d) Consider a square lamina, then 23. (c) From the conservation of angular momentum,
 l2 l2  (I1 + I 2 )ω = I1ω1 + I 2ω 2
4m  +  about
 12 12 ⇒ ( 6 + I2 ) ×
300
× 2π = 6 ×
600
× 2π + I 2 × 0
√2
60 60
2ml 2
Centre of mass = l/√2 ⇒ I 2 = 6 kg m2
3
Apply perpendicular axes theorem 24. (c) For pure rolling, friction is not necessary.
2 2
2ml  l  8 2 For pure accelerated rolling on a horizontal floor, friction is
= + 4m   = ml not necessary in every case.
3  2 3
For pure accelerated rolling on an inclined plane, friction is
18. (a) dm = 2πr dr (ρ ) = ( A + Br ) ( 2πr dr ) necessary.
a
π Aa 4 2πBa 5  A 2a  m
l = ∫ dmr 2 = + = πa 4  + B 25. (b) The mass of ring per unit angle is λ =
2 5 2 5  2π
0
m
19. (b) Given, R = 2 m, M = 100 kg ∴ dm = dθ

Speed of centre of mass,
∴The moment of inertia of dl=r2dm
v = 20 cm/s = 0.20 m/s considered element about
Work done to stop the hoop = Total kinetic energy of the z-axis passing through dm
hoop point O is
1 1 dl = r 2dm dθ
W = M v 2 + Iω 2
2 2 θ
∴The moment of inertia of O
v
But moment of inertia, I = Mr 2 and angular velocity, ω = the arc is
R mr 2 θ = 2π / 3 mr 2
1 1
W = Mv 2 + (Mr 2 ) ×  2 
v2  I =r2 = ∫
3 θ=0
dm =
3
2 2 R  26. (b) Centrifugal force does not depend upon angular
1 1 velocity of particle. It depends upon angular velocity of
= Mv 2 + Mv 2 = Mv 2
2 2 frame of reference.
= 100 × (0.20)2 = (100 × 0.04) J = 4.0 J Q Centrifugal force is F = mω 20a

@iitjeehelps
ROTATIONAL MOTION 105
27. (a) Since, in this case, instantaneous axis of rotation is If perpendiculars are drawn at the velocities of centre of
always below the centre of mass. This is possible only mass and lower end. They are passing through point O.
when point of contact moves with a velocity equal to centre So, rod is rotating about instantaneous axis passing
of mass. through point O.
28. (a) For pure rolling motion of sphere, relative motion of O
point of contact of sphere and plank should be zero. For
this, the point of contact has a velocity equal to the velocity
of plank. C
∴ v 0 = v CM − rω
∴ v CM = v 0 + rω v
29. (c)According to figure, 38. (a)
39. (a) Loss in kinetic energy = Gain in
potential energy
1 2 I
∴ Iω = mg + mgI …(i)
2 2
C v0
mI 2 5
Here, I = + mI 2 = mI 2 C
x
4 4
t vC = Iω
In this case, the force of friction has a tendency to rotate From Eq. (i),
1  5 2 2 mgl 3
the cube in clockwise direction. So, normal reaction will  ml  ω = + mgl = mgl
shift in forward direction to balance the torque due to force 2 4  2 2
of friction. 12  12gl 
ml 2ω 2 = mgl ∴ vC =  
30. (b) 31. (d) 5  5 
32. (a) The greater height of the point of applying force from 40. (c) (a) This is possible, when balls are smooth.
ground, more chance of toppling. (b) ω A = ωB is not always possible. It depends upon
33. (c) The body has smaller base, greater chance of toppling. conditions. But no conditions are mentioned.
When the value of n increases, base size decreases. 41. (a) Angular impulse, m
34. (d) J = ∆L = I∆ω = I (ω − 0) = Iω O A
b
35. (b) From figure, the angular r Here, px = Iω
θ px px px
momentum is θ m v
∴ ω= + 2
= 2
L = r × mv I M ( 2a ) 4Ma
d r mb 2 + + mb 2
Q L = rmv sin θ = mv (r sin θ ) 3 3
= mvd = constant 42. (c) The angular impulse about centre of mass is
O A
J = Lf − L i = I ω − 0
36. (b) The angular momentum is ∴ J = Iω
L=r ×p r
I mI 2 mI
θ or p = ω or p = ω
L = rmv cos θ sin θ v cosθ
2 12 6
But, H = r sin θ H 6p C
θ Q ω=
L = mvH cosθ mI
v 2 sin2 θ Q θ = ωt l
= mv × θ π π πmI 2
2 2g ∴ t= = = =
ω 2ω 2 × 6p 12p
But, θ = 45° ml
B
P
mv 3
∴ L= 43. (c) Here,v CM = Rω
4 2g
The resultant velocity of point A A θ
vCM
37. (d) Since, no force is present in is
horizontal direction. θ Rω
2
v CM + (Rω )2 + 2v CM(Rω )
So, acceleration of centre of vA =
α cos (90° − θ )
mass in horizontal direction is
vCM
zero. Hence, the centre of mass v = v CM
2
+ v CM
2
+ 2v CM
2
sin θ
of rod moves in vertically
= 2 v CM 1 + sin θ
downward direction.

@iitjeehelps
106 SELF STUDY GUIDE BITSAT

2πR For rotational motion about CM


44. (a)Q v CM T < 2πR or v CM <
T 1
2Tr = Mr 2α
∴ v CM < Rω 2
45. (a)Q v CM T > 2πR 4T = Mrα
2πR where, r = radius of cylinder
or v CM > ∴ v CM > Rω 1
T ⇒ 2T = Ma …(ii)
46. (d) 2
From Eqs. (i) and (ii), we get
47. (d) For pure rolling motion, the relative motion between
2 mg
point of contact and surface should be zero. a = g andT =
3 6
This is possible only when,
Now, from equation of motion,v 2 = u 2 + 2gh, we get
a p = a 0,v p = v 0
2  4gh
48. (b) From the equation of centre of mass of the cylinder, v 2 = 2  g h ⇒ v =
3  3
Mg − 2T = Ma …(i)

BITSAT Archives
1. (a) 1 2 1
mgh = Iω + mu 2
R nR 2 2
 u  1
2
1 1
2 × 10 × 4 = ×  mR 2 ×  2  + mu 2
2 2  R  2
l1=2πR
3
I1 = 2πnR 80 = mu 2 …(i)
4
Ratio of moment of inertia of the rings 1
2 2 2 ∴ Rotational KE = Iω 2
l1  M1   R1   λL   R  2
=    =  1  1 1 1 80 × 4 80
l2  M 2   R2   λL2   R2  = mu 2 = × = J
2
4 4 3 3
 2 πR   R 
=   [λ = linear density of wire = constant] 1 2
 2πnR   nR  Rotational KE

K2
6. (a) = 2 = 2 [Q I = MK 2, v = Rω]
L1 1 1 Translational KE 1 Mv 2 R
⇒ = = [given]
L2 n 3 8 2
∴ n3 = 8 7. (d) Moment of inertia of a circular disc about an axis
⇒ n=2 passing through centre of gravity and perpendicular to its
2. (b) Angular moment of particle w.r.t. y plane
origin = linear momentum × 1
v I = MR 2 …(i)
perpendicular distance on line of m 2
action of linear momentum from origin a From Eq. (i),
[= mv × a = mva = constant] x MR 2 = 2I
O
1
3. (a) Rotational kinetic energy = Iω 2 Then, moment of inertia of disc about tangent in a plane
2 5 5 5
1 = MR 2 = ( 2I ) = I
According to question, Iω 2 = 1500 4 4 2
2 1
1 8. (b) I CM = MR 2
I(αt )2 = 1500 2 I
2
Now, moment of inertia about an
(1.2) × ( 25)2 × t 2 = 3000 axis is given as
1.2 × 625 × t 2 = 3000 1
I = I CM + Mh 2 = MR 2 + MR 2
M
3000 2
t2 = =4 [ as h = R]
1.2 × 625
3 R
I = MR 2
Ig= 1 MR2
t = 2s 2 2
4. (c) The angular momentum will be directed along the axis 9. (d) According to law of conservation of angular
of rotation. momentum, if there is no torque on the system, then the
5. (c) At foot of the plane, angular momentum remains constant.

@iitjeehelps
9
Gravitation

Introduction
Gravitation is the name given to the force of attraction between any two massive bodies of the
universe.

Universal Law of Gravitation


Every body in the universe attracts every other body. Force of gravitational attraction between two
bodies of mass m1 and m2 is given as
mm
F = G 12 2
r
The proportionality constant G is called universal gravitational constant. In SI system, the value of
gravitational constant G is 6.67 × 10−11 Nm 2 kg −2 . Dimensional formula of G is [M −1L3T −2 ].

Gravity
In Newton’s law of gravitation, we consider the gravitation as the force. But in case of two
gravitationally interacting masses (or bodies), if one is the earth, then the gravitational force is called
as the gravity.

Acceleration due to Gravity (g)


At the earth’s surface, the value of g is found to be 9.8 ms−2 or 32 ft s−2 .
F GMm
Mathematically, g= = 2
m Rm
where, F = force exerted by the earth on a mass m.
GM
⇒ g= 2
R
Here, M = mass of earth, R = radius of earth.

@iitjeehelps
108 SELF STUDY GUIDE BITSAT

Variation in Acceleration due to These laws are


Minor axis
Gravity (g)
The value of g is variable and can vary in some cases as Swept area
Planet
mentioned below (R is treated to be radius of the earth, i.e.
Major axis
6400 km)
S
(i) Variation in g with the Height from the Earth’s Perihelion Aphelion
Surface The new value of g at height h from the or perigee or apogee
a
earth’s surface, is
g
g′ =
 h
2 1. Law of Orbits
 1 + 
 R Every planet moves in an elliptical orbit around the sun,
with sun situated at one of the foci of the ellipse.
and if h << R, then
 2h  2. Law of Areas
g ′ = g 1 − 
 R
The line joining a planet to the sun sweeps equal areas in
(ii) Variation in g with Depth Inside the Earth The new equal intervals of time, howsoever small these time
value of g at a place inside the earth with depth h intervals may be. Mathematically,
from the earth’s surface, is dA 1 L
= r v = constant =
 h dt 2 2m
g ′ = g 1 − 
 R where, L = angular momentum of planet and
m = mass of planet.
(iii) Variation in g Due to Daily Rotation of the Earth
The new value of g at point P with latitude θ, is
g ′ = g − ω 2 R cos2 θ
3. Law of Periods
The square of the time period of revolution of a planet is
(a) At the pole, proportional to the cube of the semi-major axis of the
cosθ = 0 [Qθ = 90° ] ellipse traced out by the planet. Thus,
2 3
g′ = g T  r 
T ∝ r or  1  =  1 
2 3

(b) At the equator,  2


T  r2 
cosθ = 1 [Qθ = 0° ]
g′ = g − ω R 2
Gravitational Field
Pole Gravitational field of a given body is the space around it,
within which gravitational effect due to that body may be
mP
r m ω2 r experienced.
GMm/R2
θ
Equator
Gravitational Field Intensity
Gravitational field intensity at any point in the field of given
body is defined as the force experienced by a unit mass
placed at that point. Mathematically,
F
Gravitational field intensity, I = lim
m 0 →0 m
0

where, m0 is a small test mass. The SI unit of gravitational


Kepler’s Laws of intensity is N kg −1 .

Planetary Motion Gravitational intensity at a point situated at a distance r


from a point mass M is given by
Kepler discovered three empirical laws which accurately GM
describe the motion of planets. I= 2
r

@iitjeehelps
GRAVITATION 109
GM
Gravitational Field in Different Cases EP = r
a3
The calculation of gravitational field in some situations are
mentioned below
● Field due to point mass The field at point P, at a a
distance r from the point mass ( M ) is
− GM C P
EP = er
r2
where, e r is the unit vector along r.
A r
● Field at an axial point of a circular
ring z=√a2+r2
a At the centre, r = 0, E P = 0
The field at an axial point P is α
O r P (b) When point P is an external point, then
GMr GM
EP = 2 EP = 2
(a + r 2 )3 / 2 r
● Field at an axial point of a disc
a
Thefield at an axial point P is a
2GM θ P
EP = (1 − cosθ ) O r C
x P
a2
dx
r
● Field due to a uniform spherical shell
In the case of uniform solid sphere, the graph of E
There may be two conditions as
versus r is shown below
(a) When point P is an internal point of the shell
( i.e. r < a ), then E

EP = 0
GM/a 2

C P o a r

r Gravitational Potential
The amount of work done by the applied force required to
Field at any point inside the shell is zero, which bring a body of unit mass from reference position (i.e.
implies that the potential inside the shell is constant infinity) to a given position within the field is known as
everywhere. gravitational potential of the field (created by mass).
(b) When point P is an external point of the shell
( i.e. r ≥ a ), then Gravitational Potential in Different
GM Cases
EP = 2
r 1. Potential due to a point mass
Centre of mass
a

C P P
M
r r

● Field due to a uniform solid sphere There may be two Gravitational potential at P ,
conditions as GM
VP = −
(a) When point P is an internal point, then r

@iitjeehelps
110 SELF STUDY GUIDE BITSAT

2. Potential at an axial point of a


uniform ring a
z Gravitational Potential
Gravitational potential at point P, O r
θ
P Energy
− GM
VP = Gravitational potential energy of a body or system is
a2 + r 2 negative of work done by the conservative gravitational
forces, F in bringing it from infinity to the present position.
where, a = radius of the ring
Mathematically, gravitational potential energy
M = mass of the ring r
3. Potential due to a uniform U = − W = − ∫ F ⋅ dr

thin spherical shell
The gravitational potential energy of two particles of
There may be two conditions masses m1 and m2 separated by a distance r is given by
as P
Gm1m2
a U =−
(i) When point P is an r
r
internal point ( i.e. r < a ) The gravitational potential energy of mass m at the surface
− GM of the earth is
VP =
a GMm
U =−
(ii) When point P is an external point (i.e. r ≥ a ) R
Difference in potential energy of mass m at a height h from
r
the earth’s surface and at the earth’s surface is
mgh ~
U( R + h ) − U R = − mgh, if h << R
P h
a 1+
R
M
● For three particles system,

−GM Gm1m2 Gm1m3 Gm2m3 


VP = U = − + + 
r  r 12 r 13 r 23 
4. Potential due to solid sphere There may be two n (n − 1)
conditions ● For n -particles system, pairs form and total
2
(i) Potential at an internal point P potential energy of the system is sum of potential
energies of all such pairs.
a

x r P The Orbital Speed, Time


Period and Energy of the
dx
Satellite
− GM Let the mass of the satellite be m and that of the planet
VP = 3
(3a 2 − r 2 ), (r ≤ a ) under study be M and a is radius of the orbit.
2a
v
When point is at the centre of sphere, then
−3 GM a
V = P S
2a
(ii) Potential at an external point P
M
(i) Orbital Speed
The speed with which a satellite revolves in its orbit is
P known as its orbital speed (v). The gravitational force
a
between earth-satellite provides required centripetal force.
r
i.e.
GMm  v2 
−GM = m   ⇒ v = GM / a
VP = , (r ≥ a ) 2
r a a

@iitjeehelps
GRAVITATION 111

(ii) Time Period E = Potential energy + Kinetic energy


GMm GMm
The time taken by satellite to just complete a revolution =U + K = −
around the earth is known as its time period (T ). 2a a
GMm
2 πa 4π 2 3 ⇒ E=−
T = = a 2a
v GM

(iii) Height of Satellite in Terms of Binding Energy of Satellite


Period It is the energy required to remove the satellite from its
The height of the satellite (from the earth’s planet) can be orbit and take it to infinity.
determined by its time period and vice-versa. GMm
Binding energy = − E = +
As the height of the satellite in terms of time period, 2a
1 /3
 gR2T 2 
h =a − R=  2 
−R
 4π  Escape Velocity
The minimum velocity with which a particle or body should
(iv) Angular Momentum of Satellite be launched from the earth’s surface to just cross the earth’s
Angular momentum of a satellite, L = mv0a = m 2GMa . gravitational field (i.e. it never comes back to the earth), is
During the motion of planet and satellite, the angular known as escape velocity.
momentum will remain conserved. It is given by the equation,
2GM
ve ≥
Energy R
The total mechanical energy (E) associated with sun-planet where, R = radius of the earth and M = Mass of the earth.
(earth-satellite) system (i.e. sum of kinetic energy and At the earth’s surface, the value of escape velocity is
potential energy) is treated to be energy of the system. 11.6 kms−1 .

Practice Exercise
1. Calculate the gravitational force of attraction between 6Gm 2 6Gmm0
a. b. c. zero d. None
two spherical bodies, each of mass 1 kg placed at a2 a2
10 m apart (G = 6.67 × 10−11 Nm 2 /kg2).
5. Two particles each of mass m are placed at A and C
a. 6.67 × 10−13 N b. 6.67 × 10−11 N as such AB = BC = L. Calculate the gravitational force
c. 6.67 × 10−7 N d. None of these on the third particle placed at D at a distance L metre
on the perpendicular bisector of the line AC.
2. If the distance between the two particles is increased
m D
by 2%, then the force of attraction between them will
a. decrease 6% b. decrease 4%
c. increase 4% d. increase 6%
L
3. How the gravitational constant will change if a brass
plate is introduced between two bodies?
a. No change b. Decreases m m
c. Increases d. Insufficient data A L B L C

4. Six particles each of mass m are placed at the corners Gm 2 Gm 2


a. along BD b. along DB
of a regular hexagon of edge length a. If a point mass 2L2 2L2
2
m 0 is placed at the centre of the hexagon. Then, the Gm
c. 2 along AC d. None of these
net gravitational force on the point mass m 0 is L

@iitjeehelps
112 SELF STUDY GUIDE BITSAT

6. In a hypothetical concept, electron of mass me 12. A point mass of 10 kg is placed at the centre of earth.
revolves around nucleus due to gravitational force of The weight of the point mass is
attraction between electron and proton of mass m p . If a. zero b. 98 N c. 49 N d. 38 N
the radius of circular path of electron is r , then the
13. How will you weight the sun, i.e. estimate its mass?
speed of electron is
You will need to know the period of one of its planets
 Gmpme   Gmpme  and the radius of the planetary orbit. The mean orbital
a.   b.  
 4r   r  radius of the earth around the sun is 1.5 × 108 km ,
 Gmp  then the mass of the sun would be calculated as
c.   d. None of these a. 2 × 1015 kg
 r 
b. 2 × 1020 kg
7. Three point masses each of mass m rotate in a circle c. 2 × 1027 kg
of radius r with constant angular velocity ω due to their d. 2 × 1030 kg
mutual gravitational attraction. If at any instant, the 14. A particle hanging from a massless spring stretches it
masses are on the vertex of an equilateral triangle of by 2 cm at earth's surface. How much will the same
side a, then the value of ω is particle stretch the spring at height 2624 km from the
 Gm   3Gm   Gm  surface of earth? (radius of earth = 6400 km)
a.  3  b.  3  c.  3  d. None
a   a   3a  a. 1 cm b. 2 cm
8. A gravitational field is present in a region. Point A c. 3 cm d. 4 cm
mass is shifted from A to B, from different paths 15. There is a mine of depth 2 km. The conditions as
shown in the figure. IfW1, Path (i) compared to those at the surface of the earth are
W 2 and W 3 represent work a. lower value of g b. higher value of g
done by gravitational force c. Both (a) and (b) d. None of these
for respective paths. Then,
Path (ii) 16. In order to make the weight of a 5 kg body to zero at
a. W1 = W2 = W3 A B the equator. The angular speed of the earth would
b. W1 > W2 > W3
must be (take, g = 10 m/ s 2 and radius of the earth,
c. W1 > W3 > W2 Path (iii)
R = 6400 km)
d. None of the above
a. 0.00125 rad s−1 b. 0.0125 rad s−1
9. A point mass m 0 is placed at distance R / 3 from the c. 0.125 rad s−1 d. 0.0325 rad s−1
centre of a spherical shell of mass m and radius R.
The gravitational force on the point mass m 0 is
17. At what height, the weight of the body is same as that
at same depth from the earth’s surface? (take, earth’s
4Gmm0 radius = R)
a. b. zero
R2 ( 5 − 1)R
9Gmm0 a. R/2 b.
c. d. None of these 2
R2 ( 3R − 1) 5
c. d. R
10. A uniform ring of mass M and radius R is placed 2 3
directly above a uniform sphere of mass 8 M and of
18. At the surface of a certain planet acceleration due to
same radius R. The centre of the ring is at a distance
gravity is one-quarter of that on the earth. If a brass
of d = 3 R from the centre of the sphere. The ball is transported on this planet, then which one of
gravitational attraction between the sphere and the the following statements is not correct?
ring is a. The brass ball has the same mass on the other planet
GM 2 3 GM 2 2 GM 2 3 GM 2 as on the earth
a. b. c. d.
R2 2R 3 2 R2 R2 b. The mass of the brass ball on this planet is a quarter of
its mass as measured on the earth
11. A mass m is placed at P at a c. The weight of the brass ball on this planet is a
distance h along the normal r quarter of the weight as measured on the earth
through the centre O of a thin P d. The brass ball has the same volume on the other planet
circular ring of mass M and radius r O h m as on the earth
as shown in figure.
19. If both the mass and radius of the earth, each
If the mass is removed further away decreases by 50%, the acceleration due to gravity
M
such that OP becomes 2h, by what would
factor the force of gravitation will
a. remain same
decrease, if h = r ?
b. decrease by 50%
3 2 5 2 4 3 4 2 c. decrease by 100%
a. b. c. d.
4 3 3 5 5 5 d. increase by 100%

@iitjeehelps
GRAVITATION 113
20. A body is suspended on a spring balance in a ship 29. The change in potential energy of a body of mass m
sailing along the equator with a speed v ′. If ω is the when it elevated upto height of nR from the earth’s
angular speed of the earth and ω 0 is the scale reading surface, is
when the ship is at rest, the scale reading when the  n 2 + 1
ship is sailing, is a. n mgR b. mgR =  2 
 n 
a. ω 0 b. zero
 n − 1  + 1
n
 2ωv ′   g  c. mgR   d. mgR  
c. ω 0 1 ±  d. ω 0 1 −   n   n 
 g   2ω 
30. Suppose the gravitational force varies inversely as the
21. The maximum vertical distance through which a full nth power of the distance. Then, the time period of a
dressed astronaut can jump on the earth is 0.5 m. planet in circular orbit of radius R around the sun will
Estimate the maximum vertical distance through be proportional to
which he can jump on the moon, which has a mean a. Rn b. R(n+1)/ 2 c. R(n−1)/ 2 d. R −n
density 2/3rd that of the earth and radius one-quarter 31. The time period of a simple pendulum at the centre of
that of the earth. earth is
a. 1.5 m b. 3 m c. 6 m d. 7.5 m a. zero b. infinity
22. Two satellites A and B revolve round the same planet c. less than zero d. None of these
in coplanar circular orbits lying in the same plane. 32. If a rocket is fired with a speed v = 2 gR near the
Their periods of revolutions are 1 h and 8 h, earth's surface and coasts upwards, its speed in the
respectively. The radius of the orbit of A is 104 km. inter-steller space is
The speed of B relative to A when they are closed, in a. 4 gR b. 2gR c. gR d. 4gR
kmh −1 is
33. In Q. 32. if the satellite is stopped suddenly in its orbit
a. 3π × 104 b. zero c. 2π × 104 d. π × 104
and allowed to fall freely onto the earth, the speed
23. A satellite is moving on a circular path of radius r with which it hits the surface of the earth is
around earth has a time period T. If its radius slightly (Take, g = 9. 8 m/ s 2 and R = 6400 km)
increases by ∆r , determine the change in its time a. 4 km/s b. 8 km/s c. 2 km/s d. 6 km/s
period.
3 T  T  34. A projectile is fired vertically upward from the surface
a.   ∆r b.   ∆r of the earth with a velocity kv e , where v e is the escape
2r r
velocity and k < 1. If R is the radius of the earth, the
3 T 2 
c.  2  ∆r d. None of these maximum height to which it will rise measured from
2r  the centre of earth will be (neglect air resistance)
24. The gravitational field in a region is 10 N/kg ( $i − $j ). 1− k 2 R R
a. b. c. R (1− k 2 ) d.
Find the work done by gravitational force to shift R 1− k 2 1+ k 2
slowly a particle of mass 1 kg from point (1m, 1m) to a
35. A satellite of mass M revolving in a circular orbit of
point (2 m, − 2 m).
radius rs around the earth of mass M has a total
a. 10 J b. − 10 J c. − 40 J d. + 40 J
energy E. Then, its angular momentum will be
25. In previous problem, find the work done by external E E
agent. a. b. c. ( 2Emsrs2 )1/ 2 d. 2Emsrs
msrs2 2msrs2
a. 40 J b. − 40 J c. zero d. + 10 J
26. The gravitational force in a region is given by 36. A planet revolves in elliptical orbit around the sun.
(see figure). The linear speed of the planet will be
E = ay $i + ax $j maximum at
Find the work done by gravitational force to shift a B
point mass m from (0, 0, 0) to (x 0, y 0, z 0 ).
a. ma x 0y 0z 0 b. ma x 0y 0
c. −ma x 0y 0 d. Zero A S C

27. The work done by an external agent to shift a point


mass from infinity to the centre of earth is
D
a. = 0 b. > 0 c. < 0 d. ≤ 0 a. A b. B c. C d. D
28. The work done in shifting a particle of mass m from 37. Two bodies each of mass 1 kg are at a distance of
centre of earth to the surface of earth is 1 m. The escape velocity of a body of mass 1 kg
mgR which is midway between them is
a. −mg R b.
2 a. 8 × 10−5 m/s b. 2.31× 10−5 m/s
c. zero d. None of these c. 4.2 × 10−5 m/s d. zero

@iitjeehelps
114 SELF STUDY GUIDE BITSAT

38. If an artificial satellite is moving in a circular orbit 40. The binding energy of earth-sun system is (neglecting
around the earth with a speed equal to half the the other planets)
magnitude of the escape velocity from the earth, the a. 2.60 × 1033 J b. 6.33 × 1016 J
height of the satellite above the surface of the earth is c. 3.40 × 1033 J d. 8.60 × 1024 J
a. 2R b. R /2
41. If one of the satellites of jupiter has an orbital period of
c. R d. R /4
1.769 days and the radius of the orbit is 4.22 × 108 m.
39. A particle of mass m is projected from the surface of The mass of jupiter is about
earth with a speed v 0( v 0 < escape velocity). Find the a. one thousandth that of the sun
speed of particle at height h = R (radius of earth). b. one hundredth that of the sun
(Take, R = 6400km and g = 9.8 m/ s 2) c. one tenth that of the sun
a. gR d. half of that of the sun

b. v 02 − 2gR 42. A satellite is orbiting closely to earth and having


kinetic energy K. The kinetic energy required by it to
c. v 02 − gR just overcome the gravitational pull of the earth, is
d. None of these a. 2 K b. 2K c. 3K d. 2 2 K

BITSAT Archives
1. The total energy of a revolving satellite around the 6. The satellite of mass m revolving in a circular orbit of
earth is − K J. The minimum energy required to throw radius r around the earth has kinetic energy E. Then,
it out of earth’s gravitational fields is [2013] its angular momentum will be [2011]
K E E
a. K J b. J a. b.
2 mr 2 2mr 2
c. 2K J d. None of these
c. 2Emr 2 d. 2Emr
2. There is a shell of mass M and density of shell is
uniform. The work done to take a point mass from 7. If a new planet is discovered rotating around sun with
point A to B is (AB = r ) [2013] the orbital radius double that of the earth, then what
will be its time period? (in earth’s days) [2011]
A a. 1032 b. 1023
m r
M c. 1024 d. 1043
R 8. If the radius of the earth were to shrink by one
B percent, its mass remaining the same, the value of g
on the earth’s surface would [2010]
GmM GmM GmM
a. b. c. − d. zero a. increase by 0.5% b. increase by 2%
r R r c. decrease by 0.5% d. decrease by 2%
3. A satellite is in a circular orbit round the earth at an 9. The mean radius of the earth’s orbit around the sun is
altitude R above the earth’s surface, where R is the . × 1011 m and that of the orbit of mercury is
15
radius of the earth. If g is the acceleration due to
6 × 1010 m. The mercury will revolve around the sun
gravity on the surface of the earth, the speed of the
satellite is [2012] is nearly [2010]
2 3/ 2
a. 2 Rg b. Rg 2 2  2  2
a. yr b. yr c.   yr d.   yr
5 5  5  5
Rg Rg
c. d.
2 4 10. A satellite of mass m is orbiting around the earth at a
height equal to twice the radius of the earth (R). Its
4. Which is constant, the earth revolving around the
potential energy is given by [2009]
sun? [2012]
R −2 R
a. Angular momentum b. Linear momentum a. −2mgR b. −mg c. mgR d. −mg
2 3 2
c. Rotational kinetic energy d. Kinetic energy
5. Suppose the gravitational force varies inversely as the 11. A small mass m is moved slowly from the surface of
nth power of distance. Then, the time period of a earth to a height h above the earth.The work done in
planet in circular orbit of radius R around the sun will doing this is [2009]
be proportional to [2011] a. − mgR for h << R b. mgh for all values of h
(n + 1)/ 2 (n − 1)/ 2 1 1
a. R b. R c. − mgR for h = R d. mgR for h = R
c. Rn d. R(n − 2)/ 2 2 2

@iitjeehelps
GRAVITATION 115
12. The orbit of geostationary satellite is circular, the time 14. A satellite of mass m is placed at a distance r from the
period of satellite doesnot depends on [2008] centre of earth (mass M ). The mechanical energy of
a. mass of the satellite the satellite is [2006]
b. mass of the earth GMm GMm GMm GMm
a. − b. c. d. −
c. radius of the orbit r r 2r 2r
d. height of the satellite from the surface of earth
15. If M is the mass of the earth and R its radius, the ratio
13. There are two planets. The ratio of radius of the two of the gravitational acceleration and the gravitational
planets is K but ratio of acceleration due to gravity of constants is [2005]
both planets is g. What will be the ratio of their escape
R2 M M
velocity? [2007] a. b. c. MR 2 d.
M R2 R
a. (Kg )1/ 2 b. (Kg )−1/ 2 c. (Kg )2 d. (Kg )−2

Answer with Solutions


Practice Exercise 8. (a)
9. (b)
1. (a) 2. (b) 3. (a)
10. (d) Gravitational intensity due to the ring at a distance
4. (c) If equal masses are placed at respective corners of a d = 3 R on its axis is
regular polygon. Then, the gravitational force on a point GMd GM × 3 R 3 GM
mass placed at centre of the regular hexagon is zero. I= = =
(d 2 + R 2 ) 3/ 2 ( 3R 2 + R 2 ) 3/ 2 8R 2
Gm 2 Gm 2
5. (b) Here, F0 = = 3 GM 3 GM 2
( 2L) 2
2L2 Force on sphere = ( 8M ) I = ( 8M ) × =
8R 2 R2
F0 sin 45° D F0 sin 45°
11. (d) Gravitational force acting on an
object of mass m, placed at point P r
F0 at a distance h along the normal
45° 45° F0 P
through the centre of a circular ring
√2 L 2F0 sin 45° O h m
L of mass M and radius r is given by
√2 L GMmh
F = 2
A
L B L
C (r + h 2 )3/ 2
M
∴ Resultant force, When mass is displaced upto distance 2h, then
F = 2F0 cos 45° (along DB) GMm × 2h 2GMmh
F′ = 2 =
1 Gm 2 1 Gm 2 (r + ( 2h )2 )3/ 2 (r 2 + 4h 2 )3/ 2
= 2F0 × =2 2
× = (along DB)
2 2L 2 2L2 When h = r , then
6. (c) GMm × r GMm
Gm 2 F = =
7. (b) Here, F0 = 2 (r 2 × r 2 )3/ 2 2 2r 2
a 2GMmr 2GMm
2Gm 2 3 and F′ = =
∴ F = 2F0 cos 30° = (r 2 + 4r 2 )3/ 2 5 5 r 2
a2 2
m F′ 4 2 4 2
∴ = or F ′ = F
F 5 5 5 5
12. (a)
a a
13. (d) Mean orbital radius of the earth around the sun,
r O
F0 r = 1.5 × 108 km = 1.5 × 1011 m
2F0 cos 30°
m m Time period of the earth around the sun
a
F0 = 1 yr = 365 days
= 365 × 24 × 60 × 60 s
3 Gm 2 3 Gm 2 As, required centripetal force is obtained from the
or mrω 2 =
=
a 2
a2 gravitational force, therefore
2
a 2 3 Gm  3Gm 
or m ω = ∴ω=  3 
3 a 2  a 

@iitjeehelps
116 SELF STUDY GUIDE BITSAT

Centripetal force = Gravitational force  g′ − g 


∴ % increase in g =   × 100
mv 2 GMsm 2 GMs  g 
= ,v =
r r2 r  2g − g 
=  × 100 = 100%
(rω ) =
2 GMs
or ω = 3
2 GMs
[Qv = r ω]  g 
r r
20. (c)

But, ω= 4
T 21. (b) On the moon, gm = π G (R/4)( 2ρ/3)
3
2
 2π  GM 4π 2r 3 14  1
∴   = 3 s or Ms = =  π GRρ = g
T  r GT 2 6 3  6
4 × (3.14)2 × (1.5 × 1011)3 Work done in jumping = m × g × 0.5 = m × (g/6) h1
= h1 = 0.5 × 6 = 3.0 m
6.67 × 10−11 ( 365 × 24 × 60 × 60 )2
22. (d) From the Kepler’s law,
≈ 2 × 1030 kg
T12 r13
GM =
14. (a) g1 = T22 r23
(R + h )2
1 r13
2 2 ⇒ = ⇒ r2 = 4 × 104 km
g1  R   R  64 r23
∴ =  ∴ g1 =   g
g R + h R + h 2πr1 2πr2
∴ v1 = and v 2 =
 
2 T1 T2
6400 g
g1 =   ×g = ⇒ v1 = 2 × 104 π kmh −1
 6400 + 2624 2
and v 2 = π × 104 kmh −1
Q At surface of earth, mg = kx 0
At the height h = 2624 km, mg1 = kx Thus, speed of B relative to A is v 21.
x g1 i.e. v 21 = π × 104 kmh−1
∴ =
x0 g 23. (a) 24. (d)
g 1 25. (b) Since, particle is shifted slowly. So, change in kinetic
∴ x = 1 × x 0 = × 2 = 1cm
g 2 energy is zero.
15. (a) According to work-energy theorem
W + Wext = 0
16. (a) As g ′ = g − ω 2R cos2 φ
Wext = − W = − 40 J
At the equator, φ = 0, i.e. cos φ = 1
26. (b) Here, F = m E = ma (y$i + x $j )
For weight to be zero g ′ = 0
Q r = x$i + y$j
g
∴ ω= = 0.00125 rad s −1 ∴ d r = dx$i + dy $j
R
x 0, y 0, z 0
17. (b) The value of g at two places will must be same ∴ W = ∫ F ⋅ d r = ma ∫ (y $i + x $j ) ⋅ (dx $i + dy $j )
0, 0, 0
gR 2  h = ma ∫ d (xy ) = ma [xy ] 0x,00,,y00, z 0 = ma x 0y 0
⇒ = g 1 − 
(R + h )2  R
27. (c)
 h  h2 2h 
⇒ 1 −  1 + 2 +  =1 28. (b)
 R  R R 1 1
29. (c) Change in PE = ∆U = Uf − Ui = GmM  − 
h 3
h h 2  R nR 
⇒ + − =0
R3 R2 R GMm  n − 1  n − 1  Gm 
=   = mgR   Q g = 2 
R  n   n   R 
h  h2 h 
⇒  + − 1 = 0 2πR
R R2 R  30. (b)T =
v
5 R − R R ( 5 − 1)  2GM 
1/ 2
⇒ h= = 1 GMm
E = mv 2 = n − 1 or v = n − 1
2 2 2 R  R 
18. (a) 2 πR 2π
∴ T= = × R(n + 1)/ 2
19. (d) Here, g = GM /R 2 and 2GM 2GM
G(M /2) 2GM Rn −1
g′ = = = 2g
(R/2)2 R2 ∴ T ∝ R(n + 1)/ 2

@iitjeehelps
GRAVITATION 117
31. (b) = − 6.67 × 10−11( 2 + 2)
32. (b) Applying law of conservation of energy, = − 4 × 6.67 × 10−11
1 GMm 1
mv 2 − = mv12 + 0 Uf = 0 at infinity
2 R 2 1
1 1 GMm ∴ mve2 = ∆Uf − Ui = 4 × 6.67 × 10−11
or mv1 = mv 2 −
2
2
2 2 R
2GM ∴ ve = 2 × 4 × 6.67 × 10−11 = 2.31× 10−5 m/s
or v12 = v 2 − = v 2 − ve2
R 2GM
38. (c)ve = escape velocity =
∴ v1 = v 2
− ve2 = 4gR − 2gR = 2gR R +h

33. (b) GM
vo = orbital velocity =
34. (b) If a body is projected from the surface of the earth with R +h
a velocity v and reaches a height h, then according to the ve
Given, vo =
law of conservation of energy, 2
1 mgh GM 1 2GM
mv 2 = ∴ =
2 h R +h 2
1+ R
R
GM 1 2GM
Here, v = kve = k 2gR or = ×
R +h 4 R
1 mg (r − R )
∴ mk 2 ⋅ 2gR = On solving, we get h = R
2 (r − R )
1+ 39. (c) During motion of the particle, total mechanical energy
R
remains constant.
2  r −R 
k R 1+ =r −R At the surface of earth, total mechanical energy is
 R 
GmM 1
or k 2r = r − R Ei = − + mv 02
R 2
R GM 1
or r= = − 2 mR + mv 02
1− k 2 R 2
GMms msvs2 1
35. (c)Q = = − gmR + mv 02
rs2 rs 2
1/ 2 Total mechanical energy at height h = R is
 GM  − GmM 1
vs =   Ef = + mv 2
 rs  2R 2
Angular momentum = msvsrs = J gmR 1
=− + mv 2
 GM 
1/ 2 2 2
∴ J =  msrs = (GMmr2rs )1/ 2 According to conservation principle of energy,
 rs 
Ei = Ef
1 1 GM
Q KE =msvs2 = ms ∴
1
− gmR + mv 02 = −
gmR 1
+ mv 2
2 2 rs 2 2 2
GMms − 2gR + v 0 = − gR + v
2 2
PE = − or
rs
∴ v = v 02 − gR
GMms
Total energy, E = −
2rs 40. (a) 41. (a)
∴ J = ( 2Ems rs2 )1/ 2 42. (a) Orbital speed closer to the earth is gR .
1 mgR
36. (a) K = KE = m ( gR )2 =
2 2
6.67 × 10−11 × 12 6.67 × 10−11 × 12
37. (b)Ui = − − Now, if the velocity is changed to 2gR (escape speed),
 1  1
    then the kinetic energy K′ will be
 2  2
1
1kg 1kg 1kg K ′ = m ( 2gR )2 = mgR
2
1
2
m ⇒ K ′ = 2K

@iitjeehelps
118 SELF STUDY GUIDE BITSAT

BITSAT Archives
1. (a) If the satellite could be through outside the earth’s 7. (a) By Kepler’s third law, T 2 ∝ R 3
gravitational field, its minimum total energy = 0. 2
 T2   2r 
3
So, − K + K′ = 0 ∴   = 
 365 days r 
K′ = K J
2. (d) T2 = 365 × 2 2 = 1032 days
A M GM
8. (b) g =
m R2
B
∆g  ∆R 
× 100 = 2  × 100 = 2 × 1% = 2%
g  R 
Therefore, the value of g will increase by 2%.
The gravitational field at A and B are zero. So, no work 9. (d) According to Kepler’s law,
required to make change between the points A and B. T12 r13
=
3. (c) Orbital velocity (vo ) at a height h above the earth’s T22 r23
surface is given by 3
1  15. × 1011
3
 5
g =   =  
vo = Re T22  6 × 1010   2
Re + h
3/ 2
Given, h = Re  2
T2 =   yr
g  5
∴ vo = R 3R GMm
2R 10. (c) PE = ∫ − dr
Rg
R r2
⇒ vo = 3R
2  r −1   1 1
= − GMm   = GMm −
− 1  3R R 
4. (a) Kepler’s second law  R
dA L
= 2 GMm 2 gR 2m 2
dt 2m =− =− = − mgR
3 R 3 R 3
dA
= constant 11. (d) When h = R
dt
L = constant GMm  GMm  GMm 1
Work done = − − − = = mgR
5. (a) The necessary centripetal force required for a planet (R + R )  R  2R 2
to move round the sun 12. (a) Time period of satellite,
= gravitational force exerted on it 1
1/ 2 T ∝ 1/ 2 , where M is mass of earth.
mv 2 GMem  GM  M
= or v =  n − 1
R R n R  T ∝ (R + h )3/ 2, where R is radius of the orbit, h is the height
1/ 2
2 πR  Rn − 1 of satellite from the earth’s surface.
As, T = = 2 πR ×  
v  GM  13. (a)v = 2gR
 (n + 1)  v1 g R
∴ = 1 × 1 = g × K = (Kg )1/ 2
 R 2  v2 g 2 R2
T = 2π 
(GMe )1/ 2 
  14. (d) If a satellite of mass m is placed at a distance r from
 
(n + 1)/ 2 the centre of the earth (of mass M), the mechanical
T ∝R −GMm
energy of the satellite is given by .
6. (c) KE of a satellite 2r
1 GM
E= mv 2 15. (b) Gravitational acceleration is given by g = 2
2 R
or mv = 2Em where, G = gravitational constant
Angular momentum, g M
∴ =
L = mvr = ( 2Em ) × r = 2mEr 2 G R2

@iitjeehelps
10
Simple Harmonic
Motion
Periodic Motion
A motion which repeats itself over and over again after a regular interval of time is called a periodic
motion. A periodic motion in which a body moves back and forth repeatedly about a fixed point
(called mean position) is called oscillatory or vibratory motion.
Period The regular interval of time after which periodic motion repeats itself is called period of the
motion.
Frequency The number of times of motion repeated in one second is called frequency of the periodic
motion. Every oscillatory motion is periodic but every periodic motion is not an oscillatory motion.

Displacement as a Function of Time


In a periodic motion each displacement value is repeated after a regular interval of time, displacement
can be represented as a function of time y = f (t ).

Periodic Function
A function which repeats its value after a fix interval of time is called a periodic function.
y(t ) = y(t + T )
where, T is the period of the function.
Trigonometric functions sin θ and cosθ are simplest periodic functions having period of 2π.

Simple Harmonic Motion (SHM)


It is a special kind of oscillatory motion in which particle moves to and fro about a mean position
(on a straight line) under a restoring force which is directed towards mean position and its magnitude
(of restoring force) is directly proportional to the displacement of particle (at all instants). For simple
harmonic motion, displacement should be very small.
Simple harmonic motion is a special form of oscillatory or vibratory motion.

@iitjeehelps
120 SELF STUDY GUIDE BITSAT

It is of two types Restoring force, F = − kx , where x is displacement, k


1. Linear simple harmonic motion is spring constant.
F a
2. Angular simple harmonic motion
F = – kx F = – ω2x
Linear SHM x x

In linear SHM, a particle moves to and fro (on a straight


line) about fixed point under a restoring force (always
directed towards mean position) (a) (b)
m Slope = – k Slope = – ω2
whose magnitude is directly
proportional to displacement of the (iii) Force constant (Spring constant)
particle (at all instants). We know that, defining equation of SHM is
e.g. Motion of a block connected to a spring on a smooth F
surface. a = − ω 2 x , also a =
m
Restoring force ∝ Displacement F
∴ =−ω x 2

F ∝ − x (if displacement is along X-axis) m


∴ Acceleration, a ∝ − x F = − mω 2 x ⇒ F = − kx

d 2x k is called spring constant or force constant.


or ∝− x
dt 2 Also, k = mω 2 .
⇒ a = − ω 2x (iv) Amplitude Maximum Extreme Extreme
(Mean position) position
displacement of position
where, a is acceleration and ω is angular frequency.
particle from mean
Negative sign indicates direction of restoring force and position on either side
acceleration is towards equilibrium position, but in is defined as
opposite direction of displacement. amplitude. It is a scalar Amplitude Amplitude
quantity.
Angular SHM 1
Amplitude = × (Distance between extreme points
In angular SHM, the restoring torque (or angular 2
acceleration) acting on the particle is proportional to the or position)
angular displacement of the particle and directed towards (v) Instantaneous displacement Displacement of
equilibrium position. particle from mean position in a particular direction
(at any instant of time) is defined as instantaneous
displacement.
θ
It is given by x = Asin(ωt + φ )
where, A = amplitude
φ = initial phase or phase constant

Restoring torque ∝ Angular displacement Velocity


τ∝−θ The velocity of a particle executing SHM at an instant is
defined as the time rate of change of its displacement at that
⇒ α ∝−θ
instant.
d θ
2
or ∝−θ Velocity, v = ω A2 − y 2
dt 2
At the mean position ( y = 0), during its motion
Some Terms Related to SHM v = Aω = vmax and at the extreme positions ( y = ± A)v = 0.
Velocity amplitude = vmax = Aω
(i) Mean position It is a point at which restoring force
on the particle is zero.
(ii) Restoring force The force which is acting on the Acceleration
particle tends to bring the particle towards its mean The acceleration of a particle executing SHM at an instant is
position is known as restoring force. defined as the time rate of change of velocity at that instant.

@iitjeehelps
SIMPLE HARMONIC MOTION 121
Acceleration, a = − ω2y
The acceleration is also a variable. At the mean position
Oscillations of a Spring Block
( y = 0), acceleration a = 0 and at the extreme position System
( y = ± A), the acceleration is amax = − Aω 2 . A spring pendulum consists Elongation
∴ Acceleration amplitude, amax = Aω 2 of a point (small sized) mass x1
m either suspended from a
massless (or light) spring or
Phase and Phase Relationship placed on a smooth
Natural position
Compression
Phase is that physical quantity which tells about the horizontal plane attached
x2
position and direction of motion of any particle at any with a spring.
moment. It is denoted by φ. In SHM, the velocity is ahead of Here, x1 = x2 = l
π
the displacement by a phase and the acceleration is If the mass is once pulled so as to stretch the spring and is
2
π then released, then a restoring force acts on it which
further ahead of the velocity by a phase of . continuously tries to restore its mean position, restoring
2 force F = − kl, where k is force constant and l is the change
in length of the spring under the restoring force the spring
Phase Difference pendulum oscillates simple harmonically having time
The difference of total phase angles of two particles period and frequency given by
executing SHM with respect to their mean positions is m 1 k
T = 2π and ν =
known as phase difference. If phase angles of two particles k 2π m
executing SHM are(ωt + φ 1 ) and(ωt + φ 2 )respectively, then
where, k is the force constant of the spring and it is
∆φ = (ωt + φ 2 ) − (ωt + φ 1 ) numerically equal to the force required to increase the
∆φ = φ 2 − φ 1 length of the spring by unity.
If the spring is not light but has a mass ms , then
Time Period
m + 1 / 3 ms
The time taken by a particle to complete one oscillation is T = 2π
k
called time period. It is denoted by T .
Acceleration in SHM, a = − ω2y NOTE If two masses m1 and m2 , connected by a spring are made to
oscillate on a horizontal surface, then its period will be
|a |
Hence, |a | = ω 2 | y | ⇒ ω = T = 2π
µ
| y| k
∴ Time period of SHM, k
2π | y| Displacement m1 m2
T = = 2π = 2π
ω |a | Acceleration
m1m2
where, µ = = reduced mass of the system.
m1 + m2
Equation of Simple Harmonic Motion
A simple harmonic motion may be mathematically Series Combination of Springs
expressed by a single sinusoidal (sine or cosine) function of
If two springs of spring constants k1 and k2 are joined in
time. The equation of simple harmonic motion is given by
series (horizontally and vertically), then their equivalent
y = Asinωt spring constant ks is given by
and x = Acosωt
● Differential equations of SHM for linear SHM, k1 k1
d2y d 2θ
2
+ ω 2 y = 0, for angular SHM, 2 + ω 2 θ = 0 k1 k2 m
dt dt
k2 k2
● All sine and cosine functions of t are simple harmonic in
nature, i.e. for the function
m
● y = Asin(ωt ± φ ) or y = Acos(ωt ± φ ) m
2 (a) (b) (c)
d y

2
is directly proportional to − y. Hence, they are
dt 1 1 1
= +
simple harmonic in nature. ks k1 k2

@iitjeehelps
122 SELF STUDY GUIDE BITSAT

k1k2
⇒ ks = 2. Potential energy This is an account of the
k1 + k2 displacement of the particle from its mean position.
m m(k1 + k2 ) 1
∴ T = 2π = 2π U = mω 2 x 2
ks k1k2 2
1
= mA2ω 2 sin 2 (ωt + φ )
2
Parallel Combination of Springs
Thus, potential energy has its minimum at the centre
If the two springs of spring constants k1 ( x = 0) and increases as the particle approaches either
and k2 are joined in parallel as shown in extreme of oscillation ( x = ±A).
the figure then their equivalent spring
constant k p = k1 + k2 , hence k1 k2 3. Mechanical energy This is an account of kinetic and
potential energy.
m
T = 2π Thus, total mechanical energy = kinetic energy +
kp
potential energy
m 1
= 2π m or E= mω 2 A 2
(k1 + k2 ) 2
(b)
Obviously, the total energy is constant and is
proportional to the square of amplitude (A) of motion.
Spring Pendulum Figures show the variations of total energy (E),
A point mass suspended from a massless potential energy (U) and kinetic energy (K) with
(or light) spring constitutes a spring pendulum. displacement (x).
If the mass is once pulled downwards so as to
stretch the spring and then released, the k Total energy = U ( t) + K ( t)
system oscillates up and down about its mean E
position simple harmonically. Time period (T ) Energy
U ( t)
and frequency (ν) of oscillations are given by (a)
m K ( t)
m 1 k
T = 2π or ν = O t
k 2π m T/ 2 T
If the spring is not light but has a definite mass ms , then it
can be easily shown that period of oscillation will be 1 2 U, K
E= kA 1
m 2 U = kx 2
2
m+ s
T = 2π 3
k
(b) 1
K= k(A2 – x2)
2
Energy in Simple Harmonic
Motion – O +A
Displacement
If a particle executes SHM, its kinetic energy changes into
potential energy and vice-versa keeping total energy
constant (if friction of air is neglected). Some Important Points
● KE and PE also varies periodically.
A particle executing SHM possesses two types of energy
● Frequency of PE or KE is double than that of SHM.
1. Kinetic energy This is an account of the velocity of the ● The frequency of total energy in SHM is zero.
particle. ● The average values of KE and PE in one complete revolution
1 1 1 1
K = mv 2 = mA2ω 2 cos2 (ωt + φ ) = mω 2 ( A2 − x 2 ) is mω 2 A 2 .
2 2 2 4
● The average value of total energy in one complete revolution
From this expression, we conclude that kinetic energy is 1
maximum at the centre ( x = 0 ) and zero at the extremes of is mω 2 A 2 .
2
oscillation ( x = ± A ).

@iitjeehelps
SIMPLE HARMONIC MOTION 123
said to be free vibrations. The natural frequency of free
Simple Pendulum vibrations depends on the nature and structure of the body
A simple pendulum, in practice, consists of a heavy but and in ideal situation, the amplitude, frequency and the
small sized metallic bob suspended by a light, inextensible energy of the vibrating body remain constant.
and flexible string. The motion of a simple pendulum is
simple harmonic for very small angular displacement (α) Forced Vibrations
whose time period and frequency are given by
The vibrations in which a body oscillates under the effect of
l 1 g
T = 2π and ν = an external periodic force, whose frequency is different
g 2π l from the natural frequency of oscillating body are called
where, l is the effective length of the string and g is forced vibrations. In forced vibrations, the oscillating body
acceleration due to gravity. vibrates with the frequency of external force and amplitude
of oscillations is generally small.
(i) If a pendulum of length l at temperature θ°C has a
time period T , then on increasing the temperature by If an external driving force is represented by
∆θ°C its time period changes to T × ∆T , F ( t ) = F0 cosω d t
∆T 1 The motion of particle is under combined action of
where, = α ∆θ
T 2 restoring force ( −kx ), damping force ( −bv ) and driving force
where, α is the temperature coefficient of expansion F ( t ).
of the string. Now, ma = −kx − bv + F0 cosω d t
(ii) A second’s pendulum is a pendulum whose time d 2x kx b dx F0 cosω d t
or =− − +
period is 2 s. At a place where g = 9.8 m/s 2 , the dt 2 m m dt m
length of a second’s pendulum is 0.9929 m (or 1 m
The solution of this equation gives x = x0 sin (ω d t + φ ) with
approx).
amplitude
(iii) If the bob of a pendulum (having density ρ) is made
F0 / m ω 20 − ω 2d
to oscillate in a non-viscous fluid of density σ , then it x0 = , tan θ =
can be shown that the new period is  bω 
2 bω d / m
(ω 20 − ω 2d )2 +  
l  m
T = 2π
 σ k
g 1 −  and ω0 = = natural frequency
 ρ m
(iv) If a pendulum is in a lift or in some other carriage
moving vertically with an acceleration a, then the Damped Vibrations
effective value of the acceleration due to gravity
When a body is set in free vibrations, generally there is a
becomes ( g ± a ) and hence,
dissipation of energy due to dissipative causes like viscous
l drag of a fluid, frictional force, hysteresis, electromagnetic
T = 2π
( g ± a) damping force, etc., and as a result amplitude of vibration
Here, positive sign is taken for an upward regularly decreases with time. Such vibrations of
accelerated motion and negative sign for a continuously falling amplitudes are called damped
downward accelerated motion. vibrations. In these oscillations, the amplitude of oscillation
decreases exponentially and hence, energy also decreases
(v) If a pendulum is made to oscillate in a freely falling
exponentially. If the velocity of an oscillator is v, the
lift or an orbiting satellite then the effective value
damping force
of g is zero and hence, the time period of the
pendulum will be infinity and therefore pendulum Fd = −bv
will not oscillate at all. where, b = damping constant.
Resultant force on a damped oscillator is given by
Free, Forced, Damped and F = FR + FD = −kx − bv or
md 2 x bdx
+ + kx = 0
dt 2 dt
Resonant Vibrations Displacement of a damped oscillator is given by
x = xm e − bt / 2 m sin (ω ′t + φ )
Free Vibrations
If a given body is once set into vibrations and then let free to ω ′ = ω02 − (b /2m )2
vibrate with its own natural frequency, the vibrations are where, ω ′ = angular frequency of the damped oscillator

@iitjeehelps
124 SELF STUDY GUIDE BITSAT

For a damped oscillator, if the damping is small then the (iii) When a ball of mass m is made to oscillate in the
mechanical energy decreases exponentially with time neck of an air chamber having volume V and neck
1 2 − bt / m area A, then
as E = kx m e
2  mV 
T = 2π  2 
 pA 
Resonant Vibrations
(iv) If a person sitting on an oscillating swings stands up,
It is a special case of forced vibrations in which frequency
the time period of the swing decreases.
of external force is exactly same as the natural frequency of
oscillator. As a result the oscillating body begins to vibrate (v) When a pendulum is kept in a car which is sliding
with a large amplitude leading to the resonance down, then
phenomenon to occur. Resonant vibrations play a very  1 
important role in music and tuning of station/channel in a T = 2π  
 g cosθ 
radio/TV.
where, θ is an angle of inclination.
Some Important Points (vi) If time period of one spring is T 1 and that of second
spring is T 2 and if they are connected in series, then
 K2 
l +  T series = T 12 + T 22 . If they are connected in parallel,
(i) Physical or compound pendulum, T = 2 π  l 
 g  then
 
  T 1T 2
T parallel =
where, l = distance of centre of gravity form point of T 12 + T 22
supension and
(vii) The time period of a simple pendulum having long
K = radius of gyration about an axis passing through
centre of gravity.  lR 
length is T = 2π  
(ii) In the case of water oscillating in a U-tube, (l + R) g 
h where, R= radius of earth
T = 2π  
 g  R
If length is infinite, then T = 2π  
where, h is the height of liquid column in each limb.  g

Practice Exercise
1. A particle executing simple harmonic motion has 5. A particle moves along Y-axis according to equation
amplitude of 1 m and time period 2 s. At t = 0, net force y = 3 + 4 cos ωt . The motion of particle is
on the particle is zero. The equation of displacement a. not SHM b. oscillatory but not SHM
of particle is c. SHM d. None of these
a. x = sin πt b. x = cos πt c. x = sin 2πt d. x = cos 2πt 6. In Q. 5, the amplitude of vibration is
2. In Q. 1, find maximum velocity and maximum a. 3 units b. 4 units
acceleration. c. 5 units d. None of these
a. 1 m/s, π m/s 2 b. π m/s and π 2 m/s 2 7. If s = a sin ωt $i + b cos ωt $j, the equation of path of
c. π m/s and π m/s 2 d. None of these particle is
3. A particle executes simple harmonic motion. The x2 y2
a. x 2 + y 2 = a 2 + b 2 b. + =1
amplitude of vibration of particle is 2 cm. The b2 a2
displacement of particle in one time period is x2 y2
c. + =1 d. None of these
a. 1 cm b. 2 cm c. 4 cm d. zero a2 b2
4. In Q. 3, the distance travelled by the particle is 8. In previous question, the amplitude of vibration is
a. 8 cm b. 2 cm a. 4 units b. 8 units
c. 4 cm d. zero c. 10 . 58 units d. None of these

@iitjeehelps
SIMPLE HARMONIC MOTION 125
9. The motion of a particle varies with time according to 16. A point mass m = 20 kg, is suspended by a
the relation
massless spring of constant 2000 N/m.
y = a sin ωt + a cos ωt . Then,
The point mass is released when
a. the motion is oscillatory but not SHM
b. the motion is SHM with amplitude a elongation in the spring is 15 cm. The
c. the motion is SHM with amplitude 2a equation of displacement of particle as m
d. None of the above function of time is (take, g = 10 m/s 2)
10. A particle executes SHM along a straight line so that a. y = 10 sin 10t b. Y = 10 cos 10t
its period is 12 s. The time it takes in traversing a
 π
distance equal to half its amplitude from its equilibrium c. y = 10 sin 10t +  d. None of these
 6
position is
a. 6 s b. 4 s 17. A springs of spring constant 200 g
c. 2 s d. 1 s 3
N/m has a block of mass 1 kg
11. A particle executes SHM with an amplitude of 10 cm hanging at its one end and from
and frequency 2 Hz. At t = 0, the particle is at a point other end spring is attached to a
ceiling of an elevator. The
where, potential energy and kinetic energy are same. elevator rising upwards with an
The equation of displacement of particle is acceleration of g/3. When m
 π acceleration is suddenly cease,
a. 0.1sin  4πt + 
 4 then what should be the angular
b. 0.1sin 4πt frequency and elongation during
 π the time when the elevator is accelerating?
c. 0.1cos  4πt + 
 4 a. 14.14 rad/s, 0.07 m b. 14 rad/s, 0.1m
d. None of the above c. 14.14 rad/s, 0.05 m d. 10 rad/s, 0.07 m
12. A particle executes simple harmonic motion with a 18. A spring of force constant k is cut into two pieces such
frequency f. The frequency with which the potential
that one piece is double the length of the other. Then,
energy oscillates is
the long piece will have a force constant of
a. f b. f / 2 2 3
c. 2f d. zero a. k b. k c. 3 k d. 6 k
3 2
13. A particle of mass m is executing oscillation about the
19. A solid copper sphere is suspended from a massless
origin on the X-axis. Its potential energy is u (x ) = k | x |3 ,
spring. The time period of oscillation of the system is
where k is a positive constant. If the amplitude of 4 s. The sphere is now completely immersed in a
oscillation is a, then its time period T is liquid whose density is 1/8th that of brass. The sphere
1 remains in liquid during oscillation. Now, the time
a. proportional to b. independent of a
a period is
c. proportional to a d. proportional to a 3/2 a. 4 s b. 2 s
14. A particle of mass m is free to move along the x-axis c. 3 s d. None of these
2
and has potential energy given by u (x ) = k [1− e − x ] for 20. A load of mass m falls from a height h on to
− ∞ ≤ x ≤ ∞, where k is positive constant of appropriate the scale pan hung from a spring as shown
dimensions. Then, in the adjoining figure. If the spring constant
k
is k and mass of the scale pan is zero and
a. at points away from the origin, the particle is in
the mass m does not bounce relative to the
unstable equilibrium
b. for any finite non-zero value of x, there is a force
pan, then the amplitude of vibration is m
h
directed away from the origin mg mg  1+ 2hk 
k a. b.  
c. its total mechanical energy is , it has its minimum k k  mg 
2
kinetic energy at origin mg mg  1+ 2hk  mg  1+ 2hk mg 
c. +   d.  − 
d. for small displacement from x = 0, motion is SHM k k  mg  k  mg k 
15. A simple harmonic oscillator has amplitude A, angular 21. In Q. 20, find the amplitude of vibration.
velocity ω, and mass m. Then, average energy in one
time period will be  mv 2   mv 2 
a.   b.  
1 1  k   2k 
a. mω 2 A 2 b. mω 2 A 2
4 2  mv 2 
c. mω 2A 2 d. zero c.   d. None of these
 4k 

@iitjeehelps
126 SELF STUDY GUIDE BITSAT

22. Two point masses of 3 kg and 1 kg are attached to l  l 


opposite ends of a horizontal springs whose spring a. T = 2π   b. T = 2π  2 
g  a0 + g 
2
constant is 300 N/m
as shown in figure.  l   l 
1 kg 3 kg c. T = π   d. T = 2π  
The natural vibration  a2 + g2   a2 −g2 
 0   0 
frequency of the
system is of order 29. There is a clock which give correct time at 20° C is
a. 4 Hz b. 3 Hz c. 2 Hz d. 1 Hz subjected to 40°C. The coefficient of linear expansion
23. Two blocks of the pendulum is 12 × 10−6 per ° C, how much is gain
m1 m2 F
connected by a or loss in time?
spring rest on a a. 10.3 s/day b. 19 s/day c. 5.5 s/day d. 6.8 s/day
smooth horizontal plane as shown in figure. If a
constant force F starts acting on the block m 2 30. There are two pendulums of length l1 and l 2 start
a. length of spring increases continuously, if m1 > m2 vibrating. At some instant, the two are in mean
position in the same phase. Calculate after how many
b. blocks start performing SHM about centre of mass
of the system with increasing amplitude
vibrations of shorter pendulum, the two will be in
c. blocks start performing SHM about centre of mass phase in the mean position?
of the system which moves rectilinearly with constant [(l1 > l 2 ), l1 = 121cm, l 2 = 100 cm]
acceleration a. 11 b. 10 c. 9 d. 8
d. acceleration of m2 is maximum at initial moment of
31. A clock is performing SHM along a vertical line with
time only
amplitude of 40 cm on a horizontal plank. The block
24. There is a spring with nature length L0. Two just lose the contact with plank when plank is
m1
masses m1 and m 2 are connected to both of momentarily at rest. Then, (Take, g = 10 m/s 2)
its ends as shown in figure. The whole 2π
system is held at rest. At any time t = 0, m 2 is g a. the period of its oscillation is s
5
released and system starts free fall. Initial 2π
b. the period of its oscillation is s
stretched length of spring before fall is L0. 6
What is the displacement of centre of mass π
m2 c. the period of its oscillation is s
as a function of time? 5
1 2 g 2 m1 + m2 d. None of the above
a. gt 2 b. gt c. t d. ×t
2 k m1m2 32. There is a ring or mass m and radius R is pivoted at a
point O on its periphery. It is free to rotate about an
25. A second’s pendulum has time period 2s. The axis perpendicular to its plane. What is the period of
spherical bob which is empty has mass of 50 g. This is ring?
replaced by another solid bob of same radius but
R  2R 
having mass of 100 g. The new time period will be a. T = 2π   b. T = 2π  
g  g 
a. 4 s b. 1 s
c. 2 s d. 8 s  2R   3R 
c. T = π   d. T = 2π  
 g   g 
26. A clock pendulum is adjusted for giving correct time in
Patna. This clock pendulum also gives correct time in 33. There is a rod of length l and mass m . It is hinged at
a. Delhi b. Kota one end to the ceiling. Find the period of small
c. Hyderabad d. None of these oscillation.
27. A simple pendulum of length L and mass M is  2l   l 
a. T = 2π   b. T = π  
oscillating in a plane about a vertical line between  3g   3g 
angular limits − φ and + φ. For an angular
 l  l
displacement, the tension in the string and the velocity c. T = 2π   d. T = 2π  
 3g  g
of the bob are T and v respectively. The following
relation holds good under the above conditions 34. A particle of mass m is allowed to y
a. T = Mg cosθ b. T cosθ = Mg oscillate near the minimum of a g m
Mv 2 vertical parabolic path having the
c. T − Mg cosθ = d. None of these
L equation x 2 = 4ay . The angular
frequency of small oscillations is
28. From the ceiling of a train, a pendulum of length ‘l’ is
given by
suspended. The train is moving with an acceleration
a 0 on horizontal surface. What must be the period of g g
a. gh b. 2gh c.   d.  
oscillation of pendulum?  2a  a

@iitjeehelps
SIMPLE HARMONIC MOTION 127
35. A highly rigid cubical block of mass m and side L is h
fixed rigidly on to another cubical block B of the same p
dimensions and of low modulus of rigidly η such that M
the lower face of A completely covers the upper face A
of B. The lower face of B is rigidly held on a horizontal
surface. A small force F is applied perpendicular to
one of the side faces of A. After the force is withdrawn,
The cylinder is kept with its axis horizontal. If the
block A executes small oscillation, the period is
piston is disturbed from its equilibrium position, it
 η  mL m oscillates simple harmonically. Find its period.
a. 2π ηmL b. 2π m  c. 2π   d. 2π  
 L  η  ηL
 Mh   MA 
a. T = 2π   b. T = 2π  
36. A cylindrical piston of mass M slide smoothly inside a  pA   ph 
long cylinder closed at one end, enclosing a certain  M 
mass of gas. c. T = 2π   d. T = 2π MphA
 pah 

BITSAT Archives
1. A particle of mass m = 5 g is executing simple d 2y
bdy
7. The equation + + ω 2y = 0 represents the
harmonic motion with an amplitude 0.3 m and time dt 2 dt
period π/5 second. The maximum value of force acting equation of motion for a [2010]
on the particle is [2013] a. free vibration b. damped vibration
a. 5 N b. 4 N c. 0.5 N d. 0.15 N c. forced vibration d. resonant vibration
2. Pulse rate of a normal person is 75 per minute. The 8. A pole is floating in a liquid with 80 cm of its length
time period of heart is [2013] immersed. It is pushed down a certain distance and
a. 0.8 s b. 0.75 s c. 1.25 s d. 1.75 s then released. Time period of vertical oscillation is
3. A simple wave motion represented by [2010]
y = 5 (sin 4 πt + 3 cos 4 πt ). Its amplitude is 4π 3π 2π π
[2012] a. s b. s c. s d. s
7 7 7 7
a. 5 b. 5 3 c. 10 3 d. 10
9. A pendulum has a ball fo mass m attached to the
4. If the displacement of simple pendulum at any time is string and is suspended from the roof of a trolley. If the
2
0.02 m and acceleration is 2 m/s , then in this time trolley rolls upwards with acceleration a, then what is
angular velocity will be [2012]
the angle made by the string with the inclined plane?
a. 100 rad/s b. 10 rad/s c. 1 rad/s d. 0.1 rad/s (Given that, a = 5 m/s 2, g = 10 m/s 2 and the angle of
5. If ks and k p respectively are effective spring constant inclination of the plane is 30°) [2009]
in series and parallel combination of springs as shown  2 −1  2  −1  2  −1  2 
k a. cos−1   b. cot   c. sin   d. tan  
in figure, find s .  3  3  3  3
kp [2012]
10. A particle is executing simple harmonic motion with an
amplitude A and time period T. The displacement of
the particles after 2T period from its initial position is
a. A b. 4A [2008]
c. 8A d. zero
11. In a second's pendulum, mass of bob is 30 g. If it is
9 3 2 7 replaced by 90 g mass, then its time period will be
a. b. c. d.
2 7 9 3 a. 1 s b. 2 s [2007]
c. 4 s d. 3 s
6. What is the maximum acceleration of the particle
doing the SHM? 12. A simple pendulum hanging from the ceilling of a
stationary lift has time period t1. When the lift moves
 πt 
y = 2 sin  + φ, where 2 is in cm downward with constant velocity, the time period is t 2,
 2  [2011] then [2006]
π π2 π π a. t 2 is infinity b. t 2 > t1
a. cm/s 2 b. cm/s 2 c. cm/s 2 d. cm/s 2
2 2 4 4 c. t 2 < t1 d. t 2 = t1

@iitjeehelps
Answer with Solutions
Practice Exercise 8. (c)
 π
1. (a)Q In the case of SHM, 9. (c) y = a sin ωt + a cos ωt = a sin ωt + a sin ωt + 
 2
x = a sin(ωt + φ )
2π 2π The resultant motion is SHM but resultant amplitude is
Here, a = 1m,ω = = = π rad/s π
T 2 A = a 2 + a 2 + 2a 2 cos = 2a
At t = 0, net force on the particle is zero. 2
Hence, particle is at mean position at t = 0 10. (d) Since, particle starts from mean position.
∴ x = 0 at t = 0 ∴ x = A sin ωt
∴ 0 = a sin (ω × 0 + φ ) ⇒ φ = 0 ∴
A 1
= A sin ωt 0 or = sin ωt 0
∴ x = a sin ωt 2 2
∴ x = 1× sin π t 2π π
or sin t 0 = sin
∴ x = sin πt T 6
2π π
2. (b)v max =aω = 1× π = π m/s 2 or t0 =
T 6
a max =− ω 2a = − π 2 m /s 2 T 12
∴ t0 = = = 1s
∴ | a max | = π 2 m/s 2 12 12
3. (d)Qx = A sin ωt 11. (a) Let x = A sin (ωt + φ )
 2π  dx
∴ x = A sin  ×T  ∴ v= = A ω cos(ωt + φ )
T  dt
∴ x = A sin 2π = 0 1 1
∴ KE = mv 2 = mA 2ω 2 cos2 (ωt + φ )
4. (a) Since, the distance travelled in T/4 time is S1 = A 2 2
1
∴The distance travelled in one time. ∴ ( KE)max = mA 2ω 2
2
s = 4s 1 = 4A 1
∴ s = 8 cm (Q A = 2 cm) ∴ PE = mA 2ω 2 − KE
2
5. (c) y = 3 + 4 cosω t 1 1
= mA 2ω 2 − mA 2ω 2 cos2 (ωt + φ )
2 2
7 units 1
= mA 2ω 2 sin2 (ωt + φ )
3 units 2
y According to problem, KE = PE (at t = 0)
1
∴ = mA ω sin (ωt + φ )
2 2 2
2
t 1
= mA 2ω 2 cos2 (ωt + φ )
The y-t graph is shown in the figure. Since, graph is 2
sinusoidal, hence motion is SHM. ∴ tan2 (ωt + φ ) = 1
6. (b) y − 3 = 4 cosωt π
⇒ tan2 (ωt + φ ) = tan2
Comparing with x = A cosωt 4
π
⇒ Amplitude of SHM, A = 4 units ⇒ ωt + φ =
4
7. (c) Here, x = a sin ωt π
x ⇒ φ= (Qt = 0)
∴ sin ωt = 4
a
Q x = A sin (ωt + φ )
Similarly, y = b cosωt
y Here, A = 10 cm= 0.1 m
∴ cosωt =
b ω = 2π f = 2π × 2 = 4π rad/s
x2 y2 π
∴ sin2 ωt + cos2 ωt = 2 + 2 φ=
a b 4
 π
x2 y2 ∴ x = 0.1sin  4πt + 
∴ + =1 (Ellipse)  4
a2 b2

@iitjeehelps
SIMPLE HARMONIC MOTION 129
12. (c) If x = A sin ωt 20. (b) According to conservation principle
1 1
Then, PE = mA 2ω 2 sin2 ωt mgh = kx 02 − mgx 0
2 2
1  1− cos 2ωt  where, x 0 is maximum elongation in spring (when particle
∴ PE = mA 2ω 2  
2  2  is in its lowest extreme position).
1 2
∴ ω′ = 2ω or kx 0 − mgx 0 − mgh = 0
2
or 2 πf ′ = 2 × 2 πf
2mg 2mg
∴ f ′ = 2f or x 02 − x0 − h =0
k k
13. (a) u = k | x |3
2mg  2mg  2 2mg 
u max = ka 3 ±   + 4× h
k  
1  k k 
The oscillation energy is given by ma 2ω 2. ∴ x0 =
2 2
1 Q Amplitude = elongation in spring for lowest extreme
∴ ma 2ω 2 = ka 2a position − elongation in spring for equilibrium position
2
2k mg  2hk 
or ω2 = a = x 0 − x1 = 1+ 
m k  mg 
 2k  
or ω =   a   k 
21. (b) Here, ω =  
 m    2m 
2π m 1
∴ T= = 2π ∴ E0 = ( 2m ) A 2ω 2
ω 2ka 2
1 mv 2 mv 2
∴ T∝ or = mA 2ω 2 or = mA 2
k
a 4 4 2m
14. (d)  mv 2 
T ∴ A=  
∫0 u dt = 1 T u dt  2k 
T ∫0
15. (a) Average energy = T
∫0 dt 22. (b) The reduced mass of the system is µ =
m1m2
m1 + m2
1 T 1
= ∫ 0 mω A 2 cos2 (ωt + φ )dt = mω 2 A 2
2
2T 4  µ  m1m2 
T = 2π   = 2π  
k  (m1 + m2 ) k 
16. (c)
17. (a) The angular frequency under all g 1
3
∴ f = = 3 Hz
circumstances is T
T 23. (c) In this case, a constant force F acts on the system.
k  200
ω=   =   = 14.14 rad/s So, the centre of mass of system moves with constant
 m  1  acceleration.
When elevator is moving up, the F
m a CM =
equation of motion is m1 + m2
mg 4mg
T − mg = ⇒T = mg In the frame of centre of mass, particles execute SHM
3 3 because force on each particles varies linearly.
This tension elongates the spring by x 24. (b)
T = kx 25. (c) Time period of second's pendulum does not depend
4mg
⇒ x= = 0.07 m on the mass of bob.
3k
26. (d) The time period of clock pendulum at different places
1 1 1 1 2 1 3
18. (b) = + or = + = are different due to variation of g.
k k1 k 2 k k2 k2 k2
27. (c) The bob moves on a circular path
Q k 2 = 3k
k 3 θ
Q k1 = 2 = k φ φ
2 2
m
19. (a) Since,T = 2π Mg sinθ
k Mg Mg cosθ
Here, m and k remain constant. Mv 2

Hence, T remains constant. ∴ T − Mg cosθ =


L

@iitjeehelps
130 SELF STUDY GUIDE BITSAT

28. (b) Let train is moving towards right. The pseudo force is 33. (a)
acting in left direction = ma 0 34. (c) At the vertex, tangential acceleration is zero. When
a0
particle is at mean position, it is at vertex. When it is
θ displaced for small angle θ,

g2
T
−mf = mg sin θ

+
0
a
ma0 or f = − g sin θ = − g θ tan θ
mg  dy 
g or f = −g  
 dx 
Effective acceleration, a eff = a 02 +g 2
dy
Q x 2 = 4ay or 2x = 4a
 l  dx
∴ Time period = 2π   dy x x
 a eff  or = ∴ tan θ =
dx 2a 2a
 l 
T = 2π  
 a2 + g2 
 0 
N
∆t 1 ∆L 1
29. (a) = = α ∆θ
t 2 L 2
1 θ
= × 12 × 10−6 × ( 40° − 20° ) = 12 × 10−5
2 mg cosθ
mg
∆t = t × 12 × 10−5 = 86400 × 12 × 10−5 = 10.3 s/day mg sinθ

l  l  x 
30. (a)T1 = 2π  1  ,T2 = 2π  2  (l1 > l2 ) ∴ f = −g  
g g  2a 
Let shorter pendulum makes n vibrations, then the longer x 
or − ω2 x = − g  
will make one less than n to come in phase again.  2a 
nT2 = (n − 1)T1
g
l  l  ∴ ω=  
2π  2  n = (n − 1) 2π  1   2a 
g g
F F
∴ n = 11 35. (d) Stress = , η= 2
L2

31. (a) The block just loose the contact from the plank if
F = L2ηφ
acceleration of plank is just equal to g. (Vertically
downward)  −x   x
F = L2η   Q Strain = φ =–
Since, block just loses contact, when the plank is  L  L 
momentarily at rest. Therefore, at that instant the plank is F = (Lη)( −x )
at its upper extreme position. ⇒ ma = (Lη)( −x )
But at extreme position acceleration = aω 2
 Lη
where, a = 40 cm = 0.4 m a =   ( −x )
m
Hence, 0.4 ω 2 = g = 10 m/s2
2π ∴ a ∝ − ω 2x
ω = 5 rad/s ⇒ T= s Lη
5 ( −x ) = − ω 2x
m
32. (b) d = R and I = 2mR 2

 Lη m
 I   2mR 2  or ω =   ∴T = 2π  
Q T = 2π  T = 2 π  mgR  m  ηL
mgd   
1  BA 2 
 2R  36. (a) f =  
⇒ T = 2π  2π  MV0 
 g 
O Pivot where, B = Bulk modulus of elasticity
 MV0 
T = 2π  
 BA 2 
CM  M (hA )
= 2π  
 pA 2 
 Mh 
T = 2π  
 pA 

@iitjeehelps
SIMPLE HARMONIC MOTION 131

BITSAT Archives
4π 2 Acceleration of particle is
1. (d) We know maximum acceleration, a max = ω 2A = A
T2 a = ω 2x [numerically]
4π 2 At x = + A, a = a max
= 2
× 0.3 = 30 m/s 2
 π  π
2
  ∴ a max = ω 2A =   × 2
 5  2
5
Maximum force, Fmax = ma max = × 30 = 0.15 N π2 π2
1000 =2× = cm/s2
4 2
2. (a) The beat frequency of heart is
7. (b) This equation represented damped vibration.
75 75
ν= = = 1.25 = 1.25 Hz limmersed
(1min) 60 s 8. (a)T = 2π
g
The time period of heart is
0.80 4π
1 1 ⇒ T = 2π =
T = = = 0.8 s 9.8 7
ν 125 . s− 1
9. (b) In equilibrium,
3. (d) y = 5 (sin 4πt + 3 cos 4πt )
T cos θ = ma + mg sin α
y = 5 (sin 4πt + 5 3 cos 4πt )
and T sin θ = mg cos α
A = A12 + A22 T cosθ
T sinθ
A = (5)2 + (5 3 )2
T
= 25 + 75 = 100 = 10 a θ
4. (b) Acceleration | α | = ω x 2 α
sin
mg
α α
or ω=
x
mg cosα
2
= = 10 rad/s
0.002 a + g sin α
This gives, cot θ =
5. (c) The effective spring constant ks of this arrangement is g cos α
1 1 1 1 2+1 3  1
= + ⇒ = = 5 + 10 ×  
ks k 2k ks 2k 2k  2 2
= =
2k 3 3
ks = 10 ×
3 2
The effective spring constant k p of this arrangement is  2
∴ θ = cot −1  
k p = k1 + k 2 = k + 2k = 3k  3
ks 2k /3 2
∴ = = 10. (d) It is the least interval of time after which the periodic
kp 3k 9 motion of a body repeats itself.
 πt  Therefore, displacement will be zero.
6. (b) y = 2 sin  + φ
2  11. (a) Time period is independent of mass of bob of
Comparing the equation with the standard equation pendulum.

y = A sin(ωt + φ ) 12. (d) The lift is moving with constant velocity, so there will
π be no change in the acceleration. Hence, time period will
So A = 2 cm, ω = remain same.
2

@iitjeehelps
11
Fluid Mechanics

Fluid Statics
The substances which flow are called fluids. Fluids include both liquids and gases. The science of
fluids at rest is called fluid statics.

Pressure in a Fluid
Consider a point A inside the fluid (see figure alongside). Imagine a small area
∆S containing the point A. We define pressure of fluid at point A as
F ∆S
p = Lt 2
∆S → 0 ∆ S
A
2
where, p = pressure of fluid, F = force exerted by fluid
The SI unit of pressure is Nm −2 called pascal which is abbreviated as Pa.
Common unit of pressure is 1 atmosphere (atm), i.e. the pressure exerted by the
atmosphere at sea level.
1 atm = 1013
. × 105 Pa
Other unit of pressure is torricelli’s, 1 torr = 133 Pa
Another important property of fluid is its density, which is defined as the mass of fluid per unit volume
occupied by it, i.e.
Mass of fluid
Density of fluid =
Volume occupied by the fluid
It is denoted by ρ and its SI unit is kg/m3 .
Relative density is defined as the ratio of the density of a substance to the density of water at 4°C, i.e.
Density of substance
Relative density =
Density of water at 4° C
Relative density has no unit and no dimension.

Pressure due to Fluid Column


Hydrostatic pressure at a point A depth h below the fluid surface is given by
p = hρg
where, ρ = density of fluid.

@iitjeehelps
FLUID MECHANICS 133

Gauge Pressure Case III The density of body is lesser than that of liquid
( i.e. ρB < σ ). In this case, w < F , so the body will
The pressure difference between the real hydrostatic
move upwards and in equilibrium will float
pressure and the atmospheric pressure is known as the
partially immersed in the liquid, such that
gauge pressure.
w = V in σg [V in is the volume of body in the liquid]
∴ Gauge pressure
or VρB g = V in σg [asw = mg = ρRV g ]
= real pressure (p) − atmospheric pressure ( p0 )
or VρB = V in σ …(i)

Pascal’s Law Equation of Continuity


This law can be defined as “if the pressure in a liquid Consider a streamline flow Q
P
is changed at a particular point, then this change is
of a fluid through a tube v2
transmitted to the entire liquid without being diminished in
of varying cross-section v1
A2
magnitude”. Assuming liquid to be incompressible and
(see figure alongside).
non-viscous. This law has various interesting applications
Now, according to equation A1 Streamline flow of fluid
in the form of hydraulic machines such as hydraulic lift,
hydraulic brakes etc. of continuity,
A1v1 = A2v2 (or Av = constant)

Archimedes' Principle where, A1 and A2 are the cross-section areas and v1 and v2
are the velocities of the fluids at two section respectively.
Whenever a body is partly or wholly immersed in a fluid at
rest, there is a decrease in its weight and this decrease in
weight is equal to the weight of the fluid displaced by the Energy of a Flowing Fluid
immersed part of the body. In fact, when a body is There are three types of energies in a flowing liquid, which are
immersed in a fluid, it experiences an upthrust due to the defined below
fluid and as a result the apparent weight of the body is
reduced. Pressure Energy
∴ Apparent weight of the body
If p is the pressure at the cross-section area A of a fluid and
= weight of the body – upthrust due to fluid the fluid moves through a distance l against this pressure.
= weight of the body – weight of the fluid displaced Then, the pressure energy stored in fluid = ( pA) × l = pA l
e.g. for a floating body, the volume of a body (V − V s ) Volume of liquid moved = Al
remaining outside the liquid will be given by ∴ Pressure energy (stored in fluid) per unit volume = p
ρ  ρ
V0 = V − V s = V − V = V 1 −  Kinetic Energy
σ  σ
If any fluid element of mass m and volumeV is flowing with
velocity v, then
Laws of Floatation ∴
1 m 1
KE per unit volume =   v 2 = ρv 2
 m
Qρ = 
When a body of density ρB and volume V is immersed in a 2 V  2  V 
liquid of density σ, the forces acting on the body are Here, ρ is the density of the fluid.
(i) The weight of body w = mg = VρB g acting vertically
downwards through the centre of gravity of the Potential Energy
body. If a fluid element of mass m and volumeV is at a height of h
(ii) The upthrust F = Vσg acting vertically upwards above the ground level (reference level), then the potential
through the centre of gravity of the displaced liquid energy stored in fluid is mgh. So, potential energy per unit
i.e. centre of buoyancy. mgh
volume is given by = = ρgh
So, the following three cases are possible. V

Case I The density of body is greater than that of liquid


(i.e. ρB > σ ). In this case, as weight will be more than Bernoulli’s Theorem
upthrust, the body will sink. Bernoulli’s theorem is an outcome of the principle of
conservation of energy, when applied to the flowing fluid. It
Case II The density of body is equal to the density of liquid states that “for a streamline F F
( i.e. ρB = σ ). In this case, w = F , so the body will flow of an ideal fluid θ t θ
float fully submerged in neutral equilibrium (incompressible,
anywhere in the liquid. non-viscous), the sum of

@iitjeehelps
134 SELF STUDY GUIDE BITSAT

pressure energy per unit volume, kinetic energy per unit The time of making empty a hemispherical tank through an
volume and potential energy per unit volume remains orifice at its bottom. Initially, tank is full of water and finally
constant at every cross-section of the flow”. tank is completely made empty.
1
i.e. p + ρgh + ρv 2 = constant R5/ 2
2 T = 14π
15a 2 g

Applications of Bernoulli’s Theorem where, R = the radius of the hemispherical tank


a = cross-sectional area of the orifice
(i) The action of carburetor, paintgun, scent sprayer,
atomiser is based on Bernoulli’s equation.
(ii) Action of Bunsen’s burner, gas burner, oil stove is
also based on it.
Surface Tension
Each molecule (or atom) of liquid
(iii) Motion of spinning ball (Magnus effect) is based on
Bernoulli’s equation. attracts the neighbouring molecule
with the some force (called
(iv) Aerofoil or lift of aircraft wing is also based on it.
cohesive force). However, as every
When aerofoil moves against the wind, the flow of
molecule is gathered by a number
speed of air on top is more as compared to the speed
of molecules in every direction, the
of air below it, which results into an upward force
net effect on the molecule is zero
providing lift to the aircraft.
(v) Torricelli’s law and venturimeter are also based on (figure).
this equation. Now, if we consider a molecule on the surface of the liquid,
as clear from the figure, the molecule would feel a net
Limitations of Bernoulli’s Theorem downward force due to which the free surface behaves like
a stretched membrane under tension.
(i) When a fluid is at rest, i.e. its velocity is zero
everywhere, Bernoulli’s equation becomes If we draw an imaginary line of a length l on the surface of
p1 + ρgh1 = p2 + ρgh2 ⇒ ( p1 − p2 ) = ρg (h2 − h1 ) the liquid, the net value of force acting on this line per unit
length is called surface tension.
(ii) Bernoulli’s equation ideally applies to fluids with
zero viscosity or non-viscous fluids. Force F
Surface tension, S = ⇒ S=
(iii) Bernoulli’s equation applies to fluids which must be Length L
incompressible, as the elastic energy of the fluid is SI unit of surface tension (S or T ) is Nm −1 and its
also not taken into consideration. dimensional formula is[MT −2 ]. It is the characteristic of the
(iv) Bernoulli’s equation does not hold for steady or two media in contact.
turbulent flows, because in that situation velocity
and pressure are constantly fluctuating in time.
Applications of Surface Tension
(Drop, Bubble)
Torricelli’s Law As every system tends to have minimum potential energy,
This law states that “speed of the surface of a liquid too tends to have minimum surface
p2 = p v2
liquid coming out through a area. As, sphere has the minimum surface area for a given
hole at a depth h below the free 2 volume, drops and bubbles tend to have spherical shape.
surface of a tank is same as h A2
1 A1 If any other force such as gravitation, air resistance is
that of a particle fallen freely v1 absent, then the drops and bubbles would be perfect
through height h under p1
spheres.
gravity”. The speed of the
liquid coming out is called the Another interesting consequence of surface tension is that
speed of efflux. pressure inside a spherical drop or bubble is more than the
pressure outside.
In the above figure, p1 , v1 , A1 are the pressure, velocity and
2S
area of orifice at 1 and p2 , v2 , A2 be pressure, velocity and Excess pressure for liquid drop ( ∆p )d =
area at 2 respectively. r
4S
Now, speed of efflux, v1 = 2 gh Excess pressure for soap bubble ( ∆p )b =
r
∴ Volume of liquid coming out of orifice per second where, S is the surface tension of the liquid and r is the
= A1v1 = A1 2 gh radius of drop and bubble.

@iitjeehelps
FLUID MECHANICS 135
(iii) If the capillary tube is of
Surface Energy insufficient length, the
Surface energy of a liquid is the potential energy of the liquid rises upto the upper h
α l
molecules of a surface film of the liquid by virtue of its end of the tube and then the
position. radius of its meniscus
When the surface area of a liquid is increased, work is done changes from R to R′ such
against the cohesive force of molecules and this work is that hR = h ′ R′ , where
stored in the form of additional surface energy. h ′ = insufficient length of
Increase in surface potential energy the tube.
∆U = Work done ( ∆W ) = S∆ A
(iv) After connection due to the weight of liquid
where, ∆ A is the increase in surface area of the liquid. contained in the meniscus, the formula for the height
2S r
is given by h = −
ρrg 3
Angle of Contact
The angle of contact
between a liquid and a
Viscosity
solid is defined as the θ
? Viscosity is the property of
angle enclosed between θ? a fluid in motion, by virtue v + ∆v
∆z
the tangent to the liquid of which a force of v
surface and the solid Water
Mercury resistance exists between
surface which is in the different layers of the
contact with the liquid as fluid whenever there is relative motion between them.
shown in the figure given (A) Acute angle (B) Obtuse angle
When a fluid is flowing in a pipe or a tube, then the velocity
below. of the liquid layer along the axis of the tube is maximum
The angle of contact varies between 0° to 180°. and decreases gradually as we move towards the walls,
where it becomes zero.
For the liquids which wet the surface of solid (such as water
and glass), the angle of contact is acute, while those for The relative speed of the liquid layer (closest to boundary)
liquids which do not wet the surface of solid (such as is assumed to be zero.
mercury and glass), the angle of contact is obtuse. Hence, due to viscosity, liquid flows in layers. This type of
Angle of contact decreases with the addition of impurities flow is called laminar flow.
whereas it increases if temperature is increased.
Poiseuille's Formula
In case of steady flow of liquid of viscosity η in a capillary
Capillary Rise or Capillarity tube of length L and radius R under a pressure difference p
Capillarity is the phenomenon of rise or fall of a liquid in a across it, the volume of liquid flowing per second is given
capillary tube as compared to that in a surrounding liquid. by
The height h upto which a liquid will rise in a capillary tube dQ πpR4
=
2 S cosθ 2S dt 8 ηL
is given by h = =
rρ g r ρg This is called poiseuille’s formula.
where, r = radius of the capillary tube and
Stokes’ Law
r
R= = radius of liquid meniscus. Stokes proved that for a small spherical body of radius r
cosθ
moving with a constant speed v called terminal velocity
1
(i) The rise in capillary tube h ∝ ⋅ through a fluid having coefficient of viscosity η, the viscous
r force F is given by F = 6πηrv. It is known as the Stokes’ law.
(ii) If a capillary tube, dipped in a liquid is tilted at an
angle α from the vertical, the vertical height h of the Terminal Velocity
liquid column remains the same. However, the
If a small spherical body is dropped in a fluid, then initially
length of the liquid column (l ) in the capillary tube
increases to it is accelerated under the action of gravity. However, with
h an increase in speed, the viscous force increases and soon it
l= balances the weight of the body. Now, the body moves with
cosα

@iitjeehelps
136 SELF STUDY GUIDE BITSAT

a constant velocity, called the terminal velocity. Terminal


velocity vt is given by Reynold’s Number
2 r 2 (ρ − σ ) g Reynold defined a dimensionless number called Reynold’s
vt =
9 η number (Re ), whose value gives an approximate idea
whether the flow would be laminar or turbulent.
where, r = radius of the falling body, Flow is streamline, if Re < 2000
ρ = density of the falling body and
Flow is turbulent, if Re > 3000
σ = density of the fluid.
Between 2000 to 3000, the flow is transient.

Practice Exercise
1. The pressure at the bottom of a tank of liquid is not 6. If the weight of a body in vacuum is w and w1 and w 2
proportional to are weights when it is immersed in a liquid of specific
a. the density of the liquid gravity ρ1 and ρ2 respectively, then find the relation
b. the area of the liquid surface among w, w1and w 2.
w 1 ρ 2 + w 2 ρ1 w 1 ρ 2 − w 2 ρ1
c. the height of the liquid a. w = b. w =
w1 + w 2 ρ 2 − ρ1
d. the acceleration
w 1 ρ1 + w 2 ρ 2 w 1 ρ 2 + w 2 ρ1
2. If a vessel containing a fluid of density ρ upto height h c. w = d. w =
ρ1 + ρ 2 ρ1 + ρ 2
is accelerated vertically downwards with acceleration
a 0. Then, the pressure by fluid at the bottom of 7. A closed rectangular tank 10 m long, 5 m wide and
vessel is 3 m deep is completely filled with an oil of specific
a. p = p0 + ρgh + ρha 0 gravity 0.92. Find the pressure difference between the
rear and front corners of the tank, if its moving with an
b. p = p0 + ρgh
acceleration of 3 m / s 2 in the horizontal direction
c. p = p0 + ρh(g − a 0 )
a. 27. 6 kPa b. 50 kPa
d. p = p0 − ρgh c. 60 kPa d. 70 kPa
3. In each heart beat, a heart pumps 80 ml of blood at an
average pressure of 100 mm of Hg. What will be the 8. A U-tube having a liquid
power output of the heart? (Assume 60 heart beat per of density ρ is
accelerated at a m/s 2, so h1 – h2
minute)
as to create be the height =l / 2
a. 1 W b. 2.75 W h
c. 1.06 W d. 0.5 W difference between two 1
columns of l/2 (as shown h2
4. One end of a U-tube of uniform bore in figure). If l is the length
(area A) containing mercury is x of the base of U-tube, l
connected to a sunction pump. Because the value of acceleration
of it the level of liquid of density ρ falls in given to the system is
one limb. When the pump is removed, a. 4.9 m/s 2 b. 9.8 m/s 2
the restoring force in the other limb is c. 5.6 m/s 2 d. 6.4 m/s 2

a. 2xρAg b. xρg 9. A body weighs 5 N in air and 2 N when immersed in a


liquid. The buoyant force is
c. Aρg d. xρAg
a. 2 N b. 3 N
5. A cylindrical vessel of radius r containing a liquid is c. 5 N d. 7 N
rotating about a vertical axis through the centre of 10. A neckless weighing 50 g in air, but it weight 46 g in
circular base. If the vessel is rotating with angular water. Assume copper is mixed with gold to prepare
velocity ω, then what is the difference of the heights of the neckless. Find how much copper is present in it.
liquid at centre of vessel and edge? (Specific gravity of gold is 20 and that of copper is 10)
rω r 2ω 2 a. m = 25 g
a. b.
2g 2g b. m = 30 g
ω2 c. m = 35 g
c. 2g rω d.
2gr 2 d. m = 20 g

@iitjeehelps
FLUID MECHANICS 137
11. If air of weight w is filled in a empty balloon which 20. Water from a tap emerges vertically downward with an
weights w1 the weight of balloon will become w 2 initial speed of 1m/s. The cross-sectional area of the
Suppose the density of air inside and out side the tap is 10−4 m 2. Assume that the pressure is constant
balloon is same, then throughout the stream of water and that the flow is
a. w 2 = w 1 + w b. w 2 = w 1w steady. The cross-sectional area of the steam 0.15 m
below the tap is
c. w 2 = w 1 d. w 2 = w 1 − w
a. 5 × 10−4 m2
12. A soft plastic bag of weight w 0 is filled with air at STP b. 1 × 10−5 m2
Now, weight of the bag is w in air. Then, c. 5.83 × 10−5 m2
a. w > w 0 b. w = w 0 c.w >_ w 0 d. w < w 0 d. 2 × 10−5 m2

13. A block of ice of area A and thickness 0.5 m is floating 21. A tube of flow is shown in the figure.
in the fresh water. In order to just support a man of B
100 kg, the area A should be (specific gravity of ice
0.917 and density of water = 1000 kg/m 3 )
. m2
a. 124 b. 4.21m2 c. 2.41m2 d. 7.23 m2

14. A piece of ice is floating in water. Find the fraction of


volume of the piece of ice outside the water A
(Given, density of ice = 900 kg /m 3 and density of
water = 1000 kg / m 3 ) a. The fluid particles must be accelerated from A to B
a. 0.21 b. 0.01 c. 0.1 d. 0.9 b. Fluid particles may accelerate from A to B
15. A block of wood floats with 1/4 of its volume under c. The fluid particles must be decelerated from A to B
d. The fluid particles may be decelerated from B to A
water. What is the density of the wood? (Density of
water = 1000 kg/m 3 ) 22. A pipe GB is fitted with C
A
a. 750 kg/m3 b. 250 kg/m3 c. 300 kg/m3 d. 260 kg/m3 two pipes C and D as
2
shown in the figure. The
16. A block weighs 15 N and 12 N in air and water pipe has area A = 24 m 2 B
respectively. When it is immersed in another liquid, it at G and velocity of water G
weights 13 N, then find the relative density of the at G is 10 m/s, and at C is
block. 6 m/s. The velocity of A
a. 5 b. 6 c. 10 d. 2 3
water at D is D
17. In English, the phrase ‘tip of the iceberg’ is used to a. 21 m/s
mean a small visible fraction of something that is
b. 3.3 m/s
mostly hidden. For a real iceberg, what is this fraction,
c. 30 m/s
if the density of sea water is 1.03 g/cc and that of ice is
d. None of these
0.92 g/cc?
a. 0.106 b. 10.6 c. 0.901 d. 0.801 23. Bernoulli’s equation is applicable to points
a. in a steadily flowing liquid
18. A vessel contains oil (density 0.8 g/cc) over mercury
b. in a stream line
(density 13.6 g/cc). A homogeneous sphere floats with
c. in a straight line perpendicular to stream line
half its volume immersed in mercury and the other half
d. in any non- viscous liquid
in oil. The density of the material of the sphere in g/cc is
a. 3.3 b. 6.4 c. 7.2 d. 12.8 24. The horizontal flow of fluid depends upon
a. pressure difference
19. In a steady incompressible flow of a liquid
b. amount of fluid
a. the speed does not change, if the area of
c. density of fluid
cross-section changes
d. All of the above
b. the speed increases, if the area of cross- section
increases 25. In steady horizontal flow,
c. the speed decreases, if the area of cross- section a. the pressure is greatest where the speed is least
increases b. the pressure is independent of speed
d. bubbles are produced when the area of the c. the pressure is least where the speed is least
cross-section increases d. Both a and c are correct

@iitjeehelps
138 SELF STUDY GUIDE BITSAT

26. From a horizontal tube with area of cross-section A1 Q is h height above the ground, then the height of hole
and A2 as shown in figure, liquid is flowing with P above the ground will be
velocities v 1 and v 2 respectively. The difference in the H
a. 2h b.
level of the liquid in the two vertical tubes is h. h
H
c. H − h d.
2
h 32. A liquid having area of free surface A and has an
orifice at a depth of h with an area a, below the liquid
v1 surface, then find the velocity v of flow through the
A1 v2 orifice.
A2
A2
a. v = 2gh b. v = 2gh
A − a2
2

a. The volume of the liquid flowing through the tube in A A2 − a 2


time is A1 v1 c. v = 2gh d. v = 2gh
A −a A2
b. v 2 − v1 = 2gh
33. A capillary tube of area of cross- section A is dipped in
c. v 22 − v12 = 2gh
water vertically. Calculate the amount of heat evolved
d. The energy per unit mass of the liquid is the same in as the water rises in the capillary tube upto height h.
both sections of the tube The density of water is ρ
27. A vessel is filled with water and kerosene oil. The Aρgh 2
a. b. Agh 2ρ
vessel has a small hole in the bottom. Neglecting 2
viscosity if the thickness of water layer is h1 and c. 2Agh 2ρ d. None of these
kerosene layer is h2, then the velocity v of flow of
water will be (density of water is ρ1 g/cc and that of 34. The radius of the biggest metal coin of thickness t and
kerosene is ρ2 g/cc) density ρ, which would be able to float on water
surface of surface tension S, is
  ρ  S S
a. v = 2g (h1 + h2 ) b. v = 2g h1 + h2 2   a. b.
  ρ1   2 ρgt ρgt
2S 4S
 ρ  c. d.
c. v = 2g (h1ρ1 + h2 ρ 2 ) d. v = 2g h1 1 + h2 ρgt 3 ρgt
 ρ2 
35. An open glass tube is immersed in mercury in such a
28. Mark the correct option(s). way that a length of 8 cm extends above the mercury
a. two stream lines may cross each other level. The open end of the tube is then closed and
b. two stream lines must cross each other sealed and the tube is raised vertically up by
c. two stream lines never cross each other additional 46 cm. What will be length of the air column
d. None of the above above mercury in the tube now?
29. Water flows along a horizontal pipe whose (Atmospheric pressure = 76 cm of Hg)
cross-section is not constant. The pressure is 1 cm of a. 38 cm b. 6 cm
Hg, where the velocity is 35 cm/s. At a point where the c. 16 cm d. 22 cm
velocity is 65 cm/s, then pressure will be 36. While measuring surface tension of water using
a. 0.89 cm of Hg b. 8.9 cm of Hg capillary rise method, height of the lower meniscus
c. 0.5 cm of Hg d. 1 cm of Hg from free surface of water is 3 cm, while inner radius
30. A pilot tube was inserted in a pipe to measure the of capillary tube is found to be 0.5 cm. Then, compute
velocity of water in it. If the water rises in the tube is tension of water using this data. [Take, contact angle
200 mm. Find the velocity of water. between glass and water as 0 and g = 9.81 m/s 2]
a. 9.8 m/s b. 1.98 m/s c. 19.6 m/s d. 196 m/s a. 0.72 N/m
31. A cylindrical vessel filled b. 0.77 N/m
with water to a height H. c. 1.67 N/m
A vessel has two small P d. None of the above
holes in the side, from 37. To what depth must a rubber ball be taken in deep sea
which water is rushing H Q so that its volume is decreased by 0.1%. (The bulk
out horizontally and the h modulus of rubber is 9.8 × 108 N/m 2, and the density
two streams strike the
ground at the same of sea water is 103 kg/m 3 )
point. If the lower hole Q a. 100 m b. 60 m c. 75 m d. 65 m

@iitjeehelps
FLUID MECHANICS 139
38. A wooden block of mass m and density ρ is tied to a 46. A drop of radius r is broken into n equal drops.
string, the other end of the string is fixed to the bottom Calculate the work done if surface tension of water isT.
of a tank. The tank is filled with a liquid of density σ a. 4rπR 2nT
with σ > ρ. The tension in the string will be b. 4πR 2T (n 2/ 3 − 1)
 σ − ρ  σ − ρ c. 4πR 2T (n1/ 3 − 1)
a.   mg b.   mg
 σ   ρ  d. None of the above
ρ mg σ mg
c. d. 47. What will happen if n drops of a liquid each has
σ ρ
surface energy E , combine to form a single drop.
39. Assume that a drop of liquid evaporates by decrease a. No energy will be released in the process
in its surface energy, so that its temperature remains b. Some energy will be absorbed in the process
unchanged. What should be the minimum radius of c. Energy released or absorbed will be E (n − n 2/ 3 )
the drop for this to be possible? The surface tension is d. Energy released or absorbed will be nE ( 22/ 3 − 1)
T, density of liquid is ρ and L is its latent heat of
vaporisation. 48. If a bigger drop of liquid at temperature t, breaks up
a. ρL /T b. T /ρL c. T /ρL d. 2T /ρL into number of small droplets, then what is
temperature of the droplets? (Assume, bigger drop is
40. Water is flowing continuously from a tap having an isolated from its surroundings)
internal diameter 8 × 10−3 m. The water velocity as it a. Equal to t
leaves the tap is 0.4 ms −1. The diameter of the water b. Greater than t
stream at a distance 2 × 10−1 m below the tap is close to c. Less than t
a. 7.5 × 10−3 m b. 9.6 × 10−3 m d. Either (a), (b) and (c) depending on surface tension of
c. 3.6 × 10−3 m d. 5.0 × 10−3 m liquid
49. The excess pressure inside a soap bubble of radius
41. In an experiment a capillary tube is kept vertical, then
water rises up in the tube upto 3 mm height. When the 4 cm is 30 dyne/cm 2. The surface tension is
tube is tilted at an angle of 60º? with vertical, what a. 30 dyne/cm b. 20 dyne/cm
should be the height of water rise c. 40 dyne/cm d. 80 dyne/cm
a. 6 mm b. 4 mm 50. Calculate the work done against surface tension in
c. 3 mm d. None of these formation of a drop of mercury of radius 4 cm (surface
tension for mercury = 465 dyne/cm)
42. Calculate for the rise of water in a capillary tube when
a. 9.34 × 10−3 J b. 10 × 10−2 J
kept vertical in water whose radii is 1/4th of that
capillary tube which when kept vertical water rise in it c. 4 × 10−3 J d. 466 J
upto a height of 3 mm. 51. Calculate the energy required to increase the radius of
a. 12 mm b. 10 mm c. 4 mm d. 3 mm a soap bubble from 1 cm to 2 cm (The surface tension
43. Calculate the heat evolved for the rise of water when is 30 dyne/cm).
one end of the capillary tube of radius r is immersed a. 240 π erg b. 720 π erg
vertically into water. Assume surface tension = T and c. 480 π erg d. None of these
density of water to be ρ.
2πT πT 2 52. A film of a liquid is held on a circular ring of radius r , if
a. b. the surface tension of the liquid is T. The surface
ρg ρg
tension of the liquid is
2 πT 2
c. d. None of these a. πr 2T b. 2 πr2T
ρg c. 4πr 2T d. None of these
44. In a liquid there is air bubble of radius 1 mm at a depth 53. A small uniform tube is bent into a
10 cm below the free space. The surface tension of circle of radius r whose plane is
liquid 0.075 N/m and density is 1000 kg/m 3 . By what vertical. The equal volumes of
σgV
amount is the pressure inside the bubble greater than two fluids whose densities are ρ
θ
the atmospheric pressure? and σ (ρ > σ ), fill half the circle.
a. 1130 pascal b. 1200 pascal Find the angle that the radius
c. 1100 pascal d. 1000 pascal passing through the interface ρgV
45. Calculate the work done by a boy in making a soap makes with the vertical.
bubble of diameter 1.4 cm by blowing, if the surface ρ−σ ρ−σ
a. cot θ = b. tan θ =
tension of soap solution is 0.03 N/m. ρ+σ ρ+σ
a. 3 × 10−5 J b. 3.696 × 10−5 J ρ+σ ρ
c. sin θ = d. sin θ =
c. 2 × 10−5 J d. 4.2 × 10−5 J ρ−σ σ

@iitjeehelps
140 SELF STUDY GUIDE BITSAT

54. A liquid drop of radius R breaks into N smaller droplets 59. Water is flowing in a river. If the velocity of a layer at a
of radii r . If liquid has density ρ, specific heat s and distance 10 cm from the bottom is 20 cm/s. Find the
surface tension T, then the drop in temperature is velocity of layer at a height of 40 cm from the bottom.
given by a. 10 m/s b. 20 cm /s c. 30 cm/s d. 80 cm/s
NT  1 1 NT  R 
a.  −  b.  − 1 60. A horizontal plate(10 cm × 10 cm) moves on a layer
ρs  R r  ρs  r  of oil of thickness 4 mm with a constant speed of
3 T  1 1 3T  1 1 10 cm/s. The coefficient of viscosity of oil is
c.  −  d.  − 
4 ρs  R r  ρs  R r  4 poise. The tangential force applied on the plate
55. A body of density ρ is dropped from height h into a to maintain the constant speed of the plate is
liquid having density σ (σ > ρ) . If the body just a. 103 dyne b. 104 dyne
touches the base of the container, then the distance of c. 105 dyne d. None of these
fallen would be proportional to (Neglect viscous 61. A liquid is flowing through a narrow tube. The
forces) coefficient of viscosity of liquid is 0.1308 poise. The
h h length and inner radius of tube are 50 cm and 1 mm
a. b.
σ −ρ σ+ρ respectively. The rate of flow of liquid is 360 cm 3 /min.
hρ Find the pressure difference between ends of tube.
c. h × ( σ − ρ ) d.
σ −ρ a. 106dyne/cm2
b. 104dyne/cm2
56. A block of mass m and density ρ is hanging from a
c. 10 dyne/cm2
string. If it is lowered into a vessel of cross-sectional
d. None of the above
area A containing a liquid of density σ (< ρ) and gets
fully immersed, the increase in pressure at the bottom 62. Find the terminal velocity of solid sphere of radius
of vessel would be 0.1 m moving in air in vertically downward direction.
m ρg m σg . × 10−5 Ns/m 2, density of sphere = 1000 kg/m 3
( η = 18
a. b.
σA ρA and g = 10 m/ s 2)
mg a. 2 m/s b. 1.2 cm/s
c. d. zero
A c. 4 cm/s d. None of these
57. A solid sphere falls with a terminal velocity of 32 m/s in 63. Eight equal drops of water each of radius r = 2 mm
air. If it is allowed to fall in vacuum, then are falling through air with a terminal velocity of
a. the terminal velocity will be 32 m/s 16 cm/s. The eight drops combine to from a big
b. the terminal velocity will be less than 32 m/s drop. Calculate the terminal velocity of big drop.
c. the terminal velocity will be greater than 32 m/s a. 16 cm/s b. 32 cm/s
d. there will be no terminal velocity c. 64 cm/s d. None of these
58. Find the common radius of curvature r when two soap 64. At 20º C, to attain the terminal velocity how fast will an
bubbles with radii r1 and r 2(r1 > r 2 ) come in contact aluminium sphere of radii 1 mm fall through water.
r +r r r Assume flow to be laminar flow and specific gravity
a. r = 1 2 b. r = 1 2
2 r1 − r2 (Al) = 2.7, ηwater = 8 × 10−4 Pa.
r1 r2 a. 5 m/s b. 4.6 m/s
c. r = d. r = r1 r2
r1 + r2 c. 4 m/s d. 2 m/s

BITSAT Archives
1. The work done in blowing a soap bubble of surface 3. The relative humidity on a day when partial pressure
tension 0.06 Nm −1 from 2 cm radius to 5 cm radius is of water vapour is 0.012 × 106 Pa at 12°C is
a. 0.004168 J b. 0.003168 J [2014] (Take, vapour pressure of water at this temperature
c. 0.003158 J d. 0.004568 J as 0.016 × 105 Pa) [2013]
2. A ball whose density is 0.4 × 103 kg/m 3 falls into water a. 70% b. 40% c. 75% d. 25%
from a height of 9 cm. To what depth does the ball 4. In the absence of intermolecular forces of attraction,
sink? [2013] the observed pressure p will be [2013]
a. 9 cm a. remain same
b. 6 cm b. decrease
c. 4.5 cm c. increase
d. 2.25 cm d. zero

@iitjeehelps
FLUID MECHANICS 141
5. 10000 small balls, each weighting 1g, strike one 10. Water is flowing through a tube of non-uniform
square centimetre of area per second with a velocity cross-section. If the radii of the tube at the entrance
100 m/s in a normal directions and rebound with the and the exit are in the ratio 3 : 2, then the ratio of the
same velocity. The value of pressure on the surface velocities of flow of water at the entrance and the exit is
will be [2013] [2010]
a. 2 × 1010 N/m2 b. 2 × 105 N/m2 a. 9 : 4 b. 4 : 9
c. 107 N/m2 d. 2 × 107 N/m2 c. 8 : 27 d. 27 : 8

6. At a given place where acceleration due to gravity is 11. A liquid X of density 3.36 g/cm 3 is poured in a U-tube,
2
g m/ s , a sphere of lead of density d kg/m is gently 3 which contains Hg. Another liquid Y is poured in left
arm with height 8 cm, upper levels of X and Y are
released in a column of liquid of density ρ kg /m 3 . If
same. What is density of Y ? [2010]
d > ρ, the sphere will [2013]
a. 0.8 g/cc b. 1.2 g/cc
a. fall vertically with an acceleration g m/s2 c. 1.4 g/cc d. 1.6 g/cc
b. fall vertically with no acceleration
 d − ρ 12. The surface tension of soap solution is 0.03 N/m. The
c. fall vertically with an acceleration g   work done in blowing to form a soap bubble of surface
 d 
area 40 cm 2, (in J), is [2008]
 ρ
d. fall vertically with an acceleration g   . × 10−4
a. 12 b. 2.4 × 10−4
d 
c. 12 × 10−4 d. 24 × 10−4
7. Motion of a fluid in a tube is best described by [2011]
a. Bernoulli’s theorem b. Poiseuille’s principle 13. Two rain drops reach the earth with different terminal
c. Archimedes’ principle d. Stokes’ law velocities having ratio 9 : 4. Then, the ratio of their
volumes is [2008]
8. The velocity of efflux of a liquid through an orific in the
a. 3 : 2 b. 4 : 9
bottom of the tank does not depend upon [2011]
c. 9 : 4 d. 27 : 8
a. size of orifice
b. height of liquid 14. Water falls from a tap down the streamline, [2006]
c. acceleration due to gravity a. area decreases
d. density of liquid b. area increases
c. velocity remains same
9. Density of sea water is 1.03 gcc −1. A ship passes from
d. area remains same
fresh water into sea water. It will [2010]
a. rise
15. If a liquid does not wet glass, its angle of contact is
[2005]
b. sink
c. remain at the same depth a. zero b. acute
d. rise or sink depending on its shape and size c. obtuse d. right angle

Answer with Solutions


Practice Exercise 3. (c) Work done by the heart = pdV
dV = ( 60) × ( 80 × 10−6 m3 )
1. (b)
. × 105 Pa
101
2. (c) Here, effected gravitational acceleration is p = (100 mm of Hg) ×
760 mm of Hg
= 1.33 × 104 Pa
Work done
Power =
Time
h 1.33 × 104 Pa × 60 × 80 × 10−6 m3
=
60 s
Power = 106
. W
4. (a)Q The force due to excess pressure = restoring force
a0
or ρghA = restoring force
g′ = g − a 0 or ρg( 2x )A = restoring force
∴ p = p 0 + ρg′ h = p 0 + ρ(g − a 0 )h ∴ Restoring force = 2 ρg xA

@iitjeehelps
142 SELF STUDY GUIDE BITSAT

5. (b) From Bernoulli’s theorem, ρ = 1000 kg/m3 ρVg


1 1 h = 0.5 m
p A + dv A2 + dghA = pB + dvB2 + dghB man
2 2 ∴ 100g + 917Vg = ρVg = 1000Vg
Here, hA = hB ω
100g
1 2 1 P
h ∴ V = Ice
p A + dv A = pB + dvB2 1000g − 917g
2 2 A r C B 100
1 ∴ Ah =
p A − pB = d (vB2 − v A2 ) 83
2 100
∴ A= mg
Now, v A = 0,vB = rω 83 × 0.5
p A − pB = hdg
1 = 2.41m2
hdg = dr 2ω 2
2 14. (c)
r 2ω 2 15. (b) Let the volume of wood ρVg
or h=
2g piece isV0.
The dipped portion of the piece
6. (b) Loss in weight of liquid of density ρ1 = (w − w 1) kg, ifV
is
is the volume of the body, then V
w − w1 V = 0
Vρ1 = w − w 1 ⇒ V = 4
ρ1
∴For equilibrium of wood piece,
w −w2
Similarly, Vρ 2 = w − w 2 ⇒ V = mg = ρVg
ρ2 ρV
or σV0g = 0 g
w − w1 w − w 2 4
= mg
ρ1 ρ2 ρ density of water
∴ σ= =
w 1 ρ 2 − w 2 ρ1 4 4
w =
ρ 2 − ρ1 =
1000
= 250 kg/m3
4
7. (a) We consider a fluid
body of cross-sectional 16. (a)
area A (shown in p2 A 17. (a) For equilibrium,
figure) a Q weight = force of buoyancy
p 1A
p1A − p 2A = ma or mg = Vinσg
or ( p1 − p 2 )A = ρAla ρV
ρV = Vinσ ∴ Vin =
or p1 − p 2 = ρla σ
= 0.92 × 10 × 3 l  ρ
∴ Vout = V − Vin = V 1 − 
= 27.6 kPa  σ
8. (a) Pressure difference between two columns, V ρ  0.92
Fraction = out = 1 − = 1 −  = 0.106
l V σ  1.03 
∆p = pg(h1 − h2 ) = pg
2 18. (c) For equilibrium, the total upward pull will be equal to
∴ Force on the liquid contained in the horizontal portion of the downward pull. IfV is the volume of the sphere, we
the tube = ( ∆p ) × area. Now, this force must be equal to have
the product of mass of liquid (in base tube) and V  V 
  (13.6)g +   ( 0.8)g = Vρg
acceleration a of the system  2  2
l
⇒ pg × area = (volume × density) × a  13.6 + 0.8 −3 −3
2 ∴ ρ=  g cm = 7.2 g cm
 2 
l
⇒ pg × area = area × l × p × a 19. (c) 20. (c) 21. (c) 22. (a)
2
⇒ a = g /2 = 4.9 m/s2 23. (d) 24. (a) 25. (a)
p v2 p v2
9. (b) 10. (b) 11. (c) 12. (b) 26. (c) 1 + 1 = 2 + 2
ρ 2 ρ 2
13. (c) For equilibrium,
ρ 2
(m1 + m2 )g = ρVg or p1 − p 2 = (v 2 − v12 )
2
Here, m1 = mass of man = 100 kg
ρ
m2 = mass of ice But p1 − p 2 = ρgh = (v 22 − v12 )
2
= 0.917 × 1000 V
or v 22 − v12 = 2gh
= 917 V

@iitjeehelps
FLUID MECHANICS 143
27. (b) Net pressure = h1 ρ1 g + h2 ρ 2 g 1  av 
2
1 2
  + gh = v

2 A  2
According to Bernoulli’s theorem, this pressure energy
will be converted into KE, while flowing through the hole is  A2 − a 2
v = 2gh  
1  A2 
h1 ρ1 g + h2ρ 2g = ρv 2
2
33. (a) Heat evolved = Potential energy of water
 ρ  mgh ρAh gh ρAh 2g
v = 2g h1 + h2 2  = = =
 ρ1  2 2 2
28. (c) If two stream lines cross each other, then at a given 34. (c) For the biggest coin, the F F
point, there are two directions of motion of fluid particles. force due to surface tension
This is physically not possible. shall be vertical (i.e. θ = 0) θ t θ
Hence, (c) is correct, ∴ Upward force exerted on
the coin at the verge of its
29. (a) In horizontal pipe,
sinking
1 1 F = (S ) × ( 2πr ) (r = radius of coin)
p1 + ρv12 = p 2 + ρv 22 …(i)
2 2 Now, F = weight of coin
Here, p1 = ρmgh1 = 13600 × 9.8 × 10−2 2S
∴ S × 2 πr = πt 2 τ × ρg ⇒ r =
p 2 = 13600 × 9.8 × h ρgt

ρ = 1000 kg/m3 35. (c) Assuming, temperature remains same.


Hence, we can write p1V1 = p 2V2
v1 = 35 × 10−2 m/s
⇒ l = p 2l2 ⇒ ( 76)( 8) = (54 − x )( 76 − x )
p11
v 2 = 65 × 10−2 m/s ⇒ x = 38 cm
∴ From Eq. (i), we get air
−2 1
13600 × 9.8 × 10 + × 1000 × ( 0.35)2
2 46+8=54
1 x
= 13600 × 9.8 × h + × 1000 × ( 0.65)2
2 8 air 8
air
After solving, 0.89 cm of Hg.
1
30. (b) ρv 2 = ρgh
2

Length of air column = 54 − 38 = 16 cm


 r
r  h +  ρg
h  3
36. (b) As,T =
2 cos θ
 0.5 
0.5 × 10−2 3 + × 10−2 × 103 × 9.81
 3 
= = 0.77 N/m
2
37. (a)
38. (b) From free body diagram of the wooden block,
Vσg = mg + T [V is the volume of block]
v = 2gh = 2 × 9.8 × 200 × 10−3 = 1.98 m/s
31. (c)
32. (d) Applying Bernoulli’s theorem, T
p 1 p 1
+ (v ′ )2 + gh = + v 2 + 0 Vσg
ρ 2 ρ 2 σ mg
where,v′ is velocity of all surfaces of liquid andv is
velocity of efflux.
From equation of continuity T = Vσg − mg
Av ′ = av ⇒ v ′ =
av m  σ − ρ
T = σg − mg = mg  
A ρ  ρ 

@iitjeehelps
144 SELF STUDY GUIDE BITSAT

39. (d) When radius is decreased by dr. 44. (a) Let atmospheric pressure is p 0.
Decrease in surface energy = Heat required for Pressure of liquid just outside the bubble is
vaporisation ( 4πrdr ) × T × 2 = 4πr 2 drρ
p = p 0 + hdg
2T
⇒ r = Pressure of liquid inside the bubble
ρL
2T 2T
1 p′ = p + = p 0 + hdg +
40. (c) From Bernoulli’s theorem, ρgh = ρ (v 22 − v12 ) r r
2 1 2 × 0.075
v  2  1  A  2  p′ − p 0 = × 1000 × 9.8 +
1 10 1 × 10−3
⇒ gh = v12  2  − 1 = v12  1  − 1
2  v1   2  A2   p′ − p 0 = 1130 Pa
45. (b) Initial area of soap bubble = 0
1 1 = A2v 2 )
(Q Av
2 Soap bubble has two surfaces internal and external.
 A1  2hg
⇒   = 1+ 2 Increase in surface area = ∆A = 2 × 4πr 2
 A2  v1
22
2 =2×4× × ( 0.7 × 10−2 )2
 D12  2gh 7
⇒  2 = 1+ 2
 D2  v1 = 12.32 × 10−4 m2
D1 8 × 10−3 Work done = W − S∆A
⇒ D2 = = = 3.6 × 10−3 m

1/ 4 1/ 4 = 0.03 × 12.32 × 10−4
2gh   2 × 10 × 0.2
1 + 2  1 +  = 3.696 × 10−5 J
 v1   ( 0.4)2 
46. (c) The volume of n smaller drop = volume of bigger drop
41. (a) Since, the atmospheric pressure and temperature are
4 4
same. ∴ n πr 3 = πR 3
3 3
Q R = n1/ 3r
h1 60° h2
R
∴ r =
n1/ 3
Here, R = radius of bigger drop
r = radius of smaller drop
h1 3 Q W = 4π(nr 2 − R 2 )T
∴ h2 = = = 6 mm
cos 60° 1/ 2   R 2  nR 2 
= 4π n  1/ 3  − R 2  T = 4π  2/ 3 − R 2 T
2S cos θ n 
42. (a) h =    n 
rgρ
= 4πR 2(n1 − 2/ 3 − 1)T = 4πR 2T (n1/ 3 − L)
2S cos θ
⇒ hr =
ρg 47. (c) Let radius of small drop = r
For both experiment S, θ, ρ, g are same Radius of single drop = R
2S cos θ Surface tension S = surface energy per unit area
hr = = constant
ρg 4 4
n × × πr 3 = πR 3
h1r1 = h2r2 3 3
 r  R = n1/ 3r
h2 =   3 = 12 mm
 r / 4 Initial surface energy = Ei = n × 4πr 2 × S = nE
2T Final surface energy = Ef = 4πR 2S
43. (c) Water rise to height = h =
ρgr
= 4πr 2n 2/ 3S = n 2/ 3E
mgh 2πT 2
Potential energy of water column U = = Energy released = Ei − Ef = E(n − n 2/ 3 )
2 ρg
The work performed by force of surface tension is
4 πT 2 48. (c) 49. (a) 50. (a) 51. (b)
W = 2πrTh = 52. (b)
ρg
53. (b) Taking torque about the point O.
From conservation of energy, the heat evolved
∴ ρVr sin ( 45° − θ )g = σVr sin ( 45° + θ )g
2 πT 2 ρ−σ
Q =W −U = tan θ =
ρg ρ+σ

@iitjeehelps
FLUID MECHANICS 145
54. (d) Volume will remain same, hence we can write By Newton’s third law of motion, force exerted by block on
R3 mσg
4 4
πR 3 = N × πr 3 ⇒ N = 3 liquid surface =
3 3 r ρA
mσg
Increase in surface energy due to breaking of drop into N ∴ Increase in pressure =
droplets, ∆U = 4πT (R 2 − Nr 2 ) ρA
As this energy is provided at the cost of lowering of 57. (d)
temperature by ∆θ 58. (b) Let p 0 = atmosphere pressure r 2
ms ∆θ = ∆U p1 and p 2 are pressure difference across
∆U 4πT (R 2 − Nr 2 ) the common surface. r1
∆θ = = Let r = radius of curvature of the common
ms 4 3 
 πR ρ s surface.
3  4S
p 2 − p1 =
3T  1 r 2  3T  1 1  R3 r
⇒ ∆θ =  − N =  −  Q N = 3 
ρs  R R 3  ρs  R r   r  4S 4S 4S
= − (S = surface tension)
r r2 r1
55. (a) Buoyant force, F ∝ ( σ − ρ )
1 1 1 rr
∴ Deceleration ∝ ( σ − ρ) ⇒ a = − k ( σ − ρ ) = − ⇒r = 12
r r2 r1 r1 − r2
Now, as initial velocity, u = 2gh dv
59. (d)QF = − ηA
and final velocity,v = 0 dx
Distance fallen by the body, dv ∆v
∴ F ∝ or F ∝
2gh  u2  dx ∆x
s = Qs = 2a  v v
2 × k ( σ − ρ)   F ∝ or x ∝
x F
h
⇒ s ∝ But F is constant,
σ −ρ
∴ x ∝v
m v2 x2 v 40
56. (b) Volume of block = = or 2 =
ρ v1 x1 20 10
∴ Buoyant force on block = Volume × Density of liquid × g ∴ v 2 = 80 cm/s
m 60. (b) 61. (a) 62. (b) 63. (c)
= × σ ×g
ρ 64. (b)

BITSAT Archives
1. (b) As given, s = 0.06 Nm−1, The retardation,
r1 = 2 cm = 0.02 m, r2 = 5 cm = 0.05 m apparent weight
a=
Since, bubble has two surface mass of ball
V (ρ − σ ) g (ρ − σ ) g
Initial surface area of the bubble = 2 × 4π r22 = =
Vρ ρ
= 2 × 4π × (0.02)2
 0.4 − 1 3
= 32 π × 10−4 m2 = g=− g
 0.4  2
Final surface area of the bubble = 2 × 4π r22 If h be the depth upto which ball sin k, then
= 2 × 4 × π × ( 0.05)2  3 
0 − v 2 = 2 ×  − g × h
= 200 × π × 10−4 m2  2 

So, work done = s × increase in surface ⇒ 2g × 9 = 3gh


= 0.06 × ( 200 π × 10−4 − 32 π × 10−4 ) ∴ h = 6 cm

= 0.06 × 168 π × 10−4 3. (c) Relative humidity at a given temperature (R )


partial pressure of water pressure
= 0.003168 J =
vapour pressure of water
2. (b) The velocity of ball before entering the water surface
0.012 × 105
v = 2gh = 2g × 9 =
0.016 × 105
When ball enters into water, due to upthrust of water, the
velocity of ball decreases (or retarded) = 0.75 = 75%

@iitjeehelps
146 SELF STUDY GUIDE BITSAT

4. (c) In the absence of intermolecular forces, there will be 11. (a) As shown in figure, in the two arms of a tube pressure
no stickness of molecules. Hence, pressure will increase. remains same on surface PP′. Hence,
F n{mv − ( −mv )} 2mnv 8 × ρ y × g + 2ρ Hg × g = 10 × ρ x × g
5. (d) p = = =
A A A 8ρ y = 2 × 13.6 = 10 × 3.36
2 × 10−3 × 104 × 102 Y X
= = 2 × 107 N/m2
10−4 8 cm 10 cm
2 cm
6. (c) Apparent weight = actual weight − upthrust P
P'
Vdg′ = Vdg − Vρg
 d − ρ
⇒ g′ =  g
 d 
Hg
7. (a) Bernoulli's theorem is applicable only for tube flow of
non-uniform cross-section.
39.6 − 27.2
8. (a)v = velocity of efflux through an orifice or ρy =
8
= 2g H = 0.8 g/cc
v=0 p=0 12. (b) In case of soap bubble,
W = T × 2 × ∆A
= 0.03 × 2 × 40 × 10−4
= 2.4 × 10−4 J
H
13. (d) Terminal velocity,vT ∝ r 2
vT1 r12
or =
vT2 r22
It is independent of the size of orifice. 9 r1 r 3
∴ = or 1 =
9. (a) Fresh water has lesser density as compared to sea 4 r2 r2 2
water. So, upthrust in sea water will be more as 4 3 V r 3 27
compared to fresh water. ∴ V = πr or 1 = 13 =
3 V2 r2 8
10. (b) From equation of continuity
14. (b) If water falls from a tap down the streamline, then its
Av1 1 = A2v 2
area increases.
⇒ πr12v1 = πr22v 2
15.(c) For the liquids, which do not wet the glass, the liquid
v1 r22 4
⇒ = = meniscus is convex upward, so angle of contact is
v 2 r12 9 obtuse.

@iitjeehelps
12
Elasticity

Introduction
Elasticity is the property of body by virtue of which a body regains or tends to regain its original
configuration (shape as well as size), when the external deforming forces acting on it, is removed.

Cause of Elasticity
When a solid is deformed by a force, atoms or molecules get displaced from their equilibrium position
causing a change in interatomic/intermolecular distances. When deforming force is removed, the
interatomic forces tend to drive back the molecules to their original positions. This gives rise to the
property elasticity.

Elastic Behaviour of Solids


If a body completely regains its original configuration on removal of external deforming forces, it is
called a perfectly elastic body.
If a body has no tendency to regain its original configuration and tends to maintain its deformed state
even after the removal of the deforming force, the body is called a plastic (or non-elastic) body.

Stress
The internal restoring force per unit area of cross-section of the deformed body is called stress. Thus,
Restoring force F
Stress, s = =
Area A
Stress is numerically equal to the deforming force per unit area of cross-section.
The SI unit of stress is Nm -2 or pascal (Pa).

Types of Stress
Stress are of two types normal stress or longitudinal stress and tangential stress or shearing stress.

@iitjeehelps
148 SELF STUDY GUIDE BITSAT

Also the longitudinal stress can be of two types (iii) When the deforming force, applied tangentially to a
surface, produces a change in shape of the body, the
(i) If a solid in the form of
F F strain developed is called shearing strain or shear.
a wire/rod is stretched F F
by applying a force Shearing strain = f
Tensile stress
normally, then the
stress is called tensile stress (first type of
longitudinal stress). Hooke’s Law
(ii) If the solid wire/rod is According to the Hooke’s law, for any body within the
F F
compressed, then the F F elastic limit, stress developed is directly proportional to the
stress is called Fig (a) Compressive stress
strain produced.
compressive stress. Stress µ strain
(Other type of longitudinal stress) Stress = E ´ strain
(iii) For a bulk solid or liquid or gas if the deforming The ratio of stress to strain, within the elastic limit, is called
force is applied normal to the surface such that the coefficient (or modulus) of elasticity for the given
magnitude of force on any small area is proportional material.
to the area, the stress is called volumetric stress. Depending on the type of stress applied and resulting strain,
∆x we have the following three of elasticity given as
F Stress
E=
L
q Strain

F
Fig (b) Volumetric stress Stress-Strain Relationship
When a wire is stretched by an applied force, then a typical
The given Fig. (b). Here, force is applied tangentially
graph is obtained (especially in case of metals) as shown
to the cross-section of the body. So, the restoring
below
force developed due to applied tangential force is
called tangential or shearing stress. Yield
point D (Ultimate tensile strength)
σu
C
Strain σy B
E (Fracture point)
Proportional
Stress

A
When the size or shape of a body is changed under an σp limit
external force, the body is said to be strained. The change Permanent set
occurred in the unit size of the body is called strain. O
<1% Strain 30%
Change in dimension Dx
i.e. Strain = =
Original dimension x
where, Dx = Change (may be in length, in volume etc) Modulus of Elasticity
x = Original value of quantity in which change has Within the proportional region (region OA in stress-strain
occurred graph), the ratio of stress and strain remains constant. This
It is a dimensionless quantity. ratio is called modulus of elasticity.
It has same units and dimensions as that of stress.
Types of Strain As there are three types of stresses and corresponding
(i) For a wire or rod, linear strain is defined as the ratio strains. So, there are three different modulus of elasticity
of change in length to the original length. which are given below.
Change in length ( DL ) 1. Young’s Modulus
\ Longitudinal strain =
Original length ( L ) Young’s modulus of elasticity (Y ) is defined as the ratio of
(ii) When the deforming force causes a change in normal stress (either tensile or compressive) to the
volume, the strain is called volumetric strain. longitudinal strain within a elastic limit.
Change in volume ( DV ) Normal stress F /A FL
Volumetric strain = Y = = =
Original volume (V ) Longitudinal strain D L / L AD L

@iitjeehelps
ELASTICITY 149
2. Bulk Modulus
It is defined as the ratio of the normal stress to the
Elastic Potential Energy of a
volumetric strain. Strained Body
Coefficient of volume elasticity When force is applied on a wire, it gets under tension and
F /A pV work is done against the internal forces which is stored in
B= =-
DV /V DV the form of potential energy in the body.
F Stress
where, p = = the pressure or stress negative sign signifies Q Young’s modulus, Y =
A Strain L
that for an increase in pressure, the volume will decrease. AY
Reciprocal of bulk modulus is called compressibility. or F = ×x
L
x
\Work done is equal to potential energy
3. Modulus of Rigidity (Shear Modulus)
l AY AY 2
It is defined as the ratio of tangential stress to shearing W = ò0 L
× x dx =
2L
× l (QdW = F × dx ) F
stress.
Tangential stress F / A F FL 1
h= = = = U = ´ Stress ´ Strain ´ Volume
Shearing strain f Af Ax 2
1
\ Energy per unit volume = ´ Stress ´ Strain
2
Poisson's Ratio ( s )
In general, a deforming force in one direction can produce Inter-relations between
strains in other directions such as when wire is stretched,
then force will produce longitudinal strain (obviously) but Elastic Constants
it will also change its lateral dimensions, (i.e. radius of Y = Young’s modulus, h = Rigidity modulus,
cross-section) as described in figure given below K = Bulk modulus, s = Poisson’s ratio
Y = 2 h (1 + s )
F F d d – ∆d
Y = 3 K (1 - 2 s )
L 9 3 1
L+∆L = +
Y h K
The fractional change in transverse length is proportional 9 Kh
or Y =
to the fractional change in longitudinal length and is h + 3K
constant for a given material. This constant of
3 K - 2h
proportionality is called Poisson's ratio and is given by s=
Dd 6K + 2h
Transverse strain Volume elasticity of a gas under an isothermal condition is
s=- Þ s=- d
Longitudinal strain DL equal to the pressure exerted by the gas B iso = p
L Adiabatic elasticity of a gas Badia = g p, where g is the ratio
Theoretical value of s varies from - 1 to 0.5. of the two principal specific heats of the gas

@iitjeehelps
Practice Exercise
1. Calculate the stress for one litre of a perfect gas, at a on the rod as shown. The value of x, so that the stress
pressure of 72 cm of Hg, when it is compressed in A is same as that in B, is
isothermally to a volume of 900 cc. L L 2L 3L
a. b. c. d.
a. 9.88 ´ 103 N/m2 b. 10.88 ´ 103 N/m2 3 2 3 4
c. 1.088 ´ 103 N/m2 d. 2 ´ 103 N/m2 7. Two wires, one made of copper and other of steel are
2. If a metal wire of length L, having area of cross-section joined end to end. (as shown in figure). The area of
A and Young’s modulus Y , behaves as a spring of cross-section of copper wire is twice that of steel wire.
spring constant k. The value of k is They are placed under compressive force of
YA YA 2YA YL magnitudes F. Find the ratio of their lengths such that
a. b. c. d.
L 2L L A change in lengths of both wires are same
(Ys = 2 ´ 1011 N/m 2 and YC = 11
. ´ 1011 N/m 2).
3. In the figure three identical springs are shown. From
spring A, a mass of 4 kg is hung and spring shows
elongation of 1 cm. But when a weight of 6 kg is hung Copper Steel F
F
on B, the Hooke’s descends
a. 2.1 b. 1.1 c. 1.2 d. 2
A A 8. Find the total elongation of the bar, if the bar is
subjected to axial forces as shown in figure. The
cross-sectional area of bar is 10 cm 2.
B [Take E = 8 ´ 102 dyne/cm 2]
A B C D
5t 3t 1t 1t
a. 1 cm b. 2 cm c. 3 cm d. 4 cm
60cm 100cm 120cm
4. A system consists of two springs and a
mass m = 1kg as shown in figure. If mass
k a. 0.01 cm b. 0.5 cm c. 0.0675 cm d. 0.775 cm
m is displaced slightly along vertical and
released. The system oscillates with a 9. When tension in a metal wire is T1, its length was l1.
period of 2 s. Then, the spring constant k m and when tension is T2, the length is l 2. Find its
is unstretched length.
k l1 + l2 (l1T2 - l2T1) l T + T1l2
p2 p2 a. l1l2 b. c. d. 1 2
a. b. 2 T2 - T1 T1 + T2
4 6
p2 p2 m 10. One end of a steel wire is fixed to
c. d. a0=2 m/s2
8 2 ceiling of an elevator moving up
with an acceleration 2 m/ s 2 and a
5. A spring of constant k is cut into two parts of length in load of 10 kg hangs from other
the ratio 2 : 3. The spring constant of larger spring is end. Area of cross-section of the
a.
5
k b.
2
k wire is 2 cm 2. Find the longitudinal
3 3 strain in the wire (Take g = 10 m/ s 2
3 and Y = 2 ´ 1011 N/m 2)
c. k d. k
5
a. 4 ´ 1011 b. 3 ´ 10-6 c. 8 ´ 10-6 d. 2 ´ 10-6
6. A light rod of length L is A B
suspended from a support 11. Two wires one of copper and other of steel having same
cross-sectional area end lengths 1.0 m and 0.5 m
horizontally by means of two
respectively, are fastened end to end and stretched by
vertical wires A and B of
a load M . If copper wire is stretched by 1 mm, the total
equal lengths as shown in the L
extension of the combined wire is
figure. Cross-section area of A
(Given, Young’s modulii are Ycopper = 1 ´ 1011 N/m 2,
is half that of B and Young’s x
modulus of A is double than and Ysteel = 2 ´ 1011 N/m 2)
W
that of B. A weight W is hung a. 1.125 cm b. 0.2 cm c. 0.120 cm d. 0.25 cm

@iitjeehelps
ELASTICITY 151
12. A body of mass 1 kg is fastened to one end of a steel 19. A body of mass m = 10 kg is attached to a wire of
wire of cross-sectional area 3 ´ 10-6m 2 and is rotated length 0.3 m. Calculate the maximum angular velocity
in horizontal circle of radius 20 cm with a constant with which it can be rotated in a horizontal circle
speed 2 m/s. Find the elongation of the wire (Breaking stress of wire = 4.8 ´ 107 N/m 2 and area of
(Y = 2 ´ 1011N / m 2 )
cross-section of a wire = 10-6m 2)
a. 0.33 ´ 10-5 m b. 0.67 ´ 10-5 m
-5
a. 4 rad /s b. 8 rad /s c. 1 rad/s d. 2 rad /s
c. 2 ´ 10 m d. 4 ´ 10-5 m
20. Find the velocity of projection of a missile of mass 5 g.
13. If a conical wire is stretched by two forces F applied When a rubber cord is stretched to 12 cm and then
parallel to its length and in opposite direction. Normal released to project the missile (Given, area of
to end faces. The length of wire is L and its end radius cross-section of cord = 1mm 2, total unstretched length
are r1 and r 2. Find out the extension produced.
(Given, Y = Young’s modulus of wire) = 10 cm, Young’s modulus of rubber = 5 ´ 108 N/m 2)
FL FL FL FLY a. 20 m/s b. 25 m/s
a. b. c. d.
pr12Y prY
1 pr1r2Y pr1r2 c. 22 m/s d. 18 m/s

14. If for a material, Y and B are Young’s modulus and 21. A rubber cord catapult has a cross-sectional area of
Bulk’s modulus, then 25 mm 2 and the initial length of rubber cord is 10 cm.
a. Y < 3B b. Y = 3B c. Y > 3B d. 3Y = B It is stretched by 5 cm and then released to project a
missile of mass 5 g. Taking, Yrubber = 5 ´ 108 Nm -2,
15. When a sphere is taken to bottom of sea 1 km deep, it velocity of the projected missile is
contracts by 0.01%. Find the bulk modulus of elasticity
a. 20 ms-1 b. 100 ms-1 c. 250 ms-1 d. 200 ms-1
of the material of sphere. (Given, density of water
= 1g/ cm 3 ) 22. One end of a wire 2 m long and diameter 2 mm is
10
a. 9.8 ´ 10 N /m 2
b. 10.2 ´ 10 N/ m 10 2 fixed in a ceiling. A naughty boy of mass 10 kg jumps
to catch the free end and stays. there. The change in
c. 0.98 ´ 1010 N/ m2 d. 8.4 ´ 1010 N/ m2
length of wire is (Take g = 10 m/ s 2,Y = 2 ´ 1011 N/m 2)
16. The Young’s modulus of brass and steel are . ´ 10-5m
a. 3185 b. 2 mm
1.0 ´ 1011 Nm -2 and 2.0 ´ 1011 Nm -2 respectively.
c. 3 mm d. 4 m
A brass wire and a steel wire of the same length extend
by 1 mm, each under the same force. If radii of brass 23. In above problem, if Poisson’s ratio is s = 01
. . Find the
and steel wires are RB and RS respectively, then change in diameter.
a. RS = 2 RB
R
b. RS = B a. 3.184 ´ 10-5 m . ´ 10-5 m
b. 3184
2 c. 3.184 ´ 10-8 m . ´ 10-8 m
d. 3184
RB
c. RS = 4RB d. RS =
2 24. Each of three blocks shown in figure has a mass 3 kg.
The wire connecting blocks A and B has area of
17. The adjacent graph shows
the extension D l of a wire cross-section 0.005 cm 2 and Young’s modulus of
of length 1 m, suspended 4 elasticity Y = 2 ´ 1011 N/m 2. Neglect friction. Find the
∆l (×10– 4) m

from the top of a roof at elastic potential energy stored per unit volume in wire
3
one end and with a load w connecting blocks A and B in steady state.
connected to the other 2 (Take g = 10 m/ s 2 )
end. If the cross-sectional
area of the wire is 10-6 m 2, 1 A B
calculate the Young’s 0 20 40 60 80
modulus of the material of w (N)
the wire.
a. 2 ´ 1011 Nm-2 b. 2 ´ 10-11 Nm-2 C
c. 3 ´ 10-12 Nm -2 d. 2 ´ 10-13 Nm-2

18. A wire of mass m and length l is suspended from a


ceiling. Due to its own weight it elongates, consider a. 500 J/m3 b. 1000 J/m3
cross-section area of wire as A and Young’s modulus c. 2000 J/m3 d. 3000 J/m3
of material of wire as Y . The elongation in the wire is 25. A steel rod of Young’s modulus 2 ´ 1011 N / m 2
2mg mg l undergoes an elastic strain of 0.05%. The energy per
a. b.
3YA YA unit volume stored in J/m 3 is
mgl a. 12500 b. 5000
c. d. Cannot be calculated
2YA c. 10000 d. 25000

@iitjeehelps
152 SELF STUDY GUIDE BITSAT

26. If F is the force applied to an elastic bar to produce an 28. Which of the following correctly gives the elastic
extension of Dl. Then, the energy lost in the process is energy stored in the metal bar?
F × Dl (s = stress, e = strain, Y = Young’s modulus,
a. F × Dl b.
2 L= length, Dl = extension, F = load, A=
3F × Dl cross-sectional area).
c. zero d.
2 1 2
a. sY
27. If the work done in stretching a wire by 1 mm is W . 2
Then, the work required to stretch another wire of 1 s2
b.
same material but with half the radius of cross- section 2 Y
and double the length by 2 mm is 1
c. × e 2Y × ( AL)
1 1 2
a. 4W b. W c. W d. W 1
4 2 d. × s 2Y × ( AL)
2

BITSAT Archives
1. The strain-stress curves of three wires of different how much force is needed to stretch wire 2 by the
material are shown in the figure. P
same amount? [2011]
P , Q, R are the elastic limits of the Q a. 4F b. 6F
Strain

wires, the figure shows that [2014] R c. 9F d. F


a. elasticity of wire P is maximum 6. The Poisson’s ratio of a material is 0.5. If a force is
b. elasticity of wire R is maximum Stress applied to a wire of this material, there is a decrease in
c. tensile strength of R is maximum the cross-sectional area by 4%. The percentage
d. None of the above increase in the length is [2011]
2. A steel wire of length 4.7 m and cross-section a. 1% b. 2%
3.0 ´ 10-5 m 2 stretches by the same amount as a c. 2.5% d. 4%
copper wire of length 3.5 m and cross-section 7. A load of 1 kg weight is a attached to one end of a
4.0 ´ 10-5 m 2 under a given load. What is the ratio of steel wire of area of cross-section 3 mm 2 and Young’s
the Young’s modulus of steel so that of copper? [2014] modulus 1011 N/m 2. The other end is suspended
a. 1.5 : 2 b. 1.8 : 2 vertically from a hook on a wall, then the load is pulled
c. 1.5 : 1 d. 1.8 : 1 horizontally and released. When the load passes
3. The average depth of Indian Ocean is about 3000 m. through its lowest position the fractional change in
Bulk modulus of water 2.2 ´ 104 N / m 2,
is length is (Take g = 10m/ s 2 ) [2008]
DV a. 0.3 ´ 10-4 b. 0.3 ´ 10-3
g = 10 m/ s 2, then fractional compression of water
V c. 0.3 ´ 103 d. 0.3 ´ 104
at the bottom of the Indian Ocean will be [2014]
8. There is some change in length when a 33000 N
a. 1.36% b. 20.6% tensile force is applied on a steel rod of area of
c. 13.9% d. 0.52%
cross-section 10-3 m 2. The change of temperature
4. One end of steel wire is fixed to ceiling of an elevator required to produce the same elongation, if the steel
moving up with an acceleration 2 m/s 2 and a load of rod is heated, is (The modulus of elasticity is
10 kg hangs from other end. Area of cross-section of 3 ´ 1011 N/m 2 and the coefficient of linear expansion of
the wire is 2 cm 2. The longitudinal strain in the wire is . ´ 10-5 / ° C).
steel is 11 [2008]
(g = 10 m/ s 2 and Y = 2 ´ 1011 Nm -2 ) [2014]
a. 20 ° C b. 15 ° C
a. 4 ´ 1011 b. 3 ´ 10-6
c. 10 ° C d. 0 ° C
c. 8 ´ 10-6 d. 2 ´ 10-6
9. Which one of the following is not a unit of Young’s
5. Two wires are made of the same material and have modulus? [2005]
the same volume. However wire 1 has cross-sectional
a. Nm-1 b. N m-2
area A and wire 2 has cross-sectional area 3A. If
length of wire 1 increased by Dx on applying force F, c. dyne cm- 2 d. mega pascal

@iitjeehelps
Answer with Solutions
Practice Exercise 7. (b)YS =
FLS
andYC =
FLC
AS DlS AC DlC
1. (b) Let p = pressure of the gas in compressed state.
YC AC Dl C
Condition for isothermal process is LC æ Y ö æ A ö æ Dl ö
\ = F =ç C÷ ç C÷ ç C÷
p ´ 900 = 72 ´ 1000 Þ p = 80 cm LS YS AS DlS èYS ø è AS ø è DlS ø
Stress = increase in pressure = ( 80 - 72) = 8 cm of Hg F
= 0.08 ´ 13.6 ´ 103 ´ 10 = 10.88 ´ 103 N/m2 AC Dl Y 11
.
Here = 2, C = 1, C =
AS DlS YS 2
2. (a) Let the extension of wire be x for an external force F .
F LC .ö
æ 11
Stress = \ = ç ÷ ( 2) (1) = 11
.
A LS è 2ø
x
Þ Strain = 8. (c) Let dl = total elongation of the bar, A = area,
L
F / A FL E = 8 ´ 102. Let make the free body diagram.
Y = = A B B C
x / L xA
F YA 5t 5t 1t 1t
Equivalent force constant = k = =
x L
F
3. (c) x = C D
k
1t 1t
If spring constant is k for the first case,
k
it is for second case. 1
2 Using equation dl = l + F2l2 + F3l3 )
(F11
AE
4
For first case, 1= …(i) 1
k dl = (5 ´ 60 + 2 ´ 60 + 1 ´ 120)
10 ´ 8 ´ 102
6 12
For second case, x¢ = = …(ii) dl = 0.0675 cm
k /2 k
12 / k 9. (c) Let unstretched length = l
Dividing Eqs. (i) and (ii), we get x ¢ = = 3 cm
4 /k T1 = k (l1 - l ) ÞT2 = k (l2 - l )
æmö 2p 2m p 2 æQT = 2 sö T1 l1 - l æT l - T l ö
4. (d)T = 2p ç ÷ Þ k = = ç ÷ = Þl = ç 21 12 ÷
è 2k ø T 2
2 èm = 1kg ø T2 l 2 - l è T2 - T1 ø
Fl 10. (b) Here,T = m(g + a 0 ) = 10(10 + 2) = 120 N T
5. (a) As, Y =
A × Dl T 120
æ AY ö Q Stress = =
Þ F =ç ÷ × Dl A 2 ´ 10-4
è l ø
= 60 ´ 104 N / m2
As, | F | = kl m(g+a0)
stress
AY 1 Q Y =
So, k µ \ k µ strain
l l
stress 60 ´ 104
Length of bigger spring, l ¢ =
3
l Q Strain = = = 30 ´ 10-7 = 3 ´ 10-6
5 Y 2 ´ 1011
k¢ l 5 F l
So, = = 11. (a) Dl =
3
k l 3 AY
5 F
is identical for both the wire.
T A
6. (c) Stress in A = A , where AA is cross-sectional area of
AA Dl1 (copper) l1 Y2
= ×
wire A. Dl2 (steel) Y1 l2
T
æ 0.5 ö æ 1 ´ 10 ö
11
Stress in B = B , where AB is cross-sectional area of wire B. Y1 l2
AB Dl2 = Dl1 × = (1) ç ÷ç ÷ = 0.25 mm
Y2 l1 è 1 ø è 2 ´ 1011 ø
A T T 2L
It is given, AA = B , A = B which gives x = Total extension = 1 + 0.25 = 1125
. mm
2 AA AB 3

@iitjeehelps
154 SELF STUDY GUIDE BITSAT

12. (b) 21. (c) Elastic potential energy stored in the catapult
FL FL FL FL æ 1 1 YA 2 ö
13. (c) DL = = = , DL = çU = FDL = DL ÷ is converted into the kinetic
YA Y A1A2 Y pr 2 × pr 2 Ypr1r2 è 2 2 L ø
1 2
Y æ 1 ö YADL2
14. (a) = 1 - 2s energy of the projectile çK = mv 2 ÷ Þv =
3B è 2 ø Lm
1/ 2
1 é (5 ´ 108 ) ´ ( 25 ´ 10-6 ) ´ (5 ´ 10-2 ) 2 ù
and 0 <s < =ê = 250 ms-1
2 ú
ë (10 ´ 10-2 ) ´ (5 ´ 10-3 ) û
Þ 0 < 1 - 2s < 1
Y 22. (a) For equilibrium of the boy,
Hence, < 1 \ Y < 3B Tension in wire,T = weight of boy = mg = 100 N
3B
Dp DV 0.01 100 ´ 106 DL DL
15. (a) Bulk modulus, B = Þ- = = 10-4 = N / m2 Þ Strain = =
-DV / V V 100 3.14 L 2
6
Dp = pressure of water = Hrg = (10)3 ´ (1 ´ 103 ) ´ (9.8) stress 100 ´ 10 ´ 2
\ Y = =
9.8 ´ 106 strain 3.14 ´ DL
Dp = 9.8 ´ 106 Þ B = = 9.8 ´ 1010 N/ m2
10-4 100 ´ 106 ´ 2
\ DL = . ´ 10-5 m
= 3185
16. (b) 3.14 ´ 2 ´ 1011
17. (a) (D l - w ) graph is a straight line, where DdL
23. (c)Qs =
w ( 80 - 20) dD L
= = 2 ´ 105 Nm–2
Dl ( 4 - 1) ´ 10-4 sdDL 0.1´ 2 ´ 10-3 ´ 3184
. ´ 10-5
\ Dd = = = 3.184 ´ 10-8 m
L 2
Moreover, L = 1 m and A = 10-6m2
24. (b) a a T1
FL wL 2 ´ 105 ´ 1
Hence, Y = = = = 2 ´ 1011 Nm-2
ADl ADl 10-6 3 kg T T 3 kg T1
a
3 kg
18. (c) Consider an element of wire of width dx
at a distance x from bottom end of wire. mg
The force experienced by this element is dx dx From force diagram,
due to the gravitational force of portion of T = 3a …(i)
wire lower to it. x
T1 - T = 3a …(ii)
T /A
So,Y = , where D(dx ) is the 3g - T1 = 3a …(iii)
D(dx )
After solving Eqs. (i) and (ii), we get T = 10 N
dx
T 10
\ Stress = =
elongation in A 0.005 ´ 10-4
this element. \The elastic energy stored per unit volume
T mg 1 1 stress æ stress ö
Now, D (dx ) = dx = ´ xdx = ´ stress × strain = ´ stress ´ çQY = ÷
YA YAl 2 2 Y è strain ø
l mg ´ xdx mgl
Total elongation, Dl = ò D(dx ) = ò = . æ 10 ö
2
0 YAl 2YA ç ÷
19. (a) Breaking strength = tension in the wire = mr w2 stress2 è 0.005 ´ 10-4 ø
= =
4.8 ´ 107 ´ 10-6 = 10 ´ 0.3 ´ w2 2Y 2 ´ 2 ´ 1011
48 100 100
w2 = = 16 Þ w = 4 rad/s = =
0.3 ´ 10 25 ´ 10-6 ´ 10-8 ´ 4 ´ 1011 100 ´ 10-14 ´ 1011

20. (a) Equivalent rubber constant of rubber cord = 103 J/m3 = 1000 J/m3
YA (5 ´ 108)(1 ´ 10-6 ) 25. (d) 26. (b) 27. (c)
k = = = 5 ´ 103 N/m
l ( 0.1) 1
28. (c) Energy stored = ´ stress ´ strain ´ volume
From conservation of energy 2
1 1 1
Elastic PE of cord = KE of missile k ( Dl )2 = mv 2 = ´ s ´ e ´ ( AL)
2 2 2
s
æk ö æ 5 ´ 103 ö As, Y = Þ s = eY
v = ç ÷ . Dl = ç ÷ ´ (12 - 10) ´ 10-2 e
èm ø è 5 ´ 10-3 ø 1
So, E = × e 2 Y × ( AL)
v = 20 m/s 2

@iitjeehelps
ELASTICITY 155

BITSAT Archives
1. (b) As stress is shown on x-axis and strain on y-axis. where, AL = V = volume of wire, Young modulus in the
1 1 same as both the wires are made of same material. It is
So, we can say that y = cot q = = given that both the wires have same volume and same
tan q slope
extension in length.
So, elasticity of wire P is minimum and R is maximum.
F ¢ A ¢ 2 ( 3 A )2
2. (d) As given for steel wire \ = 2 = =9
F A A2
A1 = 3 ´ 10-5 m2, l1 = 4.7 m, Dl1 = Dl ; F1 = F F ¢ = 9F
For copper wire, 6. (d) Poisson’s ratio = 0.5
A2 = 4 ´ 10-5 m2,
Since, density is constant therefore change in volume is
l2 = 3.5 m, Dl2 = Dl, F2 = F zero, we have
Let Y1 and Y2 be the Young’s modulus of steel wire and V = A ´ l = constant
copper wire, respectively. log V = log A + log l
F l F 4.7 dA dl
So, Y1 = 1 ´ 1 = -5
´ or + =0
A1 Dl1 3 ´ 10 Dl A l
F2 ´ l 2 F ´ 3.5 dl dA
and Y2 = = =-
A2 ´ Dl2 4 ´ 10-5 ´ Dl l A
Y1 4.7 ´ 4 ´ 10-5 \Percentage increase in length = 4%
= = 1.8 mgl
Y2 3.5 ´ 3.0 ´ 10-5 7. (a) Y=
A Dl
So, Y1 : Y2 = 1.8 : 1
3. (a) The pressure exerted by a 3000 m column of water on D l mg
Þ =
the bottom layer. l AY
p = hrg = 3000 ´ 1000 ´ 10 Dl 1´ 10
\ =
= 3 ´ 107 kg m-1s-2 = 3 ´ 107 N/m2 l 3 ´ 10-6 ´ 1011
æ DV ö = 0.3 ´ 10-4
Fractional compression ç ÷
è V ø
Force l
8. (c) Modulus of elasticity = ´
Stress 3 ´ 107 Area D l
= = = 1.36 ´ 10-2
B 2.2 ´ 109 33000 l
DV 3 ´ 1011 = ´
´ 100 = 1.36% 10-3 Dl
V
D l 33000 1
4. (b) As,T = m (g + a 0 ) = 10 (10 + 2) = 120 N = ´
l 10-3 3 ´ 1011
T 120
Stress = = = 60 ´ 104 Nm-2
A 2 ´ 10-4 = 11´ 10-5
Stress Stress Dl
and Y = , Strain = Change in length, = a DT
Strain Y l
60 ´ 104 11´ 10-5 = 11
. ´ 10-5 ´ DT
=
2 ´ 1011 Þ DT = 10 K or 10° C
= 30 ´ 10 -7
= 3 ´ 10 -6 stress
9. (a)Y = = N / m2 or pascal (in SI system)
strain
FL
5. (c) Y = dyne
ADL and Y= (in CGS System)
cm2
YADL YA 2DL YA 2DL YA 2Dx
or F = = = = Thus, Nm-1 is not the unit of Young’s modulus.
L AL V V

@iitjeehelps
13
Wave Motion

Wave
In physics, a wave is a disturbance or oscillation that travels through space and matter, accompanied
by a transfer of energy. In simple terms, we can say that wave motion involves transfer of disturbance
(energy) from one point to the another with particles of medium oscillating about their mean
positions, i.e. particles of the medium do not travel themselves along with the wave.

Types of Waves
Types of waves are shown as below
Waves

Mechanical wave Non-mechanical wave

Mechanical wave

Transverse wave
Waves

Non-mechanical wave

Longitudinal wave
Transverse wave Longitudinal wave

Mechanical Waves Waves which can be produced or propagated only in material medium are
defined as mechanical waves, e.g. waves on water surface, waves on strings, sound waves, etc.
Non-mechanical Waves Waves which can be independent of propagation medium are defined as
non-mechanical waves, e.g. visible light, ultraviolet light, X-rays, etc.

Transverse Waves
A wave motion in which the particles of the medium oscillate in simple harmonic motion in a
direction perpendicular to its direction of propagation, are defined as transverse waves.
e.g. Movement of string of a sitar or violin. On propagation of transverse waves, medium is divided
into alternate crests and troughs. Crest Wavelength Crest
The distance between two consecutive crests or (λ)
Amplitude
troughs is called wavelength of the transverse wave.
It is denoted by λ. Trough Trough

@iitjeehelps
WAVE MOTION 157

Speed of Transverse Wave (v) A particle velocity at a given position at a given time
is equal to product of wave velocity and negative of
The expression for speed of transverse waves in a solid and slope of the wave curve at the given position and
in case of a stretched string can be obtained theoretically time.
given as d
η u ( x , t ) = [ y( x , t )]
(i) In solids, v = dt
d d2y
where, η is the modulus of rigidity and d is the (vi) Acceleration of a particle at ( x , t ), a =
dt 2
density of the medium.
Also, |amax | = − ω 2 A
T Mg
(ii) In a stretched string, v = =
m πr 2d (vii) Intensity A flow of energy per unit area of
cross-section of the string in unit time is defined as
where, T = the tension in the string,
intensity.
m= the mass per unit length of the string,
Power
M= mass suspended from the string, Thus, I=
Area of cross-section
r = radius of the string and
d= density of the material of the string. P 1
= = ρω 2 A2v
S 2
Progressive Waves where, ρ is density of string and v is speed of the wave.
A wave which travels continuously in the same direction
NOTE From the formula for intensity, we can write I ∝ A 2 , where A is
without any change in its amplitude is called a progressive amplitude.
wave or a travelling wave. A progressive wave may be
transverse or longitudinal.
Principle of Superposition of
Displacement Relation for a Progressive
Wave
Waves
(i) If a plane wave travels in a medium along the When two or more than two waves of similar type
positive x-direction, then the displacement y of a propagate in a medium simultaneously, then resultant
particle located at x at time t is given by displacement of any particle of the medium is equal to the
vector sum of displacements produced by individual waves
y( x , t ) = Asin(ωt − kx )
separately. This principle is called principle of
where, A = amplitude of the wave superposition of waves.

ω= or 2πν is angular frequency y = y 1 + y 2 + y3 + K
T
 2π 
k =   , i.e. angular wave number Interference of Waves
λ
When two waves of same frequency A2 A
(ii) If a wave is travelling along the negative x-direction,
(or same wavelength) travelling along
then
same path superimpose each other,
y( x , t ) = Asin(ωt + kx ) φ
there occurs redistribution of energy
(iii) Phase change with position At any instant t , if φ 1 in the medium. If at a given position (x θ
and φ 2 are the phase of two particles whose distance being constant) displacement due to A1
from origin are x 1 and x 2 . two waves be
Then, φ 1 = (ωt − kx 1 ) and φ 2 = (ωt − kx 2 ) y 1 = A1 sin ωt and y 2 = A2 sin (ωt + φ )
φ 1 − φ2 = k( x 2 − x 1 ) Then, resultant displacement
2π y = y 1 + y 2 = A sin (ωt + φ )
So, phase difference, ∆φ = × path difference (∆ x )
λ where, A = A12 + A22 + 2 A1 A2 cos φ
(iv) Phase change with time If a particle at a distance x A2 sin φ
from origin has phase φ 1 at t 1 and φ 2 at t 2 . and tan θ =
A1 + A2 cos φ
Then, φ 1 = (ωt 1 − kx ) and φ 2 = (ωt 2 − kx )
φ 1 − φ2 = ω (t 1 − t 2 ) At those points where phase difference φ = 0° or 2nπ, i.e. an
2π integer multiple of 2 π(n = 1, 2 , 3 , . . . ),
So, phase difference, ∆φ = × time difference (∆t ) A = A1 + A2 = Amax
T

@iitjeehelps
158 SELF STUDY GUIDE BITSAT

This is known as constructive interference. (ii) Law of tension If l and m are constants, then
At those points where φ = (2n − 1)π , i.e. phase difference is frequency of vibration of stretched string ( f ) is
an odd multiple of π and hence, directly proportional to the square root of tension
A = A1 ~ A2 = Amin (T ) of the string.
This is known as destructive interference. i.e. f ∝ T
If I 1 and I 2 are intensities of the interfering waves and φ is (iii) Law of mass If l and T are constants, then frequency
the phase difference, then resultant intensity is given by of vibration of stretched string ( f ) is inversely
I = I 1 + I 2 + 2 I 1 I 2 cos φ proportional to the square root of mass per unit
length of wire.
I max = I 1 + I 2 + 2 I 1 I 2 = ( I 1 + I 2 )2 for φ = 2 πn 1
i.e. f ∝
and I min = I 1 + I 2 − 2 I 1 I 2 m
I min = ( I 1 − I 2 )2 for φ = (2n + 1) π (iv) Law of density If l, T and D are constants, then
frequency of vibration of stretched string ( f ) is
inversely proportional to the square root of the
Stationary or Standing density of wire.
Waves in String 
Q f =
n T
=
n Tl A n
=
4T 
, i.e. f ∝
1
When two sets of progressive waves of the same type  2 l m 2 l MA 2 l ρ D π 
2
ρ
having same amplitude, same frequency and same speed
along the same straight line in opposite directions
superimposed, then a new wave is formed. This is called
stationary wave or standing wave. Consider two waves of
Fundamental Mode
the same frequency, speed and amplitude, which are and Harmonics
travelling in opposite directions along a string. Two such Normal modes of vibration in a string fixed at both ends are
waves may be represented by the equations discussed as below
y 1 = Asin(kx − ωt ) and y 2 = Asin(kx + ωt )
First harmonic λ1
l ⇒ l=
Hence, the resultant may be written as (string plucked at
l 2
y = y 1 + y 2 = Asin(kx − ωt ) + Asin(kx + ωt ) 2 ⇒ λ 1 = 2l
to get 1 loop) v
y = 2 Asin kx cosωt ⇒ f1 =
λ1
This is the equation of a standing wave. 1 T v
N A N ⇒ f1 = =
2l m 2l
λ1
Characteristics of Stationary Waves 2
(i) There are certain points in the medium in a standing 2λ 2
Second harmonic or ⇒ =l
wave, which are permanently at rest. These are first overtone 2
N
called as nodes. Distance between two consecutive (string plucked at
l ⇒ λ2 = l
λ 4
nodes is ⋅ to get 2 loops) ⇒ f2 =
2 T
2 N A A N 2l m
λ2 v 
(ii) Some points in the medium in a standing wave, have ⇒ f2 = 2 f1 = 2  
 2l 
maximum amplitude. These are called as ant-nodes.
λ 3λ 3
Distance between two consecutive ant-nodes is ⋅ Third harmonic or ⇒ l=
2 second overtone 2
l 2l
(string plucked at ⇒ λ3 =
6
Laws of Vibrations of to get 3 loops)
NA
N
A
N
A N

3

f3 =
3 T
3λ3
Stretched Strings 2
2l
v 
⇒ f3 = 3 f1 = 3  
m

(i) Law of length If T and m are constants, then  2l 

fundamental frequency of vibration of stretched


Note (i) For string fixed at both ends, the waves nth harmonic
string ( f ) is inversely proportional to the length ( l ) of
produced having all harmonics. (ii) For a string fixed at 2l
the string. fn = nf1 ⇒ λ n =
only one end antinode will be formed at free end and n
1 node at fixed end, only odd harmonics are present.
i.e. f ∝
l

@iitjeehelps
Practice Exercise
1. The equation of a wave travelling on a stretched string wave velocity
−(bx + ct )
a. wave velocity × strain b.
along the x-axis is y = ae . The direction of strain
propagation of wave is angular frequency
c. wave velocity d.
a. along negative y-axis b. along positive y-axis propagation constant
c. along negative x-axis d. along positive x-axis 9. In a wave motion y = a sin (kx − ω t ), y can represent
2. In (Q. 1.) the maximum displacement of particle of a. electric field b. magnetic field
string is c. displacement, pressure d. All of these
a. a b. b c. c d. c /b
10. The equation of a wave travelling on a stretched string
3. In (Q. 1.) the speed of wave is  t x 
is y = 4 sin 2π  − 
a. c /b b. b /c c. a d. c  0.02 100
4. The wave travels along a string whose equation is Here, x and y are in cm and t is in second. The speed
p3 of wave is
y = , where p = 2 unit and q = 0.5 unit.
p 2 + ( px − qt )2 a. 50 m/s b. 40 m/s c. 50 cm/s d. 40 cm/s
Find the direction of propagation of wave. 11. In (Q. 10.) the ratio of particle velocity amplitude and
a. along + y-axis b. along − x-axis wavelength is
c. along + x-axis d. None of these a. π b. 2π c. 3π d. 4π

5. If wave y = A cos (ωt + kx ) is moving along x-axis. 12. In (Q. 10.) the relative deformation amplitude of
medium is
The shape of pulse at t = 0 and t = 2 s
a. 0.02π b. 0.08π
a. are different b. are same
c. may not be same d. None of these c. 0.06π d. None of these

6. At any instant a wave travelling along the string shown 13. Along a stretched string equation of transverse wave
in figure. Here, point A is moving upward. Which of the  x t 
is y = 3 sin 2π  − 
following statement is true?   20 0.01 
B
where, x , y are in cm and t is in second. Calculate
A C wave velocity.
O
x a. 20 m/s b. 30 m/s c. 15 m/s d. 25 m/s
14. A transverse wave along a string is given by
a. The wave is travelling to the right  π
y = 2 sin  2π ( 3t − x ) + 
b. The displacement amplitude of wave is equal to  4
displacement of B at this instant
where x and y are in cm and t in second. Find
c. At this instant C also directed upward.
acceleration of a particle located at x = 4 cm at t = 1s.
d. None of the above
a. 36 2π 2 cm/s2 b. 36π 2 cm/s2
7. In the given figure, c. −36 2π 2 cm/s2 d. −36π 2 cm/s2
B
F  x
15. If y = y 0 sin 2π ft −  is the equation of transverse
A
C
E  λ
wave, then for what value of λ the maximum particle
D velocity is equal to four times the wave velocity?
y 0π
a. the velocity of particles B and C are same a. y 0π b. c. 2y 0π d. 1.5 y 0π
2
b. the velocity of particles A, C and E are maximum
c. the particle F moves upward 16. Along a stretched wire a transverse wave passes with
speed 3000 m/s. If the tension in the wire increased
d. all particles have same velocity
four times, then the velocity of the wave is
8. If a wave propagates through a medium. Then, the a. 1500 m/s b. 3000 m/s
velocity of particle of medium is given by c. 6000 m/s d. 9000 m/s

@iitjeehelps
160 SELF STUDY GUIDE BITSAT

17. A long rubber tube having mass 25. In (Q. 24.) the tension in string is T and the linear
0.9 kg is fastened to a fixed mass density of string is µ. The ratio of magnitude of
support and the free end of the maximum velocity of particle and the magnitude of
tube is attached to a cord which maximum acceleration is
passes over a pulley and 1 µl 2 µl 2 1 T 1 µl 2
supports an object, with a mass Cord a. b. 2π c. d.
2π T T 2π µl 2 4π T
of 5 kg as shown in figure. If the m
12 m
tube is struck by a transverse 26. In (Q. 24.) if at t = 0, y = 2.5 mm, the equation of
blow at one end, find the time Rubber
tube standing wave is
required for the pulse to reach 
2π  T 
the other end. a. (2.5 mm) sin x cos  2π  2   t
a. 5 s b. 0.47 s c. 4.7 s d. 3.2 s l   µl  
π
18. The time taken by a transverse wave going on a wire b. 5 mm sin x cos 2πt
having mass 5 g, from one end to another end of wire l
is 0.5 s. The area of cross-section of wire is 1 mm 2 2π   T 
c. 5 mm sin x cos  2π  2   t
and Young’s modulus of elasticity is 16 × 1011 N/ m 2. l   µl  
The speed of wave is 80 m/s. The strain in wire is 
2π  T 
a. 5 × 10−7 b. 2 × 10−7 c. 3 × 10−6 d. 4 × 10−6 d. 5 mm cos x cos  2π  2   t
l   µl  
19. Find the time taken by a transverse wave to travel the
full length of a uniform rope of mass 0.1 kg and length 27. If a string fixed at both ends vibrates in four loops. The
2.45 m hangs from the ceiling wavelength is 10 cm. The length of string is
a.1 s b. 0.5 s c. 2 s d. 1.5 s a. 5 cm b. 15 cm c. 30 cm d. None
20. A transverse wave of equation y = 2 sin ( 0.01x + 30 t ) 28. In (Q. 27.) the distance of plucking poing from the
moves on a stretched string from one end to another fixed end is
end. In the equation of wave, x and y are in cm and t a. 5 cm b. 10 cm c. 2.5 cm d. 7.5 cm
is in second. The time taken by wave to reach from 29. A stretched wire carries a body of density
one end to another end of string is 5 s. The length of σ = 8000 kg/m 3 at its end. The fundamental frequency
string is of vibration of wire is 280 Hz. The body is dipped
a. 10 m b. 100 m c. 150 m d. 160 m completely in a vessel of water. Find the new
frequency of fundamental mode of vibrations.
21. A sinusoidal wave travelling in the same direction
(Density of water is ρ = 1000 kg/m 3 )
have amplitudes of 3 cm and 4 cm and difference in
phase by π /2. The resultant amplitude of the a. 262 Hz b. 260 Hz c. 243.2 Hz d. 255.5 Hz
superimposed wave is 30. An elastic string of length 2 m is fixed at its end. The
a. 7 cm b. 5 cm string starts to vibrate in third overtone with a
c. 2 cm d. 0.5 cm frequency 1200 Hz. The ratio of frequency of lower
22. Two simple harmonic motions are represented by the overtone and fundamental is
a. 1 b. 2 c. 3 d. 4
 π
equations y 1 = 10 sin  3 πt + 
 4 31. If a string is stretched with a weight 4 kg then the
fundamental frequency is equal to 256 Hz. What
and y 2 = 5 ( 3 sin 3 πt + 3 cos 3 πt ). Their amplitudes
weight is needed to produce its octave?
are in the ratio of a. 4 kg-wt b. 12 kg-wt c. 16 kg-wt d. 24 kg-wt
a. 3 b. 1/ 3 c. 2 d. 1/6 32. Calculate the minimum possible length of the string
2πx  2πvt  when both ends of string are fixed and has
23. Predict for the wave y = A cos sin   consecutive standing wave mode for which distance
λ  λ 
between adjacent nodes are 18 cm and 16 cm
a. It is a progressive wave respectively.
b. It is a transverse progressive wave a. 150 cm b. 144 cm c. 140 cm d. 142 cm
c. It is a longitudinal progressive wave
d. It is a stationary wave 33. A sonometer wire of length 114 cm is fixed at both the
ends. Where should the two bridges be placed so as
24. A string of length l is fixed at both ends and is vibrating to divide the wire into three segments whose
in second harmonic. The amplitude at anti-node is fundamental frequencies are in the ratio 1:3:4?
2 mm. The amplitude of a particle at distance l /8 from a. At 36 cm and 84 cm from one end
the fixed end is b. At 24 cm and 72 cm from one end
5 10 c. At 48 cm and 96 cm from one end
a. 5 2 mm b. mm c. 5 mm d. mm
2 2 d. At 72 cm and 96 cm from one end

@iitjeehelps
WAVE MOTION 161
34. A wave of angular frequency ω propagates so that a 37. The transverse displacement y (x , t ) of a wave on a
certain phase of oscillation moves along x-axis, string is given by y (x , t ) = e − (ax
2
+ bt 2 + 2 ab xt )
. This
y -axis, z-axis with speeds c1, c 2 and c 3 respectively.
Find the propagation constant k. represents a
ω ω $ ω $ ω $ a. wave moving in − x-direction with speed
b
a. ( $i + $j + k$ ) b. i+ j+ k
a
c12 + c 22 + c 32 c1 c2 c3
b. standing wave of frequency b
1
c. (ω $i + ω $j + ω k$ ) d. None of these c. standing wave of frequency
1
c
b
35. Wave of frequency 500 Hz has a phase velocity d. wave moving in + x-direction with speed a / b
360 m/s. The phase difference between two
  x π
displacement at a certain point at time 10−3 s apart 38. For a wave y = 0.0002 sin 2π 110t −  + 
will be 
 3 3 
π π
a. π rad b. rad c. rad d. 2π rad is travelling in a medium. Find energy per unit volume
2 4 being transferred by wave if density of medium is
π x 1.5 kg/m 3 .
36. Equation of a plane wave is given by 4 sin 2t + .
4  8 a. 14 × 10−4 J / m3
The phase difference at any given instant of two b. 143.2 × 10−4 J / m3
particles 16 cm apart is c. 14.3 × 10−4 J / m3
a. 60° b. 90° c. 30° d. 120° . × 10−4 J / m3
d. 143

BITSAT Archives
1. The equation of a simple harmonic motion is given by a. y = A sin (ωt − kx ) b. y = A cos (kx − ωt )
π c. y = A cos (ωt − kx ) d. y = A sin (kx − ωt )
y = 3 sin (50 t − x ), where x and y are in metres and
2
t is in seconds, the ratio of maximum particle velocity 5. A wave has velocity v in medium P and velocity 2v in
to the wave velocity is [2014] medium Q. If the wave is incident in medium P at an
a. 25 m/s b. 30 m/s angle of 30°, then the angle of refraction will be [2007]
c. 50 m/s d. None of these a. 30°
2. A stretched string of length 2 m vibrates in 4 segments. b. 45°
c. 60°
The distance between consecutive nodes is [2010]
d. 90°
a. 0.5 m b. 0.25 m c. 1.0 m d. 0.75 m
6. The equation of progressive wave is y = 0.2 sin 2π
3. At t = 0, a transverse wave pulse in a wire is described
 t x 
by the function y = 2
6
, where x and y are in  0.01 − 0.3 , where x and y are in metre and t is in
x −3
second. The velocity of propagation of the wave is
metre. The function y (x , t ) that describes this wave [2007]
equation, if it is travelling in the positive x-direction a. 30 m/s
with a speed of 4.5 ms –1 is [2010] b. 40 m/s
6 6 c. 300 m/s
a. y = b. y =
(x + 4.5 t )3 − 3 (x − 4.5 t 2 ) + 3 d. 400 m/s
6 6
c. y = d. y = 7. Two progressive waves having equation x1 = 3 sin ωτ
(x + 4.5 t )2 − 3 (x − 4.5 t )2 − 3 and x 2 = 4 sin (ωτ + 90° ) are superimposed. The
4. In a sine wave, position of different particles at time amplitude of the resultant wave is [2006]
t = 0 is shown in figure. The equation for this wave a. 5 unit
travelling along positive x-axis can be [2009] b. 1 unit
y
c. 3 unit
d. 4 unit
x 8. A particle on the trough of a wave at any instant will
come to the mean position after a time
(T = time period) [2005]
a. T /2 b. T /4 c. T d. 2 T

@iitjeehelps
Answer with Solutions
Practice Exercise Compare the given equation with standard equation
 x t 
1. (c) y = A sin 2π  −  
  λ T 
2. (a)Qy = ae −(bx + ct )
We get A = 3 cm
At t = 0,y = ae −bx λ = 20 cm
The value of y is y max = a. 1
f = = 100 Hz
3. (a) Q bx + ct = constant T
20 × 100
∴ b
dx
+c
dt
=0 v = λf = = 20 m/s
dt dt 100
dx c 14. (c)
∴ Wave speed = =
dt b 15. (b) ω = 2πf and v max = ωr = 2πfy 0
p3 p Wave velocity = fλ
4. (c) At t = 0, y = =
( p + p 2x 2 ) 1 + x 2
2 2π fy 0 = 4fλ
y = f ( px − qt ) (Given, velocity of particle is four times of wave velocity)
y π
y = f (ωt − kx ) λ= 0
2
Here, sign of f and x are opposite, so direction should be
+ x-axis. T 
16. (c) From relation,v =   = 3000 m/s
5. (b) 6. (b) 7. (b)  m
∂y  4T  T 
8. (a) Strain = , v′ =   = 2 ×  
∂x m  m
∂y
As we know, v particle = − c v′ = 2 × 3000 = 6000 m/s
∂x
= wave velocity × strain 17. (b) Let µ = mass per unit length of rubber tube
9. (d) 10. (a) 0.9
µ= = 0.075 kg/m
11. (d) We know that, 12
2π Tension in tube,T = mg = 5 × 9.8 = 49 N
k =
100 Speed of wave on the tube
2π 2π T 
or = v =   =
49
= 25.56 m/s
λ 100 µ 0.075
∴ λ = 100 cm
AB 12
∂y Time required = t = = = 0.47 s
But v Particle = = aω cos (ωt − kx ) v 25.56
∂t
2π T 
(v Particle)max = aω = 4 × = 400π cm/s 18. (a) c =  
0.02 µ
(v Particle)max 400π T
∴ = = 4π ∴ T = µc 2, y =
λ 100 A strain
12. (b) The relative deformation is
T µc 2 mc 2
∂y ∂ ∴ Strain = = =
= [ 4 sin (ωt − kx )] AY AY lAY
∂x ∂x
= − 4k cos (ωt − kx ) mc 2 mc 5 × 10−3 × 80
= = = −6
The relative deformation amplitude (ct ) AY AYt 10 × 16 × 1011 × 0.5
 ∂y  2π = 5 × 10−7
  = ak = 4 × = 0.08π
 ∂x  max 100
19. (a) Speed of transverse wave at lower end = 0.
13. (a) Let wave velocity = v Speed of tranverse wave at the upper end = lg
v = λf Let, acceleration = a, time to travel = t
The given equation of wave  gl  1
 x t  a=  ∴ s = at 2
y = 3 sin 2π  −   t  2
  20 0.01 

@iitjeehelps
WAVE MOTION 163
1  gl  2 T 
I=  t ω = 2πν = 2π  2
2 t   µl 
l  2.45 2π T 
t =2   =2   ∴ y = (5 mm) sin x cos 2π  2t
g  9.8  l  µl 
t = 1s 27. (b) 28. (c) 29. (c)
20. (c) 21. (b) 30. (b)
22. (b) 23. (d) 31. (c) To produce an octave, force should be 4 times the
24. (b) The equation of stationary wave on a string fixed at initial value as octave has double frequency.
both ends is 32. (b) Let length of string = l
y = 2a sin kx cos ωt
l/2
The amplitude of a particle at distance x from fixed end is
A = 2a sin kx .
For second harmonic, string vibrates in two loops (shown
in figure). 18 cm n loops
Loops
λ I
∴ =
2 2
16cm
l/2

(n+1) loops
λ/2 18n = l
∴ λ =l 16 (n + 1) = l
2π 2π
∴ k = = n=8
λ I l = 144 cm
2π l π
∴ A = 2a sin kx = 2a sin × = 2a sin 33. (c)
l 8 4
The amplitude at antinode is 2a = 5 mm 34. (b) Propagation constant is a vector quantity.
π 5 ∴ k = k $i + k $j + k k$
∴ A = 5 sin = mm x y z
4 2 ω ω
Here, kx = =
25. (a) Here,v max = Aω = 2aω (at antinode A = 2a) and v x c1
acceleration is ω ω
ky = =
fmax = ω 2A = 2 aω 2 vy c2
2aω ω ω
∴ Ratio = =
1 and kz = =
2aω 2
ω vz c3
ω ω $ ω $
n T  2 T  ∴ k = $i + j+ k
Q Frequency, ν =   =   c1 c2 c3
2l  µ  2l µ
35. (a) ∆x = ct …(i)
(for second harmonic, n = 2)
2π 2π
T  ∴ ∆φ = ∆x = ∆x
=  2 λ c /f
 µl 
2πct
Q ω = 2πν = = 2πft [from Eq. (i)]
c /f
1
∴ Ratio = = 2π × 500 × 10−3
2πν
= π rad
1  µl 2 
=   π  x
2π T  36. (b) y = 4 sin  2t +  …(i)
4  8
26. (c) The equation of standing wave on a string fixed both π πx 
or y = 4 sin  t +  …(ii)
ends is 2 32 
y = 2a sin kx cosωt The standard equation is
Here, 2a = 5 mm y = 4 sin (ωt ± kx )

k = Comparing the Eq. (i) with Eq. (ii), we get
l

@iitjeehelps
164 SELF STUDY GUIDE BITSAT

π 2 2 2
37. (a) y (x , t ) = e − ( ax + bt + 2 ab xt ) = e − ( a x + b t )
k =
32 It is a function of type
2π π
∴ = y = f (ω t + kx )
λ 32 ∴ y (x , t ) represents wave travelling along − x-direction.
λ
∴ = 32 ω b b
2 Speed of wave = = =
k a a
∴ λ = 64 1
2π 38. (b) We know E0 = 2π ρA v = ρA 2ω 2
2 2 2
∴ ∆φ = ∆x 2
λ 1
2π π = × 15 . × ( 0.02)2 ( 220π )2 × 10−4
= × 16 = = 90° 2
64 2 E0 = 143.2 × 10−4 J/m3

BITSAT Archives
1. (c) The given wave equation is dy / dt − ω A cos (kx − ωt )
∴ =
π dy / dx kA cos (kx − ωt )
y = 3 sin (50t − x )
2 dx ω
 π  = − = −v
⇒ y = 3 sin 25πt − x  …(i) dt k
 2  dy  dy 
∴ = −v  
Comparing with standard equation dt  dx 
y = a sin(ωt − kx ) …(ii) i.e. particle velocity = − (wave speed) × slope.
ω = 25π , k = π / 2 And slope at x = 0 and t = 0 is positive in figure. Therefore,
Wave velocity, particle velocity is in negative y-direction. Therefore, (d) is
ω 25π correct option.
v = = = 50 m/s
k π /2 sin i v1
5. (a) v = =
2. (a) Since, it vibrates in 4 segments, so we must get sin r v 2
5 nodes and 4 antinodes. 2v
⇒ sin r = sin 30° ×
v
1
N A A A A N ⇒ sin r = × 2 × 1
N N N 2
2m ⇒ r = 90°
⇒ 2 = 2λ ⇒ λ = 1m Coefficient of t 2π / 0.01
6. (d) v = = = 30 m/s
Distance between two consecutive nodes is Coefficient of x 2π / 0.3
λ 1
= = 0.5 m 7. (d) x1 = 2 sin ωt and x 2 = 4 sin (ωt + 90° )
2 2
The phase difference between the two waves is 90°.
a
3. (c) y (x , t ) = is another form of progressive So, resultant amplitude
(x ± vt )2 + b
wave equation propagating with a speedv. Negative sign a = ( 3)2 + ( 4)2 = 9 + 16 = 25
to be taken for propagation along +x-axis and positive = 5 unit
sign to be taken for propagation along − x-axis.
8. (b) The time taken by the particle to come to mean
4. (d) It is clear from figure that at t = 0, x = 0 displacement position from the trough =
T
y = 0. Therefore, option (a) or (b) may be correct. 4
Crest
In case of (d), y = A sin (kx − ωt )
dy
= A cos (kx − ωt )( −ω )
dt O T/4 T/2 T
dy
= A cos (kx − ωt )(k ) 3T/4
dx Trough

@iitjeehelps
14
Sound Wave

Introduction
Sound may by defined as the physical cause which enables us to have the sensation of hearing. Sound
wave is a longitudinal wave.

Longitudinal Waves
In this wave motion, the particles of the medium oscillate about their mean or equilibrium position
along the direction of propogation of the wave motion itself. Longitudinal wave travels in the form of
compression and rare factions.
● Compression It is a region of the medium in
which particles are compressed (i.e. come
closer), so distance between particles become
less than the normal distance between them.
● Rarefaction It is a region of the medium in
which particles of the medium are rarefied, i.e.
particles get farther apart than the normal Compression Rarefaction Compression
distance between them. Propagation of longitudinal wave

Speed of Longitudinal Wave (or Sound Wave)


According to Newton’s formula, speed of sound in a gas is

(i) Newton’s Formula


B
v= , where B denotes bulk modulus of elasticity and D denotes density of medium.
D

(ii) Laplace’s Correction


For gases, E = coefficient of adiabatic elasticity.
E = γp
Here, p is the pressure of the gas.
γp Cp
v= , where γ denotes adiabatic constant =
D CV

@iitjeehelps
166 SELF STUDY GUIDE BITSAT

(iii) Effect of Temperature on Velocity 2π


Phase difference ( φ ) = × time difference (t )
T
With rise in temperature then velocity of sound increases 2πt
⇒ φ=
γRT v2 T2 T
as v= ; =
M v1 T1 Tφ
⇒ t=

i.e. v∝ T
T
Time difference (t ) = × path difference ( x )
(iv) Effect of Pressure for Gases Medium λ
p Tx λt
remains constant. Pressure has no effect on the velocity of ⇒ t= ⇒ x=
D λ T
sound, provide temperature remains constant. (i) For a wave, velocity
v = frequency (n ) × wavelength ( λ )
(v) Effect of Humidity
⇒ v = nλ
When humidity in air increases, its density decreases and so
velocity of sound increases. 2π 2πv
(ii) Angular speed, ω = 2 πn = =
T λ
Y K
For solids, v = . For liquids, v =
D D
where, Y = Young’s modulus of elasticity Reflection and Transmission
K = bulk modulus of elasticity. of Waves
(vi) Effect of Wind Velocity When sound waves are incident on a boundary separating
The velocity of sound in a particular direction is the two media, a part of it is reflected back into the initial
algebraic sum of the velocity of sound and the component medium while the remaining is partly absorbed and partly
of wind velocity. transmitted into the second medium.

NOTE The density of a gas mixture can be calculated as Characteristics


ρ V + ρ2 V2 + L + ρn Vn (i) In case of reflection and transmission of sound, the
ρm = 1 1
V1 + V2 + L + Vn frequency of the wave remains unchanged, i.e.
where, ρm = Density of the gas mixture ωi = ω r = ωt = ω.
ρ1 K ρn = Density of each of the components (ii) The incident ray, the reflected ray, normal and the
V1, V2 K Vn = Volume share of each of the components refracted ray are always in the same plane.
(iii) In case of reflection of sound,

Sound Waves angle of incidence = angle of reflection


sin i v
Mechanical waves in air having a frequency ranging from (iv) In case of refraction of sound, = i
sin r ′ vt
20 Hz to 20 kHz are known as audio waves or sound waves.
Waves having frequencies less than 20 Hz are known as the (v) When a longitudinal wave is reflected from a denser
infrasonic waves. Waves in air having frequencies greater medium, compression and as rarefaction do not
than 20 kHz are known as ultrasonic waves. invert after reflection.
Sound waves do not undergo polarisation. Transverse
waves only get polarised.
Sound exhibits reflection, refraction, interference and
Standing Wave in Organ
diffraction but not polarisation. A person can emit Pipes
5 syllables in 1s. Each syllable is produced in 0.2 s.
Organ pipes are the musical instruments which are used for
producing musical sound by blowing air into the pipe.
Relation between Phase Difference, Organ pipes
Path Difference and Time Difference
2π Closed organ pipes Open organ pipes
Phase difference ( φ ) = × path difference ( x )
λ Organ pipes which are closed Pipes which are opened
2πx φλ at one end and open at other end at both ends
⇒ φ= ⇒ x=
λ 2π

@iitjeehelps
SOUND WAVE 167

Closed Organ Pipes Beats


If one end of a pipe is closed, the reflected wave is 180° out ● When two sound waves of nearly equal (but never equal)
of phase with the incoming wave. The displacement of the
or slightly different frequencies and equal or nearly equal
small volume elements at the closed end must always be
amplitudes travelling along the same direction
zero. Hence, the closed end must be a displacement node.
superimpose at a given point, the resultant sound
Various modes of vibrations of air column in a closed organ
intensity alternately rises and falls. This alternate rise and
pipe are shown below
fall of sound at a given position is called beats.
First harmonic λ1
⇒l = ● One beat is said to be formed when starting from
(fundamental mode) 4
⇒ λ 1 = 4l minimum sound the intensity once rises to its maximum
⇒ f1 =
v value and then falls back to the minimum level.
4l ● Number of beats formed per second is called the
Third harmonic (first ⇒l=
3λ 3 frequency of beats. If two sound waves of frequencies
overtone) 4 ν 1 and ν 2 superimpose, then frequency of beats
4l
⇒ λ3 =
3
= ( ν 1 − ν 2 ), i.e. either ( ν 1 − ν 2 ) or ( ν 2 − ν 1 ).
λ3 λ3 3v ● For formation of distinct beats then difference between
⇒ f3 =
4 2 4l the frequencies of two superposing notes should be less
⇒ f3 = 3 f1 than 10 Hz.
Fifth harmonic 5λ 5
⇒l = ● If source of sound is linear, then at a distance d from the
(second overtone) 4
P
⇒λ5 =
4l line source, l =
5 2π / d
5λ 5 5v
⇒ f5 = ● Our perception of loudness is better corelated with the
4 4l
λ ⇒ f5 = 5 f1
second level measured in decibel (dB) and defined as
I
Note In open organ pipe, only odd harmonics are present nth harmonic follows β = 10 log 10   , where I0 = 10−12 Wm2 at 1 kHz.
λn =
4l  I0 
( 2n − 1 )

ν n = ( 2n − 1 ) ν 1

Doppler’s Effect in Sound


Open Organ Pipes When apparent change in the frequency of sound caused by
If both ends of a pipe are open and a system of air is directed the relative motion between source of sound and the
against an edge, standing longitudinal waves can be set up observer, it is called Doppler’s effect. Doppler’s effect is a
in the tube. The open end has a displacement antinode. wave phenomena, it holds not only for sound waves but
also for electromagnetic waves. So, change in frequency
Various modes of vibration of air column in an open organ
pipe are shown below can be analysed under three different situations
(i) Observer is stationary but the source is moving.
First harmonic λ1
⇒l = (ii) Observer is moving but the source is stationary.
(fundamental mode) 2
(iii) Both the observer and source are moving.
⇒ λ 1 = 2l
v
⇒ f1 =
2l Expression for Apparent Frequency
Second harmonic or l=λ2 ⇒l = λ2 Consider a source of sound and an observer are moving
first overtone
⇒ f2 =
2v with velocity vs and vo , respectively. The source is provided
2l a sound of frequency f and the frequency of sound heard by
N N A
⇒ f2 = 2 f1 the observer be f ′.
Third harmonic or ⇒l =
3λ 3 Velocity of sound w.r.t. source = v − vs
second overtone 2
v − vs
3v Apparent wavelength, λ ′ =
⇒ f3 = f
2l
⇒ f3 = 3 f1 Velocity of sound wave w.r.t. listener = v − vo
Wavelength of wave reacting by the listeners = λ ′
Hence, apparent frequency of sound wave heard by listener is
nth harmonic
fn = nf1 v − vo v − vo
λ n = 2l/n
f′ = = f
λ′ v − vs

@iitjeehelps
168 SELF STUDY GUIDE BITSAT

Change in frequency can be analysed under different


situations, which are given below
(i) If source is moving towards v − (–vo )  v + vo 
listener but listener is at rest, ∴ f′=  × f =  v − v  × f
 v − v s  s
then vs is positive and
vo = 0 i.e. f′> f
v (vi) If source and listener are moving away from each
∴ f′= × f , i.e. f ′ > f
v − vs other, then vs is negative and vo is positive.

(ii) If source is moving away


from listener but listener is
 v − vo   v − vo 
at rest, then vs is negative ∴ f′=  × f =  × f
and vo = 0 . v − ( −vs )  v + vs 
 v   v  i.e. f′< f
∴ f′=  × f =  × f
 v − ( −vs )  v + vs  (vii) If source and listener both are in motion in the same
direction and with same velocity, then
i.e. f′< f
vs = vo = v ′
(iii) If source is at rest and
listener is moving away vs = +v′ v = +v′
from source, then vs = 0 and S L
vo is positive. (v − v ′ )
∴ f′= × f
 v − vo  (v − v ′ )
∴ f′=  × f
 v  i.e. f′= f
i.e. f′< f (viii) If source and listener moves at right angles to the
(iv) If source is at rest and direction of wave propagation, then velocity of
listener is moving towards source and listener along the direction of wave
the source, then vs = 0 and propagation is vs cos90° = 0 and vo cos90° = 0
vo is negative. (v − 0)
∴ f′= × f or f ′ = f
v − ( −vo )  v + vo  (v − 0)
∴ f′=  × f =  v  × f
 v NOTE ● Velocity of source and observer measured in above
expressions are with respect to the medium in which the
i.e. f′> f sound wave travels.
(v) If source and listener are approaching each other, ● If the medium carrying the sound wave is itself moving
then vs is positive and vo is negative. then we have to consider speed of the wave relative to the
speed of the medium.

Practice Exercise
1. A physicist points out that glass is rarer than water. 4. If copper has modulus of rigidity 4 × 1010 N/m 2 and
a. This statement is correct in the case of sound Bulk modulus 12 . × 1011 N/m 2 and density 9 g/cm 3 ,
b. This statement is always wrong then find the velocity of longitudinal wave, when
c. This statement is correct in the case of light set-up in solid copper.
d. This statement is always correct a. 4389 m/s b. 5000 m/s
2. When height increases, velocity of sound decreases c. 4000 m/s d. 4300 m/s
a. this is due to decrease of pressure 5. A piezo electric quartz plate of Young’s modulus of
b. this is due to decrease in temperature . × 103 kg/m 3 is
elasticity 8 × 1010 N/m 3 and density 265
c. this is due to both decrease in temperature and vibrating in resonant condition. The fundamental
pressure frequency of vibrating is 550 kHz. What is thickness of
d. statement is wrong
the plate?
3. The velocity of sound is not affected by change in a. 0.05 cm b. 0.5 cm
a. temperature b. medium c. pressure d. wavelength c. 1.25 cm d. 0.55 cm

@iitjeehelps
SOUND WAVE 169
6. The value of adiabatic constant γ for oxygen and a. 3.63 × 10−5 m b. 3 × 10−5 m
nitrogen is same. The speed of sound in oxygen is c. 4.2 × 10−5 m d. 6.4 × 10−5 m
470 m/s at STP. The speed of sound in nitrogen at 16. When a wave is propagated from rarer medium to
STP is denser medium. Which of the following will remain
a. 340 m/s b. 580 m/s unchanged?
c. 502 m/s d. None of these a. Wave speed b. Propagation constant
7. At STP, the speed of sound in hydrogen is 1324 m/s c. Frequency d. None of these
then the speed of sound in air 17. A sonar system fixed in a submarine operates at a
a. 331 m/s b. 220 m/s c. 340 m/s d. 230 m/s frequency 40.0 kHz. An enemy submarine moves
towards the sonar with a speed of 360 km/h. What is
8. If the speed of sound is changed by 1 per cent, how the frequency of sound reflected by the submarine?
much must the temperature of air near 0°C be (Take the speed of sound in water to be 1450 m/s)
changed a. 52.3 kHz b. 45.93 kHz c. 62.49 Hz d. 54.34 kHz
a. 5°C b. 6°C c. 5.5°C d. 6.5°C
18. A small speaker has a capacity of power 3 W.
9. If the speed of sound wave in a stretched string is v A microphone is placed at distance 2 m from the
and Hooke’ law is obeyed, then the extenion in string speaker. Find the displacement amplitude of particles
is x. Find the extension in the string if the speed of of air near the microphone. The frequency of sound
sound wave will become 1.22 v . emitted by speaker is 1.0 kHz.
a. 1.5x b. 1x c. 0.5x d. 2x (Density of air = 1.2 kg/m 3 ) and speed of sound in
air = 330 m/s)
10. A boy watches a jet plane flying from north to south.
a. 2.76 × 10−4 cm b. 4 × 10−4 cm
When the jet is just seen above his head, the sound of
jet appears to reach him making some angle with c. 10 × 10−4 cm d. 3.8 × 10−3 cm
horizontal from north. If the velocity of sound is v and 19. From a height of 2 m, a drop of water of radius
velocity of jet is v /2, then find the angle. 2 × 10−3 m fall and produces a sound. The sound
a. 60° b. 30° c. 45° d. 15° produced can be heard upto a distance of 20 m. If the
gravitational energy is converted into sound energy in
11. A stone is dropped into a lake from a tower of 500 m
0.5 s, then calculate the intensity at a distance of
high. The sound of the splash will be heard by the
20 m.
man approximately after (Take g = 10 ms −2)
a. 2 × 10−7 W/m2 b. 2.6 × 10−6 W/m2
a. 11.5 s b. 21 s c. 10 s d. 14 s c. 2.6 × 10−7 W/m2 d. 3 × 10−7 W/m2
12. A light pointer fixed to one prong of a tuning fork
20. Velocity of sound wave in air is 330 m/s for a
touches a vertical plate. The fork is set vibrating at a
particular sound in air. A path difference of 40 cm
frequency of 56 Hz and allowed to free fall. Calculate is equivalent to a phase difference of 1.6 π. The
how many complete oscillation are counted when frequency of this wave is
plate falls at 10 cm.
a. 165 Hz b. 150 Hz c. 660 Hz d. 330 Hz
a. 10 b. 9 c. 8 d. 7
21. A tuning fork A of frequency 512 Hz produces 4 beats
13. The equation of a sound wave in air is per second when sounded with a tuning fork B. Due to
P = 0.01 cos (1000 t − 3x ) where, P , x and t are in SI. filing of the prongs of the tuning fork B, the number of
The bulk modulus of elasticity is 1.4 × 105 N/m 2. The the beats per second becomes 6. The actual
displacement amplitude is frequency of B is
a. 0.24 m b. 0.24 × 10−7 m a. 516 Hz b. 508 Hz
c. 512 Hz d. None of these
c. 8 × 10−7 m d. 10 m
22. A tuning fork A of frequency 260 c/s produces 4 beats
14. A sound wave of pressure amplitude 14 pascal per second with tuning fork B. When the tuning fork A
propagates through the air medium. The normal is loaded with wax, then the number of beats
pressure of air is 1.0 × 105 N/m 2. The difference produced per second becomes 3. Then, what is the
between maximum and minimum pressure in the frequency of tuning fork B?
medium is a. 264 b. 263 c. 256 d. 260
a. 5 × 105 N/m2 b. 10 × 105 N/m2 23. When temperature of air is 20°C, a tuning fork
c. 10 N/m2 d. None of these sounded over the open end of an air column produces
4 beats per second, the tuning fork given a lower note.
15. A sound wave having a frequency of 100 Hz and
If the frequency of tuning fork is 34 Hz, then find how
pressure amplitude of 10 Pa, then calculate the
many beats will be produced by the tuning fork if
displacement amplitude (Given speed of sound in air temperature falls to 5°C?
= 340 m/s and density of air = 1.29 kg/m 3 )
a. 2 beat/s b. 4 beat/s c. 1 beat/s d. 3 beat/s

@iitjeehelps
170 SELF STUDY GUIDE BITSAT

24. Two wires A and B are of same length, radius and 33. When two waves of almost equal frequencies n1 and
same material are in unison. If tension in A is n2 are produced simultaneously, then the time interval
increased by 4%, 4 beats are heard, then the between successive maxima is
frequency of the note produced when they were in 1 1 1 1 1 1
unison, will be a. b. − c. + d.
n1 − n 2 n1 n 2 n1 n 2 n1 + n 2
a. 50 Hz b. 100 Hz c. 150 Hz d. 200 Hz
34. A metal rod of length 1.5 m is clamped at the centre.
25. The velocity of sound in air at NTP is 331 m/s. Find its When it is set with longitudinal vibrations it emits a
velocity when temperature rises to 91°C and its note of 1000 Hz. Determine the Young’s modulus if
pressure is doubled. the density of material = 8 × 103 kg/m 3
a. 372 m/s b. 382.1 m/s c. 423 m/s d. 392.5 m/s
a. 7 × 1010 N/m2 b. 7.2 × 1010 N/m2
26. The fundamental frequency of a closed organ pipe is c. 0.7 × 1010 N/m2 d. 6.8 × 1010 N/m2
equal to second overtone of an open organ pipe. If the
length of closed organ pipe is 15 cm. The length of 35. A long tube of length l = 25 cm and diameter equal to
open organ pipe is 2 cm is taken and at its mouth air is blown as shown in
a. 90 cm b. 30 cm c. 15 cm d. 20 cm figure. The sound emitted by tube will have all the
frequencies of the group (velocity of sound = 330 m/s)
27. An organ pipe closed at one end resonates with a
tuning fork of frequencies 180 Hz and 300 Hz. It will Air
also resonate with tuning fork of frequencies
a. 360 Hz b. 420 Hz c. 480 Hz d. 540 Hz
28. Figures shows the vibrations of four air coloumns. The
ratio of frequencies np : nq : nr : ns is

p q a. 660, 1320, 2640 Hz b. 660, 1000, 3300 Hz


c. 302, 684, 1320 Hz d. 330, 990, 1690 Hz
36. In the above situation, bus is at rest blowing horn of
r frequency f0. A boy is at rest at some distance. What
will be apparent frequency of sound, if the air start
moving with the speed of 20 m/s from bus towards
boy?
s
f0
a. 12 : 6 : 3 : 4 b. 1 : 2 : 4 : 3
c. 4 : 2 : 3 : 1 d. 4 : 3 : 2 : 1 vw = 20 m/s
29. A steel rod 100 cm long is clamped at its middle. The Bus at rest Boy at rest
fundamental frequency of longitudinal vibrations of the
rod are given to be 2.53 kHz. What is the speed of a. < f0 b. > f0 c. = f0 d. None of these
sound in steel?
37. The first overtone of an open organ pipe beats with
a. 6.2 km/s b. 5.06 km/s c. 7.23 km/s d. 7.45 km/s the first overtone of a closed organ pipe with a beat
30. Two organ pipes are emitting their fundamental notes, frequency of 2.2 Hz. The fundamental frequency of
when each closed at end, give 5 beat/s. If their closed organ pipe is 110 Hz. Find length of the open
fundamental frequencies are 250 Hz and 255 Hz, then pipe. (Given, sound in air = 330 m/s)
find the ratio of their lengths. a. 2.43 m b. 0.73 m c. 0.993 m d. 2.93 m
49 49 50 51 38. The apparent frequency is f1 when a source of sound
a. b. c. d.
50 51 51 50 approches a stationary observer with a speed u and f2
31. In the case of vibration of closed end organ pipe in when the observer approches the stationary source
fundamental mode of vibration, the pressure is with same speed. If v is the velocity of sound, then
maximum at a. f1 = f2 b. f1 > f2 if u < v
a. open end b. closed end c. f2 > f1 if u < v d. f2 > f1 if u < v
c. at middle d. None of these
39. A locomotive engine approaches a railway station and
32. An air column in a pipe which is closed at one end will whistles at a frequency of 400 Hz. A stationary
be in resonance with a vibrating tuning fork of observer on the platform observes a changes of 40 Hz
frequency 264 Hz if the length of the air column in cm as the engine passes across him. If the velocity of
is (Speed of sound in air = 340 m/s) sound is 330 m/s. The speed of the engine is
a. 32.19 cm b. 64.39 cm c. 100 cm d. 140 cm a. 33 m/s b. 18 m/s c. 16.5 m/s d. 24 m/s

@iitjeehelps
SOUND WAVE 171
40. Two trains, one moving at a speed of 30 mile/h and v + vm  v + vm
a.  f b. f
other at 60 mile/h, approaching each other. When a  v + vb  v − vb
faster train blows a whistle, the apparent frequency of
the note heard by an observer at rest behind the faster 2vb (v + vm ) 2vm (v + vb )
c. f d. f
train is 1852 Hz. Calculate the frequency of note (v 2 − vb2 ) (v 2 − vm2 )
produced by faster train (Assume speed of sound to
be 1100 ft/s) 42. There is a road between two parallel rows of building
a. 2000 vib/s b. 1500 vib/s c. 1000 vib/s d. 2500 vib/s and distance between the rows of building is 106 m.
Find the velocity of car if a car blows a horn whose
41. A boy with a radio, playing a music at a frequency ‘f ’ echo is heard by the driver after 1 s.
is moving towards a wall with velocity v b . A motorist is
(Given, speed of sound = 340 m/s)
following the boy with a speed v m . Find the expression
for the beat frequency heard by the motorist., if the a. 180 m/s b. 165 m/s
speed of sound is v c. 323 m/s d. 150 m/s

BITSAT Archives
1. A sound source is moving towards stationary listener and minima. If the speed of sound is 330 ms −1, then
1 the frequency at which the first maximum is observed
with th of the speed of sound. The ratio of apparent
10 is [2010]
to real frequency is [2012] L1
2 40 m
9 10
a.   b.
 10 9
2
11  11 9m
c. d.  
10  10

2. The produced rays in sonography are [2012]


L2
a. microwaves
b. infrared waves a. 165 Hz b. 330 Hz c. 495 Hz d. 660 Hz
c. sound waves 6. When a sound wave of wavelength λ is propagating in
d. ultra sound a medium, the maximum velocity of the particle is
3. The source of sound generating a frequency of 3 kHz equal to the wave velocity. The amplitude of wave is
reaches an observer with a speed of 0.5 times the λ λ λ
a. λ b. c. d. [2008]
velocity of sound in air. The frequency heard by the 2 2λ 4λ
observer is [2011] 7. The disc of a siren containing 60 holes rotates at a
a. 1 kHz b. 2 kHz constant speed of 360 rpm. The emitted sound is in
c. 4 kHz d. 6 kHz unison with a tuning fork of frequency [2005]
4. If v m is the speed of sound in moist air and v d is the a. 10 Hz b. 360 Hz c. 216 Hz d. 60 Hz
speed of sound in dry air under identical conditions of 8. The ratio of velocity of sound in hydrogen and oxygen
pressure and temperature, then [2011] at STP is [2005]
a. vm > vd a. 16 : 1 b. 8 : 1 c. 4 : 1 d. 2 : 1
b. vm < vd
9. In an experiment with sonometer a tuning fork of
c. vm = vd
frequency 256 Hz resonates with a length of 25 cm
d. v m ⋅v d = 1 and another tuning fork resonates with a length of
5. Two loudspeakers L1 and L2, driven by a common 16 cm. Tension of the string remaining constant the
oscillator and amplifier, are arranged as shown. The frequency of the second tuning fork is [2005]
frequency of the oscillator is gradually increased from a. 163.84 Hz b. 400 Hz
zero and the detector at D records a series of maxima c. 320 Hz d. 204.8 Hz

@iitjeehelps
Answer with Solutions
Practice Exercise 12. (c) Let n = frequency
and N = number of oscillations
1. (a) 2. (b) 3. (c) 4. (a)
Time taken to full freely to a distance 10 cm.
Y   8 × 1010 
5. (b) Here, c =   =   1 2 × 10
 ρ  2.65 × 103  s = gt 2 ⇒ t =
2 g
 80 × 106  The frequency of oscillation during this time
=   = 5494.4 m/s
 2.65  N 2 × 10
n = ⇒ N = nt ⇒ N = 56 ×
c 5494.4 t 980
∴ λ= = N = 7.999 = 8
f 550 × 103
∴ For fundamental mode of vibration, thickness 13. (b) The standard equation is
λ p = p 0 cos (ωt − x ) …(i)
t = = 0.4995 = 0.5 cm
2 The given equation is
 γRT  P = 0.01cos (1000 t − 3x ) …(ii)
6. (c) c =  
 M  Comparing the Eq. (i) and Eq. (ii), we get
M  c nitrogen p 0 = 0.01N/m2
c2  32  8
∴ =  1  or =   =   ω = 1000 rad/s
c1  M2 c oxygen  28  7
k = 3 per metre
 8  8
Q c nitrogen =   c oxygen =   × 470 But p 0 = BAk
 7  7
p
= 502.45 m/s ≈ 502 m/s ∴ A= 0 [B = Bulk modulus of elasticity]
Bk
7. (a) Molecular weight of hydrogen = 2 0.01
= = 0.2381 × 10−7 m ≈ 0.24 × 10−7 m
Molecular weight of oxygen = 32 . × 105 × 3
14
γ P  γ HPH 14. (c) The minimum pressure is p min = (105 − 5) N/m2
Both are diatomicv air =  air air  , v H =
 ρ air  ρH The maximum pressure is p max = (105 + 5) N/m2
ρ air M air 32 16 ∴ ∆π = p max − p min = 10 N/m2
= = =
ρH MH 2 1 15. (a) 16. (c)
v air  γ Pρ H  v air ρ  17. (b) Sonar frequency,
=  air 
 ⇒ =  H  νs = 40 kHz = 40 × 10 3 Hz
vH  γ H Pρ air  1324  ρ air 
Speed of enemy submarine,
1324
v air = = 331m/s 5  5 
4 vs = 360 km/h = 360 × m/s =100 m/s Q 1km/h = m/s
18  18 
8. (c)
Speed of sound in water = 1450 m/s
9. (a)T ∝ x
Apparent frequency received by the energy submarine,
So, T ′ ∝ x′ v + v 0   1450 + 100
ν′ =   ν=  × 40 × 10
3
T  T ′   v   1450 
v =   ; v′ =  
 m m
= 42.76 × 10 3 Hz
v′ T ′   x ′  122
. Now, the reflected waves received by sonar having a
=   ⇒   = ⇒ x ′ ≅ 1.5x
v  T   x  v different frequency
 v 
10. (a) Distance covered by sound to reach the boy ν′′ =   ν′
Distance covered by jet v − vs 
=
cos θ Here, vs = 100 m/s is velocity of enemy submarine.
Velocity of jet = (velocity of sound) × cos θ  1450 
∴ ν′′ =   × 42.76 × 10
3
v  1450 − 100
⇒ = v cos θ ⇒ θ = 60°
2 = 45.93 × 103 Hz
11. (a) = 45.93 kHz

@iitjeehelps
SOUND WAVE 173
P 3 3 v 3v l
18. (a)QI = = = ∴ f1 = f2 ∴ = ⇒ 2 =6
4πr 2 4π × 22 16π 4l1 2l2 l1
I ∴ l2 = 6l1 = 6 × 15 = 90 cm
But I = 2π 2A 2v 2ρ 0c or A 2 =
2π 2v 2ρ 0c 27. (b) Resonate frequencies are n1f and n 2f , where n1 and n 2
 I   3  are integers and f is fundamental frequency.
∴ A=  2 2  =  
 2π v ρ 0c   16π × 2π × 10 × 12
2 6
. × 330 Here, n1f = 180 and n 2f = 300
n1 180 18 3
= 0.002764 × 10−3 m = 2.76 × 10−4cm ∴ = = = ∴ n1 = 3 and n 2 = 5
n 2 300 30 5
19. (c) Change in gravitational energy = mgh 180
∴ 3f = 180 Q f = = 60 Hz
4  3
=  πr 3 ρgh joule
3  Possible harmonics in the case of closed end organ pipe is
4 3 1, 3, 5, 7, 9, 11, 13, ....
πr ρgh
Hence, power = 3 watt The corresponding frequencies are 60, 180, 300, 420, ...
t 28. (b)
4 3
πr ρgh 29. (b) In fundamental mode,
and intensity = 3 = 2.6 × 10−7 W/m2  λ λ
( 4πR 2 )t l = 2  = ⇒ λ = 2l
 4 4

20. (c) As, phase difference = × Path difference Given, l = 100 cm, ν = 2.53 kHz N
λ
2π Using v = νλ A A
⇒ 1.6 π = × 40 ⇒ λ = 50 cm = 0.5 m
λ ⇒ v = 2.53 × 103 × 2 × 1000 × 10−2 l/4 l/4
v 330 = 5.06 × 103 m/s = 5.06 km/s l
Now, as v = λf ⇒ f = = = 660 Hz
λ 0.5
30. (c) 31. (b)
21. (a) Due to filing, frequency of tuning fork B increases.
32. (a) For fundamental mode of vibration,
Since, after filing beats frequency increases.
λ
Q fB > fA Q fB − fA = 4 Q = l Q λ = 4l
4
∴ fB = 4 + fA = 4 + 512 = 516 Hz
Q v = fλ or 340 = 264 × 4l
22. (c) When tuning fork A is loaded with wax, then its
Q l = 32.19 cm
frequency decreases
∴ fA − fB = 4 ∴ 260 − fB = 4 33. (a) Number of beats per second = n1 − n 2
∴ fB = 256 Hz 1
∴Time interval between two successive maxima =
n1 − n 2
23. (d) Let beat produced at 5°C = b
Frequency of air column at 20°C = 34 + 4 34. (b)
Frequency of air column at 5°C = 34 + b 35. (a) n =
v
=
330
× 100 = 660 Hz
Ratio of velocity of sound at temperature 20°C and 5°C 2l 2 × 25
n1 = 2 × 660 = 1320 Hz
v 20°  273 + 20 34 + 4  293
=   ⇒ =   n 2 = 3 × 660 = 2460 Hz
v 5°  273 + 5  34 + b  278
36. (c)
b = 3 beat/s
37. (c) Fundamental frequency of closed organ pipe,
2
104  n + 4 104 n + 4 v v
24. (d) =  or = ⇒ n = 200 Hz νc = ⇒ Lc = = 0.75 m
100  n  100 n 4Lc 4 × 110
25. (b) Given velocity of sound in air v 0 = 331 m/s Frequency of first overtone of open organ pipe,
t = 91° C v v
ν1 = 2 × =
273 + t 273 + 91 91 2L0 L0
vt = v 0 = 331 = 331 1 +
273 273 273 Frequency of first overtone of closed organ pipe, ν 2 = 3 νc
v 3 × 330
1
vt = 331 1 + = 331 ×
2 =3× = = 330 Hz
3 3 4Lc 4 × 0.75
v
= 382.1m/s ∴ ν1 − ν 2 = 2.2; − 330 = 2.2
v L0
26. (a) For closed organ pipe f1 = [for fundamental] v
4l1 ⇒ = 332.2
L0
3v
For open organ pipe, f2 = [second overtone] 330
2l2 ⇒ L0 = = 0.993 m
332.2

@iitjeehelps
174 SELF STUDY GUIDE BITSAT

38. (d) When source approaches the observer,


 
 v   v   1  vm vb
f1 =  f =  f =  f
v − vs  v − u   1 − u  
  v  
When observer approaches towards the source ⇒ Sound reflected from wall
v + v 0  v + u   u v + vm 
f2 =  f =   f = 1 −  f v + vm 
 v   v   v f ′′′ =   ×f″ =  f
 v   v + vb 
∴ f2 > f1
Beat frequency f ′′′ − f ′
 v   v 
39. (c) Here, n′ =   n and n ″ =  n v + vm  v + vm  2v (v + vm )
v − vs  v + vs  = f −  f = b2 f
 v − vb   v + vb  (v − vb2 )
According to problem,
 v   v  42. (c) Signal goes from A to B. Let the velocity of car = v.
40 = n′ − n ″ =  n− n
v − vs  v + vs  B
 330 × 400  330 × 400 x
or 40 =   −  53
 330 − vs   330 + vs  170 170
Building v
After solving, vs = 16.5 m/s O 170
v 2
40. (a)Q 60 mile/h = 88 ft/s x 2
 v   1100 
n′ =   n′ or   × n = 1852 A
v + vs   1100 + 88 v
1852 × 1188 Distance travelled AB = v , OA =
n= = 2000.16 = 2000 vib/s 2
1100 2x
= 340 ⇒ x = 170 m
41. (c) Motorist will receive two sounds. 1
v + vm  v2
For direct sound waves, f ′ =  f (170)2 = (53)2 +
 v + vb  4
 v  4( 28900 − 2809 ) = v ⇒ v = 323 m/s
For reflected sound waves f ″ =   ×f
v − vb 

BITSAT Archives
1. (b) Given, ns =
n 5. (b) Let ∆x = path difference
10 ⇒ ∆x = L2D − L1D ⇒ ∆x = 402 + 9 2 − 40
 v  ⇒ ∆x = 41 − 40 ⇒ ∆x = 1m
Apparent frequency n′ = n  
v − vs  λ
For first maximum, ∆x = ( 2n ) (where n = 1)
where, n = real frequency of source 2
λ v
v = velocity of sound, vs = velocity of source ⇒ 1 = 2 (1) ⇒ λ = 1m ⇒ f = = 330 Hz
2 λ
n′ v 10 λ
So, = = 6. (c)v max = v ⇒ Aω = v ⇒ A × 2πv = vλ or A =
n v − v 9 2π
10
360
2. (d) The produced rays in sonography are ultrasound. 7. (b) Speed = 360 rev/min = rev/s = 6
60
 v  ∴ Frequency = 6 × 60 = 360
3. (d) Apparent frequency heard will be n′ = n  
v − vs  γ RT
8. (c) Velocity of sound,v =
v = velocity of sound, vs = velocity of source of sound M
n = frequency = 3 kHz vH
=
MO
=
16
= 4 :1
v v vO MH 1
∴ n′ = 3 × =3× = 6 kHz
v − 0.5v 0.5v 1
9. (b) For sonometer, n ∝
4. (a) Under identical pressure and temperature condition, l
speed of sound in moist air is more than that in dry air, n1 l2 256 16 256 × 25
∴ = ⇒ = ⇒ n2 = = 400 Hz
i.e. vm > vd n 2 l1 n2 25 16

@iitjeehelps
15
Heat, Temperature
and Calorimetry

Heat
Heat is the energy, which is transferred between system and surroundings due to the temperature
difference. In other words, heat is a form of energy that flows from one body to another because of
temperature difference between them.
The conventional unit of heat is called the calorie. It is defined as the amount of heat required to raise
the temperature of 1 g of water through 1°C. 1 cal = 4.186 J.

Temperature
Temperature is the measure of degree of hotness or coldness of a body. So, a body at higher
temperature would be hotter relative to a body at lower temperature.
Whenever, there is a difference of temperature between a body and its surroundings, then exchange of
energy takes place. This energy is called heat energy. So, heat is a form of energy transferred between
two (or more) systems or a system and its surroundings by virtue of its temperature difference.

Measurement of Temperature
Temperature is measured by using a thermometer. Various temperature scales such as celsius scale,
fahrenheit scale and kelvin scale is used for measuring the temperature.
Properties of matter such as volume expansion or contraction with the change of temperature is the
generally used principle of thermometers. In commonly used thermometers (liquid in glass), mercury
or alcohol are the generally used liquids. Whereas, the Kelvin scale was derived by using the gas
thermometers which is based on ideal gas equation.
The minimum temperature for an ideal gas at which its pressure is reduced to zero is called absolute
zero (0 K ), which is found to be equal to −273.15° C.

@iitjeehelps
176 SELF STUDY GUIDE BITSAT

Relation between the three mentioned temperature scales is Different Thermometers


given below
T − 273.15 tC − 0 t f − 32 Constant Volume Gas Thermometer
= = If p0 , p100 , p tr and pt are the pressures of gas at
100 100 180
temperatures 0°C, 100°C, triple point of water and unknown
where, T = Absolute temperature (in kelvin),
temperature (t°C) respectively keeping the volume
tC = Celsius temperature
constant, then
t F = Fahrenheit temperature
 p − p0   p
Since, at melting point of water tC = 0°C t= × 100 ° C or T = 273.16 K
 100
p − p0   p tr 
So, T melting = 273.15 K and t F , melting = 32 ° F
And at boiling point of water tC = 100° C Platinum Resistance Thermometer
So, Tboiling = 373.15 K and t F ,boiling = 212 ° F If R0 , R100 , Rtr and Rt are the resistances of a platinum wire
at temperatures 0°C, 100°C, triple point of water and
unknown temperature (t°C) respectively, then
Thermometry  R − T0 
t= t × 100 °C
The branch dealing with measurement of temperature is  R100 − R0 
called thermometry and the devices used to measure
R  R 
temperature are called thermometers. or T =  T × T tr  K =  t × 273.16 K
 Rtr   Rtr 
At changing temperature, change in pressure of a gas at
constant volume, change in electric resistance of a metallic Mercury Thermometer
wire etc. Such property of a substance is called
In this thermometer, the length of a mercury column from
thermometric property. Let thermometric properties at
some fixed point is taken as thermometric property. Thus,
temperatures 0°C (ice point), 100°C (steam point) and t°C
 l −l  l 
(unknown temperature) are X0 , X 100 , and Xt respectively. t =  t 0  × 100° C or T =  t × 273.16 K
Then,  l100 − l0   ltr 
Xt − X0 X 100 − X0
=
t 100
Xt − X0 t Thermal Expansion
or ` =
X 100 − X0 100 A change in temperature of a body causes change in its
dimensions. When the body’s temperature is increased,
 X − X0 
Thus, t= t  × 100° C body expands in dimensions. It is called thermal
 X 100 − X0  expansion.

Triple Point Linear Expansion


Triple point is a state in which ice, water and water vapour (Expansion in Length of a Solid)
can stay together in equilibrium. It refers to temperature at
Consider a rod of length l1 at a temperature θ 1 . Let it be
the equilibrium. heated to a temperature θ 2 and the increased length of the
∴ The temperature scale by the equation rod be l2 , then
p l2 = l1 (1 + α∆θ )
T = lim × 273.16 K
p tr → 0 p
tr where, α = coefficient of linear expansion and θ = θ 2 − θ 1
where, p = pressure
ptr = pressure at equilibrium and K stands for Kelvin Areal Expansion
scale of temperature. (Expansion in Surface Area)
If values of thermometric properties at 0 K, 273.16 K and T K If A1 is the area of solid atθ 1 °C and A2 is the area atθ 2 °C, then
are 0, X tr and X respectively, then A 2 = A 1 (1 + β∆θ )
T X X  X  where, β = coefficient of areal (superficial) expansion and
= or T = T tr =  × 273.16 K
T tr X tr X tr  X tr  ∆θ = θ 2 − θ 1

@iitjeehelps
HEAT, TEMPERATURE AND CALORIMETRY 177
∆V ∆T
Volume Expansion So, =
V T
(Expansion in Volume) 1  ∆V  1
or γ= ⋅  =
If V 1 is the volume of solid at θ 1 °C and V 2 is the volume at ∆T  V  T
θ 2 °C, then
V 2 = V 1 (1 + γ ∆θ )
where, γ = coefficient of cubical (volume) expansion and
Specific Heat Capacity
∆θ = θ 2 − ∆θ 1 The specific heat of a substance is the quantity of heat in
calorie required to raise the temperature of 1 g of that
substance by 1°C. Its unit is cal g −1 C −1 .
Apparent Expansion of Liquid
The quantity of heat ∆Q required to change
If in a beaker (container) a liquid is fully filled and if the
temperature of the system increases, then because of the fact the temperature of a body of mass m by ∆T ,
that γ liquid > γ solid , the expansion in liquid is more than the is approximately proportional to the product of m and ∆T
expansion in solid and thus the liquid overflows from the i.e. ∆Q ∝ m∆T or ∆Q = ms ∆T , where s is the specific heat
container. This is termed as apparent expansion of liquid. capacity of the material. The product of mass of the body
and specific heat capacity is termed as heat capacity.
Consider a vessel of volume V0 fully filled with a liquid of
∆Q
coefficient of cubical expansion γ l . If temperature of the s= i.e. heat capacity is defined as the amount of heat
system is increased by ∆T , then ∆T
required to raise the temperature of a body by 1°C.
∆Vl = V0 γ l ∆T , ∆V c = V0 γ c ∆T , where subscript c denotes
the container.
Principle of Calorimetry
The volume of overflowing liquid is
Let two bodies of masses m1 and m2 , specific heats s1 and s2
∆V = V0 ( γ l − γ c ) ∆T = V0 γ l c ∆T
and at temperatures θ 1 and θ 2 are brought in contact with
where, γ lc = γ l − γ c is termed as the apparent coefficient of each other. Assuming θ 1 > θ 2 , heat will flow from body 1 to
cubical expansion. body 2. If θ is the common temperature of the two bodies at
Dulong and Petit’s method In this method, a column of the state of thermal equilibrium, then (assuming no heat is
experimental liquid at t °C is balanced against other gained or lost from or to the surroundings)
column of the experimental liquid at 0 °C is by taking them Heat lost by body 1 = Heat gained by body 2
in U-tube.
ht − h0 m1s1 (θ 1 − θ ) = m2s2 (θ − θ 2 ) Q (θ 2 < θ < θ 1 )
Here, γ=
h0t
Latent Heat
Anomalous Behaviour of Water The heat required to change the state of a system is
Water exhibits an anomalous behaviour. It contracts on proportional to the mass of the system, i.e.
heating between 0°C and 4°C. But it shows normal Q ∝ m ⇒ Q = mL, where L is the latent heat.
behaviour above 4°C i.e. it expands above 4°C. For this
reason, water has maximum density at 4°C (see graph).
Water Equivalent of a Substance
Water equivalent of certain amount of substance is defined
Volume as the amount of water, which when replaced by the
substance requires the same amount of heat for the same
rise in temperature.
O 4°C mS
Temperature mw = ,
Sw
Since, pV = nRT where, mw = water equivalent of substance whose mass is m,
So, at constant pressure, S = specific heat capacity of substance and
p ⋅ ∆V = nR ⋅ ∆T S w = specific heat capacity of water.

@iitjeehelps
178 SELF STUDY GUIDE BITSAT

Heating Curve dp Jl
=
A plot of temperature versus heat energy for 1 kg of water is dt T (V 2 − V 1 )
shown below
Hygrometry It is a branch in which we study and measure
of water vapours in atmosphere.
Temperature (ºC)

Boiling point Phase change Steam m


100 RH = × 100
M
Melting point
0
Phase change Here, m = mass of water, M = mass of water vapour to
(Ice) Water
saturate the same volume of air.

Latent heat of Latent heat of Heat Let p and P are the actual and maximum vapour pressures
fusion vaporisation energy respectively at the temperature, then

Debyer T 3 law: Specific heat of a solid varies with  Maximum vapour pressure at dew point 
RH =   × 100%
temperature. It is 3R at higher temperature and near  Maximum vapour pressure at t ° C 
absolute zero CV ∝ T 3 . Clausius Clapeyron’s equation (or p
latent heat equation) representing change of MP or BP with = × 100%
P
pressure is

Practice Exercise
1. The temperature at which Centrigrade thermometer 5. The graph between two temperature scales A and B
and Kelvin thermometer gives the same reading, is is shown in figure given below. Between upper fixed
a. 4° b. 273° point and lower fixed point, there are 150 equal
c. not possible d. 0 divisions on scale A and 100 on scale B. The
relationship for conversion between two scales is
2. What is the change in the temperature on Fahrenheit given by
scale and on Kelvin scale, if a iron piece is heated
from 30 to 90°C? 180
Temperature (°A)

a. 108°F, 60 K b. 100°F, 55 K
c. 100°F, 65 K d. 60°F, 108 K 150
3. At 30°C, the hole in a steel plate has diameter of
0.99970 cm. A cylinder of diameter exactly 1 cm at 30
30°C is to be slide into the hole. To what temperature
the plate must be heated? 0 Temperature (°B) 100
. × 10−5 ° C−1)
(Given α steel = 11
t A − 180 t t A − 30 t
a. 58°C b. 55°C c. 57.3°C d. 60°C a. = B b. = B
100 150 150 100
4. A monoatomic ideal gas, initially at temperature T1, is t − 180 t t − 40 t
c. B = A d. B = A
enclosed in a cylinder fitted with a frictionless piston. 150 100 100 180
The gas is allowed to expand adiabatically to a 6. Heat given to a system can be associated with
temperature T2 by releasing the piston suddenly. If
a. kinetic energy of random motion of molecules
L1 and L 2 be the lengths of the gas column before and
b. kinetic energy of orderly motion of molecules
T
after expansion respectively, then 1 is given by c. total kinetic energy of random and orderly motion of
T2 molecules
2/ 3 d. kinetic energy of random motion in some cases and
 L1   L1 
a.   b.   kinetic energy of orderly motion in other
 L2   L2 
2/ 3
7. At what temperature (in °C), the fahrenheit and celsius
 L2   L2  scale gives same reading?
c.   d.  
 L1   L1  a. 40 b. − 40 c. 8 d. − 8

@iitjeehelps
HEAT, TEMPERATURE AND CALORIMETRY 179
8. Which one of the following would raise the [Given, Y1 and Y2 are Young’s modulus of materials of
temperature of 40 g of water at 20°C most when the rods, α1 and α 2 are coefficient at linear expansion.]
mixed with?
a. α 1Y2 = α 2Y1 b. α 1Y1 = α 2Y2
a. 20 g of water at 40°C b. 30 g of water at 30°C
c. 10 g of water at 60°C d. 4 g of water at 100°C c. α 1 = α 2 d. Y1 = Y2

9. If same amount of heat is supplied to two identical 15. What will be the stress at − 20°C, if a steel rod with a
spheres (one is hollow and other is solid), then cross-sectional area of 150 mm 2 is stretched between
a. the expansion in hollow is greater than the solid two fixed points? The tensile load at 20°C is 5000 N
b. the expansion in hollow is same as that in solid (Assume, α = 117. × 10−6 /°C and Y = 200 × 1011 N/m 2)
c. the expansion in hollow is lesser than the solid
a. 12.7 × 106 N/m2 . × 106 N/m2
b. 127
d. the temperature of both must be same to each other
c. 127 × 106 N/m2 d. 0.127 × 106 N/m2
10. At 30°C, a lead bullet of 50 g, is fired vertically
upwards with a speed of 840 m/s. The specific heat of 16. In an anisotropic medium, the coefficients of linear
lead is 0.02 cal/g°C. On returning to the starting level, expansion of a solid are α1, α 2 and α 3 in three
it strikes to a cake of ice at 0°C. Calculate the amount mutually perpendicular directions. The coefficient of
of ice melted (Assume all the energy is spent in volume expansion for the solid is
melting only) α1 + α 2 + α 3
a. α 1 − α 2 + α 3 b.
a. 62.7 g b. 55 g 3
c. 52.875 kg d. 52.875 g c. α 1 + α 2 + α 3 d. None of these
11. A second’s pendulum clock having steel wire is 17. A liquid when heated in a copper vessel and when
calibrated at 20°C. When temperature is increased to heated in a silver vessel, the apparent coefficient of
30°C, then calculate how much time does the clock expansion is C and S, respectively. If coefficients of
lose or gain in one week? linear expansion of copper is A, then coefficient of
. × 10−5 (°C−1)]
[α steel = 12 linear expansion of silver is
C + S − 3A C + 3A − S
a. 0.3628 s b. 3.626 s a. b.
c. 362.8 s d. 36.23 s 3 3
3A − S − C C + S + 3A
c. d.
12. A plate composed of welded sheets of aluminium and 3 3
iron is connected to an Fe 18. Using the following data, calculate at what
electrical circuit as shown Al
temperature will the wood just sink in benzene
in figure. What will happen
if a fairly strong current be Density of wood at 0°C = 8.8 × 102 kg/m 3
passed through the circuit? Density of benzene at 0°C = 9 × 102 kg/m 3
a. Strip bends upward Cubical expansivity of wood = 1.5 × 10−4 K −1
b. Strip bends downward
c. Strip remains in its initial condition Cubical expansivity of benzene = 1.2 × 10−3 K −1
d. None of the above a. 27°C b. 21.7°C c. 31°C d. 31.7°C
13. A copper rod of length l 0 at 0°C is placed on smooth 19. The radius of metal sphere at room temperature T is
surface. Now, the rod is heated upto 100°C. Find the R and the coefficient of linear expansion of the metal
longitudinal strain developed. is α . The sphere is heated a little by a temperature T,
so that new temperature is T + ∆T . The increase in
(α = coefficient of linear expansion) volume of sphere is approximately
100l0α a. 2πR α∆T b. π R 2α ∆T
a. b. 100 α
l0 + 100l0α c. 4πR 3α∆T / 3 d. 4πR 3α∆T
c. zero d. None of these
20. A copper rod and steel rod having length Lc and Ls
14. Two metal rods are fixed end to end between two rigid
supports, as shown in respectively at certain temperature. It is observed that
figure. Each rod is of difference between their length remains constant at all
length ‘l’ and area of
l l temperatures. If αc and αs are their respective
cross-section is A. When coefficient of linear expansions. Then, ratio of Ls is
α1y1 α2y2
the system is heated up, Lc
determine the condition αc αs  α   α 
when the junction a. b. c. 1 + s  d. 1 + c 
αs αc  αc   αs 
between rods does not shift.

@iitjeehelps
180 SELF STUDY GUIDE BITSAT

21. One mole of an ideal monoatomic gas is heated at a 27. The molar heat capacity of rock salt at low
constant pressure of 1 atmosphere from 0°C to temperatures varies with temperature according to
100°C. Work done by the gas is Debye’s T 3 law.
a. 8.31 × 103 J b. 8.31 × 10−3 J T3
Thus, C = k 3 , where k = 1940 Jmol−1K −1, θ = 281K
c. 8.31 × 10−2 J d. 8.31 × 102 J θ
Calculate how much heat is required to raise the

TemperatureºC
22. The following figure represents
the temperature versus time plot f temperature of 2 moles of rock salt from 10 K to 50 K?
for a given amount of a d a. 800 J b. 373 J
substance when heat energy is b e c. 273 J d. None of these
c
supplied to it at a fixed rate and a
28. Equal masses of two liquids A and B contained in
O Time
at a constant pressure. Which vessels of negligible heat capacity are supplied heat at
part of plot represent a phase the same rate. The temperature-time graphs for the
change? two liquids are shown in the figure. If S represents
a. a to b and e to f specific heat and L represents latent heat of liquid, then
b. b to c and c to d Y
c. a to e and e to f

Temperature
B A
d. b to c and d to e
23. A sphere A is placed at smooth table. An another
sphere B is suspended as shown in figure. Both
spheres are identical in all respects. Equal quantity of
heat is supplied to both spheres. All kinds of heat loss
O X
are neglected. The final temperatures of A and B are Time
TA and TB respectively, then a. S A > SB , LA < LB b. S A > SB , LA > LB
c. S A < SB , LA < LB d. S A < SB , LA > LB
A 29. The specific heat of a substance at temperature t °C is
B s = at 2 + bt + c . Calculate the amount of heat required
to raise the temperature of m g of the substance from
0°C to t 0°C
a. TA = TB b. TA > TB mt 03a bt 02 mt 03a mbt 02
c. TA < TB d. None of these a. + + ct 0 b. + + mct 0
3 2 3 2
mt a mbt 02
3
24. Calculate the resulting temperature when 20 g of c. 0 + d. None of these
boiling water is poured into an ice-cold brass vessel 3 2
(specific heat = 01
. cal/g°C) of mass 100 g. 30. In Q. No. 29 find the average value of specific heat.
a. 66.66°C b. 6.66°C at 03 bt 02 at 0 bt 02
c. 0.66°C d. 50°C a. + + ct 0 b. + +c
3 2 3 2
25. In similar calorimeters, equal volumes of water and c. at 0 + bt 0 + c
2
d. zero
alcohol, when poured, take 100 s and 74 s
31. A drilling machine of 10 kW power is used to drill a
respectively to cool from 50°C to 40°C. If the thermal
bore in a small aluminium block of mass 8 kg. If 50%
capacity of each calorimeter is numerically equal to
of power is used up in heating the machine itself or
volume of either liquid, then calculate the specific heat
lost to the surroundings then how much is the rise in
capacity of alcohol.
temperature of the block in 2.5 min?
(Given, the relative density of alcohol as 0.8 and (Given, specific heat of aluminium = 091
. J/g°C)
specific heat capacity of water as 1 cal/g/°C)
a. 103°C b. 130°C c. 105°C d. 30°C
a. 0.8 cal/g°C
b. 0.6 cal/g°C 32. A thermally insulated piece of metal is heated under
c. 0.9 cal/g°C atmosphere by an electric current so that it receives
d. 1 cal/g°C electric energy at a constant power P . This leads to an
increase of the absolute temperature T of the metal
26. The ratio of thermal capacities of two spheres A and
with time t as follows T = a 1/ 4
B, if their diameters are in the ratio 1 : 2, densities in
the ratio 2 : 1, and the specific heat in the ratio of 1 : 3, Then, the heat capacity C p is
will be 4PT 3 4PT 2
a. b.
a. 1 : 6 b. 1 : 12 a4 a3
c. 1 : 3 d. 1 : 4 c. 4PT 2 d. None of these

@iitjeehelps
HEAT, TEMPERATURE AND CALORIMETRY 181
33. If in 1.1 kg of water which is contained in a calorimeter a. 1/15 b. 1/17
of water equivalent 0.02 kg at 15°C, steam at 100°C is c. 2/15 d. 1/8
passed, till the temperature of calorimeter and its 36. It takes 20 minutes to melt 10 g of ice, when ray from
contents rises to 80°C. The mass of steam condensed the sun are focussed by a lens of diameter 5 cm on to
in kilogram is a block of ice. Calculate the heat received from the
a. 0.131 b. 0.065 c. 0.260 d. 0.135 sun on 1 cm 2 per minute.
34. 5 g of water at 30°C and 5 g of ice at − 20°C are (Given, L = 80 kcal/kg)
mixed together in a calorimeter. The water equivalent a. R = 2.04 cal/cm2-min b. R = 3.04 cal/cm2-min
of calorimeter is negligible and specific heat and latent
c. R = 0.204 cal/cm2-min d. R = 204 cal/cm2-min
heat of ice are 0.5 cal/g°C and 80 cal/g respectively.
The final temperature of the mixture is 37. In an energy recycling process, X g of steam at 100°C
a. 0°C b. − 8°C c. − 4°C d. 2°C becomes water at 100°C which converts Y g of ice at
0°C into water at 100°C. The ratio of X / Y will be
35. Water at − 10°C is present in a thermally insulated
a.1/3 b.2/3
container. Calculate the ratio of mass of ice formed c. 3 d. 2
and initial mass of water, if a small crystal of ice is
thrown into it

BITSAT Archives
1. In an experiment on the specific heat of a metal a 6. One junction of a thermocouple is at a particular
0.20 kg block of the metal at 150°C is dropped in a temperature Tr , and another is at T. Its thermoemf is
copper calorimeter (of water equivalent 0.025 kg)  1 
expressed as E = k (T − Tr ) T0 − (T + Tr ) .
containing 150 cc of water at 27°C. The final  2 
temperature is 40°C. Calculate the specific heat of the T0
metal. If heat losses to the surroundings are not At temperature T = , the value of thermo electric
2
negligible, is our answer greater or smaller than the
power will be [2010]
actual value of specific heat of the metal? [2014] 1
a. 0.02 b. 0.2 c. 0.01 d. 0.1 a. kT0 b. kT0
2
2. A partition wall has two layers of different materials A 1 1
c. kT02 d. k (T0 − Tr )2
and B in contact with each other. They have the 2 2
same thickness but the thermal conductivity of layer 7. In a 10 m deep lake, the bottom is at a constant
A is twice that of B. At steady state, if the temperature of 4°C. The air temperature is constant at
temperature difference across the layer B is 50 K, – 4°C. The thermal conductivity of ice is 3 times that of
then the corresponding difference across the layer water. Neglecting the expansion of water on freezing,
A is [2013] the maximum thickness of ice will be [2010]
a. 50 K b. 12.5 K c. 25 K d. 60 K a. 7.5 m b. 6 m c. 6 m d. 2.5 m
3. A monoatomic gas is suddenly compressed to (1/8)th 8. A metal string is fixed between rigid supports. It is
of its initial volume adiabatically. The ratio of its final initially at negligible tension. Its Young’s modulus is Y ,
pressure to the initial pressure is (Given, the ratio density is ρ and coefficient of thermal expansion is α. If
of the specific heats of the given gas to be 5/3) [2012] it is now cooled through a temperature = t, transverse
a. 32 b. 40/3 c. 24/5 d. 8 wave will move along it with speed. [2009]
4. 1g of water (volume 1 cm 3 ) becomes 1671 cm 3 of αt Y Yαt Yα
a. Y b. α t c. d. t
steam when boiled at a pressure of 1 atm. The latent ρ ρ ρ ρ
heat of vaporisation is 540 cal/g, then the external 9. There is some change in length when a 33000 N
work done is (1 atm = 1013
. × 105 N/m 2 ) [2011] tensile force is applied on a steel rod of area of
a. 499.7 J b. 40.3 J cross-section 10−3 m 2. The change of temperature
c. 169.2 J d. 128.57 J required to produce the same elongation, if the steel
5. The end A of rod AB of length 1 m is maintained at rod is heated is (The modulus of elasticity is
80°C and the end B at 0°C. The temperature at a 3 × 1011 N / m 2 and the coefficient of linear expansion
distance of 60 cm from the end A is [2010] . × 10−5 / ° C)
of steel is 11 [2008]
a. 16°C b. 32°C c. 48°C d. 64°C a. 20 ° C b. 15 ° C c. 10 ° C d. 0 ° C

@iitjeehelps
Answer with Solutions
Practice Exercise New time period, T ′ = T + ∆T ⇒ T ′ = 2.00012 s
Time lost in a week
1. (c) ∆T . × 10−4
12
2. (a) ∆TC = 90° − 30°C = 60°C ∆t = ×t = × ( 7 × 24 × 3600) = 36.23 s
T′ 2.00012
9 9
∆TF = ∆TC = ( 60° ) = 108° F 12. (a) 13. (c)
5 5
14. (b) Since, each rod is prevented from expansion so they
∆T = ∆TC = 60 K are under compression and mechanical strain. The net
3. (c) ∆l = 1 − 0.99970 = 0.00030 cm strain in each rod
Fl Fl
∆l = α l ∆t ε1 = α 1l∆T − ⇒ ε 2 = α 2l∆T −
∆l 0.0003 AY1 AY2
∆t = = = 27.3°C ε1 = ε 2 = 0
α l 11 . × 10−5 × 0.9997
Fl Fl
So, plate temperature must be raised to α 1l∆T − = 0 and α 2l∆T −
AY1 AY2
30° + 27.36 = 57.3°C
Solving, we get
4. (d) For an adiabatic process,
γ −1 α 1Y1 = α 2Y2
TV
1 1 = T2V2γ − 1
15. (c) Let L = free length at 0°C
or T1L 1γ − 1 = T 2L 2γ − 1 (Q A = constant)
L0 = final stretched length in each case.
γ −1
T1  L2  L1 and L2 are free lengths at + 20°C and − 20°C,
or =  
T 2  L1  respectively.
5
∴ γ = for a monoatomic gas, L
3
2 δ1
Q γ − 1= L1 αL∆T
3
2/ 3
T1  L2 
Thus, =   L2 αL∆T
T 2  L1 
δ2
5. (b) 6. (a) 7. (b) L0
8. (c) If m is the mass of water added at θ°C, then resultant
temperatureT is given as We know, s 2 = s 1 + 2αL∆T
Heat lost = Heat gained Here, s 1 and s 2 are load deformation
800 + m ⋅ θ F2L F1 L
⇒ m (θ − T ) = 40 (T − 20 ) ⇒T = = + 2αL∆T
40 + m AY AY
800 + 20 × 40 F F
So, for option (a), T = = 26.6° C ⇒ σ 2 = 2 = 1 + 2α∆TY
40 + 20 A A
800 + 30 × 30 =
5000
. × 10−6 )( 20)( 2 × 1011)
+ ( 2)(117
For option b, T = = 24.28° C
40 + 30 150 × 10−6
800 + 10 × 60 = 127 × 106 N/m2
For option (c), T = = 28° C
40 + 10 16. (c)
800 + 400 17. (b) According to relation,
For option (d), T = = 27.27° C
40 + 4 Coefficient of apparent expansion = coefficient of real
9. (a) 10. (d) expansion − coefficient of volume expansion
C = γL − 3 A …(i)
11. (d) Time period of second’s pendulum = 2 s
S = γL − 3A0 …(ii)
Change in time period,
1 where, A0 = linear expansion of coefficient of silver.
∆T = Tα∆θ
2 Subtracting Eq. (i) from Eq. (ii), we get
 1  3A + C − S 
. × 10−5 )( 30° − 206) = 12
∆T =   ( 2)(12 . × 10−4 s S − C = − 3A0 + 3A ∴ A0 =  
 2  3 

@iitjeehelps
HEAT, TEMPERATURE AND CALORIMETRY 183
18. (b) ρ1 + ρ1γ 2∆T = ρ 2 + ρ 2γ1∆T 28. (d)
ρ1 − ρ 2 29. (b) ∆H = ∫ ms dt
∆T =
ρ 2γ1 − ρ1γ 2 t
0
at 3 bt 2 
900 − 800 = m ∫ (at 2 + bt + c ) dt = m  0 + 0 + ct 0 
∆T2 = ⇒ ∆T = 217
. °C
880 × 12 . × 10−3 − 900 × 15
. × 10−4 0  3 2 
t0 t0
19. (d) Since, coefficient of volume expansion, at 03 bt 02
30. (a)s = ∫ s dt = ∫ (at 2 + bt + c ) dt = + + ct 0
γ = 3 × coefficient of linear expansion 0 0
3 2
⇒ γ = 3α W
1  ∆V  31. (a) P =
Now, ⋅  = 3 α ⇒ ∆V = 3 V ⋅ α ⋅ ∆T t
∆T  V  Total work done by drill machine in 2.5 × 60 s
4 = (10 × 103 )( 2.5 × 60) = 15 × 105 J
⇒ ∆V = πR 3 × 3α × ∆T = 4πR 3α ∆T
3 ∴ Energy lost = 50% of 15 × 105 J = 7.5 × 105 J
20. (a) If rods are heated to ∆t ° C, the increases in length of Energy taken by its surroundings, i.e. aluminium block.
steel and copper rods are ∆Q = mc∆t = 8 × 103 × 0.91 × ∆T J
∆Ls = Ls α s ∆t , and ∆Lc = Lc α c ∆t
Energy given = Energy taken
Difference between their lengths will remain constant.
7.5 × 105 = 8 × 103 × 0.91 × ∆T
⇒ ∆Ls = ∆Lc
L α ∆T = 103°C
⇒ Lsα s ∆t = Lc α c ∆t or s = c Pdt P
Lc α s 32. (a)Q dH = Pdt ∴ Cp = =
dT dT
21. (a) dW = dQ − dU dt
= Cp (T2 − T1) − CV (T2 − T1) = R [T2 − T1] T T4
Here, T = at 1/ 4 or = t 1/ 4 ∴ t = 4
= 8 . 31 × 100 = 8 .31 × 102 J a a
−3 / 4 −3
22. (d) 23. (c) 24. (a) dT a T 4  a T  a4 P 4PT 3
∴ =  4 =   = ⇒ Cp = =
3
25. (b) LetV cm be volume of either liquid dt 4 a  4 a 4T 3 dT a4
Mass of water = V × 1g dt
Mass of alcohol = V × 0.8 = 0.8V g 33. (a)
Rate of cooling of the water calorimeter 34. (a) Total heat required to convert the temperature of ice
1 1 from − 20°C to 0°C water is
= [V × (50° − 40° ) + V × 1(50° − 40° )] = V cal/s
100 5
∆H = mL + ms ∆θ = 5 × 80 + 5 × 0.5 × [ 0° − ( − 20° )]
Rate of cooling of alcohol calorimeter
1 = 400 + 2.5 × 20 = 400 + 50 = 450 cal
= [V × (50° − 40° ) + 0.8 V × s (50° − 40° )]
74 The total heat released to loss the temperature of water
1 from 30°C to water at 0°C is
= (10V + 8Vs ) cal/s
74 ∆H′ = 5 × 1 × ( 30 − 0) = 150
As, rate of cooling of both is same Since, heat released in lesser than heat required. Hence,
1 total ice is not converted into water. Hence, final
5V = (10V + 8Vs ) ⇒ s = 0.6 cal/g°C
74 temperature of mixture is 0°C.
 4 3 35. (d) Let initial mass of water in the container be m.
  πr ρ c 3
mAc A  3 A A A  rA  ρ Ac A
26. (b) = =  m1 = the mass of water converted into ice.
mBcB  4 3  rB  ρBcB
  πrB ρBcB At final thermal equilibrium state, temperature of mixture
 3
3
should be zero.
mAc A  1 2  1 1
=  × ×  = Q Heat loss by water = heat of fusion of water
mBcB  2 1  3 12
or ms [ 0 − ( − 10)] = m1L
T3 m1 10 1
27. (c) dQ = nCdT , dQ = nk 3 dT or m × 1 × 10 = m1 × 80 or = =
θ m 80 8
T2
nk nk T 4 − T14 

36. (a) Let R = rate at which heat is received
Q = 3 ∫ T 3dt = 3  2 
θ T θ  4 
Q1 = R × A × t = R × π ( 2.5)2 × ( 20 min) = 392.5 R cal
1

2 × 1940(504 − 104 ) Heat required to melt the 10 g of ice


= = 272.79 J ⇒ Q = 273 J
( 281)3 × 4 Q 2 = mL = 80 × 10 = 800 cal

@iitjeehelps
184 SELF STUDY GUIDE BITSAT

Q1 = Q 2 ⇒ 392.5 R = 800 Specific latent heat of vaporisation

R=
800
= 2.04 cal / cm2 -min = 22.68 × 105 J/kg = X × 10−3 × 22.68 × 105
392.5
= Y × 10−3 × 3.36 × 105 + Y × 10−3 × 4200 × 100
37. (a) Specific heat of water = 4200 J/kg-K
X 7.56 1
∴ = =
Specific latent heat of fusion = 3.36 × 105 J/kg Y 22.68 3

BITSAT Archives
1. (d) Mass of metal, m = 0.2 kg = 200 g  1  1 
S = k (T − Tr ) ×  −  + T0 − (T + Tr ) × k
 2  2 
Fall in temperature of metal ∆T = 150 − 40 = 110° C
T
If s is specific heat of metal, the heat lost by the metal When T = 0
2
∆Q = ms ∆T = 200 × 110 × s
k  T0   1 T 
Volume of water, = 150 cc S = −  − Tr  + T0 −  0 + Tr   × k
22   22 
Mass of water m′ = 150 g
− kT0 kTr kT0 kTr 1
Water equivalent of calorimeter = + + kT0 − − = kT0
4 2 4 2 2
w = 0.025 kg = 25 g
 4−0  0 − ( − 4)
Rise in temperature of water in calorimeter 7. (a) KA   = 3KA  
 10 − x   x 
∆T ′ = 40 − 27 = 13° C
Heat gained by water and calorimeter –4°C
∆Q ′ = (m′ + w ) ∆T ′ = (150 + 25) × 13
x
∆Q ′ = 175 × 13 = ∆Q 3K
So, 200 × s × 100 = 175 × 13 10 0
175 × 13
⇒ s = ≈ 0. 1 K
200 × 100 (10 – x)

2. (c) LetT be the junction temperature 4°C


Here, K A = 2KB , T − TB = 50 K
TA T TB
At the steady state HA = HB ⇒ x = 7.5 m
K A A(TA − T ) KB A(T − TB ) F /A T .L
⇒ = 8. (c) From, Y = =
L L A B ∆ L /L A ∆ L
⇒ 2KB (TA − T ) = KB (T − TB )
where, F = T = tension.
T − TB 50 L
⇒ TA − T = = = 25 K On cooling, ∆ L = α L ( ∆ θ ) = α Lt
2 2
TL
3. (a) In an adiabatic process, ∴ Y =
pV γ = constant A α Lt
γ T = YA α T
p1 V2 
5/ 3
p  1
⇒ =  ⇒ 1 =  Also, mass per unit length of string
p 2  V1  p 2  8
5/ 3 m = Aρ
p1  1  1 p
⇒ =  = ∴ 2 = 32 T  YA α t Y α t 
p 2  23  32 p1 As wave velocity =   ∴ v = =  
 m Aρ  ρ 
4. (c) Work done, W = p∆V = 1013
. × 105 × (1671 − 1) × 10−6
Force l
= 1013
. × 105 × 1670 × 10−6 = 169.2 J 9. (c) Modulus of elasticity = ×
Area ∆l
∆T 80 − 0
5. (b) = = 80 cm−1 33000 l
l 1 3 × 1011 = ×
10−3 ∆l
∆T1 = ( 0.60) ( 80) = 48° C
∆ l 33000 1
Decrease in temperature after a distance of 60 cm is 48°C = × = 11 × 10−5
and hence actual temperature is ( 80 − 48)° C = 32° C. l 10−3 3 × 1011
6. (a) Thermoelectric power, ∆l
Charge in length, = α ∆T
dE d   1  l
S = = k (T − Tr ) ⋅ T0 − (T + Tr )
dT dT   2  11 × 10−5 = 11
. × 10−5 × ∆ T ⇒ ∆ T = 10 K or 10 ° C

@iitjeehelps
16
Physics for
Gaseous State

Ideal Gas or Perfect Gas


An ideal gas or perfect gas is that gas which strictly obeys the gas laws such as Boyle’s law, Charles’
law, Gay-Lussac’s law, etc., at all values of temperature and pressure.

Gas Laws
There are some laws given below

Boyle’s Law
It states that for a given mass of an ideal gas at constant temperature (called isothermal process), the
volume of a gas is inversely proportional to its pressure, i.e.
1
V∝
p

(where, V = volume, p = pressure, m = mass and T = temperature or pV = constant)


or p1V 1 = p2V 2 = p3V3 = K
This law can also be shown graphically as

p p pV 1/p log p

V 1/V p or V log V
1/ V
(a) (b) (c) (d) (e)

(m and T are constants in all cases)

@iitjeehelps
186 SELF STUDY GUIDE BITSAT

Charles’ Law Ideal gas equation is a form of combined effect of above


four laws, thus its equation is given by
It states that for a given mass of an ideal gas at constant m
pressure, (called isobaric process) volume of a gas is pV = nRT = RT
M
directly proportional to its absolute temperature
i.e. V ∝T [if m and p are constants]

or
V V V
= constant or 1 = 2 = L
Kinetic Theory of Gases
T T1 T2 The kinetic theory of gases is based on the following
This law can also be shown graphically as assumptions
1. Molecules of a gas are small hard spheres and these
V V
In this graph, molecules are very far apart in comparison to their
V0 is volume at sizes.
V0
0°C 2. The total volume of the molecules is negligible as
–273
compared to the size of the gas.
T t (°C)
(a) (b) 3. The molecules collide elastically with each other.
4. The molecules exert no force on each other except
V/T 1/T V/T
during collisions.

Concept of Pressure
V
(c) V or T (d) (e) 1/V or 1/T The pressure of the gas is equal to the force exerted per unit
area of the walls by the gas.
(m and p are constants in all cases) Pressure of an ideal gas
1 mn 2 1 2
p= vrms = ρvrms
Gay-Lussac’s Law or Pressure Law 3 V 3

It states that for a given mass of an ideal gas at constant


volume (called isochoric process), pressure of a gas is Kinetic Interpretation of Temperature
directly proportional to its absolute temperature According to kinetic theory of gases, for 1 mole of an
i.e. p ∝T [if m and V are constants] ideal gas
p 1M
or = constant p= (vrms )2
T 3V
p1 p2 1
or = =L ⇒ pV = M (vrms )2 …(i)
T1 T2 3
(here, M = total mass)
Here, temperature is in kelvin.
3 RT
This law can also be shown graphically as So, (vrms )2 = (for 1 mole, i.e. n = 1) …(ii)
M
p p
T Also, pV = RT
3 RT
vrms =
M

T p or T
⇒ vrms ∝ T
(a) (b) The absolute temperature of an ideal gas is directly
proportional to mean square velocity of its molecule.
(m and V are constants in all cases)
1
Also, 2
M vrms = RT [From Eqs. (i) and (ii)]
3
Ideal Gas Equation ⇒
1 M 2
vrms =
R 1
T ⇒ mvrms 2 3
= k BT
An ideal gas is one whose molecules are free from 3 N0 N0 2 2
intermolecular attraction and obeys gas laws at all values of  R 
 kB = = Boltzmann constant
temperature and pressure.  N0 

@iitjeehelps
PHYSICS FOR GASEOUS STATE 187
3
⇒ (KE) trans = k BT Most Probable Speed
2
It is defined as speed which is possessed by maximum
1 2
where, mvrms is average translational energy per fraction of total number of molecules of gas.
2
molecule According to kinetic theory of gases, it can be shown that
2kBT
(KE) trans ∝ T vmp =
m
Relation between Pressure and
Kinetic Energy of the Gas Degree of Freedom
Degree of freedom represents the number of independent
By using the expression for pressure ( p ) in terms of mean possible ways in which the system can have energy (due to
square speed (v 2 ), we can find the relation between its motion or configuration). A system can possess energy
pressure and kinetic energy ( E ) as shown below. due to translational, rotational or vibrational motion or due
to vibrational configuration or any combination of these.
1
Since, p= ρ(v 2 ) …(i) (i) For an ideal monoatomic gas, f = 3 due to
3
translational motion in three directions.
Mass of unit volume of gas = 1 × ρ = ρ
(ii) For an ideal diatomic gas at room temperature f = 5
Mean kinetic energy of translation per unit volume of the due to 3 translational and 2 rotational. At high
gas is temperature, f = 7 because of 2 additional ways due
1 to vibration (one kinetic energy and other potential
E = ρ(v 2 ) …(ii) energy).
2
(iii) For an ideal polyatomic gas at room temperature,
From Eqs. (i) and (ii), we get
f = 6 due to 3 translational and 3 rotational. At high
p 2 2 temperature, the vibrational mode will also come
= or p = E (for unit volume)
E 3 3 into existence. For a polyatomic gas, number of
vibrational terms can be greater than two.
The pressure exerted by an ideal gas is numerically equal to
two-third of the mean kinetic energy of translation per unit (iv) For solids, f = 6 due to vibrational motion in all three
volume of the gas. directions.

Law of Equipartition of Energy


Various Speeds of Gas Total internal energy of an ideal gas distributes equally in
Molecules all degrees of freedom and energy per degree of freedom of
1
one mole of gas is RT , where T is the absolute temperature
2
RMS Speed of Gas Molecules of the gas.
The square root of the mean of the squares of the speed of If f is the degree of freedom and n is number of moles of the
gas molecules is called their root mean square speed (vrms ). gas, then the internal energy of the gas is given by
3 RT n
vrms = [vrms ∝ T ] U = fRT
m 2
where, m = molecular weight.
Mean Free Path
Mean Speed or Average Speed Every gas consists of a large number of molecules
undergoing frequent collisions. These molecules are in a
It is the arithmetic mean of the speeds of molecules in a gas.
state of continuous random motion. They undergo
If v1 , v2 , K, vn are individual speeds of n molecules, then perfectly elastic collision against one another. The zig-zag
v1 + v2 + K + vn path of different lengths is called free
vav = path and their mean is called mean
n λ3
λ2 λ4
free path. If λ 1 , λ 2 , λ 3 , K, λ n are
According to kinetic theory of gases, it can be shown that λ1
successive free paths, then
8kBT λ + λ 2 + λ 3 +... + λ n
vav = Mean free path, λ = 1
πm n

@iitjeehelps
188 SELF STUDY GUIDE BITSAT

So, mean free path of gas molecules is average distance Here, M = molar mass of gas, n = number of moles,
travelled by a molecule between two successive collisions. C = molar specific heat and c = specific heat
Mathematically, it can be expressed as (or gram specific heat)
1
λ= It is of two types
2 π d 2n
where, n = number of molecules per unit volume Specific Heat at Constant Volume (CV )
d = diameter of molecules When heat is supplied to gas at constant volume, the entire
heat supplied just increases the internal energy of gas, so
specific heat at constant volume is
Specific Heat Capacity 1  ∆Q 
CV =  
It is the amount of heat required to raise the temperature of n  ∆T  V
unit mass of a substance through 1 °C. Consider a container
containing m gram of gas of molecular mass M. If n is 1  ∆U 
⇒ CV =  
number of moles of gas in a container, ∆Q is the heat n  ∆T 
supplied and rise in temperature is ∆T , then
1 ∆Q Specific Heat at Constant Pressure (C p )
Specific heat, c = and SI unit = J kg −1 K −1
m ∆T
When heat is supplied to the gas at constant pressure, a part
of it increases the internal energy of the gas and remaining
Molar Specific Heat of Gases does an external work, so specific heat at constant
pressure is
Molar specific heat (C) of a substance is defined as the
1  ∆Q 
amount of heat required to raise the temperature of 1 mole Cp =  
of substance through one degree celsius n  ∆T  p
1 ∆Q At constant pressure, to increase internal energy of gas by
C=
n ∆T the same amount (as in case of heat supplied at constant
Also, relation between specific heat and molar specific heat volume), more amount of heat has to be supplied, so we
conclude C p > CV .
C = Mc

Practice Exercise
1. In an ideal gas without preferred direction of motion of 3. In troposphere, temperature varies linearly with
molecules, elevation as T = T0 − ay , where T0 is the temperature
a. v x = v y = v z at the earth’s surface, then
b. v x2 = v y2 = v z2 a. the pressure does not change with elevation in
troposphere
c. v x2 = v y2 = v z2 b. the variation of pressure with elevation is linear
d. None of the above c. the dimension of a is [M0L−1θ ]
2. Two perfect gases at absolute temperatures T1 and T2 d. the pressure is independent of variation with
temperature in the given situation
are mixed. The absolute temperature of the mixture is
T. There is no loss of energy. If m1 and m 2 are masses 4. A vessel contains a mixture of nitrogen of mass 7 g
of molecules and n1 and n2 are number of molecules, and carbon dioxide of mass 11 g at temperature 290 K
then and pressure 1 atm. Find the density of the mixture.
T1 + T2 a. 1.1 g/L b. 1.2 g/L c. 1.515 g/L d. 1.6 g/L
a. T =
2 5. 12 g of gas occupy a volume of 4 × 10−3 m 3 at a
n T + n 2T2
b. T = 1 1 temperature of 7°C. After the gas is heated at constant
n1 + n 2
pressure, its density becomes 6 × 10−4 g/ cm 3 . What is
n1T1 + n 2T2
c. T = the temperature to which the gas was heated?
T1 + T2
a. 1000 K b. 1400 K c. 1200 K d. 800 K
d. None of the above

@iitjeehelps
PHYSICS FOR GASEOUS STATE 189
6. A closed vessel with a capacity of 1 m 3 contains 14. Figure shows graph of
0.9 kg of water and 1.6 kg of O 2. Find the pressure in pressure versus density for T1
the vessel at a temperature of 500 °C at which all the an ideal gas at two p
water will be converted into steam. temperatures T1 and T2, then
a. T1 > T2 T2
a. 3.2 × 105 N/m2 b. 6.4 × 105 N/m2
. × 105 N/m2
c. 16 d. 9.6 × 105 N/m2 b. T1 = T2
b. T1 < T2
7. The pressure of a gas kept in an isothermal container ρ
d. any three is possible
is 200 kPa. If half the gas is removed from it, the
pressure will be 15. A gas is contained in a closed vessel at 250 K, then
a. 100 kPa b. 200 kPa c. 400 kPa d. 800 kPa calculate the percentage increase in pressure, if the
gas is heated through 1°C.
8. The pressure inside a tyre is 4 atm at 27 °C. If the tyre
a. 0.4% b. 0.6%
bursts suddenly, its final temperature will be
c. 0.8% d. 1.0%
a. 300 ( 4)7/ 2 b. 300 ( 4) 2/ 7 c. 300 ( 2)7/ 2 d. 300 ( 4)−2/ 7
16. What volume will be occupied by the molecules
9. A uniform tube is shown in figure. Which is open at contained in 4.5 kg water at STP, if the intermolecular
one end and closed at the other. To enclose a column forces vanish away ?
of air inside the tube, a pellet of mercury is introduced. a. 5.6 m3 b. 4.5 m3
If the length of air column at 27 °C is 18 cm, at what c. 11.2 m3 d. 5.6 L
temperature its length will be 21.6 cm?
17. Find the pressure exerted by 6 × 1023 hydrogen
molecules which will strike per second a wall of area
10−4 m 2 at 60° with normal. The mass of hydrogen
molecules and speed are 3.32 × 10−27 kg and 103 m/s
respectively.
Mercury
a. 19.92 × 103 N/m2
a. 87 °C b. 91 °C c. 85 °C d. 97 °C b. 18.2 × 103 N/m2
c. 1. 992 × 103 N/m2
10. Pressure of an ideal gas is increased by keeping
temperature constant. What is its effect on kinetic d. 0.1992 × 103 N/m2
energy of molecules? 18. Four molecules of a gas have speeds 1, 2, 3 and
a. Increase b. Decrease 4 km/s. The value of the root mean square speed of
c. No change d. Cannot be determined the gas molecules is
11. How many cylinders of hydrogen at atmospheric 1 1
a. 15 km/s b. 10 km/s
pressure are required to fill a balloon whose volume is 2 2
500 m 3 , if hydrogen is stored in cylinders of volume  15
c. 2.5 km/s d.   km/s
0.05 m 3 at an absolute pressure of 15 × 105 Pa?  2
a. 700 b. 675 c. 605 d. 710 19. The temperature of H2 at which the rms velocity of its
12. Two identical containers A and B have frictionless molecules is seven times the rms velocity of the
pistons. Both contain same volume of ideal gas at molecules of nitrogen at 300 K, is
same temperature. The gas in each cylinder is a. 2100 K b. 1700 K c. 1350 K d. 1050 K
allowed to expand isothermally to double the initial 20. Choose the correct order of the root mean square
volume. The mass of the gas in A is m A and the mass velocity (v rms ), the average velocity (v av ) and the most
of the gas in B is mB . The changes in the pressure in A probable velocity (v mp ).
and B are ∆p and 1. 5 ∆p , respectively, then
a. v mp > v av > v rms b. v rms > v av > v mp
a. 4mA = 9mB b. 2mA = 3mB c. 3mA = 2mB d. 9mA = 4mB
c. v av > v mp > v rms d. v mp > v rms > v av
13. The figure shows, the p-V T T
diagram of two different masses
21. The average kinetic energy of a gas molecule at 27 °C
m1 and m 2 drawn at constant is 6.21× 10−21 J. Its average kinetic energy at 127 °C
temperature T, then will be
p
a. m1 > m2 m2 a. 12.2 × 10−21 J
b. m2 > m1 m1 b. 8.28 × 10−21 J
c. m1 = m2 V c. 10.35 × 10−21 J
d. insufficient data d. 11.35 × 10−21 J

@iitjeehelps
190 SELF STUDY GUIDE BITSAT

22. Ratio of v mp (most probable speed), v av (average pV


28. Given the graph between and p for 1 g of oxygen
speed), v rms (root mean square speed) of gas T
molecules are related as gas at two different temperatures T1 and T2 (in figure).
8 π Given, density of oxygen = 1.427 kg/m3 . The value of
a. 3: 2: b. 2 : 3 : pV
π 8 at point A and the relation between T1 and T2 are
8 8 T
c. 2: 3: d. 2: : 3 respectively
π π
pV –1
JK A
23. Five gas molecules chosen at random are found to T
have speeds of 500, 600, 700, 800 and 900 m/s.
Then, T1
Ideal gas
a. the rms speed and the average speed are the same
b. the rms speed is 14 m/s higher than the average T2
speed
c. the rms speed is 14 m/s lower that the average
speed p
d. the rms speed is 14 m/s higher than the
average speed a. 0.256 JK −1 and T1 < T2
24. In case of molecules of an ideal gas, which of the b. 8.314 J mol−1 K −1 and T1 > T2
following, average velocities cannot be zero? c. 0.256 JK −1 and T1 > T2
a. < v > b. < v 3 > d. 4.28 JK −1 and T1 < T2
c. < v 4 > d. < v 5>
29. Which of the following quantities is zero on an
25. Choose the correct relation between the rms speed average for the molecules of an ideal gas in
(v rms ) of the gas molecules and the velocity of sound equilibrium?
in that gas (v s ) in identical situations of pressure and a. Kinetic energy b. Momentum
temperature. c. Density d. Speed
 3
a. v rms = vs b. v rms =  vs 30. On a fast moving train, a container is placed enclosing
 γ some gas at 300 K, while the train is in motion, the
 γ temperature of the gas
c.v rms =   vs d. γ v rms = 3 vs
 3 a. rises above 300 K b. falls below 300 K
c. remains unchanged d. becomes unsteady
26.At what temperature is the effective speed of gaseous
hydrogen molecules equal to that of oxygen
31. If at a pressure of 106 dyne/cm 2, one gram mole of
molecules at 47 °C? nitrogen occupies 2 × 104 cc volume, then calculate
the average energy of a nitrogen molecules in erg.
a. 50 K b. 20 K (Given, Avogadro's number = 6 × 1023 )
c. 40 K d. 100 K
a. 14 × 10−13 b. 10 × 10−12 c. 106 d. 2 × 106
27. Volume versus temperature graphs for a given mass 32. Calculate the temperature at which average
of an ideal gas are shown in figure at two different translational KE of a molecule is equal to the KE of an
values of constant pressure. What can be inferred electron accelerated from rest through a potential
about relation between p1 and p 2? difference of 1 V.
V(l) a. T = 7729 K b. T = 8879 K
p2
40 c. T = 7.72 K d. T = 772.9 K

30 33. In a model of chlorine (Cl2), two Cl-atoms are rotated


p1
about their centre of mass as shown. Here the two
20 Cl-atoms are 2 × 10−10 apart and angular speed
10 ω = 2 × 1012 rad/s. If the molar m ω
m
mass of chlorine is 70 g/mol,
T(K) then what is the rotational Cl Cl
100 200 300 400 500
kinetic energy of one Cl2 r r
molecule?
a. p1 > p 2 b. p1 < p 2
a. 2.32 × 10−20J b. 2.32 × 10−21J
c. p1 = p 2 d. Data is insufficient
c. 2.32 × 10−19J d. 2.32 × 10−22J

@iitjeehelps
PHYSICS FOR GASEOUS STATE 191
34. At 20 °C temperature, an argon gas at atmospheric 35. In the case of saturated vapour,
pressure is confined in a vessel with a volume of 1 m 3 . a. pressure depends upon volume at constant
The effective hard sphere diameter of argon atom is temperature
3.10 × 10−10 m. Determine mean free path. b. pressure varies non-linearly with temperature at
constant volume
a. 100 nm
c. pressure becomes less than one atmosphere at
b. 90 nm boiling point
c. 93.6 nm d. pressure varies linearly with temperature at constant
d. 95 nm volume

BITSAT Archives
1. Two balloons are filled one with pure He gas and 6. From the following V -T diagram, what is true about
other with air respectively. If the pressure and pressure? [2009]
temperature of these balloons are same, then the
B
number of molecules per unit volume is [2014]
a. more in He filled balloon b. same in both balloons
A C
c. more in air filled balloon d. in the ratio 1: 4 T

2. A vessel containing 1 mole of O2 gas (molar mass 32)


D E
at temperature T. The pressure of the gas is p. An V
identical vessel containing one mole of He gas (molar
mass 4) at temperature 2T has a pressure of [2013] a. p1 < p 2 b. p1 > p 2
a. p /8 b. p c. 2 p d. 8 p c. p1 = p 2 d. Cannot predict
3. The temperature of an ideal gas is increased from 7. One litre of oxygen at a pressure of 1 atm and two
27 °C to 127 °C, then percentage increase in v rms is litres of nitrogen at a pressure of 0.5 atm, are
[2013] introduced into a vessel of volume 1 L. If there is no
a. 37 % b. 11% c. 33% d. 15.5% change in temperature, the final pressure of the
4. The ratio of the adiabatic bulk modulus to the mixture of gas (in atm) is [2008]
isothermal bulk modulus of a perfect gas with f a. 1.5 b. 2.5 c. 2 d. 4
degrees of freedom is [2010]
2 1 2 (f − 1) 8. The ratio of velocity of sound in hydrogen and oxygen
a. b. 1 + c. 1 + d. 1 + at STP is [2005]
f f f 4
5. If v is the molecular speed and l is the mean free path a. 16 : 1
of molecule of a gas, then the collision frequency is b. 8 : 1
v l c. 4 : 1
a. vl b. c. d. vl d. 2 : 1
l v [2010]

Answer with Solutions


Practice Exercise After mixing, the average KE of both gases
3
1. (c) An isotropic gas is one which have same properties = k (n1 + n 2 ) T
2
throughout and their molecules are all moving in random
directions, so only average values of square of their where, T is temperature of mixture.
velocity components are equal. Q No loss of energy.
3
2. (b) Average KE per molecule of a gas = kT ∴
3 3 3
k (n1 + n 2 ) T = kn1T1 + kn 2T2
2 2 2 2
Before mixing two gases, the average KE of all molecules n1T1 + n 2T2
of 2 gases T =
(n1 + n 2 )
3 3
= kn1T1 + kn 2T2 3. (c)
2 2

@iitjeehelps
192 SELF STUDY GUIDE BITSAT

7 12. (c)
4. (c) p1V = × R × 290
28 13. (b) We know that,
11
p 2V = × R × 290 m
44 pV = nRT = RT
M
 7 11
∴ ( p1 + p 2 )V =  +  R × 290 M 
 28 44 m = ( pV )  
 RT 
Q p1 + p 2 = 1 atm
m ∝ pV
 7 11
 +  × 8.3 × 10 × 290
7
p 2V2 > p1V1
 28 44
V = From the curve, m2 > m1
.
1013 × 106
mass 18 14. (a) According to ideal gas equation,
∴ Density = = = 1.515 g/L pV = nRT
volume V
m
pV = RT
5. (b) Volume at 7 ° C = 4 × 10−3 m3 M
ρRT
Let it be heated to x K. or p=
M
12
Volume = cm3 = 2 × 104 cm3 RT
6 × 10−4 or p= ρ
M
= 2 × 10−2 m3 Hence, more temperature means more slope.
V  dθ 
Since, = constant 15. (a) pt = p 0 (1 + γ dθ ) = p 0 1 + 
T  237
4 × 10−3 2 × 10−2 pt − p 0 1
∴ = × 100 = × 100
280 x p0 273
2 × 10−2 × 280
∴ x = = 1400 K = 0.36% ≅ 0.4%
4 × 10−3
16. (a) If intermolecular forces vanish away, the liquid will
6. (b) convert into vapours.
7. (a) For isothermal process, temperature remains Molecular mass of water vapours
constant. According to ideal gas equation, = 18 g = 18 × 10−3 kg
pV = nRT
So, 18 × 10−3 kg of water vapour will occupy 22400 m3
For a given container,V remains constant.
∴ p ∝n then, 4.5 kg of water vapour will occupy
22.4 × 10−3
When half of gas is removed, number of moles becomes × 4.5 = 5.6 m3
half. Hence, pressure becomes half. 18 × 10−3
8. (d) In an adiabatic process, 17. (a) Change in momentum = 2mv cos θ
p(21 − γ ) T2γ = p1(1 − γ ) T1γ Force, F = 2mv cosθ × n
(1 − γ )/ γ (1 − 7 / 5 ) F 2mv n cosθ
p   4 7 / 5 p= =
T2 = T1  1  = 300   = 300 ( 4)−2/ 7 A A
 p2   1
1
2 × 3.32 × 10−27 × 103 × × 6 × 1023
9. (a) From Charles’ law, L ∝ T = 2
L1 L2 10−4
=
T1 T2 = 19.92 × 10 N/m
3 2


L
T2 = 2 × T1 v 2 + v 22 + .... + vn2 
L1 18. (d)Qv rms =  1 
 n 
216.
T2 = × 300
18  12 + 22 + 32 + 42   1 + 4 + 9 + 16
v rms =   =  
T2 = 87 °C  4   4 
10. (c) As kinetic energy depends on temperature, so there  30  15
will be no change. =   =   km/s
 4  2
11. (b) 15 × 105 × V2 = 500 × 1013
. × 105
 15
V2 = 33.77 m3 ∴ v rms =   km/s
 2
33.77
∴ Number of cylinders = = 675 19. (d)
0.05

@iitjeehelps
PHYSICS FOR GASEOUS STATE 193
 3kT   8kT  29. (b) The average of speed, density and kinetic energy can
20. (b)v rms =   ,v =  ,
 m  av  πm  never be zero. Because these quantities have only
positive values. But momentum is a vector quantity. So,
 2kT  average value of momentum may be zero.
v mp =  
 m 
30. (a) Random motion of molecules is not ordered motion,
∴ v rms > v av > v mp causes rise of temperature.
E (127 + 273) 1 1 3p 1 M 3p
21. (b) As, KE ∝ T ⇒ 127 = 31. (a) Average KE = mv rms
2
= m = ×
E27 (27 + 273) 2 2 ρ 2N ρ
400
E127 = 6.21 × 10−21 × =
1
×
28 3 × 106
300 2 6 × 10  1 
23
= 8.28 × 10−21 J  
 2 × 104 
8
22. (d)v mp : v av : v rms = 2 : : 3 = 14 × 10−13 erg
π
500 + 600 + 700 + 800 + 900 32. (a)
23. (b)Q v av =
5 33. (b) Let I = moment of inertia
3500
= = 700 m/s I = 2 (mr 2 )
5
70 × 10−3
(500)2 + ( 600)2 + ( 700)2  m= = 5.81 × 10−26 kg
  2 × 6.02 × 1023
+ ( 800)2 + (900)2 
and v rms = 
 5  2 × 10−10
r = = 1 × 10−10 m
  2
 
= 714 m/s I = 1.16 × 10−45 kg/m2
Hence, the rms speed is 14 m/s higher than the average 1 2
∴ k = Iω
speed. 2
24. (c) The average velocity of even power quantity cannot 1
= (1.16 × 10−45 ) ( 2 × 1012 )2
be zero. 2
 3RT 
25. (b)v rms = 
 γRT  = 2.32 × 10−21 J
 and vs =  
 m   m 
34. (c) Mean free path,
 3 kT
∴ v rms =   vs λ=
 γ 2πρd 2
26. (b) (1381
. × 10−23 ) ( 293)
λ=
27. (a) When pressure is constant, Charles’ law is obeyed 2 π (1.013 × 105 ) ( 3.10 × 10−10 )2
V 1 λ = 93.6 nm
i.e. V ∝ T or = constant =
T p
35. (b) The p-T graph for saturated vapour is shown in figure.
From slope of curves, p1 > p 2.
m
28. (c) pV = nRT = RT p
M
1atm V=constant
where, m = mass of gas
m
and = n = number of moles
M
pV
⇒ = nR = constant for all values of p (ideally, it is
T
a straight line). T
Boiling point
pV 1g
∴ = × 8.31 J mol−1 K −1 = 0.256 JK −1
T 32 g
From graph, it is clear that pressure varies non-linearly
Also, T1 > T2 with temperature at constant volume.

@iitjeehelps
194 SELF STUDY GUIDE BITSAT

BITSAT Archives
1. (b) Assuming the balloons have the same volume, as Es Cp 2
4. (c) = = γ = 1+
pV = nRT . If p, V and T are the same, n the number of ET CV f
moles present will be the same, whether it is He or air. v
Hence, number of molecules per unit volume will be same 5. (b) Collision frequency =
l
in both the balloons.
6. (a) AsV-T graph is a straight line with positive slope,
2. (c) Applying gas equation, pV = nRT therefore its equation, may be written as
We can write, p1V = n1RT1 V = aT + b
From standard gas equation,
and p 2V = n 2RT2 RT RT R
p= = =
p 2 n 2 T2 V aT + b a + b / T
⇒ = ×
p1 n1 T1 As, T2 > T1
1 2T ∴ p 2 > p1 or p1 < p 2
= × =2
1 T 7. (c) Ideal gas equation is given by
⇒ p 2 = 2p pV = nRT …(i)
3RT For oxygen, p = 1atm, V = 1 L, n = n 0
3. (d) We know, v rms = Therefore, Eq. (i) becomes
M
1
⇒ % increase in ∴ 1 × 1 = n O2RT ⇒ n O2 =
RT
3RT2 3RT1 1
− and 0.5 × 2 = n N2RT ⇒ n N2 =
M M × 100 RT
v rms =
3RT1 For mixture of gas,
M p mixVmix = n mixRT
T2 − T1 Here, n mix = n O2 + n N2
= × 100 p mixVmix 1 1
T1 ∴ = +
RT RT RT
400 − 300 ⇒ p mixVmix = 2
= × 100
300 γRT
8. (c) Velocity of sound,v =
20 − 17.32 M
= × 100
17.32 vH MO 16
= = = 4 :1
= 15.5% vO MH 1

@iitjeehelps
17
Laws of
Thermodynamics

Thermal Equilibrium
When there is no exchange of heat between two objects placed in contact, then both are said to be in
thermal equilibrium. Thus, the temperature remains constant throughout all the portions.
Thermodynamics is a branch of science which deals with transformation of heat energy into other
forms of energy and vice-versa.

Zeroth Law of Thermodynamics


If two objects A and B are separately in thermal equilibrium with another
object C, then objects A and B will also be in thermal equilibrium. C
Cylinder has a cross-sectional area A and pressure exerted by system on the
piston face is p. The work done by the system on the surroundings for small A B
displacement dx is dW = pAdx as shown in figure.
dx

pA

.
Vf
W = ò dW = òV pdV
i

Work
Work done can be defined as ability to do work. In thermodynamics, work is actually done due to
pressure of the gas.
If a gas placed in a cylinder with frictionless piston is allowed to expand from the state A to the state B,
when pressure and change in volume are p and DV . If during expansion, piston moves by small
distance dx, then work done is
V2
dW = pA (dx ) = pdV Þ W = òV
1
pdV

@iitjeehelps
196 SELF STUDY GUIDE BITSAT

If V increases, then work done by the gas is positive. If V


decreases, then work done by the gas is negative. First Law of Thermodynamics
Work done = area bounded by p -V curve. This law is based on law of conservation of energy. If dQ is
heat supplied to a thermodynamical system and dW is
Work done in Different Cases work done by the thermodynamical system and resulting
dU is the change in internal energy, then
Case I When volume is constant.
dQ = dW + dU
p p
B A This equation is the first law of thermodynamics.

or Sign Conventions for First Law of


Thermodynamics
A B
V V dQ , dW and dU must be in the same units.
V = constant WAB = 0 Heat supplied to the system = Positive
Heat rejected by the system = Negative
Case II When volume is increasing. Work done by the system = Positive
Work done on the system = Negative
p p
B A If temperature increases, dU = Positive
If temperature decreases, dU = Negative
A or
B

V V Different Thermodynamical
V is increasing WAB > 0 WAB = shaded area
Processes
Case III When volume is decreasing. We are going to discuss different types of thermodynamics
processes.
p p
B A
B
1. Reversible and Irreversible
A or
Processes
V V A reversible process means, if a process takes up the path
V is decreasing WAB < 0 WAB = – shaded area AB (as shown in figure), then on reversing the conditions it
comes back by BA.
Case IV Work done in cyclic process. A thermal process could be reversible, if the change is
p p extremely small (infinitesimally small).
E A In irreversible process, one will not reach back to A, if the
D S R
B process AB has occurred.
p p
C P Q B B
x
V V
(a) (b) y
A A
V V
(i) Work done by clockwise cycle
Reversible process Irreversible process
ABCDE = + shaded area [In Fig. (a)]
(ii) Work done by anti-clockwise cycle 2. Isothermal Process
PQRS = - shaded area [In Fig. (b)] ● In this process, temperature of the system is kept
Case V Work done in incomplete cycle. constant during the change of state.
Q
p p ● As Q = nC isodT Þ C iso = = ¥, i.e. molar heat
ndT
C B A capacity for an isothermal process is infinity.
B or
C D
A
● From dU = nCV dT as dT = 0, so dU = 0, i.e. internal
V V energy is constant.
WABC = + shaded area WABCD = – shaded area ● Gas equation is pV = constant.

@iitjeehelps
LAWS OF THERMODYNAMICS 197
● From first law of thermodynamics, dQ = dW i.e. heat ● dU = nCV dT
given to the system is equal to the work done by p
system on the surroundings.
æV f ö æp ö
W = nRT ln ç ÷ = nRT ln ç i ÷
è Vi ø è pf ø p

● After differentiating p
pV = constant, we have Isotherm V
dp p Vi Vf
pi
= - (slope of p-V
dV V pV=constant ● From the first law of thermodynamics,
curve) and - dp = p, Rectangular dQ = dU + dW ; dW = pdV = nRdT
dV /V pf hyperbola
Þ W = p(V f - Vi ) = nR(T f - Ti )
i.e. bulk modulus of a Vi Vf V V
gas in isothermal
● Gas equation is = constant.
T
process, B = p ● p -V curve is a straight line parallel to the volume
● p-V curve is a rectangular hyperbola. axis.

3. Adiabatic Process 5. Isochoric Process


● In this process, no heat exchange takes place
● This is the process in which volume is kept constant.
between the system and the surroundings, i.e. dQ = 0 ● dQ = nCV dT , molar heat capacity for isochoric
or Q = constant proces isCV .
p
● From dQ = nCdT , Cad = 0 as dQ = 0, i.e. molar heat A
capacity for an adiabatic process is zero.
● From first law, dU = - dW i.e. work done by the
system is equal to decrease in internal energy. When
B
a system expands adiabatically, work done is V
positive and hence internal energy decreases i.e. the
system cools down and vice-versa. ● Volume is constant, so dW = 0
● Gas equation is pV g = p ● From the first law of thermodynamics, dQ = dU . As
constant. Gas equation heat is supplied to the system, internal energy
can be written in many Isothermal increases and hence the temperature increases.
other ways as TV g - 1 = ●
p
Gas equation is = constant.
constant or p 1- gT g = Adiabatic T
Cp ● p-V curve is a straight line parallel to the
constant, where g = .
CV V pressure axis.
● Work done in an adiabatic process is 6. Cyclic Process
nR(T 1 - T 2 ) p1V 1 - p2V 2
W = = In cyclic process,
g -1 g -1
final state of gas is ive tiv
e
● Using pV = nRT and pV g = constant, we can have same as the initial p o sit ega
P p N
= =
dp gp state of gas. For one W W
=- i.e. slope of p - V curve in an adiabatic
dV V complete cycle
process is g times the slope of p - V curve in process, dU = 0. V V
isothermal process. Work done by the gas
Cyclic process diagram

is equal to the area enclosed by the p -V curve.


4. Isobaric Process Here, net heat in the process is given by
● This is the process in which press ure is kept Net heat = Total heat supplied + Total heat rejected
constant.
Efficiency of cyclic process
● Molar heat capacity of the process is C p and Total work done in cycle
dQ = nC pdT h= ´ 100%
Total heat supplied

@iitjeehelps
198 SELF STUDY GUIDE BITSAT

(i) Clausius statement It is impossible to make any


Heat Engine such machine that can transfer heat from an object
It is a device which is used to convert heat energy into with low temperature to an object with high
mechanical energy, in a cyclic process. temperature without any external source.
Components of a heat engine are (ii) Kelvin statement It is impossible to obtain work
continuously by cooling an object below the
(i) A body at higher temperature T 1 from which heat is
extracted is called the source. temperature of its surroundings.
(ii) Body of the engine contains a working substance (iii) Kelvin-Planck statement It is impossible to
which performs mechanical work, when heat is construct any such machine that works on a cyclic
supplied to it. process and absorbs heat from a source, converts all
(iii) A body at lower temperature T 2 to which heat can be that heat into work and rejects no heat to sink.
rejected is called the sink.

Efficiency of Heat Engine Carnot Cycle


Carnot engine is an ideal reversible heat engine that
Efficiency of heat engine (h) is defined as the fraction of
operates between two temperatures T 1 (source) and T 2
total heat supplied (Q1 ) to the engine which is converted
(sink).
into work (W ).
As the engine works, the working substance of the engine
Mathematically, undergoes a cycle, known as Carnot cycle.
W Q - Q2 Q T é Q2 T 2 ù
h= or h = 1 =1- 2 =1- 2 êQ Q = T ú p
Q1 Q1 Q1 T1 ë 1 1û A(p1,V1,T1)

Adiabatic compression
Iso
exp therm
an a
sio l
Refrigerator n
B(p2,V2,T1)
Refrigerator is a device which takes heat from a cold body,
work is done on it and the work done together with the heat Adiabatic
absorbed is rejected to the source. An ideal refrigerator can expansion
be regarded as an ideal heat engine working in the reverse
D(p4,V4,T2)

direction. Isoth
e C(p3,V3,T2)
com rmal
pres
sion
Coefficient of Performance of a
Refrigerator (b) E F
V
G H
It is defined as the ratio of quantity of heat removed per p -V plot for a Carnot engine
cycle (Q2 ) to the work done on the working substance per
cycle to remove this heat.
Q
b= 2 =
Q2
or b =
T2
=
1-h Net Work done by the Gas per Cycle
W Q1 - Q2 T1 - T2 h During the four strokes,W 1 ,W 2 are the work done by the gas
andW3 ,W 4 are the work done on the gas. Therefore, the net
work performed by the engine,
Second Law of W = W 1 + W 2 - W3 - W 4
Thermodynamics = Area ABGE + Area BCHG - Area CDFH – Area ADEF
Second law of thermodynamics tells us whether in a given = Area ABCD
process conservation of energy will actually take place or Thus, net work done by the engine during one cycle is equal
not. This law gives the direction of heat flow. It can be stated
to the area enclosed by the indicator diagram of the cycle.
as follows

@iitjeehelps
Practice Exercise
1. A boy weighing 50 kg eats bananas. The energy 8. The work done for the cycle shown in given figure,
content of banana is 1000 cal, if this energy is used to will be
lift the boy from ground, then the height through which p a
50
he is lifted, is
a. 8.57 m b. 10.57 m c. 6.57 m d. 5.57 m 40

2. 1 kg mass of water, boils at standard atmosphere N/m2 30


pressure, turns completely into saturated vapour. 20 b
c
Assume saturated vapour to be an ideal gas. Find the 10
increment of internal energy of the system and 0
internal work done. 1 2 3 4 5
(Given, specific latent heat of vaporisation of
V(m3)
water = 2250 kJ/kg)
a. 3 ´ 106 J b. 2.1 ´ 106 J c. 3.2 ´ 106 J d. 4.2 ´ 106 J a. 45 J b. 54 J c. 22.5 J d. 32.5 J
3. An ideal gas is heated at constant pressure and
9. An ideal monoatomic gas is (2p,V) (2p,2V)
absorbs amount of heat Q. If the adiabatic exponent is
taken around the cycle A B
g, then find the fraction of heat absorbed in raising the
ABCDA as shown in the p -V
internal energy and performing the work is diagram. The work done p
1 1 2 2
a. 1 - b. 1 + c. 1 - d. 1 + during cycle is given by
g g g g D C
1 (p,V) (p,2V)
4. The latent heat of vaporisation of water is 2240 J. If a. pV b. pV
2
the work done in the process of vaporisation of 1 g is V
c. 2pV d. 4pV
168 J, then increase in internal energy is
a. 2408 J b. 2240 J c. 2072 J d. 1904 J 10. A balloon that is initially flat, is inflated by filling it from
a tank of compressed air. The final volume of the
5. The molar heat capacity in a process of a diatomic balloon is 5 m 3 . The barometer reads 95 kPa. The
gas if it does a work of Q / 4, when Q amount of heat is
work done in this process is
supplied to it is
2 5 10 6 a. 475 ´ 105 J b. 4.75 ´ 107 J
a. R b. R c. R d. R c. 4.75 ´ 103 J d. 4.75 ´ 105 J
5 2 3 7
6. Starting with the same initial conditions, an ideal gas 11. What work will be done, when 3 moles of an ideal gas
expands from volume V1 to V2 in three different ways. are compressed to half the initial volume at a constant
The work done by the gas is W1 if the process is purely temperature of 300 K?
isothermal, W 2 if purely isobaric and W 3 if purely a. -5188 J b. 5000 J c. 5188 J d. - 5000 J
adiabatic. Then,
a. W2 > W1 > W3 b. W2 > W3 > W1 12. A gas is contained in a cylinder and expands
c. W1 > W2 > W3 d. W1 > W3 > W2 according to the relation pV 1. 3 = Constant. The initial
pressure and initial volume of the gas are 30 atm and
7. Calculate the work done (W AB ) by the gas, if 5 moles 30 L respectively. If the final pressure is 15 atm, then
of an ideal gas is carried by a quasi state isothermal calculate the work done on the face of piston by the
process at 500 K to twice its volume. pressure force of the gas.
a. 5 ´ 104J b. 4.36 ´ 104 J
p A
pA c. 3 ´ 104 J d. 4 ´ 104J

13. A vertical cylinder divided into two parts by a


pB frictionless piston in the ratio of 5 : 4. The piston is free
B
to slide along the length of the vessel and length of the
vessel is 90 cm. Each of the two parts of the vessel
contains 0.1 mole of an ideal gas and the temperature
VA VB V of gas is 300 K. Calculate the mass of the piston.
a. 1500 J b. 14407 J c. 13380 J d. 14890 J a. 14 kg b. 12.7 kg c. 16 kg d. 15 kg

@iitjeehelps
200 SELF STUDY GUIDE BITSAT

14. During an isothermal expansion of an ideal gas, 21. An ideal gas is taken from the state A (pressure p,
a. its internal energy decreases volume V ) to the state B (pressure p /2, volume 2V ),
b. its internal energy does not change a long straight line path in the p-V diagram. Select
c. the work done by the gas is equal to the quantity of the correct statement from the following.
heat absorbed by it a. The work done by the gas in the process A to B,
d. both (b) and (c) are correct exceeds the work that would be done by it, if system
15. In the given graph, adiabatic and isothermal curves were taken along the isotherm
are shown. Then, b. In the T -V diagram, the path AB becomes a part of
a hyperbola
B A c. In the p-T diagram, the path AB becomes a part of a
hyperbola
d. In going from A to B, the temperature T of the
p gas first decreases to a minimum value and then
increases
22. During adiabatic change, specific heat is
a. zero b. greater than zero
V
c. less than zero d. infinity
a. the curve A is isothermal b. the curve B is isothermal
c. the curve A is adiabatic d. the curve B is adiabatic 23. Molar heat capacity is directly related to
a. temperature b. heat energy
16. 0.2 moles of an ideal gas, c. molecular structure d. mass
is taken around the cycle b
abc as shown in the figure. 24. If at NTP, velocity of sound in a gas is 1150 m/s, then
The path b -c is adiabatic find out the rms velocity of gas molecules at NTP.
process, a -b is isovolumic p (Given, R = 8.3 J/mol/K, C p = 4.8 cal/mol/K)
process and c -a is isobaric a. 1600 m/s b. 1532.19 m/s
process. The temperature a c c. 160 m/s d. 16 m/s
at a and b are Ta = 300 K 25. What is the molar heat capacity for the process, when
and Tb = 500 K and 10 J of heat added to a monoatomic ideal gas in a
pressure at a is 1 V
process in which the gas performs a work of 5 J on its
atmosphere. Find the volume at c. surrounding?
Cp 5
(Given, g = = , R = 8.205 ´ 10-2 L /atm/mol-K) a. 2R b. 3R c. 4R d. 5R
CV 3
26. A gaseous mixture consists of m1 = 2 moles of oxygen
a. 6.9 L b. 6.68 L c. 5.52 L d. 5.82 L
and m 2 = 3 moles of carbon dioxide. Assume gases to
17. At 27°C a motor car tyre has pressure of 2 atmosphere. Cp
Find the temperature, if the tyre suddenly bursts. be ideal. Calculate, g = for the gaseous mixture.
CV
(Given, g air = 14
. )
a. 2.33 b. 1.33 c. 0.33 d. 3.33
a. 24.1 K b. 250 K c. 246.1K d. 248 K
18. In an adiabatic expansion, a gas does 25 J of work 27. Find the molar specific heat of mixture at constant
while in an adiabatic compression 100 J of work is volume, if one mole of a monoatomic gas is mixed
done on a gas. The change of internal energy in the with three moles of a diatomic gas.
two processes respectively are a. 3.33R b. 2.25R c. 115
. R d. 6.72R
a. 25 J and -100 J b. -25 J and 100 J 28. If a gas heated at constant pressure, then what
c. -25 J and -100 J d. 25 J and 100 J percentage of total heat supplied is used up for
19. The molar heat capacity of oxygen gas at STP is 4
external work? (Given, g for gas = )
nearly 2.5R. As the temperature is increased, it 3
gradually increases and approaches 3.5 R. The most a. 25% b. 50% c. 75% d. 80%
appropriate reason for this behaviour is that at high 29. One mole of a gas isobarically heated by 40 K
temperature, receives an amount of heat 1.162 kJ. What is the ratio
a. oxygen does not behave as an ideal gas of specific heats of the gas?
b. oxygen molecules dissociate in atoms a. 1.7 b. 1.4 c. 1.3 d. 1.5
c. the molecules collide more frequently
d. molecular vibrations gradually become effective 30. In a process PT = constant, if molar heat capacity of a
20. p-V diagram of a diatomic gas is a straight line gas is C = 37.35 J/mol-K, then find the number of
passing through origin. The molar heat capacity of the degrees of freedom of molecules in the gas.
gas in the process will be a. n = 10 b. n = 5
a. 4 R b. 3 R c. 4 R/3 d. 2.5 R c. n = 6 d. n = 7

@iitjeehelps
LAWS OF THERMODYNAMICS 201
31. If in an adiabatic process, the pressure is increased by t + Dt , its moment of inertia also increases from I to
2 æ Cp 3 ö (I + DI ). If coefficient of linear expansion of the body is
%, then volume decreases by ç Assume = ÷ DI
3 è CV 2ø a, then find the ratio .
4 2 9
I
a. - % b. % c. 4% d. % Dt 2Dt
9 3 4 a. b. c. a Dt d. 2a Dt
t t
32. A monoatomic ideal gas is expanded adiabatically to n
37. A Carnot engine, whose efficiency is 40%, takes in
times of its initial volume. The ratio of final rate of
heat from a source maintained at a temperature of
collision of molecules with unit area of container walls
to the initial rate will be 500 K. It is desired to have an engine of efficiency
a. n -4/ 3 b. n 4/ 3 c. n 2/ 3 d. n -5/ 3 60%. Then, the intake temperature for the same
exhaust (sink) temperature must be
33. In the case of solid, number of degrees of freedom is a. efficiency of Carnot engine cannot be made larger
a. 3 b. 5 c. 6 d. 7 than 50%
34. A given quantity of an ideal gas is at the pressure p b. 1200 K c. 750 K d. 600 K
and the absolute temperature T. The isothermal bulk 38. The inside and outside temperatures of a refrigerator
modulus of the gas is are 273 K and 303 K respectively. Assuming that
2 3 refrigerator cycle is reversible, for every joule of work
a. p b. p c. p d. 2p
3 2 done, the heat delivered to the surrounding will be
35. A non-conducting cylindrical vessel of cross-sectional a. 10 J b. 20 J c. 30 J d. 50 J
area 29.15 ´ 10-5 m 2 and volume 103 m 3 contains 39. Find coefficient of performance, if in a mechanical
10-4 kg oxygen in it. The cylinder is closed by a refrigerator, the lower temperature coils of a
frictionless non-conducting piston which is free to evaporator are at -23° C and compressed gas in the
move in vertical direction. If the piston is slightly condenser has a temperature of 77° C.
depressed and released, it will execute simple a. 70% b. 20% c. 0.23% d. 2.5%
harmonic motion, then find its frequency. (Assume
Cp 40. A refrigerator works between the temperature of
= 14
. for oxygen and ignore atmosphere pressure) melting ice and room temperature (17°C). The amount
CV
of energy (in kWh) that must be supplied to freeze
a. 0.13 s b. 0.318 s c. 0.23 s d. 0.36 s 1 kg of water at 0° C is
36. At a temperature t, the moment of inertia of a body is I. a. 1.4 b. 1.8
When the temperature of the body is increased from c. 0.058 d. 2.5

BITSAT Archives
1. The heat energy [2014] 4. A sample of ideal monoatomic gas is taken round the
a. is a state variable cycle ABCA as shown in the figure. The work done
b. does not depend on the state of the system during the cycle is [2012]
c. is equal to internal energy of the system
d. None of the above B(4p, 3V)

2. In a thermodynamic process, the pressure of a fixed


mass of a gas is changed in such a manner that the
p
gas releases 20 J of heat and 8 J of work is done on
the gas. If the initial internal energy of the gas was
30 J, then the final internal energy will be [2014]
A(p, V) C(p, 3V)
a. 2 J b. 42 J c. 18 J d. 58 J
V
3. A thermodynamical system is changed from state
( p1, V1) to ( p 2, V2 ) by two different process, the quantity
a. 3 pV b. zero
which will remain same will be [2013]
c. 9 pV d. 6 pV
a. DQ b. DW c. DQ + DW d. DQ - DW

@iitjeehelps
202 SELF STUDY GUIDE BITSAT

5. An ideal refrigerator has a freezer at a temperature of 8. Initially two gas samples 1 and 2 are at the same
-13 ° C. The coefficient of performance of the engine is condition. The volume of the two are halved, one
5. The temperature of the air (to which heat is isothermally and the other adiabatically. What is the
rejected) will be [2011] relation between the final pressures p1 and p 2? [2006]
a. 325°C b. 325 K c. 39°C d. 320°C a. p1 = p 2
6. In the adiabatic compression, the decrease in volume b. p1 > p 2
is associated with [2008] c. p 2 > p1
a. increase in temperature and decrease in pressure d. Cannot be determined
b. decrease in temperature and increase in pressure
9. A can is taken out from a refrigerator at 0°C. The
c. decrease in temperature and decrease in pressure
atmospheric temperature is 25°C. If t1 is the time taken
d. increase in temperature and increase in pressure
to heat from 0°C to 5°C and t 2 is the time taken from
7. Which of the following is true in the case of an 10°C to 15°C, then [2006]
Cp a. t1 > t 2
adiabatic process, where g = ?
CV [2008] b. t1 < t 2
a. p1 - gT g = constant b. p1T 1 - g = constant c. t1 = t 2
c. pT g = constant d. p gT = constant d. there is no relation

Answer with Solutions


Practice Exercise From first law of thermodynamics, dU = dQ - dW
Q 3Q
1. (a) or dU = Q - =
4 4
2. (b) We know, DQ = DU + DWex + DWin Now, molar heat capacity,
DU = 0, DQ = mL dQ Q 5 QR 10
C= = = = R
DWex = p 0(Vvapour - Vwater ) = p 0Vvapour dT æ dU ö æ 3Q ö 3
2ç ÷ 2 ç ÷
m è5R ø è 4 ø
PV = RT
M
6. (a)
m p
Þ DWex = p 0Vvapour = RT Isobaric (W2)
M
m
DWin = DQ - DWex = mL - RT
M
é RT ù Isothermal (W1)
= m êL - = 2.1 ´ 106J
ë M úû Adiabatic (W3)
3. (a) Heat absorbed by the system at constant pressure V
Q - nCp DT and change in internal energy
DU = nCV DT It is clear from the graph that on the basis of area under
W = Q - DU p-V curve for different processes W2 > W1 > W3 .
W Q - DU DU 7. (b)
Fraction = = = 1-
Q Q Q 8. (a) The area of p - V diagram gives work.
DU æ 1ö = 45 J = work done
= 1- = ç1 - ÷
Q è gø 9. (b)W = ( 2V - V )( 2p - p ) = pV
4. (c) Given, L = 2240 J, m = 1 g 10. (d)
dW = 168 J 11. (a) As process is isothermal
From first law of thermodynamics, æV ö
dU = 2072 J W = 2.303 nRT log ç 2 ÷
è V1 ø
æ5 ö
5. (c) dU = CV dT = ç R ÷ dT æ 1ö
è2 ø W = 2.303 ´ 3 ´ 8.315 ´ 300 ´ log ç ÷
è 2ø
2(dU )
Þ dT =
5R W = - 5188 J

@iitjeehelps
LAWS OF THERMODYNAMICS 203
V2
12. (b)W = ò pdV and pV 1. 3 = C ,p = CV -1. 3 20. (b) As p-V diagram is a straight line passing through
V1 origin, therefore, p µ V or pV -1 = constant
V2 -C 0. 3 V2 In the process, pV x = constant, molar heat capacity is
W = ò CV -1. 3dV = [V ]V1
V1 0.3 R R
given by C= +
C = p1V11. 3 = p 2V21. 3 g - 1 1- x
p 2V2 - p1V1 where, x = - 1and g = 14
. for diatomic gas
W = R R 5 R
-0.3 C= + = R+
1/1. 3 . - 1 1 - ( -1) 2
14 2
æp ö
Final volume, V2 = V1ç 1 ÷ C = 3R
è p2 ø
1æ pö 3
21. (a)W = ç p + ÷ V = pV = 0.75pV
= 30 ´ 10-3( 2)0. 77 = 30 ´ 10-3 ´ 171
. 2è 2ø 4
p
. ´ 10-3 m3
V2 = 513
p0
. ´ 10-3 - 30 ´ 105 ´ 30 ´ 10-3
15 ´ 105 ´ 513 p
W =
0.3
p/2
W = 4.36 ´ 104 J
13. (b)
V
14. (d) In isothermal process, temperature remains constant V 2V V0
Q DU = nCV DT = 0 (Q DT = 0) æ 2V ö
Wiso = RT ´ 2.3026 log10 ç ÷ = 0.693 pV
According to the first law of thermodynamics, èV ø
DH = DU + DW Thus, statement (a) is correct.
\ DH = DW (Q DU = 0)
22. (a)
Hence, (d) is correct.
23. (c) Since, Cp - CV = R
15. (b, c)
Cp
16. (b) From a - b, volume is constant = g or Cp = gCV
p a pb CV
=
Ta Tb Q Cp - CV = R
500 5 CV ( g - 1) = R
pb = ´ 1 = atm R
300 3 \ CV =
For b - c , adiabatic process ( g - 1)
g -1 5/ 3 - 1 gR
Tc æ pc ö
2/ 5 Also, Cp =
g é 1 ù 5/ 3 æ 3ö ( g - 1)
=ç ÷ =ê =ç ÷
Tb è pb ø ë 5 / 3 úû è5ø
But R is universal constant. The value of g depends upon
Also, pbVbg = pcVcg molecular structure.
1/g 8.3
æp ö 24. (b) R = cal/mol/K
Vc = (Vb ) ç b ÷ 4.2
è pc ø
æ 8.3 ö
3/5 CV = Cp - R = ç 4.8 - ÷ = 2.824
æ 5/3 ö è 4.2 ø
= (4.928)ç ÷ = 6.68 litre
è 1ø Cp 4.8
g= = = 169
.
17. (c) CV 2.824
18. (b) In adiabatic process, DH = 0 æ 3ö
Since, V = ç ÷ ×Vs = 1532.19 m/s
\For expansion, ègø
DH = DU + DW 25. (b)
0 = DU + 25
26. (b) Let n = n1 + n 2
U = - 25 J
nCp = n1Cp1 + n 2Cp 2
For adiabatic compression, n1Cp1 + n 2Cp 2
DH ¢ = DU ¢ + DW ¢ Cp =
n
0 = DU ¢ - 100 n1CV1 + n 2CV2
Similarly, CV =
DU = 100 J n
Hence, (b) is correct. Cp n1Cp1 + n 2Cp 2
g= =
19. (a) CV n1CV1 + n 2CV2

@iitjeehelps
204 SELF STUDY GUIDE BITSAT

g1R g 2R Cp 3
But Cp1 = and Cp 2 = 31. (a) =g=
( g1 - 1) ( g 2 - 1) CV 2
R R Let k = constant
also CV1 = and CV2 =
g1 - 1 g2 - 1 We know for an adiabatic process, pV g = k
é gR ù é g R ù pV 3/ 2 = k
n1ê 1 ú + n 2 ê 2 ú
( g - 1) ë ( g 2 - 1) û
g= ë 1 û 3
log p + logV = log k
n1R n 2R 2
+
( g1 - 1) ( g 2 - 1) Dp 3 DV DV 2 Dp
+ =0 Þ =-
After solving, g = 133
. p 2 V V 3 p
27. (b) Let Cvm = molar specific heat of mixture at constant DV æ 2 ö æ Dp ö
´ 100 = ç - ÷ ç ´ 100÷
volume. V è 3øè p ø
For mixture, (n1 + n 2 )Cvm = n1CV1 + n 2CV2 2 2 4
=- ´ =- %
3R 3 3 9
Here, CV1 = , n1 = 1mole
2 32. (d) The rate of collision of molecules with unit area of
5R walls of container is proportional to pressure.
CV2 = , n 2 = 3 moles
2
p1V1g = p 2V2g
3 5
(1 + 3)Cvm = 1 ´ R + 3 ´ R
2 2 V2 = nV1
9 g g
Cvm = R = 2.25R p 2 æ V1 ö æV ö 1
4 \ =ç ÷ =ç 1 ÷ = g
p1 èV2 ø è nV1 ø n
28. (a) Heat supplied, dQ = nCpdT
p 2 = p1n -g
Heat used for work = dW = nRdT
2 5
dW R Cp - CV C For monoatomic gas, g = 1 + =
= = = 1- V 3 3
dQ Cp Cp Cp
p 2 = p1n -5/ 3
æ 1ö æ 3ö
= ç1 - ÷ = ç1 - ÷ 33. (c)
è gø è 4ø
Dp
1 34. (b) b =
= ´ 100 = 25% DV
4 V
29. (b) \In isothermal process ( p + Dp )(V - DV ) = pV
30. (b) pV = nRT and pT = k = constant or pV - pDV + VDp - DpDV = pV
k But DpDV may be neglected.
Þ p=
T Dp
\ VDp = pDV or =p
nR 2 DV
V = T ,
k V
2nRTdT mg
dV = 35. (b) In equilibrium state, p =
k A
T
For n = 1,dU = ò PdV Also Poisson’s law, pV g = constant as process is
0
Tk
adiabatic.
2R
=ò TdT = 2RdT When it is depressed, volume will decrease by DV and
0Tk
R R pressure increase by Dp
dW = nCV dT = n dT = dT pV g = ( p + Dp )(V - DV )g
g -1 g -1
According to law, DQ = DU + DW æ DV ö
Dp = gp ç ÷
è V ø
R é 2g - 1ù
= dT + 2RdT = R ê údT When piston is released, restoring force will act on it.
g -1 ë g -1û
é 2g - 1ù æ DV ö
dQ F = - Dp( A ) = - gpA ç ÷
C= = Rê ú è V ø
dT ë g -1û
7 2 7 DV = An
g= and 1 + =
5 n 5 -gpA 2
F = ×n
n =5 V

@iitjeehelps
LAWS OF THERMODYNAMICS 205
F æ -gpA 2 ö I + DI = S [mr 2(1 + 2aDt )] = I (1 + 2aDt )
Acceleration of piston, a = =ç ÷x = - w2x
m è mV ø DI
= 2aDt
I
w 1 ggA é mg ù
n= Þn= êëQ p = A úû 37. (c)
2p 2p V
n = 0.318 s 38. (a)
36. (d) I = S mr 2 39. (c)
40. (c)
\ I + DI = Sm [r (1 + aDt )] 2

BITSAT Archives
1. (b) Heat energy is not a state variable, it is just energy in It can also be written as
transition. é nRT ù
TV g - 1 = constant ê as p =
2. (c) Given, Q = - 20 J, W = - 8 J ë V úû
Using Ist law, Q = DU + W Þ - 20 = DU + ( - 8) é nRT ù
and p1-gT g = constant ê as V =
Þ DU = - 12 J Þ Uf - Ui = - 12 J ë p úû
Þ Uf - 30 = - 12 J Þ Uf = 18 J 8. (c) For isothermal process
3. (d) For all processes, change in internal energy pV = K
DU = ( DQ - DW ) does not change. It depends only on K
initial and final states. Þ p1 = …(i)
V
4. (a) The work done = area of p-V graph K
= area of triangle ABC
1 = V = 2K
= ´ 3p ´ 2V = 3pV 2
2
For adiabatic process
5. (c) Given,T2 = - 13°C = 260 K and b = 5
pV g = K (constant)
T2
b= K
T1 - T2 \ p2 = …(ii)
260 Vg
5=
T1 - 260 =
K
= K ( 2g )
g
T1 = 312 K or T1 = 312 K = 39°C æV ö
ç ÷
6. (d) In adiabatic compression, temperature and hence è 2ø
internal energy of the gas increases. In compression From Eqs. (i) and (ii), we have
pressure will increase.
p 2 > p1
7. (a) For adiabatic change, equation of state is
9. (b) According to Newton’s law of cooling, t1 < t 2
pV g = constant

@iitjeehelps
18
Transmission
Modes of Heat Transfer
The heat can be transferred within the body and from one body to the other body through the
following modes
(i) Conduction (ii) Convection (iii) Radiation

Conduction
This process of heat transfer from one end of rod to other end, which does not involve any mass
movement of molecules is called conduction.

Thermal Conductivity
The amount of heat transmitted through a conductor is given by
dl
KA∆T∆ t
∆Q =
l T1 A T2

or in differential form, we can write l


dQ dT
= − KA
dt dx
dT
(where, = temperature gradient, A = area of cross-section, ∆T = temperature difference,
dx
∆t = time elapsed, K = thermal conductivity and l = length)
The rate of transmission of heat by conduction is given by
∆Q KA∆T
H= =
∆t l
The unit of thermal conductivity is Wm −1K −1 .
dQ dT dm
We can write = mc =±L
dt dt dt

Convection
Convection is the mode of heat transfer by actual motion of the matter (or particles of matter).
Convection is not possible in solids and can takes place in fluids only.

@iitjeehelps
TRANSMISSION 207

Radiation where, Q represents the energy of thermal radiation.


Conduction and convection require some material as a Qabsorbed
Absorptivity or absorptive power, a =
transport medium. However, the third mechanism for heat Qincident
transfer needs no medium. It is called radiation. The Q reflected
electromagnetic waves which require no medium for its Reflectivity, r =
Qincident
propagation is the carrier of this radiant energy.
Q transmitted
Transmissivity, t=
Some Common Terms and Points Qincident
The thermal radiation emitted by a body comprises of all Qs Qv Qt
+ + = a +r +t = 1
the wavelengths; intensities of radiation corresponding to Q Q Q
different wavelengths are different. For a perfect black body, a = 1 , r = t = 0
● Absorptive power (a) It is defined as the ratio of the For a perfect reflector, a = t = 0, r = 1
radiant energy absorbed by it in a given time to the total For a perfect transmitter, a = r = 0, t = 1
radiant energy incident on it in the same interval of time
energy absorbed
a=
energy incident Perfectly Black Body
As a perfectly black body absorbs all radiations incident A perfectly black body is the one which completely absorbs
on it, the absorptive power of a perfectly black body is the radiations of all the wavelengths that are incident on it.
maximum and unity. Thus, absorbing power of a perfectly black body is
● Spectral absorptive power (aλ ) The spectral absorptive 1 (i.e. a = 1).
power is the ratio of radiant energy absorbed by a No material body is a perfectly black body. However, 1amp
surface to the radiant energy incident on it for a black and platinum black are nearly perfectly black bodies.
particular wavelength λ. It may have different values for
different wavelengths for a given surface. The spectral
absorptive power aλ is related to absorptive power through Kirchhoff ’s Law

the relation a = ∫0 aλ dλ . It states that the ratio of emissive power to absorptive
power is the same for all the bodies at a given temperature
● Emissive power (e) For a given surface it is defined as and is equal to the emissive power of a black body at that
the radiant energy emitted per second per unit area of temperature.
the surface. It is the total amount of energy radiated by a e (body)
body per second per unit area of surface Thus, = E (black body) = constant
a (body)
1 ∆Q
i.e. e= So, a body with high emissive power, should also have high
A ∆t
● Spectral emissive power (e λ ) It is emissive power for a absorptive power and vice-versa.

particular wavelength λ. Thus, e = ∫ e λ dλ Absorptive power is dimensionless quantity but emissive
0 power is not.
● Emissivity ( ε ) Emissivity of a body at a given
temperature is defined as the ratio of the total emissive
power of the body (e) to the total emissive power of a Stefan-Boltzmann Law
perfect black body (E) at that temperature. It states that “energy emitted per unit area per unit time of
e perfectly black body is proportional to T 4 ,” where T is
i.e. ε=
E temperature on absolute scale.
i.e. U(J/sm 2 ) = σT 4
Absorption, Reflection and Here, σ is Stefan’s constant.
Measured value of σ is 5.67 × 10−8 Wm −2 K −4 .
Transmission
If A is the area, then radiant energy per unit time of perfectly
When radiations are incident on a surface, then three things
happenA part of the radiation is absorbed, some is black body is U (J/s) = σAT 4 .
reflected back, and remaining is transmitted. For a body which is not perfectly black, energy radiated per
Qincident = Qabsorbed + Q reflected + Q transmitted second is given by U = e σ AT 4 .

@iitjeehelps
208 SELF STUDY GUIDE BITSAT

where , e is emissivity which is equal to absorptive power a The area, in accordance with the Stefan’s law of radiation, is
of the body. (i.e. e = a ) directly proportional to the fourth power of the
temperature.
Also, if a body is placed where surrounding temperature is ∞
T0 , then energy absorbed by the body will be given by Emissive power, E = ∫ E λd λ = Area under E λ- λ graph = σT 4
0
U = e σ AT04 From E λ λ graph, we find that as the temperature T of a
Thus, net loss of thermal power is given by black body increases, the wavelength λ m corresponding to
the maximum emission decreases such that
∆U = U − U 0 = eσA(T 4 − T04 )
λm ∝ or λ mT = constant = b
T
Wien’s Displacement Law where, b is known as the Wien’s constant and its value is
× − mK.
The black body spectrum is a continuous spectrum as
shown in the figure. At a given temperature, initially the
intensity of thermal radiation increases with an increase in
wavelength and reaches a maximum value at a particular
Newton’s Law of Cooling
wavelength λ m . On increasing the wavelength beyond λ m , According to the Newton’s law of cooling, the rate of
the intensity of radiation E λ starts decreasing. cooling of a body is directly proportional to the
temperature difference between the body and the
surroundings, provided the temperature difference is small.
λ
T4 > T3 dT dT
Mathematically, ∝(T − T0 ) or − = k(T − T0 )
dt dt
T3 > T2
where, k is a constant.
T 2 > T1
Newton’s law of cooling is a special case of Stefan’s law
under the condition that the temperature difference is
small enough.
1 If a body cools by radiation through a small temperature
λ difference from T 1 to T 2 in a short time t when the
λm4 λm λm λm surrounding temperature is T0 , then
3 2 1

dT T 1 − T 2 T + T 2 
The total area under E λ -λ curve gives the total intensity of = =k  1 − T0 
radiation at that temperature. dt t  2 

Practice Exercise
1. One end of a metal rod is kept in steam. In steady 3. Cylindrical rod of copper of length 2 m and
 d θ cross-sectional area 2 cm 2 is insulated at its curved
state, the temperature gradient  
 dx  surface. The one end of rod is maintained in steam
a. may be variable chamber and other is maintained in ice at 0°C (The
b. must be constant thermal conductivity of copper is 386J/m-s °C).
c. must be variable Find the temperature at a point which is at a distance
d. None of the above of 120 cm from the colder end.
a. 80°C
2. Calculate the daily loss of energy by the earth, if the b. 60°C
temperature gradient in the earth's crust is 32°C per c. 50°C
km and mean conductivity of the rock is 0.008 of CGS d. None of the above
unit. (Given, radius of earth = 6400 km)
a. 1040cal 4. In previous problem, the amount of ice melts per
b. 1030cal second is
c. 1018cal a. 48.25 g/s b. 80 g/s
d. 1010cal c. 20.8 g/s d. None of these

@iitjeehelps
TRANSMISSION 209
5. Cylindrical copper rod of length 1 m and a cylindrical 11. A compound rod is formed of a steel core of diameter
steel rod of length 1.5 m are joined together end to 1cm and outer casing is of copper, whose outer
end. The cross-sectional area of each rod is 3.14 cm . diameter is 2 cm. The length of this compound rod is
The free ends of steel rod and copper rods are 2 m and one end is maintained as 100°C, and the end
maintained at 0°C and 100°C, respectively. The is at 0°C. If the outer surface of the rod is thermally
surfaces of rods are thermally insulated. Find the insulated, then heat current in the rod will be
temperature of copper-steel junction. (Given, thermal (Given, thermal conductivity of steel = 12 cal/m/K/s,
conductivity of steel = 46 J/m-s °C and the thermal thermal conductivity of copper = 92 cal/m/K/s)
conductivity of copper = 386 J/m-s °C) a. 2 cal/s b. 1.13 cal/s c. 1.42 cal/s d. 2.68 cal/s
a. 40°C b. 60°C
12. An aluminium rod of length L and cross-sectional area
c. 93°C d. 80.64°C
2A is joined with a copper rod of length 2L and area of
6. In previous problem, the rate of loss of heat at free cross-section is A, as shown in figure. Find the
end of copper is temperature of aluminium-copper junction in the
a. 2 J/s steady state of the system.
b. 0.89 J/s Insulating
c. 1.07 J/s material
d. None of the above
7. A block of ice at 0°C rest on the upper surface of the Furnace Air
slab of stone of area 3600 cm 2 and thickness of T1=400°C T2=20°C
10 cm. The slab is exposed on the lower surface to
steam at 100°C. If 4800 g of ice is melted in one hour, Aluminium Copper
then calculate the thermal conductivity of stone.
(Given, thermal conductivity K Al = 240 J/m/s/°C,
(Given, the latent heat of fusion of ice = 80 cal/g) K Cu = 400 J/m/s/°C)
a. K = 2 . 96 × 10−3cal/cm s°C a. 300°C b. 400°C c. 288.24°C d. 275.4°C
b. K = 1. 96 × 103 cal/cm s°C 13. A uniform metal ring with centre C have two points A
c. K = 0 . 96 × − 103 cal/cm s°C and B, such that angle ACB is θ . A and B are
d. None of the above maintained at two different constant temperature.
8. The ice is filled in a hollow glass sphere of thickness If the angle between A and B, i.e. θ = 180°, the rate of
2 mm and external radius 10 cm. This hollow glass heat flow from A to B is 1.2 W, then what will be the
sphere with ice now placed in a bath containing rate, when θ = 90°?
boiling water at 100°C. Calculate the rate at which ice a. 0.6 W b. 0.9 W c. 1.6 W d. 1.8 W
melts. Neglect volume change in ice. 14. What amount of ice at − 14 °C required to cool 200 g of
(Given, thermal conductivity of glass 1.1 W/m/K, latent water from 25°C to 10 CC?
heat of ice = 336 × 103 J/kg) (Given, C ice = 0 . 5 cal/g°C, Lf for ice = 80 cal/g)
m m a. 31 g b. 41 g c. 51 g d. 21 g
a. = 0.01 kg/s b. = 0.002 kg/s
t t 15. A metallic sphere having inner and outer radii a and b
m m
c. = 0.02 kg/s d. = 0.001 kg/s respectively has thermal conductivity
t t
K
K = 0 (a ≤ r ≤ b )
9. 5 cm thick walls of a box like cooler is made of plastic r
2
foam. Its total surface area is 1.5 m . If outside Find the thermal resistance between inner surface
temperature is 30°C, then how much ice melts each and outer surface.
hour inside the cooler to hold its temperature at 0°C. (b − a ) (b 2 − a 2 )
(Given, K for plastic = 0.04 W/mK, L0 = 80 cal/g and a. b.
4 πK 0 4πK 0ab
1 kcal = 4.184 kJ/kcal) 4πK 0
a. 4 kg b. 0.39 kg c. d. None of these
(b − a )
c. 3.9 kg d. 0.2 kg
16. In above problem, if the temperature of inner and
10. If in two identical containers, equal quantities of ice
outer surfaces are θ1°C and θ 2 ° C (θ 2 > θ 2 ). The rate of
melts completely in 30 and 20 minutes respectively,
flow of heat in steady state is
then find the ratio of the thermal conductivities of the 4πK 0(θ1 − θ 2 ) 4πK 0 (b − a )
material of two containers. a. b.
(b − a ) (θ1 − θ 2 )
a. 1 : 1 b. 1 : 2 4πK 0ab (θ1 − θ 2 )
c. 3 : 2 d. 2 : 3 c. d. None of these
(b 2 − a 2 )

@iitjeehelps
210 SELF STUDY GUIDE BITSAT

17. A body initially at 60°C cools to 50°C in 10 minutes. 25. Assuming the sun to be a spherical body of radius R
What will be its temperature at the end of next at a temperature of T K, evaluate the total radiant
10 minutes, if the room temperature is 25°C? Assume power, incident on earth, at a distance r from the sun.
Newton's law of cooling. Given, r 0 is the radius of earth and σ is the Stefan’s
a. 42.85°C b. 45°C c. 40.46°C d. 44.23°C constant.
18. Find the time in which a layer of ice of thickness h will a. 4πr02R 2σT 4 / r 2 b. πr02R 2σT 4 / r 2
grow on the surface of the pond of surface area A,
c. r02R 2σT 4 / 4πr 2 d. R 2σT 4 / r 2
when the surrounding temperature falls to − T °C.
(Assume, K = thermal conductivity of ice, ρ = density 26. What will be the increment in heat energy radiated
of water, L = latent heat of fusion) when the temperature of hot body is raised by 5%?
ρL 2 ρL 2 a. 5% b. 6% c. 11.65% d. 21.55%
a. t = h b. t = h
2KT KT 27. Two spheres of the same material having radii r and
2
ρLh ρLh 2 4r and temperatures 2T0 and T0 respectively. The ratio
c. t = d. t =
3KT 4KT of rate of radiation of energy by the spheres is
19. A black body maintained at a certain temperature a. 1 : 1 b. 1 : 2 c. 2 : 1 d. 3 : 1
radiates heat energy at the rate Q watt. If its surface 28. A sphere, a cube and a thin circular plate all of same
is smoothened so as to lower its emissivity by 10%, material having same mass are initially heated to
what will be the increase in its rate of radiation at 200°C. Which of these will cool fastest?
double the initial temperature? a. Circular plate b. Sphere
a. ( 0.9 × 24 − 1) Q watt b. 0.9 × 24 Q watt c. Cube d. All of these
c. ( 0.9 × 2)4Q watt d. ( 0.9 )4 × 2Q watt 29. A body at a temperature of 727°C and has surface
20. The thermal radiation emitted by a body per second area 5 cm 2, radiates 300 J of energy each minute.
∆H Calculate the emissivity.
per unit area is = kT 4 . If σ is Stefan’s constant,
A ∆t (Given, Boltzmann constant= 5 . 67 × 10−8 Wm 2K 4 )
then body a. e = 0.18 b. e = 0.02 c. e = 0.2 d. e = 0.15
a. may be polished b. may be black body
c. must be black body d. must not be black body 30. Choose the correct relation, when the temperature of
an isolated black body falls from T1 to T2 in time ‘t ’, and
21. If temperature of black body increases from 300 K to assume ‘c ’ to be a constant.
900 K, then the rate of energy radiation increases by  1 1  1 1
a. 81 b. 3 c. 9 d. 2 a. t = c  −  b. t = c  2 − 2 
T2 T1  2
T T1 
22. Three objects coloured black, grey and white can  1 1  1 1
withstand hostile conditions at 2800°C. These objects c. t = c  3 − 3  d. t = c  4 − 4 
T2 T1  T2 T1 
are thrown into furnace where each of them attains a
temperature of 2000°C. Which object will have the 31. The temperature and the surface area of the body are
brightest glow? 227°C and 0.15 m, respectively. If its transmitting
a. The white object power is negligible and reflecting power is 0.5, then
b. The black object calculate the thermal power of the body.
c. All glow with equal brightness (Given, σ = 5 . 67 × 10−4 J/m 2/s/K)
d. Grey object
a. 300 W b. 265.78 W c. 201 W d. 320.89 W
23. The power P is received by a surface at temperature
T0 K from a small sphere at temperature T (T > > T0 ) 32. The surface temperature of the sun is ‘T ’K and the
and at a distance ‘d ’. If both ‘T ’ and ‘d ’ are doubled, solar constant for a plate is ‘ s ’. The sun subtends an
then power received by surface will become angle θ at the planet. Then,
a. P b. 2P c. 4P d. 16P a. s ∝ T 4 b. s ∝ T 2
c. s ∝ θ 2 d. s ∝ θ
24. A cylindrical rod is having temperatures T1 and T2 at its
ends. The rate of flow of heat is Q1. If all the linear 33. When a blackened platinum wire is heated gradually,
dimensions are doubled keeping the temperature it appears
constant, then rate of flow of heat Q2 will be a. first blue, then red and finally white
a. 4 Q1 b. 2 Q1 b. first red, then blue and finally white
Q Q c. first white, then blue and finally red
c. 1 d. 1 d. first red, then white and finally blue
4 2

@iitjeehelps
TRANSMISSION 211
34. Surface temperature of the sun as estimated is 35. The plots of intensity of radiation versus wavelength of
6032.25 K. Find the wavelength at which sun radiates 3 black bodies of temperatures T1, T2 and T3 as shown
maximum energy (Given, Wien's constant = 0.2898 in the figure, then
cm-K) a. T1 > T2 > T3
a. λ m = 5000 Å b. T3 > T2 > T1
b. λ m = 4804.2 Å c. T1 > T3 > T2
c. λ m = 3809.5 Å d. T1 < T3 < T2
d. λ m = 28916
. Å

BITSAT Archives
1. A partition wall has two layers of different materials A 4. In a 10 m deep lake, the bottom is at a constant
and B in contact with each other. They have the same temperature of 4°C. The air temperature is constant at
thickness but the thermal conductivity of layer A is – 4°C. The thermal conductivity of ice is 3 times that of
twice that of B. At steady state, if the temperature water. Neglecting the expansion of water on freezing,
difference across the layer B is 50 K, then the the maximum thickness of ice will be [2010]
corresponding difference across the layer A is [2013] a. 7.5 m b. 6 m c. 6 m d. 2.5 m
a. 50 K
5. Two slabs A and B of equal surface area are placed
b. 12.5 K
one over the other such that their surfaces are
c. 25 K
completely in contact. The thickness of slab A is
d. 60 K
twice that of B. The coefficient of thermal conductivity
2. The rate at which a black body emits radiation at a of slab A is twice that of B. The first surface of slab A
temperature T is proportional to [2012] is maintained at 100°C, while the second surface of
1 slab B is maintained at 25°C. The temperature at the
a. b. T
T contact of their surfaces is [2008]
c. T 3 d. T 4 a. 62.5°C b. 45°C
3. A hot and a cold body are kept in vacuum separated c. 55°C d. 85°C
from each other. Which of the following cause 6. The wavelength of the radiation emitted by a body
decrease in temperature of the hot body? [2011] depends upon [2005]
a. Radiation a. the nature of the surface
b. Convection b. the area of the surface
c. Conduction c. the temperature of the surface
d. Temperature remains unchanged d. All of the above factors

Answer with Solutions


Practice Exercise 3. (b) In steady state, temperature gradient of rod remains
constant.
1. (b) In steady state,
100°C θ°C 0°C
∆H dθ
= − KA
∆t dx
∆H ∆θ
Here, = constant ∴ = constant 80 cm 120 cm
∆t dx
dθ 32 32 100° − θ θ − 0
2. (c) Temperature gradient = ° C/m = 5 ° C/cm ∴ =
dx 1000 10 80 120
Loss of energy by earth is given by  80 
or 100° − θ =  θ
dθ  120
Q = KA × 86400 2 5
dx or 100° − θ = θ or 100° = θ
4 × 22 32 3 3
= 0.008 × × ( 64 × 108 )2 × 5 × 86400 100 × 3
7 10 ∴ θ= = 60° C
. × 1018cal = 1018 cal
= 11 5

@iitjeehelps
212 SELF STUDY GUIDE BITSAT

4. (a)Q Rate of transmission of heat 10. (d)


∆H KA (100 − 0) 386 × 2 × 10−4 × 100 11. (b) Q = Q steel + Q copper
= =
∆t l 2
12π ( 0.5)2 × 10−4 × 100 92π (12 − 0.52 ) × 10−4 × 100
∆mL 386 × 2 × 10−4 × 100 = +
or = 2 2
∆t 2
= 150π × 10−4 + 3450π × 10−4 = 3600 π × 10−4 = 113 . cal/s
∆m 386 × 2 × 10−4 × 100
∴ = (∴L = 80) 12. (c) In steady state condition rate of heat flow in both rods
∆t 2 × 80
must be equal. Let junction temperature = T
= 4.825 × 10−2 kg/s = 48.25 g/s Q K AlAAl( 400° − T ) K CuACu (T − 20° )
H= = =
5. (c) Since, both rods are in series. So, the rate of flow of T LAl LCu
heat through both wires will be same. 240 ( 2A ) ( 400° − T ) 400 × A × (T − 20° )
= =
100°C θ°C 0°C L 2L
Copper Steel T = 288.24° C
13. (c) Let ∆T = temperature difference between the rings
∴ Rate of heat transfer R = total thermal resistance of the ring when θ = 180°.
∆H K1A (100 − θ ) K 2A (θ − 0)
= = The resistance will be R/2, R/2 and both are in parallel, so,
∆t l1 l2 equivalent resistance = R/4
 l  K   1   46  Q ∆T
or 100 − θ =  1   2  θ or 100 − θ =    θ Rate of total heat flow = 1 = 1.2 = ⇒
 l2   K1   1. 5  386 t (R / 4)
∴ θ = 92.64 ~ = 93°C When θ = 90° there are two sections with resistances, R /4
and 3R / 4 in parallel.
6. (b) The rate of flow of heat is equal to rate of loss of heat
3R
of free end of copper So, equivalent resistance =
∆H K1A (100 − θ ) 16
∴ = Q2 ∆T
∆t l1 Rate of heat flow = =
t  3R 
386 × 3.14 × 10−4 (100 − 92.64)  
= = 0.89 J/s  16 
1
KA ∆T 16  ∆T 
7. (a) We know, θ = ⇒ I2 =  
∆x
t 3  R 
Quantity of heat flowing through the stone in one hour I 2 = 16
. W
= 4800 × 80 = 384 × 103 cal ⇒ I 2 = 1.6 W
θ ∆x 1 384 × 103 10 1 14. (a) Heat given by water in cooling from 25° to 10°C.
K = × = × ×
t ∆T A 3600 100 3600 Q1 = (cm ∆T )w = 200 × 1 × ( 25 − 10) = 3000 cal
K = 2.96 × 10−3 cal/cm s °C Heat absorbed by m g of ice at − 14° C
dQ 4πKr1r2∆θ Q 2 = (mc ∆T )ice + mLf + (mc ∆T )w
8. (c) Rate of transmission of heat =
dt (r2 − r1) = m [( 0.5) [ 0 − { − 14}] + 80 + 1(10 − 0)]
dQ 4 × π × 11 . × 0.098 × 0.1 × 100 = 97 m cal
∴ = = 6769.8 ≈ 6770 W
dt 0.002
Q 2 = Q1
m  m dθ
Rate of ice melt = ⇒   L= 97m = 3000 = 30.93 g
t  t  dt
m = 31g
 m 3 m
  × 336 × 10 = 6770 ⇒ = 0.02 kg/s 15. (a) We consider a spherical
t t
∆Q element of radius r and thickness
9. (b) Heat flow into the box = dr. The resistance of considered
∆t element is r
∆Q  ∆ T   30  dr dr dr
= KA   = (0.04) (1.5)   = 36 W dR = = = dr
∆t  ∆x   0.05 KA K 4πr 2 K 0 4πr 2
∆Q = In an hour total energy r2
= 36 × one hour = 36 × 60 × 60 J = 129.6 kJ 1
Q dR = dr
Q ∆Q = mL = m (80 kcal/kg) (4.184 kJ/kcal) 4 πK 0
∆Q 129.6 1 b (b − a )
∴m = =
80 × 4.184 80 × 4.184
kg = 0.39 kg ice will melt The equivalent resistance, R =
4 πK 0 ∫adr = 4 πK 0

@iitjeehelps
TRANSMISSION 213
16. (a) Rate of transmission of heat flow 27. (a) Energy radiated per second P = e σ AT 4
∆H θ1 − θ 2 4π K 0 (θ1 − θ 2 ) 4
 4 πr 2  16T04 r2 16T04
= = ∴
P1 AT
= 1 14 =  = × = 1: 1
∆t R b −a 2 4 2
P2 A2T2  4π( 4r )  T0 16r T04
17. (a) According to Newton’s law of cooling, 28. (a) Radiation is directly proportional to the radiating area
dθ cooling fast.
= − K (θ − θ 0 )
dt For a given mass, area of circular plate is maximum. So,
where, θ = temperature of body it cools first.
θ 0 = temperature of surroundings 29. (a) Q = EAt = eσ (T 4 − T04 ) At
θ 2 dθ 1
where, t = time
∫θ1 θ − θ 0 = − K ∫0dt
T0 = temperature of surrounding when,T > T0
 θ − θ0  Q = eσT 4 At
log  2  = − Kt
 θ1 − θ 0 
300 = e × (5.67 × 10−8 )(1000)4 (5.00 × 10−4 ) ( 60)
 60 − 25 e = 0.18
log   = − K × 10 × 60
 50 − 25 
30. (c) We know, P = 4 πr 2 σ T 4 = mS  −
dT 

Let α = temperature at end of 10 minutes.  dt 
 50 − 25 4 dT dT
log  = − K × 10 × 60 = − πr 3 ρ and R =
 α − 25  3 dt dt
α = 42.85° C 2 1
P ∝ r and R ∝
r
18. (a) T 2 dT 1
19. (a) For a black body, ∫T1 T 4 = (constant) ∫0dt
Rate of radiation, Q = σT 4
 1 1
After smoothing and doubling the temperature = Rate Q t =c  3 − 3
T2 T1 
= 0.9 σ ( 2T )4 = 0.9 × 104 Q
31. (b) Absorbing power of a body is equal to emissivity of the
Charge = ( 0.9 × 24 − 1) Q watt body.
20. (c) 21. (a) 22. (b) And also a + r + t = 1 ⇒ 0 + 0.5 + a = 1
T4 a = 0.5, ε = a = 0.5
23. (c) Radiation ∝ 2
d Q
= ε σ AT 4 = ( 0.5) (5.67) × 10−8 ( 0.15) × ( 273 + 227)4
T02 t
P ∝ 2
d0 Q
= 265.78 W
When T and d are doubled t
( 2)4T 4 4T 4 32. (a) Let radius of sun = R
p′ ∝ 2 20 ∝ 20 ∝ 4P
2 d0 d0 Distance of earth from the sun = d
Therefore, power received by surface will become 4P. Power radiated from the sun = ( 4πR 2 ) σ T 4 = P
KA (T − T2 ) P
24. (b) Initially, Q1 = 1 1 but on doubling all Energy received/area/s = S =
l1 4 πd 2
dimensions l2 = 2l1 and A 2 = 4 A1. T4 R2 1  2R 
2
= 4 πR 2 σ = σT 4 2 = σ T 4  
K A2(T1 − T2 ) K 4 A1(T1 − T2 ) 4 πd 2
d 4  d 
Hence, Q 2 = =
l2 2l1 2R
Angle subtended by sun at earth, α =
K A1(T1 − T2 ) d
=2 = 2Q1
l1 S = constant × T 4 × α 2
25. (b) ∴ S ∝T 4
4
E + ∆E [T + (5 / 100)T ] 33. (b) According to Wien’s law
26. (d) =
E T4 λ mT = constant
 ∆E  4
1 + . )4
 = (1 + 0.05) = (105 When temperature increases, wavelength decreases.
 E 
Due to this the colour of object appears first red (of longer
∆E wavelength) then blue (shorter wavelength) and finally
= 12155
. − 1 = 0.2155
E white (all wavelengths).
∆E
× 100 = 0.2155 × 100 = 21. 55% 34. (b) 35. (c)
E

@iitjeehelps
214 SELF STUDY GUIDE BITSAT

BITSAT Archives
1. (c) LetT be the junction temperature  4 − 0  0 − ( − 4 )
4. (a) KA   = 3KA  
Here, K A = 2KB , T − TB = 50 K  10 − x   x 
TA T TB
– 4°C

A B x
3K
10 0
L
K
10 – x
At the steady state, HA = HB
K A A(TA − T ) KB A(T − TB ) 4°C
⇒ =
L L
⇒ x = 7. 5 m
⇒ 2KB (TA − T ) = KB (T − TB )
5. (a) The temperature at the contact of the surface
T − TB
⇒ TA − T = k d θ + k 2d1θ 2
2 = 2 2 1
k1d 2 + k 2d 2
50
= = 25 K 2k 2d 2 × 100 + 2d 2 × k 2 × 35
2 =
2k 2d 2 + k 2 2d 2
2. (d) From Stefan’s law, the rate of emission of energy per
unit surface area of a black body is inversely proportional 200 + 50
= = 62.5 ° C
to the fourth power of absolute temperature (T ) of the 4
body. 6. (c) According to Wien’s law,
E = σT 4( σ = Stefan’s constant) 1
λ∝
3. (a) Heat flow through vacuum is possible in radiation T
mode due to which temperature of hot body falls. i.e. it depends on the temperature of the surface.

@iitjeehelps
19
Ray Optics

Reflection of Light
The phenomenon of bouncing back of light rays in the same medium on striking a smooth surface is
called reflection of light.

Laws of Reflection
It follows two laws, i.e. Normal
(i) Angle of incidence, ( ∠i ) is equal to angle of reflection, ( ∠r ) Incident ray
Reflected ray
i.e. ∠i = ∠r
(ii) Incident ray, normal ray and reflected ray lie in the same plane.
Angle of i r Angle of
incident reflection

Reflection from a Plane Mirror


1. If a ray is incident on a plane mirror at an angle of incidence i, then it suffers a deviation of
( π − 2i ).
2. While keeping an object fixed, a plane mirror is rotated in its plane by an angle θ, then the
reflected ray rotates in the same direction by an angle 2θ.
3. Focal length as well as the radius of curvature of a plane mirror is infinity. Power of a plane
mirror is zero.
4. If two plane mirrors are inclined to each other at an angle θ, the total number of images formed of
2π  2π 
an object kept between them, is n = or  − 1 whichever is odd.
θ  θ 
5. The minimum size of a plane mirror fixed on a wall of a room, so that a person at the centre of the
1
room may see the full image of the wall behind him, should be rd the size of the wall.
3

@iitjeehelps
216 SELF STUDY GUIDE BITSAT

v h′
Reflection of Light at a m=− =
u h
Spherical Surface or where, m = magnification, v = image distance, u = object
distance, h ′ = height of image and h = height of object.
Spherical Mirror
Mirrors having their reflecting surface spherical are called
spherical mirrors. Spherical mirrors are of two types Refraction of Light
(i) Concave mirror If reflection takes place from the When light passes from one medium, say air, to another
inner surface, then the mirror is called concave medium, say glass, a part is reflected back into the first
[Fig. (a)]. medium and the rest passes into the second medium. When
(ii) Convex mirror If reflection takes place from the it passes into the second medium, it either bends towards
outer surface, then the mirror is called convex the normal or away from the normal. This phenomenon is
[Fig. (b)]. known as refraction.
A
Incident Incident
light light Rarer Denser i
medium
i medium
C
P Denser Rarer
P C medium medium r
r

B (a) (b)
(a) Concave mirror (b) Convex mirror

(iii) Focal length It is the distance between focus F and


pole P of the mirror. It is represented by f . Laws of Refraction
R (i) When light goes from one medium to another, the
i.e. f = , where R = radius of curvature frequency and phase of light do not change, however
2
velocity and wavelength of light changes.
(ii) The incident ray, the refracted ray and normal to the
Mirror Formula interface at point of incidence, all lie in the same
Mirror formula is the relation between object distance (u ), plane.
image distance (v ) and focal length ( f ) for a spherical mirror
(iii) The product of refractive index and sine of angle of
and it is given by
incidence/refraction at a point in a medium is
1 1 1 constant
+ =
v u f
Normal
Incident ray Reflected ray
A
M Interface of
i
two media
B′ F
P (µ1) Medium 1 i i
B C
f
A' (µ2) Medium 2
v
r
u N
Mirror formula rays incidence Refracted ray

Magnification of Image in a Spherical i.e. µ × sini = constant


µ 2 sin i
Mirror So, from figure = =1 µ 2
µ 1 sin r
The size of an image reflected through a spherical mirror
varies with the original size of object. This is called linear where, 1 µ 2 is refractive index of medium 2 with respect to
magnification of the image, which is the ratio of height of medium 1.
image to the height of object given by This law is also known as Snell’s law.

@iitjeehelps
RAY OPTICS 217

Apparent Shift of an Object due to Lens


Refraction The lens formula is given by
1 1
− =
1
Due to bending of light at the interface of two different v u f
media, the image formation due to refraction creates an For a thin object of height h placed perpendicular to the
illusion of shifting of the object position. principal axis at a distance u, if the height of image formed
Rarer
ish ′, then lateral or transverse magnificationm is given by
µ1 h′ v f f −v
θ2 m= = = =
h u f +u f
A
depth
Apparent

B θ2
Real depth

For a small sized object placed linearly along the principal


θ1 Denser axis, its axial or longitudinal magnification is given by
I θ1 µ1
2 2 2
dv  v   f   f − v
O Axial magnification = − =  =  = 
du  u   f + u  f 
Bending of light ray The power of a lens is mathematically given by the
reciprocal of its focal length,
Real depth 1
= 1 1
Apparent depth µ2 i.e. power, P =
f (in m )
Total Internal Reflection SI unit of power is dioptre (D). Power of a converging lens is
positive and that of a diverging lens is negative.
When light travelling in a
denser medium incident on the i < θc
interface between denser
mr Lens Maker’s Formula
md Focal length of the lens is given by
medium (medium with higher θc
i>
refractive index) and rarer i=θc 1 µ 2   1 1 
O = − 1  − 
medium (medium with lesser f µ1   R1 R2 
refractive index), then refracted ray bends away from the
normal forming a greater angle of refraction than angle of If µ 2 = µ and µ 1 = 1, then
incidence. 1  1 1 
⇒ = (µ − 1)  − 
In this situation, when the angle of incidence exceeds a f  R1 R2 
particular angle, the incident ray comes back into first
medium (denser medium). This angle is called critical angle Combination of Thin Lenses in Contact
(θC ). If two or more lenses are placed in contact, then
At critical angle, there is no refracted ray found because
∠ r = 90° .
f1 f2
sin θC µ µ
= rarer = r
sin 90° µ denser µ d
µr µ 
⇒ sinθC = ⇒ θC = sin −1  r  Combination of convex lenses
µd µ d 
Total focal length of the combination,
where, µ r = refractive index of rarer medium n
1 1 1 1 1
and µ d = refractive index of denser medium = + + +K= ∑
f f1 f2 f3 f
i=1 i
Refraction from a Spherical Surface Total power of combination,
Let an object be placed in a medium of refractive index n 1 at n

a distance u from the pole of a spherical surface of radius of P = P1 + P2 + P3 + K = ∑ Pi


i=1
curvature R and after refraction, its image is formed in a
medium of refractive index n 2 at a distance v, then Total magnification of combination,
n2 n1 n2 − n1 M = m1 × m2 × m3 × Kor M = m1 m2 m3 K
− =
v u R
NOTE If the lenses are separated by a distance d, then
The relation is true for all surfaces, whether the image 1 1 1
= + −
d
and P = P1 + P2 − P1 P2 d
formed is real or virtual. F f1 f2 f1 f2

@iitjeehelps
218 SELF STUDY GUIDE BITSAT

Deviation by a Prism Simple Microscope (Magnifying Glass)


A prism is a homogeneous, It consists of a A′
A
transparent medium bounded single convex lens
by two plane surfaces inclined of small focal L
n n′ length and forms a
at an angle A with each other. δ A
These surfaces are called as magnified image of A'' β
i i′ an object placed B' α F
refracting surfaces and the r F B
r′ C
angle between them is called between the optical
angle of prism A. Deviation centre and the
produced by a prism is principal focus of f
B C
δ = i + i′ − A ⇒ r + r ′ = A the lens. If the image is formed at the near point of eye.
 D
For grazing incidence i = 90°, and grazing emergencei′ = 90°. Then m = 1 +  but if the image is formed at infinity, then
 f
For minimum deviation,
D
(i) i = i ′ and r = r ′ m = , where D = normal distance (25 cm), f = focal length
f
 δ + A
sin  m  of magnifying lens.
 2 
(ii) µ =
A Compound Microscope
sin
2
It has two converging (convex) lenses i.e. one near object,
(iii) S = i + e − A, where i is the angle of incidence, e is called objective lens and other near eye, called eyepiece.
the angle of emergence and A is the angle of prism. Objective has smaller focal length and aperture than
In case of minimum deviation, ray is passing through eyepiece.
prism symmetrically.
fo
For maximum deviation (δ ), i = 90° or i′ = 90° uo fe
Eyepiece
For thin prism, δ = (µ − 1) A

F
Dispersion by a Prism h
h′ θ
o
Dispersion of light is the phenomenon of splitting of white h′′
light into its constituent colours on passing light through a
prism. This is because different colours have different
wavelength, and hence different refractive indices vo ue
Angular dispersion = δv − δ r = (nv − n r ) A, where n v and n r Set up of compound microscope
represent refractive index for violet and red lights.
n − nr
Dispersive power, ω = v Magnifying Power of Compound Microscope
n −1
Magnification produced by the compound microscope is
nv + n r
where n = is the mean refractive index. given by,
2 v  D
By combining two prisms with angles A and A′ and RI n and m=  
a  fe 
n ′ respectively we can create conditions of
vo  D 
(n − 1) A For relaxed eye, M∞ = −  
(a) Dispersion without deviation when, A′ = − uo  fe 
(n ′ − 1)
and length of microscope, L∞ = vo + f e . When final image
 n − nr 
(b) Deviation without dispersion when A′ = −  v A is formed at least distance of distinct vision, then
n ′v − n ′r  vo  D
MD = − 1 + 
uo  fe 
Microscope and length of microscope, LD = vo + u e
It is an optical instrument which forms a magnified image where, vo = distance of first image from objective lens
of a small nearby object and thus, increases the visual angle u o = distance of object from objective lens
subtended by the image at the eye so that the object is seen f e = focal length of eyepiece.
to be bigger and distinct.

@iitjeehelps
RAY OPTICS 219

Astronomical Telescope Magnification of Terrestrial Telescope


It is an optical device which increases the angle at the eye by (a) For relaxed eye, M ∞ = f o / f e
forming the image of a distant object at the least distance of In this position, length of telescope,
distinct vision, so that the object is seen distinct and bigger. L∞ = f o + 4 f + f e
B Object (b) When a final image is formed at least distance of
Eyepiece distinct vision,
α
β f  fe 
A′ F Fe Mo = o 1 + 
A″ e fe  D
A O
B′ Length of telescope,
L∞ = f o + 4 f + u e
B″
D where, f o = Focal length of objective lens
ue f e = Focal length of eyepiece.
Set up of astronomical telescope

Galilean Telescope
A simple model of Galilean telescope is shown in figure. A
Magnifying Power of convergent lens is used as the objective and a divergent lens
Astronomical Telescope as the eyepiece.
fo fo
For relaxed eye, M ∞ = − Parallel rays B′′
fe from object
at ∞ ue
Length of telescope, L∞ = f o + f e
When a final image is formed at least distance of distinct A′′
vision, then, α E A′
α
f  fe 
MD = − o 1 + 
fe  D B′

Length of telescope, LD = f o + u e
Set up of Galilean telescope
where, f o = Focal length of objective
f e = Focal length of eyepiece.
Magnification of Galilean Telescope
fo
(a) For relaxed eye, M ∞ =
Terrestrial Telescope fe
In an astronomical telescope, the final image is inverted In this position, length of telescope, L∞ = f o − f e
with respect to the object. To remove this difficulty, a (b) When final image is formed at least distance of distinct
convex lens of focal length f is included between the vision,
objective and the eyepiece in such a way that the focal plane f  f 
of the objective is a distance 2f away from this lens. M D = o 1 + e 
fe  D
Parallel rays B′′
from object Length of telescope, LD = f o − u e
at ∞
B′′
α A′ A′′
A′′
Resolving Power of an
B′
Optical Instrument
L Resolving power of an optical instrument is its ability to
fo 2f 2f produce distinct images of two points of an object (or two
Set up of terrestrial telescope nearby objects) very close together.

@iitjeehelps
220 SELF STUDY GUIDE BITSAT

The minimum distance (or angular distance) between two


points of an object whose images can be formed distinctly Resolving Power of a Microscope
by the lens of an optical instrument, is called its limit of The least distance (d) between two points, whose images
resolution. are just seen distinctly by a microscope, is given by
1. 22 λ
d=
Resolving Power of a Telescope 2n m sin θ
If the aperture (diameter) of the telescope objective be the where, λ = wavelength of light used to illuminate the object,
D, then the minimum angular separation (dθ ) between two n m = refractive index of the medium between the
distant objects, whose images are just resolved by the object and the objective lens
telescope, is θ = semi angle of the cone of light from the point
1. 22 λ object
dθ =
D The term n m sinθ is generally called the numerical aperture
of the microscope the objective.
and resolving power of the telescope,
1 D ∴ Resolving power of the microscope,
RP = = 1 2 n m sin θ (NA)
dθ 1. 22 λ RP = = =
d 1. 22 λ . λ
061

Practice Exercise
l0
1. A hair dresser stand with her nose 20 cm infront of a 5. A pole 5 m high is n
plane mirror. For what distance must she focus her situated on a horizontal
eyes in order to see her nose in the mirror? surface. Sun rays are 30°
a. 40 cm b. 50 cm incident at an angle 30°
d
c. 30 cm d. 60 cm with the vertical. The 30° 30°

2. On a plane mirror,a ray of light is size of shadow on


incident at an angle of 30° with horizontal surface is
horizontal. To make the reflected 30° 30° 5
a. 5 m b. m
ray vertical, at what angle with 3
horizontal must a plane mirror be 10
c. m d. None of these
placed? 3
a. 30° b. 60° 6. A beautiful girl with two normal eyes wants to see full
c. 45° d. 54° width of her face by a plane mirror. The eye to eye
3. A mirror is inclined at an angle of θ and ear to ear distances of her face are 4 inch and 6
M inch respectively. Find the minimum width of the
with the horizontal. If a ray of light
θ required mirror.
is incident at an angle of incidence
θ, then the reflected ray makes the a. 1 inch b. 2 inch c. 3 inch d. 4 inch
following angle with horizontal θ H 7. A ray of light falls on a plane mirror. When the mirror is
a. θ b. 2θ turned, about an axis at right angle to the plane of the
θ
c. d. None of these mirror through 20°, the angle between the incident ray
2 and new reflected ray is 45°. Find the angle between
4. Two mirrors each 1.6 m long are facing each other. the incident ray and original reflected ray.
The distance between the mirrors is 20 cm. A light ray a. 15° b. 30° c. 45° d. 60°
is incident on one end of the mirror at an angle of 8. The shortest height of a vertical mirror required to see
incidence of 30°. How many times in the ray reflected
the entire image of a man, will be
before it reaches the other end?
a. one-third the man’s height
a. There are 15 reflections counting the first one
b. half the man’s height
b. There are 13 reflections counting the first one
c. two-third the man’s height
c. There are 12 reflections counting the first one
d. data insufficient
d. None of the above

@iitjeehelps
RAY OPTICS 221
9. Two plane mirrors are perpendicular to each other. A 16. If u represents object distance from pole of spherical
ray after suffering reflection from the two mirrors will be mirror and v represents image distance from pole of
a. perpendicular to the original ray mirror and f is the focal length of the mirror, then a
b. parallel to the original ray straight line u = v will cut u versus v graph at
c. parallel to the first mirror a. (f , f ) b. ( 2f , 2f ) c. (f , 2f ) d. ( 0, 0)
d. at 45° to the original ray
17. A short linear object of
10. A vessel consists of two plane length l lies on the axis of
mirrors at right angles (as shown a spherical mirror of focal P
in figure). The vessel is filled with length f , at a distance x l
water. The total deviation in from the mirror. Then,
90° x
incident ray is the length of the image
a. 0° (P ) so obtained will be
b. 90° lf lf 2 lf l (x − f )
c. 180° a. b. c. d.
(x − f ) (x − f )2 x x
d. None of the above
11. A light ray is incident on a horizontal plane mirror at an 18. A rear view mirror of a vehicle is cylindrical having
angle of 45°. At what angle should a second plane radius of curvature 10 cm and length of arc of curved
mirror be placed in order that the reflected ray finally surface is 10 cm. Find the field of view in radian, if it is
be reflected horizontally form the second mirror, as assumed that the eye of the driver is at a large
shown in figure? distance from the mirror.
a. 0.5 b. 1 c. 2 d. 4
C G 19. In the figure, AB and BK
represents incident and M
S K
α D reflected rays. If angle N
α B
A
45°
N BCF = θ, then ∠BFP will A θ
45°
be equal to P F C
45°
P a. θ b. 2θ
c. 3θ d. 3.5 θ
a. θ = 30° b. θ = 24°
20. Find the position of 1 cm tall object which is placed
c. θ = 22.5° d. θ = 67.5°
8 cm infront of a concave mirror of radius of curvature
12. If two mirrors are inclined at some angle and an object 24 cm.
is placed between the mirrors and there are 7 images a. 24 cm b. 25 cm c. 26 cm d. 27 cm
formed for an object. Then, what is angle between the
21. A convex driving mirror of focal length 20 cm, is fitted
mirrors?
in a motor car. If the second car 2 m broad and 1.6 m
a. 54° b. 50° c. 60° d. 64° high is 6 m away from first car and overtake the first
13. Two plane mirrors are placed at some angle. There car at a relative speed of 15 m/s, then how fast will the
are five images formed when an object is placed image be moving?
symmetrically between them. Find the angle between a. 0.016 m/s b. 0.0257 m/s c. 0.162 m/s d. 0.0073 m/s
the mirrors.
22. When an object is placed at a distance of 25 cm from
a. 60° b. 65° c. 30° d. 45°
a mirror, the magnification is m1. But the magnification
14. If two adjacent walls and the ceiling of a rectangular becomes m 2, when the object is moved 15 cm farther
room are mirror surfaced, then how many images of m
himself, a man can see? away with respect to the earlier position. If 1 = 4,
m2
a. 3 b. 5 c. 6 d. 9
then find the focal length of the mirror and what type of
15. A convex mirror of focal mirror it is?
length 10 cm is shown in a. 20 cm, convex b. 20 cm, concave
figure. A linear object B A c. 10 cm, convex d. 10 cm, concave
AB = 5 cm is placed
along the optical axis. 23. Two objects A and B when placed in turns infront of a
5 cm
Point B is at distance concave mirror, give images of equal size. The focal
25 cm from the pole of 25 cm length of the concave mirror is 7.5 cm and size of
mirror. The size of image object A is three times the size of object B. Find the
of AB is distance of B from the mirror, if A is placed 30 cm from
the mirror.
a. 2.5 cm b. 0.64 cm c. 0.36 cm d. None of these
a. 18 cm b. 15 cm c. 20 cm d. 25 cm

@iitjeehelps
222 SELF STUDY GUIDE BITSAT

24. An object of height 5 cm is placed in midway between 32. A ray of light is incident on the surface of separation of
a concave mirror of radius of curvature 30 cm and a a medium at an angle 45° and is refracted in the
convex mirror of placed opposite to each other and medium at an angle 30°. What will be the speed of
are 60 cm apart. Find the position of the image formed light in the medium?
by reflection at convex mirror. . × 108 ms−1
a. 196 b. 2.12 × 108 ms−1
a. 10 cm b. 20 cm c. 15 cm d. 13 cm c. 3.18 × 108 ms−1 d. 3.33 × 108 ms−1
25. The focal length of plano-convex lens, the convex 33. The optical path of a monochromatic light is same if, it
surface of which is silvered is 0.3 m, if µ of the lens is goes through 4.0 cm of glass or 4.5 cm of water. If the
7/4, the radius of curvature of the convex surface is refractive index of glass is 1.53, the refractive index of
a. 0.45 m b. 1.05 m c. 3 m d. 0.9 m the water is
26. The magnification of a compound microscope is 30 a. 1.30 b. 1.36
c. 1.42 d. 1.46
and the focal length of its eye piece is 5 cm. Calculate
the magnification produced by the objective, when the 34. A small coin is resting on the 3 cm
image is to be formed at least distance of distinct bottom of a beaker filled with a
vision (25 cm). liquid. A ray of light from the

4 cm
a. 5 b. 6 c. 8 d. 10 coin travels upto the surface of
the liquid and moves along its Coin
27. A convergent doublet of separated lens, corrected for surface (see figure).
spherical aberration, are separated by 2 m and has an
equivalent focal length of 10 cm. Calculate the focal How fast is the light travelling in the liquid?
length of its component lenses. . × 108 ms−1
a. 18 b. 2.4 × 108 ms−1
a. f1 = 18 cm, f2 = 10 cm c. 3.0 × 108 ms−1 . × 108 ms−1
d. 12
b. f1 = 20 cm, f2 = 28 cm 35. If c is the velocity of light in vacuum, then find the time
c. f1 = 20 cm, f2 = 18 cm taken by the light to travel through a glass plane of
d. f1 = 24 cm, f2 = 18 cm thickness t and having refractive index µ.
 t  µt tc
28. A ray of light falls on a transparent glass slab of a.   b. t µc c. d.
 µc  c µ
refractive index 1.62. What is the angle of incidence, if
the reflected ray and refracted ray are mutually 36. Considering normal incidence of ray, find equivalent
perpendicular? refractive index of combination of two slabs shown in
−1 −1 
1  figure.
a. tan (1.62) b. tan  
 1.62 Air
1
c. −1
d. None of these
tan (1.62) 10 cm µ 1=
4
3
29. A ray of light travelling in glass having refractive index 3
µ 1=
a µg= 3/ 2, is incident at a critical angle C on the 2 15 cm
glass-air interface. If a thin layer of water is poured on
Air
glass air interface, then what will be the angle of
emergence of this ray in air when it emerges from a. 1.8 b. 1.43
water-air interface? c. 2 d. None of these
a. 180° b. 0° 37. A tank contains two different liquids which do not mix
c. 90° d. 45° with each other. The lower and upper liquid are at
depth h1 and h2
30. Calculate the time taken by the light to travel a C Q i
distance of 500 m in water of refractive index of 4/3. respectively and of
refractive indices µ 1 h
(Given, velocity of light in vacuum = 3 × 1010 cm/s) 2 E µ2
and µ 2. An object O is B
a. 3 × 10−10 s b. 2.22 × 10−6 s located at the bottom, P µ1
h D
c. 4.3 × 10−5 s d. 3 × 10−6 s when seen vertically 1
from above. Locate the O
31. In a medium of refractive index n1, a monochromatic position of image of the
light of wavelength λ 1 is travelling. When it enters in a object O as seen from above.
denser medium of refractive index n2, the wavelength h1 h2 h1 h2
of the light in the second medium is a. − b. +
µ1 µ 2 µ1 µ 2
n  n  λ 1(n 2 − n1) λ (n − n1) h h h h
a. λ 1 1  b. λ 1 2  c. d. 1 2 c. 1 + 2 d. 1 − 2
 n2   n1  n2 n1 µ1 µ1 µ 2 µ1

@iitjeehelps
RAY OPTICS 223
38. In a lake, a fish rising vertically to the surface of water 46. Find the refractive index of the material, if a prims
uniformly at the rate of 3 m/s, observe a bird diving having an angle A = 60° which produces a minimum
vertically towards the water at a rate of 9 m/s vertically deviation of 30°.
above it. Calculate the actual velocity of the dive of the 1
a. 3 b. 2 c. 5 d.
bird. (Given, refractive index of water = 4/3) 2
a. 9.2 m/s b. 4.5 m/s c. 9.0 m/s d. 3.2 m/s
47. Find the power and type of the lens by which a person
39. A compound microscope has an eyepiece of focal can see clearly the distant objects, if a person cannot
length 10 m and an objective of focal length 4 cm. see objects beyond 40 cm.
Calculate the magnification if an object is kept at a a. −2.5 D and concave lens b. − 2.5 D and convex lens
distance of 5 cm from the objective then final image is
c. − 3.5 D and concave lens d. − 3.5 D and convex lens
formed at the least distance of distinct vision.
a. 10 b. 11 c. 12 d. 13 48. If the resolution limit of the eye is 1 minute and at a
distance x km from the eye, two persons stands with a
40. A simple microscope consists of a concave lens of
lateral separation of 3 m, then calculate x for which the
power − 10 D and a convex lens of power + 20 D in
two persons can be just resolved by the nacked eye.
contact. If the image formed at infinity, then calcualte
the magnifying power (D = 25 cm ). a. 10 km b. 15 km c. 20 km d. 30 km
49. The prism shown in the figure has A
a. 2.5 b. 3.5 c. 2.0 d. 3.0
one side silvered. The angle of the
41. A telescope consists of two lenses of focal lengths
prism is 30° and µ = 2. What 30°
10 cm and 1 cm. Calculate the length of the telescope,
when an object is kept at a distance of 60 cm from the should be the angle of incidence, if
D
objective, and the final image is formed at least the incident ray retraces its initial i
distance of distinct vision. path?
a. 15.05 cm b. 12.96 cm c. 13.63 cm d. 14.44 cm a. 50°
b. 45° µ=√2 C
42. Calculate the limit of resolution of microscope, if the c. 60° B
numerical aperture of microscope is 0.12 and the d. 75°
wavelength of light used is 600 nm.
50. If a crown glass prism of refracting angle 10° have
a. 0.3 µm . µm
b. 12 c. 2.3 µm d. 3 µm
refractive indices for red and violet rays 1.514 and
43. An equilateral prism deviates a ray through 45° for the 1.523 respectively, then find the dispersion caused by
two angle of incidence differing by 20°. The angle of a crown glass prism.
incidence is a. 0.07° b. 0.08° c. 0.09° d. 0.10°
a. 60° b. 40° 51. A thin prism of angle 6° made of glass of refractive
c. 120° d. None of these index 1.5 is combined with another prism made of
44. Find the refractive index of the material of prism, if a glass of µ = 175. to produce dispersion without
thin prism of angle A = 6 °, produces a deviation δ = 3 °. deviation. The angle of second prism is
a. 1.5 b. 1.2 a. 7° b. 4.67° c. 9° d. 5°
c. 1.1 d. 1.25 52. A small object is enclosed in a sphere of solid glass
45. Figure shows, a glass prism ABC (refractive index 8 cm in radius. It is situated 2 cm from the centre and
1.5), immersed in water (refractive index 4/3). A ray of is viewed from the side to which it is nearer. Where
light incident normally on face AB . If it is totally will it appear to be if µ of glass = 15
. ?
reflected at face AC, then a. 6 cm from the centre
b. 4 cm from the nearer surface
1
c. 3 cm from the nearer surface
5
B A 2
θ d. 3 cm from the centre
3
53. In a glass sphere, there is a small bubble 2 × 10−2 m
from its centre, if the bubble is viewed along a
C diameter of the sphere, from the side on which it lies,
how far from the surface will it appear, the radius of
8 2 glass sphere is 5 × 10−2 m and refractive index of
a. sin θ ≥ b. sin θ ≥ glass is 1.5?
9 3
3 2 8 a. 2.5 × 10−2m b. 3 . 2 × 10−2 m
c. sin θ = d. < sin θ < c. 6.5 × 10−2 m d. 0.2 × 10−2 m
2 3 9

@iitjeehelps
224 SELF STUDY GUIDE BITSAT

54. A ray incident at a point at an angle of incidence 60° 61. On the axis of a spherical mirror of focal length f , a
enters a glass sphere of µ = 3 and is reflected and short linear object of length L lies on the axis at a
refracted at the further surface of the sphere. The distance µ from the mirror. Its image has an axial
angle between the reflected and the refracted rays at length L′ equal to
this surface is 1/ 2 2
 f  (µ + f ) 
1/ 2
(µ − f ) 
2
 f 
a. 50° b. 90° a. L  b. L c. L d. L 
 f   f 
c. 60° d. 40° (µ − f )  (µ − f ) 

55. The focal length of a thin convex lens for red and 62. The radius of curvature of the face of plano-convex
violet colour are 44.6 cm and 42.5 cm. Calculate the lens is 12 cm and its refractive index is 1.5. If the
focal length for the mean colour and dispersive power plane surface of the lens is now silvered, then find the
of the lens. focal length of the lens.
a. Focal length = 43.53 cm, dispersive power = 0.048 a. 26 cm b. 22 cm c. 24 cm d. 20 cm
b. Focal length = 28.53 cm dispersive power = 0.048 63. The speed at which the image of the luminous point
c. Focal length = 63.53 cm, dispersive power = 8.48 object is moving, if the luminous point object is
d. Focal length = 30.43 cm, dispersive power = 4.8 moving at speed v o towards a spherical mirror, along
its axis is
56. A wire mesh consisting of very small squares is (Given, R = radius of curvature, u = object distance)
viewed at a distance of 8 cm through a magnifying 2
 R 
lens of focal length 10 cm, kept close to the eye. The a. vi = − vo b. vi = − vo  
magnification produced by the lens is  2u − R 
a. 5 b. 8  2u − R   R 
c. vi = − vo   d. vi = − vo  
c. 10 d. 20  R   2u − R 
57. A double convex lens made of glass (refractive index 64. When a thin convergent glass lens (µ g = 15
. ) and has
n = 15
. ) has the radii of curvature of both the surfaces power of + 5.0 D, is immersed in a liquid of refractive
as 20 cm. Incident light rays parallel to the axis of the index µ I , it acts as a divergent lens of focal length
lens will converge at a distance L such that 100 cm. find the value of µ I .
a. L = 20 cm b. L = 10 cm a. 4/3 b. 5/3 c. 5/4 d. 6/5
20
c. L = 40 cm d. L = cm 65. Find the change in the focal length of the lens, if a
3
convex lens of focal length 20 cm and refractive index
58. An object approaches a convergent lens from the left 1.5, is immersed in water having refractive index 1.33.
of the lens with a uniform speed 5 m/s and stops at
a. 62.2 cm b. 5.82 cm c. 58.2 cm d. 6.22 cm
the focus. The image
a. moves away from the lens with a uniform speed 66. The reflective surface is given by y = 2 sin x . The
5 m/s reflective surface is facing positive axis. What is the
b. moves away from the lens with a uniform least value of coordinate of the point where a ray
acceleration parallel to positive x-axis becomes parallel to positive
c. moves away from the lens with a non-uniform y -axis after reflection?
acceleration π  π  π  π 
a.  , 3 b.  , 2 c.  , 2 d.  , 3
d. moves towards the lens with a non-uniform 3  2  3  4 
acceleration
67. An object is placed at the focus of convex mirror. The
59. A concave lens and a convex lens have the same image will be at
focal length of 20 cm and both are kept in contact. The
a. c b. f
combination is used to view an object 5 cm long kept
c. infinity d. None of these
at a distance of 20 cm from the lens combination. As
compared to the object, the image will be 68. A 4.5 cm needle is placed 12 cm away from a convex
a. magnified and inverted mirror of focal length 15 cm. Give the location of the
b. diminished and erect image and the magnification.
c. of the same size and erect a. 6.7 cm, 5/9 b. 7.5 cm, 5/9
d. of the same size and inverted c. 6.7 cm, 9/5 d. 7.5 cm, 9/5
60. A source of light lies on the angle bisector of two plane 69. With a concave mirror, an object is placed at a
mirrors inclined at angle θ. The values of θ, so that the distance x1 from the principal focus, on the principal
light reflected from one mirror does not reach the axis. The image is formed at a distance x 2 from the
other mirror will be principal focus. The focal length of the mirror is
a. θ ≥ 120° b. θ ≥ 90° x1 + x 2 x1
a. x1 x 2 b. c. d. x1 x 2
c. θ ≤ 120° d. θ < 30° 2 x2

@iitjeehelps
RAY OPTICS 225
70. Electromagnetic radiation of frequency n, wavelength 72. A layered lens is made of materials indicated by
λ, travelling with velocity v in air, enters a glass slab of shades in the figure. The number of images formed is
refractive index µ. The frequency, wavelength and
velocity of light in the glass slab will be respectively
n λ v λ v v n λ
a. , , b. n, , c. n, λ , d. , ,v
µ µ µ µ µ µ µ µ
71. When a glass slab is placed on a cross made on a
sheet, the cross appears to be raised by 1 cm. The
thickness of the glass is 3 cm. The critical angle for
glass is
 3 a. 1 b. 2
a. sin−1( 0.33) b. sin−1( 0.5) c. sin−1( 0.67) d. sin−1  c. 3 d. 4
 2

BITSAT Archives
1. Resolving power of a telescope will be more, if the A point object is placed at the focus of the bio-convex
diameter (a) of the objective is [2014] lens. What should be the value of X , so the final image
a. larger forms at infinity? [2013]
b. smaller a. 10 cm b. 20 cm
c. resolving power does not depend on a c. 15 cm d. None of these
d. None of the above
7. The image formed by a concave spherical mirror [2013]
2. A glass piece is dipped in a liquid of refractive index a. is always virtual b. is always real
4/3, it gets dissappeared in the liquid. The refractive c. is always inverted d. may be erect
index of the glass piece is [2013]
3 5 4 4 8. A ray of light is incident on the interface between
a. b. c. d. water and glass at an angle i and refracted parallel to
4 3 5 3
the water surface, then value of µ g will be [2012]
3. If the bio-convex lens is cut as shown in the figure, the
new focal length f ′ is [2013] r µw=4/3
Water r
i
Glass
f f′=? 1 4
a. ( 4 / 3) sin i b. c. d. 1
sin i 3
9. A mica slit of thickness t and refractive index µ is
a. 2 f b. f c. f /2 d. infinite
introduced in the ray from the first source S1. By how
4. Refractive index of a medium depends [2013] much distance of fringes pattern will be displaced?
a. on the medium only d D
a. (µ − 1) t b. (µ − 1) t
b. on the incident light only D d [2012]
c. on both the conditions given in options a. and b. d D
c. d. ( µ − 1)
d. None of the above ( µ − 1) D d
5. A point object is placed at the focus of a convex mirror, 10. The refractive index of water is 4/3 and that of glass is
the image will be formed at [2013] 5/3. What will be the critical angle for the ray of light
a. infinity b. centre of curvature entering water from the glass? [2012]
c. at focus itself d. None of these  4  5  1  2
a. sin−1  b. sin−1  c. sin−1  d. sin−1 
6. f1=10 cm f2=10 cm  5  4  2  1

11. A film projector magnifies a 100 cm 2 film strip on a


O screen. If the linear magnification is 4, the area of the
F magnified film on the screen is [2011]
a. 1600 cm2 b. 400 cm2
d c. 800 cm2 d. 6400 cm2

@iitjeehelps
226 SELF STUDY GUIDE BITSAT

12. A convex lens makes a real image 4 cm long on a 17. The optical path of a monochromatic light is same if it
screen. When the lens is shifted to new position goes through 4.0 cm of glass of 4.5 cm of water. If the
without disturbing the object, we again get a real refractive index of glass is 1.53, the refractive index of
image on the screen which is 16 cm tall. The length of the water is [2007]
the object must be [2010]
1 a. 1.30 b. 1.36 c. 1.42 d. 1.46
a. cm b. 8 cm c. 12 cm d. 20 cm
4 18. What is the refractive index of a prism whose angle
13. Rays of light fall on a glass A = 60° and angle of minimum deviation dm = 30°?
slab (µ > 1) as shown in figure. A B [2006]
If µ at A is maximum and at B 1 1
a. 2 b. c. 1 d.
it is minimum, then what will 2 3
happen to these rays? [2009]
19. An object is placed at a distance 20 cm from the pole
a. They will tilt towards A of a convex mirror of focal length 20 cm. The image is
b. They will tilt towards B produced at [2006]
c. They will not deviate
a. 13.3 cm b. 20 cm c. 25 cm d. 10 cm
d. There will be total internal reflection
20. The plano-convex lens of focal length 20 cm and
14. The maximum value of index of refraction of a material 30 cm are placed together to form a double convex
of a prism which allows the passage of light through it lens. The final focal length will be [2006]
when the refracting angle of the prism is A, is [2009]
a. 12 cm b. 60 cm
 A  A c. 20 cm d. 30 cm
a. 1 + sin   b. 1 + cos  
 2  2
21. Which mirror is to be used to obtain a parallel beam of
 A  A light from a small lamp ? [2005]
c. 1 + tan   2
d. 1 + cot   2
 2  2 a. Plane mirror b. Convex mirror
15. The focal lengths of objective and the eyepiece of a c. Concave mirror d. Any one of these
compound microscope are fo and fe respectively. 22. Which of the following is a wrong statement ? [2005]
Then, [2009] a. D = 1/f , where f is the focal length and D is called
a. fo > fe b. fo < fe c. fo = fe d. None the refractive power of a lens
b. Power is expressed in a diopter when f is in metres
16. The refractive index of a material of a plano-concave
c. Power is expressed in diopter and does not depend
lens is 5/3, the radius of curvature is 0.3 m. The focal
on the system of unit used to measure f
length of the lens in air is [2008]
d. D is positive for convergent lens and negative for
a. −0.45 m b. −0.6 m c. −0.75 m d. −10
. m divergent lens

Answer with Solutions


Practice Exercise 5. (b) From figure,
BC
1. (a) From the figure, image is situated at 40 cm from eye. tan 30° =
5
E′ E
30°

Image Object A
A′ A
20 cm 20 cm

2. (a) 3. (d) 30°


5 cm
4. (a) From the figure given in question,
l 20
tan 30° = 0 ⇒ l0 = d tan 30° =
d 3
B C
 
Horizontal surface
. × 100 cm
 L  16
∴ Number of reflections =   =   = 14
l0   20 cm  ∴ BC = 5 tan 30°
 3  5
= m
If first reflection is considered, then n = 14 + 1 = 15 3

@iitjeehelps
RAY OPTICS 227
6. (a) Here the minimum size of mirror and one due to their combination. With three images right
b −a 6 − 4 angles to each other, one image is produced by each
= = = 1inch plane mirror and one by each of the three pairs of
2 2
combination of two mirrors each.
7. (c) When mirror is rotated through an angle θ about an
axis in the place of mirror, then reflected ray is rotated 15. (c) For point A, u = − 25 cm, y
through an angle 2θ. f = + 10 cm
When mirror is rotated about an axis perpendicular to the 1 1 1
∴ + =
plane of mirror. The angle of incidence and angle of v u f
reflection does not change. 1 1 1 x′ x
or − =
8. (b) Let MN = height of a man v 25 10
1 1 1 1 5+ 2
E = position of eye or = + or =
L v 10 25 v 50 y′
M′ M 50
∴v = = 7.14 cm
S P 7
For point B, u′ = − 30 cm
f ′ = 10 cm
E 1 1 1
+ =
v ′ u′ f ′
Q 1 1 1 1 1 1 1+ 3 4
G or − = or = + = =
v ′ 30 10 v ′ 30 10 30 30
N′ N 30
K ∴ v′ = = 7.5 cm
As, ∆NGQ and ∆QGE are similar, 4
∆ESP and ∆PSM are congruent. ∴ A′B′ = |v ′ − v | = |7.5 − 7.14| = 0.36
MP = PE = EQ = QN 16. (b) From u-v graph, (b) is correct.
Now, SG = PQ = PE + EQ
ME EN
SG = +
2 2
1
SG = MN (2f, 2f)
2
9. (b) Here angular deviation is δ = 2π − 2θ
17. (b) Here, u = − x , F = − f
π
= 2π − 2 × =π 1 1 1
2 Q + =
v u F
Hence, final emergent ray is parallel to incident ray.
1 1 1 1 1 1 f −x
10. (c) The phenomenon of reflection is independent of or − = − or = − =
v x f v x f xf
medium. xf
The deviation produced by combination of two plane ∴ v =
f −x
mirrors is 2
δ = π − 2θ ∆v v 2  xf  1 f2
π ∴ = 2 =  × 2 =
or δ = 2π − 2 (Q θ = 90° ) ∆u u f − x  x (f − x )2
2
∴ δ = π = 180° f 2l
∴ ∆v = [Q ∆u = l]
11. (c) From figure, CD = emergent ray and CD is parallel to (x − f )2
PQ and BC is a line intersecting these parallel lines. l 10
18. (c) θ = ⇒ θ= = 2 rad
So, ∠DCB + ∠CBQ = 180° r 5
∠DCN + ∠NCB + ∠CBQ = 180° 10 cm
α + α + 45° = 180°
α = 67.5°
But ∠NCS = 90°
So, second mirror is at an angle of 22.5° with horizontal. 5 cm
12. (b) 13. (a)
14. (c) Total number of images formed = 6 , because with an θ
object placed between two plane mirrors inclined at an
angle of 90°, one image is produced due to each mirror

@iitjeehelps
228 SELF STUDY GUIDE BITSAT

19. (b) Here, incident ray is parallel to principal axis, so 2 1


25. (b) Here, P = 2PL + Pm = −
∠AMN = ∠BCF = θ fL fm
also ∠CBF = ∠KMN = ∠AMN 2 2
P = +
∴ ∠BFP = θ + ∠CBF = θ + θ = 2θ fL R
1 1 1
20. (a) + = 1  1 1  7   1 1
Here, = (µ − 1)  −  =  − 1  + 
v u f fL  r1 r2   4   ∞ R 
1 1 1
− =− ⇒ v = 24 cm 1 3
v 8 12 =
fL 4R
1 1 1
21. (a) Mirror formula + = 1 6 2 6 + 8 14
v u f ∴ P =− = + = =
F 4R R 4R 4R
Differentiating w.r.t. t, we get
1 14
1 dv 1 du or − =
0=− 2 − 0.3 4R
v dt u 2 dt
14 × 0.3 4.2
du v 2 du ∴ R=− =− = − 105
.
=− 2 4 4
dt u dt
26. (a) When final image is formed at least distance of distinct
du
But = 15 m/s vision, then the magnifying power
dt  D
1  1 1  = M 0 × 1 + 
Now = +   fe 
v  20 600
 25
600 30 = M 0 1 + 
600 v 1  5
∴ v = cm and = 31 =
31 u 600 31 M0 = 5
2
dv  1 27. (c) Since, the doublet is corrected for spherical
∴ Speed of image = = −   × 15 = 0.016 m/s
dt  31 aberration, it satisfies the following condition
f1 − f2 = d = 2 cm
22. (b)
f1 = f2 + 2 cm
IA I
23. (b) Let mA = and mB = B Let the equivalent focal length = F
OA OB
f1f2
Here, I A = O A and O A = 3OB F = = 10 cm
f1 + f2 − d
mA I O I O 1
= A × B = A × B = Solving it, f1 = 20 cm, f2 = 18 cm
mB O A IB IB O A 3
28. (a) Here, ∠i + 90° + ∠r = 180°
f f
also, mA = and mB = i + r = 90°
f − uA f − uB
r = (90° − i )
mA  f − uB 
=  From Snell’s law
mB  f − u A 
sin i sin i Air
1 −7.5 − uB µ= =
= sin r sin (90° − i ) Incident i i Reflected
3 −7.5 − ( −30) sin i
ray
90°
ray
= = tan i
uB = − 15 cm cos i
µ=1.62 r Reflected
30 µ = tan i
24. (a) u1 = 30 cm, f1 = − = − 15 cm, u 3 = v1 = ? ray
2 i = tan−1(µ )
i = tan−1(162
. )
1 2
29. (c) sin C = =
l1 µ
a g 3 C
O Glass
I 4
sin C a µw 3 8
Here, = = = r1
Water
1 1 1 sin r1 a µg 3 9 r1
− = + 2
15 30 v1
9 9 2 3 r2
v1 = − 10 cm sin r1 = sin C = × =
8 8 3 4
Image formed is virtual which is formed behind the convex Let r2 = angle of refraction for water-air interface
mirror.

@iitjeehelps
RAY OPTICS 229
sin r1 1 3 ∑ ti
= = µ=
sin r2 a µw 4 t
∑ i
4 4 3 µi
sin r2 = sin r1 = × = 1 = sin 90°
3 3 4 t1 + t 2 10 + 15
In this case, µ = =
r2 = 90° t1 t2 10 15
+ +
30. (b) µ1 µ 2 4 3
c  3 2
  25 25
λ air  v  = = = 143
.
31. (a) λ medium = = 7.5 + 10 17.5
nm nm
c c 37. (b) Let ∠BOP = ∠i, ∠BDP = ∠r, ∠CEQ = ∠i′
λ1 = , λ2 = or λ 1n1 = λ 2n 2
vn1 vn 2 Q i
C
n 
λ = λ 1 1  h2
 n2  E µ2
B
sin i c P µ1
32. (b) n = = h1 D
sin r v
O
c sin r 3 × 108 × sin 30°
Hence, v = =
sin i sin 45° Refraction at P
3 × 108 µ1 sin i = µ 2 sin r
= = 2.12 × 108 ms–1 µ1 sin r tan r h
2 = = = 1
µ 2 sin i tan i BD
33. (b) As optical paths are equal, hence
h1 µ 2
xg 4.0 ⇒ BD =
ng ⋅ xg = nw ⋅ xw ⇒ nw = ng ⋅ = 153
. × = 1.36 µ1
xw 4.5
At Q, µ 2 sin r = sin i′
34. (a) As shown in figure, a light ray A R B sin i′ tan i′ BD + BC
from the coin will not emerge out µ1 = = =
sin r tan r CE
of liquid, if i > C. i>C
C BD + BC h1 h2
Therefore, minimum radius R h ⇒ CE = = +
µ2 µ1 µ 2
corresponds to i = C. In ∆SAB,
Coin
R s 38. (b) Let at any time, fish at depth of the x and y height at
= tan C or R = h tan C
h which bird was above the surface of water
h d AC µ1 y 1
or R= = ⇒ =
n2 −1 d AP µ 2 d AP µ
n d AP = µy
 1 sin C  1 or
Q sin C = and tan C =  ∴ Total apparent distance of P
 n 1 − sin2 C  c
the bird which was seen by
√n2–1 B
Given, R = 3 cm , h = 4 cm a fish x distance in water.
3 1 h = x + µy µy
Hence, = dh dx dy
4 n −1
2
= +µ C
25 5 dt dt dt
or or n =
n2 = dy y h
9 3 9 =3+µ A
dt
c c 3 × 108
But n = or v = = = 1.8 × 108 ms−1 dy 6
v n 5/ 3 = = 4.5 m/s
dt  4 x x
 
35. (c) µ =
Velocity of light in vacuum
=
c  3
Velocity of light in glass plate c ′ 1 1 1 F
c 39. (c) We know, + =
or c′ = v0 5 4
µ
v 0 = 20 cm
distance t µt
Time taken = = = v  D
velocity  c  c and magnification (M ) = 0 1 + 
  u0  fe 
 µ
20  20 
36. (b) The equivalent refractive index of combination of slab M = 1+ = 12
5  10 
for normal incidence is

@iitjeehelps
230 SELF STUDY GUIDE BITSAT

40. (a) Power of combination 47. (a) Here, in this case, the lens used by the person should
P = P1 + P2 form the image of distant object of 40 cm.
P = + 20 − 10 = + 10 D 1 1 1
∴ u = ∞, v = − 40 cm and = −
1 1 100 f v u
F = = m= cm = 10 cm 1 1 1
P 10 10 = −
For final image at infinity f − 40 −∞
D 25 1 1
M = = = 2.5 =−
F 10 f 40
1 1
41. (b) Two lenses used are eyepiece and objective. Power = = − = − 2.5 D
For eyepiece, f 40
fe = 1 cm, D = ve = 25 cm Negative sign shows that lens used is concave lens.
1 1 1 1 1 l
− = or − − =1 48. (a) θ = , where θ = 1minute
ve ue fe 25 ue r
1  π 1
ue = −
25 θ= = ×  rad and l = 3 m
cm 60°  180 60
26
and for objective, uo = − 60 cm,fe = 10 cm l 3m
x =r = = = 10318 ≈ 10 km
1 1 1 θ  π 1
∴ − =  × 
vo uo fe  180 60
1 1 1 49. (b) To retrace the path ray must be A
+ =
vo 60 10 incidented normally on the silvered
face of the prism.
1 1 1
= − In ∆AED, 30°
vo 10 60
30° + 90° + ∠D = 180° D D E
1 5 i
= ∠D = 60° r
vo 60
also ∠D + ∠r = 90°
60
vo = ∠r = 90° − 60° = 30°
5 B
From Snell’s law
vo = 12 cm C
1sin i = ( 2 ) sin 30°
 25
∴Length of telescope = L = vo – ue = 12 +  = 12.96 cm  1
 26 i = sin−1 
 2
42. (d) Let x = limit of resolution of a microscope
i = 45°
0.61 λ
x = 50. (c) We know, angular dispersion
µ sin θ
= δV − δR = (µV − µR ) A = (1523
. − 1514
. ) × 10°
( 0.61) ( 6 × 10−7 )
x = = 0.009 × 10° = 0.09 °
( 0.12)
51. (b) δ = 0
x = 3 µm
∴ δ1 − δ 2 = 0
43. (a) Since, prism is equilateral
or (µ1 − 1) A1 − (µ 2 − 1) A2 = 0
∴ Angle of prism = 60°
or . − 1) 7° = (175
(15 . − 1) A2
Also, i − i′ = 20 …(i)
7 3
Q δ = i + i′ − A or δ + A = i + i′ or = A2
2 4
or 40° + 60° = i + i′ or i + i′ = 100 …(ii) 7 4
Adding Eqs. (i) and (ii), we get ∴ A2 = ×
2 3
2i = 120 14
= = 4.67°
∴ i = 60° 3
44. (a) Condition for angle of deivation for prism 52. (c) Here, µ1 = 1. 5, µ 2 = 1, u = + 6 cm,
δ
δ = A (µ − 1) ⇒ µ = 1 + r = + 8 cm
A
µ 2 µ1 µ 2 − µ1
3 − =
µ = 1 + = 15 . v u r
6
1 1. 5 1. 5 − 1 x′ x
µ = 1. 5 − = O P′ P C
v 6 8
45. (a) 1
∴ v = 3 cm
46. (b) 5

@iitjeehelps
RAY OPTICS 231
53. (a) For spherical surface 360
60. (a) n =
µ 2 µ1 µ 2 − µ1 θ
− =
v u R Number of images, N = n, which is odd = n − 1
Here, µ1 = 15. , µ2 = 1 For the given condition, no successive reflection takes
u = (5 × 10−2 − 2 × 10−2 ) place. So, the number of images will be N ≤ 2
I O
= 3 × 10−2 cm n − 1≤ 2
µ R n≤3
and R = 5 × 10−2 cm 360
≤3
1 .
15 1 − 15
. θ
∴ − −2
=
v 3 × 10 5 × 10−2 120 ≤ θ ⇒ θ ≥ 120°
v = 2.5 × 10−2 m fu
61. (d) For spherical mirrorv =
Image will form at a distance of 2.5 × 10 −2
m to the right of (u − f )
nearer surface. The value of u for two ends of the objects of length L are
L L
54. (b) The angle between the u1 = µ − and u 2 = µ +
° –θ
reflected ray and refracted ray 2 2
120 Then, the axial length of the image
is (120° − θ ). 60°
For calculation of θ, θ  L  L
60° f µ −  f µ + 
 2  2
µ1 sin i = µ 2 sin r L′ = v1 − v 2 = −
 L   L 
1 × sin 60° = 3 sin θ µ − − f  µ + − f 
 2   2 
3  
= 3 sin θ 2
2  Lf   f 
1 =f  2
=L  
sin θ = = sin 30° (µ − f )2 − L  µ − f 
2  
4
θ = 30°
1  1 1
Hence, (120° − 30° ) = 90° 62. (c) = (µ − 1)  − 
f  r1 r2 
55. (a) Mean focal length = f = fr fv
1  1 1
f = 44.6 × 42.5 = 43.53 cm = (1.5 − 1)  − 
f  12 ∞ 
fr − fv 44.6 − 42.5 f = + 24 cm
∴ Dispersive power = = = 0.048
f 43.33 1 1 1
63. (b) + =
1 1 1 v u f
56. (a) = − …(i)
f v u 1 dv 1 du
Differentiating both sides − =
Given, f = 10 cm (as lens is converging) v 2 dt u 2 dt
u = − 8 cm (as object is placed on left side of the lens) dv
2
 v  du v 
2
= vi = −   = −   v0
Substituting these values in Eq. (i), we get dt  u  dt u
1 1 1 1 1 1 1 8 − 10 1 1 1 2 1 2u − R
= − ⇒ = − ⇒ = Again, = − = − =
10 v − 8 v 10 8 v 80 v f u R u Ru
80 uR
∴ v = = − 40 cm v =
−2 2u − R
2 2
Hence, magnification produced by the lens v   R 
vi = −   vo = − vo  
v − 40 u  2u − R 
m= = =5
u −8
1  µg − µ a   1 1
57. (a) Here, n = 15
. , as per sign convention followed 64. (b) Pa = =   − 
fa  µ a   R1 R2 
R1 = + 20 cm and R2 = − 20 cm 1  µg − µl   1 1
 1 Pl = =    − 
1 1  1 1  fl  µl   R1 R2 
∴ = (n − 1)  −  = (1.3 − 1)  − 
f  R1 R2  ( + 20) ( − 20)  . − 1  1
 15 1
2 1 =5    −  …(i)
= 0.5 × = ⇒ f = 20 cm  1   R1 R2 
20 20
 15
. − µl   1 1
Incident rays travelling parallel to the axis of lens will − 1=    −  …(ii)
 µl   R1 R2 
converge at its second principal focus. Hence, L = + 20 cm
5
58. (c) 59. (c) From Eqs. (i) and (ii), we get µl =
3

@iitjeehelps
232 SELF STUDY GUIDE BITSAT

1  1 1 Distance of image from the mirror,v = 6.7 cm


65. (c) When the lens is in air, = (µ − 1)  − 
fa  R1 R2  The positive sign shows that the image is formed behind
the mirror.
1  1 1  1 1 1
= (15
. − 1)  −  or  −  = Using the formula of magnification,
20  R1 R2   R1 R2  10 v I
m=− =
When lens is in water, u O
1  µg − µw   1 1   15
. − 133
.  − 6.7 I
=   −  =  ×
1 =
fw  µw   R1 R2   133 .  10 − 12 4.5

fw = 78.2 cm Size of image, I = 2.5 cm


The change in focal length = 78.2 − 20 = 58.2 cm As I is positive, so image is erect and virtual.
n Magnification m is given by,
66. (a) Let the incidence point is 45° I 2.5 25 5
P (x , y ) 45° P m= = = =
O 4.5 45 9
m = tan 45° = 1
1 1 1
(from law of reflection) 69. (d) As, + =
u v f
y = 2 sin x
1 1 1
dy ∴ + =
∴ m= = 2 cos x = 1 (f − x1) (f − x 2 ) f
dx
1 π f − x 2 + f − x1 1
cosx = ⇒ x = ⇒ =
2 3 (f − x1) (f − x 2 ) f
π ⇒ f 2 − fx 2 − fx1 + x1x 2 = 2f 2 − f (x1 + x 2 )
The corresponding value of y is 2 sin = 3
3 ⇒ f 2 = x1x 2 ⇒ f = x1 x 2
67. (d) If the object is placed on the right of the convex mirror,
no image will be formed. 70. (b) In going from one medium to another, frequency
remains same but wavelength and velocity decreases
Reflecting with the increase in refractive index i. e. wavelength
surface λ v
becomes λ ′ = and velocity becomes,v ′ = .
F µ µ
 1
71. (c) We can write shift, 1 = 3 1 − 
 µ
68. (a) Given, focal length of convex mirror f = + 15 cm
1 1
(focal length of convex mirror is taken as positive) ⇒ 1− =
µ 3
Distance of object, u = − 12 cm
1 1 2 3
Size of object, O = 4.5cm or = 1− = ⇒ µ =
µ 3 3 2
Using the mirror formula,
1 3 2
1 1 1 Now, = ⇒ sin ic =
= + sin ic 2 3
f v u
−1  2  −1
1 1 1
= − or ic = sin   ⇒ ic = sin ( 0.67)
 3
15 v 12 F C
1 1 1 4+5 9 72. (d)
⇒ = + = =
v 15 12 60 60

BITSAT Archives
1. (a) 2
f = (µ − 1) …(i)
2. (d) The glass piece will dissappear only, if the refractive R
 1 1
index of the glass and liquid is same. and f ′ = (µ − 1)  − 
 R ∞
4
So, refractive index of glass piece must be ⋅ (µ − 1)
3 f′ = …(ii)
R
 1 1
3. (a) f = (µ − 1)  −  From Eqs. (i) and (ii), we get
 R1 R2 
f′ = 2f
1 1
f = (µ − 1)  +  4. (c) Refractive index of a medium depends on the medium
R R as well as on the wavelength of the incident light.

@iitjeehelps
RAY OPTICS 233
5. (d) Image will not form, because object is placed on the 15. (b) For a microscope fo < fe .
side from where reflection is not possible. 1 1 1
16. (a) Lens maker's formula = (µ − 1)  − 
f R1 R2 
Object
Where, R2 = ∞, R1 = 0.3 m
F 1 5   1 1
∴ =  − 1  − 
f  3   0.3 ∞ 
6. (b) 1 2 1
⇒ = ×
f 3 0.3
F or f = 0.45 m
O F1<F2 17. (b) Optical path, µx = constant
i.e. µ1x1 = µ 2x 2 ⇒ 153
. × 4 = µ 2 × 4.5
10+10=20 cm
. ×4
153
The final image will be at infinity only, if the focii of lens ⇒ µ2 = = 136
.
4.5
and mirror coincides. The situation could be understood on
the basis of given diagram. 18. (a) Refractive index of prism
A + δm 60° + 30°
7. (d) The image formed by a concave mirror could be real, sin sin
virtual, erect and inverted. µ= 2 = 2
A 60°
sin i sin sin
8. (b) g µw = 2 2
sin r
sin r sin 45° 1/ 2 1 2
gµa =
= = = × = 2
sin 90° sin 30° 1/ 2 2 1
sin i sin r 19. (d) u = − 20 cm, f = 20 cm
⇒ g µw × wµ a = × = sin i
sin r sin 90° 1 1 1
µw µ a 1 From mirror formula, = +
or × = sin i ⇒ µg = f v u
µg µw sin i
1 1 1
D(µ − 1) t = +
9. (b) Fringe displacement, x 0 = 20 v −20
d 1 1 1
= +
µ1 5/3 5 v 20 20
10. (a) µ = = =
µ 2 4/3 4 1 1 1
= +
1 4  4 v 20 20
sin C = = ⇒ C = sin–1  
µ 5  5 1 2
=
11. (a) As linear magnification m = 4 v 20
⇒ v = 10 cm
Hence, a real magnification ma = m 2 = ( 4)2 = 16
20. (a) Equivalent focal length
∴ Surface area of film image on the screen 1 1 1 1 1
= 16 × 100 = 1600 cm2 = + = +
F f1 f2 20 30
12. (b) 0 = I1I 2 = 4 × 16 = 8 cm 20 × 30 600
F = = = 12 cm
13. (c) For normal incidence, whatever be refractive index. 20 + 30 50
i = 0, r = 0, so there is no deviation of ray. 21. (c) If lamp is placed at the focus of concave mirror, then
14. (d) Situation is shown in figure. we get parallel beam of light.
A
r1 = C, r2 = C
r1 + r2 = A
A r1=C
∴ 2C = A, C =
C
r=

2 P
2

F
1 1
∴ µ= =
sin C  A
sin  
 2
 A  A
sin2   + cos2   22. (c) Power is expressed in dioptre and does not depend
 2  2  A
= = 1 + cot 2   on the system of unit used to measure F . So, among the
 A  2
sin   given options, option (c) is wrong.
 2

@iitjeehelps
20
Wave Optics

Wave Optics
Wave optics describes the connection between waves and rays of light. According to wave theory of
light, the light is a form of energy which travels through a medium in the form of transverse wave
motion. The speed of light in a medium depends upon the nature of medium.

Huygens’ Principle
Huygens’ principle is used to determine the shape of a wavefront at a time t, if the shape of that
wavefront is known at time t = 0.
It states that ‘‘each point of the wavefront is the source of a secondary disturbance and the wavelets
emanating from these points spread out in all the directions with the speed of the wave known as
secondary wavelets. If we draw a common tangent to all these spheres, then we obtain the new
position of the wavefront at a later time.’’

Laws of Reflection and Refraction


using Huygens’ Principle
Law of Reflection of a Plane Wave
by a Plane Surface
Consider the diagram given below showing reflection of a plane Incident
wave by a plane surface constructed using Huygens’ principle. wavefront
E B
Here, AB is a plane wave. MN is the reflecting surface, v is the Reflected
speed of wave, t is the time taken by the wavefront to reach i i wavefront
from B to C. So, BC = vt = AE . i r
M N
The law of reflection states that angle of incidence is equal to A C
the angle of reflection, which can be proved by the diagram
constructed using Huygens’ principle.

@iitjeehelps
WAVE OPTICS 235
In the above diagram, we can see that ∆AEC and ∆ABC are If a1 and a2 be the amplitudes of two light waves of same
congruent and hence their corresponding angles are equal, frequency and φ be the phase difference between them, then
i.e. ∠i = ∠r . the amplitude of resultant wave is given by

Law of Refraction for a Denser Medium AR = a12 + a22 + 2a1a2 cos φ


Consider the given diagram showing refraction of a plane and in terms of intensity of light
wave, constructed using Huygens’ principle. I R = I 1 + I 2 + 2 I 1 I 2 cos φ
Here, XX′ is the boundary between two mediums i.e.
medium 1 ( µ 1 ) and medium 2 (µ 2 ), where v1 , v2 are the Condition for Constructive Interference
respective speed of light (v2 < v1 ), t is the time taken by the If at some point in space, the phase difference between two
wavefront in travelling distance BC. So, CE represents the waves, φ = 0° or 2nπ (or path difference between two waves,
refracted wavefront. ∆x = 0 or nλ),
Incident
where, n is an integer, then
B
wavefront Medium 1 (µ1) AR = a1 + a2
A′
v1 v1 t or I R = I 1 + I 2 + 2 I 1 I 2 is maximum.
i
X i X′ Such an interference is called constructive interference.
A v2 t r C
r Medium 2 (µ2)
Refracted
v2 < v1 E wavefront Condition for Destructive Interference
If at some point in space, the phase difference between
λ
From the figure, we have two waves, φ = (2n − 1)π [or path difference, ∆x = (2n − 1) ,
2
BC AE then at such points AR = (a1 − a2 ) and I R = I 1 + I 2 − 2 I 1 I 2 is
sini = and sin r =
AC AC
minimum leading to a destructive interference.
sin i BC AC BC
So that, = × = Imax I 1 + I 2 + 2 I 1 I 2  I 1 + I 2 
2
a 1 + a 2 
2
r + 1 
2
sin r AC AE AE = =  =
Imin I 1 + I 2 − 2 I 1 I 2  I 1 − I 2  a − a  = r − 1 
sin i v1 t v1 c /µ 1 µ 2  1 2  
⇒ = = = = a1
sin r v2 t v2 c /µ 2 µ 1 where, r = = amplitude ratio.
a2
⇒ µ 1 sin i = µ 2 sin r
This figure proves the Snell’s law of refraction.
i.e. µ 1 sin i = µ 2 sin r . Here, µ 1 and µ 2 are refractive indices of Young’s Double Slit
both the media, respectively. Experiment
When a ray passes from an optically denser medium to an
This experiment is a demonstration that matter and energy
optically rarer medium, the angle of refraction r is greater can display characteristics of both waves and particles and
than the angle of incidence i. For a certain angle of demonstrates the fundamentally probabilistic nature of
incidence i = iC , angle of refraction r = 90°, then this angle quantum mechanical phenomena.
of incidence iC is called critical angle. The arrangement is shown in figure. Monochromatic light
NOTE From law of refraction (Snell’s law), of same wavelength is used.
µ 1 sini = µ 2 sinr ⇒ µ 1 siniC = µ 2 sin 90° P
–1
⇒ siniC = µ 2 / µ 1 ⇒ iC = sin (µ 2 /µ 1 )
x
S1
d
Interference of Light Light S
S2 O
source D
Interference of light is the phenomenon of redistribution of
light energy in space when two or more light waves of same
For light waves reaching a point P, situated at a distance x
frequency (or same wavelength) emitted by two coherent
from central point O, the path difference
sources, travelling in a given direction, superimpose on xd
each other. ∆x = S 2 P − S 1 P =
D

@iitjeehelps
236 SELF STUDY GUIDE BITSAT

xd
(i) If = nλ, then we get the nth bright fringe. Hence, Fringe Shift
D
position of bright fringes on the screen are given by (i) If a refracting slab of refractive index µ and thickness
nDλ t is placed in front of one of the two slits of Young’s
the relation x = . double slit experiment, then the whole fringe pattern
d
gets shifted towards the slab by distance
xd λ
(ii) If = (2n − 1) , then we get the nth dark fringe. ∆y = (µ − 1)t
D 2
(2n − 1)Dλ (ii) If both slits are covered by refracting surfaces of
Hence, for nth dark fringe x = .
2d thickness t 1 and t 2 and refractive indices µ 1 and µ 2
respectively, then fringe pattern shifted by
Fringe Width ∆y = (µ 2 − µ 1 )t . Net shift will be towards the slit
having greater refractive index.
It is the distance between two consecutive bright or two
consecutive dark fringes. It is denoted by β. Fringe width
between two consecutive bright or dark fringes is same.
λD
Fresnel’s Biprism
Expression for fringe width, β =
d Fresnel’s biprism is a device for obtaining coherent sources.
where, β = fringe width The biprism is a glass prism of very small base angles.
λ = wavelength of light
Biprism
D = distance between the screen and plane of two S1 P
slits
d = distance between the two slits S O Fringe
d pattern
Path difference between the waves coming from S 1 and S 2 ,
meets on the screen at a distance Yn from central maxima S2 Q
dYn
(O) = ∆x ≈ D
D
(i) Path difference for bright fringes, Comparing the situation geometrically with the double slit
∆x =
dYn
= nλ Dλ
experiment fringe pattern width G = W =
D d
Distance of the nth bright fringe from the centre of where, D = distance of screen from the line joining S 1 S 2 .
the fringe system is
λ = wavelength of light used,
nλD d = distance between S 1 and S2 .
Yn = ; n = 0, 1, 2 , ... , etc.
d
(ii) Path difference for dark fringes,
Calculation of Deviation
dYn  1
∆x = = n +  λ Let a = distance of source S from biprism
D  2
Deviation = (µ − 1)α (for small angle prism)
Distance of the nth dark fringe from the centre of the
fringe system is ⇒ d = 2aδ = 2a (µ − 1)α
 1  λD
Yn =  n +  ; n = 0,1, 2 , 3 , ... , etc.
 2 d
Diffraction of Light
(iii) Angular separation between nth bright fringe and the
central fringe is Diffraction of light is the deviation of light waves when it
Y nλ encounter any obstacle in its way.
θn = n = ; θ n is in radian.
D d Diffraction happens due to bending of light around the
(iv) Angular separation between nth dark fringe and the edges of obstacles.
central fringe is The phenomenon of diffraction is divided mainly in the
 1 λ following two classes
θ n = n + 
 2 d (i) Fresnel diffraction

where, n = 0,1 , 2 , 3, ...etc. (ii) Fraunhofer diffraction

@iitjeehelps
WAVE OPTICS 237

Diffraction due to a Single Slit Malus Law


To study the diffraction phenomenon, Young’s double slit When a beam of completely plane polarised light is incident
experiment was modified to single slit. on an analyser, the resultant intensity of light ( I )
transmitted from the analyser varies directly as the square
of cosine of angle ( θ ) between plane of transmission of
analyser and polariser.
S
i.e. I ∝ cos2 θ
Angle with planes of
transmission, θ = 90°

Slit Screen Unpolarised


light Polarised Analyser
light
From this experiment, it was observed that when a parallel
beam of monochromatic light falls normally on a narrow
slit, the diffraction pattern on a screen has bright central Intensity = I0
maxima borderd on both sides by secondary maxima of
Intensity = I0
rapidly decreasing intensity. Suppose, λ is the wavelength Polariser
2
of light and a is width of slit. Intensity = I0 cos2θ
2
I
3 λ 5λ here, 0 = Intensity of
(i) Thus, for bright fringes, sin θ = 0, , , K, etc. 2 polarised light
2a 2a
λ 2λ 3 λ
(ii) For dark fringes, sin θ = , , , K, etc.
a a a Brewster’s Law
According to this law, when unpolarised light is incident at
Width of Central Maxima an angle called polarising angle, i p on an interface
Angular width of central maxima =
2λ separating air from a medium of refractive index µ, then the
a reflected light is fully polarised (perpendicular to the plane
2fλ of incidence), provided
Linear width of central maxima = µ = tani p
a
P R
Intensity
Un l

ht ed
po igh

lig ris
la t

la
ris

Po
ed

ip ip
Air
X Y
Glass Q

–2λ –λ 0 λ 2λ
a a a a sin θ
Width of central maxima

where, λ = wavelength of light, a = width of slit


S
f = focal length of the convex lens placed close to
the slit This relation represents Brewster’s law.

@iitjeehelps
Practice Exercise
1. Light propagates 2 cm distance in glass of refractive 6. Newton’s rings are observed by keeping a spherical
index 1.5 in time t 0. In the same time t 0, light surface of 100 cm radius on a plane glass plate. The
propagates a distance of 2.25 cm in a medium. The wavelength of light used is 5880 Å. If the diameter of
the 15th bright ring is 0.590 cm, then calculate
refractive index of the medium is
diameter of the 5th ring.
a. 4/3
a. 0.226 cm b. 0.446 cm c. 0.336 cm d. 0.556 cm
b. 3/2
c. 8/3 7. If n coherent sources of intensity I 0 are superimposed
d. None of the above at a point, the intensity of the point is
2. Two non-coherent sources emit light beam of I0
a. nI 0 b. n 2I 0 c. n 3I 0 d.
intensities I and 4I. The maximum and minimum n
intensities in the resulting beam are 8. In Young’s double slit experiment with monochromatic
a. 9I and I b. 9I and 3I light of wavelength 600 nm, the distance between slits
c. 5I and I d. 5I and 3I is 10−3 m. For changing fringe width by 3 × 10−5 m,
3. In an interference pattern, the position of zeroth order a. the screen is moved away from the slits
maxima is 4.8 mm from a certain point P on the by 5 cm
screen. The fringe width is 0.2 mm. The position of b. the screen is moved by 5 cm towards the slits
second minima from point P is c. the screen is moved by 3 cm towards the slits
a. 5.1 mm b. 5 mm d. Both (a) and (b) are correct
c. 4 mm d. 5.2 mm 9. If Young’s double slit experiment, is performed in
4. A beam of light AO is incident on a glass slab (µ = 154
. ) water
in a direction as shown in the figure. The reflected ray a. the fringe width will decrease
OB is passed through a nicol prism. On viewing b. the fringe width will increase
through a nicol prism, we find on rotating the prism that c. the fringe width will remain unchanged
d. there will be no fringe
A N B 10. In Young’s double slit experiment, the spacing
between the slits is ‘d ’ and wavelength of light used is
6000 Å. If the angular width of a fringe formed on a
33° 33°
distant screen is 1°, then calculate ‘d ’.
O
a. 1 mm b. 0.05 mm c. 0.03 mm d. 0.01 mm
a. the intensity is reduced down to zero and remains zero 11. In Young’s double slit experiment, when violet light of
b. the intensity reduces down somewhat and rises again wavelength 4358 Å used, then 84 fringes are seen in
c. there is no change in intensity the field of view, but when sodium light of certain
d. the intensity gradually reduces to zero and then again wavelength is used, then 62 fringes are seen in the
increases field of view, calculate wavelength of sodium light.
5. A parallel beam of light of intensity I 0 a. 6893 Å b. 5904 Å c. 5523 Å d. 6429 Å
is incident on a glass plate, 25% of Air 12. In a Young’s double slit experiment, 5th dark fringe is
light is reflected by upper surface and formed opposite to one of the slits. The wavelength of
50% of light in reflected from lower light is
surface. The ratio of maximum to d2 d2 d2 d2
a. b. c. d.
minimum intensity in interference 6D 5D 15D 9D
region of reflected ray is
2 2
13. In the ideal double slit experiment, when a glass-plate
1 3 1 3 (refractive index 1.5) of thickness t is introduced in the
 +   + 
path of one of the interfering beams (wavelength λ ),
a.  2 8 b.  4 8
1 3 1 3 the intensity at the position where the central
 −   −  maximum occurred previously remain unchanged.
2 8 2 8
The minimum thickness of the glass plate is
5 8
c. d. 2λ λ
8 5 a. 2λ b. c. d. λ
3 3

@iitjeehelps
WAVE OPTICS 239
14. For the same objective, find the ratio of the least 18. In Fresnel’s biprism experiment, the distance between
separation between two points to be distinguished by biprism and screen is 4 m. The angle of prism is
a microscope for light of 5000 Å and electrons 2 × 10−3 rad, the refractive index of glass of biprism is
accelerated through 100 V used as the illuminating 1.5. The fringe width observed on the screen is
substance. 15 × 10−4 m. Find the number of fringes on the screen.
a. 1× 10−4 b. 0.2 × 10−3 c. 5 × 10−2 d. 7 × 10−4
a. 3 b. 2 c. 6 d. 8
15. One slit of a double slit experiment is covered by a 19. The first diffraction minimum due to single slit
thin glass plate of refractive index 1.4 and the other by diffraction is θ, for a light of wavelength 5000 Å. If the
a thin glass plate of the refractive index 1.7. The point width of the slit is 1 × 10−4 cm, then the value of θ is
on the screen, where the central maximum fall before
a. 30° b. 45°
the glass plate was inserted, is now occupied by what c. 60° d. 15°
had been the fifth bright fringe was seen before?
20. A screen is at a distance of 1m away from the
Assume the plate have the same thickness t and
aperture. If light of wavelength 500 nm falls on an
wavelength of light 480 nm. Then, the value of t is
aperture, then calculate the area of first HPZ and
a. 2.4 mm b. 4.8 mm c. 8 µm d. 16 mm
radius of third HPZ.
16. In a Young’s experiment, one of the slit is covered a. 1.57 mm 2, 1.22 mm
with a transparent sheet of thickness 3.6 × 10 cm −3 b. 1.22 mm 2, 1.57 mm
due to which position of central fringe shifts to a c. 1.65 mm 2, 2.79 mm
position originally occupied by 30th bright fringe. Find d. 2.63 mm 2, 0.22 mm
the refractive index of the sheet, if λ = 6000 Å. 21. The sodium yellow doublet has wavelengths 5890 Å
a. 1.5 b. 1.2 c. 1.3 d. 1.7 and λ Å and resolving power of a grating to resolve
these lines is 982, then value of λ is
17. A beam of light parallel P R
a. 5896 Å b. 5880 Å
Un

ht ed
po ligh

to central line AB is
lig ris
la t

la
ris

c. 5869 Å d. 5876 Å
Po
ed

incident on the plane of i i p p

Air
22. An unpolarised beam of light is incident on a group of
slits. The number of
X Y
Glass Q

minima obtained on the four polarising sheets which are arranged in such a
large screen is n1. Now, way that the characteristic direction of each polarising
if the beam is tilted by sheet makes an angle of 30° with that of the preceding
sheet. The percentage of incident light transmitted by
S

same angle ( ≠ 90° ) as


shown in figure, then the number of minima obtained the first polariser will be
is n2. Then, a. 100%
b. 50%
a. n1 = n 2 b. n1 > n 2 c. 25%
c. n 2 > n1 d. n 2 will be zero d. 125%

BITSAT Archives
1. The angular size of the central maxima due to a single 3. A mica slit of thickness t and refractive index µ is
slit diffraction is (a → slit width) [2014] introduced in the ray from the first source S1. By how
λ 2λ 3λ λ much distance of fringes pattern will be displaced?
a. b. c. d.
a a 2a 2a [2012]
d D
2. Find the final intensity of light (I ′′ ) , if the angle a. (µ − 1) t b. (µ − 1) t
D d
between the axes of two polaroids is 60°. [2014] d D
c. d. ( µ − 1)
( µ − 1) D d
Unpolarised I′
Light I′′ 4. A single slit diffraction is obtained using a beam of red
I0
light. If red light is replaced by blue light, what
happens ? [2009]
1st polaroid 2nd polaroid
a. The diffraction pattern disappears
3I I b. There is no change in the diffraction pattern
a. 0 b. 0 c. Diffraction fringes become narrower and crowded
2 2
I0 I0 together
c. d. d. Diffraction fringes become broader and farther apart
4 8

@iitjeehelps
240 SELF STUDY GUIDE BITSAT

5. Statement Using Huygens’ eyepiece measurements 7. The optical path of a monochromatic light is same if it
can be taken but are not correct. goes through 4.0 cm of glass of 4.5 cm of water. If the
Reason The cross wires, scale and final image are refractive index of glass is 1.53, the refractive index of
not magnified proportionately because the image of the water is [2007]
the object is magnified by two lenses, whereas the a. 1.30 b. 1.36 c. 1.42 d. 1.46
cross wire scale is magnified by one lens only. 8. In Young’s double slit experiment a minima is
Identify the correct one of the following. [2008] observed when path difference between the
a. Both (S) and (R) are true, (R) explains (S) interfering beam is [2006]
b. Both (S) and (R) are true, but (R) cannot explain (S) a. λ b. 1.5 λ c. 2 λ d. 2.25 λ
c. (S) is false, (R) is false
d. (S) is false, (R) is true
9. Two coherent light beams of intensity I and 4I are
superposed. The maximum and minimum possible
6. In Fraunhofer diffraction experiment, L is the distance intensities in the resulting beam are [2005]
between screen and the obstacle, b is the size of a. 9 I and I b. 9 I and 3 I c. 5 I and I d. 5 I and 3 I
obstacle and λ is wavelength of incident light. The
general condition for the applicability of Fraunhofer 10. When unpolarised light beam is incident from air onto
diffraction is [2008] glass (n = 1.5 ) at the polarising angle [2005]
b2 b2 a. reflected beam is polarised 100%
a. >> 1 b. =1
Lλ Lλ b. reflected and refracted beams are partially polarised
b 2
b 2 c. the reason for (a) is that almost all the light is
c. << 1 d. =/ 1 reflected
Lλ Lλ
d. All of the above

Answer with Solutions


Practice Exercise 5. (a) The intensity of light reflected from upper surface is
25 I 0
1. (a) For a given time, optical path remains constant I1 = I 0 × 25% = I 0 × =
100 4
∴ µ1x1 = µ 2x 2 Intensity of transmitted light from upper surface is
or 1.5 × 2 = µ 2 × 2.25 I 3I
1.5 × 2 2 20 4 I = I0 − 0 = 0
∴ µ2 = = = = 4 4
2.25 1.5 15 3 ∴ Intensity of reflected light from lower surface is
2. (d) When two coherent light beams of intensity l1 and l2
3I 0 50 3I I ( I1 + I 2 )2
superimpose, then maximum intensity is ( I1 + I 2 ) 2 and I2 = × = 0 ⇒ max =
4 100 8 I min ( I1 − I 2 )2
minimum intensity is ( I1 − I 2 ) 2. But when two
2 2
incoherent light beams of intensities I1 and I 2  I0 3I 0  1 3
 +   + 
superimposed, then maximum intensity is (I1 + I 2 ) and I max  4 8  2 8
minimum intensity is (I1 − I 2 ). = 2
= 2
I min  I 3I 0  1 3
3. (a) The distance between zeroth order maxima and  0 −   − 
second order minima is  4 8  2 8
β 3 3 6. (c) Let Dn+ p = diameter of (n + p )th ring
y1 = + β = β = × 0.2 mm = 0.3 mm
2 2 2
Dn = diameter of nth ring
∴ The distance of second maxima from point P is D15 = 0.590 cm, Ds = ?
y = ( 4.8 + 0.3) mm = 5.1 mm λ = 5880 Å , R = 100 cm
4. (d) As ip = tan−1 (1.54) = 57° and in the figure given P = 10
i = 90° − 33° = 57° = ip Dn2+ p − Dn2
λ=
A N B 4PR
−5 ( 0.59 + Dn ) ( 0.59 − Dn )
5880 × 10 =
4 × 10 × 100
33° 33°
Dn = 0.336 cm
O
7. (b) The resultant amplitude is A = na 0
∴ Reflected light along OB is plane polarised. On rotating
the nicol prism, intensity gradually reduces to zero and ∴ I ∝ A2
then increases again. ∴ I = kA 2 = k n 2a 02 = n 2I 0

@iitjeehelps
WAVE OPTICS 241
D ∆D Ratio of the least separation
8. (d) β = λ ⇒ ∆β = λ
d d d ′min 0.12 × 10−9
∴ = = 0.2 × 10−3
d∆β 3 × 10−5 × 10−3 d min 5500 × 10−10
∆D = =
λ 600 × 10−9
15. (c) Due to the introduction of the glass plate, the change
= 5 cm away or towards the slits in path difference is (µ 2 − µ1)t.
9. (a) As we know, ∴ Before inserting the glass plate, the path difference for
D
β= λ …(i) central maxima is zero.
d After introducing glass plate, the change in path difference
1 is equal to 5λ.
and λ∝ …(ii)
µ (µ 2 − µ1)t = 5λ
From Eqs. (i) and (ii), we get 5λ 5 × 480 × 10−9
1 1 t= = = 8 µm
β∝λ∝ ∴ β∝ µ 2 − µ1 . − 14
17 .
µ µ
16. (a) The position of 30th bright fringe
y 30λD
10. (c) Here, sin θ ≈ θ =  
D y 30 =
d
∆y
So, ∆θ = Now, position shift of central fringe is
D 30λD
Angular fringe width θ 0 = ∆θ (width ∆y = β) y0 =
d
β Dλ 1 λ
θ0 = = × = D
D d D d But we know, y 0 = (µ − 1)t
d
π
θ 0 = 1° = rad and λ = 60 × 10−7 m 30λD D 30λ
180 = (µ − 1)t ⇒ (µ − 1) =
d d t
λ 180
d = = × 6 × 10−7 30 × ( 6000 × 10 ) −10
θ0 π = = 0.5 ⇒ µ = 1.5
( 3.6 × 10−5 )
= 3.44 × 10−5m = 0.03 mm
11. (b) 17. (a) Fringe size will not change due to tilting
12. (d) For dark fringe, ∴ n1 = n 2
P
Yd λ S2 18. (c) The deviation,
= ( 2m − 1) Y=
d
D 2 d 2 S = (µ − 1) A = 0.5 × 2 × 10−3 = 10−3 rad
δ
d 2 δ C
Here, m = 5,Y = The length of band on either side
2 S1 of centre of screen
d d λ
∴ = ( 2 × 5 − 1) D = l = bδ = 4 t × 10−3 m b=4m
2D 2
d2 ∴The number of fringe on either side is
or = 9λ l 4 × 10−3
D n1 = =
d2 β 15 × 10−4
∴ λ=
9D 40
= = 2 .67
(µ − 1) tD λD 15
13. (a) We can write, shift = = fringe width, β =
d d ∴ Total number of fringes
∴ Thickness, t = 2λ = 2 [n1] + 1 = 2 [ 2 .67] + 1 = 6
. λ
122 19. (a) 20. (a)
14. (b) d min =
2 sin β 5890 + λ
21. (a) Mean wavelength, λ m =
where, λ is the wavelength of light and β is the angle 2
subtended by the objective at the object. Wavelength difference dλ = λ − 5890
For the light of wavelength 5500 Å Resolving power R of a grating
1.22 × 5500 × 10−10 5890 + λ
d min = …(i) λm
2 sin β R= = 2 = 982 ⇒ λ = 5896 Å
dλ λ − 5890
For electrons accelerated through 100 V, the de-Broglie
wavelength 22. (b) First polariser just polarises the unpolarised light.
12.27 12.27 Therefore, intensity of polarised light transmitted from first
λ= = = 0.12 × 10−9 m
V 100 polariser is
1.22 × 0.12 × 10−9 1
dmin = I 0 = 50% I 0
2 sin β 2

@iitjeehelps
242 SELF STUDY GUIDE BITSAT

BITSAT Archives
1. (b) λb
⇒ β blue = × β red
1st minima λr
Q λb < λr
θ=λ/a ∴ β blue < β red
θ′=2λ/a
∴ Diffraction fringes become narrower and crowded
together.
5. (a) Using Huygens’ eyepiece, measurements can be
taken but not accurately due to the reason given.
 λ  2λ 6. (c) The general condition for Fraunhofer diffraction is
So, angular size of central maxima is = 2   =
a a b2
<< 1
2. (d) From first polaroid the unpolarised light will become Lλ
polarised with half the intensity.
7. (b) Optical path µx = constant
I
So, I′ = 0 i.e. µ1x1 = µ 2x 2
2
From second polaroid ⇒ 1.53 × 4 = µ 2 × 4.5
I I 1 I 1.53 × 4
I ′′ = I ′ cos2 θ = 0 cos2( 60) = 0 = 0 ⇒ µ2 = = 136
.
2 2 4 8 4.5
D(µ − 1) t 8. (b) In Young's double slit experiment, a minima is
3. (b) Fringe displacement, x 0 =
d observed when path difference between the interfering
4. (c) For red light, λ = λ r beam is 1.5 λ.
D 9. (a) As I ∝ a 2 or a ∝ I
∴ β red = λ r …(i)
a a1 I a 1
∴ = ⇒ 1 =
For blue light, λ = λ b a2 4I a2 2
D 2 2
∴ β blue = λ b …(ii) I max  a1 + a 2   1 + 2 9
a =  =  =
D I min  a1 − a 2   1 − 2 1
λ
β blue a b λ b ∴ I max = 9I , I min = I
∴ = =
β red D
λr λr 10. (a) If unpolarised light is incident at polarising angle, then
a reflected light is completely, i.e. 100% polarised.

@iitjeehelps
21
Electric Charge

Electric Charge
Charge is the property associated with matter due to which it produces and experiences electric and
magnetic effects. Electric charge is of two types–positive and negative. Loss of electrons gives positive
charge and gain of electrons gives negative charge to a body. The charge on any body will be an
integral multiple of e, i.e. Q = ± ne where n = 1, 2 , 3...
The SI unit of charge is ampere ´ second = coulomb (C). Charge can also be described in mC, mC, nC.
i.e. 1 mC = 10-3 C, 1 mC = 10-6 C, 1 nC = 10-9 C
Its CGS unit is coulomb or esu. Its dimensional formula is [ AT] .

Continuous Charge Distribution


● When charge is distributed along a line, straight or curved, then it is called linear charge
distribution and its charge per unit length is called linear charge density ( l ).
Dq
i.e. l=
Dl
Its unit is coulomb/metre (Cm -1 ).
● When charge is distributed over a surface area, then it is called surface charge distribution and
charge over its per unit area is called surface charge density (s ).
Dq
i.e. s=
DS
Its unit is coulomb-metre -2 (Cm -2 ).
● When charge is distributed over entire volume of a body, then it is called volume charge
distribution and charge per unit volume is called volume charge density (r).
Dq
i.e. r=
DV
Its unit is coulomb-metre -3 (Cm -3 ).

@iitjeehelps
244 SELF STUDY GUIDE BITSAT

Coulomb’s Law Motion of a Charged Particle in an Electric


If q1 and q2 be two stationary point charges in free space Field
separated by a distance r , then the force of attraction or In this condition, some cases arise as below
repulsion between them is given by
Case I A charged particle is released from rest in an
k|q1 ||q2 | electric field E.
F =
r2 Then force on charged particle is given by F = qE
1 |q1 ||q2 | é 1 ù The acceleration produced by this force is given by
= × êQk = 4pe ú
4pe0 r2 ë 0û F qE
a= =
9 ´ 109 ´ | q1 || q2 | é 1 9ù
m m
= 2 êQ 4pe = 9 ´ 10 ú
r ë 0 û Since, E is constant, the acceleration a is also constant.
The term e0 is called the electric permittivity of free space Hence, the particle is uniformly accelerated. Let the particle
having a value of 8.85 ´ 10 –12 C 2N –1m –2 . Its dimensional starts from rest, then velocity of charged particle after time
t is given by v = u + at
formula is [M –1L–3T 4A 2 ].
æ qE ö æ qE ö
If some dielectric medium is completely filled between the v=ç ÷t çu = 0and a = ÷
èmø è mø
given charges, then the Coulomb’s force between them
becomes The distance travelled by the particle is given by
1 q1q2 1 1 qE 2
Fm = s = ut + at 2 Þ s = t
4p e r 2 2 2 m
1 q1q2
= The kinetic energy gained by the particle
4pe0 e r r 2
1 qq 1 q 2 E 2t 2
= × 1 22 = mv 2 =
4pKe0 r 2 2m
e Case II A charged particle y-axis
= e r or K –
e0 enters the field in
perpendicular direction.
FFree space é q1q2 ù Let a charged particle of P (x, y)
= êQ FFree space = ú
K ë 4p r 2 e0 û mass m and charge q, enters
the electric field along x-axis O x-axis
Here, e0 = absolute electric permittivity of the given u
with speed u. The electric
medium, K dielectric constant and e r is the relative
field E is along y -axis is
permittivity of the given medium.
given by
NOTE ● Force of two charges exerts on each other is not changed F y = qE
by the presence of a third charge.
and force along x-axis remains zero, i.e. Fx = 0
● Coulomb’s force between two protons is 1036 times the
\ Acceleration of the particle along y-axis is given by
gravitational force between them.
Fy qE
ay = =
Electric Field m m
The initial velocity is zero along y-axis (u y = 0).
The space surrounding an electric charge q in which
another charge q0 experiences a force of attraction or \ The deflection of charged particle along y-axis after time
1 qE 2
repulsion is called the electric field of charge q. t is given by y = u y t + a y t 2 = t
2 2m
Electric field vector E (also known as the electric field
intensity) at any point is given by Along x-axis there is no acceleration, so the distance
F covered by particle in timet along x-axis is given by x = ut
E = lim Eliminating t, we have
q0 ® 0 q
0
æ qE ö
where, q0 is a small positive test charge which experiences a y=ç x2÷
è 2mu 2 ø
force F at a given point.
y µ x2
The SI unit of electric field is NC -1 and it is also known as
This shows that the path of charged particle in
Vm -1 . The dimensional formula for electric field is
perpendicular field is a parabola.
[MLT –3 A –1 ].

@iitjeehelps
ELECTRIC CHARGE 245
l
Electric Field due to a Point Charge E= , for r > R,
2 pe0r
1. Electric Field at a Distance r from a Point l
Charge q is E= , for r = R
1 q 2 pe0 R
E= × 2
4pe0 r and E = 0, for r < R
If q1 and q2 are two like point charges, separated by a (ii) For a non-conducting charged cylinder, for r £ R,
distance r , a neutral point between them is obtained at a lr
point distant r1 from q1 , such that E=
r 2 pe0 R2
r1 =
é q2 ù l
ê1 + ú and E= for r > R
ë q1 û 2 pe0r
If q1 and q2 are two charges of opposite nature separated
3. Electric Field due to a Uniformly Charged
by a distance r , a neutral point is obtained in the
extended line joining them, at a distance r1 from q1 , such Thin Spherical Shell
that For a charged conducting sphere/ shell of radius R and
r total charge Q, the electric field is given by
r1 =
é q2 ù
ê - 1ú E
q
ë 1 û r<R

2. Electric Field due to a Charged Cylinder


(i) For a conducting charged cylinder of linear charge
density l and radius R, the electric field is given by O
R
E Emax E Emax
Case I E = 0, for r < R
r>R
R

Q
r<

r>R
Case II E = , for r = R
r

E ∝ 12
<

r 4pe0 R 2
E

O r=R r O r=R r Q
Case III and E = , for r > R
(a) Conducting cylinder (b) Non-conducting cylinder 4pe0r 2

@iitjeehelps
Practice Exercise
1. One brass plate is inserted between two charges. The 7. Two equal and oppositely charged particles are kept
force between two charges will be at some distance apart from each other. A spherical
a. remain the same b. increase surface having radius equal to separation between the
c. decrease d. fluctuate particles and concentric with their mid-point is
considered. Then,
2. A sure test of electrification is
a. electric field is normal to the surface at two points
a. attraction b. repulsion c. friction d. induction only
m0 b. electric field is zero at no point
3. In relativistic mechanics m = the equivalent
æ v2ö c. electric potential is zero at every point of one circle
ç1 - 2 ÷ only
è c ø d. All of the above
relation in electricity for electric charge is 8. Two bodies A and B of definite shape are placed near
a. q = q 0 b. q =
q0 one another. Electrostatic attraction is found between
æ v2 ö the bodies, then
ç1 - 2 ÷ a. both bodies must be positively charged
è c ø
b. both bodies must be negatively charged
q qv
c. q 0 = d. q = 0 c. both bodies must be oppositely charged
æ v2 ö c d. body A may be neutral
ç1 - 2 ÷
è c ø 9. If s = surface charge density, e = electric permittivity,
s
4. Two positively charged particles each having charge the dimension of are same as
Q and are d distance apart. A third charge is e
introduced in midway on the line joining the two a. electric force b. electric field intensity
charges. Find nature and magnitude of third charge, c. pressure d. electric charge
so that the system is in equilibrium. 10. What is the magnitude of a point charge due to which
+Q q +Q the electric field 30 cm away has the magnitude 2?
[1/ 4 pe 0 = 9 ´ 109 Nm 2 / C2 ]
d/2 d/2
a. 2 ´ 10-11 C b. 3 ´ 10-11 C
-Q Q 3Q 3Q c. 5 ´ 10-11 C d. 9 ´ 10-11 C
a. q = b. q = c. q = d. q = -
4 4 4 4
11. A charged particle of mass m and charge q is released
5. As shown in the adjoining figure two charge particles from rest from position (x ,0, 0) in a uniform electric field
^
each having charge q and mass m are d distance E 0 j. The angular momentum of the particle about
apart from each other. If two particle are in equilibrium origin
under the gravitational and electric force, then a. is zero b. is constant
determine the ratio q / m . c. increases with time d. decreases with time
m m 12. An infinite plane sheet of positive charge has surface
q q
d charge density s. A metallic ball of mass m and
-8 -10 10 charge +Q is attached to a thread and is tied to a point
a. 10 b. 10 c. 10 d. None A on the sheet PQ. The angle q made by the string
6. Charges + 2Q and - Q are placed as shown in figure. with the plane sheet is in equilibrium is
The point at which electric field intensity is zero will be æ Qs ö
a. zero b. tan-1ç ÷
è 4e 0mg ø
–Q +2 Q
æ Qs ö æ Qs ö
E c. tan-1ç ÷ d. tan-1ç ÷
è 2e 0mg ø è e 0mg ø
a. somewhere between - Q and + 2Q
13. A dimensionless body having a physical quantity
b. somewhere on the left of - Q varies as 1 / r 2, where r is distance from the body. This
c. somewhere on the right of + 2Q physical quantity may be
d. somewhere on the right bisector of line joining - Q a. gravitational potential b. electric field
and +2 Q c. gravitational field d. None of these

@iitjeehelps
ELECTRIC CHARGE 247
14. The electric field inside a conductor 20. A small element l is cut from a circular ring of radius a
a. must be zero and l charge per unit length. The net electric field at
b. may be non-zero the centre of ring is
c. must be non-zero ll
d. Both (a) and (c) are correct a. zero b.
4pe 0a 2
A l
15. A point charge Q is situated at point B on c. ¥ d.
the ground. A point charge q of mass m 4pel
is vertically dropped along line AB from 21. A circular ring carries a uniformly distributed positive
a multi-storey building of height h. Find Q charge and lies in XY -plane with the centre at origin of
the position of the point charge q when it B coordinate system. If at a point ( 0, 0, Z ) electric field isE ,
is in equilibrium which of the following graphs is correct?
æ qQ ö Q E E
a. ç ÷ b.
è 4pe 0mg ø h2
qh a. b.
c. d. None of these O Z O Z
m
16. Two point charges q1 and q 2 are released from rest in a
gravity free hall when distance between them is a. Find E E
the maximum speeds of charged particle. The mass of O
each charged particle is m. c. d.
O Z Z
qq æ q1q 2 ö
a. 1 2 b. ç ÷
4p e 0a è 4p e 0ma ø
22. In the Q.21 find the separation of the point mass and
æ 2q1q 2 ö
c. ç ÷ d. None of these wall at the equilibrium position of the mass.
è 4p e 0ma ø qE qE
a. L + b. L -
–q +q k k
17. Six point charges are arranged at qE
c. -L d. None of these
the vertices of a regular hexagon k
of side length a (shown in figure). – q O +q
Find the magnitude of electric field 23. In the Q.21 find energy stored in spring at the
at the centre of regular hexagon. equilibrium position of the point mass.
q +q +q q 2E 2 1 2
a. b. zero a. b. kE
4p e 0a 2 2k 2
q 2E 2
c.
q
d. None of these c. d. None of these
k
2p e 0a 2
18. A positively charged particle P enters the region 24. A particle of mass m and having a charge q is placed
between two parallel plates with a velocity u, in a on a smooth horizontal table and is connected to walls
direction parallel to the plates. There through unstressed springs

of constant k (shown in E
is a uniform electric field in the k k
region. P emerges from this region P figure). A horizontal electric m
u
with a velocity v . Taking C as a field E parallel to spring is
constant, v will depend on u as + switched On. The maximum
speed of the particle is
a. v = Cu b. v = u 2 + Cu
C C qE qE
c. v = u 2 + d. v = u 2 + a. b.
u u2 2mk mk
qE
q c. d. None of these
19. A point charge Q is placed at the centre m
of a circular wire of radius R having Q 25. A point charge is projected along the axis of circular
charge q. Find the force of electrostatic ring of charge Q and radius 10 2 cm. The distance of
O
interaction between point charge and the point charge from centre of ring, where acceleration
the wire. of charged particle is maximum, will be
qQ
a. b. zero a. 10 cm
4pe 0R 2 b. 20 cm
q2 c. at infinity
c. d. None of these
4pe 0R d. None of the above

@iitjeehelps
248 SELF STUDY GUIDE BITSAT

26. If a charged particle is projected on maximum distance covered by an electron in vertical


E
a rough horizontal surface with q direction above its initial level.
speed v 0. Find the value of dynamic m v0 a. 14.2 mm
coefficient of friction, if the kinetic b. 15 mm
energy of the system is constant. c.12.6 mm
qE qE d. 14.2 cm
a. b.
mg m
q 31. A pendulum bob of mass m and charge q is
c. d. None of these
g suspended by a thread of length l. The pendulum is
placed in a region of a uniform electric field E directed
27. Two charges of value 2 mC and - 50 mC are placed at
in distance 80 cm apart. Calculate the distance of the vertically upward. If the electrostatic force acting on
point from the smaller charge, where the intensity will the sphere is less than that of gravitational force,
be zero. calculate the period with which the pendulum
a. 20 cm b. 35 cm c. 30 cm d. 25 cm oscillates (Assume small oscillation).
28. Two charged particles of charge + 2q and +q have l l
a. T = 2p b. T = 2p
masses m and 2m, respectively. They are kept in qE qE
g+ g-
uniform electric field and allowed to move for the m m
same time. Find the ratio of their kinetic energies. l l
c. T = p d. T = p
a. 1 : 8 b. 16 : 1 c. 2 : 1 d. 3 : 1 g-
qE
g+
qE
m m
29. An oil drop of charge of 2 electrons fall freely with a
terminal speed. Calculate the mass of oil drop so, it 32. Identical charges of magnitude Q are placed at (n - 1)
can move upward with same terminal speed, if electric corners of a regular polygon of n sides each corner of
field of 2 ´ 103 V/m is applied. the polygon is at a distance r from the centre. The
a. 3.0 ´ 10-17kg b. 3.2 ´ 10-17kg field at the centre is
c. 2.5 ´ 10-17kg d. 3.3 ´ 10-17 kg kQ kQ
a. b. (n -1)
r2 r2
30. An electron is projected with velocity 107m/s at an n kQ (n - 1) Q
angle q ( = 30° ) with horizontal, in a region of uniform c. . 2 d. k 2
(n -1) r n r
electric field of 500 N/C vertically upwards. Find the

BITSAT Archives
1. A ring shaped conductor with radius a carries a net 2. The electrostatic force of repulsion between two
positive charge q uniformly distributed on it as shown positively charged ions carrying equal charge is
in figure. A point P is situated at a distance x from its 3.7 ´ 10-9 N, when they are separated by a distance of
centre. Which of following graph +++++ 5 Å. How much electrons are missing from each ion?
shows the correct variation of electric + + +x P
+ + a. 10 b. 8 c. 2 d. 1 [2014]
field (E ) with distance (x )? [2014] + +
+a + + 3. Two equal charges q are kept fixed at a and+ a
+ +
+++ q
along the x-axis. A particle of mass m and charge is
2
E E brought to the origin and given a small displacement
along the x-axis, then [2012]
a. x a. the particle executes oscillatory motion
b. x b. the particle remains stationary
c. the particle executes, SHM along x-axis
d. the particle executes SHM along y-axis
4. The magnitude of electric field intensity E, such that
E
an electron placed in it would experience an electrical
c. x d. None of these force equal to its weight is given by [2011]
mg e e2
a. mge b. c. d. g
e mg m2

@iitjeehelps
ELECTRIC CHARGE 249
5. Which one of the following is correct statement? [2009] 7. An electric line of force in the xy -plane is given by
a. Electric field is always conservative equation x 2 + y 2 = 1 . A particle with unit positive
b. Electric field due to varying magnetic field is charge, initially at rest at the point x = 1, y = 0 in the
conservative
xy -plane [2007]
c. Electric field is conservative due to electrostatic
charges while non-conservative due to time varying a. not move at all
magnetic field b. will move along straight line
d. Electric field lines are always closed loops c. will move along the circular line of force
d. information is insufficient to draw any conclusion
6. A charge of 1 mC is divided into two parts such that
8. Identify the wrong statement in the following.
their charges are in the ratio of 2 : 3. These two
Coulomb’s law correctly described the electric force
charges are kept at a distance 1 m apart in vacuum.
that [2005]
Then, the electric force between them (in N) is [2008]
a. binds the electrons of an atom to its nucleus
a. 0.216
b. binds the protons and neutrons in the nucleus of an
b. 0.00216 atom
c. 0.0216 c. binds atoms together to form molecules
d. 2.16 d. binds atoms and molecules to form solids

Answer with Solutions


Practice Exercise 11. (b) Angular momentum, L = r ´ p = rp sin 90°
= rp = rmv = constant
1. (b) 2. (b) 3. (a)
4. (a) Condition for equilibrium qE
v = —0 t j
m
kQ 2 kqQ kQ
+ = 0 Þ 2 (Q + 4q ) = 0 y E0
d2 (d / 2)2 d
-Q
q= (x, 0, 0)
4 x
5. (b) Condition for equilibrium Fe = Fg
kq 2 Gm 2 12. (c) By using Lami’s theorem,
=
d2 d2 QE Qs
2 Þ tan q = =
1 q Gm 2 mg 2 e 0mg
Þ =
4pe 0 d 2 d2
q2 + +
or = G ( 4pe 0 ) + + ++
m2 + +θ ++
+ + ++
q G ´ 4pe 0 + + ++
= T + + ++
m 1 + + ++
θ + + ++
6.67 ´ 10-17 ++
= » 10-10 QE
Q + +
9 ´ 109 + +
++
++
+ +
++
6. (b) mg
7. (d)
æ Qs ö s
8. (d) Attraction may take place between like charged q = tan-1ç ÷ (QE = field of plate = )
bodies, unlike charged bodies and charged body and è 2 e 0mg ø 2 e0
neutral body. Hence, (d) is correct.
13. (b) 14. (b) Fe
9. (b)
q 15. (a) For equilibrium, q
10. (a) Electric field due to a point charge, E = qQ
4pe 0 r 2 mg = Fe or mg = mg y
4pe 0y 2
2
1 æ 30 ö
\ q = E ´ 4pe 0 r 2 = 2 ´ ´ç ÷ æ qQ ö
9 ´ 109 è 100 ø \ y = ç ÷ Q
è 4pe 0mg ø
Þ q = 2 ´ 10-11C

@iitjeehelps
250 SELF STUDY GUIDE BITSAT

16. (b) When separation between charges is infinitely large, 25. (a) 26. (a)
potential energy is zero and kinetic energy of system is 27. (a) E2
maximum. According to conservation principle of energy, E1
q2 = 50 µC q1 = 2 µC
a
F F 80 cm x
q1 q2 (0.8 + x) m
1 2 1 2 qq 2 qq
mv max + mv max = 1 2 or mv max = 1 2 \ E = E1 - E2 or 0 = E1 - E2 \ E1 = E2
2 2 4pe 0a 4pe 0a
kq1 kq 2
or =
æ qq ö x 2
( 0.8 + x )2
\ v max = ç 1 2 ÷
è 4pe 0ma ø 2 ´ 10-6 50 ´ 10-6
or =
17. (c) From figure, the resultant electric field is x2 ( 0.8 + x )2
–q a +q \ x = 0.2 m = 20 cm
a E0 a 28. (b) Let E = uniform electric field
a E0 Let m1 = m, m2 = 2m
–q +q
E0 E0 Force on particles,
a a
E0 E0 F1 = ma1 = 2qE
+q a +q Þ F2 = 2ma 2 = qE
2qE qE
2q q a1 = and a 2 =
E = 2E0 = 2
= m 2m
4pe 0a 2pe 0a 2
Final velocities of the particle
l l
18. (d) u x = u , t = v1 = u1 + a1t
u
Eq al P u 2qE qE
Þ ay = = a Þ v y = at = Þ v1 = t Þ v2 = t
m u m 2m
a 2l 2 C C Let K = kinetic energy
Þ v 2 = u2 + 2 = u2 + 2 Þ v = u2 + 2 2 2
u u u 1 1 æ 2qE ö æ qE ö
K1 = m1v12 = m ç t ÷ = ( 2m ) ç t÷
2 2 è m ø èm ø
19. (b)
2
20. (b) Enet = electric field due to ring at centre 1 æ qE ö 2 K 16
Similarly, K 2 = m ç ÷ t Þ 1 =
without cutting portion electric field due to cutting 2 è 2m ø K2 1
C
element is 29. (b) mg = F = 6phrv …(i)
q0 ll
Enet = 0 - =- qE - mg = 6phrv = F …(ii)
4pe 0a 2 4pe 0a 2
ll From Eqs. (i) and (ii), we get
\ | Enet | =
4pe 0a 2
qE = 6phrv + mg = 2mg
21. (b) At Z = 0, E = 0 and between Z = 0 and Z = ¥ and E 2mg 2mg mg
E= = =
grows to a maximum value, which can be found by q 2e e
dE
= 0. Also, graph (b) should be symmetric, which is (b). 2 ´ 103 ´ 16
. ´ 10-19
dt m= = 3.2 ´ 10-17kg
10
22. (a) At equilibrium position,
30. (a)
qE
qE = k x 0 \ x 0 = 31. (b) Resultant force vertically downwards (mg - qE )
k
qE Net acceleration
\ Separation = L +
k æ qE ö qE
g ¢ = çg - ÷ θ
23. (a) è mø l

24. (a) The compression in spring at equilibrium = x 0 æ l ö


T = 2p ç ÷ o sθ
qE è g ¢ø E)c
\ 2k x 0 = qE Q x 0 = = A = amplitude
θ

–q
in

2k
)s

g
(m
qE

l
k + k2 2k T = 2p mg

w= 1 æ qE ö
g

Also, =
(m

m m çg - ÷
è mø
1 2 1 qE
mv max = mA 2w2Þ v max = 32. (a)
2 2 2mk

@iitjeehelps
ELECTRIC CHARGE 251

BITSAT Archives
1. (c) The net electric field at point P is given by 3. (c) From Coulomb’s law, q q/2 q
qx $i 1 q1q 2 (–a,0) x (+a,0)
E= F =
4pe 0(x 2 + a 2 )3/ 2 4pe 0 r 2
\At the centre of ring x = 0 Þ E0 q q
q´ q´
1 2 - 1 2
1 q F = ×
If x >> a, E = 4pe 0 (a + x )2 4pe 0 (a - x )2
4pe 0 x 2
1 q2 é 1 1 ù
= ê 2
- ú
E 4pe 0 2 ë (a + x ) (a - x )2 û
Emax 1 q2 é 4ax ù
a = × ê- ú
4pe 0 2 ë (a 2 - x 2 )2 û
2 O
x
a When x << a, then
2 2q 2
Emax F =- x ÞF µ -x
4pe 0a 3
Hence, SHM is along x-axis.
and E will be maximum where,
4. (b) Force on electron
dE a mg
= 0 or x = ± | F | = qE = eE = mg Þ E =
dx 2 e
1 2q 5. (c) Electric field is conservative due to electrostatic
and Emax =
4pe 0 3 3a 2 charges, while non-conservative due to time varying
magnetic field.
2. (c) Here, F = 3.7 ´ 10-9 N
6. (b) Ratio of charges = 2 : 3
Let, q1 = q 2 = q 2 3
\ q1 = ´ 5 ´ 1mC and q 2 = ´ 1mC
Þ r = 5 Å = 5 ´ 10-10 m 5 5
1 q1q 2 Electrostatic force between the two charges
Q F = 1 q1q 2
4p e 0 r 2 F =
4 pe 0 r 2
q ´q
Þ 3.7 ´ 10-9 = 9 ´ 109 9 ´ 109 ´ 2 ´ 10-6 ´ 3 ´ 10-6
(5 ´ 10-10 )2 =
5 ´ 5 ´ (1)2
3.7 ´ 10-9 ´ 25 ´ 10-20
q2 = = 2.16 ´ 10-3 N
9 ´ 109
~
- 0.00216 N
= 10.28 ´ 10-38 7. (c) Charge will move along the circular line of force
or q = 3.2 ´ 10-19 C because x 2 + y 2 = 1 is the equation of circle in xy-plane.

q 3.2 ´ 10-19 8. (b) Coulomb’s law is applicable for charged particles, it is


Now, q = ne \ n= = =2 not responsible to bind the protons and neutrons in the
. ´ 10-19
e 16 nucleus of an atom.

@iitjeehelps
22
Gauss’s Law and Electric
Potential Theory

Electric Flux
The electric flux linked with any surface in an electric field is basically a measure of total number of
lines of forces passing normally through the surface. Flux is an imaginary scalar physical quantity.
Electric flux through an elementary area dS is defined as the scalar product of dS
area of field.
i.e. dφ = E ⋅ dS = E dS cosθ E
θ
Hence, flux from complete area (S),
φ = ∫ E dS cosθ

Outward flux is taken to be positive, while inward flux is taken to be negative.


N-C
SI unit of electric flux is (volt × metre) or and its dimensional formula is [ ML3T −3 A −1 ].
m2

Gauss’s Law
Gauss’s law states that flux is produced by charge. So, total electric flux through a closed surface
Q 1
enclosing a charge = en , φ net = Qen
ε0 ε0

Gauss’s law is the consequence of Coulomb’s law. According to Gauss’s law,


Q
∫ E ⋅ dS = εen0

@iitjeehelps
GAUSS'S LAW AND ELECTRIC POTENTIAL THEORY 253
NOTE In figure shown below, E is the net electric field due to both Electric Field due to Charged Conducting
the charges Q1 and Q 2 but Qen = Q1.
Sphere
dS
If a charge on a conducting sphere of radius R is Q and σ is
θ the surface charge density as shown in figure, then electric
Q1
E field in different situations are
Q2
+Q P +Q +Q
+ ++ + + +P + ++
+ + + + + +
Applications of Gauss’s Law +
r
+ + r + +
r
P +
+ + + + + +
R R R
Electric Field due to Infinitely Long Line of +
+ +
+ +
+ +
+ +
+ +
+
+ + +
Charge (a) Outside (b) At the surface (c) Inside
Let us consider a uniformly charged wire of an infinite
length having a constant linear charge density, i.e. (i) Outside the sphere If point P lies outside the
 Charge  sphere, then
λ =  . We have to calculate electric field at point P 1 Q σR2
 Length  E out = ⋅ 2 = (Q = σ × A)
4πε0 r ε0r 2
which is at a distance r as shown in figure.
+
+ dS
(ii) At the surface of sphere At the surface, r = R
+ 1 Q σ
+ So, ES = ⋅ 2 =
+a 4πε0 R ε0
+
+ dS (iii) Inside the sphere Inside the conducting charge
l +
+
r P sphere, electric field is zero.
+
+
+ R
+ dS O
+
Q Q
∫ E dS = ε0 ⇒ E ∫ dS =
ε0
E
Eout ∝
1

Ein= 0
r2
Q
⇒ E × 2 π rl = O
ε0
∴ E in = 0
Q λ 2kλ  1 
⇒ E= = = Q k = 
2 πε0rl 2 πε0r r  4πε0  Uniformly Charged Non-conducting Sphere
Suppose a charge Q is uniformly distributed in the volume
Electric Field due to Charged Circular Ring of a non-conducting sphere of radius R as shown in figure
Suppose we have a charged circular ring of radius R and below.
P
charge Q. On its axis, electric field is to be determined at a
point x distance away from the centre of the ring. +++++++++++++++++ P+
+++++++++
++++++++ ++++++++ ++ ++++++++++
+ ++ r
+++++ + +++++++ +++ r +++++r +P++
+
+ +
+ +++++ ++++++++ ++++ +++++ ++++
+ + E +++++R ++ +++++++R+ +++ +++++R+ +++
+ R + ++++++++++++++++++ +++++++++
+ + P (a) Outside (b) At the surface (b) Inside
–x +x
+ x + E O R (i) Outside the sphere If point P lies outside the
+ + x– R
+ + √2 √2 sphere, then
+ + 1 Q
+ + E out = ⋅
++ + 4πε0 r 2
kQx
E= If the sphere has a uniform volume charge density,
( x + R2 )3 / 2
2
Q ρR3
ρ= , then E out =
At centre, x = 0, so E centre = 0. 4 3 3 ε0r 2
πR
kQ 3
At a point on the axis such that x >> R, =
x2 (ii) At the surface of sphere
R kQ 1 Q ρR
If x = ± , then E max = At the surface, r = R , so E S = ⋅ 2 =
2 3 3R 2
4πε0 R 3 ε0

@iitjeehelps
254 SELF STUDY GUIDE BITSAT

(iii) Inside the sphere At a distance r from the centre, Electric Field due to a Dipole
1 Qr ρr
E in = ⋅ = ( i.e. E in ∝ r ) 1. At a point distant r from the centre of a dipole, along its
4πε0 R3 3 ε0 axial line
1 2 pr
Infinite Thin Plane Sheet of Charge E= ⋅
4πε0 (r 2 − a 2 )2
In this case, field is uniform and does not depend on the
distance from the plate. (direction of E is the same as that of p)
For a short dipole, i.e. r > > a.
++ 1 2p
++ + E= ⋅
+ ++ 4πε0 r 3
++ ++
E ++ E
A ++
+
2. At a point distant r from the centre of a dipole along its
+
++ ++ equatorial line
++ 1 p
E=− ⋅
4πε0 (r 2 + a 2 )3 / 2
σ
So, E= ( i.e. E ∝ r 0 ) (direction of E is opposite to that of p)
2 ε0
For a short dipole,
Electric Field due to Two Thin Infinite Plane 1 p
E=− ⋅ 3 [r > > a]
4πε0 r
Parallel Sheets of Charge
Consider two large uniformly charged parallel sheets A and 3. At a point distant r from the centre of a short dipole
B having surface charge densities σ A and σ B respectively. along a line inclined at an angle θ with the dipole axis
1 p
1 E= ⋅ 3 3 cos2 θ + 1
At P, E P = − ( E A + E B ) = − (σ A + σ B ) 4πε0 r
2 ε0
E Er
+ +
+++ ++
++ B
EA ++
+++
++ ++
++ ++
+++++ EA EB +++++ EA
+++++ +++++ EQ P (r, θ)
EB P +++++
+++++ Q ++++
++++
+R EB
+
+++ +++
+ +
z

1 θ
At Q, EQ = ( E A − E B ) = (σ A − σ B ) –q O +q
2 ε0
2d
1
At R, E R = ( E A + E B ) = (σ A + σ B ) 1
2 ε0 4. E subtends an angle β from r such that, tan β = tan θ
2
NOTE Gauss’s law is easily applicable to symmetric charge
distribution.
Torque on a Dipole in a Uniform
Electric Field
Electric Dipole When a dipole is placed in an external electric field making
An electric dipole consists of two equal and opposite an angle θ with the direction of the uniform electric field E ,
charges separated by a small distance. it experiences a torque given by τ = qE × AC
The dipole moment of a dipole is defined as the product of A
qE
the magnitude of either A p +q
B E
charges and the distance –q +q 2d θ
between them. Therefore, 2a
dipole moment p = q(2a ).
qE – C
q B
Dipole moment is a vector whose direction is from negative
to positive charge. Its SI unit is coulomb-metre, i.e C-m and τ = p × E = pE sin θ
has the dimensional formula [LTA].
or qE × 2 d sinθ = (q × 2d ) E sin θ

@iitjeehelps
GAUSS'S LAW AND ELECTRIC POTENTIAL THEORY 255

Work Done 2. Electric Potential due to a System of


If an electric dipole initially kept in an uniform electric field Charges
E making an angle θ 1 is rotated so as to finally subtends an If a number of charges q1 , q2 , q3 …are present in space, then
angle θ 2 , then the work done for rotating the dipole is the electric potential at any point will be
W = pE (cos θ 2 − cos θ 1 ) V = V 1 + V 2 + V3 +…
1 q1 q2 q3  1 n
 qi 
Potential Energy of a Dipole = 
4πε0  r1
+ + +… = ∑  
r2 r3  4πε0 i = 1  ri 
It is the amount of work done in rotating an electric dipole
from a direction perpendicular to electric field to a
particular direction. Hence, U = − pE cos θ or U = − p ⋅ E.
3. Electric Potential due to an Electric
Obviously, potential energy of an electric dipole is a scalar
Dipole
quantity. It is measured in joule. At any general point,
1 p cosθ
V =
4πε0 r 2
Electric Potential On the dipole axis, θ = 0°
Electric potential at a point in an electric field is defined as 1 p
and V = ⋅ 2
the amount of work done in bringing a unit positive charge, 4πε0 r
without any acceleration from infinity to that point, along
On the equatorial axis, θ = 90° and V = 0
any arbitrary path.
Mathematically, if W work is to be done to bring a test 4. Electric Potential due to Some Common
charge q0 from infinity to a point, then the potential of that Charge Distributions
point is Potential at a point distant r from an infinitely long wire
W λ
V = having linear charge density λ is V = ⋅ ln r
q0 2 πε0
1J For a charged conducting V r < R
SI unit of potential is volt, where 1 V = . Its dimensional
1C sphere/shell having total charge
Q and radius R, the potential at a
formula is [ML2T –3 A –1].
point distant r from the centre of r>R
Electric potential is a state function and does not depend on the sphere/shell is
the path followed. Q
(i) V = , for r > R O
4πε0r r=R r
1. Electric Potential due to a Point 1 Q
(ii) V = , for r = R
Charge 4πε0 r
Potential due to a point charge Q at a distancer is given by Q
(iii) V = , for r ≤ R
1 Q 4πε0 R
V = ⋅
4πε0 r For a charged non-conducting (dielectric) sphere of radius
R, the charge Q is uniformly distributed over the entire
● At the centre of the line joining two equal and opposite
volume.
charges,V = 0 but E ≠ 0.
Hence,
● At the centre of the line joining two equal and like Q
(i) V = , for r > R
charges, E = 0 butV ≠ 0. 4πε0r
● If four identical charges q each are placed at the four Q
(ii) V = , for r = R
vertices of a square then the net electric field at the 4πε0 R

centre of the square is zero, butV =


2q Q 3 R2 − r 2 
(iii) V =  , for r < R
πε0a 4πε0  2 R3 

@iitjeehelps
256 SELF STUDY GUIDE BITSAT

At the centre of the sphere (r = 0)


Relation between E and V
V r<
R Because E is force per unit charge and V is work per unit
charge. E and V are related in the same way as work and
r>R force. If ∆V = (V B − V A ) is the increase in potential over a
short displacement ∆s , ∆V = − E∆s
O where the negative sign indicates that the work is done
r=R r
against the field. Ifα is the angle between E and ∆s, we have
3Q 3  Q 
V = = Vs V s = 4πε R ∆V = − E ( ∆s )cosα
8 πε0 R 2  0 
= − E∆ x (along E)
where, ∆x = ∆s cosα is the component of ∆s along E.
Electric Potential Energy Therefore, E=−
∆V
The electric palatial energy of a system of charges is the work ∆x
that has been done in bringing those charges from infinity to For small changes, above expression may be written as
near each other to form the system. For two point charges q1 −dV
and q2 separated by distance r12 , the potential energy is given E=
dx
by
1 q1q2 Thus, the electric field intensity E is the gradient of potential.
U =
4πε0 r12 This means that the decrease in potential is along the
direction of E. The SI unit of E is volt per metre (Vm −1 ).
Electric potential energy is a scalar quantity.

Practice Exercise
1. A surface S = 10$j is kept in an electric field. 4. A point charge Q is placed at the centre of a
E = 2$i + 4 $j + 7k$ . How much electric flux will come out hemisphere. Find the ratio of electric flux passing
through curved surface and plane surface of the
through the surface?
hemisphere.
a. 40 units
a. 1 : 1 b. 1 : 2
b. 50 units
c. 2π : 1 d. 4π : 1
c. 30 units
d. 20 units 5. What should be the flux linked with the cube, if a point
charge q is placed at one corner of a cube?
2. Electric field at point P is given by E = r E 0. The total q q q q
flux through the given cylinder of radius R and height h is a. b. c. d.
ε0 2ε 0 3ε 0 8ε 0
6. Find the electric flux passing through the sphere, if an
electric dipole is placed at the centre of a sphere.
r
1 2
P a. b.
ε0 ε0
c. Zero d. None of these
O
7. A point charge q = 2 × 10−7 C
a. E0πR 2h b. 2E0πR 2h
is placed at the centre of a
c. 3E0πR 2h d. 4E0πR 2h spherical cavity of radius 3 cm
3. A point charge Q is placed at the centre of a in a metal piece. Points a and
b
hemisphere. Find the electric flux passing through flat b are situated at distances 1.5
surface of hemisphere. cm and 4.5 cm respectively
a.
Q
b. Zero from the centre of cavity.
ε0 The electric field intensities at a and b are
Q
c. d. None of these a. 8 × 106 N/C and zero b. zero and zero
2 ε0
c. zero and 8.9 × 105 N/C d. None of these

@iitjeehelps
GAUSS'S LAW AND ELECTRIC POTENTIAL THEORY 257
q1
8. In the given figure, two point 14. Calculate the earth's potential. Assume earth has a
charges q1 and q 2 are placed at a surface charge density of 1 electron/metre 2.
distances a and b from centre of a (Given, the electronic charge = −1.6 × 10−19 C, earth's
metallic sphere having charge Q. R radius = 6.4 × 106 m, ε 0 = 8.9 × 10−12 C 2/Nm 2)
Find electric field due to the O P b q2
metallic sphere at the point P . a. −0.115 V b. 0.215 V c. − 0.225 V d. 0.185 V

1  q1 
2
q 
2 15. An electron is released from rest at one point in
a.  2  +  22  uniform field and moves a distance of 10 cm in 10−7 s.
4πε 0 a  b 
What is the voltage between the points?
1 Q
b. a. 10 V b. 7 V c. 11.4 V d. 8 V
4πε 0 R2
2 2 16. In the figure, the charge appears on the sphere is
1 Q q q 
c.   +  12 + 22 
4πε 0  R 2  a b 
d. None of the above R q
9. Find the minimum surface density of charge on the d
plate, so that a body of mass 2 kg/m 2 may just be
lifted. qd qr
a. 2.84 × 10−5 C/m 2 b. 2 . 25 × 10−5 C/m 2 a. q b. c. − d. 0
r d
c.1.86 × 10−5 C/m 2 d. None of these
17. At the eight corners of a cube of side 10 cm, equal
10. Find the surface density of electric charge at a place charges each of value 10 C are placed. Calculate the
on the earth’s surface, where the rate of fall of potential at the centre of the cube.
potential is 2.5 V. a. 83 .14 × 1011 V b. 16.62 × 1011 V
−9 2 −9 2
a. 2.0 × 10 C/m b. 2.21 × 10 C/m c. 1.66 × 1011 V d. 1662.7 × 1011 V
c. 3.36 × 10−9 C/m 2 d. 3.5 × 10−9 C/m 2
18. Calculate the work required to bring a unit positive
11. Four point charges q1, q 2, q 3 and q 4 are placed at the charge from infinity to a mid-point between two
corners of the squares of side a, as shown in figure. charges 20 µC and 10 µC separated by a distance of
Calculate the potential at the centre of the square. 50 m.
q1 a q2 a. 10.8 × 104 J b. 10.8 × 103 J
. × 106 J
c. 108 d. 0.54 × 105 J

a P a 19. Two charges +q and −3q are placed at a distance of


1 m apart. Find out the points on the line joining two
charges, where electric potential is zero.
q4 a q3 a. 0.25 m, 0.5 m b. 1 cm, 0.50 m
c. 0.35 cm, 24 cm d. None of these
(Given, q1 = 1× 10−8 C, q 2 = − 2 × 10−8 C, 20. Two points are at distance r1 and r 2 (r1 < r 2 ) from a long
q 3 = 3 × 10− 8 C, q 4 = 2 × 10−8 C and a = 1m) string having charge per unit length σ. The potential
a. 507 V b. 607 V c. 550 V d. 650 V difference between the points is proportional to
r  1 r2
12. Over a thin ring of radius R a charge q is distributed a. σ b. log 2  c. d.
 r1  σ r1
non-uniformly. Calculate the work done of the force
field in displacing a point charge q ′ from centre of the 21. A semicircular wire of radius a having λ as charge per
ring to infinity. unit length is shown in the figure. Find the electric
qq′ qq′ potential at the centre of the semicircular wire.
a. b.
4πε 0R 2πε 0R
qq′
c. d. None of these
πε 0R

13. Two drops of water each with a charge of 3 × 10−9C


λ λ
having surface potential 500 V form a single drop. a. b.
What is the surface potential of the new drop? ε0 4πε 0R
λ
a. 794 V b. 1000 V c. d. None of these
c. 250 V d. 750 V 4ε 0

@iitjeehelps
258 SELF STUDY GUIDE BITSAT

22. If a charged particle starts from rest from one 30. At the corners of an equilateral triangle of side a = 1m,
conductor and reaches the other conductor with a three point charges are placed (each of 0.1 C). If this
velocity of 109cm/s, then calculate the potential system is supplied energy at the rate of 1 kW, then
difference between the two conductor. The mass of calculate the time required to move one of charges to
the charge particle is 9 × 10−28 g and the charge is the mid-point of the line joining the other two.
A
4.8 × 10−10 esu.
a. 0.94 stat volt b. 1 stat volt
c. 1.2 stat volt d. 0.2 stat volt a a
23. A charge Q is uniformly distributed over the surface of
two conducting concentric spheres of radii R and r
(R > r ). Then, potential at common centre of these B D C
a
spheres is
kQ (R + r ) kQ (R + r ) a. 50 h b. 60 h c. 48 h d. 54 h
a. b.
Rr (R 2 + r 2 ) 31. If Q charge is given to a sphere of radius R, the energy
kQ  1 1 of the system is
c. d. kQ  − 
R2 + r 2 R r  Q2 Q
a. b.
8πε 0R 4πε 0R
24. If a charged particle starts from rest from one
Q2
conductor and reaches the other conductor with a c. d. None of these
velocity 109 cm/s, if the potential difference between 15πε 0R
the two conductors is 0.94 stat volt, then calculate the 32. Two balls with charges 5 µC and 10 µC are at a
charge of the charged particle.
distance of 1 m from each other. In order to reduce
(Given, mass of charged particle = 9 × 10−28 g) the distance between them to 0.5 m, the amount of
a. 5.8 × 10−10 esu b. 4.8 × 10−10 esu work to be performed is
c. 3.8 × 10−10 esu d. 2.75 × 10−10 esu a. 45 J b. 0.45 × 10−6 J
. × 10−4 J
c. 12 d. 0.45 J
25. Calculate the potential of the big drop, if eight charged
water drops with a radius of 1 mm and a charge of 33. Three small conducting spheres each of radius a and
10−10 C merge into a single drop. charge q is placed at the corners of an equilateral
triangle of side length l. The side length l is
a. 3200 V b. 4000 V c. 3600 V d. 4200 V
considerably larger than dimensions of the spheres.
26. A charge q = 2 µC is moved by some external force Find the electrical potential energy of system.
from infinity to a point where electric potential is 104 V.
Calculate the work done by external force.
a. 1 × 10−2 J b. 2 × 10−2 J
c. 0.2 × 10−2 J d. 12 × 10−2 J

27. Find the electric potential energy of electron-proton


system of hydrogen atom. (Given, the radius of electron
orbit = 0.53 Å, electronic charge = 1.6 × 10−19 C) 3q 2  1 1 3q 2
a.  +  b.
4πε 0  2a l  4πε 0l
a. −27.17 eV b. −20.18 eV
c. 36.55 eV d. None of these 3q 2
c. d. None of these
8πε 0a
28. Three charges Q, 2Q, 8Q are to be placed on a line
whose length is R metre. Locate the position where 34. Due to a charge inside a cube, the electric field is
these charges should be placed such that the E x = 600 x 1/ 2, E y = 0, E z = 0. The charge inside the
potential energy of the system is minimum.
cube is
R 2R 3R 4R y 0.1m
a. b. c. d.
3 3 2 3
29. A particle of mass 2 g and charge 1 µC is held at rest
on a frictionless horizontal surface at a distance of 1 m x
from a fixed charge 1 mC. If the particle is released, it
z 0.1m
will be repelled. The speed of the particle when it is at
a distance of 10 m from the fixed charge is a. 600 µC b. 60 µC
a. 100 m/s b. 90 m/s c. 60 m/s d. 45 m/s c. 7 × 10− 6 µC d. 6 µC

@iitjeehelps
GAUSS'S LAW AND ELECTRIC POTENTIAL THEORY 259
35. What is the electric field intensity at a point at a 39. What work must be done to rotate an electric dipole
distance 20 cm on line making an angle of 45° with the through an angle θ with the electric field, if an electric
axis of the dipole of moment 10 C-m? dipole of moment p is placed in an uniform electric
. × 1013 V/m
a. 177 b. 0.177 × 1013 V/m field E with p parallel to E ?
c. 17.7 × 1013 V/m d. 177 × 1013 V/m a. W = pE(1 − cos θ ) b. W = pE(1 + cos θ )
c. E = 2pE(1 − cos θ ) d. None of these
36. What is the electric potential at a point distance
100 cm from the centre of an electric dipole of 40. Find the force of interaction of two dipoles, if the two
moment 2 × 10−4 C-m on a line making an angle of dipole moments are parallel to each other and placed
60°? at a distance x apart.
a. 7 × 105 V b. 8 × 105 V
c. 9 × 105 V d. 10 × 105 V p1 p2

37. Two point charges q1 = − 10 × 10−6 C and x

q 2 = 15 × 10−6 C are 40 cm apart as shown in figure.


Find the potential difference between the points P
and Q. 3p1p 2 p1p 2 p1p 2 p1p 2
a. b. c. d.
A P B Q 4πε 0x 4 4πε 0x 4 4πε 0x 4 3πε 0x 4
20cm 20cm 20cm 41. Electric dipole moment of combination shown in the
figure is
a. −945 × 103 V b. −1000 × 103 V q q
a
c. − 880 × 103 V d. None of these

38. An electric dipole made up of a positive and negative


charge, each of 1 mC and placed at a distance 2 cm a a
apart. If the dipole is placed in an electric field of 105
N/C, then calculate the maximum torque which the
field can exert on the dipole, if it is turn from a position a
–3q q
θ = 0° to θ = 180°.
−3 −3
a. 2 × 10 N-m b. 3 × 10 N-m a. qa + qa 2 + qa b. 2 2qa
c. 4 × 10−3 N-m d. 2.8 × 10−3 N-m c. 2qa d. ( 2 + 1)qa

BITSAT Archives
M  $ 3. Two charges + q and − q are kept apart. Then, at any
1. Electric field in a region is given by E =   i , then
x 3  point on the right bisector of line joining the two
the correct expression for the potential in the region is charges [2008]
(assume potential at infinity is zero) [2014] a. the electric field strength is zero
M M b. the electric potential is zero
a. b. Mx 2 c. d. None c. both electric potential and electric field strength are
2x 2 3x 4 zero
2. A charge ( − q ) and another charge ( + Q ) are kept at d. None of the above
two points A and B respectively, keeping the charge 4. The work done in carrying a charge q once round a
( + Q ) fixed at B, the charge ( − q ) at A is moved to circle of radius r with a charge Q at the centre is [2005]
another points C, such that ABC forms an equilateral qQ
a.
triangle of side l. The net work done in moving the 4π ε 0r
charge ( − q ) is [2009] qQ
b.
1 Qq 1 Qq 4π ε 20r
a. b.
4 π ε0 l 4 π ε0 l 2 c.
qQ
1 4π ε 0r 2
c. Qql d. zero
4 π ε0 d. None of the above

@iitjeehelps
Answer with Solutions
Practice Exercise 12. (a) Work done = qV
where, V = change in potential
1. (a) The electric flux,
V0 = potential at the centre of the ring
φ = ΣE ⋅ d S = E ⋅ S
1 q
4πε 0R ∫
= (2$i + 4$j + 7k$ ) ⋅ (10$j ) = 40 units V0 = dq = , V∞ = 0
4πε 0R
2. (c) Here, E = E0r qq′
W = q′ (V0 − V∞ ) =
r = x$i + y$j + z k$ 4πε 0R

∴ E = E0x$i + E0y $j + E0z k$ 13. (a) Here, q1 = 3 × 10−9C,V1 = 500 V


φ 0 = the electric flux per unit volume q1 q1 3 × 10−9 × 9 × 109 27
V1 = ⇒ r1 = = = m
∂Ex ∂Ey ∂Ez 4πε 0r1 4πε 0V1 500 500
= + +
∂x ∂y ∂z Let radius of bigger drop is R.
φ 0 = E0 + E0 + E0 = 3E0 4 4 2q1
∴ πR 3 = 2 × πr13 ∴ R = r121/ 3 ∴V = = 794 V
∴ Net flux = φ 0 × volume = φ 0 × πR 2h = 3E0πR 2h 3 3 4πε 0R

3. (c) Total flux passing through whole sphere 14. (a) Let R = 6.4 × 106 m
Q σ = surface charge density
= = half flux from curved surface
ε0 Total charge on the surface of earth = q
+ half flux from plane surface q = 4 πR 2 σ
Then, the potential V at a point on the earth′s surface is
same as if the entire charge q were concentrated at its
Q centre. Thus,
1 q 1 4πR 2σ Rσ
V = = =
4πε 0 R 4πε 0 R ε0
. × 10−19 )
( 6.4 × 106 )( −16
Q V = = − 0.115 V
∴The flux passing through plane surface = ( 8.9 × 10−12 )
2ε0
Q 1 2
4. (a) Net electric flux = 15. (c) Q s = at
ε0 2
1
= half flux through curved surface or 10 × 10−2 = a × (10−7 )2
2
+ half flux through plane surface
10 × 10−2 × 2
Ratio = 1 : 1 ∴ a= = 2 × 1013 m/s 2
10−14
5. (d) The flux linked with each face will be one-eight of the
q ∴ F = ma or eE = ma
flux .
ε0 ma 9.1 × 10−31 × 2 × 1013
∴ E= =
q e 16. × 10−19
Flux associated with given cube =
8ε 0 But uniform electric field,
6. (c) potential difference
E=
7. (a) distance between points

8. (a) The net electric field at P is zero. ∴ Potential difference,V = Es


9.1 × 10−31 × 2 × 1013 × 10 × 10−2
∴ EP = E1 + E2 + Esphere or 0 = E1 + E2 + Esphere = = 114
. V
. × 10−19
16
∴ Esphere = − ( E1 + E2 ) ∴ Esphere = E12 + E22
16. (c)
2 2 2 2
 q1   q2  1  q1  q  17. (a) LetV1 = potential at the centre of the cube due to one
=   +  =  2  +  22 
 4πε 0a 2   4πε 0b 2  4πε 0 a  b  charge
1 q
9. (c) V1 =
4πε 0 x
10. (b) 3
11. (a) and x =L
2

@iitjeehelps
GAUSS'S LAW AND ELECTRIC POTENTIAL THEORY 261
Potential due to 8 charges, 24. (b)
  25. (c) Assume a drop to be perfect sphere of radius r.
 1 q  1 q
⇒ V = 8V1 ⇒V = 8   V =
 4 πε 3 4πε 0 r
0
× 10 × 10−2 
 2 
The charge of n drops,
16q Q 0 = nQ = 8 × 10−10 C
V = = 83.14 × 1011 V
4πε 0 3 × 10 × 10−2 Let radius of bigger drop = R
18. (b) 4  4
n  πr 3ρ = πR 3ρ ⇒ R = r (n )1/ 3 (Q n = 8)
3  3
19. (a) Let P is the point on the axis where potential is zero.
+q –3q R = 2 × 10−3 m
r (1–r)  9 × 109 × 8 × 10−10 
Hence, V =  = 3600 V
+q –3q  2 × 10−3 
r 1m
26. (b) 27. (a)
q 3q
VP = − = 0 ⇒ r = 0.25 m 28. (a) For minimum potential energy, the largest charge
4πε 0r 4πε 0(1− r )
should be farthest. Thus, the potential energy,
q 3q 2Q 2 8Q 2 16Q 2
VP ′ = − = 0 ⇒ r = 0.5 m 4πε 0U = + +
4πε 0r 4πε 0(1 + r ) x (R − x ) R
V2 r2
20. (b) Here, − ∫ dV = ∫ Edr For minimum,
dU
=0
V1 r1
dx
σr2 σ r
∴ V1 − V2 = ∫ dr ∴ V1 − V2 = In 2 dU 2Q 2 8Q 2 R
2πε 0r
r1 2πε 0 r1 4πε 0 =− 2 + + 0=0⇒ x =
dx x (R − x )2 3
r
Hence, potential difference is proportional to In log 2 . 29. (b) According to conservation principle of energy.
r1
21. (c) In the case of circular arc (uniformly or non-uniformly
charge), the potential at centre is q B
Q v
1m
10 m
O
Σq Ui + Ki = Uf + Kf
V =
4πε 0R Qq Qq 1
+ 0= + mv 2 …(i)
λπR λ 4πε 0r1 4πε 0r2 2
Here, Σq = λπR ∴ V = =
4πε 0R 4ε 0 Here, r1 = 1 m, r2 = 10 m,
1 1 mv 2 q = 10−6 C, m = 2 × 10−3 kg, Q = 10−3 C
22. (a) QV = mv 2 ∴ V =
2 2 Q Putting the values in Eq. (i), we get
9 × 10−28 × (109 )2 10−3 × 10−6 × 9 × 109 10−3 × 10−6 × 9 × 109
V = = 0.94 stat volt −
2 × 4.8 × 10−10 1 10
23. (b) Let the surface charge density is σ. 1 −3 2
= × 2 × 10 × v
∴ Q = q A + qB 2
Q ⇒ v = 90 m/s
or Q = 4πr 2σ + 4πR 2σ ∴ σ =
4π (r + R 2 )
2
30. (a)Ui = initial potential energy of the system
qA qB C
∴ VC = VA + VB ∴ VC = + 1 q 2 q 2 q 2  1  3q 2 
4πε 0r 4πε 0R A =  a + a + a  = 4πε  a 
B 4πε 0
  0  
1 q A qB   4 πr 2 σ 4 πR 2 σ 
+ =k  +  = k ( 4πrσ + 4πRσ )  ( 0.1) 
2
4πε 0  r R   r R  = 9 × 109  3 ×  = 27 × 107J
 1 
 Qr QR  kQ (R + r )
= 4 πk  2 2
+ 2 2 
= 2 2 Let charge at A is moved to mid-point D.
 4π(R + r ) 4π (r + R )  (R + r )
1  2q 2 q2  1 q2
 1  Uf =  +  =5 ×   = 45 × 107 J
 Here, k =  4πε 0 (a / 2) a  4πε 0  a 
 4πε 0 

@iitjeehelps
262 SELF STUDY GUIDE BITSAT

Work done = Uf − Ui = 18 × 107 J Potential at Q


Also, energy supplied per second 1 q 2 q1  15 × 10−6 10 × 10−6 
VQ =  −  = 9 × 109  − 
= 1000 J 4πε 0  r2 r1   0.1 0.5 
18 × 107 = 117 × 104 V ∴ VP − VQ = − 945 × 103 V
Time taken =
1000
38. (a) τ = pE sin θ and τ is a maximum, when θ = 90°
= 18 × 104 s = 50h
∴ τ max = pE and P = (q )( 2l ) = (1 × 10−6) )( 0.02)
31. (a) 32. (d)
E = 105 N/C
33. (a)U = U1 + U 2 + U 3 + U12 + U13 + U 23
τ max = (1 × 10−6 × 0.02) × 105 = 2 × 10−3 N-m
1
39. (a) The initial potential energy = Ui = − pE
The final potential energy = Uf = − pE cosθ
But Wext = change in potential energy.
2 3 Wext = ∆U = Uf − Ui
Wext = − pE cos θ − ( − pE )
q2 Wext = pE(1 − cos θ )
U1 = self potential energy of first sphere = =U 2 =U 3 40. (a)
8πε 0a
U12 = interaction energy of first sphere and second sphere 41. (b) Here, p1 = qa = p 3
q2 p 2 = 2qa
= = U13 = U 23
4πε 0l q
1 2
q
2
y
3q  1 1
∴ U =  + 
4πε 0  2a l 
√2a


P1 P1
P2
Q P2
34. (c) φnet = en, Q en = φ net × ε 0
ε0 45°
x
= 600 [(0.2)1/2 − (0.1)1/2] (0.1)2 × 8.85 × 10−12 q
4 3
q P3
P3
= 7 × 10−12 C
From vector diagram shown in figure,
35. (a) 36. (c) p x = p 3 + p 2 cos 45°
37. (a) Potential at P 1
= qa + 2qa = 2qa
1 q 2 q1  2
VP = − 
4πε 0  r2 r1  p y = p1 + p 2 cos 45° = 2qa
 15 × 10−6 10 × 10−6  ∴ p = p x2 + p y2 = 2 2qa
= 9 × 109  −  = 225 × 103 V
 0.2 0.2  Σp = 0

BITSAT Archives
( x, y, z )
1. (a)V (x , y , z ) = − ∫ E ⋅ dr 3. (b) At equatorial point,
∞ 1 p
( x , y , z ) Mdx M Ee =
=−∫ = 4πε 0 r 3
∞ x3 2x 2
(directed from + q to − q) and Ve = 0.
2. (d) Net work done = Final potential energy − Initial
4. (d) The potential at each point on the circular path will be
potential energy equal.
Qq Qq
= − =0 So, work done = q × potential difference
4π ε 0l 4π ε 0l
=q × 0= 0

@iitjeehelps
23
Electric Capacitor

Capacitor
Capacitor is a device that stores electrical energy. It consists of conductors of any shape and size
carrying charges of equal magnitudes and opposite signs and separated by an insulating medium.

Capacity of a Capacitor
Ability to store charge by a conductor is called capacity of the capacitor.
The amount of charge required to raise the potential of the capacitor by one unit (in SI one volt) is
called capacitance of the capacitor.
If q = amount of charge given and V = potential of capacitor, then capacitance of the capacitor is
given by
q
C=
V
SI unit of capacitance is farad or F, 1 farad = 1 coulomb/volt
Dimensions of capacitance = [M -1L-2T 4 A 2 ]
Capacitance is a scalar quantity.

Parallel Plate Capacitor (without Dielectric)


Most commonly used capacitor is parallel plate capacitor, which consists of two charged parallel
plates separated by a small distance. Capacity of a parallel plate c
+ –
apacitor with vacuum between the plates is given by
e A e0 KA + –
C= = farad
d d + –
e0 A
or C vacuum = (QK = 1, for vacuum) + –
d
+ –
where, K = dielectric constant of the medium between the plates
+σ –σ
A = area of plates and d = distance between the plates Parallel plates with vacuum

@iitjeehelps
264 SELF STUDY GUIDE BITSAT

(i) If a dielectric slab fills completely the gap between


the plates, then capacitance increases by K times. Combination of Capacitors
Ke0 A (i) Capacitors connected in series
i.e. C¢ =
d C2
C1
Þ C ¢ = KC
where, K = dielectric constant.
1 1 1
(ii) If a dielectric slab of thickness E = +
+ – +– C C1 C2
t (t < d ) is inserted between the plates + – + –
as shown, then E = electric field + – + – (ii) Capacitors connected in parallel
+ – + –
between the plates, Ei = induced A – –
+ C1
electric field in dielectric. + – Ei + –
E ¢ = ( E - Ei ), the reduced value of + – + –
+ – + –
electric field in the dielectric. + – + –
Now, capacitance of the capacitor d
C2

Q
C¢ = C = C1 + C2

e0 A
Þ C¢ =
t
(d - t ) +
K Energy of a Charged
NOTE When dielectric is inserted between the plates, voltage and
electric field reduces by a factor of K between the plates. Conductor/Capacitor
(iii) As dielectric constant of metal t When a conductor/capacitor is charged, its potential
is infinite. increases from 0 toV and work is done against repulsion,
between charge stored in the conductor and charge
Hence, field inside the slab does coming from the source (battery). This work is stored as
not exist. Capacitance,
e0 A A K=∞ electrostatic potential energy. The graph of rise of
C¢ = , in case of partial potential of the capacitor with charge is shown as(Q µ V ).
(d - t )
filling and in case of complete V
filling, with metallic slab C ¢ = ¥,
capacitor is said to be short d
Parallel plates
circuited as shown in the
figures below.
C=∞ C=∞ Q

Work done = Area of (Q -V ) graph


A K=∞ A
1
= QV
2
1
d d Hence, potential energy, U = QV
NOTE (i) The capacity of a spherical capacitor is given below 2
4pe0R1R 2 By using Q = CV , we can write
C=
R 2 - R1 1 1 Q2
U = QV = CV 2 =
where, R1 is radius of inner spherical conductor and R 2 is 2 2 2C
radius of outer spherical conductor.
The energy is stored in electric field between plates of
(ii) Capacity of an isolated – – capacitor, which depends on capacitance, charge and
–E R2 –
spherical capacitor.
+
+

potential difference. This energy is obtained at cost of


+
+

+

When a smaller conductor is + chemical energy of battery.


+
R1

shielded by a conducting

+ +
shell, then capacity of the
+ +

+
Energy Density
+
combination depends upon


+

+
+
+

radius of shell and is given – –


– – The energy stored in a capacitor is not localised on the
by C = 4pe0R
charges or the plates but is distributed in the field. In case of
where, R is radius of the shell.
a parallel plate capacitor, electric field is only between the

@iitjeehelps
ELECTRIC CAPACITOR 265
plates in a volume ( A ´ d ), which is so called energy If Charge is Held Constant, i.e. Battery is
density. Disconnected After Charging
1
CV 2 q
energy 2
Hence, energy density = = +++++++
volume Ad
1 é e0 A ù V 2 e0 A ù C, V, E, U
é
= êëQC = d úû
2 êë d úû Ad –––––––
2
1 æV ö 1
= e0 ç ÷ = e0 E 2
2 èd ø 2 Charge remains unchanged, q = q0
Energy density is the property of electric field and above Capacity increases as C = KC0
formula is valid in all cases. The unit of energy density is PD between the plates decreases as
J/m3 . q q V
V = = 0 = 0
C KC0 K
Particular Cases of Capacitors Field between the plates decreases as
V V E
Force between Plates of Capacitor E= = 0 = 0
d Kd K
The two plates of capacitor attract each other, because they
are oppositely charged. Energy stored in the capacitor decreases as
s Q q2 q2 U
Electric field due to positive plate, E = = U = = 0 = 0
2 e0 2 e0 A 2C 2 KC0 K
Q2
Force on negative charge -Q, F = - QE = - If Potential is Held Constant, i.e. Battery
2 e0 A
Remains Attached
Q2
Magnitude of force, F =
2 e0 A
q0 q
+++++++
Capacity of an Isolated Sphere +++++++
V0 C0, V0, E0, U0 V0 C, V, E, U K
+ + +
+ ––––––– –––––––
+
+ R
Q
+ +
+ +
+ + PD remains constant, i.e. V = V0 , as battery is a source of
constant potential difference
Q Capacity increases as,
Vsurface = ; Q = 4pe0 R(V )
4pe0 R C = KC0
So, C = 4pe0 R Charge on capacitor increases as

Effect of Dielectric on Charge, Potential q = CV = ( KC0 )V = Kq0


Difference, Field and Energy of Capacitor Electric field remains unchanged as
When a dielectric is introduced between plates, it polarizes éV ù V
E = ê ú = 0 = E0
and reduces potential difference between plates and so ëd û d
increases its capacity. Energy stored in the capacitor increases as
1
q0 U = CV 2
+++++++
2
1
V0 C0, V0, E0, U0 = KC0V02
2
–––––––
1
= KU 0
2

@iitjeehelps
Practice Exercise
1. In which form of the following, the energy is stored in 8. Find the electric field in region II as shown in figure.
the capacitor? +σ –σ
a. Charge b. Magnetic field ×

– –––––– ––––
×

– –––––– ––––
× ×

– – – – – – – – –– –– –– –
c. Capacitance d. Electric potential energy × ×
×××

– ––––––
2. A sheet of aluminium foil of (I) ×× × (II) (III)
d/2 ××
negligible thickness is placed × ×
×××
between the plates of a Foil ×
d/2
capacitor of capacitance C as
shown in the figure, the s
a. zero b.
capacitance of capacitor becomes. 4pe 0
C s
a. 2C b. C c. d. zero c. d. infinite
2 e0
3. Calculate the ratio of 9. Two capacitors A and B having capacitances 10 mF
capacitance of two
and 20 mF are connected in series with a 12 V battery.
capacitors filled with
The ratio of the charge on A and B is
dielectrics of same
dimensions but of a. 0.5 : 1 b. 1 : 1
K (i) (ii) c. 2 : 1 d. 2 : 4
different values K and
4 10. A 6 ´ 10-4 F parallel plate air capacitor is connected
arranged in two ways as shown in Fig. (i) and (ii). to a 500 V battery. When air is replaced by another
dielectric material, 7 . 5 ´ 10-4 C charge flows into the
a. 5 : 2 b. 25 : 16
c. 5 : 4 d. 2 : 5 capacitor. The value of the dielectric constant of the
material is
4. A conducting sphere of radius 10 cm is given a charge
a. 1.5 b. 2.0
of + 2 ´ 10-8C. What will be its potential? c. 1.0025 d. 3.5
a. 0.03 kV b. 0.9 kV
11. The 90 pF capacitor is connected to a 12 V battery.
c. 1.8 kV d. 3.6 kV
How many electrons are transferred from one plate to
5. If n identical drops of mercury are combined to form a another?
bigger drop then find the capacity of bigger drop, if . ´ 10 9
a. 11 b. 6.7 ´ 109
capacity of each drop of mercury is C.
c. 4 ´ 1019 d. 5 ´ 1019
a. n1/ 3 C b. n 2/ 3 C
c. n1/ 4 C d. nC 12. In the given figure, the capacitor of plate area A is
charged upto charge q. Find the k2 k1
6. Two spherical conductors A1 and A2 A2
ratio of elongation (neglect force
of gravity) in spring C and D at
of radii r1 and r 2 are placed equilibrium position.
concentrically in air. The two are A1 k1 k2
connected by a copper wire as r1 a. b.
k2 k1
shown in figure. Then, the r2 c. k1k 2 d. None of these
equivalent capacitance of the
system is 13. A potential difference of 500 V is applied to a parallel
4p e 0 K r1r2 plate condenser. The separation between plates is
a. b. 4p e 0 (r2 + r1)
r2 - r1 2 ´ 10-3 m. The plates of the condenser are vertical.
An electron is projected vertically upwards between
c. 4pe 0r2 d. 4pe 0r1
the plates with a velocity of 105 m/s and it moves
7. Calculate amount of charge flow, when a conducting undeflected between the plates. The magnetic field
sphere of radius R and carrying a charge Q, is joined acting perpendicular to the electric field has a
to an uncharged conducting sphere of radius 2R. magnitude of
Q Q Q 2Q a. 1.5 Wb/m 2 b. 2.0 Wb/m 2
a. b. c. d.
4 3 2 3 c. 2.5 Wb/m 2 d. 3.0 Wb/m 2

@iitjeehelps
ELECTRIC CAPACITOR 267
14. Two capacitors C1 and C 2 are connected in a circuit as 20. A parallel plate capacitor of plate separation d and
shown in figure. The potential difference (VA - VB ) is plate area A is charged to a potential difference V and
then the battery is disconnected. To fulfill the space
C1
between the plates of the capacitor, a slab of dielectric
+ constant K is inserted. If the magnitude of the charge
+ 2 µF E2 –24 V on each plate, electric field between the plate (after
E1 – –
–12 V C the slab is inserted) and work done on the system in
2
the process of insertion of a slab are Q, E, W
4 µF respectively, then
a. 8 V b. - 8 V AV AV 2 æ 1ö
a. Q = e 0 b. W = e 0 ç1 - ÷
c. 12 V d. 12 V d 2d è Kø
V
15. In the given figure, the equivalent capacity between A c. C = d. All of these
and B is Kd
21. A capacitor of capacitance 10 µF 20 Ω
B D E
10 mF is charged by
A C C C connecting through a
resistance of 20 W and
C battery of 20 V. What is the 20 V
a. 3C b.
3 energy supplied by the
3 battery?
c. d. infinity
2C a. Less than 2 mJ b. Equal to 2 mJ
16. For the circuit, the equivalent capacitance between P c. More than 2 mJ d. Cannot be predicted
and Q is 22. A capacitor of capacitance C is charged to a potential
P Q difference V0. The charged battery is disconnected
and the capacitor is connected to a capacitor of
C C C C C C
unknown capacitance C x . The potential difference
a. 6C b. 4C across the combination is V . The value of C x should
3C 6C be
c. d.
2 11 C(V0 - V ) C(V - V0 ) CV CV0
a. b. c. d.
17. A parallel plate capacitor is connected to a battery of V V V0 V
constant emf. Let the electric field at a given point 23. A parallel plate capacitor is made of two dielectric
between the plate be E 0, when there is no medium blocks in series. One of the blocks has thickness d1
between the plates. The new electric field at the point, and dielectric constant K 1 and the other has thickness
if a medium of dielectric constant A is introduced d 2 and dielectric constant K 2 as shown in figure. This
between them, is arrangement can be thought as a dielectric slab of
E0 E0
a. b. thickness d ( = d1 + d 2 ) and effective dielectric
4 2 constant K . Constant K is equal to
c. E0 d. 4E0
d1 K1
18. Force acting upon a charged particle, kept between
the plates of a charged condenser is F. If one of the
plates of the condenser is removed, force acting on d2 K2
the same particle will become
a. 0 b.
F K1d1 + K 2d 2 K1d1 + K 2d 2
a. b.
2 d1 + d 2 K1 + K 2
c. F d. 2F K1K 2(d1 + d 2 ) 2K1K 2
c. d.
19. A parallel plate capacitor has K1= 2 K2= 6 (K1d1 + K 2d 2 ) K1 + K 2
two layers of dielectrics as
shown in figure. 24. A copper plate of thickness b is placed
inside a parallel plate capacitor of plate
This capacitor is connected distance d and area A as shown in figure.
across a battery, then the The capacitance of capacitor is
ratio of potential difference Ae 0 Ae 0
across the dielectric layers is d 2d a. b.
d b d
4 1 1 3 Ae 0
a. b. c. d. c. d. ¥
3 2 3 2 d -b

@iitjeehelps
268 SELF STUDY GUIDE BITSAT

25. For the circuit shown below, capacitors A and B have q2 q


a. W = (x 2 - x1) b. W = (x 2 - x1)
identical geometry, but a material of dielectric e 0S e 0S
constant 3 is present between the plates of B. The q2 q2
potential difference across A and B are respectively c. W = (x 2 - x1) d. W = (x 2 - x1)
2e 0S 4e 0S

10 V 28. Five identical plates are connected across a battery


as follows.
1
2
3
4
A B 5
a. 2.5 V, 7.5 V b. 2 V, 8 V
c. 7.5 V, 2.5 V d. 8 V, 2 V If the charge on plate 1 is +q, then the charges on the
26. For the network shown we have three identical plates 2, 3, 4 and 5 are
capacitors. Each of them can withstand a maximum a. -q, + q, - q, + q b. -2q, + 2q, - 2q, + q
100 V potential difference. What maximum voltage c. -q, + 2q, - 2q, + q d. None of these
can be applied across A and B that no capacitor gets
spoiled? 29. The equivalent capacitance of the combination of
C three capacitors each of capacitance C between A
C and B as shown in figure is
A C B
C
a. 150 V b. 120 V A B
c. 180 V d. 0.200 V
27. Calculate the work done against the electric force, if the
separation of the capacitor of area S is increased from C
x1 to x 2. Assume charge q on the capacitor is constant. a. C b. 2C c. C /2 d. 3C

BITSAT Archives
1. Three capacitors X = 1mF, Y = 2 mF and Z = 3 mF are the space between the plates is filled with a dielectric
connected as shown in figure, then the equivalent of relative permittivity er , then [2009]
capacitance between points A and B is [2014] S

X Y Z
A B
A B

a. the potential difference as well as charge on each


a. 6 mF b. 12 mF capacitor goes up by a factor er
c. 3 mF d. None of these b. the potential difference as well as the charge on
each capacitor goes down by a factor er
2. A capacitor of capacity 0.1 mF connected in series to a
c. the potential difference across A remains constant
resistor of 10 MW is charged to a certain potential and and the charge on B remains unchanged
then made to discharge through resistor. The time in d. the potential difference across B remains constant,
which the potential will take to fall to half its original while the charge on A remains unchanged
value is (Given, log10 2 = 0. 3010) [2012] 5. Which of the following is discontinuous across a
a. 2 s b. 0.693 s charged conducting surface ? [2009]
c. 0.5 s d. 1.0 s a. Electric potential
-18 b. Electric intensity
3. The work done in placing a charge of 8 ´ 10 C on a
c. Both electric potential and intensity
capacitor of capacity 100 mF is [2011]
d. None of the above
a. 32 ´ 10-32 J b. 16 ´ 10-32 J
6. Capacitance of a capacitor made by a thin metal foil is
c. 3.1 ´ 10-26 J d. 4 ´ 10-10 J 2mF. If the foil is folded with paper of thickness
4. Two identical air filled parallel plate capacitors are 0.15 mm, dielectric constant of paper is 2.5 and width
charged to the same potential in the manner shown by of paper is 400 mm, the length of foil will be [2007]
closing the switch S. If now the switch S is opened and a. 0.34 m b. 1.33 m c. 13.4 m d. 33.9 m

@iitjeehelps
Answer with Solutions
Practice Exercise 13. (c) In this case, electric force is balanced by magnetic
force.
1. (d) E V æ V ö
\ qE = qvB \ E = vB \ B = = 0 çQ E = 0 ÷
2. (b)
1
=
1
+
1 v vd è d ø
C ¢ C1 C2 500 2
= = 2. 5 Wb /m
e 0A e A 2 ´ 10-3 ´ 105
C1 = , C2 = 0
d /2 d /2
1 d e A 14. (b) Consider the diagram below going through the path
\ = \ C¢ = 0 = C shown
C ¢ e 0A d
12 - VA + VB - 24 + VD - VC = 0
e 0K1A e 0K 2A æ Q ö
3. (b) Here, C1 = +
2d 2d Þ VA - VB = 12 + VD - VC - 24 = - 12 + ç ÷
è 4 mF ø
e 0KA e 0KA æ Given, K1 = K ö C1 = 2 µF
= + ç K ÷
2d 8d ç and K 2 = ÷ A B
è 4 ø
e 0KA æ 1 1 ö e 0KA æ 4 + 1ö 12 V +24 V
= ç + ÷= ç ÷
d è 2 8ø d è 8 ø
5e KA C
Þ C1 = 0 ...(i) D
C2 = 4 µF
8d
e 0A e 0A ( 2K )e 0A -Q Q é 1 1ù
and C2 = = = ...(ii) Þ = - 12 + Þ Q ê + ú = 12 mC
d d d d 5d 2 mF 4 mF ë 2 4û
+ +
2K1 2K 2 2K 2K 12 ´ 8
Þ Q= mC = 16 mC
4 6
Dividing Eq. (i) by Eq. (ii), we get -Q - 16 mC
Þ VA - VB = = = - 8V
5e 0KA C1 2 mF
C1 5 ´ 5 25
= 8d = = 15. (d) From figure, VA = VD = VE = VB
C2 2Ke 0A 2 ´ 8 16
q q
5d VA - VB = 0 C = = =¥
VA - VB 0
4. (c) 5. (a) 6. (c)
16. (d)
7. (d) Capacitance of first sphere, C1 = 4p e 0R A C B D
Capacitance of second sphere P E G F Q
C2 = 4pe 0 ( 2R ) = 2C1
Then, when they are in contact with the common potential F
=E=
between them C C=D C
Q B
=V = A=
C1 + C2 P= P C Q
C G C
Q C2 2
Charge on C2 = C2V = = Q C
C1 + C2 3
8. (c) 9. (b) 3C 2C C
10. (c) q 0 = C0V0
P Q
q = CV0
1 1 1 1 2 + 3 + 6 11 6C
\ Dq = q - q 0 = (C - C0 ) V0 \ = + + = = \ Ceq =
Ceq 3C 2C C 6C 6C 11
or Dq = ( er C0 - C0 ) V0 or Dq = ( er - 1) C0V0 ...(i)
Here, Dq = 7. 5 ´ 10-4C 17. (c) 18. (b)
C0 = 6 ´ 10- 4 F 19. (d) As it is a series combination, so charge on each
capacitor is same.
V0 = 500 V
Putting the value in Eq. (i), we get er = 10025
. ⇒

11. (b) 12. (b) K = 2K = 6 K=2 K=6

@iitjeehelps
270 SELF STUDY GUIDE BITSAT

Q Q 23. (c) 24. (d)


So, V1 = and V2 =
C1 C2 25. (c) If C0 = capacitance with air between plates. Then,
A 3 C0 = capacitance with dielectric. As capacitors are in
e 0K 2
V1 C2 2d = 6 =3 series, equivalent capacitance is given by
Þ = =
V2 C1 e 0K1
A 2´2 2 1 1 1 3
= + Þ Ceq = C0
d Ceq C0 3 C0 4
20. (b) When battery is disconnected, then charge remains Q 3
the same. Now, Ceq = Þ Q = 10 ´ C0
V 4
e AV
Q = CV = 0 Also, charge on each capacitor is same. Charge on each
d 15
Potential is reduced by factor of K and electric field also. capacitor is C0 = Q
2
V
E= \Potential drop across capacitors is
Kd 15
Energy before dielectric material is introduced × C0
Q 15
1Q 2 1 e 20A 2V 2 d e AV 2 VA = = 2 = = 7 .5 V
U1 = Þ U1 = = 0 ...(i) CA C0 2
2C 2 d 2
Ae 0 2d 15
C0
Q 5
Energy after dielectric introduced VB = = 2 = = 2 .5 V
CB 3 C0 2
1 Q2
U2 =
2 C¢ 26. (a) 27. (c)
e 0KA 1 e 20A 2V 2 d e AV 2 28. (b) As plates are identical.
Here, C ¢ = Þ U2 = . = 0 ...(ii)
d 2 2d 2
e 0KA 2dK Charge on plate 5 is + q.

From Eqs. (i) and (ii), we get Charge on plate 3 is + 2q = + q + q.


e 0AV æ2
1ö Charge on plate 2 is - 2q = - q - q.
U1 - U 2 = ç1 - ÷
2d è Kø Charge on plate 4 is - 2q = - q - q.
21. (c) 29. (b) Third capacitor is short-circuited as its both ends are
connected to B. Equivalent circuit is
22. (a) The initial charge on first capacitor is Q = CV0
After connection, voltages of both capacitors become
same, i.e. V C
A B
\ Q = q1 + q 2 or CV0 = CV + CxV
C
CV0 C(V0 - V )
V = \ Cx =
C + Cx V
Ceq = C + C = 2C

BITSAT Archives
1. (a) The equivalent circuit of the X or t = 2.3026 ´ 0.3010 or t = 0.693 s
following figure is as follow. Y q2 ( 8 ´ 10-18 )2
3. (a) Work done,W = =
\ Ceq = X + Y + Z = 1 + 2 + 3 = 6 mF Z 2C 2 ´ 100 ´ 10-6
2. (b) By equation of charging, = 32 ´ 10-32 J
q = q 0(1 - e -t /CR )
4. (c) When switch S is opened, capacitor A is still
According to question, connected with battery. Therefore, potential difference
q 1 across A remains constant. As B is disconnected from
= = 0.50 \ 0.50 = 1 - e -t /CR
q0 2 battery, charge on B remains fixed.

e -t /CR = 1 - 0.50 = 0.50 Þ et /CR = 2 5. (b) Electric intensity inside the conductor is zero.
Ke (l ´ b )
t t 6. (d) If length of the foil is l, then C = 0
or = loge 2 or = 2.3026 log10 2 d
CR CR -12
2 . 5 ´ 8.85 ´ 10 (l ´ 400 ´ 10-3 )
or t = CR ´ 2.3026 log10 2 Þ 2 ´ 10-6 =
0.01 ´ 10-3
or t = 0.1 ´ 10-6 ´ 10 ´ 106 ´ 2.3026 log10 2
Þ l = 33.9 m

@iitjeehelps
24
Current Electricity

Electric Current
The time rate of flow of charge through any cross-section of wire per unit time is called current. So, if
∆q
∆q charge passes through a cross-section in time ∆t, then iav = and instantaneous current
∆t
∆q dq q
i = lim = ⋅ If rate of flow is uniform, then i = ⋅ Thus, current is a scalar quantity. Its SI unit is
∆t → 0 ∆ t dt t
ampere (A).

Current Density
In case of flow of charge through a cross-section, current density is defined as a vector having
magnitude equal to current per unit area surrounding that point. Remember area is held normal to the
direction of charge flow (or current passes) through that point. Current density at point P is given by
di
J= n
dA
Current density J is a vector quantity having SI unit A/m 2 and dimension [L−2 A ].
i
In case of uniform flow of charge through a cross-section normal to it asi = nqvA so, J = n = (nqv ) n.
A
E
Current density related with electric field is J = σE = , where σ = conductivity and ρ = resistivity or
ρ
specific resistance of substance. The direction of current density J is same as that of electric field E.

Drift Velocity
Drift velocity is the average uniform velocity acquired by free electrons inside a metal by the
application of an electric field which is responsible for current through it. Drift velocity is very small, it
is of the order of 10−4 m/s as compared to thermal speed (~ − 105 m/s) of electrons at room temperature.
The current related with drift velocity is i = neAvd .
i J σE E V
We can also write vd = = = = = ⋅
neA ne ne ρne ρ ne

@iitjeehelps
272 SELF STUDY GUIDE BITSAT

where, n = number of electrons per unit volume of the Substances which do not obey Ohm’s law are called
conductor non-ohmic resistors. e.g. electrolytes, gases, thermionic
A = area of cross-section tubes, transistors, rectifiers, etc.
V = potential difference across the conductor V-I graph for them is non-linear.
E = electric field inside the conductor
i = current, J = current density,ρ = specific resistance
 1
σ = conductivity σ =  I I
 ρ

V V
Ohm’s Law
According to Ohm’s law, physical conditions (temperature,
mechanical strain, etc.) remaining unchanged, the current
Colour Code for Resistors
flowing through a conductor is directly proportional to the The value of resistance used in electric and electronic
potential difference across its ends. circuit vary over a wide range.
Thus, I ∝ V orV = IR Such high resistances used are usually carbon resistances
and the values of such resistances are marked on them
where, R is a constant known as the electrical resistance of
according to a colour code.
given conductor.
Colour Figure
Electrical Resistance Black 0

The obstraction offered by any conductor in the path of Brown 1

flow of current is called electrical resistance (R). It is equal Red 2


to the ratio of the potential difference (V ) applied across the Orange 3

ends of the conductor and the electric current (I) flowing Yellow 4

through it. Green 5

V Blue 6
R= Violet 7
I
Grey 8
The resistance Rt of a metal conductor is given by
White 9
Rt = R0 (1 + αt + βt 2 )
Rt − R0
where, α=
R0 × t
Its unit is ohm (Ω ).
In terms of free electron, density n and relaxation time τ, the
resistance of a conductor is given by
m l
R= 2 Colour Tolerance
ne τ A Gold 5%
m
where, ρ = 2 is the resistivity of material. Silver 10%
ne τ No colour 20%
Resistivity is a characteristic of material, i.e. it does not
depend upon dimensions of the material. Although, it
depends on the temperature. Series and Parallel
The SI unit of resistivity is Ω-m and its dimensional formula Combinations of Resistors
is [ML3T −3 A −2 ]. ● Series grouping In series grouping of resistances, same
current I flows through all the resistances.
V -I Characteristics of Ohmic and
Non-ohmic Conductors
Substances obeying Ohm’s law are called ohmic resistors,
e.g. metals and their alloys. V-I graph for an ohmic resistor
is an inclined straight line.

@iitjeehelps
CURRENT ELECTRICITY 273
The potential difference applied across the combination is Fig. (b) shows a cell of emf E, internal resistance r connected
distributed across various resistors in the direct ratio of
to an external resistance R. Total resistance of the circuit
their resistances,
= R + r.
i.e. V = V 1 + V 2 + V3 + … E
The current in the circuit is given by I =
and V 1 : V 2 : V3 … = R1 : R2 : R3 … (R + r )
Total equivalent resistance in series grouping is equal to Potential difference across r is V = Ir
the sum of individual resistances. Thus,
Potential difference across R is V = IR. V is called the
Rs = R1 + R2 + R3 + …
terminal voltage and V ′ is the potential drop across the
● Parallel grouping In parallel
I1 internal resistance.
grouping same potential difference R1
Thus,
V appears across each resistance. I2 R2 E = V + V ′ = I( R + r )
The current is distributed among I3
I I Terminal voltage is V = E − Ir .
various resistors in the inverse ratio R3
of their resistances. Thus, + –
Discharging of a Cell
I = I 1 + I 2 + I3 + … V K
When an external resistance is connected across a cell, a
1 1 1
and I 1 : I 2 : I3 … = : : current flows in external circuit, this is also called
R1 R2 R3 discharging of the cell then current given by the cell
Equivalent resistance in parallel grouping Rp is given by E
i=
1 1 1 1 R+r
= + + +…
Rp R1 R2 R3 Potential difference across the resistance R, V = iR
Potential drop inside the cell = ir
Electric Cell Equation of cell, E = V + ir (E > V )
E 
The device which converts chemical energy into electrical Internal resistance of the cell, r =  − 1 ⋅ R
V 
energy is known as electric cell.
Power dissipated in external resistance (load)
2
V2  E 
+ P = Vi = i 2 R = =  ⋅R
A R R + r
Anode Cathode – R
Power delivered will be maximum when R = r .
+ – R
i
V=iR
i
A E r B
Electrolyte
Cell
(a) Electric cell (b) Schematic diagram of cell
E, r

E2
Emf, Terminal Voltage and Internal So, Pmax =
4r
Resistance of a Cell
NOTE During charging of a cell, current flows into positive terminal
The potential difference between the terminals of a cell of the cell and terminal voltage V = E + ir
when it is on an open circuit, i.e. when no current is drawn
from it is called its emf (E). Combination of Cells in Series and in
The potential difference between the terminals of a cell Parallel
when it is in a closed circuit, i.e. when a current is drawn
A group of cells is called a battery. Two common grouping of
from it is called its terminal voltage (V).
cells are
Every cell has a resistance of its own called its internal
resistance (r). The value of r depends upon the nature of 1. Series Grouping
electrodes, the nature of the electrolyte, size of electrodes In series grouping, if all the cells are joined so as to supply
and the distance between them. current in the same direction, then resultant emf

@iitjeehelps
274 SELF STUDY GUIDE BITSAT

Eeq = E 1 + E 2 + E3 + … The rate at which work is done by the source of emf in


maintaining the effect of current in a circuit is called
E1 E2 E3
+ve –ve electric power of the circuit
r1 r2 r3
P = VI
where,V is the potential difference across the conductor, I
However, if one or more cells are joined so as to supply is the current flowing through the conductor.
current in reverse direction, then emf of that/those cells is Other expressions for power are given as
taken as negative while calculating the equivalent emf. V2
The equivalent internal resistance of cell P = I 2R ⇒ P =
R
req = r1 + r2 + r3 + …
The SI unit of electric power is watt.
If n cells, each of emf E and internal resistance r, are joined Another important unit is horse power (hp).
in series, then 1 hp = 746 watt.
E eq = nE and req = nr

2. Parallel Grouping Kirchhoff’s Laws and their


In parallel grouping, if positive
terminals of all cells have been Applications
joined at one point and all Sometimes complex electric circuits cannot be reduced
negative terminals at another to simple series parallel combination.
point, then
1 1 1 1 Kirchhoff ’s First Law
= + + +…
req r1 r2 r3 The algebraic sum of currents meeting at a junction is zero.
i.e. Σi = 0
The equivalent emf of the
i1 + i2 = i3 + i4
parallel grouping is given by
This law is simply a statement of
E eq E 1 E 2 E3
= + + +… conservation of charge as if current
req r1 r2 r3
reaching a junction is equal to the be
If n cells, each of emf E and internal resistance r, all joined in current leaving the junction, charge will conserved.
r
parallel, then req = Kirchhoff’s Second Law
n
The algebraic sum of the changes in potential in complete
But, E eq = E
traversal of a mesh (closed loop) is zero, i.e. ΣV = 0
e.g. In the following closed loop
Electric Energy and Power Applying Kirchhoff’s second rule to closed mesh ABFGA
The total work done (or energy supplied) by the source of we have I 1 R1 − I 2 R2 = E 1 − E 2
emf in maintaining the electric current in the circuit for Similarly, for the closed mesh BCDFB we have
a given time is called electric energy consumed in the I 2 R2 + ( I 1 + I 2 )R3 = E 2 .
circuit. I1 R1
A G
The SI unit of electric energy is joule but another unit is E1 I
I2 R2
watt-hour. B F
The bigger unit of electric energy is kilowatt-hour (kWh). It i1 + i2 E2 R3 II
is known as Board of Trade Unit (BTU). C D
l1 + l2
The electric energy consumed in kWh is given by
V (in volt) × I (in amp) × t (in hour) This law represents conservation of energy as if the sum of
W =
1000 potential changes around a closed loop is not zero,
unlimited energy could be gained by repeatedly carrying a
Whenever the electric current is passed through a
charge around a loop.
conductor, it becomes hot after short time. This indicates
that the electric energy is being converted into heat energy.
This effect is known as heating effect of current or Joule
Sign Convention for the Applications
heating effect. of Kirchhoff ’s Law
I 2 Rt For the application of Kirchhoff’s laws, the following sign
H = W = I 2 Rt joule = cal
4.18 conventions are to be considered

@iitjeehelps
CURRENT ELECTRICITY 275
X
(i) The change in potential in traversing a resistance in R.B.
the direction of current is −iR while in the opposite R
D
S
direction is +iR.
G
A i R B A i R B P B Q
–iR A C
+iR l1 l2
(ii) The change in potential in traversing an emf source
+ –
from negative to positive terminal is +E while in the
opposite direction is −E irrespective of the direction V K1
of current in the circuit.
If by sliding the tapping point a null point is obtained on
bridge wire at point B , then in balanced condition
R P l1 l
= = ⇒ X=R2
(iii) The change in potential in traversing a capacitor X Q l2 l1
from the negative terminal to the positive terminal is
q q
+ while in opposite direction is − .
C C Potentiometer
– C+ – C+
A B A B Potentiometer is an instrument which can be used for
q q different electric measurements. It is commonly used to find
q q
+ –
C C emf of a given cell and to find internal resistance of a cell.
Rh
(iv) The change in voltage in traversing an inductor in + – – +
di A
the direction of current is −L while in opposite E0 K
dt
di l J
A B
direction it is + L .
dt
G
+ –
E
Here, AB is a long uniform resistance wire (length AB may
be ranging from 1 m to 10 m). E0 is a battery whose emf is
known supplying a constant current I for flow through the
Wheatstone’s Bridge potentiometer wire. If R be the total resistance of
It is a sensitive arrangement to potentiometer wire and L its total length, then potential
determine the value of an unknown gradient, i.e. fall in potential per unit length along the
resistance. The bridge is said to be potentiometer will be
balanced if on switching the keys K 1 V IR E0 R
k= = =
and K 2 there is no deflection in L L ( R0 + R) L
galvanometer. It is possible when where, E0 = emf of battery,
V B = V D . In balanced condition,
R0 = resistance inserted by means of rheostat Rh.
P R
=
Q S Applications of Potentiometer
In balanced bridge, Q and R can be interchanged without The several applications of potentiometer are given below
affecting the balance condition. Similarly, P and S can be ● Determination of emf of a cell If with a cell of emf E on
interchanged. Moreover, cell and galvanometer may also sliding the contact point we obtain zero deflection in
be interchanged. If an electric circuit resembles a balanced galvanometer G when contact point is at J at a length l
Wheatstone's bridge, then resistance of branch BD may be from the end where positive terminal of cells have been
ignored (or removed from the circuit) as no current is joined, then fall in potential along length l is just
flowing through it. balancing the emf of cell. Thus, we have E = kl
● Comparison of emfs of two cells If with a given
potentiometer arrangement we obtain balancing lengths
Meter Bridge l1 and l2 for cells of emfs E 1 and E 2 , then
A meter bridge is a practical arrangement to realise E 1 l1
=
Wheatstone’s bridge. E 2 l2

@iitjeehelps
276 SELF STUDY GUIDE BITSAT

● Determination of internal resistance of a cell The


arrangement is shown in figure. If the cell E is in open

Thermo-emf
circuit and balancing length is l1 , then

Rh
+– – +
E0 K1 A
T
A J B T0 Tn Ti

E
+– G bt 2
E = at +
2
R. R K2
B. where, a , b are constants and E is emf.
Thermoelectric power, P
dE
But if by inserting key K 2 circuit of cell is closed, then = a +2bt
dt
potential difference V is balanced by a length l2 of
At neutral temperature,
potentiometer, where V = kl2
dE
⇒ Internal resistance of a cell is given as =0
dt
E −V
r = R a + 2bT n = 0
V
a
l −l Tn = −
= 1 2 R 2b
l2
Inversion temperature,
a
Ti = 2T n = −
Variation of Thermo-emf b
Peltier effect The absorption or evolution of heat at a
with Temperature junction of two dissimilar metals, when current is passed
When temperature of hot junction increases by keeping the through it is known as Peltier effect. It is a reversible
temperature of cold junction constant, thermo-emf phenomenon.
increases and graph is parabolic. At certain temperature H = Heat energy evolved or absorbed when one coulomb of
emf becomes maximum but thermo electric power is zero. charge flows across junction
This temperature is known as neutral temperature (T n ). The  dE 
π = Peltier coefficient = T  
neutral temperature is independent of temperature of cold  dt 
junction. On further increasing, the temperature of hot If Peltier emf at junction under consideration is E voIt, then
junction, the thermo-emf decreases and becomes zero. energy absorbed or evolved is Eit
Again further increase in temperature it reverses sign, this
so, πit = Eit or π = E
temperature is known as temperature of inversion Ti .
Seebeck effect is reversible. Thomson’s effect The absorption or evolution of heat
energy along the length of a wire, when current is passed
∴ T n − T c = Ti − T n through a wire whose ends are kept at different
T + Tc temperatures.
⇒ Tn = i
2  d 2E 
Thomson coefficient, σ = −T  
Graph between emf and T can be represented as  dT 2 

@iitjeehelps
Practice Exercise
1. Two uniform wires A and B of same metal and have 9. Three equal resistors connected in series across a
equal masses, the radius of wire A is twice that of wire source of emf, together dissipated 10 watts of power.
B. The total resistance of A and B when connected in What will be the power dissipated if the same resistors
parallel is are connected in parallel across the same source of
a. 4 Ω, when resistance of wire A is 4. 25 Ω emf?
b. 5 Ω, when the resistance of wire A is 4 Ω a. 10 W b. 30 W c. 90 W d. (10 / 3) W
c. 4 Ω, when the resistance of wire B is 4.25 Ω 10. Reciprocal of specific resistance is
d. 5 Ω, when the resistance of wire B is 4 Ω a. conductive resistance b. specific conductance
2. A battery of emf 10V and internal resistance 05 . Ω is c. conductive reactance d. plate resistance
connected across a variable resistance R. The value 11. A heating coil is labelled 100W, 220 V, the coil is cut in
of R for which the power delivered in it is maximum, is half and two pieces are joined in parallel to the same
given by source. The energy now liberated per second is
a. 2 Ω b. 0.25 Ω c. 1Ω d. 0.5 Ω a. 200 J b. 400 J c. 25 J d. 50 J
3. If R1 and R 2 are respectively the filament resistances 12. Two resistors of 6 Ω and 9 Ω are connected in series
of a 400 W bulb and a 200 W bulb, designed to to a 120 V source. The power consumed by the 6 Ω
operate on the same voltage, then resistor is
a. R1 = 2 R2 b. R2 = 2R1 a. 384 W b. 576 W c. 1500 W d. 1800 W
c. R2 = 4R1 d. R1 = 4R2 13. A 100 W, 200 V bulb is connected to a 160 V supply.
4. Two electric bulbs rated P1 watt V volt and P2 watt V The power consumption would be
volt are connected in parallel across V volts mains, a. 64 W b. 80 W c. 100 W d. 150 W
then the total power is
14. The thermistors are usually made of
a. P1 + P2 b. P1P2
a. metals with low temperature coefficient of resistivity
P + P2
c. P1P2 / (P1 + P2 ) d. 1 b. metals with high temperature coefficient of resistivity
P1P2 c. metal oxides with high temperature coefficient of
resistivity
5. In Bohr's model of H2 atom, the electrons move
d. semiconducting materials having low temperature
around the nucleus in a circular orbit of radius coefficient of resistivity
5 × 10−11 m. Its time period is 15
. × 10−16 s, the current
15. Two wires of same material have lengths, L and 2L
associated with electron motion is and cross-sectional area 4A and A, respectively. The
a. zero . × 10−19 A
b. 16 ratio of their resistances would be
c. 0.17 A . × 10−3 A
d. 107 a. 1: 1 b. 1: 8 c. 8 : 1 d. 1: 2
6. A 5°C rise in temperature is observed in a conductor 16. Three bulbs A, B and C are
by passing a current. When the current is doubled, connected as shown in C
then rise in temperature will be approximately figure, B and C are identical,
a. 20° C b. 16° C c. 12° C d. 10° C if bulb C is fused, then A B

7. A uniform wire when connected directly across a 220V a. both A and B will glow more
line produces heat H per second. If wire is divided into brightly
n parts and all parts are connected in parallel across a b. both A and B will glow less
brightly than before
200 V line, then the heat produced per second will be
c. A will glow less brightly and B more brightly
a. Hs b. nHs d. None of the bulbs will glow
c. n 2Hs d. Hs / n 2
17. Two wires of the same metal have the same length, but
8. A wire of 50 cm long, 1mm 2 in cross-section carries a their cross-sections are in the ratio of 3:1. They are
current of 4 A, when connected to a 2 V battery, the joined in series. The resistance of the thicker wire is
resistivity of wire is 10 Ω. The total resistance of the combinations will be
a. 2 × 10−7 Ω-m b. 5 × 10−7 Ω-m a. 40 Ω b. 40 / 3 Ω
c. 4 × 10−6 Ω-m d. 1× 10−6 Ω-m c. 5 / 2 Ω d. 100 Ω

@iitjeehelps
278 SELF STUDY GUIDE BITSAT

18. Three copper wires of length and cross-sectional 25. The 80 Ω galvanometer deflects full scale for a
 A potential of 20 mV. A voltmeter deflecting full scale of
areas are (L, A )  2L,  , (L / 2, 2A ), resistance is
 2 5 V is to made using this galvanometer. We must
A connect
a. minimum in wire of cross-sectional area
2 a. a resistance of 19.92 k Ω parallel to the galvanometer
b. minimum in wire of cross-sectional area A b. a resistance of 19.92 kΩ in series with the
c. minimum in wire of cross-sectional area 2A galvanometer
d. same in all three cases c. a resistance of 20 kΩ parallel to the galvanometer
19. There is a current of 1.344 A in a copper wire whose d. a resistance of 20 kΩ in series with galvanometer
area of cross-sectional normal to the length of wire is 26. Consider the circuit shown in figure, the current I 3 is
1mm 2. If the number of free electrons per cm 2 is
equal to
8.4 × 1028m 3 , then the drift velocity would be 28 Ω 54 Ω
a. 1mm/s b. 1mm/s c. 0.1mm/s d. 0.01mm/s
20. In the given figure, 45 Ω 6V
the potentiometer l3
wire AB has a 5V
resistance of 5 Ω and J 8V 12 V
A B
length 10 m. The G 5
balancing length AJ a. 5 A b. 3 A c. − 3 A d. − A
6
for the emf 0.4 V is
0.4 V 27. Two cells having an internal resistance of 0.2 Ω and
a. 8 m
b. 0.8 m 0.4 Ω are connected in parallel, the voltage across the
c. 4 m battery is 1.5 V. If the emf of one cell is 1.2 V, then the
d. 0.4 m emf of second cell is
21. In the following figure, the a. 2.1 V b. 2.7 V c. 3 V d. 4.2 V
V
reading of an ideal 28. On a six fold increase in external resistance of a
voltmeter V is zero. Then, E E circuit the voltage across the terminals of the battery
the relation between R, r1 r1 r2 has increased from 5 V to 10 V. The emf of battery is
and r 2 is a. 15 V b. 18 V c. 12.5 V d. 11 V
a. R = r2 − r1 R
29. In figure, AB is a
b. R = r1 − r2
potentiometer wire; E
c. R = r1 + r2 length 10 m and J
rr
d. R = 1 2 resistance 2 Ω with A B
r1 + r2 open the balancing E1
22. Three equal resistors, each equals to r are connected length is 5.5 m.
as shown in figure. Then, the equivalent resistance However, on closing
between points A and B is key K the balancing 1Ω K
r 2r length reduces to 5 m.
a. r b. 3r c. d. The initial resistance of the cell E 1 is
3 3
a. 0.01Ω b. 0.1Ω c. 0.2 Ω d. 1Ω
23. When a resistance of 2 Ω is connected across the
terminals of a cell, the current is 0.5 A. When 30. A battery of emf E E
r
resistance is increased to 5 Ω, the current is 0.25 A. and internal
The emf of the cell is resistance r is
a. 1V b. 1.5 V c. 2 V d. 2.5 V connected to a R
variable resistor R
24. The potential difference between points A and B from as shown in figure.
the figure is Which one of the
5Ω 5Ω
A B following is true?
E a. Potential difference across the terminals of the
5Ω 5Ω
battery is maximum, when R = r
2V b. Power delivered to the resistor is maximum when
R =r
D C c. Current in the circuit is maximum when R = r
5Ω 5Ω
d. Current in the circuit is maximum when R > > r
a. 2 / 3 V b. 8 / 9 V c. 4 / 3 V d. 2 V

@iitjeehelps
CURRENT ELECTRICITY 279
31. The voltmeter in figure has a resistance of 200 Ω. The a. D is at 5 V
reading of voltmeter is b. E is at zero potential
c. The current in the circuit will be 0.5 A
3V
d. None of the above
38. The figure shows a network of 15 A
currents. The magnitude of 3A
200 W
A currents is shown here. The
100 W magnitude of current I will be 8A
V a. 3 A
b. 13 A
a. 2 V b. 1 V c. 1.5 V d. 3 V l
c. 23 A 5A
32. For the circuit, the 500 Ω d. −3 A
G
galvanometer G A 2V 39. Two wires having resistance R and 2R are connected
shows zero R in parallel, the ratio of heat generated in 2R and R is
12 V B
deflection. If the a. 1:2 b. 2:1 c. 1:4 d. 4:1
batteries A and B
have negligible 40. If the deflection of galvanometer in B
internal resistance, the value of the resistor R will be Wheatstone circuit is zero, the 2 4
a. 200 Ω b. 100 Ω c. 500 Ω d. 1000 Ω value of resistance will be
a. 2 Ω A C
33. For a potentiometer experiment, the emf of a battery
b. 4 Ω 3
in the primary circuit is 20 V and its internal resistance X
is 5 Ω. There is a resistance box in series with the c. 6 Ω
D
battery and the potentiometer wire, whose resistance d. 8 Ω
can be varied from 120 Ω to 170 Ω. Resistance of the
41. In which property of free electrons causes increase in
potentiometer wire is 75 Ω. The following potential
the resistance of a conductor with rise in temperature?
differences can’t be measured using this potentiometer
a. Number density b. Relaxation time
a. 5 V b. 6 V c. 7 V d. 8 V
c. Mass d. None of these
34. A 6 V battery is connected to the terminals of a 3 m
42. In an electroplating experiment, mg of silver is
long wire of uniform thickness and resistance of
100 Ω. The difference of potential between two points deposited when 4 A of current flows for 2 min. The
on the wire separated by a distance of 50 cm will be amount of silver (in g) deposited by 6 A of current for
a. 2 V b. 3 V c. 1 V d. 15 V 40 s will be
a. 4 m b. m/2 c. −m/4 d. 2m
35. A cell has emf of 2.2 V, when connected to a
resistance of 5 Ω, the potential difference between the 43. A, B , C are identical bulbs. How does the brightness of
terminals of the cell becomes 2.1 V, the internal A and B change when the switch S is closed?
resistance for the cell is + –
a. 0.12 Ω b. 0.48 Ω c. 0.24 Ω d. 0.50 Ω
36. A uniform wire of 16 Ω resistance is made into the A S C
form of a square. Two opposite corners of square are
connected by a wire of resistance 16 Ω. The effective B
resistance between the other two opposite corners is a. The brightness of A increases and that of B
a. 32 Ω b. 16 Ω c. 8 Ω d. 4 Ω decreases
b. The brightness of A does not change and that of B
37. For the circuit shown in figure. The point F is decreases
grounded. Which of the following is wrong statement? c. The brightness of both A and B decreases
5Ω d. The brightness of A increases and that of B does not
B C change
44. For a thermocouple, Tc , Tn and Ti denote the
2Ω 3Ω temperatures of cold junction, the neutral temperature
and the temperatures of inversion respectively. Which
A D
one of the following relation is correct ?
10 V 3V Tc + Tn
4Ω a. Ti = b. Ti = 2Tn − Tc
F E 2
T −T
c. Ti = n c d. Ti = Tn − Tc
2

@iitjeehelps
280 SELF STUDY GUIDE BITSAT

45. If nearly 105 C liberate 1g equivalent of aluminium, 53. Two heater coils made of the same material are
then the amount of aluminium (equivalent wt. 9) connected in parallel across the mains, the length and
deposite through electrolysis in 20 min current of the diameter of one coil is double of the other. Which
50 A will be will produce more heat?
a. 0.6 g b. 10.8 g c. 0.09 g d. 5.4 g a. Thinner coil b. Thicker coil
c. Both produce same heat d. None of these
46. For the network shown in figure, points A,B and C are
at potentials of 70 V, zero and 10 V respectively 54. Cell of emf 1 V is connected across a potentiometer,
balancing length is 600 cm. What will be the balancing
B 0V
length for 25 V?
20 Ω a. 400 cm b. 600 cm c. 1500 cm d. 1200 cm
10 Ω 55. A moving coil galvanometer has a resistance of 9.8 Ω
A D
and gives a full scale deflection when a current of
70 V
10mA is passed through it. The value of the shunt
30 Ω required to convert it into a milliameter to measure
10 V currents upto 500mA is
C
a. 0.02 Ω b. 0.2 Ω c. 2 Ω d. 0.4 Ω
a. point D is at a potential of 40 V
b. the currents in the sections AD, DB and DC are in the 56. The total electrical resistance between the points
ratio 4 : 3 : 2 A and B of the circuit shown, is
c. the currents in the sections AD, DB and DC are in the 10 Ω 10 Ω 10 Ω
ratio 1: 2 : 3 A 20 Ω B
d. the network draws a total power of 100 W
20 Ω 20 Ω
47. When copper voltmeter is connected with a battery of
emf 12 V, 2 g of copper is deposited in 30 min. If the a. 9.23 Ω b. 15 Ω c. 30 Ω d. 100 Ω
same voltmeter is connected across a 6 V battery,
then the mass of copper deposited in 45 min would be 57. A wire when connected to 220 V mains supply has
a. 1 g b. 1.5 g c. 2 g d. 2.5 g power dissipation P1. Now, the wire is cut into two
equal pieces which are connected in parallel to same
48. The emf in a thermoelectric circuit with one junction at supply. Power dissipation in this case is P2. Then,
0°C and other at t°C is given by E = At − Bt 2, the P1 : P2 is
neutral temperature is, then a. 1 b. 2 c. 3 d. 4
a. A / B b. −A / 2B c. −B / 2A d. A / 2B
58. The resistance of a 50 cm long wire is 10 Ω. The wire
49. For the adjoining circuit is stretched of uniform wire of length 100 cm. The
diagram, the readings of resistance now will be
ammeter and voltmeter are 2A a. 15 Ω b. 30 Ω c. 20 Ω d. 40 Ω
and 120 V respectively. If the
value of R is 75 Ω, then the 59. For the given circuit, the potential difference across
voltmeter resistance will be P and Q will be nearest to
a. 100 Ω b. 150 Ω c. 300 Ω d. 75 Ω
100 Ω
50. An electric heater rated as (500 W and 200 V) raises 48 V
the temperature of 1 kg water from 15° C to its boiling 80 Ω Q
point in 15 min. The heat efficiency of the heater is
100 Ω 20 Ω P
a. 79% b. 97% c. 69% d. 96%
51. The same mass of copper is drawn into two wires
2 mm and 3 mm thick. The two wires are connected in
series and current is passed through them. The ratio a. 9.6 V b. 6.6 V c. 4 V d. 3.2 V
of heat produced in the two wires is 60. A voltmeter having resistance of 1800 Ω is employed
a.
9
b.
3
c.
2
d.
81 to measure the potential difference across 200 Ω
4 2 3 16 resistance which is connected to DC power supply of
52. An electric bulb rated 500 W at 100 V is used in a 50V and internal resistance 20 Ω. What is the
circuit having a 200 V supply. What resistance R must percentage change in potential difference across
be put in series with the bulbs so that the bulb delivers 200 Ω resistance as a result of connecting voltmeter
500 W? across it?
a. 20 Ω b. 40 Ω c. 10 Ω d. 5 Ω a. 1% b. 5% c. 10% d. 20%

@iitjeehelps
CURRENT ELECTRICITY 281
61. In an experiment to measure the internal resistance of 63. The reading of ammeter shown in figure is
a cell by potentiometer, it is found that the balance
point is at a length of 2 m when the cell is shunted by 2Ω
a 4 Ω resistance and is at a length of 3 m when the
6Ω 3Ω
cell is shunted by a 8 Ω resistance. Then the internal
resistance of the cell is, then A
a. 12 Ω b. 8 Ω c. 16 Ω d. 1 Ω 3Ω 6Ω
62. The V -I graph for a conductor at

Voltage (V )
T2
(14 V 0.8 Ω)
temperatures T1 and T2 are as
shown in the figure, The term T1 a. 6.56 A
T2 − T1 is proportional to b. 3.28 A
θ
a. cos 2θ b. sin 2θ c. 2.18 A
θ d. 1.09 A
c. cot 2θ d. tan 2θ
Current (I)

BITSAT Archives
1. Ohm’s law says [2014] 5. Three bulbs X , Y and Z are
a. V =IR connected as shown in figure. The
Y
b. V /I = constant bulbs Y and Z are identical. If bulb
c. Both a and b are correct Z gets fused, then [2013] X
d. Both a and b are incorrect a. Both X and Y will glow more Z

2. A current 4.0 A exist in a wire of cross-sectional area brightly


2.0 mm 2. If each cubic metre of the wire contains b. Both X and Y will glow less brightly
12.0 × 1028 free electrons, then the drift speed is [2014] c. X will glow less brightly andY will glow more brightly
d. X will glow more brightly andY will glow less brightly
a. 2 × 10−8 m/s b. 0.5 × 10−3 m/s
−4 6. In the circuit shown, the value of I in ampere is [2012]
c. 1.04 × 10 m/s d. None of these
4Ω
3. What will be the value of current i in the circuit shown?
4Ω
[2014] I 4Ω
B 5 Ω 10 V i
i 4Ω
6Ω 4Ω
4V 1.6 Ω

a. 1 b. 0.60 c. 0.4 d. 1.5


A D
7. In the circuit shown below, the ammeter reading is
a. 0.67 A b. 1 A zero. Then, the value of the resistance R is [2012]
c. 0.32 A d. None of these 12 V 2V
4. In the given circuit (as shown in figure), each capacitor
has a capacity of 3 µF. What will be the net charge on
each capacitor? [2013] 500 Ω R A

4Ω

4Ω a. 50 Ω b. 100 Ω c. 200 Ω d. 400 Ω


A B
4Ω 8. A steady current flow in a metallic conductor of
non-uniform cross-section. The quantity/quantities
remaining constant along the whole length of the
10 V 1 Ω conductor is/are [2011]
D C a. current, electric field and drift speed
b. drift speed only
a. 48 µC b. 24 µC c. current and drift speed
c. 12 µC d. None of these d. current only

@iitjeehelps
282 SELF STUDY GUIDE BITSAT

9. Two bulbs which consume powers P1 and P2 are 17. When a battery connected across a resistor of 16 Ω,
connected in series. The power consumed by the the voltage across the resistor is 12 V. When the
combination is [2010] same battery is connected across a resistor of 10 Ω,
P1P2 2P1P2 voltage across it is 11 V. The internal resistance of the
a. P1 + P2 b. P1P2 c. d. battery (in ohm) is [2008]
P1 + P2 P1 + P2
10 20 25 30
a. b. c. d.
10. Three conductors draw respectively currents of 1 A, 7 7 7 7
2 A and 4 A when connected in turn across a battery. 18. If a rod has resistance 4 Ω and if rod is turned as half
If they are connected in series across the same circle, then the resistance along diameter is [2007]
battery, the current drawn will be [2010]
. Ω
a. 156 b. 2.44 Ω c. 4 Ω d. 2 Ω
2 3 4 5
a. A b. A c. A d. A
7 7 7 7 19. In the circuit, the potential difference across PQ will be
nearest to [2007]
. Ω,
11. 24 identical cells, each of internal resistance 05 100 Ω
are arranged in a parallel combination of n rows, each
row containing m cells in series. The combination is 48 V 80 Ω
connected across a resistor of 3 Ω. In order to send 100 Ω Q
maximum current through the resistor, we should 20 Ω
P
have [2010]
a. 9.6 V b. 6.6 V c. 4.8 V d. 3.2 V
a. m = 12 , n = 2 b. m = 8, n = 3
c. m = 2 , n = 12 d. m = 3, n = 8 20. Each resistance shown in figure is
2Ω 2Ω
2 Ω. The equivalent resistance
12. A wire is stretched as to change its diameter by between A and B is [2006] A B
0.25%. The percentage change in resistance is [2009] a. 2 Ω b. 4 Ω 2Ω
a. 4.0% b. 2.0% c. 1.0% d. 0.5% c. 8 Ω d. 1 Ω 2Ω 2Ω

13. If in the circuit in figure, the 20 V 1.5Ω


21. A cell of constant emf first connected to a resistance
internal resistance of the R1 and then connected to resistance R 2. If power
. Ω and VP and VQ
battery is 15 delivered in both cases is same, then the internal
3Ω P 2Ω
are potential at P and Q P Q resistance of the cell is [2006]
respectively. What is the 2Ω Q 3Ω R1 R1 − R2 R1 + R2
potential difference between a. R1R2 b. c. d.
the points P and Q? [2009] R2 2 2
a. Zero b. 4 V (VP > VQ ) 22. Ampere - hour is the unit of [2006]
c. 4 V (VQ > VP ) d. 2.5 V (VQ > VP ) a. quantity of charge b. potential
c. energy d. current
14. When the potential difference applied across a solid
conductor is increased, the rate of flow of electrons 23. A 5.0 A current is set up in an external circuit by a
a. remains same b. decreases [2009]
6.0 V storage battery for 6.0 min. The chemical
energy of the battery is reduced by [2005]
c. increases d. decreases sharply
. × 104 J
a. 108 . × 10− 4 J
b. 108
15. A box with two terminals is connected in series with a
. × 104 J
c. 18 . × 10−4 J
d. 18
2V battery, an ammeter and a switch. When the
switch is closed the needle of the ammeter moves 24. The current in a simple series circuit is 5.0 A. When an
quickly across the scale and drops back to zero. The additional resistance of 2.0 Ω is inserted, the current
box contains [2009] drops to 4.0 A. The original resistance of the circuit in
a. 20 Ω resistor b. a strip of copper ohms was [2005]
c. a diode d. a short length of fuse wire a. 1.25 b. 8 c. 10 d. 20
16. A current of 2 A flows in an electric circuit as shown in 25. Two resistances are connected in two gaps of a Meter
figure. The potential difference (VR − VS ), in volts bridge. The balance point is 20 cm from the zero end.
(VR and VS are potentials at R and S respectively) is A resistance of 15 Ω is connected in series with the
[2008] smaller of the two. The null point shifts to 40 cm. The
R
value of the smaller resistance in ohms is [2005]
3Ω 7Ω
a. 3 b. 6 c. 9 d. 12
P Q
26. By using only two resistance coils - singly, in series or
2A 2A
in parallel one should be able to obtain resistance of
7Ω 3Ω
3, 4, 12, and 16 Ω. The separate resistances of the
S coil are [2005]
a. − 4 b. + 2 c. + 4 d. − 2 a. 3 and 4 b. 4 and 12 c. 12 and 16 d. 16 and 3

@iitjeehelps
Answer with Solutions
Practice Exercise From Eq. (i), V = V1 + V2 = V1 +
R2
V1
R1
1. (a) rA = 2rB
R + R2  120 × 6
ρ A = ρB (specific resistance) V = V1 1  ⇒V1 = 15 = 48 V
 R1 
1 1 1
In parallel, = + …(i) V12 48 × 48
R RA RB Power in 6 Ω, P1 = = ⇒ P1 = 384 W
R1 6
A AlA = A BlB ⇒ πr 2AlA = πr B2 lB 160
13. (a) P = V 2 /R = 160 × = 64 W
l 400
ρ A
lA 1 AA RA AA 14. (c) 15. (b)
= and =4 ⇒ =
lB 4 AB RB ρ lB 16. (c) Bulbs B and C are connected in parallel. Suppose,
AB voltage across A and B is same. If bulb C is fused, bulb B
RA lA AB 1 1 1 will work
= × = × =
RB lB AA 4 4 16 V2 V2
PB = ⇒ PA =
16RA = RB …(ii) RB RA
1 1 1 RBRC
= + R=
R RA RB RB + RC
1 1 16 + 1 17 16RA RB < RA ∴ PB > PA
= + = = ⇒R =
RA 16RA 16RA 16 × RA 17 17. (a)
16 × 4.25 68 18. (c) l1 = L,A1 = A
If RA = 4.25 Ω ⇒ R = =
17 17 A
l2 = 2L, A2 =
∴ R =4Ω 2
2. (d) For maximum power, R1 l1 A2
= × ⇒ R3 < R1 < R2
∴ R = 0.5 Ω R2 l2 A1
3. (b) 4. (a) 19. (c) 20. (a)
5. (d) r = 5 × 10−11m, t = 15
. × 10−16 s 2E
21. (b) I =
. × 10−19
q 16 r1 + r2 + R
i= = . × 10−3 A
⇒ i = 107
. × 10−16
t 15 V1 = E − Ir1 = 0
⇒ I = E / r1
6. (a) 2Er1
7. (c)V = 200 V E=
r1 + r2 + R
H = i 2Rt ( i =V / r ) r1 + r2 + R = 2r1
V 2 ⇒ R = r1 − r2
Hs = t
nReq 22. (c)
In parallel, equivalent resistance of n parts is 23. (b) E = i (R + r )
r E = 15
. V
Req =
n 24. (a) By Kirchhoff's voltage law, for branch ABCEA
where, r is the resistance of each part. 2
15l1 = 2 ⇒ I1 = A
V2 nV 2 15
H= t= t
(r / n ) r For branch ADCEA,
2
∴ H = n 2Hs 15l2 = 2 ⇒ I 2 = A
15
8. (d) 9. (c) 10. (b) 11. (b) 2 2
VAD = i2R = ×5 = V
12. (a)V = 120 V 15 3
V = V1 + V2 …(i) 25. (b)
V1 = iR1 26. (d) For I1, by Kirchhoff's voltage law
V2 = iR2 ⇒
V1 V2
= −14 1
28I1 = − 6 − 8 ⇒ I1 = =−
R1 R2 28 2

@iitjeehelps
284 SELF STUDY GUIDE BITSAT

Here, I1 is −ve means opposite in direction as assumed. So, current in potentiometer wire
 1 1 E 20
∴ I 3 = − (I1 + I 2 ) = −  +  =I= = = 0.08 A
 2 3 (R + r ) 250
 5 Potential drop occurs across wire of potentiometer = IR
I3 = −   A
 6 = 0.08 × 75 = 6 V
27. (a) r1 = 0.2 Ω, r2 = 0.4 Ω Also, minimum resistance = 120 + 75 Ω
V = IR 20
∴ I= = 0.1A
rr rr 200
R = 1 2 , I = I1 + I 2 ⇒ V = 1 2 (I1 + I 2 )
r1 + r2 r1 + r2 Potential drop = IR = 0.1 × 75 = 7.5 V
I1 = E1 / r1 and l2 = E2 / r2
So, upto 7 V it can be measured with the help of
E E  r r (E r + E1r2 )  r1r2  potentiometer.
V =  1 + 2 1 2 = 2 1  
 r1 r2  r1 + r2 r1r2  r1 + r2  34. (c)
E2r1 + E1r2 E × 0.2 + 12
. × 0.4 35. (c) E = 2.2 V, R = 5 Ω,V = 2.1 V,
V= ⇒ 15. = 2
r1 + r2 0.6 E −V
E = iR + ir ⇒ R =r
0.42 V
0.9 = 0.2 E2 + 0.48 ⇒ = E2 ⇒ E2 = 2.1V 2.2 − 2.1 0.1
0.2 ×5 =r ⇒ ×5 =r
E −V E −5 2.1 2.1
28. (c) R =r ⇒ R =r …(i)
V 5 r = 0.24 Ω
E − 10 36. (d)
6R = r …(ii)
10 37. (a) Effective emf of circuit = 10 − 3 = 7 V
Equating Eqs. (i) and (ii), we get Total resistance of circuit = 2 + 5 + 3 + 4 = 14 Ω
E −5 E − 10
R= 6R ⇒ E − 5 = 3E − 30 Current I = 7/14 = 0.5 A
5 10
Potential difference between A and D = 0.5 × 10 = 5 A
2E = 25 ⇒ E = 12.5 V Potential at D = 10 − 5 = 5 V
29. (b) lAB = 10 m, RAB = 2 Ω, RAJ = 1Ω Potential at E = 5 − 3 = 2 V
E1 = ρL (When key is open) Hence, E cannot be at zero potential, as there is a
potential drop at E.
E1 = ρ × 5.5 ⇒ V1 = 5ρ
38. (c) ∑ I = 0 (at a junction)
E −V 5.5ρ − 5ρ 0.5
R =r × 1= r × 1= r
V 5ρ 5 ( 7 + 3) + ( 8 + 5) + I = 0
I = − 23A ⇒ I = 23 A
r = 0.1Ω
39. (b) In parallel circuit potential across each resistor is
30. (c) 31. (c) same.
32. (b) The galvanometer shows zero deflection, i.e. current V2
through XY is zero ∴ P1 = power across R =
R
500 Ω X
G Y V2
P2 = power across 2R =
2R
2V P1
12 V R = 2 :1
P2
40. (c) 41. (b)
As a result potential drop across 500 Ω 42. (b) m = z × 4 × 2
R is 2 V. Circuit can be redrawn I z × 6 × 40
as m′ =
R 2V 60
12
I= 12 V m 8z × 60 m
500 + R = = 2 ⇒ m′ =
m′ 240 z 2
Voltage across R,
12 43. (a)∴ Illumination of A is more than previous.
V = IR ⇒ 2 = ×R
500 + R 44. (b)Ti − Tn = Tn − Tc
or 1000 + 2R = 12R or R = 100 Ω 2Tn = Ti + Tc ⇒ Ti = 2Tn − Tc
33. (d) Maximum external resistance put across the 45. (d) m = zit
potentiometer battery is (170 + 75 ) Ω. 9 54
m = 5 × 50 × 20 × 60 = ⇒ m = 5.4 g
As external resistance = r = 5 Ω 10 10

@iitjeehelps
CURRENT ELECTRICITY 285
46. (a) Consider the current distributions in the circuit as 55. (b)
shown below 56. (a) In branch, (1) resistors are in series.
0V R1 = 10 + 10 + 10 = 30 Ω
B
In branch (3), 20 and 20 are in series
I2
∴ R3 = 20 + 20 = 40 Ω
20 Ω
A 10 Ω I1 D The equivalent circuit of (1), (2) and (3) branches are in
(70V ) parallel.
I1 l1 l2
I3 1 1 1 1 4 + 6 + 3 13
30 Ω
∴ = + + = =
Req 30 20 40 120 120
C 120
10 V Eeq = = 9.23 Ω
13
By KVL, VA − I110 − I 2 20 − VB = 0 57. (d)
⇒ I110 + I 2 20 = 70 …(i) 58. (d) When length of wire is stretched, its area and length
VA − I110 − I 3 30 − VC = 0 changes in such a way that volume always remains
⇒ I110 + I 3 30 = 60 …(ii) same.
Also, I 3 = I1 − I 2 Wire A has resistance R, length l, radius r
So, Eq. (ii) becomes Wire A when stretched resistance = R′, length = l′,
radius = r1
I1 10 + 30 (I1 − I 2 ) = 60
∴ Volume of original wire = volume of stretched wire
I1 40 − 30 I 2 = 60 …(iii)
V =V ′
Solving Eqs. (i) and (iii), we get
Al = A′ l ′ ⇒ πr 2l = πr12l′
I 2 = 2 A, I1 = 3 A, I 3 = 1A
Potential drop across branch AD, l r12 50 1
= = =
VA − VD = I1 × 10 ⇒ 70 − VD = 30 ⇒ VD = 40 V l′ r 2 100 2
47. (b) m = zit l l′
R =ρ⋅ , R′ = ρ
m = 15 . g A′ A′
R l /A l A′ l r 2 50 50 1
48. (d) = = × = ′ × 12 = × =
R′ l′ / A′ A l′ l r 100 100 4
49. (c)VAB = (I − Ig ) R = Ig ⋅ G I – Ig R
A A R′ = 4R = 4 × 10 = 40 Ω
where, G = voltmeter I B
Ig 59. (d)
(resistance)
VAB = 120, I = 2, R = 75 V 50
60. (a)V1 = E − Ir = 50 − × 20 = 50 − 4.5 = 45.5 V
⇒ 120 = ( 2 − Ig )75 ⇒ Ig = 0.4 A 220
120 50
Now, VAB = IgG ⇒ G = = 300 Ω Now, V2 = 50 − × 20 = 45
0.4 200
50. (a) Heat absorbed by water = ms∆T % change in potential difference
Heat supplied by heater = VIt = Pt (45.5 − 45)
= × 100 = 1%
ms∆T 50
∴ Efficiency = × 100  l − 2  l − 3
Pt 61. (b) As,   4 = 8 ⇒ l=6
 2   3 
By putting given value, efficiency ≈ 79%
d2  l − 2
51. (d) m = A ⋅ lx = π l ⋅ x , x = Density Therefore, r =  4 = 8Ω
4  2 
⇒ d12 l1 = d 22 l2 (as mass is same) 62. (c) R1 = tan θ = R0 (1 + α T1)
In series, current will be same. and R2 = cot θ = R0 (1 + α T2 )
l  d 22 cot θ − tan θ = R0 (1 + αT2 ) − R0 (1 + αT1) = R0 α (T2 − T1)
ρ 1  2 π
H1 i R12
 A  l A  d 4 or T2 − T1 =
1
(cot θ − tan θ )
So, = = 1
= 1 2 =  2 α R0
H2 i 2R2  l2  l2 A1  d12  d12
ρ  π 1
 A2  4 = (cot θ − tan θ )
4 α R0
d 24  3 81
= =  = 1  cos θ sin θ 
d1 4  2  16 =  − 
α R0  sin θ cos θ 
52. (a) 53. (b) 2 cos 2θ 2 cos 2θ
= =
54. (c) For a potentiometer α R0 sin θ α R0 sin 2θ
E1 E2 2
= = 1500 cm ⇒ = cot 2θ
l1 l2 α R0

@iitjeehelps
286 SELF STUDY GUIDE BITSAT

63. (c) The equivalent circuit of the given circuit will be 2Ω


reduced to as shown in figure.
2Ω 4Ω
A (c)
6Ω 6Ω
(a)
A 14 V, 0.8 Ω
3Ω 3Ω
2×4
Total resistance of the circuit = + 0.8
14V, 0.8 Ω 2+ 4
2Ω
8 12.8
= + 0.8 = Ω
6 6
2Ω 2Ω 14 84
A (b) Main current in the circuit = = A
(12.8 /6) 12.8
84 2
14 V, 0.8 Ω Reading of ammeter = × = 2.18 A
12.8 6

BITSAT Archives
V So, the brightness of bulb X decreases,
1. (c) Ohm’s law says = R = constant
I Now, bulb Y gets more current than before fusing the bulb,
2. (c) The current density in the wire is Z.
i 4 ∴ Brightness of bulbY will increase.
J = = = 2 × 106 Am−2
A 2 × 10−6 6. (c) We can simplify the network as shown in figure.
The drift speed is So, net resistance,
J 2 × 106 R = 2.4 + 1.6 = 4.0 Ω
v = =
hc 12 × 1028 × 1. 6 × 10−19 Therefore, current from the battery
V 4
106 i = = = 1A
= = 1.04 × 10−4 m/ s R 4
6 × 1.6 × 109
Now, from the circuit (b),
3. (a)VA − VD = − 6i − 5i + 10 − 4i 3
4I ′ = 6I ⇒ I ′ = I
Here, VA = VD 2
Since, points A and D are centred.
− 6i − 5i + 10 − 4i = 0 4Ω I 4Ω
10
⇒ 15i = 10 ⇒ i = = 0.67 A 4Ω
15 I 4Ω I 4Ω
(a) (b)

4. (c) Net resistance between ABCD is 4Ω

R = 4 + 1= 5 Ω
4V 1.6 Ω 4V 1.6 Ω
V 10
∴ Current I = = = 2A
R 5 ⇑

Potential difference across A and B


2.4Ω
=I × 4 = 2 × 4 = 8V
Two capacitors of 3 µF each are in series
(c)
∴ Potential difference across each capacitor
8
= =4V 4V 1.6 Ω
2
Charge on each capacitor, q = CV = 3 × 4 = 12 µC 3 5
But, i = I + I′ = I + I= I
5. (c) If bulb Z is fused, the current stops flowing through Z . 2 2
The effective resistance of the circuit due to bulbs X andY 5
in series becomes more as compared to before. Due to ∴ 1= I
2
which, the current in the circuit decrease.
2
∴ brightness ∝ (current) 2 ⇒ I = = 0.4 A
5

@iitjeehelps
CURRENT ELECTRICITY 287
7. (b) Given circuit is shown in figure VA − VQ = 2 × 2.5 = 5 V
12 V 2V VP − VQ = (VA − VQ ) − (VA − VP )
+ – + –
= 5.0 − 7.5 = −2.5 V
14. (c) When potential difference applied across a conductor
500 Ω

1 R 2 A increases, the current flowing through the conductor


increases. Therefore, drift velocity of the electrons
increases as
I ∝ vd
In loop (1),
12 − 500i1 − Ri1 = 0 15. (d) When a box contains fuse wire, a strong current flows
through fuse when circuit is closed. Due to which a very
⇒ 12 = i1(500 + R ) …(i) strong heating effect takes place, resisting the breakage
In loop (2), of fuse wire.
12 − 500i1 − 2 = 0 ⇒ 10 = 500 i1 16. (c) Current through each arm
1
or i1 = A …(ii) PQR and PSQ = 1 A
50
VP − VR = 3 V
From Eqs. (i) and (ii), we get
VP − VS = 7 V
1
12 × = (500 + R ) From Eqs. (i) and (ii), we get
i1
VR − VS = + 4 V
⇒ 12 × 50 = 500 + R ⇒ R = 100 Ω
17. (b) Here,V < E
8. (d) In a metallic conductor of non-uniform cross-section, ∴ E = V + Ir ...(i)
only the current remains constant along the entire length For first case,
of the conductor. 12
E = 12 + r
9. (a) For a series circuit, 16
Rs = R1 + R2 For second case,
11
⇒ i Rs = i 2R1 + i 2R2 ⇒ Ps = P1 + P2
2 E = 11 + r ...(ii)
10
10. (c) Let the potential of the battery beV From Eqs. (i) and (ii), we get
V V V 12 11 20
R1 = , R2 = , R3 = 12 + r = 11 + r ⇒ r = Ω
1 2 4 16 10 7
On connecting in series 18. (c)
Rs = R1 + R2 + R3 A
 1 1 7
⇒ Rs = V 1 + +  ⇒ Rs = V
 2 4 4
By definition R=4 Ω ⇒ R=4 Ω
V 4
I= ⇒ I= A
Rs 7
11. (a) N = 24 = mn B
For current to be maximum B
Rinternal = Rexternal 19. (d) Potential difference across PQ, i.e. potential
mr m m difference across the resistance of 20 Ω, which is
⇒ = 3 ⇒ ( 0.5) = 3 ⇒ = 6
n n n V = i × 20 and
⇒ m = 6n, substituting the values, we get 48
i=
24 = 6n 2 (100 + 100 + 80 + 20)
⇒ n = 2 ⇒ m = 12 = 0.16 A
12. (c) ∴ V = 0.16 × 20 = 3.2 V
5 ×5
13. (d) Effective resistance of circuit = + 15
. =4Ω 20. (a) Given circuit is a balanced Wheatstone bridge.
5+5
So, diagonal resistance of 2 Ω will be ineffective.
Total current in circuit,
20 2Ω
I==5 A 2Ω
4 2Ω
1 A B
Current in arm APB or AQB = = 2.5 A
2
VA − VP = 3 × 2.5 = 7.5 V 2Ω 2Ω

@iitjeehelps
288 SELF STUDY GUIDE BITSAT

Equivalent resistance of upper arms 24. (b) Let the original resistance is R Ω
=2+ 2=4Ω V =I R
Equivalent resistance of lower arms V =5 ×R × =5R ...(i)
=2+ 2=4Ω When 2 Ω resistance is inserted, then total resistance
4×4
RAB = =2Ω = (R + 2) Ω
4+ 4
∴ V = I ′ (R + 2) = 4 (R + 2) ...(ii)
21. (a) Current given by cell From Eqs. (i) and (ii), we get
E 5 R = 4 (R + 2)
I=
R+r ∴ R =8Ω
Power delivered in first case 25. (c) Let S be the large and R be the smaller resistance.
2
 E  From formula for meter bridge
P1 = I 2R1 =   R1
 R1 + r   100 − l  100 − 20
S = R = R = 4R
Power delivered in second case  l  20
 E 
2
 100 − l 
P2 = I 2R2 =  Again, S =   (R + 15)
 R2  l 
 R2 + r 
100 − 40
Power delivered is same in the both cases. = (R + 15)
2 2 40
 E   E  3
  R1 =   R2 = (R + 15)
 R1 + r   R2 + r  2
R1 R2 3
= ∴ 4 R = (R + 15)
(R1 + r )2 (R2 + r )2 2
8R
R1 (R22 + r 2 + 2R2r ) = R2 (R12 + r 2 + 2R1r ) ⇒ − R = 15
3
R1R22 + R1r 2 + 2R1R2r = R2R12 + R2r 2 + 2R1R2r 5R
⇒ = 15
R1R22 − R2R12 = R2r 2 − R1r 2 3
R1R2 ( R2 − R1) = r 2 (R2 − R1) ⇒ R =9 Ω
r = R1R2 26. (b) If we take R1 = 4 Ω, R2 = 12 Ω, then in series
q resistance
22. (a) I =
t Rs = R1 + R2= 4 + 12 = 16 Ω
q =I ×t 4 × 12
In parallel, resistance, Rp = =3Ω
23. (a) Chemical energy reduced 4 + 12
=V I t = 6 × 5 × 6 × 60 So, Rs = 4 Ω
= 10800 = 108
. × 104 J and Rp = 12 Ω

@iitjeehelps
25
Magnetic Field

Concept of Magnetic Field


In 1820, Oersted observed that a compass needle suffers a deflection when brought near a current
carrying wire. This means that electric current (electric charge in motion) gives rise to magnetism.

Biot-Savart’s Law
Biot-Savart’s law gives the magnitude of magnetic field at any point due to a current carrying
conductor. This law is although for an infinitesimally small conductor, yet it can be used for long
conductors. According to Biot-Savart’s law, magnetic field dB at any point P due to a current element
idl is
µ i(dl × r$ ) µ 0 i(dl × r )
dB = 0 ⋅ = ⋅
4π r2 4π r3
µ idlsin θ idlsinθ  µ0 
dB = 0 ⋅ or dB = k Q = k
4π r 2
r 2  4π 
As a moving charge is equivalent to a current, then
q dl
idl = dl = q = qv
dt dt
µ (v × r ) P
So, in terms of charge and its velocity, dB = 0 q θ dB
4π r3 dl
r
Total field can be found by summing via integration.
µ i dlsin θ
B = ∫ dB = 0 ⋅ ∫
i
i.e.
4π r2 Current carrying
where, µ 0 = absolute permeability of air or vacuum. conductor

It has exact value of 4π × 10−7 Wb/ A -m or 1.26 × 10−6 T-m/ A .


Its other units are H/m and N/ A 2 .

@iitjeehelps
290 SELF STUDY GUIDE BITSAT

Applications of Biot-Savart’s Law (b) For non-coplanar and concentric coils, magnetic
field is resultant of fields of both the coils. If planes of
Let us consider few applications of Biot-Savart’s law coils are perpendicular to B2 B
each other, then magnetic
1. Magnetic Field due to a Circular Current field at common centre is
i1 2
Carrying Loop µ
B = B 12 + B 22 = 0 i12 + i22 q
If a coil of radius r carries a current i, then magnetic field on 2r B1
its axis at a distance x from its centre is given by 2 i2
Also, the direction of B with
µ 2 π Nir 2 B
B axis = 0⋅ 2 direction of B 1 is tanθ = 2 .
4π ( x + r 2 )3 / 2 B1
where, N = number of turns in the coil.
Ampere's Circuital Law
r According to Ampere's circuital in
P law, the line integral of the
B B
O x magnetic field B around any dI i2
i closed path is equal to µ 0 times
i1
the net current I threading i3
through the area enclosed by the i4
2. Magnetic Field due to a Current Carrying closed path. Mathematically,
Circular Arc ∫ B ⋅ d l = µ 0 ΣI
Magnetic field at centre O of a circular arc as given by
Now, consider the diagram shown
Biot-Savart’s law is
Here, Σ I = i1 + i2 − i3
i
∫ B ⋅ dl = µ 0 ⋅ (i1 + i2 − i3 )
i i
Hence,

θ r O r Applications of Ampere's Circuital law


O r O θ
µ0 πi µ0 i , µ θi µ (2π–θ)i The following are the few important applications of
B = 4π × = B= 0 × , B= 0 ×
r 4r 4π r 4π r Ampere's circuital law given as below:
If magnetic field at the centre of circular coil is denoted by 1. Magnetic Field due to a Current Carrying
B0 , then
Straight Wire
µ 2 πi
B0 = 0 ⋅ The magnetic field due to a current carrying wire of finite
4π r
length at a point P situated at a normal distance r is given
Magnetic field at the centre of arc which is making an angle
B 
from Ampere's circuital law, ∫ B ⋅ d l = µ 0Σ I
θ at the centre is B arc =  0  ⋅ θ. X
 2π 
I

3. Magnetic Field in Concentric Circular φ2


r φ1
Loops P
i
(a) For coplanar and concentric loops,
i
magnetic field is either in same or in r2 Y
different directions. r1
µ 0I
If currents are in same direction, then B= (sin φ 1 + sin φ 2 )
4π r
magnetic field
If point P lies symmetrically on the perpendicular bisector
µ 1 1
B 1 = 0 × 2 πi  +  of wire XY , then φ 1 = φ 2 = φ (say) and hence
4π  r1 r2 
µ I µ I sin φ
i B = 0 ⋅ 2 sin φ = 0
If currents are in opposite directions, 4π r 2 πr
then magnetic field i
r2 For a wire of infinite length, φ 1 = φ 2 = 90° and hence
µ 1 1 r1
B 2 = 0 × 2 πi  −  µ I
4π  1
r r 2 B= 0 .
2 πr

@iitjeehelps
MAGNETIC FIELD 291
2. Magnetic Field due to a Thick (Cylindrical)
v
Wire B

Let us consider a thick cylindrical


wire of radius R and infinite
length, which carries a current I.
Then, magnetic field at a point So, path of particle is a straight line and it passes
µ I undeflected through the region of magnetic field.
outside the wire B = 0 , where r B
2 πr r P
Case II v ⊥ B, θ = 90°
is the distance of given point from
Force is maximum | F | = Bqv and it provides
centre of wire and r > R.
centripetal force and path of the particle is a circle
l as given below.
Magnetic field at a point inside
the wire at a distance r from R F v B
v
centre of wire (r < R) is given by Thick cylindrical wire F
B
µ I r F
B= 0 ⋅ 2
2π R
v
Magnetic field inside a hollow current carrying conductor
is zero. mv 2 v Bq
So, = Bqv ⇒ = ω =
3. Magnetic Field due to a Solenoid r r m
2 π 2 πm
For a solenoid coil of infinite length at a point on its axial ⇒ T = Time period of rotation = =
line, the magnetic field is given as B = µ 0nI ω Bq
1 Bq
where, n is the number of turns per unit length. and f = frequency of rotation = =
T 2 πm
1
At the end of solenoid, B = µ 0nI which is independent of radius of path and
2
velocity of charged particle.
At the end field is half of at the centre this is called end
Case III v is at some angle B
effect.
except 90° with B. Path of particle
Toroidal Solenoids
If velocity is at
Pitch
For a toroid (i.e. a ring shaped closed solenoid), magnetic some angle θ with
field at any point within the core of toroid is B = µ 0nI , direction of B, then
N due to parallel
where, n = and R = radius of toroid. component v cosθ ,
2πR v
charged particle
v cos θ

moves parallel to
Motion of a Charged Particle the direction of
field B.
θ

in a Uniform Magnetic Field Also, due to


v sin θ

perpendicular component v sinθ, it moves in a


If a particle carrying a positive
B circular path. So, path of particle is spiral in shape.
charge q and moving with v
q,m For the particle, T = time period = 2πm / Bq
velocity v enters a magnetic field
2π m
B, then it experiences a force F and pitch = × v cos θ
which is given by the expression Bq
F = q( v × B )or F = qvB sinθ
The force F is always perpendicular to both the velocity v Cyclotron
and the field B. It is a device used to accelerate positively charged particles
Trajectory (path) of the charged particle depends on the e.g. proton, deuteron, α-particle and other heavy ions to
angle between v and B. high energy of 100 MeV or more.
Case I v || B , θ = 0 ° Time taken by charged particle to describe a semi-circular
path in the dee is given by
Then, force on charged particle πr πm
t= =
F = q( v × B ) = 0 v qB

@iitjeehelps
292 SELF STUDY GUIDE BITSAT

If T is time period of oscillating High where, l is the length of that portion of


frequency
charged particle, then oscillator the conductor on which force is to be
2πm calculated. i1 i2
T = 2t =
qB Hence, force per unit length
The cyclotron frequency, D2 D1 F µ 0 2i1i2
= ⋅ N/m a
1 Bq l 4π a
ν= =
T 2 πm
Maximum energy gained by the Torque Experienced by a Current
 B 2r02q 2  W
charged particle,E max =   Carrying Loop in a Uniform Magnetic
 2m 
Target
where, r0 = maximum radius of
Field
the circular path followed by the positive ion. In the given figure, when a current carrying coil (loop)
having N turns and area A, is placed in a uniform field B , in
such a way that the normal (n)$ to the coil makes an angle θ
Magnetic Force on a Current with the direction of B , then the coil experiences a torque
given by τ = NBiA sin θ
Carrying Conductor PQ = RS = a ⇒ QR = SP = b or τ = M × B
If a current carrying conductor is placed in a magnetic field where, M = NiA called magnetic moment of the loop.
B, then a small current element I dl experiences a force P
given by B
dFm = IdI × B
S
and the total force experienced by whole current carrying ^
n
conductor will be
Fm = ∫ dFm = ∫ I (dl × B )

The direction of force when current element Idl and B are Q


perpendicular to each other can also be determined by R
applying Fleming’s left hand rule or right hand thumb rule.
Case I τ is zero, when θ = 0, i.e. when the plane of the coil
is perpendicular to the field.
Force on a Straight Wire Case II τ is maximum, when θ = 90° , i.e. the plane of the
If a current carrying straight F coil is parallel to the field. τ max = NBiA.
conductor (length l) is The above expression is valid for any shape of
placed in a uniform coil.
magnetic field B such that it B
makes an angle θ with the Work Done
i l
direction of field, then force
experienced by it is If the coil is to be rotated through an angle θ from its
equilibrium position (θ = 0° ), then required work done,
F = Bilsinθ.
W = MB (1 − cosθ ). It is maximum when θ = 180 °
(i) If θ = 0° , then F = 0 W max = 2 MB
(ii) If θ = 90° , then Fmax = Bil
(iii) In general, F = i( l × B ) or | F | = Bil sin θ
Moving Coil Galvanometer
Force between Two Parallel Current (MCG)
Carrying Conductors In a moving coil galvanometer, the coil is suspended
When two long straight conductors carrying currents i1 and between the pole pieces of a strong horse-shoe magnet. The
i2 are placed parallel at distance of a from each other, then a pole pieces are made cylindrical and a soft iron cylindrical
mutual force of attraction acts between them which is given core is placed within the coil without touching it. This makes
by the field radial. In such a field, the plane of the coil always
µ 2i i remains parallel to the field. When current will pass through
F12 = F21 = F = 0 ⋅ 1 2 × l
4π a this coil, then torque will be experienced by this coil.

@iitjeehelps
MAGNETIC FIELD 293

Conversion of Galvanometer
Spring into Ammeter
Magnet Moving
(fixed) N S An ammeter is made by connecting a low resistance S in
coil
parallel with a moving coil galvanometer G. S is known as
F shunt.

N S
ig G
i i
F
i – ig
Moving coil galvanometer

F = NBil ⇒ τ def = NBiA S


Suppose, N = number of turns in the coil, Ammeter
A = area of the coil,
B = magnetic field induction of radial magnetic Then, from circuit
field in which the coil is suspended. ig × G = (i − ig ) × S
As the coil deflects, a restoring torque is set up in the ig
suspension fibre. If α is the angle of twist, then the restoring ⇒ S= G
i − ig
torque is τ rest = Cα
where, C is the torsional constant of the fibre. So, S << G, only a small fraction of current goes through the
galvanometer.
When the coil is in equilibrium, then NBiA = Cα
C
i= α ⇒ i = kα
C
NBA Conversion of Galvanometer
where, k = is the galvanometer constant. This linear
NBA into Voltmeter
relationship between i and α makes the moving coil
A voltmeter is made by connecting a resistor of high
galvanometer useful for current measurement and
detection. resistance R in series with a moving coil galvanometer G.

(i) Current sensitivity ( Si ) The current sensitivity of a From the circuit


galvanometer is defined as the deflection produced in V
the galvanometer per unit current flowing through it. a b

α NBA
Si = ⇒ Si =
i C ig
Thus, in order to increase the sensitivity of a moving
coil galvanometer, N , B and A should be increased G R
and C should be decreased.
Voltmeter
Quartz fibres can also be used for suspension of the
coil because they have large tensile strength and
V
very low value of k. ig =
G+R
(ii) Voltage sensitivity (SV ) Voltage sensitivity of a
galvanometer is defined as the deflection produced V
⇒ G + R=
in the galvanometer per unit potential difference ig
applied to it. V
α α S NBA R= −G
SV = = = i ⇒ SV = ig
V iR R RC

@iitjeehelps
Practice Exercise
1. For the arrangement as shown in the figure, the 7. In the given figure, the loop is fixed but
magnetic induction at the centre is straight wire can move. The straight I1
wire will
a a. remain stationary
O b. move towards the loop
90°
c. move away from the loop
I d. rotates about the axis
3 µ 0Iπ µ 0I µ I 3 µ 0I 1
a. b. (1 + π ) c. 0 d. 8. The dimensions of are same as
4a 4 πa 4 πa 8a µ 0ε 0
2. If the resistance of the upper half of a rigid loop is E B
twice that of the lower half, the magnitude of magnetic a. b.
B E
induction at the centre is equal to E2  E
c. 2 d.  
B  B
I a I
O 9. A charged particle is moving in a uniform magnetic
field and losses 4% of its KE. The radius of curvature
µ I of its path changes by
a. zero b. 0
4a a. 2%
µ 0I b. 4%
c. d. None of these
12a c. 10%
3. The earth's magnetic field at a certain point is d. None of the above
0.70 gauss. This field is to be annulled by the 10. A charged particle of mass m and charge q, a uniform
magnetic field at the centre of a circular conducting magnetic field B acting into the plane. The plane is
loop 5.0 cm in radius. The required current is about frictional having coefficient of friction µ. The speed of
a. 0.66 A b. 5.6 A c. 0.28 A d. 2.8 A charged particle just before entering into the region is v 0.
v
4. Three circular concentric wires of radii a, 2a and 3a The radius of curvature of the path after the time 0
are carrying currents 3I , 2I and I in same manner. Find 2µg
the magnetic field at the common centre. is
13µ 0I µ 0I a.
mv 0
b.
mv 0
a. b.
6a 6a qB 2qB
µ I mv 0
c. 0 d. None of these c. d. None of these
a 4qB
5. A charge q(> 0) moves B 11. A γ-ray photon is passing near a nucleus and breaks
towards the centre of a x into an electron and positron. The region contains a
R
circular loop of radius R + uniform magnetic field B perpendicular to the plane of
v
along its axis. The magnitude q motion. Find the time after which they again converted
of B along the periphery of into γ-ray. The force of electrostatic interaction and
the loop is gravitational interaction may be neglected
µ0 qvR 2πm πm
a. zero b. a. b.
4π (R + x )
2 2 3 eB 2eB
4πm
qvR c. d. None of these
c. d. None of these eB
R +x
2 2
12. If a charged particle of charge 5 µC and mass 5 g is
6. A current carrying loop is free to turn in a uniform moving with constant speed 5 m/s in a uniform
magnetic field. The loop will then come into magnetic field B on a curve x 2 + y 2 = 25, where x and
equilibrium when its plane is inclined at y are in metre. The value of magnetic field will be
a. 0° to the direction of the field a. 1 tesla
b. 45° to the direction of the field b. 1 kilo tesla along z-axis
c. 90° to the direction of the field c. 5 kilo tesla along the x-axis
d. 135° to the direction of the field d. 1 kilo tesla along any line in the x-y plane

@iitjeehelps
MAGNETIC FIELD 295
13. A charged particle of mass m and charge q is 19. The magnitude of magnetic
accelerated through a potential difference of V volt. It moment of the current loop in a
enters a region of uniform magnetic field which is the figure is I
directed perpendicular to the direction of motion of the
a. Ia 2
particle. The particle will move on a circular path of
b. 2Ia 2
radius given by
c. zero a
a
 Vm  2Vm d. None of the above
a.  2  b.
 qB  qB 2 20. A length l of a wire is bent to form a circular coil of
 2Vm   1 Vm   1 some turns. A current I is then established in the coil
c.    d.   
 q   B  q   B and it is placed in a uniform magnetic field B. The
maximum torque that acts on the coil is
14. A circular flexible loop of wire of radius r carrying a Il 2B
current I is placed in a uniform magnetic field B . If B is a. IBl 2 b. 4πIBl 2 c. d. zero

doubled, then tension in the loop
a. remains unchanged b. is doubled 21. The ratio of the energy required to set up in a cube of
c. is halved d. becomes 4 times side 10 cm, a uniform magnetic field of 4 Wb/ m 2 and
a uniform electric field of 106 V/m is
15. A non-relativistic proton beam passes without
deviation through the region of space where there are a. 144 . × 10− 5 c. 144
. × 107 b. 144 . × 106 d. 144
. × 103
uniform transverse mutually perpendicular electric and 22. A magnetic field 4 × 10−3 kT exerts a force
magnetic fields with E = 120 kV/m and B = 50 mT.
Then the beam strikes a grounded target. Find the ( 4 $i + 3 $j ) × 10−10 N on a particle having a charge
force with which the beam acts on the target if the 10−9C and going on the x-y plane. The velocity of the
beam current is equal to I = 080
. mA. particle is
a. 80 µN b. 25 µN c. 20 µN d. 35 µN a. − 75 $i + 100 $j b. − 100 $i + 75 $j
16. An α-particle and a proton are both simultaneously c. 25 $i + 2$j d. 2$i + 25 $j
projected into a region of constant magnetic field,
perpendicular to the direction of the field but in 23. A uniform magnetic field B = B 0$j exists in space. A
opposite directions. After 10− 8 s, it is found that the particle of mass m and charge q is projected towards
velocity of α-particle has changed in the direction by X -axis with speed v from a point (a , 0, 0). The
45°. The angle between the velocity vectors of the maximum value of v for which the particle does not hit
α-particle and the proton is the Y-Z plane is
( 45°+90° ) Bqa Bqa Bq Bq
a. 90° b. 45° c. 45° + 90° d. a. b. c. d.
2 m 2m am 2 am
17. Four wires each of length 2 m are bent into four loops 24. Through two parallel wires A and B, 10 A and 2 A of
P , Q, R and S and then suspended into uniform currents are passed respectively in opposite
magnetic field. Some current is passed in each loop. directions. If the wire A is infinitely long the length of
Which statement is correct? the wire B is 2 m, then force on the conductor B, which
is situated at 10 cm distance from A, will be
P Q S a. 8 × 10−7 N b. 8 × 10−5 N
R
c. 4 × 10−7 N d. 4 × 10−5 N

a. Couple on loop P will be the highest 25. Two parallel long wires A and B carry currents I1 and
b. Couple on loop Q will be the highest I 2 (< I1). When I1 and I 2 are in the same direction, the
c. Couple on loop R will be the highest magnetic field at a point mid-way between the wires is
d. Couple on loop S will be the highest 10 µT. If I 2 is reversed, the field becomes 30 µT. The
ratio I1 / I 2 is
18. The magnetic moment of the a. 1 b. 3
O
current carrying loop shown in c. 2 d. 4
the figure is equal to a θ 26. A circular coil of 100 turns and effective diameter
I (b + 2ab )θ
2
Q 20 cm carries a current of 0.5 A. It is to be turned in a
a.
2 P magnetic field B = 2T from a position in which θ equals
b. Iab θ b zero to one in which θ equals 180°. The work required
I (a 2 + ba )θ in this process is
c. S R
2 a. π joule b. 2π joule
d. None of the above c. 4π joule d. 8π joule

@iitjeehelps
296 SELF STUDY GUIDE BITSAT

27. Two long parallel wires are placed on a smooth 28. Two long straight parallel conductors are separated by
horizontal table. They have equal and opposite a distance of 5 cm and carrying current 20 A. What
charges. Work required to increase separation work per unit length of a conductor must be done to
between wires from a to 2a if magnitude of charge per increase the separation between conductors to 10 cm,
unit length on them is λ, will be if the current flows in the same direction?
λ2 ln 2 λ2 λ2 λ2 a. 8 × 10− 5 loge 2 b. loge 2
a. b. ln 2 c. d. ln 2
4πε 0 πε 0 4πε 0a 2πε 0 c. 10− 7 loge 2 d. None of these

BITSAT Archives
1. A narrow beam of protons and deuterons, each 7. Two parallel conductors carry current in opposite
having the same momentum, enters a region of directions as shown in figure. One conductor carries a
uniform magnetic field directed perpendicular to their current of 10.0 A. Point C is a
direction of momentum. The ratio of the radii of the d
distance to the right of the 10.0
circular paths described by them is [2014] 2 l 10.0 A
a. 1: 2 b. 1: 1 c. 2 : 1 d. 1: 3 A current. If d = 18 cm and l is
adjusted so that the magnetic field C
2. A solenoid of length 1.0 m has a radius of 1 cm and at C is zero, the value of the d
has a total of 1000 turns wound on it. It carries a current I is [2013]
current of 5 A. If an electron was to move with a speed
of 104 ms −1 along the axis of this current carrying a. 10.0 A b. 30.0 A
c. 8.0 A d. 18.0 A
solenoid, then force experienced by this electron is
a. 2 N b. 1.2 N [2014] 8. Two long, parallel conductors carry currents in the
c. zero d. 2.5 N
same direction, as shown in figure. Conductor A
carries a current of 100 A and is
3. Magnetic field [2014] held firmly in position. Conductor IA
a. can increase the speed of charged particle B carries a current IB and is
b. can accelerate a charged particle allowed to slide freely up and
A
c. Both a and b are correct down (parallel to A) between a
d. Both a and b are incorrect set of non-conducting guides.
4. A square coil of side 10 cm has 20 turns and carries a The mass per unit length of IB B
current of 12 A. The coil is suspended vertically and conductor B is 0.1 g/cm and the
the normal to the plane of the coil, makes an angle θ distance between the two
with the direction of a uniform horizontal magnetic conductors is 5 cm. If system of conductors is in
field of 0.80 T. If the torque, experienced by the coil, equilibrium, the value of current IB is [2013]
equals 0.96 N-m, the value of θ is [2014] a. 250 A b. 240 A c. 220 A d. 230 A
π π π 9. A straight wire of mass 200 g and length 1.5 m carries
a. 0° b. rad c. rad d. rad
2 3 6 a current of 2 A. It is suspended in mid-air by a
5. For the circuit (figure), the A uniform horizontal magnetic field B. The magnitude of
current is to be measured. The B (in tesla) is (assume, g = 9.8 ms −2) [2012]
3.00Ω

ammeter shown is a a. 2 b. 1.5 c. 0.55 d. 0.65


galvanometer with a resistance
10. The intensity of the magnetic induction field at the
R G = 60.00 Ω converted to an
centre of a single turn circular coil of radius 5 cm
ammeter by a shunt resistance 3.00 V carrying current of 0.9 A is [2012]
rS = 0.02 Ω. The value of the
current is a. 36π × 10−7 T b. 9 π × 10−7 T
a. 0.79 A b. 0.29 A [2014] c. 36π × 10−6 T d. 9 π × 10−6 T
c. 0.99 A d. 0.8 A 11. The maximum current that can be measured by a
6. A square shaped current loop of side length L and galvanometer of resistance 40 Ω is 10 mA. It is
carrying current I lies in a uniform magnetic field B converted into a voltmeter that can read upto 50 V.
acting perpendicular to the plane of square loop and The resistance to be connected in series with the
directed inward. The net magnetic force acting on galvanometer (in ohms) is [2012]
current loop is [2013] a. 2010 b. 4050
a. lBL b. 4IBL c. zero d. 2IBL c. 5040 d. 4960

@iitjeehelps
MAGNETIC FIELD 297
12. A galvanometer of resistance 100 Ω gives full scale 19. In a galvanometer, 5 % of the total current in the
deflection with 0.01 A current. How much resistance circuit passes through it. If the resistance of the
should be connected in parallel to convert it into an galvanometer is G, the shunt resistance S connected
ammeter of range 10 A? [2011] to galvanometer is [2008]
a. 0.100 Ω . Ω
b. 100 c. 10.00 Ω d. 100.00 Ω G G
a. 19 G b. c. 20 G d.
19 20
13. A galvanometer of 50 Ω resistance has 25 divisions. A
current of 4 × 10−4 A gives a deflection of one division. 20. Two concentric coils of 10 turns each are placed in the
To convert this galvanometer into a voltmeter having a same plane. Their radii are 20 cm and 40 cm and
range of 25 V, it should be connected with a carry 0.2 A and 0.3 A current respectively in opposite
resistance of [2011] directions. The magnetic induction (in tesla) at the
a. 2500 Ω as a shunt b. 2950 Ω as a shunt centre is [2008]
3 5 7 9
c. 2550 Ω in series d. 2450 Ω in series a. µ0 b. µ0 c. µ0 d. µ0
4 4 4 4
14. The cyclotron frequency of an electron gyrating in a 21. A current carrying small loop behaves like a small
magnetic field of 1 T is approximately [2011, 2007] magnet. If A be its area and M its magnetic moment,
a. 28 MHz b. 280 MHz c. 2.8 GHz d. 28 GHz the current in the loop will be [2007]
15. A current carrying circular coil is bent so as to convert it a. M/A b. A/M
into a double loop, both the loops being concentric and c. MA d. AM 2
are carrying current in the same direction. If B is the 22. A magnet of magnetic moment 20 CGS units is freely
initial magnetic field at the centre, the final magnetic field suspended in a uniform magnetic field of intensity
at the centre will be [2010] 0.3 CGS unit. The amount of work done in deflecting it
a. zero b. B c. 2B d. 4B by an angle of 30° in CGS unit is [2007]
16. In order to increase the current sensitivity of moving a. 6 b. 3 3
coil galvanometers, the [2010] c. 3 ( 2 − 3 ) d. 3
a. number of turns of the coil should be increased
23. In a magnetic field of 0.05 T area of coil changes from
b. the strength of the magnetic field should be increased
101 cm 2 to 100 cm 2 without changing the resistance
c. area of coil should be increased
which is 2 Ω. The amount of charge that flow during
d. All of the above
this period is [2006]
17. In an ammeter 10% of main current is passing through a. 2.5 × 10−6 C
the galvanometer. If the resistance of the
galvanometer is G, then the shunt resistance, in ohms b. 2 × 10−6 C
is [2005] c. 10−6 C
a. 9 G b.
G
c. 90 G d.
G d. 8 × 10−6 C
9 90
24. An electric field of 1500 V/m and a magnetic field of
18. Due to magnetic field of earth, charged particles 0.40 Wb / m 2 act on a moving electron. The minimum
coming from outer space. [2009] uniform speed along a straight line, the electron could
a. required greater kinetic energy to reach the equator have is [2005]
than the poles
. × 1015 m/s
a. 16
b. required lesser kinetic energy to reach the equator
than the poles b. 6 × 10−16 m/s
c. can never reach the equator c. 3.75 × 103 m/s
d. can never reach the poles d. 3.75 × 102 m/s

@iitjeehelps
Answer with Solutions
Practice Exercise 10. (b) Here, centripetal acceleration is provided by magnetic
force but tangential acceleration is provided by force of
1. (d) friction
2. (c)Q I1R1 = I 2R2 mv 2
= qvB
I1 R2 R 1 I1 r
= = 2 = a
I 2 R1 2R2 2 mv
P O Q ∴ r =
∴ I 2 = 2I1 qB
I2
I = I1 + I 2 = I1 + 2I1 = 3I1 − µmg
and Tangential acceleration = a = .
I 2 m
∴ I1 = and I 2 = I
3 3  v  v
v = − µg  0  = 0
Q Magnetic field at the centre of coil  2µg  2
µ I µ I ×π mv mv 0
= 01 × π − 0 2 ∴ r = =
4 πa 4 πa qB 2qB
µ 0π µ I 2  µ  − I µ I µ I
∴B = (I1 − I 2 ) = 0  − I  = 0   = − 0 = 0 11. (a) Both position and electron are moving on circular
4 πa 4a  3 3  4a  3  12a 12a paths.
3. (b) 2πm
∴ T =
4. (a) B = B1 + B2 + B3 eB
2I I 12. (b) 13. (c) 14. (b)
µ ( 3I ) µ 0( 2I ) µ 0I I
= 0 + + 15. (c) Since, proton moves with constant velocity. So,
2a 2( 2a ) 2( 3a )
magnetic force on the proton is balanced by electric force.
3 µ 0I µ 0I µ 0I
= + + ∴ qE = qvB or E = vB
2a 2a 6a 3I
∆p mnv
9µ 0I + 3 µ 0I + µ 0I 13 µ 0I The force on the target is F = =
= = ∆t ∆t
6a 6a
where, n = number of protons striking the target in time dt.
µ0 v × r
5. (d) B = q 3 m = mass of proton
4π r
v = speed of each proton before striking the target
µ 0q vr sin θ
B= r dr
ne n I
3 I= ⇒ =
4π(R +2
x 2 )2 ∆t ∆t e
mvI mI E 
µ 0q vR ∴ F = =
B= [Q r sin θ = R] e e B 
4π 3
(R + x ) . × 10− 27 × 120 × 1000 × 0.8 × 10− 3
2 2 2
16
F =
6. (a) 7. (b) 8. (a) 50 × 10− 3 × 16
. × 10− 19

9. (a) r =
mv = 20 × 10−6 N = 20 µN
qB ( 2e )B 1 eB
1 16. (c) ω α = =
KE = K = mv 2 4m 2 m
2 eB
ωp =
∴ mv = 2Km m
mv 2Km θ α = 45°
∴ r = = or r ∝ K
qB qB θ p = 90° from their original directions.
1
17. (d) Concept Couple = I( A × B)
or r = eK 2 , where e is a constant.
More area means more couple.
1
dr dK e ec2 ec dK ∴For a given perimeter, circle has maximum area.
or =e = or = K
dr dr 22 K 2 K dr 18. (a) Magnetic moment
∆K ∆r c∆ K ∆K M = IA = I ( A2 − A1)
or e = 2 K or = =
∆r r 2 Kc K 2K (a + b )2 a 2  l(b 2 + 2ab )θ
= I θ− θ =
∆r ∆K  ∆K   2 2  2
or × 100 = × 100 = 2% Q × 100 = 4%
r 2K  K 
19. (b)

@iitjeehelps
MAGNETIC FIELD 299
20. (c) τ = NIAB = NIπr 2B ∴ ( 4$i + 3$j ) × 10−10 = 10−9(a $i + b$j ) × ( 4 × 10−3 ) k$
But, N × 2πr = l Solving, we get
l a = − 75, b = 100 ⇒ v = − 75 $i + 100$j
∴ r =
2πN
2
23. (a) 24. (b)
 l  I2 Il 2B µ 0 2I1 µ 0 2I 2
∴ τ = NIπ   B = NIπ B = 25. (c) − = 10 µT
 2 πN  4π 2N 2 4 πN 4π r 4π r
Il 2B µ 0 2I1 µ 0 2I 2
∴ τ max = [Nmin = 1] + = 30 µT
4π 4 πr 4π r
1 On solving, I1 = 20 A and I 2 = 10 A
21. (c)UE = ε 0E 2 × volume
2 So, I1 / I 2 = 2
B2 26. (b)
UB = × volume
2µ 0 2a λ. λdr λ2
27. (d)W = ∫ = ln 2
UB  B 
2
1  4 
2 a 2πε 0r 2πε 0
∴ =  =  ( 3 × 10 )
8 2
UE  E  µ 0ε 0  106  28. (a) The force per unit length is
16 µ II
= 12 × 9 × 1016 = 144 × 104 = 144 . × 106 F0 = 0 1 2
10 2 πr
22. (a) From Lorentz force, µ I I 10 × 10− 2 dr
∴ W = ∫ Fdr = 0 1 2 ∫
F = q(v × B ) 2π 5 × 10− 2 r
µ 0I1I 2
Given, F = ( 4$i + 3$j ) × 10−10 N, q = 10−9C = ln 2 = 2 × 10− 7 × 20 × 20 ln 2

B = 4 × 10−3 kT $
= 8 × 10− 5 ln 2 = 8 × 10− 5 loge 2

BITSAT Archives
1. (b) Since, the radius of circular path of a charged particle Torque, experienced by the coil, τ = 0.96 N-m
mv p Since, total torque on the coil, τ = (NIA )B sin θ
in magnetic field is r = =
qB qB Substituting the values in above formula, we get
Now, the radius of circular path of charged particle of given 0.96 N-m = 20 × 12 A × 0.01m2 × 0.80 T × sin θ
1
momentum p and magnetic field B is given by r ∝ 0.96 1
q sin θ = =
.
192 2
But charge on both charged particles, protons and π
deuterons, is same. Therefore, θ = rad
6
rp qD 1
= = 5. (c) RG = 60.00 Ω; shunt resistance, rs = 0.02 Ω
rD q p 1
Total resistance in the circuit is RG + 3 = 63 Ω.
2. (c) Here, L = 1m, N = 1000 Hence, I = 3 / 63 = 0.048 A
The number of turns per unit length Resistance of the galvanometer converted to an ammeter
n = N/L = 1000 turn/m is,
Magnetic field inside the solenoid RGrs 60 Ω × 0.02 Ω
= = 0.02 Ω
B = µ 0nI = µ 0 × 1000 × 5 = 2π × 10−3 T RG + rs ( 60 + 0.02) Ω
The direction of magnetic field is along the solenoid. Total resistance in the circuit = 0.02 + 3 = 3.02 Ω
For electron, q = − e, v = 104 ms −1 Hence, l = 3 / 3.02 = 0.99 A
Magnetic Lorentz force, F = − evB sin 0° = 0 as the angle 6. (c) Let the current be flowing in A ⊗l D
between B and v is 0°. anti-clockwise direction as shown ⊗B
in figure. l
3. (b) Magnetic field can accelerate a charged particle by
changing the direction of its velocity but it cannot change Now, magnetic force on ⊗
l
the speed of charged particle as magnetic force always AD = F1 = ILB inwards
acts perpendicular to the velocity of charged particle. (by Fleming’s left hand rule) ⊗
C
4. (d) Area of coil, A = side 2 = (0.1)2 = 0.01 m2, Similarly, magnetic force on B l
BC = F2 = ILB inwards
Number of turns, N = 20, current, I = 12 A,
Since, two forces are equal in magnitude and opposite in
Normal to the coil make an angle θ with the direction of B,
direction, therefore, they cancel out each other.
magnetic field, B = 0.80 T

@iitjeehelps
300 SELF STUDY GUIDE BITSAT

Also, magnetic force on CD = F3 = ILB inwards 13. (d) Here, Ig = 25 × 4 × 10−4A = 10−2 A
(by Fleming’s left hand rule) To convert the galvanometer into a voltmeter, we must
Similarly, magnetic force on AB = F4 = IlB inwards. join a series resistance of
Again, two forces are equal in magnitude and opposite in V 25
R = − G = −2 − 50 = 2500 − 50 = 2450 Ω
direction, therefore, they cancel out each other. Ig 10
7. (b) The magnetic field at C due to first conductor is
µ I d 3d Bq . × 10−19
1 × 16
B1 = 0 (since, point C is separated by d + = 14. (d) Cyclotron of frequency, ν = =
2π 3d / 2 2 2 2πm 2π × 9.1 × 10−31
from 1st conductor). The direction of field is perpendicular = 2.8 × 1010 Hz = 28 GHz
to the plane of paper and directed outwards.
µ 0i
The magnetic field at C due to second conductor is 15. (d) Binitial =
2r
µ 10 d
B2 = 0 (since, point C is separated by from 2nd µ0i µ0 i 2µ0 i
2 π d /2 2 Bfinal = + ⇒ Bfinal = = 4Binitial
r r r
conductor) 2  2 
 2  2
The direction of field is perpendicular to the plane of
paper and directed inwards. 16. (d) Current sensitivity of the galvanometer can be
Since, direction of B1 and B2 at point C is in opposite increased by all three factors.
direction and the magnetic field at C is zero, therefore, Ig G 0.1G G
B1 = B2 17. (b) Shunt resistance S = = =
I − Ig 1 − 0.1 9
µ0 I µ 10
= 0
2 π 3d / 2 2 π d / 2 18. (c) Due to magnetic force the charged particles get
deviated. For positively, charged particles, due to
On solving I = 30.0 A east-west asymmetry of particles, more particles get
8. (a) When system of conductors is in equilibrium. The deviated towards east, then towards west, but since they
magnetic force of attraction per unit length between are deviated, hardly any charged particle will reach
conductors = weight of conductor B per unit length. equator.
µ 0 I A × IB mg  m  19. (b) Shunt of an ammeter,
= =  g Ig × G
2π d L  L 5 ×G G
S = = =
I − Ig 100 − 5 19
µ 0 I A × IB  m 
=  g
2π d  L 20. (b) Two coils carry currents in opposite directions, hence
net magnetic field at centre will be difference of the two
100 × IB
2 × 10−7 × = (0.01 kg/m) × 10 fields.
0.05 i
µ i  10 µ 0  0.2 0.3  5
On solving, we get IB = 250 A i.e. Bnet = 0 . 2 π N  1 − 2  = − = µ0
4π 1
r r 2 2  0.2 0.4  4
9. (d) Magnetic force on straight wire
M
F = Bil sin θ = Bil sin 90° = Bil 21. (a) M = iA ⇒ i =
A
For equilibrium of wire in mid-air,
22. (c) Work done,W = MBµ (1 − cos θ )
F = mg
Bil = mg  3
= 20 × 0.3 (1 − cos 30° ) = 6 1 −  = 3 (2 − 3)
mg 200 × 10−3 × 9.8  2
∴ B= = = 0.65 T
il 2 × 15
. 23. (a) B = 0.5 T, A1 = 101 cm2 = 101 × 10−4 m2
10. (a) The intensity of magnetic induction field A2 = 100 cm2 = 100 × 10−4 m2
µ i 4π × 10−7 × 0.9 R =2Ω
B= 0 =
2r 2 × 5 × 10−2 Amount of charge flowing is
B = 36π × 10−7 T B∆A 0 . 50 × (101 × 10−4 − 100 × 10−4 )
q= =
R 2
11. (d) To convert a galvanometer into voltmeter, the −4
0.05 × 1 × 10
necessary value of resistance to be connected in series = = 2.5 × 10−6 C
with the galvanometer is 2
V 50 24. (c) Here, E = 1500 V / m, B = 0.4 Wb / m2
R = −G= − 40 = 5000 − 40 = 4960 Ω
Ig 10 × 10−3 E
Minimum speed of electron along the straight line,v =
12. (a) Shunt is given by B
Ig × Rg 0.01 × 100 0.01 × 100 =
1500
= 3750 = 3.75 × 103 m/s
S = = = = 0.100 Ω 0.4
I − Ig 10 − 0.01 9.99

@iitjeehelps
26
Magnetostatics

Current Loop as a Magnetic Dipole


A current loop is equivalent to a magnetic dipole. If A( = pa 2 ) be the area of the loop, then the
magnitude of its dipole moment is
pm = iA = ipa 2
where, a is radius of coil and i is current flowing through it.
p
\ i = m2 …(i)
pa
Magnetic field at the centre of a circular current loop is given by
mi
B= 0 …(ii)
2a
Putting the value of i from Eq. (i) in Eq. (ii), we get
m p
B= 0 m
2 p a3
This is the expression for the magnetic field at the centre of the current loop in terms of its dipole
moment. Instead of a circular loop, if there is a circular coil havingn turns, its dipole moment would be
pm = niA = ni pa 2 .

Torque on a Magnetic Dipole Placed in a


Uniform Magnetic Field
In the given figure, a magnetic dipole is placed in a uniform magnetic field B represented by
equidistant parallel lines.
Now, torque exerted on magnetic dipole by magnetic field is
Þ tm = m ´ B
Here, t m = magnetic torque
Þ t m = mB sin q

@iitjeehelps
302 SELF STUDY GUIDE BITSAT

where, B = magnetic field, m = magnetic moment of the bar e


and ml (minimum) = ×h
magnet, q = the angle between m and B. 4pme
So, torque is zero when q = 0° or 180° and it is maximum = 9.27 ´ 10-24 A -m 2 ( n = 1)
when q = 90°. This value is called Bohr’s magneton.
N
B
τm
θ Potential Energy of a Magnetic Dipole
S
in a Uniform Magnetic Field
Dipole in magnetic field When we rotate a dipole with the help of external torque,
we have to do some external work.
\ W ext = DU …(i)
Magnetic Dipole Moment of Þ W ext = ò dW = ò t dq
a Revolving Electron = òq
q2

1
mB sinq dq
A revolving electron constitutes a current,
W ext = - m B (cosq 2 - cosq 1 ) …(ii)
e– From Eqs. (i) and (ii), we get
+Ze me -m B (cosq 2 - cosq 1 ) = DU …(iii)
Let at q = 90° , U = 0
Revolving electron So, if q 1 = 90°, q 2 = q
e and U i = 0, then
i.e. i=
T W ext = - mB (cosq 2 ) = U f
2p Þ U = - mB cosq = - m × B
As, T = time period of revolution, T = ,
w
where, w = electron’s orbital angular speed
Magnetic Dipole Moment Associated
ew ev
\ i = equivalent current = = ( v = rw) with Charged Rotating Bodies
2 p 2 pr
So, magnetic moment associated with this circulating As a charged rotating body constitutes an equivalent
current current. So, they have a magnetic dipole moment
evr e associated with them. For geometrically symmetric
ml = i p r 2 = = (mevr )
2 2me charged bodies, magnetic moment is given by
e
where, r = radius of an orbit = × l, ω ω
2me
+ + +
where, l = angular momentum ( l = mevr ) + ++ +
+ ++ + + ++
+ +
+
-e + + + +
and vector form m l = l +
2me
or the angular momentum vector and magnetic dipole A disc Ring Rod
moment vectors are opposite in direction. Charged rotating bodies
m e
The ratio l = is called gyromagnetic ratio. Its value
l 2me qL qIw
m= = ( L = Iw)
is 8.8 ´ 1010 C/kg. 2m 2m

According to Bohr’s postulate, | l | =


nh where, L= angular momentum,
2p I= moment of inertia,
nhe w= angular velocity,
\ ml =
4p m e m= mass of rotating body

@iitjeehelps
MAGNETOSTATICS 303

Bar Magnet Earth’s Magnetic Field and


A bar magnet may be viewed as a combination of two
magnetic poles, north pole and south pole, separated by Magnetic Elements
some distance. The distance is known as the magnetic Earth’s magnetic field on the earth’s surface is around
length of the given bar magnet. 0.4 gauss and its value shows variations with time. At
If a bar magnet is broken, the fragments are independent present, magnetic south pole is located at northern Canada
magnetic dipoles and not isolated magnetic poles. 70.5° N latitude and 96° W longitude. North magnetic is at
70.5° S and 84° E at Arctic. The magnetic poles are around
Magnetic Field due to a Bar Magnet 2000 km apart.
A bar magnet has a magnetic field around it. Magnetic field Axis of rotation
is mathematically measured by a vector term B, whose Geographical of the earth
north pole 11.5°
SI unit is 1 tesla (1 T). Magnetic south pole
The magnetic field in free space, at a point at a distance r S Magnetic
from the given bar magnet (or magnetic dipole) equator
N
(i) Along its axial line Magnetic
north pole
m0 2 Mr
B= Geographical
4p (r - l 2 )2
2
south pole

and the direction of B is the same as the direction of


Magnetic equator intersects the geographic equator at 6° W
M. For a short dipole (or for a far away point on the
longitude and 174° E latitude. Earth’s field reverses itself in a
axis) when r > > l , the above relation is simplified as
span of around one million years. Declination shows a
m0 2 M yearly variation. As per the most established theory, it is
B=
4p r 3 due to the rotation of the earth whereby various charged
(ii) Along the equatorial line of a magnetic dipole,the ions present in the molten state in the core of the earth
magnetic field B in free space is given by rotate and constitute a current.
m0 M Magnetic equator divides the earth into two hemispheres.
B=
4p (r + l 2 )3 / 2
2 The hemisphere containing south polarity of the earth’s
m0 M magnetism is called northern hemisphere. While the other
If r > > l , the relation is modified as B = hemisphere containing north polarity of the earth’s
4p r 3
magnetism is called the southern hemisphere. Direction of
However, along the equatorial line, the direction of B the earth’s magnetic field is from S (geographical south) to
is opposite to that of M. N (geographical north).
In general, in a direction making an angle q with the The magnitude and direction of the magnetic field of the
magnetic axis, the magnetic field is given by earth at a place are completely given by certain quantities
m0 M known as magnetic elements.
B= (3 cos2 q + 1)
4p r 3 Three magnetic elements of the earth are
In these relations, m0 is a constant having a value of
(i) Magnetic declination (f) It is the angle between
4p ´ 10-7 T mA -1 and it is known as the magnetic geographic and the magnetic meridian planes.
permeability of free space. Geographical
For solenoid B = m0ni, where n is number of turns of north BH f
Magnetic θ
solenoid and i is the current flowing through it. north Bv
NOTE Time period of oscillations of a bar magnet in a uniform Geographical B Magnetic
magnetic field. meridian meridian
I
T = 2p
mB

@iitjeehelps
304 SELF STUDY GUIDE BITSAT

Declination at a place is expressed at f° E or f° W Relative magnetic permeability m r is a unitless and


depending upon whether the north pole of the dimensionless term. For magnetic substance like iron,
compass needle lies to the east or to the west of the m r > 1000
geographical axis. Relative permeability of a diamagnetic substance is less
than 1 (m r < 1), but it can never be negative. Thus, 0 £ m r < 1,
(ii) Angle of inclination or magnetic dip (q) It is the
for a diamagnetic material.
angle between the direction of intensity of total
magnetic field of the earth and a horizontal line in
the magnetic meridian. If l is magnetic latitude at a 3. Intensity of Magnetisation (I)
place, then angle of dip q at the place is given by Intensity of magnetisation of a substance is defined as the
tan q = 2 tan l . magnetic moment induced in the substance per unit
volume, when placed in the magnetising field.
(iii) Horizontal component of the earth’s magnetic field Thus,
( B H ) Earth’s magnetic field is horizontal only at the M
magnetic equator. I=
V
At any other place, the total intensity can be
It is a vector quantity and its SI unit is Am -1 .
resolved into horizontal component ( B H ) and
vertical component ( BV ).
Also, B H = B cos q and BV = B sin q 4. Intensity of Magnetising Field
BV or Magnetic Intensity (H)
So, B = B H2 + BV2 and tan q =
BH It is a measure of the capability of external magnetising
æB ö field to magnetise the given substance and is
Þ q = tan -1 ç V ÷
è BH ø mathematically defined as
At equator, q = 0 Þ B H = B , BV = 0, while at poles B B B
H = 0 or H = or H = -I
q = 90° m0 m m0
Þ B H = 0, BV = B Magnetic intensity H is a vector quantity and its SI unit is
Am -1 .

Important Terms Used in 5. Magnetic Susceptibility ( c m )


Magnetism Magnetic susceptibility of a substance is the ratio of the
intensity of magnetisation I induced in the substance to the
1. Magnetic Induction or Magnetic I
magnetic intensity H. Thus, c m = , it is a scalar term and
Flux Density (B) H
Whenever a piece of magnetic substance is placed in an has no unit or dimension.
external magnetising field, the substance becomes Relation (i) BI = m 0( H + I ) and (ii) m r = 1 + c m
magnetised. If B0 is the magnetic field in free space, then on
placing the given magnetic substance at that place, the
magnetic field changes from B0 to B, where B = m r B 0 Magnetic Behaviour of
ò B × dS is magnetic flux which is equal to m0m inside , where Materials
m inside is the net pole strength inside a closed surface.
For a dipole, Gauss’s law for magnetism is ò B × dS = 0 for a Diamagnetic Materials
closed surface. These are materials which show a very small decrease in
magnetic flux when placed in a strong magnetising field.
2. Magnetic Permeability (µ ) Hydrogen, water, copper, zinc, antimony, bismuth, etc. are
It is the degree or extent to which the magnetic lines of the examples of diamagnetic materials.
induction may pass through a given distance.
Magnetic permeability of free space m0 has a value of
4p ´ 10-7 TmA -1. However, for a material substance, Paramagnetic Materials
absolute permeability ( m ) has a value different than m 0. These are those materials which show a small increase in
m B the magnetic flux when placed in a magnetising field.
For any magnetic substance = = m r = relative Oxygen, air, platinum, aluminium, etc., are examples of
m0 B 0
magnetic permeability of that substance. paramagnetic materials.

@iitjeehelps
MAGNETOSTATICS 305

Ferromagnetic Materials Initially as H is increased, the I


These are those materials which are strongly attracted by a intensity of magnetisation I
B A
magnetic field and can themselves be magnetised even in a developed in the material
weak magnetising field. Iron, steel,nickel and cobalt are increases non-linearly along Retentivity
the curve OA and reaches a C
ferromagnetic materials. H
O F
maximum, known as saturated
magnetism. Now on reducing E
Curie’s Law and Curie H, I follows the path AB. Point B
D

Temperature corresponds to H = 0 but I has a


finite positive value. This value
Coercivity

of I is called remanence or retentivity or residual


Curie’s Law magnetisation.
According to Curie’s law, the magnetic susceptibility of
paramagnetic substances is inversely proportional to If direction of H is reversed and is gradually increased, H
absolute temperature, curve is along BC. Point C represents the situation when I is
1 finally reduced to zero. The reversed value of I, represented
i.e. cµ
T by OC, is known as the coercivity of the material.
Here, T = absolute temperature. On increasing H in the reverse direction further, graph CD is
On increasing temperature, magnetic susceptibility of obtained. Now, if H is taken back from its negative
paramagnetic substances decreases or vice-versa. saturation value to its original positive saturation value a
similar I-H curve represented by DEFA is traced.
Curie Temperature or Curie Point The whole graph ABCDEFA is a closed loop and known as
The magnetic susceptibility of these substances decreases hysteresis loop.
on increasing the temperature and above a particular
temperature, a ferromagnetic substance behaves like a
paramagnetic substance.
Vibration Magnetometer
This particular temperature is called the Curie temperature
of the substance. (a) If small magnet is placed in magnetic maridian and it
vibrates in horizontal plane, then
e.g. The Curie temperature of iron is 770 °C. It follows that at
a temperature below 770 °C, the iron is ferromagnetic and at I
T = 2p
a temperature above 770 °C, the iron is paramagnetic. MH
Similarly, the Curie temperature of nickel is 369 °C and that æl2 + b2 ö
of cobalt is 1150 °C. where, I = Mç ÷
è 12 ø
Curie-Weiss Law Its breadth is negligible,
At temperature above Curie point, the C M0l 2
magnetic susceptibility of ferromagnetic I=
12
substances is inversely proportional to
(T - TC ), i.e. (b) If magnet is placed parallel to magnetic meridian and
1 C T oscillates in vertical plane, then
TC
cµ or c =
T - TC T - TC æ I ö
T = 2p ç ÷
Here, TC = Curie temperature. è MBe ø
(c) If magnet is placed perpendicular to magnetic
Hysteresis Curve meridian and oscillates in vertical plane

A great deal of information can be learned about the magnetic æ I ö


T = 2p ç ÷
properties of a material by studying its hysteresis loop. A è MV ø
hysteresis loop shows the relationship between the induced
(d) Comparision of magnetic moment by sum and
magnetic flux density (B) and the magnetising force (H). It is
difference method.
often referred as B-H loop.
A ferromagnetic material can be easily magnetised by M 1 T 12 + T 22
=
placing it in an external magnetising field H. M 2 T 12 - T 22

@iitjeehelps
Practice Exercise
1. Calculate the force acting between two small 10. Two small magnets of each of magnetic moment M 0 is
magnets, placed in end on position 0.1 m apart from placed parallel to each other (shown in figure). The
their centres. (Given, magnetic moment of each magnetic field at point O is
magnet is 5 Am 2). M0
2 cm
a. 0.6 N b. 0.8 N c. 0.15 N d. 0.2 N O
2 cm
2. Two similar equal magnetic poles when separated by M0
a distance of 1 m, they repel with a force of 10-3 N.
a. zero b. 4 ´ 10-4N
The pole strength is
a. 10 Am b. 20 Am c. 50 Am d. 100 Am c. 2 ´ 10-4N d. None of these

3. Three similar magnetic south poles each of strength 11. Two short magnets of magnetic moment 2 Am 2 and
10 Am are placed at the corners of an equilateral 5 Am 2 are placed along two lines drawn at right angle
triangle of side 20 cm. Find the magnetic force on one to each other on the sheet of paper as shown in the
figure. What is the magnetic field at the point of
of the pole.
intersection of their axis?
a. 0.25 ´ 10-3 N b. 10-3 N
c. 10 ´ 10-3 N d. None of these S N P
1 r1=0.3 m
4. Six similar magnetic poles are placed on six corners of
a regular hexagon of side10 cm. A south pole of r2=0.4 m
strength 10 Am is placed at the centre of hexagon. N
Find the magnetic force on the south pole.
a. Zero b. 4p ´ 10-4N 2 S
-5
c. 10 N d. None of these a. 2 .15 ´ 10 T b. 215 ´ 10-5 T
5. A magnetic wire of dipole moment 4 p Am 2 is bent in c. 2 .15 ´ 10-3 T d. 21.5 ´ 10-5 T
the form of semicircle. The new magnetic moment is 12. Calculate the magnetic induction at P , for the
a. 4p Am2 b. 8p Am2
arrangement shown in the figure, when two similar
c. 4 Am2 d. None of these short magnets of magnetic moment M are joined at
the middle. So that, they are mutually perpendicular.
6. Current I is flowing in a conducting wire of length l. It is
bent in the form of a circular coil of single turn. Its B N
magnetic moment will be
Il I 2l Il 2 I 2l 2 d P
a. b. c. d. A
4p 4p 4p 4p S N
7. The magnetic induction inside a solenoid is S
6.5 ´ 10-4 T. When it is filled with iron medium, then
the induction becomes 1.4 T. The relative permeability m0 M 3 m 0 2M m 0M 5 m 0 2M
a. b. c. d.
of iron will be 4p d 3 4p d 3 4 pd 3 4 pd 3
a. 1578 b. 2355 13. A small magnet of dipole moment M is kept on the
c. 1836 d. 2154 arm of a deflection magnetometer set in tan A position
8. If a magnetising field of 1600 A/m produces a at a distance of 0.2 m. If the deflection is 60°, find the
magnetic flux of 14 . ´ 10-5 Wb in an iron bar of value of P (BH = 0.4 ´ 10-4 T).
cross-sectional area 0.2 cm 2. Then, a. 2.77 Am2 b. 8 Am2
a. magnetic permeability of iron rod is around 1000 c. 0.2 Am2 d. None of these
b. magnetic susceptibility is very larger than unity 14. A short bar magnet of magnetic dipole moment
c. magnetic susceptibility is 340
m = 0.32 JT-1 is placed in a uniform external magnetic
d. None of the above
field of 0.15 T. If bar is free to rotate in the field, then
9. Two dissimilar poles of strength x mWb and 2 mWb a. when m is anti-parallel to B, U = 4.8 ´ 10-2J
are separated by a distance 12 cm. If the null point is b. when m is perpendicular to B, U = 0
at a distance of 4 cm from 2 mWb, then calculate x. c. when m is perpendicular to B, U = 4.8 ´ 10-2J
a. 5 mWb b. 6 mWb c. 7 mWb d. 8 mWb d. None of the above

@iitjeehelps
MAGNETOSTATICS 307
15. A magnet of dipole moment 2 Am 2 is deflected 23. Two like poles of strength m1 and m 2 are far distance
through 30° from magnetic meridian. The required apart. Calculate the energy required to bring them
deflecting torque is (BH = 0.4 ´ 10-4 T) r 0 distance apart.
a. 0.4 ´ 10-4 Nm b. 0.4 Nm m 0 m1 m2 m 0 m1 m2
a. b.
c. 0.2 ´ 10 -4
Nm d. None of these 4pr0 8pr0
m 0 m1 m2
16. If the areas under the I-H hysteresis loop and B-H c. d. None of these
16pr0
hysteresis loop are denoted by A 1 and A2, then
a. A 2 = m 0 A1 b. A 2 = A1 24. Calculate the work done in deflecting a small magnet
A of magnetic moment 10Am 2 through 180° from a
2
c. A 2 = 1 d. A 2 = m 0A 1 uniform magnetic field of strength 0.4 ´ 10-4 T.
m0
a. 8 ´ 10-4J b. Zero
17. Consider the plane S formed by the dipole axis and the c. 4 ´ 10-4J d. None of these
axis of the earth. Let P be point on the magnetic
equator and in S. Let Q be the point of intersection of 25. At a place the values of BH and BV are 0.4 ´ 10-4 T
the geographical and magnetic equators. The and 0.3 ´ 10-4 T respectively. The resultant earth’s
declination and dip angles at P and Q are magnetic field is
a. 0° and 11.3° b. 0° and 0° a. 0.5 ´ 10-4 T b. 10-4 T
. ° and 6.5°
c. 113 d. 11.3° and 11.3° c. 2 ´ 10-4 T d. None of these
18. A magnet of length 14 cm and magnetic moment M , is 26. In previous problem, the angle of dip is
broken into two parts of length 6 cm and 8 cm
a. tan-1 ( 0.75) b. tan-1 ( 0.5)
respectively. They are put at right angles to each
other, with opposite poles together. The magnetic c. tan-1 ( 0.8) d. None of these
moment of the combination is 27. Calculate the real angle of dip, if a magnet is
a. M/10 b. M suspended at an angle of 30° to the magnetic
c. M/1.4 d. 2.8 M meridian, the dip needle makes an angle of 45°
19. A uniform magnetic needle of strength of each pole is with horizontal.
98.1 amp. cm, is suspended from its centre by a æ 3ö æ 3ö æ 2 ö
a. tan-1 ç ÷ b. tan-1 ( 3 ) c. tan-1 ç ÷ d. tan-1 ç ÷
thread. When a mass of 50 mg is loaded to its upper è 2 ø è 2ø è 3ø
end, the needle becomes horizontal, then the vertical
component of earth’s magnetic induction is 28. A magnet is cut in three equal parts by cutting it
( g = 981 cm/ s 2 ) perpendicular to its length. The time period of original
magnet is T0 in a uniform magnetic field B. Then, the
a. 0.50 gauss b. 0.25 gauss
time period of each part in the same magnetic field is
c. 0.05 gauss d. 0.005 gauss T0 T0
a. b.
M 2 3
20. M and M / 3 are the magnetic T0
dipole moments of the two c. d. None of these
4
magnets, which are joined to 1
2 29. A magnet is cut in four equal parts by cutting it parallel
form a cross figure. Find the θ
inclination of the system with to its length. What will be the time period of each part,
B
the field, if their combination is if the time period of original magnet in the same field is
suspended freely in a uniform T0?
M√3 T0 T0 T0
external magnetic field B. a. b. c. d. 4 T0
2 2 4
a. q = 30° b. q = 45°
30. The current on the winding of a toroid is 2 A. It has
c. q = 60° d. q = 15° 400 turns and mean circumferential length is 40 cm.
21. Calculate the couple acting on a magnet of length With the help of search coil and charge measuring
10 cm and pole strength 15 Am, kept in a field of instrument the magnetic field is found to be 1 T. The
susceptibility is
B = 2 ´ 10-5 T at an angle of 30°.
a. 100 b. 290 c. 398 d. 397
a. 1.5 ´ 10-5 Nm b. 1.5 ´ 10-3 Nm
31. The magnetic needle of a tangent galvanometer is
c. 1.5 ´ 10-2 Nm d. 1.5 ´ 10-6 Nm deflected at an angle 30° with respect to the magnet.
22. A bar magnet of magnetic moment 2.5 J/T, is placed The horizontal component of the earth’s magnetic field
in magnetic field 0.2 T. What work is done in turning is 0. 34 ´ 10-4 T along the plane of the coil. The
the magnet from parallel to anti-parallel position magnetic intensity is
a. 1.96 ´ 10-4 T b. 1.96 ´ 104 T
relative to field direction?
a. 1 J b. 2 J c. 3 J d. 4 J c. 1.96 ´ 10-5 T d. 1.96 ´ 105 T

@iitjeehelps
308 SELF STUDY GUIDE BITSAT

32. Calculate the angle of dip, if a dip needle oscillating in a. 2.356 SI units b. 1.335 SI units
a vertical plane makes 40 oscillations per minute in a c. 3.664 SI units d. 1.664 SI units
magnetic meridian and 30 oscillations per minute in a 34. An iron rod is subjected to cycles of magnetisation at
vertical plane at right angle to the magnetic meridian. the rate of 50 Hz. Given the density of the rod is
a. q = sin-1( 0.5625) b. q = sin-1( 0.325)
8 ´ 103 kg/m 3 and specific heat is
c. q = sin-1( 0.425) d. q = sin-1( 0.235) . ´ 10-3 cal/kg ° C. Find the rise in temperature per
011
33. Inside a long solenoid wounded with 300 turns/m, an minute. If the area enclosed by the B-H loop
iron rod is placed. An iron rod is 0.2 m long, 10 mm in corresponds to energy of 10-2 J. Assume there is no
diameter and of permeability 103 . Calculate the radiation losses.
magnetic moment of the rod, if 0.5 A of current is a. 78° C b. 88° C
passed through the rod. c. 8.1° C d. None of these

BITSAT Archives
1. The susceptibility of a magnetism at 300 K is when one piece is used (in second) (bar magnet
1.2 ´ 10-5 . The temperature at which the susceptibility breadth is small) is [2008]
increases to 1.8 ´ 10-5 is [2014] a. 16 b. 8 c. 4 d. 2
a. 150 K b. 200 K c. 250 K d. 20 K
6. The magnetised wire of moment M and length l is bent
2. Magnetic moment of bar magnet is M . The work done in the form of semicircle of radius r . Then, its magnetic
to turn the magnet by 90° of magnet in direction of
moment is [2008]
magnetic field B will be [2012] 2M M
1 a. b. 2M c. d. zero
a. zero b. MB c. 2 MB d. MB p p
2
7. Susceptibility of ferromagnetic substance is [2006]
3. If M is magnetic moment and B is the magnetic field,
a. > 1 b. < 1 c. zero d. 1
then the torque is given by [2011]
| M| 8. Among the following properties describing
a. M × B b. c. M ´ B d. | M| | B |
| B| diamagnetism, identify the property that is wrongly
4. At the magnetic north pole of the earth, the value of stated. [2005]
horizontal component of earth’s magnetic field and a. Diamagnetic material do not have permanent
angle of dip are, respectively [2009] magnetic moment
a. zero, maximum b. maximum, minimum b. Diamagnetism is explained in terms of
c. maximum, maximum d. minimum, minimum electromagnetic induction
c. Diamagnetic materials have a small positive
5. With a standard rectangular bar magnet the time susceptibility
period of a vibration magnetometer is 4 s. The bar d. The magnetic moment of individual electrons
magnet is cut parallel to its length into four equal neutralise each other
pieces. The time period of vibration magnetometer

Answer with Solutions


5. (c) Here,
1. (c) 2. (d) 3. (a)
Mi = 4p
4. (a) From force diagram shown in figure, the net magnetic
force is zero. In general, if similar poles are placed on the mL = 4p
corners of a regular polygon. Then, the net magnetic 4p
m=
force on a pole placed at centre is zero. L
m1 m1
The new distance between poles is
L L
F0 F0 L1 = 2r = 2 =
(–m) 2p p
m1 m1 mL 4p
F0 F0 \ Mf = mL1 = =
F0 F0 p p
= 4 Am2
m1 m1

@iitjeehelps
MAGNETOSTATICS 309
l 16. (a) As, B = m 0(H + I)
6. (b) Length of a conducting wire, 2pr = l, r = ,
2p
Þ dB = m 0 dH + m 0 dI
2p × l2 l2
Area of coil, a = pr = =
4p 2
4p
or ò HdB = m 0 ò HdH + m 0 ò H dI
So, m = IA =
Il 2 ò HdH = 0
4p
7. (d) Bair = m 0nI, Bmedium = m 0mr nI = mr B0
ò HdB = m 0 ò H × dI
Area of the B - H loop = m 0 ´ area of I - H loop
1.4 = mr ´ 6.5 ´ 10-4 i.e. A2 = m 0A 1
1.4
\ mr = = 2154 17. (a) P is in the plane S, needle ω
6.5 ´ 10-4 is in north, so the declination is N
f zero.
8. (c) Magnetic field, B= = 0.7 Wb/m2, S 11.3° ME P
A P is also on the magnetic
B 0.7 equator, so the angle of dip
m= = = 4.3 ´ 10-4 N/A2
H 1600 = 0, because the value of angle
of dip at equator is zero Q is
m = m 0(1+ c m)
also on the magnetic equator, Q
Þ cm = 340 thus the angle of dip is zero.
S
æ 4ö 4 4 As earth tilted on its axis by
9. (d) ç 2 ÷ = = 11.3°, thus the declination at Q
è x ø 4 + 12 16
is 11.3°.
4 1
= M
x 2 16 18. (c) Pole strength of the original magnet, m =
14
x = 8 mWb
Effective distance between the poles = AB
m M
10. (a) B1 = 0 30 = 62 + 82
4 pr B=0
m 0M 0 = 10 cm
B2 = B1=B0 B2=B0
4 pr 3 \ Magnetic moment of the combination
\ B = B1 - B2 = 0 M M
M ¢ = m × 2l = ´ 10 =
11. (a) Magnetic field due to magnet (1), 14 14
.
m 2M 10-7 ´ 2 ´ 2 19. (b)
B1 = 0 3 1 = . ´ 10-5 T
= 148 m Bv
4p r1 ( 0.3)3
O l S
Magnetic field due to magnet (2),
N
m æ 2M ö 10-7 ´ 2 ´ 5
B2 = 0 ç 3 2 ÷ = 2l
4p è r2 ø ( 0.4)3
= 1. 56 ´ 10-5 T m Bv 50 mg

\ Net field at P,
Taking moment about O,
B = B12 + B22 = 2.15 ´ 10-5 T Mgl = mBV L + mBV L
12. (c) Mg = 2mBV
13. (a) B = BH tan q 50
´ 981 = 2 ´ 98.1 ´ BV
m 0 2M 1000
or = 0.4 ´ 10-4 tan 60°
4p r 3 BV = 0.25 gauss
m 0M 20. (c) Torque ( t ) acting on magnet (1),
\ = 0.4 ´ 10-4 tan 60°
2p ( 0.2)3 t1 = MB sin q
0.4 ´ 4p ´ ( 0.2)3 ´ 10-4 ´ 3 Similarly, t 2 = 3 MB sin q
M =
2 ´ m0 \ q = 60°
0.00277 ´ 10-4 But for equilibrium, t1 = t 2
=
10-7 \ MB sin q = 3 MB cos q
\ M = 2.77 Am2 tan q = 3 = tan 60°
14. (a) 15. (a) \ q = 60°

@iitjeehelps
310 SELF STUDY GUIDE BITSAT

21. (a) C = MB sin q 400 400


30. (d) As, n = = = 1000
= (m ´ 2l ) ´ 2 ´ 10-5 sin 30° 2pR 40 ´ 10-2
1 m = ni = 1000 ´ 2 = 2000
= 15 ´ 10 ´ 10-2 ´ 2 ´ 10-5 ´ l l
2 1
= 1.5 ´ 10-5 Nm
2l Þ m 0 mr = = 5 ´ 10-4
L=2l 2000
22. (a) 23. (a) 24. (a) 25. (a) 26. (a) 27. (a) 5 ´ 10-4 5 ´ 10-4
Original time period Þ mr = = = 398
28. (b) In the case of vertical cutting,T1 = m0 4p ´ 10-7
n
Þ c = mr -1 = 397
T0
T1 = (Here, n = number of equal parts) 31. (c) 32. (a) 33. (a)
3
l 34. (c) Energy loss per unit volume per cycle
29. (a) For each part, l1 =
4 = area of hysteresis loop
M Energy loss per second per unit volume
and M1 =
2 = 10-2 ´ 50 J
æ I ö Also, heat produced Q in one minute per volume
\ T = 2p ç ÷
è MB ø 30
Q = 10-2 ´ 50 ´ 60 J = cal
æ I1 ö æ I ´2 ö 4.2
and T1 = 2p ç ÷ = 2p ç ÷ Let q = rise in temperature
è M1B ø è 4M ´ B ø
30
T1 1 8 ´ 103 ´ 0.11 ´ 10-3 ´ q =
\ = 4.2
T 2 30
T T q= = 8.1 °C
\ T1 = = 0 4.2 ´ 8 ´ 0.11
2 2

C Then,
1. (b) c =
T M
New magnetic moment, M ¢ =
c1 T2 1.2 ´ 10-5 T 4
Þ = Þ -5
= 2
c 2 T1 1.8 ´ 10 300 I
Moment of inertia, I¢ =
12 4
Þ T2 = ´ 300 = 200 K
18 I¢
\New time period, T ¢ = 2p
2. (d) Work done, W = MB(1 - cos q) M ¢B
q = 90° Þ New time period, T =T ¢ = 4 s
\ W = MB 6. (a) On bending a wire its pole strength remains
unchanged, whereas its magnetic moment changes.
3. (c) Torque, t = M ´ B
New magnetic moment,
4. (a) At the magnetic north pole, the magnetic needle will æ 2l ö 2M
point vertically. There is no component of earth’s M ¢ = m( 2r ) = m ç ÷ =
èpø p
magnetic field in the horizontal direction and the angle of
dip (the angle that the resultant magnetic field at the place
makes with the horizontal) is 90°. S N
⇒ S N
H = 0, d = 90° (maximum) l l′=2
I 7. (a) Susceptibility of the ferromagnetic substance is
5. (c) Time period of magnet,T = 2p
MB always greater than 1.
When magnet is cut parallel to its length into four equal 8. (c) Diamagnetic materials have negative susceptibility.
pieces. Thus, (c) is wrongly stated.

@iitjeehelps
27
Electromagnetic
Induction (EMI)

Electromagnetic Induction
Electromagnetic induction is the phenomenon due to which an induced emf is set up in a conductor or
in an electric circuit, on changing the magnetic flux linked with it.

Magnetic Flux
The total number of magnetic lines of force passing normally through an area placed in a magnetic
field is equal to the magnetic flux linked with that area.
Net flux through the surface, φ = ∫ B ⋅ dA = BAcosθ

where, φ = magnetic flux passing through the surface


B = magnetic field at the surface and θ = angle between B and dA
dA = a vector perpendicular to the surface and has a magnitude equal to the area dA
For N turns coil, φ = NBAcosθ
Magnetic flux is a scalar quantity. Its SI unit is weber (Wb) and CGS unit is maxwell or gauss × cm 2 .
1 Wb = 10 8 maxwell. It is also expressed in tesla × m 2 .
Its dimensional formula is [φ] = [ ML2T −2 A −1 ].

Faraday’s Laws of Electromagnetic Induction


Faraday had introduced two laws of electromagnetic induction which are as follows

Faraday’s First Law


This law states that, whenever the number of magnetic lines of forces (magnetic flux) passing through
a circuit changes (or a moving conductor cuts the magnetic flux), an emf is produced in the circuit
(or emf induces across the ends of the conductor) called induced emf. The induced emf persists only
as long as the flux is changing.

@iitjeehelps
312 SELF STUDY GUIDE BITSAT

Faraday’s Second Law Rotational Emf


This law states that, induced emf in the circuit is equal to Let a conducting rod of length l rotate about an axis passing
the rate of change of magnetic flux linked with the circuit through one of its ends (that end may be fixed), with an
dφ angular velocity ω in a plane perpendicular to the magnetic
i.e. e=− field B, then an induced emf is set up between the ends of the
dt
rod, whose magnitude is given by
−d φ
where, e = induced emf, = change in magnetic flux. 1
dt |e | = Bl 2ω
2
Ndφ
For N turns, e=−
dt
Self-Induction
Here, negative sign indicates that induced emf opposes the
Whenever the electric current passing through a coil or
change of flux in the circuit.
circuit changes, the magnetic flux linked with it will also
change. As a result of this, in accordance with Faraday’s
Lenz’s Law laws of electromagnetic induction, an emf is induced in the
coil or in the circuit which opposes the change that
This law gives the direction of induced emf/induced causes it.
current. According to this law, i This phenomenon is called self-induction and the emf
the direction of induced emf or induced is called back emf, current so produced in the coil
induced current in a circuit is is called induced current.
N S
such as to oppose the cause If no magnetic materials are present near the coil, then
that produced it. This law is i
number of flux linkages with the coil is proportional to the
another form of law of
current i, i.e. Nφ ∝ i or Nφ = L i (N is the number of turns in
conservation of energy.

e.g. If a magnet approaches a coil as shown, then an coil and Nφ = total flux linkage), where L = , coefficient of
i
anti-clockwise current is induced in the coil and the coil self-induction. If i = 1 A , N = 1 then, L = φ i.e. the coefficient
opposes approaching magnet. So, some work has to be of self-induction of a coil is equal to the flux linked with the
done to move magnet towards the coil and this work coil when the current in it is 1 A and number of turns is one.
appears in the form of electrical energy. dφ
By Faraday’s second law, induced emf, e = − N , which
dt
Motional Emf gives e = − L
di
dt
Let a conducting rod of length l be Then, the voltage (V ) induced across the inductor,
X X + X X
moving with a uniform velocity v
di
perpendicular to a uniform l X X v X X V = |e | = ×L
magnetic field B, an induced emf dt
X X – X X
is set up. The magnitude of the weber
Its SI unit is .
induced emf will be |e | = B lv. amp
If the rod is moving such that it makes an angle θ with the But its practical unit is henry (H). Its dimensional formula
direction of the magnetic field, then | e | = B lv sin θ. Hence, is [L] = [ML2T −2 A −2 ].
for the motion parallel to B, the induced emf is zero.
When a conducting rod moves horizontally, then an Inductance of a Solenoid
induced emf is set up between its ends due to the vertical
Let us find the inductance of a uniformly wound solenoid
component of the earth’s magnetic field. However, at the
having N turns and length l. Assume that l is much longer
magnetic equator, induced emf will be zero because BV = 0.
than the radius of the windings and that the core of the
solenoid is air. We can assume that the interior magnetic
Motional Emf in a Loop field due to a current i is uniform and given by equation,
If a conducting rod moves on two parallel conducting rails N
B = µ 0ni = µ 0   i
then an emf is induced whose magnitude is |e | = B lv and  l
the direction is given by the Fleming’s right hand rule. N
|e | B lv where, n = is the number of turns per unit length.
Induced current, | I | = = l
R R

@iitjeehelps
ELECTROMAGNETIC INDUCTION (EMI) 313
The magnetic flux through each turn is, In case of mutual inductance for two coils situated close to
NS each other, total flux linked with the secondary coil due to
φB = BS = µ 0 i current in the primary coil is
l G
N 2 φ2
Here, S is the cross-sectional area of the solenoid. Now,
and N 2 φ 2 ∝ i1 ⇒ N 2 φ2 = Mi1 coil
NφB N  µ 0 NSi  µ 0 N S
2
L= =   = where, N 1 is the number of turns
i i  l  l
in primary coil; N 2 is the number
µ 0N 2S of turns in secondary coil; φ2 is
∴ L=
l the flux linked with each turn of
secondary coil; i1 is the current flowing through primary
This result shows that L depends on dimensions ( S ,l ) and is
coil; M is the coefficient of mutual induction or mutual
proportional to the square of the number of turns.
inductance.
L ∝N2 dφ di
Because N = nl, we can also express the result in the form, Emf induced in secondary coil, e 2 = − N 2 2 ; e 2 = − M 1 ;
dt dt
(nl )2 di1
L = µ0 S If = 1 A/s, then |e 2 | = M . Hence, coefficient of mutual
l dt
= µ 0n 2 Sl = µ 0n 2V or L = µ 0n 2V induction is equal to the emf induced in the secondary coil
when rate of change of current in primary coil is unity.
Here, V = Sl is the volume of the solenoid.

Mutual Inductance between Two Coils


Energy Stored in an Inductor
(i) If two coils (1 and 2) also called primary and
The energy of a capacitor is stored in the form of electric secondary coils are placed close to each other
field between its plates. Similarly, an inductor has the (maximum coupling), then mutual inductance
capability of storing energy in the form of magnetic field. between them
i (increasing) BN A
M= 1 2 2
i1

di where, N 1 and N 2 = number of turns in primary and


|e| = L
dt secondary coils respectively, φ2 = flux linked with
di
|e| = L each turn of secondary coil, N 2 φ2 = total flux linkage
dt
with secondary coils, M = mutual inductance
An increasing current in an inductor causes an emf between between two coils.
its terminals. (ii) If the coils are two concentric i1
The work done per unit time is power. coplaner circular coils, then mutual R
dW di inductance between them P
P= = – ei = – Li r
dt dt πµ 0 N 1 N 2 r 2 S
M=
dW dU 2R
From, dW = – dU or =– r2
dt dt ⇒ M∝
dU di R
we have, = Li or dU = Li di (iii) For two solenoids tightly wound to each other as
dt dt
shown.
The total energy U supplied while the current increases Secondary
from zero to a final value i is,
i 1 1
U = L ∫ idi = Li 2 ∴ U = Li 2
0 2 2
Primary

Magnetic field inside the primary solenoid,


Mutual Induction B 1 = µ 0n 1 i1

n 1 =
N1

Whenever the current passing through a coil or circuit  l 
changes, the magnetic flux linked with a neighbouring coil µ 0N 1N 2A
Thus, M=
or circuit will also change. Hence, an emf will be induced in l
the neighbouring coil or circuit. where, A = area of each solenoid,
This phenomenon is called mutual induction. l = length of each solenoid

@iitjeehelps
314 SELF STUDY GUIDE BITSAT

Combination of Inductors (a) When the switch is connected to position b, the rise
If two coils of self-inductances L 1 and L 2 are placed quite of current in R-L circuit takes place.
far apart and are arranged in series, then their equivalent The growth of the current in the circuit is given by
inductance
i = i0 (1 − e −t / τ )
L s = L1 + L 2
E L
However, if the coils are placed quite close to each other, so where, i0 = and τ = = time constant of the circuit.
as to mutually affect each other, then R R
E  R 
L s = L1 + L 2 ± 2 M ∴ i = 1 − exp.  − t  
R  L 
Here, M has been written with ± sign depending on the fact
whether currents in the two coils are flowing in same sense In case of growth of current, when t = λ, then
or opposite sense. i = 0632
. i0
If two coils of self-inductances L1 and L 2 are connected in (b) When the switch S is connected to position a, decay
parallel, then equivalent inductance L p is given by of the current in R-L circuit takes place. The current
1 1 1 L1 L 2 in this case is given by
= + ⇒ Lp =
L p L1 L 2 L1 + L 2  t  t
i = i0 exp.  −  = i0exp.  − 
 τ  τ
In a parallel arrangement, effect of mutual inductance may
be omitted because value of k in this arrangement is quite In case of decay of current, when t = λ, then
small. i = 0.368 i0
The growth and decay of the current is shown in
Growth and Decay of Current figure.

in an Inductance Resistance i0
(0.63 I0)
Circuit Growth
of current
Consider a circuit containing a Decay
resistance R and inductance L, R L S a of current
Current (i)
a two-way key and a battery of
b (0.37 I0)
emf E connected in series as E Time (t)
shown in figure. We consider
the following cases

Practice Exercise
1. The magnetic flux φ (in weber) in a closed circuit of 3. Three resistances of magnitude R each are connected
resistance 10 Ω varies with time t (in second) in the form of an equilateral triangle of side a. The
according to equation φ = 6t 2 − 5t + 1. The magnitude combination is placed in a magnetic field B = B 0e − λt
perpendicular to the plane. The induced current in the
of induced current at t = 0.25 s
circuit is given by
a. 1.2 A b. 0.8 A c. 0.6 A d. 0.2 A
× A ×
2. A metallic circular loop of radius r is placed in uniform
magnetic field B acting perpendicular to the plane of × ×
the loop. A naughty boy pulls dimetrically opposite
R

corner so that after sometime, the loop changes into × ×


an ellipse of major and minor radius a and b. If total × ×
resistance of loop is R and it remains constant during B
R
C
the pulling. Find average charge flowing through loop × ×
during pulling.
 a 2λ   a 2λ 
B( πab ) B( πab − πr 2 ) a.  B0 e − λt b.  B0 e − λt
a. b.  2 3R   4( 3 )R 
R R
Bπr 2 Bπbr  a 2B0  − λt  a 2B0R  − λt
c. d. c.  e d.  e
R R  λ 4 3R   λ4 3 

@iitjeehelps
ELECTROMAGNETIC INDUCTION (EMI) 315
4. A helicopter rises vertically with a speed of
10 ms −1. If helicopter has a length of 10 m and 11. In the figure shown, a coil of
the horizontal component of the earth’s magnetic field single turn is wound on a
is 1.5 × 10−3 Wbm −2, the emf induced between the tip sphere of radius r and mass
of the nose and the tail of the helicopter, is m. The plane of the coil is O
a. 0.15 V b. 125 V c. 130 V d. 5 V parallel to the inclined plane B mg
θ
5. An air-cored solenoid with length 30 cm, area of and lies in the equatorial
cross-section 25 cm 2 and number of turns 500, carries plane of the sphere. If sphere is in rotational
a current of 2.5 A. The current is suddenly switched equilibrium, the value of B is (current in the coil is I)
off in a brief time of 10−3 s. How much is the average mg mg sin θ
back emf induced across the ends of the open switch a. b.
πIr πl
in the circuit? Ignore the variation in magnetic field mgr sin θ
near the ends of the solenoid. c. d. None of these
πl
a. 6.5 V b. 7.4 V c. 8.2 V d. 9.3 V
12. The loop ABCD is moving with velocity ‘v ’ towards
6. A fan blade of length 2a rotates with frequency f cycle right. The magnetic field is 4 T. The loop is connected
per second perpendicular to magnetic field B. Then, to a resistance of 8Ω. If steady current of 2 A flows in
potential difference between centre and end of the loop then value of v if loop has a resistance of 4 Ω,
blade is (Given, AB = 30 cm, AD = 30 cm) is
a. πBa 2f b. 4πBaf c. 4πa 2Bf d. 2πaBf
x x x x x x C
D
7. A metal rod AB of length l A O B x x x x x x v
is rotated with a constant x x x x x x 8Ω
angular velocity ω about I/4 3I/4 x x x37°x x x
an axis passing through ‘ B
O’ and normal to its length. Potential difference x x x x x x
between ends of rod in absence of external magnetic 50
field (where, e = electric charge) a. m/s b. 20 m/s
3
mω 2l 2 mω 2l 2 mω 2l 2 100
a. zero b. c. d. c. 10 m/s d. m/s
4e 2e 8e 3
8. A wire is sliding as shown in the figure. The angle 13. Calculate the self-inductance of the air cored solenoid
between the acceleration and velocity of the wire is of length 80 cm and has 500 turns and its circular
cross-section has diameter of 2 cm.
a. 150.6 µH b. 162.2 µH
c. 123.3 µH d. 102.5 µH
B
v d 14. The inductance per unit length of a double tape line as
30° shown in the figure.

h
a. 30° b. 40°
c. 120° d. 90°
d
9. A fan blade of length 1/ π metre rotates with I
frequency 5 cycle per second perpendicular to a b
magnetic field 10 tesla. I
What is potential difference between the centre and
the end of blade? µ0 h b
a. b.
a. − 50 V b. + 50 V b µ0 h
c. − 2.0 V d. + 0.02 V µ0b hb
c. d.
10. A coil has an area of 0.05 m 2 and has 800 turns. After h µ0
placing the coil in a magnetic field of strength 15. What is the mutual inductance of coil and solenoid if a
4 × 10−5 Wbm −2, perpendicular to the field, the coil is solenoid of length 0.50 m and with 5000 turns of wire
rotated by 90° in 0.1 s. The value of average emf has a radius 4 cm and a coil of 700 turns is wound on
induced is the middle part of the solenoid?
a. zero b. 0.016 V a. 44.17 mH b. 48.98 mH
c. 0.01 V d. 0.032 V c. 34.34 mH d. 36.73 mH

@iitjeehelps
316 SELF STUDY GUIDE BITSAT

16. When the current changes from + 2 A to − 2 A in 22. The time constant for the given circuit is
0.05 s, an emf of 8 V is induced in a coil. The
coefficient of self-induction of the coil is 6Ω 2H
a. 0.1 H b. 0.2 H
c. 0.4 H d. 0.8 H
12 Ω 4Ω
17. A closed circuit consists of a source of emf E and an
inductor coil of inductance L, connected in series. The
active resistance of whole circuit is R. At the moment
t = 0, inductance of coil abruptly decreased to L / n. a. 4 s b. 1/4 s c. 2 s d. 1/2 s
Then, current in the circuit immediately after, is 23. With usual notations, the energy dissipation in an
a. zero b. E / R ideal inductor is given by
nE E a. LI
c. d. 1
R nR b. LI
2
18. Three pure inductors each of 2H are connected as 1
shown in the figure. The equivalent inductance of the c. LI 2
2
circuit is
d. None of the above
24. A non-conducting ring of radius r has charge per unit
2H
length λ. A magnetic field perpendicular to plane of
2H the ring changes at rate dB / dt . Torque experienced
2H
by the ring is

a. 8 H/6 b. 6 H ×
×× ×
c. 2 H d. None of these ×
× × ×
19. The sum and the difference of self-inductances of two ×
R = 200 cm ×
coils are 13 H and 5 H respectively. The maximum
× 10 cm× ×
mutual inductances of two coils is ×
× × ×
a. 6 H b. 5 H
× × ×
c. 65 H d. 18 H × ×
×
20. In the figure, the steady state current through the
dB dB
inductor will be a. λπr 3 b. λ 2πr 3
dt dt
4Ω dB
c. λ2( 2πr )2r d. zero
dt
1Ω
5V 3 mH 25. Figure shows a uniform magnetic field B confined to a
cylindrical volume of radius R. If B is increasing at
4 µF
constant rate of 0.01 T/s. Instantaneous acceleration
experienced by electron at r = 10 cm (< R) as shown
a. zero in the figure.
b. 1 A a. 8.79 × 10−12 m/ s2
c. 1.25 A
b. 8.79 × 10−11 m/ s2
d. Cannot be determined
c. 8.79 × 10−10 m/ s2
21. Determine the value of time constant for the given
circuit. d. 8.79 × 10−9 m/ s2

R1
26. Find the energy stored in the magnetic field if current
of 5 A produces a magnetic flux of 2 × 10−3 Wb
through a coil of 500 turns.
L R2 a. 2.5 J b. 0.25 J
E, r c. 250 J d. 1.5 J
27. The inductance of a coil in which a current of 0.1 A
increasing at the rate of 0.5 A/s represents a power
L L 1
a. b. flow of watt, is
R1 + r + R2 (R1 + r ) 2
L(R1 + R2 + r ) a. 2 H b. 8 H
c. d. None of these
(R1 + r )R2 c. 20 H d. 10 H

@iitjeehelps
BITSAT Archives
1. Lenz’s law of electromagnetic induction corresponds 4. If emf induced in a coil is 2 V by changing the current
to the [2011] in it from 8 A to 6 A in 2 × 10 −3 s. Then, the coeffcient
a. law of conservation of charge of self-induction is [2006]
b. law of conservation of energy a. 2 × 10−3 H b. 10−3 H
c. law of conservation of momentum
d. law of conservation of angular momentum c. 0.5 × 10−3 H d. 4 × 10−3 H

2. A toroidal solenoid with an air core has an average 5. If in a triode valve amplification factor is 20 and plate
radius of 15 cm, area of cross-section 12 cm 2 and resistance is 10 kΩ, then its mutual conductance is
1200 turns. Ignoring the field variation across the [2006]
cross-section of the toroid the self-inductance of the a. 2 milli mho
toroid is [2011] b. 20 milli mho
a. 4.6 mH b. 6.9 mH c. (1/2) milli mho
c. 2.3 mH d. 9.2 mH d. 200 milli mho
3. A coil of inductance L is carrying a steady current I.
What is the nature of its stored energy? [2011]
6. The induction coil works on the principle of [2005]
a. Magnetic a. self-nduction
b. Electrical b. mutual induction
c. Both magnetic and electrical c. Ampere's rule
d. Heat d. Fleming's right hand rule

Answer with Solutions


Practice Exercise e = 500 × 25 × 10−4 × 4 × 3.14 × 10−7×
500
−2
2.5
× −3
30 × 10 10
1. (d) 2. (b) = 6.5 V
− λt 3 2
3. (b) φ = BA = B0e a 6. (a)
4
dφ mω 2x
e=− =−
3 2 d − λt
a B0 (e ) 7. (b) eV = mω 2x ⇒V =
dt 4 dt e
3l / 4
mω 2 mω 2 mω 2l 2
=−
3 2
a B0( − λ )e − λt
=
3 2
a B0λe − λt dV =
e
dx ⇒ V = ∫
e 1/ 4
x dx =
4e
4 4
e 3 2  a 2λB0  − λt 8. (c) The magnetic force on the
∴ I= = a B0λe − λt =  e F
3R 12R  4 3R  wire is perpendicular to wire.
(shown in the figure)
4. (a) 30°
∴ Angle = 30° + 90° = 120°
−2
5. (a) Given, length of solenoid l = 30 cm = 30 × 10 m VD a v0
Area of cross-section A = 25 cm2 = 25 × 10−4 m2 9. (a) ∫ de = − ∫ B xω dx ω
V0 0
Number of turns, N = 500
1
Current, I1 = 21.5 A, I 2 = 0 VD − V0 = − Ba 2ω
2
Brief time d t = 10−3 s
1 1 x
Induced emf in the solenoid = − × 10 × 5 × 2π × = − 50 V
2 π
dφ d
e= = (BA) [Q φ = BA] 10. (b)
dt dt
Magnetic field induction B at a point well inside the long 11. (a) Torque due to magnetic force = torque due to
solenoid carrying current I is gravitational force
 N
MB sin (180 − θ ) = mgr sin θ
B = µ 0nI  where, n = number of turns per unit length =  or
 I
or MB sin θ = mgr sin θ
dB d  N  µ N dl mgr mgr mg
e = NA =A µ 0 I  = A 0 ⋅ ∴ B= = =
dt dt  l  l dt M Iπ r 2 Iπ r

@iitjeehelps
318 SELF STUDY GUIDE BITSAT

12. (d) e = BAD sin 37°v = 4 × 0.3 sin 37°v Adding Eqs. (i) and (ii), we get
4 × 0.3 × 0.6v 24 × 10 100 2L1 = 18 ∴ L1 = 9 H
I= =2 ⇒ v = = m/s ∴ L2 = L1 − 5 = 9 − 5 = 4 H ∴ M = K L1L2
( 4 + 8) 4 × 0.6 × 3 3
∴ M max = K max 9 × 4 = 1 × 36 = 6 H [QK max = 1]
13. (c)
20. (c)
14. (a) Neglecting end effects of magnetic field, we have (R + r )R2 L L(R1 + R2 + r )
µ I 21. (c) Here, Req = 1 ∴τ = =
B= 0 R1 + r + R2 R (R1 + r )R2
b
22. (b) 6Ω
Flux φ per unit length of the plates is
µ 0I µ hI 2H
×h ×l = 0 12 Ω 4Ω
b b
µ 0h
Also, φ = L I ⇒ L =
b


15. (a) Flux linked with coil L
6Ω
µ N N πr 2I
φ 2 = N2BlA = 0 1 2 also φ = MI ⇒ 8Ω
l 12 Ω 2H

Comparing the equations,


4Ω
µ N N πr 2 L 2 1
M = 0 1 2 τ= = = s
l R 8 4
4 × 3.14 × 10−7 × 5000 × 700 × 3.14 × ( 4 × 10−2 )2 23. (d)
M = dφ dB dB
0 .5 24. (a) e = = πr 2 ⇒ E( 2πr ) = πr 2
M = 44.17 mH dt dt dt
∆I 1 dB
16. (a) e = L E= r ⇒ dF = λdSE
∆t 2 dt
e 8 8 × 0.05 ⇒ dτ = rλdSE ⇒ τ = λrE ∫ dS = λrE 2πr
∴ L= = = = 0.1H
∆I 4 4
1 dB dB
∆t 0.05 = 2 πr 2 λ r = πr 3 λ
17. (c) Due to change in inductance, magnetic flux remains 2 dt dt
constant. 2 dB 1 dB
25. (b) E( 2πr ) = πr ⇒ E= r
But, φ = LI = constant dt 2 dt
L1I1 = L2I 2 e  dB  1
r   16 . × 10−19 × × 0.1 × 0.01
L1 E LI nE eE 2  dt  2
Here, L2 = and I1 = ∴ I2 = 1 1 = a= = =
n R L2 R me me 9.1 × 10−3
18. (a) = 8.79 × 10−11 m/s2
19. (a) L1 + L2 = 13 26. (a) 27. (d)
L1 − L2 = 5

BITSAT Archives
1. (b) Lenz’s law of electromagnetic induction corresponds 4. (a) Induced emf, e = 2 V
to the law of conservation of energy. i1 = 8 A, i2 = 6 A ⇒ ∆t = 2 × 10−3
2. (c) For a solenoid,
B = µ 0nI Coefficient of self-induction
e −2 −2 × 2 × 10−3
n=
N L= = −3
= = 2 × 10−3 H
Q
2πr ∆i / ∆t ( 6 − 8) / 2 × 10 −2
µ NI 5. (a) Amplification factor µ = 20
∴ B= 0
2 πr Plate resistance Rp = 10 k Ω = 10 × 10 −3 Ω
Flux linked with the solenoid µ
φ = NBA ∴Mutual conductance gm =
Rp
µ N 2IA 4π × 10−7 × (1200)2 × 12 × 10−4
φ= 0 = 20
2 πr 2π × 15 × 10−2 =
10 × 103
L = 2.3 × 10−3 H
= 2 × 10−3 mho or 2 milli mho
= 2.3 mH
3. (a) Energy is stored in an inductor in the form of magnetic 6. (b) The induction coil works on the principle of mutal
potential energy. induction.

@iitjeehelps
28
Alternating Current
and EM Wave

Alternating Current
An alternating current is the current (or voltage) whose
magnitude keeps on changing continuously with time, between V I
zero and a maximum value and the direction also reverses I0
periodically. V0 t
O
Alternating current or voltage varying as sine function can be T T 3T T
4 2 4
written as

i = i0 sin ωt = i0 sin 2 πνt = i0 sin t
T

and V = V0 sin ωt = V0 sin 2 πνt = V0 sin t
T
where, i and V = instantaneous values of current and voltage,
i0 and V0 = peak values of current and voltage respectively.
ω = angular frequency in rad/s, ν = Frequency (in Hz) and T = time period.
2 2
Mean Square Value (V or i )
The average of square of instantaneous values in one cycle, is called mean square value. It is always
positive for one complete cycle.
2 1 T V02 i2
e.g. V =
T ∫0 V 2dt =
2
or i 2 = 0
2

Root Mean Square (rms) Value


Root mean square of voltage or current in an AC circuit for one complete cycle is called rms value. It is
denoted by V rms or irms .

@iitjeehelps
320 SELF STUDY GUIDE BITSAT

T 2 pure inductance does not oppose the flow of DC (ω = 0) but


irms
i 2 + i22 + K
= 1 = i2 =
∫0 i dt =
i0 opposes the flow of AC.
T
n
∫0 dt 2
O π
V
2
= 0. 707i0 = 70. 7% of i0 (a)
L
(b)
Similarly,V rms = V0 / 2 = 0. 707 V0 = 70. 7% ofV0
V = V0 sin ωt
The rms value of alternating current is also called virtual I
value or effective value. V XL

Mean or Average Value (iav orVav ) (c) (d)


The average of instantaneous values of current or voltage π
in one cycle is called its mean value. The average value of 2
O I O V
alternating quantity for one complete cycle is zero.
The average value of AC over half cycle (t = 0 to T /2 ) is V
T/2
Current flowing, I =

iav =
∫0 idt =
2i0
= 0.637i0 = 63 . 7% of i0
XL
T/2
π The current decreases with an increase in frequency.
∫0 dt π
The current lags behind the voltage by (or voltage leads
2V0 2
Similarly, V av = = 0.637 V0 = 63 . 7% of V0
π π
the current by ) and is thus given by
2
π
AC Circuit Elements 
I = I0 sin ωt − 
 2
The circuits consist of resistor, capacitor and inductor are
callled pure resistive, pure inductive and pure capacitive
circuits. These circuits with their respective phasor 3. Pure Capacitive Circuit
diagram are given below. Let an alternating voltage V = V0 sinωt be applied across a
pure capacitance C. Then,
1. Pure Resistive Circuit I
Let an alternating voltage V = V0 sinωt be applied across a
pure resistance R. Then, (a) C (b)
π
2
Irms

R I V = V0 sin ωt V
O V XC
I
π
2
V = V0 sin ωt (c) (d)
V
(a) (b) (c)
V V V V
Current, I = or I rms = rms
R R
The capacitance offers some opposition to the flow of DC
Current and voltage are in the same phase, i.e. current is
current but allows AC to pass through it. The opposition
given by I = I0 sinωt .
offered is known as the capacitive reactance.
1
2. Pure Inductive Circuit XC = Ω

Let an alternating voltage V = V0 sinωt be applied across a 1
pure inductance L. Then, the average power = Ω
C × 2 πν
π
= V rms I rms cos = 0 V
2 Current flowing, I =
XC
Such a current, for which average power as well as power
factor is zero, is called as wattless current. For DC circuit, frequency ν = 0 and hence, X L = 0 but
The inductance offers some opposition to the flow of AC. It XC = ∞
is known as inductive reactance X L = 2πνL = Lω. Thus, a The current increases with an increase in frequency.

@iitjeehelps
ALTERNATING CURRENT AND EM WAVE 321
π
The current leads the voltage by (or voltage lags behind 3. Series L-C Circuit
2
π  π E0
the current by ) and is thus given by I = I0 sin ωt +  For E = E0 sinωt , I = sin(ωt − φ )
2  2 Z
where, Z = X L − XC
Out of the three elements in an AC circuit, the resistor is
called the active element because it always opposes the X − XC
and tan φ = L =∞
flow of current under all conditions. 0

L C VL
Series AC Circuits VL – VC
Some of the series AC circuits are given below
I
1. Series L-R Circuit E = E0 sin ωt
VC
The potential difference across a resistance in AC is in
phase with current and it leads in phase by 90° with current
π π
across the inductor. For X L > XC , φ = and for X L < XC , φ = −
L R 2 2
For E = E0 sinωt
1
E VL VR If X L = XC i.e. at ω = , Z = 0 and I0 becomes infinity.
I = 0 sin(ωt − φ ) LC
Z
This condition is termed as the resonant condition and this
where, Z = R2 + (ωL )2 frequency is termed as natural frequency of the circuit.
E = E0 sin ωt
ωL I Z
and tan φ =
R
V = V R2 + V L2
VL V
where,V R = voltage across resistor R.
φ ω0 ω ω0 ω
V L = voltage across inductor L. VR I Variation of I with Variation of Z with
ω in L-C circuit ω in L-C circuit
Current lags behind the voltage by φ.

2. Series R-C Circuit 4. L-C-R Series Circuit


Potential difference across a capacitor in AC lags in phase As shown in the figure, inductance L, capacitance C and
by 90° with the current in the circuit. resistance R are connected together in series and supplied
with an alternating voltage. In such an arrangement, the
For E = E0 sinωt , same circuit current flows through all the components of the
E0 circuit and V R , V L and VC indicate the voltages across the
I= sin (ωt + φ)
Z resistor, inductor and capacitor. Fig. (b) shows the circuit
C conditions when the inductive reactance ( X L ) is greater than
R
VR the capacitive reactance ( XC ).
I
VC VR φ In this case, since both L and C carry the same current and
V X L is greater than XC , it follows thatV L must be greater than
VC VC , (V L = I S X L and VC = I S XC )
E = E0 sin ωt Remember thatVC andV L are in
IS
anti-phase to each other due to
2
 1  their 90° leading and lagging C VC
where, Z = R2 +  
 ωC  relationship with the circuit
L
current I S . AsV L andVC directly VL
−1 / ωC VS
and tan φ = oppose each other, a resulting
R voltage is created, which will be R
Current leads the voltage by φ. the difference between VC and VR
V L . This is called the reactive
V 2 = V R2 + VC2 ⇒ V = V R2 + VC2 voltage. L-C-R series circuit

@iitjeehelps
322 SELF STUDY GUIDE BITSAT

1
VL ⇒ ω0 = rad/s
VL LC
VL – VC VS 1
φ ⇒ ν0 = Hz (or cps)
VR IS 2 π LC
VR IS
Resonant frequency doesn’t depend
upon the resistance of the circuit. Pmax
VC VC Half power frequencies and band
P
(a) Phasor diagram for (b) Phasor diagram for width The frequencies at which the Pmax
P=
V L and V C R-L-C circuit power in the circuit is half of the 2
maximum power (the power at ω1 ω0 ω2 ν
Phasors for V L and VC are in anti-phase. So, the circuit in resonance), are called half power Resonance curve for
which V L is greater than VC , behaves like an inductive frequencies. R-L-C circuit
circuit. (i) The current in the circuit at
The phasors for (V L − VC ), V R andV S in the figure form right 1
Half Power Frequencies (HPF) is or 0.707 or
angled triangle, a number of properties and values in the 2
circuit can be calculated using Pythagoras theorem or 70.7% of maximum current (current at resonance).
some basic trigonometry. Like
(ii) There are two half power frequencies, which are
V S2 = (V L − VC )2 + V R2 or V S = (V L − VC )2 + V R2 related as
Total circuit impedance ( Z ), Z = ( X L − XC )2 + R2 ω0 = ω 1 ω 2

The phase angle between (V L − VC ) and V R can be found Quality Factor


using trigonometry E0
(V − VC ) (V − VC ) At resonance, I0 =
∴ tan φ = L ⇒ φ = tan −1 L R
VR VR
and V L = I0 X L = VC = I0 XC
V ωL 1
Also, Ohm’s law states that R (or X ) = i.e. VL = × E0 = × E0 = QE0
I R ωRC
If (V L − VC ) and V R are each divided by the current I S . This ωL
where, Q=
allows the phase angle θ to be found using the resistances R
and reactances.
1
 X − XC  or is termed as the Quality factor of the circuit. It
φ = tan −1  L  ωRC
 R 
determines the sharpness of resonance. Higher the value of
Q, sharper is the resonance.
At Resonance (Series Resonant Circuit)
(i) X L = XC ⇒ Zmin = R, i.e. circuit behaves as resistive
circuit. Parallel Resonant Circuit
(ii) V L = VC ⇒ V = V R , i.e. whole applied voltage Figure shows a parallel R L
appeared across the resistance. resonant circuit in which
resistor R and inductor L iC
(iii) Phase difference, φ = 0° ⇒ Power factor = cos φ = 1 have been connected in
C
i
1 series and this combination is
(iv) Power consumption, P = V rms irms = V0i0 connected in parallel with the
2
capacitor C. V =V0 sin ωt
(v) Current amplitude in the circuit is maximum and it is
V To this combination, an
given by i0 = 0 alternating source of V = V0 sinωt is applied.
R
From the figure,
(vi) This circuit is used for voltage amplification and as
i = iL + iC
selector circuits in wireless telegraphy.
V V V
or = +
Resonant Frequency (Natural Frequency) Z R + jωL − j/ωC
1 V ωCV
At resonance X L = XC ⇒ ω0 L = = −
ω0C R + j ωL j

@iitjeehelps
ALTERNATING CURRENT AND EM WAVE 323
V The termV rms I rms is known as the apparent or virtual
= + j(ωC )V
R + j ωL power butV rms I rms cos φ is called the true power.
1 1
∴ = + j ωC (iii) The term cos φ is known as the power factor of the
Z R + j ωL R
given circuit. Thus, cos φ = = power factor
1 Z
is known as admittance (Y ). Thus,
Z true power
=
1 R − j ωL apparent power
Y = = 2 + j ωC
Z R + ω 2 L2 (iv) For a pure resistive circuit, V and I are in phase
R + (ωCR + ω L C − ωL )
2 2 3 2 2 (φ = 0°) hence, cos φ = 1 and average power = V rms I rms
∴ Y = For a pure inductive or a pure capacitive circuit,
R2 + ω 2 L2 π π
current and voltage differ in phase by  i.e. φ =  .
The admittance will be minimum when 2 2
ωCR2 + ω3 L2C − ωL = 0

1 R2
Wattless Current
or ω= − 2 Average power is given by Pav = E rms I rms cos φ
LC L
The phase difference between E rms and I rms is φ. We can
ω 1 1 R2
∴ f = = − 2 resolve I rms into two components
2 π 2 π LC L
I rms cos φ and I rms sin φ
is known as resonance frequency. At resonance frequency Here, the component I rms cos φ contributes towards power
admittance is minimum and impedance is maximum. dissipation and the component I rms sin φ does not
Thus, the parallel circuit does not allow this frequency from contribute towards power dissipation. Therefore, it is called
the source to pass in the circuit. Due to this reason, the wattless current.
circuit with such a frequency is known as rejector circuit.
We have dynamic resistance Choke Coil
1 L
Z max = = Let us consider a choke coil (used in tube lights) of large
Ymax CR
inductance, L and low resistance R. The power factor for
V0 V CR such a coil is given by
∴ Peak current through the supply = = 0
L/CR L R R
cos φ = ≈ (as R << ωL)
V0 R +ω L
2 2 2 ω L
The peak current through capacitor = = V0ωC
1/ωC
As R << ωL, cos φ is very small. Thus, the power absorbed by
V0ωC ωL the coil V rms i rms cos φ is very small. On account of its large
Hence, Q-factor = =
V0CR/L R
impedance Z = R2 + ω 2 L2 , the current passing through
This is basically the measure of current magnification. the coil is very small. Such a coil is used in AC circuits for
the purpose of adjusting current to any required value
without waste of energy. The only loss of energy is due to
Power in an AC Circuit hysteresis in the iron core, which is much less than the loss
Let a voltage V = V0 sinωt be applied across an AC and of energy in the resistance that can also reduce the current
consequently a current I = I0 sin(ωt − φ ) flows through the if placed instead of the choke coil.
circuit. Then,
(i) Instantaneous power = V I
= V0 I0 sin ωt sin(ωt − φ ), and its value varies with
AC Generator
time. An electrical machine used to convert mechanical energy
Here, φ is known as phase difference betweenV and I. into electrical energy is known as AC generator/alternator.
It works on the principle of electromagnetic induction, i.e.
(ii) Average power over a full cycle of AC is
1 when a coil is rotated in uniform magnetic field, an induced
Pav = V rms I rms cos φ = V0 I0 cos φ emf is produced in it.
2

@iitjeehelps
324 SELF STUDY GUIDE BITSAT

ω
B C Transformer is of two types
Step-up transformer Step-down transformer
This transformer is used to increases This transformer is used to decreases
voltage and decreases current. voltage and increases current.
t
VS > VP VS < VP
N A D S
NS > NP NS < NP
B1
ES > EP ES < EP
R1
iS < iP P S iS > iP P S
RL Output
RS > RP RS < RP

R2 tS > tP tS < tP
B2
k >1 k <1
AC generator
Ndφ where, N P and N S = Number of turns in primary and
Induced emf, = NBAω sin ωt = e0 sinωt
e=−
dt secondary coil
where, e0 = NBAω V P and V S = Applied (input) voltage to
e e primary and secondary (load
Current in the coil, i = = 0 sinωt = i0 sinωt
R R voltage) coil
where, R = Resistance of the circuit. e P ande S = Induced emf in primary and
secondary coil
θ = 0º θ = 90º θ = 180º θ = 270º θ = 360º
φ = Flux linked with primary as well
B C B C B C B as secondary coil
C B C B iP andiS = Current in primary and
S secondary coil (or load current)
N
A A RP and RS = Resistance of primary and
D A D A D D secondary coil
D e=0 e =e 0 A e = 0 e = –e 0 e = 0
(1) (2) (3) (4) (5) t P and t S = Thickness of turn in primary and
+e 0 secondary coil
As in an ideal transformer, there is no loss of power i.e.
Pout = Pin and e = V
0º 90º 180º 270º 360º So, V S iS = V PiP and V P ≈ e P , V S ≈ e S
θ
According to Faraday’s law,
–e 0
dφ dφ
eS = − N S , eP = − N P
Output of a single loop of AC generator for one cycle of rotation dt dt
e N V i
Hence, S = S = S = P = k ;
e P N P V P iS
Transformer where, k = transformation ratio
It is a device which raises or lowers the voltage in AC
Efficiency of transformer (η) Efficiency is defined as the
circuits through mutual induction. It works on the basic
principle of mutual induction i.e. The alternating current ratio of output power and input power
passing through the primary coil creates a continuously P
( η%) = out × 100
changing flux through the core. This changing flux induces i.e. Pin
an alternating emf in the secondary coil. VS IS
Laminated sheets = × 100
V P iP

Input
Electromagnetic Waves
P S Load Output Electromagnetic waves are those waves in which electric
AC
Source
and magnetic fields vary sinusoidally in space and with
time. The electric and magnetic fields are mutually
Iron core
perpendicular to each other and each field is perpendicular
Laminated core to the direction of propagation of the wave.

@iitjeehelps
ALTERNATING CURRENT AND EM WAVE 325

Characteristics of EM Waves 2. Gauss’s law for magnetism


(i) This law shows that the number of magnetic lines of
Maxwell’s theory predicted that electromagnetic waves of
force entering a closed surface is equal to the
all frequencies (and hence all wavelengths) propagate in
1 number of magnetic lines of force leaving that closed
vacuum, with a speed given by c = surface.
µ 0ε0
(ii) This law tells that the magnetic lines of force form a
where, µ 0 is the magnetic permeability and ε0 is the electric continuous closed path.
permittivity of vacuum. Now, for the vacuum, (iii) This law also predicts that the isolated magnetic
µ 0 = 4π × 10−7 TmA −1 and ε0 = 8.85 × 10−12 C 2N −1m −2 . monopole does not exist.
Substituting these values in the above relation, we have Mathematically, ∫ B ⋅ dS = 0
1 S
c= ~ 3.0 × 108 ms−1
[( 4π × 10 )(8.85 × 10−12 )]1 / 2
−7 3. Faraday’s law of electromagnetic induction
(i) This law gives a relation between electric field and a
changing magnetic flux.
Maxwell’s Displacement (ii) This law tells that the changing magnetic field is the
Current source of electric field.

If the plates of a charged capacitor are joined through a Mathematically, ∫ E ⋅ dl = − B
conductor, then the charge arises due to the flow of dt
electrons and thus conduction current flows in the 4. Ampere-Maxwell's law
conductor. (i) This law states that the magnetic field can be
Maxwell proposed that as the charge on the plates of the produced by a conduction current as well as by
capacitor decreases, the electric field between the plates displacement current.
also decreases with time and the time varying electric field (ii) This law also states that the conduction current and
produces magnetic field which signifies that there must be displacement current together have a property of
some current flowing between the plates. Maxwell termed continuity.
this as displacement current (id ) given as (iii) At an instant, in a circuit, the conduction current is
dφ E equal to the displacement current.
id = ε0
dt  dφ 
Mathematically, ∫ E ⋅ dl = µ 0  ic + ε0 E 
where, φ E = electric flux,  dt 
ε0 = permittivity of the free space These equations are collectively called Maxwell’s
equations.
Maxwell’s Equations Results from Maxwell’s Equations
Maxwell in 1862, gave the basic laws of electricity and
(i) The average of energy density associated with
magnetism in the form of four fundamental equations. In
electromagnetic wave is found to be
the absence of any dielectric and magnetic material, which
are known as Maxwell’s equations. It is based on 1
u av = ε0 E02 = Electric field energy density
experimental observations followed by all electromagnetic 2
phenomena, may be stated in the integral form as below 1
u av = B02 = Magnetic field energy density
1. Gauss’s law for electrostatics 2µ 0
(i) This law gives the total electric flux in terms of
(ii) Intensity is defined as energy crossing through unit
charge enclosed by the closed surface.
area in unit time.
(ii) This law states that the electric lines of force start
∴ I = u av c
from positive charge and end at negative charge i.e.
1
the electric lines of force do not form a continuous = ε0c E02
closed path. 2
q 1
Mathematically, ∫ E ⋅ dS = I= cB02
s ε0 2µ 0

@iitjeehelps
Practice Exercise
1. An AC source of voltage V = 100 sin 100πt is
connected to a resistor of resistance 20 Ω. The rms e e
value of current through resistor is a. b.
10 O 1 2 O 1 2
a. 10 A b. A
2 t (s) t (s)
5
c. A d. None of these
2 e e
c. d.
2. In (Q. No. 1), average value of current for long time is
5 O 1 2 O 1 2
a. zero b. A t (s) t (s)
2
c. 10 A d. None of these 10. A uniform but time-varying
B(t)
3. In (Q. No. 1), the average value for half cycle is magnetic field B (t ) exists in a P
x x x x x x
10 5 circular region of radius a and is x x x x
a. A b. A directed into the plane of the paper x xa x x x x
π π x x x x x
c. zero d. None of these as shown. The magnitude of the x x xO x x
induced electric field at point P at a x x x x x x
4. In (Q. No. 1), total charge transferred through resistor x x x x x x
distance r from the centre of the
in long time is
2I 0
circular region
a. zero b. a. is zero b. decreases as 1/r
π
I
c. 0 d. None of these c. increases as r d. decreases as 1/r 2
25π
11. Calculate the peak and rms value of current in AC
5. In (Q. No. 1), total charge transferred in 1/100 second is circuit. The current is represented by the equation
1 1  π
a. C b. C i = 5 sin  300t −  , where t is in seconds and i in
10π 5π  4
c. zero d. None of these
ampere.
6. In (Q. No. 1), total heat generated in one cycle is a. 5 A, 3.535 A b. 5 A, 5.53 A
a. 2 J b. 5 J c. 4 2 J d. zero c. 3 A, 3.53 A d. 6.25 A, 5.33 A

7. In (Q. No. 1), power factor is 12. The average value for half cycle in a 200 V AC source
a. 1 b. 0 is
c. 1/2 d. None of these a. 180 V b. 200 V c. 220 V d. None
8. A metallic square loop ABCD is A B 13. Two alternating currents are given by
moving in its own plane with velocity v I1 = I 0 sin ωt and I 2 = I 0 cos (ωt + φ )
in a uniform magnetic field v
The ratio of rms value is
perpendicular to its plane as shown in
the figure. An electric field is induced
D C a. 1 : 1 b. 1 : φ
c. 1 : 2 d. None of these
a. in AD, but not in BC
b. in BC, but not in AD 14. A current I = 3 + 8 sin 100 t is passing through a
c. neither in AD nor in BC resistor of resistance 10 Ω. The effective value of
d. in both AD and BC current is
a. 5 A b. 10 A c. 4 2 A d. 3/ 2 A
9. A circular coil is placed in a uniform
magnetic field such that its plane is 15. An alternative voltage
perpendicular to field. The radius of r
V = 30 sin 50t + 40 cos 50t
coil changes with time as shown in
the figure. Then, which of the O 1 2 is applied to a resistor of resistance 10 Ω. The rms
t (s)
following graph represent the value of current through resistor is
induced electromotive force in the 5 10 7
a. A b. A c. A d. 7 A
coil with time? 2 2 2

@iitjeehelps
ALTERNATING CURRENT AND EM WAVE 327
16. An alternating voltage V = 140 sin 50 t is applied to a 26. An AC circuit with f = 1000 Hz consists of a coil of
resistor of resistance 10 Ω. This voltage produces ∆H 200 mH and negligible resistance. Calculate the
heat in the resistor in time ∆t. To produce the same voltage across the coil, if the effective current of 5 mA
heat in the same time, required DC current is is flowing.
a. 14 A b. about 20 A a. 7.64 V (rms) b. 7.452 V (rms)
c. about 10 A d. None of these c. 6.28 V (rms) d. 74.62 V (rms)
17. An AC voltage is represented by e = 220 sin (100π) t 27. Find the average power per unit area at distance of
volt and is applied over a resistance of 110 ohm. 2 m from a small bulb, if the bulb emits 20 W of
Calculate the heat produced in 7 min. electromagnetic radiation uniformly in all directions.
a. 11 × 103 cal b. 22 × 103 cal a. 0.69 W/m 2 b. 0.56 W/m 2
c. 0.78 W/m 2 d. 0.39 W/m 2
c. 33 × 103 cal d. 25 × 103 cal
28. If a circuit made up of a resistance 1 Ω and inductance
18. The reactance of a capacitor connected with DC 0.01 H, an alternating emf of 200 volt at 50 Hz is
voltage is connected, then find the phase difference between the
a. zero b. ∞ current and the emf in the circuit.
c. 1 Ω d. None of these
 π  π  π
a. tan−1( π ) b. tan−1  c. tan−1  d. tan−1 
19. The reactance of an inductor connected with DC  2  4  3
voltage is
29. Choose the correct option. If speed of gamma rays,
a. zero b. ∞
X-rays and microwaves are v g , v X and νm .
c. 1 Ω d. None of these
a. vg < v X < vm b. vg > v X > vm
20. An AC voltage e = e0 sin 50t − e0 cos 100πt is
c. vg > v X < vm d. vg = v X = vm
connected in series with a resistor and capacitor. The
steady state current through circuit is found to be 30. A condenser of capacitance of 2.4 µF, is used in a
I = I 0 sin(50πt + φ ) + I 0 ′ cos(100πt + φ 2 ) transmitter to transmit at λ wavelength. If the inductor
Then, the ratio of 0 is
I of 10−8H is used for resonant circuit, then value of λ is
I0 ′ a. 292 m b. 400 m c. 334 m d. 446 m
a. greater than 1 b. equal to 1 31. If a dry cell of emf = 1.5 V is connected across the
c. less than 1 d. None of these
primary of a step-up transformer of turn ratio 3 : 5,
21. An alternating voltage V = V0 sin ωt is connected to a then calculate the voltage developed across the
capacitor of capacity C 0 through an AC ammeter of secondary.
zero resistance. The reading of ammeter is a. 30 V b. 5 V
V0 V0 c. zero d. None of these
a. b.
2 ωC 2 32. If at a certain instant, the magnetic induction of the
V0ωC electromagnetic wave in vacuum is 6.7 × 10−12T, then
c. d. None of these
2 the magnitude of electric field intensity will be
22. Calculate the maximum current in the circuit, if a a. 2 × 10−3 N/C b. 3 × 10−3 N/C
capacitor of capacitance 1 µF is charged to a potential c. 4 × 10−3 N/C d. 1 × 10−3 N/C
of 2 V and is connected in parallel to an inductor of
inductance 10−3 H. 33. Find the energy of photon of electromagnetic radiation
a. 4000 mA b. 2000 mA c. 1000 mA d. 5000 mA of wavelength 200 Å.
. × 10−18 J
a. 176 b. 0.99 × 10−18 J
23. In a circuit consisting of inductor (L), capacitor (C) and
1 c. 0.54 × 10−18 J d. 0.63 × 10−18 J
resistor (R) are in series, if ωL < , then the emf
ωC 34. Find the speed of light in air, if an electromagnetic
a. leads the current b. lags behind the current wave is travelling in air whose dielectric constant is
c. is in phase with current d. is zero k = 1006
. .
24. Find the resonant frequency of a series circuit consist a. 3 × 108 m/s b. 3.88 × 108 m/s
of an inductance 200 µH, a capacitance of 0.0005 µF c. 2 .5 × 108 m/s d. 4.6 × 108 m/s
and a resistance of 10 Ω.
35. An object is placed at some distance from a radio
a. 480 kHz b. 503 kHz c. 406 kHz d. 607 kHz
station. If the interval between transmission and
25. Find the frequency of voltage for an AC circuit. The reception of pulses is 266. × 10−2 s, then find the
equation of alternating voltage is V = 200 sin 314 t . distance.
a. 50 Hz b. 60 Hz c. 55 Hz d. 65 Hz a. 4000 km b. 2000 km c. 3000 km d. 2500 km

@iitjeehelps
328 SELF STUDY GUIDE BITSAT

36. Calculate the wavelength of a radio wave of frequency 38. A radio wave of intensity I is reflected by a surface.
of 1 MHz. Find intensity (I ), if pressure exerted on the surface is
a. 400 m b. 300 m c. 350 m d. 200 m 2 × 10−8 N/m 2.
37. The electric field in an electromagnetic wave is given a. 3 N/m 2 b. 4 N/m 2 c. 6 N/m 2 d. 7 N/m 2
 x 39. A TV tower has a height of 100 m. Find the area
by E = (100 N/C) sin ω t −  .
 C covered by the TV broadcast, if radius of the earth is
6400 km.
If the energy contained in a cylinder of cross-section
10 cm 2 and length 50 cm along the X-axis is a. 380 × 107 m2 b. 402 × 107 m2
4.4 × 10−8J / m 3 , then find the intensity of the wave. c. 595 × 107 m2 d. 440 × 107 m2
a. 12.4 W / m 3 40. An electromagnetic wave with pointing vector 5 W/m 2
b. 13.2 W / m 3 is absorbed by a surface of some area. If the force on
c. 15.7 W / m 3 the surface is 10−7 N, then area is
d. 11.9 W / m 3 a. 6 m 2 b. 3 m 2 c. 60 m 2 d. 4 m 2

BITSAT Archives
1. Voltage V and current i in AC circuit are given by 7. An AC source is
 π connected in
V = 50 sin (50 t ) volt, i = 50 sin 50 t +  mA C
 3 parallel with an VS VL L VCR VR
L-C-R circuit as
The power dissipated in the circuit is
shown. Let IS , IL ,
a. 5.0 W b. 2.5 W
c. 1.25 W d. zero
IC and IR denote the currents through and VS , VL , VC
and VR the voltages across the corresponding
2. The produced rays in sonography are components. Then,
a. microwaves b. infrared waves a. IS = IL + IC + IR
c. sound waves d. ultrasound b. VS = VL + VC + VR
3. The ratio of secondary and primary turns of step-up c. (IL, IC , IR ) < IS
transformer is 4 : 1. If a current of 4 A is applied to the d. IL, IC may be greater than IS
primary, the induced current in secondary will be
a. 8 A b. 2 A 8. The number of turns in primary and secondary coils of
c. 1 A d. 0.5 A a transformer is 50 and 200, respectively. If the
current in the primary coil is 4 A, then current in the
4. The impedance of a circuit consists of 3 Ω resistance secondary coil is
and 4 Ω reactance. The power factor of the circuit is a. 1 A b. 2 A c. 4 A d. 5 A
a. 0.4 b. 0.6
9. An inductor of 2 H and a resistance of 10 Ω are
c. 0.8 d. 1.0
connected in series with a battery of 5 V. The initial
5. The time taken by the current to rise to 0.63 of its rate of change of current is
maximum value in a DC circuit containing inductance a. 0 . 5 A/s b. 2.0 A/s c. 2.5 A/s d. 0.25 A/s
(L ) and resistance (R ) depends on
a. L only b. R only 10. The transformation ratio in the step-up transformer is
L a. 1
c. d. LR
R b. greater than one
c. less than one
6. Figure represents two bulbs B1 d. the ratio greater or less than depends on the
R
B1 and B 2, resistor R and and other factors
inductor L. When the switch S
S is turned off, then 11. In a step-up transformer, if ratio of turns of primary to
a. both B1 and B2 die out L secondary is 1 : 10 and primary voltage is 230 V. If the
B2 load current is 2 A, then the current in primary is
promptly
b. both B1 and B2 die out with a. 20 A b. 10 A c. 2 A d. 1 A
some delay 12. The square root of the product of inductance and
c. B1 dies out promptly but B2 capacitance has the dimension of
with some delay a. length b. mass
d. B2 dies out promptly but B1 with some delay c. time d. no dimension

@iitjeehelps
Answer with Solutions
Practice Exercise 13. (a)
I 01
I rms1 =
I I I
= 0 , I rms 2 = 02 = 0
2 2 2 2
V0 100 I 5
1. (c) I 0 = = =5 A ∴ I rms = 0 = A ∴ Ratio = 1: 1
R 20 2 2 T 2
V 100
2. (a) I = = sin 100πt = 5 sin 100πt 14. (a) 2
I effective
∫ I dt
= I rms 0 T
R 20

T
5∫ sin 100πt dt
∫0dt
∴ I = 0
T
=0 After solving, I effective = I rms = 5 A
∫ dt
0
V 30
15. (a) I = = sin 50t +
40
cos 50t 4
2l 2 × 5 10 R 10 10
3. (a) I av = 0= = A or I = 3 sin 50 t + 4 cos 50 t
l0 = √42+32
π π π = 5A
t = 1/50 s t = 1/50 s  π
4. (a) q = ∫ I dt = ∫ 100 sin 100πt = 0 or I = 3 sin 50 t + 4 sin 50t + 
0 0  2 3
T 1 I 5
5. (a) When t ≤ , i.e. t ≤ s ∴ I rms = 0 = A
2 100 2 2
2I 10 1 1
Then, q = I avt = 0 t = × = 16. (c)
π π 100 10π
Iv2RT
 5  1
2 17. (b) H = cal
2
6. (b) ∆H = I rms RT =   × 20 × =5J J
 2 50 ( 2 )2 × 110 × 7 × 60  I 220 / 110 
R R R = Q I rms = 0 = = 2
7. (a) cos φ = = = =1 4.2  2 2 
Z R + (XL − XC )2 R2 + 0 3
= 22 × 10 cal
⇒ cos φ = 1 18. (b) For DC source, frequency is zero.
8. (d) In both AD and BC, because the electrons and 1 1
∴ XC = = ×C = ∞
positive ions will be separated by magnetic force in these ωC 0
two arms. 19. (a) XL = ωL = 0 × L = 0
9. (b) Flux linked with the circular coil, φ = BA E0 E0
dφ dr dφ dr 20. (c) I 0 = =
2
= Bπ 2r ∴ ∝ R + XC2  1 
2
dt dt dt dt R2 +  
dr  50 C
Emf of coil ∝ and e ∝ r E0
dt Similarly, I ′ 0 = Hence, I 0 < I ′ 0
In 1 and 3 regions, r = constant ⇒ emf = 0  1 
2
2
dr R + 
In 2 region, = constant  100 C
dt
Emf induced is increasing at constant rate, because r is 21. (c)
increasing. 22. (a) Charge on capacitor q 0 = CV , q 0 = 2 × 10−6 C
Here, q = q 0 sin ωt
dq
r r For maximum current = I 0 = = ωq 0
dt
1
1 2 3 1 2 3 But ω = = (109 )1/ 2, I 0 = (109 )1/ 2 ( 2 × 10−6 )
LC
t t
dφ = 10 × 104 × 2 × 10−6 = 2 10 × 10−2 A = 4000 mA
10. (b) Using, ∫ E ⋅ d l = − 1 π
dt 23. (a) When ωL < , emf leads the current by .
1 ωC 2
⇒ E ( 2πr ) = constant ⇒ E ∝
r 24. (b)
11. (a) 25. (a) ComparingV = 200 sin 314t with the standard
V0
12. (a) Vrms = ∴ V0 = 2Vrms equation V = V0 sin ωt , we get ω = 314 s−1
2 ω 314
2V 2 2 2 × 200 Linear frequency, f = = = 50 Hz
∴ Vav = 0 = 2Vrms = = 180 V 2π 2 × 3.14
π π π

@iitjeehelps
330 SELF STUDY GUIDE BITSAT

26. (c) Inductive reactance, XL = Lω = 2πfL hc 6.62 × 10−34 × 3 × 108


33. (b) E = = = 0.0099 × 10−16
E 5 λ 2000 × 10−10
also, = Lω = XL or E = IXL ⇒ E= × 1256
I 1000 E = 0.99 × 10−18 J
E = 6.28 volt (rms) 1 c
P 20 34. (a)v = ⇒
27. (d) I = , I= = 0.39 W/m 2 µ 0kε 0 k
4 πr 2 4π( 2)2
c 3 × 108
X  v = = ⇒ v ≅ 3 × 108 m/s
28. (a) tan φ =  L  k 1006
.
R
35. (a) 2x = ct
and XL = ωL = ( 2πfL) = ( 2π )(50)( 0.01) = πΩ
ct 3 × 108 × 2.66 × 10−2
also R = 1Ω ⇒ φ = tan−1( π ) x = = ⇒ x ≅ 4000 km
2 2
29. (d)
36. (b)
c 3 × 108 1 2
30. (a) Frequency = n = = 37. (b) I = εEo c = ( 4.4 × 10−8) × ( 3 × 108 ) = 13.2 W/m 2
λ λ 2
1
also, resonant frequency = n = 38. (a)
2π LC
( )
2 2
1 λ 39. (b) Area = πr 2 = π = ( 2hR )1/ 2
or LC = or C =
4π 2n 2 4π 2c 2L Area = 2πRh = 2 × π × 6400 × 103 × 100 = 402 × 107 m 2
2 2
or λ = 4π c . LC I 5
40. (a) We know , P = =
or λ = 4π 2 × ( 3 × 108 )2 × 2.4 × 10−6 × 10−8 ⇒ λ = 292 m c 3 × 108
F
31. (c) Transformer is an AC device, it does not work on DC, and F =P × A ⇒ A =
so, output voltage is zero. P
10−7
32. (a) A= × 3 × 108 ⇒ A = 6 m2
5

BITSAT Archives
1. (c) Given,V = 50 sin(50 t ) V 8. (a) In a transformer,
 π NP IS 50 I
Maximum voltage, V0 = 50 V, i = 50 sin 50 t +  mA ∴ = = = S ⇒ IS = 1 A
 3 NS IP 200 4
Maximum current, i0 = 50 mA = 50 × 10−3 A  Rt 

i V 9. (c) i = i0 1 − e L 
Power dissipated, P = 0 × 0  
2 2
50 × 50 × 10−3 di d d  −Rt  −
Rt
= ⇒ = i0 − i0e L  = 0 + i0R e L
2  
dt dt dt   L
2500 × 10−3
= = 1.25 W Initially, t = 0
2
di i0 × R E 5
2. (d) The produced rays in sonography are ultra sound. ⇒ = = = = 2.5 A/s
dt L L 2
I N 1 1
3. (c) S = P = ⇒ IS = × 4 = 1 A N V
10. (b) Transformation ratio, k = S = S
IP NS 4 4
NP VP
X 4
4. (b) tan φ = = For step-up transformer,
R 3
NS > NP , i.e., VS > VP , hence, k > 1.
3
Power factor = cos φ = = 0.6 N 1
5 11. (a) P =
NS 10
L
5. (c) It depends upon its time constant, τ = . VP = 230 V, IS = 2 A ,
R
IP NS I 10
6. (c) Bulb B1 dies out promptly, but bulb B2 dies out with = ⇒ P = ⇒ IP = 20 A
IS NP 2 1
some delay. This is because of self-induced emf across
L, during decay. 1 1
12. (c) We know f = or LC = = time
7. (d) In parallel resonant circuit, current through L and C 2π LC 2πf
may be greater than the source current, i.e. IL, IC may be Thus, LC has the dimension of time.
greater than IS .

@iitjeehelps
29
Cathode Ray,
Photoelectric Effect
of Light and X-Rays

Cathode Rays
It consists of fast moving electrons. If discharge tube is operating at voltage V , then
1
E k = eV = mv 2
2

Dual Nature of Radiation


The concept of wave nature of matter arose from the dual character of the radiation which sometimes
behaves as a wave and sometimes as a particle.
In the phenomena where light interacts with itself, such as dispersion, interference, diffraction and
polarisation, the wave nature of light dominates. In the phenomena where the light interacts with
matter, such as in Photoelectric effect, Compton effect, Raman effect, the nature of particle of light
dominates.
Matter waves travel as a wave packet with decreasing amplitude on either side of the present position.

Wave packet

de-Broglie Wavelength Relation


According to de-Broglie, a wave is associated with energy moving particle. These waves are called
de-Broglie waves or matter waves.
(i) In the nth orbit, we have
nh h 2 πr
L = mvr = ⇒ λ= =
2π mv n

@iitjeehelps
332 SELF STUDY GUIDE BITSAT

2 πr E ≥ W0 or ν ≥ ν0 or λ ≤ λ 0
(ii) Number of waves in nth orbit is N = =n
λ where, ν0 = threshold frequency, λ0 = threshold
Thus, the number of de-Broglie waves in nth orbit wavelength.
is n.
Expression for the wavelength associated with charged Dependence of Photocurrent on
particles accelerated through a potential differenceV
12 . 27
Intensity
(i) Electron, λ e = Å On keeping the frequency of incident radiation and anode
V
potential fixed, if the intensity of incident light is increased,
0. 286
(ii) Proton, λ p = Å then the photoelectric current increases.
V

Photocurrent (i)
0. 202
(iii) Deuteron, λ d = Å
V
0. 101
(iv) α-particle, λ α = Å
V
(v) The de-Broglie wavelength of a particle of mass m Intensity (I )
and moving with velocity v is given by Variation of photocurrent
h with intensity of incident
λ=
mv
(vi) If m0 be the rest mass of the particle, then i.e. photocurrent (i) ∝ intensity (I )
h[1 − v 2 /c 2 ]1 / 2
λ= Dependence of Stopping Potential on
m0v Intensity
(vii) The de-Broglie wavelength of a particle of mass m The stopping potential does not change for a given metal
and kinetic energy K is given by and given frequency of radiation, if we change the intensity
h of incident light.
λ=
2 mK
current
Photo

I3 > I2> I1
(viii) If a particle of mass m carrying charge q0 is I3
accelerated through a potentialV , then its de-Broglie I2
Stopping
wavelength is given by potential
I1
h
λ=
2 mq0V – V0 V

(ix) The de-Broglie wavelength of a gas molecule of Retarding potential Collector plate potential
mass m at temperature T (in kelvin) is given by
( I 1 , I 2 , I3 are intensities)
h
λ=
3 mkT
Dependence of Stopping Potential on
Here, k is Boltzmann constant. Frequency
Greater is the frequency of incident radiation, more is the
Photoelectric Effect magnitude of stopping potential.
For emission of electrons from a metal surface, a minimum
current
Photo

amount of energy (E) is required to overcome the potential n3 > n2 > n1


barrier (W0 : work function) provided by the metal. When
energy is provided in the form of electromagnetic waves of
Saturation

photons, then emitted electrons are called photoelectrons


current

and this effect is called photoelectric effect. As photon


n3 n
energy ( E = hν ) is proportional to its frequency, a minimum 2 n1
frequency is required so that photon has energy greater
– V01 – V02 – V03
than or equal to work function. This minimum frequency is V
Retarding potential Collector plate potential
called threshold frequency ( ν0 ).
Variation of stopping potential with
For photoemission to take place, either of the following frequency (n 1 , n 2 , n 3 are frequencies)
conditions must be satisfied

@iitjeehelps
CATHODE RAY, PHOTOELECTRIC EFFECT OF LIGHT AND X-RAYS 333

Einstein’s Photoelectric Equation Graph between V 0 and ν


According to Einstein, Let us plot a graph between the stopping potential (V0 ) and
photoelectric effect is the result of Incident photon
the incident frequency (ν ). The equation between them is
one to one elastic collision K Work eV0 = hν − W
between photon and electron in function W0
e– h W 
which photon is completely W or V0 =   ν −  
absorbed. So, if an electron in a –
e e
e Metal
metal absorbs a photon of energy Again comparing with y = mx + c, the graph between V0
E ( = hν ), then it uses the energy in h
and ν is a straight line with positive slope (a universal
the following ways e
W
(i) Some energy (say W ) is used in shifting the electron constant) and negative intercept (which depends on the
from interior to surface. e
metal). The corresponding graph is shown as below:
(ii) Some energy (say W0 ) is used in removing electron
V0
from metal’s surface. 1 2
(iii) Rest energy will appear as kinetic energy ( K ) of the h
(Slope)1 = (Slope)2 =
emitted photoelectrons. Hence, W + W0 + K = hν e

For the electrons emitting from surface W = 0, so kinetic ν


energy of these emitted electrons will be maximum. W1
e
Hence, hν = W0 + K max W2
e
So, Einstein’s photoelectric equation is given by
⇒ K max = hν − W0
where,W0 is a work function.
Graph between V 0 and ν
If V0 is stopping potential,
then eV0 = K max = hν − W0 .

Graph between K max and f


Particle Nature of Light– The
Let us plot a graph between Kmax 1 2
Photon
maximum kinetic energy According to Einstein’s quantum theory, light propagates in
(K max ) of photoelectrons and the bundles (packets or quanta) of energy. Each bundle is
frequency ( ν ) of incident light. f called a photon which possesses energy.
(f0 )1 (f0 )2
The equation between K max W1 (i) Energy of photon Energy of each photon is given by
and ν is K max = hν − W W2 hc
E = hν =
Comparing it with y = mx + c , λ
the graph between K max and f
Graph between K max and f where, c = speed of light
is a straight line with positive
slope and negative intercept. h = Planck’s constant = 6.6 × 10−34 J-s

From the graph, we observe that Energy of photon in electron-volt


hc 12375 12400
(i) If K max = 0, then ν = ν0 E (eV ) = = ⇒ E(eV ) ≈
eλ λ (Å ) λ (Å )
(ii) Slope of the straight line is h, a universal constant, i.e.
if graph is plotted for two different metals 1 and 2, (ii) Mass of photon Actual rest mass of the photon is
then slope of both the lines is same. zero. But its effective mass is given as
E hν h
(iii) The negative intercept of the line is W (the work E = mc 2 = hν ⇒ m = 2 = 2 ⇒ m =
c c cλ
function) which is characteristic of a metal,
i.e. intercepts for two different metals will be This mass is also known as dynamic mass of the
different. photon.
(iii) Momentum of the photon Momentum,
If W 2 > W 1 , then ( ν0 ) 2 > ( ν0 )1 as W = hν0 , where ν0 =
E hν h
threshold frequency p =m×c = = ⇒ p=
c c λ

@iitjeehelps
334 SELF STUDY GUIDE BITSAT

(iv) Number of emitted photons The number of


photons emitted per second from a source of
Compton Effect
monochromatic radiation of wavelength λ and (a) Compton shift in wavelength
power P is given as h
∆λ = (1 − cos θ)
P m0c
n=
E h o
P (Here, = 0.024 A , θ is angle of scattering.)
= m0c
hν hc hc
Pλ (b) KE of recoil electron, E k = −
⇒ n= λ λ0
hc
λ sin θ
where, E = energy of each photon. (c) Direction of recoil electron, tan θ =
λ 0 − λ cos θ

Bragg’s Equations Mosley Law


For normal incidence, b sin φ = nλ Frequency ν of characteristic X-rays spectrum
For glancing incidence, 2d sinθ = nλ ν = n ( z − σ)
Also, φ + 2 θ = 180°
where, a and σ are constants and screening constant for K α
line, σ = 1, Screening constant for Lα line, σ = 7.4.

Practice Exercise
1. In an oil drop experiment, the following charges 6. If the velocity of an electron is doubled, its de-Broglie
(in arbitrary units) were found on a series of oil frequency will be
droplets: 2.30 × 10− 15 , 6.90 × 10− 15 , 1.38 × 10− 14 , a. half b. remain same
5.75 × 10− 15 , 1.955 × 10− 14 . The charge on electron c. doubled d. become four times
(in the same unit) should be 7. An electron is at rest. Its wavelength is
a. 2.30 × 10− 15 b. 1.15 × 10− 15 a. 1
c. 1.38 × 10− 14 d. 1.955 × 10− 14 b. infinity
h
2. In Wilson cloud chamber experiment, two particles were c.
me
found to show equal deviation but in opposite directions.
The names positron and negatron were given to these d. it has no wave character
particles by Anderson. Negatron should be 8. If the de-Broglie wavelength of a proton is 10− 13 m,
a. neutron b. neutrino the electric potential through which it must have been
c. electron d. proton accelerated is
3. An α-particle when accelerated through a potential of a. 4.07 × 104 V b. 8.2 × 104 V
V volt has a wavelength λ associated with it, but if a c. 8.2 × 103 V d. 4.07 × 105 V
proton in order to have same wavelength λ by what 9. A moving electron has numerical relation λ = h. Then,
potential difference it must be accelerated?
1 1
a. 8 V b. 6 V c. 4 V d. 12 V a. me = b. ve =
ve me
4. The cathode ray particles originate in a discharge tube c. both a and b d. None of these
from the
10. The de-Broglie wavelength of a bus moving with
a. cathode b. anode
speed v is λ. Some passengers left the bus at a
c. source of high voltage d. residual gas
stopage. Now, when the bus moves with twice its
5. Three particles having charges in the ratio of 2 : 3 : 5, initial speed, its kinetic energy is found to be twice its
produce the same point on the photographic film in initial value. What will be the de-Broglie wavelength,
Thomson’s experiment. Their masses are in the ratio of now?
a. 2 : 3 : 5 b. 5 : 3 : 2 a. λ b. 2λ
c. 15 : 10 : 6 d. 3 : 5 : 2 c. λ / 2 d. λ / 4

@iitjeehelps
CATHODE RAY, PHOTOELECTRIC EFFECT OF LIGHT AND X-RAYS 335
11. An electron of mass m and charge q is accelerated 21. An electron (mass m ) with an initial velocity
from rest in a uniform electric field of strength E. The v = v 0i$(v 0 > 0) is in an electric field E = − E 0 i$
velocity acquired by it as it travels a distance l is (E 0 = constant > 0). Its de-Broglie wavelength at time t
a. 2Eq l/m b. 2 Eq /ml c. 2Em /ql d. Eq /ml is given by
12. Protons of an electromagnetic radiation has an energy λ0  eE0t 
a. b. λ 0 1+ 
11 keV each. To which region of electromagnetic  eE0 t   mv 0 
1+ 
spectrum does it belong?  m v0
a. X-ray region b. Ultra violet region c. λ 0 d. λ 0t
c. Infrared region d. Visible region
13. Two photons of same frequencies moving in same 22. A ruby laser produces radiations of wavelengths
medium have 662.6 nm in pulses whose duration are 10− 9 s. If the
a. same linear momenta and wavelengths laser produces 0.39 J of energy per pulse, how many
b. same linear momenta and same speeds photons are produced in each pulse?
c. same energies and same linear moment a. 1.3 × 109
d. None of the above b. 1.3 × 1018
14. The voltage applied to an electron microscope to c. 1.3 × 1027
produce electrons of wavelength 0.50 Å is d. 3.9 × 1018
a. 602 V b. 50 V c. 138 V d. 812 V
23. At one time, the metre was defined as 1650763.73
15. A certain molecule has an energy level diagram for its wavelengths of the orange light emitted by a light
vibrational energy in which two levels are 0.014 eV source containing Kr 86 atoms. What is the
apart. Find the wavelength of the emitted line for the corresponding photon energy of this radiation?
molecule as it falls form one of these levels to the
a. 3.28 × 10− 19 J/quanta b. 1.204 × 10− 31 J/quanta
other
c. 1.09 × 10− 27 J/quanta d. 4.01 × 10− 40 J/quanta
a. 8.9 × 10− 5 m b. 1.2 × 10− 6 m
c. 173.6 m d. 4.6 × 10− 7 m 24. From the figure describing photoelectric effect, we
16. How many photons are emitted by a laser source of may infer correctly that
5 × 10− 3 W operating at 632.2 nm in 2 second Y Na Al
Stopping potential

(h = 6.63 × 10− 34 Js)? 4


16 16
a. 3.2 × 10 b. 1.6 × 10 3
c. 4 × 1016 d. None of these 2
17. Only a fraction of the electrical energy supplied to a 1
tungsten light bulb is converted into visible light. If a θ θ
X
100 W light bulb converts 20% of the electrical energy 5 10
into visible light (λ = 662.6 nm), then the number of Frequency
photons emitted by the bulb per second is a. Na and Al both have the same threshold frequency
a. 6.67 × 1019 b. 2 × 1028 c. 6 × 1036 d. 6.30 × 1019 b. Maximum kinetic energy for both the metals depend
linearly on the frequency
18. Calculate the number of photons emitted by a 60 W c. The stopping potentials are different for Na and Al for
bulb per second, if 10% of the electrical energy the same change in frequency
supplied to an incandescent light bulb is radiated as d. Al is a better photosensitive material than Na
visible light. 25. The stopping potential for the photoelectrons emitted
a. 1.8 × 1019 b. 1.8 × 1016 c. 1.8 × 1011 d. 1.8 × 1021 from a metal surface of work function 1.7 eV is
19. The momentum of a photon having energy equal to 10.4 eV. Identify the energy levels corresponding to
the rest energy of an electron is the transitions in hydrogen atom which will result in
emission of wavelength equal to that of incident
a. zero b. 2.73 × 10− 22 kg ms− 1
radiation for the above photoelectric effect.
c. 1.99 × 10− 24 kg ms− 1
d. infinite
20. A perfectly reflecting solid hemisphere of radius R is –0.85 eV
n=4
–1.51 eV
placed in the path of a parallel beam of light of large n=3
–3.4 eV
aperture. If the beam carries an intensity I, find the n=2
force exerted by the beam on the hemisphere. n=1
–13.6 eV
2πR 2I πR 2I
a. b.
c c a. n = 3 to 1 b. n = 3 to 2
4πR 2I c. n = 2 to 1 d. n = 4 to 1
c. d. None of these
c

@iitjeehelps
336 SELF STUDY GUIDE BITSAT

26. The work function of a certain metal is 2.3 eV. If light 35. Ultraviolet beam of wavelength 280 nm is incident on
of wave number 2 × 106 m − 1 falls on it, the kinetic lithium surface of work function 2.5 eV. The maximum
energies of fastest and slowest ejected electron will velocity of electron emitted from metal surface is
be respectively. a. 8.2 × 105 m/s b. 106 m/s
a. 2.48 eV, 0.18 eV b. 0.18 eV, zero c. 7 × 105 m/s d. None of these
c. 2.30 eV, 0.18 eV d. 0.18 eV, 0.18 eV
36. In previous problem, the stopping potential is
27. When the electromagnetic radiations of frequencies a. 1.9 V b. 10 V c. 3 V d. None
4 × 1015 Hz and 6 × 1015 Hz fall on the same metal, in
37. Threshold frequency for photoelectric effect from a
different experiments, the ratio of maximum kinetic
metal surface of work function 4.5 eV is
energy of electrons liberated is 1: 3. The threshold
frequency for the metal is a. 1.1 × 109 Hz b. 540 Hz
15 15 c. 1.1 × 1015 Hz d. None of these
a. 2 × 10 Hz b. 1 × 10 Hz
c. 3 × 1015 Hz d. 1.67 × 1015 Hz 38. If nr and nb are the number of photons of red and blue
light respectively with same energy, then
28. A surface is irradiated with ultraviolet radiation of
a. nr > nb
wavelength 0.2 µm. If the maximum velocity of
electron liberated from the surface is 8.8 × 105 m/s, b. nr < nb
then find the work function of the surface. c. nr = nb
a. 3 eV b. 4 eV c. 5 eV d. 6 eV d. no relation between nr and nb

29. Choose the correct option for the graph between the 39. The number of photoelectrons emitted per unit time
frequency of incident light and the stopping potential. depends on (v > v 0 )
a. It is a parabola b. It is a straight line a. threshold frequency
c. It is a hyperbola d. It is a circle b. frequency of the incident radiation
c. intensity of the incident radiation
30. Specific heat of water is 4.2 J/g °C. If light of frequency d. density of the metal irradiated
3 × 109 Hz is used to heat 400 g of water from 20°C to
40. An X-ray tube is operating at 15 kV. The lower limit of
40°C, the number of moles of photons needed will be
the wavelength of X-rays produced is
a. 1.69 × 1029 b. 1.69 × 1028 c. 2.80 × 104 d. 2.80 × 105
a. 0.82 × − 7 m
−3
31. 10 W of 5000 Å light is directed on a photoelectric b. 0.82 × 10− 8 m
cell. If the current in the cell is 0.16 µA, then the c. 0.83 × 10− 10 m
percentage of incident photons which produce d. 0.82 × 10− 13 m
photoelectrons, is
a. 0.4% b. 0.04% c. 20% d. 10% 41. Ultraviolet light of wavelength 66.26 nm and intensity
2 W /m 2 falls on potassium surface by which
32. The potential energy of a particle of mass m is given by
photoelectrons are ejected out. If only 0.1% of the
E ; 0 ≤ x < 1 incident photons produce photoelectrons, and surface
U (x ) =  0
 0; x>1 area of metal surface is 4 m 2, how many electrons are
λ 1 and λ 2 are the de-Broglie wavelengths of the emitted per second?
particle, when 0 ≤ x ≤ 1 and x > 1 respectively. If the a. 2.67 × 1015
λ b. 3 × 1015
total energy of particle is 2E 0, then the ratio 1 will be c. 3.33 × 107
λ2
d. 4.17 × 1016
a. 2 b. 1 c. 2 d. 1/ 2
42. The stopping potentials are V1 and V2. Calculate the
33. A radiation is incident on a metal surface of work (V1 − V2 ), if the λ 1 and λ 2 are wavelength of incident
function 2.3 eV. The wavelength of incident radiation lights, respectively.
is 600 nm, then the number of photoelectrons is
hc  1 1
a. zero b. > 104 a.  − 
e  λ1 λ 2 
c. = 104 d. None of these
hc  1 1
b.  + 
34. At its closest approach, the distance between the e  λ 1 λ 1
mars and the earth is found to be 60 million km. When
e  1 1
the planets are at this closest distance, how long c.  + 
would it take to send a radio message from a space hc  λ 1 λ 2 
probe of mars to earth? e  1 1
d.  − 
a. 5 s b. 200 s c. 0.2 s d. 500 s hc λ
 1 λ 2

@iitjeehelps
CATHODE RAY, PHOTOELECTRIC EFFECT OF LIGHT AND X-RAYS 337
43. Calculate the work function of the metal, if the kinetic 46. A red bulb and violet bulb of equal power emits nR and
energies of the photoelectrons are E 1 and E 2, with nV number of photons in a given time, then
wavelengths of incident light λ 1 and λ 2 a. nR = nV b. nR > nV
E1λ 1 − E2λ 2 E1E2
a. b. c. nR < nV d. nR ≥ nV
λ 2 − λ1 λ1 − λ 2
(E1 − E2 )λ 1λ 2 λ 1λ 2E1 47. When a surface 1 cm thick is illuminated with light of
c. d.
( λ1 − λ 2 ) ( λ 1 − λ 2 ) E2 wavelength λ, the stopping potential is V0, but when
the same surface is illuminated by light of wavelength
44. What is the energy of photon of wavelength 24800 Å? V
a. 0.5 eV b. 0.9 eV 3λ, the stopping potential is 0 . Find the threshold
6
c. 1.1 eV d. 0.75 eV
wavelength for metallic surface.
45. For a certain metal v is five times of v 0 and the a. 4λ b. 5λ c. 3λ d. 2λ
maximum velocity of coming out photons is
48. Photoelectric effect show
8 × 106 m/ s. If v = 2v 0, then maximum velocity of
a. wave-like behaviour of light
photoelectrons will be b. particle-like behaviour of light
6 6
a. 4 × 10 m/s b. 6 × 10 m/s c. both wave-like and particle-like behaviour of light
c. 2 × 106 m/s d. 1 × 106 m/s d. neither wave-like nor particle-like behaviour of light

BITSAT Archives
1. Graph of stopping potential for most energetic emitted 3. A and B are two metals with threshold frequencies
photoelectron (VS ) with frequency of incident radiation 1.8 × 1014 Hz and 2.2 × 1014 Hz. Two identical photons
on metal is given below. [2014] of energy 0.825 eV each are incident on them. Then,
photoelectrons are emitted by
VS A
(Taking, h = 6.6 × 10−34 J-s) [2012]
a. B alone b. A alone
c. Neither A nor B d. Both A and B
4. Energy conversion in a photoelectric cell takes place
C from [2011]
B
ν
a. chemical to electrical b. magnetic to electrical
c. optical to electrical d. mechanical to electrical
AB
The value of , in graph is 5. Of the following, the one which has the largest
BC de-Broglie wavelength for the same speed is [2010]
(h = Planck’s constant, e = electronic charge) a. electron b. proton
a. h b. e c.
h
d.
e c. α-particle d. oxygen atom
e h
6. The energy that should be added to an electron to
2. The number of photoelectrons in a photoelectric effect reduce its de-Broglie wavelength from 1 nm to 0.5 nm
experiment depends on the [2013] is [2005]
a. frequency of light a. four times the initial energy
b. intensity of light b. equal to the initial energy
c. Both a and b are correct c. twice the initial energy
d. Both a and b are incorrect d. thrice the initial energy

@iitjeehelps
Answer with Solutions
Practice Exercise 17. (a) 18. (a)
h E
1. (b) 2. (c) 19. (b) For photon, p = =
λ c
3. (a) λ p = λ α But from questions,
h h E = rest energy of electron = m0c 2
=
2mpQ pVp 2mαQ αVα
m0c 2
∴ p= = m0c = (9.109 × 10− 31 kg) × (3 × 108 ms− 1)
∴ mpQ pVp = mαQ αVα c
m  Q  = 2.7327 × 10− 22 kg ms− 1 = 2.73 × 10− 22 kg m/s
∴ Vp =  α   α  Vα
 mp   Q p  20. (b) Effected area = πR 2
Vp = ( 4) ( 2) Vα ⇒ Vp = 8 V I
Pressure = , where, c is speed of light
4. (a) 5. (a) 6.(c) 7. (d) 8. (b) c
h πR 2I
9. (c) λ = ∴ Force =
meve c
∴ meve = 1 [Q λ = h (given)] eE
21. (a) Acceleration, a = 0
1 m
∴ me = Final velocity after t second,
ve
eE0
10. (a) Momentum, v =v0 + t (From,v = u + at )
1 m
mv × v
2 2KE h h
p = mv = = ∴ de-Broglie wavelength, λ = =
1 v mv  eE0t 
×v m v 0 + 
2  m 
If KE as well as speed are doubled, momentum p remains h λ0
= =
unchanged.  eE0t   eE0t 
h mv 0 1 +  1 + 
Q λ=  mv 0   mv 0 
p
22. (b)
Hence, de-Broglie wavelength will be unchanged.
23. (a) Given,v = 1650763.73 m− 1
11. (a) As, v 2 = u 2 + 2 as ,
Now, E = hcv
qEl 2 qEl
So, v2 = 0+ 2 or v = = (6.626 × 10− 34 ) × (3 × 108 ) × 1650763.73
m m
= 3.28 × 10− 19 J/quanta
12. (b) 13. (d)
h h 24. (b)
14. (a) de-Broglie wavelength is λ = =
mv 2mE 25. (a) If incident radiation has wavelength λ, then
But E = eV eV0 =
hc
− φ 0 or
hc
= 10.4 + 1.7 = 12.1 eV
h h2 λ λ
λ= ⇒V = 1240 eV - nm
2meV 2meλ2 or λ=
12.1 eV
( 6.62 × 10−34 )2
V = −10 2
–0.85 eV
( 0.5 × 10 ) × 2 × 9.1 × 10−31 × 16
. × 10−19 n=4
–1.51 eV
n=3
⇒ V = 60198
. V ≈ 602 V –3.4 eV
n=2
15. (a) n=1
–13.6 eV
16. (a) Energy of each photon and for an excited H2 atom, energy levels are
hc 6.63 × 10− 34 × 3 × 108 Eg = Ei − Ef = − 1.51 − ( −13.6) = 12.1 eV
E0 = = = 3.15 × 10− 19 J
λ 632.2 × 10− 9
Total energy = E = Pt So, energy of transition from n = 3 to n = 1 corresponds to
energy of incident photons.
∴ nE0 = Pt
26. (b)
Pt 5 × 10− 3 × 2
∴ n= = = 3.2 × 1016 27. (c) hν = hν 0 + KEmax
E0 3.15 × 10− 19

@iitjeehelps
CATHODE RAY, PHOTOELECTRIC EFFECT OF LIGHT AND X-RAYS 339
For first experiment, hc 1 2
35. (a) = φ + mv max
h × 4 × 1015 = hν 0 + x ...(i) λ 2
For second experiment,  hc   1242 eV 
2 − φ 2 − 2.5 eV
 λ   280 
h × 6 × 1015 = hν 0 + 3x ...(ii) ∴ v max = =
15
m 9.1 × 10− 31
From Eqs. (i) and (ii), we get ν 0 = 3 × 10 Hz
 2 × 1.9 × 1.6 × 10− 19 
hc =   = 8.2 × 105 m/s
28. (d) We know that E = hν =  9.1 × 10− 31 
λ
6.6 × 10− 34 × 3 × 108 36. (a) 37. (c)
E=
0.2 × 10− 6 VIBGYOR
38. (a)    → E = n ⋅
hc
1.98 × 10− 25 λ (increases) λ
= = 9.9 × 10− 19 J
0.2 × 10− 6 From equation, Er = Eb (Given)
1 hc hc nr λr λr
Also, E − W0 = mv 2 or nr ⋅ = nb ⋅ or = Q >1
2 λr λb nb λ b λb
1 nr
9.9 × 10− 19 − W0 = × 9.1 × 10− 31( × 105 )2 Hence, > 1 or nr > nb
2 nb
∴ W0 = 9.9 × 10− 19 − 3.52 × 10− 19 39. (c) 40. (c)
W0 = 6.38 ≈ 6 eV 41. (a) Number of photons falling on metal surface,
1 intensity × area
29. (b) hν = hν 0 + mv 2 )max np =
2 energy per quanta
1 hν h
But,  mv 2 = eVS ∴ VS = − ν0 (2 Js−1 m) × (4 m2 )
2  max e e =
(6.626 × 10−34 Js) × (3 × 108 ms−1) 
Hence, it is a straight line.  
 (66.26 × 10− 9 m) 
30. (d)
q Ne = 2.67 × 1018 per sec
31. (b) Current, I = = = nee
t t From equation, ne = 0.1 of n p
where, ne = number of electrons per second 0.1
= × 2.67 × 1018 = 2.67 × 1015 per sec
e = 1.6 × 10−19 C 100
As, each electron is emitted due to absorption of a photon, hc hc hc  1 1
42. (a) eV1 = and eV2 = ⇒ (V1 − V2 ) =  − 
∴ ne = n 2 = number of photons absorbed per second λ1 λ2 e  λ1 λ 2 
I 0.16 × 10−6 hc
⇒ nr = = per sec = 1012 photons/s 43. (a) We know that E1 = −W ...(i)
e 1.6 × 10−19 λ1
And power of source, hc
E2 = −W ...(ii)
P = n1E, n1 = number of photons emitted per second λ2
P
⇒ n1 = From Eqs. (i) and (ii), we get
E E1 + W λ E λ − E2λ 2
hc 12400 ∴ = 2 or W = 1 1
where, E= = eV E2 + W λ1 ( λ 2 − λ 1)
λ 5000
⇒ E = 2.48 eV = 2.48 × 1.6 × 10−19 J = 3.968 × 10−19 J 44. (a)
1
P
⇒ n 1 = = 2.52 × 1015 photons/s 45. (a) m ( 8 × 106 )2 = h(5ν 0 − ν 0 ) ... (i)
E 2
1
n
∴ % of photons absorbed = 2 × 100 and mv 2 = h( 2ν 0 − ν 0 ) ...(ii)
n1 2
1012 × 100 Dividing Eq. (ii) by Eq. (i), we get
= = 0.03968% ≈ 0.04% ( 8 × 106 )2 4ν 0 ( 8 × 106 )2
2.52 × 1015 = ⇒ v2 =
2
v ν0 4
2E − E0 = E0, for 0 ≤ x ≤ 1
32. (c) KE =  0 8 × 106
 2E0, for x > 1 v = ⇒ v = 4 × 106 m/s
h h λ 2
∴ λ1 = and λ 2 = ⇒ 1 = 2 hc
2mE0 4mE0 λ2 46. (b) We know that, E =
λ
33. (a) Let, power = P
34. (b) then, energy in time t

@iitjeehelps
340 SELF STUDY GUIDE BITSAT

Pt Ptλ Dividing Eq. (i) by Eq. (ii), we get


nE = Pt and n = =
E hc 1 1
Ptλ R  − 
∴ nR = …(i)  λ λ0
hc 6=
 1 1
Ptλ V  − 
nv = ...(ii)  3λ λ 0 
hc
6 6 1 1
Dividing Eq. (i) by Eq. (ii), we get − = −
nR λ R 3λ λ 0 λ λ 0
=
nV λ V 1 5
=
We know, λ R > λ V λ λ0
∴ nR > nV λ
λ= 0
5
1 1
47. (b) eV0 = hc  −  ...(i) or λ 0 = 5λ
 λ λ 0
48. (b)
eV0  1 1
= hc  −  ...(ii)
6  3λ λ 0 

BITSAT Archives
1. (c) By Einstein’s photoelectric equation 4. (c) In a photoelectric cell, optical energy is being
KEmax = eVs = hν − hν 0 transformed into electrical energy because light photons
h hν are being absorbed and photoelectric current is being
⇒ Vs =   ν − 0 produced.
e e
h
h 5. (a) λ =
Graph of Vs with ν is straight line whose slope = mv
e
AB AB h So, electron is the lightest particle of these four. So, λ e is
Slope of graph = ⇒ = maximum.
BC BC e
6. (d) de-Broglie wavelength,
2. (b) Number of photoelectrons depends on the intensity of h
light only. λ=
2mE
3. (b) Threshold energy of A is λ1 E2
EA = hν A =
λ2 E1
= 6.6 × 10−34 × 1.8 × 1014
1 × 10−9 E2
= 11.88 × 10−20J ⇒ −9
=
0.5 × 10 E1
11.88 × 10−20
= eV E2
1.6 × 10−19 ⇒ 2=
E1
= 0.74 eV
E2
Similarly, EB = 0.91 eV ⇒ =4
E1
Since, the incident photons have energy greater than EA
but less than EB . ∴ E2 = 4E1
So, photoelectrons will be emitted from metal A only. ∴ Energy to be added = E2 − E1 = 4E1 − E1 = 3E1

@iitjeehelps
30
Atomic Structure

Theory of Atom
Atom is the smallest particle of an element which contains all properties of element. Molecule is a
single atom or a group of atoms joined by chemical bonds. It is the smallest unit of a chemical
compound that can have an independent existence. Nuclei refers to a nucleus of an atom having a
given number of nucleons. It is a general term referring to all known isotopes, both stable and unstable
of the chemical elements. Thus, O16 and O17 are different nuclides.

Rutherford’s Atomic Model


Rutherford and his two associates, H Geiger and E Marsden performed classical a-particle scattering
experiment in order to investigate the structure of the atom.
214
In this experiment, a beam of 5.5 MeV of a-particles emitted from a 33
Bi, a radioactive source was
made to fall at thin metal foil made of gold.

a-Particle Scattering Experiment


Set up for Rutherford’s a-particle scattering experiment is

Gold foil (10–8 m thick)


θ
ZnS screen
α-source
Microscope
Pb screen
The scattered a-particles were observed through a rotatable detector consisting of zinc sulphide
screen and a microscope. It produces bright light flash which can be seen by microscope.

Main Observations
(i) Most of the a-particles pass through the foil straight away (undeflected).
(ii) Some of them are deflected through small angles.
(iii) Few a-particles (1 in 1000) are deflected through the angle more than 90°.

@iitjeehelps
342 SELF STUDY GUIDE BITSAT

(iv) Some a-particles (very few) returned back, i.e. æq ö


Ze 2 cot ç ÷
deflected by 180°. è2 ø
From figure, b=
(v) The number of a-particles scattered at an angle q by 1
4pe0 ´ mv 2
a target is given by 2
N 0nt (2 Ze 2 )2 1 æq ö
Nq = ´ Þ b µ cot ç ÷
4( 4pe0 ) r2 2
(mv02 )2 æq ö è2 ø
sin 4 ç ÷
è2 ø
(ix) If t is the thickness of the foil and N is the number of
where, N 0 = total number of a-particles that a-particles scattered in a particular direction
strike the unit area of the scattered, N
n = number of target atoms per m3 , t = thickness of (q = constant), then it was observed that = constant
t
target, Ze = charge on the target nucleus, 2e = charge N1 t
on a-particle, r = distance of screen from the target Þ = 1
N2 t2
and v0 = velocity of a-particle at nearest distance of
approach.
The number of a-particles scatterd per unit area, N q
q Bohr’s Model
at scattering angle q varies inversely as sin 4 .
2 Bohr added the following postulates to the Rutherford’s
1 model of the atom
i.e. Nq µ 4 (i) The electrons revolve around the nucleus only in
sin (q / 2 )
certain permitted orbits, in which the angular
(vi) In an atom, there is maximum empty space and the momentum of the electron is an integral multiple of
electrons revolve around the nucleus in the same h / 2p , where h is the Planck’s constant.
way as the planets revolve around the sun.
Taking electrostatic force as only force of attraction (ii) The electrons do not radiate energy while revolving
on electrons, in the permitted orbits.
mv 2 kZe × e i.e. the permitted orbits are stationary, non-radiating
= orbits.
r r2
(iii) The energy is radiated only when the electron jumps
where, Z = number of protons of target nucleus
from an outer permitted orbit to some inner
(vii) Distance of closest approach (Nuclear dimension) The permitted orbit. (Absorption of energy makes the
minimum distance from the nucleus upto which the electron jump from inner orbit to outer orbit).
a-particle approaches is called the distance of (iv) If energy of the electron in nth and mth orbits be E n
closest approach (r0 ). and E m respectively, then while the electron jumps
α from nth to mth orbit the radiation frequency n is
r0
Nucleus emitted, such that E n - E m = hn
This is called the Bohr’s frequency equation.
● Radius of the orbit of electron in a hydrogen atom in its
From figure,
stable state, corresponding to n = 1, is called Bohr‘s radius.
1 2 Ze 2 The value of Bohr‘s radius is r0 = 0. 529 Å » 0. 53 Å .
r0 = ×
4pe0 E ● The time period of an electron in orbital motion in the
1 Bohr’s orbit is given by
where, E = mv 2 = KE of a-particle
2 2 p r 2 p ´ 053.
T = = = 1.52 ´ 10- 6 s
(viii) Impact parameter The perpendicular distance of the v c
velocity vector (v) of the a-particle from the centre 137
of the nucleus when it is far away from the nucleus is and the frequency of revolution is
known as impact parameter. It is given as in the 1
f = = 6.5757 ´ 1015cps
figure. T

Some Characteristics of an Atom


α θ (i) The orbital radius of the electron is
b
Nucleus n 2h 2
rn = 4pe0
4 p 2me 2

@iitjeehelps
ATOMIC STRUCTURE 343
(ii) The orbital velocity of electron is n=∞
0
1 2 pe 2 – 0.28 n=7
vn = – 0.38 n=6
4pe0 nh Pfund
– 0.54 n=5

E (eV)
(iii) The orbital frequency is given by – 0.85
Brackett n=4
Paschen n=3
1 v me 4 –1.51
f = = = Infrared
T 2pr 4 e20n 3h 3 Balmer n=2
–3.40
(iv) The total energy of the orbital electron is Visible light
2 Lyman
æ 1 ö é 2 pme 4 ù é 1 ù series
E = -ç ÷ ê ú
è 4 pe0 ø ë h 2 û êë n 2 úû –13.60
n=1
Ultraviolet
me 4 me 4
KE = or PE = -
8n 2h 2 e02 4 n 2h 2 e02 Total number of emission spectral lines from some excited
(v) The velocity of the orbital electron may be written as state n, to another energy n 2 ( < n 1 ) is given by
(n 1 - n 2 )(n 1 - n 2 + 1)
1 é 2 pe 2 ù æ 1 ö c é 1 2 pe 2 ù c
vn = ê ú çè ÷ø = ê . ú= a 2
4pe0 ë h û n n ë 4pe0 ch û n
n (n - 1)
1 2 pe 2 e.g. Total number of lines from n 1 = n to n 2 = 1 are.
Here, = a is called fine structure constant. 2
4pe0 ch The five spectral series of hydrogen atom are given as
It is a dimensionless quantity. below
1
The value of a comes out to be = 0.0073.
137 1. Lyman Series
(vi) The kinetic, potential and total energies of the Spectral lines of Lyman series correspond to the transition
electron varies with r (the radius of the orbit) as
of electron from higher energy levels (orbits) ni = 2 ,3 ,4,¼to
1 é 1 e2 ù 1 e2
follows KE = ê ú, PE = - ground energy level (1st orbit) n f = 1.
2 ë 4pe0 r û 4pe0 r
1 é 1 1 ù
é 1 e2 ù
1 For Lyman series, = v = R ê 2 - 2 ú , where n = 2 , 3 , 4,¼
and E=- l ë (1 ) n û
ê ú
ë 4 p e0 r û
2
It is found that a term Rch = 13.6 eV = 2 .17 ´ 10-18 J. The
Therefore, they are related to each other as
follows term Rch is known as Rydberg's energy.
KE = - E and PE = 2 E
(vii) For a hydrogen atom,
2. Balmer Series
1 1 Electronic transitions from ni = 3 , 4, 5, ¼ to n f = 2, give rise
rn µ n 2 , v n µ and | E | µ 2
n n to spectral lines of Balmer series.
The difference in angular momentum associated Thus, for a Balmer series line
with the electron in the two successive orbits of 1 é 1 1 ù
h nh h = n = R ê 2 - 2 ú , where n = 3 , 4, 5, ¼
hydrogen atom is DL = (n + 1) - = . l ë (2 ) n û
2p 2p 2p

3. Paschen Series
Hydrogen Spectrum Spectral lines of this series lie in the infrared region and
Hydrogen spectrum consists of spectral lines classified as correspond to electronic transition from ni = 4,5,6 ,¼ to
five spectral series of hydrogen atom. Out of these five, n f = 3.
Lyman series lies in the ultraviolet region of spectrum, é 1
1 1 ù
Balmer series lies in the visible region and the remaining Thus, = n = R ê 2 - 2 ú , where n = 4, 5, 6...
three series, lie in the infrared region of spectrum. l ë (3 ) n û

@iitjeehelps
344 SELF STUDY GUIDE BITSAT

4. Brackett Series Thus, ionisation energy of hydrogen atom in the ground


state
It lies in the infrared region and corresponds to transition = E ¥ - E 1 = 0 - ( -13.6 eV ) = + 13.6 eV
from
The potential through which an electron is to be
ni = 5, 6 , 7, ¼ to n f = 4. accelerated so that it acquires energy equal to the
Thus, for Brackett series ionisation energy is called the ionisation potential.
1 é 1 1 ù Therefore, ionisation potential of hydrogen atom in its
= n = R ê 2 - 2 ú, where n = 5, 6 , 7, ¼
l ë ( 4) n û ground state is 13.6 V.

5. Pfund Series
It lies in the far infrared region of spectrum and Excitation Energy and
corresponds to electronic transitions from higher orbits
ni = 6 , 7,8 ,K to orbit having n f = 5. Thus, we have
Potential
1 é 1 1 ù Excitation energy is the energy required to excite an
= n = R ê 2 - 2 ú , where n = 6 , 7, 8 , ... electron from a lower energy level to a higher energy level.
l ë (5) n û The potential through which an electron is accelerated so
● Radius of the nth orbit in hydrogen atom is the as to gain requisite ionisation energy is called the
rn = 0.53 n 2 . ionisation potential.
● If elements with n > 4 were not allowed to exist, then Thus, first excitation energy of hydrogen atom
total number of elements would be
2 ´ [ 12 + 2 2 + 3 2 + 42 ] = 60 = E 2 - E 1 = - 3. 4 - ( - 13 .6 )eV
= + 10.2 eV
Ionisation Energy and Similarly, second excitation energy of hydrogen atom
Potential = E3 - E 1 = - 1.51 - ( -13.6)
Ionisation energy of an atom is defined as the energy = 12 .09eV
required to ionise it, i.e. to make the electron jump from its
present orbit to infinity.

Practice Exercise
1. a-particles are projected towards the nuclei of the 4. Which of the following is incorrect regarding
following metals with the same kinetic energy. Rutherford’s atomic model?
Towards which metal, the distance of closest a. Atom contains nucleus
approach is minimum? b. Size of nucleus is very small in comparison to that of
a. Cu(Z = 29) b. Ag (Z = 47) atom
c. Au (Z = 79) d. Pd(Z = 46) c. Nucleus contains about 90% mass of the atom
d. Electrons revolve around the nucleus with a uniform
2. An a-particle accelerated through V volt is fired speed
towards a nucleus. Its distance of closest approach is
r . If a proton accelerated through the same potential is 5. In Rutherford’s experiment, the number of a-particles
fired towards the same nucleus, then distance of scattered through an angle of 90° is 28 per minute.
closest approach of proton will be Then, the number of particles scattered through an
angle of 60° per minute by the same nucleus is
a. r b. 2r c. r /2 d. r /4
a. 28 per minute b. 112 per minute
3. The distance of closest approach of an a-particle fired c. 12.5 per minute d. 7 per minute
towards a nucleus with momentum p is r . What will be 6. Find the equivalent current due to motion of electron
the distance of closest approach when the momentum in first orbit of H-atom.
of a-particle is 2p?
a. 0.7 ´ 10-3 A b. 9 ´ 10-3 A
a. 2r b. 4r c. r /2 d. r /4
c. 10-3 A d. None of these

@iitjeehelps
ATOMIC STRUCTURE 345
7. If the radius of first Bohr’s orbit is x, then de-Broglie æ 0 . 51 ´ 10-10 ö
wavelength of electron in 3rd orbit is nearly 16. If ç ÷ metre is the radius of smallest
è 4 ø
a. 2px b.6px
electron orbit in hydrogen like atom, then this atom is
c. 9x d. x / 3
a. hydrogen atom b. He+
8. How many times larger is the spacing between the c. Li2+ d. Be3+
energy levels with n = 3 and n = 4, then the spacing
between the energy levels with n = 8 and n = 9 for a 17. How many different wavelengths may be observed in
hydrogen like atom or ion? the spectrum from a hydrogen sample, if the atoms
are excited to third excited state?
a. 0.71 b. 0.41
c. 2.43 d. 14.82 a. 3 b. 4 c. 5 d. 6
18. Find the maximum number of photons emitted by an
9. The circumference of the second orbit of an atom or
-9 H-atom, if atom is excited to states with principal
ion having single electron, is 4 ´ 10 m. The
quantum number four.
de-Broglie wavelength of electron revolving in this a. 4 b. 3 c. 2 d. 1
orbit should be
a. 2 ´ 10-9 m b. 4 ´ 10-9 m
19. In (Q. 18) problem, the minimum number of photons
emitted by the H-atom is
c. 8 ´ 10-9 m d. 1 ´ 10-9 m
a. 1 b. 2 c. 3 d. 4
10. In each of the following atoms or ions, electronic 20. The kinetic energy of an electron in hydrogen atom is
transition from n = 4 to n = 1take place. The frequency 3.40 eV. The minimum energy required to ionise the
of the radiation emitted out will be minimum for hydrogen atom is
a. hydrogen atom a. - 3.40 eV b. 6.40 eV c. - 6.80 eV d. 3.40 eV
b. deuterium atom
c. He+ ion 21. Two H-atoms in the ground state collide inelastically.
d. Li2+ ion The maximum amount by which their combined kinetic
energy is reduced is
11. If an electron is revolving around the hydrogen
a. 10.20 eV b. 20.40 eV c. 13.6 eV d. 27.2 eV
nucleus at a distance of 0.1 nm, what should be its
speed? 22. Calculate the ratio of the frequencies of the long
a. 2.188 ´ 106 m/s b. 1094
. ´ 106 m/s wavelength limits of the Balmer and Lyman series of
hydrogen.
c. 4.376 ´ 106 m/s d. 1. 59 ´ 106 m/s
a. 27 : 5 b. 5 : 27 c. 4 : 1 d. 1 : 4
12. The angular speed of an electron revolving around the 23. For a certain atom, there are energy levels A, B, C
H-nucleus is proportional to corresponds to energy values E A < E B < E C . Choose
a. 1/r b. 1/r 3/ 2 the correct option if l1, l 2, l 3 are the wavelength of
c. 1/r 2 d. r 3/ 2 radiations corresponding to the transition from C to B,
13. Calculate the angular momentum of the electron in B to A and C to A respectively.
l1l 2
third orbit of hydrogen atom, if the angular momentum a. l 3 = l1 + l 2 b. l 3 =
in the second orbit of hydrogen atom is L. l1 + l 2
a. L b. 3L c. l1 + l 2 + l 3 = 0 d. 3l 2 = l 3 + 2l 2
3 2 24. Calculate the energy required to excite an electron in
c. L d. L
2 3 hydrogen atom from the ground state to the next
14. If an electron is moving around a nucleus of charge 2e higher state, if the ionisation energy for the hydrogen
in a circular orbit of radius 10-10 m, then calculate the atom is 13.6 eV.
initial frequency of light emitted by the electron. a. 3.4 eV b. 10.2 eV c. 12.1 eV d. 1.3 eV
15 15 25. Find the wavelength of the emitted radiation, if
a. 4.2 ´ 10 Hz b. 0.36 ´ 10 Hz
c. 3.6 ´ 1015 Hz d. 4.2 ´ 1015 Hz electron in hydrogen atom jumps from the third orbit to
second orbit.
15. An electron of hydrogen atom is revolving in third 36 5R 5 R
Bohr’s orbit (n = 3). How many revolutions will it a. l = b. l = c. l = d. l =
5R 36 R 6
undergo before making a transition to the second orbit
(n = 2). Assume the average life time of an excited 26. Any radiation in the ultraviolet region of hydrogen
state of the hydrogen atom is of the order of 10-8 s. spectrum is able to eject photoelectrons from a metal.
What should be the maximum value of threshold
(Given, Bohr radius = 5.3 ´ 10-12 m)
frequency for the metal?
a. 2.5 ´ 106 revolutions b. 3.5 ´ 106 revolutions a. 3.288 ´ 1015 Hz b. 2.466 ´ 1015 Hz
c. 4.5 ´ 106 revolutions . ´ 106 revolutions
d. 15 c. 4.594 ´ 1014 Hz d. 8.220 ´ 1014 Hz

@iitjeehelps
346 SELF STUDY GUIDE BITSAT

27. Balmer gives an equation for wavelength of visible 34. 29 electrons are removed from Zn-atom (Z = 30) by
kn 2 certain means. The minimum energy needed to
radiation of H-spectrum as l = . The value of k
n -4 2 remove the 30th electron, will be
in terms of Rydberg constant R is a. 12.24 keV b. 408 eV
c. 0.45 eV d. 765 eV
a. R b. 4R c. R /4 d. 4/R
35. An electron of kinetic energy E 0 is scattered by an
28. When an electron jumps from higher orbit to the
atomic hydrogen sample in ground state. Find the
second orbit in He+ ion, the radiation emitted out will minimum value of E 0, so that a photon of wavelength
be in (R = 1.09 ´ 107 m -1) 656.3 nm may be emitted by H-atom.
a. ultraviolet region b. visible region a. 12.09 eV b. 13.6 eV
c. infrared region d. X-ray region c. 14.6 eV d. None of these
29. Deuterium atoms in the ground state are radiated by 36. A H-atom moving with speed v makes a head on
photons of energy 12.8 eV. What will be the energy of collision with a H-atom in rest. Both atoms are in
induced radiation of longest wavelength? Ionisation ground state. Find the minimum value of velocity v for
energy of deuterium is 14.4 eV. which one of atom may excite.
a. 12.8 eV b. 10.8 eV c. 1.6 eV d. 2.00 eV a. 6.25 ´ 104 m/s b. 8 ´ 104 m/s
30. Calculate the ionisation energy of Li2+ atom in ground c. 7.25 ´ 104 m/s d. 13.6 ´ 104 m/s
state. 37. A photon of energy 15 eV collides with H-atom. Due to
a. 13.6 ´ 9 eV b. 13.6 J this collision, H-atom gets ionised. The maximum
c. 13.6 erg d. 13.6 ´ 10-19 kinetic energy of emitted electron is
a. 1.4 eV b. 5 eV
31. The first excitation potential of a given atom is 10.2 V, c. 15 eV d. 13.6 eV
then the ionisation potential is
a. 10.2 V b. 13.6 V c. 30.6 V d. 20.4 V 38. Find the minimum frequency of light which can ionise
a hydrogen atom.
32. For a single ionised helium atom, the longest a. 3.28 ´ 1015 Hz b. 5 ´ 1015 Hz
wavelength in ground state will absorb
c. 91.1 Hz d. None of these
a. 912 Å b. 304 Å
c. 606 Å d. 1216 Å 39. In the case of Compton effect, which of the following is
applicable?
33. If an electron drops from 4th orbit to 2nd orbit in an
a. Energy conservation b. Momentum conservation
H-atom, then
c. Charge conservation d. All of these
a. it gains 2.55 eV of potential energy
b. it gains 2.55 eV of total energy 40. The number of orbitals in 3rd orbit are
c. it emits a 2.55 eV electron a. 3 b. 10
d. it emits a 2.55 eV photon c. 18 d. None of these

BITSAT Archives
1. In hydrogen atom, an electron jumps from bigger orbit 3. If l is the wavelength of hydrogen atom from the
to smaller orbit, so that radius of smaller orbit is transition n = 3 to n = 1, then what is the wavelength
one-fourth of radius of bigger orbit. If speed of electron for doubly ionised lithium ion for same transition?
in bigger orbit was v , then speed in smaller orbit is l
v v a. b. 3l
a. b. 3 [2006]
4 2 [2014] l
c. d. 9 l
c. v d. 2v 9
2. In hydrogen atom, if l1, l 2, l 3 are shortest 4. In Bohr’s atom model, [2005]
wavelengths in Lyman, Balmer and Paschen series a. the nucleus is of infinite mass and is at rest
respectively, then l1 : l 2 : l 3 equals [2013] b. electrons in a quantised orbit will not radiate energy
a. 1 : 4 : 9 b. 9 : 4 : 1 c. mass of electron remains constant
c. 1 : 2 : 3 d. 3 : 2 : 1 d. All of the above

@iitjeehelps
Answer with Solutions
Practice Exercise 7. (b) r3 = r1 ´ n 2 = x ´ 32 = 9x

1. (a) For distance of closest approach, Q 2pr = nl


(KE)a = PE at distance of closest approach 2 pr 2 p ´ 9 x
\ l= = = 6 px
1 q1q 2 n 3
or KE = ×
4pe 0 r r4 - r3 r1 ´ 42 - r1 ´ 32 16 - 9
8. (b) = = = 0.41
where, q1 = charge of a-particle r9 - r8 r1 ´ 9 2 - r1 ´ 82 81 - 64
q 2 = charge of nucleus 9. (a) 10. (a) 11. (d)
1 q1q 2
\ r = × Þ r µ q2 12. (b) We know that,
4pe 0 KE
1 Ze 2 mv 2
Smaller q 2(= Ze ), smaller will be radius of closest approach Q × =
4pe 0 r 2 r
(r).
2. (a) KE = PE æ 1 Ze 2 ö k
\ v = ç × ÷ =
1 q1q 2 è 4 pe 0 mr ø r
or q1V = ×
4pe 0 r æ Ze 2 ö
1 q2 ç Let k = ÷
or r = × = constant in this problem è 4pe 0m ø
4pe 0 V
v k k
Angular speed, w = = = 3/ 2
3. (d) KE = PE r r ×r r
p2 1 q1q 2 1
or = × Þr µ 2 13. (c)
2m 4pe 0 r p
14. (c) We know that,
Hence, momentum of a-particle will be the closest 1/ 2
approach of r/4. mv 2 2e 2 æ 2e 2 ö
= 2
or v = ç ÷
4. (c) r 4pe 0r è 4pe 0mr ø
5. (b) According to Rutherford’s scattering formula, if the 1/ 2
v æ 2e 2 ö 1
a-particle scattered at angle q is directly proportional to Also, frequency = f = =ç ÷ ´
1 2pr è 4pe 0mr ø 2 pr
4
,
sin (q / 2) (1414
. )(9 ´ 109 )1/ 2 ´ 16
. ´ 10-19
\ f =
Then, Nq =
K -31 1/ 2
(9.1 ´ 10 ) ´ 2p (10-10 )3/ 2
sin4(q/ 2) @ 3.6 ´ 1015 Hz
-1
When q = 90° ,Nq = 28 min v3
15. (a) Number of revolutions made in one second =
K 2pr3
Þ 28 = = 4K Þ K = 7
sin ( 45° ) (for n = 3)
7 ( 0.73) ´ 106
Thus, Nq = = = 2437 ´ 1011
sin4(q / 2) 2 ´ p ´ 47.7 ´ 10-11
Hence, the number of a-particles scattered at an angle of Number of revolutions taken in 10-8 s
7 7
60° per minute is N0¢ = = = 2437 ´ 10-8 ´ 1011 @ 2.5 ´ 106 revolutions.
sin4 30° (1/ 2)4
16. (d)
= 7 ´ 16 = 112
n(n - 1)
e e e w ev 17. (d) N =
6. (a) I = = = = 2
T 2p / w 2p 2pr
For third excited state, n = 4
e2 . ´ 10-19 )2
(16
Here, v = = 4( 4 - 1)
2e 0h 2 ´ 8.85 ´ 10-12 ´ 6.63 ´ 10-34 \ N= =6
2
= 2.18 ´ 106 m/s 18. (b) For maximum number,
2 2 -10
r = n r0 = (1) ´ 0 . 53 Å = 0.53 ´ 10 N = n - 1= 4 - 1= 3
. ´ 10-19 ´ 2.18 ´ 106
16 19. (a) When transition takes place from n = 4 to n = 1. One
\ I= = 0.7 ´ 10-3 A
3 ´ 3.14 ´ 0 . 53 ´ 10-10 photon will be emitted.

@iitjeehelps
348 SELF STUDY GUIDE BITSAT

20. (d) Given, KE = 3.40 eV 912


32. (b) l=
æ 1 1ö
So, total energy = - 3.40 eV Z 2ç 2 - 2 ÷
When H-atom is ionised, its minimum total energy will è n1 n 2 ø
become zero. 912
l= = 304 Å
So, ionisation energy = 0 - (–3.40) = 3.40 eV 3
21. (a) 22. (a) 33. (d) 34. (a) 35. (a)
hc hc 36. (a) Forv min, collision should be completely inelastic.
23. (b) EB - EA = 2 Þ EC - EB =
l l2
According to energy conservation principle,
1 1 1 ll 1 1 1
or = + Þ l3 = 1 2 2
mv min = mv 2 + mv 2 + DE …(i)
l 3 l1 l 2 l1 + l 2 2 2 2
24. (b) 25. (a) According to momentum conservation principle,
26. (b) For minimum frequency in Lyman series of H-atom, mv min = mv + mv
transition n = 2 to n = 1takes place. v
\ v = min …(ii)
Now, hn = DE = E2 - E1 2
After solving Eqs. (i) and (ii), we get
é æ 13.6 ö æ 13.6 ö ù 1
or h n = ê ç - 2 ÷ - ç - 2 ÷ ú eV 2
mv min = 2 DE
ëè 2 ø è 1 øû 2
. ´ 10-19
10.2 ´ 16 æ 4DE ö
n= \ v min = ç ÷
6.63 ´ 10-34 è m ø

\ n = 2.466 ´ 1015 Hz Here, DE = minimum excitation energy = 10.2 eV


. ´ 10-27 kg
m = 167
If the threshold frequency of metal is n or less,
photoelectron will come out. . ´ 10-19 ö
æ 4 ´ 10.2 ´ 16
\ v min = ç ÷ = 6.25 ´ 104 m/s
27. (d) è . ´ 10-27
167 ø
28. (b) 37. (a) 38. (a)
29. (d) 30. (a) 31. (b) 39. (b) 40. (d)

BITSAT Archives
1. (d) Radius of nth orbit, rn µ n 2 For Balmer series, n1 = 2, n 2 = ¥
1 R
rn big n2big 4 Þ =
= = (given) l2 4
rn small n2small 1
For Paschen series, n1 = 3, n 2 = ¥
n big
Þ =2 1 R
n small Þ =
l3 9
n small 1
Þ = 1 4 9
n big 2 So, l1 = , l 2 = , l 3 =
R R R
Velocity of electron in nth orbit l1 : l 2 : l 3 = 1: 4 : 9
1 1 æ 1 1ö
vn µ 3. (c) For wavelength, = R Z 2 ç 2 - 2 ÷
n l è n1 n 2 ø
vn big n 1
= small = Here, transition is same
vn small n big 2 1
So, lµ 2
Þ vn small = 2(vn big) = 2v Z
1 æ 1 1ö l H ( Z Li )2 ( 3)1
2. (a) For hydrogen atom, = R ç 2 - 2 ÷ , n 2 > n1 = = =9
l è n1 n 2 ø l Li ( Z H )2 (1)2
l l
For Lyman series, n1 = 1,n 2 = ¥ Þ l Li = H =
1 9 9
Þ =R
l1 4. (d)

@iitjeehelps
31
Nucleus

Nucleus
In every atom the positive charge and mass is densely concentrated at the centre of the atom forming
its nucleus. Nuclear radius is of the order of 10−15 m. In nucleus the number of protons is equal to the
atomic number of that element and the remaining particles to fulfill the mass number are the
neutrons i.e. number of protons = atomic number Z (say) and number of neutrons.

Composition and Size of Nucleus


As mass of an atom is very small, hence we define a new unit of mass, called as 1 atomic mass unit (1u),
1
which is th the mass of one atom of carbon-12.
12
1 u = 1.660539 × 10– 27 kg ~
− 1.66 × 10– 27 kg = 931.5 MeV
A nucleus has a structure of its own. It consists of protons and neutrons. Electrons cannot exist inside
the nucleus. A proton is a positively charged particle having mass (m p ) of 1.007276 u and charge
( +e ) = + 1.602 × 10– 19 C.
Number of protons Z inside the nucleus of an atom is exactly equal to the number of electrons
revolving around the nucleus of that atom. This number is called the atomic number. A neutron is a
neutral particle having mass mn = 1.008665 u. The number of neutrons in the nucleus of an atom is
called the neutron number N . The sum of the number of protons and neutrons is called the mass
number A. Thus, A = N + Z.

Properties of Nucleus
The nucleus properties are described below
● Nucleus size

(a) Size of the nucleus is of the order of fermi (1 fermi = 1015 m )


(b) The radius of the nucleus is given by R = R0 A1 /3
where, R0 = 1.3 fermi and A is the mass number.
(c) The size of the atom is of the order of 10−19 m.

@iitjeehelps
350 SELF STUDY GUIDE BITSAT

● Volume The volume of nucleus is 1. α-decay


4 4
V = π R3 = π ( R0 A1 /3 )3 Anα-particle is a helium nucleus. Alpha(α ) emission occurs
3 3
principally with nuclei that are too large to be stable. When
● Density a nucleus emits an α-particle, its N and Z values each
Mass of nucleus Am p decrease by two and A decreases by four.
(a) Density = =
Volume of the nucleus 4 π ( R A1 /3 )3 Few examples of α- decay are
0
3 238
92 U → 234
90 Th + 2 He
4

mp
= 226
88 Ra → 86
222
Rn + 42 He
4
πR 3
3 0 In general, the decay can be written as
where, m p = 1.6 × 10 −27
kg = mass of proton and
A
ZX → A–4
Z –2 Y + 42 He
R0 = 1.3 fermi. where, X = parent nucleus, Y = daughter nuclues
17 3
(b) Density of nuclear matter is of the order of 10 kg/m .
(c) Density of nuclear matter is independent of the mass 2. β-decay
number. A beta minus particle (β − ) is like an electron. Emission of
β − involves transformation of a neutron into a proton, an
electron and a third particle called an anti-neutrino ( ν ).
Isotopes, Isobars and 1
0n → 11 p + 0
–1 e +ν
Isotones β-decay is spontaneous process with a half-life of about
few days.
Isotopes β − -decay usually occurs with nuclides for which the
Isotopes of an element are nuclides having same atomic neutron to proton ratio( N / Z ratio)is too large for stability.
number Z but different mass number A (or different neutron In β + -decay, a proton changes into a neutron with the
number N). Isotopes of an element have identical electronic emission of a positron (and a neutrino) p → n + e + + ν
configuration and hence, identical chemical properties. β + -decay usually occurs with nuclides for which the
1 2 3 11 12 14
1 H, 1 H, 1 H and 6 C, 6 C, 6 C, etc., are isotopes. neutron to proton ratio (N/Z ratio) is too small for stability.

Isobars 3. γ-decay
Nuclides having same mass number A but different atomic When a nucleus is placed in an excited state, either by
number Z are called isobars. Isobars have different bombardment with high-energy particles or by a
chemical properties. In isobars number of protons Z as well radioactive transformation, it can decay to the ground state
as number of neutrons N differ but total nucleon (or mass) by emission of one or more photons called gamma rays or
number A = N + Z is the same. 31 H, 32He and 614 C, 14
7 N are gamma ray photon, with typically energies of 10 keV to
isobars. 5 MeV. This process is called γ-decay.
In general, the decay can be written as
Isotones A
ZX → AZ X + γ
Nuclides with different atomic number Z and different
All the known conservative laws are obeyed in γ-decay.
mass number A but same neutron number are called
isotones.
Thus, for isotones N = ( A − Z ) is constant. 3 4
and
Radioactivity Decay Law
1 H, 2 He
198 197 According to Rutherford and Soddy, law for radioactive
80 Hg, 79 Au are examples of isotones. decay is as follows.
At any instant, the rate of decay of radioactive atoms is
Radioactivity proportional to the number of atoms present at that instant.
dN
The phenomenon of spontaneous emission of radiations by i.e. − ∝N
dt
heavy elements, is called radioactivity.
dN
An unstable nucleus undergoes a decay to form a stable ⇒ = − λN
dt
nucleus. Three types of decay occurs in nature, which are
defined as follows. So, N = N 0 e − λt

@iitjeehelps
NUCLEUS 351
Activity
It is defined as the rate of disintegration (or count rate) of
Mass Defect and Binding
the radioactive substance (or the number of atoms of any Energy
material decaying per second) ● The difference in mass of a nucleus and its constituent
dN
i.e. R=– = λ N = λ N 0 e − λt nucleons is called the mass defect of that nucleus. Thus,
dt mass defect, ∆M = Zm p + ( A − Z )mn − M
R = R0 e − λt where, M is the mass of a given nucleus.
● Packing fraction of an atom is the difference between
where, R0 = activity at t = 0, R = activity after time t.
mass of nucleus and its mass number per nucleon. Thus,
Units of Activity The SI unit of decay rate is becqueral (Bq) M−A
packing fraction =
but curie (Ci) and rutherford (Rd) are often used in practice. A
1 becquerel = 1 disintegration/s, 1 rutherford = 106 dis/s, ● The energy equivalent of the mass defect of a nucleus is
1 curie = 3. 7 × 10 dis/s.
11 called its binding energy.
Thus, binding energy ∆E b = ∆M c 2
Half-life (T1 / 2 )
= [ ZM p + ( A − Z )M n − M ]c 2
Time interval in which the N
mass of a radioactive If masses are expressed in atomic mass units,
N0
substance or the number of its then ∆E b = ∆M × 931.5 MeV
atom reduces to half of its = [ ZM p + ( A − Z )M n − M ] × 931.5 MeV
initial value, is called the Half-life = T ● Binding energy per nucleon ( ∆E bn ) is the average energy
half-life of the substance.
N0 /2 needed to separate a nucleus into its individual nucleons.
N ∆E b
i.e. if N = 0 , then t = T 1 / 2 N0 /4
Thus, ∆Ebn =
2 N0 /8 A
0 T 2T 3T t
Hence from, N = N 0 e − λt

We have,
N0
= N0 e
− λ(T 1 / 2 )
⇒ T1/ 2 =
log e 2 Nuclear Fission and Fusion
2 λ
0.693 Nuclear Fission
T1/ 2 =
λ The process of a splitting of a heavy nucleus into two lighter
After n half-lives, number of undecayed atoms nuclei of comparable masses (after bombardment with a
n t /T 1 / 2
energetic particle) with liberation of energy, is called
 1  1 nuclear fission.
i.e. N = N0   = N0  
 2  2 Fission reaction of U 235 is
Mean (or average) Life (τ) 92 U
235
→ 0n 1 → 92 U
236
→ 56 Ba 141 + 36 Kr 92
(unstable nucleus)
The time for which a radioactive material remains active, is + 3 0n1 + Q
defined as mean (average) life of that material. It is defined as
the sum of lives of all atoms divided by the total number of The energy released in U 235 fission is about 200 MeV or
atoms. 0.8 MeV per nucleon.
sum of the lives of all the atoms 1
i.e. τ= = Nuclear Fusion
total number of atoms λ
In nuclear fusion, two or more lighter nuclei combine to
form a single heavy nucleus. The mass of single nucleus so
(The average or mean life is the reciprocal of the decay formed is less than the sum of the masses of parent nuclei.
constant) This difference in mass results in the release of tremendous
0.693 amount of energy.
Also, from T1/ 2 =
λ e.g. 1H
2
+ 1 H2 → 1 H3 + 1 H 1 + 4 MeV
1 1 1H
3
+ 1 H → 2 He 4 + 0 n 1 + 17.6 MeV
2
⇒ =τ= ⋅ (T 1 / 2 )
λ 0.693 or 2
+ 1 H 2 → 2 He 4 + 24 MeV
1H
= 1.44 (T 1 / 2 ) For fusion, high pressure (≈ 106 atm) and high temperature
Thus, mean life is about 44% more than that of half-life, (of the order of 107 K to 108 K) is required and so the
which gives us τ > T( 1 / 2 ). reaction is called thermonuclear fusion.

@iitjeehelps
Practice Exercise
1. The radius of Na 23 nucleus is 10. O19 → F19 + e + ν
− 15 − 15
a. 3.125 × 10 m b. 23 × 10 m In this decay, the rest mass energy of O19 and F19 are
c. 11 × 10− 15 m . × 10− 15 m
d. 11 17692.33 MeV and 17687.51 MeV respectively. The Q
factor of the decay is
2. A heavy nucleus (mass number = A) splits into two a. 4.82 MeV b. 7 MeV
new nuclei, whose mass numbers are in the ratio 3 : 2. c. 17.69 MeV d. None of these
The ratio of radii of these new nuclei is
11. The binding energy expressed in MeV is given for the
a. 3 : 2 b. 2 : 3 c. 31/ 3 : 21/ 3 d. 21/ 3 : 31/ 3
following nuclear reactions
3. Find the rest mass energy of electron. 3
+ 0 n1 → 2He4 + 20 MeV
2 He
a. 0.8 MeV b. 1.66 amu
c. 0.5119 MeV d. None of these 2 He
4
+ 0 n1 → 2He5 + 09
. MeV
4. Find the mass of electron in atomic mass unit. Which of the following conclusion is correct?
a. 0.0005498 b. 0.5119 c. 0.5498 d. None a. 2 He4 is less stable than both 2 He3 and 2 He5
27
5. The atomic mass of Al is 26.9815 amu. The mass of b. 2 He4 is less stable than 2 He3 but more stable than
electron is 0.0005498 amu. The rest mass energy of 5
2 He
Al27 nucleus is 4
c. 2 He is less stable than 2 He5 but more stable than
a. 1862 MeV b. 25119.78 MeV 3
c. 25113.12 MeV d. None of these 2 He
4
d. 2 He is more stable than both 2 He3 and 2 He5
6. The atomic mass of B10 is 10.811 amu. Find the
binding energy of B10 nucleus. The mass of electron is 12. Calculate the energy of the reaction,
0.0005498 amu. The mass of proton is
m p = 1.007276 amu. The mass of neutron is Li7 + p → 2 2He4
mn = 1.008665 amu. If the binding energy per nucleon in Li7 and He4 nuclei
a. − 678.932 MeV b. 678.932 MeV are 5.60 MeV and 7.06 MeV, respectively.
c. 378.932 MeV d. None of these a. 19.6 MeV b. 2.4 MeV
c. 8.4 MeV d. 17.28 MeV
7. Find the binding energy of Na 23 . Atomic mass of Na 23
is 22.9898 amu and that of 1H1 is 1.00783 amu. The 13. A sample contains 1 kg O19 nuclei. The sample
mass of neutron = 1.00867 amu. decays according to following equation
a. 931 MeV b. 186.54 MeV O19 → F19 + e + ν
c. 5.38 MeV d. None of these
The mass of sample after one half-life period is
8. The binding energy per nucleon are 53 MeV, 6.2 MeV a. lesser than 1/2 kg b. equal to 1/2 kg
and 7.4 MeV for the nucleus with mass numbers, 3, 4 c. slightly less than 1 kg d. equal to 1 kg
and 5 respectively. If one nucleus of mass number 3
combines with one nucleus of mass number 5 to give 14. The number of C14 atoms in a sample is 100. The
two nuclei of mass number 4, then half-life period of C14 is 5730 year. The number of C14
a. 0.3 MeV energy is absorbed atoms in the sample after 5730 year.
b. 0.3 MeV energy is released a. must be equal to 50 b. must be equal to 100
c. 28.1 MeV energy is absorbed c. may be equal to 90 d. must be equal to 90
d. 3.3 MeV energy is absorbed 15. Half-life of an element A is 25 days. After 25 days,
12
9. What is the binding energy per nucleon of 6C three atoms of A become
nucleus? a. 1 b. 2 c. 3 d. all may be
Given, mass of C12 (mC )m = 12.000 u 16. The half-life of Tc is 6 h. The activity of Tc 99 in a
99

Mass of proton (m p ) = 1.0078 u patient, 60 h after receiving an injection containing this


Mass of neutron (mn ) = 1.0087 u radioisotope is at least 0.125 µCi. What was the
and 1 amu = 931.4 MeV minimum activity (in µci) of the sample injected?
a. 5.26 MeV b. 10.11 MeV a. 1.25 µCi b. 12.5 µCi
c. 15.65 MeV d. 7.68 MeV c. 128 µCi d. 125 µCi

@iitjeehelps
NUCLEUS 353
17. A radioactive sample has an initial activity of 50 dpm, 27. Calculate the ratio of half-life to the mean life of a
20 minute later, the activity is 25 dpm. How many radioactive sample. If λ be the decay constant of a
atoms of the radioactive nuclide were there originally? radioactive sample.
a. 20 b. 1000 c. 1443 d. 2 a. 0.693 b. 0.746
1
18. A radioactive sample has a disintegration rate of c. d. ( 0.693)2
0.693
36 × 10 disintegrations per minute. The sample itself
5

consisting of 10−5 µ mole of the active nuclei. The 28. Calculate the number of β-particles, if a radioactive
disintegration constant, λ is given by element 90 X
238
decays into 83 γ
222
.
−7 −1 −1
a. 6 × 10 s b. 6 × 10 15
s a. 4 b. 6 c. 2 d. 1
c. 6 × 109 s−1 d. 10−8 s−1 29. A radioactive nucleus (initial mass number A and
19. A radioactive element undergoes two B atomic number Z ) emits 3α-particles and 2 positrons.
λ1 the ratio of number of neutrons to that of protons in
different types of radioactive
the final nucleus will be
disintegration, one with disintegration A
A−Z −8 A−Z −4
constant λ 1 and the other with λ 2. The a. b.
λ2 Z −4 Z −8
half-life of the element is
C A − Z − 12 A−Z −4
c. d.
0.693 0.693 Z −4 Z −2
a. b.
λ1 + λ 2 ( λ1 + λ 2 ) / 2 30. How long will it take for 75% of the atoms of a certain
λ 1λ 2 0.693 λ 1λ 2 radioactive element, originally present to disintegrate?
c. 0.693 d.
λ1 + λ 2 2 ( λ1 + λ 2 ) The half-life of the element is 10 days.
a. 240 days b. 3.6 days
20. Half-life period of a given radioactive sample is τ. Its
c. 15.6 days d. 4.15 days
average life would be
τ 1 ln 2 31. For measuring the activity of a radioactive sample, a
a. τ ln 2 b. c. d.
ln 2 τ τ count rate meter is used. At certain observation, count
rate meter recorded 5050 counts per minute but after
21. Choose the correct option, if Tn and Tm denotes the 10 minute later, the count rate showed 2300 counts
half-value period and the mean-value period, per minute. Calculate the disintegration constant ( λ )
respectively of a radioactive element. a. 0.065 per min
a. Tn = Tm b. Tn > Tm c. Tn < Tm d. Tn ≥ Tm b. 0.078 per min
c. 0.24 per min
22. The half-life of radium is 1600 years. Calculate the d. 0.868 per min
number of atoms that will decay from 1 g sample of
−α −α
radium per second (given, atomic weight of radium 32. Consider x → y → z , where half-lives of x and y
= 226)
are z year and one month. The ratio of atoms of
a. 3.6 × 1010 b. 7.2 × 1010 c. 4.2 × 1010 d. 14.6 × 1010 x and y when transient equilibrium [T1/ 2(x ) > T1/ 2( y )]
23. At certain time, the activity of three radioactive has been established is
materials are in the ratio of 3 : 4 : 5. What will be the a. 1 : 22 b. 1 : 26
mark ratio of their activities at any further date? c. 26 : 1 d. 23 : 1
a. 1 : 2 : 3 b. 2 : 3 : 4 c. 3 : 4 : 5 d. 5 : 6 : 8 33. Analysis of potassium and argon atoms in a moon
24. 20% of a radioactive substance decay in 10 days. rock sample by a mass spectrometer shows that the
Calculate the amount of the original material left after ratio of the number of stable Ar 40 atoms present to the
30 days. number of radioactive K 40 atoms is 7 : 1. Assume that
a. 51.2% b. 62.6% c. 15% d. 21.27% all the argon were produced by the decay of
potassium atoms, with a half-life of 1.25 × 109 year.
25. Calculate in how many months, (3/4) th of the How old is the rock?
substance will decay, if half-life of the radioactive
a. 1.25 × 109 yr b. 3.75 × 109 yr
substance is 2 months.
c. 8.75 × 109 yr . × 1010 yr
d. 100
a. 4 months b. 6 months c. 8 months d. 14 months
26. The half-life of a freshly prepared radioactive sample 34. A radioactive sample decays by two different
is 2 hours. If the sample emits radiation of intensity processes. Half-life for the first process is t1 and for the
which is 32 times the permissible safe level, then second process is t 2. The effective half-life is
calculate the minimum time taken after which it would a. t1 + t 2 b. t1 − t 2
be possible to work safely with source. t1 t 2
c. ( t1 + t 2 ) / 2 d.
a. 8 h b. 10 h c. 16 h d. 2 h t1 + t 2

@iitjeehelps
354 SELF STUDY GUIDE BITSAT

37. Calculate the total energy released during a fission


35. reaction.
The alongside is a plot of
binding energy per
1
0n + 235
92 U → 236
92 U → 98
40Zr + 136
52 Te + 210n
nucleon E b , against the Eb B C D E The resulting fission fragments are unstable hence,
nuclear mass decay into stable end products 98 136
A
F 42 M0 and 54 Xe by
M , A, B , C , D , E , F
correspond to different sucessive emission of β-particles. Take mass of
M
nuclei. Consider four neutron = 10087
. 92 U = 236.0526 amu,
amu, mass of 235
reactions. mass of 42 M0 = 979054
98
. amu and mass of
(i) A + B → C + ε (ii) C → A + B + ε 136
54 Xe = 135.9170 amu.
(iii) D + E → F + ε (iv) F → D + E + ε
a. 198 MeV b. 220 MeV
where, ε is the energy released. In which reactions, is c. 185 MeV d. 230 MeV
ε positive?
38. Calculate the energy released per nucleon of the
a. (i) and (iv) b. (i) and (iii) c. (ii) and (iv) d. (ii) and (iii) reactant, in the thermonuclear reaction
235
36. If92 U reactor takes 30 day to consume 4 kg of fuel 31H 2 → 2He4 +1 H1 + 0n1 + 21.6 MeV
and each fission gives 185 MeV of usable energy, a. 21.6 MeV b. 7.2 MeV
then calculate the power output. c. 3.6 MeV d. 1.8 MeV
a. 2.75 × 1010 W b. 0.012 × 1010 W
c. 3.5 × 10 10
W d. 7.63 × 1010 W

BITSAT Archives
1. Let binding energy per nucleon of nucleus is denoted 5. Fusion reaction takes place at high temperature
by E bn and radius of nucleus is denoted as r . If mass because [2006]
number of nuclei A, B are 64 and 125 respectively, a. KE is high enough to overcome repulsion between
then [2014] nuclei
a. rA < rB , EbnA < EbnB b. rA > rB , EbnA > EbnB b. nuclei are most stable at this temperature
4 c. nuclei are unstable at this temperature
c. rA = rB , EbnA < EbnB d. rA < rB , EbnA > EbnB d. None of the above
5
6. Two nucleons are at a separation of one fermi.
2. Half-lives of elements A and B are 1h and 2 h Protons have a charge of + 16 . × 10−19 C. The net
respectively. Which of the following is correct? [2013]
nuclear force between them is F1, if both are neutrons,
a. Element A decays slower F2 if both are protons and F3 if one is proton and the
b. Decay constant of A is smaller other is neutron. Then, [2005]
c. If initial number of nuclei are same, then activity of A
a. F1 = F2 > F3 b. F1 = F2 = F3
is more
d. Mean-life of A is more c. F1 < F2 < F3 d. F1 > F2 > F3
1 7. Mean life of a radioactive sample is 100 s. Then, its
3. If a radioactive substance reduces to of its original
16 half-life (in min) is [2005]
mass in 40 days. Its half-life is [2011] a. 0.693 b. 1
a. 10 days b. 20 days c. 10−4 d. 1.155
c. 40 days d. None of these 8. Consider two nuclei of the same radioactive nuclide.
4. If the total binding energies of 1H2 and 2 He4 , 56 One of the nuclei was created in a supernova
26 Fe
235 explosion 5 billion years ago. The probability of decay
and 92 U nuclei are 2.22, 28.3, 492 and 1786 MeV
during the next time is [2005]
respectively, identify the most stable nucleus out of a. different for each nuclei
the following. [2009] b. nuclei created in explosion decays first
a. 1H2 b. 2 He4 c. nuclei created in the reactor decays first
c. 26 Fe56 d. 92 U235 d. independent of the time of creation

@iitjeehelps
Answer with Solutions
Practice Exercise 9. (d) Carbon nucleus has 6 protons and 6 neutrons. Total
mass of carbon
−15
1. (a) R = R0A 1/ 3
= 1.1 × 10 × ( 23) 1/ 3
( 6mp + 6mn ) = 12.099 amu
− 15
= 3.125 × 10 m and mC = 12.000 amu
1/ 3 mass defect = ∆m = 0.099 amu
 3 
× A Binding energy BE ( ∆m ) 9314
  5 .
1/ 3
r1 r0 × ( A1)1/ 3  A1   31/ 3 = =
2. (c) = =  =  = Nucleons A 12
r2 r0 × ( A2 )1/ 3  A2    × A  21/ 3
2
( 0.099 ) (9314
. )
 5  = = 7.68 MeV
12
3. (c) E = mec 2 = 9.1 × 10− 31 × ( 3 × 108 )2 10. (a) 11. (a)
= 9.1 × 10− 31 × 9 × 1016 12. (d) Binding energy of Li = 39.20 MeV
− 15
= 81.9 × 10 J Binding energy of He4 = 28.24 MeV
− 15
81.9 × 10 Binding energy of 2 He4 = 56.48 MeV
= MeV
1.6 × 10− 19 × 106 ∴ Energy = 56.48 − 39.20 = 17.28 MeV
= 0.5119 MeV 13. (c) 14. (c)
4. (a) E = (mass in amu) × 931 MeV 15. (d) After one half-life, out of three atoms of A, 1, 2, or 3
may decay.
E
∴ mass in amu = t 60
931 MeV 16. (c) Number of half-lives, x = = = 10
t1/ 2 6
0.5119 MeV
= x
 1
931 MeV Now, r = r0  
 2
= 0.0005498 amu x
5. (c) The mass of Al nucleus is ∴ r0 + r 2 = 0.125 µCi × 210 = 128 µCi
m0 = atomic mass − mass of total electrons 17. (c) Rate decreases from 50 to 25 dpm in 20 min, hence
= 26.9815 − 13 × 0.0005498 half-life is 20 min.
= 26.974353 amu 0.693
Now, r0 = λN0 = 1/ 2 × N0
∴ E = m0 × 931 MeV t
r0 × t1/ 2 50 × 20
E = 26.974353 × 931 MeV or N0 = = = 1443
= 25113.12 MeV 0.693 0.693

6. (a) The mass of B10 nucleus 18. (d)


0.693
m0 = 10.811 − 5 × 0.0005498 19. (a) λ eff = λ 1 + λ 2, hence t1 2 =
= 10.808251 amu λ1 + λ 2
Binding energy 20. (b) The relation between half-life (T1/ 2 ) and mean-life ( τ ) is
= {(5mp + 5mn ) − mass of B10 nucleus} × 931MeV τ
. T1/ 2 or T1/ 2 =
τ = 144
ln 2
= {(5 × 1007276
. + 5 × 1008665
. ) − 10.808251} 931 MeV
= (10.079 − 10.808251) 931 MeV 0.693
21. (c) We have, Tn =
= − 678.932 MeV λ
1
7. (b) The mass of nucleus = M Na − 11me Tm = or Tn = 0.693Tm
λ
Binding energy ∴ Tn < Tm
= {11mp + 12mn − (M Na − 11me )} 931 MeV
6.02 × 1023
= {11(mp + me ) + 12mn − M Na} 931 MeV 22. (a) N0 = = 2.65 × 1021
266
= {(11MH + 12mn − M Na ) 931 MeV 0.693
= [(11 × 100783
. + 12 × 100867
. )] − 22.9898} 931 MeV λ= . × 10−11 s−1
= 137
1600 × 3.16 × 107
= (1108613
. + 12.10404 − 22.9898) 931 MeV
We can assume N ≅ N0 as the half-life is much larger.
= 0.20037 × 931 MeV = 186.54 MeV
dN
8. (d) X 3 + Y 5 → 2Z 4 = λN 0
dt
∆E = ( 3 × 5.3 + 5 × 7.4) − 2 ( 4 × 6.2) = 3.3 MeV . × 10−11) ( 2.65 × 1021) × 1 = 3.6 × 1010
dN = (137

@iitjeehelps
356 SELF STUDY GUIDE BITSAT

23. (c) Activity = −


dN
= Nλ λ 1N 10e − λ 1 t λ 1N1
∴ N2 = =
dt λ 2 − λ1 λ 2 − λ1
∴ Activity does not depend upon the time. 0.693 0.693

Hence, the ratio of their activities will be 3 : 4 : 5. N1 λ 2 − λ 1 1 2 × 12 23
∴ = = =
24. (a) Fraction of the substance left N2 λ1 0.693 1
20 2 × 12
= f = 1− = 0.8
100 33. (b) K 40 → Ar 40
Let, t1 = 10 day, t 2 = 30 day Initial number N0 0
t 30 of atoms
fraction, n= 2 = =3 N0 − x x
t1 10 Number of atoms ↓ ↓
N after time t
but = (f )′′ = ( 0.8)3
N0 Initial number of atoms N0 0
N Number of atoms after time t N0 − x x
× 100 = 0.512 × 100 N0 − x 1
N0 But from question, =
x 7
= 51.2%
7
25. (a) x = N0
8
1 1
26. (b) = x ∴ (N0 − x ) =
N0
32 2 8
Time to work safely = 5 × 2 = 10 h N0 N0 N
N0 → → → 0
0.693 2 4 8
27. (a) Half -life =
λ So, time = 3 × T1/ 2 = 3.75 × 10 year
9
1
Average life or mean life = 1 1 1 t +t t1t 2
λ 34. (d) As, λ = λ 1 + λ 2 ⇒ = + = 2 1 or t =
 0.693 t t1 t 2 t1 t 2 t1 + t 2
 
 λ  35. (a) First reaction is fusion and 4th reaction is fission.
Ratio = = 0.693
 1 36. (b) 235 amu of uranium gives energy of 185 MeV.
 
 λ 185
or . × 10− 13 J
× 16
238 − 222 235
28. (d) Number of α-particles emitted = =4
4 also, . × 10− 27 kg
1 amu = 166
and β- particle emitted = 1(as atomic number is 83) . × 10− 27 kg of
so, energy released by 1 amu or 166 235
92 U
29. (b) 30. (d)
. × 10− 13
185 × 16
dM =
31. (b) = λN 0 235
dt
Energy released by 4 kg of 92 U235
N0 dN1 / dt 5050
∴ = = = 2.19 . × 10− 13 × 4
185 × 16
N dN2 / dt 2300 ⇒ W =
. × 10− 27 × 235
166
N0
Also, In = λt = 3.035 × 1014 J
N
3.035 × 1014
In ( 2.19 ) = λt Power of reactor =
30 × 24 × 60 × 60
ln( 2.19 )
λ= = 0.078 per min = 0.012 × 1010 W
10
37. (a) 10n + 235
92 U → 98
42 Mo + 136
54 Xe + 210n
λ 1 N10 −λ t
32. (d) N2 = (e 1 − e − λ 2 t ) (10087
. + 235.0439 ) = (97.9054 + 135.917 + 2.0174)
λ 2 − λ1
∆m = 0.2128
When (T1/ 2 )1 > (T1/ 2 ) 2 at transient equilibrium ∴ Total energy released during a fission reaction
λ1 < λ2 = 0.2128 × 931 MeV = 198 MeV
−λ 2 t −λ1 t
e < <e 38. (c)

@iitjeehelps
NUCLEUS 357

BITSAT Archives
1. (d) r = r0 ( A )1/ 3 N0 = original number of atom
4 n
r increases with increasing A mass number  1  1
  = 
So, rA < rB as mass number of A is smaller Ebn decreases  2  2
with increasing A for A > 56. 56 Fe has highest Ebn value. ⇒ n=4
So, Ebn for A = 64 is larger as compared to Ebn for nucleus 4 half-lives
with A = 125 ∴ 4T1/ 2 = 40
EbnA > EbnB T1/ 2 =
40
= 10 days
2. (c) Let initial number of nuclei of each element = N0 4
0.693 −1 0.693 −2 4. (c) Binding energy per nucleon for 1H2 and 2 He4, 26 Fe56
Decay constants, λ A = h , λB = h
1 2 and 92 U235 are respectively,
λ A > λB 2.22 28.3 492 1786
, , ,
2 4 56 235
Activities, RA = λ A N0
492
RB = λ B N0 Out of them all is the greatest. Therefore, 26 Fe56 is
56
⇒ RA > RB as λ A > λ B the most bound or most stable nucleus.
Less half-life of element A implies faster decay. 5. (a) Fusion reaction takes place at high temperature
1 because KE is high enough to overcome repulsion
Mean-life, τ = between nuclei.
λ
1 1 6. (b) Nuclear forces are charge independent
τA = , τB =
λA λB So, F1 = F2 = F3
τ A < τB as λ A > λ B T 100
7. (d) Half-life,T / 2 = = s
3. (a) From the formula, .
1.44 144
n = 69.44 s
 1
N = N0   69.44
 2 = ≈ 1115
. min
n 100
N  1
= N0   8. (d) Radioactive decay does not depend upon the time of
16  2
creation.

@iitjeehelps
32
Semiconductor Devices
and Logic Gates

Introduction
Energy bands When covalent bond is broken down by valence electron, hole is created in valence
band and valence electron goes to conduction band.
Conduction band has nothing to do with holes. Conduction electrons are found with free flow in
conduction band. Holes exist and flow in valence band. Conduction electrons move twice as fast as
the holes.
Valence
electron
Conduction
band
Energy gap
Valence
band
Hole

Insulators Here, valence electrons are bound very tightly to their parent atom and require a large
energy to remove them.
● Insulators have a full valence band.

● Conduction band is empty.

● It has a large energy gap ( E ), i.e. 6 eV and above.


g

Conduction
band
Large
energy gap Eg
Full valence
band

@iitjeehelps
SEMICONDUCTOR DEVICES AND LOGIC GATES 359
Conductors In the conductors, there are plenty of free (i) Intrinsic semiconductor The pure semiconductor is
electrons for conduction, here valence and conduction called the intrinsic semiconductor. The conductivity
bands overlap each other. of intrinsic semiconductor is because of their own
● Energy gap is absent in conductors. internal charge carriers. At room temperature, few
● No hole is created. covalent bonds break because of thermal collision.
● The flow of current is the flow of electrons. For each broken bond, a pair of a free electron in
conduction band and a hole in valence band is
created. So, intrinsic semiconductor has an equal
Semiconductors number of holes and electrons. Therefore, in an
intrinsic semiconductor, the conductivity is due to
Semiconductors are Conduction
holes and electrons both, which increases with rise
materials whose conductivity band
of temperature. They are also called carriers. Holes
and resistivity lie in between
Eg < 3 eV move in valence band parallel to the direction of
metals and insulators.
external field. Electrons move in anti-parallel
Its resistivity is ρ = 10− 5 to Valence direction to the external field.
106 Ω m −1 and conductivity, band
(ii) Extrinsic semiconductor When a small quantity of
σ = 10+5 to 10−6 Sm −1 . impurity is mixed in a pure or intrinsic
Band gap structure of a
Energy band gap semiconductor, the conductivity of semiconductor
diagram at T=0 K increases. Such an impure semiconductor is called
semiconductor at T = 0 K,
resembles that of a insulator but with a lower energy gap extrinsic semiconductor. The conductivity of
( E g < 3 eV ). resultant crystal depends on the nature and quantity
of the impurity added called dopant.
For silicon, E g ≈ 11
. eV
For germanium, E g ≈ 0. 7 eV In an extrinsic semiconductor, if n e is the number
density of electrons in conduction band and the
At temperature higher than Conduction density of holes in valence band is n h in a pure
T > 0 K, some of the band semiconductor, then
electrons are excited from Eg
valence band to conduction n en h = ni2
band creating an equal Valence Depending upon the nature of impurity added in
number of holes (vacancy band intrinsic semiconductor, the extrinsic
for electrons) in valence semiconductors are of two types
band. (a) n-type semiconductor or donor and (b) p -type
Energy band gap
An increase in temperature diagram at T=0 K semiconductor or acceptor
results in creation of more
electron hole (intrinsic pairs), resulting in an increase in
conductivity of the semiconductor. n-type Semiconductor
Si and Ge are the good examples of semiconductors. Other To prepare an n-type semiconductor a pentavalent
than these, we have inorganic semiconductors such as CdS, impurity, e.g. P, As, Sb are used as a dopant with Si or Ge.
GaAs, CdSe, organic semiconductors such as anthracene Such an impurity is called donor impurity because each
and doped semiconductors such as phalocyanines. For a dopant atom provides one free electron.
pure (intrinsic) semiconductor, the number of free electrons
in conduction band is equals to number of holes in valence +4
Ge
band, i.e. n e = n h = ni , where ni = number of intrinsic pairs.
The highest energy level in the conduction band occupied +4 +5 +4
by electrons at absolute zero temperature is called fermi Ge Sb Ge
level and the energy corresponding to the fermi level is
called fermi energy (E f ). +4
Ge
Free electron

Types of Semiconductors Fig. (a) Formation of n-type semiconductor


Semiconductors are of two types In n-type semiconductor n e >> n h , i.e. electrons are
(i) Intrinsic semiconductor majority charge carriers and the holes are minority charge
(ii) Extrinsic semiconductor carriers, such that n e ⋅ n h = ni2 .

@iitjeehelps
360 SELF STUDY GUIDE BITSAT

An n-type semiconductor is electrically neutral and is not Conductivity and Resistivity of


negatively charged.
Semiconductor
CB (i) Resistivity According to Ohm’s law, at constant
temperature V ∝ I
ρI
Donor i.e. V = IR R=
Free e – Hole energy A
+ Immobile donor positive ion state ⇒ (where, ρ is resistivity)
AR
Fig. (b) Schematic ∴ ρ=
representation of n-type VB I
semiconductor
Fig. (c) Discrete energy levels So, the SI unit of resistivity ρ is Ω-m.
(ii) The conductivity which is reciprocal of resistivity is
given by
p-type Semiconductor 1
σ= …(i)
To prepare a p-type ρ
semiconductor a trivalent +4 σ = e (n e µ e + n h µ h )
impurity, e.g. B, Al, In, Ga, etc., are Ge
So, the SI unit of conductivity σ is Sm −1 .
used as a dopant with Si or Ge.
Such an impurity is called +4 +3 +4 From Eq. (i), the resistivity of the semiconductor is
Ge in Ge also given by
acceptor impurity as each
Hole 1
impurity atom wants to accept an +4 ρ=
electron from the crystal lattice. Ge e(n e µ e + n h µ h )
Thus, effectively each dopant
atom provides a hole. In p-type
semiconductor n h >> n e , i.e. holes Fig. (a)semiconductor
Formation of p-type Effect of Temperature on the
are majority charge carriers and Conductivity of Semiconductors
electrons are minority charge carriers such that n h ⋅ n e = ni2 . The conductivity of a semiconductor is given by
A p-type semiconductor is electrically neutral and is not
σ = e (n e µ e + n h µ h )
positively charged. The number of free electrons in a
semiconductor varies with temperature as T 3 / 2 . As the temperature increases, the mobilities µ e and µ h of
electrons and holes decreases due to the increase in their
CB collision frequency. But due to the small energy gap of
− E / 2 kT
semiconductors, more and more electrons (n ∝ e g )
from the valence band cross over to the conduction band.
Acceptor
Free e – Hole The increase in carrier concentration, n e and n h is so large
energy
– Immobile donor negative ion state that the decrease in the values of µ e and µ h has no
Fig. (b) Schematic influence.
representation of p-type
semiconductor
VB In any semiconductor due to thermal effect, generation of
Fig. (c) Discrete energy levels
free electrons and holes take place. Apart from the process
of generation, a process of recombination also occurs
simultaneously, in which free electron further recombine
Mobility of a Charged Carrier with hole. At equilibrium, rate of generation of charge
Electrical mobility The drift velocity acquired by a charged carrier is equal to the rate of recombination of charge
carrier in a unit electric field is called its electrical mobility carriers.
and is denoted by µ. Larger the value of n e or n h , higher is the probability of their
In a semiconductor, recombination.
Drift velocity of a charged carrier ∝ Applied electric field At equilibrium, n e × n h = ni2
i.e. v ∝ E or v = µ E
v
∴ µ=
E
Semiconductor Diode
A p-n junction is obtained by joining a small p-type crystal
Hence, the electrical mobility (µ) is the drift velocity per
with a small n-type crystal without employing any other
unit electric field. Its SI unit is [m 2 V −1s−1 ].
binding material in between them.

@iitjeehelps
SEMICONDUCTOR DEVICES AND LOGIC GATES 361
Whenever a p -n junction is formed, electrons from I -V Characteristics of Forward
n-region diffuse through the junction into p-region and the and Reverse Bias Diode
holes from p-region diffuse into n-region.
The following circuit diagrams show the arrangement for
As a result of which neutrality of both n and p-regions is studying V -I characteristics of a p -n junction diode.
disturbed and a thin layer of immobile negative charged (i) Forward bias
ions appear near the junction in the p-crystal and a layer of Voltmeter (V)
positive ions appear near the junction in n-crystal. This
layer containing immobile ions is called depletion layer.
The thickness of depletion layer is approximately of the p n
order of 10− 6 m. Milliammeter
(mA) Switch
The potential difference developed across the p-n junction + –
due to diffusion of electrons and holes is called the
potential barrier V b (or emf of fictitious battery). For (ii) Reverse bias
Germanium diode barrier potential is 0.3 V but for Si diode Voltmeter (V)
its value is 0.7 V. The barrier electric field developed due to
it is of the order of 105 Vm –1 .
p n
Forward Bias Microammeter
(µA)
In forward biasing, a positive potential is applied to p -side Switch
and negative potential is applied to n-side. – +

p n
(iii) Typical V-I characteristics of a silicon diode
e– I (mA)
e–
100
V 80
Forward
+ – 60
bias
40
Forward bias 20 VBr
100 80 60 40 20
In forward biasing, in a p -n junction, the following changes V (Volt)
0.2 0.4 0.6 0.8 1.0
occur. VBr (Breakdown 10
(i) Potential barrier reduces. voltage) 20
30
(ii) Width of depletion layer decreases. Reverse
bias I (mA)
(iii) p-n junction provides very small resistance.
(iv) Forward current flows in circuit. ( p to n )
(v) Order of forward current is in mA.
(vi) Forward current is due to the flow of majority charge Applications of p-n Junction
carriers. Diode
Reverse Bias Diode as a Rectifier
p n
In reverse biasing, a positive Junction diode allows current to pass only when it is
potential is applied to n-side e– forward biased. This property is used to rectify alternating
e–
and negative potential to voltages and the circuit used for this purpose is called as
p-side. rectifier and this process of converting AC to DC is known
Due to reverse biasing, the V as rectification. Junction diodes are also used to modify
– +
following changes occur. waveform.
Reverse bias
(i) Potential barrier
increases.
Half-Wave Rectifier
(ii) Width of depletion layer increases. In half-wave rectifier, a p -n junction is used with a
(iii) p -n junction provides high resistance. transformer.
(iv) Very small reverse current flows. (n to p ) In half-wave rectifier of the single phase supply, either the
(v) Order of reverse current is in µA. positive or negative half of the AC wave is passed while the
(vi) Reverse current is due to the flow of minority charge other half is blocked. Because, only one-half of the input
carriers. waveform reaches the output, mean voltage is lower.

@iitjeehelps
362 SELF STUDY GUIDE BITSAT

Half-wave rectifier requires a single diode in a single phase Full-Wave Rectifier


supply. Half-wave rectifier produces more ripple than
full-wave rectifier, and much more filtering is needed to In full-wave rectifier two diodes are used, one for each half
eliminate harmonics of the AC frequency from the output. of the cycle with a centre tapped transformer.Output
frequency of a full-wave rectifier is twice that of input.

Input +
D1
voltage Input AC
AC Vm
at A RL V0
time mains
Input waveform
220 V,
50 Hz
Voltage
Output voltage
across RL
Centre tap D2 –
Time transformer
V0
+ Full-wave rectifier
D Vmax
AC Vm
RL
mains V0 VDC =Vmax
220 V, p
T
50 Hz Resultant output waveform

Transformer Full-wave rectifier have some fundamental advantages


Half-wave rectifier over their half-wave rectifier counterparts. The average
(DC) output voltage is higher than for half-wave, the output
During Positive Half Cycle of the full-wave rectifier has much less ripple than that of
Diode (D) → Forward biased → On switch → Output due to the half-wave rectifier producing a smoother output
waveform.
diode D received.
Since, this rectifier produces an output during both half
During Negative Half Cycle
cycles, so it is 100% efficient.
D → Reverse biased → Off switch → No output due to diode 2V
Here, V DC = max
D received. π
Richardson-Dushmann Equation = 0637
. V max
φ / kT
J = At 2 e During Positive Half Cycle
D1 → Forward biased → On switch → Output due to diode D1
shows relationship between current density J and absolute
D2 → Reverse biased → Off switch → No output due to
temperature T. diode D1
where, φ = work function of emitting surface in eV.
k = Boltzmann constant. During Negative Half Cycle
A = constant = 60 A /cm 2 K for pure metals D1 → Reverse biased → Off switch → No output due to
T = temperature in K. diode D1
Child Langmuir Law For a space charge limited current. D2 → Forward biased → On switch → Output due to diode
D2
l p ∝ (V p )3 / 2
The no-load output DC voltage of an ideal half-wave
Ripple and Ripple Factor
rectifier is In the output of rectifier, some AC components are present.
V peak V peak They are called ripple and their measurement is given by a
V rms = ,V DC = factor, so it is called ripple factor. For good rectifier, the
2 π
ripple factor must be very low.
where, V DC ,V av = DC or average output voltage
(i) For half- wave rectifier ripple factor, r = 1.21
V peak = peak value of the input voltage (ii) For full-wave or bridge wave rectifier ripple factor,
V rms = root mean square value of output voltage. r = 0. 48

@iitjeehelps
SEMICONDUCTOR DEVICES AND LOGIC GATES 363
Before LED can emit any form of light it needs a current to
Special Purpose Diodes flow through it, as it is a current dependent device with
The special purpose diode that performs many different their light output intensity being directly proportional to
functions, e.g. diodes are used to regulate voltage (Zener the forward current flowing through the LED.
diode), to produce light, Light Emitting Diode (LED) are
given below.
Photodiode
Zener Diode It is a special diode used in reverse bias which conducts
only when light of suitable wavelengths is incident on the
It is a highly doped p-n junction diode which is n junction of diode. The energy of incident light photon must
not damaged by high reverse current. It is always be greater than the band gap of semiconductor. Materials
used in reverse bias in break down voltage region used are Cds, Se, Zns.
and is chiefly used as a voltage regulator. p

Zener Diode as a Voltage Regulator Solar Cell


The following circuit is used for stabilizing voltage across a It is a special p-n junction in which one of the
load RL . The circuit consists of a series voltage-dropping semiconductors is made extremely thin so, that solar
resistance R and a Zener diode in parallel with the load RL . radiation falling on it reaches junction of diode without any
+ i R absorption. A solar cell directly converts solar energy into
+ electric energy. Popularly used solar cells, are Ni-cd, PbS
n cell.
Fluctuating Constant
p n Zener VZ RL DC output
DC input
voltage diode p voltage


Junction Transistor
– Transistor is a three terminal device which is formed, when
The Zener diode is selected with Zener voltageV z equal to thin layer of one type of extrinsic semiconductor p or
the voltage desired across the load. The fluctuating DC n-type is sandwitched between two thick layers of other
two types of n-type or two types of p-type layers extrinsic
input voltage may be the DC output of a rectifier. Whenever
semiconductor. Each transistor have three terminals which
the input voltage increases, the excess voltage appears as
are as follows
increased voltage across the resistance R. This causes an
increase in the input current i. This increase is taken away (i) Emitter It is the left most part of the transistor. It
by the Zener diode while the current through the load and emits the majority carriers towards the base. It is
hence the voltage across it remains constant atV z . Likewise highly doped and medium in size.
a decrease in the input voltage causes a decrease in the (ii) Base It is the middle part of transistor which is
input current i. The current through the diode decreases sandwitched by emitter (E) and collector (C). It is
correspondingly, again maintaining the load current. lightly doped and very thin in size.
(iii) Collector It is right part of the transistor which
Light Emitting Diode (LED) collect the majority carrier through base which is
It is a specially designed diode made of GaAsP, GaP, etc. emitted by emitter. It has large size and moderately
When used in forward biased, it emits characteristic, almost doped.
monochromatic light. In reverse biased, it works like a There are two semiconductor junctions, which are defined as
normal diode. below.
I-V Characteristics of LED (a) The junction between emitter and base is known
as emitter-base junction (J EB).
LEDs are current dependent devices with its forward
voltage drop (V F ) depending on the forward biased LED (b) The junction between base and collector is
current. I-V characteristics of LED is given below known as base-collector junction (J CB).
Amber
Yellow
Green
Blue

I (mA)
Red

Infrared Types of Transistors


Transistors are of two types
Forward current

(i) n-p-n transistor If a thin layer of p -type


semiconductor is sandwitched between two thick
layers of n-type semiconductors, then it is known as
n - p -n transistor.
VF

@iitjeehelps
364 SELF STUDY GUIDE BITSAT

n-p-n connected with emitter-base junction enters the


E B C E C
n p n
region. The emitter current (I E ) is approximately
equal to the collector current ( IC ). The base current
is the difference between I E and IC .
B i.e. I E = I B + IC
– + – + It may be pointed out that the arrows point in the
(ii) p-n-p transistor If a thin layer of n-type direction of conventional current or hole current in
semiconductor is sandwitched between two thick spite of the fact that in the n-p -n transistor, the
layers of p -type semiconductor, then it is known as current is carried by electrons.
p-n-p transistor. (ii) Working of p-n-p transistor When emitter-base
p-n-p junction is forward biased, holes (majority charge
E B C E C carriers) in the emitter are repelled towards the base
p n p
and diffuse through the emitter-base junction. The
barrier potential of emitter-base junction decreases
E → Emitter
and holes enter the n-region (i.e. base).
+ – + – B → Base B
C → Collector Emitter Base Collector
p n p
IE IC
Transistor Action C
The action of both types of transistors, i.e.n-p -n and p -n - p Emitter Collector
mA mA
is similar except that the majority and minority charge junction IB junction
carriers in the two cases are of opposite nature.
(i) Working of n-p-n transistor The emitter-base
junction is forward biased and collector-base VEE IE IC VCC
junction is kept reversed biased for n-p -n transistor IE IC
in a circuit. Its symbolic representation is shown in
given figure.
Emitter Base Collector IB
n p n
IE IC + – + –

A small number of holes (≈ 5%) combines with


Emitter Collector electrons of n-region resulting in small current ( I B ).
mA
junction IB junction
mA The remaining holes ( ≈ 95%) enter into the collector
_ region, because they are attracted towards negative
+ _ + terminal of the battery connected with the
VEE IC VCC collector-base junction. These holes constitute the
collector current (IC ).
IE IC
As one hole reaches the collector, it is neutralised by
the battery. As soon as one electron and a hole
neutralised in collector, a covalent bond is broken in
emitter region. As a result, electron-hole pair is
IB produced. The released electron enter the positive
– + – + terminal of battery (V EE ).
When the emitter-base junction is forward biased,
electrons (majority charge carriers) in emitter are Characteristics of a Transistor
repelled towards base. The barrier of emitter-base In common emitter configuration, the variation of current
junction is reduced and the electrons enter the base. on the input side with input voltage ( I E versus V BE ) is
About 5% of these electrons recombine with holes in known as the input characteristics and the variation in the
base region resulting in small current (I B ). The output current with output voltage ( IC versus VCE ) is known
remaining electrons ( ≈ 95%) enter the collector
as output characteristics. From these characteristics, we
region, because they are attracted towards the
obtain the values of following parameters.
positive terminal of battery (VCC ).
● Input resistance
For each electron entering the positive terminal of ∆V BE
the battery connected with collector-base junction, ri =
∆I B V = constant
an electron from the negative terminal of the battery CE

@iitjeehelps
SEMICONDUCTOR DEVICES AND LOGIC GATES 365
● Output resistance  ∆i 
∴ β= C
∆VCE  ∆ iB  V
ro = CE = constant
∆IC I B = constant
The value of β is always greater than 1.
● AC current gain Voltage gain The ratio of change in output voltage to the
DIC change in input voltage is called voltage gain. It is denoted
β = by AV .
DI B V
CE = constant ∆VC ∆i R
Voltage gain, AV = = C × out
The current gain for common-emitter configuration β ∆ V B ∆ iB Rin
ranges from 20 to 200. Rout
● Transconductance
⇒ AV = β ×
Rin
∆IC β
gm = = Power gain The ratio of change in output power to the
∆V BE ri change in input power is called power gain.
A transistor can be used as an amplifier. The voltage gain P
∴ Power gain = out
of an amplifier will be given by Pin
V R
AV = o = β ⋅ C Also,
R
power gain = β 2 × out
Vi RB Rin
where, RC and RB are net resistances in collector and base The relation between α and β is given by
circuits respectively. α β
β= or α =
In common base configuration, AC current gain is defined 1−α 1+β
as
C IC
∆I
α= C IB B
∆I E V = constant Input signal
n-p-n Amplified output
CE
signal voltage
voltage VBB
The value of α is slightly less than 1. In fact,
E
0.95 ≤ α ≤ 1.
IE
Current gains α and β are correlated as VCC
180° out of phase
α β
β= or α = with input signal
1−α 1+β IB IC

Transistor as an Amplifier Transistor as an Oscillator


The process of increasing the amplitude of input signal An electronic oscillator is a device that generates electrical
without distorting its wave shape and without changing its oscillations of constant amplitude and of a desired
frequency is known as amplification. frequency without any external input. The circuit providing
The amplifier is a device which amplifies the power of such oscillation is known as a tank and oscillator using
varying current or alternating current. positive feedback.

Input signal V0 L¢
Amplifier t
C
Output signal Inducting B
n -p - n
coupled B2
As an amplifier, a transistor can be used in the following N
three configurations. C
(i) Common base amplifier L
(ii) Common emitter amplifier
(iii) Common collector amplifier K
Common emitter amplifier The given circuit diagram Some of the properties of the oscillator are
shows an n-p -n transistor which has been used in
1. Oscillator is using positive feedback.
common-emitter mode. The input and output waveforms
2. To work as an oscillator
are shown along with the circuit.
Current gain in CE configuration, at constant collector to |Aβ| = 1, β → feedback factor
emitter voltage ratio of change in collector current the 1 1
3. f = frequency of oscillation = ×
change in base current. It is denoted by β. 2π LC

@iitjeehelps
366 SELF STUDY GUIDE BITSAT

Transistor as a Switch OR Gate


Consider CE configuration of a transistor is shown as It has two inputs, and one output.
below in figure. Logic symbol of OR gate is shown in Fig.(a). Its Boolean
expression is Y = A + B (being read as Y equals A or B).
IC D1
RB 1 A
B C RC A
E Y
IB Vo B Y
V
Vi
IE VCC B R
D2
0

(a) (b)

Applying Kirchhoff’s voltage rule to the input sides of this An OR gate can be realised by using two p-n junction
circuit, we get diodes as shown in Fig. (b).
Truth table of OR gate is
Cut- off region
Vo A B Y
0 0 0
Active Saturation
VCC 1 0 1
region region
0 1 1
1 1 1

AND Gate
Vi It also has two inputs and one output.
0.6 V 1.0 V Boolean expression of AND gate isY = A ⋅ B (being read asY
equals A and B ). Truth table of AND gate is
Vi = I B ⋅ RB + V BE [Vi = DC input voltage] A B Y
0 0 0
and V o = VCC − IC ⋅ R C [Vo = DC output voltage]
1 0 0
Now, we can analyse that how V o changes as Vi increases 0 1 0
from zero onwards. In case of silicon transistor, if Vi is less 1 1 1
than 0.6 V, I B will be zero, hence, IC will be zero and
D1
transistor will be said to be in cut-off state and V o = VCC . 1
A

When Vi becomes greater than 0.6 V, some I B flows, so


some IC flows (transistor is in active state now) and output Y
V
V o decreases as the term IC ⋅ RC increases. With increase in A
Y B
B R
Vi,the IC increases almost linearly and so, V o decreases
0 D2
linearly till its value becomes less than about 1.0 V.
V

Logic Gates (a) (b)

Logic symbol of AND gate is shown in Fig. (a) and the actual
A logic gate is a digital electronic circuit which follows a circuit arrangement used to realise AND gate in practice
logical relationship between its input and output. A logic has been shown in Fig. (b).
gate may have one or more inputs but has only one output.
Logic gates follow Boolean algebra, which consists of three NOT Gate
basic operations, namely AND (A ⋅ B = Y ), OR ( A + B = Y ) It has one input and one output.
and NOT ( A = Y ). Boolean expression of NOT gate A = Y (being read as A
equals Y ).
Three Basic Logic Gates Truth table of the NOT gate is
A Y
On the basis of types of operations, i.e. OR, AND, NOT etc.,
0 1
there are three basic logic gates. These gates with their
detailed descriptions are given below. 1 0

@iitjeehelps
SEMICONDUCTOR DEVICES AND LOGIC GATES 367
– +
If both the inputs of a NAND gate A
V
R are joined together, it will behave Y
as a NOT gate as shown in figure. B
C Y
1 A
Two/Three NAND gates may be NAND as NOT gate
Rb B joined so as to have the same
A Y E effect as a single AND gate or OR gate.
V

0 NOR Gate
If the output of OR gate is joined to input of a NOT gate, the
(a) (b) combination is a NOR gate.
The logic symbol and actual circuit for realisation of NOT Boolean expression of NOR gate is A + B = Y . Its logic
gate have been shown in above figure. symbol is shown in figure.
(i) A + 0 = A (ii) A + A = A
A Y′ = A + B
(iii) A + 1 = 1 (iv) A + A = 1 OR NOT Y
B
(v) A ⋅ 0 = 0 (vi) A ⋅ A = A
(a)
(vii) A ⋅ 1 = A (viii) A ⋅ A = 0
(ix) A + B = A ⋅ B (x) A ⋅ B = A + B A
NOR Y
B
In the above expressions, A and B are to be assigned the
values 0 or 1. (b)

Here, if 0 implies low, then 1 implies high. Similarly, if 0 Truth table of NOR gate is
implies off, then 1 implies on. Also, if 0 stands for down, A B Y
then 1 stand for up vice-versa may also be true. It may be
0 0 1
kept in mind that in digital electronics, 0 and 1 represent
two states and not two values. 1 0 0

0 1 0

1 1 0
Combination of Logic Gates
A
Some common combinations obtained from three Y
fundamental gates OR, AND and NOT are as follows B
NOR as NOT gate
NAND Gate If both inputs of a NOR gate are joined together, it will
If the output of AND gate is joined to input of a NOT gate, behave as a NOT gate as shown in figure. Two/ Three NOR
the combination is a NAND gate. gates may be combined so as to have the same effect as a
A Y ′ = A .B
single OR/AND gate.
AND NOT Y
B NAND and NOR gates are known as universal gate.
(a)
A XOR Gate
NAND Y
B The logic gate which gives high output (1) when either input
(b) A or input B , but not both of them, are high is called
Boolean expression of NAND gate is A ⋅ B = Y . Its logic exclusive OR gate or the XOR gate. If both inputs A and B
symbol is shown in above figure. are either high (1) or low (0), then the output is zero.
Truth table of NAND gate is A Y1= A.B
A NOT AND
A B Y B
A
0 0 1 OR Y Y
B
1 0 1 A
0 1 1 B NOT AND
B Y2= A.B
1 1 0 (a) (b)

@iitjeehelps
368 SELF STUDY GUIDE BITSAT

XOR gate can be obtained by having a combination of The Boolean expressions obey the commutative law,
two NOT gates, two AND gates and one OR gate as shown in associative law as well as distributive law.
Fig. (a). The logic symbol is shown in Fig. (b). Commutative law
Boolean expression of XOR gate is (i) A + B = B + A (ii) A ⋅ B = B ⋅ A
Y =A ⊕ B = A ⋅B + A⋅B Associative law
Truth table of XOR gate is (iii) A + ( B + C ) = ( A + B ) + C
A B Y (iv) ( A ⋅ B ) ⋅ C = A ⋅ ( B ⋅ C )
Distributive law
0 0 0
(v) A ⋅ ( B + C ) = A ⋅ B + A ⋅ C
1 0 1
(vi) A + A ⋅ B = A + B (vii) A + A ⋅ B = A
0 1 1 (viii) A ⋅ ( A + B ) = A (ix) A ⋅ ( A + B ) = A ⋅ B
1 1 0 (x) A ⋅ B = A + B

Practice Exercise
1. If the resistivity of copper is 1.7 × 10−6 Ω-m, then the 6. The dimension of LC is
mobility of electrons in copper, if each atom of copper a. ( s)2 b. volt-s/amp
contributes one free electron for conduction is [the
c. amp-s/volt d. s
atomic weight of copper is 63.54 and density is
8.96 g/cc] 7. Given, β = 49 and Ie = 2 mA, Ic for a BJT is
a. 23.36 cm2/ Vs b. 503.03 cm2/ Vs 2
a. 2 mA b. 1.96 mA c. 98 mA d. mA
c. 43.25 cm2/ Vs d. 88 cm2/ Vs 49
2. The conductivity of a semiconductor increases with 8. A single ended class A transformer coupled amplifier
increase in temperature because delivers a power of 50 mW. VCC = 10 volts. Assuming
a. number density of free current carriers increases ideal conditions, the reflected primary resistance is
b. relaxation time increases given by
c. both number density of carriers and relaxation time a. 2000 Ω b. 1000 Ω c. 500 Ω d. 200 Ω
increase
d. number density of current carriers increases, 9. In a vacuum diode, when the plate voltage is 40 V, the
relaxation time decreases but effect of decrease in plate current is 10 mA. For a plate voltage of 160 V,
relaxation time is much less than increase in number the plate current is
density a. 20 mA b. 40 mA
3. In an n-type silicon, which of the following statement is c. 80 mA d. 160 mA
true? 10. A triode is operating at E c = − 4V, E b = 175V and
a. Electrons are majority charge carriers and trivalent Ib = 10 mA. Around this operating point, the plate
atoms are the dopants characteristics are linear and the parameter values
b. Electrons are minority charge carriers and pentavalent are µ = 20, gm = 2.5 mA/V.
atoms are the dopants If E c is changed to − 6 V with E b = 175 V, the value of
c. Holes are minority charge carriers and pentavalent Ib is given by
atoms are the dopants
a. 10 mA b. 20 mA
d. Holes are majority carriers and trivalent atoms are the
c. 15 mA d. 5 mA
dopants
4. Application of a forward bias to a p-n junction 11. Given the following set of values for a triode, the value
of gm and µ are given below in the table:
a. increases the number of donors on the n-side
b. increases the electric field in the depletion zone Ec Eb Ib
c. increases the potential difference across the − 12 V − 4V − 4V
depletion zone 230 V 230 V 150 V
d. widens the depletion zone
25 mA 35 mA 25 mA
5. In a three phase full-wave rectifier of 50 Hz, the ripple
frequency is a. 1.25 mA/V and 8 b. 0.8 mA/V and 8
a. 300 Hz b. 50 Hz c. 400 Hz d. 600 Hz c. 1.25 mA/V and 10 d. 8 mA/V and 10

@iitjeehelps
SEMICONDUCTOR DEVICES AND LOGIC GATES 369
12. The table gives values of E b , E c , Ib for a triode 17. In figure , V0 is the potential barrier across a p -n
junction, when no battery is connected across the
Eb Ec Ib junction.
1
250 V −35 V 10 mA
2
300 V −45 V 10 mA 3

300 V −35 V 25 mA
V0
The value of r p in k Ω and gm in mA/ V are given by
a. 50 and 0.1 b. 10 and 3.0
c. 5 and 1.5 d. 3.33 and 1.5
a. 1 and 3 both correspond to forward bias of junction
13. A triode is operating at E c = − 4 V, E b = 175 V and b. 3 corresponds to forward bias of junction and 1
Ib = 10 mA. Around this operating point, the plate corresponds to reverse bias of junction
characteristics are linear and the parameters are c. 1 corresponds to forward bias and 3 corresponds to
µ = 20, r p = 8 kΩ. E is changed to − 6 V. To obtain the reverse bias of junction
same plate current the value of E b should be d. 3 and 1 both correspond to reverse bias of junction
a. 40 b. 135 c. 191 d. 215 18. In BJT, maximum current flows in which of the
14. A triode has r p = 10 kΩ and µ = 20. Table gives the following?
operating point with blanks a. Emitter region b. Base region
c. Collector region d. Equal in all the regions
Ec Eb Ib
19. A tungsten emitter works at 2500 K. To increase the
−8 V 250 V 8 mA emission current density by 20%, how much change
Eb 250 V 12 mA in the work function is required (Given, log 2 = 0.3,
1
log 3 = 0.477)
Ec Eb1 8 mA
1 a. 0.016 eV b. 0.039 eV c. 2.54 eV d. 0.254 eV
The values of E b1 and E c1 are 20. In an unbiased p-n junction, holes diffuse from the
p-region to n-region because
a. 290 V, − 6 V b. 210 V, − 10 V
a. free electrons in the n-region attract them
c. 290 V, − 10 V d. 210 V, − 6 V b. they move across the junction by the potential
15. For a vacuum triode, the values of E c , E b and Ib are difference
c. hole concentration in p-region is more as compared
given as under
to n- region
Ec Eb Ib d. All of the above
−1 V 100 V 5 mA 21. Zener breakdown in a semiconductor diode occurs
−1 V 140 V 9 mA
when
a. forward current exceeds certain value
−6 V 200 V 5 mA b. reverse bias exceeds certain value
c. forward bias exceeds certain value
The values of µ and r p are given by d. potential barrier is reduced to zero
a. µ = 12,rp = 10 kΩ b. µ = 12,rp = 20 kΩ 22. The voltage gain of an amplifier state is 100. The gain
c. µ = 10,rp = 20 kΩ d. µ = 20,rp = 10 kΩ expressed in db is
a. 100 b. 20 c. 40 d. 10
16. The temperature (T) dependence of resistivity (ρ) of a
semiconductor is represented by 23. If A = B = 1, then in terms of Boolean algebra what is
not equal to the value of A ⋅ B + A ?
ρ ρ a. B ⋅ A + B b. B + A c. B d. None

a. b. 24. The Boolean equation for the circuit given in figure is

O O A
T T Y
ρ ρ
B
C
c. d.
a. Y = A ⋅ B + C b. Y = A ⋅ (B + C )
O
T
O
T c. Y = A ⋅ (B + C ) d. Y = A ⋅ (B + C )

@iitjeehelps
370 SELF STUDY GUIDE BITSAT

25. In the above circuit, if the polarity is reversed of 33. In a diode vacuum tube, the plate current is 5 mA,
battery, the current flowing would be When the plate voltage is 160 V, a grid is introduced
a. 0 mA b. 2 mA c. 5 mA d. 10 mA between the plate and cathode and a voltage of − 2 V
is applied to it. The plate current will become
26. The given figure shows a rectifier of alternating
current (f = 50 c/s), the number of pulses of rectified (If gm = 5 × 10−3 Ω −1)
current obtained in 1s is a. 20 mA b. 10 mA c. 4 mA d. 7.5 mA
l
34. In common emitter amplifier, the c is 0.98, then the
value of β is le
a. 98 b. 0.98
c. 49 d. None of these
35. For the given circuit of p-n
junction diode, which of R p n
the following statements is
correct? + –
a. In forward biasing the V
voltage across R is V
a. 50 b. 25 c. 100 d. 200 b. In forward biasing the voltage across R is 2 V
13
27. In a sample of pure silicon 10 atom/ cm 3 is mixed of c. In reverse biasing the voltage across R is V
phosphorus. If all atoms are active, then what will be d. In reverse biasing the voltage across R is 2 V
the resistivity at 20° C if mobility of electron is 36. When forward bias is applied to a p-n junction, what
1200 cm 2 / V-s? happens to the potential barrier VB and the width of
a. 0.5209 Ω-cm b. 5.209 Ω-cm charge depleted region x?
c. 52.09 Ω-cm d. 520.9 Ω-cm
a. VB increases, x decreases
28. For the given combination of gates, if the logic states b. VB decreases, x increases
of inputs A, B and C are as follows. A = B = C = 0 and
A = B = 1, C = 0, then the logic states of output D are c. VB increases, x increases
A d. VB decreases, x decreases
B 37. Carbon, silicon and germanium atoms have four
valence electrons each. Their valence and conduction
D bands are separated by energy band gaps
C
a. 0, 0 b. 0, 1 c. 1, 0 d. 1, 1 represented by (E g )C , (E g )Si and (E g )Ge, respectively.
Which one of the following relationship is true in their
29. The plate voltage of a triode is increased from 225 V case?
to 250 V, the grid voltage is changed from 4 to − 4.7 V a. (Eg )C > (Eg )Si b. (Eg )C = (Eg )Si
to maintain plate current constant. The amplification
factor of the tube is c. (Eg )C < (Eg )Ge d. (Eg )C < (Eg )Si
a. 35.7 b. 2 c. 70 d. 20 38. What is the plate current in a diode valve under the
30. In a triode amplifier, the load resistance is equal to the space charge limited operation, when the plate
plate resistance r p . If µ is the amplification factor, the potential is 60 V ? In a diode valve, the plate current is
stage gain of the amplifier is 320 mA, when the plate potential is 240 V.
µ µ a. 30 mA b. 20 mA c. 40 mA d. 10 mA
a. b. µ c. 2 µ d.
2 4
39. In the following common emitter configuration, an
31. The triodes P and Q have the same amplification n-p-n transistor with current gain β = 100 is used. The
factor 40. Their plate resistances are 4 kΩ and 8 kΩ, output voltage of the amplifier will be
respectively. If an amplifier circuit is designed using
anyone of them and a load resistance is of 8 kΩ, the
ratio of the voltage gain obtained from them will be 10 kW
a. 2 : 3 b. 4 : 3 c. 3 : 4 d. 1 : 2 1 kW
10 mV V
32. When the plate voltage of a triode is 150 V, its cut-off
voltage is − 5 V. On increasing the plate voltage to
200 V, the cut-off voltage can be
a. − 4.5 V b. − 5.0 V
c. + 2.3 V d. − 6.66 V a. 10 mV b. 0.1 V c. 1.0 V d. 10 V

@iitjeehelps
SEMICONDUCTOR DEVICES AND LOGIC GATES 371
40. In the case of forward biasing of p-n junction, which 45. The exclusive-OR of A, B is represented by A ⊕ B . An
one of the following figures correctly depicts the equivalent form is
direction of the flow of charge carriers? a. AB + A B b. AB + A + B
V VB c. AB + AB d. None of these
p n p n
a. b. 46. Given the truth table relating Y to A, B .
A B Y
VB 0 0 1
0 1 0
p n
1 0 0
c. d. None of these
1 1 0
Then, Y is given by
41. In a forward biased p-n junction diode, the potential a. A + B b. AB c. AB d. A + B
barrier in the depletion region will be of the form
47. Given the following truth table, where A, B are inputs
n p
p n and Y the output
Potential Potential
barrier barrier A B Y
a. b.
0 0 1
1 0 1
0 1 1
n p n
1 1 0
Potential Potential
p barrier barrier
c. d. The output Y is
a. AB b. AB c. AB d. AB
48. What will be the input of A and B for the Boolean
42. The input resistance of a common emitter transistor expression ( A + B ) ⋅ ( A ⋅ B ) = 1?
amplifier, if the output resistance is 500 kΩ, the a. 0, 0 b. 0, 1 c. 1, 0 d. 1, 1
current gain α = 0.98 and the power gain is
6.0625 × 106 is 49. Which of the following gates will have an output of 1?
1 0
a. 198 Ω b. 300 Ω a. b.
c. 100 Ω d. 400 Ω 0 1

43. In a figure given A R D1 0 0


alongside assuming –10 V c. d.
1 1
the diodes to be ideal
D2 50. Current in the circuit will be
a. D1 is forward biased
and D2 is reverse B
biased and 20 W
hence, current flows from A to B 30 W
b. D2 is forward biased and D1 is reverse biased and
hence, no current flows from B to A and vice-versa 5V
20 W
c. D1 and D2 are both forward biased and hence current
flows from A to B
a. 5 / 40 b. 5 /50 c. 5 /10 d. 5 / 20
d. D1 and D2 are both reverse biased and hence no
current flows from A to B and vice-versa 51. Find the value of VAB .
44. In the circuit shown in figure, if the A 0.2 mA
diode forward voltage drop is 0.3 V,
10 W
then the voltage difference between 5 kΩ + + –
A to B is, 30 V –
a. 1.3 V
VAB 10 W 10 W
b. 2.3 V
c. zero 5 kΩ
d. 0.5 V a. 10 V b. 20 V
B c. 30 V d. None of these

@iitjeehelps
372 SELF STUDY GUIDE BITSAT

52. A two volt battery forward biased and a diode. a. OR gate b. AND gate
However, there is a drop of 0.5 V across the diode c. XOR gate d. None of these
which is independent of current. Also, a current 55. The circuit shown below will act as
greater than 10 mA produces large joule loss and
damages diode. If diode is to be operated at 5 mA, A
then the series resistance to be put is y
2V B
+ –
K a. AND gate b. OR gate
c. NAND gate d. XOR gate
+ – R
56. In Boolean algebra, A + B = Y implies that
a. 3 kΩ b. 300 kΩ c. 300 Ω d. 200 kΩ a. sum of A and B is Y
b. Y exists when A exists or B exists or both A and B
53. Which of the following statements concerning the exist
depletion zone of an unbiased p-n junction is(are) c. Y exists only when A and B both exist
true? d. Y exists when A or B exist but not when both
a. The width of the zone is independent of the densities A and B exist
of the dopants (impurities)
b. The width of the zone is dependent on the densities 57. In the Boolean algebra, which of the following is
of the dopants wrong?
c. The electric field in the zone is produced by ionised a. 1 + 0 = 1 b. 0 + 1 = 1
dopant atoms c. 1 + 1 = 1 d. None of these
d. The electric field in the zone is provided by electrons
in the conduction band and the holes in the valence 58. The output of a 2-input OR gate is fed to a NOT gate,
band the new gate obtained is
a. OR gate b. NOT gate c. NOR gate d. XOR gate
54. The circuit shown in figure below will act as
59. Digital circuit can be made by the repetitive use of
A Y¢
a. OR gates b. AND gates
B Y
c. NOT gates d. NAND gates

BITSAT Archives
1. The circuit is equivalent to C C
+ – + –
+ – + –
A Y p n
B [2013] n p
a. AND gate b. OR gate
c. Not gate d. None of these R R
2. Active state of n-p-n transistor, in circuit is achieved (i) (ii)
by [2013] CV CV VC VC
a. , b. CV , CV c. , VC d. VC,
a. low input voltage b. high input voltage e e e e
c. both a and b d. Neither a nor b 5. A transistor is preferable to a triode valve when used
3. If a semiconductor has an intrinsic carrier in amplifier because it
concentration of 141 . × 1016 / m 3 when doped with (i) can withstand large changes in temperature
21 3
10 /m phosphorous atoms, then the concentration (ii) has a higher input impedance
(iii) can handle larger powers
of holes / m 3 at room temperature will be [2009]
(iv) does not require powers
a. 2 × 1021 b. 2 × 1011 . × 1010 d. 141
c. 141 . × 1016
which of the above statements is correct? [2009]
4. Two identical capacitors each of capacitance C are a. Only (i), (ii) and (iii) are correct
charged to the same potential V and are connected in b. Only (i) and (iii) are correct
two circuits (i) and (ii) at t = 0 as shown. The charges c. Only (ii) and (iv) are correct
on the capacitor at t = CR are [2009] d. Only (vi) is correct

@iitjeehelps
Answer with Solutions
Practice Exercise 9. (c)Vp = 40 V, I p = 10 mA,V ′ p = 160 V
σ (Vp )3
1. (c) Mobility of electron, µ = …(i) Ip
ne ∝ 2
1 I ′p 3
Resistivity, ρ = …(ii) (V ′p ) 2
σ
3
From Eqs. (i) and (ii), we get 10  40  2
= 
µ=
1
…(iii) I ′p  160
neρ 3

n = number of free electrons per unit volume I ′p = ( 4) 2 × 10 = 80 mA


N0 × d 6.023 × 1023 × 8.96 10. (d) Ec 1 = − 4 V, Ib = 10 mA, Eb = 175 V, µ = 20,
n= =
atomic weight 63.54
gm = 2.5 mA/V
= 8.5 × 1022 …(iv) Ec 2 = − 6 V, Eb = 175 V
From Eqs. (iii) and (iv), we get At constant plate potential 175 V
1  ∆I 
µ= = 43.25 cm 2/ Vs gm =  b 
. × 10−19 × 17
8.5 × 1022 × 16 . × 10−6  ∆Ic  V
b
2. (d) 3. (c) 4. (a) −3 ∆Ib
2.5 × 10 =
5. (a) For full-wave rectifier. −4+ 6
For single phase, ripple frequency 5 × 10− 3 = ∆Ib
= 2 × f ( f = frequency of input) Ib − Ib ′ = 5 mA
f ′ = 2 × 50 = 100 Hz (frequency of supply) Ib ′ = 10 − 5 = 5 mA
For 3-phase, ripple frequency  ∆Vp  80
= 3 × f ′ = 3 × 100 = 300 Hz 11. (c) µ =   =
1  ∆Vg  I 8
6. (d) At resonance, ωL = p

ωC µ = 10
1  ∆I p 
ω2 = 10 mA
LC gm =   = = 125
. mA/V
1  ∆Vg  I 8
⇒ ω= p

LC  ∆Vp  50
1 12. (d) rp =   = = 3.33 kΩ
2πf =  ∆I p  V 15
LC g

Thus, dimensions of LC is second.  ∆I p  15


gm =   = = 1.5 mA/V
7. (b) β = 49, Ie = 2 mA, Ic = ?  ∆Vg  10
I I
β = c,α = c  ∆V 
Ib Ie 13. (d) µ =  b 
β 49 49  ∆Vg  I
α= = = p

1 + β 1 + 49 50 ∆Vb = 20 × 2 = 40
49 Ic
= ∆Vb = Vb ′ − Vb = 40
50 Ie Vb ′ = 175 + 40 = 215
49 × 2  ∆Vp 
Ic = mA 14. (d) rp =  
50  ∆Ib 
Ic = 1.96 mA 250 − Eb1
2 10 k Ω =
V 4 mA
8. (b) For ideal condition, P =
R+R 10 × 4 = 250 − Eb1
10 × 10 Eb1 = 210 V
2R = = 2000
50 × 10− 3 ∆Vp 40
µ= =
R = 1000 Ω ∆Vg Ec 1 + 8

@iitjeehelps
374 SELF STUDY GUIDE BITSAT

8 + Ec 1 =
40 32. (d)Vp1 = 150 V,Vp 2 = 200 V, Vg1 = − 5 V,Vg 2 = ?
20
µ = amplification factor of a triode valve, it is a
8 − 2 = − Ec 1 characteristic of valve remains constant.
Ec 1 = − 6 V So long as I p is constant.
 ∆V   ∆V  Vp Vp
15. (d) µ =  b  =  b  (Max. ∆Vp ) ∴ µ=− 1 =− 2
 ∆Vg  I  ∆Vc  I Vq1 Vg 2
b b

200 − 100 150 200


= = 20 =
5 − 5 Vg 2
 ∆Vp  20
140 − 100 40 Vg 2 = − = − 6.66 V
rp =   (Min. ∆Vp ) = = = 10 kΩ 3
 ∆I p  V 9 −5 4
g 33. (c) Vg1 = 0
16. (c) 17. (b) 18. (a) I p = plate current = 5 mA
19. (b) 20. (c) 21. (b) Vp = plate voltage = 150 V
22. (c) Vg = grid voltage = − 2 V
23. (d) AB + A = 1⋅ 1+ 1 = 1+ 1 = 1. When Vg = 0, it is a diode
Same is true for B ⋅ A + B as well as B + A. When Vg = − 2, it is a triode
 ∆I p 
24. (d)Y = A ⋅ (B + C ) ∴ gm =  
 ∆Vg  V
25. (a) 26. (c) p = constant

1 I −I I − 5 × 10− 3
27. (d) = e (neµe + nhµh ) = 520.9 Ω- cm [By putting values] 5 × 10− 4 = 2 1 = 2
ρ V2 − V1 −2−0
28. (d) The output D for the given combination − 10 × 10− 4 + 5 × 10− 3 = I2
D = (A + B)⋅C = (A + B) + C − 1 × 10− 3 + 5 × 10− 3 = I 2
I 2 = 4 mA
If, A = B = C = D, then D = ( 0 + 0) + 0 = 0 + 0 = 1 + 1 = 1
34. (c) 35. (a) 36. (d) 37. (a)
If A = B = 1, C = 0, thenD = (1 + 1) + 0 = 1 + 0 = 0 + 1 = 1
38. (c) Using Child Langmuir law, I ∝ V 3/ 2

29. (a)Vp1 = 225 V,Vp 2 = 250 V,Vg1 = − 4 V, 320 ∝ ( 240)3/ 2 …(i)


Vg 2 = − 4.7 V I ∝ ( 60)
3/ 2
…(ii)
 ∆Vp  25 Dividing Eq. (i) by Eq. (ii), we get
µ =   = = 35.7

 g I
V 0.7 320
= ( 4 )3 / 2 ⇒ I =
320
⇒ I = 40 mA
p
I 8
µ
30. (a) A = 39. (c) 40. (c) 41. (d)
rp
1+ 42. (a) Power gain = Current gain × Voltage gain
R
R = rp R
Voltage gain, A V = β 2
µ R1
A=
2 α 0.98
Current gain β = = = 49
31. (b) µ = 40 = amplification factor 1 − α 1 − 0.98
µP = µQ = 40  500 × 103 
AV = 49  
rP = 4 kΩ, rQ = 8 kΩ  R1 
RL = 8 kΩ Power gain, R1 = 6.0625 × 106
µRL
Voltage gain for triode A =  500 × 103 
rP + RL = 49 ×   × 49 = 198 Ω
 R1 
40 × 8
For triode P = AP =
4+ 8 43. (b) IfVp − Vn > 0, then diode is in forward biased. In case
40 × 8 of D1,Vp − Vn = − 10 − 0 = − 10 < 0, so D1 is in reverse
AQ = biased and for D2, Vp − Vn = 0 − ( −10 ) = 10 > 0, so D2 is in
8+ 8
forward biased.
40 × 8
AP 4 + 8 16 4 44. (b) Applying KVL across points A and B,
= = =
AQ 40 × 8 12 3 VA − 0.2 × 10−3 × 5 × 10 3 − 0.3 − 0.2 × 5 − VB = 0
8+ 8 ⇒ VA − VB = 2.3 V

@iitjeehelps
SEMICONDUCTOR DEVICES AND LOGIC GATES 375
45. (c) Exclusive OR, 49. (c) For option (c), it is a NAND gate, its output = 0.1 = 0 = 1
Y = A + B , Y = AB + AB 50. (b) Equivalent circuit is
46. (d) A + B = 0 + 0 = 1 30 Ω
+ −
= 0 + 1= 0 = 1+ 0 = 0 = 1+ 1= 0

A B Y
20 Ω 5V
+ −
0 0 1

0 1 0 V 5
Req = 50 Ω ⇒ I = =
1 0 0 R 50

1 1 0
51. (a) Equivalent circuit is
∴ Req = 15 V
47. (d) AB = ( 0.0) = 0 = 1 V 30
A Circuit current, I = = = 2A
Y=AB R 15
AB = (10
. )= 0 =1 B

AB = ( 0.1) = 0 = 1 10 Ω
+
AB = (11
.)= 1=0 30 V
– 10 Ω 10 Ω
48. (a) The given Boolean expression can be written as
Y = ( A + B ) ⋅ ( AB ) = ( A ⋅ B ) ⋅ ( A + B )
So, VAB = I AB ⋅ RAB = 1 × 10 = 10 V
= ( A ⋅ A ) ⋅ B + A (B ⋅ B ) = A ⋅ B + A ⋅ B = A ⋅ B
52. (c) By KVL, 2 = 0.5 + IR = 0.5 + 5 × 10−3R
So, the truth table is 1.5 1500
∴ R= ,R = = 300 Ω
A B Y 5 × 10−3 5

0 0 1 53. (b) 54. (a) 55. (a)


1 0 0 56. (d) 57. (a) 58. (c)
0 1 0 59. (d) The repetitive use of NAND and NOR gate gives digital
circuits.
1 1 0

BITSAT Archives
1. (b)Y = A + ( A ⋅ B ) = ( A + A ) ⋅ ( A + B ) 3. (d) Doping will increase the number of electrons only and
= t ⋅ ( A + B ) = A + B OR gate not the holes. So, number of holes will be equal to
number of intrinsic carrier concentration.
Here, t has value 1 for all inputs.
= 1.41 × 1016 / m3
2. (d) Transfer characteristic of transistor is
4. (c) In figure (i) the p-n junction diode is forward biased
V0 and represents a very low resistance, the capacitor,
Cut-off state
therefore discharge itself through resistor R according to
Active state relation,
q = q 0e −1/CR and q 0 = CV at t = CR
CV
∴ q = q 0e −1 =
Saturation e
state
In figure (ii), the p-n junction diode is reverse biased, the
V
Low intermediate High i capacitor therefore holds the charge intact.
input input input
voltage voltage voltage ∴ q = q 0 = CV
5. (d) A transistor is preferable to a triode valve when used
Active state is achieved at intermediate input voltage. in amplifier because it does not require a power.

@iitjeehelps
33
Universe

Introduction
1. Solar system It includes the Sun at centre, eight planets (Mercury, Venus, Earth, Mars, Jupiter
Saturn, Uranus and Neptune) revolving around the Sun with 59 natural satellites revolving
around the planets.
Mercury and Venus have no satellite, while Saturn have maximum number of satellites
twenty one (21).
(a) Stars These are very much like the Sun. They may occur as single stars, binary stars or in
clusters.
(b) Galaxies A large group of stars is called a galaxy. The galaxy containing our Sun and hundred
billion (10 11 ) stars is called milky way. There are billions of galaxies. Galaxies may be spiral,
elliptical and irregular in shape. Most galaxies are spiral like milky way.
2. Mass (M) of Milky way is given by
v 2 GM v 2r
= 2 , i.e. M =
r r G
where, v = orbital speed of Sun.
r = distance of Sun from centre of galaxy.
The mass M comes out to be nearly 3 ´ 1041 kg.
3. Asteroids The groups of small objects revolving around the Sun between orbits of Mars and
Jupiter are called asteroids. It is believed that asteroids are pieces of planets, possibly due to
gravitational effect of Jupiter.
4. Comets A small rock like material surrounded by large masses like water, ammonia and
methane and revolving in highly elliptical orbits around the Sun are called comets.
5. Meteors and meteorites These objects which get completely burnt and blazing through the
atmosphere are called meteors. The larger objects which are not completely burnt before
reaching the earth are called meteorites.
6. Colour of a star The colour of a star is determined by Wien's displacement law, according to
which
l m = constant
where, l m = wavelength corresponding to maximum energy of emission.
Accordingly, a blue star is hottest and the red star is coolest.

@iitjeehelps
UNIVERSE 377
The expanding universe and observed red shift are in compressed core is made only of neutrons. Neutron stars
favour of Big-bang theory. produce very high magnetic fields. A spinning neutron star
emitting electromagnetic waves is called Pulsar.
7. Pulsating theory According to this theory, the
boundary of universe moves in and out with a period Black Hole
of 8 ´ 109 years. If the original mass of the star is greater than 5 solar masses,
8. The steady state theory According to this theory, the the core dies into black holes. The mass of a black hole is
total number of galaxies in the universe has attained a greater than the mass of the Sun but its size is very small,
steady value. If one is lost due to annihilation in therefore the gravitational pull of a black hole is so strong
observable part of universe, one new is formed. This that even the photon or radiation emitted by it, cannot
theory is based on interconversion of mass and energy. escape from its surface. Since, no radiation is received from
a black hole, it cannot be seen through a telescope.
9. Hubble's law It states that the speed of recession v is
related to distance (r ) of galaxy by the relation
The Milky Way
v = Hr
The Milky way or Akash-ganga is the name of the galaxy to
where, H is called Hubble's constant. Its value
which our solar system belongs. It is composed of a few
1017 kms-1 / million light years.
hundred billion stars including our own Sun. The
systematic study of the Milky way was undertaken in the
Star eighteenth century by William Herschel, the same Herschel
who discovered the planet Uranus.
Dust particles, hydrogen and helium gas molecules present
in the intersteller space first come together (at about To the naked eye, Milky way looks like a stream of milk
—173°C) to form a cloud. Then, they start contracting. As a across the sky. Some of the important features of Milky way
result of compression, heating of cloud takes place. When are given below
temperature of the core becomes about 10 7 K, the fusion of Central region
hydrogen atoms is initiated with the release of energy. This
energy keeps the star shining for millions of years.

Death of a Star
A star lasts until the hydrogen in the core of the star is (i) Milky way is a convex lens shaped disc with a thick
exhausted. The core now starts contracting, it results in the centre and thinning outwards.
rise in temperature of the star. As a result of rise in (ii) The Milky way contains about 150 billions of stars.
temperature, the outer layer of the star expand. Expansion of
outer layer brings about cooling effect in them. The process (iii) The space in the Milky way is filled with dust and
continues till the temperature of outer layer falls enough to gases called the interstellar matter. About 90% of the
intersteller matter is in the form of hydrogen.
make the star appear red. It is then called red giant.
At this stage, a violent explosion called nova or supernova (iv) Some regions of the Milky way appear dark not
occurs in the star. Due to the explosion its outer layers are because they are empty but because the intervening
thrown into intersteller space leaving behind the core of the dust and gas obstructs the light from the stars behind
star. The core of the star may further end up into one of the them. These dark regions are called dark nebulae.
following three steller dead materials. (v) Sometimes the regions of dust show a great variety
of brightness. They shine in the light of the nearby
White Dwarf stars and such bright regions of dust are called bright
The core dies as white dwarf, if the original mass of the star nebulae. Orion is the example of bright nebulae.
was 1.4 solar masses. It was discovered by S Chandrasekhar
(vi) A remarkable feature of the Milky way is its rotation
in 1930 and it is known as Chandrasekhar limit. The core is
about its centre. This rotation is not rigid.
composed of protons and electrons. The core keeps on
emitting heat and light for millions of years. As it cools Our Sun, alongwith the planetry system is revolving
steadily, its colour changes from white to yellow, then to around the centre of the Milky way with a speed
red and finally it becomes black. It then becomes invisible v = 250 km/s. It takes about 250 million years for the
forever as black dwarf and neither emits heat nor light. Sun to go round once.
(vii) The mass of Milky way is estimated to be 150 billion
Neutron Star solar masses or nearly 3 ´ 1011 kg. The intersteller
The core of the star finishes up as neutron star, if the mass matter forms nearly 1.5% of the total mass of the
of the star was between 1.5 to 5 solar masses. The Milky way

@iitjeehelps
Practice Exercise
1. Which planet is closest to Sun? 13. Hubble’s law is expressed as [v = speed of recession,
a. Earth b. Mercury c. Mars d. Jupiter r = distance r of galaxy, H = Hubble constant]
H H
2. Which planet is farthest to Sun? a. v = Hr 2 b. v = Hr c. v = d. v =
a. Saturn b. Uranus c. Neptune d. Pluto r r2

3. The group of small pieces of rock revolving round the 14. The group of 100 to 1000 stars held in position by
Sun between the orbits of Mars and Jupiter are called mutual gravitational forces are called
a. meteors b. comets c. meteorites d. asteroids a. galactic clusters b. globular clusters
c. comets d. asteroids
4. The tail of a comet points away from the Sun due to
15. The most common stars like the Sun are called
a. centrifugal force
b. electrical repulsion a. dwarfs b. white dwarfs
c. attraction of comet due to other planets c. milky way d. neutron stars
d. radiation pressure 16. Very small stars having diameters 1/5th that of sun
5. A star which appears blue will be are called
a. as hot as the Sun a. dwarfs b. white dwarfs
b. cooler than the Sun c. milky way d. neutron stars
c. very cold indeed 17. The brightest planet in the solar system is
d. much hotter than the Sun
a. Mars b. Jupiter c. Venus d. Mercury
6. Sun radiates continuously and maintains its
18. The planet which has no atmosphere is
brightness because
a. Venus b. Mercury c. Mars d. Earth
a. helium is converted into iron in its core
b. of fusion of hydrogen nuclei in helium 19. The spectrum of stars is most closely related to
c. fusion of helium in hydrogen a. colour b. pressure
d. burning of carbon in its core c. distance from Earth d. mass
7. One astronomical unit (AU) is equal to 20. Venus appears brighter than other planets because
a. 10-10 m b. 1496
. ´ 1011 m a. it is heavier than other planets
c. 9.45 ´ 1015 m d. 4.964 ´ 1011 m b. its density is more than other planets
c. it is nearest to Earth than other planets
8. The planet which has no satellite is d. fusion take place at its surface
a. Neptune b. Mercury
c. Jupiter d. Mars 21. One main characteristic of black hole is that, it
a. emits a photon
9. The galaxy to which our solar system belongs is called b. absorbs a photon
a. milky way b. radio galaxy c. changes photon into mass
c. solar galaxy d. elliptical galaxy d. changes all colours into black one
10. Galaxy in which we live is 22. The spectrum of a star is usually
a. milky way b. radio galaxy a. continuous emission spectrum
c. circular galaxy d. irregular galaxy b. continuous absorption spectrum
11. Milky way is c. line absorption spectrum
d. line emission spectrum
a. a planet of our solar system
b. a sun 23. The solar constant at earth’s surface is
c. one of the stars of solar system a. 1.4 watt/m 2 b. 14 watt/m 2
d. one of the enormous galaxies of universe c. 0.14 watt/m 2 d. 1400 watt/m 2
12. The universe is 24. If r denotes the mean distance of a planet from the
a. expanding sun and T is the time period of planet, then
b. contracting
a. r µ T 2/ 3 b. r µ T 3/ 2
c. constant in size
d. increasing northwards and decreasing southwards c. r µ T 1/ 3 d. r µ T

@iitjeehelps
UNIVERSE 379
25. Which of the following theories is the most satisfactory distance of the Sun from the Earth = 1496
. ´ 1011 m,
9
about the origin of the universe? diameter of the Sun 1393
. ´ 10 m and the distance of
a. Big-Bang theory b. Pulsating theory the Moon from the Earth = 3.845 ´ 108 m, then
c. Steady state theory d. None of these calculate the diameter of the moon.
. ´ 106 m
a. 126 b. 4.30 ´ 106 m
26. Hubble’s law states that the velocity with which the
c. 3.58 ´ 106 m d. 6.86 ´ 106 m
galaxy is moving away from earth is proportional to
a. square of distance of milky way 29. Suppose the sun shrank from its present size so that
b. distance of milky way from the Earth its radius is halved. What would energy? (Given that
c. mass of milky way mass of Sun = 1989. ´ 1030 kg and radius of the Sun
d. product of mass of milky way and its distance from 8
earth = 6. 95 ´ 10 m)
a. 4.321 ´ 1050 J b. 2.275 ´ 1041 J
27. Albedo is
c. 1102
. ´ 1030 J d. 8.123 ´ 1010 J
a. a star
b. a galaxy 30. When the Jupiter is at a distance of 824.7 million km
c. a constellation from the earth, its angular diameter is measured to be
d. reflecting power of a planet 35.72 sec of an arc. Calculate the diameter of Jupiter.
28. During a total solar eclipse, the disc of the Moon a. 234265.54 km b. 142745.38 km
almost completely covers the disc of the sun. If the c. 312108.82 km d. 121310.12 km

Answer with Solutions


Practice Exercise
17. (c) Venus is the brightest planet in the solar system.
1. (b) Mercury is the only planet closest to Sun.
18. (b) Mercury is the planet among the given options which
2. (d) In comparison to the respective distances of all the
has no atmosphere.
planets from the Sun, Pluto is farthest to Sun.
19. (a) The spectrum of star is most closely related to colour.
3. (d) Asteroids are the group of small pieces of rock
revolving around the Sun between the orbits of Mars and 20. (c) Venus appears brighter than other planets because it
Jupiter. is nearest to Earth than the other planets.
4. (d) Radiation pressure is only a reason due to which the 21. (b) One main characteristic of black hole is that it absorbs
tail of the comet point away from the Sun. a photon.
5. (d) A star which appears blue will be much hotter than the 22. (c) The spectrum of a star is usually line absorption
Sun. spectrum.
6. (b) Sun radiates continuously and maintain its brightness 23. (d) The solar constant at Earth’s surface is 1400 walt/m2.
because of fusion of hydrogen nucleus in helium. 24. (a) According to Kepler’s third law, the square of the time
7. (b) One astronomical unit is equal to 1496
. ´ 1011 m. taken by the planet to revolve around the Sun is directly
proportional to the cube of semi-major axis of its elliptical
8. (b) Mercury is the planet which has no satellite. path.
9. (a) The galaxy to which our solar system belongs is called i.e. T 2 µ r 3 Þ r µT 2/ 3
milky way.
25. (a) According to Big-Bang theory of universe, all the
10. (a) Galaxy in which we live is milky way. It is the name of constituents of our universe were originally together as a
our galaxy to which our solar systems belongs. single mass. A big explosion occured at some instant and
11. (d) Milky way is one of the enourmous galaxies of universe. the single mass burst in to a large number of fragments
moving with different velocities.
12. (a) 13. (b)
26. (b) Hubble’s law states that the velocity with which the
14. (a) galaxy is moving away from the Earth is proportional to
15. (a) The most common stars like the Sun are called dwarfs. distance of milky way from the Earth.
1 27. (d) Albedo is reflecting power of a planet.
16. (b) Very small stars having diameter th that of Sun are
5
called white dwarfs. It was discovered by 28. (c) Given, for Sun
S.chandrasekhar in 1950 and it is known as d = 1393
. ´ 109 m
Chanrdrasekhar limit. D = 1496
. ´ 1011 m

@iitjeehelps
380 SELF STUDY GUIDE BITSAT

From the formula, In case Sun shrank form its present size till its radius is
d æR ö
\ a= halved ç ÷ , then
D è 2ø
.
1393 ´ 109 3 GM 2 6 GM 2
= V¢ = - =-
.
1496 ´ 1011 5 R/2 5 R2
1393
. \Change in gravitation potential energy of the Sun
= ´ 102 rad
1496
. 3 GM 2 æ 6 GM 2 ö
V -V ¢ = - × - ç- . ÷
1393
. 5 R è 5 R ø
For moon, a= ´ 10-2 rad
1496
.
3 GM 2 3 6.66 ´ 10-11 ´ (1989
. ´ 1030 )2
D = 3.845 ´ 108 V -V ¢ = × = ´
5 R 5 6.95 ´ 108
\ d = Da
= 2.275 ´ 1041 J
1393
.8 -2
3.845 ´ 10 ´ 10 = 3.58 ´ 106 m
1496
. 30. (b) Given, a = 35.72 s of arc
29. (b) Given, 35.72 ´ p
So, a= rad
M = 1989
. ´ 30 kg, 60 ´ 60 ´ 180
R = 6.95 ´ 208 m D = 824.7 ´ 106 km
G = 6.66 ´ 10-11 Nm2 kg2 Diameter of Jupiter, d = Da
Present gravitational potential energy of the Sun is given by 824.7 ´ 106 ´ 35.72 p
\ d =
3 GM 2 60 ´ 60 ´ 180
V =-
5 R = 142745.38 km

@iitjeehelps
@iitjeehelps
@iitjeehelps
1
Some Basic Concepts
of Chemistry

Introduction
Chemistry is a branch of science which deals with the study of the composition of matter and the
chemical changes involved in it. In other words, chemistry is the science of matter and its
transformation.

Laws of Chemical Combination


These are the simple statements which have the wide applications during chemical reactions (though
some of these laws are not perfect in today’s context)

1. Law of Conservation of Mass (Lavoisier, 1774)


According to this law during any chemical or physical change, the total mass of the system
(reactants + products) remains constant or we can say that, mass can neither be created nor be
destroyed.

2. Law of Definite/Constant Proportion (Proust, 1799)


This law states that a given chemical compound always contains its component elements in a fixed
ratio (by weight) and does not depend on its source or method of preparation.
e.g. H 2O obtained from any source always contains H and O in the ratio 1 : 8 (by mass).

3. Law of Multiple Proportion (Dalton, 1808)


It states that if two elements combine to form more than one compound, then the different masses of
one element which combine with a fixed mass of the other element, bear a simple ratio to one another.

@iitjeehelps
384 SELF STUDY GUIDE BITSAT

e.g. Oxides of nitrogen Sometimes, the amount of a substance is expressed in


Fixed weight of N = 28 number of gram atoms. One gram atom contains N A atoms
of an element .
Compound N 2O NO N 2O3 N 2O4 N 2O5
Weight of 1 gram atom of an element whose atomic number
Weight of oxygen 16 32 48 64 80 is A = A g
Weight of one atom = A amu
Ratio = 1 : 2 : 3 : 4 : 5

4. Law of Reciprocal Equivalent and Molecular Mass


Combining Proportion (Richter, 1794) It is the relative mass of one molecule of a compound with
respect to 1/12th of the mass of one atom of C-12.
According to this law, when two or more elements (say
X , Y , Z, etc) combine with a fixed mass of another element i.e. Molecular mass
(say A), then the ratio of their combining masses is same Weight of one molecule of compound
=
(or a simple multiple) as the proportion in which they 1/12th part of the weight of one atom of C -12
combine with each other (XY , YZ , XZ, etc).
NOTE For compounds, molecular weight is the sum of the atomic
e.g. CH 4 ,CO2 and H 2O weights of all atoms present in the molecule.
5. Gay- Lussac’s Law of Combining Average Atomic/Molecular Mass
Volumes (Gay- Lussac, 1808) If an element exists in the isotopic forms having atomic
In a gaseous phase reaction at the same temperature and masses m1 , m2 and m3 in the ratio x , y and z respectively, the
pressure, the volume of reactant and product gases bear a average atomic mass
simple whole number ratio. m ´ x + m2 ´ y + m3 ´ z
M av = 1
e.g. N 2 + 3H 2 ¾® 2NH3 x+ y+z
1 vol 3 vol 2 vol
Equivalent Weight
Dalton’s Atomic Theory Equivalent weight of an element or of a compound is the
weight of an element or of a compound which would
On the basis of laws of chemical combinations, Dalton
combine with or displace (by weight) 1 part of hydrogen or
(1803) proposed atomic theory.
8 parts of oxygen or 35.5 parts of chlorine.
The main postulates of this theory are : Atomic weight or molecular weight
Equivalent weight =
1. Matter is made up of indivisible and indestructible n- factor
particles, called atoms.
where, n-factor = acidity (number of ionisable H + )
2. Atoms are neither be created nor destroyed in the
course of an ordinary chemical reaction. = basicity (number of ionisable OH - )
3. Atoms combine with each other to form compounds in = charge of ions
simple whole number ratio. = number of electrons lost or gained
4. Atom is the smallest portion of matter which takes part
in chemical reaction. Mole Concept
5. All atoms of an element have identical mass and Mole is the amount of substance which contains
similar chemical properties. Avogadro's number (N A = 6.023 ´ 1023 ) of particles and has
Atomic Weight the mass equal to gram atomic mass or gram molecular
mass.
It may be defined as the relative weight of an atom of an
element with respect to 1/12th of the mass of an atom One mole = 6.023 ´ 1023 particles
of C-12. One mole of atom = 6.023 ´ 1023 atoms
i.e. Atomic weight For 1 mole of atoms,
Mass of one atom of element Mass of 1 mole of a substance = Gram atomic mass
=
1/12th part of the mass of one atom of C -12
For 1 mole of molecules,
The weight of one atom of C-12 is 12 amu, where amu Mass of 1 mole of molecules = Gram molecular mass
stands for atomic mass unit.
For 1 mole of ions,
1
1 amu = = 1.667 ´ 10-24 g Mass of 1 mole of ions = Gram formula mass
NA

@iitjeehelps
SOME BASIC CONCEPTS OF CHEMISTRY 385
Number of particles Mole ratio 2 mol 1 mol 2 mol
Number of moles of particles =
Avogadro's number Molecule ratio 2 ´ 6.022 ´ 10 23 6.022 ´ 10 23 2 ´ 6.022 ´ 10 23
molecules molecules molecules
Mole of gas at STP occupies = 22.4 L volume
Weight ratio 4g 32 g 36 g
Volume of gas at STP (in L)
Number of moles of gas = Volume ratio 2 volumes 1 volume 2 volumes
22.4 L
Volume ratio is valid for gaseous state at same temperature
Percentage Composition and pressure.
The percentage of any element or constituent in a
compound is the number of parts by mass of that
Limiting Reagent
element or constituent present in 100 parts by mass of The substance which is completely consumed in a reaction, is
the compound. called limiting reagent. It determines the amount of product.
Mass per cent of an element Actual yield ´ 100
Reaction yield =
Mass of element in 1 mole of compound ´ 100 Theoretical yield
=
Mass of 1 mole of compound It must be noted that in stoichiometry, if the quantities of two
or more reactants are given, the amounts of products formed
Dividing percentage by atomic mass, gives molar ratio
depend upon the limiting reactant (the reactant which
from which empirical formula in obtained.
consumed first in the reaction).
Molecular mass
n=
Empirical formula mass
Concentration Terms
Molecular formula = n ´ Empirical formula
Molar mass = 2 ´ Vapour density (a) Mass%
It is defined as the amount of solute in gram present in
Empirical Formula 100 grams of solution.
It shows the simplest relative whole number ratio of Mass of solute
atoms of each element present in the molecule of the Mass% of solute = ´ 100
Mass of solution
substance.
e.g. CH is the empirical formula of benzene (C6H6 ), C6H6 (b) % Mass by Volume
is its molecular formula. It is defined as the mass of solute present in 100 mL of solution.
Molecular Formula (c) ppm
It expresses the actual number of atoms of each element It is defined as the quantity of solute in grams present in
present in the molecule. 106 grams of solution.
e.g. Molecular formula of hydrazine is NH 2 NH 2 or
N 2H 4 while its empirical formula is NH 2 . (d) Mole Fraction
One more type of chemical formula is seen, i.e. Suppose n A moles of solute A and n B moles of solvent B are
structural formula. It represents the relative position of present in a solution, then
atoms how they are combined in a molecule. nA
Mole fraction of A = c A =
nA + nB
Chemical Equations and Mole fraction of B = c B =
nB
nA + nB
Stoichiometry
cA + cB = 1
A balanced chemical equation with suitable
stoichiometric coefficients represents the ratio of (e) Molarity (M)
number of moles of reactants and products. It is defined as the number of moles of solute per litre
The equation provides qualitative and quantitative of solution
information about a chemical change in a simple n W / MA
M= A = A
manner. V V
e.g. 2H 2 ( g ) + O2 ( g ) ¾® 2H 2O( g ) where,V = volume of solution in litres

@iitjeehelps
386 SELF STUDY GUIDE BITSAT

(f) Molality (m) (g) Normality (N )


It is defined as the number of moles of solute present in 1 kg It is defined as the number of gram equivalents of solute
of the solvent present in 1 litre of solution.
é WA ù WA
m=ê ú ´ 1000 N =
ë M AW B û E AV
where, W A = mass of solute (in gram) where, E A = equivalent mass of A
W B = mass of solvent (in gram) NOTE Normality (N) = n ´ molarity
M A = molecular mass of solute where, n=
Molecular mass
Equivalent mass

Practice Exercise
2.568 ´ 5.8
1. The answer of the calculation in
4.168 Student Readings
significant figures will be
(i) (ii)
a. 3.570 b. 3.6
c. 3.57 d. 3.579 A 3.01 2.99
2. Number of significant figures in 78.000 g, 0.0206 g B 3.05 2.95
and 3.002 g respectively are
a. 3, 4, and 5 b. 2, 5, and 4 a. Results of both the students are neither accurate nor
c. 3, 3, and 4 d. 5, 3, and 4 precise
b. Results of student A are both precise and accurate
3. Which of the following statements is true about the c. Results of student B are neither precise nor accurate
science of atoms and molecules? d. Result of student B are both precise and accurate
a. We can see, weigh and perceive the atoms and
molecules through naked eyes
6. What is the SI unit of density?
a. gm-3 b. kg / m3
b. It is possible to count the number of atoms and
molecules in a given mass of matter through naked c. g cm-3 d. kg/cm3
eyes and manually
7. What temperature is 75° F on the Kelvin scale?
c. We can establish a quantitative relationship between
a. 24 K b. 348 K
the mass and number of these particles (atoms and
c. 297 K d. 215 K
molecules)
d. Physical properties of matter can be qualitatively 8. A jug contains 2 L of milk. Calculate the volume of the
described using numerical values with suitable units milk in m 3 .
a. 2 ´ 10-2 m3
4. Which of the following statements about a compound
is incorrect? b. 2 ´ 10-1 m3
a. A molecule of a compound has atoms of different c. 2 ´ 10-3 m3
elements d. 2 ´ 10-4m3
b. A compound cannot be separated into its constituent
elements by physical methods of separation 9. Law of constant composition does not hold good for
c. A compound retains the physical properties of its a. endothermic compounds
constituent elements b. exothermic compounds
d. The ratio of atoms of different elements in a c. stoichiometric compounds
compound is fixed d. non-stoichiometric compounds

5. Two students performed the same experiment 10. One gram mole of a gas at NTP occupies 22.4 L. This
separately and each one of them recorded two fact is derived from
readings of mass which are given below. Correct a. law of gaseous volumes
reading of mass is 3.0 g. On the basis of given data, b. Avogadro’s hypothesis
mark the correct option out of the following c. Dalton’s atomic theory
statements. d. Berzelius hypothesis

@iitjeehelps
SOME BASIC CONCEPTS OF CHEMISTRY 387
11. One part of element A reacts with two parts of another 22. What is the equivalent weight of SnCl2 in the reaction,
element B. 6 parts of element C reacts with 4 parts of SnCl2 + Cl2 ¾® SnCl4 ? (mol. wt. of SnCl2 = 190)
element B. If A and C combine together, the ratio of a. 95 b. 45
their weight be governed by c. 60 d. 30
a. law of conservation of mass
b. law of reciprocal proportions 23. 5.6 L of a gas at NTP weighs equal to 8 g. The vapour
c. law of definite proportions density of gas is
d. law of multiple proportions a. 32 b. 16
c. 8 d. 40
12. In which of the following numbers all zeroes are
significant? 24. The number of atoms present in 0.1 mole of
a. 30.000 b. 0.700 c. 0.00050 d. 0.0030 P4 (at. mass= 31) are
a. 2.4 ´ 1023 atoms
13. Which of the following is a homogeneous mixture?
b. same as in 0.05 mole of S8
a. Mixture of soil and water
b. Sugar solution c. 6.02 ´ 1022atoms
c. Mixture of sugar, salt and sand d. same as in 3.1 g of phosphorus
d. Iodised table salt
25. The number of water molecules present in a drop of
14. Formation of CO and CO2 illustrates the law of water (volume = 0.0018 mL) at room temperature is
a. conservation of mass b. multiple proportion a. 1.084 ´ 1018 b. 6.023 ´ 1019
c. reciprocal proportion d. constant proportion c. 4.84 ´ 1017 d. 6.023 ´ 1023
15. Which of the following reactions is not correct 26. Number of molecules in 4.25 g of NH3 , is
according to the law of conservation of mass?
a. 1.505 ´ 1023 b. 6.02 ´ 1023
a. 2Mg(s ) + O2(s ) ¾® 2MgO(s )
c. 3.01 ´ 1023 d. None of these
b. C3H8(g ) + O2(g ) ¾® CO2(g ) + H2O(g )
c. P4(s ) + 5O2(g ) ¾® P4O10(s ) 27. Choose the wrong statement.
d. CH4(g ) + 2O2(g ) ¾® CO2(g ) + 2H2O(g ) a. Molar mass is the mass of one molecule
b. Molar mass is the mass of one mole of substance
16. Carbon dioxide contains 27.27% of carbon, carbon c. 1 mole means 6.023 ´ 1023 particles
disulphide contains 15.79% of carbon and sulphur d. Molar mass is the molecular mass (g)
dioxide contains 50% of sulphur. This data is an
28. A sample of AlF3 contains 3.0 ´ 1024 F- ions. The
agreement with
a. law of conservation of mass number of formula units of the sample are
b. law of definite proportions a. 9.0 ´ 1024 b. 3.0 ´ 1024
c. law of multiple proportions c. 0.75 ´ 1024 d. 1.0 ´ 1024
d. law of reciprocal proportions
29. Which of the following pairs contain equal number of
17. One atom of an element weighs 1.8 ´ 10-22g, its atoms?
atomic mass is a. 22.4 L (STP) of nitrous oxide and 22.4 L of nitric
a. 18 b. 29.4 c. 108.39 d. 154 oxide
18. 27 g of Al (at. mass = 27) will react completely with b. 1 millimole of HCl and 0.5 millimole of H2S
oxygen equal to c. 1 mole of H2O2 and 1 mole of N2O4
a. 24 g b. 8 g c. 40 g d. 10 g d. 11.2 cc (STP) of nitrogen and 0.015 g of nitric oxide
19. Insulin contains 3.4% sulphur. What will be the 30. Vapour density of a gas is 11.2. Volume occupied by
minimum molecular weight of insulin? 2.4 g of this at STP will be
a. 94.117 b. 1884 c. 941.176 d. -976 a. 2.4 L b. 2.24 L
c. 22.4 L d. 11.2 L
20. Molecular weight of a tribasic acid is W. Its equivalent
weight will be 31. Which of the following weigh the most?
W W a. One mole of water
a. b. W c. d. 3 W b. One gram atom of nitrogen
2 3
c. One mole of sodium
21. Compounds having same empirical formula always d. One molecule of H2SO4
have same
a. molecular mass 32. The total number of electrons present in 18 mL of
b. molecular formula water (density =1g mL-1) is
c. number of atoms a. 6.02 ´ 1023 b. 6.02 ´ 1023
d. percentage composition by mass c. 6.02 ´ 1024 d. 6.02 ´ 1025

@iitjeehelps
388 SELF STUDY GUIDE BITSAT

33. If 1 mL of water contains 20 drops, what is the number 44. Rearrange the following (I to IV) in the order of
of water molecules in the one drop of water? increasing masses and choose the correct answer
(A = Avogadro’s number) (Atomic masses : O = 16, Cu = 63 and N = 14)
0.5 0.05 I. 1 molecule of oxygen
(a ) A b. 0.05A c. A d. 0.5 A
18 18 II. 1 atom of nitrogen
34. 3 g of an oxide of a metal is converted to chloride III. 1 ´ 10-10 g molecular weight of oxygen
completely and it yielded 5 g of chloride. The IV. 1 ´ 10-10g atomic weight of copper
equivalent weight of the metal is a. II < I < III < IV b. IV < III < II < I
a. 33.25 b. 3.325 c. 12 d. 20 c. II < III < I < IV d. III < IV < I < II
35. Assuming that the density of water to be 1g/cm 3 , 45. How many moles of magnesium phosphate,
calculate the volume occupied by one molecule of Mg3 (PO4 )2 will contain 0.25 mole of oxygen atoms?
water. a. 0.02 b. 3.125 ´ 10-2
a. 2 . 989 ´ 10-23 mL
c. 1.25 ´ 10-2 d. 2.5 ´ 10-2
b. 6.023 ´ 1023 cm3
c. 22400 mL 46. If we consider that 1/6 in place of 1/12, mass of
d. 18 cm-3 carbon atom is taken to be the relative atomic mass
unit, the mass of one mole of a substance will
36. If 0.5 mole of BaCl2 are mixed with 0.2 mole of a. to be a function of the molecular mass of the
Na 3 PO4 , the maximum number of moles of substance
Ba 3 (PO4 )2 that can be formed, is b. remain unchanged
a. 0.7 b. 0.5 c. 0.3 d. 0.1 c. increase two fold
21 d. decrease twice
37. If 10 molecules are removed from 200 mg of CO2,
the number of mole(s) of CO2 left is/are 47. If 6.3 g of NaHCO3 are added to 15.0 g CH 3 COOH
-3 -3
a. 2.88 ´ 10 b. 28.8 ´ 10 solution, the residue is found of weigh 18.0 g. What is
c. 0.288 ´ 10-3 d. 1.66 ´ 10-2 the mass of CO2 released in the reaction?
a. 4.5 g b. 3.3 g c. 2.6 g d. 2.8 g
38. Which of the following pairs of gases contains the
same number of molecules?
48. Consider the following reaction,
a. 16 g of O2 and 14 g of N2 Na 2CO3 + 2HCl ¾® 2NaCl + H2 O + CO 2
b. 8 g of O2 and 22 g of CO2 Equivalent weight of Na 2CO3 is
c. 28 g of N2 and 22 g of CO2 M M
a. b. M c. 2M d.
d. 32 g of O2 and 32 g of N2 2 4

39. 10 dm 3 of N2 gas and 10 dm 3 of gas X at the same 49. The weight of lime obtained by heating 200 kg of 95%
pure lime stone is
temperature contain the same number of molecules.
a. 98.4 kg b. 106.4 kg
The gas X is
c. 112.8 kg d. 122.6 kg
a. CO b. CO2 c. H2 d. NO
50. What will be the weight of CO having the same
40. A metal oxide contains 53% metal and carbon dioxide number of oxygen atoms as present in 22 g of CO2?
contains 27% carbon. Assuming the law of reciprocal a. 28 g b. 22 g c. 44 g d. 72 g
proportions, the percentage of metal in the metal
carbide is 51. An organic compound containing C and H has 92.3%
a. 75 b. 25 c. 37 d. 66 of carbon, its empirical formula is
a. CH b. CH3 c. CH2 d. CH4
41. The number of gram molecules of oxygen in
6.02 ´ 1024 CO molecule is/are 52. Mass of 0.1 mole of methane is
a. 1 g b. 16 g c. 1.6 g d. 0.1 g
a. 10 b. 5 c. 1 d. 0.5
42. The weight of 1 ´ 1022 molecules of CuSO4 × 5H2O is 53. A person adds 1.71 g of sugar (C12H22O11) in order to
a. 41.59 g b. 415.9 g
sweeten his tea. The number of carbon atoms added
c. 4.159 g d. None of these are (molecular mass of sugar = 342)
a. 3.6 ´1022 b. 7.2 ´1021
43. The sulphate of a metal M contains 9.87% of M . The c. 0.05 d. 6.6 ´1022
sulphate is isomorphous with ZnSO4 × 7H2O . The
atomic weight of M is 54. The one which has least mass, is
a. 40.3 b. 36.3 a. 2 g atom of N b. 3 ´ 1023atoms of C
c. 24.3 d. 11.3 c. 1 mole of S d. 7.0 g of Ag

@iitjeehelps
SOME BASIC CONCEPTS OF CHEMISTRY 389
55. 19.7 kg of gold was recovered from a smuggler. How c. 67.2 L H2 (g ) at STP, is produced for every mole of Al
many atoms of gold were recovered? that reacts
a. 100 b. 6.02 ´ 1023 d. 11.2 L H2 (g ) at STP, is produced for every mole of
c. 6.02 ´ 1024 d. 6.02 ´ 1025 HCl (aq ) consumed
56. If 8.4 g of hexane burns completely in oxygen, how 64. Two solutions of a substance (non-electrolyte) are
many moles of CO2 is/are produced? mixed in the manner, 480 mL of 1.5 M of first solution
a. 6 b. 0.6 c. 0.9 d. 1.2 with 520 mL of 1.2 M of second solution.
The molarity of final solution is
57. The number of atoms in 4.25 g of NH 3 is a. 1.20 M b. 1.50 M c. 1.344 M d. 2.70 M
approximately
65. Amount of oxalic acid present in a solution can be
a. 1 ´ 1023 b. 1.5 ´ 1023
determined by its titration with KMnO4 solution in the
c. 2 ´ 1023 d. 6 ´ 1023 presence of H2 SO4. The titration gives unsatisfactory
58. Two elements x (at. mass = 75) and y (at. mass = 16) result when carried out in the presence of HCl
because HCl
combine to give a compound having 75.8% x. The
a. gets oxidised by oxalic acid to chlorine
formula of the compound is b. furnishes H+ ions in addition to those from oxalic acid
a. xy b. x 2 y c. x 2 y 2 d. x 2 y 3 c. reduces permanganate to Mn2+
59. If we take 44 g of CO2 and 14 g of N2, what will be the d. oxidises oxalic acid to carbon dioxide and water
mole fraction of CO2 in the mixture? 66. Consider the following reaction,
a. 1/5 b. 1/3 c. 2/3 d. 1/4 CH 4 ( g ) + 2O2( g ) ¾® CO2( g ) + 2H 2O( g )
60. One mole of calcium phosphide on reaction with How many moles of methane are required to produce
excess of water gives 22 g CO2 (g) after combustion?
a. one mole of phosphine a. 1 mole b. 0.5 mole
b. two moles of phosphoric acid c. 0.25 mole d. 1.25 moles
c. two moles of phosphine 67. Which of the following gases will have least volume if
d. one mole of phosphorus pentoxide 10 g of each gas is taken at same temperature and
61. The volume of water to be added to 100 cm3 of 0.5 N pressure?
H2SO4 to get decinormal concentration, is a. CO2 b. N2 c. CH4 d. HCl
a. 100 cm3 b. 450 cm3 68. A solution is made by dissolving 49 g of H2SO4 in
c. 500 cm3 d. 400 cm3
250 mL of water. The molarity of the solution prepared is
62. 5 mL of N HCl, 20 mL of N / 2 H2SO4 and 30 mL of a. 2 M b. 1 M c. 4 M d. 5 M
N /3 HNO3 are mixed together and volume made to
1 L. The normality of resulting solution is 69. What volume of water is to be added to 100 cm 3 of
a. 0.45 b. 0.025 c. 0.9 d. 0.05
0.5 M NaOH solution to make it 0.1 M solution?
a. 200 cm3 b. 400 cm3 c. 500 cm3 d. 100 cm3
63. Consider the following reaction,
70. What will be the molality of the solution made by
2AI(s ) + 6HCl(aq ) ¾® 2Al3 + (aq ) + 6Cl- (aq ) + 3H2( g ) dissolving 10 g of NaOH in 100 g of water?
a. 2.5 m b. 5 m c. 10 m d. 1.25 m
Which of the following statements is incorrect?
a. 6 L HCl (aq) is consumed for every 3 LH2(g) produced 71. How much of NaOH is required to neutralise
b. 33.6 L H2(g ) is produced regardless of temperature
1500 cm 3 of 0.1 N HCl? [Na = 23]
and pressure for every mole of Al that reacts a. 40 g b. 4 g c. 6 g d. 60 g

BITSAT Archives
1. 10 g of sample of mixture of CaCl2 and NaCl are 2. The volume respectively of 10 N and 4 N HCl required
treated to precipitate all the calcium as CaCO3 . This to make 1 L of 7 N HCl respectively are [2014]
CaCO3 is heated to convert all the Ca to CaO and the a. 0.75 L of 10 N HCl and 0.25 L of 4 N HCl
final mass of CaO is 1.62 g. The per cent by mass of b. 0.50 L of 10 N HCl and 0.50 L of 4 N HCl
CaCl2 in the original mixture is [2014] c. 0.65 L of 10 N HCl and 0.35 L of 4 N HCl
a. 32.1% b. 16.2% c. 21.8% d. 12.0% d. 0.85 L of 10 N HCl and 0.15 L of 4 N HCl

@iitjeehelps
390 SELF STUDY GUIDE BITSAT

3. 0.1 g of metal combines with 46.6 mL of oxygen at 7. How much water should be added to 200 mL of semi
STP. The equivalent weight of metal is [2013] normal solution of NaOH to make it exactly
a. 12 b. 24 decinormal? [2010]
c. 18 d. 36 a. 200 mL b. 800 mL c. 1000 mL d. 11200 mL
4. 5 moles of Ba(OH)2 are treated with excess of CO2. 8. Match the following columns. [2008]
How much Ba(OH)2 will be formed? [2012] Column I Column II (At STP)
a. 39.4 g b. 197 g
A D 1. 0.224 L CO 2
c. 591 g d. 985 g 10 g CaCO 3 ¾¾¾¾®
Decomposition
5. One mole of P2O5 undergoes hydrolysis as
B Excess HC l 2. 4.48 L CO 2
1.06 g Na 2CO 3 ¾¾¾¾®
P2O5 + H2 O ¾® H3 PO4
Excess O
The normality of the phosphoric acid formed is C 2.4 g C ¾ ¾¾¾¾
2
® 3. 0.448 L CO 2
combustion

(Volume of the solution is 1 L) [2011] D 0.56 g CO ¾ ¾¾¾¾


2Excess O
® 4. 2.24 L CO 2
combustion
a. 2 b. 12 c. 24 d. 4
5. 22.4 L CO 2
6. 1 g of hydrogen is found to combine with 80 g of bromine
and 1 g of calcium combines with 4 g of bromine.
The correct match is
Equivalent weight of calcium is [2010]
A B C D A B C D
a. 16 b. 20 a. 4 1 2 3 b. 5 1 2 3
c. 40 d. 80 c. 4 1 3 2 d. 1 4 2 3

Answer with Solutions


Practice Exercise 1m
= 1=
100 cm
100 cm 100 cm
2.568 ´ 5.8
1. (b) = 3.5735 To get m3 from the above unit factors, the first unit factor is
4.168
taken and it is cubed.
Thus, insignificant figures = 3.6 3
æ 1m ö
2. (d) ç ÷
è 100 cmø
3. (c)
1m3
4. (c) Physical properties of a compound are different from Þ = (1)3 = 1
those of elements. 106 cm3
3.01 + 2.99 Now, 2 L = 2 ´ 1000 cm3
5. (b) Average of reading of student A = = 3.00
2 The above is multiplied by unit factor
3.05 + 2.95
B= = 3.00 1 m3 2 m3
2 2 × 1000 cm3 × 6 3
= = 2 ´ 10-3m3
10 cm 103
Correct reading = 3.0
9. (d ) Law of constant composition does not hold good for
Results of student A are both precise and accurate.
non-stoichiometric compounds.
6. (b) Density of a substance is defined as its amount of
10. (b) One gram mole of a gas at NTP occupies 22.4 L. This
mass per unit volume.
fact is derived from Avogadro’s hypothesis.
kg
Sl unit of density = 3 or kg m -3 11. (b) Ratio will be governed by the law of reciprocal
m proportions.
9 43 ´ 5 12. (a) 30.000 has all significant zeroes.
7. (c) 75° F = ° C + 32 = ° C = 23.9° C
5 9
13. (b) Sugar solution is a homogeneous mixture.
= ( 23.9 + 273) K
14. (b) Formation of CO and CO2 illustrates the law of
= 296.9 ~- 297 K multiple proportion.
8. (c) 2 ´ 10-3 m3 15. (b) C3H8(g ) + O2(g ) ¾® CO2(g ) + H2O (g ) is not correct.
Q 1 L = 1000 cm3 Correct equation is
and 1 m = 100 cm which gives C3H8(g ) + 5O2(g ) ¾® 3CO2(g ) + 4H2O (g )

@iitjeehelps
SOME BASIC CONCEPTS OF CHEMISTRY 391
16. (d) In CO2 24. (a)Q 1 mole of P4 contains phosphorus atoms
Weight of CO2 = 100 = 4 ´ 6.02 ´ 1023
Weight of carbon = 27. 27 \ 0.1 mole of P4 contains phosphorus atoms
Weight of oxygen = 72 . 73 = 4 ´ 6.02 ´ 1023 ´ 0.1
27.27 g of carbon combines with 72.73 g of oxygen
= 2.4 ´ 1023 atoms
72.73
1g of carbon combines with = = 2.66 g of oxygen Weight
27.27 25. (b) Number of moles =
Molecular weight
\Ratio of C and O in CO2 = 1 : 2.66
0.0018
Similarly, in CS2 = = 1 ´ 10-4
18
Ratio of carbon and sulphur in CS2 = 1 : 5.3
[Q 0.0018 mL = 0.0018 g]
If sulphur and oxygen were to combine to form SO2,
Number of water molecules = 1 ´ 10-4 ´ 6.023 ´ 1023
according to law of reciprocal proportions. They must
combine in 5.3 : 2.66 or 2 : 1. = 6.023 ´ 1019
In SO2, the ratio of S:O is 1:1. 26. (a)Q 17 g of NH3 contains = 6.023 ´ 1023 molecules of NH3
2 1
Thus, the ratio between the two ratios is : = 2 : 1 6.023 ´ 1023
1 1 \ 4.25 g of NH3 will contain = ´ 4.25
17
Since, the ratio of S : O is a simple whole number ratio,
therefore law of reciprocal is proved. = 1.505 ´ 1023 molecules of NH3
17. (c) Atomic mass = NA ´ mass of one atom 27. (a) Molar mass is the mass of 1 mole or 6.022 ´ 1023
= 6.022 ´ 1023 ´ 1.8 ´ 10-22 = 108.39 molecules of the substance
18. (a) The equation of the reaction of aluminium and 28. (d )Q 3 F - = 1formula unit of AlF3
oxygen, is as \ 3.0 × 1024 F – = 1 ´ 10 24 formula units of AlF3
4 Al + 3O2 ¾® 2 Al2O3
4 ´ 27 6 ´ 16 29. (d ) Number of atoms in N2
= 108 = 96
11.2 ´ 10-3 ´ 6.023 ´ 1023 ´ 2
Q 108 g of Al reacts with 96 g of O2. = = 6.023 ´ 1020
96 ´ 27 22.4
\ 27 g of Al will react with = = 24 g of O2
108 0.015 ´ 2 ´ 6.023 ´ 1023
Number of atoms in NO =
19. (c) For minimum molecular mass, there must be one 30
S-atom per insulin molecule. = 6.023 ´ 1020
If 3.4 g of S is present, the molecular mass = 100 30. (a) Molecular mass = 2 ´ vapour density
\ If 32 g of S is present, the molecular mass will be = 2 ´ 11. 2 = 22.4
100 ´ 32 2.4
= = 941.176 Number of moles of gas =
3.4 22 .4
Molecular weight W
20. (c) Equivalent weight = = Q 1 mole occupies = 22 .4 L volume
Basicity 3
2.4 2.4
21. (d ) Compound having same empirical formula always \ mole will occupy = 22.4 ´
22.4 22.4
have same percentage composition by mass.
= 2.4 L
22. (a) SnCl2 + Cl2 ¾® SnCl4
190 71 31. (c) One mole of water = 18 g
Eq. wt. of SnCl2 = Eq. wt. of Cl2 1 g atom of nitrogen = 14 g
190 71 1 mole of sodium = 23 g
\ = or E1 = 95 98
E1 35.5 1 molecule of H2SO4 = g
6.023 ´ 1023
23. (b)Q 5.6 L of gas weighs at NTP = 8 g
= 1. 627 ´ 10-24g
8 ´ 22.4
\22.4 L of gas will weighs at NTP = = 32 g Hence, one mole of sodium weigh the most.
5.6
\Molecular weight of gas = 32 32. (c) 18 mL H2O = 18 g H2O =1 mol
We know that, = 6.02 ´ 1023 molecules
Molecular weight = 2 × vapour density = 10 ´ 6.02 ´ 1023 electrons
32
\Vapour density = = 16 = 6.02 ´ 1024 electrons
2

@iitjeehelps
392 SELF STUDY GUIDE BITSAT

33. (c)Q 18 mL = 18 g of water contains = 20 × 18 drops 42. (c)Q 6.02 ´ 1023 molecules of CuSO4 × 5H2O
= A molecules = 63.5 + 32 + 64 + 90 = 249.5 g
A 0.05 A \ 1022molecules of CuSO4 × 5H2O
\1 drop contains = molecules = molecules
20 ´ 18 18 249.5
= = 4.15 g
Weight of metal oxide 6.02 ´ 1022
34. (a)
Weight of metal chloride 43. (c) As the given sulphate is isomorphous with
Eq. wt. of metal + Eq. wt. of oxide ZnSO4 × 7H2O, its formula would be M SO4 × 7H2O. If m is
= the atomic weight of M, molecular weight of
Eq. wt. of metal + Eq. wt. of chloride
M SO4 × 7H2O = m + 32 + 64 + 126 = m + 222
3 E+8
= m
5 E + 35.5 Hence ,% of M = ´ 100 = 9.87 (given)
m + 222
or 5E + 40 = 3E + 106.5 or 2E = 66.5
\ E = 33.25 or 100 m = 9.87 m + 222 ´ 9.87
35. (a)Q Density of water = 1 g/cm 3 or 90.13 m = 222 ´ 9.87 or m = 24.3
32
\6.023 ×1023 molecules of H2O contain = 18 cm3 volume 44. (a) I. 1 molecule of O2 = = 5.3 ´ 10-23g
6.022 ´ 1023
18 14
\ 1 molecule of H2O will contain = II. 1 atom of N = = 2.3 ´ 10-23g
6.023 ´ 1023 6.022 ´ 1023 g
= 2.989 ´ 10-23 mL III. 10-10 g molecular weight of oxygen
36. (d) 3BaCl2 + 2Na 3PO4 ¾® Ba 3(PO4 )2 +3NaCl = 10-10 ´ 32 = 3.2 ´ 10-9g
Limiting reactant is Na 3PO4. IV. 10-10 g atomic weight of copper
1
0.2 mole of Na 3PO4 will give = ´ 0. 2 = 10-10 ´ 63 = 6.3 ´ 10-9g
2
= 0.1 mole of Ba 3 (PO4 ) 2 \ Order of increasing mass is
0.2 II < I < III < IV
37. (a) 200 mg CO2 = 0.2 g = mol = 0.00454 mol
44 45. (b) Mg3(PO4 )2 : 1 mol
-3
= 4.54 ´ 10 mol 8 moles of O -atom are contained by 1 mole of Mg3(PO4 )2.
1021 Hence, 0.25 mole of O-atom are contained by
1021 molecules of CO2 = = 1.66 ´ 10-3 mol
6.02 ´ 1023 1
= ´ 0.25 = 3.125 ´ 10-2 mole of Mg3(PO4 )2
8
\Number of moles left = ( 4.54 - 1.66) ´ 10-3 = 2.88 ´ 10-3
46. (b) Mass of the given amount of a substance is a constant
NA
38. (a) 16 g of O2 = 0.5 mol = molecules quantity, i.e. remains unchanged.
2
NA 47. (b) According to law of conservation of mass,
14 g of N2 = 0.5 mol = molecules
2 Mass of reactants = Mass of products
39. (a) Number of moles of N2 and X should be equal. This \ 6.3 + 15.0 = 18.0 + x
can be so if X has same molecular weight as N2. or x = 21. 3 - 18.0 = 3.3 g
CO has same molecular weight as N2. 48. (a) Na 2CO3 + 2HCl ¾® 2NaCl + H2O + CO2
40. (a) In metal oxide, metal = 53%, O = 47% M
In the above reaction, equivalent weight of Na 2CO3 is
In CO2, C = 27% , O = 73% 2
Q 73 parts of oxygen combines with 27 parts of carbon. because 2 moles of Na+ being transferred per mole of
27 Na 2CO3.
\ 47 parts of oxygen will combine = ´ 47
73 95
49. (b) 200 kg of 95% pure means ´ 200 = 190 kg
= 17.38 parts of C 100

Thus, metal and carbon will be present in the ratio of CaCO3 ¾¾ ® CaO + CO2
(40 + 12 + 3 ´ 16) (40 + 16 )
53 : 17.38. 100 56
53
Hence, % of metal = ´ 100 = 75.3% Q 100 g of CaCO3 on heating gives lime = 56 g
53 + 17.38
56 ´ 190
41. (b) 6.02 ´ 1024 CO molecules = 10 moles of CO \190 g of CaCO3 on heating will give lime =
100
= 10 g atoms of O
= 5 g molecules of O2 = 106.4 g

@iitjeehelps
SOME BASIC CONCEPTS OF CHEMISTRY 393
50. (a) Number of O-atoms in 22 g of CO2 = 44 g 44
59. (c) Number of moles of CO2 = =1
Molar mass of CO2 = 44 g 44
44 g = 1 mol, 22 g = 0.5 mol 14
Number of moles of N2 = = 0.5
1 mole of CO2 contains = 2 ´ 6.023 ´ 10 O-atoms 23 28
1 1 2
0.5 mole of CO2 will contain = 6.023 ´ 1023 O-atoms \ Mole fraction of CO2 = = or =
1 + 0.5 1.5 3
Number of O-atoms in CO = 6.023 ´ 1023molecules of CO
60. (c) Ca 3P2 + 6H2O ¾® 3Ca(OH)2 + 2PH3
= 1 mole of CO
61. (d ) N1V1 = N2V2
1 mole of CO = 6.023 ´ 1023 O-atoms
i.e. 0.5 ×100 = 0.1 ×V2 or V2 = 500 cm3
Molar mass of CO = 12 + 16 = 28 g
\Water to be added to 100 cm3 solution
51. (a) Element % atomic weight Simplest ratio = 500 - 100 = 400 cm3
C 92.3 7 .69 62. (a) Normality equation is
= 7.69 =1
12 7.69 N1V1 + N2V2 + N3V3 = N4 (V1 + V2 + V3 )
H 7.7 7.70 1 1
= 7.70 =1 or 1 ´ 5 + 20 ´ + 30 ´ = N4 (5 + 20 + 30)
1 7.69 2 3
\ Empirical formula = CH (HCl) (H2SO4 ) (HNO3 )
52. (c) Mass of 1 mole of methane (CH4 ) = 16 g 25
\Resulting normality (N4 ) = = 0.45 N
55
Mass of 0.1 mole of methane = 16 ´ 0.1 g = 1.6 g
1.71 63. (c) 2 moles of Al produce = 3 ´ 22.4 L of H2 gas
53. (a) Number of moles of sugar = = 0.005
342 3 ´ 22.4
\1 mole of Al will produce = = 33.62 L of H2 gas
Q 1 mole of sugar contains 2
= 12 ´ 6.02 ´ 1023atoms of carbon 64. (c) For solution I, millimoles = MV = 480 ´ 1.5 = 720
\0.005 mole of sugar will contain For solution II, millimoles = MV = 520 ´ 1.2 = 624
= 12 ´ 0.005 ´ 6.02 ´ 10 23
Total millimoles = 720 + 624 = 1344
1344
= 0.36 ´ 1023 = 3.6 ´ 1022 C-atoms \ Molarity = = 1.344 M
480 + 520
54. (b) 2 g atom of N2 = 2 ´ 14 = 28 g
65. (c) Titration of oxalic acid by KMnO4 in the presence of
3 ´ 1023
3 ´ 1023 C atoms = ´ 12 = 6 g HCl gives unsatisfactory result because HCl is a better
6 ´ 1023 reducing agent than oxalic acid and HCl reduces
1 mole of S = 32 g preferably MnO-4 to Mn2+ .
19700 66. (b) According to the chemical equation
55. (d) Atoms recovered = ´ NA
197 CH4(g ) + 2O2 (g ) ¾® CO2(g ) + 2H2O(g ),
= 100 ´ 6.02 ´ 1023 = 6.02 ´ 1025 44 g of CO2(g) is obtained from 16 g of CH4(g).
56. (b) C6H12 + 9O2 ¾® 6 CO2 + 6H2O [Q 1 mole of CO2 (g) is obtained from 1 mole of CH4 (g)]
6 ´ 12 + 12 = 84 22 g CO2(g )
Mole of CO 2(g) = = 0.5
Q 84 g C6H12 gives = 6 moles of CO2 44 g CO2(g )
6 ´ 8.4 Hence, 0.5 mole of CO2(g) would be obtained from
\ 8.4 g C6H12 will give = = 0.6 mole of CO2
84 0.5 mole of CH4(g ) or 0.5 mole of CH4(g) would be
required to produce 22 g of CO2(g).
57. (d) Mol. wt. of NH3 = 14 + 3 = 17
67. (a) Number of moles µ
1
\17 g of ammonia contains atoms = 4 ´ NA Molecular mass
(Q one molecule of NH3 has 4 atoms) Molecular masses : CO2 = 44, N2 = 28, CH4 = 16,
\4.25 g of ammonia contain atoms and HCl = 36.5
4 ´ 6.02 ´ 1023 ´ 4.25 CO2 will have least volume.
= = 6.02 ´ 1023 atoms
17 Weight of solute
68. (a) Molarity (M ) =
75.8 Molecular weight of solute
58. (d) Mole ratio of x = = 1.01
75 1000
24.2 ´
Mole ratio of y = = 1.5 Volume of solution (mL)
16 49 1000
= ´ = 2M
Simple ratio = x : y = 1: 1.5 = 2 : 3 = x 2 y 3 98 250

@iitjeehelps
394 SELF STUDY GUIDE BITSAT

69. (b) M1V1 = M 2V2 10 1000


= ´ = 2.5 m
40 100
0.5 × 100 = 0.1´ V2
V2 = 500 cm3 71. (c) 1500 cm3of 0.1 N HCl
0.1
Volume of water to be added to 100 cm3of solution = ´ 1500 = 0.15 g eq
1000
= 500 - 100 = 400 cm3
= 0.15 g eq. of NaOH
70. (a) Molality (m) = 0.15 ´ 40 = 6 g NaOH
Weight of solute 1000
= ´
Molar weight of solute Weight of solvent

BITSAT Archives
1. (a) CaCl2 + NaCl = 10 g 6. (b) 1g H2combines with Br2 = 80 g
Let, weight of CaCl2 =x g Equivalent weight of Br2 = 80
CaCl2 ¾® CaCO3 ¾® CaO 4g of Br2 combine with = 1 g Ca
1 mol 1 mol 1 mol 1 ´ 80
x x x \80 g of Br2 will combine with = = 20 g Ca
mol mol mol
111 1 00 56 4
1 1
1.62 x 1.62 7. (b) N1V1 = N2V2, ´ 200 = ´ V2
Mole of CaO = Þ = , x = 3.21 g 2 10
56 111 56
1
3.21 V2 = ´ 200 ´ 10 = 1000 mL
% of CaCl2 = ´ 100 = 32.1% 2
10
\ Volume of water added = 1000 - 200 = 800 mL
2. (b) Let,V litre of 10 N HCl be mixed with (1-V) litre of ∆
4 N HCl to give (V + 1 - V ) = 1 L of 7 N HCl 8. (a) (A) CaCO3 ¾¾ ® CaO + CO2
100 g Decomposition 22.4 L
As we know that, N1V1 + N2V2 = NV
Q 100 g CaCO3 on decomposition gives = 22.4 L CO2
10V + 4(1- V ) = 7 ´ 1
3 \10 g CaCO3 on decomposition will give
10V + 4 - 4V = 7, 6V = 7 - 4, V = = 0.50 L
6 22.4 ´ 10
Volume of 10 N HCl = 0.50 L = L CO2 = 2.24 L CO2
100
Volume of 4 N HCl = 1- 0.50 = 0.50 L (B) Na 2 CO3 ¾ Excess
¾¾¾ HCl
¾® 2NaCl + H2O + CO2
3. (a) 1 mole of O2 = 4 equivalent of oxygen 106 g 22.4 L

22400 mL of O2 = 4 equivalent of oxygen 106 g Na 2CO3 gives = 22.4 L CO2


4 22.4 ´ 1.06
46.6 mL of O2 = ´ 46.6 = 0.00832 eq. 1.06 g Na 2CO3 will give = L CO2 = 0.224
22400 106
L CO2
Equivalent of metal = Equivalent of oxygen
Excess O
Weight (C) C ¾¾¾¾®
2
CO2
= 0.00832 12 g combustion 22 . 4 L
Equivalent
0.1 0.1 12 g carbon on combustion gives = 22.4 L CO2
= 0.00832 Þ E = = 12.0
E 0.00832 2.4 g carbon on combustion will give
22.4 ´ 2 .4
4. (d) Ba(OH)2 + CO2 ¾® BaCO3 + H2O = L CO2 = 2 × 2.24 L CO2 = 4.48 L CO2
12
Q 5 moles of Ba(OH)3 = 5 moles of BaCO3 Excess O
2
(D) 2CO ¾¾¾¾
¾ ® 2CO2
\Mass of BaCO3 2 [12 + 16] combustion 2 ´ 22.4 L
= 56 g
= Moles of BaCO3 ´ Molecular mass of BaCO3
56 g carbon monoxide on combustion gives
= 5 ´ 197 = 985 g
= 2 × 2.24 L CO2
5. (b) 2 P2O5 + 6H2O ¾® 4H3PO4 0.56 g carbon monoxide on combustion will give
2 mol 4 mol
2 ´ 22.4 ´ 0 .56
\1 L solution contains 4 moles of H3PO4. = L CO2
56
\ Molarity of H3PO4 = 4 M = 0.448 L CO2
Normality = Molarity × Basicity = 4 ´ 3 N = 12 N Hence, A ® 4, B ® 1, C ® 2, D ® 3

@iitjeehelps
2
Atomic Structure

Introduction
In the later half of the 19th century and in the beginning of the 20th century the discovery of several
scientists like Faraday, Thomson, Rutherford etc. clearly revealed that the atom is divisible and
composed of several smallest particles like electrons, protons, neutrons etc. although still it is the
smallest particle that takes part in a chemical reaction. After the discovery of these particles, several
questions arise, about the nature of these particles, their arrangement in an atom etc. Our study of
inorganic chemistry must therefore begin with a review of the structure and properties of atoms.
In this chapter we consider the development of our understanding of atomic structure and the
behaviour of electrons in atoms.

Subatomic Particles
A large number of subatomic particles have been discovered so far but only electron, proton and
neutron are of great importance among them and hence, are called fundamental particles.

Atom
Atomic structure is the study of structure of atom. The word atom is derived from a Greek word
‘atomio’ which means indivisible and indestructible.
The main facts related to atom are as follows:
1. Atom is the building block particle of matter.
2. Atom is made mainly from three subatomic particles viz. electron, proton and neutron.
3. Charge to mass ratio (e / m ) is determined by using discharge tube and applying electric and
magnetic field. Deviation of particle takes place when electric and magnetic field is applied to it.
4. Millikan oil drop model is used to measure charge of the electron and it was found that electrical
charge is always an integral multiple of charge on electron.
q = ne
where, q = total charge, e = charge of one electron = 1. 602 ´ 10-19 C and
n = positive integer = 0, 1, 2 , 3.

@iitjeehelps
396 SELF STUDY GUIDE BITSAT

Cathode Rays Thomson Model (1898)


Cathode rays were discovered by Sir J J Thomson. Cathode This is also known by some other
rays were a stream of fast moving negatively charged s s
names like plum pudding, Electron
particles, called electrons. s ⊕
s
watermelon and raisin pudding
The specific charge is the ratio of charge to mass of an ⊕ ⊕
model.
electron. It is found by Thomson and denoted as e/m ratio. ⊕ Proton
Thomson visualised atom as a
The e/m ratio of electron was found to be same for all gases.
pudding or cake of positive charge
e/m ratio = - 1. 758 ´ 1011 C/kg. with electrons embedded into it.

Anode Rays (Positive Rays or It fails to explain the result of gold foil scattering
experiment.
Canal Rays)
Positive rays were discovered by Goldstein. These rays
consist of positively charged particles, called protons.
Rutherford Model
Unlike cathode rays, their e/m value depends upon the (a-particle Scattering Experiment)
nature of gas taken in the tube. This model was set up by Rutherford to discover the
The e/m value is maximum, when hydrogen gas is taken in nucleus. He used gold foil alpha particles set up and found
the tube, i.e. 9 . 58 ´ 107 C / kg. the following results.
α-particles
deflected by
Neutrons small angles
Neutrons are neutral particles and discovered by Nucleus
Undeflected
Chadwick. Neutrons are the heaviest particles of the atom. α-particles

Types of Atomic Species ++


++ α-particles
There are various types of atomic species depending upon ++ deflected
their different distribution of subatomic particles back
++
(e , p and n ) within an atom. Incident ++ ++
α-particles
(i) Isotopes are atoms having same atomic number
but different mass number. Representation of Rutherford Experiment
e. g. 1 H 1 , 1 D2 and 1 T3 are three isotopes of hydrogen.
(ii) Isobars are atoms having same mass number but (i) Most of the a-particles passed through the atom
different atomic number. without any deflection.
e. g. 18 Ar 40 , 19K 40 are isobars. (ii) A small proportion of a-particles were deflected by
small angles.
(iii) Isotones are atoms having same number of
neutrons but different atomic number and mass (iii) A very few a-particles (about 1 in 20,000) returned
back to their direction, i.e. reflected by 180°.
number, e. g. 1 H3 and 2 He 4 are isotones.
(iv) Isoelectronic are atoms or species having same Conclusion of Rutherford Experiment
number of electrons. (i) Atom consists of a very small sized nucleus where
e.g. F - , Ne and Na + are isoelectronic to each other. the entire mass of atom resides (diameter of
(v) Isodiaphers are atoms/species having same nucleus = 10-15 m). Nucleus is made up of nucleons
isotopic number and can be calculated as (neutrons + protons).
Isotopic number (ii) Very large space inside the atom is empty which
= mass number ( A) - 2 ´ atomic number ( Z ) was confirmed when most of a-particles passed
e. g. 19 K39 and 9 F 19 are isodiaphers. straight without any deflection.
(iii) Volume occupied by the nucleus is very less as
compared to total volume of atom.
Atomic Models (iv) An electron in an atom revolves around the nucleus
Various atomic models are proposed to explain the structure in stationary orbit and occupies almost all the space
of atom which are as follows: around nucleus (same as solar system).

@iitjeehelps
ATOMIC STRUCTURE 397
Limitations of Rutherford Model Threshold energy (work function) E = hn0
(i) Main drawback of this model is that it does not tell Energy of photon = hn
anything about electronic structure of atom. Energy transferred from photon to electron
(ii) According to classical theory of electromagnetism, = Energy of photoelectrons = hn - hn0 = h ( n - n0 )
if a charged particle revolves around oppositely 1
charged particle, it looses or radiates energy Kinetic energy of photoelectron = mev 2
2
continuously resulting the decrease in its speed.
Energy transferred from photon to electron
Hence, the circular orbit gradually becomes spiral
and electrons finally falls into the nucleus but in = Kinetic energy of electron
real sense, this does not happen. 1
\ h ( n - n0 ) = mev 2
(iii) When spectrum of an atom is observed, it has been 2
found that the spectrum consists of well defined
lines rather than continuous spectrum (electron
radiates energy continuously which gives Bohr’s Model
continuous spectrum).
Bohr’s model is a quantitative measure of general features
of structure of hydrogen atom and its spectrum such as
radius, energy, velocity of electron, etc.
Development Leading to the
Bohr's Model of Atom Postulates of Bohr’s Model
Interaction of EMR with matter provides us various (i) Electrons revolve around the nucleus in specific
important information such as structure, energy, radius etc. circular path known as orbit or stationary energy
EMR have dual nature, i.e. particle as well as wave nature. levels, i.e. energy of each orbit is constant.
(ii) When an electron absorbs required amount of
Black Body Radiation and energy, it undergoes transition from lower energy
level to higher energy level and when it emits
Planck’s Quantum Theory energy, it comes to the lower energy level.
A body when heated emits radiation over a wide range of (iii) Energy absorbed or emitted during transition is
wavelength. The ideal body which emits or absorbs given by
radiations of all possible frequency is known as black body hc é 1ù
and the radiation emitted by black body is called black body DE = E 2 - E 1 = hcn = êQn = ú
l ë lû
radiation.
Quantum is a measure of emission or absorption of energy where, n = wave number
in discrete quantity. Energy of radiation is directly Frequency of radiation absorbed or emitted during
proportional to frequency of light transition is given by n.
E - E1
E µ n Þ E = hn n= 2 [Q n = c n]
h
where, h = Planck’s constant = 6 .625 ´ 10-34 Js
where, E 2 = higher energy level
The emitted energy is integral multiple of hn such as E 1 = lower energy level
2hn , 3hn , 5hn, i.e. energy is quantised.
(iv) The angular momentum of an electron present in
Photoelectric Effect h
any orbit must be an integral multiple of .
When the light of certain frequency is allowed to fall on a 2p
metal surface, electrons are ejected from metal surface, this h
mvr = n ´
phenomenon is known as photoelectric effect and such 2p
ejected electrons are known as photoelectrons. where, n = 1, 2 , 3 , ....
When a photon (the smallest particle of light) of certain
energy strikes electron in an atom, it transfers its energy to 4
Nucleus 3 Electrons not
electron. Greater the energy of photon greater will be 2 allowed between
energy transferred to electron. 1
orbits
+
The minimum amount of energy required to eject an Electrons
permitted
electron from the metal surface is known as threshold
in circular orbits
energy and the frequency is known as threshold frequency
( n0 ). Bohr’s representation of an atom

@iitjeehelps
398 SELF STUDY GUIDE BITSAT

Bohr’s Parameters (iv) Zeeman effect, i.e. splitting of spectral line in


magnetic field.
(i) Radius of Bohr Orbit (v) Stark effect, i.e. splitting of spectral line in an
Radius is the distance between nucleus and orbit, it is also electric field.
known as Bohr’s radius. The radius of an orbit is given by (vi) Fine structure of spectral line.
n 2h 2
Bohr radius, rn = = 0.529 n 2 Å (for H-atom)
4p 2kme 2 Z
where, m = mass of electron, h = Planck’s constant, Bohr-Sommerfeld Model
e = charge of electron, p = 3 .14, In order to explain fine E4
n = orbit number, Z = nuclear charge, structure of H-atom, E3 E2
Sommerfeld suggested
k = 1 (in CGS system),
a model which states E1
= 9 ´ 109 Nm 2 C -2 (in MKS system) that electrons are
present in elliptical
(ii) Energy of Electron in Bohr’s Orbit orbit along with
Bohr energy is the sum of kinetic energy and potential circular orbit. They n=4
energy. nf = E1, E2, E3, E4
revolve around the
Bohr energy, E = KE + PE nucleus in two velocity
2 p 2mZ 2e 4k 2 - 13 .6 components viz. radial velocity and angular velocity.
=- = eV
n 2h 2 n2 Radial velocity along the radius vector and angular velocity
- 13 .6 æ Z2 ö which is perpendicular to radius vector give rise to radial
For H-like atom, E n = eV or - 13 .6 ´ ç 2 ÷ eV and angular momentum simultaneously.
n2 èn ø
h
where, Z = atomic number Radial momentum = n r
2p
h
Ionisation energy of electron present in first orbital of Angular momentum = n f ´
2p
H-atom is equal to energy of first orbit of H-atom. As the
where, n r = radial quantum number
value of n (orbit number) increases, the value of Bohr’s
energy increases and becomes zero when n = ¥. n f = angular quantum number

(iii) Velocity of an Electron


Velocity of electron in nth orbit is calculated by given Spectrum of Hydrogen Atom
formula. A spectrum is defined as “pictorial representation of
2 pe 2 kZ æZö
± arrangement of radiation in the increasing order of
v= = 2 .1847 ´ 106 ç ÷ ms-1 wavelength or decreasing order of frequency”.
nh èn ø
Velocity of electron present in first orbit of H Bohr’s Explanation of Hydrogen
æ1ö Spectrum
= vH = 2 .18 ´ 106 ç ÷ = 2 .18 ´ 106 ms-1
è1ø Line spectrum of hydrogen was obtained when a beam of
Velocity of electron present in first orbit of He light was passed through a discharge tube containing
hydrogen gas at low pressure.
2
= vHe = 2 .18 ´ 106 ´ = 4.36 ´ 106 ms-1 Line spectrum of hydrogen atom is classified into the
1
following six series:
i.e. velocity of electron present in first orbit of He is double
the velocity of electron present in first orbit of H. Name of series n1 n2
Lyman series ¬ UV 1 2, 3, 4, ...
Limitations of Bohr’s Model Balmer series ¬ visible 2 3, 4, 5, ...

Bohr’s model was not able to explain the following Paschen series ¬ IR 3 4, 5, 6, ...
phenomena. Brackett series ¬ IR 4 5, 6, 7, ...
(i) Heisenberg uncertainty principle Pfund series ¬ IR 5 6, 7, 8, ...
(ii) Dual character of particle
Humphery series ¬ IR 6 7, 8, 9, ...
(iii) Spectra of multielectron system

@iitjeehelps
ATOMIC STRUCTURE 399
Spectrum of H-atom is expressed in terms of wave number n,
æ 1 1 ö é- 1 ù
Quantum Mechanical Model
n = RH ç 2 - 2 ÷ ên = l ú
è n1 n2 ø ë û and its Important Features
where, R = 1 ,09 ,677 cm -1 (Rydberg constant) On the basis of dual nature of matter and Heisenberg
n 1 = emission to lower energy level uncertainty principle, E Schrodinger
&& , in 1926 gave quantum
n 2 = emission from higher energy level mechanical model for an atom.
For elements other than hydrogen, According to this model, yet exact position and momentum
æ 1 of particle cannot be determined although the probability
1 ö
n = R HZ 2 ç 2 - 2 ÷ of finding an electron can be determined.
è n1 n2 ø
where, Z = atomic number of atom Schrodinger
&& Wave Equation
Schrodinger wave equation of any particle along all three
Modern Structure of Atom directions may be written as
¶2y ¶2y ¶2y 8 p 2m
Following relationship and principle lead to explain the 2
+ 2
+ 2
+ (E - V )y = 0
¶x ¶y ¶z h2
structure of atom
where, y = acceptable wave function
Dual Nature of Matter E = total energy of electron = KE + PE
de-Broglie proposed that microscopic particle show dual V = potential energy of electron
nature similarly as the radiation. Thus, de-Broglie relation h = Planck’s constant
can be stated as “All microscopic particles such as electron, 8 p 2m
proton, etc, shows wave as well as particle like behaviour, or, Ñ 2y + (E - V ) y = 0
h2
i.e. dual nature of behaviour.”
h h h h On rearranging we get Hy = Ey
Mathematically, l= = = =
p mv 2mE 2mqV H = Hamiltonian operator or total energy operator
Now, for hydrogen-atom, it may be solved using wave
Q E = q ×V
function in polar coordinates
and p = 2mE y = R(r )× q (q ) × f ( f )
where, h = Planck’s constant = 6 .626 ´ 10-34 Js Thus, an atomic orbital ( y ) is a product of two factors
m = mass of microscopic particle (i) Radial part, i.e. R(r); depends upon distance and
p = momentum of particle, v = velocity of particle tells about the size of atom.
E = energy of particle, q = charge and (ii) Angular part, i.e. q (q ). f f depends upon angle
V = potential difference variable q and f . It gives an idea about the shape of
orbital.
This equation is known as de-Broglie relation and
applicable for microscopic particle only. Radial part depends upon principal quantum number (n )
and azimuthal quantum number ( l ), while angular part
Wave like nature of microscopic particle is confirmed by
depends upon azimuthal quantum number (l) and
diffraction pattern of electron beam.
magnetic quantum number (m ).

Heisenberg Uncertainty Principle Variation of y and y 2 with r for 1s and 2s


Heisenberg states that exact position and momentum of y 2 has physical significance and it measures the probability
microscopic particle cannot be determined
simultaneously. of finding electron around nucleus.
h y is a wave function of any orbital along 3 directions of
Mathematically, D x × D p ³
4p polar coordinates may be written as
h y (r , q , f ) = R(r ) , q (q ), f ( f )
or D x × Dv ³
4p m Radial Angular
part part

@iitjeehelps
400 SELF STUDY GUIDE BITSAT

There are two ways to represent probability distribution Number of nodal planes = l
curve: where, l = 0 for s-subshell,
(i) Radial probability distribution curve
= 1 for p -subshell,
(ii) Angular probability distribution curve
= 2 for d-subshell,
Radial probability distribution curve tells about the
= 3 for f -subshell,
variation of y 2 (probability of finding electron) with
respect to radial distance (r ), without any reference to its
direction from nucleus.
Electronic Configuration
For both 1s and 2s the shape of orbital is spherical, so it is
more useful to discuss the probability of finding an
electron between the sphere of radius (r + dr ) and r .
Distribution of Electrons in Different
4 4 Orbits (Shells)
Volume of shell = p (r + dr )3 - pr 3 = 4pr 2 dr
3 3 It is the arrangement of electrons in various shells, subshells
and orbitals of an atom.

Bohr Burry Scheme


ψ ψ
The distribution of electrons into different orbits of an atom
was suggested by Bohr and Burry in 1921. According to them,
r r the electrons enter in the orbit in accordance with the
Graph of y vs r following rules
(i) Maximum number of electrons in a shell is given by
For 1s For 2s
formula 2n 2 (n = 1, 2 , 3 , K for K , L , M K shells)
ψ2 ψ2 (ii) The outermost shell can have maximum of
8 electrons.
r
2
r
nl x Types Scheme
Variation of y with respect to r
It is written as 2, 8, 8, 18, 32 or as nl x (where, n indicates the
principal quantum number, l indicates the azimuthal
quantum number or subshell and x is the number of
4π2r2ψ 2 4π2r2ψ 2 electrons).

Rules for Filling Electrons in Orbitals


0.329 (Å) 0.529 2.645 (Å)
r r Various terms and rules have to be followed while filling up
2 2
Variation of 4 pr y with respect to r electrons in orbitals:

Aufbau Rule
Conclusion of Curves Aufbau is a German word having mean ‘‘building up’’, i.e. the
(a) Electron finding probability is zero at nucleus. electrons are filled up in orbitals in the order of increasing
(b) As distance from the nucleus increases, probability energies, i.e. orbital having smaller energy will be filled first
of finding electron increases as 1s < 2s < 3s < 4s .... followed by higher energy orbitals. The arrangement of
(c) Total number of peaks appearing in the curve for orbitals in the increasing order of energy are given as
s , p and d-orbitals are equal to n , (n - 1) and (n - 2 ) 1s , 2s , 2 p , 3s , 3 p ,4s , 3d , 4 p , 5s , 4d , 5 p , 4 f , 5d , 6 p , 7s ...
respectively.
The subshell having lowest (n + l ) value will be filled up first
(d) The region where the probability density function followed by higher value of (n + l ) . If both the orbitals have
( y 2 ) becomes zero is called nodal surface or same (n + l ) value then the orbital having lower value of n
simply node. will be filled up first, this is known as (n + l ) rule.
(e) For ns-orbital, number of nodes = n - 1
Pauli Exclusion Principle
Calculation of Number of Nodes According to Pauli exclusion principle, ‘‘No two
Number of radial nodes = (n - l - 1 ) electrons can occupy same orbital with same value
of all the four quantum numbers’’. An orbital can s-orbital
Number of angular nodes = l
accommodate two electrons with opposite spin as
Total nodes = (n - 1 ) shown in the given figure:

@iitjeehelps
ATOMIC STRUCTURE 401
Two electrons can have same value of n , l and m but value of s (ii) Angular Momentum or
can never be same.
Azimuthal Quantum Number (l)
e.g. in N-atom ( Z = 7) : 1s 2 , 2s 2 , 2 p3
It gives the shape of the orbitals. Values of l varies from
Q.No. 1s 2s 2 px 2 py 2 pz 0 to (n - 1 ) .
Value of l Subshell Shape of orbital
n 1 2 2 2 2
0 s Spherical
l 0 0 1 1 1
1 p Dumb-bell
m 0 0 +1 0 -1 2 d Double dumb-bell
3 f Complicated
s 1 1 1 1 1 1 1
+ , - + , - + + +
2 2 2 2 2 2 2
Orbital angular momentum,
h
Hund’s Rules for Maximum Spin Multiplicity mvr = l( l + 1 )
2p
No electron pairing takes place in p, d and f-orbitals until each
orbital in the given subshell contains one electron having (iii) Magnetic Quantum
parallel spin, e.g. N(7) has electronic configuration 1s 2 , 2s 2 , 2 px1 ,
Number (m l )
2 p 1y , 2 pz1 , according to Hund’s rule and not 1s 2 , 2s 2 , 2 px2 , 2 p 1y .
It says about orientation or distribution of atomic
Extra Stability of Half-filled and orbital.
Completely Filled Orbitals Values of ml = - l to +l
In case of half-filled and fully filled electronic configuration, Note
the extra stability is due to the possibility of larger number (i) Total number of orbitals present in any subshell = 2l +1.
of exchange of electron in degenerate energy levels which (ii) The middle orbital of a subshell has a value of zero.
causes evolution of more exchange energy. Orbitals to the left of middle orbital have –ve numbers; to
the right have +ve numbers.
Me = 0 –1 0 +1 –2 –1 0 +1+2 –3 –2 –1 0 +1+2+3
Incomplete unstable Half-filled stable
p-orbital d-orbital
s p d f
l =0 1 2 3

Incomplete Complete
unstable f-orbital fully-filled stable f-orbital (iv) Spin Quantum Number (ms )
It gives the spin motion of electron and describes the
behaviour of electron.
Quantum Numbers Value of ms Þ for clockwise spin (¼),
There are four quantum numbers 1
s=+
2

(i) Principal Quantum Number (n) For anticlockwise spin (¿),


1
It is the measure of the effective volume of the electron cloud. It s=–
is represented by ‘ n ’. Greater the value of ‘n’ larger is the 2
average distance of an electron in the orbital from the nucleus These spin produces spin angular momentun
and therefore, larger is the orbital. The maximum number of h
electrons possible in a given shell is 2n 2 . = s(s + 1)
2p

@iitjeehelps
402 SELF STUDY GUIDE BITSAT

Practice Exercise
1. The charge on electron was determined by 11. Which of the following electromagnetic radiation has
a. Schrodinger b. Chadwick maximum wavelength?
c. Thomson d. Millikan a. Infrared b. Ultraviolet rays
c. X-rays d. Radiowave
2. Deflection back of a few particles on hitting thin foil of
gold shows that 12. The kinetic energy of proton, whose mass is 140 times
a. nucleus is heavy that of electron and potential difference V is
b. nucleus is small a. 1 KeV b. 1840 KeV
c. Both a and b 1
c. 920 KeV d. KeV
d. electrons create hindrance in the movement of 1840
a-particles
13. In first Bohr orbit of hydrogen atom, the velocity of
3. A photon of frequency n has momentum electron would be (given that radius of first Bohr orbit
h nc
a. hn/c b. hc/n c. d. is 0 . 53 ´ 10-10m)
nc h
a. 2.2 ´ 106 m/s b. 3.3 ´ 106 m/s
4. The energy of one quantum of light with a wavelength
of 6500 Å (1Å = 10-10 cm ) c. 1.1 ´ 106 m/s d. 4.4 ´ 106 m/s
a. 9.04 ´ 10-24 J b. 3.02 ´ 10-20 J 14. Bohr atomic model explains
c. 3.06 ´ 10-19 J d. 6.02 ´ 10-20 J a. finer details of atomic spectra
b. splitting of spectral lines in the presence of magnetic
5. Sodium lamp emits yellow light of wavelength 5800 Å. and electric field
The wave number of the light is c. spectra of hydrogen or hydrogen like atoms
a. 1. 72 ´ 106/ m b. 17.2 ´ 108/ m d. intensity of various spectral lines
c. 5.17 ´ 1014/ s d. 17.2 ´ 107/ m 15. The expression for Bohr radius of nth orbit of an atoms
16 is
6. The ultraviolet radiation has frequency 6 ´ 10 / s. The
n 2h 2 nh
wavelength would be a. r = b. r =
6 -7 4p 2mZe 2 4p 2mZe 2
a. 2 ´ 10 cm b. 5 ´ 10 cm
c. 18 ´ 10-26 cm d. 2 ´ 107 cm n 2h 2 n 2h 2
c. r = d. r =
4p 2mZ 4p 2me 2
7. A radiation of 2000 Å falls on the metal whose work
function is 4.2 eV. Then the kinetic energy of the 16. The potential energy of the electron in the hydrogen
fastest photo electron is atom is - 6.8 eV. The excited state in which the
a. 6.4 ´ 10-10 J b. 16 ´ 10-10 J electron is present is
a. first b. second
c. 1. 6 ´ 10-19 J d. 3 . 2 ´ 10-19 J
c. third d. None of these
8. The threshold wavelength for the ejection of electron 17. The energy required for the ionisation of excited
from metal is 330 nm, then work function for the hydrogen atom would be (in eV)
photoelectric emission is a. < 13.6 b. > 13.6
-12 -19
a. 6 ´ 10 J b. 6 ´ 10 J c. 13.6 d. None of these
c. 1.2 ´ 10-20 J d. 1. 2 ´ 10-18 J
18. For hydrogen atom, radius of first Bohr’s orbit is a.
2 . 859 For Li2+ , the radius of first Bohr’s orbit is
9. It is given that E = cal / mol. The energy
l a. 27a b. 3a c.
a
d.
a
associated with radiation of wavelength 4 ´ 10-5 m will 27 3
be 19. If the velocity of an electron in the first orbit of
a. 71. 5 kcal / mol b. 35.75 kcal / mol hydrogen atom is approximately 2 . 2 ´ 108 cm / s , is
c. 32 . 0 kcal / mol d. 7.15 kcal / mol velocity in the fourth orbit would be
10. The ratio of energy of a photon of 2000 Å radiation to a. 5.5 ´ 107 cm/s
that of 4000 Å radiation. b. 4.4 ´ 107 cm/s
1 1 c. 3.3 ´ 107 cm/s
a. 2 b. 4 c. d.
4 2 d. 6.6 ´ 107 cm/s

@iitjeehelps
ATOMIC STRUCTURE 403
20. Energy of an electron in hydrogen atom is given by 30. In Bohr’s series of lines of hydrogen spectrum, the
13 . 6 third line from the red end corresponds to which one
E =- eV. If n is changed from 1 to 4, then
n2 of the following inter orbit jumps of the electron for
energy of 1 is Bohr’s orbit in an atom of hydrogen?
a. four times that of 4 b. five times that of 4 a. 5 ® 2 b. 4 ® 1 c. 2 ® 5 d. 3 ® 2
c. sixteen times that of 4 d. equal that of 4
31. The wavelength of the radiation emitted when the
21. The radii of nuclei and atoms are known to be of the electron jumps from 4th shell to 2nd shell is
order of 10-13 cm and 10-8 cm respectively assuming a. 4862 Å b. 2056 Å c. 5241Å d. 109700 cm
them to be spherical. The fraction of atomic volume
occupied by the nucleus would be 32. First line of Paschen series has wave number
a. 10-15
b. 10 -17 (R H = 109700 / cm )
c. 10-14 d. 10-16 a. 2854 ´ 108 (Å)-1 b. 6243 ´ 108 (Å)-1
22. Which of the following statements is false? c. 6856 ´ 108 (Å)-1 d. 3452 ´ 108 (Å)-1
a. Spin quantum number corresponds to two possible 33. Which of the following statement is false?
directions of spin
a. Violet radiations have a longer wavelength than red
b. No two electrons in an atom may have the same set
radiations
of values for the four quantum numbers
b. The energy of light with l = 600 nm is lower than that
c. Number of electrons in an atom is equal to the
number of orbitals of l = 500 nm
d. The momentum and position of an electron cannot c. Spectrum of an atom is known as line spectrum
be known simultaneously d. The wavelength associated with an electron is longer
than that of proton if they have the same speed
23. For hydrogen atom, radius of first Bohr orbit would be
34. The effect of splitting of spectral lines under the
a. 9.63 ´ 10-10m b. 0.43 ´ 10-10m
influence of magnetic field is called
c. 0 . 50 ´ 10-10m d. 0 . 53 ´ 10-10m a. photoelectric effect b. Zeeman effect
24. The relation between radius of third orbit r 3 and radius c. Raman effect d. Stark effect
of first orbit r1 in hydrogen atom would be 35. In the influence of electric field the splitting of spectral
a. r3 = 3r1 b. r3 = 9r1 c. r1 = 3r3 d. r1 = 9r3 lines is called
a. Zeeman effect b. Stark effect
25. The electron of hydrogen atom is excited to certain c. photoelectric effect d. None of these
level. When the electron returns to the first Bohr orbit,
the wavelength of line emitted, if the energy difference 36. The visible part of electromagnetic radiations is
is 11.0 eV would be a. 2000 Å to 4000Å b. 2000 Å to 5000 Å
a. 11. 25 ´ 10 m-7 -6
b. 2 . 2 ´ 10 m c. 4000 Å to 8000 Å d. 1000 Å to 2000 Å
c. 9.1176 ´ 10-8m d. 1.22 ´ 10-7m 37. In hydrogen spectrum wave number of different lines
26. Ionisation energy of He+ is – 19 . 6 ´ 10-18 J. The 1 é 1 1ù
is given by = RH ê 2 - 2 ú
energy of the first stationary state (n = 1) of Li2+ is l ë ni nf û
a. 4.41 ´ 10-16 J atom-1 b. - 4.41 ´ 10-17 J atom-1 where, R H = 1.090678 ´ 107m -1
c. -2.2 ´ 10-15 J atom-1 d. 8.82 ´ 10-17 J atom-1
The wavelength of first line of Lyman series would be
27. Assuming atom to be a sphere, the radius of atom a. 122 ´ 10-7m b. 9.1176 ´ 10-8m
would be [if the apparent volume of atom of a metal is c. 12 . 2 ´ 10-7m d. 1. 22 ´ 10-7m
1. 23 ´ 10-23 mL] 38. The mass of a photon of wavelength 1. 54 Å is
a. 1. 401Å b. 1. 942 Å c. 1. 642 Å d. 1. 541Å a. 2 . 5 ´ 10-32 kg b. 1. 42 ´ 1032 kg
28. In hydrogen an electron transition takes place from c. Both of these d. None of these
n = 2 level to n = 3 level. The wavelength of the line in 39. The uncertainty in the velocity of moving bullet of
the hydrogen spectrum would be mass 10 g, when uncertainty in its position is 10-5 m is
7 -7
a. 5.485 ´ 10 m b. 6.56 ´ 10 m
a. 5 . 2 ´ 10-28 m/s b. 5.2 ´ 10-22 m/s
c. 4.57 ´ 1014m d. 0.529 ´ 10-10m
c. 3 ´ 10-28 m/s d. 3 ´ 10-22 m/s
29. The line spectra are characteristics of
40. The momentum of a particle having de-Broglie
a. molecules in ground state
b. atoms in ground state
wavelength of 6Å is
c. molecules in the excited state a. 1. 1 ´ 1034kg - m/ s b. 39.6 ´ 10-34kg - m/ s
d. atoms in excited state c. 1.1 ´ 10-24kg - m/ s d. 39.6 ´ 10-24kg - m/ s

@iitjeehelps
404 SELF STUDY GUIDE BITSAT

41. When an electron is moving uniformly, it produces 51. For the valency electron in copper, the four quantum
a. both electric and magnetic fields numbers are
b. an electric field only a. n = 4, l = 0, m = 0, s = + 1/2
c. a magnetic field only b. n = 4, l = 2, m + 2, s = - 1/2
d. no such fields 1
c. n = 4, l = - 2, m = 2 , s = +
42. Which of the following scientists demonstrated the 2
wave nature of electron? 1
d. n = 4, l = 2, m = 0, s = +
a. Davisson b. Heisenberg 2
c. de-Broglie d. Schrodinger
&&
52. What will be the uncertainty in velocity of a cricket ball
43. Which one of the following statements is most of 100 g if the uncertainty in its position is 1. 65 Å?
appropriate?
a. Electron spins around its own axis only 10-23 6.6 -45 -1
a. ms-1 b. 10 ms
b. Electron moves around the nucleus in spherical p p
-26
orbits 10
c. 4.65 ´ 10-43 ms-1 d. ms-1
c. Electron moves around the nucleus in elliptical orbits p
d. Electron moves around the nucleus in spherical or
elliptical orbits spins around its own axis 53. The electrons, identified by quantum numbers n and l,
(i) n = 4, l = 1 (ii) n = 4, l = 0 (iii) n = 3, l = 2
44. The maximum number of electrons that can be taken (iv) n = 3, l = 1 can be placed in increasing order of
by a subshell with l = 3 is energy, from the lowest to highest, as
a. 8 b. 14
a. (iv) < (ii) < (iii) < (i) b. (ii) < (iv) < (i) < (iii)
c. 10 d. 12
c. (i) < (iii) < (ii) < (iv) d. (iii) < (i) < (iv) < (ii)
45. The spectrum of He+ is expected to be similar to that of 54. An element has atomic number 37. The electronic
a. hydrogen b. Li+
configuration of the element is
c. Na d. He
a. ( 2, 8), 3s 2 3p 6 3d 10, 4s 25s 6 4p 5
46. The orbital with n = 3 and l = 2 is
b. ( 2, 8), 3s 2 3p 6 3d 10, 4s 2 4p 6, 5s 1
a. 5d b. 3 d
c. ( 2, 8), 3s 2 3p 6,4s 2 3d 95s 14p 5
c. 4d d. 5s
d. None of the above
47. If n and l are respectively the principal and azimuthal 55. The number of unpaired electrons in ferrous ion is
quantum numbers, then which of the following a. 5 b. 2
expression represent the total number of electrons in c. 4 d. 3
any energy level
I =n l =n -1 56. The triad of the nuclei that is isotonic
a. å 2( 2l + 1) b. å 2( 2l + 1) a. 6 C14, 7 N14, 9 F19 b. 6 C 14, 7 N15, 9 F17
l =0 l =1 c. 6 C 14, 7 N14, 9 F17 d. 6 C 12, 7 N14, 9 F19
l =n + 1 l =n -1
c. å 2( 2l + 1) d. å 2( 2l + 1) 57. In which of the following orbital diagram Aufbau
l =0 l =0 principle is not violated
48. Which of the following set of quantum number is not a.
applicable for an electron in an atom?
a. n = 2, l = 0, m = 0, s = + 1/2 b.
b. n = 1, l = 0, m = 0, s = - 1/2
c. n = 1, l = 0, m = 0, s = + 1/2 c.
d. n = 1, l = 1, m = 1, s = + 1/2
d.
49. The maximum number of electrons that d-orbital can
contain is
a. 10 b. 2 58. Nitrogen has electronic configuration
c. 14 d. 6
1s 2 2s 2 2p 1x 2p 1y 2p 1z . This is because of
50. The shape of the orbital is determined by a. Pauli’s exclusion principle
a. spin quantum number b. (n + 1) rule
b. magnetic quantum number
c. azimuthal quantum number c. Hund’s rule
d. principal quantum number d. Uncertainty principle

@iitjeehelps
ATOMIC STRUCTURE 405

BITSAT Archives
1. Which of the following relation is incorrect regarding 7. An electronic transition in hydrogen atom result in the
Bohr’s theory? [2014] formation of Ha line of hydrogen in Lyman series, the
1 energies associated with the electron in each of the
a. Velocity of electron µ
n orbits involved in the transition (in kcal mol–1) are [2008]
1 a. -313.6 , - 34.84 b. -313.6, - 78.4
b. Frequency of revolution µ
n2 c. -78.4, - 34.84 d. -78.4, - 19.6
c. Radius of orbit µn 2 / Z
1 8. The velocities of two particels A and B are 0.05 and
d. Force on electron µ 4
n 0.02 ms –1 respectively. The mass of B is five times the
2. Bohr theory is applicable to [2013]
mass of A. The ratio of their de-Broglie’s wavelength
is [2008]
a. He b. Li2+
c. He2+ d. None of these a. 2 : 1 b. 1 : 4 c. 1 : 1 d. 4 : 1

3. If the radius of H is 0 . 53 Å then what will be the radius 9. Cr has electronic configuration as [2007]
2+ a. 3s 2 3p 2 3d 4, 4s 1 b. 3s 2 3p 6 3d 5, 4s 1
of 3 Li ? [2012]
c. 3s 2 3p 6 3d 6 d. None of these
a. 0.17 Å b. 0.36 Å c. 0.53 Å d. 0.59 Å
10. The number of waves in an orbit are [2006]
4. Which of the following has the largest de-Broglie a. n 2 b. n
wavelength, given that all have equal velocity? [2011] c. n - 1 d. n - 2
a. CO2 molecule b. NH3 molecule
c. Electron d. Proton 11. The probability of finding the electron in the orbital is
a. 100% b. 90-95% [2006]
5. The wave number of a spectral line is 5 ´ 105 m –1. The
c. 70-80% d. 50-60%
energy corresponding to this line is [2010]
a. 3.49 ´ 10-23 kJ b. 4.45 ´ 10-24 J 12. The velocity of electron in first orbit of H-atom as
c. 5. 50 ´ 10-22 J d. 9 . 93 ´ 10-23 kJ compared to the velocity of light is [2005]
1 1
a. th b. th
6. Energy of third orbit of Bohr’s atom is [2009] 10 100
a. -13.6 eV b. -3.4 eV 1
c. th d. same
c. -1. 5 eV d. None of these 1000

Answer with Solutions


Practice Exercise 6. (b) l =
c
=
3 ´ 108
= 0 . 5 ´ 10-8 m = 5 ´ 10-7 cm
n 6 ´ 1016
1. (d)
2. (c) 7. (d) Kinetic energy ( KE) = hn - W
h hc 6.62 ´ 10-34 ´ 3 ´ 108
3. (c) l = = - 4.2eV = - 4 . 2 ´ 1.6 ´ 10-19
p l 2000 ´ 10-10
where, p = momentum = 9.9 ´ 10-19 J - 6.7 ´ 10-19 J = 3.2 ´ 10-19 J
h h
h = Planck’s constant Þ P = = 8. (b) W = hn 0
l cn
hc 6.62 ´ 10-34 ´ 3 ´ 108
hc 6.625 ´ 10-34 ´ 3 ´ 108 W = = = 6 ´ 10-19 J
4. (c) E = hn = = l0 330 ´ 10-9
l 6500 ´ 10-10
2.859
E = 3.06 ´ 10-19 J 9. (a) Given, E = cal / mol
l
1
5. (a) Wave number = l = 4 ´ 10-7
l
1 2.859 cal / mol
= = 1.724 ´ 106 / m E= = 71. 5 kcal /mol
5800 ´ 10-10 m 4 ´ 10-5 m

@iitjeehelps
406 SELF STUDY GUIDE BITSAT

hc v1 r2 ær ö 1
10. (a) E = = Þ v 2 = ç 1 ÷ v1 Þ v 2 = v1
l v 2 r1 è 2ø
r 4
E1 l 2 4000 Å 1
= = =2 v2 = ´ 2 .2 ´ 108
E2 l1 2000 Å 4
11. (d) Radiowave has maximum wavelength of the order of v 2 = 0 . 55 ´ 108 cm/s
102 in metre.
= 5 . 5 ´ 107 cm/s
12. (a) According to law of conservation of energy, ‘‘Sum of
potential energy and kinetic energy remain constant’’. 1
20. (c) Eµ
Now initially proton starts from rest. Hence, kinetic energy n2
of proton at initial point is zero but it has potential energy. E1 ( 4)2
=
PE = qv E4 (1)2
where, q = 1. 6 ´ 10-19 C and V = 1000 V E1
E4 =
Assuming the proton no longer experiences the potential 16
energy and it is all converted to kinetic energy Volume of nucleus
21. (a) = fraction of atom
1 Volume of an atom
PE* = 0, KE* = mv 2
2 Volume occupied by nucleus
4
Since, KE + PE = Total energy = PE* + KE* p (10-13 )3
3 = 10-15
1 4
qv + 0 = 0 + mv 2 or KE = qv = 1.6 ´ 10-16 J = 1KeV -8 3
p (10 )
2 3
13. (a) From Bohr postulate 22. (c) Number of electrons in an atom is equal to the number
nh nh of orbitals.
mvr = or v =
2p 2pmr n 2h 2
23. (d) r =
1´ 6.625 ´ 10-34 4p 2me 2z
v = = 2 . 2 ´ 106 m/s
2 ´ 3.14 ´ 9.1´ 10-31 ´ 0 . 53 ´ 10-10 n = 1, h = 6.62 ´ 10-34 J s–1
14. (c) Spectra of hydrogen or hydrogen like atoms such as \ r = 0 .53 ´ 10-10 m = 0 . 53 Å
He+ , Li2+ were successfully explained by Bohr's model.
n 2h 2
n 2h 2 n2 n2 24. (b) r = , i.e. r µ n 2
15. (a) r = 2 = R 0 = 0 . 529 Å 4p 2me 2z
4p mZe 2 Z Z hc
1 25. (a) E2 - E1 = DE =
16. (a) Total energy = En = (Potential energy) l
2
hc 6.62 ´ 10-34 ´ 3 ´ 108
- 6.8 l= =
En = = - 3.4 eV DE 11. 0 ´ 1. 6 ´ 10-19
2
13.6 eV l = 11.25 ´ 10-7m
We know that, En = -
n2 26. (b) IE = – E1
-13.6 eV E1 for He+ = - 19.6 ´ 10-18 J atom-1
-3.4 eV =
n2 (E1) (Z )2
He+ He+ -19.6 ´ 10-18 4
n2 = 4 = Þ =
(E1)Li2+ ( Z Li2+ )2 (E1) 2+ 9
Li
n = 2 First excited state
-19.6 ´ 9 ´ 10-18
17. (a) (n = 2) ® Energy of first excited state = - 3.4 eV or E1( Li2+ ) =
4
(n = 3) ® Energy of second excited state = – 1. 51 eV = - 4.41´ 10-17 J atom–1
r (H) Z 2+ 4 3
18. (d) 2+
= Li 27. (a) V = pr
r (Li ) ZH 3
1/ 3
ZH 1´ a a 3V æ 3V ö
r (Li+2 ) = rH = = =r3 Þ r =ç ÷
Z Li+2 3 3 4p è 4p ø

nh 1 é 3V ù
19. (a) mvr = log r = log ê
2p 3 ë 4p úû
hn 1 é 3 ´ 1. 23 ´ 10-23 ù
v = log r = log ê ú
2pmr 3 ë 4 ´ 3.14 û

@iitjeehelps
ATOMIC STRUCTURE 407
1 é 1 1ù 42. (c) Electron behaves as a material as well as wave, this
28. (b) = 109677 ê 2 - 2 ú = 6 . 56 ´ 10-7 m was proposed by de-Broglie.
l ë n1 n 2 û
h
29. (d) Radiations obtained by exciting a substance has l=
mv
dark space between two consecutive lines. This is line
spectrum. 43. (d) Electron moves around the nucleus in spherical or
elliptical orbits and spins around its own axis.
30. (a) The lines at the red end belong to Balmer series.
These are obtained for jumps to n = 2 from outer orbits 44. (b) When l = 3, m = - 3, - 2 - 1, 0, 1, 2, 3. Each m can have
(1st line from n = 3, 2nd line from n = 4, 3rd line from two electrons with opposite spins, thus maximum number of
n = 5). electrons = 2 ´ 7 = 14
31. (a) According to Balmer equation 45. (a) Both H and He+ have one electron.
æ 1 1ö él = 0, 1, 2, 3ù
Wave number (v ) = 109677 ç 2 - 2 ÷ cm–1 46. (b) 3d orbital ê ú
è n1 n 2 ø ë = s , p, d , f û
æ 1 1 ö –1 47. (d) e.g. when n = 3, l = 0, 1, 2
v = 109677 ç 2 - ÷ cm 2
è ( 2 ) ( 4 )2 ø
109677 ´ 3
Total electrons = å 2( 2l + 1)
= cm–1 I =0
16 = 2 + 2 [ 2 ´ 1 + 1] + 2 [ 2 ´ 2 + 1]
1 16 = 2 + 6 + 10 = 18 electrons
l= = = 4862 ´ 10–8 cm
n 109677 ´ 3 48. (d) For n = 1, l = 1 not possible
= 4862 ´ 10–10 m = 4862 Å Since, l = 0 to (n - 1)
32. (c) ni = 3, nf = 4 49. (a) 10 electrons
+2 +1 0 –1 –2
1 é 1 1ù
= 109677 ê 2 - 2 ú = 6856 ´ 108( Å)-1 d-orbital
l n
ë i nf û
50. (c) The shape of the orbital is determined by azimuthal
33. (a) Violet radiations have a lower wavelength than red quantum number.
radiations.
51. (a) The valence electronic configuration of copper is
34. (b) The splitting of spectral lines under the influence of 3d 10 4s 1
magnetic field is called Zeeman effect.
Hence, n = 4, l = 0, m = 0, s = + 1/ 2
35. (b) In the influence of electric field the splitting of
spectral lines is called Stark effect. 52. (a) According to Heisenberg uncertainty principle,
h
36. (c) The visible part of electromagnetic radiation is D x ´ mDv =
4000 Å to 8000 Å. 4p
37. (d) For first line in Lyman series 100 6.625 ´ 10-34
1.65 ´ 10-10 ´ ´ Dv =
1000 4p
ni = 1, nf = 2
h h 6.625 ´ 10-34 + 10 10-23
38. (b) l = = Dv = ´ 10 = m/ s
mv lv 1 .65 ´ 4 p p
where, v = 3 ´ 108 m / s; h = 6.625 ´ 10-34 Js 53. (a) (i) 4p (ii) 4s (iii) 3d (iv) 3p
l = 1. 54 ´ 10-10m Order of increasing energy is 3p < 4s < 3d < 4p
\ m = 1.42 ´ 10-32 kg 54. (b) Rb = ( Z = 37);( 2, 8), 3s 2 3p 6 3d 10, 4s 2 4p 6, 5s 1
h h 55. (c) Ferrous ( Fe2+ ) ion. Its electronic configuration is d 6.
39. (a) D x . Dp = or Dx mD v =
4p 4p
h 6.626 ´ 10-34 Js Number of unpaired electrons = 4
Dv = =
4p D xm 4 ´ 3.14 ´ 10-5m ´ 10–2 kg 56. (b) Number of neutrons are
= 5.2 ´ 10-28 m/s C ¾¾® 14 - 6 = 8
h 6.62 ´ 10-34 N ¾¾® 15 - 7 = 8
40. (c) p = =
l 6 ´ 10-10 F ¾¾® 17 - 9 = 8
-24
p = 1.1 ´ 10 kg - m/ s 57. (b)
1s 2s 2px 2py 2pz
41. (a) Moving electron produces both electric and
magnetic fields. 58. (c) N—

@iitjeehelps
408 SELF STUDY GUIDE BITSAT

BITSAT Archives
1. (c) Radius of orbit is directly proportional to ratio of For H-atom,
square of principal quantum number and atomic number Z =1
n2 For Lyman series,
i.e. Radius of orbit a
Z n1 = 1, n 2 = 2
n 2 ´ 0 . 529 ´ 10-8 Energy of electron in n1 orbit
For H-atom rn = cm
Z 313 . 52 ´ (1)2
=- kcal mol–1
2. (b) Bohr’s theory is applicable to H-like species containing (1)2
one electron only, e.g. Li 2+ .
= - 313 .52 kcal mol–1
3. (a) The radius of hydrogen atom = 0 . 53 Å. 3 Li2+ ion also
has only one electron but it has 3 protons in nucleus, » - 313.6 kcal mol–1
hence, its electron feels three times more attraction from Energy of electron in n 2 orbit
nucleus in comparison to hydrogen atom. Thus, the
313 . 52 ´ (1)2
radius of 3 Li2+ will be =- kcal mol–1
( 2 )2
0 . 53
= = 0.17 Å 313 .52
3 =- kcal mol–1
4
4. (c) l = h / mv
= - 78 . 38 kcal mol–1
For same velocity l a 1/ m
8. (a) Given velocity of particle A = 0.05 ms–1
Electron has the least mass, so its wavelength is
maximum. Velocity of particle B = 0.02 ms–1
hc – æ 1ö Let the mass of particle A = x
5. (d) Energy E = = hc v çQv = ÷
l è lø \The mass of particle B = 5x de-Broglie's equation is
= 6.62 ´ 10-34 ´ 3 ´ 108 ´ 5 ´ 105 h
l=
= 9 .93 ´ 10-23 kJ mv
h
- 13.6 Z 2 For particle A lA = ...(i)
6. (c) En = eV x ´ 0.05
n2
h
where, Z = atomic number and n = number of orbit For particle B lB = ...(ii)
5x ´ 0.02
For H, Z = 1, and for 3rd orbit n = 3
13.6 13.6 l A 5x ´ 0.02
\ E3 = - =- = - 1. 5 eV Eq. (i)/(ii) =
( 3 )2 9 lB x ´ 0.05
lA 2
7. (b) Energy of an electron in nth orbit, =
lB 1
2p 2Z 2e 4
En = or 2:1
n 2h 2
On substituting the values of k, m, e and h, we get 9. (b) Cr ( 24) = 1s , 2s 2p 6, 3s 2 3p 6 3 d 5, 4s 1
2 2

2.172 ´ 10-18 Z 2 10. (b) The number of waves in an orbit is equal to n.


En = - J atom–1 (principal quantum number)
n2
1311. 8 Z 2 313 . 52 Z 2 11. (b) The probability of finding the electron in the orbital is
= 2
kJ mol–1 = - kcal mol–1 90-95%.
n n2
12. (b)
[Q1 kcal = 4.184 kJ ]

@iitjeehelps
3
Nuclear Chemistry

Radioactivity
‘‘It is a process in which nuclei of certain elements undergo spontaneous disintegration without
excitation by any external means.’’ And such elements whose atoms disintegrate and emit radiations
are called radioactive elements. On applying electric or magnetic field, these radiations were split into
three types of rays: a, b and g-rays.
Comparison of a, b and g-rays
Properties a-particle or rays b-particle or rays g-rays

Charge and It carries 2 unit positive charge and It carries 1 unit negative charge These are electromagnetic rays
mass 4 unit mass. and has no mass. with very short wavelength
(approx. 0.05 Å).
Nature It is represented as helium nucleus It is represented as It is represented as 0 g 0 .
or helium ion, 2 He 4 or He 2+ . electron -1 e 0 .

Action of a These are deflected towards the These are deflected towards the These are not deflected.
magnetic field cathode. anode.
Velocity 2 ´ 10 9 cm/s or 2 ´ 10 7 m/s 2.36 to 2.83 ´ 10 10 cm/s or 2.36 Same as that of light,
1 to 2.83 ´ 10 8 m/s 3 ´ 10 10 cm / s or ( 3 ´ 10 8 m/s)
( th of velocity of light).
10 1 9
( rd to th of velocity of light)
3 10
Ionising power Very high, nearly 100 times to that of Low, nearly 100 times to that of Very low
b-rays. g-rays.
Effect on ZnS They cause luminescence. Very little effect. Very little effect
plate
Penetrating Low, easily stopped by air. 100 times to that of a-particles. 10,000 times to that of
power b-particles.
Kinetic energy High due to their high mass. Very less due to their small Zero
mass.
Nature of Product obtained by the loss of Product obtained by the loss of There is no change in the atomic
product 1 a-particle has atomic number less 1 b-particle has atomic number number as well as in mass
by 2 units and mass number less by 4 more by 1 unit, without any number, e.g.
units, e.g. change in mass number, e.g.
n-4
y A n ¾® y - 2A + 2 He 4 y A n ¾® y + 1 An + -1 e
0
y A n ¾® y A n + 0g 0

NOTE Rate of radioactive disintegration is independent of temperature, pressure and chemical combination, etc.

@iitjeehelps
410 SELF STUDY GUIDE BITSAT

Group Displacement Law Units of Radioactivity


It was first stated by Fajans and Soddy in 1913. According to 1 curie = 3.7 ´ 1010 dps
them, “in an a-emission, the parent element will be 1 rutherford = 106 dps
displaced to a group two places to the left and in a
b-emission, it will be displaced to a group one place to the Becquerel is the SI unit of radioactivity
right.’’ becquerel = 1 dps
222 - 2 He 4 218
e.g. 88 Ra ¾ ¾ ¾® 86 Rn
Group II Group 0
Nuclear Stability
0 The stability of the nucleus is decided by the following factors :
84 Po
209
¾-¾¾
-1b
® 85 At 209
Group IV Group V
(i) Neutron-Proton Ratio
It has been observed that the stability of nucleus depends
Disintegration Series upon the neutron to proton ratio (n / p ) .
There are four radioactive disintegration series: It has been found that
(i) The thorium series (4n series) (a) n/p ratio for stable nuclei = 1 (for elements with atomic
number £ 20) and n/p ratio for stable nuclei > 1
(ii) The neptunium series (4n + 1 series)
(for elements with atomic number > 20)
(iii) The uranium series (4n + 2 series)
(b) When n/p ratio is higher than the ratio required for
(iv) The actinium series (4n + 3 series) stability, the nuclei have the tendency to emit b-rays,
i.e. a neutron is converted into proton.
Kinetics of Radioactive Disintegration (c) When n/p ratio is lower than the ratio required for
The rate of radioactive disintegration is found , to decrease stability, the nuclei increases the ratio either by
with time. Radioactive disintegration is found to follow first emitting a-particle or by emitting a positron or by
order kinetics and hence, for first order radioactive K-electron capture.
disintegration, the rate constant or specific reaction rate (k (ii) Mass Defect and Binding Energy
or l) is given by Mass of an atom is always less than the sum of masses of its
2.303 N constituent nucleons (neutrons and protons). This loss of
l or k = log 0 ...(i)
t N mass is called mass defect ( Dm ). This mass defect, in fact,
where, N 0 = amount of element at start, changes into energy according to Einstein equation
N = amount of element remained after t time E = D m ×c 2 ...(iv)
and k = disintegration constant (also denoted by l ) We can calculate the binding energy (BE), if mass defect is
known as BE equivalent to ( Dm = 1 amu ) is 931 MeV.
Half-life Period
Hence, BE = Dm ´ 931 MeV …(v)
Half-life is the period, in which a radioactive element
The total binding energy of a nucleus divided by the
disintegrates to half of its original mass. It is represented by
number of nucleons, is called specific binding energy or
t 1/ 2.
N binding energy per nucleon. It is denoted by B.
when, t = t 1 / 2 then, N = 0 , Total binding energy of nucleus
2 i.e. B=
2.303 N0 Number of nucleons
Hence, k= log
t 1/ 2 N0 / 2 Greater is the value of specific binding energy (B) of a
nucleus, greater is its stability. Nuclei having specific
2.303 log 2
= binding energy, ranging from 7. 5 - 9.0 MeV per nucleon, are
t 1/ 2 most stable.
0.693
t 1/ 2 = ...(ii)
k Artificial Transmutation
It is defined as the conversion of one element into another
Average Life Period
or one type of atoms into another. When this conversion is
It is the reciprocal of disintegration constant and is achieved by artificial means, i.e. by means of bombarding
represented by t av . with some fundamental particles, it is known as artificial
1 t transmutation,
Average life (t av ) = = 1 / 2 = 1.44 ´ t 1 / 2 ...(iii)
k 0693
. e.g. 7N
14
+ 2 He 4 ¾® 8 O17 + 1 H 1

@iitjeehelps
NUCLEAR CHEMISTRY 411
Fundamental particles used in the bombardment of different Nuclear Fusion
elements are It is a reaction, in which lighter nuclei combine to form a
a-particle ( 2 He 4 ), proton ( 1 H 1 ), deutron ( 1 H 2 ) or ( 1 D2 ) single heavier nucleus. In this process also, some mass
and neutron (0 n 1 ) defect changes into energy called nuclear fusion energy.
2
e.g. 1H + 1H 2 ¾® 2He 4 + 23 . 7 MeV
Artificial Radioactivity Nuclear fusion differs from nuclear fission in mainly two
Many stable nuclei when bombarded with high speed respects :
particles produce unstable nuclei that are radioactive. The (i) Unlike nuclear fission, nuclear fusion require very high
radioactivity produced in this manner by artificial means is temperature (of the order of 106° C) and high pressure.
known as artificial radioactivity or induced radioactivity.
(ii) Nuclear fusion reaction cannot be controlled in
contrast to nuclear fission.
Nuclear Reactions Thus, nuclear fusion is much hazardous. Hydrogen bomb is
A nuclear reaction is the process, in which a change in the based on this reaction. Also the source of solar energy is
composition of nucleus occur. It results in the formation of nuclear fusion reactions between hydrogen nuclei, which is
another element. While writing a nuclear reaction, the sum continuously occurring on sun.
of the mass numbers and atomic numbers should be same
on both sides of the reaction.
Other Important Terms
Nuclear reactions are of various types as given below :
1. Magic Numbers
Particle-particle Reactions The nuclei with protons and/or neutrons equal to 2, 8, 20,
In these reactions, bombarding particle ( 2 He 4 , 0 n 1 , 1 H 1 , 1 H 2 , 28, 50, 82, and 144 have been found much stable. These
0
1 b , etc.) is absorbed by stable nucleus to become unstable numbers are called magic numbers.
and it emits other particle and form stable nuclei.
24 2. Packing Fraction (PF)
e.g. 11 Na + 2 He 4 ¾® 13 Al
26
+ 2 0n 1
Actual isotopic mass - mass number
PF = ´ 104
Spallation Reactions Mass number
In these reactions, a high speed projectile chips a fragment PF depends upon the manner of packing of the
off from a heavier nucleus and a large number of lighter nucleons within the nucleus. Its value can be negative,
particles are emitted. positive or even zero. Higher the negative PF value,
e.g. 118
+ 1 H 1 ¾® 31 Ga 72 + 10 2 He 4 + 7 0n 1 more stable is the nucleus.
50 Sn
NOTE
Nuclear Fission (i) Lower the packing fraction, greater is the binding energy per
In 1939, German scientists, Otto Hahn and F Strassman nucleon and hence, greater is the stability.
discovered these type of reactions. They bombarded U-235 (ii) Packing fraction is highest for H and least for Fe.
nuclei with slow moving neutrons and found that this
nucleus splitted into two lighter nuclei with the liberation
3. Uses of Isotopes
14 238
of three neutrons and a large amount of heat energy, i.e. 6C is used for radiocarbon dating. 92 U is used for

92 U
235 1
+ 0 n ¾® 56 Ba
141
+ 36 Kr 92 1
+ 3 0n + D H determining age of rocks, earth or an ore, etc. Co-60 is used
in the treatment of cancer.
Such reactions are called nuclear fission reactions and the 131
energy produced is called nuclear energy. About 199 MeV of 53 I is used to detect functioning of thyroid gland. 11 Na 24 is
energy is released by the fission of one nucleus of uranium. used to detect blood clot and position of tumour. Besides
The source of this nuclear energy is actually mass defect these uses, various isotopes are used as radioactive tracers
( Dm ) occurring in the reaction, which changes into the in various fields of research.
energy according to Einstein equation.
4. Age of Old Piece of Wood
In this reaction, two or three neutrons are also produced,
which split other uranium nuclei. Thus, this reaction goes 0693
. 2.303 C 14 in living plants
l= = ´ log 14
on increasing and then it is called a chain reaction. t 1/ 2 t C in dead plants
In chain reaction, a huge amount of nuclear energy is where, t = age of wood
obtained which can be used either for destructive purpose and t 1 / 2 = half-life of C 14
or for constructive purpose.

@iitjeehelps
412 SELF STUDY GUIDE BITSAT

5. Age of the Earth or Rock 6. Relation between Disintegration


It is determined by uranium-lead method using the Per Second and Amount of Substance
formula. dN - dN 6.023 ´ 1023 ´ w
- = l ×N, = l×
0.693 dt dt M
l=
t 1/ 2 - dN
where, = number of disintegration per sec.
2.303 (U 238 + Pb 206 ) dt
= log
t U 238 N = 6.023 ´ 1023 per gram mole of substance
NOTE 206 g Pb 206 º 238 g U238 w
= 6.023 ´ 1023 ´ atoms
M
\ x g Pb 206 º æç ´ x ö÷ g U238
238
è 206 ø w = weight and M = molecular weight

Practice Exercise
1. The nuclear radius as compared to the atomic radius, 8. Loss of b -particle is equivalent to
is of the order of a. decrease of one neutron only
a. 10-2 b. 10-15 b. increase of one proton only
c. 10-4 d. 10-8 c. Both (a) and (b)
2. Arrange the following particles in the increasing order d. None of the above
of their penetration capacity as the projectiles for 9. Which of the following processes cause the emission
artificial transmutation of elements : of X-rays?
Proton (1 H 1) , alpha particle ( 2 He4 ), deuteron ( 1 H 2 ), a. a-emission b. g-emission
c. Positron emission d. Electron capture
neutron ( 0n1) 235
a. 2 He4 < 1H2 < 1H1 < 0n1 b. 1H1 < 1H2 > 0n1 < 2He4
10. What will be the product, if 92 U emits two a-and
one b-particle?
c. 1H1 < 1H2 < 2He4 < 0n1 d. 0n
1
< 1H 2 < 2 He4 < 1 H 2 211 235
a. 87 Ac b. 89 Ac
225 227
3. Which of the following radiations is most easily c. 89 Ac d. 89 Ac
stopped by air? -a -b
a. b -rays b. a-rays
11. In the reaction, Po ¾® Pb ¾® Bi, if Bi belongs to
group 15, to which group Po belongs?
c. X-rays d. g-rays
a. 13 b. 14 c. 15 d. 16
4. b -particle is emitted in radioactivity by 12. Which of the following transformation is not correct?
a. conversion of proton into neutron a. 33 As75 + 2 He4 ¾®35 Br 78 + 0 n1
b. from outermost orbit
b. 3 Li7 + 1H1 ¾® 4 Be7 + 0n1
c. conversion of neutron into proton
d. b-particle is not emitted c. 8 O16 + 0n1 ¾® 6C14 + 2He4
d. 83 Bi209 + 1H 2 ¾® 84 Po210 + 0n1
5. 7 N13 changes to 6 C13 by the emission of
a. electron b. neutron 13. For the nuclear reaction, 0n1 ¾® 1H
1
+ ?,
c. positron d. proton the missing nuclide is
0
6. Which of the following processes are feasible? a. -1e b. + 1e 0 c. 1H 2 d. 2 He4
1 1 2
a. 1p + 0n ¾® 1D + n 14. The end product of the series, starting with 232
, is
90 Th
b. 1H1 + -1e 0 ¾® 0n1
a. Pb-208 b. Pb-206 c. Bi-209 c. Pb-207
c. 0n1 ¾® 1p1 + -1e 0 + n
d. None of the above
15. The number of a-and b -particles emitted in the
228 212
24 nuclear reaction, 90 Th ¾® 83 Bi , respectively
7. 11Na is a radioactive and it decays to
are
a. 12 Mg24 and b-particles b. 11Na 21 and neutron a. 4 a and 1b b. 3 a and 7 b
c. 13 F24 and positron d. 9 F20 and a-particles c. 8 a and 1b d. 4 a and 7 b

@iitjeehelps
NUCLEAR CHEMISTRY 413
16. The radioactive disintegration follows the kinetics of 29. The decay constant of a radioactive sample is l . The
a. zero order b. first order half-life and mean life of the sample respectively are
c. second order d. third order 1 ln 2 ln 2 1 1 l 1
a. , b. , c. l ln 2, d. ,
17. The amount of substance that gives 3.7 ´ 10 dps is 7 l l l l l ln 2 l
a. one becquerel b. one curie 30. Which is the most stable nucleus among these?
c. one millicurie d. one rutherford a. 2 He3 b. 11Na 24 210 207
c. 83 Bi d. 82 Pb
214
18. The weight of 1 curie 82 Pb (t1/ 2 = 26.8 min) in grams
31. Sometimes, the ejection of an a-particle does not
is completely stabilise the nucleus. In such a case, more
a. 3 .1 ´ 10-8 g b. 1. 55 ´ 10-8 g a-particles may be emitted. The a-decay
c. 6.2 ´ 10-8 g d. 3.1 ´ 10-10 g a. raises the n/p ratio and is often followed by
b-emission
19. Radium has atomic weight 226 and a half-life of b. lowers the n/p ratio and is often followed by positron
1600 yr. The number of disintegrations produced per emission
second from one gram are c. raises the n/p ratio and is often followed by neutron
a. 4.8 ´ 1010 b. 9.2 ´ 106 c. 3.7 ´ 1010 d. zero emission
d. lowers the n/p ratio and is often followed by g-rays
20. Only 1/8th of the original amount of a radioactive emission
element remains after 96 min. The value of t1/ 2 of this
element is 32. Which of the following nuclear reaction is incorrect?
a. 12.0 min b. 32.0 min c. 24.0 min d. 48.0 min a. 20 Ca 40 + 0n1 ¾® 19 K 40 + 1H1
21. The half-life of radium is 1600 yr, after how much time b. 12 Mg24 + 2 He4 ¾® 14 Si27 + 0n1
will 1 g radium be reduced to 125 mg? c. 48 Cd113 + 0n1 ¾® 48 Cd112 + -1e0
a. 800 yr b. 1600 yr d. 20 Co43 + 2 He4 ¾® 21Sc46 + 1H1
c. 3200 yr d. 4800 yr
33. The easily fissionable isotope of uranium is with the
22. If one starts with 1 curie of radioactive substance number
(t1/ 2 = 12 h), the activity left after a period of 1 week a. 236 b. 235 c. 237 d. 238
will be about 34. Atom bomb is based on the principle of
a. 1 curie b. 129 microcurie a. carbon dating b. nuclear transformation
c. 60 microcurie d. 8 millicurie c. nuclear fission d. nuclear fusion
23. 20 mg of C-14 has a half-life of 5760 yr. 100 mg of 35. The compound used in the enrichment of the uranium
sample containing C-14 is reduced to 25 mg in in nuclear power plant is
a. 57600 yr b. 1440 yr a. UCl4 b. UO2(NO3 )2
c. 17280 yr d. 11520 yr
c. UF6 d. U3O8
24. In the case of a radioisotope, the value of t1/ 2 and l
are identical in magnitude. The value of t1/ 2 is
36. If uranium (mass number = 238 and atomic number
0.693
= 92) emits an a-particle, the product has mass
a. 1/0.693 b. ( 0.693)1/ 2 c. d. 0.693 number and atomic number respectively are
2 a. 236 and 92 b. 236 and 90
25. The half-life of a radioactive isotope is 3 h. If the initial c. 238 and 90 d. 234 and 90
mass of the isotope was 300 g, the mass which 37. In a nuclear reactor, heavy water is used
remained undecayed in 18 h would be a. to provide high speed neutrons
a. 1.17 g b. 9.36 g b. to increase the speed of neutrons
c. 2.34 g d. 4.68 g c. to capture neutrons produced by nuclear fission
26. The half-life period of a radioactive element is d. to transfer the heat from the nuclear reactor
140 day. After 560 days, 1 g of the element will reduce 38. Which one of the following is an exact example of
to artificial radioactivity?
a. 1/8 g b. 1/16 g a. 0n1 + 11Na 23 ® 11Na 24 + g;11Na 24 + 1H1 ® 12 Mg24 + 0n1
c. 1/4 g d. 1/2 g
b. 2 He4 + 7 N14 ® 8 O17 + 1H1; 8 O17 + 0n1 ® 7 O18 + g
27. If 5 g of a radioactive substance has t1/ 2 = 14 h, 10 g of c. 2 He4 + 13 Al27 ® 15 P30 + 0n1; 15 P30 ® 14 Si30 + 1e 0
the same substance will have t1/ 2 equal to d. 89 Ac228 ® 90 Th228 + b; 90 Th228 ® 88 Ra 224 + a
a. 70 h b. 14 h c. 28 h d. 50 h
39. The function of the cadmium rod in a nuclear reactor
28. Choose the incorrect one. is
a. 1 curie = 3.7 ´ 1010 ds-1 b. 1 rutherford = 106 ds-1 a. to produce rays b. to produce neutrons
c. 1 becquerel = 1 ds-1 d. 1 fermi = 103 ds-1 c. to absorb neutrons d. to speed up neutrons

@iitjeehelps
414 SELF STUDY GUIDE BITSAT

40. The nuclear reaction 12 H + 12 H ¾® 42 He is called 44. An isobar of 20 Ca


40
is
40
a. fission reaction b. fusion reaction a. 18 Ar b. 20 Ca 38
c. chain reaction d. thermal reaction c. 20 Ca 42 d. 18 Ar 38
41. Which of the following nuclei is not doubly magic? 45. An isotope of 231
can be converted into 227
by
4 16 208 238 90Th 90Th
a. 2 He b. 8 O c. 82 Pb d. 92 U the emission of
42. Atom A possesses higher values of packing fraction a. one a-particle
than atom B. The relative stabilities of A and B are b. one b-particle
a. A is more stable than B c. two a and one b-particle
b. B is more stable than A d. one a and 2 b-particles
c. A and B both are equally stable
d. Stability does not depend on packing fraction 46. Nuclides having the same difference in mass number
and atomic number, are known as
43. Match the Column I and Column II and pick the a. isotopes b. isobars
correct matching from the codes given below. c. isotones d. isomers
Column I Column II 47. Which of the following statements is wrong?
228 228 a. P32 is used in the treatment of leukemia
A. Isotope 1. 88 Ra and 89 Ac b. I131 is used the treatment of thyroid gland cancer
39 c. Co59 cannot be used in the treatment of cancer
B. Isobar 2. 18 Ar and 19 K 40
d. Excessive use radioactive elements is responsible of
2 cancerous growth
C. Isotone 3. 1H and 1 H3
48. The isotope of carbon used in radiocarbon dating is
D. Isosters 4. 92 U
235
and 90 Th
231
a. 6 C12 b. 6 C13
c. 6 C14 d. 6 C15
E. Isodiaphers 5. CO 2 and N2O
49. Which one of the following radioisotopes is used in the
The correct matching is treatment of blood cancer?
a. Na 24 b. I131
Codes
c. Co62 d. P32
A B C D E
a. 5 4 1 2 3 50. The proper rays for radio carbon dating are
b. 3 1 2 5 4 a. cosmic rays b. X-rays
c. 2 1 4 5 3 c. UV rays d. IR rays
d. 2 5 1 4 3

BITSAT Archives
11
1. 1 g of U-235 is converted into UF6. The radioactivity of 3. If the mass defect of 5B is 0.081 u, its average
UF6 thus obtained is [2011] binding energy (in MeV) is [2008]
a. zero a. 8.60
b. less than that of 1 g of U-235 b. 6.85
c. more than that of 1 g of U-235 c. 5.60
d. same as that of 1 g of U-235 d. 5.86
2. A sample of radioactive substance shows an intensity 4. When radioactive minerals like clevelte, monozite and
of 2.3 millicurie at a time t and an intensity of pitch blende are heated to 1273 K in vacuo the noble
1.62 millicurie after 600 s. The half-life period of the gas obtained is [2007]
radioactive metal is [2010] a. Rn b. Kr
a. 1000 s b. 1187 s c. 1200 s d. 1500 s c. He d. Ne

@iitjeehelps
Answer with Solutions
Practice Exercise =
228 - 212 16
= =4
4 4
1. (c) The nuclear radius is of the order of 10-14 m while that
Number of b-particles = 2 ´ a-particle
of atom is of the order of 10-10 m. Hence,
- (difference in atomic number)
Nuclear radius 10-14
= = 10-4 = 2 ´ 4 - (90 - 83) = 8 - 7 = 1
Atomic radius 10-10
Hence, a and b-particles, emitted are 4 and 1 respectively.
2. (a) The penetrating power increases with increase in
velocity of particles. Thus, the correct order of penetrating 16. (b) The radioactive disintegration follows the first order
power of given particles is kinetics, for which decay constant is given by
1 1 2
0 n > 1 H > 1 H > 2 He
4 2.303 N
l= log 0
t N
3. (b) a-rays have less penetrating power.
17. (c) The amount of substance, which give 3.7 ´ 1010
4. (c) It is considered that b-particle is emitted from the
disintegration per second, is known as 1 curie. Hence,
nucleus by the conversion of neutron into proton.
1 3.7 ´ 107 dps = 1millicurie
0n ¾® 1H1 + -1 b
0

-dN 6.023 ´ 1023 ´ w


5. (c) 7 N13 ¾® 6C13 + 1e 0 18. (a) We know that, =l
dt M
Positron
- dN
Positron is an anti-particle of electron ( -1e 0 ). Here, = 1 curie = 3.7 ´ 1010 dps
dt
6. (c) The electron does not exist in the nucleus but it is 0.693 0.693
formed at the time of emission and indicates as l= = s- 1
t1/ 2 26.8 ´ 60
1
0n ¾® 1p1 + -1e
0
+n
0.693 6.023 ´ 1023 ´ w
24 24 \ 3.7 ´ 1010 = ´
7. (a) 11Na is a radioactive isotope and it decays to 12 Mg 26.8 ´ 60 214
and b-particles, which is shown as
24
or w = 3.1 ´ 10-8 g
11 Na ¾® 12Mg24 + -1 b 0
19. (c) Given, M = 226, t1/2 = 1600 yrs,w = 1 g
8. (c) 0n1 ¾® 1H1 + -1e 0
- dN 6.023 ´ 1023 ´ w 0.693 6.023 ´ 1023 ´ 1
So, it is equivalent to increase of one proton and =l = ´
dt M t1/ 2 226
decrease of one neutron.
9. (d) X-rays are produced when high energy electrons are 0.693 ´ 6.023 ´ 1023
=
captured by a metal of high atomic number. 1600 ´ 226 ´ ( 365 ´ 24 ´ 3600)

10. (d ) 235 -2 a
¾® 227 -b
¾® 227 = 3.7 ´ 1010 disintegrations per second
92 U 88X 89 Ac
1
209 -a -b 20. (b) Given that , N0 = 1, N = , t = 96 min
11. (d ) 84 Po ¾® 82 Pb205 ¾® 83 Bi
205
8
(16 group) (14 group) (15 group)
t 96
Thus, according to group displacement law, Po belongs to æ 1ö t 1 æ 1ö t
We know that, N = N0 ç ÷ 1/ 2 or = ç ÷ 1/ 2
16 group. è 2ø 8 è 2ø
12. (c) Mass number are not balanced in 96
3
æ 1ö æ 1ö t 96 96
8O
16 1
+ 0n ¾® 6 C 14
+ 2 He
4 or ç ÷ = ç ÷ 1/ 2 or = 3 or t1/ 2 = = 32 min
è 2ø è 2ø t1/ 2 3
Therefore, it is not correct transformation 125
21. (d ) Given, t1/ 2 = 1600 yr, N0 = 1 g, N = 125 mg = g
13. (a) 0n1 ¾® 1H
1
+ -1e
0
1000
14. (a) 90 Th232 is in ( 232 = 4 ´ 58 = 4n ) series. The end t
æ 1ö t
product of this series will be, thus, Pb- 208 as We know that, N = N 0 ç ÷ 1/ 2
è 2ø
( 208 = 4 ´ 52 = 4n ) .
t /1600 3 t /1600
15. (a) Number of a-particles 125 æ 1 ö æ 1ö æ 1ö æ 1ö
=ç ÷ , ç ÷ or ç ÷ = ç ÷
æmass number - mass number ö 1000 è 2 ø è 8ø è 2ø è 2ø
ç of emitter of end product ÷ø t
è Hence, = 3 or t = 3 ´ 1600 = 4800 yr
= 1600
4

@iitjeehelps
416 SELF STUDY GUIDE BITSAT

22. (c) Given, t1/ 2 = 12 h. 207 125


n / p ratio for 82 Pb = = 1.53
t 7 ´ 24 82
æ 1ö t æ 1 ö 12 Hence, 82 Pb207 is the most stable as its n/p ratio lies
t = 1 week = 7 ´ 24 h, N = N0 ç ÷ 1/ 2 = 1 ´ ç ÷
è 2ø è 2ø between the stability range, i.e. (1.00 - 1.53).
14
æ 1ö 31. (a) The n/p ratio increases in case of a-decay while
=ç ÷ = 6.1 ´ 10-5 curie = 60 microcurie
è 2ø decreases during b-decay.
23. (d ) Half-life period for a nucleus is a constant quantity. It 32. (c) A nuclear reaction must be balanced in terms of
does not change with amount or mass. Hence, atomic numbers and mass number. Hence, following
t reaction is unbalanced.
æ 1ö t 48 Cd
113
+ 0n1 ¾® 48 Cd
112
+ -1e 0
t1/ 2 = 5760 yr, N = N0 ç ÷ 1/ 2
è 2ø 235
33. (b) The most unstable isotope of uranium is 92 U .
t t
2
25 æ 1 ö 5760 æ 1ö æ 1 ö 5760 34. (c) Atom bomb is based on the principle of nuclear fission.
=ç ÷ or ç ÷ = ç ÷
100 è 2 ø è 2ø è 2ø 35. (c) The fuel used in nuclear power plant is enriched in
t uranium-235 in the form of UF6.
\ = 2 or t = 2 ´ 5760 = 11520 yr 238
5760 36. (d ) 92 U ¾® 90 Th234 + 2 He4
0.693
24. (b ) We know that, t1/ 2 = or (t1/ 2 )2 = 0.693 (as, t1/ 2 = k ) 37. (d)
k
38. (c) Nucleus bombarded should be stable and nucleus
or t1/ 2 = ( 0.693)1/ 2 produced should be radioactive.
n 6
æ 1ö æ 1ö 39. (c) Cadmium is used to absorb neutron.
25. (d) N = N0 ´ ç ÷ = 300 ´ ç ÷ = 4.68 g
è 2ø è 2ø 40. (b)
Total time 560 41. (d ) The magic numbers for protons are 2, 8, 20, 28, 50,
26. (b) Number of half-lives (n ) = = =4
t1/2 140 82, 114 and for neutrons are 2, 8, 20, 28, 50, 82, 126,
n 4 184, 196.
æ 1ö æ 1ö 1
Amount left = ç ÷ ´ Initial amount = ç ÷ ´ 1g = g 42. (b) The atom which has lower value of packing fraction is
è 2ø è 2ø 16
more stable.
0.693
27. (b) t1/ 2 = and it is independent of initial amount or 43. (b)
l
mass. 44. (a)
28. (d ) The nuclear radius is expressed in fermi (Fm) units. 45. (d) Isotopes of an element can be obtained by emitting
-13 -15 one a- and 2b-particles from its nucleus.
1 fermi = 10 cm = 10 m
231 -a -b -b
loge 2 1 90 Th ¾ ¾¾® 88 Ra 227 ¾ ¾¾® 89 Ac227 ¾ ¾¾® 90 Th227
29. (b) t1/ 2 = and average life =
l l 46. (c)
1
30. (d ) n / p ratio for 2 He3 = = 0.5 ( < 1) 47. (c)
2
48. (c)
13
n / p ratio for 11Na 24 = = 1.19 (greater than that of 49. (c) Co62 is used in the treatment of blood cancer.
11
23 50. (a)
stable isotope 11Na )
210 127
n / p ratio for 83 Bi = = 1.54 ( > 1.53)
83

BITSAT Archives
1. (d ) Radioactivity does not depend upon the state of Binding energy = 931 ´ Dm MeV
combination so it remains unaffected. = 931 ´ 0.081 = 75.411 MeV
N 2.30 2.303 2.30 Binding energy
2. (b) 0 = , Now, l = log = 0.000584 Average binding energy =
N 1.62 600 1.62 Number of nucleons
0.693 0.693 75.411
\ t1/ 2 = = = 1187 s =
l 0.000584 11
3. (b) Given, D m for 5 B11 = 0.081u = 6.85 MeV
Number of nucleons = 11 4. (c)

@iitjeehelps
4
Chemical Bonding

Bond or Chemical Bond


A bond is a kind of interatomic, intermolecular or inter-ionic attraction which holds two or more
constituents (atoms, molecules or ions) together, e.g. ionic bond, covalent bond etc. It is formed so that
the atoms acquire stability.

Valency
The combining capacity of different constituents is called their valency. In case of elements of I A and
II A group, it is equal to their group number, i.e. + 1 and + 2 respectively.
For other elements, it varies from group number to group number − 8,
i.e. Valency = group number to (group number − 8)
Here, group number are in accordance with modern periodic table, e.g. nitrogen an element of group V
A, has valency from + 5 to − 3 ( = 5 − 8 ).

Lewis Octet Rule


The tendency of atoms to acquire 8 electrons in their outermost shell is known as Lewis octet rule.

Exceptions
(a) Incomplete octet BF3 , BeCl 2 etc. (b) Octet expansion PCl 5 , BF6 etc.

Ionic Bonding
Electrons are completely transferred from electropositive element to electronegative element
forming charged ions which are held together by strong electrostatic forces.

e.g. Na + •• Cl •• → Na+ + Cl − ≡ NaCl
(2, 8, 1) •• (2, 8) (2, 8, 8)
(2, 8, 7)

@iitjeehelps
418 SELF STUDY GUIDE BITSAT

Conditions Favourable for Ionic Covalent Bonds


Bonding Such chemical bonds are formed by sharing of electrons
(a) The electropositive element should have low ionisation between the elements of almost same electronegativity or
energy. between the elements having less difference in
(b) The electronegative element should have higher electronegativity.
electron affinity. e.g. Formation of O 2 molecule
(c) Lattice energy should be high. 8O = 2, 6
O == O
Lattice Energy
A lattice is a regular arrangement of cations and anions in an O O
ionic crystal. The amount of energy released, when free ions
combine to form one mole of a crystal lattice, is known as
lattice energy. Dipole Moment (µ)
Factors affecting of Lattice energy
µ = q ×d
(a) Size of ions
where, q = charge
(b) Charge on ions and d = distance between positive and negative charges
NAZ + Z −e 2 ue 2 Unit → Debye, 1 D = 10−18 esu cm
Lattice energy (LE) = − + n
r r
In symmetric molecules, µ = 0
(Born Lande equation)
Higher the value of µ , more is the polarity of bond.
where, Z + , Z − are the charges on cation and anion respectively.
Observed µ
e = electronic charge, N = Avogadro’s number %Ionic character = × 100
Calculated µ
r = interionic distance, A = Madelung constant
Covalency → number of electrons contributed by an
u = Born exponent
elements
Born Haber Cycle e.g. Formation of Cl 2 molecule
•• •• •• ••
e.g. Formation of NaCl • • • Cl •
1 • Cl • → •• Cl •• Cl •• ≡≡ Cl Cl or Cl 2
Na(s) + Cl2(g) •• •• •• ••
2 ∆Hform
Formation of CO2 → molecule
Sublimation 1/2 DE
∆Hs energy •• ••

NaCl (crystal) • O •• •• C •• •• O •• ≡≡ O == C == O or CO2
Na(g) Cl(g)
The above structures are known as electron-dot
IE –EA structures or Lewis structure.
Na+(g) + Cl–(g) LE
1 Properties of Covalent Compound
∆Hform=∆Hs + IE+ DE+EA+LE
2 (a) They generally exist in gaseous or liquid form or
More is the negative value of ∆H form, more stable will be the sometimes as soft solids (e.g. S,P etc). Sometimes, they
compound. exists as hard solids, e.g. SiC, BN etc.
(b) They have low boiling point and melting point except
some covalent solids such as SiC, diamond, SiO2 etc.
Characteristics of Ionic Compounds
(c) In general, covalent compounds are bad conductors
(a) They are crystalline in nature with 3-D solid aggregates
of heat and electricity due to the absence of ions or
due to non-directional force of attraction.
free electrons. An exception is graphite, since in
(b) They are good conductors of electricity and heat, but graphite, electrons can pass from one layer to
only in aqueous phase of molten state. In solid state, another.
there is no free electron.
(d) Non-polar covalent compounds are soluble in
(c) They have very high melting and boiling point. non-polar solvents such as CCl 4 , C6H6 , CS2 etc.
(d) They are soluble in polar solvents. (e) Covalent substances show slower chemical reactions.
(e) Ionic reactions are very fast due to presence of ions. (f) Covalent substances show isomerism.

@iitjeehelps
CHEMICAL BONDING 419

Fajan’s Rule Pi ( π ) Bond


The partial covalent character of an ionic bond has It is formed by incomplete, sidewise or parallel overlapping of
been explained by Fajan as follows: orbitals.
When a cation approaches an anion, the electrons
cloud of the anion is deformed which is called +
polarisation. Greater the polarisation, more is the
covalent character.
Covalent character ∝ polarising power of cation p-orbital p-orbital p-p overlapping
(sidewise)
1

size of cation Double bond has one σ-bond and one π-bond. Triple bond has two
π-bonds and one σ-bond.
Polarisability of anion ∝ size of anion
Coordinate Covalent (Dative) Bond 2. Valence Shell Electron Pair
A bond in which complete pair of shared electrons
is contributed by the same element. Atom
Repulsion Theory (VSEPR theory)
contributing its extra electron pair is called donor Main Postulates
while other atom is called acceptor and the bond is
(a) In polyatomic molecules, one atom is called central atom to
represented by an arrow (→ ) from donor to
which all other atoms (or group of atoms) are linked.
acceptor.
(b) The shape of any molecule depends upon the total number of
e.g. NH +4 : NH3 → H + valence shell electron pairs on central atom.
H3O+ : H 2O → H + (c) If all the atoms linked to central atom (having no lone pair) are
same then molecule is symmetrical.
(d) The relative order of repulsion is as follows:
Theories Developed for lp − lp > bp − lp > bp − bp
Covalent Bonding
Concept of Hybridisation
1. Valence Bond Theory (VBT) (Involving s, p, d and f-orbitals)
(Given by Heitler and London) Intermixing of orbitals of almost similar energy belonging to the
It explains bond formation in terms of same atom to form same number of orbitals of exactly equal energy
overlapping of orbitals, e.g. the formation of H 2 is known as hybridisation. The new orbital thus formed are known
molecule from two hydrogen atoms involves the as hybrid orbitals.
overlap of 1s-orbital of two H-atoms which are
Features of Hybridisation
singly occupied.
Only valence shell atomic orbitals of an atom or ion takes part in
Because of orbital overlap, electron density
hybridisation. Types of hybridisation along with geometry and
between the nuclei increases which helps in
bond angles are listed below.
bringing them closer.
Types of Hybridisation
The overlapping of orbitals may results in two types of
bonds given below: Number of Bond
Hybridisation Geometry
hybrid orbitals angle
Sigma (σ ) Bond 2 sp Linear 180°
It is the result of end to end overlapping or axial 3 sp 2
Triangular planar 120°
overlapping between s-s, p -p, s-p orbitals,
3
e.g. single bond. 4 sp Tetrahedral 109.5°
dsp 2 Square planar 90°
The electron density accumulates between the
centres of the atoms being bonded. 5 sp 3 d Trigonal 120°, 90°
bipyramidal
σ-bond
6 sp 3d 2 or d 2sp 3 Octahedral 90°
+
7 sp 3d 3 Pentagonal 72°, 90°
s-orbital s-orbital s-s overlapping bipyramidal
(axial)

@iitjeehelps
420 SELF STUDY GUIDE BITSAT

The hybridisation of the molecule can be determined by +


+ + +
1
H = [V + Y − C + A] σ1s
2 1s 1s

where, V = valence shell electrons of the central metal


+ –
Y = number of monovalent atoms +
+ –
C = total positive charge σ
* 1s
1s 1s
A = total negative charge on the molecule
H 2 3 4 5 6 7 When atomic orbitals overlap laterally, the resulting
Hybridisation sp sp 2
sp 3
sp d3 3
sp d 2
sp d 3 3 molecular orbital energetic is called a π-molecular orbital.
It has no cylindrical symmetry and contains a nodal plane
along the internuclear axis. π-orbital is more energetic
Resonance than σ-orbital.
Single Lewis structure of some molecules is not adequate to
explain the properties. Hence, the molecule could be + + +
represented by more than one structural molecule. These
+ Nodal
structure having similar energy are called resonance plane
structures.
– – –
e.g. CO32− (carbonate ion) can be represented as

– – 2p 2p π 2p
O O O
O C O C O C
O– O– O + –
+ –
I II III
2– +

O
≡≡ O C – + + –
O–
Resonance hybrid 2p 2p π2 2p
IV

I, II and III are resonating structures of CO32− ion. The


structure IV is known as resonance hybrid which describes
Relative Energy of MO
the structure of CO32− more accurately. Case I Total e − ≤ 14

σ1s < σ* 1s < σ2s < σ* 2s < π2 p y


3. Molecular Orbital Theory
= π 2 pz < σ 2 px < π* 2 p y = π* 2 pz < σ* 2 px
(1932 Mulliken and Hund)
When the atoms to be bonded come close together, the Case II Total e − > 14
orbitals of the bonded atoms loose their individual
σ1s < σ* 1s < σ2s < σ* 2s < σ2 px < π2 pz
character and overlap to form larger orbitals called
molecular orbitals, in which electron move under the = π 2 p y < π* 2 pz = π* 2 p y < σ* 2 px
influence of many nuclei. Molecular Orbitals (MO) possess
1
different energy levels and their shapes depend on the Bond order = (number of electrons in bonding orbitals
atomic orbitals from which they are formed. The 2
combining orbitals must be of comparable energy and − number of electrons in anti-bonding orbitals)
should overlap to a larger extent. The filling of electrons in 1
= (N b − N a )
molecular orbital is governed by Aufbau principle, Hund’s 2
rule and Pauli’s exclusion principle.
If there are unpaired electrons in any molecular orbitals,
Formation of bonding and antibonding molecular then the substance is paramagnetic otherwise it is
orbitals A sigma molecular orbital has cylindrical
diamagnetic. If N B ≤ N A , the molecule is unstable.
symmetry around the internuclear axis and has no nodal
plane along this axis.

@iitjeehelps
CHEMICAL BONDING 421
e.g. (a) He 2
Total electrons = 4
Some Weaker Bonds
Molecular orbital electronic configuration = σ1s 2 σ* 1s 2 Hydrogen bond
1
Bond order = (2 − 2 ) = 0 (cannot exist) It can be defined as the attractive force which binds hydrogen
2 atom of one molecule with the electronegative atom (F, O or N)
(b) O2 of another molecule. Cl has some electronegativity as nitrogen
Total electrons = 16 but it does not form strong H-bonds due to its large size.
Strong H-bond exist in KHF2 .
Molecular orbital electronic configuration
δ+ δ− δ+ δ− δ+ δ−
σ1s 2 σ* 1s 2σ2s 2 σ* 2s 2σ 2 px2 π 2 p 2y π 2 pz2 < π* 2 p 1y π* 2 pz1 −−−−− H— X −−− H— X −−− H— X −−−−

1 Hydrogen bond
Bond order = (10 − 6 ) = 2
2 1. Intermolecular hydrogen bond
O2 is paramagnetic molecule as it contains R— O KH — O KH — O K
unpaired electrons.   
H H R
Non-Bonding Combination of AOs 2. Intramolecular hydrogen bond
In such an overlap of AOs, any stabilisation due to OH
positive-positive combination is destabilised by an
equal amount of positive negative overlap. C O
Hydrogen bond
O H

+ Effects of hydrogen bond


(a) Association
+ (b) High melting point and boiling point
– (c) Solubility in water
s (d) Viscosity and surface tension
py
van der Waals’ Forces
These are effective only in non-polar molecules.
Metallic Bond van der Waals’ force ∝ molecular weight
Attraction between the metallic cations and the mobile ∝ number of electrons
electrons which hold the crystal together, is known as
∝ atomic weight
metallic bond.
Boiling point ∝ van der Waals’ forces
Some properties of metals are :
Metallic luster, electrical conductivity, thermal NOTE The relative strength of various bond is as follows:
conductivity, malleability, ductility, high tensile lonic bond > covalent> metallic > hydrogen bond >
van der Waals’ forces
strength etc.

@iitjeehelps
Practice Exercise
1. Which of the following is not correct? 9. Which one has covalent as well as ionic valency?
a. Na + + Cl− → NaCl a. NaCl b. HCl c. H2O d. NaOH
b. Ca 2+ + 2 F − → CaF2
10. In NO−3
ion, the number of bond pairs and lone pairs of
c. Na + + F − → NaF
electrons on nitrogen atom respectively are
d. Ca + + 2Na+ → CaNa 2
a. 2 and 2 b. 3 and 1 c. 1 and 3 d. 4 and 0
2. Which postulation provide the basic for the modern 11. Which one of the following contains both ionic and
concepts regarding ion formation by electron transfer
covalent bonds?
and the formation of ionic crystalline compounds?
a. C6H 5Cl b. H2O
a. Kossel’s postulations b. Langmuir’s postulations
c. NaOH d. CO2
c. Newton’s postulations d. Lewis postulations
3. In chlorine atom, how many electrons are short of the 12. Lewis dot structures of CO, NO−2 and CO2−
3 are
argon configuration? I, II and III respectively given below.
a. One b. Two c. Three c. Four O 2–

4. Which is not paramagnetic? –


C O [O N O] O C O
a. O 2 b. O+2 c. O−2 d. O2−
2
I II III

5. Which of the following is the weakest bond? Which of these structure(s) is/are wrong?
a. Hydrogen bond b. Covalent bond a. Only I b. Only II
c. Ionic bond d. Metallic bond c. Only III d. I, II and III
6. Which of the following compounds has the smallest 13. In the following electron dot structure, calculate the
bond angle? formal charge from left to right nitrogen atom
a. H2O b. H2S c. NH3 d. CO2 respectively :
•• ••
7. The molecule which contains ionic as well as covalent N == N== N
•• ••
bond, is
a. −1, −1and +1
a. NH4Cl b. H2O
b. −1, +1and −1
c. CaCl2 d. CCl4
c. +1, −1and −1
8. Consider the following Lewis structure of HNO3 , d. +1, −1and +1
(2)
O 14. Lattice energy of a solid increases if
H O N a. size of ions is small
(3)
b. charges of ions are small
(1) O
c. ions are neutral
d. None of the above
The formal charge on O(1) , O(2) and O(3) are given in
column II. Match the following and choose the correct 15. If the electronic configuration of an element is
option from the codes given below. 1s 2 2s 2 2p 6 3s 2 3 p 6 3d 2 4s 2, four electrons involved in
Column I Column II chemical bond formation will be……. .
a. 3 p 6 b. 3 p 6, 4 s 2
A. O1 1. Zero c. 3 p 6, 3 d 2 d. 3 d 2, 4 s 2
B. O2 2. −1
16. The electronic configuration of the outermost shell of
C. O3 3. +3
the most electronegative element is
a. 2s 2 2p 5 b. 3s 2 3 p 5
Codes
c. 4s 2 4p 5 d. 5s 25p 5
A B C
a. 1 2 3 17. An electronic arrangement is said to be stable if its
b. 2 1 2 outer shell consists
c. 1 1 2 a. doublet of electrons b. triplet of electrons
d. 3 2 1 c. octet of electrons d. singlet of electron

@iitjeehelps
CHEMICAL BONDING 423
18. Which is the most covalent? 25. In which of the following molecule/ion, all the bonds
a. C—F b. C—O are not equal?
c. C—S d. C—Br a. XeF4 b. BF4−
19. Which of the following molecules is correct regarding c. C2H4 d. SiF4
BeCl2?
26. If the electronegativity difference between two atoms
a. It violates octet rule and hassp 2-hybridisation A and B is 2.0, then the percentage of covalent
b. It has sp-hybridisation and follow octet rule character in the molecule is
c. It violates octet rule and has linear structure a. 54% b. 46%
d. All of the above are true c. 23% d. 72%

20. The correct order of increasing covalent character of 27. Which of the following is a favourable factor for cation
the following is formation?
a. SiCl4 < AlCl3 < CaCl2 < KCl a. Low ionisation potential
b. High electron affinity
b. KCl < CaCl2 < AlCl3 < SiCl4
c. High electron negativity
c. AlCl3 < CaCl2 < KCl < SiCl4 d. Small atomic size
d. None of the above
28. Match the reaction given in Column I with enthalpy in
21. The metallic lustre exhibited by sodium is explained Column II and choose the correct option from the
by codes given below.
a. diffusion of sodium ions
b. excitation of free protons Column I Column II
c. oscillation of loose electrons A. Mg → Mg 2+
+ 2e − 1. Electron gain enthalpy
d. existence of body centred cubic lattice B. O → O 2− − 2e − 2. Ionisation enthalpy
22. Arrange the following ionic compounds in order of C. M (g ) → M + (g ) + e −
increasing ionic character:
D. X (g ) → X − (g ) − e −
KF KCl KBr KI
A B C D Codes
a. A<B<C<D A B C D
b. D<C<B<A a. 1 2 1 2
c. B<A<C<D b. 2 1 2 1
c. 1 1 2 2
d. C<A<B<D
d. 2 2 1 1
23. Consider the Born-Haber cycle for the formation of an
29. In the formation of a molecule, only the outer shell
ionic compound given below.
electrons take part in chemical combination and are
∆H1 ∆H 2 + known as
M (s ) → M ( g ) → M ( g ) a. valence electrons
∆H 3 ∆H 4 ∆H5 b. inner electrons
1
X 2 ( g ) → X ( g ) → X − ( g ) → Z c. inert electrons
2 d. reactive electrons
Here, Z refers to 30. How many double bonds are present in carbon
a. M + X − (l ) b. M + X − (s ) dioxide molecule?
c. MX 2 d. M + X − (g ) a. One b. Two c. Three d. Four
24. Which of the following structures is/are correct? 31. Which of the following hybridisations is not possible?
a. sp 3
Cl 8e–
b. sp 3d 3
Cl C Cl Cl Cl O C O c. sd 3
8e– 8e– 8e– 8e– 8e– 8e– 8e– d. None of the above
Cl
8e– orCl Cl
I II III 32. The hybridisation of atomic orbitals of nitrogen in
NO+2 , NO–3 and NH +4 respectively are
H O H
a. sp 2, sp 3 and sp 2
2e– 8e– 2e–
IV
b. sp, sp 2 and sp 3
a. I, II and IV b. II, III and IV c. sp 2, sp and sp 3
c. II and III d. I, II, III and IV d. sp 2, sp 3 and sp

@iitjeehelps
424 SELF STUDY GUIDE BITSAT

33. What is the type of hybridisation of carbon atoms 45. In PO34 − ion, the formal charge on the oxygen atom of
marked with star? P— O bond is
* * a. +1 b. −1 c. − 0.75 d. +0.75
CH 2 == CH  C  O  H
46. Which of the following is not the correct representation

O of resonance?
2 2 2 + −
a. sp and sp b. sp and sp I. C C O C C O C CO
2
c. sp and sp d. None of these
O O O
34. Among the three molecules XeF4 , SF4 , SiF4 , which II.
has/have tetrahedral structure? C C C
− − − −
a. All three b. SiF4 and SF4 O O O O O O
c. Only SiF4 d. Only SF4 I II III

35. What is the structure of XeF6? a. Only I b. Only II


a. Tetrahedral b. Distorted octahedral c. Both I and II d. None of these
c. Octahedral d. None of these 47. Which of the following molecules represent the
36. Which of the molecules has trigonal bipyramidal resonance?
a. O3 b. CO 2−
geometry with bond angles 120° and 90°? 3

a. SF6 b. PCl5 c. CH 4 d. BF3 c. CO2 d. All of these

37. Which of the following species has tetrahedral 48. The structure represents the molecules more
geometry? accurately, is called
a. BH −4 b. NH −2 a. resonance hybrid b. canonical structure
c. resonating structure d. None of these
c. CO2−
3 d. H3O+
49. Match the following columns and choose the correct
38. The species having pyramidal shape is option from the codes given below.
a. SO3 b. BrF3 c. SiO 2−
3 d. OSF2
Column I Column II
39. Which of the following molecules have same A. BeCl2 1. Linear
molecular geometry? B. BF3 2. Trigonal planar
I.CH4 II. BF3 III. NH +4 IV. SF4 C. CO 2
a. I and II b. III and IV
c. I and III d. I, III and IV Codes
A B C A B C
40. The structure of IF 7 is a. 1 1 2 b. 2 2 1
a. square pyramidal b. trigonal bipyramidal c. 1 2 1 d. 2 1 2
c. octahedral d. pentagonal bipyramidal 50. The compound MX 4 is tetrahedral. The number of
41. Based on VSEPR theory, the number of 90 degree ∠XMX angles formed in the compound is
F—Br—F angles in BrF5 , is a. three b. four c. five d. six
a. 0 b. 1 c. 2 d. 3
51. In accordance to molecular theory,
42. A σ-bonded molecule MX 3 is T-shaped. The number a. O+2 is diamagnetic and bond order is more than O2
of non-bonding pairs of electrons is b. O+2 is diamagnetic and bond order is less than O2
a. 0 c. O+2 is paramagnetic and bond order is less than O2
b. 2 d. O+2 is paramagnetic and bond order is more than O2
c. 1
d. Can be predicted only if atomic number of M is known 52. Which bond angle, θ would result in the maximum
dipole moment for a triatomic molecule XY2 as shown
43. An atom X has three valence electrons and atom Y
below?
has six valence electrons. The compound formed Y
between them will have the formula
a. X 2Y6 b. XY2 c. X 2Y3 d. X 3Y2
θ
44. The number of oxygen atoms bonded to one Y
phosphorus atom in P4 O6 is X
a. θ = 90° b. θ = 120°
a. 4 b. 3
c. 6 d. 5 c. θ = 150° d. θ = 180°

@iitjeehelps
CHEMICAL BONDING 425
53. The bond angles of NH3 , NH+4 and NH–2 are in the 61. The correct stability order of the following resonance
order structure is
a. NH−2 > NH3 > NH +4 b. NH4+ > NH3 > NH2− + – + –
H2C == N == N H2 C — N == N
c. NH3 > NH2− > NH4+ d. NH3 > NH4+ > NH2−
I II
– + – +
54. The molecule having zero dipole moment is H2 C — N ≡≡ N H2C — N ≡≡ N
a. ClF3 b. CH4 c. PH3 d. CH2Cl2 III IV
a. I > II > IV > III b. I > III > II > IV
55. Which of the following show correct structure of ICl2?
c. II > I > III > IV d. III > I > IV > II
Cl
Cl Cl
62. The correct order of bond angles in NH3 , PCl3 and
a. I b. I BCl3 is
a. PCl3 > NH3 > BCl3 b. NH3 > BCl3 > PCl3
Cl c. NH3 > PCl3 > BCl3 d. BCl3 > NH3 > PCl3
Cl
c. I Cl d. None of these 63. The correct order of decreasing polarity is
a. HF > SO2 > H2O > NH3
b. HF > H2O > SO2 > NH3
56. The net dipole moment of H2O molecule is c. HF > NH3 > SO2 > H2O
a. zero b. 1.85 D d. H2O > NH3 > SO2 > HF
c. 4.90 × 10−30 cm d. 0.80 × 10−30 cm
64. In case of sodium and chlorine, the electron transfer
57. Deviation of O — O bond length in ozone molecule takes place from
from the normal bond length seems as a. sodium to chlorine b. chlorine to sodium
a. single bond length increases while double bond c. Both (a) and (b) d. None of these
length decreases
65. The bond order of superoxide ion O2−
2 is
b. single bond length decreases while double bond
length increases a. 2.5 b. 1.5 c. 1.0 d. 2
c. single bond length increases while double bond 66. AlCl3 is covalent while AlF3 is ionic. This can be
length remains same
d. single bond length remains same while double bond justified on the basis of
length increases a. valence bond theory b. crystal structure
c. lattice energy d. Fajan’s rule
58. The decreasing order of dipole moments of the
molecules HF, H2O, BeF2, NF3 is 67. VSEPR theory was proposed by
a. Pauling b. Sidgwick and Powell
a. HF > H2O > BeF2 > NF3 c. Hund and Mulliken d. GN Lewis
b. H2O > HF > NF3 > BeF2
68. The bond order of O−2 is
c. BeF2 > NF3 > HF > H2O
a. 2 b. 1.5 c. 1 d. 2.5
d. NF3 > BeF2 > H2O > HF
69. In the case of Cl2, the bond is formed by
59. Which of the following will have largest dipole a. transfer of electrons b. sharing of electrons
moment?
c. Both (a) and (b) d. None of the above
NH2 NO2
70. Match the following species with chemical formula
and choose the correct option from the codes given
a. b. below.
Column I Column II
NH2 NH2
A. Compounds 1. NH4+
B. Atoms 2. H2O
c. d.
C. Ions 3. Li2O
NO2
4. Ne
NO2
5. Li +
60. Which one of the following pairs of molecules will have
Codes
permanent dipole moments for both the members?
A B C A B C
a. SiF4 and NO2 b. NO2 and CO2
a. 2,3 4 1,5 b. 3 1, 2 4, 5
c. NO2 and O3 d. SiF4 and CO2 c. 2 3,1 4,5 c. 1 2, 3 4, 5

@iitjeehelps
426 SELF STUDY GUIDE BITSAT

71. Sodium chloride is soluble in water but not in benzene 78. The molecule having one unpaired electron is
because a. NO b. CO c. CN − d. CO2
a. ∆Hhydration < ∆Hlattice (in water) 79. How many corners of a cube would be occupied in the
∆Hhydration > ∆Hlattice (in benzene) case of noble gas?
b. ∆Hhydration < ∆Hlattice (in benzene) a. 1 b. 2 c. 4 d. 8
∆Hhydration > ∆Hlattice (in water) 80. The noble gases have a particularly stable outer shell
c. ∆Hhydration = ∆Hlattice (in water) electronic configuration represented as
a. ns 2np 6 b. ns 2np 5
∆Hhydration < ∆Hlattice (in benzene)
c. ns 2np 4 d. ns 2np 8
d. ∆Hhydration < ∆Hlattice (in water)
∆Hhydration = ∆Hlattice (in benzene) 81. Which of the following has a linear structure?
a. CCl 4 b. SO2 c. C 2H 2 d. C 2H 4
72. Which of the following bonds is present in BF4− ?
a. Electrovalent b. Metallic 82. In XeF2, XeF4 and XeF6, the number of lone pairs of
c. Dative d. Hydrogen Xe respectively are
73. KF combines with HF to form KHF2. The compound a. 2, 3 and 1 b. 1, 2 and 3 c. 4, 1 and 2 d. 3, 2 and 1
contains the species 83. The largest bond angle is in
a. K + , F – and H + b. K + , F – and HF a. AsH3 b. NH3 c. H2O d. PH3
c. K + and [HF2] – d. [KHF] + and F2
84. If the bond enthalpy of O2, N 2 and H 2 are 498 , 946
74. Among the following, choose the correct pair which is and 435.8 kJ mol −1 respectively. Choose the correct
isostructural and isoelectronic?
order of decreasing bond strength.
a. NO−3, CO32− b. SO3, NO3− c. ClO−3, CO32− d. CO23− , ClO3− a. H2 > N2 > O2 b. N2 > O2 > H2
75. When two atoms share two electron pairs, they are c. O2 > H2 > N2 d. H2 > O2 > N2
said to be joined by a
a. single bond b. double bond 85. The one which has no coordinate bond, is
c. triple bond d. None of these a. HNO3 b. CO c. CO 2−
3 d. CH3 — NC

76. Which of the following molecules have same bond order? 86. For the given two compounds, vapour pressure of B at
H2 Cl2 CO Br2 N2 a particular temperature is expected to be
I II III IV V OH OH
a. I, II and IV have same bond order and
b. III and V have same bond order O2N NO2
c. Both (a) and (b) A B
d. None of the above a. higher than that of A
77. An ionic solid is poor conductor of electricity because b. lower than that of A
c. same as that of A
a. ions do not conduct electricity
d. Can be higher or lower depending upon the size of
b. charge on the ions is uniformly distributed
the vessel
c. ions have uniform field of influence around it
d. ion occupy fixed position in solids

BITSAT Archives
1. Which of the following pairs has identical shape? 4. Which of the following does not contain any
a. CH4 and SF4 b. BCl3 and ClF3 [2014] coordinate bond? [2011]
c. XeF2 and ZnCl2 d. SO2 and CO2 a. H3O+ b. BF4−
c. HF2− d. NH4+
2. Using MOT, which of the following pairs denote
paramagnetic species? [2013] 5. The number of unpaired electrons in nickel carbonyl,
a. B2 and C2 b. B2 and O2 is
c. N2 and C2 d. O2 and O2−
2
a. zero b. one c. four d. five [2011]

3. Which of the following is isoelectronic with carbon? 6. The highest bond strength is shown by [2010]
a. Na +
b. Al3+ a. O  O bond b. S  S bond
c. Se  Se bond d. Te  Te bond
c. O 2− d. N + [2012]

@iitjeehelps
CHEMICAL BONDING 427
7. Which of the following has maximum dipole moment? 12. Which of the following is a correct set? [2008]
a. NCl3 b. NBr3 c. NH3 d. NI3 [2009] a. H2O, sp 3, angular
b. BCl3, sp 3, angular
8. Which of the following has giant covalent structure?
a. NaCl b. SiO2 c. NH+4 , dsp 2, square planar
[2009]
d. CH4, dsp 2, tetrahedral
c. AlCl3 d. PbO2
13. The isoelectronic pair is [2007]
9. Which one of the following species has the largest
internuclear distance for its ion pair? [2009] a. Cl2O, ICl−2 b. Cl−2, ClO2
a. NaCl b. NaBr c. LiCl d. Kl c. IF2+ , I−3 d. ClO−3, ClF +2

10. The pair of species with similar shape is [2009] 14. Which of the following species has a bond order other
a. PCl3, NH3 b. CF4, SF4 c. PbCl2, CO2 d. PF5, IF5 than 3? [2006]
a. CO b. CN −
c. NO+ d. O+
11. The bond length of HCl molecule is 1.275 Å and its 2

dipole moment is 1.03 D. The ionic character of the 15. The molecular electronic configuration of Be2 is
molecule (in per cent) is
a. σ1s 2 σ* 1s 2σ 2s 2 σ* 2p 2 b. KKσ2s 2
(Charge of the electron = 4.8 × 10−10 esu) [2008]
[2005]

a. 100 b. 67.3 c. 33.66 d. 16.83 c. σ1s 2 σ* 1s 2σ 2s 2 σ* 2s 2 d. None of these

Answer with Solutions


Practice Exercise For O( 3) Formal charge = 6 −
1
( 2) − 6 = − 1
2
1. (d) Both Ca and Na are metals and form positive ions,
9. (d) NaOH has following structure
which repel each other. Hence, the given reaction is not
feasible. Na + [O — H] −
2. (a) Kossel’s postulations provide the basic for the modern Hence, it has both ionic as well as covalent bonds.
concepts regarding ion formation by electron transfer and 10. (d) For NO−3 ion,
the formation of ionic crystalline compounds.
Os
3. (a) 17 Cl + e − → Cl− O ← N
One [ 18 Ar]
electron
O
Chlorine is short of one electron only in electronic Bond pairs = 4 ( 3σ + 1π )
configuration in comparison with argon. Lone pair = 0
4. (d) O2−
2 has no unpaired electron, hence it is diamagnetic.
11. (c) In NaOH, Na+ and OH− ions are bonded together by
ionic bond while in OH− ion, oxygen and hydrogen atoms
2 * 2 2 * 2 * 2 *
O 2− 2 2 2 2
2 = σ1s σ s σ 2s σ 2s σ 2 p x π 2 p y π 2 p z π p y π 2 p z are bonded together by covalent bond Na+ [O — H] − .

5. (a) H-bond is only an interaction between electronegative 12. (a) I may be correctly represented as
element and hydrogen therefore, it is the weakest bond
among all. C O C O or C O
(Correct)
6. (b) Bond angle of H2S is smallest because S-atom is large
1 3
in size and has low electronegativity. •• 2 ••
+ 13. (b) N ==N==N
7. (a) 3 N—H bonds are covalent. H •• ••

1 N—H bond is coordinate and bond − For N(1) and N (3),


N Cl
between NH4+ and Cl− is ionic. H 4 
H Formal charge = 5 − + 4 = 5 − 6 = −1
(2) H  2 
O 1
For N(2) , Formal charge = 5 − × 8 − 0 = + 1
8. (c) H O N (3)
2
(I) O 14. (a) According to Born-Lande equation,
1 Z + Z −e 2 NAM 1 
For O(1) Formal charge = 6 − ( 4) − 4 = 0 U =  − 1
2 rnode n 
1 ∴Ions should be of small size to have high lattice energy.
For O( 2) Formal charge = 6 − ( 4) − 4 = 0
2

@iitjeehelps
428 SELF STUDY GUIDE BITSAT

15. (d) The outer shell electron takes part in chemical 27. (a) Low ionisation potential indicates that element can
reaction to complete the octet. Therefore, four electrons easily lose electron to form cation.
involved in chemical bond formation will be 3 d 2, 4s 2. Mg → Mg2+ + 2e − 
28. (b)  Ionisation enthalpy involved
16. (a) The electronegativity order of halogens is M (g ) → M + (g ) + e − 
F > Cl > Br > I
Fluorine is the most electronegative element of the O → O 2− − 2e − 
 Electron gain enthalpy involved
periodic table. X (g ) → X − − e − 
17. (c) Eight electrons (octet) in outer shell show stable 29. (a) Outer shell is known as valence shell and its electrons
electronic arrangement. are called valence electrons.
1
18. (c)Q Covalent character ∝ 30. (b)
Ionic character O C O O C O
1 or

difference in electronegativity O C O
∴ C — S is the most covalent. 31. (d )sp 3 = tetrahedral geometry
2+ 2
19. (c) For BeCl2, H = =2 sp 3d 3 = pentagonal bipyramidal geometry
2
⇒ Hybridisation = sp sd 3 = it is possible, e.g. → MnO−4 (tetrahedral)
⇒ Linear shape 1
32. (b) Hybridisation in NO+2 = (5 + 0 − 1) = sp
20. (b) EN difference in SiCl4 = 3.0 − 1.8 = 1.2 2
1
in AlCl3 = 3.0 − 1.5 = 1.5 Hybridisation in NO−3 = (5 + 0 + 1) = 3 = sp 2
2
in CaCl2 = 3.0 − 1.0 = 2.0 1
Hybridisation in NH4+ = (5 + 4 − 1) = 4 = sp 3
in KCl = 3.0 − 0.8 = 2.2 2
∴ KCl < CaCl2 < AlCl3 < SiCl4 33. (b)
This order is also obtained by applying Fajan’s rule.
34. (c) For SiF4
i.e. covalent character ∝ charge on ion. F
21. (c) Metallic lustre is due to oscillation of loose electrons.
Si
22. (b) Higher the electronegativity difference, higher is its
ionic character. F F F
Hence, the correct order is σ -bond = 4, lp = 0
D<C<B<A ∴ Structure is tetrahedral.
23. (b) The Born-Haber cycle takes place as follows: 35. (b) For XeF6
 ∆H 1 ∆H 2 +

M1(s ) → M (g ) → M (g )  F F
 
 1 ∆ H 3 ∆ H 4 −  Xe
 X 2(g ) → X (g ) → X (g )  F
 2  F F
 ∆f H + − ∆H 5  F
 → M X (s ) ←  1
  H = [v + y − c + A]
2
24. (d ) I. Represents Lewis-dot structure of CCl4.
1
II. Represents Lewis-dot structure of Cl2. = [ 8 + 6 − 0 + 0] = 7
2
III. Represents Lewis-dot structure of CO2. ⇒ Hybridisation = sp 3d 3
IV. Represents Lewis-dot structure of H2O. Due to the presence of one lone pair of electron, XeF 6 has
25. (c) In C2H4 molecule, there are two types of bonds arise distorted octahedral geometry.
from the central metal atom. Cl
C—H (107 pm bond length) Cl
C == C (133 pm bond length) 90°
36. (b) Cl P 120°
While in rest of the cases, all bonds from the central atom
are similar.
Cl Cl
26. (a) If the electronegativity difference between two atoms
A and B is 2.0, then the percentage of covalent character 37. (a) BH −4 has tetrahedral geometry.
in the molecule is 54%. 38. (d)

@iitjeehelps
CHEMICAL BONDING 429
+ +
39. (c) CH4 and NH+4 both have tetrahedral shape while BF3
47. (d) (i) O O
has trigonal planar and SF4 has see-saw shape. O O
−O O−
40. (d) According to VBT, structure of IF7 can be determined
I II
as − −
O O O
5s2 5p5 5d 0
I (53) (ii) C C C
− −
O O −
O O O O−
(ground state)
I II III
5s1 5p3 5d 3
⊕ s
(in excited state) (iii) O C O O C O
I II

sp3d3-hybridisation in excited state


s ⊕
F O C O
F
Thus, all three molecules represent resonance.
F
90° 48. (a)
F I 72°
49. (c)
F
50. (b) Since, MX 4 is tetrahedral, therefore, total number of
F ∠ XMX are four.
F
Pentagonal bipyramidal structure
51. (d) O+2 contains 1 unpaired electron and therefore it has
bond order of 2.5 while O2 contains only 2 unpaired
41. (a) In BrF5, number of electron pairs electrons. So, it possesses bond order of 2.
(1lp + 5 bp ) 52. (a) µ = µ12 + µ 22 + 2µ1 µ 2 cos θ

F F If θ = 90°, µ is maximum.
Because cos 90° = 0, since, the angle increases from
Br 90 to 180, the value of cosθ becomes more and more
negative and hence resultant decreases. Thus, dipole
F F
moment is maximum, when θ = 90°.
F 53. (b) As the number of lone pairs of electrons increases,
So, the structure is supposed to be square pyramidal but bond angle decreases. Therefore, the order of bond angle
will be distorted because of additional lp-bp interaction. is
Additional lp-bp interaction reduced the all bond angle and
NH +4 > NH3 > NH −2
do not let any angle to be 90°.
42. (b) no lp 1lp 2lp
43. (c) If an atom X has three valence electrons and atom Y 54. (b)
has six valence electrons, the compound formed between 55. (b) To minimise lp − lp repulsion, they exist as shown in
them will have the formula X 2Y3 like Al2O3. option (b).
44. (b) One phosphorus is bonded to three O-atoms as 56. (b) The dipole moment of a molecule is the vector sum of
represented by 1,2,3 for P-atom present in the structure the dipole moments of various bonds.
given below
e.g. In H2O molecule which has a bent structure, two
O P O1 O — H bonds are oriented at an angle of 104.5° with
2
O dipole moment 1.85D.
P P
O 57. (b)
P 58. (b) H2O > HF > NF3 > BeF2
O O3 1.85 D 1.78 D 0.23 D 0D
Structure of P4O6 NH2
45. (b)
46. (d) Both representation of resonating structures in 59. (d)
molecules of CO2 and CO2− 3 are correct. NO2

@iitjeehelps
430 SELF STUDY GUIDE BITSAT

Above compound has the highest dipole moment among 4+ 2


H= =3
the given structures because it is totally unsymmetrical. 2
Usually symmetrical molecules have less dipole moment ⇒ Hybridisation = sp 2
in comparison to unsymmetrical molecules.
∴Both NO 2− 2−
3 and CO 3 are isoelectronic and isostructural
60. (c) NO2 and O3 both having irregular geometry due to
species.
which they have permanent dipole moments.
75. (b) When two atoms are linked to each other by sharing of
61. (b) I has maximum covalent bond and negative charge electrons pair, they are said to be joined to each other by
on electronegative nitrogen, most stable. III has more a double bond.
covalent bond than both II and IV, III is second most
stable. II is more stable than IV, since it has negative
charge on nitrogen. So, the order is A A A A or A A
I > III > II > IV
62. (d) NH3 (107°) 76. (c) In CO (three shared electron pairs between C and O),
the bond order is 3. For N2, bond order is 3.
PCl3 (93°)
BCl3 (120°) Cl2, Br2 and H2 have same bond order, i.e. 1.
63. (b)Q Polarity ∝ difference in electronegativity 77. (d) An ionic solid is a good conductor of electricity only in
molten or solution state, when ions are free to move.
∴ Correct order of polarity is
78. (a) NO has 15 electrons in its molecular orbitals which are
HF > H2O > SO2 > NH3 distributed as follows
•• ••
64. (a) Na × + •Cl •• → Na+ ו Cl •• ≡≡ Na+Cl− NO = σ1s 2 σ* 1s 2σ 2s 2 σ* 2s 2π 2p y2 π 2p z2 σ 2p x2 π* 2p1y
•• ••

Nb − Na 6 − 4 As it contains one unpaired electron, hence it is


65. (c) BO = = = 1.0 paramagnetic.
2 2
66. (d) According to Fajan's rule, smaller the anion, lesser will 79. (d ) Eight corners of a cube would be occupied in the case
be its polarisibility. Hence in AlF3, F − ions are polarised to of noble gas.
very little extent, therefore it has more ionic character. 80. (a) The noble gas have a particular stable outer shell

In AlCl3, Cl are polarised to greater extent (due to larger electronic configuration represented as ns 2np 6.
size), hence AlCl3 has more covalent character. 81. (c) C2H2 has linear structure.
67. (b) VSEPR theory was first of all proposed by Sidgwick 82. (d) Xe-atom has 8 electrons in its outermost shell. In case
and Powell in 1940. Later on, it was developed by of XeF2 out of these 8 electrons, 2 are used for bond
Gillespie and Nyholm in 1957. formation while 3 pairs remain as such, that’s why it has 3
1 lone pairs.
68. (b) BO = ( 6 − 3) = 1.5
2 In case of XeF4, 4 electrons of Xe are used for bonding,
69. (b) therefore number of lone pairs (non-bonding electrons) is 2.
70. (a) In case of XeF6, 6 electrons are involved for bond
formation, thus number of lone pair is only 1.
71. (b) For a compound to be soluble, the hydration energy
must be greater than the lattice energy. Since, NaCl is 83. (b) The largest bond angle is present in NH3 (107°). All
soluble in water but insoluble in benzene. the given molecules aresp 3-hybridised but due to the
∆H hydration < ∆H lattice (in benzene) presence of 2 lone pairs in H2O , its bond angle is less
(104.5°). While in PH3 and AsH3, due to less difference in
and ∆H hydration > ∆H lattice (in water)
electronegativities, the bonded e − pair remain between
72. (c) BF −4 can be considered as being made by BF3 by P and H (in PH3) and As and H (in AsH3) so that large
accepting lone pair of F − , lp − bp repulsion makes bond angle smaller.
i.e. F3B ← F − (thus, it has dative bond). 84. (b) Larger the bond dissociation enthalpy, stronger will be
the bond in the molecule. So,
73. (c) F− forms H-bond with HF, therefore the species
N2 > O2 > H2
[H ..... F  H] − or HF − exists and hence, option (c) is
946 kJ mol–1 498 kJ mol–1 435.8 kJ mol–1
correct. −
O
74. (a) For NO−3 ion, number of electrons = 7 + 3 × 8 + 1 = 32 85. (c) Structure of CO2−
3 ion is C O.

O
V + M −C + A 5 + 1
H= = =3 Hence, it has no coordinate bond. In it, carbon is
2 2
sp 2-hybridised and hence the ion has trigonal planar
⇒ Hybridisation = sp 2 geometry.
For CO23 − ion, number of electrons = 6 + 3 × 8 + 2 = 32 86. (a)

@iitjeehelps
CHEMICAL BONDING 431

BITSAT Archives
1. (c) XeF2 and ZnCl2 both have linear shape. 8. (b)
2. (b) Among given four pairs, B2 and O2 are paramagnetic due 9. (d) Amongs the given compounds, K + is the largest
to presence of unpaired electron. cation and I− is the largest anion, therefore KI has the
largest internuclear distance.
MO electronic configuration of B2
10. (a) Both PCl3 and NH3 have pyramidal shape, since
= σ1s 2 σ* 1s 2 σ 2s 2 σ* 2s 2 π 2p12 ≡ π 2p1y both havesp 3-hybridisation and one lone pair of
MO electronic configuration of O2 electrons. In contrast, all other pairs have difference
hybridisations and hence, different shapes, i.e. CF4
= σ1s 2 σ* 1s 2 σ 2s 2 σ* 2s 2 σ 2p 22
hassp 3 while SF4 hassp 3d -hybridisation, PbCl2 is ionic
π 2p x2 ≡ π 2p y2 π* 2p1x ≡ π 2p1y while CO2 hassp-hybridisation, PF5 hassp 3d while IF5
3. (d )∴N + ( 7 − 1 = 6 e − ) is isoelectronic with C as both have 6 hassp 3d 2-hybridisation.
electrons. 11. (d) Theoretical value of dipole moment = e × d
4. (c) Among the given, only HF −2 has H-bonding [F  H K F] − . = 4.8 × 10−10 × 1275
. × 10−8 esu cm
Rest all the molecules have coordinate bonds. = 6.12 × 10−18 esu cm
0
5. (a) In nickel carbonyl Ni(CO)4, Ni is present as Ni. = 6.12D
Ni = [Ar] 3d 4s8 2 %Ionic character
Observed dipole moment
CO being strong field ligand, shifts electrons from 4s to 3d = × 100
Theoretical value of dipole moment
orbital.
4s0 4p 103
.
3d10 = × 100 = 16.83%
Ni(CO)4 = [Ar] × × × × 6.12
CO COCOCO
14243 12. (a)
sp 3 13. (d) ClO−3 and ClF +2 contain 34 electrons each hence
Therefore, number of unpaired electrons in nickel carbonyl is they are isoelectronic.
0. 14. (d) The configuration of the given species can be
6. (b) As the size increases, bond strength decreases. But written as
S—S bond is stronger than O—O bond because of smaller CO or CN− or NO+ (14 e − )
size of O-atoms.
= σ1s 2, σ* 1s 2, σ 2s 2, σ* 2s 2, σ 2p z2 , π 2p x2 = π 2p y2
7. (c) Electronegativity difference between N(3.0) and Cl (3.0)
is zero and hence, N—Cl bonds are non-polar. As a result, O+2 ( 8 + 8 − 1) = σ1s 2, σ* 1s 2, σ 2s 2, σ* 2s 2, σ 2p x2
the overall dipole moment of NCl3 molecule and its direction
is just the dipole moment of the lone pair of electrons. { π 2p 2y = π 2p Z2 π* 2p1y }
1
Bond order = (Nb − Na )
N N 2
H H 1
Cl Cl CO or CN − or NO+ = (10 − 4) = 3
Cl 2
H
1 5
On the other hand, N—Br, ( 3.0 − 2.8), N—I ( 3.0 − 2.5) and O+2 = (10 − 5) = = 2.5
2 2
N—H ( 3.0 − 2.1) are polar and hence, contribute towards the
overall dipole moment of the respective molecules. Since, the 15. (c) Molecular electronic configuration of Be2 is
electronegativity difference is highest in case of N—H bonds,
σ1s 2, σ* 1s 2, σ 2s 2, σ* 2s 2
therefore NH3 has the highest dipole moment.

@iitjeehelps
5
Periodic Properties

Classification of Elements
The elements are considered as the basic units of all types of matter. As the more elements were
discovered in the recent times, it becomes a very difficult task to study the chemistry of elements and
their compounds. In order to remove this difficulty, we need to organise the elements in a systematic
pattern.
The efforts to classify the elements are as follows :

Laws for Classifying the Elements


Many attempts were made to classify the known elements from time to time.
These are as follows:

Dobereiner’s Triads (1800, Johann Dobereiner)


Dobereiner arranged the elements in several groups of three elements each called triads, with similar
properties. He found that in triads, the atomic weight of middle element is about half of the sum of the
first and third element. This relation is known as law of triads.
Li Na K Ca Sr Ba Cl Br I
e.g.
7 23 39 40 88 137 35.5 80 127

Law of Octaves (1865, John Alexander Newlands)


Newland arranged the elements on the basis of the increasing order of their atomic weights and
noted that the properties of every eighth element resembles the properties of the first element just
like the octaves of music. This is known as Newlands law of octaves.

Lothar Meyer’s Curve


Lothar Meyer plotted the physical properties such as atomic volume, melting and boiling points etc.,
with atomic weight and observed a periodic repetition in their properties. In 1868, Lothar Meyer had
developed a table of elements which show clear resemblance with the modern periodic table but his
work was not published.

@iitjeehelps
PERIODIC PROPERTIES 433
Mendeleef’s Periodic Law root of frequency of X-ray ( n) emitted against atomic
According to him, ‘‘the physical and chemical properties of number (Z ), gave a straight line and not the plot of n vs
elements are the periodic function of their atomic weight’’. atomic masses. Hence, he concluded that atomic
In order to arrange the elements in the increasing order of their number is more fundamental property of an element
atomic weight, he constituted a table containing 12 horizontal than its atomic mass. The modern periodic law can be
rows (series) and 8 vertical columns. He arranged the elements stated as “the physical and chemical properties of
in such a way that (zero group was added later on) the elements changes periodically with their atomic
properties of elements belong to same group were similar. numbers.”
1. 9 groups are notified as roman numberic I, II, III, IV, V, VI, Q Atomic number = Number of protons
VII, VIII and 0. = Number of electrons
2. Each group has been divided into 2 subgroups A and B. So, we can say that properties of any element varies
3. Series 1, 2 and 3 were short period. periodically with their electronic configuration.

4. Series 4 and 5 were long period containing transition Present Form of Periodic Table
elements. After Moseley’s finding, a modern version of periodic
table is also known as long form of the periodic table
Modern Periodic Law and Present Form of was launched. This table is just representation of
Periodic Table Aufbau principle . In this table, elements are arranged
Modern Periodic Law in increasing order of their atomic number. The table
consists of 7 horizontal rows called periods and 18
In 1913, Henry Moseley observed regularities in the
vertical columns called groups or families.
characteristic X-ray spectra of the elements. A plot of square

Long Form of Periodic Table


Main-group elements Main-group elements

1 1 Atomic number
H
18
IA Symbol
0
1.00794 Atomic weight
1 2
1 H 2 13 14 15 16 17 He
1.00794 4.002602
II A IIIA IVA VA VIA VIIA
3 4 Transition elements 5 6 7 8 9 10
2 Li Be B C N O F Ne
6.941 9.012182 10.811 12.011 14.00674 15.9994 18.9984032 20.1797

P
3 11 12 13 14 15 16 17 18
E Na Mg 3 4 5 6 7 8 9 10 11 12 Al Si P S Cl Ar
22.989768 26.981539 28.0855 30.973762 32.066 35.4527 39.948
R 24.3050 IIIB IVB VB VIB VIIB VIII IB IIB

I 25 26 27
4 19 20 21 22 23 24 28 29 30 31 32 33 34 35 36
O K Ca Sc Ti V Cr Mn
54.93805
Fe
55.847
Co
58.93320
Ni
58.6934
Cu Zn Ga Ge As Se Br Kr
83.80
39.0983 40.078 44.955910 47.88 50.9415 51.9961 63.546 65.39 69.723 72.61 74.92159 78.96 79.904
D
S 37 38 39 40 41 42 43 44 45 46 47 48 49 50 51 52 53 54
5 Rb Sr Y Zr Nb Mo Tc Ru Rh Pd Ag Cd In Sn Sb Te I Xe
85.4678 87.62 88.90585 91.224 92.90638 95.94 (98) 101.07 102.9055 106.42 107.8682 112.411 114.82 118.710 121.757 127.60 126.90447 132.29

55 56 57 72 73 74 75 76 77 78 79 80 81 82 83 84 85 86
Cs Ba La * Hf Ta W Re Os Ir Pt Au Hg Tl Pb Bi Po At Rn
6 132.90543 137.327 138.9055 178.49 180.9479 183.85 186.207 190.2 192.22 195.08 196.96654 200.59 204.3833 207.2 207.98037 (209) (210)
(222)

87 88 89 104 105 106 107 108 109 110 114 117


7 Fr Ra Ac ** Rf Db Sg Bh Hs Mt Ds Uuq Uus
(223) (226) (227) (261.11) (262.114) (263.118) (262.12) (265) (266) (269)
Inner-transition elements

58 59 60 61 62 63 64 65 66 67 68 69 70 71
* L anthanoids Ce Pr Nd Pm Sm Eu Gd Tb Dy Ho Er Tm Yb Lu
140.115 140.90765 144.24 (145) 150.36 151.965 157.25 158.92534 162.50 164.93032 167.26 168.93421 173.04 174.967

90 91 92 93 94 95 96 97 98 99 100 101 102 103


** A ctinoids Th Pa U Np Pu Am Cm Bk Cf Es Fm Md No Lr
232.0381 (231) 238.0289 (237) (244) (243) (247) (247) (251) (252) (257) (258) (259) (262)

@iitjeehelps
434 SELF STUDY GUIDE BITSAT

Notation for IUPAC Nomenclature of d


(iii) Metallic radius, r ¢ =
Elements 2
S.No. Digit Name Abbreviation r ¢ > actual size of atom
1. 0 nil n d
(iv) van der Waals’ radius, r1 = ,r1 > >> actual size of atom
2. 1 un u 2
3. 2 bi b
Covalent radius < Metallic radius < van der Waals’ radius
4. 3 tri t
5. 4 quad q In the end of the period, atomic radii of inert gases are
6. 5 pent p exceptionally higher because they do not form molecule
7. 6 hex h and their radii are simply van der Waals’ radii.
8. 7 sept s
9. 8 oct o
10. 9 enn e
2. Ionisation Enthalpy
It is the energy required when an electron is removed from
To write the name of any element with atomic number
an isolated gaseous atom. Generally left to right in periods,
more than 100, write the name of each number from the
IE increases; down the group it decreases but half-filled
table followed by “ium”.
orbitals and fully filled orbitals are stable form and have
Atomic no. Name
high IE.
101 un-nil-unium
1 0 1 Various factors affecting ionisation energy varies are:
109 un-nil-ennium
(i) Atomic size : varies inversely
1 0 9 (ii) Screening effect : varies inversely
(iii) Nuclear charge : varies directly
(iv) Helium has the highest IE 1 while Cs has the lowest.
Periodic Properties (v) IE values of inert gases are exceptionally higher due to
stable configuration.
The elements in the periodic table are arranged in order of
increasing atomic number. All of these elements display Screening Effect
several other trends and we can use the periodic law and
Shielding (or screening) effect is the repulsion of valence
table formation to predict their chemical, physical and
electrons by the electrons in penultimate shell to reduce
atomic properties.
effective nuclear charge.
Z eff = Z - s
1. Atomic Size or Atomic Radius
where, Z = atomic number, s = shielding constant
Left to right size of atom decreases; down the group size
increases. s = [0.35 ´ number of electrons in nth shell excluding last
(valence) electron] + [0.85 ´ number of electrons in
(i) Ionic radius Distance of the outermost shell of an ion
(n - 1)th shell] + [ 1.0 ´ number of electrons in inner shell]
from its nucleus.
(ii) Covalent radius
(a) In homodiatomic molecule, ( A ¾ A)
3. Electron Gain Enthalpy
dA ¾ A It is the energy released when an electron is added in an
r = , d A ¾ A = bond length in molecule isolated gaseous atom. Chlorine has the highest electron
2 affinity but oxidising power of fluorine is larger than
(b) In heterodiatomic molecule, ( A ¾ B ) chlorine.
● When ( X A - X B ) is very small Various factors affecting electron gain enthalpy are:
dA ¾B = rA + rB (i) Atomic size : varies inversely
dA ¾B = bond length (ii) Nuclear charge : varies directly
rA and rB = covalent radii of A and B respectively (iii) Configuration : Half-filled orbitals and fully filled
X A and X B = electronegativities of A and B orbitals are stable form, therefore electron gain
● When ( X A - X B ) is considerable enthalpy will be low or even sometimes energy is
dA = rA + rB - 0.09 ( X A - X B ) required rather than getting released.
¾B

@iitjeehelps
PERIODIC PROPERTIES 435
Exception Cl > F (electron gain enthalpy) Variation of Valency in a Group
S>O All elements in a group have the same valency as they have
F and O-atoms have small size and high charge density, same number of electrons in their outer shell. p-block
therefore have lower electron gain enthalpy. elements show variable valency on account of inert pair
effect.
4. Electronegativity Term oxidation state is also being used for covalency,
It is the tendency to attract the shared pair of electrons. F is which indicates the actual charge on atom in that
the most electronegative element while Cs is the least. particular molecule. It follows the same trend along period
or group as valency.
Difference in the electronegativities of two atoms( A and B )
X A - X B = 0.208 D NOTE Tin has maximum number of isotopes (10 in number) and Os
has the highest oxidation state or valency (+8).
where, D = actual bond energy
- energy for 100% covalent bond Variation of Valency in Transition Elements
\ D = EA - EA ´ EB Transition elements and inner-transition elements show
¾B ¾A ¾B
variable valency of 1, 2 or 3 as they use electrons from outer
Here, E A ¾B = dissociation enthalpy of A ¾ B (kcalmol -1 ) as well as penultimate or inner-penultimate shell.
EA ¾A = dissociation enthalpy of A ¾ A (kcal mol -1 )
6. Chemical Reactivity
EB ¾B = dissociation enthalpy of B ¾ B (kcal mol -1 )
Reactivity of metals increases with decrease in ionisation
Mulliken’s approach energy, electronegativity and increase in atomic radii and
IE + EA electropositive character.
XM =
2 Reactivity of non-metals increases with increase in
Mulliken values are » 2.8 times greater than Pauling values. electronegativity and electron gain enthalpy and decreases
IE + EA with increase in atomic radii.
XP = or X P = 0.336 [ X M - 0.615]
5.6 Variation in a Group
Unit of electronegativity = eV On moving down the group, reactivity of metals increases
For noble gases, its value is taken as zero. while for non-metals, it decreases.
Decreasing order of electronegativity
Variation in a Period
F > O > Cl »N > Br > S » C > I > H
Reactivity of metals decreases while reactivity of
In periods left to right, electronegativity increases. non-metals increases across the period.
In groups down the group, electronegativity decreases.
Some other Periodic Trends
5. Valency ● Bond energy µ electronegativity difference
The valency of an element is related to the electronic 1
configuration of its atom and usually determined by ● Bond length µ
electronegativity difference
electrons present in valence shell (outer shell).
● Acidic character of hydride µ electronegativity of central
Variation of Valency in a Period atom
Valency of the elements with respect to hydrogen increases ● Acidic character of hydra acids increases on moving left
from 1 to 4 upto 14th group and then decreases to 1. to right and top to bottom.
e.g. NaH, CaH 2 , AlH 3 , SiH 4 , PH 3 , H 2 S, HCl. ● Periodicity of oxyacids Acidic character increases left to
But valency of the elements with respect to oxygen right while decreases top to bottom.
increases from one to seven along a period. ● Nature of oxides Acidic character increases left to right
e.g. Na 2O, CaO, Al 2O3 , SiO2 , P4O10 , SO3 , Cl 2O7 while decreases top to bottom.

@iitjeehelps
Practice Exercise
1. Which of the following has firstly organise the 9. The period number in the long form of the periodic
elements according to same trend? table is equal to
a. Newland b. Mendeleef a. magnetic quantum number of any element of the
c. Lothar Meyer d. Dobereiner period
2. Johann Dobereiner gave the idea of trends among the b. atomic number of any element of the period
physical and A…… of several groups of three c. maximum principal quantum number of any element
elements. Here, A refers to of the period
a. atomic number b. atomic mass d. maximum azimuthal quantum number of any element
c. chemical properties d. electronic configuration of the period
3. Lothar Meyer plotted the physical properties such as 10. Which of the following is incorrect?
atomic volume, melting point and A……against atomic a. Henry Moseley observed regularities in the
weight. Here, A refers to characteristics X-ray spectra of elements
a. mass b. boiling point b. A plot of frequency of X-ray emitted against atomic
c. surface tension d. molecules number (Z ) give a straight line
4. Lothar Meyer drew a graph showing the relation c. The atomic number of an element is equal to the
between number of neutron(s) in a neutral atom
a. atomic number and atomic weight d. Electronic configuration of an atom determines the
b. atomic number and atomic size physical and chemical properties
c. atomic weight and atomic volume 11. Match the Column I with Column II and select the
d. atomic weight and atomic size correct answer by given codes.
5. Which of the following is incorrect explanation about Column I Column II
Mendeleef’s periodic law? (Number of periods) (Number of elements)
a. Mendeleef arranged elements in horizontal rows only
b. Mendeleef arranged elements with increasing atomic A. First period 1. 14
weight B. Third period 2. 02
c. Mendeleef’s system of classifying elements was more C. Lanthanoids 3. 14
elaborate than that of Lothar
D. Actinoids 4. 08
d. Both (a) and (b)
5. 04
6. Which basic concept was used by Mendeleef for
organising the elements?
Codes
a. They organise the metals only A B C D A B C D
b. They organise the non-metals only a. 2 4 1 5 b. 2 4 1,3 3,1
c. They organise the both metals and non-metals c. 4 2 1 3 d. None of these
according to increasing number of masses
12. Observe the following statements,
d. None of the above
I. The physical and chemical properties of
7. Which element was named as eka-silicon in elements are the periodic functions of their
Mendeleef classification of elements? electronic configuration.
a. Germanium b. Gallium II. Electronegativity of fluorine is less than the
c. Thallium d. Selenium electronegativity of chlorine.
8. The statement that is not correct for the periodic III. Electropositive nature decreases from top to
classification of elements, is bottom in a group.
a. the properties of the elements are the periodic The correct answer is
function of their atomic number a. I, II and III b. Only I
b. non-metallic elements are lesser in number than c. I and II d. II and III
metallic elements 13. In an element 19
, there are 10 neutrons in the
ZM
c. the first ionisation energies of elements along a period
nucleus. It belongs to
do not vary in a regular manner with increase in
a. s -block
atomic number
b. d -block
d. for transition elements, the d-subshells are filled with
c. f -block
electrons monotonically with increase in atomic
d. None of the above
numbers

@iitjeehelps
PERIODIC PROPERTIES 437
14. The third alkaline earth metal ion contains number of 28. Chloride ion and potassium ion are isoelectronic, then
electrons and protons as a. their sizes are same
a. 20e –, 20p b. 18e –, 18p b. Cl- ion is bigger than K + ion
c. 18e –, 20p d. 19e –, 20p c. K + ion is relatively bigger
d. their sizes depend on other cation and anion
15. A metal having electronic configuration
29. Which one of the following is correct increasing order
1s 2, 2s 2 2p 6, 3s 2 3 p 6 3d 10, 4s 2 is in of size?
a. s -block b. d -block a. Mg < Na + < F - < Al b. Na + < Al < Mg < F -
c. p-block d. None of these c. Na + < F - < Al < Mg d. Na + < F - < Mg < Al
16. Which of the following orbitals are in the process of 30. Ionic radii vary in
filling in the 6th period? a. inverse proportion to the effective nuclear charge
a. 6s, 6p, 6d, 6f b. 6s, 5f, 6d, 6p b. inverse proportion to the square of effective nuclear
c. 6s, 4f, 5d, 6p d. 5s, 5p, 5d charge
c. inverse proportion to the screening effect
17. The configuration to second excited state of the d. direct proportion to the square of the screening effect
element is (O) electronic with O2 or P – or Cl+ is
31. The first ionisation energy of beryllium is more than
a. [ Ne] 3s 2 3p x2 3p1y 3p1z
that of boron because
b. [Ne]3s 2 a. boron has higher nuclear charge
c. [ Ne] 3s 1 3p1z 3p1y 3p1z 3d 1xy 3d 1yz b. boron has only one electron in p-subshell
d. [ Ne] 3s 2 3p1x 3p1z 3p1xy c. atomic size of boron is less than that of beryllium
d. atomic size of boron is more than that of beryllium
18. The element having atomic number 33 lies in the 32. Among the following, the element with highest
group ionisation potential is
a. 16 b. 14 c. 15 d. 13 a. boron b. carbon
19. General configuration of ultimate and penultimate shell c. oxygen d. nitrogen
is -(n - 1)s 2 (n - 1)p 6 (n - 1)d x ns 2 . If n = 4 and x = 5, 33. The isoelectronic ion having lowest ionisation energy is
the number of protons in the nucleus would be a. S2– b. Ca 2+ c. K + d. CI –
a. < 24 b. 25 c. 24 d. >25
34. The values in electron-volt per atom which represent
20. Generally, the valency of noble gases is the first ionisation energy of oxygen and nitrogen atom
a. two b. three c. one d. zero respectively are
a. 13.6 and 14.6 b. 14.6 and 13.6
21. Identify the least stable ion among the following.
- - - - c. 14.6 and 14.6 d. 13.6 and 13.6
a. Li b. Be c. B d. C
35. The pair in which the ionisation energy of first species
22. The electronic configuration of an element is
is less than that of second is
1s 2, 2s 2 2p 6, 3s 2 3 p 3 . The atomic number and the
a. N, P b. Be, Be+ c. S, P d. N, N–
group number of the element X which is just below
the above element in the periodic table respectively, 36. How many Cs-atoms can be converted to Cs+ ion by
are 1 J energy if IE1 for Cs is 376 kJ mol-1?
a. 23 and 5 b. 23 and 15 a. 1.60 ´ 1023 b. 1.60 ´ 1015
c. 33 and 15 d. 33 and 5 c. 1.60 ´ 1018 d. 16.0 ´ 1026
23. What will be the IUPAC name of element having 37. How many joules of energy must be absorbed to
Z = 106? convert Li to Li+ , all the atoms present in 1.00 mg of
a. Unnilquadium b. Unnilhexium gaseous Li? IE1 of Li is 520.3 kJ mol -1 (Li = 7).
c. Unnilheptium d. Ununhexium
a. 0.00743 kJ b. 0.520 kJ
24. The one with the largest ionic size is c. 520 kJ d. 0.0743 kJ
a. O 2– b. Mg2+ c. F – d. Na +
38. The incorrect statements among the following is
25. Fluorine and neon have atomic radii in angstrom given by a. the first ionisation potential of Al is less than the first
a. 1.60, 1.60 b. 0.72, 0.72 ionisation potential of Mg
c. 0.72, 1.60 d. None of these b. the second ionisation potential of Mg is greater than
the second ionisation potential of Na
26. Which of the following alkali metals has smallest size?
c. the first ionisation potential of Na is less than the first
a. Cs b. Rb c. Na d. K ionisation potential of Mg
27. Which of the following ionic species has largest size? d. the third ionisation potential of Mg is greater than the
a. Rb+ (aq ) b. Li+ (g ) c. Na + (aq ) d. Li+ (aq ) third ionisation potential of Al

@iitjeehelps
438 SELF STUDY GUIDE BITSAT

39. Ionisation enthalpy of Na would be same as 45. The electron affinity values (in kg mol-1) of three
+
a. value of electron affinity of Na halogens X, Y and Z are –349, –333 and – 325
b. value of electronegativity of Na respectively. Then, X, Y and Z respectively are
c. value of ionisation potential of Mg a. F2, Cl2 and Br2 b. Cl2, F2 and Br2
d. value of electron affinity of Na
c. Cl2, Br2 and F2 d. Br2, Cl2 and F2
40. Which one of the following statements is incorrect in
relation to ionisation enthalpy? 46. Which one of the following statements is false?
a. Ionisation enthalpy increases for each successive a. The electron affinity of chlorine is less than that of
electrons fluorine
b. The greatest increase in ionisation enthalpy is b. The electronegativity of fluorine is more than that of
experienced on removal of electron from core noble chlorine
gas configuration c. The electron affinity of bromine is less than that of
c. End of valence electrons is marked by a big jump in chlorine
ionisation enthalpy d. The electronegativity of chlorine is more than that of
d. Removal of electron from orbitals bearing lower n bromine
value is easier than from orbitals having higher n
47. Which of the following element has highest
value
electronegativity?
41. The formation of the oxide ion, O2- (g) from oxygen a. Oxygen b. Chlorine
atom requires first an exothermic and then an c. Fluorine d. Nitrogen
endothermic step as shown below
48. In C, N, O and F, the electronegativity
O( g ) + e - ¾®. O- ( g ); D H s = - 141kJ mol-1 a. increases from carbon to fluorine
- - 2- s
O ( g ) + e ¾® O ( g ); D H = + 780 kJ mol -1 b. decreases from carbon to oxygen and then increases
c. decreases from carbon to fluorine
Thus, process of formation of O2- in gas phase is d. increases from carbon to oxygen and then decreases
unfavourable even though O2- is isoelectronic with 49. Electronegativity of F on Pauling scale is 4.0. What
neon. It is due to the fact that, will be its value on Mulliken scale?
a. oxygen is more electronegative a. 10.0 b. 11.2 c. 8.54 d. 16
b. addition of electron in oxygen results in larger size of
the ion 50. Two elements whose electronegativities are 1.2 and
c. electron repulsion outweights the stability gained by 3.0 respectively, the bond formed between them
achieving noble gas configuration would be
d. O- ion has comparatively smaller size than oxygen a. ionic b. covalent c. coordinate d. metallic
atom
51. What will be the electronegativity of carbon at Pauling
42. Which one of the following arrangements represent scale? Given that, E H — H = 104.2 kcal mol-1,
the correct order of electron gain enthalpy
(with negative sign) of the given atomic species? E C — C = 83.1 kcal mol -1, E C — H = 98. 8 kcal mol-1
a. Cl < F < S < O
b. O < S < F < Cl [Electronegativity of hydrogen = 2.1]
c. S < O < Cl < F a. 0.498 b. 0.598 c. 2.134 d. 2.597
d. F < Cl < O < S 52. Which one of the following is incorrect?
43. Electronic configurations of four elements A, B, C and a. Non-metals have strong tendency to gain electron
D are given below b. Electronegativity is directly related to non-metallic
A. 1s 2, 2s 2, 2p 6 B. 1s 2, 2s 2, 2p 4 properties of elements
c. Electronegativity is inversely proportional to the
C. 1s 2, 2s 2, 2p 6, 3s 1 D. 1s 2, 2s 2, 2p 5
metallic properties of elements
Which of the following is the correct increasing order d. Increase in electronegativity down the group is
of tendency to gain electron? accompanied by a decrease in non-metallic
properties
a. A < C < B < D b. A < B < C < D
c. D < B < C < A d. D < A < B < C 53. Pauling’s equation for determining the electronegativity
of an element, is
44. In which of following order of arrangement does not
agree with variation of property indicated against it? (X A , X B are electronegativity values of elements A and
3+ 2+ + -
a. Al < Mg < Na < F (Increasing ionic size) B respectively, D represents the polarity of A —B
bond.)
b. B < C < N ³ O (Increasing first ionisation enthalpy)
a. X A - XB = 0.208 D b. X A + XB = 0.208 D
c. I < Br < Cl < F (Increasing electron gain enthalpy)
c. X A - XB = 0.208 D2 d. X A - XB = D
d. Li < Na < K < Rb (Increasing metallic radius)

@iitjeehelps
PERIODIC PROPERTIES 439
54. Match the Column I with Column II and select the 56. Which pair of elements has same chemical properties?
correct answer by given codes. a. 13, 22 b. 3, 11 c. 4, 24 d. 2, 4
Column I Column II 57. The element of second period which forms most
(Elements) (Properties) acidic oxide is
A. + 3+ 2+ + 1. DEA (Electron affinity)
a. carbon b. boron c. fluorine d. nitrogen
Li < Al < Mg <K
B. Li+ > Al3 + > Mg2+ > K + 2. Ionic radii 58. In periodic table, melting point/ boiling point increases
down the group in which of the following group?
C. Cl > F > Br >I 3. EN (Electronegativity) a. Group 13 b. Group 2
D. F > Cl > Br > I 4. ENC c. Group 17 d. Group 1
59. Considering the elements B, C, N, F and Si, the
Codes correct order of their non-metallic character is [NCERT]
A B C D A B C D
a. 2 4 3 1 b. 2 4 1 3 a. B > C > Si > N > F b. Si > C > B > N > F
c. 4 2 3 1 d. 4 2 1 3 c. F > N > C > B > Si d. F > N > C > Si > B

55. The set that contains pairs of elements that do not 60. Considering the elements B, Al, Mg, and K, the correct
belong to same group but show chemical resemblance order of their metallic character is
is a. B > Al > Mg > K b. Al > Mg > B > K
a. B, Al b. Be, Al c. Hf, Zr d. K, Pb c. Mg > Al > K > B d. K > Mg > Al > B

BITSAT Archives
1. Elements/ions having same number of electrons are 3. Which of the following choices represent the correct
known as isoelectronic species. Arrange the following order of first ionisation enthalpy? [2012]
elements incorrect order of atomic/ionic radii and a. B < C < N < O < F b. B > C > N > O > F
choose the correct option from the four choices given c. B < C < N > O < F d. B < C < N > O > F
below [2013] 4. Electron gain enthalpy of fluorine is lower than that of
O2 -, Na + , Mg2+ , F- , Al3 + chlorine is due to [2010]
a. 3+ 2+
Al < Mg < Na < F < O + - 2– a. smaller size
b. smaller nuclear charge
b. Al3+ < Na + < Mg2+ < F - < O2– c. difference in their electronic configurations
c. Al3+ > Mg2+ > Na + > F - > O2– d. its highest reactivity
d. None of the above 5. The atomic numbers of elements A, B , C and D are
2. For the properties mentioned, the correct trend for the Z - 1, Z , Z + 1and Z + 2 respectively. If B is a noble
different species is in [2012] gas, choose the correct statement among the the
a. Strength as Lewis acid— BCl3 > AlCl3 > GaCl3 following statements: [2010]
b. Inert pair effect— Al > Ga > In I. A has higher electron affinity.
c. Oxidising property— Al3+ > In3+ > Tl3+ II. C exists in +2 oxidation state.
d. First ionisation enthalpy— B > Al > Tl III. D is an alkaline earth metal.
a. I and II b. II and III c. I and III d. I, II and III

Answer with Solutions


Practice Exercise
1. (d) Dobereiner has firstly used triad system to organise 4. (c) Lothar Meyer drew a graph showing the relation
the elements. between atomic weight and atomic volume.
2. (c) Johann Dobereiner in early 1800’s was the first to 5. (a) Mendeleef arranged elements in horizontal rows and
consider the idea of trends among properties of elements. vertical columns of a table in increasing order to their
By 1829 he noted a similarity among the physical and atomic weights.
chemical properties of several groups of three elements 6. (c) Mendeleef observed that the periodic properties of an
(triads). element is a periodic function of atomic masses.
3. (b) 7. (a) Germanium was called eka-silicon in Mendeleef’s time.

@iitjeehelps
440 SELF STUDY GUIDE BITSAT

8. (d) 9. (c) 30. (a)


10. (c) The atomic number of elements is equal to the 31. (b) Boron has only one electron in p -subshell, hence it
number of electrons/number of protons in a neutral atom. can be lost easily, while beryllium has comparatively
Z = number of e - / p + stable configuration (1s 2, 2s 2 ) hence, cannot give an
A = number of proton + number of neutron electron easily, consequently its IE is greater.
11. (b) A ® 2, B ® 4, C ® 1,3, D, ® 3,1 32. (d) Nitrogen has stable half-filled orbital.
A. First period contains 2 elements. 33. (b)
B. Third period contains 8 elements. 34. (a) Ionisation energy of N > ionisation energy of oxygen,
C. D. 14 elements of both sixth and seventh periods are since, nitrogen has stable configuration due to half-filled
known as lanthanoids and actinoids respectively. 2p -orbitals.
12. (b) The physical and chemical properties of elements are 35. (c) P has higher ionisation energy due to stable half-filled
periodic functions of their electronic configuration. configuration.
Electronegativity of fluorine is more than that of chlorine. 36. (c) 376 ´ 103 J energy produces 6.023 ´ 1023 ions.
Electropositive nature increases from top to bottom in a 6.023 ´ 1023
group. \1 J energy will produce =
3.76 ´ 103
13. (d) Z = 19 - 10 = 9
= 1.60 ´ 1018 Cs+ ions
EC = 1s 2 2s 2 2p 5, hence, it belongs to p-block.
1 ´ 10-3
14. (c) Ca 2+ ; p = 20, e – = 18 37. (d) 1.00 mg = 1 ´ 10-3 g = mol Li
7
15. (b) Zn ( 3d 10 4s 2 ) is d-block element. 1 mole of Li is converted by 520.3 kJ.
16. (c) The order in which orbitals are filled, is 1 ´ 10-3
\ mole is converted to Li+ by
6s ® 4f ® 5d ® 6p 7
17. (c) In O2 or P– or CI + the electrons present are 16. The 520.3 ´ 1 ´ 10-3
= kJ = 0.0743 kJ
element is sulphur (Z = 16). The electronic configuration 7
of sulphur in second excited state is given by (c). 38. (b) IE2 of Mg is lower than that of Na because in case of
18. (c) Arsenic is in group 15. Mg2 + , 3s electron has to be removed while in case of Na + ,
19. (b) The element having outer electronic configuration an electron from the stable inert gas configuration (neon)
3s 2, 3p 6, 3d 5 4s 2, has atomic number 25. Therefore, the has to be removed.
number of protons is 25. 39. (a) Na ¾® Na + + e -
20. (d) They are chemically inert (inactive). Na + + e - ¾® –X ; Deg = - IE
2
21. (b) Be has fully filled 2s-subshell ( 2s ) and therefore,
40. (d) Higher n value has lower value of IE due to large size.
show least tendency to accept an electron. Thus, Be- is
least stable. 41. (c) The process of formation of O 2- in gas phase is
unfavourable even though O 2- is isoelectronic with neon
22. (c) Atomic number of the given element = 15
because electron repulsion outweights the stability gained
Group number = 10 + 5 (valence electrons) = 15 by achieving noble gas configuration.
Period = 3 42. (b)
Atomic number of the element just below = 33 43. (b) np 5 and ns 1 elements possess a large tendency to
23. (b) For Z = 106, Root of 1 — un, Root of 0 — nil accept the electron than fully and half-filled elements
Root of 6 — hex + ium, IUPAC name = unnilhexium 44. (c)
24. (a) O2–, F – Mg2+ and Na+ are isoelectronic species (each 45. (b) The electron affinity for halogens,
having 10 e - ). The size of isoelectronic species decreases
F = 332.6 » 333 Þ Y
with increase in nuclear charge (number of protons).
Hence, correct order of size is Cl = - 348.5 » - 349 Þ X
O2– >, F – > Na + > Mg2+ Br = - 324.7 » -325 Þ Z
X has the highest value of electron affinity. Therefore, the
25. (c) 26. (c) 27. (d) 28. (b)
correct order of electron affinity is
29. (b) Na and F ions are isoelectronic, therefore F - has
+ -
Cl2 > F2 > Br2
the largest and Na + has the lowest size. Further, Al with
46. (a) The electron affinity of fluorine is less than that of
higher nuclear charge has lower size than Mg. Thus, the
chlorine, due to very small size of fluorine in which
overall order is negative charge is highly concentrated and repels the
Na + < Al < Mg < F - incoming electrons.

@iitjeehelps
PERIODIC PROPERTIES 441
47. (c) 54. (b) A ® 2, B ® 4, C ® 1, D ® 3
48. (a) In a period. it increases from left to right. A. Li+ < Al3+ < Mg2+ < K +
49. (b) Electronegativity on Mulliken scale and Pauling scale The cation with the greater positive charge will have a
are given by smaller radius because of the greater attraction of the
X M = X P ´ 2.8 = 4.0 ´ 2.8 electrons to the nucleus. Anion with the greater
= 11. 2 negative charge will have the larger radius.
Usually, Mulliken’s value of electronegativity are 2.8 times 1
Positive charge µ
of Pauling value. ionic radius
50. (a) When the electronegativity difference is more than Negative charge µ ionic radius
1.7, the bond formed will be ionic in nature. B. Greater positive charge, increases ENC in case of
51. (d) X C -X H = 0.208 D isoelectronic species while for same group elements,
where, D = E C ¾ H - E C ¾ C ´ E H ¾ H ENC decreases down the group.
C. Cl > F > Br > I
D = 98.8 - 83.1 ´ 104.2
Electron affinity of Cl is highest in halogen family.
\ D = 5.75
D. F > Cl > Br > I
X C - 2.1 = 0. 208 5.75 Electronegativity of fluorine (F) is higher than that of Cl,
X C - 2 .1 = 0.497 Br and I.
X C = 2. 597 55. (b)
52. (d) Decrease in electronegativity down the group is 56. (b) The pair which belongs to same group, i. e. in which
accompanied by a decrease in non-metallic properties. both the elements have same outer electronic
53. (a) Pauling’s equation configuration has same chemical properties.
X A - XB = 0.208 D 3 Þ 1s 2, 2s 1
where, X A and XB are electronegativity values of elements 11 Þ 1s 2, 2s 2 2p 6, 3s 1
A and B and D = EA ¾ B – EA ¾ A ´ EB ¾ B 57. (c) 58. (c)
Here, EA ¾ B , EA ¾ A and EB ¾ B are the bond energies of 59. (d) 60. (d)
the molecules AB, A2 and B2 respectively.

BITSAT Archives
1. (a) Elements/ions having equal number of electrons are 4. (a) Because of the small size of F, electron-electron
known as isoelectronic species. Among isoelectronic repulsions present in its relatively compact 2p-subshell,
species, cations having highest charge are smallest while do not easily allow the addition of an extra electron. On
anion having highest charge are largest. the other hand , Cl because of its comparatively bigger
Cation < Neutral atom < Anion size than F, allows the addition of an extra electron more
Hence, correct choice is easily. Thus, the EA of Cl is higher than that of F.
Al3+ < Mg2+ < Na + < F- < O2- 5. (c) A = Z - 1

2. (d) As we know on moving down the group, first ionisation B = Z ¾® Noble gas (outer subshells ares or p)
enthalpy decreases top to bottom, therefore order of first C=Z +1
ionisation enthalpy for group 13 elements is D=Z +2
B > Al > Ga > In > TI Noble gas wth outers -subshell = He (1s 2)
3. (d) Ionisation energy is the minimum amount of energy Noble gas with outer p-subshell
required to remove the outermost electron from an
= ns 2 np 6 like [Ne]
isolated gaseous atom. Quantitatively, it depends on the
attraction between electron present on outermost shell In periodic table, electron affinity is highest in chlorine of
and nucleus. Greater the interaction between outermost group 17 with electronic configuration of 3s 23p 5.
electron and nucleus, higher will be its ionisation
Noble gas = 3s 2 3p 6 = [Ar]
enthalpy. So, correct order of first it must be
B<C<N<O<F B = [Ar] ¾®Z ¾® 3s 2 3p 6 (Noble gas)
But due to extra stable half-filled electronic configuration A = Z - 1 ¾® 3s 2 3p 5 (Halogen family)
of p-orbital, N has more value of first ionisation enthalpy
than oxygen hence, correct order is C = Z + 1 ¾® 3s 2 3p 6,4s 1 (Alkali metal)

B<C<N>O>F D = Z + 2 ¾® 3s 2 3p 6,4s 2 (Alkaline earth metal)

@iitjeehelps
6
States of Matter

Matter
Anything that has mass and takes up space is known as, ‘matter’. It clearly means, it is everything
around us.
In general, matter exists in three states:
(i) Solids: They have definite volume, a definite shape and hence they are rigid.
(ii) Liquids: They have definite volume, but do not have definite shape and hence they are
non-rigid.
(iii) Gases: They have neither definite shape, nor definite volume and hence they are non-rigid.
It is the simplest state and shows great uniformity in behaviour.

Gaseous State
It is the most disordered state of matter. In this state, matter neither have fixed volume nor fixed
shape. It takes the shape and volume of the container in which it is placed.

Gas Laws
Gases exhibit dependency on temperature, pressure, volume and mass. The inter relationship of
these factors can be analysed through gas laws which are given below:
1. Boyle’s Law (1662)
1
At constant temperature (T ), p µ
V
pV = constant (for given moles and T ) or p1V 1 = p2V 2

T 1 < T2 < T3
pV T3
p p T2
T1

V 1/V O p

@iitjeehelps
STATES OF MATTER 443
2. Charles’ Law (1787) 6. Dalton’s Law of Partial Pressures
At constant pressure ( p ), V µ T Total pressure of a mixture of a number of non-reacting
V gases is equal to the sum of the pressures exerted by
= constant (for given n and p) individual gases.
T
V1 V2 ptotal = p1 + p2 + p3 + p4 + K
or =
T1 T2 l Partial pressure of a gas = Its mole fraction ´ total
pressure exerted by the mixture
p2
l When a gas is collected over the water, the pressure of
p1 the gas
V
pobserved or moist = Pressure of dry gas
p 2 < p1 + aqueous tension or water VP
O T l Relative humidity ( RH ) is given by RH = Partial
Charles obtained experimentally that for 1°C change in pressure of water in air/vapour pressure of water.
temperature of a gas, the volume get changed by a fraction 7. Graham’s Law of Diffusion
1 Rate of diffusion or effusion
of . Hence, if V0 be the volume of a gas at 0°C and it is
273 1 1
recorded to be V at t°C then µ µ
Molecular weight density
1 æ T ö æ 273.15 + T ö
V = V0 + T V0 = V0 ç1 + ÷ = V0 ç ÷ Molecular weight
273.15 è 273.15 ø è 273.15 ø Density =
2
If temperature is lowered, the volume decreases at Volume effused
constant pressure and becomes zero at - 273.15° C. Further Rate of effusion (r) =
Time taken
lowering is not possible thus, it is called the lowest possible
temperature or absolute zero of temperature. r1 t 2 d2 M2
Hence, = = =
T (K) = t (° C ) + 273.15 r2 t 1 d1 M1

3. Gay-Lussac’s Law If p is not constant, r µ p

At constant volume (V ) r1 p M2
\ = 1
r2 p 2 M1
p µT
p
= constant (for given n and V) Kinetic Theory of Gases
T The molecular details regarding gases can be visualised
p1 p2 with the help of kinetic molecular theory of gases which is
or =
T1 T2 based on the following assumptions:
4. Avogadro’s Law l A gas consists of extremely small discrete molecules
so that volume of the molecules is negligible as
It states that equal volumes of all gases at same pressure
compared to the total volume of the gas.
and temperature contain equal number of molecules.
l Gas molecules are in constant random motion with
V µn (at given T and p) high velocities and change directions on collision
V1 V2 with other molecules or walls of container.
or =
n1 n 2 l The intermolecular forces and the force of gravity on
them are negligible.
5. Ideal Gas Equation
l The collisions are perfectly elastic, therefore, there is
w
pV = nRT or pV = RT or pM = dRT no lose of kinetic energy during collision.
M
l The pressure of a gas is caused by bombardment of
where, w = mass of a gas moving molecules against the walls of container.
M = molecular weight of a gas
l In a gas, different molecules have different kinetic
d = density of a gas energies but the average kinetic energy of molecules
R = gas constant = 0. 0821 L atm K -1 mol -1 is proportional to absolute temperature of the gas.
= 8.314 J K -1 mol -1 This is known as Maxwell’s generalisation.

@iitjeehelps
444 SELF STUDY GUIDE BITSAT

On the basis of these assumptions, the following Deviation from ideal behaviour
mathematical expression is derived : At very low temperature and high pressure, real gases show
1 deviation from the ideal gas behaviour.
pV = mNu 2
3 The causes of deviation are that
1 (i) at low temperature and high pressure, volume of a
or pV = Mu 2 (QM = m ´ N , for one mole, m = 1)
3 real gas is materially larger than that predicted for an
where, u = rms velocity ideal gas as the molecules of gas have their own
volume.
Average kinetic energy per molecule
Average KE per mole (ii) intermolecular forces are not negligible.
=
N Compressibility factor (Z )
3 RT 3 é R ù
= = KT êë\ N = K úû pV
2N 2 = 1 (for ideal gas)
RT
where, K = Boltzmann constant Different conditions of compressibility factor ( Z ) for real
gases
Different Types of Molecular Velocities
(i) Z < 1 (as for CH 4 , CO2 ) can be attributed to
Average velocity (v av ) predominance of attractive forces among the
It is the average of the velocities of different molecules of molecules of these gases at the temperature of
the gas at a particular temperature. experiment.
8 RT (ii) Z > 1 can be attributed to the dominance of strong
Average velocity v or vav =
pM repulsive forces among the molecules.

Root mean square velocity (v rms ) van der Waals’ Equation


It is the square root of the mean of the square of velocities van der Waals’ gave the gas equation for real gases as,
of various molecules of the gas at a particular æ an 2 ö
temperature. ç p + 2 ÷(V - nb ) = nRT (for n mole of gas)
è V ø
Root mean square velocity,
æ a ö
3 pV 3 RT ç p + 2 ÷(V - b ) = RT (for 1 mole of gas)
vrms = = è V ø
M M
Here, a and b are van der Waals’ constants whose values
3p
= [Qd = density of the gas] depend on the nature of gas.
d
Critical Phenomenon and
Most probable velocity (v mp )
Liquefaction of Gases
It is the velocity possessed by the maximum fraction of
molecules of the gas at a particular temperature. The phenomenon of converting a gas into liquid is known as
liquefaction at particular temperature, pressure and volume.
2 RT
vp or vmp = [QM = Molecule mass of the gas] When a gas changes into liquid, these conditions are known
M as critical conditions, e.g. T c , V c , pc etc.
8 8a
vrms : vav : vmp :: 3 : : 2 = 1.2248 : 1.1284 : 1 Critical temperature (T c ) =
p 27Rb
= 1 : 0.921 : 0.816 a
Critical pressure ( pc ) =
vrms = 1.085 ´ vav or vav = 0.921 ´ vrms 27b 2
vrms = 1. 224 ´ vmp or vmp = 0. 816 ´ vrms Critical volume (V c ) = 3b
vav = 1. 128 ´ vmp or vmp = 0.886 ´ vav Relation between critical p , V and T
pcV c 3
Ideal and Real Gases = , Z < 1 at critical point
RT c 8
The gas which obeys gas law exactly or at standard
condition is called an ideal gas while the gases which obey At critical point, there is no distinction between gaseous
gas laws under moderate conditions of temperature and state and liquid state as densities of a substance in gaseous
pressure are called ‘Real gases’. and liquid states are same.

@iitjeehelps
STATES OF MATTER 445

Liquid State Units of viscosity coefficient (h) are dynes cm -2 s or


poise.
It is the state of matter in which the molecules are held close to
each other and execute random motion through intervening Reciprocal of viscosity coefficient is known as fluidity
spaces. Most of the physical properties of liquids are controlled of liquid ( f ), i.e.
by the strength of intermolecular attractive forces. 1
f=
h
Evaporation
The transformation of a liquid into gaseous state, at surface, is
called its evaporation. It occurs at all temperatures but increase Solid State
with temperature. Evaporation causes a fall in temperature of The state of matter which has definite volume and a
liquid because the molecules with higher kinetic energy, leaves definite shape due to strong internuclear force of
the surface and hence, average kinetic energy and consequently attraction among the constituent particles is known as
temperature decreases. ‘solid’. Solids do not have translatory motion but can
have only vibrational motion about their mean
Vapour Pressure positions.
When evaporation takes place, the gaseous state of liquid is
called its vapour. These vapours now exert a pressure on the Classification of Solids
liquid. This pressure is called the vapour pressure. Thus, vapour On the basis of the arrangement of their atoms or ions
pressure of a liquid also increases with temperature. or molecules, solids are broadly classified into two
groups :
Boiling Point
During evaporation, the temperature at which vapour pressure 1. Crystalline Solids
of liquid becomes equal to atmospheric pressure is called They have systematic and regular arrangement of
boiling point of that liquid. Obviously, boiling point increases particles and sharp melting point.They have flat faces
with increases in atmospheric pressure. Liquids having weak and sharp edges.
attractive forces, have low boiling point. On the basis of types of bond present, they are further
The temperature of liquid always remains constant during divided into four groups:
boiling until all the liquid is changed into gaseous vapours. (i) Molecular crystal or van der Waals' crystals
Surface Tension e.g. I 2 (s ), H 2O (s) etc.
The surface tension (g) is defined as the force acting right angles (ii) Atomic crystals or covalent crystals
to the surface along unit length of surface. e.g. Graphite, diamond etc.
Due to the surface tension liquid drops are spherical. The rise of (iii) Metallic crystals e.g. All metallic solids etc.
a liquid in a capillary is also due to surface tension. (iv) Ionic crystals e.g. NaCl, CsCl, ZnS etc.
Mathematically, surface tension
Work done to expand the surface 2. Amorphous Solids
g=
Change in area In these solids, the constituent particles are not
arranged in a regular manner, e.g. rubber, glass, pitch,
Units of g are dynes per cm or Nm -1 (in SI system). fused silica, plastics, polymer of high molecular mass.
Crystalline solids are anisotropic, i.e. their physical
Viscosity
properties have different values in different directions
The resistance due to which, liquid does not flow with same while amorphous solids are isotropic, i.e. their physical
speed is called viscosity of liquid. properties have same values in different directions.
Thus, the liquid with less viscosity flow rapidly
(e.g. water, alcohol etc) and vice-versa. Bragg’s Equation
The resistance between two layers of liquid (i.e. the force of Is is used to calculate interfacial distances (d ) of a
friction, f ) is given by crystal. In this method, X-rays are used, which are
A × Dv reflected back by crystal faces in a definite manner.
f = h×
x Following reaction is the Bragg’s equation:
where, h = coefficient of viscosity nl = 2d sinq
A = surface area of each layer of liquid where, l = wave length of X-rays used
Dv = velocity difference of two layers n = order of diffraction, q = angle of diffraction
x = distance between two layers d = interfacial distance

@iitjeehelps
446 SELF STUDY GUIDE BITSAT

Space Lattice or Crystal Lattice The number of spheres surrounding a particular atom is
An array of lattice points showing arrangement of known as coordination number (CN).
constituent particles in different position in three Packing in solids may be divided into the following
dimensional space, is known as space lattice or crystal
lattice. categories:

Crystal Systems One Dimensional Packing


All the crystals found in nature, can be grouped into 7 types When the spheres are placed in horizontal row, touching
of crystal systems which have 14 types of unit cell each other, an edge of the crystal is formed, this is known as
(Bravais lattice) can be tabulated as, one dimensional packing.
Crystal system Axial ratio Axial angles Example

1. Cubic a =b =c a = b = g = 90 ° NaCl

2. Tetragonal a =b ¹c a = b = g = 90 ° TiO2 Two Dimensional Packing


3. Rhombic a ¹b ¹c a = b = g = 90 ° Rhombic It is of two types:
sulphur
(i) Square Close Packing (scp) The particles, when
4. Monoclinic a ¹b ¹c a = g = 90 ° ¹ b Monoclinic placed in the adjacent rows, show a horizontal as
sulphur
well as vertical alignment and form squares.
5. Triclinic a ¹b ¹c a ¹ b ¹ g ¹ 90 ° K 2Cr2O7
1 Coordination
6. Trigonal a =b =c a = b = g ¹ 90 ° Quartz number is 4 (scp)
4 2
7. Hexagonal a =b ¹c a = b = 90 ° , Graphite
g = 120 ° 3

This is also known as AAA... type arrangement.


Unit Cell
(ii) Hexagonal Close Packing (hcp) The particles in
It is the smallest group of lattice points which when
repeated in all directions will develop the entire lattice. every next row are placed in the depression between
the particles of the first row. The particles in the third
It can be
row will be vertically aligned with those in the first row.
(i) Simple unit cell or primitive unit cell in which
particles are present only at corners. 1 2
1 6 3
\ Number of atoms per unit cell = ´ 8 = 1
8 5 4
(ii) Face centred cell in which the particles are present
Coordination number is 6 (hcp)
at the corners as well as at the centre of each face.
1 1 This is also known as ABAB... type arrangement.
\ Number of atoms per unit cell = ´ 8 + ´ 6 = 4
8 2
Three Dimensional Packing
(iii) Body centred unit cell in which the particles are
present at the corners of the cube as well as one It is of two types:
particle is present at the centre within the body. (i) Hexagonal Closed Packing (hcp) The first layer A
1 and second layer B, are arranged as AB AB K
\ Number of atoms per unit cell = ´ 8 + 1 = 2
8 pattern is called hexagonal closed packing (hcp).
(ii) Cubic Close Packing (ccp) When the third layer is
Packing in Solids placed over the second layer in such a way that
In solid, each atom is surrounded by certain number of spheres cover the octahedral voids, a layer different
atoms leaving the minimum vacant space. from layers A and B is produced. i.e layer C.
It is expressed in terms of packing fraction (PF). This pattern of stacking spheres is called ABC ABCK
Total volume of spheres pattern or cubic close packing (ccp). It is similar to
Packing fraction = ´ 100 face centred cubic (fcc) packing.
Volume of the unit cell

@iitjeehelps
STATES OF MATTER 447
Voids
The empty space between stacked atoms is known as void. These defects are found as Schottky and Frenkel defects.
Number of tetrahedral voids S. No. Schottky Defect Frenkel Defect
= 2 ´ Number of octahedral voids
1. It is due to equal number of It is due to the missing of ions
Number of octahedral voids = Number of lattice point cations and anions missing (usually cations) from the lattice
from the lattice sites. sites and they occupy the
Sizes vary the order,
interstitial sites.
Trigonal > Tetrahedral > Octahedral
2. This results in the decrease in It has no effect on the density of
Radius Ratio Rule density of crystal. crystal.
Radius of cation
Radius ratio = 3. This type of defect is found in This type of defect is found in
Radius of anion highly ionic compounds with crystals, where the difference in
high coordination number, the size of cations and anions is
Relation between radius ratio and coordination number e.g. NaCl, CsCl, AgBr etc. very large, e.g. AgCl, AgBr, ZnS
Radius Ratio Cation Coordination Structure Example etc.
æ r+ ö occupied number
ç ÷
è r- ø
(ii) Non-Stoichiometric Defects
< 0.155 2 Linear When the ratio of cations and anions, due to imperfection,
differ from that indicated by their molecular formula, the
0 .155 - 0 .225 Trigonal 3 Planar B 2O3
defects are called non-stoichiometric defects.
void triangular
F-centre A negative ion may be missing from its lattice
0 .225 - 0 .414 Tetrahedral 4 Tetrahedral ZnS
void site, leaving a hole which is occupied by an electron,
thereby maintaining the electrical balance.
0 .414 - 0 .732 Octahedral 6 Octahedral NaCl
void
The electrons, thus trapped in the anion vacancies are
called F-centres because they are responsible for
0 .732 - 1.0 Cubic void 8 bcc CsCl imparting colour to the crystals.

Calculation Involving Unit Cell Parameters Properties of Solids


(i) Relation between edge length, radius of
Some remarkable properties of solids are described below :
sphere and packing fraction
Unit cell Side/edge length Radius Packing fraction Electrical Properties of Solids
Depending upon conduction power, solids are divided into
Simple cubic a a /2 52%
conductors conductivity 107 ( Wm )-1 , semiconductors
bcc a ( 3 / 4) a 68 % (10-6 to 104 ) ( Wm )-1 and insulators (10-20 - 10- 10 ) ( Wm )-1 .
fcc a a /2 2 74 %
Superconductivity is the property of the conductor by
which they allow electricity to pass through them
(ii) Determination of density of crystals without any resistance.
Z´ M
Density, d = The temperature at which a substance starts behaving
N A ´ a 3 (V ) as superconductor is called transition temperature,
where, Z = number of atoms per unit cell e.g. Ba 2Cu3O7 at 90K, Bi 2Ca 2Sr2Cu3O10 at 105 K,
M = molar mass Tl 2 Ca 2 Ba 2 Cu3 O10 at 125 K start behaving as
a = edge length of unit cell superconductors.
N A = Avogadro’s number = 6.022 ´10 23 Band Theory to Explain Electrical
Property of Solids
Imperfections as Defects in Solids
Atomic orbitals combine with each other to form
These are of following two types: molecular orbitals which are close to each other. These are
called band. There are two types of band which are valence
(i) Stoichiometric Defects
band and conduction band.
If imperfection in crystals are such that the ratio between the
The energy gap between valence band and conduction
cations and the anions remain the same as in its molecular
formula, the defect will be called stoichiometric defects. band is used to explain electrical properties of metal.

@iitjeehelps
448 SELF STUDY GUIDE BITSAT

(a) In a conductor, there is no gap between valence band and conduction band.
(b) In a semiconductor, the gap between valence band and conduction band is small. So, conductivity of semiconductor
is low but conductivity can be increased by supplying energy.
There are two types of semiconductors depending upon the doping of foreign elements:
n-type semiconductors Group 14 elements when doped with group 15 elements form n-type semiconductors.
p-type semiconductors Group 14 elements when doped with group 13 elements form p-type semiconductors.
(c) In an insulator, the energy gap is so large that it cannot be covered up by supplying energy.

Magnetic properties
Alignment of magnetic
Properties Description Examples Applications
dipole

Diamagnetic Which are feebly repelled by All electrons are paired. TiO2 , V2O5 NaCl, C 6 H 6 Insulator
the magnetic fields due to
­ ¯ ­ ¯ ­¯
existence of paired electron.

Paramagnetic Attracted by magnetic field At least one unpaired O2 ,Cu2+ , Fe 3 + ,TiO, Ti 2O3 , Electronic applications
due to the presence of electron. ­ ¯ ­ ­
VO, VO2 , CuO
unpaired electrons acts as a
tiny bar magnet.

Ferromagnetic Acts as a permanent magnet All unpaired electrons are in Fe, Ni, Co, CrO2 CrO2 is used in audio and
even after removing same direction. video tapes.
magnetic field. Above curie
­­­­­
temperature, no
ferromagnetism occurs.

Anti ferromagnetic Net dipole moment becomes ­ ¯ ­ ¯ ­ ¯ CoO, Co3O4 , MnO, MnO2 ,
zero due to equal and M 2O3 , FeO, Fe 2O3 , NiO
opposite alignment.

Ferrimagnetic This arises when there is no ­¯¯¯­¯ Fe 3O4 , Ferrites


dipole moment.

Practice Exercise
1. To which of the following Dalton’s law of partial 4. At 25°C and 760 mm of Hg pressure a gas occupies
pressure is not applicable? 600 mL volume. What will be its pressure at a height
a. SO2 and CO2 at room temperature where temperature is 10°C and volume of the gas is
b. N2 and H2 at room temperature 640 mL?
a. 676.6 mm Hg b. 600 mm Hg
c. SO2 and O2 at room temperature
c. 700 mm Hg d. 752 mm Hg
d. HCl and NH3 at room temperature
5. The pressure of 1 atmosphere is equal to
2. A sample of gas has a volume of 0.2 litre at 1 atm a. 760 cm b. 105 Nm-2
pressure and 0°C. At the same pressure but at 273°C c. 104 dyne cm-2 d. 1 bar
its volume will become.
a. 0.1 L b. 0.4 L 6. A vessel contains a mixture of different types of
c. 0.8 L d. 0.6 L gases. Which of the following statements is correct?
a. On the average, the heavier molecules have higher
3. A vessel is filled with a mixture of equal masses of speed
oxygen and nitrogen. What is the ratio of partial b. The average translational energy of different types
pressure of oxygen and nitrogen? of molecules is the same
a. p O2 = 0.5p N2 b. p O2 = 0.875p N2 c. The average speed of different molecules is same
d. On an average, the heavier molecules have higher
c. p O2 = p N2 d. p O2 = 1.14p N2
translational energy

@iitjeehelps
STATES OF MATTER 449
7. Diffusion of He gas is 20. The units of van der Waals’ constants a and b
a. 4 times faster than CO2 respectively are
b. 4 times faster than SO2 a. L-2 atm- 1mol- 1 and L mol- 2
c. 4 times faster than NO2 b. L2 atm mol- 2 and mol- 1 L
d. 4 times faster than ClO2 c. L atm mol2 and mol L
d. L atm 2 mol- 1 and mol L- 1
8. The vapour density of a gas is 11.2. The volume
occupied by 11.2 g of the gas at NTP is 21. If van der Waals’ parametres for gases P , Q, R and S
a. 22.4 L b. 11.2 L c. 1 L d. 2.24 L are given as
9. The volume of 2.8 g of carbon monoxide at 27°C and
Gas a (time2 atm/mol2 ) b (time/mol)
0.821 atm pressure, is
a. 30 L b. 3 L c. 0.3 L d. 1.5 L P 4.0 0.027
10. What will be the temperature when the rms velocity is Q 8.0 0.030
four times of that at 300 K?
R 12.0 0.027
a. 300 K b. 900 K c. 4800 K d. 1200 K
S 6.0 0.024
11. The density of neon is lowest at
a. STP b. 0°C, 2 atm The gas which has the highest Boyle temperature is
c. 273°C, 1 atm d. 273°C, 2 atm
a. P b. Q c. R d. S
12. When 3.2 g sulphur is vaporised at 450°C and 22. A balloon is filled with hydrogen at room temperature.
723 mm Hg pressure, the vapours occupy a volume of
It will burst, if pressure exceeds 0.2 bar. If a 1 bar
780 mL. What is the molecular formula of S vapours?
pressure the gas occupied 2.27 L volume, up to what
a. S2 b. S4 c. S6 d. S8
volume can the balloon be expanded?
13. At what temperature, the rms velocity of SO2 be same a. 6.25 L b. 11.35 L c. 8.35 L d. 10.50 L
as that of O 2 at 303 K? 23. The average velocity of an ideal gas molecule at
a. 273 K b. 606 K c. 303 K d. 403 K 300 K is 2 m/s. The average velocity at 1200 K will be
a. 6 m/s b. 4 m/s c. 2 m/s d. 8 m/s
14. The value of compression factor, Z for critical
constants, is 24. In van der Waals' equation of the gas law, the
1 3 constant ‘a’ is a measure of
a. b.
2 4 a. intermolecular repulsion
2 3 b. intermolecular attraction
c. d.
3 8 c. volume occupied by the molecules
d. relative velocity of gas molecules
15. The ratio between the root mean square velocity of
H 2 at 50 K and that of O2 at 800 K, is 25. Which of the following liquid will exhibit highest vapour
a. 4 b. 2 c. 1 d. 1/4 pressure?
a. C2H5OH(l ) b. NH3(l ) c. HF (l ) d. H2O (l )
16. A vessel contains He and H 2 in the molar ratio 1 : 5.
The ratio of mean translational kinetic energies, at the 26. Which of the following relation is correct for an ideal
same temperature, is gas regarding its pressure ( p ) and translational kinetic
a. 1 : 1 b. 1 : 2 c. 2 : 1 d. 1 : 5 energy per unit volume (E )?
3 2
17. The ratio of root mean square velocity to average a. p = E b. p = E
velocity of a gas molecule at a particular temperature 2 3
1
is c. p = E d. p = 2E
2
a. 1 : 1.086 b. 2 : 1.86
c. 1.086 : 1 d. 2.086 : 1 27. The surface tension of which of the following liquids is
18. The RMS speed of helium in ms -1 maximum?
-1 a. H2O b. CH3OH c. C2H5OH d. C6H6
(atomic mass = 4.0 g mol ) at 400 K, is
a. 16.8 b. 60 28. Which of the following is not a crystalline solid?
c. 168 d. 1580 a. KCl b. CsCl
c. Glass d. Rhombic sulphur
19. A mixture of gases having different molecular weights
is separated by which method? 29. The arrangement ABC ABC .... is referred to as
a. Atmolysis a. octahedral close packing
b. Metathesis b. hexagonal close packing
c. Ostwald and Walker method c. tetrahedral close packing
d. Reverse osmosis d. cubic close packing

@iitjeehelps
450 SELF STUDY GUIDE BITSAT

30. How many Cl- ions are there around Na + ion in a 39. A monoclinic crystal has dimensions
NaCl crystal? a. a ¹b ¹ c , a = g = 90° ; b ¹ 90°
a. 3 b. 4 b. a =b = c , a = b = g = 90°
c. 6 d. 8 c. a =b = c , a = b = 90° ; g = 120°
31. Which of the following statements is not true about d. a ¹b = c , a = b = g = 120°
NaCl structure?
a. Cl- ions are in fcc arrangement 40. In A +B - ionic compound, radii of A + and B - ions are
b. Na + ions have coordination number four 180 and 187 pm respectively. The crystal structure of
c. Cl- ions have coordination number six the compound will be
d. Each unit cell contains 4 NaCl molecules a. NaCl type b. CsCl type
c. ZnS type d. similar to diamond
32. The number of atoms in 100 g of a fcc crystal with
density = 10 g cm -3 and cell edge as 200 pm, is equal 41. A compound having bcc geometry has atomic mass 50.
Calculate the density of the unit cell, if its edge length is
to
25 24 290 pm.
a. 3 ´ 10 b. 5 ´ 10
a. 6.81 g cm-3 b. 3.40 g cm- 3
c. 1 ´ 1025 d. 2 ´ 1025
c. 13.62 g cm-3 d. None of these
33. Which of the following expressions is correct for a 42. If ‘a’ stands for the edge length of the cubic systems,
CsCl unit cell with lattice parameter, a? simple cubic, body centred cubic and face centred
a
a. rCs+ + rCl- = 2a b. rCs+ + rCl- = cubic, then the ratio of radii of the spheres in these
2 systems respectively will be
3 3 1 1 1 3 2
c. rCs+ + rCl- = a d. rCs+ + rCl- = a a. a : 3a : a b. a: a: a
2 2 2 2 2 2 2
34. In diamond, the coordination number of carbon is 1 3 1
c. 1a : 3a : 2a d. a : a: a
a. four and its unit cell has eight carbon atoms 2 4 2 2
b. four and its unit cell has six carbon atoms
c. six and its unit cell has four carbon atoms 43. Calculate the approximate number of unit cells
d. four and its unit cell has four carbon atoms present in 1g of gold. It is well known that gold
crystallises in the face centred cubic lattice (atomic
35. The molar volume of KCl and NaCl are 37.46 mL and mass of gold is 197u).
27.94 mL respectively. The ratio of the unit cube a. 7.64 ´ 1020 b. 6.02 ´ 1023
edges of the crystals is
c. 197 d. 4
a. 1.296 b. 1.116
c. 1.341 d. 0.950 44. A solid compound contains X , Y and Z atoms in a
cubic lattice with X atoms occupying the corners.
36. Calcium crystallises in a face centred cubic unit cell
Y atoms in the body centred positions and Z atoms at
with a = 0.556 nm. Calculate the density if it contained
the centres of faces of the unit cell. What is the
0.1% Schottky defects.
empirical formula of the compound?
a. 1.5463 g/cm3 b. 1.4962 g/cm3
a. XY2Z 3 b. XYZ 3
c. 1.5448 g/cm3 d. 1.5943 g / cm3
c. X 2Y2Z 3 d. X 8YZ 6
37. Calculate the ionic radius of a Cs + ion, assuming that
the cell edge length for CsCl is 0.4123 nm and that the
45. Total volume of atoms present in a fcc unit cell of a
metal is (r = atomic radius).
ionic radius of a Cl- ion is 0.181 nm. 16 3 12 3
a. 0.176 nm b. 0.231 nm a. pr b. pr
3 3
c. 0.357 nm d. 0.116 nm 24 3 20 3
c. pr d. pr
38. Iron crystallises in a bcc system with a lattice 3 3
parameter of 2.861 Å. Calculate the density of iron in
46. What will be the per cent fraction of edge length not
the bcc system.
covered by atom when a metal crystallises in bcc
(Atomic weight of Fe = 56, N A = 6.02 ´ 1023 mol- 1)
lattice?
a. 7.92 g mL-1 b. 8.96 g mL-1 a. 11.4% b. 10.4%
c. 2.78 g mL-1 d. 6.72 g mL-1 c. 13.4% d. 12.4%

@iitjeehelps
BITSAT Archives
1. The temperature 30.98°C is called critical temperature 8. A compound contains P and Q elements. Atoms Q are
(Tc ) of carbon dioxide. The critical temperature is the in ccp arrangement while P occupy all tetrahedral
[2014] sites, formula of compound is [2009]
a. lowest temperature at which liquid carbon dioxide is a. PQ b. PQ 2 c. P2Q d. P3Q
observed 9. What is the temperature at which the kinetic energy of
b. highest temperature at which gas carbon dioxide is
0.3 mole of helium is equal to the kinetic energy of
observed
0.4 mole of argon at 400 K? [2008]
c. highest temperature at which solid carbon dioxide is
observed a. 400 K b. 873 K
d. highest temperature at which liquid carbon dioxide is c. 533 K d. 300 K
observed 10. For a crystal, the angle of diffraction ( 2 q ) is 90° and
2. Point out the correct statement for the set of the second order line has a d value of 2.28Å.
characteristics of ZnS crystal. [2014] The wavelength (in Å) of X -rays used for Bragg's
a. Coordination number (4 : 4) ; ccp; Zn2 + ion in the diffraction is [2008]
alternate tetrahedral voids a. 1.612 b. 2.00 c. 2.28 d. 4.00
b. Coordination number (6 : 6) ; hcp; Zn2 + ion in all
tetrahedral voids 11. A solid has a structural in which W atom are located at
c. Coordination number (6 : 4) ; hcp; Zn2 + ion in all the corners of a cubic lattice, O atom at the centre of
octahedral voids edge and Na atoms at the centre of cube. The formula
d. Coordination number (4 : 4) ; ccp; Zn2 + ion in all for the compound is [2007]
tetrahedral voids a. Na 2WO3 b. Na 2WO2
c. NaWO2 d. NaWO3
3. KCl crystallises in the same type of lattice as does
NaCl. Given that, r Na + / r Cl- = 0.55 and 12. Which of the following is most volatile compound?
r K + / r Cl- = 0.74. Determine the ratio of the side of the [2007]
unit cell for KCl to that of NaCl [2013] a. Hl b. HCl c. HBr d. HF
a. 0.124 b. 1.1226 c. 0.891 d. 1.414 13. The root mean square velocity of a gas is double
4. A gas is heated through 1°C in a closed vessel and so when temperature is [2007]
the pressure increases by 0.4%. The initial a. increased four times
temperature of the gas was [2012] b. increased two times
a. -23° C b. + 23° C c. 250°C d. 523°C c. reduced to half
d. reduced to one - fourth
5. Barium titanate has the pervoskite structure, i.e. a 14. The molar volume of CO2 is maximum at [2006]
cubic lattice with Ba 2 + ions at the corners of the unit
cell, oxide ions at the face centres and titanium ions at a. NTP
b. 0°C and 2.0 atm
the body centred. The molecular formula of barium
c. 127°C and 1 atm
titanate is [2012]
d. 273°C and 2 atm
a. BaTiO3 b. BaTiO4 c. BaTiO2 d. BaTiO
-6
15. The RMS speed of hydrogen is 7 times the RMS
6. 1 L of a gas is at a pressure of 10 mm of Hg at 25°C. speed of nitrogen. If T is the temperature of the gas,
How many molecules are present in the vessel? then
a. 3.2 ´ 106 b. 3.2 ´ 1013 [2011] a. TH2 = TN2 b. TH2 > TN2 [2005]
c. 3.2 ´ 1010 d. 3 ´ 104 c. TH2 < TN2 d. TH2 = 7 TN2
7. An ideal gas cannot be liquefied because [2009]
16. Which gas has the highest partial pressure in
a. its critical temperature is always above 0°C
atmosphere? [2005]
b. its molecules are relatively smaller in size
c. it solidifies before becoming a liquid a. CO2 b. H2O
d. forces operating between its molecules are negligible c. O2 d. N2

@iitjeehelps
Answer with Solutions
Practice Exercise From Eq. (i),
1
=
300
4 T2
1. (d) Dalton’s law is applicable to those mixtures of gases
which don’t react. HCl and NH3 react at room temperature 1 300
=
react to give NH4Cl. 16 T2
V V T2 = 16 ´ 300
2. (b) According to Charle’s law, 1 = 2
T1 T2 = 4800 K
0.2 ´ 546 11. (c) On applying ideal gas equation we get
V2 = = 0.4 L
273 p
d µ
3. (b) Let 32 g of each gas is taken. Then, T
32 32 15 Therefore, for low density, p should be less while
Total moles = + =
32 28 7 temperature should be high. Hence, at 273°C and 1 atm
1 7 pressure, density of neon will be minimum.
Mole fraction of O2 = =
15 / 7 15 12. (d) From ideal gas equation,
8/7 8 w
Mole fraction of N2 = = pV = RT
15 / 7 15 M
7 3.2 ´ 0.0821 ´ 760 ´ 1000 ´ 723
p O2 15 ´ 1 7 15 7 \ M = = 256
\ = = ´ = = 0.875 723 ´ 780
p N2 8
´ 1 15 8 8
Q 32 g sulphur has 1 atom of S.
15
\ Molecular formula = S8
p O2 = 0.875p N2
pV pV U rms (SO2 ) T (SO2 ) M (O2 )
4. (a) According to combined gas law, 1 1 = 2 2 13. (b) = ´
T1 T2 U rms (O2 ) M (SO2 ) T (O2 )
(760 mm Hg) ´ (600 mL) ´ (283 K) T (SO2 ) 32
Þ p2 = = 676.6 mmHg i.e. 1 = ´ or T ( SO2 ) = 606 K
(640 mL) ´ (298 K) 64 303
5. (d) 1 atmosphere = 1.01 ´ 105 Nm-2 = 1bar pcVc a ´ 3b ´ 27 3
14. (d) Compressibility factor ( Z c ) = = =
RTc 27b 2 ´ 8a 8
6. (b) The average translational energy of different types of
molecules is the same because it depends only on pcVc 3
=
temperature. RTc 8
r M2 3RT
7. (b) According to the Graham’s law, 1 = 15. (c)U rms =
r2 M1 M
M2 M2 U1 3RT1 M2 T M 50 ´ 32
4= Þ 16 = \ = ´ = 1 2 = =1
4 4 U2 M1 3RT2 T 2 M1 800 ´ 2
\ M 2 = 64 3 3
16. (d) KE = pV = nRT
64 is the molecular weight of SO2 hence, (b) is correct. 2 2
8. (b) Molecular weight of gas = 2 ´ VD = 2 ´ 11. 2 = 22 . 4 3
( KE) He 2 RT æ 1ö 1
11.2 1 = ç ÷ = =1:5
Hence, number of moles of gas taken = = ( KE) H2 3 è ø 5
22.4 2 RT 5
2
1
Hence, volume of gas at NTP = ´ 22.4 L = 11.2 L 17. (c) For a gas at temperatureT , M is constant.
2
nRT 2.8 ´ 0.0821 ´ 300 v rms 3RT pM 3´ p
9. (b) pV = nRT orV = = = 3L \ Ratio of = ´ =
p 28 ´ 0.821 v av M 8RT 8
3RT 3 ´ 3.14 1.086
10. (c)U rms (velocity) = Q = =
M 8 1
u1 T 3RT 3 ´ 8.314 ´ 400
Hence, u µ T Þ = 1 …(i) 18. (d)v rms = = » 1580 m /s
u2 T2 M ( 4 ´ 10- 3 ) kg

@iitjeehelps
STATES OF MATTER 453
19. (a) Diffusion method is used to separate a mixture ZM æ 0.1 ö
36. (c) d = ; Z = 4 ´ ç1 - ÷ = 3.996
of gases having different molecular weight as rate N0a 3 è 100 ø
of diffusion varies inversely with molecular mass,
i.e. (because it contains 0.1% Schottky defect)
1 3.996 ´ 40
r µ d = = 1. 5448 g / cm3
M 6.02 ´ 1023 ´ (0.556 ´ 10-7 )3
d body 0.7141
This method is called atmolysis. 37. (a) rCs+ + rCl- = = nm = 0.3571 nm
pV 2 atm ´ L2 2 2
20. (b) \ Units of a = = = L2 atm mol- 2 rCs+ = 0.3571 - 0.181 = 0.176 nm
n2 mol2
V L ZM
\ Units of b = = = mol- 1 L 38. (a) d = (for bcc, Z = 2)
n mol NA a 3
a a (2) ´ 56.0 g mol- 1
21. (c)TB = Þ Higher the value of , higher will be d Fe = = 7.92 g cm- 3
Rb b (6.02 ´ 10 mol- 1) (2.861 ´ 10- 8 )3 cm3
23

the Boyle's temperature.


39. (a) Dimensions of a monoclinic crystal lattice are a ¹ b ¹ c and
22. (b) According to Boyle's law, p1V1 = p 2V2 a = g = 90° , b ¹ 90°.
p1V1 1bar ´ 2.27 L r + 180
V2 = = = 11.35 L 40. (b) - = = 0.96, this value lies in the range of 0.732 - 1.000.
p2 0.2 bar r 187
8RT \ Crystal structure will be of CsCl type.
23. (b) v av = Þ v av µ T
pM Z ´M
41. (a) Density of unit cell, d =
2 300 1 1 NA ´ a 3
= = = Þ v 2 = 4 m /s
V2 1200 4 2 2 ´ 50
d = = 6.81g / cm3
24. (b) ( 290 ´ 10 ) ´ 6.023 ´ 1023
-10 3

25. (b) NH3 have weakest hydrogen bonding among [QFor bcc, Z = 2 ]
these. Hence, its molecules have most escaping a 3 1
42. (d) Ratio of radius of scc, bcc, fcc will be : a: a.
tendency and therefore NH3 has greatest vapour 2 4 2 2
pressure among these liquids.
43. (a) 1 mole of gold = 197 g = 6.02 ´ 1023 atoms.
2 2
E ¢ 1 mNU rms 3 1 mNU rms 3
26. (b) E = = = ´ = p 6.02 ´ 1023
V 2 V 2 3 V 2 \ Number of atoms available in 1 g of gold =
197
2
P = E As fcc unit cell contains 4 atoms.
3
27. (a) Surface tension of H2O is maximum due to 6.02 ´ 1023
\ Number of unit cells present = = 7.64 ´ 1020
maximum hydrogen bonding among the given. 197 ´ 4

28. (c) Glass is an amorphous solid. 1


44. (b) Z x = ´ 8 =1 (at corner)
8
29. (d)
Zy = 1 [At Body centre] , Z z = 3 [At Face centre]
30. (c) In NaCl, both Na + and Cl- ions, have
coordination number 6. Hence, the formula of compound = XYZ 3
31. (b) NaCl has a fcc structure have CN 6 : 6. 45. (a) Total number of atoms in the unit cell of fcc = 4
Z ×M 4 3 4 3
32. (b) We know that, d = \Total volume of an atom = pr ´ 4 [Qv = pr ]
n ´ a3 3 3
4 ´ 100 16 3
n= = pr
10 ´ ( 200 ´ 10-10 )3 3
= 5 ´ 1024 46. (c) Edge length not covered by atom = a - 2r
3
33. (c) CsCl crystal has bcc structure, [Q For bcc, r = a]
4
3
Þ rCs+ + rCl- = a 3 é2 - 3ù
2 =a - 2 ´ a =a ê ú
4 ë 2 û
34. (a) The space lattice of diamond is fcc and has
tetrahedral geometry. Thus, the conventional unit é2 - 3ù
cube contains eight atoms with CN four. aê ú ´ 100
ë 2 û
1 \ % of fraction not covered =
d KCl æ 37.46 ö 3 a
35. (b) =ç ÷ = 1.116
d NaCl è 27.94 ø = 0.134 ´ 100 = 13.4%

@iitjeehelps
454 SELF STUDY GUIDE BITSAT

BITSAT Archives
1. (d) 7. (d) In the ideal gas, the intermolecular forces of attraction
2. (a) ZnS has zinc blende type structure (i.e. ccp are negligible and hence, it cannot be liquefied.
structure). The S2 - ions are present at the corners of 8. (c) Suppose number of atoms Q in the ccp arrangement
the cube and at the centre of each face. Zinc ions = 100
occupy half of the tetrahedral sites. Each zinc ion is
Then, number of tetrahedral sites = 200
surrounded by four sulphide ions which are disposed
towards the corner of regular tetrahedron. Similarly, As all the tetrahedral sites are occupied by atoms P,
S2 - ion is surrounded by four Zn2+ ions. therefore their number = 200
3. (b) Given that rNa+ / rCl = 0.55, rK+ /rCl- = 0.74 Hence, ratio P : Q = 2 : 1
i.e. the formula is P2 Q .
rNa+
Q = 0.55 9. (c) We know that, kinetic energy = nRT
rCl-
Kinetic energy of helium = 0.3 ´ R ´ T … (i)
rNa+
\ + 1 = 0.55 + 1 Kinetic energy of argon = 0.4 ´ R ´ 400 …(ii)
rCl-
According to question
rNa+ + rCl- Kinetic energy of helium = Kinetic energy of argon
= 1.55 …(i)
rCl- 0.3 ´ R ´ T = 0.4 ´ R ´ 400
rK+ T = 533 K
\ = 0.74
rCl- 10. (a) Given, angle of diffraction ( 2q) = 90°
rK+ + rCl- q = 45°
Similarly = 1.74 …(ii) Distance between two planes, d = 2.28 Å
rCl-
n = 2 [Q second order diffraction]
Dividing Eq. (ii) by Eq. (i)
Bragg's equation is nl = 2d sin q
rk + + rCl- 1.74
= = 1.1226 2 ´ l = 2 ´ 2.28 ´ sin 45° Þ l = 1.612
rNa+ + rCl- 1.55 1
11. (d) In a unit cell, W-atoms at the corner = ´ 8 = 1
4. (a) LetT1 = T ÞT2 = (T + 1) 8
0.4 p 100.4 1
and p1 = p Þ p 2 = p + = p O-atoms at the centre of edge = ´ 12 = 3
100 100 4
p1V1 p 2V2 pV 100.4p V Na-atoms at the centre of the cube = 1
From = , = ´
T1 T2 T 100 (T + 1) W : O : Na = 1: 3 : 1
100 T + 100 = 100.4 T Hence, formula = NaWO3
100 12. (b) Boiling point of HF is highest due to H-bonding. For
T = = 250 K other halogen acids boiling point increases in the order
0.4
HCl < HBr < Hl. Therefore, most volatile (with lower boiling
= ( 250 - 273)° C = - 23° C point) is HCl.
5. (a) Number of Ba 2+ ions at the corner of unit cell 3RT
1 13. (a)v rms = or v rms µ T
= 8 ´ =1 M
8
v rms T 1 T
1 = Þ = Þ T ¢ = 4T
Number of O2 - ions at the face centres = 6 ´ = 3 v ¢rms T¢ 2 T¢
2
nRT
Number of Ti4 + ions at the body centre = 1 14. (c) Fromv =
P
\ Molecular formula of barium titanate = BaTiO3 At high T and low p, V is high.
p1V1 p 2V2 10- 6 ´ 1000 760 ´ V2 u rms( H2 ) éT ( H2 )M ( N2 ) ù
1/ 2
6. (b) = Þ = 15. (c) = 7=ê = 7
T1 T2 298 273 u rms( N2 ) ú
ë M ( H2 )T ( N2 ) û
V2 = 1.2 ´ 10- 6 cc (at STP) T ( H2 ) M ( H2 ) æ 2ö 1
Hence, =7 = 7ç ÷ =
Number of molecules T ( N2 ) M ( N2 ) è 28 ø 2
6.02 ´ 1023
= ´ 1.2 ´ 10- 6 TH2 < TN2
22400
16. (d) N2 has the highest partial pressure in atmosphere.
= 3.2 ´ 1013

@iitjeehelps
7
Chemical
Thermodynamics

Thermodynamics
The various forms of energy are interrelated and under certain conditions these may be transformed
from one form into another. The study of these energy transformation forms the subject matter of
thermodynamics. It is based on four generalisations, i.e. zeroth, first, second and third law of
thermodynamics.

System
It is the part of the universe, which is under the thermodynamic investigation.

Types of System
(i) Open system The system, which can exchange energy as well as matter with the surroundings,
e.g. tea in a cup, human body, etc.
(ii) Closed system The system, which can exchange only energy with the surroundings but not the
matter, e.g. hot water in a sealed tube.
(iii) Isolated system The system, which is perfectly insulated and hence, cannot exchange energy
and matter with the surroundings, e.g. ice in a thermos flask.

Surroundings and Boundary


The part of the universe other than the system, is known as its surroundings. A system is separated
from the surroundings by a real or imaginary boundary.

Thermodynamic Properties
Intensive and Extensive Properties
An intensive property is that which does not depend upon the quantity of matter present in the
system, e.g. temperature, density, refractive index, boiling point, viscosity, specific heat etc. While
extensive properties are those properties, which depend upon the quantity of matter present in the
system, e.g. mass, volume, energy, heat capacity, enthalpy, entropy etc.

@iitjeehelps
456 SELF STUDY GUIDE BITSAT

State functions 2. Work done in isothermal irreversible


These are the properties which depend on the state and not expansion
on the path followed to obtain that state, e.g. internal W = - pext ´ DV
energy, enthalpy, entropy, free energy etc. If expansion occurs in vacuum, i.e. at zero pressure, the
value of work done is zero.
Processes W = - pext ´ DV = 0
The state of variable can be changed by means of an 3. Work done in adiabatic expansion
operation called process.
nR
W = (T 2 - T 1 )
These are of following types : g -1
(i) Adiabatic process The process in which system Work done by the system,W = -ve ; \ DU = -ve
does not exchange heat with its surroundings, i.e.
dQ = 0. Work done on the system,W = +ve ; \ DU = +ve
(ii) Isothermal process The process in which 4. Work done in cyclic process is always zero
temperature remains constant, i.e. dT = 0. i.e. ò dW =0
(iii) Isobaric process The process in which change of
state is brought about at constant pressure, i.e. Internal Energy (E or U)
dp = 0.
The total amount of energy associated with a molecule is
(iv) Isochoric process The process in which volume of
known as internal energy. It is denoted by U .
the system remains constant, i.e. dV = 0.
U = U trans + U vib + U elec + U rot +¼
(v) Cyclic process This is the process in which a
system undergoes a number of different states and T = constant, DU = 0
finally returns to its initial state. For such a process, Internal energy depends on temperature, pressure, volume
change in internal energy and enthalpy is zero, i.e. and quantity of matter.
dE = 0 and dH = 0.
(vi) Reversible process It is an ideal process and here,
Heat
every intermediate state is in equilibrium with It is defined as the quantity of energy, which flows between
others, if any. This is a slow process. system and surroundings on account of temperature
(vii) Irreversible process That process, cannot be difference. It is also a path function, i.e. depends upon the
reversed in which amount of energy increases. All path followed.
natural processes are irreversible in nature. It is given as
H = msDt
First Law of Thermodynamics where, m = mass of substance
It states that energy can neither be created nor be destroyed s = specific heat
although, it can be changed from one form to another. Dt = temperature difference
Mathematically,
DE = q + W …(i) Enthalpy
where, DE = change in internal energy The total heat content of a system at constant pressure is
called the enthalpy of the system. Indeed, it is the sum of
q = heat absorbed
internal energy and the product of pressure-volume work.
W = work done It is an extensive quantity and is represented by the symbol
Concept of Work H. The equation is H = E + pV
Work is a medium to transfer energy from a system to DH = DE + pDV
surrounding. DH = DE + Dn g RT
1. Work done in isothermal reversible expansion where, DH = enthalpy change
V p Dn g = gaseous moles of products
W = -2.303nRT log 2 = -2.303nRT log 1
V1 p2 - gaseous moles of reactants
where,V 1 = initial volume , V 2 = final volume If Dn g = 0, then DH = DE ; If Dn g > 0 then DH > DE and
p1 = initial pressure, p2 = final pressure if Dn g < 0 then, DH < DE .
R = real gas constant, T = temperature For reaction involving solids and liquids only, DH = DE .

@iitjeehelps
CHEMICAL THERMODYNAMICS 457
Heat Capacity Hess’s Law of Constant Heat
Heat capacity (C) of a system is defined as the amount of Summation
heat required to raise the temperature of the system by1° C.
Total enthalpy change during the complete course of
q reaction in a single step and several steps are equal.
C=
DT ∆H (Direct step)
A B
If the system consists of a single substance or a solution and
weighs 1 mole, the heat capacity of the system is known as ∆H1 ∆H3
molar heat capacity. If the system consists of a single
substance or a solution and weighs 1 g, the heat capacity of C
∆H2
D
the system is known as specific heat of the system.
s L = C ´ m ´ DT D H = D H 1 + D H 2 + D H3
where, m = mass of substance, C = specific heat capacity Applications
and DT = temperature difference (a) In determination of enthalpy of reaction, enthalpy of
formation, enthalpy of transformation etc.
Enthalpies for Different Types of Reactions
(b) In determination of bond energy,
Enthalpy Change of Formation (D f H°) The enthalpy
change during the formation of one mole of the compound D r H = S bond energy of reactants -S bond energy of
from constituent element in their standard state is known products
as enthalpy change of formation.
C(amorphous) + O2 ( g ) ¾® CO2 ( g ); D f H = -97. 6 kcal/mol Second Law of Thermodynamics
Enthalpy of Phase Transition ( Dt H ) The enthalpy change For a spontaneous process of an isolated system, the
during the conversion of one mole of a substance from one entropy change is always positive.
phase to another phase is known as enthalpy of phase DS univ = DS sys + DS surr > O
transition.
S (rhombic) ¾® S (monoclinic); Dt H = -13.14 kJ mol -1
Entropy
It is the property of a system which measures the degree of
Enthalpy of Fusion Change in heat enthalpy when 1 mole randomness in a system. It is a state function, therefore
of a substance is completely converted from solid state into change in entropy depends only on the initial and final
liquid state at its melting or freezing point is known as states. qsys rew
enthalpy of fusion. DS = S final - S initial =
T
H 2O(s ) ¾® H 2O(l ); D fus H - = 6.00 kJ mol -1
1. Entropy change for an ideal gas
Enthalpy of Sublimation (D sub H) The enthalpy change (i) For isothermal process,
during sublimation of 1 mole of solid substance to
V p
gaseous substance directly is known as enthalpy of DS = 2 .303 nR log 2 = 2.303nR log 1
sublimation. V1 p2
D sub H = D fus H + D vap H T2
(ii) For isobaric process, DS = 2 .303n C p log
Enthalpy of Ionisation ( D ion H° ) The enthalpy change T1
during complete conversion of one mole of weak acid or T2
base into its constituent ions is known as enthalpy of (iii) For isochoric process, DS = 2 .303n Cv log
T1
ionisation. It is denoted by D ion H°.
CH3COOH (aq ) s CH3COO- (aq ) + H +(aq ); 2. Entropy change during phase transition
DH
D ion H s = -55. 43 kJ/mol DS =
T

Criteria of Spontaneity : Gibbs Energy


Thermochemistry In most of the chemical reactions, change in enthalpy and
entropy occurs simultaneously.
It is the branch of physical chemistry which is concerned
with energy change between a chemical system and its Thus, a new thermodynamic function is introduced to
surroundings when a change of a phase or a chemical define spontaneity of chemical process known as Gibbs
reaction takes place within the system. It is also termed as free energy (G ).
chemical energetics. It is based on the first law of G = H – TS
thermodynamics. \ D G = D H - TD S

@iitjeehelps
458 SELF STUDY GUIDE BITSAT

This equation is also known as Gibbs Helmholtz equation. Percentage efficiency of fuel
The spontaneity of process is related to Gibbs free energy change as DG
% efficiency = ´ 100
DH
DG = +ve Non-spontaneous process
DG = 0 Equilibrium state
DG = -ve Spontaneous process
Third law of Thermodynamics
This law states that the entropy of a substance
NOTE If DH = + ve, TDS = + ve , TDS > DH becomes zero at absolute zero temperature
Then, DG < 0 hence, reaction becomes spontaneous. (perfectly crystalline state), i.e.
Effect of temperature on spontaneity of process lim S = 0
T ®0
Dr H - Dr S - D rG - Reaction will be The importance of the third law is that it is used in
spontaneous at
calculating absolute entropies of pure substances
- + - All temperature
at different temperatures.
- - - Low temperature
For a solid at T K
- - + High temperature
ST - S0 = DS
+ + + Low temperature
T C p × dT
+ + - High temperature =ò
0 T
+ - + All temperature

Practice Exercise
1. Thermodynamics is not concerned about ……… 7. 1 mole of CO2 gas at 300 K is expanded under
a. energy changes involved in a chemical reaction adiabatic conditions such that its volume becomes
b. the extent to which a chemical reaction proceeds 27 times. What is work done? (g = 1.33 and C V = 6
c. the rate at which a reaction proceeds cal mol -1 for CO2)
d. the feasibility of a chemical reaction
a. 900 cal b. 1000 cal c. 1200 cal d. 1400 cal
2. What is DE for a system that does 500 cal of work on
8. The heat of combustion of benzene determined in
surrounding and 300 cal of heat is absorbed by the
a bomb calorimeter is -870 kcal mol-1 at 298K. The
system?
value of DE for reaction is
a. -200 cal b. -300 cal c. +200 cal d. +300 cal
a. -1740 kcal mol-1 b. + 870 kcal mol-1
3. Flying bird is an example of c. - 870 kcal mol-1 d. +1740 kcal mol-1
a. closed system b. open system
c. isolated system d. microscopic system 9. Enthalpy of combustion of C6H 6(l ) is -3264.64
4. For an adiabatic process, which of the following is kJ/mol. The heat produced by burning 3.9 g of
correct? benzene is
a. pDV = 0 b. q = + W c. Dq = 0 d. DE = q a. -163.23 kJ b. - 326.4 kJ
c. -32.64 kJ d. -3.264 kJ
5. A gas expands, isothermally and reversibly. The work
done by the gas is 10. A molecule with highest bond energy is
a. zero b. maximum a. Br2 b. F2 c. Cl2 d. I2
c. minimum d. cannot be determined
11. For the reaction C + O2 ¾® CO2,
6. Which of the following statement is correct? a. DH > DE b. DH < DE
a. The presence of reacting species in a covered beaker is
c. DH = DE d. None of these
an example of open system
b. There is an exchange of energy as well as matter between 12. Given that DH comb of C(s ), H 2( g ) and CH4 ( g ) are
the system and the surroundings in a closed system
c. The presence of reactants in a closed vessel made up of -394, -284 and -892 kJ/mol respectively. The heat
copper is an example of a closed system of formation of CH4 is
d. The presence of reactants in a thermos flask or any other a. -70 kJ/mol b. -71. 8 kJ/mol
closed insulated vessel is an example of a closed system c. -244 kJ/mol d. -782 kJ/mol

@iitjeehelps
CHEMICAL THERMODYNAMICS 459
13. For which of the following reactions DH is less than 23. If the heat of neutralisation for a strong acid-base
DE ? reaction is –57.1 kJ, what would be the heat released
a. HCl (aq ) + NaOH(aq ) ¾® NaCl (aq ) + H2O (l ) when 350 cm 3 of 0.20 M of a dibasic strong acid is
b. H 2(g ) + I 2(g ) ¾® 2HI (g ) mixed with 650 cm 3 of 0.10 M monoacidic base?
c. C (s ) + O2(g ) ¾® CO2(g ) a. 57.1 kJ b. 3.71 kJ c. –57.1 kJ d. 0.317 kJ
d. N 2(g ) + 3H 2(g ) ¾® 2NH3(g ) 24. If enthalpies of formation of C2H4 ( g ), CO2( g ) and
H 2O(l ) at 250° C and 1 atm pressure be 52, –394 and
14. Enthalpy change, when 1 g water is frozen at 0°C is -286 kJ mol-1 respectively, the enthalpy of combustion
(DH fus = 1.435 kcal mol-1) of C2H4 ( g ) will be
a. 0.0797 kcal b. -0.0797 kcal a. 1412 kJ mol-1 b. -1412 kJ mol-1
c. 1.435 kcal d. -1.435 kcal c. +141. 2 kJ mol-1 d. -141.2 kJ mol-1
15. S rhombic + O2( g ) ¾® SO2( g ), DH = - 297. 5 kJ 25. The bond dissociation energies of gaseous H2, , Cl2 and
S monoclinic + O2( g ) ¾® SO2( g ), DH = - 300 kJ HCl are 104.58 and 103 kcal respectively. The
enthalpy of formation of HCl gas would be
The data can predict that a. -44 kcal b. 44 kcal c. -22 kcal d. 22 kcal
a. rhombic sulphur is yellow in colour
b. monoclinic sulphur has metallic lustre 26. The DHf° for CO2( g ), CO( g ) and H2O( g )
c. monoclinic sulphur is more stable
d. DHtransition of S of S is endothermic are -393.5, - 110.5 and -241.8 kJ mol -1 respectively,
the standard enthalpy change (in kJ) for the reaction,
16. When ammonium chloride is dissolved in water, the CO2( g ) + H2( g ) ¾® CO( g ) + H2O( g ), is
solution becomes cold. The change is a. 524.1 b. 41.2 c. -262 . 5 d. -412
a. endothermic b. exothermic
c. supercooling d. None of these 27. Heat of formation of H2O is -188 kJ/mol and H2O2 is
-286 kJ/mol. The enthalpy change for the reaction
17. If a refrigerator’s door is kept opened, then we get
2H2O2 ¾® 2H2O + O2, is
a. room cooled b. room heated
c. more heat is passed out d. no effect on room a. 196 kJ b. -196 kJ c. 984 kJ d. -984 kJ

18. Internal energy and pressure of a gas of unit volume 28. The enthalpy of dissolution of BaCl2(s ) and
are related as BaCl2 × 2H2O (s ) are -20.6 and 8.8 kJ/mol respectively.
2 E 3 The enthalpy of hydration for,
a. p = E b. p = c. p = E d. p = 2 E
3 2 2 BaCl2(s ) + 2H2O ¾® BaCl2 × 2H2O (s ) , is
19. Latent heat of vaporisation of a liquid at 500 K and a. 29.4 kJ b. -29.4 kJ c. -118
. kJ d. 38.2 kJ
1 atm pressure is 10.0 kcal/mol. What will be the 29. Df U of formation of CH 4 (g) at certain temperature is

change in internal energy ( DE ) of 3 moles of liquid at -393 kJ mol -1. The value of Df H – is
same temperature?
a. zero b. < D U -
a. 30 kcal b. – 54 kcal -
c. 27.0 kcal d. 50 kcal c. >D U d. equal to D U -
20. Water is brought to boil under a pressure of 1.0 atm. 30. Find the entropy change when 2 moles of ideal gas at
When an electric current of 0.50 A from a 12 V 27°C temperature is expanded reversibly from 2 L to
supply is passed for 300 s through a resistance in 20 L.
thermal contact with it, it is found that 0.798 g of a. 92.1 b. 0 c. 4 d. 9.2
water is vaporised. Calculate the molar internal
energy change at boiling point (373.15 K). 31. When a gas expands from 1.5 L to 6.5 L against a
a. 37.5 kJ mol -1 b. 3.75 kJ mol -1 constant pressure of 0.50 atm and during this process,
c. 42.6 kJ mol -1 d. 4.26 kJ mol -1 the gas also absorbs 100 J of heat. The change in
internal energy is
21. Enthalpy of solution of NaOH (s) in water is a. 153.3 J b. 353.3 J c. -153.3 J d. -353.3J
-41.6 kJ mol-1. When NaOH is dissolved in water,
the temperature of water 32. The reaction between cyanamide, NH 2CN(s) and
a. increases b. decreases
oxygen was allowed to complete and DU at 300 K was
c. does not change d. fluctuates indefinitely
observed to be -743 kJ mol -1. The value of DH at
300 K for the combustion reaction
22. The heat of combustion of carbon to CO2 is 3
-393.5 kJ / mol. The heat released upon formation NH2CN (s ) + O2( g ) ¾® N2( g ) + CO2( g ) + H2O (l )
2
of 35.2 g of CO2 from carbon and oxygen gas is would be
a. +315 kJ b. -31. 5 kJ a. - 741.75 kJ mol -1 b. -743 kJ mol -1
c. -315 kJ d. +31. 5 kJ c. -744.25 kJ mol -1 d. -740.5 kJ mol -1

@iitjeehelps
460 SELF STUDY GUIDE BITSAT

33. The following data (s) are given as the standard 39. Use the following data to calculate D latticeH° for
enthalpies of combustion of C(s ), H2 ( g ) and CH4 ( g ) NaBr. D subH - for sodium metal = 108.4 kJmol-1
are -393.5 kJ mol -1, - 285.8 kJ mol -1 and -890.4 kJ
lonisation enthalpy of sodium = 496 kJmol-1
mol -1 respectively at 298 K. The standard enthalpy of
formation of methane [CH4 ( g )] is Electron gain enthalpy of bromine = 325 kJmol-1
a. + 724.42 kJ mol -1 b. +74.7 kJ mol -1 Bond dissociation enthalpy of bromine = 192 kJ mol-1
c. - 114.82 kJ mol -1 d. -194.62 kJ mol -1 Df H - for NaBr (s ) = -360.1 kJ mol-1
a. -735.5 kJ mol-1 b. +735.5 kJ mol-1
34. If for a given substance, melting point is TB and
c. -735.5 J mol-1 d. +735.5 Jmol-1
freezing point is TA then correct variation of entropy
by graph between entropy change and temperature is 40. Find out the standard free energy change at 60°C
TB TB
and at 1 atm if the N2O4 is 50% dissociated.
a. -800.0 kJmol-1 b. +800.0 kJmol-1
a. ∆S b. ∆S c. -789.89 JK –1 mol-1 d. +789. 98 JK –1mol-1
TA TA
41. Moles of an ideal gas expand isothermally and
T T reversibly from pressure of 5 atm to 1 atm at 300 K.
TA TB TA T B Calculate the largest mass that can be lifted through
a height of 1 m by this expansion.
c. d.
a. 4092.76 kg b. 8730.9368 kg
c. 4492.76 kg d. 8170.2344 kg
T T
42. At 27° C, one mole of an ideal gas is compressed
35. Mark out the enthalpy for the formation of carbon isothermally and reversibly from a pressure of 2 atm
1 to 10 atm. The value of DE and q are (R = 2 cal)
monoxide (CO) Given, C(s ) + O2( g ) ® CO( g ) ;
2 a. - 965.84 cal
DH = -393.3 kJ / mol b. - 965.84 cal, - 865.58 cal
1 c. 865.58 cal, - 865.58 cal
CO( g ) + O2( g ) ® CO2( g ); DH = -282 kJ/mol
2 d. 965.84 cal, +865.58 cal
a. 110.5 kJ / mol b. 676.1kJ / mol 43. The enthalpy of vaporisation of water is 186.5J/mol.
c. 282.8 kJ / mol d. 300.0 kJ/mol The entropy of its vaporisation will be
36. In the reaction, Na(s ) ¾® Na( g ) a. 0. 5 JK -1 mol-1 b. 1.0 JK -1 mol-1
c. 1. 5 JK -1 mol-1 d. 2.0 JK -1 mol-1
the enthalpy of atomisation is same as the
a. enthalpy of dissociation b. enthalpy of sublimation 44. The reaction which proceeds in the forward direction is
c. enthalpy of association d. enthalpy of vaporisation a. Fe2O3 + 6HCl ¾® 2FeCl3 + 3H2O
37. Calculate the bond enthalpy of Xe ¾ F bond as given b. NH3 + H 2O + NaCl ¾® NH4Cl + NaOH
in the equation, c. SnCl4 + Hg2Cl2 ¾® SnCl2 + 2HgCl2
XeF4 ( g ) ¾® Xe+ ( g ) + F- ( g ) + F2( g ) + F( g ) d. 2Cul + I2 + 4K + ¾® 2Cu+ + 4KI
Dr H = 292 kcal mol-1 .
45. Ammonium chloride, when dissolved in water, leads
lonisation energy of Xe = 279 kcal mol - to a cooling sensation. The dissolution of ammonium
Bond energy (F ¾ F) = 38 kcal/mol chloride at constant temperature is accompanied by
Electron affinity of F = 85 kcal/mol a. increase in entropy b. decrease in entropy
a. 8.5kcal/mol b. 34 kcal/mol c. no change in entropy d. no change in enthalpy
c. 24 kcal/mol d. None of these 46. At 27°C, the reaction
38. Calculate the resonance energy of N 2O. 15
C6H 6(l ) + O2( g ) ¾® 6CO2( g ) + 3H 2O (l )

Df H of N2O = 82 kJ mol -1 2
proceeds spontaneously because the magnitude of
Bond energy of N == O = 607 kJmol-1 a. DH = T × DS b. DH > T × DS
Bond energy of O == O = 498 kJmol-1 c. DH < T × DS d. DH > 0 and T × DS < 0
Bond energy of N == N = 418 kJmol-1 47. The sign of DG for the process of melting of ice at
273 K and 1 atm pressure is
Bond energy of N ºº N = 946 kJmol-1
-1 a. positive
a. 82 kJ mol b. negative
b. - 88 kJ mol-1 c. neither negative nor positive
c. - 82 kJ mol-1 d. either negative or positive
d. + 88 kJ mol-1

@iitjeehelps
CHEMICAL THERMODYNAMICS 461
48. The initial state A has the temperature TA , U A as 53 DG, in process of melting of ice at -15° C, is
the internal energy of the system. By applying the a. DG = -ve b. DG = + ve
mechanical work, new state B is achieved with the c. DG = 0 d. All of these
temperature TB and having the internal energy UB . 54. Using following data
Given that TB > TA . What is the correct expression
H 2O(l ) [ 373.15 K,1atm] ¾®H2O( g )[ 373.15 K,1atm]; DS1
for the change in internal energy ( DV )?
a. U =U H 2O(s ) [273.15 K,1atm] ¾®H2O(l )[ 273.15 K,1atm]; DS 2
b. U -U Predict which of the following is correct?
c. U -U a. DS1 = DS 2
d. None of the above b. DS1 > DS 2
49. When 1.8 g of steam at the normal boiling point of c. DS1 < DS 2
water is converted into water, at the same d. DS1 may be greater or smaller than DS 2
temperature, enthalpy and entropy changes
respectively will be 55. On the basis of thermochemical equations a, b and c,
[Given, DH vap for water = 40.8 kJ mol-1] find out which of the algebraic relationships given in
option a. to d. is correct?
a. -8.12 kJ, 11.89 JK -1 a. C(graphite) + O 2(g) ¾® CO2(g ); D H = x kJ mol-1
b. 10.25 kJ, 12.95 JK -1
1
c. -4.08 kJ, - 10.93 JK -1 b. C(graphite) + O2(g ) ¾® CO(g ); D H = y kJ mol-1
d. 10.93 kJ, -4.08 JK -1 2
c. CO(g ) + 1 / 2 O2(g ) ¾® CO2(g ); D H = z kJ mol-1
50. When the heat of a reaction at constant pressure is
a. z =x +y
-2.5 ´ 103 cal and entropy change for the reaction
b. x =y -z
is 7.4 cal deg -1, it is predicted that the reaction at
25°C is c. x =y + z
a. reversible d. y = 2z - x
b. spontaneous 56. The heat of atomisation of P4 H4 (g) and PH 3( g ) are 355
c. non-spontaneous
d. irreversible
kcal/mol and 228 kcal/mol respectively. The energy of
P—P bond is
51. What will be the change of entropy Dr S° at 298 K a. 102 kcal mol -1
for the reaction in which urea is formed from NH3 b. 51 kcal mol -1
and CO2? c. 26 kcal mol -1
d. 204 kcal mol -1
2NH 3 ( g ) + CO2( g ) ¾® NH2CONH2(aq ) + H2O(l )
[Given, the standard entropy of NH2CONH2(aq ), 57. In the given equation,
CO2( g ), NH 3 ( g ) and H 2O(l ) are 174.0, 213.7, 4Fe(s ) + 3O2( g ) ¾® 2Fe2O3 (s )
192.3 and 69.9 JK - mol -1 respectively] The entropy change is -549.4 JK -1 mol-1 at 298K.
-1 -1
a. 200 JK mol
(Dr H - = -1648 ´ 103 Jmol-1). The above reactions is
b. – 35.44 JK -1 mol -1
c. – 354.4 JK -1 mol -1 a. spontaneous
d. 425.2 JK -1 mol -1 b. non-spontaneous
c. Both (a) and (b)
52. For an isomerisation reaction A 3 B , the d. None of the above
temperature dependence of equilibrium constant is
2000 58. Find out the entropy change in surroundings when
given by loge K = 4.0 - . The value of DS° at 1 mole of H 2O(l ) is formed under standard conditions
T
Df H - = - 286 kJ mol-1
300 K is therefore,
a. 4 R a. 959.7 JK -1 mol-1
b. 5 R b. 286 JK -1 mol-1
c. – 959.7 JK -1 mol-1
c. 400 R
d. -286 JK -1 mol-1
d. 2000 R

@iitjeehelps
BITSAT Archives
1. A swimmer coming out from a pool is covered with a 9. Internal energy does not include [2009]
film of water weighing about 18 g. Calculate the a. vibrational energy
internal energy of vaporisation at 100°C. [2014] b. rotational energy
-1 c. energy arising by gravitational pull
[D vapH° for water at 373 K = 40.66 kJ mol ]
d. nuclear energy
a. 35.67 kJmol-1 b. 37.56 kJmol-1
c. 36.57 kJmol-1 d. 38.75 kJmol-1 10. A reaction has both DH and DS negative. The rate of
reaction [2009]
2. The heat of combustion of sucrose, C12H 22O11(s ) at a. increases with increase of temperature
constant volume is 1348.9 kcal mol-1 at 25°C, then b. increases with decrease of temperature
the heat of reaction at constant pressure when steam c. remains unaffected by change of temperature
is produced is [2014] d. cannot be predicted for change in temperature
a. - 1348.9 kcal b. - 1342.34 kcal 11. One mole of an anhydrous salt AB dissolves in water
c. + 1250 kcal d. None of these with the evolution of 21.0 J mol-1 of heat. If the heat of
hydration of AB is - 29.4 J mol-1, then the heat of
3. At constant temperature and pressure which one of dissociation of hydrated salt AB is [2009]
the following statements is correct for the reaction?
a. 50.4 Jmol-1 b. 8.4 Jmol-1
1
CO( g ) + O2( g ) ¾® CO2( g ) [2013] c. - 50.4 Jmol-1 d. - 8.4 Jmol-1
2
a. DH = DE 12. Calculate the DH in kJ for the following reaction,
b. DH < DE C( g ) + O2( g ) ¾® CO2( g ) [2008]
c. DH > DE Given that,
d. DH is independent of physical state of reactant H2O( g ) + C( g ) ¾® CO( g ) + H2( g ); DH = + 131kJ
1
4. The equilibrium constant K p for the reaction, CO( g ) + O2( g ) ¾® CO2( g ); DH = - 282 kJ
2
N2( g ) + 3H2( g ) -2NH (g ) 1
3 H2( g ) + O2( g ) ¾® H2O( g ); DH = - 242 kJ
-4 -2 2
1.6 ´ 10 (atm) at 400°C. What will be the
equilibrium constant at 500°C, if heat of the reaction in a. - 393 b. + 393 c. + 655 d. - 655
this temperature range is - 25.14 kcal? [2013] 13. For a reversible reaction A B, E
-
a. 1.231 ´ 10-4 (atm)-2 b. 1.876 ´ 10-7 (atm)-2 which one of the following
c. 1.462 ´ 10-5 (atm)-2 d. 3.462 ´ 10-5 (atm)-2 statements is wrong from the given B
A
5. Given that energy profile diagram? [2008]
Reaction coordinate
H2O(l ) ¾® H + (aq ) + OH - (aq ); DH = 57.32 kJ
a. Activation energy of forward reaction is greater than
1 backward reaction
H2( g ) + O2( g ) ¾® H2O(l ); DH = - 286.02 kJ
2 b. The forward reaction is endothermic
Then, calculate the enthalpy of formation of OH - at c. The threshold energy is less than that of activation
25°C. [2013] energy
a. - 228.8 kJ b. - 343.52 kJ d. The energy of activation of forward reaction is equal to
the sum of heat of reaction and the energy of activation
c. + 228.8 kJ d. + 343.52 kJ of backward reaction
6. Calculate the amount of heat evolved when 500 cm 3 14. A 27°C one mole of an ideal gas is compressed
of 0.1 M HCl is mixed with 200 cm 3 of 0.2 M NaOH. isothermally and reversibly from a pressure of 2 atm
[2013] to 10 atm. The value of DE and q are (R = 2 cal) [2007]
a. 57.3 kJ b. 2.865 kJ a. 0, - 965.84 cal
c. 2.292 kJ d. 0.573 kJ
b. - 965.58 cal, - 865.58 cal
7. The mutual heat of neutralisation of 40 g NaOH and c. + 865.58 cal, - 865.58 cal
60 g CH3 COOH will be [2011] d. + 965.84 cal, + 865.58 cal
a. 57.1 kJ b. less than 57.1 kJ
c. more than 57.1 kJ d. 13.7 kJ 15. Gibbs free energy G, enthalpy H and entropy S are
interrelated as in [2007]
8. For the dissociation reaction, [2010] a. G = H + TS
H 2( g ) - 2H( g ) ; DH = 162 kcal b. G = H - TS
heat of atomisation of H is c. G - TS = H
a. 81 kcal b. 162 kcal c. 208 kcal d. 218 kcal d. G = S = H

@iitjeehelps
Answer with Solutions
Practice Exercise 17. (b) The room gets heated because heat is released to
the surroundings.
1. (c) Thermodynamics is not concerned with the rate at 3 3 2
which a reaction proceeds. 18. (a) Internal energy, E = RT = p Þ p = E
2 2 3
2. (a) DE = q + W = 300 + ( - 500) = - 200 cal 19. (c) DH = DE + Dn RT , 30 = DE + 3 ´ 2 ´ 500 ´ 10-3
3. (b) Flying bird gives up energy and takes in air (O2 ). DE = 27 kcal
4. (c) In adiabatic process, heat is not exchanged with 20. (a) DH = work done = i ´ V ´ t = 0.50 A ´ 12 V ´ 300 s
surroundings, hence Dq = 0
= 1800 J = + 1.8 kJ
5. (b) Work done, during isothermal reversible expansion of a Molar enthalpy of vaporisation,
gas, is maximum. DH DH 1.8 kJ
6. (c) The presence of reactants in a closed vessel madeup DH = = = = 40.6 kJ mol-1
Moles of H2O n H2O 0.798
of conducting material. e.g. copper or steel is an example of
18
a closed system.
g -1 1/ 3 DH = DE + p DV
T2 æ V1 ö æ 1ö 1
7. (c) For adiabatic condition, =ç ÷ =ç ÷ = DH = DE + Dn RT [Q Dn = 1]
T1 èV2 ø è 27 ø 3
DH = DE + RT
1
\ T2 = 300 ´ = 100 K Molar internal energy change, DE = DH - RT
3
= 40.6 - 8.314 ´ 10-3 ´ 373.15 = 37.5 kJ mol-1
Thus, T2 < T1, hence cooling takes place due to expansion
under adiabatic condition. 21. (a) Since, the process is exothermic, heat is evolved
\ W = - DE = - C (T2 - T1) [ DE = ( - )ve, expansion] due to this temperature of water increases.
22. (c) C + O2 ¾® CO2; DH = - 393.5 kJ / mol
= - 6 (100 - 300) = 1200 cal
Q Heat released during the formation of 44 g of CO2
8. (c) The heat change determined in a bomb calorimeter is
DE, value. Thus, DE = – 870 kcal mol–1 = - 393.5 kJ
9. (a) Since, heat evolved when 78 g benzene is burnt Heat released during the formation of 35.2 g (given) of
= - 3264.6 kJ. 393.5 ´ 35.2
CO2 = - = - 315 kJ
Therefore, heat produced, by burning 3.9 g benzene 44
3 .9 23. (b) Millimoles of dibasic strong acid
= ´ ( -3264.6) = - 163.23 kJ
78 = M ´ V = 0.20 ´ 350 = 70 mmol
10. (c) The bond energy of Cl2 is the highest among the given. \Amount of H + ions in the acid = 2 ´ 70 = 140 mmol
11. (c) For the reaction, Dn = 1 - 1 = 0 Similarly, amount of OH- ions in monobasic strong base
\ DH = DE = 0.10 ´ 650 ´ = 65 mmol
[Here, OH - is the limiting reactant]
12. (a) The formation of CH4 is represented as:
\1 mole of OH- ions produces = 57.1 kJ heat
C (s ) + 2H2(g ) ¾® CH4 (g );
DH = ( DHcomb) - ( DHcomb) 65 ´ 10-3 mole of OH- ions will produce

= [ - 394 + 2 ( - 284) - ( - 892)] kJ = 57.1 ´ 65 ´ 10-3 = 3.71 kJ

= - 962 + 892 = - 70 kJ mol -1 24. (b) C2H4 + 3O2 ¾® 2CO2 + 2H2O

13. (d) For N2(g ) + 3H2(g ) ¾® 2NH3(g ) DH reaction = [2 ´ DH°f (CO2 ) + 2 ´ DH°f (H2O)]

Hence, Dn = -2 - [ DH° (C 2H 4 ) + 3 ´ DH° (O 2 )]


\ DH < DE = [ 2( -394) + 2( -286)] - [52 + 0] = -1412 kJ mol - 1
1.435
14. (b) DH = - kcal g–1 1 1
18 25. (c) H2 + Cl2 ¾® HCl
2 2
= - 0.0797 kcal g–1
é1 1 ù
DH = ê BE(H2 ) + BE(Cl2 )ú - BE(HCl)
15. (d) Subtracting second equation from first, we get ë2 2 û
S( ) ¾® S( ) DH = + 2.5 kJ ; endothermic
éæ 1 ö æ1 öù
16. (a) The process is endothermic and takes up heat from = ê ç ´ 104÷ + ç ´ 58÷ ú - 103 = -22 kcal
solution so that the solution becomes cold. ëè 2 ø è2 øû

@iitjeehelps
464 SELF STUDY GUIDE BITSAT

26. (b) C + O2 ¾® CO2(g ) ; DH° = -393.5 kJ mol-1 ...(i) 37. (b) D H = Heat supplied - heat evolved
1 292 = (x + 279 ) - ( 38 + 85), x = 34 kcal mol-1
C + O2 ¾® CO (g ) ; DH° = -110.5 kJ mol - 1
...(ii)
2 1
1 38. (b) N ºº N(g ) + O2(g ) ¾® N == N == O
H2 + O2 ¾® H2O (g ) ; DH° = -241.8 kJ mol-1 ...(iii) 2
2
Calculated value of D H È =
Eq. (ii) + Eq. (iii) - Eq. (i)
é 1 ù
CO2(g ) + H2(g ) ¾® CO(g ) + H2O(g ) ; DH = 41.2 kJ D H - = ê BE(N ºº N) + BE(O == O)ú
1 ë 2 û
27. (a) H2 + O2 ¾® H2O; DH = -188 kJ mol-1 ...(i)
2 - [BE(N == N) + BE(N == O)]
H2 + O2 ¾® H2O2; DH = -286 kJ mol-1 ...(ii) é 1 ù
= ê946 + (498)ú - [418 + 607] = 170 kJ mol-1
Multiply Eqs. (i) and (ii) by 2, ë 2 û
2H2 + O2 ¾® 2H2O; DH = -376 kJ mol-1 ...(iii) Resonance energy = Observed D H È - Calculated D H È
2H2 + 2O2 ¾® 2H2O2; DH = -572 kJ mol-1 ...(iv) = 82 - 170 = - 88 kJmol-1
Eq. (iii) - Eq. (iv) 39. (b) By applying Hess’s law,
2H2O2 ¾® 2H2O + O2; DH = +196 kJ D H È = D subH ° + IE + D dissH ° + D eq H ° + U
28. (b) BaCl2(s ) + aq ¾® BaCl2(aq ); DH = -20.6 kJ ...(i) - 360.1 = 108.4 + 496 + 96 + ( - 325) + U
BaCl2 × 2H2O(s ) + aq ¾® BaCl2(aq ); DH = +8.8 kJ ...(ii) U = + 735.5 kJmol-1
Eq. (i) can be split as
40. (c) N2O4(g ) - 2NO2(g )
BaCl2(s ) + 2H2O(l ) ¾® BaCl2 × 2H2O(s ); DH = H1
N2O4 is 50% dissociated, the mole fraction of the
BaCl2 × 2H2O(s ) + aq ¾® BaCl2(aq ); substances is given by
DH = H1 + H2 = -20.6 ; H1 = 8.8 kJ 1 - 0.5 2 ´ 0. 5
c N2O4 = , c NO2 =
H2 = -20.6 - 8.8 = -29.4 kJ 1 + 0.5 1 + 0 .5
29. (b) D H - = D U + pDV . So, D H - < D U - 0. 5 1
V 20 p N2O4 = ´ 1atm, p NO2 ´ 1atm
30. (d) DS = 2 . 303 nR log 2 = 2.303 ´ 2 ´ 2 ´ log = 9. 2 1. 5 1. 5
V1 2
The equilibrium constant K is given by
31. (c) ( -W ) = p ext DV = 0. 5 (6.5 - 1.5 )
( p NO2 )2 1. 5 1
= 2.5 L- atm = 2.5 ´ 101.3J = 253.25J K = = = = 133
.
DU = q + W = 100 - 253.25 = -153.25J p N2O4 (1. 5)2( 0. 5) 0.75
32. (a) DH = DU + ( Dn )RT D G È = - RT ln K
1
= -743 + ´ 8.314 ´ 300 ´ 10- 3 kJ mol -1 = - 8.314 JK -1mol-1 ´ 333 K ´ 2.303 ´ 0.1239
2
= - 789 . 89 JK -1 mol-1
= -741.75 kJ mol -1
41. (d)Wexp = - 2.303 nRT log p1 / p 2
33. (b) C(s ) + 2 H2(g ) ¾® CH4(g )
-393.5 5
-285.8
-
-890.4
- -
= - 2.303 ´ 20 ´ 8.314 JK -1mol-1 ´ 300 K log
D H( CH4(g )) = D H ( CH4 ) - D H ( C) - D H ( H2(g )) ´ 2 1
= -890.4 - ( -393.5) - ( -285.8 ´ 2) = 80.15 ´ 103 J
= +74.7 kJ/mol Work done = mgh
34. (a) For a pure substance,T andT represent the same 80.15 ´ 103 = m ´ 9.81ms-2 ´ 1 m
temperature. Hence, A is correct choice. 80.15 ´ 103 kgm2s-2
m= = 8.17023 ´ 103
1
35. (a) C(s ) + O2(g ) ¾® CO(g ) 9.81 ms-2 ´ 1 m
2 = 8170. 2344 kg
The above equation can be obtained by subtraction of
42. (a) Isothermally and reversibly work
1
CO(g ) + O2(g ) ¾® CO2(g ); DH = - 282.8 kJ mol-1 W = 2.303nRT log( p 2 / p1)
2
= 2.303 ´ 1 ´ 2 ´ 300 log (10 / 2) = 965.84
from C(s ) + O2(g ) ¾® CO(g ); DH = - 393.3 kJ mol-1
At constant temperature, DE = 0, q = - W = - 965.84 cal
DH = [ + 393.3 - ( 282.8)] = 110.5 kJ mol-1 q 186.5
43. (a) DS = rev = = 0.5 JK –1 mol-1
36. (b) Na(s ) ¾® Na(g ), D H È = 108.4 kJ mol-1 T 373
The enthalpy of atomisation is same as the enthalpy of 44. (d) In (d), iodine acts as oxidising agent, so this reaction
sublimation. proceeds in forward direction.

@iitjeehelps
CHEMICAL THERMODYNAMICS 465
45. (a) In dissolution, solid changes to ions in solution, which 55. (c) C + O2 ¾® CO2(g ); D H = x kJ mol -1 ...(i)
have more randomness. 1
C (graphite) + O2(g ) ¾® CO(g ); D H = y kJ mol -1 …(ii)
46. (b) In the reaction, DS = - ve 2
Hence, for the reaction to be spontaneous, DH should be 1
CO(g ) + O2(g ) ¾® CO2(g ); D H = z kJ mol -1 …(iii)
negative and higher thanT × DS i.e. DH > T × DS 2
47. (c) At 273 K, water and ice are in equilibrium, hence, (i) = (ii) + (iii)
DG = 0. Þ x= y + z
48. (b) The change in the internal energy is DU = U - U . 228
56. (b) P¾ H bond energy = = 76 kcal mol -1 For P4H4
3
49. (c) DHcondensation for 1.8 g of steam
bond energy of 4( P ¾ H) + ( P ¾ P)
æ 1.8 ö = 355 kcal mol -1
= ( -40.8) kJ ´ ç ÷ mol = - 4.08 kJ
è 18 ø
Þ 4 ´ 76 + ( P ¾ P) = 355 kcal mol -1
DH -4.08 ´ 103J \( P ¾ P) bond energy = 51kcal mol -1
DS = = = - 10 .93 JK -1
T 373.15 K
57. (a) At temperature,T entropy change of the surroundings
50. (b) Heat at constant pressure means enthalpy, i.e. is
we have, DG = DH - TDS D HÈ
DS surr = - (at constant pressure)
= -2.5 ´ 103 - 298 ´ 7.4 = - 4705 cal T
Hence, the process is spontaneous. - ( - 1648 ´ 103 Jmol-1)
=
51. (c) For the given change 298 K
= 5530 JK -1mol-1
D S ° = Sn S ° - Sn S °
D S total = 5530 + ( - 549.4)
°
= S NH + S H° 2O - [ 2 ´ S NH
° + S °CO2 ]
2CONH2 3 = 4980.6 JK -1mol-1
= 174.0 + 69. 9 - [ 2 ´ 192.3 + 213.7] Hence, the above reaction is spontaneous.
= - 354.4 JK -1 mol-1 1
58. (a) H2(g ) + O2(g ) ¾® H2O(l )
52. (a) Variation of K with temperature is given by 2
DS ° DH ° 2000 D H È = - 286 kJmol -1
log K = - = 4.0 -
R RT T When 1 mole of H2O(l ) is formed, 286 kJ of heat is
DS ° released. This heat is absorbed by the surrounding,
On comparing, = 4 or DS ° = 4 R
R i.e. q surr = + 286 kJ mol-1
53. (b) Because melting of ice at -15°C is a non-spontaneous q surr 286 kJmol-1
process. DS = =
T 298 K
54. (b) Vaporisation of water creates more randomness.
= 959.7 JK -1mol-1
Hence, has more entropy.

BITSAT Archives
1. (b) We can represent the process of evaporation as 2. (b) The combustion equation of sucrose is
Vaporisation
18 g H2O(l ) ¾ ¾¾¾¾® 18 g H2O(g ) C12H22O11(s ) + 12O2(g ) ¾® 12CO2(g ) + 11H2O(g )
Here, Dn = 12 + 11 - 12 = 11
Number of moles in 18g H2O (l )
18 g As we know, DH = DE + DnRT
= = 1mol DH = ( - 1348.9 ´ 103 ) + 11 ´ 2 ´ 298 = - 1342.34 kcal
18 g mol-1
D vapU = D vap H - - pDV 3. (b) As we know, DH = DE + Dn RT
1
= D vapH - - Dn RT For the reaction, CO(g ) + O2(g ) ¾® CO2(g )
2
Assume steam behave as an ideal gas æ 1ö 1
Dn = 1 - ç1 + ÷ = -
D vapU = ( 40.66) - (1)( 8.314 ´ 10-3 )( 373) è 2ø 2
= 40.66 - 3.10 1
\ DH = DE - RT
2
= 37.56 kJ mol-1
Hence, DH < DE

@iitjeehelps
466 SELF STUDY GUIDE BITSAT

4. (c) Equilibrium constants at different temperature and 10. (b) DG = DH - TDS


heat of the reaction are related by the equation Negative DH favours the process. Negative DS opposes
K DH° éT2 - T1 ù
2.303 log 2
= the process. If temperature is decreased, opposing factor
ê ú decreases. Hence, rate of reaction increases.
K 1 R ë TT 1 2 û
11. (b) (i) AB (s ) + (aq ) ¾® AB (aq ); DH = - 21J mol-1
log K = - 4.835
K
2
= 1462
. ´ 10-5 (atm)-2 (ii) AB (s ) + x H2O ¾® AB × x H2O (s ); DH = - 29.4 J mol-1
2
Required equation is
5. (a) Consider the formation of H2O
AB × x H2O (s ) + (aq ) ¾® AB(aq ); DH = ?
1
H2(g ) + O2(g ) ¾® H2O(l ); DH = - 286.20 kJ Eq. (i) is equivalent to
2
1 AB(s ) + x H2O ¾® AB × x H2O(s ); DH = DH1
DH = DH [ H2O (l )] - DH [ H2(g )] - DH [O2(g)] AB × xH2O (s ) + (aq ) ¾® AB(aq ); DH = DH2
2
\ DH (H2O(l )) = - 286.20 DH1 + DH2 = - 21
Now consider the ionisation of H2O - 29.4 + DH2 = - 21 or DH2 = 8.4 J mol-1
H2O(l ) ¾® H+ (aq ) + OH- (aq ) ; DH = 57.32 kJ 12. (a) On adding Eqs. (i), (ii) and (iii) we get
DH = DH (H+ (aq ) + DH ( OH- (aq )) - DH (H2O(l)) C(g ) + O2(g ) ¾® CO2(g ); DH = ?
C(g ) + O2(g ) ¾® CO2(g )
57.32 = 0 + DH (OH_, aq ) - ( - 286.20)
DH = (131 - 282 - 242) kJ = - 393 kJ
or DH ( OH -, aq ) = 57.32 - 286.20 e 13. (c) threshold
energy (Et)

(energy of reactant)
= - 228.8 kJ Et
E′a
6. (c) HCl + NaOH ¾® NaCl + H2O
Ep (energy of product)
At t = 0, Ea
E
500 ´ 0.1 200 ´ 0.2 ∆E B energy of the reaction
Number of moles = = ER
1000 1000 A
= 0.05 = 0.04
reaction coordinate
Q During neutralisation of 1 mole of NaOH by 1 mole of
HCl, heat evolved = 57.3 kJ where,
\ To neutralised 0.04 moles of NaOH by 0.04 mole of E = activation energy of forward reaction
NaOH, heat evolved
E ¢ = activation energy of backward reaction
= 57.3 ´ 0.04
= 2.292 kJ The above energy profile diagram shows that E > E ¢
40 The potential energy of the product is greater than that of
7. (b) 40 g NaOH = = 1mol NaOH
40 the reactant, so the reaction is endothermic.
60
60 g CH3COOH = = 1 mol CH3COOH E = E ¢ + DE
60
Since, acetic acid is a weak acid, some of the heat is E = E or E > E ¢
utilised to ionise it. So, enthalpy of neutralisation of 1 mole
14. (a) Isothermally (at constant temperature) and reversibly
of NaOH by 1 mole CH3COOH is less than 57.1 kJ.
work is
Enthalpy of neutralisation of a strong acid by a strong base p 10
is always 57.1 kJ. W = 2.303 nRT log 2 = 2.303 ´ 1 ´ 2 ´ 300 log
p1 2
8. (a) DH = DH(product) - DH(reactant)
= 965.84
162 = 2 ´ DHH - DHH2
at constant temperature, DE = 0
162
DHH = = 81kcal (Q DHH2 = 0) DE = q + W , q = - W = - 965.84 cal
2
9. (c) Thus, internal energy does not include energy due to 15. (b) Gibbs free energy G1, enthalpy H and entropy S are
gravitational pull. interrelated as G = H - TS

@iitjeehelps
8
Chemical and
Ionic Equilibria

Chemical Equilibrium
The state of reversible reactions at which the concentration of the reactants and products do not
change with time, is known as chemical equilibrium.

Law of Chemical Equilibrium


This law states that, the rate at which a chemical reaction proceeds, is proportional to the product of
active masses of reacting substances with each concentration term raised to the power of
stoichiometric coefficient of that species in chemical reaction,
For a reversible reaction, aA + bB 0 cC + dD,
Rate of forward reaction, Rf µ[ A] a [ B ] b = K f [ A] a [ B ] b (law of mass action)
Rate of backward reaction, Rb µ[C ] c [ D] d = K b [C ] c [ D] d
At equilibrium, Rf = Rb
K f [ A] a [ B ] b = K b [C ] c [ D] d
Kf [C ] c [ D] d
=
Kb [ A] a [ B ] b
Kf
= KC = equilibrium constant
Kb

Equilibrium Constants (K p and K C )


For a reaction, aA + bB r cC + dD
[C ] c [ D] d ( pC )c ´ ( pD )d
Equilibrium constants, KC = , Kp =
[ A] a [ B ] b
( pA )a ´ ( pB )b

@iitjeehelps
468 SELF STUDY GUIDE BITSAT

Relation between K p and K C


D ng
Relation between K and DG°
K p = KC ( RT )
Gibbs free energy change and reaction quotient are
R = gas constant, related as
T = temperature in kelvin DG = DG ° + 2 . 303 RT log QC
Dn g = gaseous moles of products - gaseous moles At equilibrium, DG = 0
of reactants
QC = KC
Hence, (i) If Dn g = 0, K p = KC
DG° = -2 .303 RT log KC
(ii) If Dn g = +ve , K p > KC
or DG° = -2.303 RT log K p
(iii) If Dn g = -ve , K p < KC
Significance of DG° are given below :
Units of Equilibrium Constants ● If DG ° < 0, log K > 0 Þ K > 1

● If Dn g = 0, then K p = KC , no units for both KC and K p Therefore, forward reaction is spontaneous.


Dn
● If Dn g > 0, unit of KC is (mol L-1 ) g and that of K p is ● If DG ° > 0, log K < 0 Þ K < 1

Dng Therefore, backward reaction is spontaneous.


(atm) .
Dng ● If DG° = 0, log K = 0 Þ K = 1
● If Dn g < 0, unit of KC is (L mol -1 ) and that of K p is
Dng
Therefore, reaction is at equilibrium.
(atm) .
Characteristics of equilibrium constant Le-Chatelier’s Principle
Equilibrium The Le-Chatelier’s principle states that “a change in any of
Characteristics Reaction
constant the factors that determine the equilibrium conditions of a
system will cause the system to change in such a manner so
If reaction is reversed A +B r C +D K ¢ = 1/ K as to reduce or to counteract the effect of the change”.
If reaction is divided by 1 1 1 1 K¢ = n K This law is applicable to all physical and chemical
A + B r C+ D
factor n n n n n
equilibria.
If reaction is multiplied nA + nB r nC + nD K¢ = Kn
by factor n
Factors Affecting the Equilibrium
If reaction is written in K1
K ¢ = K1 ´K2 ´ Following are the various factors which affect on the state of
A + B r X +Y
n number of steps
¼K n equilibrium :
K2
X +Y r P +Q ¼ (i) Effect of change in concentration If concentration of
Kn
rC +D the reactants are increased or products are removed,
the equilibrium will shift in the forward direction and
vice-versa.
Reaction Quotient (Q) (ii) Effect of change in pressure In a reaction at
It is the ratio of the molar concentration or partial pressure equilibrium, on increasing pressure, equilibrium shifts
of the product species to that of reactant species at any in that direction where number of gaseous molecules
stage in the reaction. decreases and vice-versa.
For a general reaction, (iii) Effect of change in temperature
aA + bB r cC + dD (a) An increase in temperature for exothermic
[C ]c[ D]d pCc . pDd reactions decreases the concentration of products
QC = and Q p = at equilibrium and vice-versa.
[ A]a[ B ]b pAa . pBb
(b) An increase in temperature for endothermic
At any stage of the reaction, reactions increases the concentration of products
(i) if QC > KC , the reaction will proceed in the direction of at equilibrium and vice-versa.
reactants (reverse reaction).
(iv) Effect of catalyst Catalyst increases (or decreases) in
(ii) if QC < KC , the reaction will move in the direction of both forward and backward reactions equally,
the products. therefore equilibrium will be attained in less (or more)
(iii) if QC = KC , reaction mixture is already at equilibrium. time. Catalyst does not affect the equilibrium constant.

@iitjeehelps
CHEMICAL AND IONIC EQUILIBRIA 469
(v) Effect of the addition of inert gas Bronsted-Lowry Concept
(a) Addition of inert gas to an equilibrium (if Dn = 0) According to Bronsted-Lowry concept, an acid is a
has no effect. substance (molecule or ion) that can donate a proton and a
(b) Addition of inert gas to an equilibrium (if Dn ¹ 0) base, is a substance that can accept a proton from an acid.
has no effect, if addition is made at constant e.g.
volume, but addition at constant pressure will + –
CH3COOH + H2O l H3O + CH3COO
favour the direction of reaction where total number
Acid Base Conjugate Conjugate
of moles at equilibrium show an increase. acid base

Weak and Strong Electrolytes Generally, a strong base has a weak conjugate acid and
vice-versa.
and their Ionisation
Weak electrolytes dissociate partially in the solutions and Lewis Acids and Bases Concept
such solutions are poor conductor of electricity, e.g. An acid is a species which accepts an electron pair and base
CH3COOH, H3PO4 , H3BO3 , NH 4OH, HCN etc. which donates an electron pair, e.g.
Strong electrolytes dissociate completely into their ions in
H3 N ·· +BF3 = H3 N ® BF3
solution and such solutions are very good conductor of Base Acid
electricity, e.g. HCl, H 2SO4 , NaOH, KOH, NaCl, KCl etc. +
H + ·
·OH

= H ¬ OH
Separation of an electrolyte into their ions either on fusion
or dissolution is called ionisation or dissociation.
Relative Strength of Acids and Bases
Relative strength of two acids is written as
NaCl( aq ) ¾® Na+ (aq ) + Cl - (aq )
Strength of acid 1(HA1 ) a 1 K a1 / C K a1
= = =
Degree of Ionisation (a) Strength of acid 2(HA2 ) a 2 K a2 / C K a2
Number of moles of electrolyte dissociated as ions
a= where, K a1 and K a2 are the dissociation constants of two
Total number of moles of electrolyte dissolved
acids at the same concentration C while a 1 and a 2 are their
Amount of electrolyte dissociated
or a= degrees of dissociation. Similarly, for equimolar weak bases,
Initial amount
Strength of base 1 ( B 1OH ) K b1
Ostwald’s Dilution Law =
Strength of base 2( B 2OH ) K b2
It states that degree of dissociation of weak electrolyte is
inversely proportional to the square root of concentration. Relative strength of some of the acids are as follows :


1
, a=
k
= kV HClO4 > Hl > HBr > H 2SO4 > HCl > HNO3 > H3O+ > HSO4- >
c c
H3PO4 > HF > CH3COOH > H3CO3 > H 2S > NH 4 > HCN
where, a = degree of dissociation
Relative strength of some of the bases are as follows :
V = volume containing 1 mole of weak electrolyte
KOH > NaOH > Ca(OH)2 > NH 4OH > NH3
k = dissociation constant
> NH 2 × NH 2 > NH 2OH

Various Concepts of Acids and Bases Ionisation of Water


and their Ionisation For an equilibrium,
The various concepts of acids and bases are as follows :
+ –
H2O(l) + H2O(l) l H3O (aq) + OH (aq)
Arrhenius Concept
Acid Base Conjugate Conjugate
According to Arrhenius concept, all substances which give acid base
H + ions when dissolved in water are called acids while
those which ionise in water to furnishOH - are called bases. The dissociation constant is represented by
HA r H+ + A-, BOH r B + + OH - [H3O+ ] [OH - ]
Acid K=
[H 2O]2

@iitjeehelps
470 SELF STUDY GUIDE BITSAT

But[H2O]=1 , therefore K w =[H3O+ ] [OH - ] Salt Hydrolysis and pH of their


where, K w = ionic product of water Solution
The value of K w at 298 K The process of salt hydrolysis is actually the reverse
K w =[H3O+ ] [OH - ] = (1 ´ 10-7 )2 = 1 ´ 10-14 mol 2 L-2 process of neutralisation. The reaction of an anion or cation
with water accompanied by cleavage of O¾ H bond is
Important points related to ionic product of water are : called hydrolysis.
The value of K w is temperature dependent. It increases with Salt hydrolysis affects the pH of the solution.
increase in temperature.
For acidic solutions, [H3O+ ] > [OH - ] Neutral Salts
+ - Salts of strong acids and strong bases (i.e. neutral salts) do
For neutral solutions, [H3O ] = [OH ]
not undergo hydrolysis, e.g. NaCl, CaSO4 etc.
For basic solutions, [H3O+ ] < [OH - ] pH = 7

Acidic Salts
pH Scale
Salt of a strong acid and weak base, e.g. NH 4Cl, is called
pH of a solution is negative logarithm of hydrogen ion acidic salt, which undergoes cationic hydrolysis.
concentration of solution.
pH < 7
1
Thus, pH = - log [H + ] = log For such salts, [H3O+ ] = K h ´ C
[H + ]
Kw
or [H + ] = 10- pH Kh =
Kb
1
pOH Scale pH = 7 - [pK b + log C ]
2
pOH represents the OH - ion concentration.
where, K h = hydrolysis constant
1
\ pOH = - log [OH - ] = log K b = ionisation constant for weak base
[OH - ]
C = molar concentration of salt
or [OH - ] = 10- pOH
and pK w = pH + pOH = 14 at 25°C Basic Salts
Relationship between [H + ], [OH - ] and pH Salt of strong base and weak acid, e.g. NaNO2 , NaCN,
CH3COONa are termed as basic salts.
1
pH of a weak monobasic acid base pH = [ pK a - log C ] Such salts undergo anionic hydrolysis.
2
pH > 7
1
pOH of a weak monoacidic base pOH = [pK b - log C ] Kw
2 For basic salts, [OH - ] = K h ´ C or K h =
Ka
1
pH = 14 - pOH = 14 - [pK b - log C ] 1
2 pH = 7 + [ pK a + log C ]
Total [OH - ] in a mixture of two weak bases 2

[OH - ]= K1C 1 + K 2 C 2 Solubility Product (S)


It is defined as the product of molar concentration of its
Common Ion Effect ions in a concentrated solution, each concentration terms
raised to the power equal to the number of ions produced
It states that if to the solution of a weak electrolyte, a on dissociation of one molecule of electrolyte.
solution of strong electrolyte is added which furnishes an -
ion common to that furnished by the weak electrolyte, the Ax B y r xA+ + yB
S S
ionisation of the weak electrolyte is suppressed.
\ K sp = [ A+ ]x [ B - ] y
e.g. In IInd group of qualitative analysis, H 2 S is passed in the
presence of HCl. This is due to the fact that HCl suppresses \ K sp = S x + y

the ionisation of weakly dissociated H 2 S. Due to this, only (x + y)


or S= K sp
sulphides of II group radicals are precipitated. Sulphides
of III, IV etc., groups are not precipitated because of their where, S = solubility, K sp = solubility product
high solubility product.

@iitjeehelps
CHEMICAL AND IONIC EQUILIBRIA 471

Buffer Solution Buffer capacity is maximum when

The solution, which maintains its pH constant or reserve (a) [ salt ] = [acid ], pH = pKa for acidic buffer
acidic or basic nature even upon addition of small amounts (b) [ salt ] = [ base], pH = pK b for basic buffer
of acid or base, is called buffer solution. The ability of buffer
Greater the buffer capacity, larger is its capacity to resist the
solution to resist changes in pH on addition of acid or base
change in pH value.
is called buffer action.
Acidic buffer [HCN + NaCN]
Types of buffer (pH < 7) Acid-Base Titration
solutions Basic buffer [NH4OH + NH4Cl] Acid-Base Indicators
(pH > 7)
An acid-base indicator is a substance which possess one
colour in acidic solution and altogether different colours in
Henderson’s Equation for alkaline medium, i.e. its colour changes with pH.
Buffer Solution
Ostwald’s Theory
(i) For acidic buffer,
[salt] According to this theory :
pH = pK a + log ● Indicators are either weak acid or weak base.
[acid]
● Their unionised molecules possess different colour from
(ii) For basic buffer,
those of the ions which they give in the solution.
[salt]
pOH = pK b + log ● An acidic indicator yields a coloured anion while a basic
[base] indicator yields a coloured cation in solution.
● Since, they are weak electrolytes, they are not sufficiently
Buffer Capacity
ionised in solution. But in the presence of strong acid or
Buffer capacity
alkali, their degree of ionisation is considerably increased
Number of moles of acid / base added and they produce a large number of coloured ions.
to 1 L of buffer ● An indicator changes colour when the concentration of
=
Change in pH hydrogen ion (in mol/L) solution is equal to the
ionisation constant of the indicator, i.e. indicator is 50%
dissociated.

Practice Exercise
1. In which of the following reaction, K p and K C are 4. If the equilibrium constant for the reaction
equal? A + 2B R 2 C , is 40 then the equilibrium constant
1
a. N2(g ) + 3H2(g ) R 2NH3 (g ) for reaction C R B + A will be
b. 2SO2 (g ) + O2 (g ) R 2 SO3 (g ) 2
a. 1/ 40 b. 1/ 40 c. (1/ 40)2 d. 40
c. N2 (g ) + O2 (g ) R 2 NO (g )
d. 2 NO (g ) + O2 (g ) R 2NO2 (g ) 5. At a certain temperature in 2HI R H2 + I2, only 50%

2. The number of gram molecules of a substance HI is dissociated at equilibrium. The equilibrium


present in unit volume is termed as constant is
a. activity a. 1.0 b. 0.25 c. 3.0 d. 0.50
b. normal solution - d ( N 2O 4 )
c. molal solution 6. For the reaction, 2N2O4 R 4 NO2, =K
dt
d. active mass d ¢ ( NO2 )
and = K ¢, then
3. The active mass 10 g of CaCO3 is dt
a. 1 b. 0.1 a. K ¢ = 2 K b. K ¢ = K
c. 10 d. 100 c. 2K ¢ = K d. None of these

@iitjeehelps
472 SELF STUDY GUIDE BITSAT

7. The equilibrium constant K for the reaction 15. The ratio of K p /K C for the reaction
2HI ( g ) R H2 ( g ) +I2 ( g ), at room temperature, is 1
2.85 and that at 698 K is 1. 4 ´ 10-2. This implies that CO (g) +
2
O 2( g ) R CO2( g ) , is
the forward reaction is a. 1 b. RT c. (RT )1/ 2 d. (RT )–1/ 2
a. exothermic
b. endothermic 16. The equilibrium constant K p for the reaction
c. exergonic A R 2B, is related to degree of dissociation a of A
d. unpredictable and total pressure p as
8. When pressure is applied to the equilibrium system 4 a2p 4 a2p 2
a. b.
Ice R water, which of the following phenomenon will 1– a 2 1– a 2
happen? 4 a2p 2 4 a2p
a. More ice will be formed c. d.
1– a 1– a
b. Water will evaporate
c. Equilibrium will not be established 17. At 30°C, K p for the dissociation reaction
d. More water will be formed SO2Cl2(g ) R SO2( g ) + Cl2 is 2.9 ´ 10–2 atm. If the
9. For N 2 + 3H 2 R 2NH 3 , 1 mole of N 2 and 3 moles of total pressure is 1 atm, the degree of dissociation of
H 2 are at 4 atm. Equilibrium pressure is found to be SO2Cl2 is (assume, 1 – a 2 = 1)
3 atm. The value of K p is a. 87% b. 13% c. 0.17% d. 29%
1 1
a. b. 18. XY2 dissociates as XY2( g ) R XY ( g ) + Y ( g ).
(0.5) (0.15)3 (0.5) (1.5)3
3´ 3 When the initial pressure of XY2 is 600 mm Hg, the
c. d. None of these total equilibrium pressure is 800 mm Hg. Calculate K
(0.5) (1.5)3
for the reaction assuming that the volume of the
10. The unit of equilibrium constant, (K ) for the reaction system remains unchanged.
A+B R C , would be a. 50 b. 100
a. mol L- 1 b. mol L c. 166.6 d. 400.0
1
c. mol- 1 L d. 19. For these reactions,
mol L
[HI] 2
11. In any chemical reaction, equilibrium is supposed to I. H2 ( g ) + I2 R 2HI( g ); K C =
[H2] [I 2]
be attain when
a. mutual opposite reactions undergo [H 2] [I 2]
II. 2HI ( g ) R H2( g )+ I2 ( g ); K C =
b. velocity of mutual reactions become equal [HI] 2
c. concentration of reactants and resulting products are
equal K C and K ¢C are related as
d. the temperature of mutual opposite reactions a. K = KC ´ K ¢C b. K = KC + K ¢C
become equal 1
c. K = K ¢C - KC d. K ¢C =
12. When 3 moles of ethyl alcohol are mixed with 3 moles KC
of acetic acid, 2 moles of ester are formed at 20. For the reversible reaction,
equilibrium according to the equation N2 ( g ) + 3 H2 ( g ) R 2NH3 ( g ) at 500°C, the value of
CH 3 COOH (l ) +C2H5 OH(l ) R K p is 1.44 ´ 10-5 . When partial pressure is measured
CH3 COOC 2H5 (l ) + H2 O(l ) in atmospheres, the corresponding value of K C with
concentration in mol L-1 is
The value of the equilibrium constant for the reaction
is a.1.44 ´ 10-5 / ( 0.082 ´ 500)-2
a. 4 b. 2/9 c. 2 d. 4/9 b.1.44 ´ 10-5 / ( 8.314 ´ 773)-2
c.1.44 ´ 10-5 / ( 0.082 ´ 773)2
13. The reaction 2SO2 ( g ) + O2 ( g ) R 2SO3 ( g ), is
. ´ 10-5 / ( 0.082 ´ 773)-2
d. 144
carried out in 1 dm 3 and 2 dm 3 vessels respectively.
The ratio of the reaction velocities will be 21. PCl 5, PCl3 and Cl2 are at equilibrium at 500 K and
a. 1 : 4 b. 4 : 1 c. 1 : 8 d. 8 : 1 having concentration 1.59 M , 1.59 M and
14. The equilibrium constant for the reaction 1.41 M respectively . Calculate the K C for the following
H2 ( g ) + I2 ( g ) R 2HI( g ), is 64. If the volume of the reaction :
container is reduced to half of the original volume, the PCl 5 R PCl3 + Cl2
value of the equilibrium constant will be a. 2.79 b. 3.75
a. 16 b. 32 c. 64 d. 128 c. 4.57 d. 1.79

@iitjeehelps
CHEMICAL AND IONIC EQUILIBRIA 473
22. What is the relation between K p and K C for a general 32. For the reaction NO2 ( g ) R 2NO2 ( g ), the degree of
reaction, aA + bB R cC + dD ? dissociation at equilibrium is 0.2 at 1 atmospheric
a. K p = KC (RT )Dn b. KC = K p (RT )Dn pressure. The equilibrium constant K p will be
1 1 a. 1/2 b. 1/4
c. K p = d. KC = c. 1/6 d. 1/8
KC (RT )Dn K p (RT )Dn
33. If a 0.1 M solution of HCN is 0.01% ionised, the
23. Consider the following reactionin a closed container ionisation constant for HCN is
at equilibrium. CaCO3 (s ) R CaO(s) + CO2 ( g ) a. 10-9 b. 10-7
What would be the effect of addition of CaCO3 on the c. 10-5 d. 10-3
equilibrium concentration of CO2?
34. An aqueous solution contains the following ions:
a. Increases
b. Decreases Hg2+ 2+
2 , Hg , Pb
2+
and Cd 2+
c. Data is not sufficient to predict it On adding HCl, which of these will precipitate?
d. Remains unaffected a. Hg2Cl2 and PbCl2 b. Only Hg2Cl2
24. According to Le-Chatelier’s principle, if heat is given to c. Only PbCl2 d. PbCl2 and HgCl2
solid-liquid system, then
a. quantity of solid will reduce 35. What is correct about ionic product of an electrolyte?
b. quantity of solid will increase a. It is always equal to its solubility product
c. temperature will increase b. It can be less than or equal to K sp
d. temperature will decrease c. It is always less than K sp
25. Which of the following will not change the d. It can be less than or equal to or greater than K sp
concentration of ammonia in the equilibrium?
36. A solution was prepared by mixing 50 mL of 0.2 M HCl
4NH3 ( g ) + 5O2( g ) R 4 NO( g ) + 6 H2O2(l ); DH = +ve and 50 mL 0.10 M NaOH. The pH of the solution is
a. Increase of pressure a. 7.0 b. 2.0
b. Increase of volume c. 3.0 d. 1.3
c. Addition of catalyst
d. Decrease of temperature 37. What is [ H+ ] in mol/L of a solution that is 0.20 M in
. M CH 3 COOH ?
CH 3 COONa and 010
26. The Bronsted acid which gives the weakest conjugate
base, is K a for CH3 COOH= 1.8 ´ 10-5
a. HF b. H2S a. 9.0 ´ 10-6 b. 3.5 ´ 10-4
c. HCl d. H2O c. 1.1´ 10-5 d. 1.8 ´ 10-5
38. A week acid, HA, has a K a of 1.00 ´10-5 . If 0.100
27. Which one of the following is the strongest Lewis
acid? mole of this acid is dissolving in 1 L of water, the
a. BF3 b. BCl3 c. BBr3 d. BI3 percentage of acid dissociated at equilibrium is
closed to
28. Arrange the following acids in decreasing order of a. 99.0% b. 1.00%
acidity : c. 99.9% d. 0.100%
(I) H2SO3 , (II) H3 PO3 and (III) HClO3 39. How many gram equivalents of NaOH are required to
a. I > III > I b. I > II > III neutralise 25 cm 3 of a decinormal HCl solution?
c. III > I > II d. II > III > I a. 0.00125 b. 0.0025
29. Among the given anions, the strongest Bronsted base c. 0.0050 d. 0.025
is 40. The first and second dissociation constants of an acid
a. ClO- b. ClO-2 c. ClO-3 d. ClO-4 H2A are 1.0 ´10-5 and 5.0 ´10-10 respectively. The
30. What will be the conjugate bases for the following overall dissociation constant of the acid will be
Bronsted acids? a. 5.0 ´ 105 b. 5.0 ´ 10-5
c. 5.0 ´ 1015 d. 5.0 ´ 10-15
HF, H2SO4 and HCO-3
- 41. The pH of 0.1 M solution of a weak acid is 3. What is
a. F , and CO2-
SO2- b. F -, SO2-
4 and H2 CO3
- -
4 3 the value of the ionisation constant for the acid?
c. F , HSO4 and H2CO3 d. F -, HSO-4 and CO2-
3 a. 0.1 b. 10-3
c. 10-5 d. 10-7
31. The vapour density of N2O4 at a certain temperature is
30. What is the percentage dissociation of N2O4 at this 42. What will be the pH at which an acid indicator with
temperature? K a = 1 ´ 10-5 changes colour when the indicator
a. 53.3% b. 106.6% concentration is 1 ´ 10-3 M ?
c. 26.7% d. None of these a. 5 b. 7 c. 3 d. 6.5

@iitjeehelps
474 SELF STUDY GUIDE BITSAT

43. The pH of a 10-9 M solution of HCl in water is 52. The pH of buffer solution containing 4 ´ 10-3 and
a. 8 0.4 mole of acetic acid (pK a = 4.76 ) and sodium
b. -8 acetate respectively will be
c. between 7 and 8 a. 6.76 b. 4.76 c. 2.76 d. 0.76
d. between 6 and 7
53. For the pair of reactions given below,
44. The following concentrations were obtained for the
I. N2 ( g ) + 3 H2 ( g ) R 2NH3 ( g )
formation of NH 3 from Na and H2 at equilibrium at
1 3
500 K :[N2] = 1.5 ´ 10-2 M, [H2] = 3.0 ´ 10-2 M and II. N2 ( g ) + H2 ( g ) R NH3 ( g )
2 2
[NH3 ] = 1.2 ´ 10-2 M
If at a particular temperature, K p1 and K p 2 are the
Calculate the equilibrium constant. equilibrium constants for reaction I and II respectively,
a. 1.08 ´ 104 b. 3.98 ´ 102 then
c. 1.06 ´ 103 d. 2.93 ´ 104 a. K p1 = 2K p 2 b. K p1 = K p22
c. 2K p1 = K p 2 d. K p21 = K p 2
45. Which of the following relations represent correct
relation between standard electrode potential and 54. Solubility of BaF2 in a solution of Ba(NO3 )2 will be
equilibrium constant?
-nFE °
represented by the concentration term
nFE ° a. [Ba 2+ ] b. [F - ] 2
I. log K = II. K = e RT
2.303 RT 1
c. [F - ] d. 2[NO-3]
-nFE ° nFE ° 2
III. log K = IV. log K = 0.4342
2.303 RT RT 55. For anionic hydrolysis, pH is given by
a. I, II and III b. II and III 1 1 1
a. pH = pK a - pKb + log C
c. I, II and IV d. I and IV 2 2 2
1 1 1
46. If the ionisation constant of acetic acid is 1.8 ´ 10-5 , at b. pH = pK w + pK a - pKb
2 2 2
what concentration will it be dissociated to 2%? 1 1 1
c. pH = pKw + pK a + log C
a. 1 M b. 0.018 M c. 0.18 M d. 0.045 M 2 2 2
1 1 1
47. The dissociation constant of acetic acid at a given d. pH = - pKw + pK a + pKb
temperature is 1.69 ´ 10-5 . The degree of dissociation 2 2 2
of 0.01 M acetic acid in the presence of 0.01 M HCl is 56. The solubility of AgBrO 3 in an aqueous solution of
equal to NaBrO 3 (as compared to that in water) is
a. 0.41 b. 0.13 a. the same
c. 0.169 ´ 10-2 d. 0.013 b. more
c. less
48. The concentration of hydrogen ion in a sample of soft
d. unpredicted due to a new chemical reaction
drink is 3.8 ´ 10-3 M . What is its pH?
a. 4.32 b. 5.12 c. 3.31 d. 2.42 57. Which of the following salts undergo anionic
hydrolysis?
49. The pK a of acetic acid and pK b of ammonium a. CuSO4 b. NH4Cl
hydroxide are 4.76 and 4.75 respectively. Calculate c. FeCl3 d. Na 2CO3
the pH of ammonium acetate solution.
a. 4.765 b. 5.012 c. 7.005 d. 6.098 58. How do we differentiate between Fe3 + and Cr 3 + in
group III?
50. For a reaction,
a. By taking excess of NH4OH
CH3 COOH (aq ) R H+ (aq ) + CH3 COO- (aq ) b. By increasing NH+4 ion concentration
or HAc (aq ) R H+ (aq )+ Ac - (aq ) c. By decreasing OH- ion concentration
d. Both (a) and (c)
Evaluate the pH of the solution resulting on addition of
0.05 M acetate ion to 0.05 M acid solution 59. What will be the degree of ionisation of 0.05 M acetic
acid if its pK a value is 4.74 ?
(K a = 1.8 ´ 10-5 )
a.0.019% b. 1.9% c. 3.0% d. 4.74%
a. 5.72 b. 3.87 c. 4.24 d. 4.74
60. A certain buffer solution contains equal concentrations
51. pH of an acidic buffer is given by
[salt] [salt] of X - and HX , K b for X - is 10-10. The pH of the buffer
a. pH = pK a + log b. pH = pK a - log solution is
[acid] [acid]
1 [salt] [salt] a. 4 b. 6
c. pH = pK a + log d. pH = logK a + log c. 7 d. 14
2 [acid] [acid]

@iitjeehelps
CHEMICAL AND IONIC EQUILIBRIA 475
61. The buffering action of an acidic buffer is maximum 64. The value of pK w of water
when its pH equals to a. increases with increase in temperature
a. 5 b. 7 b. decreases with rise in temperature
c. 1 d. pK a c. does not change with variation in temperature
d. increases till 50° C and there after decreases
62. The expression for the solubility product of Ag2CrO3
will be 65. At 20°C, Ag+ ion concentration in a saturated solution
2 3
a. K sp = S b. K sp = 4S of Ag2CrO4 is 1.5 ´ 10-4 mol/L. At 20°C, the solubility
4
c. K sp = 27S d. K sp = S product of Ag2CrO4 will be
a.3 . 3750 ´ 10-12 b.1.6875 ´ 10-10
63. Amphoteric behaviour is shown by
c.1.6875 ´ 10-12 d.1.6875 ´ 1011
a. H2CO3 and Al2O3
b. HCO-3 and H2O 66. If the solubility of Ag2CrO4 is S mol/L, its solubility
c. HCO-3 and H3O+ product will be
d. H2CO3 and H2O a. S 2 b. S 3 c. 4S 3 d. 2S 3

BITSAT Archives
1. Calculate the pH at the equivalence point during the 8. The pH of 0.01 M HCN solution for which pK a is 4, is
titration of 0.1 M, 25 mL CH3 COOH with 0.05 M NaOH
[2011]
solution. [K a (CH3 COOH) = 1.8 ´ 10-5 ] [2014] a. 0.47 b. 1.2
a. 9.63 b. 8.63 c. 10.63 d. 11.63 c. 3.0 d. 4.0
2. Gastric juice in human stomach has pH value about 9. If dissociation constant of ammonia is 10-5 , its pK b
1.8 and pH of small intestine is about 7.8. The pK a
and pK a values respectively are [2010]
value of aspirin is 3.5. Aspirin will be [2013]
a. 5 and 9 b. 9 and 5
a. ionised in the small intestine and stomach
c. 7 and 7 d. 4 and 10
b. ionised in the stomach and almost unionised in the
small intestine 10. [H+ ] of 0.2 NCH3 COOH which is 4% dissociated, is
c. unionised in small intestine and stomach
d. completely ionised in small intestine and stomach [2010]
a. 0.08 M b. 0.12 M
3. At 27° C, K p value for the reaction c. 0.008 M d. 0.8 M
CaCO3 (s ) r CaO (s ) + CO2 ( g ), is 0.1 atm. K C 11. 50 mL of H2O is added to 50 mL of 1´ 10-3 M barium
value for this reaction is [2013]
hydroxide solution. What is the pH of the resulting
a. 4 ´ 10-3 b. 6 ´ 10-3 c. 2 ´ 10-3 d. 9 ´ 10-3
solution? [2008]
4. In the titration of NaOH and HCl, which of the a. 3.0 b. 3.3
following indicators will be used? [2012] c. 11.0 d. 11.7
a. Methyl orange b. Methyl red 12. Assertion (A) The aqueous solution of CH 3 COONa is
c. Both (a) and (b) d. None of (a) and (b)
alkaline in nature.
5. For 2NOBr ( g ) R 2NO( g )+ Br2 ( g ) at equilibrium, Reason (R) Acetate ion undergoes anionic hydrolysis.
p Kp
p Br2 = and p is the total pressure, the ratio will [2008]
q p a. Both (A)and (R) are true and (R) is the correct
be [2012] explanation of (A)
1 1 1 1 b. Both (A)and (R) are true and (R) is not the correct
a. b. c. d.
3 9 27 81 explanation of (A)
c. (A) is true but (R) is not true
6. For the reaction N2 ( g )+ 3H 2 ( g ) R 2NH3 ( g ), the unit d. (A) is not true but (R) is true
of K are [2011]
13. In a 500 mL flask, the degree of dissociation of PCl5 at
a. L mol-1 b. L2 mol-2
c. mol L-1 d. No units equilibrium is 40% and the initial amount is 5 moles.
The value of equilibrium constant in mol L-1 for the
7. A sulphuric acid solution has pH = 3. Its normality is decomposition of PCl5 is [2008]
[2011] a. 2.33 b. 2.66
a. 1/1000 b. 1/200 c. 1/2000 d. 1/100 c. 5.32 d. 4.66

@iitjeehelps
476 SELF STUDY GUIDE BITSAT

14. For an equilibrium reaction, N2 O 4 ( g ) R 2NO 2( g ), 16. In which of the following reactions is K p < K C ? [2005]
the concentrations of N2 O 4 and NO 2 at equilibrium are a. I2(g ) r 2I (g )
4. 8 ´ 10-2 and 1. 2 ´ 10-2 mol/L respectively. The b. 2BrCl(g ) r Cl2(g ) + Br2(g )
value of K C for the reaction is [2007] c. CO(g ) + 3H2(g ) r CH4(g ) + H2O(g )
a. 3 ´ 10-3 mol / L b. 3.3 ´ 10-3 mol / L d. All of the above
c. 3 ´ 10-1 mol / L d. 3.3 ´ 10-1 mol / L
17. The change in pressure will not affect the equilibrium
15. In 2HI R H2 + I 2, the forward reaction is not affected constant for [2005]
by change in [2006] a. N2 + 3H2 r 2NH3 b. PCl5 r PCl3 + Cl2
a. catalyst b. pressure c. H2 + I2 r 2HI d. All of these
c. volume d. temperature

Answer with Solutions


Practice Exercise \Total moles = ( 4 - 2x ) = 3
1
1. (c) Only for this reaction, Dng = 2 - 2 = 0 Hence, p NH3 = 3 ´ = 1
3
Dn g 1
Hence, K p = KC (RT ) = KC (RT )0 = KC p N2 = 3 ´ = 0.5
6
2. (d)
1
3. (a) Active mass of a pure sold is always taken unity. p H2 = 3 ´ = 1.5
2
[C] 2 p NH2
4. (a) For A + 2B R 2C, KC = …(i)
[ A] [B ] 2 Kp = 3
p N2 . p H3
1 [B ][ A ]1/ 2 2
For CR B+ A, KC¢ = …(ii) 1
2 [C] =
( 0.5)(1.5)3
On comparing Eqs. (i) and (ii), we get
1 1 [C] mol L-1
KC¢ = = 10. (c) K = =
Kc 40 [ A] [B] mol L-1 ´ mol L-1
5. (b) 2HI r H2 + I2 = (mol L-1)-1 = mol-1 L
At equilibrium (1 – 0.5) 0.25 0.25
11. (b)
[H ] [I ] 0.25 ´ 0.25 0.25 ´ 0.25
\ KC = 2 22 = = = 0.25 12. (a) CH3COOH + C2H5OH r
[HI] (0.5)2 0.25
Initial 3 mol 3 mol
1 d (N2O4 ) 1 d (NO2 ) At equilibrium 1 1
6. (a) Rate = - =
2 dt 4 dt CH3COOC2H5 + H2O
1 1 0 0
or K = K¢ 2 2
2 4 2´2
K= =4
Þ K ¢ = 2K 1´ 1
7. (a) 13. (d) 2SO2 (g ) + O2(g ) r 2SO3 (g )
8. (a) Ice has greater volume than water. Hence, equilibrium
will shift towards right. (lesser volume). For 1 dm3, R = K [SO2] 2 [O2]
2
9. (b) N2 + 3 H2 r 2NH3 é 1ù é 1ù
R =K ê ú ê ú =1
At equilibrium (1- x) ( 3 - 3 x) 2x ë 1û ë 1û
Initial pressure = 4 atm 2
é 1ù é 1ù 1
Total moles at equilibrium = (1- x ) + ( 3 - 3x ) + 2x = 4 - 2x For 2 dm3, R = K ê ú ê ú =
ë 2û ë 2û 8
Equilibrium pressure = 3 atm
So, the ratio is 8 : 1.
n2 p2
= 14. (c) H2 (g ) + l2 (g ) r 2HI (g ) ; Dng = 0
n1 p1
4 - 2x 3 \The reaction and its equilibrium constant are not affected
Þ = Þ x = 0.5 by change in volume.
4 4

@iitjeehelps
CHEMICAL AND IONIC EQUILIBRIA 477
æ 1 ö -1 21. (d) The equilibrium constant KC for the reaction can be
15. (d) Dng = 1- ç1+ ÷ =
è 2ø 2 written as,
-
1
Kp -
1
[PCl3] [Cl2] (1.59)2
\ K p = KC (RT ) 2 or = (RT ) 2 KC = = = 1.79
KC [PCl5] (1.41)

16. (a) A r 2B 22. (a)


Initially 1 0
23. (d) This is a heterogeneous equilibrium.
Hence, equilibrium constant for it is
At equilibrium 1- a 2a
KC = [CO2]
Total moles = 1 + a
Hence, CaCO3 has no effect on equilibrium concentration.
(1- a ) × p 2a
pA = , and pB = ×p 24. (b) A reaction which is in equilibrium will shift in reverse or
1+ a 1+ a
2
backward direction, when we give heat to the solid,
æ 2a ö quantity of solid will increase.
ç × p÷
pB2 è 1+ a ø 4 a2 p 25. (c)
Kp = = =
pA é 1– a ù 1 - a2
êë 1+ a p úû 26. (c)
27. (d) Among these in BI3, p p - p p back -bonding is least
17. (c) SO2Cl2 r SO2+ Cl2 effective as it occurs between 5p - 2p (B) orbitals. Hence,
Initial l y 1 0 0
At equilibrium 1– a a a BI3 is the most acidic.
Total moles = 1+ a 28. (c)
1– a a a 29. (a) ClO- > ClO2- > ClO3- > ClO4-
p SO2Cl2 = , p SO2 = , p Cl2 =
1+ a 1+ a 1+ a
30. (d) F-, HSO4- and CO2-
3 respectively.
2
æ a ö æ 1– a ö a2 31. (a) The reaction is N2O4 r 2NO2.
Kp = ç ÷ ç ÷=
è 1+ a ø è 1+ a ø 1– a 2
Molecular weight of N2O4 = 14 ´ 2 + 16 ´ 4 = 92
= a 2; (1 - a 2 = 1) \Vapour density (D) of N2O4 = 92 / 2 = 46
a = K p = 2.9 ´ 10 –2 = 0.17 D -d 46 - 30 16
a= = =
(n - 1) d ( 2 - 1) 30 30
\Degree of dissociation = 17%
\ a = 0.533 = 53.3 %
18. (b) XY2 r XY + Y
32. (c) N2O4 (g ) r 2NO2 (g )
Initially 600 mm 0 0
At equilibrium 600 – p p p At equilibrium (1- 0.2) = 0.8 2´ 0.2 = 0.4

Total pressure = 600 - p + p + p = 600 + p Total moles at equilibrium = 0.8 + 0.4 = 1.2
or 600 + p = 800 mm Þ p = 200 mm Total pressure = 1atm
and, p due to XY2 = 400 mm 0.8 2
Hence, p N2O4 = =
200 ´ 200 1.2 3
K= = 100 0.4 1
400 and p NO2 = =
1.2 3
19. (d) Equilibrium constant for the reaction,
p NO2 (1/ 3)2
H2 (g ) + I2(g ) r 2HI (g) Kp = =
p N2O4 2/3
[HI] 2
as, KC = =x 1 3 1
[H2] [I2] = ´ =
9 2 6
The equilibrium constant for the reverse reaction,
2HI (g ) r H2 + I2 (g ), at the same temperature is 33. (a) HCN r H+ + CN-
Initially 0.1 0 0
[H] [I] 2 1 1 At equilibrium ( 0.1- a) a a
K ¢C = = =
[HI] 2 x KC [H+ ] [CN- ] a×a
1 \ K= =
Thus, K ¢C = [HCN] ( 0.1- a )
KC
As, a << 0.1
20. (d) Dng = 2 - 4 = - 2 10-5 ´ 10-5 æ 0.01ö
K= ç a = 0.1´ ÷
Kp . ´ 10
144 -5 0.1 è 100 ø
KC = =
(RT )Dn ( 0.082 ´ 773)-2 K = 10-9

@iitjeehelps
478 SELF STUDY GUIDE BITSAT

34. (a) Hg2+2 and Pb


2+
have been placed in Ist group as their 42. (a) An acid indicator dissociates at equilibrium as
K sp are less than the IInd group ions (Hg2+ , Cd 2+ ). HIn r H+ + In-
Hence, only these will precipitate. [H+ ][In- ]
\ KIn =
35. (d) [HIn]
36. (d) 50 mL of 0.1 M NaOH » 25 mL of 0.2 M HCl At the mid point, [In- ] = [HIn]
Since, the resultant solution will have volume = 100 mL \ KIn = [H+ ] = 1 ´ 10-5
25 ´ 0.2
Molarity = = 0.05 \ [H+ ] = 1 ´ 10-5 or pH = 5
100
pH = - log [H+ ] = - log 0.05 = 1. 3010
43. (d) As the solution is acidic, pH < 7. This is because [H+ ]
from H2O [10-7 M] cannot be neglected in comparison to
37. (a) CH3COOH r CH3COO- + H+
x 10-9 M .
c- x x
44. (b) The equilibrium constant for the reaction
[CH3COO ] [H+ ]
-
Ka = N2 (g ) + 3H2 r 2NH3 (g ) can be written as
[CH3COOH]
[NH3 (g )] 2
[CH3COOH] =C - x » C = 0.1 M KC =
[N2 (g)] [H2 (g)]3
[CH3COO-] = 0.2 + x » 0.2 M
(1.2 ´ 10-2 )2
K [CH3COOH] 1.8 ´ 10-5 ´ 0.1 =
[H+ ] = a = (1.5 ´ 10-2 )( 3.0 ´ 10-2 )3
[CH3COO- ] 0.2
= 0.0355 ´ 104 = 3 .55 ´ 102
[H+ ] = 9 ´ 10-6
= 3.98 ´ 102
38. (b) HA r H+ + A -
45. (c) DG = - 2.303 RT log K ; -nFE° = - 2.303 RT log K
[H+ ] [ A -] [H+ ] 2
Ka = = nFE° nFE°
[HA] [HA] log K = = 0.4342 …(v)
2.303 RT RT
[ H+] = K a [ HA] = 1´ 10-5 ´ 0.1
= 1´ 10-3 nFE°
ln K =
RT
Actual ionisation 10-3
a= = = 10-2 -nFE°
Molar concentration 0.1 K = …(ii)
-2
e RT
% of acid dissociated = 10 ´ 100 = 1%
46. (d) CH3COOH r CH3 COO- + H+
39. (b) For acid-base reactions, g-equivalent of acid
(HCl) = g-equivalent of base (NaOH) Initially C 0 0
1 At equilibrium C (1- a ) Ca Ca
\g-equivalent of acid = ( 25 ´ 10-3 ) dm3 ´
10 Ca × Ca Ca 2
K = = = Ca 2
= 0.0025 = g-equivalent of (NaOH) C(1 - a ) 1 - a
40. (d) H2A r H+ + HA - K . ´ 10-5
18
or C= 2
= = 0.045 M
[H+ ] [HA - ] a ( 0.02)2
K1 = = 1.0 ´ 10-5
[H2A] . ´ 10-5
K a 169
47. (c) Or a = = = 0.169 ´ 10-2
HA- r H+ + A2- C 0.01
48. (d) pH = - log [3.8 ´ 10-3] = 2 . 42
[H+ ] [ A 2- ]
K2 = = 5.0 ´ 10-10
[ HA] Therefore, the pH of the soft drink is 2.42 and it is acidic.
- 1 1
[H+ ] 2 [ A 2 ] 49. (c) pH = 7 + [ pK a - pKb ] = 7 + [ 4.76 - 4.75]
K overall = = K1 ´ K 2 2 2
[ H2A]
= 7.005
= 1.0 ´ 10-5 ´ 5.0 ´ 10-10
50. (d) HAc (aq ) r H+(aq ) + Ac- (aq )
= 5.0 ´ 10-15 Initial conc. (M) 0.05 0 0.05
+ - At equilibrium 0.05 - x x 0.05 + x
41. (c) HA r H + A
concentration (M )
[H+ ] = 10-3 M , [ A - ] = 10-3 M
[H+ ] [Ac- ] [(x ) ( 0.05 + x )]
[H+ ] [ A - ] 10-3 ´ 10-3 Ka = =
Hence, Ka = = = 10-5 [HAc] ( 0.05 - x )
[HA] 0.1

@iitjeehelps
CHEMICAL AND IONIC EQUILIBRIA 479
As K a is small for a very weak acid, x << 0.05 60. (a) X - + H2O r OH- + HX
Hence, ( 0.05 + x ) ~
- ( 0.05 - x ) ~
- 0.05 [OH- ] [HX ]
(x ) ( 0.05 + x ) x ( 0.05) Kb =
Thus, . ´ 10-5 =
18 = = x = [H+ ] X-
( 0.05 - x ) ( 0.05)
HX r H+ + X -
-5
= 1.8 ´ 10 M
[H+ ] [X - ]
pH= - log(1. 8 ´ 10-5 ) = 4.74 Ka =
[ HX ]
51. (a) By Henderson equation \ K a ´ Kb = [H+ ] [OH - ]
[salt]
pH = pK a + log = K a = 10-14
[acid]
[salt] 0.4 Hence, K a = 10-4
52. (a) pH = pK a + log = 4.76 + log = 6.76
[acid] 4 ´ 10-3 Now, as [X - ] = [HX ], pH = pK a = 4
2
p NH 61. (d) Buffer capacity of an acidic buffer is maximum when
53. (b) K p1 = 3
…(i) the ratio of HA to A - is unity since, pH of acidic buffer
p N2 . p H32
[ A -]
p NH3 = pK a + log
Kp2 = [HA]
p1N/ 2 × p 3H/ 2
2 2
For maximum buffer capacity, [ A - ] = [ HA - ]
2
p NH3
K p22 = …(ii) \ pH = pK a
p N2 × p H3 2
62. (b) Ag2CO3(s ) r 2Ag+ + CO23-
On dividing Eq.(ii) by Eq(i) 2s s
K p1 K sp = [Ag ] [CO23- ]
+ 2
= 1or K p1 = K p22
K p22
= (2S)2 S
54. (c) Ba(NO3 )2 gives Ba and 2+
NO-3
ions, hence, K sp = 4S3
2+
concentration of Ba ion increases. To keep K sp
63. (b) Amphoteric behaviour is that any species which acts
constant, [F - ] decreases. as an acid as well as base.
1 - HCO-3 ¾® CO23- + H+
Thus, it is represented as [ F ].
2 Acid

55. (c) For anionic hydrolysis, HCO3- + H+ ¾® H2CO3


Base
1 1 1
pH = pKw + pK a + log C H2O ¾® OH- + H+
2 2 2
Acid
56. (c) NaBrO3 gives BrO-3 ions. Hence, [BrO-3] increases. To
H2O + H+ ¾® H3O +
keep K sp constant, [Ag+ ] decreases. Base

57. (d) Anionic hydrolysis means anion reacts with water. 64. (b) On increasing temperature, Kw of water increases and
Na 2CO3 + 2H2O r 2NaOH + H2CO3 hence pKw decreases.
or 2Na + CO23- + 2H2O r 2Na + + 2OH- + H2CO3
+ 65. (c) Ag2CrO4 ¾® 2Ag+ + CrO24-
1
or CO23- + 2H2O r 2OH- + H2CO3 [CrO2-
4 ]= [Ag+ ]
2
58. (d)
1.5 ´ 10-4
59. (b) CH3COOH r CH3COO- + H+ = = 0.75 ´ 10-4
1
2
0 0
1- a a a K sp = [Ag+ ] 2 [CrO24- ]
pK a = - log K a = 4.74
= (1.5 ´ 10-4 )2 ( 0.75 ´ 10-4 )
\ K a = 1.82 ´ 10-5
= 1.6875 ´ 10-12
Ca 2
From, Ka = = C a2 (Q 1- a = 1) 66. (c) Ag2CrO4 r 2Ag+ + CrO24-
(1- a )
S mol/L 2S S
Ka . ´ 10-5
182 K sp = [ Ag+ ] 2[ CrO24- ]
a= =
C 0.05
= ( 2S )2 (S ) = 4S 3
= 0.019 or 1.9%

@iitjeehelps
480 SELF STUDY GUIDE BITSAT

BITSAT Archives
1. (b) Since, at equivalence point
N1V1 = N2V2 [H+ ] = K a ´ C = 1 ´ 10-4 ´ 0.01
(for acid ) (for base)
= 10-6 = 10-3 M
\Volume of NaOH required to reach equivalence point
0.1 ´ 25 pH = -log [H+ ] = -log 10-3 = 3
= = 50 mL
0.05 9. (a) Kb of NH3 is 10-5. (given)
Millimolars of acid \ pKb = 5 and pK a = 14 - 5 = 9
\Concentration of salt formed =
Total volume in mL Hence, pKb and pK a values are 5 and 9 respectively.
25 ´ 0.1 0.1 4
= = 10. (c) a = 4% = = 0.04, Normality = 0.2
75 3 100
Kw ´ K a 10-14 ´ 1.8 ´ 10-5 ´ 3 \ [H+ ] = a ´ N = 0.04 ´ 0.2 = 0.008 M
Since, [H+ ] = =
C 0.1 11. (c) In water, barium hydroxide is hydrolysed as follows:
\ pH = 8.63 Ba(OH)2 r Ba 2+ + 2OH-
2. (a) Concentration of [OH- ] = 2 ´ 1 ´ 10-3 M = 2 ´ 10-3 M
Dn
3. (a) K p = KC (RT ) ; Dn = 1 pOH = - log[ OH- ] = - log( 2 ´ 10-3 ) = 2.69
Kp 0.1 pH + pOH = 14
Kc = =
RT 0.082 ´ 300 pH = 14 - pOH = 14 - 2.69 = 113
. » 110
.
lonisation
= 4 ´ 10-3 12. (a) CH3COONa CH3COO- + Na +
in aqueous solution Strong base
4. (c)
5. (d) 2NOBr(g ) r 2NO(g ) + Br(g ) CH3COO + H2O ¾® CH3COOH + OH-
-

æ 2p p ö Acetate ion undergoes anionic hydrolysis and the resulting


p-ç + ÷
è 9 9ø 2p p solution is slightly basic due to excess of OH- ions. Hence,
6p 9 9 both (A) and (R) are true and (R) is the correct explanation
=
9 of (A).
2
æ 2p ö æpö 13. (b) PCl5 r PCl3 + Cl2
2
( p NO ) ´ ( p Br2 ) ç ÷ ´ç ÷
è 9 ø è9 ø p Initial moles 5 0 0
and Kp = 2
= 2
= Moles at equilibrium 5(1- a) 5a 5a
( p NOBr ) æ 6p ö 81
ç ÷ Conc. at equilibrium 5(1- a) 5a 5a
è 9 ø 0.5 0.5 0.5
Kp 1 a = 40% = 0.4
\ =
p 81 æ 5 ´ 0.4 ö æ 5 ´ 0.4 ö
ç ÷ç ÷
6. (b) N2 (g ) + 3H2 (g ) r 2NH3 (g ) [PCl3] [Cl2] è 0 . 5 ø è 0.5 ø
KC = = = 2.66 mol / L
2 [PCl5] æ 5 ´ 0.6 ö
é mol ù ç ÷
2 êë L úû -2 è 0.5 ø
[NH3] é mol ù
K= = = = L2 mol-2
[N2] [H2] 3 é mol ù é mol ù 3 êë L úû 14. (a) According to law of active mass
êë L úû êë L úû [NO2] 2 [1.2 ´ 10-2] 2
KC = =
7. (a) pH = 3 [N2O4] 4.8 ´ 10-2
[H+ ] = 1 ´ 10-3 mol / L = 0.3 ´ 10-2 = 3 ´ 10-3 mol/L
H2SO4 ¾® 2H+ + SO24- 15. (a)
1 ´ 10 1 -3 16. (c) We have, K p = KC (RT )Dn
[H2SO4] = = M
2 2000 For CO (g ) + 3H2 (g ) r CH4(g ) + H2O (g )
N =2M (for H2SO4) Dn = - 2
2 1 KC
Normality = = N Kp =
2000 1000 (RT )2
8. (c) Given, pK a = 4
\ K p < KC
\ K a = 1´ 10-4 17. (d )

@iitjeehelps
9
Chemical Kinetics

Rate of a Reaction
The rate or speed or velocity of a reaction is the rate of change of concentration of reactants or
products in unit time. Consider the following reaction,
A ¾® B
Rate of reaction can be defined in two ways :
D[ A] D[ B ]
(i) Average rate of reaction, rav = – =
Dt Dt
(ii) Instantaneous rate of reaction (rinst ) can be calculated from rav in the limit Dt ® 0 and is
represented as
d[ A] d[ B ]
rinst = – =
dt dt
In general for a reaction aA + bB ¾® cC + dD,
1 d[ A] 1 d[ B ] 1 d[C ] 1 d[ D]
Rate = – =– = =
a dt b dt c dt d dt
The rate of chemical reaction depends upon a number of factors such as:
(i) Concentration (ii) Nature of reactants
(iii) Temperature ( iv) Presence of catalyst
(v) Exposure to radiations

Rate Law and Rate Constant


According to the law of mass action, the rate of a chemical reaction is directly proportional to the
product of effective concentrations of reacting species, each raised to a suitable power may or may
not be equal to the respective stoichiometric coefficients.
For a general reaction, aA + bB ¾® products
Rate = k[ A]a[ B ]b
where, k is rate constant or velocity constant or specific reaction rate.

@iitjeehelps
482 SELF STUDY GUIDE BITSAT

The above expression is called rate law as it describes the e.g. (a) n = 0, unit of k = mol L-1 s-1
functional dependence of the reaction rate upon
concentration of various reactants. Rate law cannot be (b) n = 1, unit of k = s-1
deduced from balanced equation. It is obtained (c) n =2, unit of k = mol -1 L s-1
experimentally.
(d) n = 3, unit of k = mol -2 L2 s-1

Molecularity of a Reaction
The number of atoms, ions or molecules colliding with each
Integrated Rate Expressions
other simultaneously in a single or elementary step of a (i) Zero order reaction
chemical reaction, is known as molecularity of a reaction. (a) The reaction in which the rate of a reaction is
NOTE independent of the concentration of the reactants is
is always a whole number. called zero order reaction.
(b) For a zero order reaction,
Molecularity of a reaction can never be zero or fractional. 1
or k = {[ A]0 - [ A]}
does not tell about the rate t
or mechanism of the reaction. D[ R]
Rate = - =k
Dt
Classification of the Reactions Based or,
1
k = {[ A]0 - [ A]}
on Molecularity t
The reaction can be classified on the basis of the number of (c) The graph between the concentration of reactants
molecules participating in the reaction. and time is a straight line with slope = - k

Reactions

Concentration
Unimolecular Bimolecular Trimolecular
(Involves (Involves (Involves
participation of participation of participation of Time
one molecule) two molecules) three molecules)
O2F2 O2 + F 2 2HI → H2 + I2 2NO + Cl2 → 2NOCl (d) The graph between the concentration and rate of
the reaction is shown as follows

Order of a Reaction
Rate

The sum of the powers of the concentration of the reactants


contained in the rate law expression is called order of that
chemical reaction. Consider the following reaction,
Concentration
aA + bB ¾® Products
hn
Rate = k[ A]a [ B ]b e.g. H 2 + Cl 2 ¾® 2HCl
Order of the reaction = a + b 2NH3 ¾® N 2 + 3H 2
Order of a reaction is an experimental entity. (ii) First order reaction
It may be 0, 1, 2, 3 and even a fraction. (a) The reaction in which the rate of a reaction depends
upon the concentration of one reactant is called
Units of Rate Constant for Different first order reaction.
(b) The rate constant for first order reaction is given by
Order of Reactions 2.303 a
The unit of rate constant for different order of reactions can k= log 10
t a-x
be given by a generalised formula
where, a = initial concentration of a reactant
Unit of k = (mol L-1 )1 - n s-1 [Qn = order of a reaction]
(a - x ) = concentration of reactant left after time t.

@iitjeehelps
CHEMICAL KINETICS 483
æ a ö 1
(c) The graph between log 10 ç ÷ and time is a (c) The graph between and time (t ) is a straight
èa - x ø [ A]2
straight line passing through the origin. line.

1
a —–
log10 a – x [A ]2

Time
Time

e.g. 2N 2O5 ¾® 4NO2 + O2 e.g. 2NO + O2 ¾® 2NO 2


H 2O / H + 2NO + Cl 2 ¾® 2NOCl
C 12 H 22 O11 ¾¾® C6H 12O6 + C6H 12O6
Sucrose Fructose

(iii) Second order reaction Half-life of a Reaction


(a) The reaction in which the rate of a reaction depends
The time period in which the concentration of the reactant
upon the concentration of two reactants is called
reduces to half of its initial value is called half-life of a
second order reaction.
reaction. The general expression of t 1 / 2 for n th order of the
(b) The rate constant for second order reaction is given reaction is
by 1
2.303 b (a - x ) t 1/ 2 µ
k= log 10 [ A0 ] n - 1
t (a - b ) a (b - x )
[ A0 ]
where, a = initial concentration of one reactant For zero order reaction, t 1 / 2 =
2k
b = initial concentration of second reactant
0.693
(a - x ) = concentration of first reactant left after For first order reaction, t 1 / 2 =
k
time t
(b - x ) = concentration of second reactant left after Expression for different fraction of first order reaction are
time t t3 / 4 or t 75% = 2 t 1 / 2
(c) The graph between concentration A and time t is a t87.5% = 3 t 1 / 2
straight line. Amount of the substance left after n half-lives = [ A0 ] / 2 n
The time taken for n th fraction of first order reaction is
1 given by
[A ] 2.303 a
t 1/ 2n = log
k a-x
Time
e.g. 2HI ¾® H 2 + I 2
K 2S2O8 + 2KI ¾® 2K 2SO4 + I 2
Effect of Temperature on the
(iv) Third order reaction Rate of Reaction
(a) The reaction in which the rate of a reaction depends Temperature greatly affects the rate of reaction. Generally,
upon the concentration of three reactants is called the rate of reaction becomes nearly double with every 10°C
third order reaction. rise of temperature.
(b) The rate constant for third order reaction is given When a graph is drawn between kinetic energy and fraction
by of molecules, a peak of curve is obtained which represents
1 x (2a - x ) the kinetic energy possessed by the maximum number of
k= × 2
t 2a (a - x )2 molecules in the reaction.

@iitjeehelps
484 SELF STUDY GUIDE BITSAT

This peak of curve is called probable kinetic energy. Mathematically,


t Activation energy
= threshold energy – average energy of the reactants
(t + 10 C° )
Hence, lower the activation energy, faster is the reaction
Fraction of
molecules

Area showing fraction


of additional molecules and vice-versa.
reacting at t + 10
Area showing the
fraction of molecules
reacting at t
Effect of Catalyst on the Rate
Kinetic energy of Reaction
When temperature rises to (t + 10°C), the curve broadens The substance which alters the rate of reaction without
and proportion of molecules with higher energies increases. itself undergoing any chemical change is called catalyst.
This leads to an increase in the number of effective collisions
According to this theory of intermediate complex
and hence the rate of the reaction increases.
formation, reactants combine with catalyst through
temporary bonds to form an intermediate complex which is
Arrhenius Equation short-lived and decomposes to give products and catalyst.
The effect of temperature on the rate of a reaction and rate X + Y + Catalyst ¾® X ----Y ¾® X ¾Y + Catalyst
constant (k ) was given by Arrhenius in terms of Catalyst
intermediate complex
- E a / RT
k = Ae
Catalyst provides an alternate pathway by lowering the
where, A = Arrhenius factor or pre-exponential factor or potential energy barrier through the reduction of activation
frequency factor energy between the reactants and products. Hence, catalyst
E a = activation energy, R = gas constant increases the rate of a reaction.
The term e - Ea / RT provides the fraction of molecules having Reaction path
energy equal to or greater than the activation energy. Reaction path
with catalyst
Taking natural logarithm of Arrhenius equation, without
E catalyst
ln k = ln A - a Ea
RT Without catalyst
Ea
When a graph is plotted between in k and 1/T, a straight line
Potential energy

With catalyst
is obtained.
Intercept = ln A Reactants
Products
ln k
Slope=–Ea /R
Reaction coordinate

1/T NOTE Catalyst neither alter the Gibbs free energy change [DG] of the
reaction or the enthalpy change (DH) of the reaction.
If at temperature T 1 , the value of rate constant is k1 and at T 2 ,
the value of rate constant is k2 , hence

ln
k2 E a é T 2 - T 1 ù
=
Collision Theory of
R êë T 1T 2 úû
k1
Bimolecular Gaseous Reactions
k2 E a é T2 - T1 ù This theory is based upon the kinetic theory of gases
or log =
k1 2.303 R êë T 1T 2 úû
assuming that the reactant molecules are hard spheres.
This equation is used only when rate constants at two The main postulates of this theory are as follows:
different temperatures are provided.
(i) The reaction between the molecules, occurs only when
they collide with each other in a proper orientation.
Activation Energy (ii) Only those collisions result in chemical reaction in
which the molecules acquire energy greater than the
The minimum amount of energy absorbed by the reactant
activation energy.
molecules such that this energy becomes equal to
threshold energy, thereby undergoing effective collisions (iii) For an effective collision, the colliding molecules must
for a reaction to take place is called activation energy. possess threshold energy.

@iitjeehelps
CHEMICAL KINETICS 485
(iv) An activation energy is supplied to the molecules for Rate = pZABe - Ea / RT
attaining this threshold energy. During collisions, this
where, p = probability or steric factor or orientation factor
energy is acquired by the molecules as a result of
interchange of energies. The rate of a reaction in terms Other Important Terms and Relations
of collision frequency is given by
1. Temperature coefficient The ratio of rate constants of
Rate = ZAB e - Ea / RT a reaction at two temperatures (i.e. at 298 K and 308 K)
where, ZAB refers to the collision frequency of is known as temperature coefficient i.e.
reactants A and B, e - Ea / RT represents the fraction of Rate constant at 308 K
Temperature coefficient =
molecules with energy either equal to or greater than Rate constant at 298 K
Ea. 2. Quantum efficiency of a photochemical reaction
For an effective collision, probability or steric factor is Number of molecules reacting in a given time
introduced in the above equation for proper f=
Number of quanta absorbed in the same time
orientation of colliding molecules.

Practice Exercise
1. For a chemical reaction 2X + Y ¾® Z , the rate of 6. The rate for the decomposition of NH3 on platinum
-1 -1
appearance of Z is 0.05 mol L min . The rate of surface is zero order. What are the rates of production
disappearance of X will be of N2 and H2 respectively, if k = 2.5 ´ 10-4 mol L-1s -1 ?
a. 0.05 mol L-1 min-1 b. 0.05 mol L-1 h-1 a. 1.25 ´ 10-4 and mol L-1 s-1 and
c. 0.1mol L-1 min-1 d. 0.25 mol L-1 min-1 3.75 ´ 10-4 and mol L-1s-1
-4 -1 -1
1 b. 3.75 ´ 10 and mol L s and
2. A gaseous reaction A 2( g ) ¾® B ( g ) + C ( g ), shows 1.25 ´ 10-4 and mol L-1s-1
2 -4 -1 -1
increase in pressure from 100 mm to 120 mm in c. 2.5 ´ 10 and mol L s and
5 min. The rate of disappearance of A 2 is 3.75 ´ 10-4 and mol L-1s-1
-4 -1 -1
a. 4 mm min-1 b. 8 mm min-1 d. 1.25 ´ 10 and mol L s and
2.5 ´ 10-4 and mol L-1s-1
c. 16 mm min-1 d. 2 mm min-1
k1
3. For the reaction 2N 2O4 r 4NO2, given that 7. In a reversible reaction 2NO2 r N 2O4 , the rate of
k2
-d [N2O4 ] d [ NO2] disappearance of NO2 is equal to
= K and = K ¢, then 2k1
dt dt a. [NO2] 2 b. 2k1[NO2] 2 – 2k 2[N2O4]
a. K ¢ = 2K b. K ¢ = K k2
c. 2K ¢ = K d. None of these c. 2k1[NO2] 2 – k 2[N2O4] d. ( 2k1 – k 2 ) [NO2]
4. The rate constant of a first order reaction is 8. For the reaction R ¾® P , the concentration of a
2.0 ´ 10-5 s -1 and the initial concentration is 0.10 mol
reactant changes from 0.03 M to 0.02 M in 25 min.
L-1. The initial rate is Calculate the average rate of reaction using units of
a. 2.0 ´ 10-6mol L-1 s-1 b. 1.0 ´ 10-6mol L-1 s-1 time in seconds.
c. 1.5 ´ 10-6mol L-1 s-1 d. 0.5 ´ 10-6mol L-1 s-1 a. 6.66 ´ 10-5 Ms-1
b. 6.6 ´ 10-6 Ms-1
5. The chemical reaction 2O3 ¾® 3 O2, proceeds as c. 5.67 ´ 10-5 Ms-1
follows: d. 7.26 ´ 10-6 Ms-1
O3 r O2 + O (fast)
9. Rate law for the reaction A + 2B ¾® C , is found to be
O + O3 ¾® 2O2 (slow)
Rate = k [ A] [B ]. If the concentration of reactant B is
The rate law expression should be doubled keeping the concentration of A constant, the
a. r = k ¢[O3] 2 b. r = k ¢ [O3] 2 [O2] - 1 value of rate constant will be
c. r = k ¢[ O3] [ O2] d. Unpredictable a. the same b. doubled
c. quadrupled d. halved

@iitjeehelps
486 SELF STUDY GUIDE BITSAT

10. For the reaction 2H 2 + O2 ¾® 2H 2O, 20. The rate constant of first order reaction whose half-life
n
the rate law expression is, r = k [ H 2] . When the is 480 s, is
concentration of H 2 is doubled, the rate of reaction a. 1.44 ´ 10-3 s-1 . s- 1
b. 144
found to be quadrupled. The value of n is c. 0.72 ´ 10-3 s-1 d. 2.88 ´ 10-3 s-1
a. 0 b. 1 c. 2 d. 3 21. In accordance to Arrhenius equation, the plot of log k
11. Consider the following reaction, 1
against is a straight line. The slope of the line is
d [NO2] T
2N2O5 ¾® 4NO2 + O2; = k 2 [ N 2O5 ], equal to
dt
-Ea -2.303
d [ O 2] d a. b.
= k 3 [ N2O5 ] and [ N2O5 ] = k1 R Ea × R
dt dt
-Ea -Ea
The relation between k1, k 2 and k 3 is c. d.
2.303R 2.303
a. k1 = k 2 = k 3 b. 2k1 = k 2 = 4k 3
c. 2k1 = 4k 2 = k 3 d. None of these 22. For the first order reaction, k = 5.48 ´ 10-4 s -1, the
two-third life time for this reaction is
12. The rate of formation of SO3 in the following reaction a. 2005 s b. 1000 s c. 2000 s d. 3005 s
is 100 g min -1. 2SO2 + O2 ¾® 2SO3
23. The rate constant for a zero order reaction is
The rate of disappearance of O2 is C0 C0 – Ct
a. k = b. k =
a. 29 g min -1 b. 20 g min -1 2t t
c. 50 g min -1 d. 200 g min -1 C – Ct C0
c. k = In 0 d. k =
13. Consider the following reaction, t Ct
2A + B + C ¾® Products 24. The time required for 100% completion of a zero order
How will the rate of reaction changes when the reaction is
concentration of A is doubled and that of B is tripled a
a. ak b.
while C is taken in excess? 2k
a. The rate reduces 8 times of its original value a 2k
c. d.
b. The rate reduces 12 times of its original value k a
c. The rate increases 8 times of its original value 25. Pieces of wood burn faster than a log of wood of the
d. The rate increases 12 times of its original value
same mass because
14. In the reaction 2A ¾® Products, the concentration a. surface area of log of wood is larger and needs more
of A decreases from 1.0 mol L-1 to 0.8 mol L -1 in time to burn
20 min. Calculate the rate during this interval. b. pieces of wood have larger surface area
c. all pieces of wood catch fire at the same time
a. 0.5 mol L–1 min-1 b. 0.005 mol L-1 min-1 d. log of wood has higher density than pieces of the
c. 0.05 mol L-1 min-1 d. 0.0005 mol L-1 min-1 same wood
15. The unit of the rate constant of nth order is 26. What is the order of a reaction which has a rate
a. mol1-n Ln-1 s-1 b. moln-1L1 - ns-1 expression, i.e. rate = k [ A] 3 / 2[B ] -1?
c. moln-1Ln - 1s d. moln L1 - ns-1 3 1
a. b.
16. In the presence of acid, the initial concentration of 2 2
c. Zero d. None of these
cane sugar was reduced from 0.2 M to 0.1 M in 5 h
and to 0.05 M in 10 h. The reaction must be of 27. If initial concentration is doubled, the time for
a. zero order b. first order half-reaction is also doubled, the order of reaction is
c. second order d. fractional order a. zero b. first c. second d. third
17. The rate constant of a reaction is 28. If the graph of concentration of [A ] vs T for completion
3.25 ´ 10-3 mol-2 L2 min-1. The order of raction is of reaction is a straignt line, then the order of reaction
a. zero b. 1 c. 2 d. 3 is
a. zero b. second c. first d. third
18. The half-life period of a first order process is 1.6 min. It
will be 90% completed in 29. For a reaction, temperature increases by 10°C, the
a. 0.8 min b. 3.2 min c. 5.3 min d. 1.6 min equilibrium will be attained faster
1
19. The hydrolysis of ester in alkaline medium is a a. 2 times b. same c. times d. 4 times
2
a. first order reaction with molecularity 1
b. second order reaction with molecularity 2 30. 75% of a first order reaction was completed in 32 min.
c. first order reaction with molecularity 2 When was 50% of the reaction completed?
d. second order reaction with molecularity 1 a. 16 min b. 24 min c. 8 min d. 4 min

@iitjeehelps
CHEMICAL KINETICS 487
31. The rate of a reaction becomes 4 times when k2
39. The value of log is equal to
temperature is raised from 293 K to 313 K. The k1
activation energy for such reaction would be Ea éT2 - T1 ù Ea é TT 1 2 ù
a. 50.855 kJ mol-1 a. ê ú b. ê ú
2.303R ë TT
1 2 û 2.303R ëT2 - T1 û
b. 52.849 kJ mol-1
Ea éT2 + T1 ù Ea é TT 1 2 ù
c. 54.855 kJ mol-1 c. ê ú d. ê ú
2.303R ë TT
1 2 û 2.303R ëT1 + T2 û
d. 56.855 kJ mol-1
1
32. Half-life of a hypothetical reaction is found to be 40. A graph plotted between logk vs is represented by
T
inversely proportional to the cube of initial
concentration. The order of reaction is
a. 4 b. 3 a. log k b. log k
c. 5 d. 2
33. H2O and O-atom react in upper atmosphere
bimolecularly to form two OH radicals. DH for the 1/T 1/T
reaction is 72 kJ at 500 K and energy of activation is
77 kJ mol -1. E a for bimolecular recombination of two
c. log k d. log k
OH radicals to form H2O and O-atom, will be
a. 5 kJ mol -1 b. 72 kJ mol -1
c. 77 kJ mol -1 d. 149 kJ mol -1 1/T 1/T
34. Calculate the half-life of the first order reaction,
41. Rate of a reaction can be expressed by Arrhenius
C2H4 O( g ) ¾® CH4 ( g ) + CO( g ). If the initial pressure
equation, k = Ae -E a / RT . Here, E is
of C2H4 O( g ) is 80 mm and the total pressure at the
end of 20 min is 120 mm. a. the total energy of the reacting molecules at a
a. 40 min b. 120 min temperature T
c. 20 min d. 80 min b. the energy above which all the colliding molecules
will react
35. The half-lives for two samples are 0.1 and 0.8 s c. the energy below which colliding molecules will not
whose concentrations are 400 and 50 mol L-1 react
respectively. The order of the reaction is d. the fraction of molecules with energy greater than
a. 0 b. 1 the activation energy of the reaction
c. 2 d. 3 42. Compounds A and B react according to the following
36. The rate constant for the first order reaction is 60 s -1. chemical equation:
How much time will it take to reduce the concentration A ( g ) + 2B ( g ) ¾® 2C ( g )
of the reactant to 1/16th value? Concentration of either A or B were changed keeping
a. 4.6 ´ 104 s b. 4.6 ´ 10- 4 s the concentrations of one of the reactant constants
c. 4.6 ´ 10-2 s d. 4.6 ´ 102 s and rates were measured as a function of initial
concentration. Following results were obtained.
37. The half-time of the following first order decomposition
of nitramide is 2.1 h at 15°C: Initial Initial Initial rate of
concentration concentration formation of
NH2NO2(aq ) ¾® N2O ( g ) + H2O (I ) Experiment of [A] / mol L-1 of [ B ] / mol L-1
[C ] / mol L-1s-1
If 6.2 g of nitramide is allowed to decompose then
time taken for it to decompose 99%, will be 1. 0.30 0.30 0.10
a. 2.1 h b. 12 h 2. 0.30 0.60 0.40
c. 13.96 h d. 33 h
3. 0.60 0.30 0.20
38. A first order reaction is found to have a rate constant,
k = 4.2 ´ 10-12 s -1. Find the half-life of the reaction. Choose the correct option for the rate equations for
13
a. 1.26 ´ 10 s 11
b. 1.65 ´ 10 s
this reaction.
a. Rate = k [ A] 2[B] b. Rate = k [ A][B] 2
c. 1.65 ´ 10-11 s d. 1.26 ´ 10-13 s
c. Rate = k [ A][B] d. Rate = k [ A] 2[B] 0

@iitjeehelps
BITSAT Archives
1. Following is the graph between logT50 and 4. Consider the following reaction,
log a (a = initial concentration) for a given reaction at 2N 2O5 r 4NO2 + O2.
27° C. [2014] If rate and rate constant for above reaction are
2.40 ´ 10-5 mol L-1 s -1 and3 ´ 10-5 s -1 respectively,
then calculate the concentration of N2O5 . [2012]
a. 1.4 mol L-1
log t1/2

45°
b. 1.2 mol L-1
c. 0.04 mol L-1
d. 0.8 mol L-1
log a
5. For a given reaction, t1/ 2 = 1/ ka . The order of this
Hence, order is
reaction is [2011]
a. 1 b. 2 c. 3 d. 0
a. 0 b. 1 c. 2 d. 3
2. Choose the law that corresponds to data shown for
6. The time taken for 90% of a first order reaction to be
the reaction, A + B ¾® products. [2013]
completed is approximately [2011]
Experiment [A] [B] Initial rate a. 1.1 times that of half-life
b. 2.2 times that of half-life
1 0.012 0.035 0.1 c. 3.3 times that of half-life
2 0.024 0.070 0.8 d. 4.4 times that of half-life

3 0.024 0.035 0.1 7. What is the energy of activation of a reaction if its rate
doubles when the temperature is raised from 290 K to
4 0.012 0.070 0.8
300 K ? [2010]
a. Rate = k [B ] 3 b. Rate = k [B ] 4 a. 12 kcal
c. Rate = k [ A] [B ] 3 d. Rate = k [ A] 3[B ] b. 15 kcal
c. 10 kcal
3. A reaction takes place in three steps. The rate d. 20 kcal
constants are k1, k 2 and k 3 . The overall rate constant 8. The active mass of solid is generally taken as [2010]
kk
k = 1 3 . If E 1, E 2 and E 3 (energy of activation) are 60, a. >1
k2 b. =1
30 and 10 kJ, respectively, the overall energy of c. <1
activation is [2013] d. 0
a. 40 kJ b. 30 kJ c. 400 kJ d. 300 kJ

@iitjeehelps
Answer with Solutions
Practice Exercise 10. (c) r = k [ H2]n …(i)

1. (c) 2X + Y ¾® Z On doubling the concentration of H2,

Hence, rate expression for the reaction will be 4r = k [ 2H2]n …(ii)


1 d [X ] d [Y ] d [Z ] On dividing Eq. (ii) by Eq. (i), we get,
- = =+
2 dt dt dt 4 = ( 2)n
d [X ]
\ - = 2 ´ 0.05 mol L-1min -1 ( 2)2 = ( 2)n
dt
n=2
= 0.1mol L-1 min-1
11. (b) 2N2O5 ¾® 4NO2 + O2
1
2. (b) A 2(g ) ¾® B (g ) + C(g ) 1 d [ N2O5] 1 d [ NO2] d [ O2]
Initially 100 mm 2 Rate = - = =
After 5 min
0 0 2 dt 4 dt dt
(100 - p )mm 1
p
2
p Þ 2k1 = k 2 = 4k 3
1 1 12. (b) 2SO2 + O2 ¾® 2SO 3
Total pressure = 100 - p + p + p = 100 + p
2 2 d [ O2] 1 d [ SO 3]
1 Rate = - =
\ Increase in pressure = p = 120 - 100 = 20 mm dt 2 dt
2 d [ O2] 1 æ 100 ö 1 æ 100 ö
-1 -1
\Decrease in pressure of A2 = 2 ´ 20 = 40 mm - = ç ÷ mol min = ç ÷ ´ 32 g min
dt 2 è 80 ø 2 è 80 ø
dx 40
\Rate of disappearance of A2 = = = 8 mm min-1 [Q Molar mass of O2 = 32 g/mol ]
dt 5
= 20 g min- 1
3. (a) For this reaction,
2N2O4 R 4NO2, 13. (d) 2A + B + C ¾® Products
1 d [N2O4 ] 1 d [ NO2] r1 = k [ A] 2 [B ]
Rate expression = - =
2 dt 4 dt [ QC is taken in excess, rate does not depend on C ]
d [N2O4 ] 1 d [ NO2]
or - = r2 = k [ 2A] 2 [ 3B ]
dt 2 dt
1 r2 12 k [ A] 2 [B ]
or K = K¢ = Þ r2 = 12 r1
2 r1 k [ A] 2 [B ]
or K ¢ = 2K 14. (b) Rate of reaction = Rate of disappearance of A
æ dx ö 1 D( A ) 1 (0.8 - 1.0) mol L-1
4. (a) Rate ç ÷ = Rate constant (k ) ´ Concentration =- =-
è dt ø 2 Dt 2 20 min
= 2.0 ´ 10-5 s-1 ´ 0.10 mol L -1 = 0.005 mol L-1 min-1
= 2.0 ´ 10-6 mol L-1s-1 15. (a) The units of rate constant of nth order is mol1-n Ln-1s-1
5. (b) 6. (a) 16. (b) For the first order reaction
[ A]
7. (b) Rate µ [ A] Þ = Constant (k )
Rate
8. (b) For the reaction R ¾® P ,
(i) When Dx = 0.2 - 0.1 = 0.1, Dt = 5 h, [ A] = 0.2
Average rate of reaction 0.2
Hence, = 10 = k
Change in concentration of reactant or product 0.1/ 5
=
Time taken (ii) When Dx = 0.1 - 0.05 = 0.05, Dt = 5 h, [ A] = 0.1
- D [R] [R2] - [R1] (0.02 - 0.03) M 0.1
= =- =- Hence, = 10 = k
t t (25 ´ 60) s 0.05 / 5
= 6.6 ´ 10-6 Ms-1 Hence, reaction follows first order rate law.
17. (d) The unit of rate constant for nth order reaction
9. (b) Rate of reactions, r1 = k [ A] [B]
= (mol)1 - n Ln - 1 min -1
r2 = k [ A] [ 2B]
On putting n = 3, we get the unit of given rate constant.
r2 = 2r1
Hence, the order of reaction is 3.

@iitjeehelps
490 SELF STUDY GUIDE BITSAT

0.693 30. (a) For the reaction ,


18. (c) For first order reaction, t1/ 2 =
k 2.303 a 2.303
k = .log = .log 4 = 0.04332
0.693 t a -x 32
Þ k = = 0.4332 min-1
1.6
Hence, time required for 50% completion,
2.303 a
Now, for 90% completion, k = log 2.303 a
t a -x t= .log
0.04332 a
a-
2.303 100 2
Þ 0.4332 = log Þ t = 5.3 min
t 10 2.303
= ´ 0.3010 = 16 min
19. (b) Alkaline hydrolysis of ester (saponification) is a 0.04332
bimolecular and second order reaction. 31. (b) From the Arrhenius equation,
20. (a) For the first order reaction k Ea éT2 - T1 ù
0.693 0.693 log 2 = ê ú
Þ k = = = 1.44 ´ 10-3 s-1 k1 2.303R ë T1 ×T2 û
t1/ 2 480 sec
Ea é 313 - 293 ù
21. (c) Arrhenius equation can be written as Hence, log 4 = ê ú
2.303 ´ 8.314 ë 293 ´ 313 û
Ea
log k = log A - 11.521
2.303RT Ea = = 52.849 kJ mol -1
0.000218
On comparing this equation with the straight line equation,
1
y = mx + c 32. (a) t1/ 2 µ n -1
a
Ea 1
we get, y = log k , m = - , x = , C = log A 1
2.303R T Hence, t1/ 2 µ , only when n = 4
a3
22. (a) For first order reaction, we know that
Ea
2.303 a 33. (a) H2O + O ¾® 2OH; DH = 72 kJ
k = .log
t (a - x ) E
2OH ¾®
b
H2O + O; DH = - 72 kJ
2.303 a
Hence, t = -4
´ log Also Ea - E b = DH or 77 - Eb = 72
5.48 ´ 10 2
a- a
3 \ Eb = 5 kJ mol-1
2.303 ´ 104 34. (c)
= ´ log 3
5.48 35. (c) The ratio of two half-lives and their concentrations are
2.303 ´ 104 n -1
= ´ 0.4771 = 2005 s (t1/ 2 )1 é a 2 ù
related as = [Here, n = order]
(t1/ 2 )2 êë a1 úû
5.48
23. (b) For a zero order reaction, n -1
C – Ct 0.1 é 50 ù
C0 – C t = k t Þ k = 0 Therefore, =
t 0.8 êë 400 úû
24. (c) For a zero order reaction, On taking log of both sides
x = kt 0.1 50
log = (n - 1)log
For 100% completion of the reaction, x = a 0.8 400
a 1= n - 1 Þ n = 2
Therefore, a = kt or t = 2.303 [A ]
k 36. (c) For the first order reaction, k = log 0
t [ A]
25. (b) More the surface area of reactant, more will be the
rate of reaction. 2.303 æ a ö -2
Then, k = logç ÷ = 0.0462 = 4.62 ´ 10 s
æ 3ö 1 t è a / 16 ø
26. (b) Order of reaction = ç ÷ + ( -1) =
è 2ø 2 37. (c) For the first order reaction,
27. (a) For nth order reaction, t1/ 2 µ a1 - n k =
0.693 0.693
= = 0.33 h-1
Here, t1/ 2 µ a t1/ 2 2.1

Hence, 1 - n = 1 or n = 0 (i.e. zero order reaction) 2.303 a


and k = log10
t (a - x )
28. (a) In the zero order reaction, concentration decreases
linearly with time as the rate of reaction is constant (does 2.303 100
or t= log10
not depend on concentration of reactant). k (100 - 99 )
29. (a) Rate of reaction becomes almost twice with an 2.303
= ´ 2 = 13.96 h
increase of 10°C in temperature (from 298 K to 308 K). 0.33

@iitjeehelps
CHEMICAL KINETICS 491
38. (b) For the first order reaction 0.40 = K [ 0.30] x [ 0.60] y …(iii)
0.693 0.693 x
0.20 = K [ 0.60] [ 0.30] y
t1/ 2 = = …(iv)
k 4.2 ´ 10-12 s-1
From Eqs. (ii) and (iii), we get
= 1.65 ´ 1011s x y
0.1 é 0.30 ù é 0.30 ù
éT2 - T1 ù =
k
39. (a) The value of log 2 is equal to
Ea 0.4 êë 0.30 úû êë 0.60 úû
k1 2.303 R êë TT ú.
1 2 û y
1 é 1ù
= Þy = 2
40. (c) On the basis of Arrhenius equation, 4 êë 2 úû
From Eqs. (iii) and (iv), we get
x y
0.40 æ 0.30 ö æ 0.60 ö
=ç ÷ ç ÷
log K 0.20 è 0.60 ø è 0.30 ø
x
æ 1ö
2 = ç ÷ ( 2 )y
è 2ø
1/T
On putting the values of y, we get
x
41. (c) In the Arrhenius equation for the rate of reaction, E is æ 1ö
2 = ç ÷ ( 2 )2
the energy below which colliding molecules will not react. è 2ø
42. (b) Let, Rate law, r = k [ A] x [B] y …(i) Þ x =1
On putting values, we get On putting the values of x and y in Eq. (i), we get
0.10 = K [ 0.30] x [ 0.30] y …(ii) r = k [ A]1[B] 2 = k [ A] [B ] 2

BITSAT Archives
n -1 1 1
æ 1ö 5. (c) t1/ 2 µ ×
1. (d) t1/ 2 µ ç ÷ or t1/ 2 = k (a )1 - n
èa ø (a )n - 1 k n
log t1/ 2 = log k + (1 - n )log a 1
Given, t1/ 2 µ
(It represents straight line equation, y = c + mx ) a
Slope = (1 - n ) = tan 45° = 1 On comparing, an - 1 = a1, n - 1 = 1, n = 1 + 1 = 2
\ (1 - n ) = 1 Þ n = 0 6. (c) For a first order reaction,
2. (a) It is seen that, in experiments (3) and (2), [A] is 2.303 a
constant and [B] is doubled and rates becomes 8 times, k = log
t a -x
so order with respect to [B ] = 3. In experiments (1) and
(3), [B ] is constant and [A] is doubled, but rate does not 2.303 ´ t1/ 2 100 é 0.693 ù
t= log êQ k = ú
change, so order with respect to [ A] = 0 0.693 (100 - 90) ë t1/ 2 û
Thus, rate = k [B] 3
= 3.3 t1/ 2
E a1 /RT -E a 2 /RT -E a 3 /RT
3. (a) k1 = Ae , k 2 = Ae , k 3 = Ae = 3.3 times that of half-life
kk
Overall rate = k = 1 3 k2 Ea éT2 - T1 ù
7. (a) log =
k2 k1 2.303R êë TT ú
1 2 û
Therefore, overall Ea = Ea1 + Ea 3 - Ea 2 é 300 - 290 ù
Ea
log 2 =
= 60 + 10 - 30 = 40 kJ 2.303 ´ 2 êë 290 ´ 300 úû
4. (d) For a first order reaction, Rate, = k[ N2O5]
Ea = 12062 cal
2.4 ´ 10-5 = 3 ´ 10-5 ´ [ N2O5]
=12.0 kcal
2.4 ´ 10-5
[ N2O5] = = 0.8 mol L-1 8. (b) The active mass of solids is usually taken as unity..
3 ´ 10-5

@iitjeehelps
10
Solution

Solutions in Chemistry
Solution is a homogeneous mixture of two or more substances on molecular level. A solution of two
substances is called a binary solution. The substances forming the solution are called components of
the solution. As a generalisation, the component present in smaller amount is called solute and the
other present in larger amount is called solvent.

Different Methods for Expressing Concentration of Solutions


Number of moles of solute
1. Molality (m ) =
Weight of solvent( in kg)
Number of moles of solute
2. Molarity ( M ) =
Volume of solution (in L)

Relation between molality (m) and molarity (M)


m ´d
Molarity ( M ) =
m ´ M2
1+
1000
where, M 2 = molar mass of solute
and d = density of the solution
3. Mole fraction of solute in the solution
Moles of solute (n solute )
c solute =
Total moles of solution (n solute + n solvent )
Mole fraction of solvent in the solution
Moles of solvent (n solvent )
c solvent =
Total moles of solution (n solute + n solvent )
Sum of mole fractions is always equal to 1,
i.e. c solute + c solvent = 1

@iitjeehelps
SOLUTION 493
Relation between molarity (M) and mole fraction (c) where, p1 and p2 are vapour pressures at T 1 and T 2
of the solute respectively and DH is heat of vaporisation.
MM 1
c= Factors Affecting Vapour Pressure
M (M 1 - M 2 ) + d
where, M 1 = molar mass of solvent Vapour pressure gets affected by following factors:
and M 2 = molar mass of solute Temperature
Weight of solute ´ 100 The vapour pressure of a liquid increases with increase in
4. Percentage by weight =
Weight of solution temperature. This is because on increasing the temperature,
5. Percentage by volume kinetic energy of molecules increases that results into the
fact that more molecules of the liquid can go into vapour
(a) Weight of solute per 100 mL of solution (w/V )
phase.
(b) Volume of solute per 100 mL of solution (V /V )
Effect of Adding Solute
106 ´ mass of solute
6. ppm concentration = When a liquid contains a solute, some of the solvent
Mass of (solute + solvent)
molecules are replaced by the solute particles on the
Number of gram - equivalents of solute liquid surface and therefore, the available surface area for
7. Normality =
Volume of solution (in L) the escape of solvent molecules decreases.
Due to the less available area on the surface of liquid for
Solubility escape, rate of evaporation and hence, the rate of
condensation both lowers.
Maximum amount of a solute that can be dissolved in a
given amount of solvent (usually 100 g) at a given The vapour pressure of liquid in solution is known as its
temperature is called solubility at that temperature. partial vapour pressure and is less than the vapour
pressure of the pure liquid at the same temperature.
Solubility of Gas in Liquid If p° be the vapour pressure of pure liquid and ps be that of
All gases are soluble in water and in other liquids upto a
liquid in solution then lowering of vapour pressure of the
greater or lesser extent.
liquid = p° - ps
The solubility of a gas depends upon the following factors: p° - p s
\ Relative lowering in pressure =
(i) Nature of the gas The gases which can be easily p°
liquefied, are more soluble in common solvents, e. g.
CO2 is more soluble than H 2 and O2 in water. Raoult’s Law
(ii) Nature of the solvent The gases which are capable of The partial vapour pressure of a volatile component is
producing ions in aqueous solutions, are much more directly proportional to its mole fraction in solution.
soluble than in other solvent. For a solution possessing two components A and B, the
(iii) Temperature The solubility of most of the gases in partial vapour pressure of A is
liquids decreases with rise in temperature.
pA µ c A
(iv) Pressure The solubility of a gas in liquid is influenced
pA = k × c A
by pressure. The relation between the solubility and
pressure is given by Henry’s law. It states that ‘the where, c A = mole fraction of components A
partial pressure (p) of the gas in vapour phase is If c A = 1, then k (proportionality constant) = p°A
proportional to the mole fraction of the gas (c) in the (vapour pressure of pure component A), thus,
solution’ and is expressed as p = kH c
pA = pA° c A
where, kH is the Henry’s law constant.
Similarly, pB = pB° c A
Vapour Pressure According to Dalton’s law,
The pressure exerted by the vapours of a liquid which are in ptotal = pA + pB
equilibrium with it at a given temperature, is called vapour
pressure. Vapour pressure variation with temperature is ptotal = c A pA° + c B pB° '
given as = (1 - c B ) pA° + c B pB°
p DH é T 2 - T 1 ù
2 . 303 log 10 2 =
p1 R êë T 1T 2 úû = pA° + ( pB° - pA° )c B

@iitjeehelps
494 SELF STUDY GUIDE BITSAT

Limitations of Raoult’s Law Non-ideal Solutions Showing Negative Deviation


(a) It is valid only for very diluted solutions. When the observed vapour pressure is less than that
(b) It is valid only for the solution having non-volatile and expected by Raoult's law, negative deviation is observed.
non-electrolyte solute which exists as a single p°A
molecule.
p total
(c) It is not valid for those solutes which associate or Vp p°B
dissociate in the particular solution. p
A

Ideal and Non-ideal Solutions pB


0 χA 1
Ideal Solutions 1 χB 0
The solutions which obey Raoult's law over the entire range Graphical representation of
of concentration are called ideal solutions. the solution showing negative
deviation
For ideal solutions,
DH mix = 0, DV mix = 0 For such solutions, DH mix < 0, i.e. energy is released on
Solute-solute and solvent-solvent interactions mixing and DV mix < 0, i.e. attractive forces between unlike
» solute-solvent interactions molecules are greater than the forces of attraction between
Non-ideal Solutions like molecules.
The solution which shows deviation from Raoult’s law, is e.g. Chloroform and acetone
called non-ideal solution. For a solution showing negative deviation,
For such solutions, DH mix ¹ 0 , DV mix ¹ 0 pA < pA° ´ c A or pB < pB° ´ c B

Non-ideal Solutions Showing Positive Deviation ptotal < pA° c A + pB° c B


When the observed vapour pressure is more than that ● Maximum boiling azeotropes are formed by those
expected by Raoult's law, positive deviation is observed. liquid pairs which show negative deviation from ideal
● For such a deviation, p
A > pA c A ,
° pB > pB° c B behaviour. Such azeotropes have boiling points higher
than either of the components.
ptotal > pA° c A + pB° c B
e.g. H 2O (20.22% by mass) + HCl.
In this case, the intermolecular attractive forces between
the solute-solvent molecules are weaker than those
between the solute-solute and solvent-solvent molecules.
Colligative Properties
ptotal The properties which depend only on the number of moles
p°A of non-volatile solute, are referred as colligative properties.
pA There are four types of colligative properties as given below:
Vp p°B
1. Relative Lowering of Vapour Pressure
pB Addition of non-volatile solute leads to the lowering of
vapour pressure.
0 χA 1 p°- p
1 χB 0 = csolute

Graphical representation of
the solution showing positive p°- p n A w A × M B
deviation or = =
p° N MA ×w B
For such solutions, DH mix > 0, i.e. energy is absorbed on p°- p
mixing and DV mix > 0. These are usually obtained by mixing where, = relative lowering of vapour pressure

of polar liquids with non-polar ones.
e.g. Cyclohexane and ethanol, H 2O and C 2H 5 OH. n = moles of solute
N = moles of solvent
● Minimum boiling azeotropes are formed by those
liquid pairs which show positive deviation from ideal p° = vapour pressure of pure solvent
behaviour. Such azeotropes have boiling points lower w B and wA are masses of solute and solvent respectively
than either of the components. and M B and M A are molecular weights of solute and
e.g. C 2 H5OH (95.57%) + H2O(4.43%) (by mass) solvent respectively.

@iitjeehelps
SOLUTION 495
2. Elevation in Boiling Point ( DTb ) where, n = moles of solute, C = molar concentration
For dilute solutions, V = volume of solution (in litre), R = gas constant
DT b = K b ´ molality T = temperature (in kelvin)
w RT
DT = T - T °
b b b \ MB = B
pV
where, T b = boiling point of solution
Two solutions having same osmotic pressure at same
and T b° = boiling point of pure solvent temperature, are termed as isotonic solutions. When two
solutions are being compared, the solution with higher
K b is ebullioscopic constant or molal elevation constant
osmotic pressure is termed as hypertonic and the solution
(K b depends only on solvent). with lower osmotic pressure is termed as hypotonic.
K ´ w B ´ 1000 Osmotic pressure can be determined quite accurately, hence,
\ MB = b
DT b ´ w A it is used in the determination of molecular weights of large
RT02 proteins and similar substances.
Kb =
1000 DHV
where, R = gas constant van’t Hoff Factor (i)
and DHV = latent heat of vaporisation In 1880, van’t Hoff introduced a factor i, known as the
van’t Hoff factor, to account for the extent of dissociation or
3. Depression in Freezing Point ( DT f )
association. This factor i is defined as
For dilute solutions
Number of particles after association or dissociation
DT f = K f ´ molality i=
Number of particles before association or dissociation
DT = T ° - T
f f f i -1
Degree of dissociation, a =
where, T ° = freezing point of pure solvent n -1
T f = freezing point of solution where, n = number of particles after dissociation
K f = cryoscopic constant or molal depression i -1
Degree of association, a =
1
constant -1
n
K f ´ w B ´ 1000
\ MB = where, n = number of particles after association
DT f ´ w A
RT02
Modified Expressions of Colligative Properties
Kf = 1. Relative lowering of vapour pressure
1000 DH f
p°- p nB æ nB = n ö
where, H f = latent heat of fusion =i ; ç ÷
p° nB + nA èn B + n A » N ø
4. Osmosis and Osmotic Pressure 2. Elevation in boiling point
Spontaneous flow of solvent molecules through a DT b = i × K b × m where, m = molality
semipermeable membrane from a pure solvent to the
3. Depression in freezing point
solution (or from a dilute solution to concentrated
solution) is termed as osmosis. DT f = i × K f × m
n 4. Osmotic pressure ( p ) = i × CRT
Osmotic pressure ( p ) = RT = CRT
V

@iitjeehelps
Practice Exercise
1. The molarity of 720 g of pure water is 13. A substance will be deliquescent if its vapour pressure
a. 40 M b. 4 M a. is equal to the atmospheric pressure
c. 55.5 M d. unpredictable b. is equal to that of water vapour in air
c. is less than that of water vapour in air
2. Molarity is expressed as
d. is greater than that of water vapour in air
a. g / L b. L / mol
c. mol / L d. mol / 1000g 14. The aqueous solution that has the lowest vapour
pressure at a given temperature, is
3. The formula weight of H 2SO4 is 98. The weight of the
a. 0.1 M sodium phosphate
acid in 400 mL 0.1 M solution is b. 0.1 M barium chloride
a. 2.45 g b. 3.92 g c. 4.90 g d. 9.8 g c. 0.1 M sodium chloride
d. 0.1 M glucose
4. Out of molarity (M ), molality (m), normality (N ) and
mole fraction ( c ), those independent of temperature 15. For determination of molecular mass, Raoult's law is
are applicable only to
a. M and m b. N and c c. m and c d. M and c a. dilute solutions of electrolytes
b. concentrated solutions of electrolytes
5. A c molal solution of a compound in benzene has c. dilute solutions of non-electrolytes
mole fraction of solute equals to 0.2. The value of c is d. concentrated solutions of non-electrolytes
a. 14 b. 3.2 c. 1.4 d. 2 16. 100 mL of liquid A was mixed with 25 mL of liquid B to
6. On mixing 10 mL of acetone with 50 mL of chloroform, give non-ideal solution of A-B. The volume of this
the total volume of the solution is mixture will be
a. < 60 mL b. > 60 mL a. 75 mL
c. = 60 mL d. unpredictable b. 125 mL exact
c. fluctuate between 75 mL and 125 mL
7. The normality of 10% (weight / volume) acetic acid is d. close to 125 mL but not to exceed 125 mL
a. 1 N b. 10 N c. 1.66 N d. 0.83 N
17. 6.0 g of urea (molecular weight = 60) was dissolved in
8. Normality of 2 M H2SO4 is 9.9 moles of water. If the vapour pressure of pure
N N water is p 0, the vapour pressure of solution is
a. 4 N b. 6 N c. d.
4 6 a. 0.10 p 0 b. 1.10 p 0 c. 0.90 p 0 d. 0.99 p 0
9. Suppose sea water contains 3.50 weight per cent of 18. On mixing 25 mL of CCl4 with 25 mL of toluene, the
NaCl. Calculate the molarity of sea water. total volume of the solution is
a. 6.2 m b. 0.062 m c. 0.62 m d. 0.0062 m a. = 50 mL b. > 50 mL
10. Which of the following statements is true about c. < 50 mL d. unpredictable
saturated solution? 19. An azeotropic mixture of two liquids has boiling point
a. A solution in which no more solute can be dissolved lower than either of them when it
at the same temperature and pressure a. shows a negative deviation from Raoults' law
b. The solution which is in dynamic equilibrium with b. shows no deviation from Raoults' law
undissolved solute and contains maximum amount of c. shows positive deviation from Raoult's law
solute dissolved in solvent d. is saturated
c. The solution in which more solute can be dissolved
at the same temperature 20. A sugar solution boils at 101°C. The molality of the
d. Both (a) and (b) sugar solution is (Given, K b = 0 . 52 ° C kg mol-1)
11. Calculate the mass of urea (NH2CONH2 ) required in a. 1.84 m b. 1.92 m c. 2.02 m d. 4.02 m
making 2.5 kg of 0.25 molal aqueous solution. 21. An unknown compound is immiscible with water. It is
a. 37 g b. 35 g c. 34 g d. 32 g steam distilled at 98.0°C. At 98.0°C, p and p °H 2 O are
12. One litre of N/2 HCl solution is heated in a beaker and 737 and 707 torr respectively. This distillate was 75% by
it is observed that when volume was reduced to weight of water. The molecular weight of the unknown
600 mL, 3.25 g of HCl is lost. Calculate the normality will be
of new solution. a. 318.15 g mol-1 b. 300 g mol-1
a. 0.50 N b. 0.60 N c. 0.68 N d. 0.70 N c. 306.76 g mol-1 d. None of these

@iitjeehelps
SOLUTION 497
22. The relative lowering of vapour pressure of an 31. A 0.2 molal aqueous solution of weak acid ( HX ) is
aqueous solution containing a non-volatile solute, is 20% ionised. The freezing point of this solution is
0.0125. The molality of the solution is (Given, K f = 1.86° C m -1 for water)
a. 0.69 m b. 0.50 m c. 0.80 m d. 0.40 m
a. -0.45° C b. -0.55° C c. -0.90° C d. -0.31° C
23. How many grams of H2SO4 is/are to be dissolved to
prepare 200 mL aqueous solution having concentration 32. The van’t Hoff factor for a very dilute solution of
of [H3 O+ ] ions is 1 M at 25°C temperature? Fe2(SO4 )3 is
a. 19.6 g b. 0.98 g c. 4.9 g d. 9.8 g a. 9 b. 5
c. 24 d. None of these
24. Vapour pressure of pure A is 70 mm of Hg at 25°C . If it
forms an ideal solution with B in which mole fraction of A 33. The latent heat of vaporisation of water is
is 0.8 and vapour pressure of the solution is 84 mm of Hg 9700 cal/mol and if the boiling point is 100°C, the
at 25°C, then the vapour pressure of pure B at 25°C is ebullioscopic constant of water is
a. 140 mm b. 70 mm c. 56 mm d. 28 mm a. 0.516°C b. 1.026 °C c. 10.26°C d. 1.832°C

25. Lowering of vapour pressure is highest for 34. The relative lowering of vapour pressure of an
a. 0 .1 M BaCl2 b. 0.1 M glucose aqueous solution containing a non-volatile solute is
c. 0.1 M MgSO4d. urea
0.0125. The molality of the solution is
a. 0.70 m b. 0.590 m c. 0.80 m d. 0.40 m
26. Which of the following curves represent the Henry’s 35. The osmotic pressure of one molar solution at 27°C is
law? a. 2.46 atm b. 24.6 atm c. 1.21 atm d. 12.1 atm
36. The freezing point of 0.1M solution of glucose is
Partial pressure

Partial pressure

a. b. -1.86° C. If an equal volume of 0.3 M glucose solution


of gas

of gas

is added, the freezing point of the mixture will be


a. –7.44°C b. –5.58°C c. –3.72°C d. –2.79°C
37. The van’t Hoff factor for 0.1 M Ba(NO3 ) 2 solution is
Mole fraction of gas Mole fraction of gas
in solution in solution 2.74. The degree of dissociation is
a. 91.3% b. 87% c. 100% d. 74%
Partial pressure

Partial pressure

38. In a 0.2 molal aqueous solution of a weak acid HX, the


c. d. degree of ionisation is 0.3. Taking K f for water as 1.85,
of gas

of gas

the freezing point of the solution will be nearest to


a. -0.360° C b. -0.260° C
c. + 0.480° C d. -0.481° C
Mole fraction of gas Mole fraction of gas
in solution in solution 39. Acetic acid exists in benzene solution in the dimeric
form. In an actual experiment, the van't Hoff factor
27. If p° and ps are the vapour pressures of the solvent
was found to be 0.52. Then, the degree of dissociation
and solution respectively and n1 and n2 are the mole
of acetic acid is
fractions of solvent and solute respectively. Then,
a. 0.48 b. 0.88 c. 0.96 d. 0.52
a. ps = p° n1 b. ps = p° n 2
æn ö 40. A compound X undergoes tetramerisation in a given
c. p° = ps n 2 d.ps = p° ç 1 ÷ organic solvent. The van't Hoff factor is
è n2 ø
a. 4.0 b. 0.25 c. 0.125 d. 2.0
28. Arrange the following in the increasing order of their 41. The freezing point (in °C) of a solution containing 0.1 g
solubility in n -octane based on solute-solvent of K 3 [ Fe(CN)6] (molecular weight 329) in 100 g of
interaction:
water (K f = 1.86 K kg mol-1) is
a. KCl < CH3CN < CH3OH < Cyclohexane
a. - 2.3 ´ 10-2 b. - 5.7 ´ 10-2
b. KCl < Cyclohexane < CH3OH < CH3CN
c. - 5.7 ´ 10-3 d. - 1.2 ´ 10-2
c. KCl < CH3OH < CH3CN < Cyclohexane
42. 0.004 M Na 2SO4 is isotonic with 0.01 M glucose.
d. KCl < Cyclohexane < CH3CN < CH3CN
Degree of dissociation of Na 2SO4 is
29. Which one of them is more volatile component? a. 75% b. 50% c. 25% d. 85%
a. CH2Cl2 b. CHCl3
43. Colligative pro}perties of a solution depends upon
c. Both a and b d. Not able to determine
a. nature of solute only
30. The elevation in boiling point would be highest for b. nature of both solute and solvent
a. 0.08 m BaCl2 b. 0.10 m glucose c. number of solute particles
c. 0.10 m KCl d. 0.06 m calcium nitrate d. number of solvent particles

@iitjeehelps
498 SELF STUDY GUIDE BITSAT

44. Among the following 0.10 m aqueous solutions, which 49. Elevation in boiling point was 0.52°C when 6 g of a
one will exhibit the largest freezing point depression? compound X was dissolved in 100 g of water.
a. KCl b. C6H12O6 c. Al2(SO4 )3 d. K 2SO4 Molecular weight of X is (K b of water is 5.2°C per
100 g of water)
45. To observe an elevation of boiling point of 0.05°C, the a. 120 b. 60 c. 600 d. 180
amount of a solute (molecular weight = 100) to be
added to 100 g of water (K b = 0.5 ) is 50. What does point A and B represent in the following
a. 2 g b. 0.05 g c. 1 g d. 0.75 g diagram?
A B
46. van’t Hoff factor can be applied to the 3 bar
a. relative lowering of vapour pressure of solvent 1 atm
b. elevation of boiling point
c. osmotic pressure of solution
d. all colligative properties

pressure
47. Determine the amount of CaCl2 (i = 2.47) dissolved in

Vapour
DTb
T°b
2.5 L of water such that its osmotic pressure is Tb
0.75 atm at 27°C. Temperature/K
a. 1.0 g b. 9.2 g c. 3.42 g d. 2.42 g
Point A Point B
48. Find out the osmotic pressure of 0.1 M monobasic a. Boiling point of solvent Boiling point of solution
acid if pH = 2.0 at 25°C. b. Boiling point of solution Boiling point of solvent
a. 2.69 atm b. 26.9 atm c. Boiling point of solute Boiling point of solvent
c. 0.269 atm d. None of these d. Boiling point of solvent Boiling point of solute

BITSAT Archives
1. Usually, CaCl2 is preferred over NaCl for cleaning 7. CaCl2 is preferred to NaCl for clearing snow on roads
snow on roads particulary in very cold countries. This particularly in very cold countries. This is because
is because [2014] a. CaCl2 is less soluble in H2O than NaCl [2009]
a. NaCl makes the road slippery but CaCl2 does not
b. CaCl2 is hygroscopic but NaCl is not
b. CaCl2 is hygroscopic but NaCl is not
c. eutectic mixture of CaCl2/H2O freezes at -55°C
c. CaCl2 is less soluble in H2O than NaCl
while that of NaCI / H2O freezes at -18°C
d. eutectic mixture of CaCl2/H2O freezes at -55° C d. NaCl makes the road slippery but CaCl2 does not
while that of NaCl /H2O freezes at -18° C
2. The vapour pressure of a solvent decreased by 8. 100 cc of 1.5% solution of urea is found to have an
10 mm of Hg when a non-volatile solute was added to osmotic pressure of 6.0 atm and 100 cc of 3.42%
the solvent. The mole fraction of solute in solution is solution of cane sugar is found to have an osmotic
0.2, what would be the mole fraction of solvent if the pressure of 2.4 atm. If two solutions are mixed, the
decrease in vapour pressure is 20 mm of Hg? [2013] osmotic pressure of the resulting solution will be
a. 0.8 b. 0.6 c. 0.4 d. 0.3 a. 8.4 atm b. 4.2 atm [2009]
c. 16.8 atm d. 2.1 atm
3. If the elevation in boiling point of a solution of 10 g of
solute (molecular weight = 100) in 100 g of water is 9. When 25 g of a non-volatile solute is dissolved in
DTb , the ebullioscopic constant of water is [2012]
100 g of water, the vapour pressure is lowered by
2.25 ´ 10-1 mm. If the vapour pressure of water at
a. DTb /10 b. DTb c. 10DTb d. 100DTb
20°C is 17.5 mm, what is the molecular weight of the
4. 1.2% NaCl solution is isotonic with 7.2% glucose solute? [2008]
solution. What will be the van’t Hoff factor, i ? [2012] a. 206 b. 302 c. 350 d. 276
a. 0.5 b. 1 c. 2 d. 6 10. A solution of sucrose (molar mass = 342 g/mol) is
5. K b for water is 0.52 K/m. Then, 0.1 m solution of NaCl prepared by dissolving 68.4 g of it per litre of solution,
will boil approximately at [2011] what is its osmotic pressure at 273 K?
a. 100.52°C b. 100.052°C c. 101.04°C d. 100.104°C (R = 0.082 L atm K -1 mol-1) [2007]
-1
6. The molal freezing point for water is 1.86°C mol . If a. 3.92 atm
342 g of cane sugar is dissolved in 1000 mL of water, b. 4.48 atm
the solution will freeze at [2010] c. 5.92 atm
a. 1.86°C b. -1.86° C c. 2.42°C d. -2.42° C d. 29.4 atm

@iitjeehelps
Answer with Solutions
Practice Exercise 12. (c) 1 L of N / 2 HCl contains =
1
´ 36 . 5 = 18.25 g
2
w 720
1. (c) Molarity = = = 55.5 M HCl now present = 18.25 - 3.25 = 15 g
m × V 18 ´ 720 / 1000
Now, volume = 600 mL = 0.600 L
(720 g water = 720 mL water) 15 1
Normality = ´ = 0.685 N
2. (c) 36.5 0.600 L
w
3. (b) Molarity = 13. (c)
m × V (litre)
14. (a) Na 3PO4 r 3Na+ + PO-43
w
Hence, 0.1 = 15. (c)
400
98 ´
1000 16. (d) For non-ideal solution, DVmix < 0. Hence, total volume
400 will be little less than (100 + 25 = 125 mL).
w = 0.1 ´ 98 ´
1000 6
17. (d) Mole of urea = = 0.10
= 3 . 92 g 60
4. (c) Molality (m) and mole fraction (c) involves only mass According to Raoult’s law
and do not depend upon volume. Hence, these are p solvent - p solution nA
= c solute =
independent of temperature. p solvent n A + nB
5. (b) Molality and mole fractions of solute ( c A ) are related as p 0 - p solution 0.10
Hence, =
m. MB m ´ 78 p0 0.10 + 9.9
cA = Þ 0.2 =
1000 + m. MB 1000 + m ´ 78 or p 0 - p solution = 0.01 p 0 or p solution = 0.99 p 0
200 + 15.6 m = 78 m 18. (b) It is a non-ideal solution, hence, DVmix = + ve
or 62.4 m = 200
19. (c)
m = 3.2
20. (b) We know that DTb = i × Kb × m (Here, i = 1)
6. (a) Chloroform and acetone show strong hydrogen
bonding. Hence, for such solution ( DVmix = - ve), volume \ (101 - 100) = 0 . 52 ´ m
decreases on mixing (Non-ideal solution with negative 1
\ Molality (m) = = 1.92
deviation). 0 . 52
w 10 g 1
7. (c) Normality = = ´ 21. (a) Since, unknown compound is immiscible with water.
E . V 100 mL 60 Hence, vapour pressure µ moles
10 g
= = 1.66 N Given, p °total = 737 torr
100
´ 60
1000 ° O = 707 torr
pH2
8. (a) Basicity of H2SO4 = 2
\ °
p unknown = 737 - 707 = 30 torr
Normality = Molarity ´ Basicity of acid = 2 ´ 2 = 4 N
WH2O = 100 g
9. (c) NaCl solution contains 3.5 g of NaCl in 100 g of water.
Wunknown = 75 g
Water in solution = 100 - 3.5 = 96.5 g = 0.0965 kg
3.5 °
p unknown n Wunknown ´ mH2O
Molality = = 0.62 m = unknown =
58.5 ´ 0.0965 p °water n H2O WH2O ´ munknown
10. (d) 30 75.0 ´ 18
or = or munknown = 318.15 g mol-1
11. (a) Mass of solvent = 1000 g 707 100 ´ munknown
Molar mass of urea (NH2CONH2 ) = 60 g mol-1 p° - ps m ´ M
22. (a) =
p° 1000
0.25 mole of urea = 0.25 ´ 60 = 15 g
m ´ 18
Total mass of solution = 100 + 15 = 1.015 kg 0.0125 = (molecular weight of H2O = 18)
1000
1.015 kg of solution contain urea = 15 g
15 0.0125 ´ 1000
2.5 kg of solution = ´ 2.5 = 37 g m= = 0.69
1.015 18

@iitjeehelps
500 SELF STUDY GUIDE BITSAT

w 2 ´ 1000 n é nù
23. (d) Molarity, M = 35. (b) p = RT = MRT
M 2 ´ V (in mL) V êë Molarity (M ) = V úû

where, w 2 is the mass of H2SO4 in g and M 2 is the molar = 1 ´ 0.0821 ´ 300 = 24.6 atm
mass of H2SO4. 36. (c) Kf = DTf / M = 1. 86 / 0.1 = 18.6
1 ´ 98 ´ 200
w2 = = 19.6 g 0.1 ´ V + 0.3 V = M 3 ´ 2V
1000
or M 3 = 0.2
H2SO4 + 2H2O q 2H3O+ + SO24-
DTf = 18.6 ´ 0.2 = 3.72° C
But according to equation, 1 mole of H2SO4 gives 2 moles
Tf = - 3.72° C
of [H3O+ ] ions. Thus, the amount of H2SO4 to prepare
37. (b) Ba(NO3 )2 r Ba 2+ + 2NO-3
200 mL solution having 1 M concentration of H3O+ ions is
19.6/2 = 9.8 g. Initially 1 mol 0 0
After dissociation 1- a a 2a
24. (a) p = p°A c A + p°B cB
Total number of moles = 1 + 2a
Þ 84 = 70 ´ 0.8 + p°B ´ 0.2
\ i = 1 + 2a
28
p°B = = 140 mm i - 1 2.74 - 1
0.2 or a= = = 0.87 = 87%
2 2
25. (a) Lowering of vapour pressure is directly proportional to
the van’t Hoff factor. For BaCl2, i = 3 (maximum) thus, 38. (d) HX r H+ + X -
BaCl2 has highest value for relative lowering of vapour Initially 1 mol 0 0
pressure. After dissociation 1- 0.3 0.3 0.3

26. (a) A graph between partial pressure of the gas versus Total moles = 1 - 0.3 + 0.3 + 0.3 = 1.3
mole fraction of the gas in solution is drawn, then we get 1.3
i= = 1.3
plot of straight line. 1
p - ps DTf = iKf m = 1.3 ´ 1.85 ´ 0.2 = 0.481° C
27. (a) Raoult’s law 0 = n2
p0 \ Tf = 0 - 0.481° C = - 0.481° C
p
1 - s = n2 39. (c ) 2CH 3 COOH l (CH 3 COOH)2
p0
Initially 1 0
ps
= 1 - n 2 = n1 After dissociation 1- a a/ 2
p0
a
ps = n1 ´ p 0 i = 1-
2
28. (c) KCl < CH3OH < CH3CN < Cyclohexane or a = 2 (1 - i ) = 2 (1 - 0.52) = 0.96
29. (a) Since, CH2Cl2 is a more volatile component than 40. (b) 4A r ( A )4
a
CHCl3, [p°CH Cl = 415 mm Hg and p°CHCl3 = 200 mm Hg] 1- a
2 2 4
and the vapour phase is also richer in [y CH2Cl2 = 0.82 and
a
[y CHCl3 = 0.18] 1- a +
i= 4 = 1- 3 a
30. (a) More the number of particles and concentration higher 1 4
is the elevation in boiling point. Thus, 0.08 m KCl
where, a = degree of dissociation = 1 = 100%
= - 0.08 ´ 2 = 0.24 m (highest) 3
31. (a) 32. (b) \ i = 1 - = 0.25
4
R.Tb2 2 ´ 373 ´ 373 ´ 18 41. (a) van’t Hoff factor (i ) = 4 { 3K + + (Fe(CN)6] 3-}
33. (a) Kb = = = 0.516° C
1000 . Lv 1000 ´ 9700 0.1 ´ 1000 1
Molality = =
34. (a) Relative lowering of vapour pressure of an aqueous 329 ´ 100 329
solution containing non-volatile solute a = is mole fraction 1
of solute - DTf = iKf m = 4 ´ 186
. ´ = 2.3 ´ 10-2
329
p 0 - ps n
As = = 0.0125 Þ DTf = -2.3 ´ 10-2C
p0 n +N
(As freezing point of water is 0°C)
n +N 1 N 1- 1 0.9875
Þ = - = = 42. (a) 0.004 M Na 2SO4 solution is isotonic with 0.01 M
N 0.0125 n 0.0125 0.0125
N 0 .9875 solution of glucose, so their osmotic pressures are equal
So = to each other. Osmotic pressure of 0.01 M glucose,
n 0.0125
0.0125 ´ 1000 p glucose = CRT = 0.01 ´ 0.0821 ´ T
Now, molality = = 0.70
0.9875 ´ 18 = ( p obs )Na 2SO4 = p glucose = 0.01 ´ 0.0823 ´ T

@iitjeehelps
SOLUTION 501
Na 2SO4 r 2Na+ + SO 24- 46. (d)
Initially 1 0 0 47. (c) p = iCRT p = inRT / V
After dissociation 1- a 2a a
pV 0.75 atm ´ 2.5 L
( p cal ) Na 2SO4 = CRT = 0.004 ´ 0.0821 ´ T n= =
iRT 2.47 ´ 0.0821 L atm K -1 mol-1 ´ 300 K
( p )Na 2SO4 = 0.0308 mol
By van’t Hoff factor, i = obs
( p cal )Na 2SO4
Molar mass of CaCl2 = 111g mol-1
Number of particles after dissociation 1- a + 2a + a
= = Amount dissolved = 0.0308 ´ 111 = 3.42 g
Number of particles before dissociation 1
48. (a) HA r H+ + A -
0.01 ´ 0.0821 ´ T 1+ 2 a
= C 0 0
0.004 ´ 0.0821 ´ T 1
C (1- a) Ca Ca
10 1 + 2a 10 - 4 [ H+ ] = Ca = 10- 2
= Þ a= = 0.75
4 1 8
\ Percentage of a = 75% pH = 2
10-2
43. (c) C a = 10-2 Þ a = = 0.1
44. (c) As we know, DTf = i ´ Kf ´ m 0.1
Total number of moles after dissociation
where, Kf and m are constants.
= C - Ca + Ca + Ca = C(1 + a )
DTf µi
+ -
(a) KCl(aq ) r K (aq ) + Cl (aq ); (i = 2) C(1 + a )
van’t Hoff factor = = 1 + a = 1 + 0.1 = 11
.
(b) C6H12O6 r No ions C
(c) Al2(SO4 )3 (aq ) r 2 Al3+ + 3SO24- ; (i = 5) p = iCRT = 11
. ´ 0.1 ´ 0.0821 ´ 298 = 2.69 atm
49. (b) We know that
(d) K 2SO4 (aq ) r 2K + + SO24- ; (i = 3)
100Kb ×w
Hence, Al2(SO4 )3 will exhibit largest freezing point m=
DTb × W
depression due to the highest value of i.
100 ´ 5.2 ´ 6
45. (c) Elevation of boiling point, = = 60
0.52 ´ 100
w ´ Kb ´ 1000
DTb = 50. (a) The boiling point of a solution is always higher than
M ´ W (gram)
that of the boiling point of pure solvent.
On substituting values, we get
A ® Boiling point of solvent
w ´ 0.5 ´ 1000 0.05 ´ 100 ´ 100
0.05 = or w = = 1g B ® Boling point of solution
100 ´ 100 0.5 ´ 1000

BITSAT Archives
1. (d) 1000 Kf w
6. (b) m =
DTfW
2. (b) From Raoult’s law
p° - ps = p° ´ mole fraction of solute 1000 Kf w 1000 ´ 1.86 ´ 342
DTf = = = 1.86° C
mW 342 ´ 1000
10 = p° ´ 0.2 Þ 20 = p° ´ c 2
\The solution will freezes at = 0 - 1.86° C = - 1.86° C
\ c 2 = 0.4 and c1 = 1 - 0.4 = 0.6
7. (a)
c1 = mole fraction of solvent
8. (b) p urea = 3.0 atm, p cane sugar = 1. 2 atm
1000 ´ Kb ´ w
3. (b) m = Total = 4 . 2 atm
W ´ DTb
m ´ W ´ DTb 100 ´ 100 ´ DTb 9. (c)
or Kb = = = DTb
1000 ´ w 1000 ´ 10 10. (b) Osmotic pressure ( p ) = CRT

4. (c) Here, C = concentration of solution


n 68.4
5. (d) DTb = iKb × m = 2 ´ 0.52 ´ 0.1 = 0.104° C C= =
V 342
Tb = 100 + 0.104° C 68.4
p= ´ 0.082 ´ 273 = 4.48 atm
= 100.104° C 342

@iitjeehelps
11
Adsorption and
Colloidal System

Concept of Surface Chemistry


Surface chemistry deals with the phenomenon that occurs at the surface or interfaces. There are
several properties of substances, particularly of solids, liquids and solids, gases, which depend upon
the nature of the surface of interface.

Concept of Adsorption
Retaining of the molecular species on the surface of the solid is known as adsorption. The solid
substance on the surface of which adsorption occurs is called adsorbent. The molecular species that
get adsorbed on the solid surface due to intermolecular attractions are called adsorbate.
Rate of adsorption decreases with time whereas rate of absorption remains constant with time.

Characteristics of Adsorption
(i) Adsorption occurs due to unbalanced residual force on surface which allows to hold the
adsorbate particle. These forces are generally chemical forces or physical van der Waals’ forces.
(ii) Adsorption occurs by decreasing DG° , DS ° and DH° with condition DH has more negative value
in comparison to TDS .
DG < 0, DH < 0, DS < 0, -TDS < DH
(iii) Extent of adsorption is directly proportional to surface area, i.e. increases with surface area.
That’s why finely divided metals and porous materials are better adsorbents.
NOTE At high temperature, physisorption can be converted into chemisorption. DH = - ve hence, adsorption is
always an exothermic process.

@iitjeehelps
ADSORPTION AND COLLOIDAL SYSTEM 503

Classification of Adsorption Effect of Temperature on Freundlich


Adsorption Isotherm
Adsorption is of two types:
It has been found that as temperature increases, the extent
Types of adsorption of adsorption decreases, in
Effects of Pressure on Adsorption
Physisorption Chemisorption 1 x
Case I At high pressure,= 0 or = k (constant)
● Adsorbate molecules are held ● Adsorbate are held by chemical
n m
by weak van der Waals’ forces forces. At this condition, graph becomes almost constant
of attraction. and follow zero order kinetics.
● It is reversible process. ● It is irreversible process. 1 x
● Low temperature is favourable. ● High temperature is favourable. Case II At low pressure, = 1 or = k p
n m
It decreases with increase of It increases with increase of
temperature. temperature. x
i.e. µp
● Less specific in nature. ● More specific in nature. m
● Heat of adsorption is very low ● Heat of adsorption is about Thus, the extent of adsorption is directly
about 5 kcal mol -1 . 20-80 kcal mol -1 . proportional to pressure which follow first order
● There is no appreciable ● High activation energy is kinetics. The graph between extent of adsorption
activation energy is involved. required.
and pressure is a straight line.
1
Case III At intermediate range of pressure, the value of
Adsorption Isotherm varies between 0 and 1, the graph can be n
The variation of extent of adsorption with pressure at expressed as
constant temperature is known as adsorption isotherm. T=constant
x
Freundlich Adsorption Isotherm m
=kp0
x 1
According to this isotherm, the extent of adsorption is x
m =kp n
m Saturation
related to the pressure at constant temperature by the given x pressure, ps
formula, =kp
m
x p
= k × p 1/ n ...(i) General adsorption isotherm
m
x
where, = extent of adsorption Langmuir Adsorption Isotherm
m
In order to explain the relation between extent of adsorption
p = pressure and pressure, Langmuir introduced an adsorption isotherm on
k and n are constants which depend upon the nature of the basis of the following assumptions :
adsorbent and gas. (i) Adsorption is monolayered.
On taking log on both sides of Eq.(i), we get (ii) Interaction between adjacent adsorbed molecule is
x 1 negligible.
log = log k + log p
m n (iii) Adsorption is uniform throughout the surface.
(iv) At equilibrium, rate of adsorption is equal to rate of
desorption.
x 1 On the basis of Langmuir adsorption isotherm, the fraction
log Slope =
m n of surface covered by the adsorbed molecules (q)
represented as
{log k = intercept} Kp
(i) At equilibrium, q =
1+ K p
log p
Plot of log
x
vs log p where, K = equilibrium constant, known as adsorption
m coefficient
1 k Rate constant for adsorption
The Freundlich isotherm is valid for the value of in = a =
n kd Rate constant for desorption
between 0 to 1. Probable range is 0.1 to 0.5, where maximum K¢p
adsorption is observed. (ii) q = , where, K ¢ = kK = constant
1 + Kp

@iitjeehelps
504 SELF STUDY GUIDE BITSAT

Pressure Dependence of Langmuir Adsorption 2. Heterogeneous Catalyst


Let us, consider the mathematical relation When the catalyst is in different phase than that of the
x K¢ p æ x ö reactants. It is called heterogeneous catalyst.
= çQ µ q ÷ Important examples are :
m 1+ K p è m ø
Fe( s )
3H 2 ( g ) + N 2 ( g ) ¾¾® 2NH3 ( g )
Case I At high pressure, 1 + Kp » Kp
Cu( s )
x K¢ p K¢ CO( g ) + 2H 2O( g ) ¾¾¾® CH3OH ( g )
= = = constant ZnC /Cr 2O 3
m Kp K
Enzyme Catalysis
Thus, at high pressure, the extent of adsorption
approaches to a limiting value. Enzymes are complex nitrogenous organic compounds
which are produced by living plants and animals. They
Case II At low pressure, catalyse numerous biochemical reactions. The enzymes are
The Kp becomes negligible in comparison to thus, termed as biochemical catalysts and the phenomenon
1 + K p =k p is known as biochemical catalysis.
x
= K ¢ p,
x
µp
Mechanism
m m Michaelis and Menton suggested the following steps for
Thus, at low pressure, the extent of adsorption is enzyme catalysis :
directly proportional to pressure. Step I Binding of enzyme to substrate to form an activated
Case III When pressure is moderate, then expression is in complex. k1
x 1 E + S r ES
the form of = Kp 1/ n , where, lies between 0 k -1
m n
and 1. Step II Product formation in the activated complex.
k2
Adsorption from Solutions ES r EP
Freundlich adsorption isotherm, Step III Decomposition of EP into products and enzyme
x again.
= kC 1 / n (n > 1) EP ¾® P + E
m
Langmuir adsorption isotherm, Colloidal State
x aC A substance is said to be in the colloidal state, when it is
=
m (1 + bC ) dispersed in another medium in the form of very small
Langmuir adsorption isotherm is applicable only to particles having diameter between 10-4 to 10-7 cm, i.e. 1 to
chemisorption. 100 nm.

Types of Colloidal System


Catalysis The dispersed phase or dispersion medium can be a gas,
The term catalyst defined as a substance which alter the liquid or solid. There are eight types of colloidal system.
rate of reaction without taking part in it and phenomenon is These are as follows:
called as catalysis.
S. Colloidal Dispersion Dispersion Examples
Catalyst
No. system phase medium
1. Foam Gas Liquid Soda water, froth,
shaving cream
Positive catalyst Negative catalyst Auto catalyst Induced catalyst 2. Solid foam Gas Solid Foam rubber, cork
(If a catalyst (If a catalyst (When one of the (One of the 3. Aerosol Liquid Gas Fog, mist, clouds
increases the decreases the products reactants act
rate of reaction) rate of reaction) act as catalyst) as catalyst) 4. Emulsion Liquid Liquid Milk, hair cream
5. Solid emulsion Liquid Solid Butter, cheese
(gel)
Types of Catalyst
6. Aerosol of Solid Gas Dust in air, smoke
1. Homogeneous Catalyst solids

When the catalyst are in the same phase as the reactants 7. Sol Solid Liquid Paint, ink, colloidal gold,
and products then it is called homogeneous catalyst and the 8. Solid sol Solid Solid Ruby glass some
process is called homogeneous catalytic reaction or gemstones, alloys, rock
salt
homogeneous catalysis.

@iitjeehelps
ADSORPTION AND COLLOIDAL SYSTEM 505
Classification of Colloids (ii) Electrical dispersion It is done by applying electric
spark between the electrodes of metal which vaporises
Colloids can be classified in a number of ways based upon metal and converts metal into colloidal size. This
some of their important characteristics as given below : process is known as Bredig’s arc method.
Based on Affinity of Phase e.g. Gold sol is prepared by this method.

1. Lyophilic colloids represent such colloidal systems (iii) Peptisation This is a process of conversion of
in which the particles of dispersed phase have great precipitate to the colloidal solution by means of
affinity for the dispersion medium. These are reversible shaking the solution of precipitate in the presence of
colloids, e.g. gum, gelatin, rubber, proteins etc. small amount of electrolyte. Such electrolyte is known
as peptising agent.
2. Lyophobic colloids represent such colloidal systems
e.g. Fe (OH) 3 + FeCl3 ¾® Sol
in which particles of the dispersed phase have no
affinity for the dispersion medium. These are
Chemical Methods
irreversible, e.g. sols of metals and their insoluble
compounds like sulphides and oxides. Some important examples are as follows :
If water is the dispersion medium, the terms used are (i) Double decomposition of As2O3
hydrophilic and hydrophobic colloids.
As 2 O3 + 3H 2 S ¾® As 2 S3 + 3 H 2 O
Lyophobic colloids are less stable due to the presence (Yellow sol)
of electric charge on their particles. On the other hand,
lyophilic colloids are stable due to the presence of (ii) Oxidation
charge as well as their extensive solvation. 2H 2S + SO2 ¾® 2H 2O + 3S ¯ (sol)
(iii) Reduction of AgNO3 and AuCl3 by tannic acid
Based on Molecular Size
AgNO3 + Tannic acid ¾® Ag sol
1. Multimolecular colloids are the colloids in which
colloidal particles consist of aggregate of atoms or AuCl3 + Tannic acid ¾® Au sol
small molecules with diameter less than 10-9 m or (iv) Hydrolysis of FeCl3
1 nm, e.g. a sol of gold, a sol of sulphur. FeCl3 + 3H 2O ¾® Fe(OH)3 + 3HCl
2. Macromolecular colloids are the colloids in (Red sol)
which colloidal particles themselves are large
Properties of Colloidal Solutions
molecules.
3. Associated colloids or micelles are the substances The important properties of the colloidal solutions are given
which behave as normal electrolytes at low below :
concentration but as colloids at higher concentration.
Brownian Movement
This is because at higher concentration, they form
associated particles called micelles, e.g. soap and Colloidal particles are always in a state of rapid random
synthetic detergents. motion, which is termed as Brownian movement.
The concentration above which micelle formation Tyndall Effect
occurs, is called CMC (critical micelle concentration) When a strong and converging beam of light is passed
and the temperature above which micelle formation through a colloidal solution, its path becomes visible due to
occurs is called Kraft temperature.
scattering of light by particles. It is called Tyndall effect.
Preparation of Colloid Electrophoresis
The various methods of preparation of colloids are as The phenomenon involving the migration of colloidal
follows: particles under the influence of electric field towards the
oppositely charged electrode, is called electrophoresis.
Dispersion Methods
The breaking down of a large solid particles into small Coagulation or Flocculation
colloidal particles by using mechanical or electrical The precipitation of particles of the dispersed phase in a sol
methods are known as dispersion methods. is known as coagulation.
(i) Mechanical dispersion It is done by using colloidal The minimum amount of an electrolyte required to cause
mill which causes breaking of large colloidal particles. precipitation of one litre of a colloidal solution is called
e.g. Preparation of colloidal graphite (used as coagulation value or flocculation value. The reciprocal of
lubricant), preparation of printing ink. coagulation value is regarded as the coagulating power.

@iitjeehelps
506 SELF STUDY GUIDE BITSAT

Hardy-Schulze Rule liquids. Generally, one of the two liquids is water and the
Higher the valency of the active ion, greater will be its liquid other than water is designated as oil.
power to precipitate the sol. Thus, the two types of emulsion may be formed as :
e.g. Order of coagulating power is 1. Oil dispersed in water (O/W type), e.g. milk.
3+ 2+ +
Al > Ba > Na , PO34- > SO24- > Cl - 2. Water dispersed in oil (W/O type), e.g. butter, cream.

Gold Number Characteristics of Emulsion


Protective power of any lyophilic colloid is determined in The main characteristics of emulsion are as follows :
terms of gold number which is defined as “the milligrams of (i) Commonly emulsions are unstable and stabilised by
lyophobic colloid required to just prevent the precipitation adding some third substance called emulsifying agents.
of 10 mL of gold sol on the addition of 10 mL of 10% NaCl
Some emulsifying agents are as follows:
solution. Smaller the gold number of hydrophilic colloids,
greater is its protective power. (a) O/W type emulsion, e.g. protein, gum, natural and
synthetic soaps.
Emulsion (b) W/O type emulsion, e.g. lamp black, metal salt of
fatty acids, long chain alcohols.
Emulsion is a type of colloidal solution made by the
addition or mixing of two immiscible or partially miscible (ii) Droplets of emulsion are often negatively charged.

Practice Exercise
1. The substance that gets adsorbed on the surface of a. decrease in entropy b. decrease in enthalpy
the solid is called c. decrease in free energy d. None of these
a. adsorbate b. adsorbent 10. Amount of gas adsorbed per gram of adsorbent
c. micelle d. inner phase increases with pressure, but after certain limit is
2. Adsorption is multilayered in the case of reached, adsorption becomes constant, it is due to
a. physical adsorption b. chemisorption a. multilayers are formed b. desorption takes place
c. Both (a) and (b) d. None of these c. temperature is increased d. absorption also starts
3. Which is favourable for van der Waals’ adsorption? 11. Which of the following is the variation of physical
a. High temperature, low pressure adsorption with temperature?
b. Low temperature, high pressure
Adsorption

Adsorption

c. Low temperature, low pressure


d. High temperature, high pressure a. b.
4. In physical adsorption, the forces associated are
a. ionic b. covalent T T
c. van der Waals’ d. H-bonding
5. Which adsorption takes place at higher temperatures?
Adsorption

Adsorption

a. Physical b. Chemical c. d.
c. Both (a) and (b) d. None of these
6. Which is adsorbed in minimum amount by the T T
activated charcoal?
a. H2 b. CO2 c. SO3 d. CO 12. According to Freundlich adsorption isotherm, which of
the following is correct?
7. Which of the following gas is adsorbed in maximum x
amount by charcoal? a. µ p°
m
a. SO2 b. CO2 c. CO d. Water vapour x
b. µ p1
8. Which is true about chemisorption? m
a. It is reversible in nature x
b. It is usually occurs at low temperatures c. µ p1/n
m
c. It is highly specific in nature d. All of the above are correct for different ranges of
d. The attractive forces between adsorbate and pressure
adsorbent are van der Waals’ forces
9. Adsorption is accompanied by

@iitjeehelps
ADSORPTION AND COLLOIDAL SYSTEM 507
13. 1g of charcoal adsorbs 100 mL of 0.5 M CH 3 COOH to 20. ‘‘Rate of adsorption is directly proportional to the
form a monolayer, and thereby the molarity of fraction of area uncovered and rate of desorption is
CH 3 COOH reduces to 0.49 M. What is the surface directly proportional to the fraction of area covered’’.
area of charcoal used by each molecule of acetic This statement is true for
acid? a. Freundlich adsorption isotherm
b. Langmuir adsorption isotherm
[Surface area of charcoal = 3.01 ´ 102m 2 / g] c. BET isotherm
a. 6.02 ´ 10-30 m 2 d. None of the above
b. 5.00 ´ 10-19 m 2 21. The correct relationship which represents the
c. 3.01´ 10-2 m
2
Freundlich adsorption isotherm is
-19 2
d. 2.00 ´ 10 m æx ö 1
a. log ç ÷ = log k + log p
èm ø n
14. Which of the following is not the true about the æm ö 1
adsorption? b. log ç ÷ = log k + log p
èx ø n
a. During the process of adsorption, residual attractive æx ö 1
forces decreases c. log ç ÷ = log p + log p
èm ø k
b. During the adsorption surface energy decreases
c. It is an exothermic process æx ö 1
d. log ç ÷ = log c + log k
d. It is an endothermic process èm ø n
15. Which of the following statements are correct? 22. Which of the following is correct statement?
I. Silica gel adsorbs water molecules. a. Physisorption occurs at low temperature and
II. Anhy. CaCl2 adsorbs the water molecules. chemisorption occurs at high temperature
b. Physisorption occurs at very high temperature and
III. Adsorption is a surface phenomenon.
chemisorption occurs at low temperature
a. I and II b. II and III
c. Physisorption is irreversible and chemisorption is
c. I and III d. All of these
reversible
16. Which of the following process will be observed, when d. None of the above
a chalk stick is dipped in the solution of ink? x
a. Adsorption b. Absorption 23. For Freundlich adsorption isotherm, = kp 1/ n , the
m
c. Desorption d. Both (a) and (b)
value of n is
17. A mixture of sand and water is an example of a. always greater than 1
a. true solution b. always smaller than 1
b. colloidal solution c. always equal to 1
c. suspension d. greater than 1 at low temperature and smaller than 1
d. All of the above at high temperature
18. Freundlich adsorption isotherm is represented at 24. In general, H2 gas is adsorbed on activated charcoal
temperatures T1, T2 and T3 . to a less extent in comparison to the easily liquefiable
gases due to
a. very strong van der Waals’ force and low critical
temperature
T1 b. very weak van der Waals’ force and low critical
x/m T2 temperature
T3 c. very strong van der Waals’ force and high critical
temperature
d. very weak van der Waals’ force and high temperature
p 25. Which of the following reaction requires catalyst?
Arrange the temperatures in the increasing order as a. S + O2 ¾® SO2
shown in the graph. b. 2SO2 + O2 ¾® 2SO3
a. T1 < T2 < T3 b. T3 < T2 < T1 c. C + O2 ¾® CO2
c. T2 < T3 < T1 d. T3 < T1 < T2 d. All of the above

19. Identify the gas which is readily adsorbed by activated 26. Hydrolysis of sugar to glucose and fructose is
charcoal? catalysed by
a. H2 b. N2 a. invertase b. zymase
c. SO2 d. O2 c. lactic bacilli d. diastase

@iitjeehelps
508 SELF STUDY GUIDE BITSAT

27. The efficiency of an enzyme in catalysing a reaction is 40. Which of the following has minimum gold number?
due to its capacity a. Gelatin b. Egg albumin
a. to form an enzyme substrate complex c. Gum arabic d. Starch
b. to decreases the bond energies of the substrate 41. Which of the following represents homogeneous
molecules
catalysis?
c. to change the shape of the substrate molecule Ni
d. None of the above a. Oil + H2 ¾¾ ® Saturated fat
b. N2 + 3H2 ¾ Fe
¾® 2NH3
28. In Zeigler-Natta polymerisation of ethylene, the active +
species is c. CH3COOH + C2H5OH ¾ H¾
¾® CH3COOC2H5 + H2O
a. AlCl3 b. Et 3Al d. None of the above
c. CH2CH2 d. Ti3+
42. Which of the following is not a colloid?
- a. Chlorophyll b. Smoke
29. Following reaction is catalysed by Br (aq )
c. Ruby glass d. Milk
2H2O2(aq ) ¾® 2H2O(I ) + O2( g )
This is an example of 43. Lyophilic colloids are stable due to
a. homogeneous catalysis b. heterogeneous catalysis a. charge on the particles
c. Both (a) and (b) d. None of these b. large size of particles
c. small size of particles
30. Vanishing cream is an example of d. layer of dispersion medium on the particle
a. solution b. foam
c. lyophilic solution d. emulsion 44. Which has the maximum coagulating power for ferric
hydroxide colloid?
31. Colloidal solution of gold is prepared by a. AlCl3 b. K 4[Fe (CN)6]
a. colloidal mill c. BaSO4 d. MgO
b. double decomposition method
c. Bredig’s arc method 45. Gold number is the index for
d. peptisation a. protective power of lyophilic colloid
b. purity of gold
32. When a beam of light is passed through colloidal c. metallic gold
solution, d. electroplated gold
a. it gets scattered b. it gets adsorbed
c. it is refracted d. it undergoes reflection 46. Blue colour of sky and red colour of sunsets are due to
a. scattering of light from the sun
33. Fog is a colloidal solution of b. scattering of light from particles of dust in the
a. gas in gas b. solid in gas atmosphere
c. liquid in gas d. None of these c. refraction of blue light by impurities in sea water
34. Which property is not shown by colloids? d. scattering of light due to ozone layer
a. Adsorption b. Tyndall effect 47. When dilute aqueous solution of AgNO3 (excess) is
c. Flocculation d. Paramagnetism
added to KI solution, positively charged sol particles of
35. The path of a beam of light through smoke is visible AgI are formed due to adsorption of ion
because a. K + b. Ag+ c. I- d. NO-3
a. carbon dioxide in the smoke scatters light
b. carbon dioxide in the smoke absorbs light 48. Gold sols and sulphur sols are the examples of
c. carbon particles in the smoke absorb light a. multimolecular colloids
d. carbon particles in the smoke scatter light b. macromolecular colloids
c. associated colloids
36. Four different colloids have the following gold number, d. All of the above
Which one has most effective action?
a. 10 b. 30 49. Colloidion is a
c. 20 d. 40 a. 100% solution of nitrocellulose
b. 10% solution of nitrocellulose in mixture of alcohol
37. In coagulating the colloidal solution of As 2S 3 , which and ether
has the minimum coagulation value? c. 4% solution of nitrocellulose in mixture of alcohol and
a. NaCl b. KCl c. BaCl2 d. AlCl3 ether
d. 1% solution of nitrocellulose in mixture of alcohol and
38. Which of the following indicates the charge on ether
colloidal particles?
a. Brownian movement b. Electrophoresis 50. Lyophilic solution is coagulated by
c. Electrolysis d. Tyndall effect a. adding an electrolyte
b. adding a suitable solvent
39. Which of the following is not a colloid? c. Both (a) and (b)
a. Milk b. Blood c. Latex d. Vinegar d. None of the above

@iitjeehelps
BITSAT Archives
1. The gold number of a few protective colloids are given 4. Which of the following is an example of homogeneous
x = 0.005, y = 3.5, z = 40 catalysis? [2010]
a. Haber’s process for the synthesis of NH3
The protective nature of these colloidal solutions
b. Catalytic conversion of SO2 to SO3 in contact process
follow the order : [2014]
c. Catalytic hydrogenation of oils
a. z > x > y b. x < y < z d. Acid hydrolysis of methyl acetate
c. z > y > x d. x > y > z
5. Which of the following graphs represents Freundlich
2. When equal volume of each of two sols of AgI, one adsorption isotherm? [2008]
obtained by adding AgNO3 to slight excess of KI and
another obtained by adding KI to slight excess of
AgNO3 are mixed together. It is observed that [2013]
a. log x b. log x
a. the sol particles acquired more electric charge m m
b. the sols coagulated each other mutually
c. a true solution is obtained log p log p
d. the two sols stabilised each other
3. Which of the following will be the most effective in the
coagulation of Fe(OH)3 sol? [2012] c. log x d. log x
m m
a. Mg3(PO4 )2 b. BaCl2
c. NaCl d. KCN log p log p

Answer with Solutions


Practice Exercise 11. (b) Adsorption of gases decreases with increase in
temperature.
1. (a)
12. (d)
2. (a) Adsorption is multilayered in case of physical
13. (b) 100 mL of 0.5 CH3COOH contains
adsorption while it is almost monolayered in case of
chemical adsorption. CH3COOH = 0.05 mol
3. (b) van der Waals’ adsorption is the physical adsorption After adsorption, CH3COOH remained = 0.049 mol
which is favoured by low temperature as the phenomenon \Acetic acid adsorbed = 0.001mol = 6.02 ´ 1020 molecules
is exothermic. Increase in pressure favours the
\Surface area of charcoal adsorbed by each molecule
adsorption. 2
3.01´ 10 2 m
4. (c) Adsorbate molecules are held by weak van der waals’ = = 5 ´ 10-19m2
force of attraction. 6.02 ´ 1020
5. (b) Chemical adsorption takes place at higher 14. (d)
temperature, in comparison of physical adsorption. 15. (b) Anhy. CaCl2 absorbs moisture and adsorption is a
6. (a) The gas, which is difficult to liquefy (i.e. having very low surface phenomenon.
critical temperature), is adsorbed in less amount and 16. (d) When a chalk stick is dipped in ink, the surface retains
vice-versa. Here, hydrogen has the minimum critical the colour of ink due to adsorption of coloured molecules
temperature, (-240° C) hence, is adsorbed in minimum while the solvent of the ink goes deeper into the stick due
amount. to absorption.
7. (a) Stronger the intermolecular forces, easier to liquefy 17. (c)
and more strongly. It is adsorbed.
x
8. (c) Chemisorption occurs through formation of chemical 18. (a) shows, extent of adsorption. Extent of adsorption
m
bonds, hence it is very specific in nature. increases by decrease of temperature. Order of temperature
9. (d) T1 < T2 < T3
10. (a) Langmuir showed that at low pressure, the physically 19. (c) Easily liquefiable gases like SO2, NH3 have greater
adsorbed gas forms only one molecule thick layer.
However, above a certain pressure, multimolecular thick value of critical temperature than elemental gases, i.e.
layer is formed. N2, O2, H2 thus, readily get adsorbed.

@iitjeehelps
510 SELF STUDY GUIDE BITSAT

20. (a) 21. (a) 37. (d) As2S3 is a negative sol Hence, AlCl3 with
22. (a) At high temperature, kinetic energy of adsorbate untrialent Al3+ has least coagulation value.
increases and make a strong chemical bond between 38. (b) Electrophoresis shows that the colloidal particles have
adsorbate and adsorbent, hence chemisorption occurs. charge on them as they move towards oppositely charged
1 electrode, when electric field is applied on them.
23. (a) If n > 1, then, < 1
n 39. (d) Vinegar is approximately 10% solution of acetic acid in
\n is always greater than 1. water.
24. (b) H2 is a permanent gas, hence, has low value of critical 40. (a) Gelatin is the best protective colloid as it has the least
temperature and low van der Waals’ force. Due to these gold number (about 0.005).
factors, it shows low extent of adsorption. 41. (c)
25. (b) 26. (a) 42. (a) Chlorophyll is not a colloid.
27. (a) Enzyme + Substrate ¾®[Complex intermediate] 43. (d)
¾® [Product +enzyme] 44. (b) Ferric hydroxide is a positive sol, [Fe(OH)3]Fe3+ .
28. (d) (C2H5 )3 Al + TiCl4 ¾® Active species Hence, to coagulate it, an anion is needed. K 4[Fe(CN)6]
Ti3+ has one active site vacant and thus accommodate gives the tetravalent anion [Fe(CN)6] 4- , hence it has the
one alkyl group (as (C2H5 )3 Al reduces TiCl4 to TiCl3). greatest coagulating power among these (Hardy-Schulze
- law).
29. (a) 2H2O2(aq ) ¾ Br
¾ (aq)
¾¾® 2H2O(l ) + O2(g )
45. (a) Protective character of various lyophilic substances is
This is an example of homogeneous catalysis. expressed in terms of gold number.
30. (d) Vanishing cream is an oil in water (O in W) type 46. (b) It is due to scattering of light from particles of dust in
emulsion.
the atmosphere.
31. (c) Colloids of metals are generally prepared by Bredig's
47. (b) Sol particles possess the tendency to adsorb
arc method.
preferentially the common ion present in solution.
32. (a)
48. (a) Gold sol and sulphur sol are the examples of
33. (c) Fog is colloidal solution of water droplets (liquid)
multimolecular colloids.
dispersed in air (gas) dispersion medium.
49. (c) The usual colloidion is a 4% solution of nitrocellulose
34. (d) Paramagnetism is not shown by colloids.
in the mixture of alcohol and ether.
35. (d)
50. (c) Lyophilic sol is coagulated by adding a suitable
36. (a) Lesser the gold number, greater will be its protecting
solvent or an electrolyte.
efficiency.

BITSAT Archives
1. (d) Smaller the gold number, greater is its protective \It will be coagulated by anion.
power, Hence, the order would be (a) Mg3(PO4 )2 r 3Mg2+ + 2PO34-
x >y > z (b) BaCl2 r Ba 2+ + 2Cl-
Gold number is used for calculating the protective powers (c) NaCl r Na + + Cl-
of lyophilic colloids.
(d) KCN r K + + CN-
2. (b)
4. (d) During acid hydrolysis of methyl acetate, reactants
3. (a) According to Hardy Schulze rule, coagulation power of and catalyst (H2SO4 ) are in the same phase. Thus, it is an
ions is directly proportional to charge on ion. example of homogeneous catalysis.
Q Fe(OH)3 is positively charged colloid. 5. (c) In Freundlich isotherm

@iitjeehelps
12
Redox Reactions

Concepts of Oxidation and Reduction


Loss of electron by an atom is called oxidation or de-electronation while gain of electron by an atom is
called reduction or electronation.
Oxidants or oxidising agents are the substances which
(i) oxidise other, (ii) get reduced ,
(iii) gain electrons (i.e. their oxidation number decreases during a reaction)
Reductants or reducing agents are the substances which
(i) reduce others, (ii) get oxidised,
(iii) loss electrons (i.e. their oxidation number increases during a reaction)

Oxidation Number
The real or imaginary charge, which an atom appears to have in its combined state is called oxidation
number of that atom. Valency of an element is always a whole number. It can neither be zero nor
fractional while oxidation number may be positive or negative. It can be zero or fractional.
Fractional oxidation state is only the average oxidation state of an element when two or more of its
atoms are present in different oxidation states in a given compound.

Rules for Assigning Oxidation Number


The oxidation number of an element or atom can be calculated with the help of following rules:
(i) The oxidation number of an element in its elementary state is zero, e.g. H 2 , S8 and P4
(ii) Oxidation number of an ion is equal to the electrical charge present on it.
(iii) Oxidation number of a compound is zero.
(iv) Oxidation number of fluorine is always −1 in all of its compounds.
(v) The oxidation number of alkali metals is always +1 and those of alkaline earth metals is +2.

@iitjeehelps
512 SELF STUDY GUIDE BITSAT

(vi) Oxidation number of hydrogen is +1 except in ionic Balancing of Redox Reaction


hydrides, where it is −1 .
(vii) Two oxidation numbers of N are −3 and +3, when it is 1. Oxidation Number Method
bonded with less electronegative and more In balancing redox reaction, following steps are used:
electronegative atoms respectively.
(i) Indicate the oxidation number of all the atoms
(viii) Oxidation number of oxygen is −2 except in OF2 ( + 2), involved in the equation and identify the
O2 F2 ( + 1) , peroxides ( −1) and superoxides ( −1/2 ). elements which undergo change in oxidation
(ix) The oxidation number of halogens is always −1 in metal number.
halides. (ii) Calculate the increase and decrease in oxidation
(x) In interhalogen compounds, more electronegative metal number per atom with respect to the reactants if
of the two halogens gets the oxidation number of −1. more than one atom is involved, then multiply
with the number of atoms undergoing the
change to calculate the total change in oxidation
Redox Reactions number.
The reaction which involves oxidation and reduction as its two
(iii) Equate the increase and decrease in oxidation
half-reactions is called redox reactions. A redox change occurs
simultaneously. number on the reactant side by multiplying the
suitable integers.

Reduction (+2e )
(iv) Balance the equation with respect to all atoms
Sn2++ 2Fe3+ 2Fe2++ Sn4+ except hydrogen and oxygen.
– (v) Finally, balance hydrogen and oxygen also.
Oxidation (–2e )
(vi) In acidic medium, hydrogen and oxygen are
Types of Redox Reactions balanced by H + and H 2O respectively while in
These are of three types as follows : basic medium, these are balanced by using
OH − and H 2O .
1. Intermolecular Redox Reactions
These involve the reaction between two substances, one of them is 2. Ion-Electron Method
oxidant and other is reductant, In this method, the following steps are used :
e.g. 10FeSO4 + 2KMnO4 + 8H 2SO4 → (i) Indicate the oxidation number of all the atoms
Reductant Oxidant
involved in the reaction and determine the
2MnSO4 + 5Fe 2 (SO4 )3 + K 2SO4 + 8H 2O oxidising agent and reducing agent.
2. Intramolecular Redox Reactions (ii) Now, split the reaction into oxidation
These involve oxidation of one element of a compound as well as half-reaction and reduction half-reaction.
reduction of other element of the same compound. (iii) Now, balance both the half-reactions in the
Decomposition reactions are also intramolecular redox reactions following ways:
but to be a redox reaction, it is essential that one of the (a) Balance all the atoms except H and O by
products of decomposition must be in the elemental state. multiplying the suitable integer.
e.g. (b) Now, balance H and O by using H + (in acidic
∆ medium) or by using OH − (in alkaline
(NH 4 )2 Cr2O7 → N 2 + Cr2O3 + 4H 2O
medium) and H 2O.
3. Autoredox or Disproportionation Reactions (c) Charge is balanced by using electrons in the
These involve oxidation and reduction of the same element, side deficient in negative charge.
0 −1 +5
∆ (iv) Now, combine both the half-reactions so as to
e.g. 3Cl 2 + 6NaOH(conc. ) → 5NaCl + NaClO3 + 3H 2O
cancel the electrons.

@iitjeehelps
Practice Exercise
1. When zinc is added to CuSO4 solution, copper is 12. Carbon has zero oxidation number in
precipitated. It is because of a. CH4 b. CH3Cl c. CCl4 d. CH2Cl2
a. reduction of zinc b. reduction of Cu2+
13. Oxidation states of iodine vary from
c. hydrolysis of CuSO4 d. reduction of SO2−
4 a. −1 to + 1 b. −1 to + 7
2. Which of the following is the most powerful oxidising c. +3 to + 5 d. −1 to + 5
agent?
a. F2 b. Cl2 c. Br2 d. I2 14. Oxidation number of S in Na 2S 4O6 is
a. 1. 5 b. 2 . 5 c. 3 d. 2
3. The strongest reducing agent is
a. HNO3 b. H2S c. H2SO3 d. SnCl2 15. The brown ring complex compound of iron is
formulated as [Fe(H2O)5 (NO)] SO4 . The oxidation
4. Which of the following is a reducing agent? state of iron is
a. NaHCO3 b. NaHSO3 c. Na 2O2 d. NaHSO4 a. 1 b. 2 c. 3 d. 0
5. In which one of the following reactions, hydrogen is 16. Oxidation number of oxygen in F2O is
acts as an oxidising agent? a. −1 b. +1 c. +2 d. −2
a. With Li to form LiH b. With I2 to form HI
17. The oxidation state of nitrogen in N3 H is
c. With N2 to form NH3 d. With S to form H2S
1 1
a. + b. +3 c. −1 d. −
6. Which of the following reactions has the underlined 2 3
substance been reduced?
a. Carbon monoxide + Copper oxide →
18. The oxidation state of chromium in Cr(CO)6 is
Carbon dioxide + Copper a. 0 b. + 2 c. −2 d. + 6
b. Copper oxide + Hydrochloric acid →
Copper chloride + Water 19. Phosphorus has the oxidation state + 3 in
c. Hydrogen + Iron oxide → Iron + Water a. orthophosphoric acid b. phosphorous acid
d. Steam + Iron → Iron oxide + Hydrogen c. metaphosphoric acid d. pyrophosphoric acid

7. Nitric oxide acts as a reducing agent in which of the 20. In which of the following, increasing orders the
following reaction? oxidation number of oxygen has been arranged?
a. BaO2 < O3 < OF2 < KO2
a. 4NH3 + 5O2 → 2NO + 6H2O
b. BaO2 < KO2 < O3 < OF2
b. 2NO + 3I2 + 4H2O → 2NO−3 + 6I− + 8H+ c. OF2 < KO2 < BaO2 < O3
c. 2NO + H2SO3 → N2O + H2SO4 d. KO2 < OF2 < O3 < BaO2
d. 2NO + H2S → N2O + S + H2O 21. The oxidation number of an element in a compound is
8. In acidic medium, equivalent weight of K 2Cr2O7 evaluated on the basis of certain rules. Which of the
following rules is not correct in this respect?
(molecular weight = M ) is
a. The oxidation number of hydrogen is always +1
a. M / 3 b. M / 4 c. M / 6 d. M / 2
b. The algebraic sum of all the oxidation numbers in a
9. In the reaction, I2 + 2S 2O23− → 2I− + S 4 O62− , compound is zero
equivalent weight of iodine will be equal to c. An element in the free or the uncombined state bears
a. M b. M / 2 c. M / 4 d. 2M oxidation number zero
d. In all of its compounds, the oxidation number of
(where, M is the molecular weight of iodine) fluorine is −1
10. The process in which oxidation number increases, is 22. In which of the following pairs, there is greatest
a. reduction b. hydrolysis difference in the oxidation number of the underlined
c. oxidation d. decomposition elements?
11. In which of the following, oxidation number of chlorine a. NO2 and N2O4
is +5 ? b. P2O5 and P4O10
a. Cl2O7 b. ClO–3 c. N2O and NO
c. ClO– d. ClO–4 d. SO2 and SO3

@iitjeehelps
514 SELF STUDY GUIDE BITSAT

23. A compound contains atoms A, B and C. The c. Both reactions I and II are intramolecular redox
oxidation number of A is + 2, of B is + 5 and of C is reactions
−2. The possible formula of the compound is d. intramolecular redox reactions and intermolecular
a. ABC3 b. B2( AC3 )2 redox reaction respectively
c. A3(BC4 )2 d. A3(BC3 )2 31. For the redox reaction,
24. Which of the following reactions is a redox reaction? MnO4− + C2O24− + H + → Mn2+ + CO2 + H 2O,
a. NaBr + HCl → NaCl + HBr the correct coefficients of the reactants for the
b. HBr + AgNO3 → AgBr + HNO3 balanced reaction are
c. H2 + Br2 → 2HBr MnO−4 C2O2−
4 H+
d. Na2 O + H2SO4 → Na 2SO4 + H2O a. 2 5 16
b. 16 5 2
25. White phosphorus reacts with caustic soda to form c. 5 16 2
PH3 and NaH2 PO2. This reaction is an example of d. 2 16 5
a. oxidation b. reduction
c. hydrolysis d. disproportionation 32. Consider the following reaction,
C2 H6( g ) + n O2 → CO2( g ) + H 2O(l )
26. Following reaction is an example of
In this equation, ratio of the coefficients of CO2 and
Ag2+ (aq ) + Ag(s ) q 2Ag+ (aq ) H2O is
a. reduction b. oxidation a. 1: 1 b. 2 : 3 c. 3 : 2 d. 1: 3
c. comproportionation d. disproportionation
33. Which of the following statements is true regarding the
27. Which of the following is a redox reaction? following balanced half-reaction?
a. Formation of glucose from CO2 and water
CN − → CNO−
b. Reaction of potassium cyanide with silver cyanide
a. Carbon is losing two electrons per atom
c. Hydration of rubidium
b. Oxidation number of carbon increases from +1 to +3
d. Reaction of barium chloride with sulphuric acid c. Oxidation number of nitrogen remains constant
28. Identify the disproportionation reaction. d. Both (a) and (c)
a. CH4 + 2O2 → CO2 + 2H2O 34. In the ionic equation,
b. CH4 + 4Cl2 → CCl4 + 4HCl BrO−3 + 6H + + xe − → Br 3 + + 3H 2O,
c. 2F2 + 2OH − → 2F − + OF2 + H2O
the value of x is
d. 2NO2 + 2OH − → NO−2 + NO−3 + H2O
a. 6 b. 2 c. 4 d. 3
29. Which of the following is not an intermolecular redox 35. In the redox reaction,
reaction?
x KMnO4 + NH 3 → y KNO3 + MnO2 + KOH + H 2O,
a. MgCO3 → MgO + CO2
x and y are
b. O2 + 3H2 → 2H3O+
a. x = 4, y =6
 1
c. K + H2O → KOH +   H2 b. x = 3, y =8
 2
 1 c. x = 8, y =6
d. MnBr3 → MnBr2 +   Br2 = 8, y =3
 2 d. x

30. Consider the following reactions, 36. Consider the following reaction,
I. 2Mn2O7 → 4MnO2 + 3O2 2Fe3 + + 2I− → 2Fe2+ + I2
II. SnCl2 + 2FeCl3 → SnCl4 + 2FeCl2 The half-reactions for the given reaction are
The given reactions are the examples of a. 2I− → I2 + 2e − and Fe3+ + e − → Fe2+
a. intermolecular redox reaction and intramolecular b. I2 → 2I− + 2e − and Fe2+ + e − → Fe3+
redox reactions respectively c. I2 → 2e − + 2I− and Fe2+ → Fe3+ + e −
b. Both reactions I and II are intermolecular redox
d. None of the above
reactions

@iitjeehelps
BITSAT Archives
1. The ratio of oxidation states of Cl in potassium 5. Which one of the following reactions represent the
chloride to that in potassium chlorate is [2014] oxidising property of H2O2? [2008]
a. +
1
b. −
1 a. 2KMnO4 + 3H2SO4 + 5H2O2 →
5 5 K 2SO4 + 2MnSO4 + 8H2O + 5O2
2 3 b. 2K 3[Fe(CN)6] + 2KOH + H2O2 →
c. − d. +
5 5 2K 4[Fe(CN)6] + 2H2O + O2
2. The oxidation state of sulphur in Na 2S 4 O6 is c. PbO2 + H2O2 → PbO + H2O + O2
[2013]
3 d. 2KI + H2SO4 + H2O2 → K 2SO4 + I2 + 2H2O
a. +6 b. +
2 6. The species that undergoes disproportionation in an
5
c. + d. −2 alkaline medium is [2006]
2
a. MnO2−
4 b. ClO−4
3. 2MnO−4 + 5H2O2 + 6H + → 2Z + 5O2 + 8H2O c. NO2 d. All of these
Identify Z in the above reaction. [2012] 7. A compound contains X , Y and Z atoms. The
a. Mn2+ b. Mn4+ oxidation states of X , Y and Z are +2, +2 and −2
c. Mn d. MnO2 respectively. The possible formula of the compound is
[2006]
4. The oxidation number of N and Cl in NOClO4 a. XYZ 2 b. Y2(XZ 3 )2 c. X 3(Y4Z )2 d. X 3(YZ 4 )3
respectively are [2011]
a. +2 and +7 8. The oxidation number of oxygen in hydrogen peroxide
b. +3 and +7 is [2005]
c. −3 and +5 a. +1 b. −1 c. +2 d. −2
d. + 2 and −7

Answer with Solutions


Practice Exercise +2 +5
7. (b) 2NO + 3I2 + 4H2O → 2NO3− + 6I− + 8H+
1. (b) The reaction is Hence, NO acts as a reducing agent and reduces I2 to I−
Oxidation state decreases since, the oxidation number of nitrogen changes from +2 in
NO to +5 in NO−3.
+2 0
Zn + CuSO4 ZnSO4 + Cu 8. (c) In acidic medium, K 2 Cr2 O7 gives following reaction,
2+ +6 +3
Hence, in this reaction, Cu is reduced to Cu. K 2 Cr2 O7 + 4H2SO4(Dil. ) → K 2SO4 + Cr2 (SO4 )3
2. (a) Fluorine is the strongest oxidising agent. Its reduction
potential (E°red ) is + 2.87 V, hence it is easily reduced to + 4H2O + 3O2

F by gaining electron. Q Decrease in ON of Cr per atom = 3
3. (b) H2 S is the strongest reducing agent among these as Q Decrease in ON of Cr per molecule = 2 × 3 = 6
the oxidation state of S is minimum in H2 S, i. e. − 2. M
Hence, equivalent weight of K 2 Cr2 O7 =
4. (b) NaHSO3 gives NaHSO4 during reaction in which the 6
oxidation number of S increase from +4 to +6. Hence, it is 0 −1
a reducing agent. 9. (b) I2 + 2S2O23− → 2I− + S4O26−
5. (a) 2Li + H2 2LiH Q Decrease in ON of iodine per atom = 1
0 +1
∴Decrease in ON of iodine per molecule = 2 × 1= 2
Oxidation
Hence, equivalent weight of iodine
Molecular weight of iodine M
Hence, in this reaction, hydrogen act as oxidising agent. =
Total decrease in ON of iodine per molecule 2
6. (d) Steam (H2O) is reduced to hydrogen by iron.

@iitjeehelps
516 SELF STUDY GUIDE BITSAT

10. (c) Oxidation is the process in which oxidation number of 23. (c) The oxidation states of A, B and C are +2 , + 5 and −2
effective element in the species increases. respectively.
11. (b) ClO−3 (a) In ABC3, 2 + 5 + 3 × ( −2) = +1 ≠ 0
x + 3 ( −2 ) = − 1 (b) In B2( AC3 )2, 2 × ( +2) + 2{( +5) + 3 ( −2)} = +2 ≠ 0
x − 6 = −1 (c) In A3(BC4 )2, 3 × ( +2) + 2{( +5) + 4 ( −2)} = 0
x = 6 −1 (d) In A3(BC3 )2, 3 × ( +2) + 2{( +5) + 3 ( −2)} = +4 ≠ 0
x = +5 ∴ Possible formula of compound is A 3(BC4 )2.
12. (d) CH2Cl2, 24. (c) Oxidation

x + 2 ( +1 ) + 2 ( −1 ) = 0 0 0 +1 –1
H2 + Br2 2HBr
x + 2−2=0
Reduction
x =0
25. (d ) Such reactions, in which a compound undergoes both
13. (b) lodine shows −1oxidation state in iodides
oxidation and reduction simultaneously, are called
(KI etc), +1, +3, +5 and +7 oxidation states in interhalogen disproportionation.
compounds and polyhalides like ICI, ICI3, IF5,IF7 etc.
P4 + 3NaOH + 3H2O → PH3 + 3NaH2PO2
14. (b) Na 2S4O6, 2 ( +1) + 4x + 6 ( −2 ) = 0
26. (c) Reduction
4x + 2 − 12 = 0
+2 0 +1
4x = 10 Ag2+(aq) + Ag(s) 2Ag+(aq)
⇒ x = 2. 5 Oxidation
15. (b) [Fe(H2O)5 NO]SO4,
This is an example of comproportionation reaction which is
x + 0 + 0 + ( −2 ) = 0 reverse of disproportionation reaction.
x =+2 27. (a) Oxidation
16. (c) Fluorine is the most electronegative element and +4 –2 0 0
Sunlight
oxygen is the second one. Hence, in F2O, oxygen is the 6CO2 + 6H2O C6H12O6 + 6O2
positive element.
Reduction
x + 2 ( −1 ) = 0
⇒ x =+ 2 Since, oxidation and reduction both occurs simultaneously
in the above equation, so it is a redox reaction.
Oxidation state of oxygen in F2O (or OF2 ) is +2.
28. (d) Disproportionation reaction,
17. (d) The oxidation state of hydrogen is always +1except in
+4 +3 +5
ionic hydrides (where ON of hydrogen is −1).
2NO2 + 2OH− → NO−2 + NO−3 + H2O
N3 H (hydrogen acid) is not an ionic compound, hence
29. (a) Intermolecular redox reactions and redox reactions
3x + ( +1) = 0 are same.
3x = − 1, x = − 1/ 3 +2 +4 −2 +2 −2 +4 −2
MgCO3 → MgO + C O2
18. (a) CO (carbonyl) is a neutral ligand, hence oxidation
state of Cr in Cr(CO)6 is zero. As there is no change in oxidation number of atoms,
therefore, this reaction is not an intermolecular redox
19. (b) Orthophosphoric acid (H3 PO4 ), ON of P = +5 reaction.
Phosphorous acid (H3 PO3 ) , ON of P = +3 30. (d) Same compound, e.g. Mn2O7 is dissociated into O2
Metaphosphoric acid (HPO3 ), ON of P = +5 and MnO2, where oxidation and reduction take place,
Pyrophosphoric acid (H4 P2 O7 ), ON of P = +5 i.e. 2Mn2O7 → 4MnO2 + 3O2
20. (b) BaO2 < KO2 < O3 < OF2 It is an example of intramolecular redox reaction, while
−1 −0.5 0 +2
reaction, such as
21. (a) Oxidation number of hydrogen is +1except in ionic SnCl2 + 2FeCl3 → SnCl4 + 2FeCl2
hydrides, where it is −1.
is an example of intermolecular redox reaction.
+4 +4
22. (d) (a) NO2 and N2O4; Difference = 0 31. (a) MnO4− + 8H+ + 5e − → Mn2+ + 4H2O] × 2
+5 +5
(b) P2O5 and P4O10; Difference = 0 C2O24− → 2CO2 + 2 e − × 5
+1 +2 2MnO4− + 5C2O24− + 16H+ → 2Mn2+ + 10CO2 + 8H2O
(c) N2O and NO ; Difference = +2 − 1 = +1
+4 +6
Thus, the coefficients of MnO−4, C2O2− +
4 and H in the above
(d) SO2 and SO3; Difference = +6 − 4 = +2 balanced equation respectively are 2, 5, 16.

@iitjeehelps
REDOX REACTIONS 517
32. (b) The balanced equation is 34. (b) Balanced reaction is
2C2H6 + 7O2 → 4CO2 + 6H2O BrO3− + 6H+ + 2e − → Br 3+ + 3H2O
33. (d) In the given redox reaction, ∴ Value of x is 2.
C− ≡≡ N → O−  C ≡≡ N , 35. (d) The balanced reaction is as follows:
Oxidation number of nitrogen is remaining unchanged at − 3. 8KMnO4 + 3NH3 → 8MnO2 + 3KNO3 + 5KOH + 2H2O
Oxidation number of carbon is increasing from + 2 to + 4. 36. (a) The half-reactions are oxidation reaction as in
Hence, (a) and (c) are the only correct response. 2I− → I2 + 2e − and reduction reaction as in
Therefore, option (d) is correct. Fe3+ + e − → Fe2+

BITSAT Archives
1. (b) Oxidation state of Cl in KCl = − 1 5. (d) The reaction in which H2O2 is reduced while the other
Oxidation state of Cl in KClO3 = + 5 reactant is oxidised, represents the oxidising property of
−1 H2O2.
∴ Ratio of oxidation state of Cl = Reduction
5
–1 –1 0 –2
2. (c) Oxidation number of Na = + 1 2KI + H2SO4 + H2O2 K2SO4 + I2 + 2H2O
Oxidation number of O = − 2 (Oxidising agent
oxidation)
Let oxidation number of S = x
∴ 2(ON of Na) + 4(ON of S) + 6(ON of O) = 0 6. (c) 2NO2(g)+ 2OH −(aq) →
2( +1) + 4x + 6 ( −2) = 0
NO2− (aq)+ NO3–(aq)+ H2O (l)
+2 + 4x − 12 = 0
This reaction involves disproportionation of NO2
4x = + 12 − 2
(+ 4 state) in to NO−2 (+3 state) and NO−3 (+5 state).
10
x =+ Therefore, this reaction is an example of
4 disproportionation redox reaction.
5
⇒ x =+ 7. (a) The oxidation number of X,Y and Z are +2, +2 and −2
2 respectively.
3. (a) 2MnO–4 + 5H2O2 + 6H+ → 2Mn2+ + 5O2 + 8H2O (a) In XYZ 2 = ( +2) + ( +2) + 2( −2) = 0
4. (b) NOClO4 is actually NO+ ClO−4. (b) In Y2(XZ 3 )2 = 2( +2) + 2( +2) + 6( −2) =/ 0
Let the oxidation state of N in NO+ is x. (c) In X 3(Y4Z )2 = 3 ( +2) + 8 ( +2) + 2 ( −2) =/ 0
+ (d) In X 3(YZ 4 )3 = 3 ( +2) + 3 ( +2) + 12 ( −2) =/ 0
NO: x + ( −2) = + 1
8. (b) In H2O2 Let the ON of oxygen = x
x = + 1+ 2 = + 3
Then, 2 ( +1) + 2 (x ) = 0
Let the oxidation state of Cl in ClO−4 is y.
or 2 + 2x = 0
ClO−4 : y + ( −2) × 4 = − 1 or 2x = − 2
y − 8 =1; y = + 7 or x = −1

@iitjeehelps
13
Electrochemistry

Electrochemistry is the study of production of electricity from energy released during spontaneous
chemical reactions and the use of electrical energy to bring about non-spontaneous chemical
transformations.

Section A : Conductor and Conductance


Electrical Conductors
Any substance, through which, we can pass electricity, is called electrical conductor.
Conductors are generally divided into two types with different properties which are tabulated as :
Metallic conductors Electrolytic conductors
(e.g. metals) (e.g. NaCl )

(a) In these, conduction takes place by the flow of In these, conduction takes place by the flow of
electrons. ions.

(b) It does not involve transfer of matter. It involves transfer of matter.

(c) In these, conductance decreases with increase in In these, conductance increases with increase in
temperature. temperature.

Conductance and Conductivity of Electrolytic Solutions


The power of an electrolytes conduct electric current is called conductance or conductivity.
The resistance of any conductor varies directly as its length (l) and inversely as its cross-sectional
area (a), i.e.
l
R∝
a
l
or R=ρ
a
where, ρ is called the resistivity or specific resistance.

@iitjeehelps
ELECTROCHEMISTRY 519

If l = 1 cm and a = 1 cm 2 then R = ρ Kohlrausch’s Law


1 l It states that molar conductance at infinite dilution for any
∴ Specific conductance (κ) = = × conductance (C )
ρ a electrolyte is the sum of contribution of its constituent ions
l i.e. anions and cations.
where, = cell constant
a e.g. Λ °m CH3COOH = λ °m CH3COO− + λ °m H +
The unit of (κ) is ohm –1 cm −1 or S cm –1 . Λ°m Al 2 (SO4 )3 = 2 × λ °m Al3 + + 3 × λ ° m SO24−

Molar Conductivity ( λ m or µ )
κ × 1000 Applications of Kohlrausch’s Law
λ m or µ =
M (i) For the determination of equivalent/molar
conductivity at infinite dilution.
where, M = molarity of the solution
m = molar conductivity (ii) For the determination of degree of dissociation.
Its SI units are expressed in Ω cm mol −1 2 −1 Λc
or Degree of dissociation (α ) = ∞m
mho cm 2 mol −1 or S cm 2 mol −1 . Λm
(iii) For the calculation of dissociation constant of a weak
Equivalent Conductivity (λ eq or Λ) electrolyte.
κ × 1000 Cα 2
λ eq or Λ = Ka = = Cα 2 [QFor weak electrolyte, α < < < 1]
C eq ( N ) 1−α

where, C eq = concentration of solution in equivalent per where, K a = equilibrium dissociation constant


litre (i.e. normality) and C = molar concentration of weak electrolyte
eq = equivalent conductivity (iv) For the determination of solubility of sparingly soluble
−1 −1
salt.
Its SI unit are expressed in Ω cm (equiv ) or
2
κ × 1000
Solubility =
mho cm 2 (equiv −1 ) or S cm 2 (equiv −1 ). Λ° m
● Transport number of cation (n c )
Variation of Conductivity and Molar
Current carried by cation
Conductivity with Concentration =
Total current
Equivalent as well as molar conductivity α dilution and uc
1 =
specific conductivity ∝ uc + ua
dilution ua
Strong electrolytes like KCl, have high value of conductance
● Transport number of anion (n a ) =
uc + ua
even at low concentration and there is no rapid increase in
their equivalent or molar conductance on dilution.
Section B : Conversion of
Electrical Energy into Chemical
Λm Λm CH3COOH Energy (Electrolytic Cell)
KCl (weak electrolyte)
(strong electrolyte)
Electrolysis
Molarity Molarity
It is a process in which electrical energy is used to bring some
In case of weak electrolytes, like acetic acid, have a low chemical changes. It is carried out in an electrolytic cell.
value of conductance at high concentration and there is a Electrolytic cell
rapid increase in the value of equivalent conductance
(molar conductance) with dilution. r Anode r Cathode
Oxidation Reduction
Limiting Molar Conductivity or Infinite More E°oxi , gets More E°red , gets
Conductivity (Λ°m or Λ∞m ) oxidised easily reduced easily
It is the molar conductivity of electrolyte when ° = standard oxidation potential
where, E oxi
concentration of electrolyte approaches zero
° = standard reduction potential
E red
(i.e. at infinite dilution).

@iitjeehelps
520 SELF STUDY GUIDE BITSAT

● Decreasing order of oxidation potential : Electrode and Half-Cell


− −
I − > Br − > Cl − > OH − > F − > NO2 > NO3 > SO32− > SO2−
4 A strip of metal, used in electrochemical studies, is called
● Increasing order of reduction potential : electrode. The combination of the metal electrode and
All metals, more reactive than hydrogen solution, is called a half-cell.
°
[ E red ° = zero] < metals
= less than zero] < Hydrogen[ E red Electrolytic cell Voltaic or Galvanic cell
°
less reactive than hydrogen[ E red = + ve] like Cu, Ag etc.
Anode Cathode Anode Cathode
● Besides the ions of electrolyte, if some cations or anions
are present in the solution, then ions with lower Sign + – – +
discharge potential are discharged in preference to those
Electron flow Out In Out In
which have high discharge potential.
Half-reaction Oxidation Reduction Oxidation Reduction

Faraday’s Laws of Electrolysis


Salt-bridge
There are two laws given by Faraday, are discussed below :
It is a U-shaped tube contains a gel permeated with a
First Law solution of an inert electrolyte such as Na 2SO4 . The
salt-bridge is necessary to complete the electrical circuit
Deposited mass of the substance is directly proportional to
and to maintain electric neutrality in both compartments
passed charge in a voltameter.
(by flow of ions).
w ∝Q
w = ZQ = Zit Representation of a Cell
where, w = mass, Q = charge (in coulomb) Consider the Daniell cell with following cell reaction :
i = current (in amperes) Zn(s ) + Cu 2 + (aq ) → Zn 2 + (aq ) + Cu(s )
t = time (in second) In writing a cell diagram, following points are considered.
Z = electrochemical equivalent We divide the cell into two half-cells.
1 F = 6.022 × 1023 × 1.6 × 10–19 = 96500 C (approx.) Anode Cathode

Number of gram equivalents = Number of Faraday’s of Reaction Oxidation takes place Reduction takes place
electricity 2+ −
Zn( s ) → Zn ( aq ) + 2e Cu2 + ( aq ) + 2e − → Cu ( s )
Second Law Terminal Negative Positive
The number of equivalents of any substance produced by a
Side LHS RHS
given quantity of electricity during electrolysis, are same.
2+
w A EA Diagram Zn ( s ) / Zn ( aq ) Cu2 + ( aq ) / Cu ( s )
=
w B EB Complete cell diagram may be represented as follows :
Anode half-cell Salt-bridge Cathode half-cell
Section C : Conversion of Zn(s) | Zn2+(aq) || Cu2+(aq) | Cu(s)
Chemical Energy into Electrical Electrons flow in this way
Phase boundary
Energy Phase boundary

If oxidised or reduced part is a gas, use Pt electrode


(Electrochemical or Galvanic Cells) saturated with that gas.
If oxidised and reduced parts are in ionic state, use Pt
Electrochemical Cell electrode.
An electrochemical cell is a system or arrangement in
which two electrodes are fitted in the same electrolyte or in
two different electrolytes which are joined by a salt bridge.
EMF of Cell
The difference in electrode potential of electrodes due to
Galvanic or Voltaic Cells different flow of electrons, is called electromotive force or
It is a device in which a redox reaction used to convert cell potential of a cell. This is the driving force for all the cell
chemical energy into electrical energy. reactions.

@iitjeehelps
ELECTROCHEMISTRY 521

Standard Electrode Potential Nernst Equation


Potential difference between the metal and the metal ion in The relationship between electrode potential and
which electrode is dipped, is called electrode potential concentration of solution is called nernst equation.
denoted as E, which is related to density of negative electric 2 .303 RT
charge. E cell or emf = E °cell – log Q
nF
When pressure is 1 bar and concentration is 1 M, electrode
potential is called standard electrode potential denoted as where, n = total number of electrons lost or gained
E °. Temperature is generally taken as 298 K (i.e. 25°C). [OS]
Q = reaction coefficient of overall reaction =
For half-cell reaction M n+ + ne − → M , [RS]
● If E
° = x V, then E ° = − x V where, [OS] = concentration of oxidised state
oxi red
° ° [RS] = concentration of reduced state
ECu 2+ = 0.34V , then ECu 2+ = − 0.34V
/Cu /Cu 0.0591
∴ E cell = E °cell – log Q
● The SI unit of cell potential is the volt (V) and the n
potential of a galvanic cell is defined as the positive ● The emf of a standard cell does not change with
quantity. temperature.
E °cell = E °oxi + E °red , E cell = E oxi + E red ● The standard electrode potential of a half-cell has a fixed

value. It does not change, if the half-reaction is multiplied


Electrochemical Series with an integer.
The arrangement of metals in decreasing order of tendency Applications of Nernst Equation
to lose electrons, is called electrochemical series or it is the
series in which the elements are arranged on the basis of There are two important applications of Nernst equation are
the values of their standard reduction potentials at 25°C. given below :
Standard electrode
(i) To find equilibrium constant
Electrode reaction
Element (Reduction) reduction potential E° At equilibrium, E cell = zero
(volt) 2 . 303 RT 0.0591
∴ E °cell log K= log K
Li Li+ + e– Li – 3.05 nF n
K K+ + e – K – 2.925 where, K = equilibrium constant
Ca Ca2+ + 2e– Ca – 2.87
Na Na+ + e– Na – 2.714 (ii) To find Gibbs free energy change
Mg Mg 2+
+ 2e –
Mg – 2.37 ∆G ° = – n E °cell F
Increasing tendency to accept electrons (e– )

Increasing tendency to lose electrons (e– )

Al Al3+ + 3e– Al – 1.66 ∆G ° = standard Gibbs free energy change


Increasing strength as oxidising agent

Increasing strength as reducing agent

Zn2+ + 2e–
Increasing tendency for reduction

Increasing tendency for oxidation

Zn Zn – 0.7628
Cr Cr3+ + 3e– Cr – 0.74 Relation between Standard Potentials of
Fe Fe2+ + 2e– Fe – 0.44 Half-cells Containing a Metal in Different
Cd Cd2+ + 2e– Cd – 0.403 Oxidation States
Ni Ni2+ + 2e– Ni – 0.25 If two half-reactions having potentials E °1 and E °2 are
Sn Sn2+ + 2e– Sn – 0.14 combined to give a third half-reaction having a potential
H2 2H+ + 2e– H2 0.00 E °3 , then
Cu Cu2+ + 2e– Cu + 0.337 n E ° + n 2 E °2
– E °3 = 1 1
I2 I2 + 2e– 2I + 0.535 n3
+ –
Ag Ag + e Ag + 0.799
Hg Hg2+ + 2e– Hg + 0.885 Batteries
– –
Br2 Br2 + e 2Br + 1.08 A cell or a battery (arrangement of one or more cells
Cl2 Cl2 +2e– 2Cl– + 1.36 connected in series) is basically a galvanic cell and used
Au Au3+ + 3e– Au + 1.50 where the chemical energy of redox reaction is converted
F2 F2 + 2e– 2F – + 2.87 into electrical energy.

@iitjeehelps
522 SELF STUDY GUIDE BITSAT

There are two types of batteries : Fuel Cells


1. Primary Batteries These are another means by which chemical energy
The primary batteries are those in which the cell reaction occurs may be converted into electrical energy, e.g. H 2− O2
only once and the battery becomes dead after the use over a period fuel cell. This cell was used as a primary source of
of time and cannot be reused again. electrical energy on the moon flights. The overall cell
reaction produces water, which was used for drinking
(i) Dry cell or Leclanche cell is also called primary voltaic cell.
by the astronauts.
In it, the electrode reactions are
At anode Zn (s ) → Zn 2 + + 2e– The half-reactions are
At anode 2H 2 ( g ) + 4OH − (aq) → H 2O(l) + 4 e −
At cathode MnO2 + NH 4+ + e − → MnO(OH) + NH3
At cathode O2 ( g ) + 2H 2O(l) + 4 e − → 4OH − (aq)
The cell potential is 1.6 V.
(ii) Another type of dry cell is mercury cell. Overall cell reaction 2H 2 ( g ) + O2 ( g ) → 2H 2O(l)
The electrode reactions for the cell are Corrosion and Its Prevention
At anode Zn(Hg) + 2OH − → ZnO(s ) + H 2O + 2e − Corrosion metal is oxidised by loss of electrons to
At cathode HgO + H 2O + 2e − → Hg(l ) + 2OH − oxygen and form metal oxide, e.g. conversion of iron
to rust [Fe 2O3 ⋅ xH 2O].
The overall cell reaction is
Corrosion of iron, known as rusting, occurs in the
Zn(Hg) + HgO(s ) → ZnO(s ) + Hg(l )
presence of water and oxygen.
2. Secondary Batteries At anode 2Fe(s ) → 2Fe 2 + (aq ) + 4 e –
These are also called reversible galvanic or voltaic cell. At cathode O2 ( g ) + 4H + (aq ) + 4 e – → 2H 2O (l )
Secondary batteries are rechargeable because on charging,
reaction becomes reverse, e.g. lead storage battery, nickel Overall reaction
cadmium cell etc. In lead storage battery, a solution of sulphuric 2Fe(s ) + O2 ( g ) + 4H + (aq ) → 2Fe 2 + (aq ) + 2H 2O (l )
acid surrounds the plates and acts as electrolyte. The cell
potential is 12 V. Fe 2+ ions further oxidised by atmospheric oxygen to
Fe3 + ions and form hydrated ferric oxide
The half-cell reactions, when the battery is being used up are
[Fe 2O3 ⋅ xH 2O].
At anode Pb(s) + SO24− → PbSO4 (s) + 2e −
4Fe 2 + + O2 + 4H 2O → 2Fe 2O3 (s ) + 8H + (aq )
At cathode PbO2 (s ) + SO24− + 4H + + 2e − → PbSO4 (s ) + 2H 2O
Fe 2O3 + xH 2O → Fe 2O3 ⋅ xH 2O
Overall reaction
Hydrated ferric oxide (rust)
Pb (s ) + PbO2 (s ) + 4H + → 2PbSO4 (s ) + 2H 2O + 2SO2−
4
Rusting of iron can be prevented by the following
During charging, following reactions occur
methods :
At anode PbSO4 (s ) + 2H 2O → PbO2 (s ) + SO24− + 4H + + 2e −
(i) Barrier protection through coating of paints or
At cathode PbSO4 (s ) + 2e − → Pb(s) + SO24− electroplating.
Overall reaction (ii) Galvanisation or coating of surface with tin metal.
2PbSO4 (s) + 2H 2O → Pb(s ) + PbO2 (s ) + 4H + + 2SO24− (iii) By the use of antirust solutions.

@iitjeehelps
Practice Exercise
1. Two electrodes are fitted in conductance cell 1.5 cm 9. The increase in the molar conductivity of HCl with
apart while the area of cross-section of each electrode dilution is due to
is 0.75 cm 2. The cell constant is a. increase in the self ionisation of water
a. 1.125 cm−1 b. 0.5 cm−1 b. decrease in the self ionisation of water
c. 2.0 cm−1 d. 0.2 cm−1 c. decrease in the interionic forces
d. None of the above
2. Conductivity (unit siemens) is directly proportional to
area of the vessel and the concentration of the 10. Molar conductance Λ m is plotted against C (mol L −1)
solution in it and is inversely proportional to the length for three electrolytes (NaCl, HCl, NH4 OH).
of the vessel, then the unit of constant of
proportionality is I
–1 2 –1 Λm II
a. S m mol b. Sm mol
–2 2
c. S m mol d. S2 m2 mol–2 III
√C
3. Which of the following solutions has the highest
equivalent conductance? Which one of the following is correct?
a. 0.01 M NaCl b. 0.05 M NaCl I II III
c. 0.005 M NaCl d. 0.02 M NaCl a. NaCl HCl NH4OH
4. The resistance of 0.1N solution of a salt is found to be b. HCl NaCl NH4OH
2.5 × 103 Ω. The equivalent conductance of the c. NH4OH NaCl HCl
d. NH4OH HCl NaCl
solution is (Cell constant = 1.15 cm −1)
11. The plot of molar conductance vs C in strong
a. 4.6 ohm−1 cm2 equ iv −1 b. 5.6 ohm−1 cm2 equ iv −1
c. 6.6 ohm−1 cm2 equ iv −1 d. 7.6 ohm−1 cm2 equ iv −1
electrolyte, is
a. circular b. linear
5. The equivalent conductivity of a solution containing c. parabolic d. sinusoidal
2.54 g of CuSO4 per litre, is 91.0 Ω −1 cm 2 eq−1. Its 12. The values of Λ∞eq for NH4 Cl, NaOH and NaCl are
conductivity would be 149.74, 248.1 and 126.4 Ω −1 cm 2 equiv −1. The value
a. 2.9 × 10−3 Ω −1 cm−1 b. 1.9 × 10−3 Ω −1 cm−1
of Λ∞eq of NH4 OH is
c. 2.4 × 10−3 Ω −1 cm−1 d. 3.6 × 10−3 Ω −1 cm−1
a. 371.44 Ω cm2equiv −1
6. Point out the correct statement. b. 271.44 Ω cm2equiv −1
a. Equivalent conductance decreases with dilution c. 71.44 Ω cm2equiv −1
b. Specific conductance increases with dilution d. data is insufficient to calculate it
c. Specific conductance decreases with dilution
d. Equivalent conductance increases with increase in 13. The quantity of electricity required to liberate 0.1 g
concentration equivalent of an element at the electrode is
a. 9650 C b. 96500 C
7. The equivalent conductance of solution is ......... . c. 965 C d. 96.5 C
[If cell constant is 1.25 cm −1 and resistance of N /10
14. On passing 3 A of electricity for 50 min, 1.8 g of metal
solution is 2.5 × 103 Ω].
−1 −1
deposits. The equivalent mass of metal is
a. 2.5 Ω cm equ iv 2
a. 20.5 b. 25.8 c. 19.3 d. 30.7
b. 2.5 Ω −1 cm− 2 equiv −1
15. One faraday of electricity will liberate one gram atom
b. 50 Ω −1 cm2 equiv −1 of a metal from a solution of
d. 5.0 Ω −1 cm 2 equiv −1 a. AuCl3 b. CuSO4 c. BaCl2 d. KCl
8. The increase in the molar conductivity of acetic acid 16. The charge required for the reduction of 1 mole of
with dilution is due to Cr2O72− ions to Cr 3 + is
a. decrease in interionic forces a. 96500 C b. 2 × 96500 C
b. increase in degree of ionisation c. 3 × 96500 C d. 6 × 96500 C
c. increase in self ionisation of water
d. None of the above 17. On passing 0.1 F of electricity through aluminium
chloride, the amount of aluminium metal deposited on
cathode is (Atomic weight of Al = 27)

@iitjeehelps
524 SELF STUDY GUIDE BITSAT

a. 0.27 g b. 0.3 g c. 0.9 g d. 2.7 g [ E °Cu2 + / Cu+ = + 0.15 V, E °Cu2 + / Cu = 0.34 V ]


18. The current of 2 A is passed for 5 h through a molten a. − 0.38 V b. + 0.19 V
tin salt to deposit 22.2 g tin. What is the oxidation state c. − 0.49 V d. + 0.38 V
of tin in salt? [Atomic weight of Sn = 118 .69 g ] 28. Calculate the emf of the following cell:
a. +2 b. +5 c. +3 d. +4
Cu(s ) | Cu2+ (aq ) | | Ag+ (aq ) | Ag(s )
19. What will be the weight of silver deposited, if 96.5 A of
Given that, E ° Cu2 + / Cu = 0.34 V , E °Ag / Ag+ = − 0.80 V
current is passed into aqueous solution of AgNO3 for
100 s? a. 0.046 V b. 0.46 V c. 0.57 V d. −0.46 V
a. 1.08 g b. 10.8 g c. 108 g d. 1080 g
29. A cell constituted by two electrodes
20. A certain amount of current liberates 0.5 g of H2 in A (E °A / A + = − 0.35 V ) and B (E °B / B + = 0.42 V ).
2.0 h. How many gram(s) of oxygen can be liberated Calculate the emf of the cell.
by the same current in the same time?
a. 0.5 g b. 8.0 g a. 0.07 V b. 0.77 V
c. 4.0 g d. 16.0 g c. − 0.77 V d. − 0.07 V
21. Pick the odd one out. 30. Out of Cu, Ag, Fe and Zn, the metal which can
a. Daniell cell b. Voltaic cell displace all others from their salt solution is
c. Galvanic cell d. Electrolytic cell a. Ag b. Cu c. Zn d. Fe
22. The EMF of a cell is 31. Which of the following solution will turn blue when
a. sum of two oxidation potentials placed in copper vessel?
b. sum of two potentials a. AgNO3 b. NaCl
c. difference of two electrode potentials c. ZnSO4 d. KNO3
d. None of the above
32. A gas X at 1 atm is bubbled through a solution
23. Consider the following reaction, containing a mixture of 1 M Y – and 1 M Z – at 25°C. If
Cu| Cu2+ (1M) | | (Zn2+ (1M)| Zn
the order of reduction potentials is Z > Y > X, then
A cell represented above should have emf a. Y will oxidise X but not Z
a. positive b. negative b. Y will oxidise Z but not X
c. zero d. Cannot be predicted c. Y will oxidise both X and Y
d. Y will reduce both X and Y
24. The standard reduction potential of Pb and Zn
electrodes are − 0.126 and − 0.763 V respectively. 33. Using the standard electrode potential, find out the
The cell equation will be pair between which redox reaction is not feasible.
a. Pb + Zn → Pb + Zn
2+ 2+
E ° values: Fe3 + /Fe2+ = +0.77, I2 / I− = + 0.54 V
b. Pb4+ + 2 Zn → Pb + 2 Zn2+
c. Zn2+ + Pb → Zn + Pb2+ Cu2+ / Cu = + 0.34 V, Ag+ / Ag = + 0.80 V
d. None of the above a. Fe3+ and I− b. Ag+ and Cu
c. Fe3+ and Cu d. Ag+ and Fe3+
25. Consider the following reaction,
1 34. The emf of the cell involving the reaction
H2 ( g ) + AgCl(s ) → H + (aq ) + Cl– (aq ) + Ag (s ) 2Ag+ (aq ) + H2 ( g ) → 2Ag (s ) + 2H+ (aq ) is 0.80 V.
2
Above reaction occurs in the galvanic cell The standard oxidation potential of silver electrode is
a. Ag | AgCl(s)|KCl(aq) || AgNO3(aq)|Ag(s) a. 0.80 V b. 0.40 V
b. Pt | H2(g )| HCl (aq) || AgNO3(aq )|Ag(s) c. −0.80 V d. 0.20 V
c. Pt | H2(g )| HCl (aq) || AgCl(s)|Ag(s) 35. For a reaction A (s ) + 2B + (aq ) → A 2+ (aq ) + 2B , K C
d. Pt | H2(g )| KCl (aq) || AgCl(s)|Ag(s)
has been found to be 1012. The E °cell is
26. When the sample of copper with the zinc impurity is to a. 0.354 V b. 0.708 V
be purified by electrolysis, the appropriate electrodes c. 0.0098 V d. 1.36 V
are
Cathode Anode 36. The value of E cell of hydrogen electrode at pH = 0,
a. Pure zinc Pure copper 298 K and 1 atm, is
b. Impure zinc Pure copper a. 0.59 V b. 0 V
c. Impure zinc Impure sample c. − 0.59 V d. − 0.059 V
d. Pure copper Impure sample
37. The emf of a galvanic cell is positive when free energy
27. Cu+ ion is not stable in aqueous solution because of change of reaction is
disproportionation reaction. E ° value of a. > 0
disproportionation of Cu+ is b. < 0
c. = 0

@iitjeehelps
ELECTROCHEMISTRY 525
d. no relationship of free energy change and emf a. 156 b. 125 c. 1.25 × 10–2 d. 6.4 × 10–3
38. What is the value of E °cell in the following reaction? 43. The standard emf of a cell involving one electron
Cr | Cr 3 + (0.1M) | | Fe2 + (0.01M) | Fe change is found to be 0.591 V at 25°C. The
equilibrium constant of the reaction is
Given, E °Cr3 + / Cr = − 0.74 V , E °Fe2 + / Fe = −0.44 V
(1F = 96500 C mol−1)
a. + 0.2606 V b. 0.5212 V a. 1.0 × 101 b. 1.0 × 105
c. + 01303 V d. − 0.2606 V
c. 1.0 × 1010 d. 1.0 × 1030
39. Consider the following reaction,
44. Hydrogen electrode is placed in the solution whose pH
Zn (s ) + Cu2+ (0.1M) → Zn2+ (1M) + Cu (s ) above is 10. The potential of this electrode will be
reaction, taking place in a cell, E °cell is 1.10 V. E cell for a. + 0.591 V b. − 0.591 V
 RT  c. 0 d. None of these
the cell will be  2.303 = 0.0591
 F  45. What is the Gibbs energy of the following reaction?
a. 1.80 V b. 1.07 V Zn (s )+ Cu2+ (aq ) → Zn2 + (aq )+ Cu2 + (s ) ;
c. 0.82 V d. 2.14 V
E ° cell = 1.1V
40. The Edison storage cell is represented as a. 106135.75 J mol −1
b. 21227 J mol−1
Fe (s )| FeO (s )| KOH (aq )| Ni2O3 )(s )| NiO (s )| Ni (s ) c. − 212 . 27 kJ mol−1 d. 21227 J mol−1
the half-cell reactions are
46. When a lead storage battery is charged, it acts as
Ni2O3 (s ) + H2O (l ) + 2e − r 2NiO(s ) + 2OH − ; a. primary cell b. galvanic cell
E ° = + 0.40 V c. concentration cell d. electrolytic cell
FeO(s ) + H2O(l ) + 2e − r Fe(s ) + 2OH − ; 47. In a hydrogen oxygen fuel cell, combustion of
hydrogen occurs to
E ° = − 0.87 V a. produce high purity water
b. remove adsorbed oxygen from electrode surface
What is the maximum amount of electrical energy that c. generate heat
can be obtained from one mole of Ni2O3 ? d. create potential difference between two electrodes
a. 127 kJ b. 245.11 kJ
c. 90.71 kJ d. 122.55 kJ 48. In which of the following, the corrosion of iron will be
most rapid?
41. Consider the following cell reaction . a. In pure water
Cu (s ) + 2Ag+ (aq ) → Cu2+ (aq ) + 2Ag (s ) b. In pure oxygen
c. In air and moisture
E °cell = 0.46 V By doubling the concentration of Cu2+ , d. In air and saline water
E cell is
49. On the basis of electrochemical theory of aqueous
a. doubled corrosion, the reaction occurring at the cathode is
b. halved a. O2(g ) + 4H+ (aq ) + 4e − → 2H2O (l )
c. increases but less than double
d. decreases by a small fraction b. H2(g ) + 2OH− (aq ) → 2H2O (l ) + 2e −
c. Fe2+ (aq ) + 2e − → Fe (s )
42. The value of the reaction quotient, Q for the following
d. Fe3+ (aq ) + e − → Fe2+ (aq )
cell is
Zn(s )| Zn2+ (0.01M) || Ag+ (1.25 M)| Ag (s )

@iitjeehelps
BITSAT Archives
1. A reaction, Cu2+ + 2e − → Cu is given. For this 8. When same quantity of electricity is passed through
reaction, graph between E red versus ln[Cu ] is a 2+ aqueous AgNO3 and H2SO4 solutions connected in
straight line of intercept 0.34 V, then the electrode series, 5.4 × 10−2 g of H2 is liberated. What is the mass
oxidation potential of the half-cell Cu/Cu2+ (0.1 M) will of silver (in grams) deposited? (Equivalent weight of
be [2014] hydrogen = 1008, silver = 108) [2008]
0.0591 a. 54 b. 0.54
a. 0.34 b. 0.34 + c. 5.4 d. 10.8
2
0.0591 0.0591 9. When electric current is passed through acidified
c. − 0.34 − d. − 0.34 +
2 2 water for 1930 s, 1120 mL of H2 gas is collected
2. If E ° and E ° are −0.36 V and 0.439 V (at STP) at the cathode. What is the current passed in
Fe3 + / Fe Fe2 + / Fe amperes? [2008]
respectively, then the value of E ° is a. 0.05 b. 0.50
Fe3 + / Fe2 + [2013]
a. ( − 0.036 − 0.439 ) V c. 5.0 d. 50
b. [ 3 ( −0.36) + 2( 0.439 )] V 10. Given, standard electrode potentials
c. ( −0.36 − 0.439 ) V
d. [( 3 − 0.36) − 2( − 0.439 )] V Fe2 + + 2e − → Fe, E ° = − 0. 440 V
3. The ionic conductance of Ba 2+ and Cl− respectively Fe3 + + 3e − → Fe, E ° = − 0. 036 V
are 127 and 76 Ω −1 cm 2 at infinite dilution. The The standard electrode potential (E ° ) for
equivalent conductance (in Ω −1cm 2) of BaCl2 at infinite Fe2 + + e − → Fe2 + , is [2007]
dilution will be [2012]
a. + 0.772 V b. − 0.772 V
a. 330 Ω −1cm2 b. 203 Ω −1cm2 c. + 0.417 V d. − 0.417 V
c. 139.5 Ω −1cm2 d. 51 Ω −1cm2
11. The specific conductivity of 0.1 N KCl solution is
4. The equilibrium constant (K ) for the reaction 0.0129 Ω −1cm −1. The resistance of the solution in the
Cu(s ) + 2Ag+ (aq ) → Cu2+ (aq ) + 2Ag(s ), will be cell is 100 Ω. The cell constant of the cell will be
[Given, E °cell = 0.46 V] [2011] a. 1.10 b. 1.29 [2007]
a. K c = antilog 15.6 b. K c = antilog 2.5 c. 0.56 d. 2.80
c. K c = antilog 1.5 d. K c = antilog 12.2 12. The cathodic reaction of a dry cell is represented as
5. E ° for Fe/Fe2+ is +0.44 V and E ° for Cu/Cu2+ is 2MnO2(s ) + Zn2 + + 2e − → ZnMn2O4 (s )
−0. 32 V. Then, in the cell, [2011]
a. Cu oxidises Fe2+ ion b. Cu2+ oxidises iron If there are 8 g MnO2 in the cathodic compartment
c. Cu reduces Fe2+ ion d. Cu2+ reduces Fe then the time for which the dry cell will continue to give
current of 2 milliampere, is [2006]
6. A current of 0.5 A when passed through AgNO3 a. 25.675 day b. 51.35 day
solution for 193 s deposited 0.108 g Ag. The c. 12.8 day d. 6.423 day
equivalent weight of silver is [2010]
a. 108 b. 54 c. 10.8 d. 5.4 13. The standard reduction potential of the reaction
1
7. When an aqueous solution of sodium chloride is H2O + e − → H2 + OH , at 298 K, is
2 [2005]
electrolysed using platinum electrodes, the ion RT RT
discharged at the electrodes are a. E ° = ln Kw b. E ° = ln [ p H2 ]1/ 2 [ OH − ]
F F
a. sodium and hydrogen b. sodium and chloride [ p H2 ]1/ 2
RT RT
c. hydrogen and chloride d. hydroxyl and chloride c. E ° = ln d. E ° = − ln Kw
F [H+ ] F

@iitjeehelps
Answer with Solutions
Practice Exercise +6 −2
16. (d) Cr2 O7 → Cr
+3

l 1.5
1. (c) Cell constant (x ) = = = 2 cm−1 Hence, electrons involved in this reduction will be
a 0.75 = 2 × [( +6) − ( +3)] = 2 × 3 = 6
2. (b) Sm2 mol−1
∴Charge required for one mole = 6 × 96500 C
3. (c)
17. (c)
l
4. (a) We know that, k = C   i ⋅t
a 18. (a) Equivalents of tin =
96500
1  1
= × 1.155 cm−1 C =  22 ⋅ 2 2 × 5 × 60 × 60
2.5 × 103  R or =
Eq. wt. 96500
k × 1000 1.15 × 1000
Λ eq = = ∴ Eq. wt. = 59.5
N 2.5 × 103 × 0.1 At. wt. 118.69
∴Valency of tin = = = 2 (an integer)
= 4.6 ohm−1 cm2 equiv −1 Eq. wt. 59.5
 2 . 54  19. (b) Q = it = 96.5 × 100 = 9650 C
5. (a) K = Λ eq × C = (91Ω −1 cm 2 eq−1)  eq
 159 / 2 × 1000  96500 C deposited weight of Ag = 108 g
= 2.9 × 10−3 Ω −1cm−1 108 × 9650
9650 C deposited weight of Ag = = 10.8 g
96500
6. (c)
Cell constant 1.25 20. (c) According to Faraday’s first law,w = Zit
7. (d) k = = = 5 × 10−4 Ω −1cm−1
Resistance 2.5 × 103 Given, it is same for H2and O2.
k × 1000 WH2 Z H2 EH2
Equivalent conductance = = =
M WO2 Z O2 EO2
5 × 10−4 × 1000 0.500 2
= = , WO2 = 4.00 g
1/ 10 WO2 16
= 5 Ω −1 cm2 equiv −1 21. (d)
8. (b) 22. (c)
9. (c) Due to increase in volume of water, interionic forces of 23. (b) If E°red of Cu and Zn are known from ECS.
HCl will decrease. Hence, ionic mobility will increase and
consequently, the molar conductivity will also increase. than, E ° = E °cathode − E °anode
10. (b) =E °right − E °left
11. (b) The plot of molar conductance vs C in strong = ( −0.76) − ( +0.34) (From ECS)
electrolyte is linear. = − 0.76 − 0.34 = − 110
. V
12. (b) Λ∞eq (NH4OH) = Λ∞eq (NH4Cl) + Λ∞eq NaOH − Λ∞eq(NaCl) 24. (c) The E°red of Zn is lower than that of Pb, hence Zn is a
better reducing agent than Pb.
= (149.74 + 248.1) − 126.4
∴The correct cell reaction will be
= 271.44 Ω cm2 equiv −1
Zn2+ + Pb → Zn + Pb2+
13. (a) Electricity required to liberate 0.1 g equivalent of an 25. (c) H2 undergoes oxidation and AgCl(Ag+ ) undergoes
element
reduction. Therefore, cell may be represented as
= 96500 × 0.1 = 9650 C Pt | H2(g )| HCl(aq )|| AgCl(s )| Ag(s )
14. (c) We know that, 26. (d) Impure sample is made of the anode and pure copper
F ⋅w 96500 × 1.8
E= = = 19.3 acts as the cathode.
it 3 × 50 × 60
27. (b) The reaction of disproportionation is
15. (d) KCl r K + + Cl− and K + + e − → K 2Cu+ → Cu2+ + Cu
Thus, one mole KCl need one mole electrons to liberate ∴ E °Cu+ /Cu2+ , = E °Cu+ /Cu2+ + E °Cu2+ /Cu
one gram metal atom. One mole electron charge is equal = ( −E °Cu2+ /Cu+ ) + E °Cu2+ /Cu
to 96500 C (1 F). Hence, 1 F charge will liberate one gram
atom of metal from solution of KCl. = − 0.15 + 0.34 = + 0.19 V

@iitjeehelps
528 SELF STUDY GUIDE BITSAT

28. (b) 40. (b) Given, E°FeO/ Fe = − 0.87 V and E°Ni2O3 /NiO = 0.40 V
29. (b) Since, reduction potential of electrode, A is lower than ∴ E°Fe/ FeO = + 0.87 V
that of B.
We have, E °cell = E °cathode − E °anode and E°NiO/ Ni2O3 = 0.40 V
= 0.42 − ( −0.35) = 0.77 V Since, E°oxi for Fe/ FeO > E°oxi for NiO / Ni2O3
30. (c) The metal placed above in electrochemical series, can ∴Redox changes
displace the metals placed below, from their salt solution. At anode Fe + 2OH − → FeO (s ) + H2O (l ) + 2e −
Hence, Zn can displace Fe, Ag, Cu.
At cathode Ni2O3(s ) + H2O (l ) + 2e − → NiO(s ) + 2OH−
31. (a) Cu is placed above Ag in electrochemical series.
Hence, Cu can displace Ag from AgNO3 and itself is Overall reaction Fe(s )+ Ni2O3(s ) → FeO(s )+ 2NiO(s )
oxidised to Cu+ (blue) ions. Hence, E °cell = E °anode− E °cathode = 0.87 − ( −0.40)
Cu + 2AgNO3 → Cu(NO3 )2 + 2Ag+ or E°cell = 1.27 V
32. (a) Greater the reduction potential, stronger is the and ∆G ° = nFE °cell = 2 × 1.27 × 96500
oxidising agent. Hence,Y is stronger oxidising agent than = 245110 J
X but weaker than Z .
= 245.11 kJ
33. (d) Ag+ and Fe3+ are is not feasible because they have
RT [Cu2+ ]
almost same standard electrode potential. 41. (d) Ecell = E °cell − ln
nF [Ag+ ] 2
34. (c) From the equation, given cell represented as
Doubling [Cu2+ ], decreases the emf by a small fraction.
H2 | H+ (aq )|| Ag+ (aq )| Ag
42. (d) The cell reaction is
Hence, Ecell = E Ag+ /Ag − E 1
H+ / H2 Zn(s ) → Zn2+ (0.01M) + 2e −
or 0.80 = E Ag+ /Ag − 0.0 2

[Ag+ (1.25M) + e − → Ag(s )] × 2


∴ E Ag+ /Ag = 0.80 V
[Zn (s ) + 2Ag+ (1.25 M) → Zn2+ (0.01 M) + 2Ag(s )
Hence, Eoxi = − 0.80 V
[Zn2+ ] 0.01
35. (a) We know that ∆G ° = − 2.303 RT log KC Q= + 2
= = 6.4 × 10−3
[Ag ] (1.25)2
= − 2.303 RT log 1012= − 12 × 2.303 RT
43. (c) Relation between K eq and E°cell is
Also, ∆G ° = − nFE °cell
0.0591
or E °cell =
2.303 RT
× 12 =
0.059
× 12 ; (n = 2) = 0.354 V E °cell = log K eq (at 298 K)
nF 2 n
0.0591
36. (b) Ecell = E ° − pH × 0.059 = 0.00 − ( 0) × 0.059 0 . 591 = log K eq
1
(For hydrogen electrode, E° = 0)
∴ K eq = 1 × 1010
= 0.0 V
0.0591 1
37. (b) ∆G = − nFE 44. (b) E 1 = E° 1 − log −10
H+ / H2 H+ / H2 1 10
2 2
If E = + ve, then ∆G = − ve or < 0
= − 0.0591 × 10 = − 0.591 V
38. (a) The cell reaction is
45. (c) ∆r G = − 2 × 1.1× 96487
2Cr + 3Fe2+ → 2Cr 3+ + 3Fe (n = 6)
= − 212 . 27 kJ mol−1
From Nernst equation,
0.059 [Cr 3+ ] 2 46. (d) When a lead storage battery is charged, it acts as an
Ecell = E °cell − log electrolytic cell.
n [Fe2+ ] 3
47. (d) Fuel cell works when potential difference is
0.059 ( 0.1)2 developed.
= [ −0.44 + 0.74] − × log
6 ( 0.01)3 48. (d) Corrosion is speed up by air, water, electrolytes and
= + 0.2606 V impurities in metal.
39. (b) 49. (a)

@iitjeehelps
BITSAT Archives
1. (d) Cu2+ + 2e − → Cu 9. (c) Number of moles of hydrogen collected
0.059 1 1120 × 10− 3
ECu2 + /Cu = E ° 2 + − log = = 0.05
Cu /Cu 2 [Cu2+ ] 22.4
RT Q 1 mole of hydrogen is deposited by 2
= E °Cu2 + /Cu + ln [Cu2+ ]
2F moles of electrons.
ECu2 + /Cu = 0.34 +
0.059
log 0.1 = 0.31 V ∴0.05 mole of hydrogen will be deposited by 2 × 0.05
2 = 0.10 mole of electrons
0.059 Charge, Q = nF
E Cu /Cu2+ = − ECu2 + /Cu = − 0.34 + V
2
= 0.1 × 96500
2. (d) E ° 3+ 2+ = 3 E° 3+ − 2 E° 2+ Charge, Q = it
Fe /Fe Fe / Fe Fe /Fe

= [ 3 − 0.36) − 2 ( − 0.439)] V 0.1 × 96500 = i × 1930


3. (c) Λ∞m for BaCl2 = Λ∞m Ba 2+
+ 2Λ∞m Cl − 0.1 × 96500
i= = 5.0 A
1930
1
∴ Λ∞eq for BaCl2 = Λ∞m Ba 2+ + Λ∞m Cl− 10. (a) Fe2 + + 2e − → Fe ; ∆G ° = − nFE ° ...(i)
2
1 ∆G° = − 2 × F × ( − 0.440 V ) = 0.880 F
= × 127 + 76 = 139.5 Ω −1 cm 2
2 Fe3 + + 3e − → Fe ...(ii)
0.059
4. (a) E° = log KC ∆G° = − 3 × F × ( − 0.036)
n
= 0.108 F
0.059
0.46 = log KC On subtracting Eq. (ii) form Eq. (i) we get
2
0.46 × 2 Fe3+ + e − → Fe2+
log KC = = 15.6
0.059 ∆G° = 0.108 F − 0.880 F = − 0.772 F
KC = antilog 15.6 − ∆G ° ( − 0.772F)
E° = = = + 0.772 V
5. (b) Fe + Cu2+ → Fe2+ + Cu nF 1× F
Oxidation Reduction 1
11. (b) Specific conductivity ( κ ) = × cell constant
° = E° 2+ + E° 2+
Ecell R
Fe/Fe Cu /Cu

= 0.44 + 0.32 Cell constant = κ × R = 0.0129 × 100 = 1. 29

= 0.76 V 12. (b) According to Faraday's law of electrolysis


i ×t ×E w ×F  It  M
6. (a)w = ⇒E= m= 
F i ×t F  Z
0.108 × 96500 t=
mZF
= = 108 or,
0.5 × 193 IM
7. (c) As the reduction potential of hydrogen is lower than The factor of equivalence can be found dividing the
that of sodium, it will be discharged at the cathode and number of electrons by the number of reacted particles.
2
chloride will be discharged at the anode. In case of MnO2, the factor is Z = = 1
2
8. (c) According to Faraday's second law of electrolysis,
The molar mass of MnO2 is M (MnO2 ) = 55 + 16 × 2
Weight of silver deposited Eq. wt. of silver
= = 87 g/mol
Weight of hydrogen liberated Eq. wt. of hydrogen
8 × 1 × 96500
w 108 Now, t = = 4.436 × 10 s = 1232 = 51. 3 days
6
= 0.002 × 87
5.04 × 10− 2 1.008
13. (a)The standard reduction potential of the cell reaction is
108 × 5.04 × 10− 2
w = = 5.4 g RT
1.008 E0 = ln Kw
F

@iitjeehelps
14
Hydrogen

Occurrence of Hydrogen
Dihydrogen (H 2 ) is the most abundant element in the universe (70% of the total mass of the universe)
and is the principal element in the solar atmosphere. The giant planets, Jupiter and Saturn consist of
mainly hydrogen.

Position of Hydrogen in Periodic Table


The position of hydrogen in periodic table is uncertain as it shows resemblance with alkali metals as
well as with halogens. However, on the basis of electronic configuration (1s 1 ), it is placed above
lithium in the periodic table but still it is not considered as the member of that group. It is the lightest
element known.

Isotopes and Allotropes of Hydrogen


Hydrogen has three isotopes : protium ( 11 H ), deuterium or heavy hydrogen ( 21 H or D) and tritium
(31 H or T). Out of these isotopes, only tritium is radioactive and emits low energy of b-particles. These
three isotopes have different masses, hence, their rates of reaction and equilibrium constants are
different. This is known as isotopic effect.

Dihydrogen or Hydrogen Gas (H2 )


Hydrogen being highly reactive, it exists in diatomic form, called the dihydrogen or hydrogen gas.

Methods of Preparation of Dihydrogen


Dihydrogen can be prepared by the following methods :
(i) Laboratory method In laboratory, dihydrogen is produced by the reaction of Zn with dil. H 2SO4 .
Zn + H 2SO4 ¾® ZnSO4 + H 2 ­
(ii) By the electrolysis of water In electrolysis, acidified water is electrolysed by using platinum
electrodes as
Electrolysis
Water H2↑ + O2↑
Containing small Traces of Cathode Anode
amount of acid acid/base
or alkali

@iitjeehelps
HYDROGEN 531
(iii) High purity (> 99.95%) dihydrogen is obtained by compounds (like H 2O, NH3 , HX etc) at higher
electrolysing warm aqueous barium hydroxide solution temperatures.
between nickel electrodes. (iv) Reaction with metals Under suitable condition,
(iv) By the action of certain active metals on water or any metals (like Li, Na, K, Cs, Ca etc) react with H 2 to
other protic solvent. form hydrides which are chiefly ionic, good
e.g. 2Al + 2NaOH + 2H 2O ¾® 2NaAlO2 + 3H 2 ­ reducing agents.
Zn + 2NaOH ¾® Na 2 ZnO2 + H 2 ­ (v) Reducing action Dihydrogen has the property to
reduce the oxides of less electropositive elements.
CaH 2 + 2H 2O ¾® Ca(OH)2 + 2H 2 ­
e.g. CuO + H 2 ¾® Cu + H 2O
(v) Industrial methods of preparation of dihydrogen
Certain metal chlorides also get reduced as
It involve following processes :
2AgCl + H 2 ¾® 2AgHCl
(a) Bosch process However, hydrogen does not have the ability to
C + H 2O ¾® CO + H 2 reduce the oxides of more electropositive elements
Coke Superheated Water gas
(at 400-660 °C) steam or syn gas
like alkali metals and alkaline earth metals.
Reacts with steam again
in the presence of catalyst
Uses of Dihydrogen
like Fe, Cr and Co

The important uses of dihydrogen are as follows:


CO2 + H 2
(i) In the manufacturing of vanaspati fat by the
The CO2 is removed either by washing under pressure hydrogenation of polyunsaturated vegetable oils
of 25-30 atm or by reacting with potassium hydroxide. like soyabean, cotton seeds etc.
(b) Lane process (ii) In oxyhydrogen and atomic hydrogen torches for
3 Fe + 4 H 2O ¾® Fe3O4 + 4H 2 ­ cutting and welding of substances.
(900 °C) Superheated æç CO is passed over ö÷
steam è it and reduces to Fe ø (iii) Dihydrogen is considered as very good fuel as it
does not cause any pollution and produces greater
(c) In Nelson or Castner Kellner cell, H 2 is also produced energy per unit mass of fuel as compared to
by the electrolysis of brine solution (NaCl) in them. gasoline or other fuels. H 2 -O2 fuel cells generate
electrical power with conversion efficiency
of 70-85%.
Properties
Physical Properties of Dihydrogen Hydrides
Some physical properties are given below: Hydrogen combines with a number of metals and
æ Cp ö non-metals to form as hydrides.
(i) Hydrogen is a diatomic ç g = = 1.40÷ , colourless,
è CV ø Hydrides are mainly of three types :
tasteless gas with no characteristic odour. It is lighter Ionic or Saline Hydrides
than air and insoluble in water.
These are formed by metals which are highly
(ii) Its critical temperature is very low (– 236.9°C). It can be electropositive in character, i.e. alkali and alkaline earth
liquefied at or below –236.9°C by application of suitable metals (except Be and Mg), e.g. LiH, CaH 2 etc. These are
pressure and solidifies at –259.8°C. crystalline solids with ionic lattices and behave like salts.
(iii) Certain metals like Ni, Pt, Pd, Fe, Au etc., can adsorb or
occlude large volumes of H 2 gas at different temperatures Molecular or Covalent Hydrides
(Pd can occlude 1000 times of its own volume). These are formed by elements of comparatively higher
electronegativity, generally p-block elements. In most
Chemical Properties of Dihydrogen cases, bonds are covalent in character, although in some
cases (e. g. HF), bond is partly ionic in character.
Important chemical reactions of dihydrogen are given below:
Molecules are held together by weak van der Waals’
(i) Nature Dihydrogen is neutral in nature. forces. These hydrides are soft, have low melting point
(ii) Combustion Dihydrogen is highly combustible and it and boiling point with low electrical conductivity.
burns with pale blue flame in the presence of air. These hydrides are of following three types:
2H 2 + O2 ¾® 2H 2O ; ( DH = - 285.9 kJ mol -1 ) (i) Electron deficient hydrides
(iii) Reaction with non-metals With non-metals (ii) Electron precise hydrides
(like O2 , N 2 , F2 , Cl 2 , Br2 etc) hydrogen forms covalent (iii) Electron rich hydrides

@iitjeehelps
532 SELF STUDY GUIDE BITSAT

Metallic or Interstitial Hydrides Hard and Soft Water


These are formed by Be, Mg (both are s-block elements) and
The water which lathers with soap is soft, if not, is hard.
transition (d-block) elements. These are interstitial
compounds as the hydrogen atoms occupy interstitial Hardness of water is of two types:
places in the metal lattices. These are non-stoichiometric (i) Temporary hardness
compounds (e.g. VH0.56 , NiH0.6 - 0.7 etc.) and give out (ii) Permanent hardness
hydrogen easily and thus, used as reducing agents.
Temporary Hardness of Water
It is due to the presence of magnesium and calcium
Water (H2O) hydrogen carbonates. It can be removed by Clark’s method,
Water is very essential for the existence of life. It constitutes which involves the addition of calculated amount of lime to
about three-fourth of the earth’s surface and two-third of hard water.
human body. Ca(HCO3 )2 + Ca(OH)2 ¾® 2CaCO3 ¯ + 2H 2O
Structure of Water Mg(HCO3 )2 + 2Ca(OH)2 ¾®
Water molecule is a bent molecule with a bond angle 2CaCO3 ¯ + Mg(OH)2 ¯ + 2H 2O
of 104.5°. In water molecules, the central atom (oxygen) is
sp3 -hybridised. Because of high electronegativity of oxygen,
Permanent Hardness of Water
several H 2O molecules associated through hydrogen It is due to the presence of soluble salts of magnesium and
bonding (O¾ HLLO). calcium in the form of chlorides and sulphates.
H H It can be removed by the following methods:
O
2M 2 + + Na 2 [Na 4 (PO 3 )6] ¾® Na 2[ M 2 (PO 3 )6]+ 4Na +
Calgon Soluble
H
O H (M 2+
= Ca , Mg )
2+ 2+

O 95 H O
H
pm

.7 H H ● Ion-exchange method or Permutit process is also


pm O
.7

O
95

104.5° H used to remove permanent hardness.


H H
H 2Na Z(s) + M 2+ (aq ) ¾® MZ2 (s ) + 2Na + (aq )
Structure of H 2O ( M = Mg, Ca)
Ice has a highly ordered 3 dimensional H-bonded structure where, Na Z is sodium aluminium silicate (NaAlSiO4 ).
in which each oxygen atom is surrounded tetrahedrally by This is also known as sodium zeolite. It is regenerated for
four other oxygen atoms. further use by treating with an aqueous NaCl solution
(brine).
Physical Properties of Water ● Cation exchange resins contain large organic molecules
Water is colourless, odourless and tasteless liquid. Due to with ¾ SO3H group and are water insoluble. Ion-exchange
its high dielectric constant and polar nature, water is an
excellent solvent for polar and ionic substances. resin ( RSO3H) is changed to RNa by treating it with NaCl.
2 RNa + M 2+ (aq ) ¾® R2 M (s ) + 2Na + (aq )
Chemical Properties of Water
( M 2+ = Ca 2+ , Mg 2+ )
The chemical properties of water are below:
The resin can be regenerated by adding aqueous NaCl
(i) It behaves as an amphoteric substance. solution.
H 2O(l ) + NH3 (aq ) s OH - (aq ) + NH 4+ (aq )
Acid Base
Heavy Water (D2O)
H 2O(l ) + H 2S(aq ) s H3O+ (aq ) + HS- (aq )
Base Acid It has quite similar physical and chemical properties to
(ii) It is oxidised to O2 with fluorine. those of H 2O. Dielectric constant of D2O is lower than that of
H2 O and rate of reactions are much slower than H 2O.
2F2 ( g ) + 2H 2O(l ) ¾® 4H + (aq ) + 4F - (aq ) + O2 ( g )
It is used as a moderator in nuclear reactions, as trace
(iii) The association of water is essentially of 5 types
compound for studying reaction mechanism, for the
namely coordinated water, hydrogen bonded water,
lattice water, zeolitic water and clathrate water. preparation of deuterium.

@iitjeehelps
HYDROGEN 533

Hydrogen Peroxide (H2O2 ) (iii) Oxidising action in basic medium


Mn 2+ + H 2O2 ¾® Mn 4 + + 2OH -
It is a compound with an oxygen-oxygen single bond. It is
also a strong oxidiser. (iv) Reducing action in basic medium
I 2 + H 2O2 + 2OH - ¾® 2I - + 2H 2O + O2
Preparation
BaO2 × 8H 2O(s ) + H 2SO4 (aq ) ¾® Storage
BaSO4 (s ) + H 2O2 (aq ) + 8H 2O(l ) H 2O 2 decomposes slowly on exposure to light.
H 2O2 is commercially represented in terms of volume 2H 2O2 (l ) ¾® 2H 2O(l ) + O2 ( g )
strength.
In the presence of metal surfaces or traces of alkali
(i) Volume strength = 5.6 ´ Normality (present in glass containers), the above is catalysed. It is
Percentage strength therefore, stored in wax lined glass or plastic vessels in
= 5.6 ´ ´ 10
Eq. wt. of H 2O2 (i.e. 17) dark. Urea can be added as a stabiliser.
(ii) Volume strength = 11.2 ´ Molarity Uses of H2O2
Percentage strength 1. Aqueous solution of H 2O2 is used as germicide,
= 11.2 ´ ´ 10
Mol. wt. of H 2O2 (i.e. 34) antiseptic, preservative for milk and wine, bleaching
17 agent for soft materials.
(iii) % strength = ´ Volume strength
56 30% H 2O2 is called perhydrol. Its volume strength is
100 and molarity is 8.8.
Chemical Properties 2. It is used as an antichlor and in refreshing old oil
The chemical properties are shown below: paintings which became black.
(i) Oxidising action in acidic medium Structure of H2O2
2Fe 2+ (aq ) + 2H + (aq ) + H 2O2 (aq ) ¾® °
7A H
2Fe3 + (aq ) + 2H 2O(l ) 0 .9
O
97°
PbS(s ) + 4H 2O2 ¾® PbSO4 (s ) + 4H 2O(l )
1.48 A°
(ii) Reducing action in acidic medium 97°
O
2MnO4- + 6H + + 5H 2O2 ¾® 2Mn 2+ + 8H 2O + 5O2
H

Practice Exercise
1. Why does H+ ion always get associated with other d. Hydrogen can form compounds with almost all other
atoms or molecules? elements
a. Ionisation enthalpy of hydrogen resembles to that of 3. Ortho and para hydrogens differ in the
alkali metals a. number of protons
b. Its reactivity is similar to halogens b. molecular weight
c. It resembles both alkali metals and halogens c. nature of spins of protons
d. Loss of an electron from hydrogen atom results in a d. nature of spins of electrons
nucleus of very small size as compared to other
atoms or ions. Due to small size, it cannot exist free 4. Consider the following statements about ortho and
para-hydrogen
2. Which of the following explanation justifies for not I. In ortho -hydrogen, the spin of protons are in the
placing hydrogen in either the group of alkali metals or same direction.
halogens? II. ortho -hydrogen is more stable than the para form
a. Ionisation energy of the hydrogen is too high for in the ambident condition.
group of alkali metals and too low for the halogen
group III. At ordinary temperature, ordinary hydrogen is a
mixture of about 75% para and 25% ortho forms.
b. Hydrogen atom does not contain any neutron
c. Hydrogen is much lighter than the alkali metals or IV. Two forms have similar chemical properties but
halogens differ in physical properties like specific heat and
thermal conductivity.

@iitjeehelps
534 SELF STUDY GUIDE BITSAT

Which of the statements given above are correct? c. CH3CH2CHO and CH3CH2OH
a. I, II and III b. II, III and IV d. CH3CH2CH2CHO and CH3CH2CH2OH
c. I, III and IV d. I, II and IV 13. Dihydrogen is used in the
5. Which of the following reactions increase production I. manufacturing of nitric acid and nitrogenous
of dihydrogen from synthesis gas? fertilisers.
a.CH 4(g) + H2O (g ) ¾1270
¾¾ ¾K
® CO (g ) + 3H2 (g ) II. manufacturing of vanaspati fat.
Ni
III. manufacturing of methanol.
b. C (s ) + H2O (g ) ¾1270
¾¾ ¾K
® CO (g ) + H2 (g )
IV. preparation of hydrogen chloride.
c. CO (g ) + H2O (g ) ¾ 673
¾¾ K
® CO2 (g ) + H2(g ) Choose the correct option.
Catalyst
a. I, II and IV b. II, III and IV
d. C2H6 + 2H2O ¾ ¾¾
¾® 2CO + 5 H2
1270 K
c. I, II and III d. I, II, III and IV
Ni

6. The ionisation of hydrogen atom gives 14. H2 gas is liberated at cathode and anode both by the
a. hydride ion b. hydronium ion electrolysis of the following aqueous solution except in
c. proton d. hydroxyl ion a. NaH b. KH
c. NaCl d. Both a and b
7. Pure H2 is obtained by the action of
15. Hydrogen is prepared on large scale for an industrial
a. aluminium with potassium hydroxide
use
b. sodium hydride with water
a. by Zn + H2SO4 b. by Al + NaOH
c. electrolysis of warm solution of Ba(OH)2 using Ni c. by Na + C2H5OH d. from water gas
electrodes
d. All of the above 16. Which of the following is incorrect statement?
a. s-block elements, except Be and Mg, form ionic
8. Hydrogen adsorbed on palladium is known as hydride
a. atomic H b. ortho H c. occluded H d. heavy H b. BeH4, MgH2, CuH2, ZnH2, CaH2 and HgH2 are
9. The chemical reactions of dihydrogen is accomplishes intermediate hydrides
c. p-block elements form covalent hydride
by the
d. d and f-block elements form ionic hydride
a. loss of the only one electron to give H+
b. gain of an electron to form H- 17. The correct decreasing order of basic strength of
c. sharing electrons to form a single covalent bond hydrides is
d. All of the above a. AsH3 > SbH3 > PH3 > NH3
b. SbH3 > AsH3 > PH3 > NH3
10. Consider the following reactions,
c. NH3 > PH3 > AsH3 > SbH3
I. H2( g ) + X 2( g ) ¾® 2HX ( g ) (X = F, Cl, Br, I) d. PH3 > AsH3 > SbH3 > NH3
Catalyst or
II. 2H2( g ) + O2( g ) ¾¾¾¾® 2H2O (l ) 18. The maximum possible number of hydrogen bonds in
heating
a water molecule can form in ice is
III. 3H2( g ) + N2 ( g ) ¾¾¾¾¾¾®
673 K, 200 atm
2NH3 ( g ) a. 1 b. 2 c. 3 d. 4
Fe
19. Consider the following statements about
IV. H2( g ) + 2M ( g ) ¾® 2M H(s ) (M = alkali metal)
intermolecular and intramolecular hydrogen bonds.
V. H2( g ) + Pd 2+ (aq ) ¾® Pd(s ) + 2H+ (aq ) I. Both types of H-bonds are temperature dependent.
The correct reactions are II. Intramolecular H-bond disappears on increasing
a. II, III and V b. IV and V the concentration.
c. I, II and IV d. All of these III. Intramolecular H-bond disappears on decreasing
the concentration.
11. Hydrogenation of vegetable oils using nickel as a
IV. The boiling point of compounds having
catalyst gives edible fats is known as
intramolecular H-bond are lower than that of those
a. coconut oil
compounds which have intermolecular H-bond.
b. soyabean oil
c. margarine and vanaspati ghee Which of the statements given above are correct?
d. vanaspati ghee a. I, II and IV b. III and IV
12. H2 + CO + R CH == CH2 ¾® X c. I, III and IV d. I and II
½ H2 20. The boiling point of water is high because
¯ a. water molecule is linear
Y b. water molecule is not linear
What are X and Y in the above reaction? c. water molecule possess covalent bond between H
a. R CH2CH2CHO and R CH2CH2CH2OH and O
b. R CH2CH2CHO and R CH2CH2OH d. water molecules associate due to H-bonding

@iitjeehelps
HYDROGEN 535
21. When two ice cubes are pressed over each other, 32. Which of the following is not correct regarding the
they united to form one cube. Which of the following electrolytic preparation of H2O2?
forces are responsible to hold them together? a. Lead is used as cathode
a. Ionic interaction b. 50% H2SO4 is used
b. van der Waals’ forces c. Hydrogen is liberated at anode
c. Covalent interaction d. Sulphuric acid undergoes oxidation
d. Hydrogen bond formation
33. Match the Column I with Column II and select the
22. If one assume linear structure instead of bent correct option from the codes given below.
structure of water then which one of the following
properties cannot be explained? Column I Column II
a. Formation of intermolecular hydrogen bonding in A. 10 vol H2O 2 1. Perhydrol
water
B. 20 vol H2O 2 2. 5.358 N
b. High boiling point of water
c. Solubility of polar compounds in water C. 30 vol H2O 2 3. 1.785 M
d. Ability of water to form coordinate covalent bonds D. 100 vol H2O 2 4. 3.03%
23. The strength of an oxo acid (E ¾ O ¾ H ), where E is
Codes
the central atom, depends upon the A B C D
a. electronegativity of E a. 4 3 2 1
b. atomic size of E b. 1 2 3 4
c. ability of E to share electron pair with O c. 1 3 2 4
d. atomic size and electronegativity of E d. 4 2 3 1
24. Heavy water (D2O) freezes at 34. Study the following reactions carefully
a. 0°C b. 3.8°C c. - 3.8°C d. 38°C I. HOCl + H2O2 ¾® H3 O+ + Cl- + O2
25. Boiling point of heavy water is II. PbS + 4H2O2 ¾® PbSO4 + 4H2O
a. 100°C b. 99°C c. 101.4°C d. 110°C
Point out the correct option.
26. Which of the following statements are correct a. In (I), HOCl is reduced and in (II). PbS is oxidised
regarding D2O and H2O? b. In (I), HOCl is oxidised and in (II). PbS is reduced
I. D2O reacts with Al4 C3 at a faster rate than does c. In both (I) and (II), HOCl and PbS are reduced
H2O. d. In both (I) and (II), HOCl and PbS are oxidised
II. The freezing point of D2O is higher than that of H2O. 35. 6 volume sample of H2O2
III. NaCl is more soluble in D2O than in H2O. a. would give 6 volumes of oxygen per unit volume of
IV. Ionic product of D2O is smaller than that of H2O. H2O2 sample at STP
Select the correct answer using the codes given b. will contain 6% V/V of H2O2
c. will contain 6% W/V of H2O2
below.
d. would give 6 volumes of oxygen per unit weight of
a. I and II b. I and III c. II and III d. II and IV H2O2 sample at STP
27. Mass percentage of deuterium in heavy water is 36. When zeolite, which is hydrated sodium aluminium
a. same as that of protium in water silicate, is treated with hard water, the sodium ions are
b. 11.1 exchanged with
c. 20.0 a. H + ion b. Ca 2+ ion c. SO 2- d. OH - ion
4 ion
d. Cannot be predicted
37. 1000 g aqueous solution of CaCO3 contains 10 g of
28. A mixture of hydrazine (N2H4 ) and 58-60% solution of
calcium carbonate, hardness of the solution is
H2O2 is used as a. 10 ppm b. 100 ppm c. 1000 ppm d. 10000 ppm
a. antiseptic b. fertiliser c. rocket fuel d. None
38. 100 mL of tap water containing Ca(HCO3 )2 was
29. Which of the following oxides is a peroxide?
N
a. Na 2O2 b. MnO2 c. BaO d. SO2 titrated with HCl with methyl orange as an indicator.
50
30. Hydrogen peroxide is used as If 30 mL of HCl was required, the temporary hardness
a. an oxidant only b. a reductant only
c. an acid only d. All of these of water, as parts of CaCO3 per 106 parts of water was
a. 150 ppm b. 600 ppm c. 275 ppm d. 300 ppm
31. Which of the following statements are not correct ?
a. H2O2 oxidises Fe (II) to Fe (III) 39. Excess of Kl and dil. H2SO4 were mixed in 50 mL
b. H2O2 can be obtained by the electrolysis of dil. H2SO4 H2O2 . Thus, I2 liberated requires 20 mL of 0.1 N
c. H2O2 reduces Mn (VII) to Mn (II) Na 2S 2O3 . What will be the strength of H2O2 in g L-1?
d. H2O2 is a weak base a. 0.034 b. 0.68 c. 6.80 d. 5.80

@iitjeehelps
536 SELF STUDY GUIDE BITSAT

40. H2O2 is marked ‘22.4 volume’. How much of it (in mL) 42. Both temporary and permanent hardness is removed
are required to oxidise 3.4 g H2S gas? on boiling water with
a. 10 mL b. 70 mL c. 100 mL d. 1000 mL a. CaSO4 b. Na 2CO3 c. CaCO3 d. CaO
41. Temporary hardness of water is caused due to the 43. Which one of the following is used for reviving the
presence of exhausted permutit?
a. CaSO4 b. CaCl2 a. HCl solution b. 10% CaCl2 solution
c. CaCO3 d. Ca(HCO3 )2 c. 10% MgCl2 solution d. 10% NaCl solution

BITSAT Archives
1. Which one of the following is a covalent hydride? 4. H2O2 used in rocket has the concentration [2007]
[2014]]
a. 50%
a. CaH2 b. NaH c. BH3 d. BeH2 b. 70%
2. Calcium carbide reacts with heavy water to form [2013] c. 30%
a. C2D2 b. CaD2 c. CaD2O d. CD2 d. 90%

3. H2O2 is always stored in black bottles because [2009] 5. Calgon used as water softner is [2006]
a. Na 2[Na 4(PO3 )6]
a. it is highly unstable b. Na 4[Na 2(PO3 )6]
b. its enthalpy of decomposition is high c. Na 2[Na 4(PO4 )5]
c. it undergoes autoxidation on prolonged standing d. None of the above
d. None of the above

Answer with Solutions


Practice Exercise 15. (d)
16. (d)
1. (d) 2. (a)
17. (c)The correct order of basic strength is
3. (c) ortho and para hydrogens differ in the nature of spins
of protons. NH3 > PH3 > AsH3 > SbH3
4. (d) Statement (III) is incorrect at ordinary temperature 18. (d)
(25°C), ordinary hydrogen contains 75% ortho and 25% 19. (a) Statements I, II and IV are correct.
para forms. 20. (d) Water molecules are associated with H-bonding, due
5. (c) CO (g ) + H2O (g ) ¾ 673
¾¾ K
® CO2(g ) + H2 (g ) to which boiling point of water becomes high.
Catalyst
21. (d) Two ice cubes (solid H2O ) united to form one cube
6. (c) H (g ) ¾¾® H + (g ) + e - due to the formation of H-bond.
7. (d) 8. (c) 9. (d) 22. (b) The high boiling point of water cannot be explained on
10. (d) All reactions are correct. the basis of bent structure instead of linear structure.
11. (c) 23. (a) Higher electronegativity reduces the strength of acid.
12. (a) Hydroformylation of olefins yields aldehydes which 24. (b) Heavy water freezes at 3.8°C.
further undergo reduction to give alcohols. 25. (c) Boiling point of heavy water is 101.4°C.
H2 + CO + R CH == CH2 ¾® R CH2CH2CHO 26. (d) D2O reacts slower than H2O due to greater mass, its
H2 + R CH2CH2CHO ¾® R CH2CH2CH2OH freezing point is 3.8°C. NaCl is less soluble in D2O as in
13. (d) H2O and D2O does not dissociate therefore, its ionic
product is smaller than H2O.
14. (c) Salt ionisation-3
27. (c) Formula of heavy water is D2O, i.e. molecular mass
(i) NaH r Na + + H–
= 20
(ii) HCOONa r HCOO– + Na + 4 ´ 100
% of deuterium =
(iii) NaCl r Na + + Cl– 20
(iv) LiH r Li+ + H- = 20%

@iitjeehelps
HYDROGEN 537
28. (c) A mixture of H2O2 and hydrazine (N2H4 ) is used as a w 1
´ 1000 = ´ 30
rocket fuel. 100 / 2 50
29. (a) Na 2O2 is a peroxide, all others are double oxides 3 ´ 100
w CaCO3 = = 0.03 g
(MnO2, SO2 ) and simple oxide (BaO). 5 ´ 2000
30. (d) H2O2 acts as an oxidant, reductant and an acid \100 mL of H2O contains Ca 2+ = 0.03 g
(weak). 0.03 ´ 106
106 mL H2O contains Ca 2+ = = 300 ppm
31. (d) H2O2 is not a weak base but a weak acid. 100

32. (c) H2O2 can be prepared by electrolysis of 50% H2SO4. 39. (b) For 20 mL of H2O2
In this method, hydrogen is liberated at cathode. Meq. of KI = Meq. of H2O2 in 50 mL = Meq. of Na 2S2O3
H2SO4 r 2H+ + HSO–4 w ´ 1000
= 20 ´ 0.1
34 / 2
At anode 2HSO4 ¾® H2S2O8 + 2e -
20 ´ 0.1 ´ 34
H2S2O8 + 2H2O ¾® 2H2SO4 + H2O2 \ w H2O2 in 50 mL = = 0.034
2000
At cathode 2H+ + 2e- ¾® H2 ­ 0.034 ´ 1000
\ w H 2O2 in 1000 mL = = 0.68 g /L
33. (a) 50
68 x
34. (a) Reduction 40. (c) Density of ‘x volume’ H2O2 = g / mL
22400
+1 –1 –1 0 68 ´ 22.4
HOCl + H2O2 H3O+ + Cl– + O2 Density of ‘22.4 volume’ = = 0.068 g/mol
22400
Oxidised H2S + H2O2 ¾¾® 2 H2O + S
Oxidation 34 g 68 g

34 g H2S is oxidised by = 68 g H2O2


–2 –1 +6 2– 68 ´ 3.4
PbS + 4H2O2 PbSO4 + 4H2O 3.4 g H2S is oxidised by = = 6.8 g H2O2
34
Reduction
0.068 g H2O2 is present in = 1mL
\ 6.8 g H2O2 is present in = 100 mL
35. (a) 6 volume sample of H2O2 means 6 volumes of oxygen
are given out per unit volume of H2O2 sample at STP. 41. (d) Temporary hardness of water is caused due to the
presence of bicarbonates of Ca and Mg.
36. (b)
Na 2Z + CaCl2 ¾¾® CaZ + 2NaCl 42. (b) Ca(HCO3 )2 + Na 2CO3 ¾® CaCO3 + 2 Na(HCO3 )
Sodium zeolite From hard water Calcium zeolite CaCl2 + Na 2CO3 ¾® CaCO3 + 2 NaCl
[where, Z = Al2Si2O8 × x H2O] Ca 2+ or Mg2+ ions are removed as insoluble carbonates.
10 ´ 106 43. (d)
37. (d) Hardness of solution in ppm = = 104 ppm
1000
38. (d) Meq. of CaCO3 = Meq. of Ca (HCO3 )2 = Meq. of HCl

BITSAT Archives
1. (c) Hydrides are binary compounds of hydrogen. 2. (a)
These can be classified into four groups : 3. (c) H2O2 is always stored in black bottles because H2O2
decomposes into water and oxygen in the presence of light.
(i) lonic hydrides : NaH,CaH 2, LiH
2H2O2 (l ) ¾® 2H2O (l ) + O2 (g )
(ii) Covalent hydrides : B 2H 6, NH 3, SbH 3
4. (d) H2O2 is used as an oxidant for rocket fuel and has
(iii) Polynuclear hydrides : LiAIH 4, NaBH 4
90% concentration to be used in rockets.
(iv) Interstitial hydrides are those in which hydrogen is 5. (a) Calgon used as water softner. The chemical
trapped in the interstitial spaces of transition metals. composition is Na 2[Na 4(PO3 )6].

@iitjeehelps
15
s-Block Elements

General Introduction of s-Block Elements


The components in the long form of the periodic table have been partitioned into four blocks in
particular s, p, d and f- blocks. The elements of Group 1 (I A) and 2 (II A) have their last electron in
s-orbital thus, they are called s-block elements.

Group 1 Elements : Alkali Metals


The Group 1 elements comprising Li, Na, K, Rb, Cs and Fr are commonly called alkali metals.
Francium is radioactive and has a very short life (half-life of 21 min). These are known as alkali metals
because their water soluble hydroxides are strongly alkaline. Sodium and potassium are abundant
and lithium, rubidium and caesium have much lower abundance.

Electronic Configuration of Alkali Metals


The general electronic configuration of alkali metals is [noble gas] ns1 (n = 2 to 7) .

Group 2 Elements : Alkaline Earth Metals


Group 2 elements are basically called alkaline earth metals as their oxides and hydroxides are
alkaline in nature and these metal oxides are found in the earth’s crust. Calcium and magnesium
rank fifth and sixth in abundance respectively in the earth’s crust. Strontium and barium have
much lower abundances. Beryllium is rare and radium is the rarest of all comprising only 10-10 per
cent of igneous rocks.

Electronic Configuration of Alkaline Earth Metals


The general electronic configuration of alkaline earth metals is [noble gas] ns 2 (n = 2 to 7).

@iitjeehelps
s-BLOCK ELEMENTS 539
The high conductivity and paramagnetic properties
Chemical Properties of s-Block Elements
along with blue colour is attributed to the presence of
(i) Reaction with oxygen (air) Except Li, all alkali metals these ammoniated electron as well as ammoniated
are tarnish in air due to oxide formation at their cation.
surface and hence, they are stored in kerosene or
Alkaline earth metals are also soluble in liquid
paraffin oil.
ammonia and give bright blue solutions. When these
1
2 Li + O2 ¾® Li 2O (Oxide) metal-ammonia solutions are evaporated,
2 hexammoniates [ M(NH3 )6] are formed. The
2Na + O2 ¾® Na 2O2 (Peroxide) hexammoniates slowly decompose to give amides.
M + O2 ¾® MO2 (Superoxide) ( M = K, Rb, Cs) [ M (NH3 )6 ] ¾® [ M (NH3 )2 ] + 4 NH3 + H 2
Superoxide O-2 ion is stable only in the presence of Concentrated solutions of the metals in ammonia
large cations such as K, Rb, Cs. impart bronze colour due to the formation of metal
clusters and it is diamagnetic.
Alkaline earth metals get slowly oxidised on exposure to
air forming oxides. These oxides are basic in nature. Basic (vi) Reaction with nitrogen and carbon
nature of oxides increases on moving down the group. 6 H 2O
3 Ca + N 2 ¾® Ca3 N 2 ¾¾® 3Ca(OH)2 + 2NH3 ­
NOTE Oxide of Be, i.e. BeO is amphoteric in nature due to smaller
size of Be 2 + ion. (vii) Reactivity and electrode potential Reducing character
(ii) Basic strength of hydroxides Hydroxides of alkali or reactivity of alkali metals increases from Li to Cs, i.e.
metals are strongly basic and their basic strength Li < Na < K < Rb < Cs.
increases down the group. And hydroxides of alkaline Reducing character or reactivity of alkaline earth
earth metals are less basic than alkali metals of the metals increases from Be to Ba, i.e.
corresponding period. Be < Mg < Ca < Sr < Ba
CsOH > RbOH > KOH > NaOH > LiOH Mg(ClO4 )2 is used as drying agent under the name
(iii) Reaction with hydrogen Alkali metals react with anhydrone, it is a strong oxidising agent. So, it is not
hydrogen and form ionic hydrides, M + H - . used with organic material.
2 M + H 2 ¾® 2 MH (where, M = Li, Na, K etc)
Alkali metal hydrides attacked by water to give Anomalous Properties of Lithium
hydrogen. Lithium due to its
MH + H 2O ¾® MOH + H 2 ● exceptionally small size (like Mg) and

Except Be, all alkaline earth metals react with ● high polarising power (i.e. charge/radius ratio) (like Mg)
hydrogen to form ionic hydrides of MH 2 type. The
exhibits some properties which are different from those
hydride of Mg bears some covalent character in it due
of the other members of group 1 but similar to that of
to its smaller size.
magnesium (present diagonally in the following group,
(iv) Reaction with halogens Alkali metals combine readily i.e. group 2 ). The property of showing similarity in
with halogens to form halides. properties with the element present diagonally opposite
é where, M = Li, Na, K, Rb, Csù in the periodic table is called diagonal relationship.
2 M + X 2 ¾® 2 MX ê ú These properties are :
ë X = F, Cl, Br or I û
(a) Alkali metals do not react with nitrogen except Li.
Alkaline earth metals combine with halogens at
elevated temperature forming halides, MX 2. Beryllium Heat
6Li + N 2 ¾¾® 2Li3 N
halide is covalent while others are ionic. The solubility Lithium nitride
of halides (except fluoride) decreases on moving top to (Li metal is used as scavenger in metallurgy to
bottom in the group (Lower solubility of LiF in water is remove O2 and N 2 gases.)
due to its higher lattice energy).
3Mg + N 2 ¾¾® Mg3 N 2
(v) Solubility in liquid ammonia All alkali metals are Magnesium nitride
soluble in liquid ammonia. Dilute alkali (b) Alkali metal carbonates, nitrates and hydroxides do
metals-ammonia solution is of blue colour. These not decompose on heating into their oxides except
solutions are strong reducing agents due to the lithium.
presence of free electrons. Heat
Na 2CO3 ¾¾® No reaction
M + ( x + y ) NH3 ¾® [ M (NH3 )x ]+ +[e(NH3 ) y ]-
Ammoniated Ammoniated Heat
cation electron Li 2CO3 ¾¾® Li 2O + CO2

@iitjeehelps
540 SELF STUDY GUIDE BITSAT

Heat
MgCO3 ¾¾® MgO + CO2 Properties
Heat O2 H2O CO2
4LiNO3 ¾¾® 2Li 2O + 4NO2 + O2 Na2O NaOH 2Na2CO3 + 2H2
Heat
2Mg (NO3 )2 ¾¾® 2MgO + 4NO2 + O2 H2O
NaOH
2NaNO3 ¾¾® 2NaNO2 + O2
HCl
(c) MgCl 2 and LiCl are deliquescent and crystallise as NaCl
their hydrates, LiCl × 2H 2O and MgCl 2 × 2H 2O. S Na2S
Na SiO2
Anomalous behaviour of Beryllium Si + 2Na2O
Al2O3
Beryllium due to its exceptionally small size (like Al) and Al + Na2O
high polarising power (like Al) shows similarities with
ROH
aluminium in its properties and this relationship is called NaOR
diagonal relationship. These properties are : NH3
NaNH2
● BeO like Al O , is amphoteric and covalent while oxides of
2 3
other alkaline earth metals are ionic and basic in nature.
Uses
● Both BeCl
2 and AlCl3 are soluble in organic solvents
because of covalent nature and both have a bridged (a) For extraction C, Be, Mg and Si.
polymeric structure. (b) As lab reagent.
● Beryllium hydroxide dissolves in excess of alkali to give a (c) In high temperature thermometers.
beryllate ion, [Be(OH)4 ]2- just as aluminium hydroxide
gives aluminate ion,[Al(OH)4 ]- . Compounds of Sodium
● Beryllium and aluminium ions have strong tendency to
Sodium Carbonate
form complexes, BeF 24- , AlF 36 - .
(Washing Soda, Na 2CO 3 ×10H2O)
● Carbides of Be are covalent and react with water to
Sodium carbonate is generally prepared by Solvay process.
produce methane gas whereas carbides of other
members are ionic and produce acetylene with water. 2NH3 + H 2O + CO2 ¾¾® (NH 4 )2 CO3
(NH 4 )2 CO3 + H 2O + CO2 ¾¾® 2NH 4HCO3
NH 4HCO3 + NaCl ¾¾® NH 4Cl + NaHCO3
Sodium (Na) Sodium hydrogen carbonate crystals separate out. These
Compounds found in nature are heated to obtain sodium carbonate.
NaCl, NaNO3, , Na 2SO4 ×10H 2O (Glauber’s salt) 150 ° C
2NaHCO3 ¾¾® Na 2CO3 + CO2 + H 2O
Na 2B 4O7 × 10H 2O (Borax)
In this process, NH3 is recovered when the solution
Extraction containing NH 4Cl is treated with Ca(OH)2 .
1. Castner’s Process
Uses of Na2 CO3
2NaOH r 2Na+ + 2OH –
(i) In laundries and in softening of water as washing soda.
At cathode 2Na+ + 2e – ¾® 2Na
(ii) In the manufacturing of glass, sodium silicate, paper,
At anode 4OH - ¾® 2H 2O + O2 + 4e – borax, soap powders, caustic soda, etc.
Oxygen and H 2O are produced during electrolysis. (iii) In textile and petroleum refining.
2. Down’s Process (iv) For the preparation of various carbonates of metals.
Cathode Fe Sodium Hydroxide (Caustic Soda, NaOH)
Anode Graphite Sodium hydroxide is prepared commercially by the
NaCl ¾® Na + + Cl – electrolysis of sodium chloride in Castner-Kellner cell
At cathode Na + + e - ¾® Na (or mercury cathode cell).
Hg
At anode
300 °C
2Cl – ¾¾® Cl 2 + 2e – At cathode Na + + e - ¾¾® Na -Hg (Amalgam)
Fusion temperature is reduced to 600°C by adding CaCl 2 , 1
At anode Cl - ¾¾® Cl 2 + e - (By-product)
KE. 2

@iitjeehelps
s-BLOCK ELEMENTS 541
The amalgam is treated with water to give sodium Compounds of Magnesium
hydroxide and hydrogen gas.
2Na-Hg + 2H 2O ¾¾® 2NaOH + 2Hg + H 2 Magnesium Chloride (MgCl 2 × 6H 2O)
● Sodium hydroxide is white, translucent solid and Preparation
deliquescent. It is called caustic soda because it breaks MgCO3 + 2HCl ¾® MgCl 2 + CO2 + H 2O
down proteins of skin to a pasty mass.
MgO + 2HCl ¾® MgCl 2 (hydrated)
● On exposure to atmosphere, it absorbs moisture and
D
CO2 . 2NaOH + CO2 ¾® Na 2CO3 + H 2O MgO + C + Cl 2 ¾® MgCl 2 + CO2
● It reacts with metals like Zn, Al, Sn and Pb and evolve H 2 Burning
gas. Mg + Cl 2 ¾¾® MgCl 2 (anhydrous)
e.g. Zn + NaOH ¾® Na 2ZnO2 + H 2
Properties
● It also reacts with metallic salts to form hydroxides. It is
used in soap manufacturing, purification of bauxite, It is a colourless, crystalline, deliquescent and highly
manufacturing of rayon, etc. soluble in water.
D
Sodium Chloride 2(MgCl 2 × 6H 2O) ¾® MgO × MgCl 2 + 2HCl + 11 H 2O
(Common salt or rock salt, NaCl) Uses
The three major sources of NaCl are It is used in the preparation of metallic magnesium and in
(i) Sea water (3% NaCl) making Sorel cement.
(ii) Water of inland lake Magnesium Sulphate (MgSO4 × 7H 2O)
(iii) Salt mines Preparation
● It is a white crystalline solid, soluble in water.
MgCO3 + H 2SO4 ¾® MgSO4 + H 2O + CO2
● Pure NaCl is not hygroscopic. It shows hygroscopic Magnesite
nature due to the presence of chlorides of Ca and Mg MgCO3 × CaCO3 + 2H 2SO4 ¾®
as an impurity. Dolomite
● It is used as the starting material for the preparation
MgSO4 + CaSO4 ­ + 2H 2O + 2CO2
of NaOH, Na 2CO3 , NaHCO3 , etc. Commercial method
● It is used as a preservative for meat, fish, etc. It is
MgSO4 × H 2O + 6H 2O ¾® MgSO4 × 7H 2O
also used as a salting out of soap and in making (Epsom salt)
Properties
freezing mixtures.
It is a colourless crystalline solid, soluble in water.
Sodium Hydrogen Carbonate °C
MgSO4 × 7H 2O ¾150
¾¾ ® MgSO4 × H 2O
(Baking Soda, NaHCO 3 )
°C
It is prepared as an intermediate product in the Solvay ¾200
¾¾ ® MgSO4 + H 2O
ammonia soda process. D
2MgSO4 ¾
¾® 2MgO + 2SO2 + O2
NH3 + H 2O + CO2 + NaCl ¾® NaHCO3 ¯ + NH 4Cl
2MgSO4 + C ¾® 2MgO + 2SO2 + CO2
● It is a white crystalline solid, sparingly soluble in water.
● Its aqueous solution is alkaline in nature due to Uses
hydrolysis. It is used in
NaHCO3 + H 2O r NaOH + H 2CO3 1. Medicines (as purgative).
Heat 2. Dyeing and tanning processes.
2NaHCO3 ¾¾® Na 2CO3 + H 2O + CO2
3. Ceramic, cement, match industries.
● It is used as a medicine to neutralise acidity in stomach.
4. Treatment of textile fibres to make them non-
● It is used as a constituent of baking powder and in fire
inflammable.
extinguisher.

@iitjeehelps
542 SELF STUDY GUIDE BITSAT

Compounds of Calcium Uses


It is used
Quicklime (CaO)
1. In the preparation of plaster of Paris.
Preparation 2. In the manufacturing of cement, H 2SO4 , (NH 4 )2 SO4 , etc.
D
CaCO3 ¾
¾® CaO + CO2 3. For glazing paper.
800-1000 °C

Properties Plaster of Paris (2CaSO4 × H2O)


It is white amorphous powder with melting point 2600°C. Preparation
Action of water
D
CaO + H 2O ¾® Ca(OH)2 2(CaSO4 × 2H 2O) ¾® 2CaSO4 × H 2O + 3H 2O
Gypsum Plaster of Paris
Basic nature
CaO + 2HCl ¾® CaCl 2 + H 2O Properties
CaO + CO2 ¾® CaCO3 It is a white powder, which sets to a hard mass when its
CaO + SiO2 ¾® CaSiO3 paste (with water) is allowed to stand.
2000 °C Plaster of Paris ¾® Gypsum ¾® Gypsum
CaO + 3C ¾¾® CaC 2 + CO2 (Orthorhombic) (Monoclinic)

CaO + 2NH 4Cl ¾® CaCl 2 + 2NH3 + H 2O Uses


Uses It is used
It is very useful as drying agent, in bleaching powder 1. in making chalks, casts in dentistry, etc.
production (as used for the preparation of slaked lime and 2. in surgery for setting broken/dislocated bones.
as a constituent of mortar).
3. in making casts for statues, toys etc.
Calcium Carbonate (CaCO3 ) Calcium Hydroxide or Slaked Lime [Ca(OH)2 ]
Preparation Preparation
(a) Ca(OH)2 + CO2 ¾® CaCO3 + H 2O It is prepared by adding water to quicklime as
(b) CaCl 2 + Na 2CO3 ¾® CaCO3 + 2NaCl 2CaO + 2H 2O ¾® Ca(OH)2 + 2H 2 ­
Properties Properties
CaCO3 is a white powder, insoluble in water but dissolves in 1. It is a white powder, sparingly soluble in water and the
the presence of CO2 . aqueous solution is called lime water while the
Excess
CO 2 aqueous suspension is called milk of lime. CO2 has the
Ca 2+ (OH- )2 + CO2 ¾® CaCO3 + H 2O ¾¾® Ca 2 + (HCO3- )2 ability to turn lime water milky as
(Insoluble ) (Soluble)
White ppt Ca(OH)2 + CO2 ¾® CaCO3 + H 2O
(Milky ppt)
Uses
This milkiness disappears with excess of CO2 as
It is used soluble calcium hydrogen carbonate is formed.
1. in manufacturing paints, quicklime, etc. CaCO3 + H 2O + CO2 ¾® Ca(HCO3 )2
2. in toothpastes and cosmetics. Soluble
3. as a building material. 2. Being basic in nature, it reacts with acids and acidic
gases forming their salts.
Calcium Sulphate (Gypsum) (CaSO4 × 2H2O) 3. On reaction with chlorine, it forms hypochlorite,
Preparation a constituent of bleaching powder, as
CaCl 2 + H 2SO4 ¾® CaSO4 + 2HCl 2Ca(OH)2 + 2Cl 2 ¾® CaCl 2 + Ca(OCl)2 + 2H 2O
CaCl 2 + Na 2SO4 ¾® CaSO4 + 2NaCl Uses
Properties Slaked lime is used in mortar production, lime water
● It is a white crystalline solid, sparingly soluble in water production, disinfectant, bleaching powder production, etc.
but dissolves in dilute acids. Portland Cement
Strong
heating It was discovered by an English Mason, Joseph Aspdin
● 2CaSO4 ¾¾® 2CaO + 2SO2 + O2 (1824). When the mixture of CaCO3 , clay and water is heated
● CaSO4 + 4C ¾® CaS + 4CO strongly and allowed to stand, it hardens to a stone like mass.

@iitjeehelps
s-BLOCK ELEMENTS 543
It resembles with Portland rock (a famous building stone of Setting of Cement Paste of water and cement gets hard on
England) and so, it is named as Portland cement. It is a keeping it for some time. This is called setting of cement.
mixture of aluminates and silicates of Ca with small amount Mechanism of setting of cement is as follows:
of gypsum. Hydration
3CaO × Al 2O3 + 6H 2O ¾¾¾® 3CaO × Al 2O3 × 6H 2O
Chemical Composition Tricalcium aluminate (Colloidal gel)

CaO = 50-60%, SiO2 = 20-25%, Hydrolysis


3CaO × SiO2 + H 2O ¾¾¾¾® Ca(OH)2 + 2CaO × SiO2
Al 2O3 = 5 -10%, MgO = 2 - 3%, (ppt) Dicalcium silicate
Hydration
Fe 2O3 = 1-2%, SO3 = 1-2% 2CaO × SiO2 + xH 2O ¾¾¾® 2CaO × SiO2 × xH 2O
(Colloidal gel)
Formation of Constituents of Cement
Partial
5CaO + 5SiO2 ¾® 2CaO × SiO2 + 3CaO × SiO2 3CaO × Al 2O3 + 6H 2O ¾¾¾¾® 3Ca(OH)2 + 2Al (OH)3
hydrolysis (ppt)
Dicalcium silicate Tricalcium silicate (ppt)

3CaO + Al 2O3 ¾® 3CaO × Al 2O3 4CaO × Al 2O3 × Fe 2O3 + 6H 2 O ¾®


Tricalcium aluminate Tetracalcium aluminoferrite
4CaO + Al 2O3 + Fe 2O3 ¾® 4CaO × Al 2O3 × Fe 2O3 3CaO × Al 2O3 × 6H 2O + CaO × Fe 2O3
Dialuminium tetracalcium (Colloidal gel)
iron (III) oxide or celite

Practice Exercise
1. Which of the following ions forms a hydroxide which is 8. Which of the following pairs cannot exist together in
highly soluble in water? solution?
a. Ni2+ b. K + c. Zn2+ d. Al3+ a. Na 2CO3 and NaHCO3 b. NaHCO3 and NaOH
2. Solubility of alkaline earth metal sulphates decreases c. Na 2CO3 and NaOH d. NaOH and NaCl
down the group 2 because 9. Sodium peroxide, a yellow solid, when exposed to air
a. they become more ionic becomes white due to the formation of
b. lattice energy of sulphates does not vary significantly a. H2O2 b. Na 2O
c. hydration energy decreases rapidly from Be2+ to Ba 2+ c. Na 2O and O3 d. NaOH and Na 2CO3
d. lattice energy plays more predominant role than
hydration energy 10. The products obtained on heating LiNO 3 will be
3. Which of the following compounds has the lowest a. LiNO2 + O2 b. Li2O + NO2 + O2
anion to cation size ratio? c. Li3N + O2 d. Li2O + OH + O2
a. LiF b. NaF c. CsI d. CsF 11. Identify the correct formula for halides of alkaline earth
4. Which is the most basic among the following? metals.
a. Na 2O b. BaO c. As2O3 d. Al2O3 a. BaCl2 × 6H2O b. CaCl2 × 6H2O
5. The stability of the following alkali metal chlorides c. BaCl2 × 4H2O d. SrCl2 × 4H2O
follows the order
12. LiF is almost insoluble in water whereas LiCl is soluble
a. LiCl > KCl > NaCl > CsCl b. CsCl > KCl > NaCl > LiCl
c. NaCl > KCl > LiCl > CsCl d. KCl > CsCl > NaCl > LiCl not only in water but also in acetone. This is because of
a. high hydration enthalpy of Li+
6. When CO is passed over solid NaOH heated to b. low hydration enthalpy of Li+
200°C, it forms c. more ionic character in LiCl
a. Na 2CO3 b. H2CO3 c. HCOONa d. All of these
d. None of the above
7. Which of the following does not illustrate the
anomalous properties of lithium? 13. Consider the following statements.
a. Li is much softer than the other group 1 metals I. BeO is insoluble but BeSO4 is soluble in water.
b. The melting point and boiling point of Li are II. BaO is insoluble but BaSO4 is soluble in water.
comparatively high III. LiI is more soluble than KI in ethanol.
c. Li forms a nitride Li3N unlike group 1 metals The true statements are
d. The ion of Li and its compounds are more heavily a. I and II b. I and III c. II and III d. I, II and III
hydrated than those of rest of the group 1 elements
14. Which of the following statements is/are incorrect
regarding the s-block elements?

@iitjeehelps
544 SELF STUDY GUIDE BITSAT

I. Francium is highly radioactive element. Na 2CO3 ¾® Na


D
II. Oxides and hydroxides of alkali metals and CO2 HCl
alkaline earth metals are not alkaline in nature. d. Na + H2O ¾® NaOH ¾¾® Na 2CO3 ¾¾®
III. Sodium and potassium are the only two s-block Electrolysis
elements which are found in large proportion in NaCl ¾¾¾¾® Na + Cl
(Molten)
biological fluids.
IV. Biological function of s-block elements is due to 22. The ore of potassium is
maintenance of ion balance and nerve impulse a. bauxite b. dolomite c. carnallite d. cryolite
condition. 23. Which of the following is duralumin alloy?
a. Only I b. I and III c. II and III d. Only III a. Mg + Cu + Al + Mn b. Mg + Cu
15. Match the Column I with Column II and choose the c. Cu + Al d. Mg + Al
correct option from the codes given below. 24. When sodium is reacted with limited supply of oxygen,
Column I Column II the main compound formed is
a. Na 2O4 b. Na 2O c. Na 3O d. Na 2O2
A. Sodium 1. Present in biological fluid
B. Beryllium 2. Radioactive element 25. Metallic magnesium is prepared by
C. Francium 3. Lower abundance
a. electrolysis of molten MgCl2
b. displacement of Mg by iron from MgSO4 solution
D. Calcium 4. Alkali metal c. electrolysis of aqueous solution of Mg(NO3 )2
5. Alkaline earth metal d. reduction of MgO by coke
Codes 26. The element which is not present in asbestos, is
A B C D A B C D a. Si b. Ca c. Ba d. Mg
(a) 1 4 2 5 (b) 1 2 3 4 +
(c) 3 1 2 5 (d) 4 5 2 1
27. The molarity of Na , when the average concentration
of Na + in human blood serum is about 3.4 g L -1, is
16. Which of the following is/are correct pair(s) regarding a. 8.4 b. 2.3 c. 0.68 d. 0.15
diagonally related elements?
a. Beryllium and aluminium b. Lithium and magnesium 28. Caesium is the most reactive metal in the alkali metal
c. Sodium and aluminium d. Both a. and b. series because
a. it exerts considerable attractive force on valence
17. The decomposition temperature is maximum for electrons
a. MgCO3 b. CaCO3 b. it is a heavier metal
c. BaCO3 d. SrCO3 c. its incomplete shell is nearest to the nucleus
18. Which of the following metal salts has highest d. its valence electron has the highest principal quantum
conductivity in aqueous medium? number than the valence electron of any of the others
a. Li+ b. Cs+ c. Na + d. K + 29. When oxygen reacts with potassium, which of the
following is preferentially formed?
19. In case of alkali metals, the covalent character
a. K 2O4 b. KO2
decreases in the order
c. K 2O3 d. K 2O
a. M I > M Br > M Cl > M F
b. M Cl > M I > M Br > M F 30. Which of the following is different from other three
c. M F > M Cl > M Br > M I oxides?
d. M F > M Cl > M I > M Br a. Cr2O3 b. MgO
20. Correct order of thermal stability of K 2CO3 , CaCO3 , c. SnO d. ZnO
MgCO3 and BeCO3 is 31. Nitrogen dioxide cannot be prepared by heating
a. BeCO3 < MgCO3 < CaCO3 < K 2CO3 a. KNO3 b. Pb(NO3 )2
b. K 2CO3 < MgCO3 < CaCO3 < BeCO3 c. Cu(NO3 )2 d. AgNO3
c. MgCO3 < BeCO3 < CaCO3 < K 2CO3 32. The correct decreasing order of ionisation enthalpy of
d. BeCO3 < MgCO3 < K 2CO3 < CaCO3
alkali metals is
21. Which of the following sequence of chemical reaction a. Na > Li > K > Rb b. Rb < Na < K < Li
is correct? c. Li > Na > K > Rb d. K < Li < Na < Rb
2 HCl(aq ) CO
a. Na + O2 ¾® Na 2O ¾¾¾® NaCl ¾¾® 33. The gas evolved on heating Na 2CO3 is
D a. CO2 b. water vapour
Na 2CO3 ¾® Na
c. CO d. No gas is evolved
2 O H O CO
b. Na ¾® Na 2O ¾¾®
2
NaOH ¾¾®
2
34. A mixture contains two moles of Na 2CO3 and 1 mole
D
Na 2CO3 ¾® Na of Li2CO3 . What will be the volume of CO2 formed on
HCl CO2 heating this mixture and the data is converted to STP?
c. Na + H2O ¾® NaOH ¾¾® NaCl ¾¾® a. 22.4 L b. 44.8 L c. 50.2 L d. 11.2 L

@iitjeehelps
s-BLOCK ELEMENTS 545
35. An aqueous solution of sodium carbonate is alkaline C and D . The solution of C becomes milky on
because sodium carbonate is a salt of bubbling carbon dioxide. The element A is
a. weak acid and weak base a. Li b. Mg c. Ca d. Be
b. strong acid and weak base
47. The metal ion which plays an important role in muscle
c. weak acid and strong base
d. strong acid and strong base
contraction is
a. Be2+ b. Mg2+ c. Ca 2+ d. Ba2+
36. Sodium is heated in air at 350°C to form A.
Compound A when reacts with carbon dioxide forms 48. Mix calcium sulphate with conc. HCl and forms a
sodium carbonate and Y . Here, Y is paste. Bring a pinch of this paste near to the flame,
a. hydrogen peroxide b. hydrogen what colour will you obtain?
c. ozone d. oxygen a. Brick red b. Apple green
c. Golden yellow d. Crimson red
37. On reacting with NaOH, which gives inflammable gas?
a. Zn b. S c. I 2 d. NH4Cl 49. In curing cement plasters, water is sprinkled from time to
time. This helps in
38. When washing soda is heated, a. converting sand into silicic acid
a. CO2 is released b. CO + CO2 is released b. keeping it cool
c. CO is released d. water vapour is released c. developing interlocking needle like crystals of
39. In the synthesis of sodium carbonate, the recovery of hydrated silicates
ammonia is done by treating NH 4 Cl with Ca(OH) 2. d. hydrating sand and gravel mixed with cement
The by-product obtained in this process is 50. What will be the oxidation states of nitrogen and alkali
a. CaCl2 b. NaCl c. NaOH d. NaHCO3 metal respectively when the nitrogen and alkali metal
react with each other?
40. Solvay process is not used to prepare KHCO3 . Which
a. - 3 and + 1 b. -1 and + 1
of the following reactions will produce KHCO3 ?
c. -1 and - 1 d. -3 and - 1
a. KHSO4 + KCl ¾®
b. Magnesia process 51. What will you get when you heat hydrated CaSO4 to
c. KNO3 + H2O ¾®
d. Calcium carbonate and silver chloride 125°C instead of 200°C?
a. CaO + SO3 b. CaSO4
41. The chemical name of soda ash is 3
a. sodium bicarbonate b. sodium hydroxide c. CaSO4 × H2O d. (CaSO4 )2 × H2O
4
c. sodium carbonate d. None of these
52. The ease of adsorption of the hydrated alkali metal
42. In the electrolytic process for the manufacturing of
ions on an ion-exchange resins follows the order
NaOH from NaCl solution, the ion discharged at the
anode is [CBSE-AIPMT 2012]
a. OH- b. O- a. Li < K < Na < Rb
+ + + +
b. Rb < K + < Na + < Li+
+

c. Cl- d. All of these c. K + < Na + < Rb+ < Li+ d. Na + < Li+ < K + < Rb+
43. Anhydrous magnesium chloride is prepared by
a. dissolving Mg in dil. HCl
53. Which of the following represent the correct
b. dissolving MgO in dil. HCl composition of Sorel cement?
c. passing Cl2 over red hot mixture of MgO + C a. Portland cement + MgO b. CaSiO3 × MgCO3
d. All of the above c. MgCl2 × CaSiO3 d. MgCl2 × 5MgO ×x H2O

44. A substance which gives brick red flame and breaks 54. Formula of plaster of Paris is
down on heating to give oxygen and a brown gas, is a. CaSO2 × 2H2O b. 2(CaSO4 × 1/ 2 H2O)
a. magnesium nitrate c. 2CaO × H2O d. CaCO3
b. calcium nitrate 55. Which of the following is not contained by Portland
c. barium nitrate cement?
d. strontium nitrate a. Ca 3PO4 b. CaSiO3 c. CaSiO4 d. Ca 3Al2O6
45. A chemical A is used for the preparation of washing 56. The name given to the mixture of MgCl2 and MgO is
soda to recover ammonia. When CO2 is bubbled a. double salt b. Portland cement
through an aqueous solution of A, the solution turns c. Sorel cement d. None of these
milky. It is used in white washing due to disinfectant 57. Which of the following statements is correct?
nature. What is the chemical formula of A? a. Plaster of Paris can be obtained by hydration of
a. Ca(HCO3 )2 b. CaO c. Ca(OH)2 d. CaCO3 gypsum
46. Element A burns in nitrogen to give an ionic b. Gypsum is obtained by heating plaster of Paris
compound B. Compound B reacts with water to give c. Plaster of Paris is obtained from gypsum by oxidation

@iitjeehelps
546 SELF STUDY GUIDE BITSAT

d. Plaster of Paris contains higher percentage of c. concentrated solution of Ca(OH)2


calcium than gypsum d. dilute solution of Cu(OH)2
58. Except lime, a major constituent of Portland cement is 60. Which of the following ions is/are responsible for
a. silica b. alumina c. iron oxide d. magnesium biological functions such as maintenance of ion
59. Bleaching powder is obtained by the reaction of balance and nerve impulse conduction?
chlorine with a. Na+ b. K +
a. dry CaO c. Mg2+ d. All of these
b. moist slaked lime

BITSAT Archives
BaCl2 9. Which of the following statements are correct for alkali
1. Salt A + S ¾® B ¾¾® White precipitate metal compounds?
A is paramagnetic in nature and contains about I. Superoxides are paramagnetic in nature.
55% K. Thus, A is [2013] II. The basic strength of hydroxides increases down
a. K 2O b. K 2O2 the group.
c. KO2 d. K 2SO4 III. The conductivity of chlorides in their aqueous
2. The decomposition temperature is maximum for [2012] solutions decreases down the group.
a. MgCO 3 b. CaCO3 IV. The basic nature of carbonates in aqueous
c. BaCO3 d. SrCO3 solution is due to cationic hydrolysis. [2008]
a. I, II and III b. I and II
3. When some amount of zinc is treated separately with
c. II, III and IV d. III and IV
excess of sulphuric acid and excess of sodium
hydroxide solution, the ratio of volumes of hydrogen 10. Setting of plaster of Paris involves [2007]
evolved is [2012] a. oxidation with atmospheric oxygen
a. 1 : 1 b. 1 : 2 c. 2 : 1 d. 2 : 3 b. combination with atmospheric CO2
c. dehydration
4. The product(s) of the reaction, [2011] d. hydration to yield another hydrate
Na 2CO3 + CO2 + H2O ¾® is/are 11. The ionic conductance is least for [2007]
a. 2NaOH + CO2 b. Na 2CO3 + H2CO3 a. Cs+ b. Rb+ c. K + d. Na +
c. 2NaHCO3 d. None of these
12. Which one of the following substances is used in the
5. Sodium carbonate reacts with SO2 in aqueous laboratory for a fast drying of neutral gases? [2007]
medium to give [2011] a. Phosphorus pentoxide
a. NaHSO3 b. Na 2S2O3 b. Active charcoal
c. NaHSO4 d. Na 2SO4 c. Anhydrous calcium chloride
d. Na 2PO4
6. Anhydrone, a drying agent is [2010]
a. Mg(ClO4 )2 b. Sr(ClO4 )2 13. Which out of the following compounds is called
c. Ca(ClO4 )2 d. Ba(ClO4 )2 photographers fixer? [2007]
a. Na 2SO3 b. Na 2S2O3 × 5H2O
7. Mg + NO2 on burning give [2010] c. Na 2SO2 d. Na 2S
a. N2 b. NO
c. N2O d. N2O5 14. Which of the following is called Berthelot’s salt? [2006]
a. (NaPO3 )6 b. NaOCl c. KClO3 d. KHF2
8. A metal X on heating in nitrogen gas gives Y. Y on
treatment with water gives a colourless gas which 15. Milk of magnesia is used as [2005]
when passed through CuSO4 solution gives a blue a. antichlor b. antacid
colour. Y is [2009] c. antiseptic d. food preservative
a. Mg(NO3 )2 b. Mg3N2 16. Solvay process is used for the manufacturing of [2005]
c. NH3 d. MgO a. NaOH b. Na 2CO3
c. NH3 d. NaCl

@iitjeehelps
Answer with Solutions
Practice Exercise 15. (d ) A ® 4, B ® 5, C ® 2, D ® 1

1. (b) Alkali metal hydroxides, i.e. KOH is highly soluble in 16. (d)
water. 17. (c) Greater the electropositive character, more will be the
stability and high decomposition temperature.
2. (c) Due to very large size of sulphate ions, the magnitude
of lattice energy of alkaline earth metal sulphates remains Among the given options, Ba has maximum
almost constant. Thus, their solubility is only governed by electropositive character and hence, highest
hydration energy which decreases from Be2+ to Ba 2+ . decomposition temperature.
3. (d) For anion to cation ratio to be low, the anion must be 18. (b) Cs+ has maximum ionic mobility due to low value of
small and cation must be large, i. e. Cs+ F -. hydration enthalpy. It is a large size ion.
4. (a) Alkali metal oxides (Na 2O) are most basic followed by 19. (a) Fajan’s rule states that more is the polarising power of
anion, greater is the covalent character.
alkaline earth metals. Al2O3 is however amphoteric.
20. (a) As the basicity of metal hydroxide increases down the
5. (d) Stability of a compound depends upon its enthalpy of group, thermal stability of their carbonates increases in
formation DHf . The more negative value of DHf shows
the same order. As we know, group 1 compounds are
more stability of a compound.
more stable than group 2 compounds. Hence, correct
Thus, KCl is more stable and LiCl is least stable. order is
DHf for LiCl = - 408.8 kJ mol-1 BeCO3 < MgCO3 < CaCO3 < K 2CO3
DHf for NaCl = - 412 . 5 kJ mol-1 21. (d ) 2Na + 2H2O ¾® 2NaOH + H2 ­
DHf for CsCl = - 433 kJ mol-1 2NaOH + CO2 ¾® Na 2CO3 + H2O
DHf for KCl = - 436 kJ mol-1 Na 2CO3 + 2HCl ¾® 2NaCl + H2O + CO2
200°C
6. (c) NaOH + CO ¾¾® HCOONa Electrolysis
NaCl ¾¾¾¾® Na + + Cl-
(Solid) Sodium formate
½ + e - ½ -e -
7. (a) Li is not softer than the other group 1 metals. Actually, ¯ ¯
Li is harder than other alkali metals. Na Cl
8. (b) NaHCO3 and NaOH both reacts to form Na 2CO3. In the case of options (a),(b) and (c), the last step on
heating
9. (d) In the presence of moisture and CO2, Na 2O2 is D
converted into NaOH and Na 2CO3 . Na 2CO3 ¾® Thermally stable
D 1 22. (c) KCl × MgCl2 × 6H2O (carnallite) is an ore of potassium.
10. (b) 2LiNO3 ¾® Li2O + 2NO2 + O2
2 23. (a) Cu - 4% , Mg - 0.05%,
11. (b) The correct formula of halides of alkaline earth metals Mn - 0.5% , Al -95%
is
24. (b) 4Na + O2 ¾® 2Na 2O
CaCl2 × 6H2O , BaCl2 × 2H2O, SrCl2 × 6H2O.
25. (a) Electrolysis of molten MgCl2
12. (a) LiF is almost insoluble in water due to its high lattice
energy but LiCl is soluble in water due to high hydration At anode MgCl2 r Mg2+ + 2Cl-
energy of Li+ ions. LiCl is also soluble in acetone due to its At cathode Mg2+ + 2 e - ¾® Mg
predominantly covalent nature (because covalent
character increases with increase in the size of anion). 26. (c) Asbestos is CaMg3(SiO3 )4.
13. (b) If lattice energy > hydration energy, the compound 27. (d) Molarity (M )
(Beo) is insoluble in water. Weight of solute / litre of solution
=
If lattice energy < hydration energy, the compound Molecular weight of solute
(BeSO4) is soluble in water. 3 .4 g L-1
LiI is more covalent because Li+ is smallest and polarised = = 0.15
21.2
I - (anion) to maximum extent, so it is more soluble than
KI in ethanol. 28. (d) When valence electron is in the highest quantum
shell, the ionisation energy is lowest and hence, reactivity
14. (c) Statements II and III are incorrect because oxides and is highest.
hydroxides of alkaline earth metals and alkali metals are
alkaline in nature. Apart from Na and K, Ca and Mg are 29. (b) K + O2 ¾® KO2 (superoxide)
also found in biological fluid. 30. (b) MgO is basic while other three are amphoteric oxides.

@iitjeehelps
548 SELF STUDY GUIDE BITSAT

31. (a) Only nitrates of heavy metals and lithium decompose 45. (c) For the recovery of ammonia, Ca(OH)2 is used during
on heating to produce NO2. the preparation of washing soda.
32. (c) The correct decreasing order of ionisation enthalpy of 46. (c) Carbon dioxide turns only lime water milky. Thus, the
alkali metals are as follows
compound C must be Ca(OH)2 and the element A must
Li > Na > K > Rb be Ca. The reactions are as follows:
33. (d) On heating Na 2CO3, no gas is evolved. 3Ca + N2 ¾® Ca 3N2
A B
34. (a) Decomposition of Li2CO3
D Ca 3N2 + 6H2O ¾® 3Ca(OH)2 + 2NH3
Li2CO3 ¾¾ ® Li2O + CO2 B C D
1 mol 22.4 L
Ca(OH)2 + CO2 ¾® CaCO3 + H2O
Na 2CO3 is thermally stable, i. e. it does not give CO2 gas C (Milky)
on heating.
2+
Hence, volume of CO2 formed on heating the mixture at 47. (c) Ca plays an important role in muscle contraction.
STP = 22.4 L 48. (a) The colour of the paste in the flame is brick red.
35. (c) An aqueous solution of sodium carbonate is alkaline in 49. (c) Water is sprinkled on cement plasters to bind hydrated
nature because it is a salt of weak acid and strong base.
crystals of silicates to one another.
350°C
36. (d) Na + O2 ¾¾® Na 2O2 50. (a) Alkali metal reacts with nitrogen to produce nitride.
A
2M + 3N2 ¾® 2M ×N3
2Na 2O2 + 2CO2 ¾® 2Na 2CO3 + O2
Y where, oxidation state of M = +1
37. (a) On reacting with NaOH solution, Zn gives inflammable N= - 3
gas. 51. (d) On heating hydrated CaSO4 to 125°C, (CaSO4 )2 × H2O
D D
38. (d) Na 2CO3 × 10H2O ¾® Na 2CO3 × H2O ¾® is obtained.
120 ° C
Na 2CO3 + H2O ­ 2(CaSO4 × 2H2O) ¾¾® 2(CaSO4 ) × H2O + 3H2O
39. (a) 2NH4Cl + Ca(OH)2 ¾® CaCl2 + 2NH3 + 2H2O 52. (b) Ease of adsorption of the hydrated alkali metal ions on
40. (b) Magnesia Process or Prech’s Process Carbon an ion exchange resins decreases as the size of alkali
dioxide is passed into a solution of potassium chloride at metal ions increases. Since, the order of size of alkali
290 K in the presence of hydrated magnesium carbonate, metal ions is
potassium hydrogen carbonate get precipitated. Li+ < Na + < K + < Rb+
2KCl + 3MgCO3 + 9H2O + CO2 ¾® Thus, the ease of adsorption follows the order
2(KHCO3 × MgCO3 × 4H2O) + 3MgCl2 Rb+ < K + < Na + < Li+
½ 53. (d) Mixture of MgCl2 and MgO is called Sorel cement.
¯
2KHCO3 + MgCO3 + CO2 + 9H2O Sorel cement = MgCl2 × 5MgO × x H2O
1
Potassium
bicarbonate 54. (b) The formula for plaster of Paris is CaSO4 × H2O.
2
This process is known as magnesia method.
55. (a)
41. (c) Na 2CO3 (anhydrous) is called soda ash, sodium
56. (c) When a saturated solution of MgCl2 is mixed with
bicarbonate is called baking soda and sodium hydroxide
is called caustic soda. MgO, the resulting paste sets to a hard marble like mass
which consists of oxychloride (MgO × MgCl2 × 6H2O). The
42. (c) Cl– ¾® Cl + e - paste is known as Sorel cement.
Cl + Cl ¾® Cl2 57. (d) Plaster of Paris is obtained when gypsum is heated to
43. (c) Anhydrous MgCl2 is obtained by passing chlorine gas 120°C.
over red hot mixture of MgO and C. 58. (a) Silica 20 -25%, CaO (quicklime) 60 -70%
MgO + C + Cl2 ¾® MgCl2 + CO 59. (c) Ca(OH)2 + Cl2 ¾® Ca(OCl)2 + H2O
44. (b) Brick red colour is given by Ca which O2 and brown
60. (d) Na+ , K + , Ca 2+ , Mg2+ all are responsible for
colour are given by NO2 group.
maintenance of ion balance and nerve impulse system in
D
2 Ca(NO3 )2 ¾¾® 2 CaO + O2 + 4NO2 biological system.

@iitjeehelps
s-BLOCK ELEMENTS 549

BITSAT Archives
1. (c) Among the given oxides, only KO2, i. e. potassium 6H O
3Mg + N2 ¾® Mg3N2 ¾¾®
2
2NH3 ­ + 3Mg(OH)2
superoxide is paramagnetic in nature. This is because X Colourless
peroxide ion, O-2 has three electron bonds which makes it
Y

paramagnetic and coloured. CuSO4 + 4NH3 ¾® [Cu(NH3 )4]SO4


Deep blue colour
-
O2- O — O- == O ] -
[ O &&&&&& 9. (b) The conductivity of alkali metal chlorides in their
Oxide ion Peroxide ion Superoxide ion
aqueous solution increases on moving down the group
Hence, A is KO 2. and basic nature of carbonates in aqueous solution is
2. (c) Barium salts are quite stable because of great due to anionic hydrolysis.
electropositive nature of Ba. Hence, Ba compounds 10. (d) On hydration of plaster of Paris, it converts into
possess high decomposition temperature. gypsum.
3. (a) Zn + H2SO4 ¾® ZnSO4 + H 2 ­ 1 3
CaSO4 × H2O + H2O ¾® CaSO4 × 2H2O
Zn + 2NaOH ¾® Na 2ZnO2 + H2 ­ 2 2 Gypsum
Plaster of Paris
Hence, ratio of volumes of hydrogen evolved is 1: 1. 11. (d) Due to small size of Na + , it is heavily hydrated and
4. (c) When sodium carbonate is treated with CO2 and H2O, become large molecule.Ionic conductance increases
it gets converted into sodium bicarbonate. down the group in alkali metals.
Na 2CO3 + CO2 + H2O ¾® 2NaHCO3 Order of ionic conductance is
Sodium bicarbonate or
sodium hydrogen carbonate
Na + < K + < Rb+ < Cs+
12. (c) Anhydrous CaCl2 is used for fast drying of neutral
5. (a) When SO2 is passed in the sodium carbonate solution,
CO2 gas is evolved and sodium carbonate is converted gases.
into NaHSO3 (sodium bisulphite). 13. (b) Na 2S2O3 × 5H2O (hypo) it is called photographer’s fixer
Na 2CO3 + 2SO2 + H2O ¾® 2NaHSO3 + CO2 because it removes the excess AgBr in the form of
soluble silver complex.
6. (a) Magnesium perchlorate, Mg (ClO4 ) 2 is also called
anhydrone. 14. (c) KClO3 is known as Berthelot’s salt.
7. (b) Mg + NO2 ¾® MgO + NO 15. (b) Milk of magnesia is used as an antacid.
8. (b) Only NH3 gas gives deep blue solution with 16. (b) Solvay process is used in the manufacturing of
CuSO4 × NH3 is obtained by the reaction of nitride of metal sodium carbonate. It is also known as ammonia-soda
with water. process.

@iitjeehelps
16
Metallurgy

Earth Crust
It is the source of many elements. Aluminium is the most abundant metal of earth crust and iron
comes second. The percentages of different elements in earth crust are
O-49%, Si-26%, Al-7.5%, Fe-4.2%, Ca-3.2%, Na-2.4%, K-2.3%, Mg-2.3%, H-1%
Metals occur in two forms in nature (i) in native state and (ii) in combined state.

Minerals and Ores


The substance (or compound) in the form of metal which is found in nature, is called a mineral and
the mineral from which extraction of metal is beneficial and cheap, is called an ore.
Thus, all ores are minerals but all minerals are not ores.

Types of Ores
Depending upon the nature of associated group or atom, ores are of following types:
(i) Oxide ores Haematite (Fe 2O3 ), zincite (ZnO) etc.
(ii) Sulphide ores Galena (PbS), cinnabar (HgS), argentite (Ag 2 S), ruby silver (Ag 2S ⋅ Sb 2S3 ) etc.
(iii) Carbonate ores Magnesite (MgCO3 ), siderite (FeCO3 ) etc.
(iv) Sulphate ores Gypsum (CaSO4 ⋅ 2H 2O), Glauber’s salt (Na 2SO4 ⋅ 10H 2O) etc.
(v) Silicate ores Willemite (Zn 2 SiO4 ), feldspar (NaAlSi3O8 ) etc.
Nitrate ores are rare because all nitrates are water soluble and at higher temperature, they
decompose into oxides or metal.
Gangue or Matrix
The impurities associated with the ore, are called gangue or matrix.

Metallurgy
Extraction of a metal from its ores, is known as metallurgy. Metallurgy of a metal includes several
metallurgical operations depending upon the nature of metal, its ore and impurities.

@iitjeehelps
METALLURGY 551

Steps Involved in the Extraction of Metals Sodium silicate remains in the solution and hydrated
alumina is filtered, dried and heated to give back pure
Various steps/processes involved in the extraction of a metal Al 2O3 .
are as follows :
1470 K
Al 2O3 ⋅ x H 2O(s ) → Al 2O3 (s ) + x H 2O( g )
Concentration of Ores
(ii) In the metallurgy of silver and gold
Removal of unwanted materials from the ores, is known as
concentration of ores or ore dressing. 4M (s ) + 8CN − (aq ) + 2H 2O(aq ) + O2 ( g )
Some of the important procedures are described below :
→ 4 [ M (CN)−2 ](aq ) + 4 OH − (aq )
Hydraulic Washing (Levigation) ( M = Ag or Au)
In this process, an upward stream of running water is used
to wash the powdered ore. The lighter gangue particles are 2[ M (CN)2 ] (aq ) + Zn(s ) → [Zn(CN)4 ]2 − (aq ) + 2 M (s )

washed away and the heavier ores are left behind. The
oxide ores of iron (Fe3O4 and Fe 2O3 ) are concentrated by Extraction of Crude Metal from
this method.
Concentrated Ore
Magnetic Separation
It involves in two major steps :
This is based on the differences in magnetic properties of
the ore components. If either the ore or the gangue is 1. Conversion of Concentrated Ore to Oxide
capable of being attracted by a magnetic field, this process
is used. In electrostatic separation, electrically charged The following steps are considered under the conversion of
surfaces are used to separate metallic particles from concentrated ore to oxide :
non-metallic particles of ore.
Calcination
Froth Floatation Method Strong heating of ore in the absence of air, e. g.
This method is used for the concentration of sulphide ores. ∆
In this process, a suspension of the powdered ore is Fe 2O3 ⋅ x H 2O(s) → Fe 2O3 (s ) + x H 2O( g )
made with water. Collectors and froth stabilisers are added ∆
ZnCO3 (s ) → ZnO(s ) + CO2 ( g )
to it.

NOTE CaCO3 ⋅ MgCO3 (s ) → MgO(s ) + CaO(s) + 2CO2 ( g )
(i) Collectors (e.g. pine oil, fatty acids, xanthates etc.) enhance Usually carbonates and hydroxides are converted into
non-wettability of the mineral particles and froth stabilisers oxides by this method. Volatile impurities of S, As and P are
(e.g. cresols, aniline) stabilise the froth.
removed as their volatile oxides.
(ii) Sometimes, it is possible to separate two sulphide ores by
adjusting proportion of oil to water or by using depressants. Roasting
e.g. In case of an ore containing ZnS and PbS, the depressant
Strong heating of ore in the presence of air, e. g.
used is NaCN.
2ZnS + 3O2 → 2ZnO + 2SO2
Leaching
2Cu 2S + 3O2 → 2Cu 2O + 2SO2
It is often used if the ore is soluble in some solvent (i.e. acids,
bases or other chemicals) but not the impurities. The sulphide ores of copper are heated in reverberatory
e.g. furnace.

(i) Leaching of alumina from bauxite (Baeyer’s process) 2. Reduction of Oxide to Metal
473- 523K Reduction of the metal oxide to metal, takes place when
Al 2O3 (s ) + 2NaOH(aq ) + 3H 2O(l ) →
heated with reducing agents such as C (coke) or CO or even
2Na [Al(OH)4 ](aq ) another metal. This process is known as smelting.
Sodium meta aluminate
M x Oy + y C → x M + y CO
473- 523K
SiO2 (s ) + 2NaOH(aq ) → Na 2SiO3 (aq ) +H 2O Some metal oxides get reduced easily while others are very
Sodium silicate difficult to be reduced.
The resulting solution is filtered, cooled and pH is
NOTE Some metals like Fe dissolve the reducing agent used
adjusted by neutral with CO2 causing precipitation of (carbon) in their extraction. This can be removed by heating
aluminium hydroxide. the impure metal with more of the ore. Misch metal is used as
2Na [Al(OH)4 ](aq ) + 2CO2 ( g ) → a reducing agent for extraction of pure vanadium. Hydrogen is
used as a reducing agent in extraction of pure tungsten and
Al 2O3 ⋅ xH 2O (s) ↓ + 2NaHCO3 (aq ) vanadium.

@iitjeehelps
552 SELF STUDY GUIDE BITSAT

Thermodynamics of Metallurgy Refining


To understand the variation of temperature requirement for The metal obtained from the above processes is not
thermal reactions and suitable reducing agent for a given metal 100% pure, hence called crude metal. From the
oxide ( M x O y ), Gibbs energy interpretations are made. impure metal, the metal of high purity is obtained by
Gibbs equation, ∆G = ∆H − T∆S refining. Several techniques are used in refining
depending upon the differences in properties of the
where, ∆H = enthalpy change metal.
∆G = Gibbs free energy
T = temperature
Distillation
∆S = entropy change This process is very useful for low boiling metals like
zinc, cadmium and mercury. The impure metal is
∆G ° = − 2.303 RT log K evaporated to obtain the pure metal as distillate.
and K = equilibrium constant Liquation
If ∆G = − ve, process is spontaneous, In this method, low melting metals like tin, bismuth
∆G = + ve, process is non-spontaneous, and lead can be made to flow on a sloping surface of a
reverberatory furnance and heated above their
∆G = 0, process is at equilibrium.
melting point. In this way, it is separated from higher
A reaction with positive ∆G can still be made to occur by coupling melting impurities.
it with another reaction having large negative ∆G. Such coupling
is easily understood through Ellingham diagram. Electrolytic Refining
Copper is refined by using an electrolytic method.

At anode Cu(s ) → Cu 2 + (aq ) + 2e −


D G°/ kJ/mol of O2

l O3
/3 A 2
2 At cathode Cu 2+ (aq ) + 2e − → Cu(s )
A
–900 Al + O2
4/3 2 M O
g Zone Refining
–1000
g + O2 This method is based on the principle that the
–1100 2M
impurities are more soluble in the melt than in the
–1200 solid state of the metal. This method is very useful for
273K 673K 1073K 1473K 1873K
producing semiconductors and other metals of very
Temperature high purity, e. g. germanium, silicon, boron, gallium
and indium.

Gibbs Energy ( ∆G° ) vs T plots Vapour Phase Refining


In this method, the metal is converted into its volatile
(Ellingham Diagram) compound. It is then decomposed to give pure metal,
From Ellingham diagram, it is evident that metals which have e. g.
more negative ∆ f G ° of their oxides, can reduce those metal (i) Mond process for refining nickel
oxides for which ∆ f G ° is less negative. 330-350 K
● Reduction can also be done by using Al, H 2 etc. By Al, oxides of Ni + 4CO → Ni(CO)4 Volatile
Cr, Fe and Mn are reduced. A mixture of Fe 2O3 and Al is called 450-470 K
thermite mixture. Ni(CO)4 → Ni + 4CO
● Highly electropositive metals like Na, K and Al are reduced by (ii) van-Arkel method for zirconium, titanium,
the electrolysis of their fused salts. vanadium or thorium
e.g. Zr + 2 I 2 → ZrI 4 (Volatile)
NaCl r Na + + Cl − 1800 K
Fused ZrI 4 → Zr + 2 I 2

+
Na + e → Na
Chromatographic method
Cl → Cl + e −

This method is based on the principle that different
↓ components of a mixture are adsorbed differently on
Cl 2
an adsorbent.

@iitjeehelps
METALLURGY 553

Extraction of Metals
The metals like aluminium, copper, zinc and iron can be extracted from their ores in the following ways:

Aluminium Ore-Bauxite (Al 2O 3 ⋅ 2H2O)


Flow Chart for Al Extraction
Aluminium Ore-(Bauxite) (Al2O3 . 2H2O)

Baeyer’s process Serpeck’s process


Red bauxite → Sodium White bauxite ore + Coke


hydroxide (NaOH) solution 1800°C
AlN
→

180°C 80 atm

→
H2O
Na[Al(OH)4 ] (or) NaAlO2
Al(OH)3 + NH3
→

CO2
ppt.

→
Calcination
Al(OH)3
→
1500°C
ppt. Al(OH)3 Al2O3
Anhydrous
alumina
→ Electrolytic reduction

Electrolyte Al2O3 dissolved in CaF2 [cathode-carbon lining steel


and Na3AlF6 vessel anode - graphite rods]
→

→
2Al2O3+3C 4Al + 3CO2 →
At cathode: Al
At anode: CO and CO2

Electrolytic refining (Hoope’s method)


Pure Al(≈99.98% pure)

Copper Ore-Copper Pyrites (CuFeS2 )


Flow Chart for Cu Extraction ↓
Copper pyrites (CuFeS2 ) Matte (Cu 2S+ FeS)
↓ ↓
Crushed + Sieved
Bessemerisation (in bessemer converter in the
↓ presence of air)
Concentration (froth floatation method)
FeO + SiO2 → FeSiO3 (slag)
[Powdered ore + Pine oil → Sulphide ore (in froth) + Water
2FeS + 3O2 → 2FeO + 2SO2

2 Cu2 S +3 O2 → 2 Cu2O +2SO2
Roasting (in reverberatory furnace in the presence of air)
Auto- reduction
S + O2 → SO2 2 Cu2O + Cu2 S → 6 Cu + SO2

2CuFeS 2 + O2 → Cu2 S + 2FeS + SO2


4As + 3O2 → 2As 2O3

↓ Blister copper (98% Cu, 2% impurities)
2Cu 2S + 3O2 → 2CuO + 2SO2
Electrolytic refining
2FeS + 3O2 → 2FeO + 2SO2
Cathode — Pure Cu plates
Coke + Silica + Roasted ore
↓ Anode — Impure Cu plates
Smelting (in blast furnace in the Electrolyte — CuSO4 solution + H 2 SO4
presence of air)
Cu2O + FeS → Cu2 S + FeO ↓
2 FeS + 3O2 → 2 FeO +2SO2
Pure copper (≈ 99.6 − 99.9%) deposited at cathode
FeO + SiO2 → FeSiO3 (slag)

@iitjeehelps
554 SELF STUDY GUIDE BITSAT

Zinc Chief Ore–Zinc Blende (ZnS)


Flow Chart for Zinc Extraction
Zn Ore

Electrolytic process Reduction process


Concentration
(by froth floatation method) Concentration

Concentrated ore
Gravity process if ZnCO3 Froth floatation process
Roasting is used if ZnS is used

Roasted ore
Dissolution in dil. H2SO4+ Concentrated ore
Treatment with Ca(OH)2
900°C (excess of O2)

FeSO4 + Ca(OH)2 → Fe(OH)2+CaSO4 Roasting


CdSO4 + Zn → Cd + ZnSO4 2ZnS + 3O2 → 2ZnO +2SO2
ZnCO3 → ZnO + CO2
CuSO4 + Zn → Cu + ZnSO4

Filtration
ZnO
Filtrate Reduction

Electrolysis [Al → Cathode, Pb plate-anode] (i) Vertical retort process In silicon carbide at 1300°C
(ii) Belgian process Coke + Roasted (at 1100°C) ore
Pure Zn (= 99.95% pure) ZnO + C → Zn + CO
(at cathode)

Impure Zn (spelter)
(97.8% Zn + Impurities)
Distillation (950–1000°C)

Pure zinc (at 907°C)

Iron Chief Ore-Haematite (Fe2O 3 ) Reduction zone (Upper most part of blast furnace,
Flow Chart for Fe Extraction temperature range 500 to 800 K).
573 -673 K
Iron ore 3Fe 2O3 + CO → 2Fe3O2 + CO2 ↑
↓ 573 -883 K
Fe3O4 + 4CO → 3Fe + 4CO2 ↑
Concentration (Gravity process)
573 -873 K
↓ Fe 2O3 + CO → 2FeO + CO2 ↑
(Electromagnetic separation)
Combustion zone (Temperature 2170 K)
↓ C + O2 → CO2 (g) + 97 kcal
CO2 + C → 2CO( g )
Calcination
FeO + CO → Fe + CO2

Ore + Limited supply of air → CO2 , SO2 , moisture Slag formation zone (Temperature range: 900-1500 K)
AS 2O3 are removed
CaCO3 → CaO + CO2
Oxidation
FeO → Fe 2O3 Melting zone (Temperature range 1500 -1600 K) Molten
Ferric oxide
iron (1600 K as pig iron)
Impurities − 4% C + S, P, Si and Mn in small amount.
↓ Mild steel (0.1-0.5 % C)
Smelting Steel
Hard steel (0.5-1.5 % C)
(In blast furnace – coke + ore + limestone)

@iitjeehelps
Practice Exercise
1. Among the following statements, the incorrect one is 12. How do we separate two sulphide ores by froth
a. calamine and siderite are carbonates floatation method?
b. argentite and cuprite are oxides a. By adding pine oil
c. zinc blende and pyrites are sulphides b. By adding sodium cyanide
d. malachite and azurite are ores of copper c. By adding foaming agent
d. By passing air
2. Cassiterite is an ore of
a. Mn b. Ni c. Sb d. Sn 13. Commonly used collectors in froth floatation method are
3. A metal becomes quite hard due to the presence of I. pine oil II. coconut oil
small impurity because the impurities III. fatty acids IV. xanthates
a. reduce the number of mobile electrons a. I, II, and III b. II, III, IV
b. reduce the number of slide planes c. I, III, and IV d. I, II, and IV
c. reduce the crystal symmetry 14. Removal of the unwanted materials from the ore is
d. change the lattice structure of metals known as
4. Statement I Nitrate ores are very rare. a. benefaction b. dressing
c. concentration d. Either (a) or (b) or (c)
Statement II Bond dissociation energy of N 2 is very
high. 15. The process by which lighter earthly particles are
made free from heavier particles by washing with
Which of these statement(s) is/are true? water, is called
a. Only I b. Only II a. leaching b. levigation
c. Neither (a) nor (b) d. Both (a) and (b) c. benefication d. None of these
5. The metal found always in the free state, is 16. Which of the following metals is extracted by auto
a. Au b. Ag c. Cu d. Na reduction?
6. Which of the following metals is extracted by the a. Zinc b. Iron
electrometallurgical method? c. Copper d. Aluminium
a. Cu b. Fe 17. Among the following pairs of oxides, which pair cannot
c. Na d. Ag be reduced by carbon to give the respective metals?
7. In the froth floatation process for the facilitation of a. Fe2O3, ZnO b. PbO, Fe3O4
ores, the ore particles float because c. Cu2O, SnO2 d. CaO, K 2O
a. their surface do not get easily wetted by water 18. Which of the following fluxes is used to remove acidic
b. they are light impurities in metallurgical process?
c. they bear electrostatic charge a. Silica b. Lime stone
d. they are not soluble c. Sodium chloride d. Sodium carbonate
8. The pyrometallurgical operations mainly involve the 19. Flux is used to
use of a. remove all impurities from ores
a. complexation b. high temperature b. reduce metal oxide
c. sulphide ores d. electrolysis c. remove silica
d. remove silica and undesirable metal oxide
9. Froth floatation process for the concentration of Cu
illustrates the practical application of 20. The process of converting hydrated alumina into
a. adsorption b. absorption anhydrous alumina, is called
a. roasting
c. sedimentation d. coagulation
b. smelting
10. Chemical used as a depressant in separating ZnS c. dressing
from PbS in froth-floatation process, is d. calcination
a. NaCN b. NaCl 21. The chemical process in the production of steel from
c. BaCl 2 d. ZnSO4 haematite ore involve
a. reduction
11. Which of the following ores is not concentrated by
b. oxidation
froth floatation process?
c. reduction followed by oxidation
a. Pyrolusite b. Pentlandite d. oxidation followed by reduction
c. Zinc blende d. Copper pyrites

@iitjeehelps
556 SELF STUDY GUIDE BITSAT

22. Which one of the following reactions is an example of 31. Which of the following elements is extracted
auto-reduction? commercially by the electrolysis of an aqueous
a. Fe3 O4 + 4C → 3Fe + 4CO2 solution of its compound?
b. Cu2 O + C → 2Cu + CO a. Chlorine b. Bromine
c. Cu2+ (aq ) + Fe (s ) → Cu (s ) + Fe2+ (aq ) c. Sodium d. Aluminium
1 1
d. Cu2O + Cu2S → 3Cu + SO2 32. For which metal, the property of forming volatile
2 2 compounds is taken into advantage for its extraction?
23. In order to carry out a reduction process, temperature a. Nickel b. Iron
is selected to make c. Cobalt d. Tungsten
a. ∆G positive b. ∆H positive 33. Zone refining is based on the principle that
c. ∆G negative d. ∆H negative a. impurities of low boiling metals can be separated by
24. Smelting is involved in distillation
Heat b. impurities are more soluble in molten metal than in
a. Fe2O3 + 3C → 2Fe + 3CO solid metal
Heat c. different components of a mixture are differently
b. 2PbS + 3O2 → 2PbO + 2SO2
Heat
absorbed on an adsorbent
c. Al2O3 2 H2O → Al2O3 + 2 H2O d. vapours of volatile compound can be decomposed
Heat into pure metal
d. Zn2O3 → ZnO + CO2
34. In electrorefining of copper, some gold is deposited as
25. Specific gravity of slag is a. anode mud b. cathode mud
a. always less than that of molten metal c. cathode d. electrolyte
b. always higher than that of molten metal
c. same as that of molten metal 35. Which method of purification is represented by the
d. None of the above following equation?
523 K 1700 K
26. The least stable oxide among the following, is Ti(s ) + 2I2( g ) → TiI4 ( g ) → Ti(s ) + 2I2( g )
a. Sb2O3 b. Ag2O c. CuO d. ZnO a. Zone refining b. Cupellation
27. Which of the following statements about the c. Polling d. van-Arkel
advantage of roasting of sulphide ore before reduction 36. Impure nickel can be purified by
is not true? a. electrolytic refining
a. ∆f G° of the sulphide is greater than those of CS2 and b. Mond-carbonyl process
H2S c. zone-refining process
b. ∆f G° is negative for roasting of sulphide ore to oxide d. van-Arkel process
c. Roasting of sulphide to oxide is thermodynamically
feasible 37. To obtain pure germanium,which of the following
d. Carbon and hydrogen are suitable reducing agents methods of refining is preferred?
for metal sulphides a. Liquation b. Zone-refining
c. Electrolytic method d. Poling
28. The value of ∆f G° for the formation of Cr2O3 is
38. Mond’s process is used for the purification of
–540 kJ mol −1 and that of Al2O3 is –827 kJ mol −1. a. Ni b. Ti c. Zr d. Hg
Is the reduction of Cr2O3 with Al is feasible reaction?
39. In the metallurgy of aluminium,
a. The data is incomplete
b. The reaction is feasible a. Al3+ is oxidised to Al (s)
c. The reaction is not feasible b. graphite anode is oxidised to carbon monoxide and
d. The reaction may or may not be feasible carbon dioxide
c. oxidation state of oxygen changes in the reaction at
29. Use the relationship ∆G ° = −nFE °cell to estimate the anode
minimum voltage required to electrolyse Al2O3 in the d. oxidation state of oxygen changes in the overall
Hall-Heroult process reaction involved in the process
∆f G° (Al2O3 ) = −1520 kJ mol−1 40. Al can be obtained by
a. electrolysis of Al2O3 dissolved in Na 3AlF6
∆f G° (CO2 ) = −394 kJmol−1 b. heating alumina with cryolite
a. 0.8 V b. 1.60 V c. 2.8 V d. 3.0 V c. reducing Al2O3 by coke
d. reducing Al2O3 by chromium
30. Electrolytic refining’s is used to purify which of the
following metals? 41. Aluminothermic process is used for metallurgy of
a. Cu and Zn b. Ge and Si a. Pb b. Ag
c. Zr and Ti d. Zn and Hg c. Al d. None of these

@iitjeehelps
METALLURGY 557
42. Which of the following furnaces can be used to get 44. The process used to produce tough pitch copper, is
above 3000° C temperature? a. cupellation b. distillation
a. Blast furnace b. Arc furnace c. poling d. zone-refining
c. Muffle furnace d. Reverberatory furnace
45. In Mc-Arthur Forrest method, silver is extracted from
43. The process in which 10 g of green wood is used, is the solution of Na[Ag(CN)2] by the use of
a. poling b. pickling a. zinc b. magnesium
c. anodising d. galvanising c. copper d. iron

BITSAT Archives
1. Consider the following Ellingham diagram for carbon c. Carbon reduces many oxides at elevated
temperature because ∆G° vs temperature line has a
0
negative slope
–100 1
1 d. ∆S ° [C(s ) + O2(g ) → CO(g )]
–200 C+ O → CO 2
2 2 C + O2 CO2 < ∆S ° [C(s ) + O2(g ) → CO(g )]
–300
–400
2. In the extraction of Ag, Zn is removed from (Zn-Ag)
∆G° alloy through [2013]
–500 a. cupellation b. fractional crystallisation
–600 c. distillation d. electrolytic refining
–700 3. Pyrolusite is a/an [2011]
–800 a. oxide ore b. sulphide ore
710 c. carbide ore d. None of the above
500 1000 1500 2000 2500
T(°C)
4. van-Arkel method is based on [2010]
a. cupellation method b. furnace refining method
Which of the following statements is incorrect for the c. poling method d. None of the above
above Ellingham diagram? [2014]
5. Aluminium is present in [2010]
a. Upto 710°C, the reaction of formation of CO2 is
a. gypsum b. carnallite c. asbestos d. diaspore
energetically more favourable but above 710°C, the
formation of CO is preferred 6. The extraction of which of the following metals
b. Carbon can be used to reduce any metal oxide at a involves bassemerisation? [2009]
sufficiently high temperature a. Fe b. Ag c. Al d. Cu

Answer with Solutions


Practice Exercise 9. (a) Basically, froth floatation method illustrates the
practical application of adsorption.
1. (b) Argentite (Ag2S) is a sulphide ore and cuprite (Cu2O) is 10. (a) PbS can be separated from ZnS in the presence of
an oxide ore. NaCN. It depresses the floatation property of ZnS by
2. (d) SnO2 (tin stone) is also called cassiterite. forming a complex Na 2[Zn(CN)4].
11. (a) 12. (b) 13. (c)
3. (b) 4. (d )
14. (d) Removal of the unwanted materials from the ore is
5. (a) Au is always found in free state. known as benefaction or dressing or concentration.
6. (c) Na is extracted by the electrometallurgical method. 15. (b) Levigation or hydraulic washing.
7. (a) In the froth floatation process for the facilitation of 16. (c) Cu2S + 2Cu2O → 6Cu + SO2
ores, the ore particles float because their surface do not
get wetted by water. 17. (d) Potassium and calcium are strong reductant, hence
their oxides cannot be reduced by carbon.
8. (b) High temperature is generally used in the
pyrometallurgical process. 18. (b) Lime stone (CaCO3 ) is a basic flux. It is used to
remove acidic impurities like SiO2 as CaSiO3 (slag).

@iitjeehelps
558 SELF STUDY GUIDE BITSAT

19. (d ) Flux removes silica and undesirable metal oxide as = 1858 × 103 J
silicate. ∴ nFE° = 1858 × 103 (n = 12 electrons)
20. (d) In calcination, moisture or hydrated water is removed 1858 × 103
on heating. ∴ E° = = 1.60 V
12 × 96500
Al2O3 ⋅ 2H2O → Al2O3 + 2H2O ↑
30. (a) Impurities are highly soluble in molten state than their
21. (d) Haematite (Fe2O3 ) having little FeO is removed by solid state.
acidic flux SiO2 in the form of slag (FeSiO3) .
31. (a) Electrolysis of aqueous solution of NaCl (brine) gives
1 1
22. (d) Cu2O + Cu2S → 3Cu + SO2 Cl2 at anode.
2 2
32. (a) Nickel is extracted by Mond's process.
23. (c) ∆G = ∆H − T∆S 33. (b) CaSiO3 is called slag which is fusible and lighter than
Temperature is chosen such that the sum of ∆G in the the molten metal and is removed easily.
following two combined redox processes is negative
34. (a)
MnO + Ared → XM + AOoxi
35. (d) van-Arkel (and de Boer) process is based on the
24. (a) Smelting basically involves the reduction of oxides of thermal decomposition of a volatile compound like an
metal with carbon or coke at high temperature. iodide, which is first formed by direct combination of metal
25. (a) Specific gravity of slag is always less than molten to be purified and iodine. Metal formed is in purest form.
metal. Titanium and zirconium are purified by this method.
26. (b) The least stable oxide is Ag2O. 36. (b) Mond-carbonyl process is used to purify impure Ni.
27. (d ) Both C and H2 are not suitable for reducing sulphide 37. (b) Elements such as Ge, Si, Ga, etc., which are used as
ore because ∆f G ° of metal sulphide is more than that of semiconductors, are refined by zone-refining.
CS2 and H2S which will be formed as a result of reduction. 38. (a) Mond’s process is used for the purification of Ni.
4 2 39. (b) In the metallurgy of Al, graphite anode is oxidised to
28. (b) Cr(s) + O2(g ) → Cr2O3(s ); ∆f G° =− 540 kJ ...(i) CO and CO2.
3 3
4 2 40. (a) Al can be obtained by electrolysing Al2O3 dissolved in
Al(s) + O2(g ) → Al2O3(s ); ∆f G° = −827 kJ ...(ii)
3 3 Na 3 AlF6.
On subtracting Eq. (i) from Eq. (ii) 41. (d) Thermite process is not used for the metallurgy of Pb,
2 4 2 4 Al and Ag.
Cr2O3(s) + Al (s ) → Al2O3(s ) + Cr (s );
3 3 3 3 42. (c) Muffle furnace is used to get above 3000° C
∆f G° = −287 kJ temperature.
Since, ∆G° comes out to be negative, the reaction is 43. (a) Poling is the process in which 10 g of green wood is
feasible. used. It acts as a reducing agent.
29. (b) 2Al2O3 + 3C → 4Al + 3CO2 44. (c)
∆f G ° = 3∆G ° (CO2 ) − 2∆G ° (Al2O3 ) 45. (b) In Mc-Arthur Forrest method, silver is extracted from
the solution of Na[Ag(CN)2] by the use of Mg.
= 3 × ( −394) − 2 × ( −1520) = 1858 kJ

BITSAT Archives
1. (d) Since, ∆G ° = ∆H ° − T∆S ° 3. (a) Pyrolusite is MnO2. Thus, it is an oxide ore.
where, ∆G° = standard Gibbs’ free energy of the reaction 4. (d)
∆S ° = standard entropy of the reaction 5. (d) Gypsum CaSO4 ⋅ 2H2O
∆H ° = standard enthalpy of the reaction Carnallite KCl ⋅ MgCl2 ⋅ 6H2O
T = temperature Asbestos CaSiO3 ⋅ 3MgSiO3
From the above equation, it is clear that ∆G° will be more Diaspore Al2O3 ⋅ H2O
negative when ∆S ° is less negative (or ∆S ° is high). 6. (d) In a bessemer converter, copper pyrites are oxidised
2. (c) The extraction of Ag using (Zn-Ag) alloy is called to FeO and Cu2O. FeO is slagged off. Cu2O reacts with
Parke’s process. As zinc is volatile at 920° while Ag is Cu2S left unoxidised to give Cu.
not. Thus, on heating (Zn + Ag) alloy, zinc vaporises while 3Cu2S + 3O2 → 2Cu2O + 2SO2 ↑
Ag remains at the bottom of the vessel. Hence, Zn is
removed from (Zn - Ag) alloy through distillation. 2Cu2S + 3Cu2S → 6Cu + SO2 ↑

@iitjeehelps
17
p-Block Elements-I
(Group 13 and 14)

Group-13 Elements : Boron Family


Boron (B), aluminium (Al), gallium (Ga), indium (In) and thallium (Th) are the members of group-13.
Boron is non-metal while rest of the members are metals.

Important Properties
General Configuration ns 2 , np1
Oxidation State
Stability of +3 oxidation state decreases down the group and that of +1 oxidation state increases
down the group due to inert pair effect. Fourth period and onwards effective nuclear charge
increases due to poor shielding effect, therefore the radius of gallium (135 pm) is less than that of
aluminium (143 pm) and down the group, inert pair effect increases.

Hydrides
Boron hydrides exist in dimeric or polymeric form. The simplest boron hydride is diborane, (B 2H6 ).
Aluminium forms only one colourless, solid polymeric hydride (AlH3 )n , known as alane. Hydrides of
Ga and In are not very much stable. B, Al and Ga form complex anionic hydrides, e.g. NaBH 4 , LiAlH 4
and LiGaH 4 .
All are strong reducing agents.
LiAlH 4 > NaBH 4 (reducing power)

@iitjeehelps
560 SELF STUDY GUIDE BITSAT

Halides Anomalous Behaviour of Boron


Halides have incomplete octet, therefore, have a high Boron shows anomalous behaviour due to its small size and
tendency to accept electrons and behave as Lewis acid. high nuclear charge/size ratio, high electronegativity and
BI3 > BBr3 > BCl3 > BF3 (acidic character) non-availability of d-electrons.
BF3 is a colourless gas, BCl3 is a colourless fuming liquid The main points of differences are:
while BI3 is a white fusible solid. Due to back-bonding and
resonance, the B¾ F bond of BF3 gets a bond order of 1.33. (i) It is a typical non-metal whereas other members are
metals.
3 (anhydrous) is covalent and behaves as Lewis acid
● AlCl

because Al3 + has small size and high polarising power (ii) It shows allotropy and exists in two forms: crystalline
but AlCl3 (hydrated) is ionic in nature. and amorphous. Aluminium is a soft metal and does
● Al3 + has small size due to the presence of vacant not exist in different forms.
d-orbitals, therefore AlCl3 easily gets hydrolysed in water Similarities with silicon
and form octahedral [Al(OH)6 ]3 + ion in which the
Due to its small size and similar charge/mass ratio, boron
hybridisation state of Al is sp3d 2 . shows resemblance with silicon to exhibit diagonal
Boron relationship.
Boron (B) is the first element of group 13 and occurs in two Some point of similarities
isotopic forms, i.e. 5B 10 (18%) and 5B 11 (82%). Its abundance (i) Both boron and silicon are typical non-metals, having
in the earth crust is less than 0.001% by mass. high melting and boiling points, nearly same densities
The important ores are (B = 2.35 g mL -1 , Si = 2.34 g mL -1 ), low atomic volumes
and bad conductor of heat. These are used as
Borax : Na 2 B 4O7 × 10H 2O
semiconductors.
Colemanite : Ca 2 B6O11 × 5H 2O
(ii) Both exist in amorphous and crystalline state and
Panderinite : Ca 2 B6O11 × 3H 2O exhibit allotropy.
Boracite : 2Mg3 B8O15 × MgCl 2
Borax (Na 2 B4O 7 × 10H 2O)
Kernite : Na 2 B 4O5(OH)4
[Sodium tetraborate decahydrate]
Properties (i) It is the most important compound of boron. It is
The main properties of boron are as follows: white crystalline solid.
(a) It is a hard solid (melting point = 2076°C, boiling (ii) It dissolves in water to give an alkaline solution.
point = 3927°C) and is a bad conductor of heat and Na 2B 4O7 + 7H 2O ¾® 2NaOH + 4H3BO3
electricity. Orthoboric acid
42(weak acid)
(b) It exists in two allotropic forms namely crystalline
1444 4444 3
Glassy mass
and amorphous. Crystalline form is chemically
inert and black while amorphous form is (iii) On heating, it loses the water of crystalisation and
chemically active and brown coloured solid. swells up to form a mass of a bead.
D
(c) Action of acids Na 2B 4O7 × 10H 2O ¾® Na 2B 4O7
-10H 2O
B + 3 HNO3 ¾® H3 BO3 + 3 NO2 Swell up
(sodium metaborate)
2B + 3 H 2SO4 ¾® 2H3BO3 + 3 SO2 D
¾® 2NaBO2 + B 2O3
(d) Action of alkalies On fusion with alkalies,
hydrogen is liberated. 144442Boric
4444anhydride
3
Glassy mass
2B + 6NaOH ¾® 2Na3BO3 + 3 H 2
(iv) With acids, it gives orthoboric acid, a weak acid.
Uses Na 2B 4O7 + 2HCl + 5H 2O ¾®
It is used as control rods in atomic reactors and as a 2NaCl + 4B(OH)3 or H3BO3
deoxidiser Orthoboric acid

@iitjeehelps
p-BLOCK ELEMENTS-I (GROUP 13 AND 14) 561
(v) Structure Properties
Borax contains the tetrahedral units, i.e. [B 4 O 5(OH) 4 ] 2- . (a) It is an electron deficient compound and can accept a
OH lone pair of electron, thus behaves as Lewis acid.
_ (b) BF3 combines with water to form two types of
B
hydrates, i.e. BF3 × H 2O (melting point = 10.18°C) and
O O
BF3 × 2H 2O (melting point 6.36°C). It gets hydrolysed in
HO—B O B—OH aqueous solutions yielding boric acid and
O O hydrofluoroboric acid.
_
B BF3 + 3H 2O ¾® H3BO3 + 3HF
OH If BF3 is present in excess, it is dissolved in HF to give
hydrofluoroboric acid H[BF4 ].
Boric Acid (H 3 BO 3 )
BF3 + HF ¾® H + [BF4 ]-
(i) It is a soft white crystalline solid, soapy to touch and
less soluble in cold water. Reaction of Aluminium with Acids and
(ii) It is a weak monobasic acid. It is not a protonic acid but Alkalies
acts as a Lewis acid. (a) Action of water
Heat Heat
(iii) H3BO3 ¾¾® HBO2 ¾¾® B 2O3 2Al + 6H 2O ¾® 2Al(OH)3 + 3H 2 ­
Metaboric Boric
acid anhydride (b) Action of alkalies
(iv) Structure of boric acid shows that it has a layered 2Al + 2NaOH + 2H 2O ¾® 2NaAlO2 + 3H 2 ­
Sodium meta-
structure in which planar BO3 units are joined by aluminate (soluble)
H-bonds. Fused
2Al + 6NaOH ¾ ¾¾® 2Na3 AlO3 + 3H 2 ­
(v) It is used as an antiseptic, eye wash, as food Sodium aluminate
preservative, in glass industry and in pottery. (c) Action of acids
Diborane (B 2H6 ) 2Al + 6H 2SO4 ¾® Al 2 (SO4 )3 + 3SO2 + 6H 2O
Hot conc.
It is a colourless, highly toxic gas with a boiling point of
180 K. It catches fire spontaneously upon exposure to air. Al becomes passive in HNO3 due to oxidation and
formation of a thin oxide film on its surface.
B 2H6 + 3O2 ¾® B 2O3 + 3H 2O
Thus, conc. HNO3 may be stored in Al vessels.
With ammonia, It gives borazine.
3B 2H6 + 6NH3 ¾® 3[BH 2 (NH3 )2 ]+ [BH 4 ]-
Heat
¾® 2 B3 N3H6 + 12 H 2
Group-14 Elements :
Borazine Carbon Family
Borazine (B3 N3H6 ) is known as ‘inorganic benzene’. Carbon(C), silicon (Si), germanium (Ge), tin (Sn) and
However, at high temperature, inorganic graphite (BN )x is lead (Pb) are the members of group 14. Carbon is the
obtained instead of borazine. seventeenth most abundant element by mass in the earth’s
Structure of Diboranex crust. C is non-metal, Si, Ge are metalloids and Sn, Pb are
Hb metals.
Ha Ha H H H
(3c–2e)
(2c–2e) B 97° B1 120° B B

Ha
13
4p
19
pm H
H H H Important Properties
m Hb a Banana Bond
(B2H6) General Configuration ns 2np2
Catenation
Boron Trifluoride (BF3 )
The tendency for catenation is maximum in carbon due to
It is a colourless, pungent gas. The melting point and boiling
maximum strength of C ¾ C bond (bond energy of
point are – 127.1°C and – 99.9°C, respectively. It is
C ¾ C = 354 kg/mol) and it decreases down the group as
exceedingly soluble in water, that’s why it is collected over
mercury. C >> Si > Ge > Sn >> Pb

@iitjeehelps
562 SELF STUDY GUIDE BITSAT

Allotropy Halides Group


All the members except Pb show allotropy. IV A elements form covalent tetrahalides of the type MX 4
(X = halogen) having tetrahedral structure, except PbBr4 and
There are some allotropes of carbon given below :
PbI 4.
The order of thermal stability of tetrahalides is
Diamond
CX 4 > SiX 4 > GeX 4 > SnX 4 > PbX 4
It is the purest, hardest form of carbon with high
Thermal stability and volatility of tetrahalides with a common
refractive index and density. In it, each carbon atom
central atom fall with the increase in the molecular mass of the
(sp3 -hybridised) is tetrahedrally surrounded by four
tetrahalides, e.g.
other carbon atoms. It is three dimensional polymer. It
MF4 > MCl 4 > MBr4 > M I 4
does not conduct electricity as it has no free electrons.
Except CCl 4 , other tetrachlorides are easily hydrolysed by water
Graphite due to the availability of empty d-orbitals in them.
It has two dimensional structure. sp 2 -hybridised
carbon atom forms three covalent bonds with three
Silica (SiO2 )
other carbon atoms in the same plane and the 4th It is commonly called as silica. It occurs in earth's crust as
electron of each carbon remains free and is crystalline quartz, tridymite and crystallobite. Amorphous
responsible for electrical conductivity of graphite. veriety flint is also found.
Methods of Preparation
Fullerenes 1. It is obtained by heating silicon in oxygen as
These are the only pure form of carbon. C 60 molecule Si + O2 ¾® SiO2
contains 12 five membered rings and 20 six membered 2. Pure SiO2 is prepared in its amorphous forms as a white
rings. The five membered rings are connected to six powder by reacting SiCl 4 or SiF4 with water as
membered rings while six membered rings are SiCl 4 + 2H 2O ¾® SiO2 + 4HCl
connected to both five and six membered rings.
SiF4 + 2H 2O ¾® SiO2 + 4HF
Oxidation States The latter method is normally less used as SiO2 further reacts
The common oxidation states are +4 and +2. Carbon with HF to form H 2SiF6 (fluorosilicic acid).
also exhibits negative oxidation states, i.e. -4. Down the
group, stability of +4 oxidation state decreases and of
Silicates
+2 oxidation state increases due to inert pair effect. (i) A large number of silicate minerals exist in nature. Some of
the examples are feldspar, zeolites, mica and asbestos. The
Reactivity Towards Water basic structural unit of silicates is SiO44 - .
Carbon, silicon and germanium are not affected by (ii) The SiO44 - unit is neutralised by positively charged metal
water. Tin decomposes with steam to form dioxide and
ions, if all the four corners are shared with other tetrahedral
dihydrogen gas.
units.
Heat
Sn + 2H 2O ¾¾® SnO2 + 2H 2 Silicates are of following types :
Lead remains unaffected by water, probably because (i) Orthosilicates Simple silicates containing SiO44 - tetrahedra.
of a protective oxide film formation.
(ii) Pyrosilicate Two tetrahedral units share one O-atom to
Oxides obtain Si 2O 6-
7 anion.
CO2 , SiO2 and GeO2 are acidic, whereas SnO2 and PbO2 (iii)Cyclic silicates Two tetrahedral units share two oxygen
are amphoteric in nature. atoms and form (SiO32- )n or (SiO3 )2nn- anion.
(i) Among monoxides, CO is neutral, GeO is distinctly
(iv) Chain silicates Share two oxygen atoms, (SiO32- )n or
acidic whereas SnO and PbO are amphoteric.
(SiO3 )2nn - are obtained.
(ii) Monomeric form of CO2 is stable due to
non-availability of d-orbitals. Carbon has (v) Sheet silicates Involve sharing of three O-atoms per
tendency to form a multiple bond (O == C == O) tetrahedron to form (Si 2O2-
5 )n .
but SiO2 exists in three dimensional polymeric
(vi) Three dimensional silicates All the four corners
form and has high melting point and it is solid at
(O-atoms) of SiO44 - tetrahedra are shared with other.
room temperature.

@iitjeehelps
p-BLOCK ELEMENTS-I (GROUP 13 AND 14) 563
Silicones Silicon tetrachloride (SiCl 4 )
(i) These are synthetic organosilicon compounds Preparation
which has repeated unit of R2SiO.
It is prepared by heating Si or SiC with Cl 2 .
(ii) These are prepared from alkyl halides.
¾D ® SiCl 4 (l )
Si (s ) + 2Cl 2 ( g ) ¾
Cu powder
2CH3Cl + Si ¾¾¾® (CH3 )2 SiCl 2 ¾¾®
2 + 2H O ¾D ® SiCl 4 (l ) + CCl 4 (l )
or SiC (s ) + 4Cl 2 ( g ) ¾
570 K - 2HCl
Properties
(CH3 )2 Si(OH)2
(i) It is a volatile liquid, having boiling point 330.57 K.
CH3 CH3 CH3
½ ½ ½ ¾D ® Si(OH)4 ¾
(ii) SiCl 4 + 2H 2O ¾ ¾D ® SiO2 × xH 2O
HO ¾ Si ¾ OH + HO ¾ Si ¾ OH + HO ¾ Si ¾ OH Silisic Silica gel
acid
½ ½ ½
CH3 CH3 CH3 Uses
æ CH3 ö CH3 (i) Silica gel is used as a catalyst in petroleum industry and as
Polymerisation ç | ÷ | an adsorbent in column chromatography.
¾¾¾¾¾¾¾® ¾çO — Si — O÷— Si — (ii) Si obtained after reduction of silica gel is used for making
ç ÷ |
ç | ÷ transistors, computer chips and solor cells.
è CH3 ø CH3 n
Silicone
Zeolites
Highly cross-linked silicone polymer is obtained by Zeolites are microporous alumino silicate having general
the hydrolysis of RSiCl3 . formula M x / n[ AlO2 ]x [SiO2 ] y mH 2O.
(iii) They are used as sealant, greases electrical Uses
insulators and for water proofing of fabrics. Being (i) They are used as molecular seieves to separate the
biocompatible, they are also used in surgical and molecules of different sizes.
cosmetic plants. (ii) They are used as catalyst.

Practice Exercise
1. Boron fibres are used in making 4. Heating of an aqueous solution of aluminium chloride to
a. bullet-proof jacket dryness will give
b. light composite material for aircraft a. Al(OH)Cl2 b. Al2O3 c. Al2Cl6 d. AlCl3
c. Both (a) and (b)
d. None of the above 5. When orthoboric acid (H3 BO3 ) is heated, the residue is
a. boron b. metaboric acid
2. Match the columns and choose the correct options c. boric anhydride d. borax
from the codes given below.
6. Borax on heating with cobalt oxide forms a blue bead of
Column I Column II a. Co(BO2 )2 b. CoBO2
A. Orthoboric acid 1. Na 2 B 4O7 × 4H 2O c. Co4(BO3 )2 d. Na 3Co(BO3 )2

B. Borax 2. Na 2 B 4O7 × 10H 2O


7. Borax bead test is responded by
a. divalent metals
C. Kernite 3. H 3 BO3
b. trivalent metals
c. light metals
Codes d. metal which forms coloured metaborates
A B C A B C
a. 1 2 3 b. 3 2 1 8. Which of the following is a component of ruby?
c. 2 1 3 d. 1 3 2 a. CaCO3 b. MgCO3
c. Al2O3 d. Al(OH) 3
3. Boron cannot form which one of the following
anions? 9. Tincal is
a. BF63 - b. BH-4 a. Na 2CO3 × 10H2O b. NaNO3
c. B(OH)-4 d. BO-2 c. Na 2 B4O7 × 10H2O d. NaCl

@iitjeehelps
564 SELF STUDY GUIDE BITSAT

10. Boric acid is used in carom boards for smooth gliding 17. B—F bonds in BF3 are weakly acidic. The main factor
of pawns because responsible is
a. H3BO3 molecules are loosely chemically bonded and a. pp -pp back bonding
hence soft b. pp -dp back bonding
b. its low density makes it fluffy c. three centred- two electron bonds in BF3
c. it can be powdered to a very small grain size d. large electronegativity of F
d. H-bonding in H3BO3 gives it a layered structure
18. Which of the following statements is incorrect for
11. Boric acid when burnt with ethyl alcohol gives a green aluminium chloride (Al2Cl6 ) ?
edged flame due to the combustion of
a. It exists as a dimer in solvents like benzene and
a. boron trifluoride b. metaboric acid
c. ethyl borate d. orthoboric acid carbon disulphide
b. The aluminium atom is tetrahedrally surrounded by
12. In diborane, two H—B—H angles nearly are
four chlorine atoms
a. 60°, 120° b. 95°, 120°
c. 95°, 150° d. 120°, 180° c. Each aluminium atom forms three covalent bonds
and one coordinate bond
13. Which of the following statements are correct
regarding diborane? d. In the vapour state, aluminium chloride does not exist
as a dimer
I. Two bridged hydrogen atoms and two boron
atoms lie in one plane. 19. AlCl3 achieves stability by forming a dimer. Structure
H of this dimer is
H H Al Al
a. Cl Cl Cl Cl
Cl Cl b. Al Al
B B Al Al Cl Cl Cl
H H Al Al Cl
H c. Cl Cl d. None of these
Al Al Cl
II. Out of six B—H bonds, two bonds can be
described in terms of 3 centre-2 electron bonds. 20. Which of the following reactions will not give the
III. Out of six B—H bonds, four B—H bonds can be anhydrous AlCl3 ?
described in terms of 3 centre-2 electron bonds. a. By heating AlCl3 × 6H2O
IV. Four terminal B—H bonds are two centre-two b. By passing dry HCl gas on heated aluminium powder
electrons regular bonds. c. By passing dry chlorine gas on heated aluminium
a. I, II and IV b. II, III and IV powder
c. I and II d. III and IV d. By passing dry chlorine gas over a heated mixture of
alumina and coke
14. The species which does not exist, is
21. The tendency of Ge, Sn, Pb to show +2 oxidation
a. [AlF6] 3- b. [GaF6] 3- c. [InF6] 3- d. [BF6] 3-
state, increases in the sequence
15. In Al2Cl6, a. Ge = Sn < Pb b. Ge < Sn < Pb
c. Ge > Sn > Pb d. Ge > Sn = Pb
I. six Al—Cl bonds are of same length and two of
different length. 22. The order of catenation of C, Si, Ge, Sn is
II. the angle Al—Cl—Al is 87°. a. C < Si < Ge < Sn b. C > Si > Ge > Sn
III. four Al—Cl bonds are of same length and two of c. C >> Si > Ge » Sn d. C » Si >> Ge » Sn
different length. 23. The hybridisation of the central atom in
IV. the angle Cl—Al—Cl is 93° and 110°. SiF62- , [GeCl6] 2- and [Sn(OH)6] 6- is
Which statements are correct? a. sp 3d b. sp 3d 2 c. sp 3 d. sp 3d 3
a. I, II and III b. II, III and IV
c. I, III and IV d. I, II and IV 24. Graphite conducts electricity due to the
a. highly delocalised nature of p-electrons
16. Consider the following statements, b. highly localised nature of p-electrons
I. BBr3 is stronger acid than BF3 . c. highly polarised nature of p-electrons
II. pp- pp back bonding occurs in the halides of boron d. None of the above
but not in the halides of aluminium. 25. Graphite is a soft, solid lubricant, extremely difficult to
III. Borazine is less reactive than boron. melt. The reason for this anomalous behaviour is that
IV. Al is unstable in air and water. graphite
The set of incorrect statement is a. is an allotropic form of diamond
b. has molecules of variable molecular masses like
a. I and II b. II and III
polymers
c. III and IV d. None of these

@iitjeehelps
p-BLOCK ELEMENTS-I (GROUP 13 AND 14) 565
c. has carbon atoms arranged in large plates of rings of 36. The shape of gaseous SnCl2 is
strongly bound carbon atoms with weak interplate
a. tetrahedral
bonds
b. linear
d. is a non-crystalline substance
c. angular
26. Carborundum is obtained when silica is heated at high d. T-shaped
temperature with 37. Mark the oxide which is amphoteric in character.
a. carbon b. carbon monoxide
a. CO2 b. SiO2
c. carbon dioxide d. calcium carbonate
c. SnO2 d. CaO
27. The mixture of CO and H2 is known as
38. SiH4 and O2 mixture on bubbling through water and
a. producer gas b. synthesis gas
bubbles coming in contact with air,
c. water gas d. Both (b) and (c)
a. burns with a luminous flame
28. A mixture of CO and N 2 is known as b. vertex rings of finely divided silica are formed
a. synthesis gas b. water gas c. SiH4 + 2O2 ¾® SiO2 + 2H2O, reaction occurs
c. producer gas d. All of these d. All of the above
29. Water gas is used as an industrial fuel because 39. Ionisation enthalpy (Dt Hi in kJ mol-1) for the elements
a. on combustion, it further produces CO2 of group-13 follows the order
b. on combustion, it looses heat a. B > Al > Ga > ln > Tl
c. on combustion, it produces CO2 and liberates heat b. B < Al < Ga < ln < Tl
d. None of the above c. B < Al > Ga < ln > Tl
30. Silica gel is used as a/an …… d. B > Al < Ga > In < Tl
a. dehydrating agent (drying agent) 40. Two elements P and Q react separately with highly
b. dehydrogenating agent
electropositive metal to form binary compounds,
c. reducing agent
d. oxidising agent which upon hydrolysis yield mixture of boranes and
silanes. P and Q respectively are
31. (Me)2 SiCl2 on hydrolysis will yield a. B, Al b. Si, B
a. (Me)2 Si ==O b. — ]n-
[ O—(Me)2 Si —O— c. B, Si d. Al, B
c. Me2SiCl(OH) d. (Me)2 Si(OH)2 41. Which of the following is most stable?
32. Consider the following statements, a. Sn2+ b. Ge2+
I. GeCl4 in HCl forms H2[GeCl6]. c. Si2+ d. Pb2+
II. Trisilyl amine is pyramidal. 42. The correct stability order for boron halides is
III. SnCl4 is more stable than SnCl2. a. BF3 > BCl3 > BBr3 > BI3
IV. Ge(OH)2 is amphoteric. b. BCl3 > BF3 > BBr3 > Br3
c. BI3 > BBr3 > BCl3 > BF3
The correct statements are d. BBr3 > BCl3 > BI3 > BF3
a. I and IV b. II and III c. I and II d. III and IV
43. Aluminium metal is corroded in coastal places near to
33. Name of the structure of silicates in which three the sea, it is due to protective oxide film,
oxygen atoms of [SiO4 ] 4 - are shared, is a. is removed by sea water
a. pyrosilicate b. sheet silicate b. reacts with sea water
c. linear chain silicate d. three dimensional silicate c. is attacked by salt present in sea water
d. reacts with sand particles
34. An inorganic compound X , made of two most
occurring elements in the earth’s crust and used in 44. Which of the following elements form both neutral as
building construction, when reacts with carbon, forms well as acidic oxides?
a diatomic molecule, which is poisonous in nature. a. Sn
Compound X may be b. Si
a. SiO2 b. Al2O3 c. CaO d. CO2 c. C
d. P
35. An inorganic compound containing (3c -2e) and
45. Al2O3 can be converted into anhydrous AlCl3 by
(2c -2e) bonds when reacts with NH3 at a certain heating from
temperature, gives a compound X which is a. a mixture of Al2O3 and carbon in dry Cl2 gas
isostructural with benzene and when reacts at high b. Al2O3 with Cl2 gas
temperature, forms a substanceY . The substanceY is c. Al2O3 and HCl gas
a. B2H6 b. B3N3H6 d. Al2O3 with NaCl in solid state
c. inorganic graphite d. B2H6 × 2NH3

@iitjeehelps
BITSAT Archives
1. Which of the following statements are incorrect in a. Strength as Lewis acid— BCl3 > AlCl3 > GaCl3
context of borax? [2013] b. Inert pair effect— Al > Ga > In
a. It is made up of two triangular BO3 units and two c. Oxidising property— Al3+ > In3+ > Tl3+
tetrahedral BO4 units d. First ionisation enthalpy— B > Al > Tl
b. One mole of borax can be used as a buffer
c. It is a useful primary standard for titrating against 3. Which glass has the highest percentage of lead?
acids a. Soda glass [2007]
d. Aqueous solution of borax can be used as a buffer b. Flint glass
c. Jena glass
2. For the properties mentioned, the correct trend for the
d. Pyrex glass
different species is in [2013]

Answer with Solutions


Practice Exercise 14. (d) BF63- does not exist because there is no vacant
d-subshell in boron.
1. (c)
15. (b) The structure of Al2Cl6 is shown below
2. (b) Orthoboric acid is H3BO3
Cl 93° Cl Cl
Borax is Na 2B4O7 × 10 H2O 110° Al2 Al
87°
6Å .2
Kernite is Na 2B4O7 × 4H2O Cl 1Å Cl Cl
2. 0
3. (a) Due to absence of 2d-orbital, maximum covalency is
four. Thus, BF 3-
6 is not formed.
16. (c) Borazine is more reactive than boron. Aluminium is
stable in air and water.
4. (b) Aqueous solution of AlCl3 is acidic due to hydrolysis.
17. (a) It is pp - pp bonding involving B and F-atom
AlCl3 + 3H2O r Al(OH) 3 + 3HCl
responsible for the acidic nature of boron halides as
On strong heating, Al(OH) 3 is converted into Al2O3. BF3 < BCl3 < BBr3 < BI3.
D
2Al(OH) 3 ¾® Al2O3 + 3H2O Smaller, atom shows more back bonding.
D D 18. (d)
5. (c) H3BO3 ¾¾¾® HBO2 ¾® B2O3
Orthoboric acid above 70 K Metaboric acid Boric anhydride 19. (b)

6. (a) Na 2B4O7 ¾® 2NaBO2 + B2O3


20. (a) Hydrated aluminium chloride undergoes hydrolysis to
forms Al2O3 .
B2O3 + CoO ¾® CoO × B2O3 or Co(BO2 )2 2AlCl3 × 6H2O ¾® 2Al(OH)3 + 6HCl
Blue bead
2Al(OH)3 ¾® Al2O3 + 3H2O
7. (d) Metals forming coloured bead can be identified by 21. (b) The tendency of Ge, Sn, Pb to show +2 oxidation
borax bead test. state increases in the sequence Ge < Sn < Pb.
8. (c) Al2O3 is a component of ruby (Al2O3 + Cr2O3 ), red 22. (c) The order of catenation of C, Si, Ge, Sn is
precious stone. C >> Si > Ge » Gn
9. (c) Naturally occurring crude borax is called tincal. Thus, 23. (b) The species like, SiF 2- 2-
6 , [GeCl 6 ] , [Sn(OH)6 ]
2-
exist
it is chemically Na 2B4O7 × 10H2O. where the hybridisation of the central atom issp d .3 2

10. (d) Boric acid is used in carom boards for smooth gliding 24. (a) Graphite conducts electricity due to the highly
of pawns because H-bonding in H3BO3 gives it a layered delocalised nature of p-electrons.
structure.
25. (c) Graphite has carbon atoms arranged in large
-3H2O
11. (c) H3BO3 + 3C2H5OH ¾¾® (C2H5 )3 BO3 hexagonal layers with weak van der Waals’ interactions
Boric acid Ethyl alcohol Ethyl borate between the layers.
It burns with green edged flame. High temp.
26. (a) SiO2 + 3 C ¾¾¾¾® SiC + 2CO
2300 K Carborundum
12. (b) In diborane, two H—B—H angles are nearly 95° and
120°. 27. (d) Mixture of CO and H2 is known as water gas
13. (a) The correct statements are I, II and IV. as well as syn gas.

@iitjeehelps
p-BLOCK ELEMENTS-I (GROUP 13 AND 14) 567
28. (c) Mixture of CO and N2 is known as producer gas. 36. (c) Gaseous SnCl2 is angular withsp 2-hybridisation.
29. (c) Water gas on combustion produces CO2 and 37. (c) SnO2 is amphoteric. It dissolves in acids as well as in
liberate heat. alkalies.
30. (a) Silica gel is used as dehydrating agent as it removes 38. (d) All these are characteristics noted during the
moisture from atmosphere. process.
31. (d) (Me)2 SiCl2 on hydrolysis will yield (Me)2 Si (OH)2. 39. (d) Ionisation enthalpy (Dt Hi in kJ mol-1) for the elements
32. (a) GeCl4 in HCl forms H2[GeCl6] × Ge(OH)2 is of group-13 follows the order
amphoteric.SnCl2 is more stable than SnCl4. B > Al < Ga > In < Tl
Trisilyl amine is planar molecule.
40. (c) P and Q are boron and silicon respectively.
33. (b) Sheet silicates are formed when three oxygen atoms
41. (d) Due to inert pair effect, Pb2+ is most stable.
(bridging O-atoms) of each (SiO4 )4- unit are shared. .
42. (a) The stability order for boron halides is explained in
34. (a) Two most abundant elements in the earth’s crust are terms of back-bonding.
Si and oxygen and the compound made by them is SiO2.
This compound is used in building construction. 43. (c) It is a reason for given fact.
SiO2 + 2C ¾® Si + 2CO 44. (c) CO is neutral and CO2 is acidic.
Poisonous
35. (c) 45. (a) Al2O3 + 3Cl2(dry) + 3C ¾® 2AlCl3 + 3CO

BITSAT Archives
1. (b) Borax is Na 2B4O7 × 10H2O in which 2 molecules of 2. (a) The tendency to behave as Lewis acid decreases with
water among 10 molecules form a part of structure and the increase in the size down the group. BCl3 easily
exists as Na 2 [B4O5(OH)4] × 8H2O. accepts a lone pair of electrons from ammonia to form
BCl3 × NH3. Hence, strength as Lewis acid
Methyl orange with pH value of 3.7 is used to detect end
BCl3 > AlCl3 > GaCl3
point. Aqueous solution of borax acts as buffer because
borax is salt of strong base (NaOH) and weak acid 3. (b) Flint glass or lead glass has composition of
(H3BO3). K 2O × PbO × 6SiO2. It is used in making electric bulb and
optical instruments

@iitjeehelps
18
p-Block Elements-II

Group 15 Elements
Group 15 (VA) contains 5 elements, namely nitrogen (N), phosphorus (P), arsenic (As), antimony (Sb),
bismuth (Bi), collectively called as pnicogens taken from Greek word ‘pniomigs’ meaning
suffocating.

Physical Properties
(i) Electronic configuration The general electronic configuration of group 15 is ns 2np3 .
(ii) Atomic and ionic radius Atomic and ionic radius increase down the group, however arsenic
shows exceptionally low value of atomic volume.
(iii) Ionisation energy Ionisation energy of these elements is much higher due to increased
nuclear charge and stable exactly half-filled electronic configuration.
(iv) Electronegativity On moving down the group, it decreases due to gradual increase in the
atomic radius of the elements.
(v) Non-metallic/metallic character On moving down the group, metallic character increases.
(vi) Allotropy All these elements except bismuth show allotropy.
(vii) Oxidation state Elements of group VA exhibit maximum oxidation state of + 5 by losing all
the five electrons from their outer shell. + 3 oxidation state is shown only when p-orbital
electrons are used in bonding.
On moving down the group, + 3 oxidation state becomes more stable due to inert pair effect.
Nitrogen shows +5 to −3 oxidation states.

@iitjeehelps
p-BLOCK ELEMENTS-II 569

Chemical Properties Properties of Oxides


Property Gradation Reason
Some of the important chemical properties of group 15
Acidic strength N 2O3 > P2O3 > Electronegativity of central
elements are as follows : of trioxides As 2O3 atom decreases.
(i) Hydrides General formula of hydrides of these Acidic strength N 2O5 > P2O5 > As 2O5 Electronegativity of central
elements is MH3 , e.g. NH3 , PH3 , AsH3 , SbH3 and of pentoxides > Sb 2O5 > Bi 2O5 atom decreases.
BiH3 . All these hydrides are covalent in nature and Acidic strength N 2O < NO < N 2O3 Oxidation state of central
have pyramidal structure (sp3 -hybridised). of oxides of atom increases.
nitrogen < N 2O4 < N 2O5
Properties of Hydrides
Stability of P2O5 > As 2O5 > Sb 2O5 Stability of oxides of a higher
Down the pentoxides > N 2O5 > Bi 2O5 oxidation state, i.e. M 2O5
Property Reason
group decreases with increasing
Bond angle of Decreases As the size of central atom increases, atomic number.
MH 3 electronegativity decreases, thus
repulsion of bond pair and lone pair of
central atom decreases. Dinitrogen
Basic strength Decreases As the size of central atom increases,
of MH 3 electron density decreases. Preparation
NH 4Cl(aq ) + NaNO2 (aq ) → N 2 ( g ) + 2H 2O(l ) + NaCl(aq )
Thermal Decreases As the size of the central atom
Heat
stability of MH 3 increases, its tendency to form stable (NH 4 )2 Cr2O7 → N 2 + 4H 2O + Cr2O3
M H bond decreases.
Ba(N3 )2 → Ba + 3N 2
Reducing Increases As the stability of hydrides decreases,
character the reducing character increases. Properties
Melting and Increases NH 3 has higher melting point and (i) Nitrogen does not react with alkali metals except Li
boiling point (except in boiling point than PH 3 due to but reacts with alkaline earth metals to give metal
N) hydrogen bonding. As the molecular
size increases, van der Waals’ forces
nitrides.
increase. Heat
6Li + N 2 → 2Li3 N
(ii) Halides Elements of group VA form two types of Heat
halides, i.e. trihalides and pentahalides. Trihalides 3Mg + N 2 → Mg3 N 2
are mainly basic (Lewis bases) in nature and have 2000 K
(ii) N 2 ( g ) + O2 ( g ) → 2NO( g )
lone pair of electrons (central atom is
sp3 -hybridised) with pyramidal shape. Heat
(iii) 2B + N 2 → 2BN
Pentahalides are sp3d -hybridised and trigonal
bipyramidal in shape. (iv) CaC 2 + N 2 →
1273 K
CaCN 2 + C
Properties of Halides
Uses
Property Gradation Reason
(i) Liquid N 2 is used as refrigerant.
Stability of NF3 > NCl 3 > NBr3 Large size difference between N
trihalides of and the halogens
(ii) It is used for filling electric bulbs.
nitrogen
Lewis base NF3 < NCl 3 < NBr3 Decreasing electronegativity of Phosphorus
strength < NI 3 halogens.
Preparation
Bond angle PF3 < PCl 3 < PBr3 Due to decreased bond pair-bond
among the < PI 3 pair repulsion as these move away (i) Retort process
halides of from P due to increased
Ca3 (PO4 )2 + 3H 2SO4 → 2H PO
3 4
+ 3CaSO4
phosphorus electronegativity of X
Phosphorite Orthophosphoric acid
( X = F, Cl, Br, I)
H3PO4 → HPO 3
+ H 2O
(iii) Oxides All the elements of group VA form two Metaphosphoric acid
types of oxides, i.e. M 2O3 and M 2O5 and are called
trioxides and pentoxides. 4HPO3 + 10C → P4 + 10CO + 2H 2O

@iitjeehelps
570 SELF STUDY GUIDE BITSAT

(ii) Electrothermal process Phosphorus is Important Compounds of N and P


obtained from direct reduction of mineral
phosphorite by carbon in the presence of silica. Some important compounds of N and P are as follows :

2Ca3 (PO4 )2 + 6SiO2 + 10C →


1400-1500 °C Ammonia
6CaSiO3 + P4 + 10CO Preparation
It is prepared by Haber’s process.
Allotropy of Phosphorus Fe + Mo
N 2 ( g ) + 3H 2 ( g ) s 2NH3 ( g )
Black phosphorus ←
470 K
P4 750 K
Under pressure White phosphorus 200-300 atm

560 K, inert
atmosphere
Properties
Red phosphorus The main properties of ammonia are as follows :
(i) It is a light colourless gas with characteristic pungent
(i) White phosphorus is transparent, soft, smell and highly soluble in water.
poisonous, waxy solid, shows
chemiluminescence and chemically more (ii) 2NH3 Electric arc
→ N 2 + 3H 2
773 K
reactive.
l P4 + 5O2 → P4O10 or 2 P2O 5 (iii) Ammonia has a strong tendency to donate its lone
pair of electrons to other molecules and thus acts as a
l P4 + 3NaOH + 3H 2O → 3NaH 2PO2 + PH3 strong Lewis base.
Sodium Phosphine
hypophosphite (iv) It reacts with Nessler’s reagent (alkaline K 2HgI 4 ).
l 6Mg + P4 → 2Mg3P2 2K 2HgI 4 + NH3 + 3KOH → H 2 N ⋅ HgO ⋅ HgI
Magnesium phosphide
Iodide of Millon’ s
base (Brown ppt.)
l It acts as a strong reducing agent.
P4 + 10H 2SO4 → 4H3PO4 + 10SO2 + 4H 2O + 7KI + 2H 2O
(ii) Red phosphorus is the stable form of Nitric Acid or Aqua Fortis
phosphorus. It is odourless, non-poisonous and
less reactive. Preparation
l 2P + 5Cl 2 →
Heat
2PCl 5 It is prepared by Ostwald’s process.
Pt / Rh gauge catalyst
l 2P + 3S →
Heat
P2S3 4NH3 ( g ) + 5O2 ( g ) → 4NO( g ) + 6H 2O( g )
500 K, 9 bar
P + 3Na → Na3P
2NO( g ) + O2 ( g ) q
l
2NO2 ( g )
l It is a polymer consists of chains of P4
tetrahedra linked together. 3NO2 ( g ) + H 2O(l ) → 2HNO3 (aq ) + NO( g )
(iii) Black phosphorus It is thermodynamically most Properties
stable at room temperature and has two forms :
The main properties of nitric acid are as follows:
(a) α-black phosphorus It is very stable form
(a) Pure anhydrous HNO3 is a colourless, water soluble
and does not oxidise unless heated very
pungent smelling, fuming liquid
strongly.
(boiling point– 84.1°C).
(b) β-black phosphorus It is the only form of P
(b) On exposure to light, HNO3 decomposes to NO2
whose structure is definitely known
which imparts yellow colour.
(crystalline).
(c) It is extremely corrosive in nature.
(iv) Scarlet phosphorus It resembles the red variety
in its physical properties and white P in its (d) In aqueous solution, nitric acid behaves as a strong
chemical properties. acid giving H3O+ and NO3− ions.
(v) Violet phosphorus It is crystalline in nature. (e) Conc. HNO3 is a strong oxidising agent.
2HNO3 + 2HBr → Br2 + 2NO2 + 2H 2O
Uses
2HNO3 + 2HI → I 2 + 2NO2 + 2H 2O
Red P is widely used in the match industry because of its
less reactivity and non-poisonous nature, while yellow P (f) Hot conc. HNO3 attacks non-metals also.
and zinc phosphide are used as a rat poison. 32 P is used in 4HNO3 + C → H 2CO3 + 4NO2 +H 2O
the treatment of leukemia and other blood disorders. 10HNO3 + 2P → 2H3PO4 + 10NO2 + 2H 2O

@iitjeehelps
p-BLOCK ELEMENTS-II 571
Phosphine (b) Upon heating, it sublimes below 173 K and can be
melted (melting point = 421 K) by heating under
Preparation pressure.
It is prepared in laboratory by the following reaction: (c) It has the ability to dissociate as
P4 + 3NaOH (conc. ) + 3H 2O → PH3 + 3NaH 2PO2 PCl 5 q PCl3 + Cl 2
Sodium (d) In moist air, it hydrolyses to POCl3 and finally gets
hypophosphite
converted to H3PO4 .
Properties PCl 5 + H 2O → POCl3 + 2HCl
The main properties of phosphine are as follows: POCl3 + 3H 2O → H3PO4 + 3HCl
(a) It is a colourless, poisonous garlic or rotten fish like Structure
smelling gas.
X-ray studies show that solid PCl 5 consists of ionic lattices,
(b) It is weakly basic like ammonia, gives i.e. tetrahedral [PCl 4 ]+ cations and octahedral [PCl6 ]−
phosphonium compounds with acids.
anions. In vapour state, it has trigonal bipyramidal shape
PH3 + HBr → PH 4Br
(sp3 -hybridisation).
(c) Pure PH3 does not burn spontaneously. However,
on heating in air or oxygen forms P2O5. Cl Cl Cl
150 °C Cl Cl Cl
2PH3 + 4O2 → P2O5 + 3H 2O P P Cl P
Cl Cl Cl Cl Cl
Phosphorus Trichloride
Cl Cl Cl
Preparation Tetrachlorophosphonium Hexachlorophosphate Vaporised phosphorus
(Tetrahedral cation) (Octahedral anion) pentachloride
PCl3 is obtained by passing dry chlorine over heated
white P.
Oxides of Nitrogen
P4 + 6Cl 2 → 4PCl3 (i) NH 4NO3 Heat
→ N 2O + H 2O
Properties N 2O( g ) is neutral, colourless gas, with sp hybridisation
and linear geometry (it is also called laughing gas).
Following are the properties of PCl3 :
(ii) 2 NaNO 2 + 2 FeSO 4 + 3 H 2SO 4 → Fe 2 (SO 4 ) 3
(a) PCl3 is a colourless mobile liquid (boiling point
+ 2 NaHSO 4 + 2 H 2O + 2 NO
= 349 K, freezing point = 161 K; density = 1.6 g/cm3 )
with a pungent smell. It fumes in moist air. NO( g ) is colourless, neutral gas. It also contains odd
number of electrons (paramagnetic) but in solid or
(b) It acts as a reducing agent. liquid state, it exists in dimeric form and have paired
3PCl3 + S2Cl 2 → PCl 5 + 2PSCl3 electrons (diamagnetic). It is very reactive and
3PCl3 + SOCl 2 → PCl 5 + POCl3 + PSCl3 harmful to health.
(c) It reacts with conc. H 2SO4 . 250 K
(iii) 2NO + N 2O4 → 2N 2O3
4H 2SO4 + 2PCl3 →
N 2O3 is blue solid. It is acidic and planar with
2HSO3 Cl + P2O5 + 2 SO2 + 4HCl + H 2O sp 2 hybridisation.
Chlorosulphonic
acid (iv) 2Pb(NO3 )2 673
 K
→ 4NO2 + 2PbO
Phosphorus Pentachloride (PCl 5 ) NO 2 ( g ) is brown, acidic gas with angular shape and
sp 2 hybridisation. It contains odd number of valence
Preparation
electrons (paramagnetic). On dimerisation, it gets
PCl 5 is prepared by the reaction of PCl3 with excess of dry
converted to stable N 2O 4 molecule with even number
chlorine of white phosphorus with SO2Cl 2 .
of electrons (diamagnetic).
P4 + 10SO2Cl 2 → 4PCl 5 + 10SO2 Cool
(v) 2NO2 q N 2O4
Properties Heat

Following are the main properties of PCl 5 : N 2O 4 is colourless solid/liquid. It is acidic with planar
geometry and sp 2 hybridisation. Its covalency is four
(a) PCl 5 is a colourless crystalline solid with pungent
(total number of bonds with central atom).
odour when pure.

@iitjeehelps
572 SELF STUDY GUIDE BITSAT

(vi) 4HNO3 + P4O10 → 4HPO3 + 2N 2O5


N 2O 5 is colourless, acidic solid with planar shape and
Group 16 Elements
sp 2 hybridisation and four covalency. The group VI A contains 5 elements, i.e. oxygen, sulphur,
selenium, tellurium and polonium and known as oxygen
Structure of Oxides and family.
Oxoacids of Phosphorus The first four members are non-metals and collectively
O
known as chalcogens (metal ores mainly occur in the form
143 pm of oxide, sulphides etc). The outermost shell electronic
P
O
P configuration is ns 2 , np 4 (n = number of shells).
O O 102° O
123°
O O 123°
P
O 100°
P O P
O
P O Physical Properties
O O O Group 16 elements show following trends in their physical
O
P 0p
m P m
16 16 0p properties :
P4O6 (i) Metallic character On moving down the group, the
O
P4O10 metallic character increases down the group from
The structures of some important oxyacids of phosphorus oxygen to polonium because of decrease in ionisation
are summarised below: energy.
O (ii) Oxidation states Group VIA elements show oxidation
state of + 2 , + 4 and + 6 due to the promotion of
P electrons to vacant d-orbitals. But down the group, +4
–H2O oxidation state becomes more stable.
HO O
Metaphosphoric acid
HPO3 (P=+5) (monobasic)
Chemical Properties
O O O
Following are the main chemical properties of group 16
Two molecules
P –H2O P P elements :
HO OH HO O OH (i) Hydrides All the elements of the oxygen family form
OH OH OH
stable hydrides of the type H 2 M either by directly
Orthophosphoric acid Diphosphoric acid
(or pyrophosphoric acid) combining with hydrogen or by the action of acids on
H3PO4 (P=+5) (tribasic)
H4P2O7 (P=+5) (tetrabasic) metal sulphides, selenides and tellurides.
–[O]
+[O] 2H 2 + O2 q 2H 2O
O O O FeS + H 2SO4 → H 2S + FeSO4
P P H 2O is a liquid due to hydrogen bonding and others
P are colourless gases with unpleasant smell.
HO O  OH HO OH OH OH
OH Compounds H 2O > H 2S > H 2Se > H 2Te
Peroxomonophosphoric Hypophosphoric acid (all sp3 -hybridised)
acid H4P2O6 (P=+4)
H3PO5 (P=+7) (dibasic) (tetrabasic) Bond angle 104.5° 92.5° 91° 90°
The decreasing bond dissociation enthalpy of the
O O O M Η bond is due to increase in the size of M which
Two molecules
P P P explains the increasing acidic character of hydrides
–H2O
O down the group.
OH H
H OH OH OH H (ii) Halides All the elements of the oxygen family form a
Phosphorous acid Pyrophosphorous acid
H3PO3 (P=+3) (dibasic) H4P2O5 (P=+3)
number of halides like OF2 , ClO2 , S2F2 , SF4 , etc.
(dibasic) (iii) Oxides These form oxides of the type MO2 and MO3 .
e.g. SO2 , TeO2 , etc.
–[O] O

P Anomalous Behaviour of Oxygen


H OH Oxygen, the first member of group 16 differs from rest of the
H
Hypophosphorous acid members due to its small size, high electronegativity and
H3PO2 (P=+1)
(monobasic) non-availability of d-orbitals in the valence shell.

@iitjeehelps
p-BLOCK ELEMENTS-II 573
(i) Oxygen exhibits an oxidation state of − 2 only except in l It acts as a powerful oxidising agent. It liberates
O2F2 and OF2 where it has +1 and + 2 state and in iodine from neutral KI solution and the liberated I 2
peroxides where it has − 1 state whereas other members turns starch paper blue.
show +4 and + 6 oxidation states in addition to + 2 2KI + H 2O + O3 → 2KOH + I 2 + O2
oxidation state.
I 2 + starch → Blue colour
(ii) Oxygen is capable of forming pπ- pπ bonds. The other l Alkaline KI is oxidised to potassium iodate and
elements of this group do not show the tendency to periodate.
exhibit such multiple bonding, sulphur and phosphorus
KI + 3O3 → KIO3 + 3O2
show pπ -dπ bonding. Potassium iodate

KI + 4O3 → KIO4 + 4O2


Important Compounds of O and S Potassium periodate
Some important compounds of oxygen and sulphur are : H 2S + O3 → H 2O + S + O2
Dioxygen Mercury loses its meniscus in contact with ozone
(tailing of mercury).
Preparation
2Hg + O3 → Hg 2O + O2
MnO
(i) Laboratory method 2KClO3 →
2
2KCl + 3O2
∆ Allotropic forms of Sulphur
(ii) By electrolysis of water (Acidified water) It exists in several allotropic forms.
+
H 2SO4 q 2H + SO24− Rhombic Sulphur (α-sulphur)
+ −
At cathode 2H + 2e → H 2 It is common crystalline form of sulphur, yellow in
1 colour, melting point 114.5°C and specific gravity 2.06.
At anode 2OH − → H 2O + O2 + 2e − Its crystals are prepared by evaporating sulphur
2
solution in CS2 . It is insoluble in water but readily
Properties soluble in CS2 .
(i) Colourless, odourless, tasteless gas which is slightly Monoclinic Sulphur (β-sulphur)
soluble in water. Liquid oxygen exhibits paramagnetism.
It is stable above 95.6°C. Its crystals are amber yellow in
(ii) It is non-inflammable but a supporter of combustion. colour, melting point 119°C, specific gravity 1.98. It is
(iii) O == O bond dissociation energy is high, therefore it insoluble in CS2 . It is prepared by melting rhombic
reacts with metals or non-metals after external heating to sulphur in a dish followed by cooling till crust is formed.
start the reaction. 95.6° C
Rhombic sulphur q Monoclinic sulphur
3300 K
N 2 + O2 → 2NO
At 95.6°C, both forms are stable. This temperature is
Room temp.
4Na(s ) + O2 ( g ) → 2Na 2O(s ) called transition temperature. S8 rings in both the forms
is puckered and has a crown shape.
4Al (s ) + 3O2 ( g ) → 2Al 2O3 (s )
Plastic Sulphur
Ozone It is obtained by pouring boiling sulphur into cold water.
It is formed in the upper layer of atmosphere by the action of It is amber brown in colour, specific gravity 1.95 and
UV rays from sun on oxygen. It prevents the UV rays from insoluble in CS2 .
entering the earth’s atmosphere. CFCs, common refrigerants
deplete the ozone layer. Milk of Sulphur
It is obtained by boiling milk of lime with sulphur and
Preparation
decomposing the products formed with HCl. It is used in
Silent electric discharge
3O2 ( g ) 2O3 ( g ) medicines.
Colloidal Sulphur
Properties It is prepared by passing H 2S through a solution of an
oxidising agent such as nitric acid, etc.
l It is pale blue gas with pungent odour. It is diamagnetic
and poisonous. 2HNO3 + H 2S → 2NO2 + 2H 2O + S
Colloidal sulphur changes into ordinary form on
l Ozone is unstable; 2O3 →
Heat
3O2 heating.

@iitjeehelps
574 SELF STUDY GUIDE BITSAT

Sulphuric Acid (ii) Thiosulphuric acid (H 2S2O3 )


It is called as ‘oil of vitriol’ because in early days, it was O O
prepared from ferrous sulphate crystals (green vitriol) and
has an oily appearance. H O S O H or H O S S H
Preparation S O
Industrial preparations of sulphuric acid are as follows : Reducing agent

(a) Contact process


(iii) Pyrosulphuric acid (H 2S2O7 )
V2O 5 / 700 K
2SO2 + O2 → 2SO3 O
2-3 atm O
SO3 is absorbed in 98% H 2SO4 to get oleum. HO S O S OH
SO3 + H 2SO4 → H 2S2O7
Oleum
O O
Strong oxidising agent
H 2S2O7 + H 2O → 2H 2SO4
(b) Lead chamber process (iv) Peroxomonosulphuric acid (H 2SO5 )
2SO2 + O2 + H 2O + [NO] → 2H 2SO4 + [NO]
Air Catalyst Catalyst O

Properties HO S O OH
The main properties of sulphuric acid are as follows : O
(a) Pure sulphuric acid is colourless but commercial (Caro’s acid)

acid is yellow in colour due to the presence of Strong oxidising agent


impurities.
(b) H 2SO4 has associated structure due to
intermolecular hydrogen bonding which accounts Group 17 Elements
for its high boiling point and viscosity.
Group VIIA of long form of periodic table consists of the
O O H O OH
Hydrogen non-metallic elements, fluorine (F), chlorine (Cl), bromine
S bonds S
(Br), iodine (I) and astatine (At). These non-metallic
O O H O OH elements are collectively known as halogens as their salts
(c) Conc. H 2SO4 acts as a dehydrating agent. are usually found in water (Greek word-Halo means sea
C12H 22O11 +H 2SO4 → 12C + 11H 2O salt producer).
(Conc.)

The residue of carbon is called sugar charcoal. Physical Properties


(i) Electronic configuration The outermost shell
Uses electronic configuration is ns 2 , np 5.
Sulphuric acid is used (where, n = number of shells)
(a) as a dehydrating agent and in petroleum refining. (ii) Electronegativity The halogens are the most
(b) for pickling, i.e. cleaning of metal surface before electronegative elements. On moving down the group,
electroplating, in making lead storage batteries. the electronegativity decreases.
F > Cl > Br > I
Structure of Oxoacids of Sulphur (iii) Oxidation state All these elements show an oxidation
Sulphur shows following oxyacids: state of −1 . Except fluorine, other elements also show
(i) Sulphuric acid (H 2SO4 ) +1, +3, +5 and +7 oxidation states due to the presence of
O vacant d-orbitals in their valence shell.
(iv) Non-metallic character All these elements are
HO S OH non-metallic in nature due to their high ionisation
energy and high electronegativities. On moving down
O
the group, the non-metallic character gradually
Strong dehydrating agent
decreases.

@iitjeehelps
p-BLOCK ELEMENTS-II 575

Chemical Properties Examples of Pseudohalogens and Their Formulae

(i) Oxidising power On moving down the group, the Pseudohalide ions Formulae Pseudohalogens Formulae
oxidising power decreases from F to I, i.e. Cyanide CN − Cyanogen (CN) 2

F2 > Cl 2 > Br2 > I 2 Cyanate OCN Oxocyanogen (OCN) 2
– – −
Thus, I is the strongest reducing agent, while F is Thiocyanate SCN Thiocyanogen (SCN) 2
the strongest oxidising agent. Selenocyanate SeCN − Selenocyanogen (SeCN) 2
(ii) Reaction with oxygen Halogens form binary Azidothiocarbonate SCSN –3 Azidocarbon (SCSN 3 ) 2
compounds with oxygen but most of them are disulphide
unstable. Decreasing order of stability of oxides
formed by halogens is
I > Cl > Br
Anomalous Behaviour of Fluorine
Oxides of Halogens Fluorine differs from the rest of the halogens because of its
small size, high charge density, non-availability of d-orbitals
Oxides of fluorine OF2 and O2 F2 and low bond dissociation energy.
Oxides of chlorine Cl 2 O, ClO2 , Cl 2 O6 , Cl 2 O7 Important unique behaviour of fluorine is given below :
Oxides of bromine Br2 O, BrO2 , BrO3 (i) F is very reactive because of its low bond dissociation
Oxides of iodine I 2 O5
energy. However, in Cl 2 and Br2 , XX bond is stronger.
(ii) HF has a high tendency to form H-bond. Thus, it is a
(iii) Reaction with metals Halogens react with metals
liquid (boiling point = 19°C) while HCl, HBr and HI are
to form metal halides. The ionic character of the
gases under ordinary conditions.
halides decreases in the order
(iii) Hydrofluoric acid being a dibasic acid (H 2F2 ) forms two
MF > MCl > MBr > M I (M = monovalent metal) series of salt such as NaHF2 , Na 2F2 while HCl, HBr and HI
(iv) Reaction with hydrogen Halogens react with are monobasic in nature.
hydrogen to form hydrogen halides. Their affinity
for hydrogen decreases from F to I.
Chlorine
HF < HCl < HBr < HI → Acidic strength and
It was discovered in 1774 by Scheele by the action of HCl on
reducing nature MnO2 .
increases
MnO2 + 4HCl → MnCl 2 + Cl 2 + 2H 2O
HF > HCl > HBr > HI → Stability and ionic
nature Manufacturing of Chlorine
decreases Chlorine can be manufactured by the following two processes:
HCl < HBr < HI < HF → Boiling point increases
Deacon’s Process
(v) Interhalogen compounds Halogens react with
CuCl 2
themselves to produce a number of interhalogen 4HCl + O2 → 2Cl 2 + 2H 2O
compounds (XX n , where n = 1, 3, 5 or 7). 273 K

Examples of Interhalogen Compounds Electrolytic Process


Electrolysis
XX ′ ClF, BrF, BrCl, ICl, IBr, IF 2NaCl+ 2H 2O → 2NaOH + Cl 2 ( g ) + H 2 ( g )
Brine (At anode) (At cathode)
XX 3′ ClF3 , BrF3 , IF3 , ICl 3 (I 2Cl 6 )
It is also obtained as a by-product in many chemical reactions.
XX 5′ ClF5 , BrF5 , IF5
Properties of Chlorine
XX 7′ IF7
(i) It is a greenish yellow gas with pungent and suffocating
(vi) Pseudo halogens There are many uni-negative odour which is about 2-5 times heavier than air. It is
groups which show characteristics of halide ions. highly soluble in water.
These are called pseudohalides or pseudohalide (ii) Chlorine reacts with a number of metals and non-metals
ions. The covalent dimers of pseudohalide ions are to form chlorides.
called pseudohalogens. 2Na + Cl 2 → 2NaCl
NOTE Pseudohalide ions, being made up of two heteroatoms 2Al + 3Cl 2 → 2AlCl3
can function as ambidentate ligands.

@iitjeehelps
576 SELF STUDY GUIDE BITSAT

(iii) Chlorine reacts with compounds containing Properties of HCl


hydrogen, it has great affinity for hydrogen. (i) It is a colourless and pungent smelling gas which is
8 NH3 + 3Cl 2 → 6NH 4Cl + N 2 extremely soluble in water.
(Excess)
(ii) Its boiling point is 189 K and freezing point is 159 K.
NH3 + 3Cl 2 → NCl3 + 3HCl
(Excess) (iii) It gives white fumes of NH 4Cl on reaction with NH3 .
(iv) I. 2NaOH + Cl 2 → NaCl + NaOCl + H 2O NH3 + HCl → NH 4Cl (white fumes)
(Cold, dilute) Sodium (iv) It decomposes salts of weaker acids as:
hypochlorite
Na 2CO3 + 2HCl → 2NaCl + H 2O + CO2
II. 6NaOH + 3Cl 2 → 5NaCl + NaClO3 + 3H 2O
(Hot, conc.) Sodium NaHCO3 + HCl → NaCl + H 2O + CO2
chlorate
(v) 2Ca(OH)2 + 2Cl 2 → Ca(OCl)2 + CaCl 2 + 2H 2O When three parts of conc. HCl are mixed with one part of
conc. HNO3 , aqua-regia is formed. It dissolves noble metals,
The composition of bleaching powder is i.e. Au,Pt.
Ca(OCl)2 ⋅ CaCl 2 ⋅ Ca(OH)2 ⋅ 2H 2O. Au + 4H + + NO3− + 4Cl − → AuCl −4 + NO + 2H 2O
(vi) Chlorine water on standing loses its yellow colour
3Pt + 16H + + 4NO3− + 18Cl − → 3[PtCl6 ]2 − + 4NO + 8H 2O
due to the formation of HCl and HOCl.
Cl 2 + H 2O → HCl + HOCl Uses of HCl
HOCl → HCl + [O] (i) In the manufacturing of chlorine, NH 4Cl and glucose
Nascent oxygen
(from corn starch).
Nascent oxygen formed by HOCl is responsible for (ii) In medicines and as a laboratory reagent.
oxidising and bleaching properties of chlorine. (iii) For extracting glue from bones and purifying bone
Coloured substance + [O] → Colourless substance black.
(vii) It oxidises ferrous to ferric and sulphite to sulphate, Oxoacids of halogens
sulphur dioxide to sulphur trioxide and iodine to
Fluorine forms only one oxoacid HOF, known as fluoric (I)
iodate.
acid or hypofluorous acid due to high electronegativity and
2FeSO4 + H 2SO4 + Cl 2 → Fe 2 (SO4 ) 3 + 2HCl small size. Chlorine, bromine and iodine form four series of
Na 2SO3 + Cl 2 + H 2O → Na 2SO4 + 2HCl oxoacid. They are stable only in aqueous solutions or in the
form of their salts.
Uses of Chlorine
Oxoacids of Halogens
Some important uses of chlorine are as follows :
Halic (I) acid HOF HOCl HOBr HOI
(i) In the manufacturing of dyes, drugs and some (Hypohalous (Hypofluo- (Hypochlo- (Hypobro- (Hypoio-
acid) rous acid) rous acid) mous acid) dous acid)
organic compounds like DDT, carbon tetrachloride,
Halic (III) acid — HOClO — —
refrigerants, etc.
(Halous acid) (Chlorous
(ii) In the preparation of poisonous gases such as acid)
phosgene (COCl 2 ) , tear gas (CCl3 NO2 ) and mustard Halic (V) acid — HOClO2 HOBrO2 HOIO2
gas (ClCH 2CH 2SCH 2CH 2Cl). (Halic acid) (Chloric (Bromic (Iodic acid)
acid) acid)
Compounds of Halogens Halic (VII) acid — HOClO3 HOBrO3 HOIO3
(Perhalic acid) (Perchloric (Perbromic (Periodic
Hydrogen Chloride (HCl) acid) acid) acid)
It bleaches vegetable or organic matter in the presence of
moisture. Bleaching effect of chlorine is permanent. Structures of Oxoacids of Chlorine
Preparation of HCl O O O
In laboratory, it is prepared by heating NaCl with H Cl H Cl
Hypochlorous acid Chlorous acid
conc. H 2SO4 .
O O O O
NaCl +H 2SO4 →
420 K
NaHSO4 + HCl
H Cl H Cl O
NaHSO4 + NaCl →
823 K
Na 2SO4 +HCl O O
Chloric acid Perchloric acid

@iitjeehelps
p-BLOCK ELEMENTS-II 577
Acidic strength increases as oxidation number increases. Chemical Properties
HOCl < HClO2 < HClO3 < HClO4 In general, noble gases are not very reactive.
HClO4 is the strongest acid for the same oxidation number Their inertness to chemical reactivity is attributed to the
of halide. following reasons :
HClO4 > HBrO4 > HIO4 (i) The noble gases have completely filled ns 2np6
The oxidising power of oxoacids of chlorine follows the electronic configurations in their valence shells.
order (ii) The noble gases have very high ionisation energies.
HClO4 < HClO3 < HClO2 < HClO (iii) The electron affinities of noble gases are almost zero.
It is because with the increase in the oxidation number of
halogen atom in oxoacids, oxidising power decreases. Xenon-Fluorine Compounds
Xenon forms three binary fluorides XeF2 , XeF4 and XeF6 as
follows :
Group 18 Elements 673 K
Xe( g ) + F2 ( g ) → XeF2 (s )
The zero (or 18) group of the periodic table consists of six (In excess) 1 bar pressure
gaseous elements namely helium (He), neon (Ne), 873 K
argon (Ar), krypton (Kr), xenon (Xe) and radon (Rn). Except Xe( g ) + 2F2 ( g ) → XeF4 (s )
7 bar pressure
(1 : 5 ratio)
radon, all of them are present in very small amounts in the
573 K
atmosphere, hence they are also known as rare gases or Xe( g ) + 3F2 ( g ) → XeF6 (s )
- pressure
60 70 bar
noble gases. (1 : 20 ratio)
14 3 K
Physical Properties XeF4 + O2F2 → XeF6 + O2
(i) Electronic configuration These gases have highly
Properties of Xenon-Fluorine Compounds
stable ns 2np6 configuration.
All fluorides of xenon are colourless crystalline solids and
(ii) Boiling points On moving down the group, the boiling sublime readily at 298 K. They are powerful fluorinating
point increases with increase in size. agents. These are readily hydrolysed even by traces of
(iii) Ionisation energy and electron affinity These gases water.
have stable ns 2np6 (fully-filled) configuration, thus e.g. 2XeF2 (s ) + 2H 2O(l ) → 2Xe( g ) + 4HF (aq ) + O2 ( g )
have no tendency to add or lose electron. Hence, their Xenon fluorides may react with fluoride ion acceptors to
ionisation energy is very high and electron affinity is form cationic species or fluoride ion donors to form fluoro
zero. anions.
(iv) Heat of vaporisation and polarisability These gases XeF2 + PF5 → [XeF]+ + [PF6 ]−
possess very low values of heat of vaporisation due to XeF4 + SbF5 → [XeF3 ]+ + [SbF6 ]−
the presence of very weak van der Waals’ forces XeF6 + M F → M +[XeF7 ]−
between their monoatomic molecules.
(M = Na, K, Rb or Cs)
(v) Solubility Noble gases are slightly soluble in water
and their solubility generally increases on moving Xenon-Oxygen Compounds
down the group. Hydrolysis of XeF4 and XeF6 with water gives XeO3 .
(vi) Adsorption Except helium, all the noble gases are 6XeF4 + 12H 2O → 4Xe + 2XeO3 + 24HF + 3O2
adsorbed by charcoal and the ease of adsorption XeF6 + 3H 2O → XeO3 + 6HF
increases down the group.
Partial hydrolysis of XeF6 gives oxyfluorides XeOF4 and
(vii) Liquefication Ease of liquefication increases down the XeO2F2 .
group from He to Rn due to increase in intermolecular XeF6 + H 2O → XeOF4 + 2HF
forces.
XeF6 + 2H 2O → XeO2F2 + 4HF
(viii) Conductivity Noble gases have high electrical XeO3 is a colourless explosive solid and has a pyramidal
conductivity at low pressure. molecular structure. XeOF4 is a colourless volatile liquid
(ix) These gases are monoatomic gases (C p / CV = 1.667) . and has a square pyramidal molecular structure.

@iitjeehelps
Practice Exercise
1. When conc. HNO3 is heated with P2O5 , it forms 14. Maximum covalency of nitrogen is
a. N2O b. NO c. NO2 d. N2O5 a. 3 b. 5 c. 4 d. 6

2. A hydride of nitrogen which is acidic, is 15. Which of the following statements is wrong?
a. N N single bond is stronger than the P  P single
a. NH3 b. N2H4 c. N2H2 d. N3H
bond
3. Extra pure N 2 can be obtained by heating b. PH3 can act as a ligand in the formation of
coordination compound with transition elements
a. NH3 with CuO b. NH4NO3
c. NO2 is paramagnetic in nature
c. (NH4 )2 Cr2O7 d. Ba(N3 )2
d. Covalency of nitrogen in N2O5 is four
4. The number of σ bonds in P4 O10 is
16. Which of the following elements can be involved in
a. 6 b. 16 c. 20 d. 7 pπ-dπ bonding?
5. PH3 , the hydride of phosphorus is a. Carbon b. Nitrogen
c. Phosphorus d. Boron
a. metallic b. ionic
c. non-metallic d. covalent 17. Bond dissociation enthalpy of E  H (E =element)
6. Among the following compounds, the most acidic is bonds is given below.
a. As2O3 b. P2O5 c. Sb2O3 d. Bi2O3 Compound NH 3 PH 3 AsH 3 SbH 3
7. Among the following, the number of compounds that ∆ diss(E  H) /kJ mol −1 389 322 297 255
can react with PCl 5 to give POCl3 , is
O2, CO2, SO2, H 2O, H 2SO4 , P4 O10 Which of the following compounds will act as strongest
a. 1 b. 2 c. 3 d. 4 reducing agent?
a. NH3 b. PH3 c. AsH3 d. SbH3
8. Nitrogen shows different oxidation states in the range
a. 0 to +5 b. −3 to +5 18. Density of N2 gas prepared from air is slightly higher
c. −5 to +3 d. −3 to +3 than that of nitrogen prepared by chemical reaction
9. Which of the following acids possess oxidising, from a compound of nitrogen because aerial nitrogen
reducing and complex forming properties? contains
a. HCl b. HNO2 c. H2SO4 d. HNO3 a. CO2
b. argon
10. When conc. H 2SO4 is heated with P2O5 , the acid is c. some N 2 molecules analogous to O2
converted to
d. greater amount of N2 molecules derived from
a. sulphur trioxide
b. sulphur dioxide N15 isotope
c. sulphur 19. Which of the following statements is incorrect?
d. a mixture of sulphur dioxide and sulphur trioxide a. White and red phosphorus react with chlorine at room
11. Strong reducing behaviour of H 3 PO2 is due to temperature
b. White phosphorus is metastable while red
a. low oxidation state of phosphorus phosphorus is stable
b. presence of two OH groups and one PH bond c. White phosphorus is lighter than red phosphorus
c. presence of one OH group and two PH bonds d. White phosphorus is highly poisonous while red
d. high electron gain enthalpy of phosphorus phosphorus is not
12. The molecule having smallest bond angle is 20. Red phosphorus is chemically less reactive because
a. NCl3 b. AsCl3 c. SbCl3 d. PCl3 a. it does not contain P  P bonds
b. it does not contain tetrahedral P4 molecules
13. Which of the following statements is wrong?
a. The stability of hydrides increases from NH3 to BiH3 c. it does not catch fire in air even upto 400°C
d. it has a polymeric structure
in group 15 of the periodic table
b. Nitrogen cannot form dπ-pπ bond 21. Ammonia (NH3 ) on heating with carbon dioxide under
c. N—N single bond is weaker than the P—P single pressure gives
bond a. NH4HCO3 b. (NH4 )2 CO3
d. N2O4 has two resonance structures c. NH2COONH4 d. (NH4 )2 CO

@iitjeehelps
p-BLOCK ELEMENTS-II 579
22. Ammonia is mainly manufactured for fertilisers by the Choose the most suitable option.
reaction a. Both A and R are correct, R is the correct explanation
a. 2NH4Cl + Ca(OH)2 → CaCl2 + 2H2O + 2NH3 of A
b. by passing an electric discharge in a mixture of N2 b. Both A and R are correct; R is not the correct
explanation of A
and H2 c. A is correct, R is incorrect
c. by reducing the by-product nitric acid d. R is correct, A is incorrect
d. by passing a mixture of N2 and H2 under pressure
and moderate temperature over a catalyst 31. Consider the following statements,
I. Covalency of N is restricted to four.
23. In PCl5 , phosphorus shows
II. N cannot form dπ-pπ bond as the heavier element
a. sp 2-hybridisation b. sp 3-hybridisation can.
c. sp 3d -hybridisation d. sp 3d 2-hybridisation III. P and As cannot form dπ-dπ bond with transition
24. In modern process, manufacturing of phosphorus is elements when their compounds like P(C2H 5 )3
occurred by and As(C6H5 )3 act as ligands.
a. heating a mixture of phosphorite mineral with sand Which of the above statements are true?
and coke in electric furnace a. I and II b. II and III c. I and III d. All of these
b. heating calcium phosphate with coke
c. heating bone-ash with coke 32. Complete the following reactions.
d. heating the phosphate mineral with sand Heat
6Li + ... I... → 2Li3 N
25. Which of the following compounds is not an Heat
interpseudo halogen? 3Mg + N 2 → II
a. Cl2N3 b. BrCN 773 K
c. ClCN d. ICN N2( g ) + 3H 2( g ) q ... III... , ∆f H ° = − 46.1 kJ mol–1
26. The number of hydrogen atoms attached to Here, I, II and III refer to
phosphorus atom in hypophosphorus is I II III
a. three b. one a. N2 Mg3N2 NH3
c. two d. zero b. 2N2 Mg3N NH3
27. Which of the following statements is true? c. 2N2 Mg3N 2NH3
a. H3PO3 is a stronger acid than H2SO3 d. N2 Mg3N2 2NH3
b. In aqueous medium, HF is a stronger acid than HCl
c. HClO4 is a weaker acid than HClO3 33. NH2CONH 2 + 2H 2O → …A…
d. HNO3 is a stronger acid than HNO2 q 2NH 3 + H 2O + ... B ...
28. In solid state, PCl5 is a ……… In the above equation, A and B respectively are
a. (NH4) 2 CO3 and CO2
a. covalent solid
b. octahedral structure b. (NH4) 2 CO3 and CO
c. ionic solid with [PCI6] + octahedral and [PCI4] − c. (NH4) CO3 and CO
tetrahedral d. (NH4) CO3 and CO2
d. ionic solid with [ PCI4] + tetrahedral and [ PCI6] −
34. The correct decreasing order for the acidic strength of
octahedral oxides of nitrogen is
29. Ionisation enthalpy of group 15 elements a. N2O5 > N2O3 > N2O4 > NO > N2O
I. decreases down the group due to the gradual b. NO > N2O > N2O3 > N2O4 > N2O5
increase in atomic size. c. N2O > NO > N2O3 > N2O4 > N2O5
II. is higher than that of group 14 elements in the d. N2O5 > N2O4 > N2O3 > NO > N2O
corresponding periods.
III. the order of successive ionisation enthalpies is 35. Brown ring test for nitrates depends on
∆i H1 < ∆i H 2 < ∆i H 3 I. the ability of Fe2+ to reduce nitrates to nitric oxide.
Which of the above statements is/are true? II. its reaction with Fe2+ to form a brown coloured
a. I and II b. II and III complex.
c. I and III d. I, II and III Which of the above statements regarding brown test
30. Assertion (A) Heavier elements of group 15 do not for nitrates is/are true?
form pπ -pπ bonds. a. Only I
b. Only II
Reason (R) Their atomic orbitals cannot have c. Both I and II
effective overlapping due to their large size. d. Neither I nor II

@iitjeehelps
580 SELF STUDY GUIDE BITSAT

36. The correct increasing order of acidic strength of 45. Uses of ozone (O3 ) includes
oxyacids of group 15 elements is I. as an oxidising agent in the manufacturing of
a. H3SbO4 < H3PO4 < H3AsO4 < HNO3 KMnO4 .
b. H3SbO4 < H3AsO4 < H3PO4 < HNO3 II. as a germicide, disinfectant and for sterilisation.
c. HNO3 < H3SbO4 < H3AsO4 < H3PO4 III. for bleaching oils, flour, ivory, starch, etc.
d. H3PO4 < H3AsO4 < H3SbO4 < HNO3 The correct set of uses of ozone is
37. Reaction of HNO3 with C, P, S and I respectively give a. I and II b. II and III
c. I and III d. All of these
a. CO, H3PO4, H2SO4 and HIO2
b. CO2, P2O5, SO2 and I2O5 46. Angular shape of ozone molecule consists of
a. 1 σ and 1 π bond
c. CO2, H3PO3, H2SO3 and HIO3
b. 2 σ and 2 π bonds
d. CO2, H3PO4, H2SO4 and HIO3 c. 1 σ and 2 π bonds
38. Phosphine is used in d. 2 σ and 1 π bond
a. holme’s signals b. smoke screens 47. Consider the following reaction,
c. Both a. and b. d. Neither a. nor b. Catalyst
2SO2( g ) + O2( g ) → 2SO3 ( g )
39. In trimetaphosphate ion, the number of O-atoms,
P—O—P bonds and unit negative charges Identify the catalyst
respectively are a. V2O5 b. CuCl2
a. 3, 6 and 3 b. 9, 6 and 3 c. MnO2 d. Either (a) or (b)
c. 6, 6 and 3 d. 9, 3 and 3
48. Peroxoacids of sulphur are
40. Complete the following reactions: a. H2S2O8 and H2SO5
I. 8NH 3 + 3Cl2 → I b. H2S2O8 and H2S2O7
(Excess)
c. H2S2O7 and H2S2O6
II. NH3 + 3Cl2 → II d. H2SO5 and H2S2O7
(Excess)
I II 49. Consider the following statements,
a. 6NH4Cl + N2 NCl3 + 3HCl I. S — S bond is present in H 2S 2O6.
b. NCl3 + 3HCl 6NH4Cl + N2 II. In peroxodisulphuric acid (H 2S 2O8 ), sulphur is in
c. NCl3 + N2 6NH4Cl + 3HCl +6 oxidation state.
d. 6NH4Cl + 3HCl NCl3 + N2 The correct set of statement is
a. Only I b. Only II
41. The number of S  S bonds in sulphur trioxide trimer c. Both I and II d. Neither I nor II
(S 3 O9 ) is
a. three b. two 50. On treating PCl5 with H 2SO4 , sulphuryl chloride
c. one d. zero (SO2Cl2 ) is formed as the final product. This shows
that H 2SO4
42. Oxidation states of O in OF2 and O2F2 respectively are
a. is a derivative of SO2
a. +1 and +2 b. is a dibasic acid
b. +1 and +3 c. has great affinity for water
c. +2 and +3 d. has two hydroxyl groups in its structure
d. +2 and +1
51. Carborundum is obtained when silica is heated at high
43. Water is much less volatile than H 2S because temperature with
a. H 2O has a bond angle of nearly 150° a. carbon b. carbon monoxide
c. carbon dioxide d. calcium carbonate
b. hydrogen is loosely bonded with the sulphur
c. sulphur atom is less electronegative than oxygen 52. Red lead is
atom a. PbO b. Pb3O4 c. PbO2 d. Pb4O3
d. sulphur atom is more electronegative than oxygen
atom 53. Which of the following halides is least stable and has
doubtful existence?
44. Correct decreasing order of O — O bond length in O2, a. CI4 b. GeI4 c. SnI4 d. PbI4
O3 and H2O2 is
a. O2 > H2O2 > O3 54. The mixture of conc. HCl and HNO3 made 3 : 1 ratio
b. H2O2 > O2 > O3 contains
c. H2O2 > O3 > O2 a. ClO2 b. NOCl
d. O2 > O3 > H2O2 c. NCl3 d. N2O4

@iitjeehelps
p-BLOCK ELEMENTS-II 581
55. For making good quality mirrors, plates flint glass are 65. Reduction potentials of some ions are given below.
used. These are obtained by floating molten glass
over a liquid metal which does not solidify before
Ion ClO −4 IO –4 BrO −4
glass. The metal used can be Reduction potential E o = 1.19 V E o = 1.65 V E o = 1.74 V
a. tin b. sodium Eo/V
c. magnesium d. mercury
Arrange them in decreasing order of oxidising power.
56. When I− is oxidised by MnO−4 in alkaline medium, I− a. ClO−4 > IO−4 > BrO4− b. IO−4 > BrO−4 > ClO4−
converts into c. BrO−4 > IO−4 > ClO−4 d. BrO−4 > ClO−4 > IO4−
a. IO−3 b. I2 c. IO−4 d. IO−
66. Consider the following statements,
57. The van der Waals’ forces in halogens decrease in the
I. Among halide ions, iodine is the most powerful
order reducing agent.
a. F2 > Cl2 > Br2 > I2 b. I2 > Br2 > Cl2 > F2
II. HOCl is stronger acid than HOBr.
c. Br2 > Cl2 > F2 > I2 d. Cl2 > Br2 > I2 > F2
III. HF is stronger acid than HCl.
58. Decreasing order of reducing power of hydrogen IV. Fluorine is the only halogen that does not show
halides is variable oxidation states.
a. HI > HBr > HCl > HF b. HF > HI > HBr > HCl The correct set of statements is
c. HI > HF > HBr > HCl d. None of these a. I, II and III b. II and IV
c. I, II and IV d. I, II, III and IV
59. Cl2O is an anhydride of
a. HClO4 b. HOCl c. HClO2 d. Cl2O3 67. Among the C —X bond (where, X = Cl, Br, I) the
correct decreasing order of bond energy is
60. A gas X is passed through water to form a saturated a. C — I > C — Cl > C — Br
solution. The aqueous solution on treatment with b. C — I > C — Br > C — Cl
AgNO3 gives a white precipitate. c. C — Cl > C — Br > C — I
The saturated aqueous solution also dissolves Mg d. C — Br > C — Cl > C — I
ribbon with evolution of colourless gas Y . X and Y
CuCl
respectively are 68. 4HCl + O2 →
2
2Cl2 + 2H 2O
a. CO2 and Cl2 b. Cl2 and CO2
The above reaction of chlorine preparation is of
c. Cl2 and H2 d. H2 and Cl2
a. Deacon’s process b. Contact process
61. In the oxyacids of chlorine, Cl— O bond contains c. Either (a) or (b) d. None of these
a. dπ -dπ bonding b. pπ -dπ bonding 69. Bleaching powder is an example of
c. pπ - pπ bonding d. None of these a. a complex salt b. an acidic salt
c. a basic salt d. a mixed salt
62. The stability of the halides of group 16 elements
70. I. 2NaOH + Cl2 → ... A... + ... B .. . + H 2O
decreases in the order (cold and
− − − −
a. Br > I > Cl > F dil.)

b. F − > I− > Cl− > Br − II. 6NaOH + 3Cl2 → ... C ... + ... D ... + 3H 2O
(hot and
c. F − > Cl− > Br − > I− conc.)
d. I− > Cl− > Br − > F − Here, A, B, C and D refer to
63. Arrange the following acids in the correct increasing A B C D
order of their acidic strength: a. NaCl NaClO3 NaOCl 5NaCl
b. 5NaCl NaClO3 NaOCl NaCl
HCl, HBr, HI, HF
a. HI < HBr < HCl < HF c. NaCl NaOCl 5NaCl NaClO3
d. 5NaCl NaOCl NaCl NaClO3
b. HF < HCl < HBr < HI
c. HCl < HBr < HI < HF 71. The reactions in which conc. H2SO4 is used as an
d. HBr < HI < HF < HCl
oxidising agent are
64. Consider the following statements, I. 2HI + H 2SO4 → I2 + SO2 + 2H 2O
I. Fluorine forms two oxides OF2 and O2F2. II. NaCl + 2H 2SO4 → NaHSO4 + HCl
II. OF2 is thermodynamically stable at 298 K. III. CaF2 + H 2SO4 → CaSO4 + 2HF
III. O2F2 oxidises plutonium to PuF6. IV. Cu + 2H 2SO4 → CuSO4 + SO2 + 2H 2O
The correct set of statements is, Choose the correct option.
a. I and II b. II and III a. I and II b. II and III
c. I and III d. I, II and III c. I and III d. I and IV

@iitjeehelps
582 SELF STUDY GUIDE BITSAT

72. Hydrolysis of one mole of peroxodisulphuric acid will 81. The hybrid state of halogen atom is sp 3 in
yield a. ClO−4
a. two moles of peroxomonosulphuric acid b. ClO−
b. two moles of sulphuric acid c. ClO−3
c. one mole each of sulphuric acid,
peroxomonosulphuric acid and hydrogen peroxide d. All of the above
d. one mole of sulphuric acid and one mole of 82. Interhalogen compounds are
peroxomonosulphuric acid a. covalent molecules
73. Enthalpy of molecular oxygen is almost identical with b. diamagnetic in nature
that of c. volatile solids/liquids at 298 K except ClF
a. neon b. argon c. helium d. xenon d. All of the above

74. The ease of liquefaction of noble gases increases in 83. Interhalogen compounds can be used as
the order I. non-aqueous solvents.
a. He < Xe < Ar < Kr < Ne b. Xe < Kr < Ar < He < Ne II. fluorinating agents.
c. He < Ne < Ar < Kr < Xe d. Xe < He < Ne < Ar < Kr The correct use(s) is/are
a. Only I
75. Among XeO3 , XeO2 F2 and XeF6, the molecules having
b. Only II
same number of lone pairs on Xe are c. Both I and II
a. XeO3 and XeO4 d. Neither I nor II
b. XeO3 and XeO2F2
84. Assertion (A) X — X ′ bond in interhalogens is
c. XeO4 and XeF6
weaker than X — X bond in halogens.
d. XeO3, XeO4 and XeF6
Reason (R) Interhalogen compounds are more
76. In the ring test for NO−3 ion, a brown ring is formed due reactive than halogens (except fluorine).
to the formation of a. Both A and R are correct; R is the correct explanation
a. FeSO4 ⋅ NO2 b. FeSO4 ⋅ HNO3 of A
c. [Fe(H2O)5(NO)] 2+ d. [Fe(H2O)4(NO)2] 2+ b. Both A and R are correct; R is not the correct
explanation of A
77. Which of the following statements regarding sulphur is c. A is correct; R is incorrect
incorrect? d. R is correct; A is incorrect
a. S2 molecule is paramagnetic
85. Which of the following statements are correct?
b. The vapour at 200°C consists mostly of S8 rings
I. Among halogens, radius ratio between iodine and
c. At 600°C, the gas mainly consists of S2 molecules fluorine is maximum.
d. The oxidation state of sulphur is never less than + 4 II. Leaving F—F bond, all halogens have weaker
in its compounds X—X bond than X—X ′ bond in interhalogens.
78. Which one of the following reactions of xenon III. Among interhalogen compounds, maximum
compounds is not feasible? number of atoms are present in iodine fluoride.
a. XeO3 + 6HF → XeF6 + 3H2O IV. Interhalogen compounds are more reactive than
b. 3XeF4 + 6H2O → 2Xe + XeO3 + 12HF + 1.5 O2 halogen compounds.
c. 2XeF2 + 2H2O → 2Xe + 4HF + O2 The correct option is
d. XeF6 + RbF → Rb[XeF7] a. I, II and III
b. I, III and IV
79. Which of the following acids forms three series of c. II, III and IV
salts? d. I and II
a. H3PO2 b. H3BO3 c. H3PO4 d. H3PO3 86. Assertion (A) SF6 is known but SCl6 is not.
80. The correct decreasing order of the acidic strength of Reason (R) Due to small size of F.
HClO, HClO2, HClO3 , HClO4 is a. Both A and R are correct; R is the correct explanation
a. HClO > HClO2 > HClO3 > HClO4 of A
b. HClO4 > HClO3 > HClO2 > HClO b. Both A and R are correct; R is not the correct
c. HClO4 > HClO2 > HClO > HClO3 explanation of A
c. A is correct; R is incorrect
d. HClO3 > HClO > HClO4 > HClO2 d. R is correct; A is incorrect

@iitjeehelps
BITSAT Archives
1. The reaction of P4 with X leads selectively to P4 O6. 10. Distinguishing reagent between silver and lead salt is
X is [2013] a. H2S gas [2009]
a. dry O2 b. dil. HCl solution
b. moist O2 c. [NH2Cl (solid) + NH4OH ] solution
c. mixture of O2 and N2 d. [NH2Cl (solid) + (NH4 )CO3] solution
d. O2 in the presence of aqueous NaOH
11. Identify B in the following reaction:
2. The acidic strength for the hydrides of group 15 °C
H4 SiO4 1000
 → A   → B + CO
Carbon
[2008]
follows the order [2013] − H2O ∆
a. NH3 > PH3 > AsH3 > SbH3 a. Corundum
b. NH3 < PH3 < AsH3 < SbH3 b. Quartz
c. Silica
c. NH3 > PH3 > SbH3 > AsH3
d. Carborundum
d. NH3 < PH3 < SbH3 < AsH3
12. The correct order of reducing abilities of hydrides of
3. The molecules BF3 and NF3 both are covalent group 15 elements is [2008]
compounds but BF3 is non-polar and NF3 is polar. The a. NH3 < PH3 < AsH3 < SbH3 < BiH3
reason is that [2012] b. NH3 > PH3 > AsH3 > SbH3 > BiH3
a. boron is a metal and nitrogen is a gas in uncombined c. NH3 < PH3 > AsH3 > SbH3 > BiH3
state
d. SbH3 > BiH3 > AsH3 > NH3 > PH3
b. BF3 bonds have no dipole moment whereas NF3 bond
have dipole moment 13. Which one of the following pentafluorides cannot be
c. atomic size of boron is smaller than that of nitrogen formed? [2007]
d. BF3 is symmetrical molecule whereas NF3 is a. PF5 b. AsF5
unsymmetrical c. SbF5 d. BiF5
4. Sodium carbonate reacts with SO2 in aqueous 14. Which of the following is most volatile compound?
medium to give [2011] a. HI b. HCl [2007]
a. NaHSO3 b. Na 2S2O3 c. HBr d. HF
c. NaHSO4 d. Na 2SO4 15. SO2 does not act as a/an [2006]
5. The highest bond strength is shown by [2010] a. bleaching agent b. oxidising agent
a. O  O bond b. S  S bond c. reducing agent d. dehydrating agent
c. Se  Se bond d. Te  Te bond 16. Which noble gas is not found in atmosphere? [2006]
6. Which one of the following is pyrophosphoric acid? a. Ne b. Ar c. Rn d. Kr
[2009] 17. In P4 O10, [2009]
a. H3PO4 b. H4P2O7 a. second bond in P == O is formed by pπ - dπ back
c. H4P2O5 d. H3PO4 bonding
7. Which of the following has maximum dipole moment? b. P == O bond is formed by pπ - pπ bonding
[2009] c. P == O bond is formed by dπ - dπ bonding
a. NCl3 b. NBr3 d. P == O bond is formed by dπ -dπ - 3 σ back bonding
c. NH3 d. NI3
18. Dinitrogen pentoxide (N2O5 ), a colourless solid, is
8. Chlorine acts as a bleaching agent only in the prepared by [2005]
presence of [2009] a. heating NH2NO2 with an excess of oxygen
a. dry air b. moisture b. dehydrating HNO2 with CaO
c. sunlight d. pure oxygen c. dehydrating HNO3 with P4O10
9. Helium-oxygen mixture is used by deep sea divers in d. heating a mixture of HNO2 and Ca(NO3 )2
the presence of nitrogen-oxygen mixture because
[2009] 19. Incorrect statement for pyrophosphorus acid (H4 P2O5 )
a. helium is much less soluble in blood than nitrogen is [2005]
b. nitrogen is much less soluble in blood than helium a. It contains P in + 5 oxidation state
c. due to high pressure deep under the sea, nitrogen b. It is dibasic acid
and oxygen react to give poisonous nitric oxide c. It is strongly reducing in nature
d. nitrogen is highly soluble in water d. It contains one P  O  P bond

@iitjeehelps
Answer with Solutions
Practice Exercise 28. (d ) 29. (d)

∆ 30. (a) Heavier elements of group 15 do not form pπ - pπ


1. (d ) 4HNO3 + 2P2O5 → 2N2O5 + 4HPO3 bonds as their atomic orbitals are so large and diffuse that
2. (d ) [N ≡≡ N+ — N− ]− → [N− — N+ ≡≡ N]− they cannot have effective overlapping.
Heat 31. (a) P and As form dπ - dπ bond with transition elements
3. (d ) Ba(N3 )2 → Ba(s ) + 3N2(g ) ↑ when their compounds like P(C2H5 )3 and As(C6H5 )3 act
as ligands.
4. (b)
32. (d )
5. (d ) PH3 is a covalent hydride.
33. (a) NH2CONH2 + 2H2O → (NH4 )2 CO3
6. (b) Pentoxides are more acidic than trioxides and their
acidic character decreases down the group from N to Bi. q 2NH3 + H2O + CO2
7. (d ) SO2, H2O, H2SO4, P4O10 34. (d ) N2O5 > N2O4 > N2O3 > NO > N2O
8. (b) −3 to +5 35. (c) Both the statements are correct.
9. (b) HNO2 , in it oxidation number of nitrogen is +3 (i.e. in NO−3 + 3Fe2+ + 4H+ → NO + 3Fe3+ + 2H2O
between –3 to +5). [Fe(H2O)6] 2+ + NO → [Fe(H2O)5(NO)] 2+ + H2O
10. (a) 2H2SO4 + 2P2O5 → 2SO3 + 4HPO3 Brown coloured complex

11. (c) Due to the presence of one  OH group 36. (b) H3SbO4 < H3AsO4 < H3PO4 < HNO3

12. (c) As we move down the group, the repulsion of bond 37. (d ) Reaction of HNO3 with C, P, S and I gives CO2, H3PO4,
pair and lone pair of central atom decreases. H2SO4 and HIO3 respectively.
13. (a) BiH3 < SbH3 < AsH3 <PH3 < NH3 38. (c) The spontaneous combustion of phosphine is
Least stable Most stable technically used in Holme’s signals. Containers
containing calcium carbide and calcium phosphide are
14. (c) Nitrogen does not show pentacovalency due to pierced and thrown in the sea when the gases evolved
non-availability of d -orbitals. burn and serve as a signal. It is also used in smoke
15. (a) screens.
16. (c) Due to availability of d-orbitals 39. (d) In trimetaphosphate ion, the number of O-atoms,
17. (d ) The reducing nature increases from NH3 to SbH3. P—O—P bonds and unit negative charges are 9, 3, 3
respectively.
18. (b) Air contains 1% argon which is heavier than N2. 40. (a) (I) 8NH3 + 3Cl2 → 6NH4Cl + N2
(Excess)
19. (a) Red phosphorus is less reactive than white
phosphorus. (II) NH 3 + 3Cl2 → NCl3 + 3HCl
(Excess)
20. (d) It is a reason for the given fact.
41. (d) S3O9 is as
∆ On heating
21. (c) 2NH3 + CO2 → NH2COONH4 → O O O
S S
NH2CONH2 + H2O O O
22. (d) By Haber’s process O O
23. (c) P in PCl5 has sp 3d -hybridisation. S
O O
24. (a) Ca 3(PO4 )2 + 3SiO2 → 3CaSiO3 + P2O5
42. (d ) Oxidation states of O in OF2 and O2F2 are +2 and +1
2P2O5 + 10C → P4 + 10CO
respectively.
25. (a) Cl2N3 is not an interpseudo halogen.
43. (c) Water is much less volatile than H2S because sulphur
26. (c) atom is less electronegative than oxygen atom.
+ O 44. (c) H2O2 > O3 > O2
27. (d ) H→ O →→ N ,
O− 45. (d) Uses of ozone
As a germicide, disinfectant and for sterilising water.
H→ O → N == O
Also used for bleaching oils, ivory, flour, starch, etc.
Polarity along O  H in HNO3 is more in comparison to It acts as an oxidising agent in the manufacturing of
O  H in HNO2. KMnO4.

@iitjeehelps
p-BLOCK ELEMENTS-II 585
46. (d ) Angular shape of ozone molecule consists of two σ 66. (c) HF is a weaker acid than HCl.
and one π-bond. 67. (c) Among the C—X bond (where, X = Cl, Br, I), the
V2 O 5 correct decreasing order of bond energy is
47. (a) 2SO2(g ) + O2(g ) → 2SO3(g ) C — Cl > C — Br > C — I
48. (a) Peroxoacids of sulphur are H2S2O8 and H2SO5. 68. (a)
49. (a) In H2S2O6, S—S bond is present. 69. (d) A mixed salt is one which gives more than one type of
cations or anions, e.g. Ca 2+ + OCl− + Cl− .
50. (d) When PCl5 reacts with sulphuric acid, sulphuryl
70. (c)
chloride (SO2Cl2 ) is formed as the final product. It shows
that it has two hydroxyl groups in its structure. 71. (d ) In the following reactions, conc. H2SO4 is used as an
HO — SO2 — OH + PCl5 → oxidising agent.
Cl — SO2 — Cl + POCl3 + 2HCl 2HI + H2SO4 → I 2 + SO2 + 2H2O
High temp.
Cu + 2H2SO4 → CuSO4 + SO2 + 2H2O
51. (a) SiO2 + 3C → SiC + 2CO O O
2300 K Carborundum
 
52. (b) Pb3O4 or PbO2 ⋅ 2PbO is called red lead. 72. (d ) HO— S — O— O— S — OH + HOH →
 
53. (d) Pb4+ will readily oxidise iodide and itself will get O O
reduced to Pb2+ . Thus, PbI2 will be formed, Peroxodisulphuric acid
PbI4 → PbI2 + I2 O O
54. (b) 3HCl + HNO3 → NOCl + 2H2O + 2[Cl]  
HO — S — OH + HO — O — S — OH
55. (a) Melting point of tin is lower than that of glass.  
56. (a) 2MnO−4 + H2O + I− → 2MnO2 + 2OH − + IO3− O O
Sulphuric acid Peroxomonosulphuric acid
57. (b) van der Waals’ forces increase with increase in the 73. (d) Enthalpy of molecular oxygen (1175 kJ mol −1) is
size and molecular weight of the atom.
almost identical with that of xenon (1170 kJ mol −1).
58. (a) The order of reducing power of halide ions is
74. (c) 75. (b) 76. (c)
I − > Br − > Cl− > F −
77. (d) Oxidation states of sulphur in H2S is −2. Thus,
59. (b) Cl2O + H2O → 2HClO
incorrect (valency can be less than 4).
Chlorine has +1oxidation state in Cl2O and HOCl.
78. (a) XeF6 has much tendency to hydrolyse. The reverse
60. (c) Cl2 + H2O → HOCl + HCl reaction is more spontaneous.
X

AgNO3 + HCl → AgCl ( white ppt. ) + HNO3 XeF6 + 3H2O → XeO3 + 6HF
Mg + 2HCl → MgCl2 + H2 79. (c) Structure of H3PO4 is
Y
O
61. (b) Cl issp 3-hybridised having electrons in d -orbitals and

p-electrons of oxygen gives rise to pπ - dπ bonding to P
Cl—O bond. HO  OH
62. (c) Elements of group 16 form a large number of halides OH
of the type EX 6,EX 4 and EX 2, where E is an element of
the group and X is a halogen. F − > Cl− > Br − > I− H3PO4 has 3 OH groups, i.e. has three ionisable
H-atoms and hence forms three series of salts. These
63. (b) Correct increasing order of their acidic strength is three possible series of salts for H3PO4 are as follows
HF < HCl < HBr < HI NaH2PO4,Na 2HPO4 and Na 3 PO4.
64. (d) Fluorine forms two oxides, i.e. OF2 and O2F2. OF2 is 80. (b) The correct decreasing order of the acidic strength of
thermodynamically stable at 298 K. These oxides are oxoacids of chlorine is
essentially oxygen fluorides because of the higher HClO4 > HClO3 > HClO2 > HClO
electronegativity of fluorine than oxygen. Both are strong 81. (d)
fluorinating agents. O2F2 oxidises plutonium to PuF6 and
82. (d) Interhalogen compounds are covalent molecules and
the reaction is used in removing plutonium as PuF6 from diamagnetic in nature. These are volatile solids or liquids
spent nuclear fuel. at 298 K except ClF.
65. (d ) Greater the SRP value of spcies, higher will be its 83. (c)
oxidising power. Hence, on the basis of the reduction 84. (b) Interhalogen compounds are more reactive than
potential of ions, the decreasing order of oxidising power halogens (except fluorine). This is because X X ′ bond in
is BrO−4 > ClO−4 > IO−4. interhalogens is weaker than X X bond. All these

@iitjeehelps
586 SELF STUDY GUIDE BITSAT

undergo hydrolysis giving halide ion derived from the maximum number of atoms are present in iodine fluoride.
smaller halogen and a hypohalite (when XX′ ), halite Interhalogen compounds are more reactive than halogen
(when XX′3 ), halate (when XX ′5 ) and perhalate (when compounds.
XX 7′ ) anion derived from the larger halogen.
86. (a) SF6 is known but SCl6 is not. It is due to the small size
XX ′ + H2O → HX ′ + HOX of fluorine. Six F− ions can be accommodated around
85. (b) Among halogens, radius ratio between iodine and sulphur while chloride ion is comparatively larger in size,
fluorine is maximum. Among interhalogen compounds, thus there will be interatomic repulsion.

BITSAT Archives
1. (c) The reaction of P4 and X leads selectively to P4O6 when the worker moves from high pressure (while in deep
because N2 prevents the further reaction of P4O6 into sea) to atmospheric pressure.
P4O10. 10. (b) On adding dil. HCl to silver and lead salts, white
2. (b) NH3 < PH3 < AsH3 < SbH3 precipitate of chlorides of Ag and Pb are obtained which
on heating with water are separated from each other as
3. (d) BF3 is symmetrical planar, although it has polar bonds PbCl2 is soluble in hot water but AgCl is not.
but resultant dipole moment is zero. In NF3, lone pair °C
11. (d) H4SiO4 1000
→ SiO2 
C
→ SiC + CO
cause distortion, hence polarity arises. –2H2O ∆ Carborundum

4. (a) Na 2CO3 + 2SO2 + H2O → 2NaHSO3 + CO2 12. (c) NH3 < PH3 > AsH3 > SbH3 > BiH3
5. (b) As the size increases, bond strength decreases. But 13. (d) The +5 oxidation state of Bi is unstable due to inert
S  S bond is stronger than O  O bond because of pair effect. Thus, BiF5 cannot be formed.
smaller size of O-atoms. 14. (b) Boiling point of HF is highest due to H-bonding . For
6. (b) other halogen acids, boiling point increases in the order
HCl < HBr < HI. Therefore, most volatile (with lower
7. (c) The electronegativity difference is highest in case of
boiling point) is HCl.
N H bonds, therefore NH3 has the highest dipole
moment. 15. (d) SO2 does not act as a dehydrating agent.
8. (b) Bleaching action of chlorine is only in the presence of 16. (b) The noble gas which is not found in atmosphere is
moisture. argon (Ar).
9. (a) He - O2 (80%-20%) mixture is used by deep sea divers 17. (a) The terminal bond formed is pπ - dπ bonding.
for artificial respiration. Because of low intermolecular 18. (c) P4O10 + 4HNO3 → 4HPO3 + 2N2O5
force in He, it is much less soluble in aqueous solutions
19. (a) It contains phosphorus in +3 oxidation state.
(as compared to N2) such as blood and does not cause
‘‘caisson sickness’’ or ‘‘bends’’ by bubbling out of blood

@iitjeehelps
19
d- and f-Block Elements

d-Block Elements
d-block elements are present from fourth period onwards. There are mainly three series of the
transition metals, (a) 3 d-series (Sc to Zn), (b) 4d-series (Y to Cd) and (c) 5d-series (La to Hg, omitting
Ce to Lu).
d-block elements are known as transition elements because their position in the periodic table is in
between the s-block and p-block elements.

Electronic Configuration
3rd group 4th group 5th group 6th group 7th group

ns 2( n - 1) d 1 ns 2( n - 1) d 2 ns 2( n - 1) d 3 ns 1 ( n - 1) d 5 ns 2( n - 1) d 5

Sc Ti V Cr Mn

Y Zr Nb Mo Tc

La Hf Ta W Re

8th group 9th group 10th group 11th group 12th group

ns 2( n - 1) d 6 ns 2( n - 1) d 7 ns 2( n - 1) d 8 ns 1 ( n - 1) d 10 ns 2( n - 1) d 10

Fe Co Ni Cu Zn

Ru Rh Pd Ag Cd

Os Ir Pt Au Hg

● Unlike Cr and Mo, W has electronic configuration as ns 2 (n - 1)d 4 .

@iitjeehelps
588 SELF STUDY GUIDE BITSAT

General Characteristics of d-Block Catalytic Properties


Elements Transition metals have two incomplete outermost shells
and ability to adopt multiple oxidation states and to form
(i) Transition elements show all the properties of metals complexes, therefore used as a catalyst. Transition metals
like metallic lustre, conductivity, malleability, ductility, also provide larger surface area for the reactant to be
high tensile strength except mercury which is liquid at adsorbed.
room temperature.
Magnetic Properties
(ii) Their atomic radii is intermediate between s- and p -block
When a magnetic field is applied to substances, mainly two
elements.
types of magnetic behaviours are observed : diamagnetism
(iii) These elements possess high melting and boiling and paramagnetism. The magnetic moment is determined
points, high enthalpies of vaporisation, atomisation by the number of unpaired electrons (n) which is given by
and hydration of ions.
m = n (n + 2 ) BM (BM = Bohr Magneton)
(iv) Their first ionisation enthalpy is higher than that of where, n = number of unpaired electrons
s-block elements but less than that of p -block elements.
If n = 0, the substance is diamagnetic.
(v) These elements are electropositive in nature and show If n ¹ 0, the substance is paramagnetic.
variable oxidation states.
(vi) Generally, these elements are paramagnetic in nature Interstitial Compounds
and form coloured compounds. Transition elements possess a tendency to trap atoms like
(vii)These elements form interstitial compounds with H, B, C, N in their interstitial spaces of their lattices,
elements like C, H, B and N and show catalytic behaviour. e. g. Ti H 1.7 , VH0.56 . These compounds are very hard,
chemically inert and have higher melting points than that
Metallic Character of pure metals.
With the exception of Zn, Cd and Hg, all the transition
elements are much harder and less volatile. Although I. Iron (Fe)
hardness and brittleness are associated with covalent Ores Haematite — Fe 2O3
bonding between metal atoms, while the overlapping of
unfilled d-orbitals of metals causes covalent bonding, the Magnetite — Fe3O4
valence s-electrons are responsible for metallic bonding. Iron pyrites — FeS2
Cu, Ag and Au are particularly outstanding in their thermal
and electrical conductivities.
Extraction
1. Concentration
Oxidation States (a) Gravity separation method
Some common oxidation states of 3 d-series elements are (b) Magnetic separator
shown below :
2. Roasting 4FeCO3 + O2 ¾® 2Fe 2O3 + 4CO2
Name of the elements Oxidation states shown by 3d-series
3. Reduction/Smelting
Sc +3
Fe 2O3 + 3CO ¾® 2Fe + 3CO2
Ti + 2, + 3, + 4
Fe 2O3 + 3H 2 ¾® 2Fe + 3H 2O
V + 2, + 3, + 4, + 5
Pig iron
Cr + 1, + 2, + 3, + 4, + 5 , + 6
CaO + SiO2 ¾® CaSiO3
Mn +2, + 3, + 4, + 5 , + 6, + 7
Wrought iron is prepared by ‘pudding process.’
Fe + 2, + 3, + 4, + 5 , + 6
Steel is obtained by Bessemer process.
Co + 2, + 3, + 4
Ni + 2, + 3, + 4 Properties of Iron
Cu + 1, +2 It is a grey-white lustrous metal. It is highly tensile, ductile,
Zn +2 malleable, magnetic metal etc.
Some important reactions
Ionic Radii
Fe + 2H 2SO4 ¾® FeSO4 + SO2 + 2H 2O
The trend for ionic radii is same as that of atomic radii. ¾D ® Fe3O4 + 4H 2
3Fe + 4H 2O ¾
(a) Ionic radii decreases with increase in oxidation state. Steam

(b) Ionic radii of cations present in the same oxidation 4Fe + 10HNO3 (dil. ) ¾® 4Fe(NO3 )2 + NH 4 NO3 + 3H 2O
state, decreases with increase in atomic number. Fe + 5CO ¾® Fe(CO)5

@iitjeehelps
d- AND f-BLOCK ELEMENTS 589
Compounds of Iron Properties of Zinc
1. Ferric Chloride (FeCl 3 ) ● Bluish white crystalline metal, brittle at ordinary
temperature, malleable and ductile.
Preparation
● ¾D ® 2ZnO
2Zn + O2 ¾
(a) 2Fe + 3Cl 2 ¾® 2FeCl3
● Zn + H 2O ¾® ZnO + H 2
(b) 2Fe + 4HCl + Cl 2 ¾® 2FeCl3 + 2H 2
● 4Zn + 3H 2O + CO2 + 2O2 ¾® ZnCO3 × 3Zn(OH)2
(c) Fe 2O3 + 6HCl ¾® 2FeCl3 + 3H 2O ● Zn + 4HNO3 ¾® Zn(NO3 )2 + 2NO2 + 2H 2O
Properties (conc. and hot)

Hydrated FeCl3 × 6H 2O is a yellow, deliquescent, ● Zn + 2NaOH ¾® Na 2ZnO2 + H 2


crystalline solid, soluble in water, alcohol etc., and
anhydrous FeCl3 is a dark solid. Compounds of Zn
FeCl3 + 3H 2O ¾® Fe(OH)3 + 3HCl 1. Zinc Oxide (ZnO)
D Preparation
2FeCl3 ¾® 2FeCl 2 + Cl 2
(a) 2Zn + O2 ¾® 2ZnO
2FeCl3 + H 2S ¾® 2FeCl 2 + 2HCl + S
¾D ® 2ZnO + 4NO2 + O2
(b) Zn(NO3 )2 ¾
2FeCl3 + SnCl 2 ¾® 2FeCl 2 + SnCl 4
Properties
2. Ferrous Sulphate (FeSO 4×7H 2O) (Green Vitriol) ● It is white, light powder insoluble in water. On heating,
Preparation becomes yellow.
2 ¾® Zn + H 2O
● ZnO + H
(a) FeCO3 + H 2SO4 ¾® FeSO4 + H 2O + CO2
● ZnO + 2NaOH ¾® Na ZnO + H O
(b) FeS+ H 2SO4 ¾¾® FeSO4 + H 2S (Kipp’s apparatus) 2 2 2

Properties 2. Zinc Sulphide (ZnS)


It is light green crystalline solid, soluble in water Preparation
efflorescent and isomorphous with MgSO4 × 7H 2O. (a) ZnSO4 + H 2S ¾® ZnS + H 2SO4
°C
FeSO4 × 7H 2O ¾300
¾¾ ® FeSO4 + 7H 2O (b) 2ZnO + 3S ¾ ¾D ® 2ZnS + SO2
¾D ® Fe 2O3 + SO2 + SO3
2FeSO4 ¾ (c) ZnCl 2 + H 2S + 2NH3 ¾® ZnS + 2NH 4Cl
Properties
FeSO4 + (NH 4 )2 SO4 + 6H 2O ¾®
● ZnS is a white solid insoluble in water and soluble in
FeSO4 × (NH 4 )2 SO4 × 6H 2O mineral acids. It turns yellow in light.
Mohr salt
● It also shows phosphorescence.
II. Copper (Cu)
3. Zinc Sulphate (ZnSO 4 ×7H 2O) (White Vitriol)
Ores Copper pyrites — CuFeS2
Preparation ZnCO3 + H 2SO4 ¾® ZnSO4 + H 2O + CO2
Copper glance — Cu 2S
Properties
Cuprite — Cu 2O ● It is highly soluble in water.
Malachite — CuCO3 × Cu(OH)2 100 °C °C
● ZnSO × 7H O ¾ ¾
4 2 ¾® ZnSO4 × 6H 2O ¾280
¾¾ ® ZnSO4
D D
Properties of Copper 760 °C
● ZnSO4 ¾ ¾¾® ZnO + SO3
● Copper is reddish golden coloured metal, soft, malleable, D
ductile and second best conductor next only to silver, ● It is isomorphous with epsom salt and green vitriol.
molten metal adsorbs SO2 ( g ) .
● 2Cu + H 2O + CO2 +O2 ¾® CuCO3 × Cu(OH)2 IV. Mercury (Hg )
● 8Cu + 6H 2O + 2SO2 + 5O2 ¾®CuSO4 × 3Cu(OH)2 Ore Cinnabar— HgS
● Cu + 4HNO3 (conc. ) ¾® Cu(NO3 )2 + 2H 2O + 2NO2
Alloy Formation
Alloy is the homogeneous solid solution of two or more
III. Zinc (Zn) metals. Transition metals have approximate same size,
Ores Zinc blende—(ZnS) therefore in molten form, they can fit to each other’s
crystalline structures and form homogeneous mixture to
Calamine—ZnCO3 form the alloy,e.g. brass (copper-zinc) and bronze
Zincite—ZnO (copper-tin), etc.

@iitjeehelps
590 SELF STUDY GUIDE BITSAT

+ 3+
Cr2O2– –
7 + 14H + 6I ¾® 2Cr + 7H 2O + 3I 2
Other Important Compounds +
Cr2O2–
7 + 3H 2S + 8H ¾® 2Cr
3+
+ 3S + 7H 2O
Copper Sulphate (CuSO 4 ×5H2O) Cr2O2– +
7 + 14H + 3Sn
2+
¾® 3Sn 4 + + 2Cr3 + + 7H 2O
(Blue Vitriol) The half-reactions of reductants are
C 2O2–
4 ¾® 2CO2 + 2e

Preparation
(a) CuCO3 × Cu(OH)2 + 2H 2SO4 ¾® 2CuSO4 + 3H 2O + CO2 Fe 2+ ¾® Fe3 + + e –
(b) CuO + H 2SO4 ¾® CuSO4 + H 2O NO–2 ¾® NO3– + 2e –
2I – ¾® I 2 + 2e –
Properties
Uses
● It is a deep blue coloured crystalline solid, fairly soluble in
water. (i) K 2Cr2O7 is used as an oxidising agent in volumetric
°C analysis.
● 2CuSO4 ¾750 ¾¾ ® 2CuO + 2SO2 + O2
(ii) It is used in mordant dyes, leather industry,
● CuSO4 + 2NH 4OH ¾® Cu(OH)2 + (NH 4 )2 SO4 photography (for hardening of film), cleaning glass
● 2CuSO4 + 4KI ¾® Cu 2I 2 + I 2 + 2K 2SO4 ware.
● 2CuSO4 + 4KCN ¾® 2Cu(CN)2 + 2K 2SO4 (iii) It is used in chromyl chloride test.

Uses
Potassium Permanganate (KMnO4 )
It is used
1. in electroplating, electric batteries, etc.
Preparation
2MnO2 + 4KOH + O2 ¾® 2K 2 MnO4 + 2H 2O
2. in medicine as antiseptic. (Green)
+
3. to make Fehling’s solution. 3MnO2–
4 + 4H ¾® 2MnO–4 + MnO2 + 2H 2O

Potassium Dichromate (K 2Cr2O 7 ) Properties


It acts as a strong oxidising agent.
Preparation (a) In the presence of dil. H 2SO4 , KMnO4 is reduced to
4FeO × Cr2O3 + 8Na 2CO3 + 7O2 ¾® manganous salt.
MnO–4 + 8H + + 5e – ¾® Mn 2 + + 4H 2O
8Na 2CrO4 + 2Fe 2O3 + 8CO2
(Yellow) MnO–4 + 3e – + 2H 2O ¾® MnO 2 + 4OH -
2Na 2CrO4 + 2H + ¾® Na 2Cr2O7 + 2Na + + H 2O To acidify KMnO4 , only H 2SO4 is used and not HCl or
(Orange)
HNO3 because HCl reacts with KMnO4 and produce
Na 2Cr2O7 + 2KCl ¾® K 2Cr2O7 + 2NaCl Cl 2 while HNO3 itself acts as an oxidising agent.
Sodium dichromate is more soluble than potassium Alkaline or neutral KMnO4 solution oxidises I - to
dichromate. Chromates and dichromates are IO3– , S2O32– to SO2–
4 , Mn
2+
to MnO2 , etc.
interconvertible in aqueous solution depending upon pH
Acidic KMnO 4 solution oxidises oxalate to CO2 , iron
of the solutions.
(II) to iron (III), nitrites to nitrates and iodides to
Structure iodine.

O
2–
O O
2–
Uses
O
Cr O Cr Cr O (i) In the laboratory preparation of Cl 2 .
O O O O (ii) As an oxidising agent, disinfectant.
O
(iii)In the making of Baeyer’s reagent.
Chromate ion sp3 Dichromate ion sp3
Structure of permanganate ion

Properties O

Sodium and potassium dichromates are strong oxidising Mn


agents thus, acidified K 2Cr2O7 will oxidise iodides to iodine,
O O−
sulphides to sulphur, tin (II) to tin (IV) and iron (II) salts to O
iron (III). sp3-hybridised tetrahedral

@iitjeehelps
d- AND f-BLOCK ELEMENTS 591

Mercury Halides
Mercury forms halides in two oxidation states, +1 and +2, ● AgBr + nNH3 ¾® [ Ag( NH3 )n ]+
e.g. Hg 2 Cl 2 , HgCl 2 Tris (triphenylphosphine product)

Preparation Photography
Mercury (I) chloride is prepared by heating a mixture of mercury (II) Ordinary black and white photographic film
chloride and mercury in iron vessel. consists of a celluloid strip that has been coated
HgCl 2 + Hg ¾® Hg 2Cl 2 with a gelatin emulsion containing silver halides,
It can also be obtained by reduction of mercury (II) chloride by usually AgBr.
reducing agents like tin (II) chloride in limited quantity.
2HgCl 2 + SnCl 2 ¾® Hg 2Cl 2 + SnCl 4

Properties
Inner-Transition
(i) 2Hg 2Cl 2 ¾® 2HgCl 2 + 2Hg Elements( f-Block Elements)
(ii) Hg 2Cl 2 + 2NH3 ¾® Hg(NH 2 )Cl + Hg + NH 4Cl
Lanthanoids
Silver Nitrate (Lunar Caustic) (AgNO3 ) Properties
, 5d 0 - 1 , 6 s 2
- 14
Preparation General configuration [Xe] 4 f 1
D
3Ag + 4HNO3 ¾
¾® 3AgNO3 + NO + 2H 2O All the lanthanoids are silvery white soft metals
Lunar caustic and tarnish rapidly in air.
● Atomic and ionic size, from left to right,
Properties decreases due to increase in nuclear charge. This
● Colourless crystalline compound soluble in H 2O and alcohol with is known as lanthanoid contraction.
mp 212 °C. ● The hydroxides are ionic and basic. The basic
● When exposed to light, it decomposes hence, stored in a brown strength decreases from Ce(OH)3 to Lu(OH)3
coloured bottle. due to lanthanoid contraction
D, red hot D, T >212 ° C ● Many trivalent lanthanoid ions are coloured
2Ag + 2NO2 + O2 ¾¾¾® 2AgNO3 ¾¾¾¾® 2AgNO2 + O2
both in the solid state and in aqueous solutions.
● It is reduced to metallic Ag by more electropositive metals like Neither La3 + nor Lu3 + ion shows any colour but
Cu, Zn, Mg and also by PH3 . the rest do so. Lanthanoid ions with xf -electrons
2AgNO3 + Cu ¾® Cu(NO3 )2 + 2Ag have similar colour to those with
6AgNO3 + PH3 + 3H 2O ¾® 6Ag + 6HNO3 + H3PO3 (14 - x ) f -electrons.
● The lanthanoid ions other than the f 0 type
● It dissolves in excess of KCN.
AgNO3 ¾KCN ¾¾® AgCN ¾KCN ¾
¾® K[Ag(CN)2 ] (La3 + and Ce 4 + ) and the f 14 type (Yb 2 + and Lu3 + )
White ppt Potassium argentocyanide are all paramagnetic. The paramagnetism arises
soluble
to maximum in neodymium.
2AgNO3 + Na 2S2O3 ¾® Ag 2S2O3 + 2NaNO3 Oxidation states Ce 4+ (some elements) is
White ppt
favoured by its noble gas configuration, but it is
Ag 2S2O3 + H 2O ¾® Ag 2S + H 2SO4 a strong oxidant reverting to the common +3
Black
state. The E °value for Ce 4 + / Ce3 + is +1.74 V, the
Silver Halide (Silver Bromide) reaction rate is very slow and hence, Ce(IV) is a
good analytical reagent.
Preparation Pr, Nd, Tb and Dy also exhibit +4 state but only in
AgNO3 (aq ) + KBr(aq ) ¾® AgBr(s ) + KNO3 (aq ) oxides. Eu 2+ is formed by losing two s-electrons
and its f 7 configuration accounts for the
Properties formation of this ion. However, Eu 2+ is a strong
It is a soft, pale-yellow, water insoluble salt. It has fcc structure. reducing agent changing to the common +3
 Ph  state. Similarly, Yb 2+ which has f 14
  configuration is a reductant, Tb 4+
has
 
● AgBr + (Ph)3P → BrAg  P  Ph half-filled f-orbitals and it is an oxidant.
 
  Reactivity Lanthanoids are very reactive metals
 Ph 3
Tris (triphenylphosphine product) like alkaline earth metals.

@iitjeehelps
592 SELF STUDY GUIDE BITSAT

Actinoids the 4f or 5d orbitals in the lanthanoid atoms because the


external shielding of the 5f electrons is less than that of
Properties the 4 f -electrons.
- 14 -2
● General configuration [Rn] 5 f 1 , 6d 0 , 7s 2 Comparison of Actinoids and Lanthanoids
● Actinoids exhibit a range of oxidation states due to
Lanthanoids Actinoids
comparable energies of 5 f , 6d and 7s orbitals. The
general oxidation state of actinoids is +3. These elements show +2 and +4 These elements show variable
● All the actinoids are strong reducing agents and very oxidation states along with +3. oxidation states like +4, +5, +6, +7
along with +3.
reactive.
● Actinoids also react with oxygen, halogen, hydrogen and Most of the ions of lanthanoid are The ions of actinoid are also
sulphur, etc., like lanthanoids. coloured. coloured.
● Actinoids are radioactive in nature and therefore, it is These elements show less These elements show greater
difficult to study their chemical nature. tendency towards complex tendency towards complex
● The atomic and ionic radii of actinoids decreases formation. formation.
regularly across the series due to poor shielding effect of Except promethium, these These elements are highly
5 f -electrons. This is known as actinoid contraction. elements are non-radioactive. radioactive.
● The allocation of electrons to the 5 f or 6d-orbitals in the
These are less basic. These are more basic.
actinoid atoms is more uncertain than the allocation to

Practice Exercise
1. The incorrect statement for transition elements is c. its unit is Bohr Magneton (BM). A single unpaired
a. the last electron of these elements enters in d -orbital electron has 1× 73 BM magnetic moment
b. the common oxidation state of these elements is +3 d. All of the above
c. the properties of these elements is intermediate 7. Which of the following compounds will show magnetic
between s- and p-block elements moment of 1.72 BM?
d. the transition elements with smaller atomic radii is a. [Ni(CN)4] 2- b. [CoCl6] 4-
scandium
2+
c. [Cu(NH3 )4] d. TiCl4
2. Zinc does not exhibit variable valency due to
a. presence of complete filled d -orbitals 8. Interstitial compounds are formed when small atoms
b. inert pair effect are trapped inside the crystal lattice of metals. Which
c. presence of 4s -orbital of the following are the characteristic properties of
d. None of the above interstitial compounds?
3. The metal which is considered as transition metal, is I. They have high melting points in comparison to pure
a. zinc b. cadmium metals.
c. mercury d. scandium II. They are very hard.
III. They retain metallic conductivity.
4. The ground state electronic configuration of neutral IV. They are chemically very reactive.
titanium atom is a. I, II and III b. II, III and IV
a. [Ar]4s 2 4p 2 b. [Ar]3 d 2 4s 2
c. I, III and IV d. I, II and IV
c. [Ar]4s 2p1x p1y d. [Ar]3 d 5 9. The element with maximum number of oxidation
5. Among the following electronic configurations of states in their compounds is
d-orbitals, the electronic configuration showing the a. Eu b. La c. Gd d. Am
highest magnetic moment is 10. When Fe metal is rusted, it is
a. 3d 2 b. 3d 5 c. 3d 7 d. 3d 8 a. isomerised b. decomposed
c. reduced d. oxidised
6. Magnetic moment
a. increases with increasing number of unpaired 11. Ferric sulphate on heating gives
electrons a. SO2 and SO3 b. SO2
b. gives indication about the number of unpaired c. SO3 d. S
electrons present in the atom, molecule or ion

@iitjeehelps
d- AND f-BLOCK ELEMENTS 593
12. Iron is rendered passive by treatment with conc. 26. Which of the following ions will finally give a black
a. HCl b. H2SO4 precipitate with Ag+ ion?
c. H3PO4 d. HNO3 a. SO2-
3 b. Br -
c. CrO2- d. S2O2-
13. Which of the following is an important ore of iron? 4 3

a. Smaltite b. Garnierite 27. In the cyanide extraction process of silver from


c. Pentalandite d. Haematite argentite ore, the oxidising and reducing agent used
14. The metal which corrodes readily in moist air, is respectively are
a. gold b. silver a. O2 and CO b. O2 and Zn dust
c. iron d. nickel c. HNO3 and Zn dust d. HNO3 and CO
15. Which one of the following forms of iron can give other 28. Turnbull’s blue is a compound, called
forms of iron? a. ferricyanide b. ferrous ferrocyanide
a. Wrought iron b. Steel c. ferrous cyanide d. ferri ferrocyanide
c. Pig iron d. Cast iron
29. Calomel (Hg2Cl2 ) on reaction with ammonium
16. The oxidation state of iron in [Fe(H2O)5 NO] 2+ is hydroxide gives
a. 1 b. 2 c. 3 d. 4 a. HgO b. Hg2O
17. In weak field ligand, which one of the following cations c. NH2 — Hg — Hg — Cl d. HgNH2Cl
has maximum magnetic moment?
30. Nitriding is the process of surface hardening steel by
a. Fe2+ b. Cu2+ c. Ni2+ d. Co2+
treating it in an atmosphere of
18. In the test for nitrate, the composition of brown ring is a. NH3 b. O3
a. FeSO4 × N2O b. FeSO4 × NO c. N2 d. H2S
c. FeSO4NO2 d. Fe(NO3 )2
31. Potassium dichromate is used
19. Copper can be extracted from a. in leather industry
a. kupfer nickel b. dolomite b. as an oxidant for the preparation of many azo
c. malachite d. galena compounds
20. The colour of light absorbed by an aqueous solution of c. Both (a) and (b)
CuSO4 is d. None of the above
a. orange-red b. blue-green 32. The structures of chromate and dichromate ions are
c. yellow d. violet A and B respectively. Here, A and B refer to
21. For the ions, Cu2+(3 d 9 ) and Cu+(3 d 10 ), the correct a. A - tetrahedral, B - octahedral
statement is b. A - tetrahedral, B - two tetrahedral
a. Cu2+ is more stable than Cu+ c. A - octahedral, B - two tetrahedral
b. Cu2+ is less stable than Cu+ d. A - two octahedral, B - octahedral
c. Cu2+ and Cu+ ions are equally stable
33. Consider the following statements.
d. stability of Cu+ and Cu2+ depends on the nature of
I. Sodium dichromate is less soluble than potassium
copper salts
dichromate.
22. When CuSO4 reacts with aqueous KI, the products are II. Crystals of potassium dichromate are of orange colour.
a. Cu2I2 + K 2SO4 b. Cu + K 2SO4 + I2 III. The chromates and dichromates are interconvertible
in aqueous solution depending upon pH of the solution.
c. CuI2 + K 2SO4 d. Cu2I2 + K 2SO4 + I2 IV. The oxidation states of chromate and dichromate are
23. Which of the following is formed, when copper (II) different.
sulphate is treated with excess ammonia? The correct statements are
a. A black precipitate a. I, II, III and IV b. II and IV
b. A red precipitate c. I and III d. I and II
c. A deep blue solution 34. The photographic industry relies on the special
d. A white precipitate turning black light-sensitive properties of …I…Here, I refers to
24. When copper nitrate is strongly heated, the compound a. AgCl b. AgBr
obtained is c. PdCl2 d. either (a) or (b)
a. copper nitrite b. copper
c. copper nitride d. copper oxide 35. What happens when FeSO4 solution reacts with
acidified KMnO4 solution?
25. Hair dye contains
a. Iron (II) is oxidised b. KMnO4 is oxidised
a. copper nitrate b. gold chloride
c. silver nitrate d. lead nitrate c. Iron (II) is reduced d. Iron (III) is reduced

@iitjeehelps
594 SELF STUDY GUIDE BITSAT

36. When pyrolusite is fused with KOH, the colour of the 45. Among the following lanthanoid ions, the
product is paramagnetic ion is
a. red b. pink a. Yb2+ b. Eu2+
c. black d. green c. Lu3+ d. Ce4+
37. Which of the following is wrong? 46. The actinoids include the …A… elements from …B…
a. K 2Cr2O7 ¾® Orange to …C… Here, A, B and C refer to
b. CuSO4 × 5H2O ¾® Blue a. A - fourteen, B - Th , C - Lr
c. MnSO4 ¾® Yellow b. A - twelve, B - Lr , C - Th
c. A - thirteen, B - Lr , C - Th
d. Cr2(SO4 )3 ¾® Purple
d. A - fourteen, B - Th, C - Lr
38. In which of the following compounds, Mn has highest
47. Larger number of oxidation states are exhibited by the
oxidation state?
actinoids then those by the lanthanoids, the main
a. K 2MnO4 b. MnO2
reason being
c. KMnO4 d. Mn3O4 a. 4f orbitals are more diffused than the 5f orbitals
39. KMnO4 is used b. lesser energy difference between 5f and 6d than
between 4f and 5d orbitals
a. in analytical chemistry
c. more energy difference between 5 f and 6d than
b. as a favourite oxidant in preparative organic chemistry
between 4f and 5d orbitals
c. in bleaching of wool, cotton and silk
d. more reactive nature of the actinoids than the
d. All of the above
lanthanoids
40. On addition of small amount of KMnO4 to
conc. H2SO4 , a green oily compound is obtained 48. The f -block consists of …… series.
which is highly explosive in nature. Identify the a. two b. three
compound from the following. c. four d. five
a. Mn2O7 b. MnO2 49. Actinoids possess
c. MnSO4 d. Mn2O3 a. variable valency
b. 12 elements
41. There are 14 elements in actinoid series. Which of the c. all synthetic elements
following elements does not belong to this series? d. only short-lived isotopes
a. U b. Np
c. Tm d. Fm
50. The lanthanoid contraction is related to
a. atomic radii
42. In context of the lanthanoids, which of the following b. atomic as well as ionic radii, M 3+
statements is not correct? c. valence electrons
a. There is a gradual decrease in the radii of the d. oxidation states
members with increasing atomic number in the
51. Lanthanoids and actinoids differ from each other
series
because
b. All the members exhibit + 3 oxidation state
c. Because of similar properties, the separation of a. of the presence of partially filled outermost shells
lanthanoids is not easy b. actinoids are radioactive in nature
d. Availability of 4f electrons results in the formation of c. they show common oxidation state of + 3
compounds in + 4 state for all the members of the d. both are known as inner-transition elements
series 52. The one which is not the characteristic property of
43. Knowing that the chemistry of lanthanoids (Ln) is transition element, is
dominated by its +3 oxidation state, which of the a. diamagnetic behaviour b. formation of complexes
following statements is incorrect? c. catalytic activity d. variable oxidation states
a. Because of the large size of the Ln (III) ions, the 53. Dichromate ions in alkaline medium exist as
bonding in its compounds is predominantly ionic in
a. CrO2-4 b. CrO3
character
b. The ionic sizes of Ln (III) decrease in general with c. Cr 3+ d. Cr 4–
increasing atomic number 54. The correct formula for diamine silver chloride is
c. Ln (III) compounds are generally colourless a. [Ag(NH3 )2]Cl
d. Ln (III) hydroxides are mainly basic in character
b. [Ag(NH2 )3]Cl
44. The basicity of lanthanoid hydroxides across the c. AgCl. NH3
lanthanoid series d. [Ag(NH4 )2]Cl
a. decreases
b. increases 55. Which of the following oxides is used as a whole
c. first decreases and then increases pigment?
d. first increases and then decreases a. ZnO b. FeO c. NiO d. CuO

@iitjeehelps
BITSAT Archives
1. Mercury is a liquid metal because [2014] a. oxide ore b. sulphide ore
a. it has a completely filled d -orbital that causes c. carbide ore d. Not an ore
d - d overlapping 6. When potassium ferrocyanide crystals are heated with
b. it has completely filled d -orbital that prevents d -d conc. H2SO4 ,the gas evolved is [2011]
overlapping
c. it has a completely filleds -orbital a. SO2 b. NH3 c. CO2 d. CO
d. it has a small atomic size 7. van-Arkel method is based on [2010]
2. When a solution of potassium chromate is treated a. cupellation method b. furnace refining method
with an excess of dilute nitric acid, [2013] c. poling method d. None of the above
a. Cr 3+ and Cr2O2-
7 are formed 8. Oil paintings turn blackish after some time. The salt
b. Cr2O72- and H2O are formed formed is [2010]
c. CrO2-4 is reduced to Cr
3+ a. SnS b. CuS c. PbS d. CdS
2-
d. CrO4 is oxidised to Cr2O2- 7 9. The extraction of which of the following metals
involves bessemerisation? [2009]
3. In the extraction of Ag, Zn is removed from (Zn-Ag)
alloy through [2013] a. Fe b. Ag c. Al d. Cu
a. cupellation b. fractional crystallisation 10. A metal X on heating in nitrogen gas gives Y . Y on
c. distillation d. electrolytic refining treatment with water gives a colourless gas which
when passed through CuSO4 solution gives a blue
4. Green vitriol is [2012]
colour. Y is [2008]
a. ferrous sulphate b. tin oxide
a. Mg(NO3 )2 b. Mg3N2 c. NH3 d. MgO
c. zinc oxide d. ferrous carbonate
5. Pyrolusite is a/an [2011]

Answer with Solutions


Practice Exercise 7. (c) Cu has +2 oxidation state in [Cu(NH3 )4] 2+ .
Electronic configuration of Cu2+ ( Z = 29) is
1. (d) Scandium is the first element of 3d series and
possesses largest atomic radii because atomic radii [Ar]18 3d 9 4s 0
decrease on moving along the period.
Number of unpaired electrons (n ) = 1
2. (a) Zinc contains completely filled d -orbitals and exhibit Magnetic moment = n(n + 2) BM = 1(1 + 2) = 1.73 BM
stable electronic configuration, therefore it does not easily
lose their electrons and hence, does not exhibit variable 8. (a)Interstitial compounds have high melting points in
valency. comparison to pure metals. They are very hard and retain
metallic conductivity. They are chemically inert.
3. (d) Zinc, cadmium and mercury of group 12 have full
d 10 configuration in their ground state as well as in their 9. (d)
common oxidation states and thus, are not regarded as 10. (d) Fe is oxidised, Fe ¾® Fe2+ + 2e -
transition metals. D
11. (c) Fe2(SO4 )3 ¾¾ ® Fe2O3 + 3SO3
4. (b) The ground state electronic configuration of Ti is
[ Ar] 3d 2 4s 2 12. (d) Conc. HNO3 renders iron passive by forming a thin
protective film of Fe3O4 on its surface.
5. (b)
13. (d) Haematite (Fe2O3)
6. (d) The magnetic moment is determined by the number of
unpaired electrons and is calculated by using the Garnierite is hydrated double silicate of Mg and Si having
‘spin-only’ formula, i.e. m = n(n + 2) BM 5-8% Ni. Pentalandite (NiS) ore associated with copper
and iron pyrite.
where, n is the number of unpaired electrons and m is the 14. (c) Iron is covered by a layer of hydrated iron oxide
magnetic moment in units of Bohr Magneton (BM). (Fe2O3 ×x H2O), i.e. rust on exposure to moist air.
A single unpaired electron has a magnetic moment of 15. (c)The iron produced in the blast furnace is known as pig
1.73 Bohr Magneton (BM). iron.

@iitjeehelps
596 SELF STUDY GUIDE BITSAT

16. (a) H2O is a neutral ligand but NO possesses +1 oxidation 35. (a) 5Fe2+ + MnO4- + 8H+ ¾® Mn2+ + 4H2O + 5Fe3+
state in the compound of Fe. 36. (d) MnO2 + 2KOH + [O] ¾® K 2MnO4 + H2O
Hence, oxidation state of Fe in the given compound is K 2MnO4 produced is green in colour.
calculated as
37. (c) MnSO4 is light pink.
x + 0 + 1 = 2 ; x = +1
17. (a) Fe 2+
has maximum number of unpaired electrons. 38. (c) It is +7.
2+ + In others, (a) + 6 (b) + 4 (d) + 8 / 3
18. (b) 3Fe + NO–3 + 4H ¾® NO + 3Fe 3+
+ 2H2O
39. (d) KMnO4 is used in analytical chemistry, as a favourite
Fe 3+
+ NO + 5H2O ¾® [Fe(H2O)5 NO] 2+ oxidant in preparative organic chemistry in bleaching of
(Brown complex)
wool, cotton and silk.
FeSO4 + NO ¾® FeSO4 × NO
40. (a) On addition of small amount of KMnO4 to conc. H2SO4,
19. (c) Copper can be extracted from malachite a greeny oily compound is obtained which is highly
CuCO3 × Cu(OH)2. explosive in nature. This compound is Mn2O7.
20. (a) The aqueous solution of CuSO4 consists of the
41. (c) The element, Tm does not belong to actinoid series.
complex [Cu(H2O)4] 2+ ion which absorbs in orange-red
42. (d) Formation of + 4 state requires very high energy, thus
region and impart deep blue colouration to the solution.
incorrect.
21. (a)
43. (c) Lanthanoids in +3 oxidation state usually have
22. (d) 2[CuSO4 + 2KI ¾® CuI2 + K 2SO4] unpaired electron in f -subshell and impart characteristic
2CuI2 ¾® Cu2I2 + I2 colour in solid as well as in solution state due to f -f
transition (except lanthanum and lutetium).
2CuSO4 + 4KI ¾® Cu2I2 + 2K 2SO4 + I2
23. (c) Tetraamino cupric ion is formed. 44. (a) On moving from left to right across the lanthanoid
series, basicity of lanthanoid hydroxides decreases.
Cu+ + 4NH3 ¾® [Cu(NH3 )4] 2+
Deep blue
45. (b) The valence shell electronic configuration of the given
D
ions are
24. (d) 2Cu(NO3 )2 ¾¾ ® 2CuO + 4NO2 + O2 Yb = [Xe] 4 f 14 6s 2 Þ Yb2 + = [Xe] 4f 14 Þ No unpaired e -
25. (c) Hair dye contains silver nitrate. Eu = [Xe] 4 f 7 6s 2 Þ Eu2 + = [Xe] 4f 7 Þ 7 unpaired e -
26. (d) 2Ag + S2O23- ¾®
Ag2S2O3 , white ppt which readily Lu = [Xe] 4 f 145 d 1 6s 2Þ Lu3 + = [Xe] 4f 14 Þ No unpaired e -
changes to yellow, orange, brown and finally black due to Ce = [Xe] 4 f 1 5 d 1 6s 2Þ Ce4 + = [Xe] Þ No unpaired e -
the formation of silver sulphide.
Hence, Eu2 + is paramagnetic as it contains 7 unpaired
Ag2S2O3 + H2O ¾® H2SO4 + Ag2S (black) electrons.
27. (b) The reactions involved in the extraction of silver by 46. (a) The actinoids include the fourteen elements from
cyanide process are Th to Lr.
Ag2S + 4CN- + O2 ¾® 2[Ag(CN)2] - + SO2 …(i) 47. (b) Lanthanoids [Xe] 4f 1-14 5 d 0-1 6s 2
- 2–
2 [Ag(CN)2] + Zn ¾®[Zn(CN)4] + 2Ag …(ii) Actinoids [Rn] 5f 1-14 6d 0-1 7s 2
In the first reaction, sulphide is oxidised by oxygen to SO2 Lanthanoids and actinoids use core d and f -orbitals also to
and in the second reaction, silver ion is reduced by Zn show higher oxidation state. As actinoids have
to Ag. Therefore, O2 is oxidising agent and Zn is reducing comparatively low energy difference between f and
agent. d -orbitals, thus show more oxidation states.
28. (b) Ferrous ferrocyanide, Fe2[Fe(CN)6] is Turnbull’s blue. 48. (a) The f -block consists of two series of lanthanoids and
actinoids.
29. (d) Hg2Cl2 + 2NH3 ¾® HgNH2Cl + Hg ¯ + NH4Cl
White Black
49. (a)
30. (a) When steel is treated in the presence of NH3, iron 50. (b) Lanthanoid contraction is related to both atomic and
3+
nitride on the surface of steel is formed which imparts a ionic radii, M .
hard coating. This process is called nitriding. 51. (b) All lanthanoids except promethium are non-radioactive
31. (c) K 2Cr2O7 is used in leather industry and as an oxidant whereas all actinoids are radioactive in nature.
for the preparation of many azo compounds. 52. (a) Since, the d -orbitals are partially filled in transition
32. (b) The structures of chromate and dichromate ions are metal atoms/ions, they exhibit paramagnetism and not
tetrahedral and two tetrahedrals, respectively. diamagnetism.
33. (c) 53. (a) 2OH– + Cr2O2– 2–
7 ¾® 2CrO4 + H2 O

34. (b) The photographic industry relies on the special light 54. (a) Diammine means 2NH3 in the coordination sphere.
sensitive properties of AgBr. 55. (a)

@iitjeehelps
d- AND f-BLOCK ELEMENTS 597

BITSAT Archives
1. (b) The electronic configuration of mercury is[Xe] 4f 10, 5d 10, 6s 2. Its d -subshell is completely filled, thus, it prevents the
overlapping of d -orbitals
(d -d overlapping). Hence, it is liquid at room temperature.
2. (b) The reaction of K 2CrO4 with dilute nitric acid is represented as
2CrO24- + 2H+ ¾® Cr2O27- + H2O
3. (c)
4. (a) FeSO4 × 7H2O is known as green vitriol.
5. (a) Pyrolusite is MnO2. Thus, it is an oxide ore.
6. (d) K 4[Fe(CN)6] + 6H2SO4 + 6H2O ¾® 2K 2SO4 + FeSO4 + 3(NH4 )2 SO4 + 6CO
7. (d) van-Arkel method is based on vapour phase refining.
8. (c)
9. (d) In a bessemer converter, copper pyrites are oxidised to FeO and Cu2O. FeO is slagged off. Cu2O reacts with Cu2S left
unoxidised to give Cu.
2Cu2S + 3O2 ¾® 2Cu2O + 2SO2 ­
2Cu2O + Cu2S ¾® 6Cu + SO2 ­
10. (b) Only NH3 gas gives deep blue solution with CuSO4. NH3 is obtained by the re5action of nitride of metal with water.
6H O
3Mg + N2 ¾® Mg3N2 ¾ ¾¾
2
® 2NH3 ­ + 3Mg(OH)2
X Y Colourless

CuSO4 + 4NH3 ¾® [Cu(NH3 )4]SO4


Deep blue colour

@iitjeehelps
20
Coordination Compounds
and Organometallics

Complex or Coordination Compounds


These are those molecular compounds which retain their identity in solid as well as in aqueous
solution. In these compounds, metal atoms are bound to a number of anions or neutral molecules by
coordinate bonds. A part of these compounds is not dissociated in solution and its behaviour is
different than its constituents.
e.g. K 4 [Fe(CN)6 ] + H 2O ¾® 4K + +[Fe(CN)6 ]4 -
Ionisable Non-ionisable Central metal
species coordination atom ligands
sphere

Important Terms
1. Complex Ion
It is an electrically charged species in which central metal atom or ion is surrounded by number of
ions or molecules.

2. Ligands
The ions or molecules bound to the central atom/ion in the coordination entity are called ligands.

Types of Ligands
(i) Unidentate ligand It is bound to a metal ion through a single donor atom.
e.g. H 2O, NH3 , CO, Cl - , NH 2- etc.
(ii) Didentate ligand It is bound to a metal ion through two donor atoms.
··
e.g. COO- CH 2 ¾ NH 2
½ ½ ··
COO- CH 2 ¾ N H 2
Oxalate ion
Ethylene diamine

@iitjeehelps
COORDINATION COMPOUNDS AND ORGANOMETALLICS 599
(iii) Polydentate ligand It is bound to a metal ion through 3. Neutral Complex [Pt(NH 3 )2 Cl(NO2 )]
several donor atoms, e.g. ethylene diamine tetra 2 1
acetate ion [EDTA] 4 - . Diamminechloronitrito-N-platinum (II)
(iv) Ambidentate ligands It can ligate through two ● Name of ligands in alphabetical order.
different atoms. ● Central metal atom and its oxidation state.
e.g. ¾ NO2 , ¾ ONO, ¾ SCN, ¾ NCS etc. NOTE The oxidation state of Fe is + 2 in sodium nitroprusside,
Na 2 [Fe(CN)5 NO] and it has the ligand nitrosonium ion, NO + .
(v) Chelate ligands may be di or polydentate ligands which
form closed ring with central metal ion. Closed ring is When Fe 2+ (aq) is added to K 3 [Fe(CN)6 ], the product obtained
known as chelate ring and this process is called is Turnbull’s blue but on addition of Fe 3+ (aq) to K 4 [Fe(CN)6 ],
chelation. the product Prussian blue is obtained.
More the number of chelate rings in a complex,
complex will be more stable. The number of such Isomerism in Coordination
ligating groups is called the denticity. Compounds
3. Coordination Number It is a phenomenon, in which compounds have the same
The number of chemical bonds formed by the ligands with molecular formula but different physical and chemical
central metal atom or ion is called the coordination number properties on account of different structures. These
of that complex. compounds are called isomers.
In case of monodentate ligand, it is equal to the number of 1. Stereo Isomerism
ligands while in bidentate ligand it is equal to the
It occurs due to different arrangements of ligands around
2 ´ number of ligands present in the complex.
central metal atom.
4. Coordination Sphere It is of two types, i.e. geometrical isomerism and optical
The central metal ion and the ligands attached to it are isomerism.
enclosed in square bracket which is known as coordination (i) Geometrical Isomerism It arises in heteroleptic
sphere. The ionisable group written outside the bracket is complexes due to different possible geometric
known as counter ions. arrangements of the ligands. Important examples of
5. Oxidation Number of Central Atom this behaviour are found in square planar and
octahedral complexes.
The charge of the complex if all the ligands are removed
along with the electron pairs that are shared with the (a) Square planar complex of formula[ MX 2 L2 ]
central atom, is called oxidation number of central atom. (X and L are unidentate)
Two X ligands may be arranged adjacent to each
Nomenclature of Coordination other in a cis-isomer or opposite to each other in a
trans-isomer.
Compounds
e.g. [Pt(NH3 )2 Cl 2 ]
1. Cationic Complex [Co (NH 3 )4 Cl(NO2 )]NO 3 (b) Square planar complex of the type [MABXL]
2 1 3
(where A, B, X, L are unidentate) shows three
Tetraamminechloronitrocobalt (III) nitrate
isomers, two cis and one trans. Such isomerism is
● Prefixes mono, di, tri etc.,…… are used to indicate the
number of the individual ligands and ligands are named not possible for tetrahedral geometry.
in alphabetical order. e.g. [Pt(NH3 )(Br)(Cl)(Py)]
● Central metal atom and its oxidation state is indicated by (c) Square planar complex of type M( XL) 2 , here, XL is
Roman numeral in parenthesis. unsymmetrical didentate ligand, e.g. [Pt (gly )2 ]
● Name of ionisable anion. (d) Octahedral complexes of formula [ MX 2 L4 ] in
2. Anionic Complex K 3 [Fe(CN)6 ] which the two ligands X may be oriented cis or
1 3 2 trans to each other, e.g. [Co(NH3 )4 Cl 2 ]+ .
Potassiumhexacyanoferrate (III) (e) Octahedral complexes of formula [ MX 2 A2 ] (where
● Name of ionisable metal. X are unidentate ligands and A are didentate
● Name of ligand in alphabetical order. ligands) form cis and trans-isomers, e.g.
● Central metal atom + ate and oxidation state. [CoCl 2 (en)2 ].

@iitjeehelps
600 SELF STUDY GUIDE BITSAT

(f) Octahedral coordination entities of the type (iii) Primary valencies are ionisable and are
[ Ma3b3 ] like [Co(NH3 )3 (NO2 )3 ], If three donor atoms non-directional while secondary valencies are
of the same ligands occupy adjacent positions at non-ionisable and directional. Therefore, geometry of
the corners of an octahedral face, we have the complex is decided by secondary valencies.
facial (fac) isomer. When the positions are around e.g. [Cr(H 2O)6 ]Cl3 has primary valency = 3 (ON of Cr)
the meridian of the octahedron, we get the and secondary valency = 6 (CN of Cr)
meridional (mer) isomer.
NOTE Tetrahedral complexes do not show geometrical isomerism.
Octahedral complexes of type [M(AA )3 ] , MA 6 and MA 5B do
EAN (Effective Atomic Number) Rule
not show geometrical isomerism. MA 2 X 2 Y2 shows both optical The EAN is calculated by using following formula :
and geometrical isomerism. EAN = Z - x + 2c
(ii) Optical Isomerism It arises when mirror images where, Z = atomic number of metal
cannot be superimposed on one another. These mirror
x = number of electrons lost by the metal in complex
images are called enantiomers. The two forms are
formation (oxidation number)
called dextro (d) and laevo (l).
Optical isomerism is common in octahedral complexes c = number of pairs of electrons accepted by metal
having atleast one didentate ligand. (coordination number)
e.g.[Co(en)3 ]3 + , [PtCl 2 (en)2 ]2+ etc. Complex EAN ( Z - x + 2c ) e - in next inert gas

(Complexes whose central atom obeys EAN rule)


2. Structural Isomerism
In structural isomerism, isomers have different bonding [Pd(NH 3 ) 6 ]4 + 46 - 4 + 2 ´ 6 = 54 54 (Xe)

pattern. K 4 [Fe(CN) 6 ] 26 - 2 + 2 ´ 6 = 36 36 (Kr)

Different types of structural isomerism is as follows :


Valence Bond Theory
(i) Linkage isomerism It arises in a coordination According to this theory, the metal atom or ion under
compound containing ambidentate ligand. the influence of ligands form inner orbital and outer
e.g.[Co(NH3 )5(NO2 )]Cl 2 and [Co(NH3 )5(ONO)]Cl 2 orbital complex. These hybridised orbitals are allowed to
(ii) Coordination isomerism It arises from the interchange overlap with ligand orbitals that can donate electron pairs
of ligands between cationic and anionic entities of for bonding.
different metal ions present in a complex. 1. Six Ligands (unidentate) Octahedral Entity
e.g. [Co(NH3 )6 ][Cr(CN)6 ] and [Cr[NH3 )6 ][Co(CN)6 ] Such complexes are of the following two types :
(iii) Ionisation isomerism When the ionisable anion (i) Inner orbital complexes (hyperligated complexes)
exchange with anion ligand. which are formed due to strong field ligands or low
e.g. [Co(NH3 )5SO4 ]Br and [Co(NH3 )5Br]SO4 spin ligands, has hybridisation d 2sp3 and shape of
(iv) Solvate isomerism It is also known as “hydrate complex will be octahedral.
isomerism”. In this case water is involved as a solvent. (a) Inner orbital complex [Co(NH 3 )6 ]3+
e.g.[Cr(H 2O)6 ]Cl3 , [Cr(H 2O)5Cl]Cl 2 × H 2O and orbitals of Co3+ ion

[Cr(H 2O)4 Cl 2 ]Cl × 2H 2O


3d 4s 4p

Bonding in Coordination Compounds d 2sp3-hybridised orbitals of Co3+

The bonding formation in coordination compounds can be


explained by using the two approaches as given below : 3d 4s 4p
Werner’s Theory [Co(NH3 )6]3+
(i) In complex compounds, metal atom exhibit two types d 2sp3-hybridised
of valencies—primary valency and secondary valency. orbitals of Co3+ ×× × ×××
(ii) Primary valencies are satisfied by anions only while Six pairs of
electrons from
secondary valencies are satisfied by ligands. Primary six NH3 molecules
valency depends upon oxidation number of central
metal atom while secondary valency represents the All electrons are paired, therefore complex will be
coordination number of central metal atom. diamagnetic in nature.

@iitjeehelps
COORDINATION COMPOUNDS AND ORGANOMETALLICS 601
(ii) Outer orbital complexes (hypoligated complexes) Limitations of Valence Bond Theory
which are formed due to weak field ligands or high
spin ligands, has sp3d 2 -hybridisation . It suffers from the following short comings:
- - - - - 2- ● It involves a number of assumptions.
Generally, halides (F , Cl , Br , I ), SCN , S form
● It describes bonding in coordination compounds only
outer orbital complexes and other ligands form inner
qualitatively.
orbital complexes.
● It does not describe the detailed magnetic properties of

e.g. coordination compounds.


(b) Outer Orbital Complex [CoF6 ]3- ● It does not give a quantitative interpretation of the

thermodynamic or kinetic stabilities of coordination


Orbitals of Co3+ ion
compounds.
● It does not make exact predictions regarding the tetrahedral
3d
and square planar structures of 4-coordinate complexes.
● It does not distinguish between weak and strong ligands.

4s 4p 4d Crystal Field Theory (CFT)


sp3d 2-hybridised orbitals of Co3+
The spliting of five d-orbitals into lower and higher energy
sp3d 2-hybridised
levels due to approach of ligands, is known as crystal field
3–
[CoF6] theory. The five d-orbitals in a gaseous metal atom/ion
orbitals of Co3+in
3d have same energy.
× ××× ×× Crystal Field Splitting in Octahedral
4s 4p 4d
Coordination Entities
Six pairs of electrons
from six F– ions
● Energy separation is denoted by D o (the subscript o, is for
octahedral). This is also known as crystal field splitting
Complex has unpaired electrons, therefore it will be energy (CFSE).
paramagnetic in nature. ● The energy of the two e g orbitals (higher energy orbitals)
will increase by (3/5) D o and that of the three t 2 g (lower
2. Four-ligands (unidentate), Tetrahedral and energy orbitals) will decrease by (2/5) D o .
Square Planar Entity
● If D o < P , the fourth electron enters one of the e g , orbitals
Tetrahedral complexes can also form inner orbital or outer
giving the configuration t 32 g e g1 . Ligands for which D o < p
orbital complexes, characteristics of which are shown below :
are known as weak field ligands and form high spin
Generally, halide (F - , Cl - , Br - , I - ) ligands, [Ni(CO)4 ], complexes.
[Co(CO)4 ], [Zn(NH3 )4 ]2+ complexes form outer orbital Energy
dx2 – y2, dz2
complexes and other form inner orbital complexes, e.g. eg

Type of Geometrical d-orbitals CN Common


3/5
hybridisation shape used examples
∆o
sp 3 Tetrahedral — 4 [Zn(NH 3 ) 4 ]2+
Average energy
[Ni(CO) 4 ] Metal 2/5
of the d-orbitals in
d-orbitals spherical crystal field
dsp 2 / sp 2 d Square planar dx2 - y 2 4 [Ni(CN) 4 ]2- t2g
dxy, dxz, dyz
sp 3 d Trigonal dz 2 5 [Ni(CN) 5 ]3 - dx2 – y2, dz2, dxy, dxz, dyz
Splitting of d-orbitals
bipyramidal Free metal ion in octahedral crystal field
d 2 sp 3 Octahedral dz 2 , dx 2 - y 2 6 [Fe(CN) 6 ]3 - ● If D o > P , it becomes more energetically favourable for
inner orbital
the fourth electron to occupy a t 2 g orbital with
complex
configuration t 24g e g . Ligands which produce this effect are
sp 3 d 2 Octahedral dx2 - y 2 6 [Co(H 2O) 6 ]2+
known as strong field ligands and form low spin
counter dz 2 [FeF6 ]3 - ,
complexes.
orbitals
complex [CoF6 ]3 - ● Where, P represents the energy required for electron
pairing in a single orbital (pairing energy).

@iitjeehelps
602 SELF STUDY GUIDE BITSAT

Crystal Field Splitting in Tetrahedral Application of Coordination


Coordination Entities
In tetrahedral coordination entities, Dt = ( 4/ 9 )D o .
Compounds
Consequently, the orbital splitting energies are not sufficiently Estimation of Hardness of Water
large for forcing pairing and therefore, low spin configurations EDTA is used in the estimation of hardness of water
are rarely observed. Due to less crystal field stabilisation energy, by volumetric technique as it is hexadentate ligand
it is not possible to pair electrons and so all the tetrahedral and forms complex with various metal ions such as
complexes are high spin. Ca 2+ and Mg 2+ .
Spectrochemical Series
Quantitative Analysis of Metal Ions
An arrangement of ligands in order of increasing crystal field
(i) Separation of group II B sulphides from group II
strength is spectrochemical series.
A sulphide is based upon the fact that sulphides
I - < Br - < Cl - < F - < H 2O < C 2O42- < NH3 < en < NO2- < CN - < CO of group II B forms complex sulphides with
yellow ammonium sulphide that are soluble in
Magnetic Properties
water.
The diamagnetic (repulsion from the applied magnetic field due
As2S5 + 3(NH 4 )2 S ¾® 2(NH 4 )3[ AsS4 ]
to the presence of paired electrons) and paramagnetic (attraction 2+
towards the applied magnetic field due to the presence of (ii) Cu ion is detected by the formation of deep
unpaired electrons) behaviour of coordination compound is blue coloured tetraamine copper sulphate
explained on the basis of crystal field theory. complex [Cu(NH3 )4 ] SO4 by the addition of NH3
to Cu 2+ ions solution.
e.g. Co3 + ion shows diamagnetic properties in [Co(NH3 )6 ]3 +
complex ion but in [CoF6 ]3 - , it shows paramagnetic properties. In Extraction of Metals
[Co(NH3 )6 ]3 + complex ion, NH3 acts as a strong ligand which
The property of the formation of coordination
causes pairing of electrons.
compounds by the metals is used in the extraction of
In this case, energy required to pair up the electrons is smaller metals like Ag and Au.
than D o . Whereas in [CoF6 ]3 - complex, F - acts as a weak ligand
2 K[Ag(CN)2 ] + Zn ¾® K 2[Zn(CN)4 ] + 2Ag
that does not allow the pairing of electrons as energy required to Silver
pair the electrons is higher than D o . Complex formed by nickel with carbonyl [Ni(CO) 4 ] is
used for the extraction and purification of nickel.
eg
Heating
Ni + 4CO ¾® [Ni(CO)4 ] ¾¾¾® Ni + 4CO
(Associated with
∆ o ∆o < P other elements)

In Biological System
t2g Cyclic ligands impart extra stability to coordinate
compounds. Thus, Mg in chlorophyll and iron in
Colour of the Complexes
haemoglobin are stable towards dissociation due to
The electrons can be excited from lower energy to higher energy the formation of coordination compounds.
state by the absorption of even lower energy radiations. Thus, due
Haemoglobin of the blood gets oxygenated through
to d-d transition of the electrons, coordination compounds show
the binding of dioxygen, O2 to the ferrous ions in
different colours.
haemoglobin.

Stability Constants Organometallics


Stability of a complex can be expressed in terms of stability
Compounds, in which a metal atom is directly
constant, k. If the complex is MLn and b n is the overall formation attached with carbon atom of an organic compound
constant, then or radical, are called organometallics. These
M + nL r MLn compounds are broadly divided into three types,
[ MLn ] depending upon the bonding.
bn = = k1 ´ k2 ´ k3 K kn (a) s-bonded Organometallics Grignard reagent
[ M ][ L]n
( R — Mg)X , TEL (C 2H 5 )4 Pb, TMS (tetramethyl
where, k1 , k2 , K kn are called stepwise formation constants. silane), (CH3 )4 Si, (C 2H 5 )2 Zn etc., are s-bonded
[Alternatively 1 / k is known as instability constant. organometallic compounds.

@iitjeehelps
COORDINATION COMPOUNDS AND ORGANOMETALLICS 603
(b) p-bonded Organometallics These are formed mainly Some Important Organometallics
by transition metals. Examples are ferrocene,
(i) Grignard reagent ( RMgX ) is very useful in organic
[Fe(h2 — C 5H 5 )2 ], dibenzene chromium[Cr(h6 — C6H6 )2 ],
synthesis.
Zeisse’s salt K[Pt(h2 — C 2H 4 )Cl3 ] etc.
(ii) Zeigler-Natta catalyst (TiCl 4 and aluminium) is
The prefixes h2 , h6 and h2 indicate that 2, 6 and
2-carbon atoms are bound to the metal respectively. used in catalysing polymerisation.

(c) s-and p-bonded Organometallics Transition metal (iii) TEL is used as antiknock compound in petroleum.
carbonyls are of this type, e.g.[Ni(CO)4 ],[Fe(CO)5] etc.

Practice Exercise
1. According of Lewis, the ligands are 10. Which of the following compound would exhibit
a. acidic in nature coordination isomerism?
b. basic in nature a. [Cr(H2O)6]Cl3 b. [Cr(NH3 )6][Co(CN)6]
c. neither acidic nor basic c. [Cr(en)2]NO2 d. [Ni(NH3 )6][BF4] 2
d. some are basic and others are acidic
11. The planar complex Mabcd gives
2. Which of the following represent hexadentate ligand? a. two optical isomers
a. 2, 2-bipyridyl b. DMG b. two geometrical isomers
c. Ethylenediammine d. None of these c. three optical isomers
d. three geometrical isomers
3. In CuSO4 × 5H2O, copper is coordinated to
a. 5 water molecules b. 4 water molecules 12. Which will not show geometrical isomerism?
c. one sulphate ion d. one water molecule a. [Co(NH3 )4 Cl2] + b. [Co(en)3] 3+
c. [Co(en)2 Cl2]Cl d. [Cr(NH3 )4 Cl2]Cl
4. Coordination number of Cr is six. A complex ion of Cr
with C2O2– –
4 , en and superoxide ion, O2 has the 13. Geometrical isomerism is shown by

formula, [Cr(C2O4 )x (en)y (O2 )z ] . The ratio x:y:z will a. [Ni(NH 3) 5Br] + b. [Cr(NH 3) 4(en)] 3+
3+
be c. [Co(en) 3] d. [Co(NH 3) 2 (en) 2] 3+
a. 1:1:1 b. 1:1:2 c. 1:2:2 d. 2:1:1 14. Optical isomerism is exhibited by
5. The oxidation number of Fe in K 4 [Fe(CN)6] is a. [Cr(NH 3) 6] 3+ b. [Cr(en)(H 2O) 4] 3+
a. 2 b. 3 c. 0 d. 1 c. trans-[Cr(en) 2Cl 2] + d. [Cr(en) 3] 3+

6. Identify the species with an atom in +6 oxidation state 15. The number of isomers of [Cu¢¢ (NH 3 ) 4 ] [Pt ¢¢Cl 4 ] are
present in the following complexes. a. 6 b. 3
a. CrO2Cl2 b. [Cr(CN) 6] 3- c. 4 d. 5
c. [NiF 6] 2- d. MnO -4 16. Amongst the following pair of enantiomers is given by
7. The oxidation state of cobalt in a. [Cr(NH 3) 6] [Co(CN) 6]
b. [Co(en) 2Cl 2] Cl
é ƒ NH ‚ ù c. [Co(NH 3) 4Cl 2] NO 2
ê (NH3 )4 Co ‚ Co(NH3 )4 ú(NO3 )4 is
ë û d. [Pt(NH 3) 4] [PtCl 6]
NO2 ƒ
a. 2 b. 3 c. 4 d. 6 17. [Co(NH3 )4 Cl2]NO2 and [Co(NH3 )4 Cl(NO2 )]Cl exhibit
8. The coordination number of Co in [Co(NH 3 )5 Cl]Cl 2 is which type of isomerism?
a. Geometrical b. Optical
a. 2 b. 5 c. 6 d. 1 c. Linkage d. Ionisation
9. The type of isomerism shown by [Co(en)2(NCS)2]Cl 18. The difference in colour is due to
and [Co(en)2(NCS)Cl] NCS is a. optical isomerism
a. coordination b. geometrical isomerism
b. ionisation c. linkage isomerism
c. linkage d. nuclear isomerism
d. All of the above

@iitjeehelps
604 SELF STUDY GUIDE BITSAT

19. EAN of iron in K 4 [Fe(CN)6] is 30. The value for crystal field stabilisation energy for an
a. 36 b. 34 c. 38 d. 40 octahedral complex, [CoCl 6] 4 - is 18000 cm -1. The
20. Which of the following complexes will give white value for crystal field stabilisation energy for
precipitate with BaCl2 solution? tetrahedral complex is
a. [Cr(NH3 )5 SO4]Cl b. [Co(NH3 )4 Cl2]NO2 a. 16000 cm -1 b. 8000 cm -1
c. [Cr(NH3 )5 Cl]SO4 d. Both (a) and (c) c. 20000 cm -1 d. 18000 cm -1

21. Sodium nitroprusside when added to an alkaline 31. The colour of the coordination compounds depends
solution of sulphide ions produces purple colouration on the crystal field splittings. What will be the correct
due to the formation of order of absorption of wavelength of light in the visible
a. Na[Fe(H2O)5 NOS] b. Na 4[Fe(CN)5 NOS] region, for the complexes, [Co(NH3 )6] 3 + , [Co(CN) 6] 3 -
c. Na 3[Fe(CN)5 NOS] d. Na 4[Fe(H2O)5 NOS] and [Co(H2O)6] 3 + ?
22. Which of the following has highest conductivity in a. [Co(CN) 6] 3- > [Co(NH 3) 6] 3+ > [Co(H 2O) 6] 3+
water? b. [Co(NH 3) 6] 3+ > [Co(H 2O) 6] 3+ > [Co(CN) 6] 3-
a. Fe3[Fe(CN)6] 2 b. K 3[Fe(CN)6]
c. Ag(NH3 )2 Cl d. [Cr(NH3 )6]Cl6 c. [Co(H 2O) 6] 3+ > [Co(NH 3) 6] 3+ > [Co(CN) 6] 3-
d. [Co(CN) 6] 3- > [Co(NH 3) 6] 3+ > [Co(H 2O) 6] 3+
23. How many moles of AgCl would be obtained, when
100 mL of 0.1 M Co(NH3 )5 Cl3 is treated with excess 32. Which of the following species is not expected to be a
of AgNO3 ? ligand?
a. 0.01 b. 0.02 a. No b. NH+4
c. 0.03 d. None of these c. NH2CH2CH2NH2 d. CO
24. High spin complex [FeCl6] 3 - has the d-configuration 33. Which one of the following compounds is not colured?
as a. Na 2[CuCl4]
a. t 23g eg2 b. t 25g c. t 22g eg3 d. eg5 b. Na 2[CdCl4]
c. [Cr(H2O)6]Cl3
25. Unpaired electrons in [FeCl6] 3 - and [Fe(CN)6] 3 - d. K 3[Fe(CN)6]
are respectively 34. Which of the following is paramagnetic?
a. 5 and 2 b. 5 and 1 c. 3 and 2 d. 2 and 3 a. K 4[Fe(CN)6]
2- b. K 3[Fe(CN)6]
26. Among the complexes, [Ni(CO) 4 ], [Ni(CN) 4 ] and
2-
c. Ni(CO)4
[NiCl 4 ] , d. [Co(NH3 )6]Cl3
a. [Ni(CO)4] is diamagnetic but [Ni(CO)4] and [Ni(CN)4] 2- 35. The stabilisation of coordination compounds due to
are paramagnetic chelation is called the chelate effect. Which of the
b. [Ni(CN) 4] 2- and [Ni(CO) 4] are diamagnetic but following is the most stable complex species?
[NiCl4] 2- is paramagnetic a. [Fe(CO)5]
c. [NiCl 4] 2- and [Ni(CO) 4] are diamagnetic but b. [Fe(CN)6] 3-
[Ni(CN)4] 2- is paramagnetic c. [Fe(C2O4 )3] 3-
d. [NiCl 4] 2- and [Ni(CN) 4] 2- are diamagnetic but d. [Fe(H2O)6] 3+
[Ni(CO)4] is paramagnetic 36. Which of the following is not an example of
27. Which of the following shell form an octahedral organometallic compound?
complex? a. Trimethyl boron
a. d 4 (low spin) b. d 2 (high spin) b. Trimethyl aluminium
c. d 6 (low spin) d. None of these c. Sodium exthoxide
d. Tetracarbonyl nickel
28. The geometry of the compound [Pt(NH3 )2 Cl2] is
37. Among the following, which is not the p-bonded
a. square planar
b. pyramidal
organometallic compound?
c. tetrahedral a. K[PtCl3(h 2 ¾ C2H4 )] b. Fe(h 5 ¾ C5H5 )2
d. octahedral c. (CH3 )4 Sn d. Cr(h 6 ¾ C6H6 )2
29. Which of the following is paramagnetic? 38. Which is not a p-bonded complex?
a. [Co(NH3 )6] 3+ a. Zeise’s salt
b. [Ni(CO)4] b. Ferrocene
c. [NiF4] 2- c. Dibenzene chromium
d. [Ni(CN)4] 2- d. Tetraethyl lead

@iitjeehelps
BITSAT Archives
1. Which of the following will not form optical isomers? 5. Ferrocene is an example of [2009]
a. [Co(en)3] 3+ [2014] a. sandwiched complex
b. [Co(NH3 )3(NO2 )3] b. pi-bonded complex
c. [Pt(en)2 Cl2] 2+ c. a complex in which all the five carbon atoms of
cyclopentadiene anion are bonded to the metal
d. [CrCl2(ox)2] 3- d. All of the above
2. The magnitude of Do will be highest in which of the 6. [Co(NH3 )5 SO4 ] Br and [Co(NH3 )5 Br] SO4 is a pair of
following complex. [2013]
a. [Cr(CN)6] 3- b. [Cr(H2O)6] 3+ …… isomers. [2008]
a. ionisation b. ligand
c. [Cr(NH3 )6] 3+ d. [Cr(C2O4 )3] 3-
c. coordination d. hydrate
3. Which of the following is an outer d-orbital or high spin 7. The crysal field splitting energy for octahedral ( Do )
complex? [2013]
and tetrahedral ( Dt ) complexes is related [2006]
a. [Co(NH3 )6] 3+ b. [Ni(CN)4] 2- c. [NiCl4] 2- d. [CoF6] 3-
4
a. Dt = Do
4. Which of the following is correct IUPAC name for 9
K 2[Cr(CN)2 O2(O)2 NH3 ] ? [2012] 1
b. Dt = Do
a. Potassium amminecyanoperoxodioxo-chromatic (IV) 2
b. Potassium amminecyanoperoxodioxo-chromium (V) c. Do = 2Dt
Potassium amminecyanoperoxodioxo-chromium (VI) 4
c. d. Do = Dt
d. Potassium amminedicyanodioxoper-oxochromate (VI) 9

Answer with Solutions


Practice Exercise 6. (a) The oxidation state of Cr in CrO 2Cl 2 is +6.

1. (b) All the ligands are Lewis bases. é NH ù


·· 7. (b) ê(NH3 )4 Co Co(NH3 )4 ú (NO3 )4
ê ú
2. (d) H3C — C == NOH ë NO2 û
½ ··
Q ( 4 ´ 0) + x + (1 ´ - 1) + (1 ´ - 1) + x + ( 4 ´ 0) + ( 4 ´ -1) = 0
H3C — C == NOH
\ x =+3
(DMG bidentate)
8. (c) In the complex [Co(NH 3) 5Cl]Cl 2, total number of the
ligands linked to the central atom, cobalt is 6. Therefore,
N N the coordination number of Co is 6.
2, 2-bipyridyl 9. (b) These complexes show ionisation isomerism as they
(bidentate)
give different ions in solution.
··
H2C— NH2 10. (b) Coordination isomerism is shown by the complexes,
½ ·· which have at least 2 metal atoms so that their ligand may
H2C — NH2 interchange.
Ethylene diammine (bidentate) 11. (d) Madcd type complex have three geometrical isomers.
3. (b) Blue vitriol [CuSO4 × 5H2O] is a complex compound in
which the coordination number of copper is 4, i.e. Cu2+ is 12. (b) [Co(en)3] 3+ cannot show geometrical isomerism, in it,
attached to 4 water molecules. Hence, it can be correctly Co-atom is linked with 3-same ligands.
written as [Cu(H2O)4]SO4 × H2O. 13. (d) Geometrical isomerism is exhibited by octahedral
complexes like [Co(NH 3) 2(en) 2] 3+ .
4. (b) The ratio of x,y and z be such that, total charge = –1
3 + 1( -2) + 1´ 0 + 2( -1) = -1 14. (d) Optical isomerism is exhibited by [Cr(en) 3] 3+ .
The ratio of x : y : z = 1 : 1 : 2 15. (a) The possible number of isomers are,
5. (a) K 4[Fe(CN)6] (i) [Cu (NH3 )4][PtCl4]
4 ´ ( +1) + x + 6 ( -1) = 0
(ii) [Cu (NH 3 )2Cl 2]×[Pt Cl 2(NH 3 )2] cis
4+x -6=0
x =2 (iii) [Cu(NH3)2 Cl2] × [Pt Cl2(NH3 )2] trans

@iitjeehelps
606 SELF STUDY GUIDE BITSAT

(iv) [Cu(NH3 )3 Cl] [Pt Cl3(NH3 )], unpaired electrons). d 6 can give arise to both low spin
(v) [Cu(NH3 )Cl3][Pt Cl(NH3 )3] and and high spin octahedral complexes in presence of strong
and weak ligands respectively.
(vi) [CuCl4][Pt(NH3 )4]
28. (a) The geometry of complex will be square planar.
16. (b) Pair of enantiomers are non-superimposable mirror
images. The complex [Co(en) 2Cl 2]Cl exhibits a pair of 29. (c) [NIF4] 2- has 2 unpaired electrons, hence it shows
enantiomers. paramagnetic behaviour.
17. (d) Ionisation isomerism is shown. 4
30. (b) The CFSE for tetrahedral complex is Do where Do is
18. (c) The difference in colour is due to the formation of 9
linkage isomers. CFSE for octahedral complexes. The value of Do is
18000 cm-1, therefore, the CFSE for tetrahedral complex
19. (a) EAN = z - x + 2c = 26 - 2 + 2 ´ 6 = 36 4
is of 18000 cm-1 that is 8000 cm-1.
20. (c) Only [Cr(NH3 )5 Cl]SO4 gives sulphate ion. 9
[Cr(NH3 )5 Cl]SO4 + BaCl2 ¾® [Cr(NH3 )5 Cl]Cl2 + BaSO4 31. (c) As we know that, strong field ligand split the five
White ppt. degenerate energy levers with more energy separation
21. (b) Na 2S + Na 2[Fe(CN)5 NO] ¾® NH4[Fe(CN)5 NOS] than weak field ligand, i.e. as strength of ligand increases
crystal field spliting energy increases.
Sodium nitroprusside Purple
hc
H2O Hence, DE =
22. (d) [Cr(NH3 )6]Cl6 ¾® [Cr(NH3 )6] 3+
+ 6Cl- l
This complex gives 7 ions in solution. Conductively of an 1 1
Þ DE µ Þ lµ
electrolyte is directly proportional to number of ions l DE
produced. Hence, the conductivity of this complex will be As energy separation increases, the wavelength
the maximum. decreases.
23. (b) Co(NH3 )5 Cl3 is an octahedral complex ionising in Thus, the correct order is
aqueous solution as [ Co ( H2O)] 3+ > [ Co(NH3 )6] 3+ > [ Co((CN)6] 3-
[Co(NH3 )5 Cl] Cl2 ¾¾® [Co(NH3 )5 Cl] 2+ +2Cl- Here, strength of ligand increases, DE inceaes CFSE
0.1 increases and l absored decrease.
Moles of [Co(NH3 )5 Cl] Cl2 = ´ 100 = 0.01
1000 32. (b) Ligand must donate a pair of electron or loosely held
Moles of AgCl formed = 2 ´ moles of Cl– in complex electron pair to metal and form a M ¾L bond.
· · · · · ·
= 0.01 ´ 2 = 0.02 e.g. ·
N == O, N H2 CH2CH2 N H2 , ··CO
·
24. (a) In [FeCl6] 3- complex, Cl- is a weak field ligand. It do +

not cause pairing of electrons in 3d subshell of Fe3+ . Among N H4 does not have any pair of electron.
+
Hence, configuration is t 23g eg2. Hence, N H4 is not a ligand.
-
25. (b) In the formation of FeCl3-
6 , Cl do not cause pairing 33. (b) Those complexes, in which at least one unpaired
because it is a weak field ligand; number of unpaired electron is found, show colour due to d -d transition of
electrons are five. electrons. The complex Na 2[CdCl4] has no unpaired
electron in Cd-atom. Hence, it is colourless as no
On the other hand, in Fe (CN) 63- complex, CN- is a
d -d transition can take place.
strong field ligand, it causes pairing so the number of
unpaired electron is one. 34. (b) K 3[Fe(CN)6] (potassium ferricyanide) has an unpaired
2-
26. (b) The oxidation state of Ni in [Ni(CN) 4] and [NiCl 4] 2- electron in Fe-atom, hence it show paramagnetic
is +2 while in [Ni(CO) 4] it is in zero oxidation state. behaviour.
Since, CO and CN - are strong ligands therefore, they 35. (c)
cause pairing of the unpaired electrons. 36. (c) Sodium ethoxide is not an organometallic compound
Therefore, [Ni(CO)4] and [Ni(CN)4] 2- are dimagnetic. as in it sodium metal is attached to oxygen and not to
Whereas, Cl - is a weak ligand that does not cause pairing carbon.
of electrons, resulting in paramagnetic complex. Hence, C2H5 — O- Na +
[NiCl4] 2– is paramagnetic.
37. (c) (CH3 )4 Sn is a s-bonded organometallic compound.
27. (c) d 1, d 2, and d 4 configurations cannot give arise to
octahedral complexes, as octahedral complexes are 38. (d) Zeise’s salt = K[Pt(n2 - C2H4 )Cl3]
formed either by d 2sp 3-hybridisation in presence of strong Ferrocene = [Fe(h 5 - C5H5 )2
ligands, called low spin complex as they have dibenzene chromium = [Cr(h 6 - C6H6 )2]
comparitively less unpaired electrons) or by
sp 3d 2-hybridisation (in presence of weak ligands, called These three are p-bonded organometallics only TEL
high spin complex as they have comparitively more (C2H5 )4 Pb is a s-bonded organometallic.

@iitjeehelps
COORDINATION COMPOUNDS AND ORGANOMETALLICS 607

BITSAT Archives
1. (b) Optical isomerism is shown by only those complexes Here, F - is a weak ligand so no pairing of electron takes
which lack symmetry. Complex [Co(NH3 )3(NO2 )3] shows place
facial and meridional isomerism. Both isomers of this
complex contain plane of symmetry. So, it will not form [CoF6]3–
optical isomers. (Outer orbital 3d 4s 4p 4d
or high spin
2. (a) The crystal field splitting, Do depends upon the field complex)
produced by the ligand and charge on the metal ion. In all
these complexes of chromium, charge acquired by metal
ion is +3. Therefore, Do depends upon the field produced 4. (d) The IUPAC name of K 2[ Cr( CN)2 O2( O)2( NH3 )] is
by the ligand. In accordance with the spectrochemical potassium amminedicyanodioxoper-oxochromate (VI).
series, the increasing order of field strength is
5. (d) Ferrocene is a sandwich complex compund in which
C2O24- < H2O < NH3 < CN- all the five carbon atoms of cyclopentadiene anion are
Thus, CN - is the strong field ligand and will produce linked to the metal through pi-bonds.
highest magnitude of Do . [Fe(h 5 - C5H5 )2] Ferrocene ® it has aromatic character.

3. (d) [Co(NH3 )6] 3+ and [Ni(CN)4] 2- are inner d -orbital or low 6. (a) [Co(NH3 )5 SO4]Br R [Co(NH3 )5 SO4] + + Br -
spin complex and [NiCl4] 2- complex hassp 3-hybridisation [Co(NH3 )5 Br]SO4 R [Co(NH3 )5 Br] 2+ + SO24-
and has tetrahedral geometry while [CoF6] 3- is outer
d -orbital or high spin complex. the molecular formula of both of the above compounds is
[CoF6] , Co = 4s , 3d , 4p , Co
3- 2 7 0 3+
= 3d , 4s , 4p
6 0 0 same but on ionisation they give different ions in solution,
so they are called ionisation isomers.
Orbitals of
Co3+ ion 3d 4s 4p 4d 7. (a) The crystal field splitting energy for octahedral ( Do )
and tetrahedral ( Dt ) complexes is related to
sp3d2 4
-hybridised 3d 4s 4p 4d Dt = Do
orbitals of Co3+
9

@iitjeehelps
21
General Organic
Chemistry

Concept of Organic Chemistry


Organic chemistry is a branch that deals with the study of C and H compounds and their derivatives.
A large number of organic compounds have been synthesised till date because of unique property of
C to catenate hence, the classification and nomenclature of organic compounds is necessary.

Tetravalency of Carbon
Liebel and van’t Hoff studied the tetravalent nature and tetrahedral structure of carbon. The carbon
atom is tetracovalent which suggests that it forms four covalent bonds with four monovalent atoms
or groups. Carbon forms a large number of organic compounds like methane, methyl alcohol,
methanoic acid.
H H O
  
H C  H H C OH H C OH
 
H H
Methane Methyl alcohol Methanoic acid

Structural Representation of Organic Compounds


Structural Formula
In this, all the bonds present between any two atoms are shown clearly
H H H H
   
H  C  C  C  C H
   
H Cl H H
2-chlorobutane

@iitjeehelps
GENERAL ORGANIC CHEMISTRY 609
Condensed Formula Classification Based on Functional Groups
In this type of chemical formula, all the bonds are not The compounds of only carbon and hydrogen are called
shown clearly. hydrocarbons. Hydrocarbons are considered as the
CH3 CH CH 2CH3 parents of all the organic compounds. All other compounds
 are obtained from hydrocarbons by the replacement of one
Cl or more hydrogen atoms with other atoms or groups.
or CH3CH(Cl)CH 2CH3 Various classes of compounds having some of the common
functional groups are listed in the following table.
Bond Line Formula
In this, every fold and free terminal represents a carbon and Class Functional group
lines represent the bond. Olefins/Alkenes ‚ C === C ƒ
Cl ƒ ‚

Acetylenes/Alkynes  C ≡≡ C 
e.g. CH3 CH 2 COOH ≡≡ OH
Halides  F,  Cl,  Br,  I (Halo)

O Alcohols  OH (Hydroxy)
Classification of Carbon and Hydrogen Atoms Thiols  SH
● Primary (1° ) carbon atom When carbon atom is
Aldehydes  C  H (Aldehydic)
attached with other carbon atom only.

● Secondary (2°) carbon atom When carbon atom is O
attached with two other carbon atoms. O
● Tertiary (3°) carbon atom When carbon atom is 
Ketones  C  (Ketonic)
attached with three other carbon atoms.
● Quarternary (4°) carbon atom When carbon atom is | |
Ethers  C  O  C  (Alkoxy)
attached with four other carbon atoms. | |
Reactivity order of carbon atoms are as follows: O
3° > 2° > 1° 
1° Carboxylic acid  C  OH (Carboxyl)
CH3
1° 2°  3° 1° Sulphonic acid SO3 H
°
CH3  CH 2  C 4 CH CH3
O
  ||
CH3 CH 3 Amides  C  NH 2 (Amide)
1° 1°
H
Amines N (Amino)
Classification of Organic Compounds H

The organic compounds have been classified on the basis of Cyanides/Nitriles  C ≡≡ N (Cyano)
carbon skeleton (structure) or functional groups or the
O
concept of homology, i.e. the series in which two successive Nitro compounds N (Nitro)
members differ by a CH 2 unit or 14 unit mass. O
O
Classification Based on Structure  |
On this basis, the organic compounds are classified as : Esters  C  O C  (Ester)
|
Organic Compounds
O
Open chain, acyclic Closed chain 
or aliphatic compounds or cyclic compounds Acid halides  C  X (Acylhalide)

Homocyclic compounds Heterocyclic compounds


Homologous Series
Alicyclic compounds Aromatic A homologous series can be defined as a group of
(e.g. cyclopropane) compounds
compounds in which the members have similar structural
features and similar chemical properties and the
Benzenoid aromatic Non-benzenoid
compounds aromatic compounds successive members differ in their molecular formula by
(benzene) (azulene, tropolone) CH 2 group.

@iitjeehelps
610 SELF STUDY GUIDE BITSAT

IUPAC Nomenclature for Organic Rules for IUPAC Nomenclature


Compounds (i) Longest chain rule The longest C C linkage of a
structure is its parent chain. It needs not to be straight
The IUPAC name of an organic compound denotes only one
chain, but can be of any combination which is longest.
molecular structure.
The IUPAC name of any organic compound comprises of the CH3 CH CH2 CH3
following structure.
Prefix Primary prefix CH3 CH2
Secondary prefix
Organic compound Word root
CH3 CH CH2 CH2 CH3
Primary suffix
Suffix
Secondary suffix
CH3 CH CH3
General structure of an IUPAC name
Primary Secondary  Word Primary (ii) Lowest locant rule The numbering of the parent chain
 prefix  −  prefix  – root  –  suffix  is done from that side where the value of locant
        substituent is minimum possible number.
Secondary Wrong
–  →1 2 3 4
 suffix  CH3 CH 2  CH  CH3
Word Root 1←
4 3
2 Right
This indicates the number of carbon atoms present in the CH3
parent chain [longest CC linkages]. This word root (iii) Alphabetical order law In case of multiple
includes→ meth, eth, prop, but, pent, hex, hept, oct, non, dec. substitutions, the name of the substituents must be
Suffix arranged in alphabetical order.
It is used to indicate the nature of CC linkages whether it (iv) First point difference rule If numbering cannot be
is saturated or unsaturated (primary suffix) and type of resolved by lowest locant rule, i.e. functional groups are at
functional group present in the molecule (secondary suffix). equivalent positions, then the numbering should be done
in such a way that functional group coming first in
Organic compound Functional group Suffix alphabetical order, will be having minimum possible
Alcohols  OH —ol value.
Aldehydes  CHO Carbaldehyde (v) For complex group (like branched side chain), the
(C) HO  al
nomenclature rules are same, the only difference is
Ketones (C) == O  one that the carbon through which the complex group is
Carboxylic acids  COOH  carboxylic acid associated with main chain, is given number one
 (C)OOH  oic acid always. For the purpose of distinction, the locant
Acid amides CO NH 2 — carboxamide of side chain are indicated by a prime following them
(C) O  NH 2 — amide as 1′, 2′ etc., e.g. consider the structure given below
Acid halides O  carbonyl halide CH3 CH2 CH2 CHCH2 CH2 CH3

CX 2′ 1′
CH3 C CH3
O  oyl halide

(C)  X
CH3
where, X = halogen 1,1-dimethyl group (complex group)
[F, Cl, Br, I ]
(vi) In case of simultaneous presence of double and triple
Esters O ( R ∗ ) ..... carboxylate
 bond, i.e. if bonds are present at equal distance from the
C  OR ∗ ends, then double bond must be preferred. If the position
(C) OOR ( R ∗ ) ...oate is not symmetrical, then the priority will be decided by
Amines NH 2  amine lowest locant sum rule and first point difference rule.
Nitriles  C ≡≡ N  carbonitrile (vii) If the chain is having more than one functional group,
 (C) ≡≡ N  nitrile the priority of the functional groups is given below
Sulphonic acids  SO2 ,  OH Sulphonic acid
COOH > SO3H > R(CO)2 O > COOR > COX >
R → any alkyl group CONH 2 > CN > CHO > C == O > OH > 
C → when C-atom is included in the main chain. NH 2 > O > > C == C > C ≡≡ C > R  X >  NO2

@iitjeehelps
GENERAL ORGANIC CHEMISTRY 611
IUPAC Nomenclature for Cyclic Compounds (ii) Position isomerism In such isomers, there is difference
For the nomenclature of cyclic compounds, general rules are in the position of functional group or multiple bond or
followed : branched chain attached to the parent C-chain.

(i) The root word is equal to sides of polygon representing OH OH


the compound. Primary prefix cyclo is used for
alicyclic compounds.
CH3 CH3
Cyclo → word root → 1° suffix
(iii) Functional isomerism Such isomers contain different
e.g. functional groups.
e.g. CH3 CH 2 OH and CH3 O CH3
Cyclohexane Cyclopentene Cyclobutyne
(iv) Metamerism This type of isomerism is due to unequal
(ii) If length of side chain is greater than ring size, then ring distribution of substituents on either side of bivalent
is treated as alkyl substituent where side chain is functional group, e.g. CH3CH 2OCH 2CH3 and
substituent.
CH3CH 2CH 2OCH3 , CH3CH 2 NHCH 2CH3 and
2 4 6
1
CH3CH 2CH 2 NHCH3 .
3 5 7
(v) Tautomerism This arises due to transfer of H-atom
e.g. from one polyvalent atom to another. Tautomers exist
in dynamic equilibrium.
1-butylcyclopentane 1-cyclopentylheptane
(a) Keto-enol tautomerism (α −H must be present)
Nomenclature of Aromatic Compounds O
Aromatic compounds are better known by their common ||
CH 2 == C OH r CH3  C H
name. IUPAC recognises and acknowledges the common |
nomenclature. Benzenoids are named as derivatives of H
benzene as per rules mentioned for alicyclic compounds.
Benzene with small side chain are also having special name CO NH CO NH
C O C OH
as benzyl, benzal, benzo, etc. Otherwise other common CO NH CO N
names like toluene, phenol, cresol, resorcinol, etc., are
(b) Nitro-acenitro tautomerism
equally used.
CH2Cl CHCl2 CCl3 O O
CH3 N q CH2 N
O OH
(c) Imine-enamine tautomerism
Benzyl chloride Benzal dichloride Benzochloride CH3 CH == NH r CH 2 == CH  NH 2
or benzylidene chloride

Stereoisomerism
Isomerism in Organic Chemistry In this type of isomerism, isomers possess same molecular
Organic compounds having the same molecular formula and structural formula but spatial arrangement of atoms is
but different structures or orientations in space are known different.
as isomers and the phenomenon is called isomerism. They can be sub-classified in the following types :
Isomerism can be classified as follows :

1. Structural Isomerism Reaction Mechanism


It arises because of the difference in the sequence of Types of Bond Fissions
covalently bonded atoms in the molecule without reference
to space. 1. Homolytic Fission
It is sub-classified into following types : It involves cleavage of covalent bond in such a way that
each of the bonded atom takes away one electron out of the
(i) Chain isomerism This arises due to the difference in
shared pair.
length of C-chain (straight or branched). e.g. C 4H 10
A  B → A• + B • (free radicals)
CH3 CH 2 CH 2 CH3 and CH3  CH CH3
(4-C chain) | It is also called free-radical cleavage or non-polar bond
CH 3 fission.
(3-C chain)

@iitjeehelps
612 SELF STUDY GUIDE BITSAT

2. Heterolytic Fission The above 2 structures are called resonating structures.


δ+ δ-
The shared pair of electron is taken away by one of the atom ..... .....
[H 2 C  HC  CH  CH 2 ]
.....
(more electronegative atom).
Resonance hybrid (Actual structure)
A  B → A+ + : B − (Ions) charged species.
Also called polar bond fission or ionic cleavage.
In the above example,
electronegativity of B > electronegativity of A
Mesomeric effect is of the following two types :
Electron Displacement in Covalent Bonds (a) +M-effect Direction of electron displacement is away
1. Inductive Effect from the substituent.
The displacement of an electron (shared) pair along the
C-chain due to the presence of an electron C C C C C
+
withdrawing/donating group in the C-chain, is known as –
C C C C C
inductive effect (I-effect).
It is transmitted along the chain and is permanent. +
C C

I-effect can be of the following two types :
(i) —I-effect Any atom or group of atoms which attracts Groups showing + M-effect
electrons more strongly than H is said to have negative
 NH 2 , OH, Cl , OR , etc.
inductive effect.
(b) —M-effect Direction of electron displacement is
Groups introducing − I-effect
towards the substituent.
 NO2 > CN > COOH > F
+
Cl > Br > I > OR > NH 2 > OH > C6H 5 > H O C C C O– C C C
(ii) +I-effect An atom or group which attracts electrons
CH O CH O– CH O– CH O–
less strongly as compared to H, is said to have a
positive inductive effect. +
+
Groups producing +I-effect
H < CH3 < C 2H 5 < (CH3 )2 CH  < (CH3 )3 C  (+)
2. Electromeric Effect
Groups showing −M-effect
It is a temporary effect operates only in π-electrons
(i.e. multiple bonded compounds) in the presence of some  NO2 , CHO, C ≡≡ N, COR, COOH, CONH 2 , etc.
attacking reagents.
4. Hyperconjugation
δ+ δ– – + –
C O + CN C O (Baker-Nathan effect or no bond resonance)
(–E-effect)
As a result of shifting of σ-electrons, there is a
δ+ δ– + – phenomenon of resonance in rest of the molecule.
C C +H Br C C
This is a type of σ-π resonance.
(+ E-effect)

The effect is called + E if transference of π-electrons occurs H +


H –
towards the atom where attacking species attaches. H C CH CH2 H C CH CH2
The effect is called − E if transference of electrons occurs
away from the atom where the attacking group attaches. H H
(+H-effect)
+
3. Resonance or Mesomeric Effect H2C H H2C H
Electron displacement relayed through conjugate system –
of multiple bonds in the C-chain causing permanent
polarisation. It is essential that the atom having a lone pair
of electrons or a group containing a π-bond.
X
CH2 CH CH CH2 X– +
+ C C C C C C (–H-effect)
CH2 CH CH CH – 2

@iitjeehelps
GENERAL ORGANIC CHEMISTRY 613
Applications of Hyperconjugation 3. Free Radical (FR)
(a) Relative stability of alkenes. Species having trivalent C-atom bearing no charge but
(b) Relative stability of alkyl carbonium ions or free have odd electrons or an unpaired electron, are known as
radicals. free radicals. They are paramagnetic in nature. Generally,
they are planar, formed due to homolytic bond cleavage.
(c) The o, p and m-directing influence of some alkyl
groups. e.g. C  X → C• + X •
CH•3 , CH 2 == CH CH•2 , Cl• etc.
Various Reaction Intermediates
Order of Stability
1. Carbocation C ⊕
(C6H 5 )3 C• > (C6H 5 )2 CH• > C6H 5CH•2 > CH 2 == CH CH•2
|
 C ⊕ organic species containing positively charged also 3 ° > 2 ° > 1° > CH•3
|
Free radical generators → Peroxides
C-atom which contains 6 electrons in its valence shell, are
known as carbocations. C ⊕ has planar geometry. 4. Carbene
⊕ Species containing divalent C-atom bearing no charge and
e.g. H3C ⊕ , R CH 2 , R2CH ⊕ , R3C ⊕
Methyl (1 ° ) ( 2°)
surrounded by 6 electrons. They are powerful electrophiles.
(3 ° ) • •
carbocation e.g. • CH 2 , • CCl 2 etc.

Relative Stability There are two types of carbenes:


(i) Alkyl carbocations (explained by inductive and (a) Singlet carbene (sp 2 -hybridised) Here the unshared
hyperconjugation effect)
electrons are paired and present in the same orbital.
CH3 CH3 H
H
+ + +
CH3 C
+
> CH3 C > CH3 C > CH3 e.g. C planar, diamagnetic
H
CH3 H H

i.e. 3 ° > 2 > 1° > CH3⊕ (b) Triplet carbene (sp-hybridised) Here the unshared
electrons are not paired.
(ii) Aryl carbocations (explained by resonance) ¼
Ph3C ⊕ > Ph 2CH ⊕ > PhCH ⊕
2 e.g. H C  H linear, paramagnetic
¼
Carbanion (C − ) In general, a triplet carbene is more stable than a
Species having such a C-atom that is bearing negative singlet carbene.
charge and surrounded by octet of electrons. They have 5. Nitrene
pyramidal geometry.
It is a neutral intermediate species containing N-atom
e.g. CH3− , (CH3 )2 CH , (CH3 )CH 2 etc. having one bond pair and two lone pairs. It is electron
deficient. They are analogous of carbenes.
Relative Stability •• ••
(i) Alkyl carbanions (explained by + I-effect) e.g. H  N• , RN• etc.
• •
H CH3 CH3
| | | 6. Benzyne
CH3 > CH3 C − > CH3  C − > CH3  C− It is a neutral highly reactive intermediate in which the
| | | aromatic character are not markedly distributed,
H H CH3
i.e. 1° > 2 ° > 3 ° or all C-atoms are sp2-hybridised
(ii) Benzyl, allyl and triphenyl methyl carbanions
(explained by resonance)
Sidewise overlapping of sp 2 -orbitals forms π-bonds
(iii) Stability of C − increases with s %.
− −
outside the ring and out of the plane of π-system of ring .
H  C ≡≡ C − > CH 2 ==== CH > CH 2 CH 2 Benzyne is very unstable.

@iitjeehelps
614 SELF STUDY GUIDE BITSAT

Type of Reagents (or Attacking Species) Mechanism


+ Br
1. Electrophile (E ) Br H
They are neutral or positively charged species
+
with deficiency of electrons and capable of accepting a pair + Br + +H
of electrons (Lewis acid).
Positively charged species
● During E ⊕ substitution, an electron donating group on
H+ , CH3+ , H3O+ , NO+2 , Br + , CH3 N 2+ etc.
benzene ring makes it more reactive. Such groups are
Neutral species  NH 2 , OH, OR, etc.

• CH 2 , AlCl3 , BF3 ,ZnCl 2 ,PCl 5 etc.
● A deactivating group is one which makes benzene less
− reactive. Such groups are
2. Nucleophile (Nu )
They are neutral or negatively charged species which are COOH, > C == O, CHO,  NO2 , CN etc.
capable of acting as donor of electron pair (Lewis base). 2. Addition Reactions
Negatively charged ions Reactions in which atoms or group of atoms are added to a
Cl − , Br − , CN − , NO−2 , HSO−4 , etc. molecule, are called addition reactions.
Neutral species A == B + x  y → A  B
•• •• | |
NH3 , R2 NH, R2O, H 2O x y

A  B + Y − → A  Y + B − Addition reaction are of the following types :


(Nu − )
(a) Electrophilic addition reactions
3. Amphibiles E
‚ ƒ | |
Amphibiles are those which act as both E + and Nu − . C == C + E δ + − Nu δ −
RDS
→  C C  + Nu −
O O ƒ ‚ S low | ⊕
|| || E
R C  H , R  C  R , H C ≡≡ C H etc. | |
Fast
→  C  C  (trans-addition)
Types of Organic Reactions | |
1. Substitution Reaction Nu
Such reactions involve the replacement of an atom or CH 2 == CH 2 + HBr →
+ −
group of atoms by some other atom or group.
CH3 CH 2 Br
→ CH3  C H 2
| |

C  X + A  B → C  A + BX
| | Br

e.g. R  X + NaOH (aq ) → R OH + NaX (b) Nucleophilic addition reaction (Intermediate → Nu − )
Substitution reaction are of the following types: +

(a) Free radical substitution (FR intermediates) C O + E  Nu C O E
C O
e.g. CH 4 + Cl 2 → CH3 Cl + HCl Nu Nu E

(b) Nucleophilic substitution (Attacking species Nu )
R  X + Nu − → R  Nu + X − CH3

CH3
C O + CN C O–
− −
CH3Br + CH3O → CH3OCH3 + Br CH3 CH3
CN
C 2H 5Br + OH − → C 2H 5OH + Br −
CH3
(c) Electrophilic substitution Substitution in benzene +
H
ring follows electrophilic substitution. CH3 C OH
FeBr 3
C6H6 + Br2 → C6H 5Br + HBr CN

@iitjeehelps
GENERAL ORGANIC CHEMISTRY 615
(c) Free radical addition (Intermediate — free radial) Rate equation R = k [halide] [:B]
Peroxide
CH3 CH== CH 2 + HBr   → CH3 CH 2 CH 2Br Molecularity = 2, order = 2
Mechanism C 2 H 5ONa
e.g. CH3  C H CH3 →
O O |
Br
R R 2 RO CH3 CH ===CH 2 + NaBr + C 2H 5OH
H 
Chain initiation RO• + HBr → ROH + Br• | 
......
Chain propagation Transition state C H 2  CH CH3 
•  | 
CH3 CH == CH 2 + Br• → CH3 CH CH 2Br  Br 
• • Order of reactivity : 3 ° > 2 ° > 1°
CH3 CHCH 2Br + HBr → CH3 CH 2CH 2Br +B r
(ii) E 1 reaction (Elimination unimolecular)
Chain termination Br• + Br• → Br2 X
• | ⊕ |
CH3 CHCH 2Br + Br• → CH3  C H CH 2 Br  C  C   → X − + C  C 
Slow

|  | |
Br H H
⊕ | Fast ‚ ƒ
3. Elimination Reactions C  C → C == C + H  B
| B ƒ ‚
Such reaction involves loss of atoms or groups from a
molecule to form multiple bonds. H
A  B → A == B + x  y Rate equation R = k[ RX ]
| | Molecularity = 1, order = 1
x y
Me CH2 H
+
Slow
CH3CH 2Cl + KOH (alc. ) → CH 2 == CH 2 + HCl e.g. (CH3)3C Br C
CH3CH 2OH + H 2SO4 (conc. ) → CH 2 == CH 2 + H 2 O Me
There are two types of mechanism: Fast
Me C CH2
(i) E 2 reaction Elimination bimolecular reaction involves B
one step only. Me
X 4. Rearrangement Reactions
| B
| |
 C  C  →  C == C  + HB In such reactions, atom’s sequence gets reshuffled to form a
| new structure, e.g. pinacol pinacolone rearrangement,
H Beckman rearrangement.

@iitjeehelps
Practice Exercise
1. Which of the following are aromatic compounds? 7. The kind of valency that exists in CaH2 and C2 H 2 is
CHO a. electrovalency in CaH2 and covalency in C2H2
b. electrovalency in both
N c. covalency in CaH2 and electrovalency in C2H2
I. II.
d. covalency in both
N
8. 3-methyloctane can be represented in which of the
O following forms?
a. CH3CH2 CH(CH2 )4 CH3
b. H| H H H H H H H
III. IV. CH3
S H C C C C C C C C H
O H H H H H H H
a. I. and III b. I, II, and III H C H
c. I, II, and IV d. III, and IV
2. 3° carbon is present in the compound H
a. cyclopropane b. toluene
c. benzene d. cyclohexane c.
3. Choose the compound in which all the carbon atoms d. All of the above
have 66.7% p-character. 9. Which of the following is a heterocyclic alicyclic
a. CH3  C ≡≡ CH compound?
b. CH2 == C == CH  CH3 O OH
c. CH2 == CH  CH == CH2
d. (CH3 )2 C == CH2 a. b.
4. The bond between carbon atom (1) and carbon atom NH2
(2) in compound
1 O
C N
2 c. d.

is 10. The IUPAC name of the isomer of CH3  CH == NOH


would be
a. 3 2
sp and sp -hybridised a. methanamide
b. 1-amino-2-propanone
b. sp 2 and sp 3-hybridised
c. ethanamide
c. sp and sp 2-hybridised d. None of the above
d. sp and sp -hybridised
11. Pyruvic aldehyde is nothing but
5. Electronegativity of carbon atoms depends upon their a. methyl glyoxal
state of hybridisation. In which of the following b. ethyl glyoxal
compounds, the carbon marked with asterisk, is most c. glyoxal
electronegative? d. None of the above
*
a. CH3  CH2  CH2  CH3 12. The IUPAC name for the following compound is
* *
b. CH3  CH == CH CH3
*
c. CH3  CH2  C ≡≡ CH
d. CH3  CH2  CH == CH2
6. The hybridisation of C2 carbon atom present in a. 4-ethyl-3-propyl hex-1-ene
3 2 1 b. 3-(1-ethylpropyl) hex-1-ene
CH2 == C == CH2 is c. 3-ethyl-4-propyl hex-5-ene
a. sp b. sp 3 d. 3-ethyl-4-ethylheptane
c. sp 2 d. sp 4

@iitjeehelps
GENERAL ORGANIC CHEMISTRY 617
13. The correct IUPAC name of 18. The IUPAC name of
COOH CH3
O
H 3C
is O is
a. ethylpentanoate
CH3 b. 1-ethoxy-2-pentanone
c. 5-ethoxy-4-pentanone
a. 6-methyl-3- phenyl benzene-1-oic acid d. ethyl-2-oxopentyl ether
b. 6-methyl dibenzene-1-oic acid
c. 3-(3-methyl phenyl) benzene-1-oic acid 19. The IUPAC name of
d. 3-(5-methyl phenyl) benzene-1- carboxylic acid
HO
is
14. The IUPAC name of is O
O a. 2-acetyl-1-buten-1- ol
a. 1-alkoxycyclopent-1- ene b. 3-(hydroxymethyl) 3-penten-2-one
b. oxocyclohex-2-ene c. 3-ethylidene-4-hydroxy-2-butanone
c. 2-methoxycyclopent-1-ene d. 3-acetyl-2-buten-4-ol
d. 5-oxocyclopent-1-ene 20. Which skeleton exhibit optical isomerism?
15. The IUPAC name of the compound shown below is a. C  C  C  C  COOH
Cl b. C  C  C  C
|
COOH
c. C  C  C  COOH
|
Br
C
a. 2-bromo-6-chlorocyclohex-1-ene C
b. 6-bromo-2-chlorocyclohexene |
c. 3-bromo-1-chlorocyclohexene d. C  C  COOH
d. 1-bromo-3-chlorocyclohexene |
16. What is the correct IUPAC name of the following C
compound? 21. In which isomer of C7H 7NO2, there are two functional
groups?
a. Phenyl nitromethane b. m-nitrotoluene
c. Anthranilic acid d. Benzylnitrite
22. The pair of functional group isomers is
a. 3-ethyl-1, 1-dimethylcyclohexane a. CH3CONH2, CH2CHO
b. 1-ethyl-3, 3-dimethylcyclohexane |
c. 1, 1-diemthyl-3-ethylcyclohexane NH2
d. None of the above b. CH3COOH , HCOOCH3
17. The correct IUPAC name of the given structures will c. CH3  CH2OH , CH3OCH3
be d. All of the above
O 23. The compound Cn H2n [n = 4] does not exhibit the
|| which of the following isomerism?
I. CH3  C  CH2  CH2  CH2  COOH
a. Chain b. Geometrical
II. CH ≡≡ C  CH == CH  CH == CH2 c. Position d. Optical
Choose the correct option. 24. Geometrical isomerism is shown by
I II H F H Br
‚ ƒ ‚ ƒ
a. C == C b. C == C
a. Hexane-dione Hexa-1, 3-diene-5-yne ƒ ‚ ƒ ‚
H Br H3C Br
b. 5-oxohexanoic acid Hexa-1, 3-diene-5- yne
H F H3C Cl
‚ ƒ ‚ ƒ
c. 5-oxohexanoic acid Hexadiene-5-yne c. C == C d. C == C
ƒ ‚ ƒ ‚
d. 3-oxohexanoic acid Hexadiene-5-yne H3C Br H3C I

@iitjeehelps
618 SELF STUDY GUIDE BITSAT

25. The compound 2, 3-dichlorobutane exhibits 31. Among the following four structures I to IV,
a. geometrical isomerism CH3 O CH3
b. diastereoisomerism | || |
c. structural isomerism C2H 5  CH  C 3 H 7 CH3  C  CH C2H 5
d. optical isomerism
I II
26. The number of constitutional isomers of the formula H
C5 H 11Br is CH3
| |
a. 4 b. 8
H C⊕ C2H 5  C H C2H 5
c. 6 d. 10 |
27. How many optical isomers are possible on H
monochlorination of 2-methyl butane? III IV
a. 2 b. 4
c. 6 d. 8
It is true that
a. all four are chiral compounds
28. The structural formula of 2-oxo-3-methyl-(N-bromo) b. I and II are chiral compounds
butanamide is c. III is a chiral compounds
a. CH3 — CH2 — CO — CO — NH — Br d. II and IV are chiral compounds
CH3
| 32. The correct statement about the compounds A, B and
b. CH3 — CH — CO — CO — NH — Br C is
CH3 COOCH3 COOH COOH
| H OH H OH HO H
c. CH3CH — CO — CO — NOBr H OH H OH HO H
d. (CH3 )3 C — CO — CO — NHBr COOH COOCH3 COOCH3
29. Match the items of Column I with the Column II and A B C
choose the correct option from the codes given below. a. A and B are identical
Column I Column II
b. A and B are diastereomers
(Structure of compounds) (Type of isomerism) c. A and C are enantiomers
d. A and B are enantiomers
A. CH3 CH2CH2CH2CH3 and 1. Chain isomerism
33. If CH3  CH  CHO gives up proton, the most stable
CH3
|

CH3  CH CH2  CH3 CH3
carbanion would be
B. CH3 CH2CH2OH and 2. Position isomerism
− CH3 ‚
OH CH2 ‚
| a. CH CHO b. ƒ
CH  CHO
CH3  CH CH 3 CH3 ƒ CH2
CH3 ‚ −
C. O 3. Metamerism c. C  CHO d. All of these
|| CH3 ƒ
CH3  C  CH3 and
34. In which case, ionic mechanism is not followed?
H
| a. Reaction of acetylene and bromine water
CH3  CH2  C == O b. Reaction of HBr and propene in the presence of
peroxide
D. CH3 OC3 H7 and C2H5OC2H5 4. Functional group c. Reaction of ethyl bromide and alcoholic KOH
isomerism d. Dehydration of ethyl alcohol catalysed by acid

Codes 35. Which compound is more sensitive to undergo S N 1


A B C D reaction?
a. 1 2 3 4 a. 2-bromobutane
b. 2 3 1 4 b. 2-bromo-2-methyl propane
c. 4 1 2 3 c. 2-methyl-1-bromo propane
d. 1 2 4 3 d. Bromoethane
30. Example of geometrical isomerism is 36. The enol form of acetone contains]
a. 2-butanol a. 9 σ -bonds, 2 π -bonds and 1 lone pair of electron
b. 2-butene b. 10 σ -bonds, 1 π- bond and 1 lone pair of electrons
c. butanol c. 9 σ -bonds, 1 π-bond and 2 lone pairs of electrons
d. 2-butyne d. 8 σ -bonds, 2 π- bonds and 2 lone pairs of electrons

@iitjeehelps
GENERAL ORGANIC CHEMISTRY 619
37. Consider the following carbocations, 43. The correct decreasing order of priority for the
+ functional groups of organic compounds in the IUPAC
(CH 3 )3 CCH 2 , (CH 3 )3 C+, system of nomenclature, is
I
+
II
+
a. —CONH2 > —CHO > —SO3H > —COOH
CH 3 CH 2 C H2, CH 3  CH  CH 3 b. —COOH > —SO3H > —CONH2 > —CHO
III IV c. —SO3H > —COOH > —CONH2 > —CHO
d. —CHO > —COOH > —SO3H > —CONH2
The correct order for the stability of the above
carbocations is 44. Correct order of nucleophilicity is
a. I < III < IV < II b. III < IV < I < II a. CH3− < NH2− < OH − < F −
c. IV < III < II < I d. II < IV < III < I b. F − < OH − < CH3− < NH2−
c. OH− < NH2− < F − < CH3−
38. The solvent in which enol form of ethylacetoacetate is
d. F − < OH − < NH2− < CH3−
maximum, is
a. CH3COOH b. aqueous HCl 45. Consider the following transformations
c. n-hexane d. H2O
I. CH3− X → II. CH−3 X →
39. Among the following compounds, the compound
III. CH−3 X →
containing maximum enol contents in their solution, is
O O Carbon species formed in I, II and III respectively are
  a. carbocation, carbanion and free radical
a. C6H5  C  CH2  C  CH3 b. free radical, carbocation and carbanion
c. free radical, carbanion and carbocation
O O d. carbanion, carbocation and free radical
 
b. CH3  C  CH2  C  CH3 46. Which is the most stable carbocation?
O
 a.
+
CHCH2 b. +
c. CH3  C  CH2  CH2CH3
O CH3

d. CH3  C  CH2  COOC2H5 + +
c. C CH3 CH2
40. What is the correct order of decreasing stability of the d.
following cations? CH3

CH 3  CH CH 3
I 47. Select the correct statement about the following
⊕ reaction :
CH 3  CH  OCH 3 H
NaI
II C2H5 Br Acetone

CH 3  CH  CH 2  OCH 3 CH3
III
a. II > I > III b. II > III > I a. it can proceed via SN 2 mechanism
c. III > I > II d. I > II > III b. configuration about chiral carbon is retained
c. a racemic mixture is formed
41. The correct stability order of the following free radicals d. reaction is stereospecific
is
• • • •
48. Consider the following reactions:
a. (CH3 )2 CH < (CH3 )3 C < (C6H5 )3 C < (C6H5 )2 CH CH3
• • • •
b. (C6H5 )2 CH < (C6H5 )3 C < (CH3 )3 C < (CH3 )2 CH SN 1
• • • •
I. H Cl →
c. (C6H5 )3 C < (C6H5 )2 C H < (CH3 )3 C < (CH3 )2 CH KOH

• • • •
CH2 CH3
d. (CH3 )2 CH < (CH3 )3 C (C6H5 )2 CH < (C6H5 )3 C
CH 3
42. Which one of the following species is not an SN 2
electrophile? II. H Br →
+ KOH
a. NO2 b. H3O+
CH2CH 3
c. Cl+ d. BH3

@iitjeehelps
620 SELF STUDY GUIDE BITSAT

Select the correct statement. 53. Which alkenol is stable?


a. Inversion in both cases takes place a. CH2 == CH  CH2  OH b. CH2 == CHOH
b. Retention in both cases takes place c. CH3  C == CH OH d. All of these
c. Retention in I and inversion in II takes place |
d. Racemisation in I and inversion in II takes place 54. Which of the
CH3following is not a free radical?


49. Arrange the following in increasing order of acidic a. Cl b. (C6H5 )2 CH

strength : c. •• CCl 2 d. (CH3 )2 CH
I. H2 SO 4 II. (CH3 ) 3 CH(SH)
III. CH3 CH2+ OH2 IV. CH 3 CH2 CH3 55. Among the following, the least stable resonance
a. IV < II< I < III b. IV < II < III < I structure is
c. IV < I < II < III d. IV < III < II < I r O s
r r O
a. s N b. r N
50. Choose the correct order of stability of carbocation
using concept of hyperconjugation. O
s O
CH 3 CH3 s
s
| | ⊕ ⊕ s O r O
r
CH 3  C⊕ CH3  C⊕ CH3 CH2 CH3 c. r N d. N
| |
CH 3 H O O
s s
I II III IV
a. I < II < III < IV 56. Carbocation stability
b. IV < III < II < I + + + +
c. III < IV < II < I CH3 < CH3 CH2 < (CH3 )2 C H < (CH3 )3 C ••
d. All of the above
Alkyl radical stability
51. Chloroacetic acid is a stronger acid than acetic acid. • • • •

This can be explained using a. CH(CH3 )2 < CH3 < CH2CH3 < C(CH3 )3
• • • •
a. −M- effect b. −I-effect b. C(CH3 )3 < C H(CH3 )2 < CH3 < C H2CH3
c. +M-effect d. +I- effect • • • •
c. C (CH3 )3 < CH(CH3 )2 < CH2CH3 < CH3
52. CH3 CH2Cl undergoes homolytic fission and produces • • • •
• • ⊕

d. CH3 < CH2CH3 < CH(CH3 )2 < C(CH3 )3
a. CH3 CH2 and Cl b. CH3 CH2 and Cl
⊕ • •
c. CH3 CH2 and Cl d. CH3 CH2 and Cl−

BITSAT Archives
1. Which of the following is correct order of stability of 2. Arrange these in correct order of decreasing reactivity.
carbocation? [2014] CH3
⊕ ⊕ CH3
 C  C F OH F

H OH
I II
I II H 3C CH3
⊕ ⊕
C C
OH Ph OH
H III IV
a. I > II > III > IV b. I > III > II > IV [2013]
III IV c. IV > III > II > I d. IV > III > I > II

a. IV > III > II > I 3. In which of the following species only one type of
b. I > II > III > IV hybridisation is present? [2012]
+
c. III > II > I > IV a. CH3  CH2 — CH == CH2 b. CH3  CH== CH — CH2
d. I > III > II > IV c. CH2 == CH CH == CH2 d. CH3  CH== CH CH−2

@iitjeehelps
GENERAL ORGANIC CHEMISTRY 621
4. Which among the following is likely to show geometrical a. 2-amino-3-hydroxy propanoic acid
isomerism? [2011] b. 1-hydorxy -2-amino propan-3-oic acid
a. CH3CH == NOH c. 1-amino-2-hydroxy propanoic acid
b. CH3 CH == CH2 d. 3-hydroxy-2-amino propanoic acid
c. CH 2 == CH CH == CCl 2 10. The compound which give the most stable
d. CH 3 C(Cl) == C(CH3 ) 2 carbonium ion on dehydration is [2007]
5. The bond dissociation enthalpies of a. CH3CH(CH3 )CH2OH
≡≡ CH , == CH ,  CH follows the order [2010] b. (CH3 )3 COH
sp sp 2 sp 3 c. CH2 == CHCH2CH2OH
d. CH3CHOHCH2  CH3
a. sp > sp 2 > sp 3 b. sp 3 > sp 2 > sp
c. sp 2 > sp > sp 3 d. sp > sp 3 > sp 2 11. Tautomerism is exhibited by [2007]

6. Most stable carbonium ion is [2010] a. CH CH OH


+
a. CH2
b. O O
+
b. CH3O CH2
c. C O
+
c. Cl CH2 H
CH3
+ d. O C OH
d. O2N CH2
CH3
OH

7. CH3  CH CH2  CH3 is 12. The compound whose stereochemical formula is
written below, exhibits x geometrical isomers and y
optical isomers. [2006]
CH3 ‚ ‚
H OH
ƒ
C Cƒ
a. 2-phenylbutane H CH2 CH2 C CH3
b. 3-phenylbutane
c. 3-cyclohexylbutane H
d. 2-cyclohexylbutane The values of x and y respectively are
a. 4 and 4 b. 2 and 2 c. 2 and 4 d. 4 and 5
‚CH 3
8. The IUPAC name of C2H 5  O  CH ƒ is 13. The IUPAC name of the following compound is
CH3
[2006]
a. ethoxypropane [2008]
b. 1,1-dimethyl ether
c. 2-ethoxysopropane
d. 2-ethoxypropane
H 3C CH3
9. The IUPAC name of the compound, [2007]
a. 3, 5-dimethylcyclohexene
CH2 CH  COOH is b. 3, 5-dimethyl-1-cyclohexene
| | c. 1, 5-dimethyl-5-cyclohexene
OH NH2 d. 1, 3-dimethyl -5-cyclohexene

@iitjeehelps
Answer with Solutions
Practice Exercise 14. (b) The given structure of the compound is shown below.
4
1. (c) I, II and IV are aromatic compounds 5 3
[contain 6π- electrons]. The IUPAC name of will be oxocyclohex-2-ene.
6 2
O
2. (b) In toluene, the carbon atom of the ring attached to CH3 1
group is 3°. Cl
15. (c) Unsaturation (double bond) is given
3. (c) CH2 == CH  CH == CH2 priority over halogen. So, the correct 1
IUPAC name is 2
In this compound, each carbon issp 2-hybridised. 6
3-bromo-1-chlorocyclohexene
2 5
Hence, % of p- character = × 100 16. (a)
3
3 4 Br
1
= 66.67%. 2
3
2
4. (c) C1 issp and C2 issp -hybridised.
5. (c) Triple bonded carbon atoms are more electronegative 3-ethyl-1,1-dimethylcyclohexane
than double and single bonded carbon atoms. O

6. (a) When C-atom is double bonded to adjacent carbon 17. (b) I. CH3  C  C H2  CH2  CH2  COOH
atoms, the carbon atoms become linear in shape due to 6 5 4 3 2 1
the formation of two π-bonds simultaneously with two 5-oxohexanoic acid
other C-atoms. Hence, hybridisation comes out to be sp. II. CH ≡≡ C  C H == CH  C H == C H2
6 5 4 3 2 1
7. (a) Electrovalency in CaH2 and covalency in C2H2.
Hexa-1, 3-dien-5 yne
8. (d) 3-methyloctane can be represented by all of the above
1 2 3 4 5
forms. 18. (b) CH3  CH2  O  C H2  C  C H2  C H2  CH3
9. (c) Heterocyclic alicyclic compounds are the compounds 
in which any atom other than carbon is also present in the O
ring. 1-ethoxypentan-2-one

10. (c) CH3CONH2 is the isomer of the given structure and its 19. (b)
4
IUPAC name is ethanamide. 3 5
HO CH3
11. (c) Pyruvic aldehyde has following structure 2

 CHO O 1 CH3
CH3  C === O glyoxal is | 
  3-(hydroxymethyl) pent-3-ene-2-one
|  CHO
CHO *
20. (b) CH3  C H  CH2  CH3
12. (a) The given compound is shown as |
1 COOH
2 Only this compound has a chiral carbon-atom, hence it
5
3 4 will show optical isomerism.
6
21. (c)
COOH
The IUPAC name of above compound is
4-ethyl-3-propylhex-1-ene. NH2
13. (c) The given structure of the compound is shown below. Anthranilic acid

HOOC 22. (c) is an example of position isomer while (b) is of


1 2 metamerism. CH3CH2OH and CH3OCH3 are functional
3 group isomers.
1 23. (d) C4H8 does not exhibit optical isomerism.
2 3 H‚
CH3 ‚F
24. (c) The compound, C == C ƒ exhibits
ƒ
H3C Br
The IUPAC name will be
geometrical isomerism. E (trans)-Z(cis) as isomers.
3-(3-methyl phenyl) benzene-1-oic acid.

@iitjeehelps
GENERAL ORGANIC CHEMISTRY 623
25. (d) The structure for 2, 3-dichlorobutane is 35. (b) Tertiary halides, generally undergo SN1 reaction, as
H H the tertiary carbocation, obtained in this mechanism, is
  quite stable.
∗ ∗
CH3  C  C  CH3 CH3 CH3
  | |
Cl Cl CH3  C  CH3   Slow
→ CH3  C⊕  CH3 + Br −
This compound contain two chiral centres (asymmetric | 3 ° -carbocation
carbons). Therefore, it exhibits optical isomerism. Br
2- bromo -2 -methyl
26. (b) The compound C 5 H11 Br possesses 8 constitutional propane

isomers. 36. (c) The enol form of acetone is shown below


27. (b) On chlorination of 2-methylbutane, 4 optically active ••

• O H
isomers are formed.

Cl H C == C  CH3
Cl Cl 
Chlorination
H
+ * + *
Cl2/hν It shows that this compound possesses 9 σ-bonds,
(dl) (dl) 1π-bond and 2 lone pairs of electrons on O-atom.
37. (a) Larger the number of α - H-atoms linked to a
+
carbocation, more will be the stable carbocation.
Cl
Hence, the correct order of stability is
I < III < IV < II
28. (b) 2-oxo-3-methyl-(N-bromo) butanamide is
H3C O O 38. (c)
   39. (d) The compound must possess only keto group in order
H3 C  HC  C  C  NBr to shown enol form.
4 3 2 1
 O
H 
There is a methyl group at 3rd position and 'Br' at nitrogen The compound CH3  C  CH2  COOC 2 H5 contains
atom in the compound. keto group alongwith ester group which favours the enol
29. (d) formation by this keto group.
30. (b) 2-butene contains a double bond and the groups 40. (a) Stability of carbocation increases with the dispersal of
attached with double bonded carbon are different positive charge by the donation of electron density
therefore, it exhibits geometrical isomerism. The through +I-effect. The groups showing +I-effect are
geometrical isomers are as  OCH3,  CH3,  CH2  OCH3. Thus, the correct
H3C ‚ ‚ CH3 H3C ‚ ‚ H stability order of the given carbocations is
C ==== C ƒ C === C ƒ II > I > III
Hƒ H Hƒ CH3
cis -form trans -form 41. (d) Larger the number of phenyl groups linked to a free
31. (b) Chiral compounds have at least one chiral centre, radical, more the number of resonating structure and
i.e. all four atoms or groups attached to carbon are greater is the stability of free radical. Thus, the correct
different. order for the stability of free radicals is
• • • •
32. (d)
(CH3 )2 CH < (CH3 )3 C < (C6H5 )2 C H (C6H5 )3 C
(i) Enantiomers are the pairs of optical isomers which are
related as non-superimposable mirror images of each 42. (b)
other. 43. (b) —COOH > — SO3H > — CONH2 > — CHO
(ii) Diastereomers are the pairs of optical isomers which 1
cannot be related a non-superimposable mirrror 44. (d) Nucleophilicity ∝
Electronegativity of atom
images of each other. bearing negative charge
∴The only correct statement about given structures is that •
(A) and (B) are enantiomers. 45. (b) I. CH3 — X → CH3 + X •
33. (c) This carbanion show resonance Free radicals

H3C ‚ − H3C ‚ −
II. CH3 — X → CH3 +X –
ƒ C  CH ==O ←→ ƒ C == CH O
H3C H3C Carbocation

34. (b) In the presence of peroxide, the addition of HBr to III. CH3 — X → CH3 +X ⊕
propene follows free radical mechanism. Carbanion

@iitjeehelps
624 SELF STUDY GUIDE BITSAT

46. (c) r In general, greater the number of alkyl groups attached to a


C CH3 positively charged carbon atom, the greater is the
hyperconjugation interaction and stabilisation of the cation.
CH3 Thus, we have the following relative stability of carbocation
3° carbocation is more stable CH3
| + ⊕ ⊕
47. (a) The reaction can proceed through SN 2
CH3 C⊕ > (CH3 )2 CH > CH3 CH2 > CH3
mechanism and it is stereospecific. A reaction is |
stereospecific when a particular stereoisomeric form
CH3
of the starting material reacts in such a way that it
gives a stereoisomeric form of the product. Hence, stability of carbocation is directly proportional to number
48. (d) of alkyl group directly attached to carbocationic carbon.
49. (a) Acidic strength depends upon the tendency of a 51. (b)
molecule+to give a proton. Among the given, 52. (a) In homolysis, the covalent bond is broken in such a way
CH3CH2 OH2 readily gives a proton, so it is highly that each resulting species known as free radical.
acidic. CH3CH2CH3 has the least tendency to lose a Homolytic • •
proton, so its acidic strength is least among the CH3  CH2  Cl → CH3 CH2 + C l
fusion
given. Thus, the correct order of acidic strength is
53. (b) and (c) are enols and will tautomerise into
IV < II < I < III
CH3  C  H and CH3  CH  CHO respectively.
50. (b) The stability of carbocation on the basis of || |
hyperconjugation can be explained as O CH3
hyperconjugation stabilises the carbocation because
54. (c) Dichlorocarbene ( •• CCl2 ) is an electrophile.
electron density from the adjacent π-bond helps in
dispersing the positive charge. 55. (a)
H H + H 56. (d) The observed order of alkyl radical stability is
H
– • • • •
H C C H C C CH3 < CH3 CH2 < (CH3 )2 C H < (CH3 )3 C

H H H H

H H H H
+
HC C H C+ C
H
H H H

BITSAT Archives
1. (d) Carbocation stability The carbocation is formed during the
2. (c) This problem includes conceptual mixing of reaction of alcohol by removal of OH. More stable the
carbocation, more will be its reactivity of carbocation formed
carbocation stability and reactivity of alcohol.
during reaction are as given
Remove the OH group by dehydration and then
arrange the carbocation in increasing order of 3. (c) (a) CH3  CH2 — CH2 == CH2
correctly. Order of carbocation stability is same as sp 3 sp 3 sp 2 sp 2
SN1 reactivity of alkyl halides. (b) CH3  CH == CH — CH2 +

CH3 sp 3 sp 2 sp 2 sp 2

(c) CH2 == CH CH == CH2


F sp 2 sp 2
carbocation increases

+ sp 2 sp 2
CH3 (d) CH3  CH == CH CH−2
Stability of

sp 3 sp 2 sp 2 sp 3
F +
H 3C CH3 4. (a) Among the given, only CH3CH == NOH (oxime) satisfy the
conditions essential for exhibiting geometrical isomerism. So,
+
+ it will exhibit syn-anti geometrical isomerism.

@iitjeehelps
GENERAL ORGANIC CHEMISTRY 625
H H + H+ +
H3 C H3 C
10. (b) (CH3 )3 C  OH → (CH3 )3 C
C C Tertiary Alcohol − H2 O 3 ° carbocation
  (more stable)
N N +
+ H CH == CH  CH  C H
CH2 == CH CH2  CH2OH →
+
OH HO 2 2 2
syn anti − H2O 1° carbocation
(less stable)
5. (a)
6. (b) Increasing order of stability of carbocation is
1 2 3 4 1° carbocation < 2° carbocation < 3° carbocation
7. (d) CH 3  CH CH2  CH3
11. (a) Tautomerism It is functional isomerism in which the
isomers are readily interchangeable and maintain a dynamic
equilibrium with each other.
OH
2-cyclohexylbutane CH CH l CH2CHO
1 Enol form Keto form
2 ‚ CH3
8. (d) C2H5  O  CH ƒ OH
CH3
CH3 H |
2-ethoxypropane 12. (b) C== C ‚CH2  C  CH3
The above compound is an ether and its name is H CH2 |
written as alkoxy. Oxy is attached with the lower group. H
Hence, the IUPAC name of above compound is It exihibits 2 geometrical and 2 optical isomers.
2-ethoxypropane.
13. (a)
9. (a) C H2 CH  COOH
| |
OH NH2 H 3C CH3
2-amino-3-hydroxypropanoic acid IUPAC name : 3, 5-dimethylcyclohexene

@iitjeehelps
22
Purification and
Estimation of
Organic Compounds

Purification
Methods of Purification of Organic Compounds
The methods employed for the purification of organic compounds are :

Filtration
It is based on the difference in filterability of different components of a solution. This method is used
for the separation of insoluble solid components of the mixture from soluble components in a given
solution, e.g. separation of sand from salt by dissolving in water.

Crystallisation
This method is used for the purification of organic compounds that are obtained in the pure form of
their crystals. This technique is based upon the differences in the solubility of the compound and
impurities in different solvents.
The common solvents used for dissolving organic compounds are water, alcohol, ether, benzene,
petroleum and chloroform.

Sublimation
This technique is applied to those solids which convert directly into the gaseous state upon heating
without converting them into liquid phase and the vapours give directly the pure solid substance.
Organic compounds like naphthalene, anthracene, camphor, benzoic acid, ammonium chloride,
salicylic acid, iodine are purified through sublimation.
NOTE A mixture of two or more sublimates cannot be separated by sublimation technique. Such mixtures are
separated only by chemical methods.

@iitjeehelps
PURIFICATION AND ESTIMATION OF ORGANIC COMPOUNDS 627
Distillation Chromatography
It is based upon the difference in the boiling points of two This technique is used for the separation of an organic
liquids or two components and is generally used, if an compound present in a mixture in very small amount.
organic liquid contains a non-volatile impurity.
Principle of Chromatography
Several distillation methods employed to separate mixtures
This technique is based on the principle that the components
are as follows :
of a given organic mixture is distributed between two
(i) Simple distillation Generally, the non-volatile phases : stationary and mobile phase.
impurities and organic liquids which differ in their
The stationary phase may be a solid or tightly bound liquid
boiling points by 30-50 K are separated by this
supported over a solid whereas the mobile phase may be a
process, e.g. a mixture of ether and toluene is separated
liquid or gas.
by this method.
The process of distribution depends upon the rate of
(ii) Fractional distillation This technique is employed to
differential movement of individual components through a
those organic liquids which do not possess large
stationary phase under the influence of a mobile phase
difference in their boiling points. It is carried out in a
(gas/liquid).
fractionating column which increases the surface area
for cooling the vapours of liquid. Types of Chromatography
e.g. Mixtures of acetone (329 K) and methyl alcohol Based upon the nature of stationary and mobile phase,
(338 K), benzene (353 K) and toluene (383 K) are various types of chromatography techniques alongwith their
separated through this technique. applications are listed below :
(iii) Steam distillation This technique is applied to those Type of Mobile Stationary
Application of
compounds which are volatile in steam but insoluble in chromatographic
chromatography phase phase
technique
water and contain non-volatile impurities.
Column Liquid Solid Used for the purification of
In this type of distillation, the mixture boils at that
(adsorption) organic compounds for
temperature, when the sum of vapour pressures of chromatography both commercial and small
organic liquid (p1 ) and that of steam (p2 ) becomes scale separations.
equal to the atmospheric pressure (p). High performance Liquid Solid Used for the identification
Therefore, p = p1 + p2 . liquid and characterisation of
chromatography organic compounds for
e.g. ortho and para-nitrophenols aniline, nitrobenzene, (HPLC) their quantitative and
sandal wood oil and terpentine oil etc., are purified or qualitative analysis.
obtained by this process. Thin layer Liquid Solid Only for qualitative
chromatography analysis.
(iv) Vacuum distillation Certain liquids tend to (TLC)
decompose at a temperature below their boiling point.
Gas liquid Gas Liquid For both qualitative and
Such liquids are purified by carrying out their
chromatography quantitative analysis.
distillation under reduced pressure. Therefore, the (GLC)
liquid boils at lower temperature without undergoing
Paper Liquid Solid For both quantitative and
decomposition. chromatography qualitative analysis of
polar organic and
e.g. The boiling point of glycerol is 563 K but it
inorganic compounds.
decomposes below this temperature. But when the
pressure is reduced to 12 mm, glycerol boils at 453 K
without decomposition.
Estimation of Organic
Differential Extraction
In this technique, an aqueous solution of an organic
Compounds
compound, taken in a separating funnel is shaken with a Estimation of organic compounds are as follows :
suitable organic solvent in which organic compound is
highly soluble. On shaking the funnel, organic layer Qualitative Analysis of Organic
separates out from an aqueous layer.
Compounds
The common solvents used are chloroform, (CCl 4 ), ether,
This method involves the conversion of covalently bonded
dichloromethane and ethanol. If the extraction is done in
nitrogen, sulphur or halogens present in the organic
small amounts, the extraction will be more efficient and
compounds to corresponding water-soluble ions, in the
hence, larger amounts of organic compound are extracted.
form of sodium salts.

@iitjeehelps
628 SELF STUDY GUIDE BITSAT

Detection of Carbon and Hydrogen Estimation of C and H in an Organic


Organic compound when heated with CuO, carbon Compound
changes into CO2 (tested with lime water which develops Liebig method is used to determine the presence of C and H
turbidity) and hydrogen to H 2O (tested with anhydrous in an organic compound. In this method, known weight of
copper sulphate which turns blue). This confirms the organic compound is heated strongly with dry CuO in an
presence of C and H in organic compound. atmosphere of CO2 free air or oxygen.
Detection of Other Elements D
C(s ) + 2CuO(s ) ¾® CO2 ( g ) + 2Cu(s )
Nitrogen, sulphur, halogens and phosphorus present in an
2H ( g ) + CuO(s ) ¾® H 2O( g ) + Cu(s )
organic compounds are detected by Lassaigne’s test. The
elements present in the organic compounds are converted Vapours of CO2 passed through a weighed potash bulb,
from covalent form into the ionic form by fusing the which results in increased weight of potash by the
compound with sodium metal. absorption of CO2 .
D While the vapours ofH 2O passed throughCaCl 2 which leads
Na + C + N ¾® NaCN
to increased weight of CaCl 2 . The increased weight of
D
2Na + S ¾® Na 2S potash and CaCl 2 is used to determine the percentage of C
D and H.
Na + X ¾® NaX [X = Cl, Br, I] 12 ´ WCO 2 ´ 100
Test for Nitrogen (i) % of C =
44 ´ W organic compound
6CN - + Fe 2+ ¾® [Fe(CN)6 ]4 -
2 ´ W H 2O ´ 100
4- 3+ x H 2O (ii) % of H =
3[Fe(CN)6 ] + 4Fe ¾¾® Fe 4[Fe(CN)6 ]3 × xH 2O 18 ´ W organic compound
Prussian blue

Lassaigne’s test is given by compounds carrying nitrogen Estimation of N in an Organic Compound


and carbon. So, hydrazine (NH 2 ¾ NH 2 ) and hydroxyl
There are two methods used to determine the percentage of N
amine (NH 2OH) do not give this test.
in an organic compound :
Test for Sulphur (i) Duma’s method In this method, a known weight of
(i) S2 - + Pb 2+ ¾® PbS organic compound is heated with dry CuO in a
Black
combustion tube to form CO2 and water vapour while
(ii) Na + C + N + S ¾® NaSCN N 2 is set free. Although, some oxides of nitrogen may
Fe3 + + SCN - ¾® [Fe(SCN)]2+ be formed but these oxides can be reduced to N 2 when
Blood red passed through hot reduced copper spiral.
Test for Halogens The vapours of N 2 , H 2O and CO2 pass through KOH
The sodium fusion extract is acidified with nitric acid and solution, where N 2 gets collected over KOH solution.
then treated with silver nitrate. While H 2O and CO2 are absorbed by KOH. The volume of
N 2 is recorded from the level of KOH solution in
X - + Ag + ¾® Ag X [X = Cl or Br or I]
nitrometer tube.
AgCl-white ppt, AgBr-dull yellow ppt, AgI-bright yellow ppt 28 ´ V N 2 at STP ´100
% of N =
Test for Phosphorus 22400 ´ W organic compound
The compound is heated with an oxidising agent
(ii) Kjeldahl's method This method is used for the
(like sodium peroxide), the phosphorus present in the
estimation of N present in food stuffs, soil fertilisers
compound is oxidised to phosphate. The solution is boiled
and drugs. In this method, weighed amount
with nitric acid and then treated with ammonium
of nitrogenous compound is heated strongly with
molybdate. A yellow colouration or precipitate indicates
conc. H 2SO4 in the presence of K 2SO4 (which raises
the presence of phosphorus.
boiling point of solution), N is converted quantitatively
into ammonium sulphate.
Quantitative Analysis of Elements Conc. H 2 SO 4
After the detection of elements present in the organic C + H + N ¾¾¾¾¾® (NH 4 )2 SO4 + CO2 + H 2O
From organic D
compound, the percentage of different constituent compound
elements is determined by the help of suitable methods. (NH 4 )2 SO4 + 2NaOH ¾
¾® Na 2SO4 + 2NH3 ­ + 2H 2O
The knowledge of molecular mass and percentage
composition helps in the determination of the molecular Ammonia evolved is condensed and absorbed by a
formula of the compound. known volume of standard solution of HCI or H 2SO4 .

@iitjeehelps
PURIFICATION AND ESTIMATION OF ORGANIC COMPOUNDS 629
The percentage of N present in the compound is From the weight of barium sulphate obtained, the
calculated by percentage of sulphur is calculated as
14 V acid ´ N acid 32 W BaSO4 formed
% of N = ´ ´ 100 % of S = ´ ´ 100
1000 W organic compound 233 W compound

Estimation of Halogen Present in an Organic Estimation of Phosphorus Present in an


Compound Organic Compound
Carius method In this method, halogen atom present in the The phosphorus present in an organic compound is
organic compound is converted into insoluble silver halide estimated only by Carius method. The weighed amount of
which is separated and weighed. organic compound is heated with conc. HNO3 to convert the
Conc. HNO 3
phosphorus present in the compound into phosphoric acid.
X ¾¾¾¾¾® AgX (From organic compound) Phosphoric acid reacts with magnesia mixture to form
Halogen AgNO3 Silver halide magnesium ammonium phosphate, Mg(NH 4 )PO4 .
From the weight of the silver halide, the percentage of The compound, Mg(NH 4 )PO4 is collected, washed, dried
halogens can be calculated as, and ignited to get magnesium pyrophosphate, Mg 2P2O7 .
35.5 W AgCl formed MgCl 2 + NH 4Cl + H3PO4 ¾® Mg (NH 4 )PO4 + 3HCl
% of Cl = ´ ´ 100
143.3 W compound Magnesium
ammonium
phosphate
80 W AgBr formed D
% of Br = ´ ´ 100 2 Mg(NH 4 )PO4 ¾® Mg 2P2O7 ¯ + 2NH3 + H 2O
188 W compound
Knowing the weight of Mg 2 P2O7 , percentage of P can be
127 W Agl formed calculated as
% of I = ´ ´ 100
235 W compound 62 W Mg 2 P2 O7 formed
% of P = ´ ´ 100
222 W compound
Estimation of Sulphur Present in an Organic
Compound Estimation of Oxygen Present in an Organic
The sulphur present in an organic compound is also Compound
estimated through Carius method. Generally, oxygen is estimated as left over percentage, i.e.
In this method, a weighed amount of organic compound is 100 minus percentage of other elements.
heated with fuming nitric acid in a Carius tube. Compound ¾Heat
¾¾® O2 + Other gaseous products
The sulphur of the compound is oxidised to sulphuric acid
2C + O2 ¾1373
¾¾ K
® 2CO
which is then precipitated as barium sulphate by adding
excess of barium chloride solution. I 2O5 + 5CO ¾ ¾¾® I 2 + 5CO2
Fuming
S ¾ ¾ ¾ ® H 2SO4 The percentage of oxygen can be derived from the amount
From the nitric acid
compound of CO2 or iodine produced.
32 ´ m1 ´ 100
H 2SO4 + BaCl 2 ¾® BaSO4 ¯ + 2HCl % of O = %
44 ´ m

@iitjeehelps
Practice Exercise
1. Absolute alcohol is prepared by 10. In sulphur estimation, 0.157 g of an organic
a. fractional distillation compound gives 0.4813 g of barium sulphate. The
b. Kolbe's method percentage of sulphur in the compound, is
c. azeotropic distillation a. 42.10
d. vacuum distillation b. 52.10
2. Aniline is purified by c. 21.05
a. steam distillation b. simple distillation d. 40.01
c. vacuum distillation d. extraction with a solvent 11. In Kjeldahl’s method, ammonia from 5 g of food
3. Name the technique where extraction of compound neutralises 30 cm 3 of 0.1 N acid. The percentage of
takes place based on difference in solubility. nitrogen in the food is
a. Differential extraction a. 0.84 b. 0.4
b. Chromatography c. 16.8 d. 1.68
c. Sublimation
d. Crystallisation
12. 0.765 g of an acid gives 0.535 g of CO2 and 0.138 g of
H2O. Then, the ratio of the percentage of carbon and
4. The technique used for the separation of acetone and hydrogen is
methanol, is a. 19 : 2 b. 18 : 11
a. steam distillation c. 20 : 17 d. 1 : 7
b. vacuum distillation
c. fractional distillation 13. 0.59 g of an organic compound produces 112 mL
d. simple distillation nitrogen at NTP. The percentage of nitrogen in the
compound through Duma’s method is
5. The best and latest technique for isolation, purification
a. 23.7 b. 27.5
and separation of organic compound is c. 33.07 d. 16.8
a. crystallisation b. distillation
c. sublimation d. chromatography 14. If 0.1 g of an organic compound containing
phosphorus, produces 0.222 g of Mg2PO2, the
6. The compound that does not give blue colour in
percentage of phosphorus present in the compound is
Lassaigne's test is
a. 31 b. 0.2
a. aniline b. glycine
c. 66 d. 62
c. hydrazine d. urea
7. When FeCl3 is added to a (FeSO4 + sodium extract) 15. A compound of carbon, hydrogen and nitrogen
solution of compound, blood red colour appears. It contains three elements in the ratio of 9 : 1 : 3.5. The
shows the presence of empirical formula of the compound is
a. N and S in the compound a. C2H4N
b. S in the compound b. C3H4N
c. N and P in the compound c. C3H6N
d. Br in the compound d. C2H6N

8. An organic compound on heating with CuO produces 16. A substance is found to contain 7% nitrogen. The
CO2 but no water. The organic compound may be minimum molecular weight of it, is
a. carbon tetrachloride a. 700 b. 100
b. chloroform c. 200 d. 70
c. methane 17. When FeCl3 is added to the sodium extract of an
d. ethyliodide organic compound containing both N and S, the red
9. Which of the following complex formation indicates the coloured precipitate is formed due to the formation of
presence of sulphur in the organic compound when a. [Fe(CN)6] 3-
sodium nitroprusside is added to sodium extract of the b. [Fe(CNS)] 2+
compound? c. [Fe(CNS)2] +
a. Fe4[Fe(CN)6] 3 b. Na 2[Fe(NO)(CN)5]
d. [Fe(CN)6] 4-
c. Fe4(CNS)3 d. Na 4[Fe(CN)5 NOS]

@iitjeehelps
Answer with Solutions
1. (c) Absolute alcohol (100% pure ethyl alcohol) is obtained 11. (a) From Kjeldahl’s method,
from rectified spirit (95.87% ethanol + 4.13% water) by 1.4 ´ N ´V 1.4 ´ 0.1 ´ 30
azeotropic distillation as it is an azeotropic mixture. % of N = =
m 5
2. (a) Aniline decomposes on its boiling point and it is also = 0.84
insoluble in water, hence it can be distilled with steam for 12 0.535
purification. 12. (a) % of C = ´ ´ 100 = 19.07
44 0.765
3. (a) Differential extraction is the technique where 2 0.138
extraction of compound takes place based on difference % of H = ´ ´ 100 = 2.004
in solubility. 18 0.765
4. (c) The compounds acetone and methanol possess equal Ratio of % of C : H = 19 : 2 (approx.)
boiling points therefore, these are separated by fractional 13. (a) % of N through Duma’s method is given by
distillation technique. 28 ´ Volume of N2 at STP
% of N = ´ 100
5. (d) Chromatography is the best and suitable technique used 22400 ´ Mass of an organic compound
for the separation of organic compound from their impurities. 28 ´ 112 ´ 100
= = 23.7
6. (c) Hydrazine (NH2 × NH2 ) has no carbon content, hence 22400 ´ 0.59
does not form NaCN in Lassaigne's test. Therefore, it
14. (d) The percentage of P present in the organic compound
does not give blue colour.
is given by
7. (a) When N and S, both are present in compound, it forms 62 Weight of Mg2P2O7
NaCNS with sodium extract. This gives ferric thiocyanate % of P = ´ ´ 100
222 Weight of organic compound
with FeCl3 solution, hence blood red colour develops.
62 0.222
3 NaCNS + FeCl3 ¾® Fe(CNS)3 + 3NaCl = ´ ´ 100 = 62
Blood red 222 0.1
8. (a) Since, the compound on heating with CuO produces 15. (b) The percentage (or mass) ratio = 9 : 1: 3.5
CO2 which contains carbon. Again, it does not produce 9 1 3.5
water, hence it does not contain hydrogen. So, the \ Mole ratio = : :
12 1 14
organic compound is carbon tetrachloride (CCl4 ).
= 0.75 : 1 : 0.25
9. (d) The formation of sodium thionitroprusside (purple)
= 3 : 4 :1
shows the presence of sulphur.
Hence, empirical formula = C3H4N
Na 2S + Na 2[Fe(NO)(CN)5] ¾® Na 4[Fe(CN)5 NOS]
Sodium extract Sodium nitroprusside Sodium thionitroprusside 16. (c) Atomic weight of nitrogen is 14. If one atom of nitrogen
{purple}
is present in compound, then it should have molecular
10. (a) Molecular mass of BaSO4 = 137 + 32 + 64 = 233 g 14
weight = ´ 100 = 200
233 g of BaSO4 contains 32 g of sulphur. 7
\0.4813 g of BaSO4 will contain 17. (b) When FeCl3 is added to the sodium extract of an
32 ´ 0.4813 organic compound containing both N and S, a red
= g of sulphur
233 coloured precipitate of [Fe(CNS)] 2+ is formed.
32 ´ 0.4813 ´ 100 NaCNS + FeCl3 ¾® [Fe(SCN)]Cl2 ¯ + NaCl
% of S = = 42.10
233 ´ 0.157 Red coloured complex

@iitjeehelps
23
Hydrocarbons

Concept of Hydrocarbons
Hydrocarbons means compounds containing carbon and hydrogen only. These may contain only
single bonds or double/triple bonds alongwith single bond. Only single bond containing
hydrocarbons are saturated hydrocarbons while that have multiple bond(s) (== or ≡≡ bond)
alongwith single bonds are unsaturated compounds. Hydrocarbons are mainly obtained from
petroleum, which is the major source of commercial energy.
Hydrocarbons are of different types and classified into three main categories :
(i) Saturated hydrocarbons (alkanes)
(ii) Unsaturated hydrocarbons (alkenes and alkynes)
(iii) Aromatic hydrocarbons

Alkanes
Introduction
⇒ Hydrocarbons having C C single bond.
⇒ General formula : C nH 2 n + 2
⇒ sp3 -hybrid orbitals having tetrahedral geometry and bond angle 109°28′.
⇒ Also called paraffins (less reactive) as:
(a) strong C C bond
(b) non-polar C H bond

Isomerism
(a) Chain isomerism

Butane Isobutane

@iitjeehelps
HYDROCARBONS 633
(b) Position isomerism Physical Properties
C C C C C C C C C C 1. C 2 C 4 → gaseous state
 
C 5 C 17 → liquid state (except neo - pentane)
C C
(2-methyl pentane) (3-methyl pentane) C18 and above → solid like wax.
2. Alkanes are colourless, odourless and tasteless.
Preparation
H 3. Insoluble in water and soluble in organic solvents.
1. From alkenes C nH 2 n  
2
→ C nH 2 n + 2
Ni, ∆ 4. The mp of unbranched alkanes having 2, 4, 6, ... (even)
[known as Sabatier-Sanderen’s reaction]. carbon atoms is higher than that of alkanes having
H 3, 5, 7, ... (odd) carbon atoms.
CH 2 == CH 2  
2
→ CH3 CH3
Ni , ∆ 5. Alkane float on water as they have lower density.
2H
2. From alkynes C nH 2 n − 2  
2
→ C nH 2 n + 2 6. Decreasing order of fuel capacity is :
Ni , ∆
branched > unbranched lower mol. wt. > higher mol. wt.
3. From alkyl halides As the percentage of C in a hydrocarbon increases, its
(a) By reducing alkyl halides calorific value (fuel capacity) decreases.
Reducing agents Zn - Cu couple LiAlH 4 , Al-Hg Chemical Properties of Alkanes
amalgam, Na/K/alc. Li/Ni 300° , B2H 6
1. Halogenation : RH + X 2 
hv
→ R  X + HX
R  X + 2H Zn-Cu
→ R H + H  X
R OH Reactivity order : 3 ° > 2 ° > 1° [for C H bond]
[ I2 is removed as 2P + 3I 2 → 2PI3 ] Reactivity of halogen : F2 > Cl 2 > Br2 > I 2
(b) Wurtz reaction CH 4 
hv
→ CH3Cl, CH 2Cl 2 , CHCl3 , CCl 4
Dry Cl 2
R  X + R  X + 2Na  
→ R  R + NaX
ether Mechanism Cl 2 
hv
→ Cl• + Cl•
CH3Cl + 2Na + CH3Cl → CH3 CH3 + 2NaCl CH 4 + Cl• → CH•3 + HCl
(c) Frankland’s method 2 RX + Zn → R  R + ZnX 2
CH•3 + Cl 2 → CH3Cl + Cl•
[Here, R Zn  R is formed which on reaction with
R  X gives higher alkane] CH3Cl + Cl• → CH 2Cl• + HCl
CH3 Br + 2Zn + CH3 Br → ⇒ Iodination is reversible process
CH3 Zn CH3 2CH
 

Br
→ 2CH3 CH3 + ZnBr2
3
CH 4 + I 2 r CH3I + HI [HI reduces CH3I]
4. From aldehyde and ketone To avoid this HIO3 is used which destroys HI on its
formation.
(a) Clemmensen’s reduction
5HI + HIO3 → 3I 2 + 3H 2O
R CHO + 2H 2 Zn-Hg
 → R CH3 + H 2O
HCl ⇒ Chlorination by sulphuryl chloride
RCOR + 2H 2 Zn-Hg
 → RCH 2 R + H 2O hv
HCl RH + SO2Cl 2 → RCl + SO2 + HCl
Organic peroxide
(b) Wolf Kishner reduction HNO
2. Nitration CH3CH 2CH3  
3
→ CH3CH 2CH 2 NO2
RCHO + N 2H 4 NaOH
→ RCH3 + N 2 ↑ + H 2O 400 °C
Hydrazine
+ CH3  CH  CH3 + CH3CH 2 NO2 + CH3 NO2
5. From carboxylic acid 
(a) Reduction RCOOH Red
 P
→ RCH3 + 2H 2O + 3I 2 NO2

3. Sulphonation
(b) Decarboxylation SO 3
∆ (CH3 )3 CH + H 2SO4  
→ (CH3 )3 C SO3H
RCOONa + NaOH → RH + Na 2CO3
400 °C
(CaO)
4. Pyrolysis CH 2 ==CH 2 + CH 4 ←   CH3CH 2CH3
600 °C
(c) Kolbe’s electrolysis
400 °C
2 RCOOK Electrolysis
 → R  R + 2CO 2 + 2K  → CH3CH == CH 2 + H 2
−600 °C

6. From aluminium carbide 5. Combustion CH 4 + 2O2 → CO2 + 2H 2O


Al 4C3 + 12H 2O → 4Al(OH)3 + 3CH 4  3n + 1
C nH 2 n + 2 +   O2 → nCO2 + (n + 1) H 2O + ∆
[Only CH 4 is formed by this reaction].  2 

@iitjeehelps
634 SELF STUDY GUIDE BITSAT

Cu [O] [O] Mechanism


6. CH 4 + O2 → CH3OH → HCHO → HCOOH
200 °C / 100atm +
H
CH3CH2OH CH3 CH2 OH2+

b
Cr O -Al O
7. Aromatisation n − hexane  2 
3
2
3
→ −H2O
600 °C CH2
+
CH2
CH2 + CH2
–H
Benzene
H
Alkenes
5. Wittig reaction
Introduction
6. Electrolysis of aqueous solution of sodium salts of
⇒ Hydrocarbons having C == C double bond.
carboxylic acids
⇒ General formula C nH 2 n ⇒ sp 2 -hybridisation.
CH 2COONa
⇒ Trigonal planar [(120°) bond angle].  + 2H 2O Electrolysis
 →
CH 2COONa
Isomerism Sodium succinate
Chain 1-butene, 2-methyl propene CH 2 == CH 2 + 2CO2 + 2NaOH + H 2
Position 1-butene, 2-butene
Physical Properties
Functional 1-butene, 2-butene,
1. C 2 C 4 → gases
2-methyl propene, cyclobutane etc.
C 5 C 15 → liquids > C 15 → solids
Geometrical cis-2 butene, trans-2 butene
2. Insoluble in water, soluble in polar solvents.
Preparation 3. bp ∝ molecular weight

1. Cracking of alkanes C 2H6 → C 2H 4 + H 2 4. More polar than corresponding alkanes.
600 °C

CH3CH 2CH3 → CH3CH == CH 2+ H 2 Chemical Properties of Alkenes
600 °C
1. Electrophilic addition reactions
2. Partial hydrogenation of alkynes
‚ ‚ ‚ ‚
R R C Cƒ + E Nu C Cƒ
Pd /CaCO 3 ‚ ƒ ƒ ƒ
(a) R C ≡≡ C  R    
→ C == C
or Pd / BaSO 4 ƒ ‚ E Nu
[Lindlar's catalyst] H cis H
Mechanism
(b) Using sodium in NH 3
R H ‚ ‚
‚ ƒ C ƒ + E δ+ Nuδ–
Li, Liquid NH Slow
R C ≡≡ C  R     
3
→ C == C ƒ
C
ƒ ‚
H R E E
trans
‚ ⊕‚
trans
3. Dehydrohalogenation of alkyl halides ƒC C ƒ+ Nu– C C
addition
Alc. KOH
CH3CH 2CH 2Br   → CH3CH == CH 2 Nu
Ease of reaction :3 ° > 2 ° > 1° [alkyl halide]
By dehalogenation of vicinal halide. (a) Hydrogenation
CH 2BrCH 2Br + Zn → CH 2 == CH 2 + ZnBr2 CH3 CH == CH 2 Pt,
Pd or
Ni
 → CH3 CH 2 CH3
H2 cis-addition
Conc. H SO
4. Dehydration of alcohols ROH    
2 4
→ Alkene
(b) Halogenation
Ease of dehydrogenation : 3 ° > 2 ° > 1°
Me CH == CH 2 FeCl
 3
→ Me CHCl CH 2Cl
Dehydrating agents I 2 or Cl 2

Al 2O3 [350° C], P2O5 , H3PO4 , anhyd. ZnCl 2 , conc. H 2SO4 . CH3 CH == CH 2 + I 2 r CH3 C H C H 2
Conc.
C 2H 5OH  → H 2C == CH 2  
H 2 SO 4 I I

@iitjeehelps
HYDROCARBONS 635
Mechanism Step II Demercuration
NaBH 4
Br Br CH3 CH CH 2 HgOCOCH3   →
 NaOH
‚ ‚ Brδ+ Brδ– ‚ ‚ OCOCH3
C Cƒ C Cƒ
ƒ ƒ CH3  CH CH3 + Hg

Br+ OH

C C + Br− (a) Hydroxylation of alkenes


(Oxidation by alkaline KMnO4 )
Br ‚ ‚ KMnO4
‚ C Cƒ C C
‚ ƒ OH–cold
C Cƒ
ƒ OH OH
Br (i) OsO , pyridine
 
  4   → C  C 
(c) Hydrohalogenation
(ii) NaHSO 3 / H 2O
 
HX OH OH
CH3 CH == CH 2  → CH3  CH CH3
Markownikoff's
 (b) Oxidation by hot conc. KMnO4
addition
X CH 2 == CH 2 [O]
→ 2CO2 + H 2O
Kharash effect
CH3CH == CH 2 [O]
→ CH3COOH + CO2 + H 2O
CH3 CH == CH 2 HBr
→ CH3 CH 2  CH 2 Br
Peroxide CH3 C == CHCH3 [O]
→ CH3 C == O + CH3COOH
(d) Chlorohydroxylation  
+ CH3 CH3
CH3 CH == CH 2 HOCl
 → CH3  CH CH 2Cl

OH Alkynes
(e) Hydration Introduction
Dil. H SO
CH3 CH == CH 2   2 
4
→ (CH3 )2 CHOH l Hydrocarbons having C ≡≡ C triple bond.
H 2O
l General formula CnH 2 n − 2
(f) Ozonolysis
O l sp-hybridisation [(180)° bond angle]
CH3 CH == CH 2  
3
→ Ozonide
H O
l Isomerism

2
→ CH3CHO + HCHO
Zn Chain 1-pentyne, 3-methyl-1-butyne.
(g) Hydroboration Position 1-pentyne, 2-pentyne
B2 H 6
CH3 CH == CH 2  → Functional 1-3 butadiene, 1-2 butadiene, cyclobutene,
H 2O + 1-butyne.
(CH3 CH 2 CH 2 ) 3B Preparation
H3O+ −
OH 1. From alkanes: Partial oxidation of methane
H 2O 2
4CH 4 + 3O2 → 2C 2H 2 + 5H 2O
CH3 CH 2 CH3 H3C CH 2 CH 2 OH 2. Synthesis: 2C + H 2 Electric
 → HC ≡≡ CH
arc. (1100 ° ) Ethyne
(h) Addition of NOCl
δ+ δ− 3. Dehalogenation of tetrahalides
 Cl
CH3 CH == CH 2 ON → CH3 CH CH 2
  Br Br
Cl NO  
H  C  C H + Zn → H  C == C H
(i) Oxymercuration-Demercuration    
Step I Oxymercuration Br Br Br Br
Hg(OCOCH )
CH3 CH == CH 2     →
3 2

Zn
→ CH ≡≡ CH
THF
4. From calcium carbide
CH3 CH CH 2HgOCOCH3
°C
 CaO + 3C 2500
  → CaC 2 + CO ↑
OCOCH3
CaC 2 + 2H 2O → C 2H 2 + Ca(OH)2

@iitjeehelps
636 SELF STUDY GUIDE BITSAT

Physical Properties (c) Addition of halogen acids


1. C 2 C 4 → gases CH ≡≡ CH + HBr → CH 2 == CHBr

C 5 C 14 → liquids > C 14 → solids. HBr


→ CH3 CHBr2
2. Slightly soluble in water. 3. Nucleophilic addition reaction
3. mp/bp ∝ molecular weight. C ≡≡ C  + Nu →  C == C 

Chemical Properties of Alkynes Nu
1. Acidic characters of alkynes The hydrogen atoms
linked to triple bonded carbon atoms is highly acidic in (a) Addition of water (H 2O) in presence of acids
HgSO
nature. CH ≡≡ CH+ H 2O   
4
→ CH 2 ==CHOH
H 2 SO 4
The reactions involving highly acidic H-atoms are as Ketonisation
follows: ←   → CH3 CHO
Acetaldehyde
CH Na
C  Na Na C  Na
(a)  →  →  (b) Addition of arsenic trichloride (AsCl3 )
CH − 1 H 2 C H 1
− H2 C  Na
2 2 CH ≡≡ CH + ClAsCl 2 → CHCl == CHAsCl 2
Monosodium Disodium Lewisite (poisonous gas)
acetylide acetylide

Acetylene also reacts with sodamide to form mono (c) Addition of hydrogen cyanide (HCN)
and disodium acetylide and ammonia. CH ≡≡ CH + HCN → CH 2 == CHCN
(b) Reaction with Tollen’s reagent (d) Addition of acetic acid
C H C  Ag Hg 2 +
 + 2AgNO3 + 2NH 4OH →  CH ≡≡ CH + CH3COOH  →
C H (Tollen’ s reagent) C  Ag CH COOH
H 2C == CH O  C CH3  3 
→
+ 2NH 4Cl + 2H 2O 
White ppt O
NOTE Greater the s-character present in a molecule higher is the CH3 CH (O  C CH3 )2
acidity of molecule. 
CH CH > CH2 CH2 > CH3 CH3 O
4. Polymerisation
(larger s-character) sp 2 -hybridised sp3-hybridised
sp-hybridised (33%) s-character (25%) s-character 2CH ≡≡ CH CuCl
 2
→ CH 2 == CH C ≡≡ CH
50% s-character NH 4Cl

2. Electrophilic addition reaction HCl


→ CH 2 == CH  C == CH 2
(Follows Markownikoff ’s rule) 
X Y Cl
C hloroprene
  Red hot
C ≡≡ C  XY →  C  C 
→  C == C  XY 3CH ≡≡ CH   
→ C6H6
Fe tube
    Benzene

X Y X Y 5. Oxidation of alkynes
(a) Addition of hydrogen (a) By neutral KMnO4
H2 H2
CH ≡≡ CH  → CH 2 == CH 2  → CH3  CH3 CH3 C ≡≡ C CH3 KMnO
  4

Pt / 573 K Pt / 573 K neutral

(b) Addition of halogens ( X = Cl, Br, I) O O


Cl Cl  
  CH3  C  C CH3
2Cl 2 BaCl 2
CH ≡≡ CH   → H  C  C H   → (b) By acidic KMnO4 (hot)
  −
Cl Cl R C ≡≡ C  R ′ (i)KMnO /OH
 4  →
(ii) H + heat
CCl 2 == CHCl
Trilene or westrosol
(anaesthetic) R COOH + R ′ COOH

@iitjeehelps
HYDROCARBONS 637
(c) By ozone Aromatic Hydrocarbons
O O
The parent compound of aromatic hydrocarbons, is
 
(i) O 3
R C ≡≡ C  R ′   → RC C  R′ benzene (C6H6 ). Some homologues of benzene are:
(ii) H 2O

O OH
 
→ R  C OH + R ′ C == O

Naphthalene Anthracene Phenanthrene


Petroleum
The term petroleum originates from the Latin word The chief source of aromatic compounds, is still the coaltar,
(Petra-rock, Oleum-oil) meaning rock oil, mineral oil or which is a residual component of destructive distillation of
earth oil. Petroleum is a thick viscous fluorescent liquid coal. The coaltar is further fractionally distilled, when
with dark green, brown or yellow colours. It was discovered various compounds are obtained at different range of
by Colonel Drake in 1859. temperature, as shown below in table:
Composition Temperature
Name of fraction Major compounds
range
It mostly consists of alkanes (C1 to C40 ) cycloparaffins,
cycloalkanes and aromatic hydrocarbons. Some organic 80 °−170 ° C Light oil Benzene, toluene, xylenes
compounds containing N, S and O are also present. etc.

170 °−230 ° C Middle oil Phenol, cresol, naphthalene,


Cracking pyridine etc.
The process of getting lower hydrocarbons by heating
hydrocarbons of higher boiling point. Cracking can be 230 °−270 ° C Heavy oil Phenols, naphthol etc.

carried out in liquid or vapour phase. 270 °−360 ° C Green oil or anthracene Anthracene, phenanthrene
oil
Knocking
 Pitch (left as residue) Carbon
Knocking is the metallic sound produced due to irregular
burning of the fuels and it depends upon the quality of the
fuel which can be measured in terms of octane number. Aromaticity
Ring systems whether having benzene ring or not with the
Octane Number following characteristics are said to be aromatic.
The octane number of any sample is determined by 1. Planarity
matching its knocking property with a mixture of
iso-octane and heptane in an experimental engine, 2. Complete delocalisation of the π- electrons in the ring
e.g. octane number of triptane is 124. 3. Presence of ( 4n + 2 ) π-electrons in the ring, where n is
Octane number is the percentage (by volume) of iso-octane an integer (n = 0, 1, 2 ). This is known as Huckel's rule.
present in a mixture of iso-octane and heptane which has
the same knocking performance as the sample fuel.
Naphthalene
Cetane Number
The efficiency of diesel oil is expressed in terms of cetane n=1 n=2
number. Cetane (hexadecane) is assigned cetane number (4n+2)=6π-electrons (4n+2)=10 electrons
100 and α-methyl naphthalene is assigned zero. A diesel oil
having cetane number 75 would have the same ignition Certain compounds such as pyrrole, furan etc., although do
property as a mixture; of 75% cetane and 25% α-methyl not contain benzene ring, but behaves like benzene. These
naphthalene. compounds are called non-benzenoid aromatic compounds.

@iitjeehelps
638 SELF STUDY GUIDE BITSAT

Benzene Mechanism
l Discoverer — M. Faraday (i) Cl Cl + AlCl3 → Cl + + AlCl −4
Electrophile
l Structure established by — Kekule
l Hybridisation of each carbon — sp 2
(ii) + Cl+
Preparation
1. From sodium benzoate H Cl H Cl H Cl H Cl
NaOH, ∆ +
C6H 5.COONa   → C6H6 + Na 2CO3 +
Sodium benzoate CaO Benzene +

2. From phenol +
(a) (b) (c) (d)
OH
H Cl Cl
Zn dust
+ ZnO


(iii) + + AlCl 4 + AlCl3 + HCl
Phenol Benzene

3. Synthesis

b
Electrophile in nitration is nitronium ion (NO+2 ) and in
(a) From acetylene : 3CH ≡≡CH Red
 hot
→
Fe sulphonation, it is SO3H + ion.
(b) From n-hexane Friedel-Crafts alkylation
CH3 CH3
CH2 CH3
Al2O3–Cr2O3
CH3Cl
500°C, 10 atm Anhyd . AlCl3
+ HCl
CH2 CH2
Toluene
CH2
Friedel-Crafts acylation
Physical Properties
Benzene is a colourless, volatile liquid having characteristic COCH3
odour, insoluble in water. Boiling point = 80. 4° C
CH3COCl
Chemical Properties + HCl
Anhyd. AlCl3
1. Electrophilic substitution Benzene, despite being Acetophenone
highly unsaturated compound, gives electrophilic
substitution reactions, e.g. halogenation, sulphonation, 2. Addition of Chlorine
nitration, Friedel-Crafts reactions etc.
H Cl
NO2 Cl H H
Cl2 Cl Cl
Conc. H2SO4 Cl2 or C6H6Cl6
hv
H H
Conc. HNO3 Fe
Cl Cl
Nitrobenzene Chlorobenzene H Cl
Benzene hexachloride (BHC)

∆ Conc. H2SO4 Commercially, it is known as gammaxene and is an


important insecticide.
SO3H 3. Oxidation

V2O5
CH.CO
+9[O] O + 2CO2+2H2O
450°C
Benzenesulphonic CH.CO
acid Maleic anhydride

@iitjeehelps
HYDROCARBONS 639
4. Reduction In case of phenol, electron density is more
H2/Ni/Pt concentrated on the two o and p-positions. Therefore,
200°C substitution is mainly in these positions.
Cyclohexane  NH 2 ,  NHR,  NHCOCH3 , OCH3 , CH3 , C 2H 5
are o and p-directing groups.
Carcinogenicity and Toxicity 2.  NO2 group is meta-directing.
Benzene and polynuclear hydrocarbons containing more
than two benzene rings fused together, are toxic and said to O O O O O O
N N N
possess cancer producing (carcinogenic) property. Such
polynuclear hydrocarbons are formed on incomplete
combustion of organic material like tobacco, coal and So on
petroleum.
Directive Influence of Functional Group in Resonating structures of nitrobenzene
Monosubstituted Benzene
The substituent already present in the benzene ring directs
the incoming group either to ortho and para or to l  NO2 , CN, CHO,  COR,  COOH,  COOR,
meta-position. o , p -directing groups increase the electron SO3H are meta-directing groups.
density on the ring and thus, increase the reaction rate l In case of haloarenes, halogens are highly deactivating
while m-directing groups decrease the electron density on because of − I -effect but due to resonance, electron
the benzene ring and thus, decrease the reaction rate. density on o and p-position increases, hence halogens
e.g. are o, p-directing.
1. OH group is ortho, para-directing.
⊕ ⊕ Cl
+ + Cl Cl ⊕ Cl Cl
OH OH OH

So on

Resonating structures of phenol Resonating structures of chlorobenzene

Practice Exercise
1. Chlorination of n-butane gives the product 4. A mixture of ethyl bromide and methyl bromide is
a. only 2-chlorobutane subjected to Wurtz reaction. The mixture of alkanes
b. only 1-chlorobutane so formed, consists of
c. mixture of s -butyl chloride (excess) + n-butyl chloride a. propane and butane
d. n-butyl chloride and isobutyl chloride b. ethane and propane
2. Consider the following reaction, c. ethane, propane and butane
AlCl3 / HCl d. ethane and butane
CH 3  CH 2  CH 2  CH 3    → product
5. Pure methane can be prepared by
The reaction is named as a. Wurtz reaction
a. insertion b. cracking b. Kolbe’s electrolytic method
c. inversion d. isomerisation c. soda lime decarboxylation
d. reduction with H2
3. The concentrated aqueous solution of potassium salts
of acetic acid and propanoic acid are electrolysed. 6. The reaction,
Which of the following hydrocarbons is/are produced? C6 H5 Br + 2Na + BrCH 3 → C6H5 ⋅ CH 3 + 2NaBr is
a. Only CH3CH2CH3 known as
b. CH3CH3 and CH3CH2CH2CH3 a. Wurtz reaction
c. CH3CH2CH2CH3, CH3CH2CH3 and CH3CH3 b. Wurtz-Fittig reaction
d. Only CH3CH3 c. Friedel-Crafts reaction
d. Berthelot synthesis

@iitjeehelps
640 SELF STUDY GUIDE BITSAT

7. LPG contains a. Zn(Hg)/HCl (conc.) and KMnO4


a. methane b. ethane b. KMnO4 and Zn(Hg)/conc. HCl
c. butane d. None of these c. H2 / Ni and KMnO4
d. CH3OH and C2H 5OH
8. The reagent used for the conversion
CH3 CH2COOH → CH 3 CH 2CH 3 , is 15. When 2-methyl butane-1-ol is dehydrated to give an
alkene, the preferred product is
a. LiAlH4 a. 2-methyl but-1-ene b. but-1-ene
b. soda lime c. 2-methyl but-2-ene d. but-2-ene
c. red P and concentrated HI
d. amalgamated zinc and concentrated HCl 16. A cylinder of compressed gas that bears no label is
supposed to contain either ethane or ethene.
9. The compound with the highest boiling point is Combustion of the sample shows that 16 cm 3 of the
a. n-hexane b. n-pentane gas require 48 cm 3 of oxygen for complete
c. 2,2-dimethylpropane d. 2-methylbutane
combustion. This shows that the gas is
10. The major product of reaction between n-butane and a. only ethane
bromine at 130°C is b. only ethene
a. CH3CH2CH2CH2Br b. CH3CH2CHBr c. 1 : 1 mixture of two gases
 d. some unknown mixtures of the two gases
CH3 17. Cold and dil. KMnO4 reacts with but-2-ene to form
c. CH3  CH2CHBr d. CH3CH2CHBr a. ethane-1, 2-diol b. butane-1, 4-diol
  c. butane-1, 3-diol d. butane-2, 3-diol
CH2Br CH3
18. The conversion of ClCH == CH  Cl to
11. Isomeric pentanes have different value of boiling point Cl2CH  CHCl2 can be carried out with
for example, n-pentane has highest the boiling point a. Cl2 b. Cl2 / hν
among it three isomers. This is due to c. Cl2 / AlCl3 d. Cl2 / aq. NaOH
a. no branching 19. Iso -butyl magnesium bromide with dry ether and
b. weak intermolecular force of attraction ethyl alcohol gives
c. large area of contact
d. None of the above a. CH3 CHCH2 OH and CH3CH2MgBr

12. Arrange the correct sequence for mechanism of CH3
chlorination of methane.

b. CH3 CHCH3 and MgBr(OC2H5 )
hν 
I. CH4 + Cl• → CH3 + H — Cl
CH3


II. Cl — Cl → 2Cl c. CH3 CHCH == CH2 and Mg(OH)Br
• 
III. CH 3 + Cl2 → CH3 — Cl + Cl• CH3
• •
IV. Cl + Cl → Cl — Cl d. CH3 CHCH3 and CH3CH2OMgBr
• • 
CH 3 + CH3 → CH3 — CH3 CH3

Choose the correct option is 20. Which of the following will not show geometrical
a. II, I, III and IV b. I, II, III and IV isomerism?
c. IV, III, II and I d. II, III, I and IV F F
F H
13. The reactants involved in general combustion formula a. C C b. C C
for any alkane is represented as Cl D Cl Cl
 3n +2  3n −1
a. CnH2n + 2 +   O b. CnH2n +   O
 2  2  2  2 H 3C CH3 CH3
C 2H5
 3n +2  3n +1 c. C C d. C C
c. CnH2n +   O d. CnH2n + 2 +  O
 2  2  2  2 H 2C 2 CH3 H3 C C2 H5
14. Consider the following reactions, 21. Anti-Markownikoff’s addition of HBr is not observed in
A a. propene
(i) CH3  COCH3 → CH3  CH2  CH3 b. butene
B c. but-2-ene
(ii) (CH3 )3 CH → (CH3 )3 COH d. pent-2-ene
Here, A and B respectively are

@iitjeehelps
HYDROCARBONS 641
22. A compound X when passed through dil. H2SO4 a. oxidation reaction
containing HgSO4 gives a compound Y which on b. reduction reaction
reaction with HI and red phosphorus gives C2H 6. The c. α-elimination reaction
compound X is d. β-elimination reaction
a. ethene b. ethyne c. 2-butene d. 2-butyne 28. According to Markownikoff’s rule, the major product
23. The addition of halogen to an alkene involves the formed by addition of HBr with propene is
formation of a. 1-bromopropane
a. carbocation as the intermediate b. 1-bromo,1-methyl ethane
b. carbanion as the intermediate c. 2-bromopropane
c. free radical as the intermediate d. 2,2-dibromopropane
d. halonium ion as the intermediate 29. What will be the product of the following reaction?
24. The addition of HBr to 1-butene gives a mixture of (C 6 H 5 CO) 2 O2
products A, B and C CH3 — CH == CH2 + HBr → Product
Br C 2H 5 a. CH3CH2CH2Br b. CH3CH(Br)CH3
c. CH3CBr2CH3 d. CH3 — C == CH2
C C 
H5C2 CH3 H CH3 H
H Br O
(A ) (B )
Zn + H2O
30. A + O3 CH3—HC CH2
CH3 — CH2 — CH2 — CH2 — Br
(C ) O—O
B + HCHO
The mixture consists of
a. A and B as major and C as minor products Here, A and B respectively are
b. B as major, A and C as minor products a. propene and methanal b. propene and ethanal
c. B as minor, A and C as major products c. propene and ethanol d. propene and ethanal
d. A and B as minor and C as major products
NaNH2 Red hot iron
25. The principle organic product formed in the reaction. 31. CH2 CHBr A
Tube, 873 K
Peroxide
CH 2 == CH (CH 2 ) 8 COOH + HBr → … is
a. CH3  CHBr(CH2 ) 8 COOH Here, A is
a. CH ≡≡ CH b. CH3  C ≡≡ CH
b. CH2 == CH(CH2 ) 8 COBr
c. CH3  CH3 d. CH ≡≡ C  NH2
c. CH2BrCH2(CH2 ) 8 COOH
d. CH2 == CH(CH2 ) 7 CHBrCOOH 32. Which one of the following compounds will react with
two mole of CH3 MgBr?
26. Identify of Z in the sequence a. CH3COOH b. CH3  C == C  CH3
HBr/ H 2 O2 c. HC ≡≡ C  CH2OH d. CH3CH2CH2OH
CH3  CH2  CH == CH2 → Y
33. An organic compound on treatment with Br2 / CCl4 ,

C 2 H 5 O Na + gives a bromoderivative alkene. The compound will
→ Z
be
a. CH3  CH  CH2  O  CH2CH3 a. CH3CH == CH2 b. CH3CH == CHCH3
 c. HC ≡≡ CH d. CH2 == CH2
CH3
34. Which of the following shows less reactivity towards
b. CH3 CH2  CH O  CH2CH3 Br2?

a. CH3CH2CH == CH2 b. CH3CH == CH2
CH3
c. CH3  C ≡≡ C  CH3 d. CH ≡≡ CH
c. CH3 (CH2 )3  O  CH2  CH3
35. Which one of the following does not dissolve in the
d. CH3  (CH2 )4  O  CH3
conc. H2SO4 ?
27. The following reaction is an example of a. CH2 == CH2 b. CH  C ≡≡ C  CH3
H H c. CH3CH2C ≡≡ CH d. CH ≡≡ CH
β α Conc. H2SO4 36. n-propyl bromide on treatment with ethanolic
H—C—C—H ∆
CH2 CH2 + H2O potassium hydroxide produces
Ethene
a. propanol-1 b. propene
H OH
Ethanol
c. propanol-2 d. ethyl propyl ether

@iitjeehelps
642 SELF STUDY GUIDE BITSAT

37. When an alkyne R C ≡≡ CH is treated with cuprous ion 46. Which of the following is non-aromatic?
in an ammoniacal medium one of the products is a. Benzene b. Tropylium cation
a. R C ≡≡ CCu b. CuC ≡≡ CH c. Cyclopentadienyl anion d. Cyclooctatetraene
c. CuC ≡≡ CCu d. R C ≡≡ CR 47. Toluene, on oxidation with KMnO4 gives
38. Ozonolysis of 2,3-dimethyl-1-butene followed by a. benzaldehyde b. phenol
reduction with zinc and water gives c. nitrotoluene d. benzoic acid
a. methanoic acid and 3-methyl-2-butanone 48. Ozonolysis of benzene gives
b. methanal and 2-methyl-2-butanone a. 8 moles of glyoxal b. glycol
c. methanal and 3-methyl-2-butanone c. 6 moles of glyoxal d. 3 moles of glyoxal
d. methanoic acid and 2-methyl-2-butanone 49. Benzene was discovered by
39. Which of the following is the predominant product in a. Faraday b. Berthelot c. Kekule d. Huckel
the reaction of HOBr with propene? 50. In nitration of benzene, the active species is
a. 2-bromo-1-propanol a. NO−2 b. ONO− c. NO+2 d. NO−3
b. 3-bromo-1-propanol
c. 2-bromo-2-propanol
51. Give the major product of the following reaction.
d. 1-bromo-2-propanol CF3

40. When propyne is treated with aqueous H2SO4 in


HNO3, H2SO4
presence of HgSO4 , the major product is Major
a. propanal
b. n-propyl hydrogen sulphate
c. acetone CF3 CF3
d. propanol
41. What is the best way to carry out the following a. b.
transformation?
1-pentyne → pentanal NO2
a. HgSO4 / H2SO4 NO2
b. H2 /Lindlar’s catalyst, O3, Zn-H2O
CF3
c. HIO4 / H2O
d. BH3, H2O2 /NaOH O2N
c.
42. On vigorous oxidation by permanganate solution d. Cannot say
(CH3 ) 2 C == CHCH2CHO gives
a. (CH3 )2 CO and OHCCH2CHO 52. PhCH3 on reaction with Cl2 + hν followed by Na /ether
b. (CH3 )2 C  CH  CH2CHO will give
 
OH OH
a.
c. (CH3 )2 CO and OHCCH2COOH
d. (CH3 )2 CO and CH2(COOH)2 Me Me
43. Acidic hydrogen is present in
a. ethyne b. ethene b.
c. benzene d. ethane
44. Which of the following order is correct regarding acidic
character of hydrocarbons given below? c. Me Me
I. CH ≡≡ CH > CH2 == CH2 > CH3  CH3
II. HC ≡≡ CH > CH3 C ≡≡ CH >> CH3 C ≡≡ CCH3 d. CH2 CH2
a. Only I b. Only II
c. Both I and II d. None of these
53. The reaction of 1,3-butadiene and acetylene gives
45. The hydrocarbon, which can react with sodium in
liquid ammonia is a. b.
a. CH3CH2CH2C ≡≡ CCH2CH2CH3
b. CH3CH2C ≡≡ CH
c. d. None of these
c. CH3CH≡≡ CHCH3
d. CH3CH2C ≡≡ CCH2CH3

@iitjeehelps
HYDROCARBONS 643
54. When nitrobenzene is treated with Br 2 in the 58. Consider the following statements,
presence of FeBr3 , the major product formed is I. The — OH group present in the phenol is ortho
m-bromonitrobenzene. Statements which are related and para-directing.
to obtain the m-isomer are II. Directive influence of a functional group in
a. the electron density on meta-carbon is less that on monosubstituted benzene depends on the nature
ortho and para-positions of the substituent already present in the benzene
b. the intermediate carbonium ion formed after initial
ring.
attack of Br + at the meta-position is least destabilised
c. lose of aromaticity when Br + attacks at the ortho and III. The — OH group activates the benzene ring for
para-positions and not at meta-position the attack by an electrophile.
d. easier loss of H+ to regain aromaticity from the IV. Groups such as — NH2, — NHR, — NHCOCH3 ,
meta-position than from ortho and para-positions — OCH3 , — CH3 , — C2H5 ., etc, are the examples
55. A hydrocarbon reacts with HI to give X which on of activating group.
reaction with aqueous KOH forms Y . Oxidation of Y Select the correct option.
gives 3-methyl, 2-butanone. a. I and II
The hydrocarbon is b. II and III
CH3 c. II, III and IV
 d. I, II, III and IV
a. CH3CH == C  CH3
NO2
b. CH2 == CH  CH  CH3 Zn/NaOH Cold
 59. C2H5OH
X
Conc. HCl
Y
CH3
c. CH3  CH2  CH  CH3 X and Y are
d. CH ≡≡ C  CH  CH3 a. C6H 5 NHOH, C5 H4 (OH)NH2
 CH3 Cl Cl
CH3  
b. C6 H5 — N2H4 — C6 H5, C6 H4 — N C6 H4 — NH
56. A Friedel-Crafts reaction of benzene with chloroform
produces c. Both (a) and (b)
d. None of the above
Cl
 60. In the reaction,
a. C6H5CHCl2 b. C6H5  C  C6H5

H Cl2 ∆
X + H2SO4 Y,
C6H5
Fuming sulphuric
 acid
c. C6H5  C  C6H5 d. All of these
 X and Y are respectively,
H
a. C6 H5Cl and C6 H5 SO3 H
X b. C6 H5Cl and C6H5 OH
c. C6 H4 Cl 2 and C6 H5 SO2
Anhyd. AlCl3 d. C6 H5 Cl and C6 H5 CHO
57. + X2 + A,
61. Which of the following organic materials damage DNA
of our body?
Here, A is a. Tobacco
a. H2 b. X − b. Coal
c. HX d. Both b and c c. Petroleum
d. All of the above

@iitjeehelps
BITSAT Archives
1. What will be the product of the reaction? [2014] 5. Lindane can be obtained by the reaction of benzene
with [2012]
CH 3
| a. CH3Cl /anhyd. AlCl3 b. C2H5I /anhyd. AlCl3
H3 C  C  Br + Na  O  CH3 → c. CH3COCl /anhyd. AlCl3 d. Cl2 in sunlight
| 6. What will be the main product when acetylene reacts
CH3 with hypochlorous acid? [2012]
CH3 a. Trichloro acetaldehyde
| b. Acetaldehyde
a. CH3  C == CH2 b. CH3  O  C  CH3 c. Dichloro acetaldehyde
| | d. Chloro acetaldehyde
CH3 CH3
7. In which of the following compounds, the bond length
c. CH3  CH2  CH2  CH3 d. CH3  CH CH3 between hybridised carbon atom and other carbon
| atom is minimum? [2012]
CH3 a. Butane b. Propyne
c. Propene d. Butene
2. The types of the reactions for these are [2014]
8. The treatment of benzene with iso -butene in the
Cl presence of sulphuric acid gives [2011]
I. NaO but a. iso-butylbenzene
b. tert-butylbenzene
c. n-butylbenzene
H
d. No reaction
Cl OH 9. Which of the following carbon atoms is most
II. aq. KOH electronegative? [2011]
III II I
I CH 3  CH 2  C≡≡ CH
I2
III. a. I
b. II
I c. III
d. All are equally electronegative
IV. ∆
+
10. The reaction/method that does not give an alkane is
a. catalytic hydrogenation of alkenes [2011]
a. elimination, substitution, addition, addition b. hydrolysis of alkyl magnesium bromide
b. addition, elimination, addition, substitution c. Kolbe’s electrolytic method
c. elimination, addition, substitution, addition d. dehydrohalogenation of an alkyl halide
d. substitution, elimination, addition, addition
11. The most strained cycloalkane is [2010]
3. Which of the following reaction produces most stable
a. cyclopropane b. cyclobutane
alkene? [2013]
c. cyclopentane d. cyclohexane
a. 2-chloro butane
b. 2, 3-dichloro butane 12. Chain isomers of C6H14 will be [2010]
c. 2, 2-dichloro butane a. 4 b. 5 c. 6 d. 7
d. 2, 3-dichloro, 2, 3-dimethyl butane
13. Which of the following alkenes will react fastest with
4. IUPAC name and degree of unsaturation of
H2 under catalytic hydrogenation conditions? [2009]
compound X is [2013]
R R R H
a. b.
H H R H
X
a. 2, 3-dimethyl bicyclo [2,2,1] hept-5 ene, 2 R R R R
b. 1, 2-dimethyl bicyclo [2,2,1] hept-4 ene, 3 c. d.
c. 5, 6-dimethyl bicyclo [2,2,1] hept-2 ene, 3
R H R R
d. 4, 5-dimethyl bicyclo [2,2,1] hept-1 ene, 2

@iitjeehelps
HYDROCARBONS 645
14. Which of the following reagents would you prefer to
find out whether the hydrocarbon C3 H4 contains Cl Cl + Mg A
one-triple bond or two-double bonds? [2009]
a. Fehling’s solution a.
b. Ammoniacal AgNO3 or CuCl solution
c. Baeyer’s reagent
d. Br2/CCl4 b. Cl Mg Mg Cl
15. Which of the following compound(s) has Z
configuration? [2008] c. Both a and b
Cl Br
I. C C
d. None of the above
H F
Cl F 18. How many asymmetric carbon atoms are present in
II. C C (i) 1, 2-dimethylcyclohexane [2005]
H Br (ii) 3-methylcyclopentene and
(iii) 3-methylcyclohexene
Br CH3 a. two, one, one
III. C C b. one, one, one
Cl H c. two, none, two
d. two, none, one
a. Only I b. Only II c. Only III d. I and III
2Cl2 , Heat Fe/ Br2 Zn/ HCl 19. Bicyclo (1, 1, 0) butane is [2005]
16. C7H 8    
→ A   
→ B   
→ C
a. b.
Here, the compound C is [2008]
a. 3-bromo 2, 4, = 6 -trichlorotoluene
b. o -bromotoluene c. d.
c. p -bromotoluene
d. m -bromotoluene
17. What is the product A in the following? [2006]

Answer with Solutions


Practice Exercise • •
CH3 + CH3 → CH3CH3
• •
1. (c) Sec-butyl chloride will be formed in more quantity then CH3CH2 + CH3 CH2 → CH3CH2CH2CH3
n-butyl chloride as 2° hydrogen is more reactive.
• •
2. (d ) CH3CH2 + CH3 → CH3CH2CH3
Dry ether
AlCl3 /HCl 4. (c) CH3CH2Br + 2Na + BrCH3   →
25°C
CH3CH2CH3 + 2NaBr
n-butane iso-butane
Propane

This is known as isomerisation. Dry ether



CH3CH2Br + 2Na + BrCH2CH3  →
− −e −
3. (c) CH3  C  O  → CH3  C  O CH3CH2CH2CH3 + 2NaBr
  Butane
O O
Dry ether
− CO
2
• CH3Br + 2Na + BrCH3  → CH3CH3 + 2NaBr
 
 → CH3 Ethane

− 5. (c) Methane cannot be prepared by either Wurtz reaction,
CH3CH2  C  O− −
e
→ CH3CH2  C  O
  Kolbe’s electrolytic method or by reduction of alkenes
O O with H2. While acetic acid salt on heating with soda lime,

gives methane.
− CO
 
2
→ CH3CH2 Soda lime
CH3COONa   → CH4 + Na 2CO3

@iitjeehelps
646 SELF STUDY GUIDE BITSAT

6. (b) This reaction is called Wurtz-Fittig reaction and is a 17. (d ) Hydroxylation occurs
common method of preparing toluene.
CH3  CH == CH  CH3 + KMnO4 + H2O
7. (c) LPG (Liquified petroleum gas) contains almost 96%
298-303 K
n-butane.
8. (c) Only red P and conc. HI convert — COOH group to
CH3  C H  C H  CH3 + MnO2 + KOH
CH3.
 
9. (a) Boiling point ∝ number of carbons. OH OH
Br 2 Cl
2
10. (b) C  C  C  C → C  C  C  C  Br 18. (a) ClCH == CHCl → Cl2CH CHCl2

CH3  CH  CH2MgBr CH3
+ C  C  C  Br |

CH3
 19. (b) C2H5OH → CH3  CH  CH3 +
C Dry ether
Major Br
ƒ
As 2° free radical is more stable than 1° free radical. Mg
‚
11. (c) With increase in number of branched chains, the OC2H5
molecule attains the shape of a sphere. This results in
smaller area of contact and therefore weak intermolecular 20. (d ) When more than two similar groups are linked to
forces between spherical molecules, which are overcome double bonded carbon atoms, then alkenes does not
at relatively lower temperatures. show any geometrical isomerism.
12. (a) 21. (c) Anti-Markownikoff’s addition cannot be seen in
symmetrical alkenes, i.e. but-2-ene
 3n + 1
13. (d ) CnH2n+ 2 +  O → n CO2 + (n + 1) H2O
 2  2 2 Dil. H SO
4
22. (b) X → Y → CH3  CH3
Red P
HgSO4 HI
Zn(Hg)/HCl
14. (a) CH3 COCH 3 → CH 3 CH2 CH 3 + H 2O Clearly,Y is CH3  CHO and X is CH ≡≡CH .
2(H)

KMnO
23. (d ) Halogenation process proceeds via following
4
(CH3 )3 CH → (CH3 )3 COH mechanism forming halonium ion as intermediate.
3 ° alcohol X
15. (a) Where the structure of the alkyl group permits, X2
R CH==CH2 R CH—CH2; process is anti-addition
rearrangement takes place: CCl4
X—X −
process.
CH3 X X
 ••
RCH—CH2

SN2
CH3  CH2  CH  CH2 O H + H⊕ → X
Halonium ion
CH3
 24. (a) HBr selectively adds to the alkenes through
CH3  CH2 == C  CH3 Markownikoff's addition.
2-methyl but -2-ene
 more stable than
CH2 HBr CH2 CH3
 2 ° carbocation 
CH3 CH CH2 CH3 CH
 y y
16. (b) Cx Hy + x +  O2 → x CO2 + H2O Br
 4 2 Chiral carbon
 y Br C2H5
1 vol. x +  vol
 4
Produces C + C
 y Racemic mixture H 5C 2 CH3 H CH3
16 cm3 16 x +  cm3
 4 H Br
(A) Major (B )
 y
Now, we have 16 x +  = 48 Alongwith A and B very little amounts of CH3CH2CH2 CH2
 4
is also produced in the reaction. 
Putting x =2 Br
 y
16 2 +  = 48 HBr
 4 25. (c) CH2 == CH (CH2 )8 C  OH →
Peroxide
product

or y =4 O
Hence, the hydrocarbon is C2H4. via anti-Markownikoff’s addition.

@iitjeehelps
HYDROCARBONS 647
HBr
Cu+ /NH4 OH
26. (c) CH3  CH2  CH == CH2 → 37. (a) R C ≡≡ CH → R C ≡≡ CCu
H2O2
(Anti-Markownikoff’s addition) (i) O3 (ii) Zn/ H2O
! 38. (c) CH3  CH  C == CH2 →
C2H5O Na ⊕
CH3CH2CH2CH2Br →  
Nucleophilic CH3 CH3
substitution
2, 3-dimethyl-1-butene
CH3CH2CH2  CH2  O  C2H5
27. (d) This reaction is the example of β-elimination reaction CH3  CH  C == O + H C == O
  
since — OH group takes out one hydrogen atom from the
CH2 CH2 H
β-carbon atom. 3 − methyl-2 − butanone Methanal
Markownikoff' s
28. (c) CH3 — CH == CH2 + HBr → δ− δ+
addition HO  Br
Br 39. (d ) CH3  CH == CH2 → CH3 C H  CH2
Markownikoff’s

Propene
rule
 
CH3 — CH— CH3 OH Br
1-bromo-2-propanol
2-bromopropane
H2SO4
29. (a) Anti-Markownikoff’s addition or peroxide effect or 40. (c) CH3C ≡≡ CH + H2O →
HgSO 4
Kharash effect
Tautomerisation
6 5 2 2
CH3 — CH==CH2 + HBr → CH3 — CH2
(C H CO) O [CH3C(OH) == CH2] → CH3COCH3
Acetone

CH2Br 41. (d )
1-bromopropane
HO–
OH
O + BH3—H2O2
Anti-Markownikoff's
30. (d ) CH3 — CH CH2 + O3 CH3CH CH2 addition of water
Propene (A)
H
O—–O r
Zn/H2O
O
CH3CHO + HCHO
Ethanal (B) 42. (d) C == C bond is cleaved and oxidised to
Ozonolysis takes place in alkenes. So, A will be a double  COOH,  CHO group is also oxidised to COOH .
bond containing compound. (CH3 )2 C == CHCH2CHO → (CH3 )2 C == O
2
31. (a) CH2 ==CHBr →
NaNH
H — C ≡≡ CH + NaBr + NH3 + HOOC  CH2  COOH
( A) 43. (a) Terminal alkynes are acidic in nature.
 Red hot, Fe tube, 873 K
44. (c) Both are correct. In (i) 1s -character defines correct
↓ trend of acidic strength as
Benzene
acidic strength ∝ % s-character
32. (c) HC ≡≡ C  CH2OH + 2CH3MgBr →
2 moles
In (ii) inductive effect defines acidic strength and polarity of
2CH4 + BrMgC ≡≡ CCH2OMgBr C —H bond of alkyne. Group having + I - effect increases
CCl4 electron density around C  H bond hence decreases
33. (c) CH ≡≡ CH + Br2   → BrCH == CHBr polarity and acidic strength of alkyne.
Dibromoethylene
Na/Liq. NH3
34. (d) The three membered ring of bromonium ion and 45. (b) CH3CH2C ≡≡ CH → CH3CH2C ≡≡ CNa +

alkyne has a full double bond, causing it to be more But -1-yne
strained and less stable than the one from any alkene. Only terminal alkyne is responsible for reaction with Na in
HC CH H2C CH2 liquid ammonia.
+
46. (d ) According to Huckel’s rule planar, closed cyclic
+
: Br : : Br : structure having ( 4n + 2)π electrons are aromatic.
(Less stable) (More stable)
(i) (ii)
+
35. (d )

Benzene Tropylium Tropylium Cyclooctatetraene
36. (b) CH3CH2CH2Br → CH3CH == CH2 (6πe–) cation cation (8πe–)
Alc. KOH
n -propyl bromide Propene (6πe–) (6πe–) non-aromatic

@iitjeehelps
648 SELF STUDY GUIDE BITSAT

47. (d ) Toluene, in presence of different oxidising agent, HI Alc. KOH [ O]


55. (b) Hydrocarbon → X → Y → C  C  C  C
gives different products on oxidation.  
CHO CH3 CH3 O
As per the informationY should be C  C  C  C
CrO2Cl2 Dil. HNO3
 
Etard's reaction KMnO4
C OH
Benzaldehyde
⇒ X should be C  C  C  C
COOH
 
C
⇒ Hydrocarbon should be C  C  C == C
Benzoic acid

C
48. (d ) Ozonolysis of benzene gives 3 moles of glyoxal. C6H5
O O AlCl3 
56. (c) C6H6 + CHCl3 → H5C6  C  C6H5
O O 
3O3
O H
O O X
O O
Benzene triozonide Anhyd. AlCl3
57. (d ) + X2 + HX
CHO
3H2O
  → 3 + 3H2O2
Zn CHO 58. (d )
Glyoxal
NO2
49. (a) Benzene was discovered by M. Faraday, who Zn/NaOH
obtained this gas by the pyrolysis of whale oil. 59. (d )
50. (c) In nitration of aromatic nucleus, the attacking
species is an electrophile, i.e. nitronium ion (NO+2 ). It is H 2N — — NH2
obtained by the reaction of conc. HNO3 and H2SO4.
H2SO4 + HNO3 → H2O + HSO4− + NO2+ Cold, conc. HCl

51. (b) Cl Cl
52. (d ) Side chain substitution followed by Wurtz reaction.
H2 N — — NH2
53. (b) Diel’s Alder reaction in the presence of dienophile,
diene will produce the product via 6 MCTS
mechanism. Cl
SO3H H2SO4
C (Fuming) Cl2
+ 60. (a) ∆
C
Y X
54. (b)  NO2 is a m -directing and deactivating group,
hence σ-complex is formed by attack of Br + at 61. (d ) Tabacco, coal and petroleum damages DNA of our body
meta-position will be most stable. and causes cancer.

@iitjeehelps
BITSAT Archives
1. (a) When tertiary alkyl halide is treated with sodium IUPAC name = 5, 6 -dimethyl bicyclo [2, 2, 1] hept-2-ene
alkoxide than elimination reaction competes over Molecular formula of compound is C9H14.
substitution reaction because alkoxides are not only
Degree of unsaturation can be calculated as
nucleophiles but strong base as well. Therefore, alkenes
are formed instead of ethers. H N
u = ( C + 1) − +
2 2
2. (a) I. Elimination reaction
Cl where, u = degree of unsaturation
C = number of carbons
+ HCl
s H = number of hydrogen
NaO but
H N = number of nitrogen
Hence, for a compound having molecular formula C9H14
II. Substitution reaction the degree of unsaturation may be calculated as
Cl OH 14
u = (9 + 1) − = 10 − 7 = 3
aq. KOH 2
5. (d )
Here, Cl replaces by OH.
HO Cl
III. Addition reaction | |
− +
I 6. (c) CH≡≡ CH + HO  Cl → CH == CH
I2 OH Cl
− | + | Cl
HO  Cl − H2O ‚
→ CH  CH → CH CHO
I | | ƒ
Cl
OH Cl
IV. Addition reaction 2, 2-dichloroacetaldehyde
Unstable


+
7. (b) We know C—C bond length = 1.54 Å
that C == C bond length = 1.34 Å
3. (d ) According to Saytzeff’s rule, more substituted C ≡≡ C bond length = 1.20 Å
(alkylated) alkene are more stable. Since, propyne has triple bond, therefore, it has minimum
Cl bond length.
Cl Cl Cl
Cl CH3 8. (b)
CH3 H2SO4
CH3 + (CH3)2C CH2 (CH3)3C
Cl CH3
H Cl Benzene iso-butene
(a) (b) (c) (d )
Base Base Base Base
tert-butylbenzene
Cl CH3 III II I
9. (a) CH3  CH2  C ≡≡ CH
sp 3 sp 3 sp sp
A B C D CH3 Electronegativity ∝ s -character
Conjugation Greater the conjugation greater will be the In sp-hybrid orbitals,s -character = 50%
stability of product. and in sp 3-hybrid orbitals,s -character = 33.3%
Hence, D has maximum stability, which is stabilised by Thus, I is the most electronegative.
conjugation as well as Saytzeff’s rule.
10. (d ) Dehydrohalogenation of alkyl halide gives alkenes but
4. (c) 7 not alkane.
1 Alc. KOH
2 R  CH2  CH2X   → R CH == CH2
− HX
6
4 11. (a) The greatest strain is involved in cyclopropane with
3 5 bond angle 60°.

@iitjeehelps
650 SELF STUDY GUIDE BITSAT

12. (b) CH3  CH2  CH2  CH2  CH2  CH3 Cl F


(ii) C C
CH3
 H Br
CH3  CH  CH2  CH2  CH3 E
CH3 (Priority : Cl > H and Br > F)
 Br CH3
CH3  CH2  CH  CH2  CH3 (iii) C C
CH3 Cl H

CH3  C  CH2  CH3 Z
 (Priority : Br > Cl and CH3 > H)
CH3
Hence, compounds (i) and (iii) have ( Z ) configuration.
CH3 CH3
16. (d ) CH3 CCI3
 
CH3  CH  CH  CH3
3Cl2, 778 K
13. (a) During catalytic hydrogenation, the hydrogens
–3HCl
are transferred from the catalyst to the less
substituted double bond. Thus, option (a) with two (A)
R groups on the same side of the molecule is
Br2/ Fe
correct.
14. (b) If the hydrocarbon C3H4 contains a triple bond CH3 CCI3
it must be CH3C ≡≡ CH . Since, being a terminal
alkyne, it will react with ammoniacal AgNO3. +6H, Zn/HCl
AgNO3 solution, to given a white ppt of silver –3HCl
propynide or with ammoniacal CuCl solution to Br Br
give a red ppt of copper propynide. (C)
m-bromotoluene
15. (d ) When the groups with higher priority (i.e. with
high atomic number) are present on same side of 17. (a)
double bond, then the configuration is Z but when Cl Cl + Mg
present on opposite side of double bond, the
configuration is E.
Cl Br
(i) C C 18. (a) two, one, one
H F 19. (c)
Z
(Priority : Cl > H and Br > F) Bicyclo (1, 1, 0) butane

@iitjeehelps
24
Halogen Derivatives
of Hydrocarbon

Haloalkane and Haloarene


The replacement of hydrogen atom(s) from a hydrocarbon (aliphatic or aromatic) by halogen atom(s)
results in the formation of haloalkane or haloarene. These may be classified as mono, di or tri halo
compounds depending upon the number of halogen atoms.
−H
If R = Alkyl ; Haloarene
R H → R  X  R = Aryl ; Haloarene 
+X
 X = Halogen 

Haloalkane
(i) General formula : C nH 2 n + 1 X (monohalides)
(ii) sp3 -hybrid C, geometry : tetrahedral
(iii) Bond angle : 109° 28′

Classification of Haloalkane
1. Based on the number of halogen atoms
(a) Monohalides[CH3Cl,(CH3 )3 CCl etc]
Cl CH2 CH2
(b) Dihalides CH2 == C
Cl Cl Cl
Gem dihalide Vicinal dihalide

(c) Trihalides CHCl3 (chloroform)


(d) Tetrahalides CCl 4 (carbon tetrachloride)

@iitjeehelps
652 SELF STUDY GUIDE BITSAT

2. Based on the type of C-atom to which halogen 4. From alcohol


atom is attached (a) Using dry HX
(a) CH3 Cl → primary R OH + HX Anhydrous
 → R  X + H 2O
ZnCl 2 , ∆
CH3
(b) CH Cl → secondary (b) Action of phosphorus halides and SOCl 2 on
CH3 alcohols
CH3 3R  OH + PX 3 → 3R  X + H3PO3
|
(c) CH3  C Cl → tertiary R  OH + PX 5 → R  X + POX 3 + HX
| R  OH + SOCl 2 Pyridine
 → R Cl + SO2 + HCl
CH3
5. Using Grignard reagent
Preparation of Haloalkane X′
RMgX ′ + X 2 → R  X + Mg
1. From alkane (By halogenation) X

R  H + X 2  → R  X + HX 6. From Ag-salt of carboxylic acid
Br
SO Cl RCOOAg  
2
→ R Br + AgBr + CO2
R  H  2
2
→ R  Cl + SO2 + HCl CCl 4
hv
Number of C-atoms decreases by one.
2. From alkene (hydrohalogenation)
7. From alkyl amines
R  CH == CH  R ′ + HX → RCH 2 CHR ′
 R  NH 2 NOCl
→ R  Cl + N 2 + H 2O
X 8. From ether
Unsymmetrical alkene gives major product by more ROR + PCl 5 → 2 RCl + POCl3
stable carbocation according to Markownikoff's rule.
Physical Properties of Haloalkanes
CH3  CH = CH 2 HBr
→ CH3  CH  CH3
 General physical properties of haloalkanes are as follows:
Br (i) CH3F, CH3Cl, CH3Br, C 2H 5F and C 2H 5Cl are found in
3. From alkyl halide (Halogen exchange reactions) gaseous state at room temperature.
(a) Finkelstein reaction (In acetone solvent) (ii) Alkyl halides are normally sweet-smelling gases
R I and liquids, whereas odourless in solid state.
Nal Nal (iii) All alkyl halides are insoluble in water.
R F Nal R Cl (iv) Boiling point of alkyl halides increases with
NaBr NaBr increase in the size of halogen atom while
R Br decreases with increase in branching.

Chemical Properties of Haloalkanes


F − < Cl − < Br − < I − (Nucleophilic strength)
Haloalkanes are highly reactive compounds due to the
(b) Swart reaction (In polar aprotic solvent) presence of polar R  X bond.
R  I 
KF
→ R F The reactions of haloalkanes have been discussed below:
DMF

R F 
KI
→ No reaction Nucleophilic Substitution Reactions
DMF
In haloalkanes, these reactions proceed either by SN 1 or by
F − < Cl − < Br − < I − (Nucleophilic strength) SN 2 type of mechanism.

@iitjeehelps
HALOGEN DERIVATIVES OF HYDROCARBON 653
1. Substitution nucleophilic unimolecular In SN 2 mechanism, rate of reactions does not depend upon
reaction (S N 1) the strength of attacking nucleophile.
(i) Tertiary alkyl group and polar solvent favour SN 1. The strength of different nucleophiles is
(ii) It follows first order kinetics, rate = k [ substrate]. CN − > I − > OR− > OH −> CH3 COO− > H 2O > F −
(iii) Carbocation is formed and gets rearranged, NOTE 3° halides usually proceed through S N1 type, 1° halides
if possible, the carbocation must be stable. proceed throughSN 2 type and 2° halide may proceed either
(iv) Reaction is completed in two steps. throughSN1 or SN 2 type.
(v) Nucleophile can attack from front or back side.
Therefore, racemic mixture can be formed. Elimination Reactions
e.g.
CH3
[Synthesis of Alkenes]
Step I Reaction of haloalkane with alcoholic KOH solution leads
(CH3)3CBr + + Br–
Slow to an elimination of hydrogen atom from β-carbon atom
H3C CH3 and halogen atom from α-carbon atom. Thus, reaction
3°-carbocation termed as dehydrohalogenation reaction.
(β ) (α )
CH3 Alc. KOH
Step II
e.g. CH3  C H  CH 2  Br →

+ + OH
Fast
(CH3)3C  ΟΗ 
CH3 tert-butyl alcohol H
H3C
CH3  CH == CH 2 + H 2O + KBr
Reactivity order of halides towards SN 1 mechanism is
If dehydrohalogenation reactions produce two or more
3 °> 2 °> 1° alkenes, the preferred or major product will be that
2. Substitution nucleophilic bimolecular alkene which possess larger number of alkyl groups
linked to double bonded C-atoms. This is in accordance to
reaction (S N 2)
Saytzeff’s rule.
(i) Primary alkyl group and non-polar solvent (β ) (α ) (β )
favour SN 2. Alc. KOH
CH3 CH 2  CH CH3 →
− KBr
(ii) It follows second order kinetics, 
Br
rate = k [substrate] [nucleophile]
CH3CH == CHCH3 + CH3CH 2CH == CH 2
(iii) Carbocation is not formed. Major Minor
[highly substituted] [less substituted]
(iv) Reaction is completed in one step.
(v) It involves complete inversion in configuration as The reactivity order of alkyl halides for
the attack of the nucleophile occurs from the back dehydrohalogenation reaction is given below
side of reactant.
tertiary > secondary > primary
e.g. (3 ° ) (2 ° ) (1 ° )
H H
Reduction
H Cl + OH HO Cl
R  X 2H
→ R H + HX
H H
H Reducing agents
Transition state
H ZnHCl, LiAlH 4 , H 2 Ni / Pt / Pd
HO + Cl
s
CH3  Cl  
2 H
→ CH 4 + HCl
H LiAlH 4
H
Formation of organo-metallic compounds
(Solid bond) – towards the viewers
R  X Mg
→ RMgX (Grignard reagent)
(Wedge bond) – away from the viewers dry ether

2 R  X 2Zn
→ R2ZnX + ZnX 2
dry ether
Reactivity order of halides towards SN 2 mechanism is
R  X +2Li Dry
→ RLi + LiX
1° > 2 ° > 3 ° > neo-pentyl halides ether

@iitjeehelps
654 SELF STUDY GUIDE BITSAT

Reaction with NaPb alloy (ii) For the same aryl group, boiling and melting points
4CH3CH 2Cl + 4NaPb → (CH3CH 2 )4 Pb + 4NaCl increase with an increase in the size of the halogen
TEL atom. Boiling and melting points of haloarenes increase
[TEL → tetraethyl lead (antiknocking agent)] with increase in the size of aryl group.
I Br Cl F
Friedel-Crafts alkylation
Anhyd. AlCl 3
C6H6 + CH3 X    → C6H 5CH3 + HX
> > > >

Reaction with salts of carboxylic acid


The p -isomers possess higher melting points than
O
 ortho and meta-isomers because p-isomers fit in the
RCH 2Br + R′COOAg Ethanol
 → R′ C  OCH 2 R + AgBr crystal lattice more efficiently than ortho and

meta-isomers.
Haloarenes Chemical Properties of Haloarenes
Preparation of Haloarenes Haloarenes give three types of reactions:
General methods for the preparation of haloarenes are as 1. Nucleophilic Substitution Reactions
follows: Haloarenes are less reactive than haloalkanes towards
nucleophilic substitution reactions.
Halogenation of Benzene/Toluene e. g.
Cl Cl OH
Cl2 (i) NaOH, 623K, 300 atm
FeCl3 or
+ HCl (i) +
Anhyd. AlCl3 (ii) H

This is known as Dow’s process.


Decomposition of Diazonium Salts
Cl OH
Sandmeyer reaction
Cl/Br (i) NaOH, 443K
(ii)
(ii) H+
CuCl/HCl
+ N2
or CuBr/HBr
NO2 NO2
r s
N NCl I Cl OH
NO2 NO2
KI (aq) (i) NaOH, 368K
+ N2 + KCl (iii) +
(ii) H
Diazonium
salt Cl/Br
NO2 NO2
Gattermann reaction
Cu/HBr/
+ N2 Cl OH
or Cu/HCl/
O2N NO2 O2N NO2
(i) Warm
Raschig Process (iv) (ii) H2O

H Cl
NO2 NO2
CuCl2/25°C 2,4,6–trinitrophenol or
2 + O2 + 2HCl 2 + 2H2O picric acid (common name)

2. Electrophilic Substitution Reactions


Physical Properties of Haloarenes Haloarenes undergo usual electrophilic substitution
(i) Aryl halides (haloarenes) are heavier than water, reactions of benzene nucleus such as halogenation,
though polar in nature but immiscible with water due nitration, sulphonation and Friedel-Crafts alkylation
to the absence of H-bonding with water molecules. and acylation.

@iitjeehelps
HALOGEN DERIVATIVES OF HYDROCARBON 655
e.g. Reactions with Metals
(i) Halogenation 1. Wurtz-Fittig Reaction
Cl
A mixture of haloarene and haloalkane, when treated with
AlCl3 (anhyd.)
Na in dry ether gives alkyl arene, e.g.
+ Cl2
X R
Cl Cl + 2Na + RX Ether
+ 2NaX
Cl
Alkyl benzene
+
2. Fittig Reaction
o-dichlorobenzene
Cl (minor) Haloarenes on treatment with Na in dry ether give
p-dichlorobenzene analogous compounds, e.g.
(major)
X
(ii) Nitration Ether
2 + 2Na + 2NaX
Cl Cl Cl
Diphenyl
NO2
HNO3 (conc.)
H2SO4 (conc.) +
Polyhalogen Compounds
1-chloro-2-nitrobenzene
(minor) NO2 Carbon compounds which contain more than one halogen
1-chloro-4-nitrobenzene atom are called polyhalogen compounds.
(major)
Some important polyhalogen compounds along with uses
(iii) Sulphonation and environmental effects are given as :
Cl Cl Cl
H2SO4 SO3H Trichloromethane (Chloroform), CHCl 3
(conc.) Chloroform oxidises by air slowly in the presence of light to
∆ +
yield phosgene, an extremely poisonous gas.
2-chlorobenzene hν Cl
sulphonic acid SO3H 2CHCl3 + O2 → 2O == C + 2HCl
(minor) 4-chlorobenzene
Phosgene Cl
sulphonic acid (major) Therefore, it is stored under closed dark coloured bottles.
(iv) Friedel-Craft’s alkylation Uses
Cl Cl Cl (i) It is used as a solvent for oils, fats, rubbers and resins.
CH3 (ii) It is exclusively used in the production of freons.
AlCl3 (anhyd.)
+ CH3Cl +
Environmental Effects
1-chloromethyl Major effects of CHCl3 are as follows:
benzene (minor) CH3
4-chloromethyl (i) The presence of CHCl3 in air can cause dizziness,
benzene (major) fatigue and headache.
(v) Friedel-Craft’s acylation (ii) It may leads to the damage of liver and kidneys when
breathed with air.
Cl
Triiodomethane (Iodoform)
AlCl3 (anhyd.) The chemical formula of iodoform is CHI3 .
+ CH3COCl
Cl Cl Uses It is used as an antiseptic for dressing the wounds,
due to the liberation of free iodine by iodoform.
COCH3
+ Freons
The chlorofluorocarbon compounds of methane and
2-chloro ethane containing no H-atom are called freons. These
acetophenone COCH3
(minor) 4-chloro acetophenone compounds are highly stable, non-toxic, unreactive and
(major) easily liquefiable gases.

@iitjeehelps
656 SELF STUDY GUIDE BITSAT

Uses It is widely used in the production of aerosol Uses It is widely used as


propellants, refrigeration and air conditioning products.
(i) Solvent
Environmental effects These freons when released into
(ii) Paint remover
air, diffuse into stratosphere where it initiates radical chain
reactions for the depletion of ozone layer. (iii) Propellant in aerosols
(iv) Metal cleaning and finishing solvent
p p′-Dichlorodiphenyl Environmental effect
Trichloroethane (DDT) (i) It harms human nervous system.
The structure of DDT is (ii) It causes dizziness, nausea, tingling and numbress in
the fingers.
Cl
(iii) Direct human skin contact with CH 2Cl 2 causes intense
burning sensation and mild redness of the skin. Also, it
Cl can burn the cornea if direct contact with eyes occurs.
Cl — Cl Tetrachloro methane (Pyrene)
Cl H CuCl 2
CH 4 + 4Cl 2 → CCl 4 + 4HCl
Uses DDT is cheap but powerful insecticide used for AlCl 3
sugarcane and fodder crops in order to kill insects and CS2 + 3 Cl 2 → CCl 4 + S2Cl 2
or Fecl 3
mosquitoes.
Uses
Environmental Effects (i) It is used in the manufacturing of refrigerants and
Most of the species of insects developed resistance towards tropellants for aerosol units.
DDT and it is found to be highly toxic for animals. (ii) It is used as a cleaning solvent, a degreasing solvent, a
It is deposited and stored in the fatty tissues because it is spot remover, a fire extinguisher.
soluble in fat and chemically stable.
Environmental effects
Dichloromethane
(i) It causes dizziness, nausea, vomitting.
UV Cl 2
CH 4 + Cl 2 → CH3Cl → CH 2Cl 2 (ii) Exposure to CCl 4 may cause irregular heart beat.
light hν

[H] (iii) It depletes the ozone layer which increases the level of
CHCl3 → CH 2Cl 2 + HCl UV-rays causing.
Zn + HCl

@iitjeehelps
Practice Exercise
1. The true order of reactivity towards S N1 reaction is 8. In alkaline hydrolysis of a tertiary halide by aqueous
a. vinyl chloride > allyl chloride > isopropyl chloride
alkali, if concentration of alkali is doubled, then the
b. allyl chloride > isopropyl chloride > vinyl chloride reaction rate
c. vinyl chloride > isopropyl chloride > allyl chloride a. will be doubled
d. isopropyl chloride > vinyl chloride > allyl cloride b. will be halved
c. will remain constant
2. Consider the following reaction, d. Cannot say anything
(CH 3 )3 C  Cl + OH → (CH 3 )3 C  OH + Cl−
9. Which of the following sequence of reagent is best
The true statement about the above process is suited for the reaction shown below?
a. the rate becomes four times of the halide O CH3
becomes twice
b. the rate does not change by reducing [OH− ] to half
c. the rate does not change by doubling [halide]
Br
d. the rate becomes twice on doubling the [OH− ]
+ +
a. (i) CH3MgBr, H3O (ii) H / ∆ (iii) HBr/ H2O2
3. Consider the following reaction, b. (i) CH3MgBr, H3O+ (ii) H+ / ∆ (iii) HBr
CH3 Br + OH − → CH3 OH + Br − c. (i) CH3MgBr, H3O+ (ii) HBr
The true statement about the above process is d. (i) HBr/ROOR (ii) CH3MgBr, H3O+
a. the rate does not change on doubling [CH3Br] and 10. The order of reactivities of the following alkyl halides
making [OH− ] to half for a S N 2 reaction is
b. the rate becomes half on doubling [CH3Br] a. R F > R Cl > R Br > R I b. R F > R Br > R Cl > R I
c. the rate becomes half on doubling [OH− ] c. R Cl > R Br > R F > R I d. R I > R Br > R Cl > R F
d. All the above are correct
11. Choose the incorrect statement.
4. The product of reaction of alcoholic silver nitrite with a. An S N1 reaction proceeds with inversion of
ethyl bromide is configuration
a. ethyl nitrite b. ethene b. An S N 2 reaction proceeds with stereochemical
c. nitroethane d. ethyl alcohol inversion
c. An S N 2 reaction follows second order kinetics
5. Which does not give methyl bromide? d. The reaction of tert-butyl bromide with OH− follows
a. CH3OH + HBr b. CH3OH + Br2 first order kinetics
c. CH3OH + PBr3 d. CH3COOAg + Br2
12. An organic compound C 5 H9Br ( A ) which readily
6. Consider the following haloalkanes,
decolourises bromine water and cold alkaline KMnO4
I. CH 3I II. CH 3 F solution gives C5 H11 Br (B) on catalytic hydrogenation.
III. CH 3 Cl IV. CH 3 Br The reaction of A with alcoholic KOH first and then
The correct sequence of increasing order of dipole with NaNH2 produces C with evolution of NH3 . C
moment is reacts with Lindlar’s catalyst to give D and on reaction
a. I < II < III < IV b. IV < III < II < I with Na in liquid NH3 produces E . D and E are
c. I < IV < II < III d. III < IV < I < II isomers. The compound A is
7. Consider the following reaction, Br
∆ |
Isopropyl chloride + A → 2-ethoxy propane + NaCl a. CH3CH2CH == C— CH3
The compound A is Br
a. C2H5Cl |
b. C2H5ONa b. CH3CH2 — C == CHCH3
c. CH2N2 c. Both (a) and (b)
d. CH3ONa d. Neither (a) nor (b)

@iitjeehelps
658 SELF STUDY GUIDE BITSAT

13. Consider the following reaction, 19. Consider the following reaction,
Mg/ether D2 O CH3 Br + Nu– → CH3 — Nu + Br –
Br Cl B C
Na/ether The decreasing order of the rate of the above reaction
A (with two with Nu– is
Na/ether (with two moles of C ) I. PhO− II. AcO−
moles of A ) −
D III. HO IV. CH3 O−
E
a. IV > III > I > II b. IV > III > II > I
D and E respectively are c. I > II > III > IV d. II > IV > III > I

a. Cl CH2OH and Cl D 20. The compound A in the following reaction sequence is


PCl5 AlC. KOH
A → B → C
b. Cl Cl and D D H2/Ni

Propane
c. D D and Cl Cl a. chloroethane b. ethanol
c. 1,2-dichloroethane d. None of these
d. None of the above
21. Which of the following statements regarding S N1
14. The main product of the reaction of propane with
chlorine at 25°C in the presence of sunlight is reaction shown by alkyl halide is incorrect?
a. 1-chloropropane a. The added Nu− plays no kinetic role in SN1 reaction
b. 2-chloropropane b. S N1 reaction involves the inversion of configuration of
c. chloroethane optically active substance
d. chloromethane c. S N1 reaction on chiral starting material ends up with
racemisation of the product
15. Hunsdiecker reaction is an example of d. Polar protic solvent increases the rate of S N1
a. decarboxylation reaction
b. debromination
c. decarboxylation and bromination 22. The product of which of the following reactions is not a
d. bromination vicinal dihalide?
a. Reaction of ethylene glycol with P + Br2
16. When 3-phenyl propene is treated with HBr in the b. Reaction of HCl with ethyne
presence of peroxide, the major product formed is c. Reaction of HBr with ethyne in the presence of
a. 1-bromo-3-phenyl propane peroxide
b. 1,2-dibromo-3-phenyl propane d. Reaction of Cl2 with propene
c. 2-bromo-1-phenyl propane
d. 3-(p-bromo phenyl) propene 23. The treatment of CH3 MgX with CH3  C ≡≡ C  H
17. The alkyl halide that undergoes S N 1 reaction more produces
readily is a. CH3  CH== CH2
a. ethyl bromide b. CH3C ≡≡ C  CH2
H H
b. iso-propyl bromide
 
c. vinyl bromide c. CH3  C == C  CH2
d. t-butyl bromide
d. CH4
18. Which alkyl halide from the following would you
24. Consider the following bromides,
expect to react more rapidly by S N 2 mechanism?
I. Me
a. CH3 CHCH2CH2Br Br

CH3 II. Me
b. CH3CH2CHCH2Br Br
|
CH3 Me Me
III.
CH3 Br

c. CH3 CH2 C CH2Br The correct order of S N 1 reactivity [AIEEE 2010]
 a. II > III > I
CH3 b. II > I > III
c. III > II > I
d. CH3CH2CH2 CHBr
| d. I > II > III
CH3

@iitjeehelps
HALOGEN DERIVATIVES OF HYDROCARBON 659
25. Which of the following reactions come within the 32. The increasing order of reactivity of the following
framework of elimination? bromides in S N 1reaction is
a. (CH3 )2 CHCl + SH⊕
O
b. (CH3 )3 C  Br + ethanolic KCN I. II. III. IV.
c. CH3CH2CH2Cl + I−
d. (CH3 )2 CHBr + aq. KOH
Br Br Br
26. 20% aqueous NaCl solution containing ethyl alcohol Br
when subjected to electrolysis, produce a. IV < III < I < II b. III < IV < I < II
a. C2H5Cl c. II < III < I < IV d. II < III < IV < I
b. CH3CHO
c. CHCl3 33. The Wurtz-Fittig reaction involves condensation of
d. CCl3CHO a. two molecules of aryl halides
b. one molecule of each of aryl halide and alkyl halide
27. Alkaline hydrolysis of which of the following c. one molecule of each aryl halide and phenol
compounds give methyl ketones? d. two molecules of an alkyl halide
I. Ethylene dichloride
34. Consider the following reaction,
II. 2, 2-dichlorobutane 3 Cl2 , ∆ Br2 / Fe Zn/ HCl
III. Ethylidene chloride C 7 H8 → A → B → C
IV. Isopropylidene chloride The compound C is
a. I,II and III a. o-bromotoluene
b. II, III and IV b. m-bromotoluene
c. I and III c. p-bromotoluene
d. II and IV d. 3-bromo-2,4,6-trichlorotoluene
28. When optically active halide is attacked by CN− , the 35. Trans-2-phenyl-1-bromocyclopentane on reaction with
product obtained is a racemic mixture. Hence, the
alcoholic KOH produces
halide should be
a. 4-phenylcyclopentene b. 2-phenylcyclopentene
a. primary
c. 1-phenylcyclopentene d. 3-phenylcyclopentene
b. secondary
c. tertiary 36. Assertion (A) Exposure of ultraviolet rays to human
d. None of the above causes the skin cancer, disorder and disrupt the
29. Dipole moment of cis-2,3-dichloro-2-butene is ……… immune system.
than the dipole moment of cis-1, 2-dichloroethene. Reason (R) Carbon tetrachloride is released into air,
a. more it rises to atmosphere and deplets the ozone layer.
b. less a. Both A and R are correct and R is correct
c. neither more nor less explanation of A
d. None of the above b. Both A and R are correct but R is not the correct
30. Aryl halides are less reactive towards nucleophilic explanation of A
substitution reaction as compared to alkyl halides c. A is correct but R is incorrect
d. R is correct but A is incorrect
due to
a. the formation of less stable carbonium ion 37. IUPAC name of DDT is
b. resonance stabilisation a. 1,1-bis (4-chlorophenyl)-2,2,2-trichloroethane
c. longer carbon-halogen bond b. 1,2-bis (4-chlorophenyl)-1,1,1-trichloroethane
d. sp 2-hybridised C attach to X c. 2,2-bis (4-chlorophenyl)-1,1,1-trichloroethane
Cl aq. NaOH d. 2,1-bis (4-chlorophenyl)-1,1,1-trichloroethane
31. In the reaction Ph CH 3  
2
→ X 
→ Y,
heat 38. Which of the following statements is/are true for
the product Y is chloroform?
a. o-cresol a. Chloroform exposure causes damage to heart
b. p-cresol b. People develop sores when skin is immersed in
c. 2, 4-dihydroxytoluene chloroform
d. benzoic acid c. Chloroform has no effect on central nervous system
d. All of the above

@iitjeehelps
BITSAT Archives
1. Arrange the following compounds in increasing order 7. The correct order of reactivity of hydrogen halides with
of their boiling points: [2014] ethyl alcohol is [2008]
CH3 a. HF > HCl > HBr > HI b. HCl > HBr > HF > HI
I. CH  CH2Br c. HBr > HCl > HI > HF d. HI > HBr > HCl > HF
CH3
8. Suitable reagents A and B for the following reactions
II. CH3 CH2  CH2  CH2  Br respectively are [2005]
CH3
| Br
III. CH3  C  CH3 +A
|
Br Br
a. II < I < III +B
b. I < II < III
c. III < I < II
d. III < II < I a. Br and Br2 b. Br2 and NBS
c. NBS and NBS d. NBS and Br2
2. Which of the following is less acidic among the given
9. Following compound is treated with NBS: [2005]
halogen compounds? [2013]
a. CHF3 b. CHI3
c. CHCl3 d. CHBr3 CH2CH CH2 + NBS A

3. 2-bromopentane with alcoholic KOH yields a mixture


of three alkenes. Which of the following alkene is Compound formed A is
predominant? [2012]
a. 1-pentene a. CHCH CH2
b. Cis-2-pentene Br
c. Trans-2-pentene
d. Cis-1-pentene
4. Toluene on reaction with N-bromo-succinimide gives b. C CHCH2Br
[2011]
a. p -bromomethylbenzene
b. o -bromomethylbenzene
c. phenyl bromomethane c. CHCH2 CH2
d. m -bromomethylbenzene
Br Br
5. Which of the following will yield a mixture of
2-chlorobutene and 3-chlorobutene on treatment with
HCl? [2011] d. CH2CH CH2
a. CH2 == C == CH  CH3
b. H2C == C  CH == CH2
Br

CH3 10. Consider the following reaction,
c. CH2 == CH  CH== CH2
d. HC ≡≡ C  CH== CH2 + Br2 A
6. 3-methyl-2-butanol on treatment with HCl gives
A will have configuration [2005]
(major product) [2010]
a. 2-chloro-2-methylbutane Br Br
b. 2-chloro-3-methylbutane a. b.
c. 2, 2-dimethylpentane Br Br
d. None of the above

c. Both (a) and (b) d. None of these

@iitjeehelps
Answer with Solutions
Practice Exercise 11. (a) SN1 reactions are accompanied by racemisation.
Br Br
1. (b) Vinyl chloride is relatively less reactive
Cl 12. (a) CH3CH2CH C CH3 CH3CH2 C CHCH3
 + A A

Hydrogenation
CH2 == C H ←→ CH2− == CH

Hydrogenation
While allyl chloride is very reactive due to stabilisation
or
of allyl carbonium ion by resonance.
+ + (i) Alc. KOH
(i) Alc. KOH (ii) NH2–
CH2 == CH  CH2 ←→ CH2  CH == CH2 (ii) NH2– Br Br
Isopropyl chloride in between them is reactivity
towards SN1. CH3CH2CH2 CH CH3 CH3CH2 CH CH2 CH3
B B
2. (b) SN1 reaction depends only upon concentration of
CH3CH2C C CH3 CH3CH2 C C CH3
substrate. C
C
3. (a) S N 2 reaction rate is directly proportional to Na/liq.NH3
H2/Pd-BaSO4
concentration of substrate and nucleophile.
4. (a) CH3CH2Br + AgNO2 → CH3CH2NO2 + AgBr H H H 5C 2 H
Alcoholic Major C C C C
5. (b) CH3OH + HBr → CH3Br + H2O H 5C 2 CH3 H CH3
E (trans)
D (cis)
CH3OH + PBr3 → CH3Br + H3PO3
CH3COOAg + Br2 → CH3Br + CO2 + AgBr Mg/ether
13. (c) Cl Br Cl MgBr
6. (c) CH3I < CH3Br < CH3F < CH3Cl
1.6 D 1.79 D 1.84 D 1.94 D A B D2 O
H3 C ∆ Na/ether D
7. (b) CHCl + NaOC2H 5 → Cl
H3 C Sodium ethoxide C
Isopropyl chloride
Cl Cl Na/ether
CH3 — CH— O — C2H5 + NaCl (with two
 E moles of C )
CH3
2-ethoxy propane D D
8. (c) Reaction of t-alkyl halides with aqueous alkali D
gives S N1 reaction and rate of S N1 reaction is not Cl2
based on the concentration of nucleophile (alkali). 14. (b) CH3  CH2  CH3 → CH3  CH  CH3
Hence, reaction rate remains constant. 25 ° C |
− HCl
9. (a) Cl
O OH 2° (Major)
+
+ CH3MgBr H
CH3 + CH3  CH2  CH2Cl
H2 O 1° (mi nor )

+ aq . KOH
∆ H 15. (c) R  C  OH → R  Br + CO2
Br + CCl4 / ∆
CH3 
O
Clearly, it is decarboxylation with bromination.
HBr/H2O2 16. (a)
HBr
CH3 H 2C CH—CH2—
Peroxide
(Antimarkownikoff’s rule)

Br BrCH2—CH2—CH2—
10. (d) The order of reactivities of the alkyl halides forSN 2
17. (d) S N1 mechanism involves formation of carbocation
reaction is
intermediate.
R I > R Br > R CI > R F

@iitjeehelps
662 SELF STUDY GUIDE BITSAT

18. (a) CH3CHCH2CH2Br Cl  OH 


    
→ CH3CH2  C  OH
Aq . KOH
CH3 27. (d) CH3CH2  C  CH3  
   
Out of the given compounds, three are primary halides. Cl  CH 
The presence of methyl group closer to halide group  3 
2,2-dichlorobutane
increases the steric hindrance and decreases rate of the O
reaction. Hence, compound (a) reacts rapidly. 
− H2O
  → CH3CH2  C
19. (a) Nucleophilicity order is 
O– O CH3
Butanone - 2
CH3 –
O > H –
O > > CH3  OO– H2C H2C
aq . KOH
C  Cl  → C  OH
+I-effect increases Negative charge Negative charge H3C H3C
O
electron density on O. stabilisation comparatively
by resonance stable also 
Tautomerisation
decreases electron decreases electron → CH3  C  CH3
density on O. density on O.
28. (c) Tertiary halide follows S N1mechanism which forms
PCl5 Alc. KOH H2 /Ni
20. (d) A → B → C → propane racemic mixture.
Since, C gives propane (CH3CH2CH3), it must be an 29. (a) [Hint : It is clear from the figure itself]
alkene, CH3CH == CH2 (C). If C is formed from B through H Cl H3C Cl
alc.KOH, then B must be an alkyl halide, CH3CH2  CH2Cl
which is formed from A by PCl5 . This suggests A would be C C
alcohol, CH3CH2CH2OH (propanol).
C C
21. (b)
CH2  OH CH2  Br H Cl H3C Cl
22. (b) (a)  → 
P
30. (b) All the aryl (aromatic) compounds are less reactive
CH2  OH Br2 CH2  Br
Vicinal
towards nucleophilic substitution because of the
presence of π-electrons (resonance) due to which they
Cl
HCl
(b) CH≡≡ CH  → CH3  CH repel the incoming nucleophile.
Geminal Cl 31. (d)
HBr
CH3 CCl3
(c) CH≡≡ CH  →
peroxide
CH2  CH2  Br
 Cl2 aq. NaOH
Br Boiling HCl
Vicinal

H (X )
Cl2  O
(d) CH3  CH == CH2  → CH3  C  CH2  Cl
 C OH
OH
Cl C OH
Vicinal
OH
23. (d) –H2O
CH3 MgX + CH3C ≡≡ CH → CH4 + CH3C ≡≡ CMgX
Benzoic acid
24. (a) Unstable
32. (a) II is most reactive as it produces an aromatic
+ Me > Me + Me
(2°-allylic) (2°-alkylic) carbocation while IV is less reactive as it produces a
(B ) (C ) non-resonance stabilised carbocation. III is least reactive
+ than I as former involve an anti-aromatic carbocation.
> Me CH2 33. (b)
(1°-alkylic)
(A ) X R
25. (b) (CH3 )3 C  Br undergoes dehydrobromination by
ethanolic KCN and produces isobutylene. Na
26. (c) Electrolysis of aqueous NaCl solution gives NaOH and + RX
Dry ether
Cl2. These reactants on reaction give chloroform. –NaX
Aryl halide Alkyl halide

@iitjeehelps
HALOGEN DERIVATIVES OF HYDROCARBON 663
34. (b) Anti-elimination means —H and —Br both departing groups
CH3 CCl3 CCl3 CH3 must be present at dihedral angle of 180° (anti).
36. (a) Carbon tetrachloride rises to atmosphere and deplete
3Cl2 Br2 Zn the ozone layer. This depletion of ozone layer increases
∆ Fe HCl exposure of UV rays to human being which lead to increase
Br Br of skin cancer, eye disease and disorder with discruption of
m-bromotoluene
the immune system.
37. (c) IUPAC name of DDT is 2,2-bis (4-chlorophenyl)
s H 1,1,1-trichloroethane.
HO Alc. KOH
35. (d) H Ph E2 Ph 38. (b) Chloroform exposure may cause damage to liver and to
H Anti-elimination kidney and some people develop sores when skin in
immersed in chloroform.
Br H H
3-phenylcyclopentene

BITSAT Archives
1. (c) Boiling point decreases with increase in branching. CH3
2. (a) Due to stronger −I- effect of F than that of Cl, 
6. (b) CH3  CH  CH  CH3 + HCl →
CHF3 should be more acidic than CHCl3. But actually 
reverse is true. OH
This is due to •• CCl−3 left after the removal of a proton CH3
from CHCl3 is stabilised due to presence of d -orbitals  +
1, 2 H-shift
CH3  CH  CH  CH3 + H2O   →
in Cl than •• CF3− left after the removal of a proton from
2° carbocation (less stable)
CHF3 which is not stabilised due to the absence of
CH3 CH3
d -orbitals on F.
Alc.KOH  − 
Cl
3. (c) CH3CH2CH2 CH CH3 → CH3  C  CH2  CH3  → CH3 C  CH2  CH3
+
| 
Br Cl
3° carbocation 2-chloro-2-methyl butane
CH3CH2CH2CH== CH2 + CH3CH2CH== CHCH3
7. (d) Among hydrogen halides, as the size of halide ion
By Saytzeff’s rule, substituted alkenes are more stable. increases, its reactivity towards ethyl alcohol also increases.
Hence, out of cis and trans forms, trans product is Thus, the order of reactivity of hydrogen halides is
more stable.
HI > HBr > HCl > HF
4. (c) NBS (N-bromosuccinimide) causes bromination at 8. (d) Br
allylic position.
+ NBS
CH3 CH2Br
  Br

+ NBS + Br2

Phenyl bromomethane
9. (a) CH2CH CH2 + NBS
5. (a) CH2 == C == CH CH3 + H+ →
+ + CHCH CH2
[CH2 == C  CH2CH3 + CH2 == CH  CHCH3]
Br
Cl−
→ CH2 == CC H2CH3 + CH2 == CH  CHCH3 Br
 
Cl Cl 10. (b) + Br2
2-chlorobutene 3 -chlorobutene Br

@iitjeehelps
25
Alcohols, Phenols
and Ethers

Alcohols
The compounds in which hydroxyl group is attached to C-atoms are called alcohols.
(i) General formula: C nH 2 n + 1OH (ii) sp3-hybridised carbon , geometry: tetrahedral

Classification
Alcohols are divided into three types, i.e. monohydric, dihydric and trihydric alcohols.

Monohydric Alcohols
Monohydric alcohols may be divided into three types, i.e. 1°, 2° and 3° depending upon the carbon
atom to which OH group is attached.
Preparation of Monohydric Alcohols
1. From alkane (Indirect conversion)
Cl 2 aq . KOH
CH3CH3 → CH3CH 2Cl + HCl → CH3CH 2OH
hv

2. From alkene
(a) Hydration
Dil. H 2 SO 4
R CH==CH 2 → R  CH CH3

OH
(b) Oxo reaction
Co, p ressure LiAlH 4
CH3 CH==CH CH3 → CH3 CH 2 CH  CH3 → CH3 CH 2  CH CH3
[CO + H 2O]
 
CHO CH 2 OH

@iitjeehelps
ALCOHOLS, PHENOLS AND ETHERS 665
3. From alkyl halide (iii) Oxidation of methane
H 2O Cu-tube
R  X + NaOH → R OH + NaX CH 4 + O2 → CH3OH
100 °C / 200 atm
where, R = 1° ,2 ° or 3° alkyl, aryl etc. (b) Preparation of ethanol
4. From carbonyl compounds (i) Fermentation of molasses
(a) Reduction (ii) From starch
Ni / H 2
R CHO + H 2 →

R CH 2 OH (1° ) Physical Properties of Monohydric Alcohols
O Monohydric alcohols have many physical properties which
 are given below :
Ni / H 2
R  C  R + H 2 → ∆
R  CH  R (2 ° ) (i) Physical state Lower alcohols are colourless, toxic
 liquids with burning taste whereas, higher alcohols
OH are colourless, odourless and wax like solids.
Reducing agents Ni -H 2 , LiAlH 4 (in ether), (ii) Boiling point The boiling point of alcohols is much
NaBH 4 , Na -EtOH, Na -Hg, H 2O can be used. higher than that of corresponding hydrocarbons
MPV (Meerwein-Ponndorf-Verley) reduction due to the presence of intermolecular H-bonding in
R‚ [(CH 3 )2 CH  O]3 Al R‚ alcohols.
ƒ
C== O → ƒ CH OH Intermolecular H-bonds
R In propyl alcohol R
O H O H O H O H
+



HCHO / H 3O
(b) RMgX → RCH 2OH (1° ) R R R R
RCHO / H 3O +
The boiling point for isomeric alcohols follows the
R  MgBr → R 2 CHOH (2 ° )
order
R2CO / H 3O +
→ R3C  OH(3 ° ) 1° > 2° > 3°
[Primary [Secondary [Tertiary
alcohols] alcohols] alcohols]
Br
H O ƒ
2 RMgBr + O2 →
2
2 ROH + 2 Mg (iii) Solubility Lower members of alcohols are soluble
‚
OH in water due to the presence of intermolecular
H 2O
X hydrogen bonding.
ƒ
RMgX + H 2C CH 2 → RCH 2CH 2OH + Mg
‚ƒ ‚
OH Acidic Character of Monohydric Alcohols
O
Alcohols are weak acids due to the presence of alkyl groups
5. From acid and its derivatives
(exerting +I-effect) linked to OH group that increases the
LiAlH 4
RCOOH → RCH 2OH + H 2O electron density on oxygen thereby, decreasing the polarity
Ether of O H bond.
O •• ••
 R  O H → R  O − + H +
4H •• ••
R  C  X → RCH 2OH + HX Alkoxide ion
Na-EtOH

6. From primary aliphatic amines The increasing order of acidic character of alcohols is as
CH2 OH follows
CH2 N H2 HNO2 1° > 2° > 3°
HO N O NaNO /HCl
2
Alcohols are weaker acids than water because alkoxide ion
Benzyl amine Benzyl alcohol is a strong conjugate base of alcohols as it readily accepts
proton from a water molecule.
7. Industrial methods
(a) Preparation of methanol Chemical Properties of Monohydric Alcohols
(i) Destructive distillation of wood The chemical reactions of alcohols are due to the presence
(ii) From water gas of polar → C OH bond.

∆ Reactions Involving Cleavage of O  H bond


C + H 2O → CO + H 2
Water gas (i) Reaction with metals
300 °C 2 RO H + 2Na → 2 RO−  Na + + H 2 ↑
CO + 2H 2 → CH3OH Alcohol Metal Sodium alkoxide Hydrogen
200 atm

@iitjeehelps
666 SELF STUDY GUIDE BITSAT

(ii) Reaction with carboxylic acids (Esterification) Note The carbocation formed during dehydration undergoes
rearrangement resulting in the formation of more than one
O O
alkene.
 H 2SO4 
R  C OH + H OR′ → R  C OR′ + H 2O Reaction of ethanol (alcohol) with sulphuric acid at
Carboxylic acid Alcohol Ester Water different temperatures
H 2SO4 acts as a protonating as well as dehydrating H 2 SO4
CH3 CH 2 OH → CH3CH 2 HSO4 + H 2O
agent. 383 K

(iii) Reaction with acid derivatives H 2 SO4


CH3 CH 2 OH → CH 2 == CH 2 + H 2O
O 433 K
Alkene
 Base
R  C Cl + H O  R′ → 2CH3 CH 2 OH → C 2H 5OC 2H 5 + H 2O
H 2 SO4

Acid chloride Alcohol 413 K


O Ether
 (ii) Oxidation of alcohols Oxidation of alcohols results in
R  C OR′ + HCl the formation of C == O bond.
Ester
[O] [O]
Reactions Involving Cleavage of CO Bond R CH 2OH → RCHO → RCOOH
Alcohol Acidified Aldehyde Acid
The order of reactivity of alcohols towards these type of KMnO4
reactions is as follows: CrO 3
Tertiary alcohols >Secondary alcohols RCH 2OH → RCHO
(1 ° ) Aldehyde
(3 ° ) (2 °)
>Primary alcohols CrO 3
(1 ° ) (2º ) → Ketone
(i) Reaction with halogen acids
PCC
The reactivity order for the halogen acids is as follows : CH3 CH==CH CH 2OH → CH3 CH == CHCHO
H I > H Br > H Cl > H F Cu
NOTE CH3CH2 OH → CH3CHO
573 K
1° alcohol Aldehyde
H  Cl/ZnCl2
R — Cl + H2O [Groove’s method]
CH3  CH CH3 →
Cu
573 K
CH3  C  CH3
48% HBr  
R—OH R — Br + H2O OH O
Alcohol Conc. H2SO4
2° alcohol Ketone
HI H3C
R — I + H 2O Cu
H 3C C OH 573 K
CH3 C CH2 + H2O
H3 C
These reactions follow SN 1 mechanism. CH3
3°alcohol
(ii) Reaction with phosphorus halides and thionyl Alkene
chloride (SOCl 2 )
Identification of Primary, Secondary and Tertiary
PCl5 Alcohols
R—Cl + POCl3 + HCl
PCl3 The primary, secondary and tertiary alcohols can be
R—OH R—Cl + H3PO3
Alcohol distinguished by
[P/I2]
R—I + H3PO3 (i) Victor Meyer’s test Alcohols when treated with P/I 2
SOCl2 and then with AgNO 2 and HNO 2 , a coloured product is
R—Cl + SO2 + HCl produced with NaOH.
Primary alcohol → Blood red product
Reactions Due to  OH Group Secondary alcohol → Blue coloured product
The reactivity order would be Tertiary alcohol → No colour
3 ° alcohol > 2 ° alcohol > 1 ° alcohol (ii) Lucas test The equimolar mixture of anhydrous ZnCl 2
(i) Dehydration and conc. HCl is called Lucas reagent. When alcohols
are treated with it,
H 2 SO4 (85%)
CH3  CH CH3 → CH3  CH == CH 2 + H 2O Primary alcohols → No turbidity at room temperature
440 K
 Alkene
Secondary alcohols → Turbidity after 5 min
OH
Alcohol Tertiary alcohols → Immediate turbid solution

@iitjeehelps
ALCOHOLS, PHENOLS AND ETHERS 667
Dihydric Alcohols Superheated
2. Oil or fat → CH 2  C H  C H 2
steam
They are the compounds containing two OH groups   
either on same carbon atom or different C-atom. OH OH OH
e.g. Ethylene glycol
Chemical Reactions of Glycerol
Methods of Preparation of Ethylene Glycol Glycerol has two primary  OH groups and one secondary
1. From alkenes  OH group, which is little less reactive than the primary
Baeyer's  OH groups.
reagent
CH 2 ==CH 2 → CH 2 CH 2 1. Reaction with metal
Alk. KMnO4
  CH 2ONa
OH OH CH 2  CH  CH 2 
Na
   → CH  OH
2. From halides
OH OH OH 
aq .Na 2CO 3
CH 2  CH 2 → CH 2  CH 2 CH 2ONa
   
Br OH 2. Reaction with HX
Br OH
CH 2  CH  CH 2
HX
Physical Properties of Ethylene Glycol    → C H 2  C H  C H 2
1. Colourless, syrupy liquid. OH OH OH   
X X X
2. Soluble in water as well as in ethanol. —X 2
3. It is toxic as methanol. → CH 2 == CH  CH 2  X
Allyl halide
Chemical Reactions of Ethylene Glycol 3. Reaction with acetic acid
1. Reaction with metals CH 2OCOCH3
CH 2  OH CH 2ONa CH 2ONa CH 2  CH  CH 2 
CH 3COOH

Na
→ 
Na
→    → CHOCOCH3
CH 2  OH 50 ° C
CH 2OH 160 ° C CH 2ONa OH OH OH 
CH 2OCOCH3
2. Reaction with HCl or phosphorus halides
4. Dehydration
2HCl or
CH2 OH PCl5 CH2Cl CH 2OH
 KHSO 4
CH2 OH CH2Cl CHOH → CH 2 ==CH  CHO

Pl3 PBr3 
CH2Br CH 2OH
CH CH
CH2Br

3. Nitration
Phenols
CH 2 OH CH 2ONO2 The organic compounds in which hydroxy group ( OH) is
HNO 3
 →  directly linked to a benzene ring are called phenols or
H 2 SO4
CH 2 OH CH 2ONO2 carbolic acid.
OH
4. Reaction with HNO3 (Oxidation)
Conc. HNO 3 COOH
(CH 2OH)2 → 
[O]
COOH
Phenol
Oxalic acid

Trihydric Alcohols Preparation of Phenols


Preparation of Glycerol Phenols are prepared by the following methods:
CH 2 CH 2OH CH 2OH (i) Hydrolysis of aryl halides (Dow’s process)
 HOCl
 NaOH  Cl O – Na+ OH
1. CH → CHCl → CHOH
   aq. NaOH H
+

CH 2OH CH 2OH CH 2OH 300 atm

@iitjeehelps
668 SELF STUDY GUIDE BITSAT

(ii) From diazonium salts of electron donating groups, like CH3 etc., on the benzene
+N≡≡NX − OH ring destabilises the phenoxide ion by increasing the
negative charge, thereby, decreasing the acidic strength of
H+ phenol.
+ N2 +HX
H2O

Chemical Reactions of Phenol


Industrial Preparation of Phenols The chemical reactions of phenol are classified into two
Phenols for industrial purposes can be manufactured in categories:
two ways:
Reactions due to the cleavage of O  H bond
(i) From cumene
O—OH (a) Reaction with metals
OH O– Na+
CH3—CH—CH3 CH3—C—CH3 OH

O2 Dil. H2SO4/H2O 2 + 2Na 2 + H2


+ CH3COCH3
Catalyst (HBr)
Acetone
150°C Phenol
Cumene Cumene (b) Acylation
hydroperoxide OH OCOR

(ii) From Raschig process through benzene H2SO4


+ RCOOH
Cl OH –H2O

Phenol Phenolic ester


CuCl2/FeCl3 H2O, 750 K
+ HCl + O2 250°C, – H2O –HCl Phenolic esters when heated with AlCl3 , o and p-acyl
phenols are produced (Fries rearrangement).
O
(iii)From coaltar Phenols can be obtained by the middle OH OH
oil fraction of coaltar distillation. CR
AlCl3
OCOR +
Physical Properties of Phenols o-isomer
(major) CR
Phenols are colourless crystalline solids or liquids with
carbolic odour. The boiling point of phenols is higher than O
corresponding alcohols and arenes of compared masses p-isomer (minor)

due to the presence of intermolecular H-bonding. Phenols (c) Benzoylation This reaction is known as
possess high dipole moments.
Schotten-Baumann reaction.
Acidic Character of Phenols OH OCOC6H5
The acidic character of phenols is due to the conjugation
between lone pair of electrons of oxygen and benzene + C6H5COCl + HCl
nucleus.
+ + +
OH OH OH OH OH (d) Reaction with Zn dust

– OH

Zn dust
– + ZnO

Effect of Substituents on Phenols Reactions due to Benzene Ring


The presence of electron withdrawing groups like
 NO2 , CN etc., on the benzene ring stabilises the Phenols undergo the following reactions due to the presence
phenoxide ion by the dispersal of negative charge, thereby, of benzene ring :
increasing the acidic strength of phenol whereas presence

@iitjeehelps
ALCOHOLS, PHENOLS AND ETHERS 669
(a) Electrophilic substitution reaction Special Reactions of Phenols
OH
Br Br The special reactions of phenols can be summarised as:
Halogenation (i) Reimer-Tiemann reaction This reaction looks like
OH ONa
Br CHCl2
2,4,6-tribromophenol CHCl3, aq. NaOH NaOH
340 K
OH OH
O2N NO2 OH ONa
Nitration
CHO CHO
H3O+

NO2
o-hydroxybenzaldehyde
Picric acid
(Major product)
OH
SO3H NOTE When CCl 4 in place of CHCl 3 is used in the reaction,
salicylic acid is produced in the reaction.
K
288 (ii) Kolbe’s reaction
Sulphonation
37 OH
3K ONa OH OH
COONa COOH
400 K H+
+ CO2 4-7 atm

(Major product) Salicylic acid


SO3H (Major product)
OH OH
Friedel-Crafts CH3 NOTE Salicylic acid is used for the production of aspirin
alkylation (2-acetoxy benzoic acid), a well known analgesic.
+
OH OCOCH3
(Minor) COOH COOH
CH3 Conc. H2SO4
(Major) + (CH3CO)2O + CH3COOH

(b) Gattermann synthesis This reaction works as Aspirin


OH OH
C==NH
+HCN H+ Distinguish Tests for Phenols
H and Alcohols
Phenols can be distinguished from alcohols through the
–NH3, –H2O Hydrolysis
following tests:
OH 1. Neutral FeCl3 test Phenols react with neutral FeCl3 to
produce a blue violet or green coloured complex.
CHO
Alcohols do not react with FeCl3 .
2. Liebermann’s test When sodium nitrite dissolved in
Salicylaldehyde conc. H 2SO4 and it is added to phenol, red/brown
colouration is produced which changes to blue-green
(c) Ether formation
on the addition of a base. Alcohols do not respond to
O– Na+ OR
this test.
R X 3. Bromine water test When bromine water is added to
+ NaX phenol, a white precipitate of 2, 4, 6-tribromophenol is
Sodium phenoxide Ether produced. Alcohols do not respond to this test.

@iitjeehelps
670 SELF STUDY GUIDE BITSAT

The reactivity order of halogen acids for this reaction


Ether are
H I > H Br > H Cl
Dialkyl derivative of water is called ether. It is anhydride of
If one R group is phenyl and other is alkyl group, the
alcohol.
cleavage of C  O bond produces phenol and alkyl
halide.
Preparation OR OH

1. From alkenes 100°C


H 2 SO4 + HI +R I
CH 2 == CH 2 + ROH → CH3CH 2OR
2. From alkyl halides Cyclic ethers cleaved by HI/HBr to form dihaloalkanes.
(a) Williamson ether synthesis 100°C
+ 2HI I  CH2  CH2CH2CH2  I + H2O
RX + R′ ONa → ROR′ + NaX O
Tetrahydrofuran
(b) Heating alkyl halides with silver oxide
∆ (b) Reaction with PCl 5
2RX + Ag 2O → R  O  R + AgX

3. From alcohols CH3  O  CH3 + PCl 5 → 2CH3Cl + POCl3
(a) Dehydration of alcohols (c) Reaction with sulphuric acid
Conc .H 2 SO4
R OH + HOR → R  O  R + H 2O Dil. H 2 SO4
140 °C R O  R + H 2O → 2 R OH
Under pressure
(b) Action of diazomethane on alcohols
(d) Reaction with acid derivatives
BF3
R  OH + CH 2 N 2 → ROCH3 + N 2 AlCl 3 / Anhyd. ZnCl 2
R O  R + R′ COCl →

Physical Properties of Ethers R Cl + R′ COOR

Following are the main properties of ethers: Reactions due to Ethereal Oxygen
(i) Dimethyl and ethyl methyl ethers are gases at These reactions are due to the presence of two lone pairs on
ordinary temperature. Other lower members are ethereal O-atom.
colourless, pleasant smell, volatile liquids. (a) Reaction with concentrated acids
(ii) Ethers have lower boiling points in comparison to R ‚• • R ‚ 
corresponding alcohols due to the presence of O•• + H + Cl − →  O+ H Cl −
Rƒ R ƒ •• 
weak dipole–dipole forces among intermolecular
Oxonium salts
ethers.
These salts stabilises by the anion of acids.
(iii) Ethers are soluble in water due to the formation of
(b) Formation of coordination complexes
H–bonding with water molecules.
H3C ‚ • • H3C ‚ −
R O R O•• + BF3 → O+ BF3
ƒ ƒ ••
O ---H H---O H3C H3C
R R Boron trifluoride
etherate
Solubility decreases with increase in the number of
C–atoms. Reactions Due to Alkyl Group
(a) Action of air and light
Chemical Properties of Ethers O2
CH3 CH 2 O CH 2 CH3 → CH3 CH  OCH 2CH3
The chemical properties of ethers are classified into three hv

Ether
OOH
categories: Hydro peroxide

Reactions Involving the Cleavage of C  O (b) Halogenation


Cl 2
bond CH3CH 2OCH 2CH3 + 2HCl → CH3 CH OCH CH3
(a) Reaction with halogen acids
Dark
 
Cl Cl
100 ° C
R O  R + HI → R OH + R I hν
CH3OCH3 + 6Cl 2 → CCl3 O CCl3 + 6HCl

@iitjeehelps
Practice Exercise
1. Appropriate test to make distinction between methanol a. one OH group b. two OH groups
and methyl carbinol is c. three OH groups d. four OH groups
a. test with acetic acid 6. Arrange the following compounds in increasing order
b. test with salicylic acid and conc. H2SO4
of boiling point:
c. test with sodium
d. None of the above Propan-1-ol, butan-1-ol, butan-2-ol, pentan-1-ol
2. Which of the following alcohols is unable to turn a. Propan-1-ol < butan-2-ol < butan-1-ol < pentan-1-ol
orange colour of chromic acid to green? b. Propan-1-ol < butan-1-ol < butan-2-ol < pentan-1-ol
a. Primary alcohol c. Pentan-1-ol < butan-2-ol < butan-1-ol < propan-1-ol
b. Secondary alcohol d. Pentan-1-ol < butan-1-ol < butan-2-ol < propan-1-ol
c. Tertiary alcohol 7. An alcohol A when heated with copper gives a product
d. Allyl alcohol B not having oxygen atom. B on ozonolysis gives two
3. Match List I with List II and choose the correct code. isomeric products C and D.C on oxidation gives a
List I List II
monobasic acid E , silver salt of which contains
59.6% Ag. The structure of A is
A. [O] 1. H2C —— CH2 a. CH3 — C H — C H — CH 2 —CH3
CH3 CH2OH → Product-1
‚ƒ | |
O CH3 OH
(Functional isomer)

Reduction 2. Optical isomer OH


B. CH3 COCH2CH3 → |
Product-2 b. CH3 — C — CH2 — CH2 — CH3
|
Na / EtOH 3. Ethanol CH3
C. CH3 CONH2 →
Product-3 c. CH3 — CH— CH2 — CH2 — CH2OH
|
(i) CH3 MgI 4. Primary amine
D. CH3
CH2O → Product-4
H2 O
d. CH3 — C H — CH2 — O — CH2 — CH2OH
|
Codes CH3
A B C D 8. Rate of dehydration of alcohols follows the order
a. 1 2 4 3 a. 2° > 1° > CH3OH > 3°
b. 3 4 1 2
c. 1 2 3 4 b. 3° > 2° > 1° > CH3OH
d. 2 3 1 4 c. 2° > 3° > 1° > CH3OH
4. Place the following alcohols in decreasing order of d. CH3OH > 1° > 2° > 3°
rate of dehydration with conc. H2SO4 : 9. Consider the following reaction,
I. CH3 CH 2CH(OH)CH 2CH 2CH 3 K 2Cr 2O7
Oxidation
II. (CH 3 ) 2 C(OH)CH 2CH 2CH 3 A → Acetone → Acetic acid
H2SO4
III. (CH 3 )2 C(OH)CH(CH 3 )2
IV. CH 3 CH 2CH(OH)CH(CH 3 )2 Identify A in the above reaction.
a. 1-propanol b. 2-propanol
V. CH 3 CH 2CH 2CH 2CH 2CH 2OH
c. 2-butanol d. Ethanol
a. III > II > IV > V > I
b. III > II > IV > I > V 10. The alcohols, 1-propanol and 2-propanol are
c. III > II > I > IV > V distinguished by
a. oxidation with conc. H2SO4 followed by reaction with
d. III > II > I > V > IV
Fehling's solution
5. A compound with molecular formula C4 H10O3 is b. oxidation with alk. KMnO4 followed by reaction
converted by the action of acetyl chloride to a with Fehling's solution
compound of molecular mass 190. The original c. reaction with iodine and aq. NaOH
compound (C4 H10O3 ) has d. reaction with Grignard reagent

@iitjeehelps
672 SELF STUDY GUIDE BITSAT

11. The order for the acidic strength of 1°, 2°, 3° alcohols, (i) NaOH H+ / H2O Ac O
H 2O and R C ≡≡ CH is 17. Phenol → A → B →
2
C
(ii) CO2 /140° C
a. R C ≡≡ CH > 3 ° > 2 ° > 1 ° > H2O
In this reaction, the end product C is
b. 1 ° > 2 ° > 3 ° > H2O > R C ≡≡ CH
a. salicylaldehyde b. salicylic acid
c. H2O > 1 ° > 2 ° > 3 ° > R C ≡≡ CH c. phenyl acetate d. aspirin
d. 3 ° > 2 ° > 1 ° > H2O > R C ≡≡ CH
18. Phenol on heating with CHCl3 and NaOH gives
12. Identify the reaction conditions for the following salicylaldehyde. This reaction is called
reaction to occur. a. Reimer-Tiemann reaction
I b. Claisen reaction
→ CH 2 == CH 2
c. Cannizzaro reaction
CH 3 CH 2OH 
II d. Hell-Volhard-Zelinsky reaction
→ C2H 5 OC2H 5
a. I → H2SO4, 443 K II → H2SO4, 443 K
19. Identify Y + Z in the following reaction,
OH
b. I → H2SO4, 413 K II → H2SO4, 413 K
CHCl3
c. I → H2SO4, 443 K II → H2SO4, 413 K X
50% KOH
Y+Z
KOH
d. I → H2SO4, 413 K II → H2SO4, 443 K

13. Which of the following reagents cannot be used to OH OH


distinguish between phenol and benzyl alcohol? CH
a. Br2 / CCl4 b. NaOH a.
c. NaHCO3 d. FeCl3 (neutral)
OH
14. For the reaction, Phenol → Cyclohexanol, the s : p CH COOK
ratio of the carbon attached to — OH group changes b.
from … to … .
a.  :   1 3 b.  :   : 
1 2 1 2 1 1 OH
 :  OH
 3 3  4 4  3 3  2 2
CH2OH COOK
 1 1  1 2
c.  :   :  d. None of these c. +
 2 2  3 3
CH2OH COOK
15. In which of the following reactions, phenol is not
obtained? d. +
OH
NaOH/CaO
a. 20. Consider the following reaction,
COOH OH
+ CHCl3 + NaOH
MgBr –
O Na+
H3O+
b.
CHO
The electrophile involved in the above reaction is
Cl +
NaOH a. dichloromethyl cation, CHCl2
c. +
633 K, 300 atm
COOH b. formyl cation, CHO

+ – c. trichloromethyl anion, CCl3
N2Cl d. dichlorocarbene, : CCl2
H2 O 21. Consider the following compounds.
d.
∆ OH OH
NO2
16. Sodium phenoxide reacts with CO2 at 400 K and
4.7 atm pressure to give
B
a. catechol b. salicylaldehyde NO2
c. sodium salicylate d. benzoic acid A

@iitjeehelps
ALCOHOLS, PHENOLS AND ETHERS 673
Among the compounds A and B, the vapour pressure OH OH
of B at a particular temperature is
a. higher than that of A NO2
b. lower than that of A a. b.
c. same as that of A
d. None of the above NO2
OH
22. Consider the following reaction,
(i) Hg (OAc)2 / THF-H2O
CH==CHCH3 c. d. Both (a) and (c)
(ii) NaBH2

The major product formed in the above reaction is


NO2

a. —CH2CH2CH2OH CH CH — CH3

b. —CH2CHOHCH3
25. + HBr ?

c. —CHOHCH2CH3
OH
d. HO— —CH==CHCH3 What will be the product formed in the above
reaction?
Br
23. Which is/are the correct method(s) to synthesise
CH2 — CH — CH3 CH — CH2 — CH3
phenol?
Br
Cl O– Na+ OH a. b.

623 K HCl
a. + NaOH
300 atm OH OH
Br Br
SO3H OH
CH2 — CH — CH3 CH — CH2CH3
Oleum (i) NaOH
b. r
(ii) H c. d.
+ –
NH2 N2Cl OH
Br Br
NaNO2 + HCl H2O warm 26.
c. OH
400 K
+ CH2I2 + NaOH
+ N2 + HCl
OH
d. All of the above
In the given reaction, the product is
CH3I OCH3
OH
a. b.
ONa OH
24. + (dil.) HNO3 ?
O
c. O d.
Select the product(s) formed in the above reaction.
O

@iitjeehelps
674 SELF STUDY GUIDE BITSAT

OH The products formed in the above reaction are


a. OH and CH3Br b. Br and CH3Br
27. + FeCl3 Violet coloured complex
c. OCH3 and H2 d. Br and CH3OH
The formula of the complex formed in the above
reaction, is
33. Anisole on reaction with HI forms
a. [Fe(OC6H 5 )3] 3− b. [Fe(OC6H 5 )3] 3+
a. benzene and methyl iodide
c. [Fe(OC6H 5 )6] 3− d. [Fe(OC6H 5 )6] 3+ b. phenol and methanol
c. iodobenzene and methanol
28. The main product of the following reaction is
d. phenol and methyl iodide
Conc. H2SO4
C6H5 CH2CH(OH)CH(CH3 )2 → ? 34. The reaction of H 2C  CH2 with H 2 O/ R  MgX
‚ƒ
H 5C6 H O
‚ ƒ
a. C == C produces
ƒ ‚
H CH(CH3 )2 a. R CHOHR
C6 H 5CH2 CH3
‚ ƒ b. R CHOHCH3
b. C == C
ƒ ‚ c. R2CHCH2OH
H CH3
d. R CH2CH2OH
H 5C6 CH(CH3 )2
‚ ƒ
c. C == C 35. Assertion (A) The cleavage of C  O bond in
ƒ ‚
H H ethers takes place under drastic condition with
H 5C 6CH2CH2 excess of hydrogen halides.
‚ Reason (R) Ethers are the most reactive among all
d. C ==CH2
H3C
ƒ the functional groups.
a. Both A and R are correct and R is the correct
29. Glycerol on treatment with excess HI gives explanation of A
a. 1, 2, 3-triiodopropane b. 1, 3-diiodopropane b. Both A and R are correct but R is not the correct
c. 2-iodopropane d. 3-iodopropane explanation of A
c. A is correct but R is incorrect
30. Etherates are d. R is correct but A is incorrect
a. ethers
b. solution in ethers 36. OR
c. complexes of ethers with Lewis acid
d. complexes of ethers with Lewis base
O + HX
||
31. Compound Ph  O  C  Ph can be prepared by the
reaction of In the above reaction, the products are
a. phenol and benzoic acid in the presence of NaOH OH H
b. phenol and benzoyl chloride in the presence of
pyridine
c. phenol and benzoyl chloride in the presence of ZnCl2 a. + RX b. + RX
d. phenol and benzaldehyde in the presence of
palladium X H
32. Consider the following reaction,
HBr c. + R OH d. + R OX
— OCH3

@iitjeehelps
ALCOHOLS, PHENOLS AND ETHERS 675

BITSAT Archives
1. What will be the product of the following reaction? OH OH
H2O2
[2014] 6. –
OH
CH3
| CHO OH [2011]
H3C  C  Br + Na  O  CH3 → This reaction is called
| a. Reimer-Tiemann reaction
CH3 b. Liebermann’s nitroso reaction
CH3 c. Dakin reaction
| d. Lederer-Manasse reaction
a. CH3  C == CH2 b. CH3  O  C  CH3
| | 7. 3-methyl-2-butanol on treatment with HCl gives
CH3 CH3 (major product) [2010]
a. 2-chloro-2-methyl butane
c. CH3  CH2  CH2  CH3 d. CH3  CH CH3 b. 2-chloro-3-methyl butane
| c. 2, 2-dimethyl pentane
CH3 d. None of the above
2. Identify the correct product formed during the 8. Phenol can be tested by [2010]
following reaction: [2014] a. Liebermann’s nitroso test
CH3 b. FeCl3 solution
Conc. H2SO4 c. bromine water
CH3 A d. All of the above
OHOH 9. Phenol reacts with PCl5 to give mainly [2009]
CH3 CH3 a. p-chlorophenol
a. b. b. chlorobenzene
O CH3 CH3
c. o and p-chlorophenol
d. triphenyl phosphate
CH3 CH3
c. d. CH3 ƒ
C CH3 10. The IUPAC name of C2H5 — O— CH is
CH3 ‚
O O CH3
a. ethoxy propane [2008]
3. When 2-methyl propan-2-ol is treated with a mixture of b. 1,1-dimethyl ether
conc. HCl and ZnCl2, turbidity appears immediately c. 2-ethoxy isopropane
due to the formation of [2013] d. 2-ethoxy propane
a. 2-methyl propane
11. The compound which gives the most stable
b. 2-methyl propene
carbonium ion on dehydration is [2007]
c. 2-methyl-2-chloropropane
d. 2-chlorobutane a. CH3CH(CH3 )CH2OH
b. (CH3 )3 COH
4. What will be the correct relation between products
when 2-methyl cyclohexene is treated with (i) B 2H6 in c. CH2 ==CHCH2CH2OH
the presence of H 2O2 /OH − and (ii) H2O/H2SO4 ? d. CH3CHOHCH2 — CH3
(Also consider stereochemistry of product) [2013] 12. Pinacol is [2006]
a. They are metamers a. 3-methylbutan-2-ol
b. They are tautomers b. 2, 3-dimethyl-2, 3-butanediol
c. They are functional isomers c. 2, 3-dimethyl-2-propanone
d. They are positional isomers d. None of the above
5. The following reaction is known as [2012] 13. Grignard reagent reacts with HCHO to produce a/an
ONa OH OH a. secondary alcohol [2005]
COONa COOH b. anhydride
120-140°C H2O c. acid
+ CO2
1.5 atm d. primary alcohol

a. Friedel-Crafts reaction 14. The product obtained by heating diethyl ether with HI
b. Kolbe reaction is [2005]
c. Reimer-Tiemann reaction a. C2H5I b. C2H5OH
d. Wittig reaction c. C2H5OH + C2H5I d. C2H5 — C2H5

@iitjeehelps
Answer with Solutions
Practice Exercise CH3
|
1. (b) Since, (C ) and (D ) are isomers, D is CH3— C == O. On the
Conc.
COOH + CH3OH basis of C and D, we can get product B and finally A.
H2SO4
OH
OH | Cu / ∆
COOCH3 + H2O CH3 — C— CH2 — CH2 — CH3 →
|
OH CH3
A
2. (c) Tertiary alcohol offers resistance to oxidation. Hence, CH3
Cr 3+ ions are not formed and the solution does not turn |
green. CH3 — C == CH— CH2 — CH3
B
3. (a) Product-1: acetaldehyde (functional isomer of
epoxide), 8. (b) The order of dehydration of alcohols is
Product-2:s -butyl alcohol, 3 ° > 2 ° > 1 ° > CH3OH
Product-3 : ethylamine and 9. (b) The given reaction can be completed as
K 2 Cr2 O7 [O]
Product-4: ethyl alcohol. CH3  CH  CH3 → CH3  C  CH3 →
H2 SO 4
4. (b) The alcohols (II) and (III) both are 3°, but alcohol (III)  
gives a more substituted alkene. Alcohol (IV) and (I) both OH O
2-propanol
are 2°, but alcohol (IV) can give a more substituted alkene
and alcohol (V) is 1°. Rate of dehydration of alcohols with CH3  COOH
conc. H2SO4 follows the order ∆
10. (c) CH3  CH2  CH2OH + I2 + NaOH → No reaction
3° > 2° > 1° Propanol
5. (b) In acetylation, replacement of H of O—H occurs by ∆
CH3  CH  CH3 + 4I2 + 6NaOH →
CH3CO — group.

O OH
 Propanol-2
—O — H + Cl — C — CH3 → — OCOCH3 + HCl CHI 3 ↓ + CH3COONa + 5 NaI + 5H2O
i.e. H-atom of mass 1 amu is lost and an acetyl group of 11. (c) H2O > 1° > 2° > 3° > R C ≡≡ CH
mass 43 amu is added. Thus, there is a net gain of H2SO4
43 − 1 = 42 amu for every acetyl group introduced. Mass of → CH2 == CH2
443 K
difference of final product and original compound is 12. (c) CH CH OH 
3 2
H2SO4
190 − 106 = 84. Hence, number of — OH groups → C2H5OC2H5
413 K
84
= =2 13. (c)
42
(a) (b) and (d) all react with phenol and none of them
6. (a) With the increase in molecular mass, the boiling points reacts with benzyl alcohol. But (c) does not react with
also increase. Amongst isomeric alcohols, the boiling either phenol or benzyl alcohol.
point decreases with branching due to decrease in 14. (a) Carbon changes its hybridisation state fromsp 2 tosp 3,
surface area.
 1 2  1 3
108 i.e.  :  to  :  .
7. (b) Molecular mass of silver salt of E = × 100 = 181  3 3  4 4
59.6
RCOOAg = 181 H O+
15. (b) C6H5MgBr →
3
C6H6
R = 181 − (108 + 12 + 32) = 181 − 152 = 29 16. (c) ONa ONa
Hence, acid salt is CH3 — CH2 — COOAg.
Corresponding to this salt, acid is CH3 — CH2 — COOH , NaOH, 400 K COONa
+ CO2 4-7 atm pressure
which is obtained by oxidation of (C ), an ozonolysis
product. Thus, (C ) is CH3 — CH2 — CH == O. Sodium salicylate

@iitjeehelps
ALCOHOLS, PHENOLS AND ETHERS 677
17. (d) OH ONa α β
CH CHCH3
(i) NaOH COONa

(ii) CO2 /140°C H
25. (b)
A
OH OCOCH3
OH
+ COOH
H /H2O COOH Ac2O

Br
B (Aspirin)
Acetyl salicylic acid
CH — CH2 — CH3
18. (a) OH OH
CHO
+ CHCl3+ 3NaOH + 3NaCl +2H2O
Chloroform
Salicylaldehyde
OH
Phenol
This reaction is called Reimer-Tiemann reaction. 26. (d )
– +
19. (c) OH OH OH O Na
NaOH CH2I2
CHO
–2NaI
+ CHCl3 + KOH(aq) OH – +
O Na
X O
OH OH
CH2OH COOK O
50%
+
aq. KOH 27. (c) Fe(OC6H5 )6] 3–
Y Z
20. (d) CHCl3 + OH → CCl3 + H2O CH3 Conc. H2SO4
(Unstable) 28. (c) —CH2—CH—HC
–H2O
CCl3 → •
+ Cl− CH3
• CCl 2 OH
Dichlorocarbene
(electrophile)

21. (a) In o-nitrophenol, intramolecular H-bonding is present


between OH and NO2 groups, whereas p-nitrophenol Loss of 2(H) is Loss of
forms intermolecular H-bonding. Thus, vapour pressure H preferred because 1 (H) is
of stability less
of B is higher than that of A. preferred
22. (c) (i) Hg(OAc)2 / THF-H2O H CH(CH3)2
—CH==CHCH3 trans (Major)
(ii) NaBH4 + CH3
—CH2
CH(CH3)2
—CH—CH2CH3
H CH3
OH H H (Minor)
cis (Minor)
23. (d) All the given methods are correct.
24. (d) Nitration of phenol with dilute nitric acid at low CH2OH CH2I CH2
temperature (298 K), phenol yields a mixture of ortho and  +3HI
 −I 2

 HI
para-nitrophenol. 29. (c) CHOH → CHI → C H →
  
OH OH OH
CH2OH CH2I CH2I
NO2
Dil. HNO3 CH3 CH3 CH3
298 K
+  –I 2  HI

CHI → C H → CHI
o-nitrophenol   
NO2 CH2I CH2 CH3
p-nitrophenol 2-iodopropane

@iitjeehelps
678 SELF STUDY GUIDE BITSAT

30. (c) Etherates are the complexes of ethers with Lewis acid. 33. (d) OCH3 OH

R — O — R + BF3 →

O → BF3
..
HI
Ether Lewis acid ƒ + CH3I
R
Etherates
R  Mg X
31. (b) O 34. (d) H2C  CH2 → R  CH2CH2  OH + MgX (OH)
‚ƒ H2O
Pyridine O
OH + Cl — C —
35. (c) Ethers are the least reactive among all the functional
O groups. The cleavage of C—O bond in ethers takes place
under drastic conditions with excess of hydrogen halides.
O—C—
36. (a) OR OH
32. (a) The complete reaction is as follows
HBr + HX + RX
—O—CH3 —OH + CH3Br

BITSAT Archives
1. (a) When tertiary alkyl halide is treated with sodium 5. (b) At 120-140°C temperature and 1.5 atm pressure,
alkoxide then elimination reaction completes over sodium phenoxide reacts with CO2 to yield sodium
substitution reaction because alkoxides are not only salicylate which on further hydrolysis give to salicylic acid.
nucleophiles but strong base as well. Therefore, alkenes This reaction is known as Kolbe’s reaction.
are formed instead of ethers. 6. (c) The reduction of  CHO group to  OH by H2O2/OH −
2. (d) is called Dakin reaction.
CH3 CH3
H2SO4 OH OH
CH3 CH3 H2O2

OH OH2 OH–
OH OH
Pinacol r CHO OH
O—H
CH3 CH3 CH3

r CH3 r CH3 7. (a) CH3  CH  CH  CH3 + HCl →
O—H 
O CH3 OH
+
CH3 CH3
–H
CH3 
CH3 CH3  CH  CH2  CH3 + H2O
Relief from angle strain O 
Pinacolone
Cl
Basis of carbocationic rearrangement is due to relief from 2-chloro-2-methyl butane
angle strain.
8. (d) Phenol can be tested by all the given reagents.
3. (c) When 2-methyl propan-1-ol is treated with a mixture of
Nucleophilic attack
conc. HCl and ZnCl2 (Lucas reagent) then tert-alkyl halide 9. (d) 3C6H5OH + PCl5 → (C6H5O)3 PCl2
is formed and produced turbidity due to its less solubility. –3HCl
H2O
4. (d)
H H  H  → (C6H5O)3 PO
– 2HCl Triphenyl phosphate
+ BH3 BH 2 1
(i)
CH3
CH3  CH3 3
B 2 ƒC H3
10. (d) C2H5 — O — C H 3
‚C H
H2O2/OH– OH 3
H3BO3 or B(OH)3 + 3 2-ethoxy propane
CH3
+ H+
(A) 11. (b) (CH3 )3 C — OH → (CH3 )3 C+
Tertiary alcohol − H2O 3 ° carbocation
(ii) H (more stable)
OH
+ H2SO4/H2O 12. (b)
H O+
CH3 13. (d) HCHO + R MgX →
3
R CH2OH
B (1° alcohol)
A and B have difference in position of OH only so, A and B HI
are positional isomers. 14. (c) C2H5 — O — C2H5 → C2H5 — OH + C2H5 — I
100 ° C

@iitjeehelps
26
Aldehydes
and Ketones

Carbonyl Compounds
Carbonyl compounds contain carbon-oxygen double bond, C == O.
These are divided into three classes :

Carbonyl Compounds
¯
¯ ¯ ¯
Aldehydes Ketones Carboxylic Acids
(i) (ii) (iii)
Carbonyl group is linked Carbonyl group is linked to Carbonyl group is linked to
to alkyl or aryl group and two alkyl or aryl groups or O-atom of hydroxyl group
one H-atom RCHO. one alkyl and aryl group RCOOH.
RCOR.

‚
Nature of C == O Group
ƒ
The C == O group is highly polar in nature due to the high electronegativity of oxygen atom, so that,
C-atom acquires partially positive charge and O-atom acquires partially negative charge.
The two canonical forms of carbonyl group are:

+ –
C O C O

@iitjeehelps
680 SELF STUDY GUIDE BITSAT

Preparation of Aliphatic and Aromatic Wacker’s process [Oxidation of alkenes with


PdCl 2 /CuCl 2 ]
Aldehydes and Ketones
The reaction is as follows:
The general methods for the preparation for aldehydes and O2
ketones are as follows: CH 2 == CH 2 + H 2O + PdCl 2 ¾¾®
CuCl 2

Aldehydes and Ketones CH3CHO + Pd + 2HCl


O
These methods are as follows : CH3CH == CH 2 + H 2O + PdCl 2 ¾¾®
2
CuCl 2
(i) From alcohols The controlled oxidation of primary
O
alcohols, RCH 2OH yields aldehydes while secondary ||
alcohols yield ketones. CH3 ¾ C ¾ CH3 + Pd + 2HCl
R K 2Cr 2O 7 R
CHOH ¾¾¾¾® C == O + H 2O (b) Alkynes
[2 °] H 2 SO4 (dil.) Ketone
R Alcohol R Carbonyl compounds can be prepared from alkynes
(ii) From carboxylic acids in the following ways:
(a) Dry distillation of calcium salts of fatty acids Dry Hydration of alkynes The reaction works as
distillation of calcium formate yields formaldehyde CH
1% HgSO 4
only. ½½½ + H 2O ¾¾¾¾¾¾® CH3CHO
42% H 2 SO 4 , 333 K
D CH
(HCOO)2 Ca ¾® HCHO+ CaCO3
(iv) From Grignard’s reagents
Dry distillation of calcium formate with calcium
salts of fatty acids yields other aldehydes. (a) Cyanides Addition of RMg X on HCN yields
D aldehydes.
(HCOO)2 Ca + ( RCOO)2 Ca ¾® 2 RCHO+2CaCO3
+ s r H 2O
Pyrolysis of a mixture of calcium salt of different HC

N + R  Mg X HC NMgX 
NH4 
+
fatty acids yields mixed ketones.

D
R′
( R ¢ COO)2 Ca + ( R ¢COO)2 Ca ¾®
HC O + Mg(OH)X + H2O+NH3
O
½½ R
R — C— R ¢ + 2CaCO3
Addition of R¢ MgX on RCN gives ketones.
(b) Catalytic decomposition of acids Reaction of acids
with formic acid yields aldehyde. Preparation of Aldehydes Only
373 K
RCOOH + HCOOH ¾¾® RCHO + CO2 + H 2O (i) From acyl chloride
MnO
O
Reaction of two molecules of acid yields ketones. ½½ H2
R R ¾ C ¾ X ¾¾¾® RCHO + HX
MnO Pd / BaSO 4
RCOOH + HOOCR¢ ¾¾® C == O + CO2 + H 2O
3 73 K (R = alkyl group or C 6H 5)

This reaction is called Rosenmund reduction.
(iii) From hydrocarbons Aldehydes and ketones can
(ii) Stephen reaction
be prepared from alkenes and alkynes.
R ¾ CN +SnCl2 +HCl ¾® RCH==NH × HC l
(a) Alkenes Iminochloride
Carbonyl compounds can be prepared from alkenes H 3O +
¾¾® RCHO+ NH 4Cl + H 2O
in the following ways:
Reductive ozonolysis of alkenes (iii) By side chain chlorination followed by hydrolysis
R CH3 CHCl2
Cl /hυ
→
2
2 mol
O3 in O  CH Zn/H2O
RCH CH2 →
CHCl /CCl
O 

3 4 CHO
196 K O  CH2 RCHO + HCHO 2HO
→
373K
Ozonide

@iitjeehelps
ALDEHYDES AND KETONES 681
(iv) Etard reaction
CH3
CS2
Physical Properties of
→
+ CrO2Cl2
Chromyl
Aldehydes and Ketones
chloride (i) Boiling points Boiling points of aldehydes and
CH(OCrOHCl2)2 ketones are higher than that of hydrocarbons and
+
CHO ethers but lower than alcohols of comparable
HO
→
3
molecular masses. This is because of weak
Chromium intermolecular interactions due to dipole-dipole
complex Benzaldehyde interactions and absence of H-bonding.
(ii) Solubility Solubility decreases with increase in
(v) By oxidation of methyl benzene molecular mass or size of alkyl group. Aldehydes and
CH3
ketones containing upto 4C-atoms are soluble in
273-283K water.
+ CrO3 + (CH3CO)2O

Chemical Reactions of Aldehydes


CH(OCOCH3)2 CHO
H3O
+
and Ketones

Due to the presence of polar C == O group, aldehydes and
ketones exhibits the following reactions:
(vi) By Gattermann-Koch reaction
CHO Nucleophilic Addition Reactions
CO,HCl
Anhyd. AlCl3 Carbonyl compounds undergo nucleophilic addition
reactions in which nucleophile attack electrophilic C-atom
of C == O group.
(vii) From diazonium salts
Nu d - - E d + ¾® Nu - + E +
+ −
N2 Cl CH NOH S tep I Step II OE
C== O + Nu - ¾¾® C ¾ O- ¾¾® C
–N2 ½ E+ Nu
+CH2 NOH –HCl Nu
Benzene In general, ketones are less reactive in nucleophilic addition
diazoniumchloride CHO
reactions as compared to aldehydes. This is because
H2O (i) Aldehydes have only one electron pair releasing group
while ketones have two.
(ii) Stearic hinderance is less in aldehydes compared to
ketones.
Preparation of Ketones Only (a) Addition of HCN The reaction looks like
(i) From acid chlorides OH
C O + H+  CN– → C
2 R Mg X + CdCl 2 ¾® R 2Cd + 2Mg( X )Cl CN
O O Cyanohydrin

½½ M - ½½
+ (b) Addition of NaHSO 3 The reaction looks like
2 R ¢— C —M Cl+ R 2Cd ¾® 2 R ¢— C — R + CdCl2
M OSO2H
O +Na+  HSO3 →

(ii) By Friedel-Crafts acylation C C
ONa
COR
O H
+ OSO2Na
AlCl3 (Anhyd.) R C
transfer OH
+R C Cl
Bisulphite addition product

@iitjeehelps
682 SELF STUDY GUIDE BITSAT

Nucleophilic Addition-Elimination Reaction (ii) Addition of PCl 5 Geminal dihalides are produced.
In these reactions, additions of nucleophile on electrophilic Cl
C-atom of carbonyl group leads to the elimination of C O + PCl5 → C + POCl3
Cl
O-atom in the form of water, H 2O.
Geminal dihalide
(i) Addition of alcohols Acetals and ketals are formed.
(iii) Addition of NH 3 and its derivatives[H 2N — Z ]
R sr R OH (Alcohol)
C O + R′OH → C
H H O R′ (Ether) OH
C O + H2N  Z r C
Hemiacetal NZ

R′O—H R OR′ H
+
C + H2O → C N  Z + H 2O
H H OR′
Acetal
(stable)
Here, Z may be
R HO H+ R O O
C O+ r C + H2O ||
R HO R O ¾ H,OH, ¾ NH 2 ¾ NHC6H 5, ¾ NH ¾ C ¾ NH 2 .
Cyclic ketal

Some N-Substituted Derivatives of Aldehydes and Ketones ( C == N ¾ Z)

Z Reagent name Carbonyl derivative Product name

—H Ammonia Imine
C == NH

—R Amine Substituted imine


C == NR
(Schiff’s base)

—OH Hydroxyl amine Oxime


C == N — OH

—NH 2 Hydrazine Hydrazone


C == N — NH 2

—HN— C N  NH
Phenyl hydrazine Phenyl hydrazone

O2N O2N

—HN— —NO2 2,4-dinitrophenyl hydrazine C N  NH  NO2 2,4-dinitrophenyl hydrazone

O
½½ O
— NH — C — NH 2 Semicarbazide ½½ Semicarbazone
C == N — NH — C — NH 2

@iitjeehelps
ALDEHYDES AND KETONES 683
Reduction (iii) Oxidation with sodium hypohalite (NaOX) or
Aldehydes and ketones can be reduced to alcohols and haloform test This reaction is given by carbonyl
alkanes. O
||
(i) Reduction to alcohols compounds containing ¾ CH3 ¾ C ¾ group that is
LiAlH 4 / NaBH 4 acetaldehyde or methyl ketone only.
C == O ¾¾¾¾¾® CHOH
O O
Note Reduction of ketones by aluminium isopropoxide || ||
NaOX
[Meerwein-Ponndrof-Verley’s (MPV) reduction] R ¾ C ¾ CH3 ¾¾® R ¾ C ¾ ONa + CHX3
R R
Al[OCH(CH3 ) 2 ] 3 [ X = Cl, Br, I]
C == O ¾¾¾¾¾¾® CHOH
(2 ° alcohol)
R R Reactions Due to a-H-atom
(ii) Reduction to alkanes The reducing agents may be a -H-atom of aldehydes and ketones is highly acidic in
(a) Zn—Hg/HCl (Clemmensen’s reduction) nature due to the presence of electron withdrawing effect of
Zn —Hg / HCl
carbonyl group.
C == O ¾¾¾¾¾® CH 2 + H 2O
(i) Aldol condensation Aldehydes and ketones
(b) NH 2 ¾ NH 2 /KOH (Wolff-Kishner reduction) possessing atleast one a -H -atom reacts in the presence
NH 2 ¾ NH 2 of dilute alkali [like NaOH as catalyst] to form
C == O ¾¾¾¾® CH 2 + H 2O + N 2 b-hydroxy aldehydes (aldol)/b-hydroxyketones (ketol)
KOH / D
respectively. Further, aldol and ketol condenses to give
Oxidation a , b -unsaturated carbonyl compounds. This reaction is
(i) Oxidation of aldehydes called aldol condensation.
RCHO + 2[Ag(NH3 )2+ ] + 3OH - ¾® O H O
Tollen’ s reagent OH–
CH3  C H + CH2  C  H →

RCOO- + 2H 2O + 4NH3 + 2Ag ¯ OH O


Silver
–H2O
mirror
CH3  CH  CH2  C  H ∆
The above reaction is called Silver mirror test and used Aldol O
as a distinguish test for aldehydes and ketones.
CH3  CH CH  C  H
R¾ CHO + 2Cu(OH)2 + NaOH ¾® α, β-unsaturated aldehyde
Fehling’ s
solution
The above reaction is called Self aldol condensation.
RCOONa + Cu 2O ¯ +3H 2O
Red ppt (ii) Cross aldol condensation When aldol condensation
is carried out between two different aldehydes/ketones,
R¾ CHO + Cu 2 + (complex) ¾® it is called cross aldol condensation.
Benedict’ s
solution Dil.
CH3CHO + CH3CH2CHO NaOH
Cu 2O ¯ + Oxidation products Ethanal Propanal
Red ppt
OH OH CH3
(ii) Oxidation of ketones By the help of strong oxidising
CH3CH CH2CHO + CH3CH2  CH CH CHO
agents like conc. HNO3 / KMnO4 / H 2SO4 / K 2Cr2O7 3-hydroxybutanal 3-hydroxy-2-methylpentanal
/ H 2SO4 , their oxidation involves carbon-carbon bond
cleavage. Self aldol products
O
|| OH CH3 OH
[O]
CH3 ¾ C ¾ CH3 ¾¾¾¾® 2CH3COOH
Conc. HNO3 CH3CH CHCHO + CH3CH2CH CH2CHO
3-hydroxy-2-methyl 3-hydroxy pentanal
For unsymmetrical ketones, the cleavage takes place in butanal
such a way that keto group remains with smaller alkyl
group (popoff’s rule). Cross aldol product

@iitjeehelps
684 SELF STUDY GUIDE BITSAT

CHO
Chemical Reactions of 1. Nitration
Aldehydes only conc. HNO3/H2SO4

(i) Cannizzaro reaction This reaction is shown NO2


by those aldehydes which lacks a-H-atom and CHO CHO
undergo self oxidation-reduction reaction 2. Sulphonation
conc. H2SO4
(disproportionation).
NaOH
2HCHO ¾¾® CH3OH + HCOONa SO3H
CHO
NaOH
2C6H 5CHO ¾¾® C6H 5CH 2OH + C6H 5COONa 3. Halogenation
Cl2/FeCl3

Cross-Cannizzaro reaction Cl
NaOH(50%)
CHO + HCHO (ii) Claisen-Schmidt reaction The reaction looks like

OH−
CHO + CH3CHO CH CHCHO
CH2OH + HCOONa
OH−
CHO + CH3COCH3 CH CHCOCH3
HCHO lacks steric hinderance. Hence, it readily
oxidises acid to salt.
(iii) Perkin condensation [For aromatic aldehydes only]
(ii) Tischenko reaction The reaction looks like
CHO + CH3CO CH3COO−Na+
Al (OC 2 H 5 )3
2CH3CHO ¾¾¾¾¾® O 180°C
CH3CO
[CH3COOH + CH3CH 2OH]
-H O CH CHCOOH + CH3COOH
¾ ¾¾
2
® CH3COOCH 2CH3
Cinnamic acid

Electrophilic Substitution (iv) Benzylic acid rearrangement [For aromatic ketones]


The reaction is given by only diketones.
Reaction Reduced
(i) Electrophilic substitution reaction due to O O OH O
benzene ring Aromatic aldehydes undergo OH

CC  C  C  OH
electrophilic substitution reactions rarely due to H2O
the presence of deactivating carbonyl group.
This is m-directing group as substituent directed
to m-position.
Oxidised

@iitjeehelps
Practice Exercise
1. Ozonide of CH 2 ==CH — CH2OH on hydrolysis gives 6. The compound formed as a result of oxidation of ethyl
a. HCHO, OHC — CHO benzene by KMnO4 is
b. HCHO, HOCH2 — CHO a. benzophenone b. acetophenone
c. HCHO, HOC — CH2OH c. benzoic acid d. benzyl alcohol
d. None of the above 7. Choose the Gattermann-Koch reaction.
2. Dry distillation of calcium acetate and calcium formate CHO
leads to the formation of …… aldehydes and …
ketones CO + HCl
a. 2, 1 a.
Anhyd. AlCl3 /CuCl
b. 1, 2
c. 2, 2
CH3 CHCl2 CHO
d. 1, 1
3. In the following sequence of reactions, the final product H 2O
is b. + Cl2/hν
373 K
1. Br2 / Fe HCHO Cl2 / Fe CrO3
C6H6 ¾¾¾® (I) ¾¾® (II) ¾¾® (III) ¾¾¾® (IV)
2. Mg/ ether Pyridine CH3 CH(OCOCH3)2
a. p-chlorobenzaldehyde
b. p-chlorobenzylalcohol
273-278 K
c. p-chlorobenzoic acid c. + CrO3 +(CH3CO)2O
d. p-salicylaldehyde
4. Identify the product (Y ) in the following reaction H3 O ∆
sequence:
OMe O CHO

1. AlCl3
+ O (X )
2. H3O+
d. None of the above
O O
MeO 40% H 2 SO4 Isomerisation
Zn(Hg)/HCl H3PO4 8. CH3 — C ºº CH ¾¾¾¾® A ¾¾¾¾®
1% HgSO4
Y
CH3 — C— CH3
½½
a. MeO (CH2)3 COOH O
b. MeO (CH2)2 COOH Structure of A and type of isomerism in the above
c. MeO CH2COOH
reaction respectively are
a. prop-1-en-2-ol, metamerism
d. MeO COOH b. prop-1-en-1-ol, tautomerism
c. prop-2-en-2-ol, geometrical isomerism
5. Consider the following reaction, d. prop-1-en-2-ol, tautomerism
O
CHC6H5 (i) O3 9. Consider the following reaction,
+X
(ii) Zn/H2O Hg2 + / H +
Ph — C ºº C — CH3 ¾¾¾® A
The compounds X is A is O
CHO CHO Ph Ph
a. b. a. b. O

H3C H3C
OH COOH Ph Ph
c. d. c. OH d. O

H 3C CH2CH3

@iitjeehelps
686 SELF STUDY GUIDE BITSAT

10. Which of the substrate give the same product on the 18. Consider the following reaction,
reduction with DIBAL-H? Pyridine
a. CH3 — (CH2 )9 — CN and CH3(CH2 )9 — COOH CH3 CHO + CH2(COOH)2 ¾¾® A
D
b. CH3 — (CH2 )9 — CN and CH3(CH2 )9 — COOC2H 5
c. CH3 — (CH2 )9 — COOH and CH3(CH2 )9 — CHO A is
d. CH3(CH2 )9 — COOH and CH3(CH2 )9 COOC2H 5 a. CH3COOH
b. CH3CH2COOH
11. Identify the starting material from which the products c. CH3CH == CHCOOH
H 3 CCH== C ¾ CHO and d. HOOC — CH == CH — COOH
½
CH 3 19. In the aldol condensation of acetaldehyde and
acetone in dilute alkali, the carbanion source will be
H 3 C — CH 2CH==CH—CHO are formed? a. acetaldehyde
a. Two molecules of ethanal b. acetone
b. Two molecules of propanal c. Both (a) and (b)
c. One molecule of ethanal and two molecules of d. None of the above
propanal
20. For distinct between CH 3 CHO and C6H5 CHO, the
d. One molecule of propanal and one molecule of
ethanal used reagent is
a. KCN b. HCN c. NH2OH d. PCl5
12. Both HCHO and CH3 CHO give similar reactions with
all the reagents except 21. Oxidation of ketones with H 2O2 or with a peroxy acid
a. Schiff’s reagent b. ammoniacal AgNO3 is called Baeyer-Villiger oxidation. This oxidation
c. Fehling solution d. ammonia reaction forms
a. carboxylic acid with the fewer number of carbons
13. Which of the following carbonyl compounds gives
b. an alcohol with the same number of carbons as in the
lactic acid as the end product in the following ketone
sequence? c. an ester
HCN H 3 O+ d. carboxylic acid with the same number of carbons as
A ¾® B ¾® C in the parent ketone
a. HCHO b. CH3CHO 22. The order of reactivity of phenyl magnesium bromide
c. C6H5CHO d. CH3COCH3 with the following compounds is
14. Oximino acetone is formed in the reaction O O O
a. acetone + hydroxylamine b. acetone + ammonia
c. acetone + nitrous acid d. None of the above H3 C CH3 H 3C H Ph Ph
15. Which of the following is used to prepare a (I) (II) (III)
medicine, which is used in making an important a. II > III > I
explosive, RDX ? b. I > III > II
a. Acetaldehyde b. Acetone c. II > I > III
c. Formaldehyde d. None of these d. All react with the same rate
16. Which of the following reactions gives pentaerythritol? 23. The product of acid hydrolysis of P and Q can be
Ca(OH)2 distinguished by
a. CH3CHO + 4HCHO ¾¾®
NaOH OCOCH3 H 3C
b. CH3CH2CHO + 2HCHO ¾¾®
NaOH
H2C == ,
c. CH3 — CH — CHO + 2HCHO ¾¾® CH3 OCOCH3
½ P Q
CH3
NaOH a. Lucas reagent b. 2, 4-DNP
d. 2HCHO ¾¾® c. Fehling's solution d. NaHSO3
17. Consider the following reaction, 24. Consider the following Rosenmund reaction,
[O] Pd/ BaSO4
CH 3 — CH== CH — CHO ¾® R COCl + H2 ¾¾¾® R CHO + HCl
CH3 — CH== CH— COOH Here, BaSO4
The above reaction is completed by the reagent a. promotes catalytic activity of Pd
a. alkaline KMnO4 b. Tollen’s reagent b. removes the HCl formed in the reaction
c. selenium dioxide d. osmium tetraoxide c. deactivates palladium
d. activates palladium

@iitjeehelps
ALDEHYDES AND KETONES 687
25. Addition of water to alkynes occurs in acidic medium 29. Which of the following is the product of aldol
and in the presence of Hg2+ ions as a catalyst. Which condensation?
of the following products will be formed on addition of OH O
O
water to but-1-yne under these conditions?
O OH
a. b.
½½ O
a. CH3 ¾ CH2 ¾ CH2 ¾ C ¾ H
O O
½½ d.
c. HO
b. CH3 ¾ CH2 ¾ C ¾ CH3
OH
O
½½
c. CH3 ¾ CH2 ¾ C ¾ OH + CO2 30. Iodoform can be prepared from all, except
O O a. ethyl methyl ketone
½½ ½½ b. isopropyl alcohol
d. CH3 ¾ C ¾ OH + H¾ C ¾ H c. 3-methyl-2-butanone
d. isobutyl alcohol
26. An organic compound containing C, H and O gives
31. The Clemmensen reduction of ketones is carried out
red colouration with sodium nitroprusside solution but
in the presence of
does not reduce Tollen’s reagent and yields
a. Zn-Hg with HCl
chloroform on treating with NaOH and Cl2. The b. LiAlH4
compound is c. H2 and Pt as a catalyst
a. CH3CH2OH b. CH3 CH CH3 d. glycol with KOH
½
OH 32. Which one of the following aldehydes does not give
c. CH3COCH3 d. (CH3 ) 3 CH — CHO Cannizzaro’s reaction?
a. Formaldehyde b. Acetaldehyde
27. Maximum dehydration takes place in that of c. Trimethyl acetaldehyde d. Benzaldehyde
O O
33. When a , b-unsaturated carbonyl compounds undergo
OH
a ring closure reaction with conjugated dienes, the
a. b.
reaction is called
OH a. Claisen rearrangement
OH CH3 b. Diels Alder reaction
c. Cannizzaro reaction
d. Perkin reaction
c. d.
34. The most acidic H-atom in the following ester is
OH
O O
OH || ||
1 2 3 4 5
CH 3 CH 2 ¾ C ¾ CH 2 ¾ C ¾ O — CH 2 CH 3
28. With reference to the scheme given, which of the
given statements about T,U,V and W is incorrect? a. 1 b. 2
c. 3 d. 4
O
 35.
NH2 NH2
== O ¾¾¾®
KOH/ ethylene glycol
== NH2 ¾¾¾¾¾¾®
O LiAlH4 (excess) - H2 O heat

CH 2 + N 2
H3C CrO3/H+ (CH3CO)2O
(T) V U W
The above reaction is known as
a. T is soluble in hot aqueous NaOH a. Wolff-Kishner reduction
b. V is optically active b. Clemmensen’s reduction
c. Molecular formula of W is C10H18O4 c. Both (a) and (b)
d. V gives effervescence on treatment with aqueous d. None of the above
NaHCO3

@iitjeehelps
688 SELF STUDY GUIDE BITSAT

O
36. Identify the product formed in the following reaction:
a. b.
H CH3
CH3 + NaOCl c. d. O
H 3C
O
O
H CH3
a. C + NaCl 40. The compounds, benzaldehyde and acetone are
OH
H3 C distinguished by
O a. Fehling’s solution
H CH3 b. 2, 4-DNP
b. CH—HC + CHCl3 c. Tollen’s reagent
H3 C COONa d. sodium hydroxide solution
H CH3 41. Consider the following Cannizzaro reaction,
c. + CHCl3
H3 C COONa OH -
2Ph ¾ CHO ¾¾® Ph ¾ CH 2OH + PhCO-2
H CH3
d. CH—HC + NaCl In the above reaction, the slowest step of the reaction is
H3 C COOH a. the transfer of hydride ion to the carbonyl group
b. the deprotonation of Ph ¾ CH2OH
37. Consider the following reaction, c. the attack of OH - at the carbonyl group
Ba(O) 2 d. the abstraction of proton from the carboxylic acid
D
2CH 3 COCH 3 e X ¾¾® Y
– H 2O 42. An aromatic compound, A [C 5 H10O] undergoes
Identify X and Y in the given reaction. Cannizzaro reaction, forms 2, 4-DNP derivative,
reducesTollen’s reagent and produces
X Y
CH3 CH3
1, 2-benzenedicarboxylic acid on vigorous oxidation.
½ ½ The compound A would be
a. H3C — C == CH — COCH3 H3C — C — CH2COCH3 CHO C2H5 CHO
½ a. b.
OH
C 2H 5
CH3
½ O
b. H3C— C — CH2 — COCH3 H3C— C == CH— COCH3
CHO C  CH3
½ ½
OH CH3 c. d.
CH3 CH3 C 2H 5 CH3
½ ½
c. H3C — CH — CH — COCH3 H3C —C == C — COCH3 43. Consider the following series of reaction,
½ O
OH
OH C
½ Zn/Hg
A
d. H3C— C — CH2 — COCH3 HCl
C
½
CH3 HO O
H2C == C — CH2 — COCH3
½ SOCl2 AlCl3 LiAlH4
CH3 ¾¾¾® B ¾¾¾® C ¾¾¾® D
38. Cannizzaro’s reaction is not given by Conc. H2SO 4 NBS Alc. KOH
CHO ¾¾¾¾¾¾® E ¾¾® F ¾¾¾¾® G
D
a. CHO b.
The end product of the above series of reaction is
CH3
Br
c. CH3CHO d. HCHO a. b.
39. The product formed in the following reaction is
O
c. d.

OH
2 ∆
OH

@iitjeehelps
ALDEHYDES AND KETONES 689
44. The increasing order of the rate of HCN addition to c. X → CH3C C—CH CH—CH CH—CHO
COOH
compounds a-d is
(i) HCHO Y→
(ii) CH3COCH3
(iii) PhCOCH3 d. X → H3C—CH CH—C C—CH CH—CHO
(iv) PhCOPh COOH
a. (i) < (ii) < (iii) < (iv)
b. (iv) < (ii) < (iii) < (i) Y→
c. (iv) < (iii) < (ii) < (i)
d. (iii) < (iv) < (ii) < (i)
48. An aromatic compound X with molecular formula
45. Select the structure of chromium complex formed C9H10O gives the following chemical tests:
when the toluene reacts with chromyl chloride to give I. Forms 2, 4-DNP derivative
benzaldehyde on hydrolysis. II. Reduces Tollen’s reagent
CH(OCrCl3)2 CH2(OCrOHCl2)
III. Undergoes Cannizzaro reaction
a. b. IV. On vigorous oxidation, 1,2-benzene dicarboxylic acid
is obtained. X is
CHO
CH(OCrOHCl2)2 CH(OCrOH2Cl2) C 2H 5 CHO
c. d. a. b.

OCOCH3 C 2H 5
O CHO
Anhyd. AlCl3, CS2
46. ∆ C
CH3
c. d.
Predict the product (s) formed in the given reaction. CH3 C2 H5
OH OH
  49. Predict the product(s) formed in the following reaction:
COCH3
CHO
a. b.
NaOH (1 mol)

COCH3
CHO
c. Both (a) and (b) d. None of these
CHO
CHO CHO
47. An organic compound (x ) with molecular formula
C8H 8O forms an orange-red precipitate with 2,4-DNP
reagent and gives yellow precipitate on heating with a. b.
iodine in the presence of sodium hydroxide.
COOH COONa
It neither reduces Tollen’s or Fehling’s reagent, nor
does it decolourise bromine water or Baeyer’s CH2OH CH2OH
reagent. CHO COONa
On drastic oxidation with chromic acid, it gives a
carboxylic acid (Y ) having molecular formula C7H 6O2.
Identify the compounds X and Y. c. d.
O CH2OH CH2OH
COOH COONa CHO
CH3
a. X → Y→
50. Name the reaction which involves the conversion of
benzaldehyde to cinnamic acid in the presence of
CH2CHO COOH acetic anhydride.
b. X → Y→ a. Benzoin condensation b. Reformatsky reaction
c. Knoevenagel reaction d. Perkin’s reaction

@iitjeehelps
BITSAT Archives
1. How many chiral centres are possible for the product
of following reaction? [2014] O
O CHO
CH3 (i) Michael

 CH3
a. b.
+ addition
H – A CH3 CH3
(ii) OH
CH3 O CHO
CHO
a. 1 b. 0 c. d.
c. 3 d. 2
CHO
2. Arrange the following compounds in the increasing order
of nucleophilic addition reaction: [2014] 7. Which of the following is process used for the
I. HCHO preparation of acetone? [2012]
II. CH 3 COCH 3 a. Waber process
III. C6H5 COCH 3 b. Wacker process
c. Wolff-Kishner reduction
IV. C6H5 COC 6H5 d. Gattermann-Koch synthesis
a. I < II < III < IV b. IV < III < II < I
c. IV < II < III < I d. III < IV < II < I 8. What will be the main product when acetylene reacts
with hypochlorous acid? [2012]
3. Which of the following compounds will give positive a. Trichloro acetaldehyde b. Acetaldehye
iodoform test with I2 and NaOH? [2013] c. Dichloro acetaldehyde d. Chloro acetaldehyde
a. C6H5COC6H5 b. CH3CH2CHO
9. Which of the following reagents can be used to
c. C6H5COCH2CH3 d. C6H5 ¾ CH¾ CH3
| prepare benzaldehyde from toluene? [2009]
OH a. CrO3 in (CH3CO)2 O b. K 2Cr2O7 + conc. H2SO4
c. Hot alkaline KMnO4 d. Conc. HNO3
4. What will be the final product of the reaction? [2014]
10. Acetone on addition to methyl magnesium bromide
O3 OH
s forms a complex, which on decomposition with acid
A B
Zn/H2O ∆ gives X and Mg(OH)Br. Which one of the following is
X? [2008]
O
a. CH3OH b. (CH3 )3 COH
O c. (CH3 )2 CHOH d. CH3CH2OH
a. b. (i) X
11. CH 3 — C ºº CH3 ¾ CH3 ¾¾¾®
(ii) Zn/ H 2 O
O
O
CH 3 — C — C — CH3
½½ ½½
O O
c. d. In the above reaction, X is [2008]
a. HNO3 b. O2
O c. O3 d. KMnO4

5. The compound formed as a result of oxidation of 12. Which of the following ketones will not respond to
propyl benzene by KMnO4 is [2013]
iodoform test? [2007]
a. benzaldehyde a. Methyl isopropyl ketone b. Ethyl isopropyl ketone
b. benzyl alcohol c. Dimethyl ketone d. 2-hexanone
c. benzoic acid 13. Acetone and acetaldehyde can be distinguished by
d. acetophenone a. Molisch test b. Tollen’s test [2005]
6. What will be the correct structural formula of product c. Schiff’s test d. iodoform test
for the following reaction? [2013] 14. Cyanohydrin of which of the following forms lactic acid
CH3 a. HCHO b. CH3CHO [2005]
s
Dil. KMnO4 HIO4 OH c. CH3CH2CHO d. CH3COCH3
A B C
CH3

@iitjeehelps
Answer with Solutions
Practice Exercise 40% H2 SO 4
8. (d) H3C — C ºº CH ¾¾¾¾¾® H3C — C == CH2
O3
1% HgSO 4 ½
1. (c) CH2 == CH — CH2OH ¾® OH
prop-1-en-2-ol
O (unstable)
O O Tautomerism
Zn/H2O ¾¾¾¾¾® H3C — C— CH3
  HCHO + OHC  CH2OH
− ZnO ½½
CH2  CH  CH2OH O
Dry distillation
2. (a) Calcium formate ¾¾¾¾¾® Formaldehyde Ph H⊕ Ph

H2O
9. (a)
Dry distillation CH3 CH3
Calcium acetate ¾¾¾¾¾® Acetone
OH ⊕
D Ph OH2
Calcium formate + calcium acetate ¾® Acetaldehyde
Ph—C—CH2CH3 Ph
i.e. two aldehydes and one ketone are obtained.
CH3
3. (a) I. C6H5MgBr, II. C6H5CH2OH , III. p -ClC6H4CH2OH , O CH3
IV. p -ClC6H4CHO. Keto form Enol form

4. (a) The final product shows the p-attack.


OMe
+ − 10. (b)
O Al Cl3
  O 11. (d)
+ O MeO  O−
12. (d) Formaldehyde gives urotropine with ammonia
whereas acetaldehyde gives acetaldehyde ammonia.
 
O O O HCN OH
13. (b) CH3 — C ¾® CH3 — CH
H3O+ H CN
O OH
 OH H × OH ½
MeO  ¾® CH3 — CH — COOH

lactic acid
O
14. (c) CH3 — C — CH3 + H — O — NO ¾®
Zn/HCl ½½
O O
OH
MeO  ½½

O CH3 — C — CH == N — OH

5. (a) 15. (c) Formaldehyde on treating with ammonia, gives


O
urotropine which is used as medicine and in making an
O O
CHC6H5 important explosive, cyclonite RDX.
(i) O3 CHC6H5
6HCHO + 4NH3 ¾® (CH2 )6 N4 + 6H2O
Hexamethylene
Ozonide tetraamine (urotropine)

N
O CHO
(ii) Zn/H2O ½
+ CH2 CH2 CH2
½ 3HNO3
Benzaldehyde N ¾¾®
6. (c) Any aliphatic carbon with hydrogen attached to it, in CH2 CH2
combination with benzene ring, will be oxidised to benzoic N N
acid by KMnO4 / H+ . CH2
7. (a) Urotropine

@iitjeehelps
692 SELF STUDY GUIDE BITSAT

NO2 23. (c) OCOCH3 H2O/H+


½ H2C C
N CH3 OH
(P) H 2C C + CH3COOH
H2C CH2 + 3HCHO + NH3 CH3
| | Enol form
O2N — N N — NO2
Tautomerisation

CH2 O
RDX
H3C C CH3
16. (a) It is the product of reaction (a). Acetone (keto form)
CH2 — OH
½ Fehling’s solution is not reduced with acetone.
HO — CH2 — C —CH 2— OH
½
H 3C H 3C
CH2OH
+
H2O/H
17. (b) Tollen’s reagent converts only —CHO to — COOH. OCOCH3 OH +CH3COOH
Q Enol form
COOH Pyridine
18. (c) CH3CH == O + H2 C ¾¾¾®
COOH —H2O Tautomerisation
COO H —CO2
CH3CH ==C ¾¾® CH3CH == CH× COOH CH3—CH2 —C — H
COO H D Crotonic acid

19. (b) Acetone generates carbanion because carbonyl group O


of acetaldehyde acts as a better nucleophile acceptor
than acetone. Fehling solution is reduced with aldehyde, i. e.
- + CH3CH2CHO. Hence, the product of acid hydrolysis of
O OK P (ketone) and Q (aldehyde) can be distinguished by
½½ KCN ½ Fehling solution.
20. (a) CH3 — C — H ¾® CH3 — C — H
½ 24. (c) BaSO4 decreases the activity of Pd.
CN 25. (b) But-1-yne on reaction with water in presence of Hg2+
O O
|| || ions as a catalyst produces butan-2-one.
KCN
But Ph — C — H ¾® Ph — C — CH — Ph Hg 2+
CH3
½ H2 O
OH 
O
In other reactions, the products are similar.
O 26. (c) CH3COCH3 gives red colour with sodium nitroprusside
H2O2 ½½
21. (c) R2C == O ¾¾® R — C — OR solution but does not reduce Tollen’s reagent. Acetone
yields chloroform with NaOH / Cl2.
Cyclic ketones are converted into cyclic esters (lactones). NaOH
CH3COCH3 + Cl2 ¾® Cl3C — COCH3 ¾¾® CHCl3
H2O2
O
Acetone Chloroform
O O
‚
27. (b) Dehydrated product will be conjugated with C == O
ƒ
22. (c) From phenyl magnesium bromide, phenyl is attached
with that C-atom of carbonyl group which has low electron O O
density (higher electropositive charge). In carbonyl H
compounds, aldehydes are more reactive towards –H2O
H
nucleophile. In nucleophilic addition reactions because in
ketones alkyl groups (due to +I effect) decrease the OH
electropositive charge of carbon of carbonyl group.
Hence, attraction of nucleophile decreases. Moreover in 28. (b) (a) T is an ester, so get hydrolysed.
the tetrahedral intermediate, aldehydes have less stearic O
CH3 O
repulsion than ketones and also the aldehyde increases
the negative charge on oxygen less in comparison of O NaOH (aq)
ketones. Therefore, on the basis of above reason, the hot
HO ONa
order of reactivity of acetone (I), acetaldehyde (II) and H 3C Soluble in aqueous NaOH (hot)
benzophenone (III) with PhMgBr is II > I > III T

@iitjeehelps
ALDEHYDES AND KETONES 693
CH3 35. (a)
H CH3 NaOCl
36. (c) C CH3
H3 C
(b) T + LiAlH4 ¾® HO OH
(U) Optically inactive (no chiral carbon) O H CH3
C ONa + CHCl3
H 3C
COOH
O
+ CrO3 / H+ ¾® H3C This oxidation does not affect a carbon double bond, if
COOH
(V ) Optically inactive
present in the molecule.
CH3
Ba(OH)2 ½
OCOCH3

OCOCH3
37. (b) 2CH3COCH3 e H3C — C — CH2 —COCH3
½
OH
(c) U + (CH3CO)2 O ¾® H3C C10H18O4 (W ) Ketol
CH3
U on treatment with excess (CH3CO)2 O, forms a diester (W ). D ½
¾¾® H3C —C == CHCOCH3
– H2O
COOH Aldol condensation product
38. (c) Only those aldehydes undergo Cannizzaro reaction in
COOH the presence of concentrated base, which have lack of
H+
H3C NaHCO3 a-H atom. CH3CHO contains three a -H atoms, that’s why
(d) U + CrO3 ¾® (V) diacid ¾¾¾® CO2 ­ it does not undergo Cannizzaro’s reaction, rather it
undergoes aldol condensation.
29. (b) When two aldehydes or ketones (or carbonyl 39. (c)
compounds) having a-hydrogen atoms react in the O O
O
presence of a base, they result in the formation of H
b-hydroxy aldehyde or ketone, called the aldol or ketol H OH–
H+
and the reaction is called aldol condensation. H
OH O OH ∆
Thus, is the product of aldol condensation. O
β
α
O
||
30. (d) All carbonyl compounds of the type R ¾ C ¾ CH3 40. (a) Tollen’s reagent is used to distinguish between
formed iodoform but isobutyl alcohol do not have aldehydes and ketones. Aromatic aldehydes do not
O reduce Fehling solution and Benedict solution because
|| they are weak reagents.
R ¾ C ¾ CH3 group.
41. (a) The transfer of hydride ion to the carbonyl group is the
31. (a) The reagents used to carry out Clemmensen slowest step of Cannizzaro reaction.
reduction of ketones are Zn – Hg with conc. HCl. (i) Attack of OH- on carbonyl carbon
32. (b) Cannizzaro reaction is given by the aldehydes that do OH
not contain a-H-atom. The aldehydes like acetaldehyde, ½
CH3CHO possess a-H-atom and hence, does not undergo Ph ¾ C == O + OH ¾¾® Ph ¾ C ¾ O -
-

Cannizzaro reaction.
| ½
H H
33. (b) The reaction between a, b-unsaturated carbonyl
(ii) Transfer of hydride ion
compound and conjugated diene to undergo ring closure
OH OH
is called Diel’s Alder reaction.
½ ½ Slow
CH2 Ph ¾ C ¾ O- + Ph ¾ C == O ¾® Ph ¾ C == O +

+ ½ ½
CH.CHO CHO H H
Diene
H O-
34. (c) The H-atoms linked to C-3 atom are most acidic in ½ ½
Ph ¾ C ¾ O- ¾® Ph ¾ C == O + PhCH2OH
nature as these are attached to two electron withdrawing ½
carbonyl groups. H

@iitjeehelps
694 SELF STUDY GUIDE BITSAT

42. (a) The compound A undergoes Cannizzaro reaction X do not decolourise bromine water which suggests the
shows that it may be an aldehydic compound with no presence of unsaturation due to an aromatic ring.
a-H-atom. The formation of 2, 4-DNP suggests the Compound Y should be benzoic acid according to the
presence of C == O group and reduction of Tollen’s molecular formula of Y. The compound X must be
reagent confirms aldehydic group in the compound. The monosubstituted aromatic methyl ketone.
production of 1,2-benzene dicarboxylic acid on vigrous
Compound X is phenylmethyl ketone.
oxidation of A shows that aldehydic and ethyl group
present at 1, 2-position of benzene ring. Thus, the O
O2N
compound A is
CHO
CH3 –H2O
+ H2NHN  NO2
C 2H 5
X 2,4-dinitrophenyl hydrazine
43. (c) O C8H8 O O2N
H3C
Zn/Hg SOCl2 H2CrO4
C N—NH— —NO2
HCl
C C
COOH
HO O HO O 2,4-DNP derivative
A
AlCl3 LiAlH4 COONa
–HCl Y
C C7H6O2 + CHI3
Cl O O
B C
Conc. NBS
48. (a) I. X forms 2, 4-DNP derivatives, it shows that it has a
H2SO4 carbonyl compound.
II. It reduces Tollen’s reagent, it shows that it has an
OH aldehyde group.
D E
III. It undergoes Cannizzaro reaction, that shows the
Br
Alc. KOH presence of an aldehyde having no a-hydrogen.
IV. It produces 1, 2-benzenedicarboxylic. It shows that
F G groups are present at 1, 2-position on benzene ring.
CHO
44. (c) Addition of HCN is a nucleophilic addition reaction.
Greater the electron deficiency of carbonyl group, higher C 2H 5
the rate of reaction.
Structure is
O O O
|| || ||
Hence, Ph ¾ C ¾ Ph < Ph ¾ C ¾ CH3 < CH3 ¾ C ¾ CH3
O 49. (c) OH- can attack any of the three CHO groups, but
|| nucleophilic addition of - OH ion occurs on that CHO
< H ¾C ¾ H group which is located at para-positions with respect to
the other. This is because electron withdrawing groups
45. (c)
facilitate the release of hydride ion from the intermediate.
OCOCH3 OH OH
The p -CHO group is oxidised while the m-CHO group is
COCH3 reduced.
Anhyd. AlCl3
46. (c) CS2, ∆
+ CHO CHO CHO


OH O
COCH3
It is the Fries rearrangement. CHO C CH2OH
H
47. (a) X forms 2, 4-DNP derivative so it is an aldehyde or CHO s C COONa
ketone. X must be ketone as it does not reduce Tollen’s O H
or Fehling reagent. X must be methyl ketone as it HO
responds to iodoform test. 50. (d)

@iitjeehelps
ALDEHYDES AND KETONES 695
6. (a) CH3
BITSAT Archives Dil. KMnO4 HIO4
OH
1. (a) O OH
O
r CH3
CH3 OH vic cis diol
s
H s O
CH3 H H CH2—H CH2
s
O OH
O O
s
OO O
CH3 –
H H CH3 CH3 CH3
CH3
O O O
Os

OH
O O – H2 O
CH3 CH3
C
H OH H
CH3 s 7. (b) In Wacker process, when mixture of propene and air is
CH2 passed through mixture of Pd and CuCl2 at high pressure,
O OH acetone is formed.
s
O OH Pd + CuCl2 ¾® PdCl2 + 2CuCl
4CuCl + HCl + O2 ¾® 4CuCl2 + 2H2O
H+ *
CH3CH == CH2 + PdCl2 + H2O ¾®
Propene
CH3COCH3 + Pd + 2HCl
O O
Product Cl
H2O
The number of chiral centre in product is 1, represented by 8. (c) CH ºº CH + 2HO-¾ Cl+ ¾® CH¾ CHO
star (*). Cl
2. (b) Steric hinderance decreases the rate of reaction. This 2, 2-dichloroacetaldehyde
steric hinderance is minimum in methanal and maximum
in benzophenone. 9. (a) K 2Cr2O7 + conc. H2SO4, hot alkaline KMnO4 and
3. (d) conc. HNO3 will all oxidise toluene to benzoic acid while
CrO3 in CH3(CO2O) (mild reagent) will oxidise toluene to
4. (d) O3 benzaldehyde.
O
Zn/H2O O H3C ‚ CH3
H C ‚ d+ d- d- d+ ƒ
A 10. (b) 3 C == O + CH3Mg Br ¾® C
Now, this bicarbonyl compound undergoes intramolecular
H3C ƒ Methyl ƒ ‚
Acetone magnesium
H3C OMgBr
aldol condensation as follows: bromide Complex
s
O
H2 O
H 3C CH3 OH
s C + Mg
OH H3O+
O O H3C OH Br
O O s O X
or ( CH3 )3 COH 2-methyl propon - 2- ol
OH
O3
∆ 11. (c) CH3 — C ºº C — CH3 ¾¾¾¾¾®
Zn/H2O - H2O2
– H2O
CH3 — C — C — CH3
O O ½½ ½½
O O
5. (c) CH2CH2CH3 COOK 12. (b)
13. (b)
KMnO4
OH
KOH, ∆ KCN
H3C H+
½ H+ /H2O
14. (b) C == O ¾¾® H3C — C — CN ¾¾®
H3O+ H EtOH ½
H
COOH OH
½
H3C — C — COOH
½
H
Lactic acid

@iitjeehelps
27
Carboxylic Acid and
Its Derivatives
Introduction
Carboxylic acids are a large family of organic compounds that contain the functional group called the
carboxyl group. Any aliphatic or aromatic compound can have a carboxyl group attached to it and
represented by
O

RCOH

Alkyl/aryl Carboxyl group

carboxylic acids form numerous important derivatives including esters, amides, anhydrides and acid
halids carboxylic acid have general formula: C nH 2 n + 1COOH

Methods of Preparation
1. From Alkenes
(a) By oxidation
Hot
RCH == CH 2 → RCOOH + CO2 + H 2O
KMnO 4

e.g. CH3  CH == CH 2 → CH3COOH + CO2 + H 2O


(b) Carboxylation
H 3 PO 4
RCH == CH 2 + CO + H 2O → R  CH COOH


CH3

@iitjeehelps
CARBOXYLIC ACID AND ITS DERIVATIVES 697
2. From Grignards Reagent 2. Solubility
H 2O X Lower members of aliphatic acids with C 1 C3 atoms are highly
RMgX + CO2  → RCOOH + Mg
OH soluble in water due to H-bonding. Solubility of carboxylic
3. From Trihalogen Derivatives acids decreases with increases in the number of carbon atoms
due to an increase in hydrophobic interactions of the
( aq ) KOH H+
R  CCl3 → RCOO−K + → RCOOH + KOH hydrocarbon part.
− H 2O H 2O
3. Boiling Points
4. By Oxidation of Aldehyde, Ketones and Boiling points of carboxylic acids is much higher than those of
Alcohols hydrocarbons, aldehydes, ketones and alcohols of comparable
RCH 2OH → RCHO → RCOOH masses due to the formation of strong intramolecular H-bonds.
[O] [O]

R‚ [O] R‚ 4. Melting Points


CHOH → C==O
R′ CH 2 ƒ R′ CH 2 ƒ The first ten members of carboxylic acids show oscillation or
alternation effect in their melting points. The melting point of an
Vigorous
→ RCOOH + R′ COOH acid with even number of C-atoms is higher than that of acids
[O]
with odd number of C-atoms.
Strong oxidising agents
K 2Cr2O7 / H + or KMnO4 / H + Acidic Strength of Carboxylic Acids
Carboxylic acids are weaker than mineral acids but stronger
Mild oxidising agents than phenols because of conjugate base, carboxylate ion is more
MnO,CuO (Fehling’s solution), Ag 2O (Tollen's reagent) , stabilised than phenoxide ion.
FeSO4 + H 2O2 (Fenton's reagent)
Effect of Substituents on Acidic Strength
DCC *
e.g. CH 2 == CH  CH 2  OH → The strength of carboxylic acids depends on the substituents
bonded to carboxyl group. Electron withdrawing groups like
Ag 2O
CH 2 == CH  CHO → CH 2 == CH NO2 , CN, F, Cl, Br, OH exerting –I-effect decreases the
 concentration of negative charge from carboxylate ion, thereby
COOH making the O—H bond weaker and hence, release of proton
(Acrylic acid) easier.
5. Carboxylation of Sodium Alkoxide Larger the number of electron withdrawing groups bonded the
carboxyl group, greater is the acidic strength of acid.
∆ H 3O +
RONa + CO → RCOO− Na + → RCOOH + Na + Cl3COOH > CHCl 2COOH > CH 2ClCOOH > CH3COOH
Pressure
Electron donating groups like  R , OR exerting +I-effect
6. By Hydrolysis of Nitriles decreases the acidic strength of acids. The +I-effect of alkyl
KCN H 3O + groups decreases in the following order
RX → RCN → RCOOH
CH3 < CH3CH 2 < (CH3 )2 CH < (CH3 )3 C
KCN H 3O +
e.g. CH3  Cl → CH3CN → CH3COOH Thus, the relative strength of acids follows the following order
CH3COOH > CH3CH 2COOH > (CH3 )2 CHCOOH
Physical Properties of > (CH3 )3 CCOOH

Carboxylic Acids o-substituted benzoic acids are stronger than para and
meta-benzoic acids.
1. Physical State and Odour COOH COOH COOH COOH
Lower members of aliphatic acids with C 1 C3 atoms Me
are colourless and pungent smelling liquids. Higher > > >
carboxylic acids are colourless oily liquids. Unbranched
Me
chain of C 12 C 24 atoms are called fatty acids. (o-effect)
Me

@iitjeehelps
698 SELF STUDY GUIDE BITSAT

Chemical Reactions Some Special Reactions of HCOOH


(a) It contains CHO group, hence reduces Tollen’s
1. Reduction reagent.
LiAlH 4
R COOH → R  CH 2 OH O
Cu/CrO 3  ∆
Red P + HI R — R H  C OH + Ag 2O → 2Ag + CO2 + H 2O
R COOH → R — R
H 2 , Ni
(b) HCOOH + Hg 2Cl 2 → CO2 + 2HCl + 2Hg
Hg-black
2. α-Halogenation
(Hell-Vohlard Zelinsky Reaction) HVZ (c) 2MnO−4 + 6H + + 5HCOOH → 2Mn 2+ + 5CO2 + 8H 2O

7 + 8H + 3HCOOH → 2Cr
(d) Cr2O2- + 3+
Red P + PCl 3 + 3CO2 + 7H 2O
RCH 2COOH + Cl 2 → R CH COOH +HCl
UV
 (e) HCOOH + 2CuO → Cu 2O + H 2O + CO2
Red ppt
Cl
The above reactions can be used to differentiate
3. Reaction of Salts of Acids between formic acid and other acids.
(a) Decarboxylation Effect of Carbonyl Group in Acid Derivatives
NaOH + CaO
RCOONa → RH + Na 2CO3 (a) It provides a site for a Nu − attack.

O
(b) Kolbe’s electrolysis  Nu −
R  C Z → R  C  Nu + Z −
Electrolysis
2 RCOONa + 2H 2O → 
O
R  R + 2CO2 + 2NaOH + H 2
O
(c) Action of Heat 

Z = (Cl, NH 2 , OR ′ , O C  R ′ )
( RCOO)NH 4 → RCONH 2 + H 2O
Reactivity RCOX > RCOOCOR > RCOOR > RCONH 2
4. Formation of Acid Derivatives (b) Acidic character of α-hydrogen.
(a) Acid chloride Carbonyl group makes α — H active hence, acid
derivatives give reactions such as Claisen condensation.
CH3COOH + SOCl 2 → CH3COCl + SO2 ↑ +HCl ↑
∆ 2CH3COOC 2H 5 → CH3COCH 2COOC 2H 5 + C 2H 5OH
RCOOH + PCl 5 → RCOCl + HCl + POCl3 Ethyl acetoacetate (EAA)

3 RCOOH + PCl3 → 3 RCOCl + H3PO3 Uses
(b) Anhydride (a) Formic acid Used as reducing agent, for removing hair
P2O 5 , ∆
from skin, formation of resin, insecticide, antiseptic etc.
2CH3COOH → (CH3CO)2 O (b) Acetic acid Used in synthesis of other organic
Conc. H 2 SO 4
compounds, as a solvent, in coagulation of rubber latex,
(c) Amide for making vinegar, cellulose acetate etc.
NH 3 ∆
CH3COOH → → CH3CONH 2
—H 2O Aromatic Acids
(d) Esterification COOH
Conc. H 2 SO 4 COOH
CH3COOH + C 2H 5OH →
CH3COOC 2H 5 + H 2O COOH
Benzoic acid Phthalic acid
5. Schmidt Reaction
(Formation of 1°-amines) NH2 OH
H 2 SO 4
R — COOH + HN3 → RNH 2 + CO2 + N 2
COOH COOH
H 2 SO 4 Anthranilic acid Salicylic acid
e.g. CH3COOH + HN3 → CH3 NH 2 + CO2 + N 2

@iitjeehelps
CARBOXYLIC ACID AND ITS DERIVATIVES 699
Preparation 3. By Sandmeyer Reaction
CuCN
1. By Oxidation of Toluene C6H 5N +2 Cl + KCN − → C6H 5CN + N 2 ↑ + KCl
CH3 COOH
Chemical Properties
Dil. HNO3 /HCrO4
[O] Acidity in Benzoic Acid
Acidity of benzoic acid increases, when any group is
2. By Friedel-Crafts Reaction introduced at the ortho position, due to ortho-effect. Hence,
AlCl 3 o-nitrobenzoic acid is the strongest among nitrobenzoic
C6H6 + COCl 2 → C6H 5COCl + HCl acids.
Benzoyl chloride
Aromatic acids are stronger acid than aliphatic acids.
C6H 5COCl + H 2O → C6H 5COOH + HCl
Benzoic acid
∴ C6H 5COOH > CH3COOH

Practice Exercise
1. Highest bp would be of 5. In the reaction,
a. formic acid b. water CH 3 COONa
c. acetic acid d. ethanol C6H5 CHO + (CH3 CO)2 O → A,
2. Acidic hydrolysis of, which of the following ester will be product A is
slowest? a. acetaldehyde
a. CH3COOCH3 b. cinnamic acid
c. β-naphthol
b. CH3COOCH2CH3
d. phenol
c. CH3COO — C H — CH3
 (i) NaOH H+ / H2O Ac 2 O
CH3 6. Phenol → A → B → C
(ii) CO2 /140° C
d. CH3COOC(CH3 )3 In this reaction, the end product C is
H ydrolysis a. salicylaldehyde
3. HCN → acid-1
b. salicylic acid
Hydrolysis c. phenyl acetate
CH3 CCl3 → acid-2 d. aspirin
The heat of ionisation of HCN and acid-2 respectively ∆ NH 3
7. C8H 6O4 → X → Y
are x kcal and y kcal [x < y ]. The order of their pK a
values will be The compound X is
a. (pK a ) HCN > (pK a ) acid-2 a. o -xylene
b. (pK a ) HCN = (pK a )acid-2 b. phthalic acid
c. (pK a ) HCN < (pK a )acid-2 c. phthalic anhydride
d. All of the above d. salicylic acid

4. A compound with molecular formula C4 H10O4 on 8. The correct order of increasing acidic strength is
a. phenol < ethanol < chloroacetic acid < acetic acid
acylation with acetic anhydride gives a compound with
b. ethanol < phenol < chloroacetic acid < acetic acid
molecular formula C12H18O8. How many hydroxyl
groups are present in the compound? c. ethanol < phenol < acetic acid < chloroacetic acid
a. One b. Two c. Three d. Four d. chloroacetic acid < acetic acid< phenol < ethanol

@iitjeehelps
700 SELF STUDY GUIDE BITSAT

9. The correct order for the acidic strength of the 12. Select the missing compounds for the given reaction.
following compounds is
CH3
I. CH3 COOH II. MeOCH2COOH Dil. KMnO4 HIO4 –
OH
A B C
III. CF3 COOH IV. Me2CHCOOH D

a. II < IV < I < III CH3


b. IV < I < III < II
c. IV < I < II < III Identify A, B and C for the above reaction and choose
d. I < IV < III < II the correct option.
A B C
10. Consider the following reaction, CH3
Mg / dry ether (i) CO2
CH3
CH2 == CHCH2Br → A →
+
X OH O
(ii) H 3 O
a.
The compound X is
OH O
a. CH2 == CHCH2COOH CH3 O
b. CH3CH2CH2COOH CH3 O CH3
c. CH3CH == CH  COOH OH
CH3
d. CH2 == CHCOOH b.
11. Arrange the following derivative of carboxylic acid OH O
CH3 O
in their increasing order of their acidity. CH3
CH3 CH3
COOH COOH COOH O
H
c.
CH3
a. < < O
OH O
O H 3C OH
OCH3 NO2 CH3
d. CH3
O
COOH COOH COOH CH3
CH3 O

b. < < 13. Which of the following represents the correct order of the
acidic strength in the given compounds?
a. CH3COOH > BrCH2COOH > ClCH2COOH > FCH2COOH
OCH3 NO2 b. FCH2COOH > CH3COOH > BrCH2COOH > ClCH2COOH
c. BrCH2COOH > ClCH2COOH > FCH2COOH > CH3COOH
COOH COOH COOH d. FCH2COOH > ClCH2COOH > BrCH2COOH > CH3COOH

CHO OHC
c.
< <
(i) NaOH/100°C
14. +
(ii) H /H2O
OCH3 NO2
CHO OHC

COOH COOH COOH Major product of the above reaction is


COOH HOOC
d. ~ <

a.

OCH3 NO2 COOH HOOC

@iitjeehelps
CARBOXYLIC ACID AND ITS DERIVATIVES 701
OH HOOC 20. The compound that undergoes decarboxylation most
readily under mild condition is
b. COOH COOH
CH2COOH O
COOH HO
a. b.
O
O
COOH CH2COOH
COOH O
c.
c. d.
O
21. The acids, HCOOH and CH3 COOH are distinguished by
CH2OH HOOC
a. reaction with Na metal
b. reaction with NaOH
d. c. reaction with Tollen’s reagent
d. reaction with Schiff’s reagent
COOH HOH2C 22. The reaction given below is called
Red P/ Br
15. To distinguish between formic acid and acetic acid, CH3 CH2CH2COOH    
2
→ CH3 CH2 CH COOH
the suitable reagent is |
Br
1. acidified KMnO4
a. Cannizzaro reaction
2. Tollen’s reagent b. Hell-Volhard-Zelinsky reaction
3. ethyl alcohol c. Schrodinger reaction
d. Perkin’s reaction
4. sodium bicarbonate
23. Carboxylic acids dissociate to give resonance stabilised
The correct answer is
carboxylate anion
a. 1, 2, 3, 4 b. 1, 2, 3
c. 2, 3, 4 d. 1, 2 R — C— OH + H2O q H3 O+

16. When salicylic acid is heated with acetic anhydride, O
we get
a. aspirin b. salol  O O⊕ 
c. paracetamol d. None of these  s s 
+ R — C ←→ R — C 

17. The general formula of the compound obtained as a  O O 
result of reaction between formic acid and hydroiodic  
acid is The above statement is
a. CnH2n + 1OH b. CnH2n + 2 a. true b. false
c. CnH2nO d. CnH2n c. sometimes true d. sometimes false
24. In the given reaction,
18. Pyruvic acid as

a. α-keto acid COOH COONH4
b. β-keto acid NH3
c. γ -keto acid
d. δ -keto acid
∆ –H2O
19. The product of which of the following reaction is
capable of changing orange colour of Cr2O72− to (I)

green colour of Cr 3 + . COOH COO– NH4

a. CH2(COOH)2 →

H3 O+ –
b. CH3CN → COOH COO NH4
H3 O+ Strong heating
c. HCN →
H2O
d. CH3CONH2 → (II)

@iitjeehelps
702 SELF STUDY GUIDE BITSAT

Identify I and II for the above reaction and choose the


X Y
correct option.
I II COOH COOH
COOH NO2 Br
COOH a.
a.
COOH COOH
COOH
CONH2
b.
CONH2
b. NO2
Br
CONH2
COOH COOH
COOH O
Br
c.
c. O

NO2
O
CONH2 O COOH COOH

d. NH d.
NO2 Br

O
COOH
Ba(OH)2
25. The product formed during Hell-Volhard-Zelinsky 27. 300°C
reaction is COOH
a. R — CH— COOH
 COOH
X
b. R — CH2 — COX a. b.
OH
X O

c. R — C— COOH C
COOH

X c. d. O
d. R — CH— CH2 — COOH
C

X O
26. In the given reactions: 28. In a set of the given reactions, acetic acid yields a
COOH product C.
CH3 COOH + PCl5 → A
Conc. HNO3 C6H6 C 2 H MgBr
Conc. H2SO4
X A → B →
5
C
Anhy. AlCl3 Ether

Product C would be
COOH a. CH3CH(OH)C6H 5
C2H 5
Br2 / FeBr3 
Y b. CH3 —C(OH) C6H 5
c. CH3CH(OH)C2H 5
Identify X and Y for the above reaction. d. CH3COC6H 5

@iitjeehelps
CARBOXYLIC ACID AND ITS DERIVATIVES 703
29. Complete the synthesis by giving the missing product.
COOH
SOCl2
Heat
COOH
O O
COCl COCl
a. b. c. O d.
COOH COCl
O O

30. Role of 2,4-dichlorophenoxy acetic acid is used as


a. moth repellant b. insecticide c. fungicide d. herbicide

BITSAT Archives
1. Acetic acid on warming with hydrazoic acid in 4. Identify A and B in the following reaction. [2008]
presence of conc. H2SO4 gives [2010] B A
CH3 — CH3 ← CH3 COOH → CH3 CH2OH
a. CH3NH2
b. CH3CH2NH2 A B
c. CH3CONH2 a. HI + red P LiAlH4
d. CH3COONH4 b. Ni/∆ LiAlH4
c. LiAlH4 HI + red P
2. Which of the following on heating does not form an d. Pd-BaSO4 Zn + HCl
anhydride? [2009]
a. Oxalic acid 5. In aqueous solution amino acids mostly exit as [2007]
b. Succinic acid a. NH2 — CHR — COOH b. NH2 — CHR — COO−
c. Glutaric acid c. NH3 — CHR — COOH d. H3N+ CHR — COO−
d. Maleic acid
6. On heating with oxalic acid at 110°C, glycerine gives
3. Acetic acid reacts separately with the following
a. glyceryl trioxalate b. formic acid [2006]
alcohols. The rate of esterification is higest for [2009]
c. glyceryl dioxalate d. None of the above
a. CH3OH
b. C2H5OH 7. Glycerol is oxidised by bismuth nitrate to produce
c. (CH3 )2 CHOH a. oxalic acid b. mesooxalic acid [2005]
d. (CH3 )3 COH c. glyceric acid d. glyoxalic acid

@iitjeehelps
Answer with Solutions
Practice Exercise
1. (c) High molecular weight of acetic acid coupled with OH
polar nature due to resonance as well as presence of 
H-bonding accounts for its high boiling point.
8. (c) CH3→→  CH2→→  O  H<
2. (d) Alcoholic part of this ester offers maximum streric +I-effect of ethyl group
hinderance. Hence, is hydrolysis will be slowest. increases the electron density
on O-atom and makes the release of
3. (a) Acid-2 is acetic acid. HCN is a weaker acid as its heat H+ ion difficult.
of ionisation is less than acetic acid. Hence, O O
(pK a ) HCN > (pK a ) acid-2  Cl 
<CH3 →→  C  OH <H  C  C  OH
4. (d) R — OH → R — OCOCH3 H
Presence of Cl-atom makes the
Acylation of one hydroxyl group results in increament of release of H+ ion easier due to
—COCH2 ( or C2H2O). In the question there is an their –I-effect.
increament of (C8H8O4 ), hence there should be present O O
four—OH groups. H 3C  
H2  CHCO 9. (c) CHC  OH <CH3 C  OH
‚ CH3 COONa H 3C
5. (b) C6H5CH O + O → Two CH3 groups exerts +I-effect
ƒ 453 K, − H2O
CH3CO and makes the release of
H+ ion difficult.
C6H5CH == CHCO
‚ Boil O O
O →
ƒ H2O  F 
CH3CO CH3  O  CH2  C  OH < F C  C  OH
F
C6H5CH == CH  COOH + CH3COOH Three F groups exerts –I-effect and makes
Cinnamic acid the release of H+ ion easier.

6. (d) OH ONa Mg (i) CO2


10. (a) CH2== CHCH2Br Dry ether CH2==CHCH2MgBr
Grignard
(i) NaOH COONa reagent
O O
(ii) CO2/140°C
 (ii) H3O+
 OH
OCOCH3 C  OMgBr CH2 ==CHCH2C  OH+Mg
OH

Br
CH2CH == CH 2
+
H /H2O COOH Ac2O COOH
11. (a) The presence of electron withdrawing group on the
(Aspirin) phenyl of aromatic carboxylic acid increases their acidity
Acetyl salicylic acid
while electron donating groups decrease their acidity.
7. (c) O COOH COOH COOH
COOH C NH3 CONH2
O
COOH C COOH
Phthalic Y
O
acid
Phthalic anhydride
X OCH3 NO2
(pKa = 4.46) ( pKa = 4.19) (pKa = 3.14)

@iitjeehelps
CARBOXYLIC ACID AND ITS DERIVATIVES 705
CH3 19. (c) Product of 1, 2 and 4 is CH3COOH, where as product
CH3
12. (b) Dil. KMnO4 —OH of 3 is HCOOH which is reducing agent and will be
—OH
oxidised by Cr2O2−7 .

CH3 A CH3
20. (b) It is a β-keto acid which undergoes decarboxylation in
very mild condition, i.e. on simple heating. Ordinary
HIO4
carboxylic acid requires soda lime catalyst for
CH3 CH3 decarboxylation (for decarboxylation β-keto acid must
OH contain an α-H atom).
OH O COOH
–H2O Intermolecular O O
CH3 aldol condensation α
O β,
CH3
B

Thus, readily undergoes decarboxylation.


C O
21. (c) HCOOH gives positive Tollen’s test while CH3COOH
13. (d) The acidity of halogenated acid increases with
do not respond to this test.
increase in electronegativity of the halogen present. The
correct order is 22. (b) The α-bromination of carboxylic acids is called
FCH2COOH > ClCH2COOH > BrCH2COOH > CH3COOH Hell-Volhard-Zelinsky reaction.

14. (d) Cannizzaro reaction occur in the absence of 23. (b) Carboxylic acids dissociate in water to give resonance
α-hydrogen atom. stabilised carboxylate anions and hydronium ions.
CHO CHO O
R—C + H2O q H3O+ +
NaOH OH
100°C
 O Os  O
COOH OHC  
R — C ←→ R — C  ≡≡ R — C
COONa CH2OH
 Os O  Os
 
24. (d) Carboxylic acids react with ammonia to give
ammonium salt, which on further heating at high
CH2OH COONa temperature give amides.
–+
COOH COONH4 CONH2
H + H 2O

COOH CH2OH ∆
+ NH3 w –H2O

Ammonium Benzamide
benzoate
CH2OH COOH COOH COO–NH4+
15. (d) Formic acid decolouries acidified KMnO4 solution as + NH3 w
well as gives silver mirror with Tollen’s reagent. COOH COO–NH4+
16. (a) Acetyl salicylic acid is known as aspirin. Ammonium phthalate

OH OCOCH3 ∆ –2H2O
O
 CONH2
COOH (CH3CO)2O COOH Strong heating
∆ NH
–NH3
CONH2

O Phthalamide
17. (b) The product of the reaction is CH4 and hence, the Phthalimide
general formula is CnH2n + 2.
18. (a) Pyruvic acid is 2-oxo-propanoic acid. 25. (a) Carboxylic acids having an α-hydrogen are
O halogenated at the α-position on treatment with chlorine
CH3COCOOH or bromine in the presence of small amount of red
(α-keto acid) α phosphorus to give α-halocarboxylic acids. The reaction
CH3 COOH
β is known as Hell-Volhard-Zelinsky reaction.

@iitjeehelps
706 SELF STUDY GUIDE BITSAT

COOH COOH

Conc. HNO3 28. (b) CH3COOH + PCl5 CH3COCl


26. (d) A
Anhy. AlCl3
Conc. H2SO4
NO2 CH3
m-nitrobenzoic acid
H3CH2C—C—OH COCH3
COOH COOH

C2H5MgBr
Br2/FeBr3
Ether

Br C B
m-bromobenzoic acid
COOH COCl
COOH COOH 29. (b) + SOCl2
Ba(OH)2
27. (c) + CO2
300°C COOH COCl
COOH 30. (d) 2,4-dichlorophenoxy acetic acid is used as
Phthalic acid
herbicide.

BITSAT Archives
N3 H Red P + HI
1. (a) CH3COOH → CH3NH2 + CO2 + N2 CH3COOH → CH3 — CH3
H2SO4 Ethane
2. (a) Therefore, oxalic acid on heating does not form an Hence, reagent A and B are respectively LiAlH4 and
anhydride. Instead it decomposes to give CO2 + CO + H2O. HI/red P.
COOH 5. (d) In aqueous solutions, amino acids mostly exist as
∆ ∆
 → CO2 + [HCOOH] → CO + H2O zwitter ions.
COOH R
+ 
3. (a) As the hydrocarbon part (R ) of the alcohol increases, the N H3 — CH — COO −
tetrahedral intermediate (I ) formed during esterification Zwitter ion

becomes more and more crowded and hence, the rate of 6. (b)
esterification decreases accordingly. Thus, the smallest
alcohol, i.e. CH3OH reacts at the fastest rate. CH2OH + HOOCCOOH CH2OOCCOOH CH2OOCH
+ 100 –110°C –CO2
O OH CHOH CHOH CHOH
–H2O
R OH
CH3 C OH CH3 C OH CH2OH CH2OH CH2OH
Glycerine
OH propane-1, 2, 3- triol H 2O
O
C –H2O
H3 C OH CH3 C OR CH2OH
OR Ester

CHOH + HCOOH
Tetrahedral intermediate (I) Formic acid
CH2OH
4. (c) Acetic acid on reduction with lithium aluminiumhydride
(LiAIH4 ) gives ethyl alcohol while on reduction with HI and CH2OH O
red P gives ethane.  Bi(NO3 )3 
7. (b) CHOH → HOOC — C — COOH
LiAlH4
CH3COOH → CH3CH2OH  Meso −o xalic acid
Ethyl alcohol CH2OH

@iitjeehelps
28
Nitrogen Containing
Compounds

Amines
These are the derivatives of ammonia in which one, two or all the three hydrogen atoms are replaced
by alkyl or aryl groups. Amines are classified as primary, secondary or tertiary according to the
replacement of one, two or three hydrogen atoms from ammonia respectively.

Structure of Amines
Nitrogen in amines, is sp3 -hybridised. The sp3 -hybridised orbitals of nitrogen overlap with orbitals of
hydrogen or carbon depending upon the composition of amines. The fourth hybridised orbital of
nitrogen in all amines has an unshared pair of electrons. Due to the presence of lone pair of electrons,
the bond angle C—N—E (where, E is C or H) is less than 109.5° and shape is pyramidal.

N
108° CH3
CH3
CH3
Pyramidal shape of
trimethyl amine

Preparation of Amines
Several methods can be used to prepare aliphatic amines, out of which some are specific for 1°, 2° and
3° amines in which some gives mixtures of all the three.
1. By the Reduction of Nitro Compounds, Nitriles and Amides
Alkyl nitriles, nitroalkanes, oximes and amides, all on reduction give primary amines. Reduction of
nitriles to primary amines by using Na and alcohol is called mendius reaction.

@iitjeehelps
708 SELF STUDY GUIDE BITSAT

NO2 NH2 O
H2/Pd
(a) Ethanol HOH / (aq) NaOH C ONa
R NH2 +
C ONa
1° amine
NO2 NH2
(b) Sn + HCl O
Fe + HCl
Aromatic primary amines cannot be prepared by this
Sn + HCl
method because aryl halides do not undergo nucleophilic
(c) R ==NO2 → R  NH 2 substitution with the anion formed by phthalimide.
( R = alkyl group)
H / Ni 4. Hofmann Bromamide Degradation Reaction
(d) R C ≡≡ N →
2
R CH 2  NH 2
Na Hg + C2 H 5OH Amides on reaction with Br2 in an aqueous or ethanolic
O solution of NaOH give 1° amine with one C-atom less than
 (i) LiAlH 4
that present in the amide.
(e) R  C  NH 2 → R CH 2  NH 2 O
(ii) H 2O
 NaOH + Br 2
(R may be alkyl or aryl group) R  C  NH 2     → R  NH 2 + Na 2CO3
NaOBr
All the above reactions are used in the preparation of
(where, R = alkyl or C6H 5 ) + 2NaBr + 2H 2O
primary amines.
5. Schmidt Reaction
2. By Ammonolysis of Alkyl Halides
(Hofmann’s Method) O
 Conc. H 2 SO 4
When alkyl halide is heated with alcoholic solution of R  C  OH + N3H  → R  NH 2 + CO2 + N 2
ammonia in a sealed tube at about 393 K, a mixture of 1 ° amine
amines is obtained. This reaction is called Hofmann’s
Instead of N3H and conc. H 2SO4 , a mixture of NaN3 and
ammonolysis method.
+
conc. H 2SO4 can also be used.
NH3 + RX → RNH3 X − ( RNH 2 + HX )
Ammonium salt
Physical Properties of Amines
+ General physical properties of amines are as follows :
RX
RNH 2 → RX
R 2 NH → RX
R3 N → R4 NX −
1° − HX − HX Quaternary (i) Lower aliphatic amines are gases while lower
2° 3°
ammonium salt aromatic amines are liquids.
NOTE Ammonolysis of alkyl halides produces a mixture of (ii) Boiling point of amines is higher than
1°, 2° and 3° amines alongwith quaternary ammonium salt. hydrocarbons of comparable molecular masses due
The free amine can be obtained from ammonium salt by the to the presence of intermolecular H-bonding.
treatment with a strong base. However, H-bonding is absent in tertiary amines.
+ − (iii) Aliphatic amines are miscible with water due to
R NH3 X + NaOH → R  NH 2 + H 2O + NaX intermolecular H-bonding with H 2O molecules.
3. Gabriel Phthalimide Synthesis (iv) The lower aliphatic amines are gases with fishy
This reaction is used for the preparation of 1° amine odour. Aniline and other aryl amines are usually
(aliphatic as well as aromatic amines having electron colourless but get coloured on storage due to
withdrawing group at o- and p-positions, e. g.  NO2 atmospheric oxidation.
group). (v) Boiling points and solubility of isomeric amines
O O decrease with branching, i.e.
_ +
C KOH C CH3  CH 2  CH 2  NH 2 > CH3
N H –H2O
NK 1 ° amine
C C 2 H-bonding

O O  NH CH 2 CH3 > (CH3 )3 N


2 ° amine 3 ° amine
1 H-bonding No H-bonding
O
(vi) Amines have lower boiling point than
RX C corresponding alcohols.
N R
–KX C
Chemical Properties of Amines
O Amines are reactive and behave as nucleophiles due to the
N-alkyl phthalimide
presence of one unshared pair of electrons.

@iitjeehelps
NITROGEN CONTAINING COMPOUNDS 709
Basic Nature of Amines ( R  NH  R) and tertiary amines (R3 N) do not show this
l Amines are basic in nature, so react with acids and reaction, hence it is used as a test for primary amines.
form salt. R— NH 2 + CHCl3 + 3KOH (alc. ) →
l Larger the value of K b or smaller the value of pK b , R— NC + 3KCl + 3H 2O
stronger is the base. Carbylamine

In RNH2 RN == CHR and RCN, nitrogen is sp3 , sp 2 and Reaction with Nitrous Acid
sp-hybridised respectively. More the s-character of
hybridised orbital containing lone pair, lesser is the 1. Aliphatic primary amines when react with HNO2 , give
basic character. Hence, RNH2 is most basic and RCN alcohol.
(HNO 2 )/ NaNO 2 + HCl
is least basic among these. R—NH 2 → R— OH + N 2 + H 2O
l The order of basic strength in case of methyl 2. Aromatic primary amines when react with HNO2 , give
substituted amines and ethyl substituted amines in diazonium salts.
aqueous solution is as follows +
(C 2H 5 )2 NH > (C 2H 5 )3 N > C 2H 5NH 2 > NH3 NH2 N NCl−
HNO2 + HCl
(CH3 )2 NH > CH3 NH 2 > (CH3 )3 N > NH3 or
NaNO2 + 2HCl
+ NaCl + 2H2O
NH2 273- 278K

< RNH2 3. Secondary amines both aliphatic and aromatic on


Alkyl group
reaction with HNO2 give an oily nitroso compound.
Electron donating substituent such as CH3 , —OCH3 NH CH3 N(NO)CH3
increases the basicity while electron withdrawing group
decreases the basicity.
+ HO NO + H2O
Acylation
In this reaction, 1° amines give N-substituted amide while N-methyl-N-nitroso aniline
(yellow oily layer)
2° amines give N,N-disubstituted amides.

C 2H 5 — NH 2 + CH3CO+ Cl − →
OH 4. Tertiary aliphatic amines dissolve in a cold solution of
Acyl chloride nitrous acid to form salts which decompose on
C2 H 5 — NHCOCH 3 + HCl warming to give nitrosoamine and alcohol.
e. g.
NH2
Cold
O O (C 2H 5 )3 N + HONO → [(C 2H 5 )3 NH + ] NO−2
3 ° amine Salt
OH−
+ CH3 C+ O− C CH3
Warm
Acid anhydride → (C 2H 5 ) 2 N  N == O + C 2H 5OH
NHCOCH3
5. Aromatic tertiary amines react with nitrous acid to give
coloured substituted nitroso compound.
+ CH3COOH
H3C N CH3 H3C N CH3
Acetanilide

NOTE Aniline does not undergo Friedel-Crafts reaction (alkylation + HO− NO+ + H2O
and acetylation) due to salt formation with AlCl 3 (anhy.), a
Lewis acid, which is used as a catalyst.
3°amine
NO
Carbylamine Reaction p-nitroso–N, N-dimethyl aniline

Aliphatic and aromatic primary amines, on heating with


CHCl3 and KOH(alc.) form isocyanide or carbylamine This reaction is used to distinguish 1° , 2 ° and 3°
which is a foul smelling substance. Secondary amines.

@iitjeehelps
710 SELF STUDY GUIDE BITSAT

Electrophilic Substitution Reaction This is due to the fact that aniline gets protonated to
This is an organic reaction in which an atom that is attached form anilinium ion which is meta directing.
to an aromatic system is replaced by an electrophile. Some .. +
NH2 NH3
of the most important electrophile aromatic substitution, i.e.
bromination, nitration, sulphonation etc, are given below :
1. Bromination + H+

NH2 NH2 Basic Anilinium ion


(intermediate)
Br Br
Br2/H2O
+ 3Br2 + 3HBr However, by protecting the — NH 2 group through
acetylation with acetic anhydride, the nitration can be
Br controlled and p-nitro aniline is obtained as a major
product.
The main problem encountered during electrophilic
substitution reactions of aromatic amines is that these NH2 NHCOCH3
have very high reactivity.
(CH3CO)2O Conc. HNO3/H2SO4
2. Selective Bromination pyridine 288 K

NH2 NHCOCH3
NHCOCH3 NH2
(CH3CO)2O Br2
Pyridine H+
CH3COOH

NHCOCH3 NH2
NO2 NO2
– + p-nitroaniline
OH or H

4. Sulphonation Benzene can be treated with a very


concentrated sulphuric acid to give sulphanilic acid in
Br Br
Major a reversible reaction. The overall reaction is given
below.
3. Nitration Aniline can be treated with a nitric acid to + –
NH2 NH3HSO4
give nitrobenzene.
+
The overall reaction is given below : H /HSO−4 453 – 473 K

NH2 NH2 +
NH2 NH3
HNO3, H2SO4
+ H2O
288 K

NO2
51% SO3H SO−3
Sulphanilic acid Zwitter ion
NH2 NH2
NO2 Hinsberg’s Test
+ + Distinction between primary, secondary and tertiary
NO2 amines is made byC6H 5SO2Cl, which is known as Hinsberg’s
47% 2%
reagent.

@iitjeehelps
NITROGEN CONTAINING COMPOUNDS 711
Primary amines give an insoluble mass which is Aryl Diazonium Salt
soluble in alkali.
O O Methods of Preparation
R N H + C6H5 S Cl C6H5  S N R NH2 N N+Cl−
–HCl
H O O H NaNO2 + HCl
(Insoluble mass)
HNO2

–H2O NaOH (aq) Benzene


diazonium chloride
O
C6H5 S N− R Na+ Chemical Reactions
+ − C2H5OH
O
Soluble 1. C6H5N2Cl ∆

Secondary amines give an insoluble substance, which


remains unaffected by alkali. 2. Formation of aromatic halides
O CuCl
C6H5 Cl
R N H + C6H5 S Cl CuBr
(Sandmeyer reaction)
–HCl
C6H5 Br
R O
+ Cu, ∆
C6H5N2Cl− C6H5 Cl + C6H5 C6H5
O
KI
NaOH (aq) C6H5I
C6H5 S N R Insoluble
due to the absence HBF4
O R ∆
C6H5 F
Insoluble
of H-atoms
Cl
Tertiary amines do not react with benzene sulphonyl
chloride because of the absence of H-atoms Cu/HCl
+ N2 + CuX
Hofmann Mustard Oil Reaction
Gattermann
Amines, when warmed with CS2 , alkyl or aryl thiocyanate, a reaction
characteristic smell of mustard oil is formed.
+ CuCN
H H S 3. C6H 5N 2 CI − → C6H 5CN
| | Sandmeyer reaction
||
C 2H 5  N  H + S == C == S → C 2H 5  N  C SH HOH / H + R  OH
+
4. C6H 5N 2 CI − → C6H 5OH → C6H 5OR
HgCl 2
→ C 2H 5  N == C == S + HgS + 2HCl
Ethyl isothiocyanate + C H
5. C6H 5N 2 CI − →
6 5
C6H 5  N== N C6H5
Uses of Amines NaOH
+
Amines are used as intermediates in drug manufacturing 6. C6H 5N 2 CI − → C6H 5  N== N  OH
and as reagents in organic synthesis.
C H  NH
Aromatic amines are used for manufacture of polymers, 7. C6H 5N 2+Cl −   
6 5 2

dyes and as intermediates for additives in rubber industry.
Quaternary ammonium salts of long chain aliphatic C6H5 N N NH2
tertiary amines are used as detergents.

@iitjeehelps
712 SELF STUDY GUIDE BITSAT

8. Isomerism
OH
CH3 CH 2  NO2 and CH3CH 2 O  N==O
Nitro ethane Alkyl nitrite
C6H5N2+ Cl− +
(ethyl nitrite)

They also show chain, positional and optical isomerism.

C6H5 N N OH Methods of Preparation


1. Nitration of alkane

Reactions Involving Retention R H + HO  NO2 → R  NO2
− H 2O
of Diazo Group ∆
e.g. CH 4 + HNO3 → CH3  NO2
Coupling reactions are the examples of electrophilic
substitution reaction. 2. Action of AgNO 2 on alkyl halide
+
N

N Cl + H OH e.g. R  X + AgNO2 → R  NO2 + AgX
CH3Br + AgNO2 → CH3 NO2 + AgBr
Phenol [O]
R CH == N OH → R  CH 2 NO2

OH
N N OH
–HCl
− Physical Properties
+ Cl + H2O
p- hydroxyazobenzene
(orange dye)
Colourless, pleasant smelling compounds with low
solubility in H 2O and have very high boiling point.
+ −
N N Cl +H NH2 Chemical Reactions
1. Action of heat

OH
N N NH2 CH2 CH2

+ Cl− + H2O CH2 CH2 + HNO2
p-aminoazobenzene H N O
(yellow dye) −
O
Coupling with phenols is carried out in weakly alkaline
medium (pH 9 to 10) because phenate ion produced is 2. Halogenation
coupled with diazonium salt more readily. Above pH 10, Br 2
diazonium salt reacts with hydroxide ion to form diazotate CH3 CH 2 NO2 → CH3 CH  NO2
NaOH |
ion which does not take part in coupling reaction. Br
Uses of Diazonium Salts 3. Reduction
Diazonium salts are used for the preparation of a large ∆
R  NO2 → R  NH 2 + H 2O
number of aromatic compounds viz substitution and Ni-H 2
coupling compounds.
Ni-H 2
CH3  NO2 → CH3  NH 2 + H 2O

Nitroalkane
4. Reaction with HNO2
The nitro derivatives of alkanes are called nitroalkanes.
− CH3CH 2 NO2 + HNO2 → CH3  CH  NO2
+ O + O |
R N R N
O

O
− NO
OH
CH3 NO2 nitromethane +
CH3 C N
CH3 CH CH3 2-nitropropane
N O O–
NO2

@iitjeehelps
NITROGEN CONTAINING COMPOUNDS 713
5. Condensation with carbonyl compounds Substitution Reactions
NaOH (a) Nucleophilic substitution (o and p - direction )
CH3CHO + CH3CH 2 NO2 → CH3 CH  CH  NO2
| | NO2 NO2 NO2
OH CH3 OH

OH
and
Nitrobenzene
OH
Methods of Preparation (b) Electrophilic substitution (m - direction )
C6H6 + HNO3 + H 2SO4 → C6H 5  NO2 NO2
O
 Conc. H2SO4
CH 3 C  ONO 2 C6H5NO2
Conc. HNO3
C6H6 → C6H 5  NO2
NO2
CF3CO 3 H NO2
C6H 5  NH 2 → C6H 5  NO2
Cl2 FeCl3
+ HNO 2 C6H5NO2
C6H 5N 2 Cl 2 → C6H 5  NO2
CuO
Cl
[O] NO2
C6H 5NO → C6H 5NO2
Conc. H2SO4
C6H5NO2
Physical Properties
SO3H
It has bitter almond odour and is also known as oil of
mirbane. Test for Nitrobenzene
(Mulliken and Barker’s Method)
Chemical Properties
C6H5NO2 (alc.) + Zn + CaCl2
Sn/HCl ∆
C6H5 NH2
LiAIH4 2Ag↓ Black ppt [C6H5NHOH]
C6H5 NH2
Filter in AgNO3 + NH4OH
Ni/H2
C6H5 NO2 C6H5 NH2
Uses of Nitrobenzene
Zn/NH4OH
C6H5 NH NH C6H5 It is mainly use in
Hydroazobenzene 1. Manufacture of aniline.
Zn/NH4Cl
H2O
C6H5 NH OH 2. As solvent in Friedel-Crafts reaction.
N-phenyl hydroxylamine
3. As an oxidant in the synthesis of quinoline.

@iitjeehelps
714 SELF STUDY GUIDE BITSAT

Practice Exercise
1. Gabriel phthalimide synthesis can be used for the
NaNO2
preparation of amine from 6. (A);
a. CH3CH2Br b. (CH3 )3 CBr H2SO4
NH2
c. p - CH3OC6H4Br d. p - CH3C6H4Br
Product of this reaction is
2. Hofmann’s bromamide degradation reaction is shown
by
a. ArNH2 a. b.
b. ArCONH2
c. ArNO2
c. d.
d. ArCH2NH2

3. The positive carbylamine test is shown by


a. p -methylbenzylamine 7. Which of the following compounds does not liberate
nitrogen with HNO2?
b. N, N -dimethylaniline
a. Carbamide b. Primary amine
c. 2, 4-dimethylaniline c. Secondary amine d. Alkanamide
d. N -methyl-o -methyl aniline
8. The compound that is most reactive towards
4. What are the constituent amines formed when the electrophilic nitration is
mixture of I and II undergoes Hofmann’s bromamide a. toluene b. benzene
degradation? c. benzoic acid d. nitrobenzene
15 9. Towards electrophilic substitution, the most reactive
CONH2 CONH2 species will be

D NO3 NH2
(1) (2)
a. b.
15
a. NH2 , NH2 , + –
NH3Cl NHCOCH3
D D
15 c. d.
NH2 , NH2

10. When aniline is treated with fuming sulphuric acid at


15 475 K, it gives
b. NH2 , NH2 ,
a. sulphanilic acid
b. aniline sulphate
D c. o-aminobenzene sulphonic acid
15 15
d. m-aminobenzene sulphonic acid
c. NH2 , NH2 , 11. The correct increasing order of basic strength for the
following compounds is [NCERT Exemplar]

15 NH2 NH2 NH2


d. NHD ,

5. The primary, secondary and tertiary amines can be


best distinguished by
NO2 CH3
a. mustard oil reaction (I) (II) (III)
b. carbylamine reaction
c. exhaustive alkylation a. II < III < I b. III < I < II
d. HNO2 treatment c. III < II < I d. II < I < III

@iitjeehelps
NITROGEN CONTAINING COMPOUNDS 715
12. Amino group, —NH 2 is ortho, para-directing group in 20. The best reagent for converting 2-phenylpropanamide
case of aromatic electrophilic substitution but nitration into 2-phenylpropanamine is
of aniline produce a good amount of m-nitroaniline. a. excess H2
This is because b. Br 2 in (aq) NaOH
a.  NH2 gets converted into  NH− NO+2 which is c. iodine in the presence of phosphorus
m-directing d. LiAIH 4 in ether
+
b. NH2 gets converted into NH3 which is m-directing 21. Consider the following reaction sequence.
c.  NH2 gets converted into  NH+ SO−4 which is The final product of this reaction sequence is
m-directing CH3 CH3
d. ortho, para activity of  NH2 group is completely
destroyed during nitration Conc. HNO3 Fe (CH3CO)2O
H2SO4 H+
13. Reduction of aromatic nitro compounds using Fe and
HCl gives
NO2
a. aromatic oxime
Br2 H2O NaNO2 H3PO2
b. aromatic hydrocarbon
H+ HCl
c. aromatic primary amine
d. aromatic amide
CH3 CH3
14. The relative order of basic character of the following
compounds is
a. b.
O S
HPO3
C
N N N N N
H H H O CH3
COCH3
CH3 NHCOCH3
(I) (II) (III) (IV) (V)
a. II > I > III > IV > V b. II > III > IV > V > I
c. d.
c. II > V > IV > I > III d. II > IV > V > III > I
15. Which of the following amine does not react with Br
Hinsberg reagent? CH2NH2
a. Neopentyl amine b. Isopropyl amine
c. Triethyl amine d. Ethyl methyl amine 22. Consider the following series of reactions
CH2CONH2
16. When a primary amine reacts with chloroform in Br ∆
ethanolic KOH, the product is →
2
X → Y.
NaOH
a. an isocyanide b. an aldehyde COOCH3
c. a cyanide d. an alcohol The compound Y is
17. Tertiary nitro compounds do not tautomerise because O
a. there is no double bond a.
b. there is no α-hydrogen NH
c. oxygen is more electronegative than H
d. All of the above O
O
18. When p-toluidine reacts with sodium nitrite and
b.
hydrochloric acid at 273 K, a crystalline precipitate is
formed, which is boiled with water. The resulting
compound obtained is O
a. p -cresol b. p -nitro toluene
c. phenol d. toluic acid c. NH
19. The reagent used to distinguish p-methylaniline from
O
N-methylaniline is
a. benzenesulphonyl chloride O
b. iodoform in alc. KOH d.
c. AgCl O
d. AgNO3 O

@iitjeehelps
716 SELF STUDY GUIDE BITSAT

23. Compound A (C3 H 9N) reacts with benzene sulphonyl 27. Consider the following reaction :
chloride to form a solid insoluble in alkali. The +
N2Cl− Cl
structure of compound A is
a. CH3 — N — CH3 Cu/HCl
| → + N2 + CuCl
CH3
b. CH3 — CH2 — NH — CH3
The above reaction is called
c. CH3 — CH2 — CH2 — NH2
a. carbylamine reaction
d. All of the above b. Gattermann synthesis
24. Aniline yields a coloured product Y through the c. Sandmeyer’s reaction
following series of reactions: d. Balz-Schiemann reaction

NH2 28. Identify the final product (z) in the following sequence
of reactions
NaNO2/HCl (i) LiAlH4 LiAlH4
KCN
273-278 K
X C6H5 COOH → X → Y → Z
(ii) PBr3
N,N-dimethylaniline
Y a. C6H5CH2NH2
(Coloured
product) b. C6H5CH2CH2NH2
c. C6H5CH2CH2NH2
The structure of Y is
CH3 d. C6H5  CH  NH2
a.  N==N  N 
CH3 CH3
CH3 29. Hydrolysis of phenyl isocyanide forms
a. benzoic acid
b. HN  NH  NH
b. formic acid
CH3 c. acetanilide
d. acetic acid
c. HN   N==N  NH 30. Which of the following on reaction with nitrous acid
followed by treatment with NaOH produces a blood
CH3 CH3
red colouration?
d. H3C   N==N   NH2
a. R CH2NO2 b. R3CNO2
c. R2CHNO2 d. PhNO2
25. Consider the following statements:
(CH3)2NH
Phenyl diazonium salts form azo dye with 31. F NO2 (A)
DMF , ∆
I. aniline
(i) NaNO2/HCl,
II. phenol (B), is
0-5°C
III. N, N- dimethyl aniline (ii) H2/Ni
IV. anisole (methoxybenzene)
The correct statements is CH3
a. H2N N
a. II, III and IV are correct b. I, III and IV are correct CH3
c. I, II and IV are correct d. I, II and III are correct
26. Benzylamine may be alkylated as shown in the CH3
b. O2N N
following equation
C6H5 CH 2NH 2 + R X → C6H5 CH 2NHR H 2N
Which of the following alkyl halides is best suited for
this reaction through S N 1 mechanism? c. H2N NH2
a. CH3Br
b. C6H5Br
c. C6 H5CH2Br d. O2N NH2
d. C2 H5 Br

@iitjeehelps
NITROGEN CONTAINING COMPOUNDS 717

BITSAT Archives
1. Arrange the following in correct order of basicity [2014] 6.
O
HN
•• •• H 3C
••
N N N CH3
Bromination
Product
H H [2007]
I II III
O
a. I > II > III b. III > II > I HN
c. II > I > III d. I > III > II H3C
a. CH3
H2 SO4
2. C6H5 NH 2 → NH 2C6H4 (SO3 H)
180°C
para form [2014]
Br
The true statement about the product is
a. it does not exist as Zwitter ion O
HN
b. it does not act as inner salt
c. SO3 diminishes the basic character of  NH2 b. H3C CH3
d. NH2 displays a powerful basic character

3. Identify C in the following reaction: [2012]


Br
NO2 H O
N
Sn/HCl NaNO2 NaNH2
A B C H3C CH3
c.
Br
a. benzamide b. benzoic acid
c. chlorobenzene d. aniline H O
4. The structure of the compound formed, when N
nitrobenzene is reduced by lithium aluminium hydride CH3
(LiAlH4 ) is [2008] d.
Br
NHOH

7. Aniline reacts with conc. HNO3 to give [2007]


a. N N b.

H H a. H2N NH2
NH2

NH2 NH2
c. N N d.
NO2
b. and
5. The IUPAC name of the compound, [2007]
CH2  CH  COOH NO2
 
OH NH 2 c. O O
a. 2-amino-3-hydroxy propanoic acid
b. 1-hydroxy-2-aminopropan-3-oic acid NO2
c. 1-amino-2-hydroxypropanoic acid
d.
d. 3-hydroxy-2-amino propanoic acid

@iitjeehelps
718 SELF STUDY GUIDE BITSAT

Answer with Solutions


Practice Exercise 11. (d) Electron releasing (+I-effect) groups increase the
basicity of amines while electron attracting (−I-effect)
1. (a) By the use of Gabriel phthalimide synthesis, 1° alkyl groups decrease the basicity of amines.
halides can be converted into corresponding 1° amines.
Hence, the order is II < I < III
2. (b) Hofmann’s bromamide degradation reaction is shown +
by amides, i.e. ArCONH2. 12. (b) Protonation of NH2 leads to the formation of NH3
3. (a) Carbylamine test is given by only primary amines. which is m - directing group.
Therefore, p-methylbenzylamine gives response to Fe/HCl
carbylamine test. 13. (c) C6H5 —NO2 → C6H5 — NH2
Reduction Aromatic primary (1° ) amine
4. (b) 15
CONH2 + CONH2 14. (c)
S
D > >
Br2 + KOH 15 N N
NH2 + NH2 H H
No delocalisation Presence of S
of electron pair (electronegative element
D decreases electron density on N)
(π-electrons)
5. (d) Primary, secondary and tertiary amines on reaction O
with HNO2 produces different products.
> >
6. (b) N
NaNO2 N N
O is more electronegative 2
H2SO4 N is sp -hybridised
than S, hence further electron pair on N
C
NH2
N NHSO4– decreases availability delocalised over ring HC3 O
Diazonium salt of electron pair on N
Electron pair on N
H+ delocalised to O

15. (c) Triethyl amine is tertiary amine. It does not react with
Dehydration
Hinsberg reagent.
16. (a) R — NH2 + CHCl3 + 3KOH →
OH 1° amine
7. (c) R2NH + HNO2 → R2N⋅ NO + H2O R —N→
N-nitroso compound == C+ 3KCl + 3H2 O
Alkyl isocyanide
8. (a) Toluene has electron donating methyl group and
17. (b) Tertiary nitro compounds do not tautomerise, since
hence, reacts fastest.
there is no α-hydrogen.
9. (b) +I-effect groups increase electron density on o and
p -positions in benzene ring, hence they favour the 18. (a)
+
electrophilic substitution. NH2 N2Cl– OH
10. (a) This is the reason why aniline is most reactive among
given compounds. NaNO2 + HCl H 2O
+ N2 + HCl
+ – 273 K
NH2 NH3HSO4
CH3 CH3 CH3
+ H2SO4 p-cresol

+
Aniline Aniline NH3 19. (a) p-methylaniline is a primary amine while
hydrogen sulphate N-methylaniline is a secondary amine. Primary amines
455-475 K
can be distinguished from secondary amines by
benzenesulphonyl chloride.
20. (d) CH3 CH3
+
SO3 | LiAlH4
|
Sulphanilic acid CH—CONH2 CH—CH2NH2
Ether
(Zwitter ion structure)

@iitjeehelps
NITROGEN CONTAINING COMPOUNDS 719
+
21. (c) CH3 CH3 CH3 25. (d) Ph N ≡≡ N is a weak electrophile that undergoes
diazocoupling only with rings activated by OH , NH2,
Conc. HNO3 Fe (CH3CO)2O
NHR or NR2. The ring is not sufficiently activated by
H2SO4 H+ OCH3 and hence, anisole does not form azodye.

NO2 NH2 26. (c) The species C6H5 CH2Br on ionisation produces,
+
C6H5CH 2 (benzyl carbocation), therefore, C6H5CH2Br is
CH3 CH3 CH3 best suited for SN1 reaction.
27. (b) +
Br2 H2O N2Cl− Cl
H+
Br Br Cu/HCl
+ N2 + CuCl
NHCOCH3 NHCOCH3 NH2
CH3 CH3 The above reaction is called Gattermann reaction.
LiAlH4 PBr3
NaNO2 H3PO2 28. (b) C6H5COOH → C6H5CH2OH →
Reduction Benzyl alcohol
HCl
Br Br KCN
+
N2Cl
− C6H5CH2Br →
X
22. (c) LiAlH4
CH2CONH2 CH2NH2 C6H5CH2CN → C6H5CH2CH2NH2
(i) Br2/NaOH Y Reduction Z
OCH3
COOCH3 29. (b)  O 
O  || 
(x) Ph  NH C  H
(ii) ∆ + − H+
 
NH + CH3OH Ph  N ≡≡ C: →  
H O 2

O
(y) H+ /H2O
→ Ph  NH2 + HCOOH
23. (b)
O NOTE Isocyanides are hydrolysed only by acids, and not by alkalies.
CH3
–HCl It is because negative charge present on the carbon atom in
S— Cl + H —N isocyanides initially attracts electrophile. (i.e H+ )
CH2—CH3 + − H2O
O R  N ≡≡ C : + H + → R  N ≡≡ CH →
Benzenesulphonyl N-methylmethanamine H+
chloride (2°amine)
H O
O | Tautomerises
||
CH3 R  N == C  OH → R  NH C  H
S—N +
H / H2O
CH2—CH3
O R — NH2 + HCOOH
N-ethyl, N-methylbenzene sulphonamide 30. (a) R  C H2 + HNO2 →
(insoluble in alkali) |
24. (a) NO2

+
CH3 R — C == N  OH
– |
NH2 N2Cl N— (N, N-dimethyl
aniline) NO2
NaNO2/HCl CH3 Nitrolic acid
273-278 K Nitrolic acid dissolve in NaOH to give blood red
Aniline (X) CH3 colouration.
N==N— —N R
CH3
|
R  C R + HNO2 → No reaction
|
(Y) NO2

@iitjeehelps
720 SELF STUDY GUIDE BITSAT

R 31. (a)
| (CH3)2NH
F NO2
R C  H + HO  N == O → DMF, ∆
|
NO2
R H 3C (i) NaNO2/HCl, 5°C
| N NO2
H 3C (ii) H2/Ni
R  C  N== O + H2O
|
NO2 H 3C
N NH2
(Pseudonitrole) H 3C
Does not dissolve in NaOH but
give blue colouration

PhNO2 + HNO2 → No reaction

BITSAT Archives
1. (c) Hybridisation of N-atom in the compounds given below 3. (d) NO2 NH2
aresp 2,sp 3 andsp 2 respectively.
Sn/HCl NaNO2
HCl, 0°C
•• •• ••
N N sp 2 N
sp 2 sp 3 (A)
Aniline
H H
I II III + −
N2Cl NH2
Greater the s -character, more will be electronegativity of
N-atom and lesser will be its basicity on this basis, I is less NaNH2
basic than II.
Conjugation If lone pairs of electrons of N are involved in (B) (C)
conjugation causes decrease in basicity of compound due Diazonium Aniline
to lesser availability of lone pair for donation to show basic salt
nature.
4. (c) Nitrobenzene on reduction with lithium aluminium
s hydride (LiAlH4 ) gives azobenzene.

•• ••
NO2
•• ••
N N N Nr
LiAIH4
H H H
Lone pair involved in formation of aromatic sextet of Nitrobenzene
6π-electron (least basic).
2. (c) NH 2 NH2. H2SO4 N N
Azobenzene
H2SO4 180°C
– H2 O 5. (a) C H2CH  COOH
+ | |
NHSO3H NH2 NH3
OH NH2
∆ 2-amino-3-hydroxy propanoic acid
S
6. (b)  NH is stronger electron releasing group than
–  CH3 group, therefore bromination will take place at
SO3H SO3 p- position with respect to  NH group.
Sulphanilic acid
HNO3
Sulphanilic acid exists as a dipolar ion which has acidic, 7. (c) NH2 O O
[O]
as well as basic groups in the same molecules. Such ions
are called Zwitter ions or inner salts. Benzoquinone

@iitjeehelps
29
Polymers, Biomolecules
and Chemistry in Action

Polymers
These are defined as very large molecules having high molecular mass (103 - 107 u ). These are formed
by joining of repeating structural units, called monomers, on a large scale. The monomers units are
linked to each other by covalent bonds. The process of formation of polymer from respective
monomers is called polymerisation.
Polymerisation
Monomer ¾¾¾¾¾® Polymer
Polymerisation
e.g. nCH 2 == CH 2 ¾¾¾¾¾® ¾
( CH 2 ¾ CH 2 ¾) n
Ethylene Polyethylene

where, n = 1000

Classification
Based on Structure
(i) Linear polymer Long straight chain, joining of monomers. They have high melting point,
density and tensile strength, e.g. polythene ¾
( CH 2 ¾ CH 2 ¾)n .
(ii) Branched chain polymer Made up of a main chain and its branches. They have low melting
point, density and tensile strength, e.g. low density polythene, PMMA, neoprene etc.
(iii) Cross-linked polymer Joining of linear chains to form a 3-D network. They are hard and
brittle in nature, e.g. bakelite.

Based on Synthesis
(i) Addition polymers In this case during linking of monomers, there is no elimination of
smaller molecules. Polymerisation takes place usually in the presence of peroxide and follows
free radial mechanism.
Molecular weight of polymer = n ´ Molecular weight of monomers

@iitjeehelps
722 SELF STUDY GUIDE BITSAT

e.g. Formation of PVC (polyvinyl chloride) The addition or chain growth polymerisation can proceed
é ù by the following two mechanisms (i) By the formation of
Polymerisation ê ú free radicals, i.e. free radical polymerisation and (ii) By the
nCH 2 ==CH ¾ Cl ¾¾¾¾¾® êCH 2 ¾ C H ú formation of ionic species, i.e. ionic polymerisation.
Vinyl chloride ê ½ ú
êë Cl úû n Condensation Polymerisation
PVC
In this mode of polymerisation, there is combination of
molecules of same or different monomers in a controlled
(ii) Condensation polymers This is a step-growth
stepwise manner with the elimination of small molecules
polymerisation in which polymer is formed by
such as water, ammonia, alcohol etc., as by-product. These
linking of monomer units along with elimination of
smaller molecules, such as H 2O, NH3 , HCl etc. monomers have functional groups such as alcohol,
amine or carboxylic acid groups, instead of double bonds.
—H 2O
e.g. Phenol + Formaldehyde ¾¾® Bakelite Since, there is a repetitive condensation reaction between
bifunctional monomers, the process is also known as step
Hexamethylene —H 2O
+ Adipic acid ¾¾® Nylone-66 growth polymerisation.
diamine
Copolymerisation
Based on Source When two or more different monomer units are allowed to
(i) Natural polymers These are obtained from polymerise, a copolymer is formed which contains multiple
animals and plants, e.g. starch, protein, natural units of each monomer, used in the same polymeric chain.
rubber, nucleic acid, cellulose etc. The process by which copolymers are formed, is referred to
(ii) Synthetic polymers Man-made, synthesised in lab, as copolymerisation.
from low weight compounds, e.g. synthetic rubber,
nylon, bakelite etc. Molecular Weight of Polymers
Based on Interparticle forces (a) Number average molecular weight ( M n )
(i) Elastomers Weakest intermolecular forces. These Total weight of molecules n 1 M 1 + n 2 M 2 + K
Mn = =
have elastic character and can be stretched with Total number of molecules n 1 + n 2 + ....
retainment of structure, e.g. natural rubber. SMini
\ Mn =
(ii) Fibres Quite strong interparticle forces such as SN i
H-bonds, e.g. nylon, dacron.
(b) Weight average molecular weight ( M w )
(iii) Thermoplastics Polymers which can easily be w M + w 2M 2 + K
moulded by heating and subsequent cooling. Mw = 1 1
w 1 + w 2 + ....
Intermolecular forces are intermediate to those of
elastomers and fibres, e.g. polythene and \ Mw > Mn
polystyrene.
(iv) Thermosetting polymers These become hard and Homo and Copolymers
influsible on heating due to excessive cross-linking. Homopolymers are formed from the same monomers.
e.g. bakelite, malamine.
e.g. Polythene, PVC etc.
General Methods of Polymerisation Copolymers are formed from more than one type of
These are the following methods through which monomers monomers.

combine to give polymers : e.g. (Styrene + 1, 3-butadiene) ¾®


Styrene butadiene rubber (SBR)
Addition Polymerisation
It is the process in which molecules of same or different Atactic, Syndiotatic and Isotatic
monomers combine together to give a polymer without the
elimination of small molecules. Since, there is an increase in the
Polymers
length of chain (i.e. chain grows) by this mode of When substitutents are arranged in a random fashion, it is
polymerisation, it is also known as chain growth called an atactic polymer, e.g. polypropylene. When
polymerisation. This mode of polymerisation is observed in substituents are arranged in a regular fashion, it is called
molecules having multiple bonds, mainly in CH 2 == CHy type syndiotactic polymer, e.g. polystyrene. Isotatic polymers
are those in which all substituents are on the same side of
molecules (where,y may be ¾ H, ¾ X, ¾ CO 2 R, ¾ CN etc)
the chain, e.g. PVC

@iitjeehelps
POLYMERS, BIOMOLECULES AND CHEMISTRY IN ACTION 723

Some Important Polymers 5. Terylene (Dacron)


O O
1. Natural rubber
HOCH2CH2 OH + nHO  C  C OH
It is a natural polymer obtained from rubber latex
Ethylene glycol
(contains 35% rubber). It is a colloidal dispersion of rubber Terephthalic acid
in water and is obtained from rubber trees (Hevea
brasiliensis). O O
Polymerisation
It is a polymer of isoprene (2-methyl-1,3-butadiene) and is OCH2CH2OC C
also known as cis-1, 4-polyisoprene.
CH3 Terylene or Dacron n

½ Polymerisation
n CH 2 == C ¾ CH == CH 2 ¾¾¾¾¾® Properties and Uses
2-methyl-1,3-butadiene (isoprene) Terylene is highly resistant to the action of chemical and
æ H3C biological agents. Its fibres are quite strong and durable. It
H ö
ç ‚ ƒ ÷ can also be blended with wool or cotton to obtain fabrics of
ç C == C ÷
ƒ desired composition. Terylene is used in the manufacturing
¾çèH 2C CH 2 ÷ø¾
‚
n of a variety of clothes such as terycot, terywool and terysilk
as a result of blending with other yerns. It is also used for
NOTE The average molecular weight (Mw ) of rubber is in the range preparing magnetic recording tapes, conveyer belts, aprons
1,30,000 - 3,40,000. Its trans-form is known as gutta percha.
for industrial workers etc.
Natural rubber ¾® cis-polyisoprene
Gutta percha ¾® trans-polyisoprene. 6. Bakelite
—H 2O
2. Neoprene (Synthetic rubber) nC6H 5 ¾ OH + nHCHO ¾¾®
+
Phenol Formaldehyde H
nCH 2 == C ¾ CH == CH 2 ¾Polymerisation
¾¾¾¾ ¾®
OH OH OH
½
Cl CH2 CH2 CH2
Chloroprene
æ Cl ‚ ƒ
CH 2 ö¾
ç C == C ÷
¾¾
è H 2C ƒ ‚
H øn
CH2 OH
Uses In household and mechanical rubber articles.

3. Buna-S
nCH 2 == CH ¾ C6H 5 + nCH 2 == CH ¾ CH == CH 2 H2C CH2
Styrene 1 , 3 -butadiene
½ CH2
¯
Uses In electrical switches, fuse holders etc.
¾
( CH 2 ¾ CH == CH ¾ CH 2 ¾ CH ¾ CH 2 ¾)n
½ 7. Nylon-6 or Perlon
C6H 5
It is obtained by heating caprolactum with water at a high
Uses In rubber soles, water proof shoes, tyres, etc. temperature.
O NOH
4. Nylon—66
O O NH2OH
Oxidation
½½ ½½ O2
nH 2 N ¾ (CH 2 )6 ¾ NH 2 + nHO ¾ C ¾ (CH 2 )4 ¾ C ¾ OH
Hexamethylene Adipic acid Cyclohexanone Cyclohexanoxime
diamine O
O
C
— nH 2O ½½ H2C NH
¾¾® ( ¾ NH ¾ (CH 2 )6 ¾ NH ¾ C ¾ (CH 2 )4 ¾ C ¾ )n H2SO4
Nylon —66 ½½ (Beckmann CH2
O rearrangement)
H2C CH2
Uses In carpets, textile fibres, brushes, hosiery, etc. CH2
Caprolactum

@iitjeehelps
724 SELF STUDY GUIDE BITSAT

O
The process involves the heating of raw rubber with
C sulphur and appropriate additive at a temperature
H2C NH
373 - 415 K.
H2O
CH2
Heat
H2N(CH2)5COOH Sulphur forms cross links at reactive sites of double
H2C
bonds and makes the rubber stiffened. Rubber made
CH2
with 1-3% sulphur is soft and stretchy and rubber
CH2
Caprolactum
made with 3-10% S, is more rigid (used in making
tyre).
 O
  CH3
Heat   

 HN  (CH2 )5  C  CH2  C  CH  CH2
n
Nylon-6  
S S
8. Glyptal  
CH2  C  CH  CH2
HOOC COOH 
CH3
nHO CH2 CH2 HO + n
Glycol
Phthalic acid CH3

  CH  C 
 CH  CH2
 O O  
  S
–nH2O  O CH2CH2 O C C O  
∆  
  CH  CH 
 CH  CH2
 
 
 n
Biomolecules
Uses In paints and lacquers.
Biomolecules are the complex organic compounds
9. Melamine (Formaldehyde resin) which form the basis of life for growth and
  maintenance of living organism. e.g. carbohydrates,
N   proteins, vitamin, hormones, etc.
 HN N
H2 N NH2 Poly  NH CH2 
+ n HCHO The branch of science which deals with the study of
N N  N N 
  biomolecules and their role in living systems is called
  biochemistry.
NH2  NH2 
 n
Carbohydrates
Vulcanisation of Rubber These are polyhydric aldehydes and ketones which
can be represented by general formula C x (H 2O) y
Natural rubber becomes soft at high temperature (>335 K) and
[where x , y > 4].
brittle at low temperature (< 283 K). It has high water absorbing
capacity. It is easily attacked by oxidising agents and soluble in Classification
non-polar solvents. To improve these properties, vulcanisation of
rubber is carried out.
Carbohydrates
Depending upon the units
produced after hydrolysis

Monosaccharides Oligosaccharides Polysaccharides


Named systematically (Oligos-few) Yield 2-9 Hydrolysed to more than
according to the number of monosaccharides 9-monosaccharides.
carbon atoms they contain. on hydrolysis.
Crystalline solids, soluble in water and Amorphous solids,
sweet in taste and collectively called insoluble in water
sugars. and tasteless and
thus are called
non-sugars.
Except-sucrose, all are reducing-sugars. All polysaccharides
are non-reducing sugars.

@iitjeehelps
POLYMERS, BIOMOLECULES AND CHEMISTRY IN ACTION 725

Monosaccharides
A carbohydrate that cannot be hydrolysed further to give simpler units is called monosaccharide, e.g. glucose, fructose,
galactose are hexose while ribose and arabinose are pentose. All monosaccharides are reducing sugars, commonly known
as glucose.

Preparation of Glucose
H+
(i) C 12H 22O11 + H 2O ¾¾¾¾® C6H 12O6 + C6H 12O6
(vi) CHO CH  NNHPh
or invertase  –H2O

Sucrose Glucose Fructose
+ (CHOH)4 + H2N⋅NH⋅C6H5 H  C  OH
H
(ii) (C6H 10O5 )n + nH 2O ¾¾¾¾¾® nC6H 12O6 

393 K, 2-3 atm Glucose (CHOH)3
Starch CH2OH 
CH2OH
Properties of Glucose Glucose
phenylhydrazone
(i) Glucose is an aldohexose and is also known as
CH NNHPh
dextrose (Grape sugar).  2H2N⋅NHPh
CHO PhNH2 + NH3 + C  NNHPh
| HI, D 
(ii) (CHOH)4 ¾¾® CH3 ¾ ( CH 2 ¾ )4 CH3
(CHOH)3
| n -hexane
CH 2OH 
CH2OH
CHO COOH Glucosazone
| Br2 / water | NOTE Glycolysis is the anaerobic degradation of glucose into two
(iii) (CHOH)4 ¾¾¾¾® (CHOH)4 molecules of pyruvic acid.
| |
CH 2OH CH 2OH
Gluconic acid Structure of Glucose
CHO CHO
(i) Fisher Projections
| (CH3CO) 2O |
(iv) (CHOH)4 ¾¾¾¾® (CHOCOCH3 )4 O
1 α 1
| ½ O H C OH H C
CH 2OH ½ || 2 2
CH 2 — O— C — CH3 H OH O H OH
3 3
Glucose
glucose pentaacetate HO H HO H
4 4
H OH H OH
5 5
CHO COOH H H OH
| | 6 6
HNO 3 CH2OH CH2OH
(v) (CHOH)4 ¾¾¾® (CHOH)4 α-D-(+)-glucose Glucose
oxidation
| | 1 β
CH 2OH COOH HO C H
Glucaric acid or 2
saccharic acid H OH O
3
COOH HO H
| H
4
OH
oxidation
¾¾¾¾® (CHOH)4 5
H
| 6
CH 2 ¾ OH CH2OH
Gluconic acid β-D-(+)-glucose

@iitjeehelps
726 SELF STUDY GUIDE BITSAT

(ii) Haworth Structures Structure of Sucrose


6 6 CH2OH
CH2OH CH2OH
H 5 O H
H 5 O H H 5 O OH
H 1 Glucose unit
H H 4
4 1 α 4 1β OH H α
OH H OH H 2
3
3 2 HO O
HO OH OH 3 2 H
H OH Glycosidic linkage
H OH H OH 6
α-D-(+)-glucose β-D-(+)-glucose HOH2C O
O β
● a and b-D-glucose have different configuration at 5 2
anomeric (C-1) carbon atom, hence are called anomers. H OH
H4 3 CH OH Fructose unit
● While the pair of diastereomeric aldoses, e.g. glucose and 1 2
mannose that differ only in configuration about C-2 are OH H
called epimers. Glucose and galactose differ in
configuration at C4 and are called C4 - epimers.
Maltose (C 12H 22O 11 )
Structure of Fructose (Laevulose; Fruit Sugar)
It is obtained by partial hydrolysis of starch by diastase
It is a functional isomer of glucose and has ketone group.
enzyme present in malt, i.e. sprouted barley seeds
CH2OH (hence named as maltose or malt sugar).
O Diastase
2(C6H 10O5 )n + nH 2O ¾¾® n C 12H 22O11
HO H Maltose
H OH
● It is a white crystalline solid (with m.p. 160-165°C),
H OH
soluble in water and dextrorotatory. When it is
CH2OH hydrolysed with dilute acid or by enzyme maltose,
D-(–) – fructose maltose yields two molecules of D-(+)-glucose.
Hence, maltose is a condensation product of two
2. Oligosaccharides a-D-glucose units.

These are the carbohydrates that yield two to ten 6 6


CH2OH CH2OH
monosaccharides units on hydrolysis.
H 5 O H H 5 O H
e.g. Disaccharides: sucrose, maltose, lactose etc.
H H
Trisaccharides : raffinose; tetrasaccharides : stachyrose. 4 1 4 1
● Except sucrose, all other disaccharides are reducing in
OH H OH H
O
nature and hence, are called reducing sugars. HO 3 2 3 2 OH

H + H OH H OH
C 12H 22O11 + H 2O ¾® C6H 12O6 + C6H 12O6 (+) - Maltose
Sucrose D-(+)– glucose D-(–)-fructose
● It is a reducing sugar. It reduces Fehling’s solution,
H+
C 12H 22O11 + H 2O ¾® 2 moles of a D-glucose Tollen’s reagent, it forms an oxime and an osazone and
Maltose undergoes mutarotation. This indicates that at least one
aldehyde group is free in maltose.
(a) Sucrose
It is a non-reducing due to absence of free aldehyde or
ketone group. It is cane sugar or table sugar. Lactose (C 12H 22O 11 )
● Sucrose is also known as invert sugar. It is due to the fact It occurs in the milk of all animals (milk-sugar). It is a white
that on hydrolysis (+) sucrose gets inverted to give a crystalline solid (with m.p. 203°C), soluble in water and is
mixture of D-(+) -glucose and D-(–) –fructose. dextrorotatory.
● In sucrose, free aldehyde or ketone group is absent. It is It is hydrolysed by dilute acid or enzyme lactose, to an
shown by the facts that it does not form osazone, does equimolar mixture of D-(+)-glucose and D-(+)-galactose. It
not exist in anomeric forms and also does not show is a reducing sugar, forms an oxime and osazone and also
mutarotation. undergoes mutarotation.

@iitjeehelps
POLYMERS, BIOMOLECULES AND CHEMISTRY IN ACTION 727
It gets hydrolysed by emulsion also, an enzyme which 2. Non-essential amino acids are the amino acids
specifically hydrolyses b-glycosidic linkage. which can be synthesised in the body, e.g. glycine,
alanine, glutamic acid, aspartic acid etc. Amino
CH2OH H OH acids behave like salts rather than simple amines or
OH
5 O 5 OH carboxylic acids. This is due to the presence of both
O acidic and basic groups in the same molecule.
H OH H
O O
OH H H ½½ ½½
-
H
H H 3 O R — CH — C — OH eR— CH — C — O
H ½ ½
H OH CH2OH NH2 NH3+
(+) – lactose (N-terminal) (C-terminal) (Zwitter ion)
or dipolar ion (neutral)

Polysaccharides At a certain pH of the medium, called the


isoelectric point of an amino acid, the structure
Carbohydrates which yield a large number of
behaves as a dipolar ion and does not migrate to
monosaccharides units on hydrolysis, are called
any electrode on passing current.
polysaccharides, e.g. starch, cellulose, glycogen, gums arabic
etc. Polysaccharides are not sweet in taste, hence they are also Proteins give biuret test, Millon’s test, Ninhydrin
called non-sugars. test. Those proteins give Mollisch’s test, which
contain a carbohydrate group. Carbohydrates also
Moreover, all polysaccharides are non-reducing due to the
‚
give Mollisch’s test.
absence of free ¾ CHO or CO group.
ƒ

Starch is the main storage polysaccharides of plants. It is a


Classification of Proteins
polymer of a-D-glucose units and consists of two Proteins can be classified into two types on the basis of
components-amylose and amylopectin. their molecular shape :
Cellulose is a predominant constituent of cell wall of plant (i) Fibrous proteins Polypeptide chains form fibre
cells. It is a straight chain polysaccharide composed only of like structure, e.g. keratin and myosin etc.
b-D-glucose units which are joined together by (ii) Globular proteins This structure results when
b-1,4-glycosidic linkage, i.e. the b-glycosidic linkages between the chains of polypeptides coil around to give a
C-1 of one glucose and C-4 of the next glucose unit. spherical shape. These are usually soluble in
Glycogen is the carbohydrate (a condensation polymer of water, e.g. insulin and albumins.
a-D-glucose) which is stored in animal body. When the body
needs glucose, enzymes break the glycogen down to glucose. Structure of Proteins
N-terminal of one amino acid combines with
● Wood contains 50% cellulose, rest being lignin and resins. In
C-terminal of other amino acid and forms peptide bond
cotton, cellulose is 90%, the rest being fats and waxes.
Cellulose is not digestible by humans due to the absence of (¾ NHCO ¾ ).
enzyme cellulase in digestive system. In the same way, many amino acids combine to each
● Starch and cellulose are homopolysaccharides while gums other and form polypeptide bonds. Polypeptides with
and pectins are heteropolysaccharides. Starch consists of fewer amino acids are likely to be called proteins.
two compounds : amylose (20%) and amylopectin (80%). Structure and shape of proteins may be discussed at four
● Gums arabic are acidic polysaccharides. different levels:
Proteins 1. Primary Structure
These are the polymers of amino acids (the compounds which Proteins may have one or more polypeptide chains.
have both the acid and amino group). The total number of Specific sequence of amino acids in a chain gives the
amino acids that have been found in proteins are twenty. primary structure of that protein. Any change in this
On the basis of their synthesis, amino acids are divided into two structure gives a different protein.
classes
2. Secondary Structure
1. Essential amino acids are the amino acids which cannot
be synthesised in the body and must be obtained through The polypeptide chains are linked by hydrogen bonds.
diet, e.g. valine, leucine, lysine, isoleucine, arginine etc. They are found to exist in two different types of
structures viz a-helix and b-pleated sheet structure.

@iitjeehelps
728 SELF STUDY GUIDE BITSAT

3. Tertiary Structure Enzymes


It has polypeptide bonds, hydrogen bonds, disulphide Enzymes are globular protein bodies, which are biological
linkages, van der Waals’ forces and electrostatic forces of catalysts. Enzyme inhibitors reduce the activity of a
attraction. It gives rise to two major molecular shapes viz particular enzyme. These are mostly inorganic ions or
fibrous and globular. complex organic molecules.
NOTE Congenital and albinism diseases are caused by the
4. Quaternary Structure deficiency of the enzymes, phenyl ketone urea and tryosinase
Some of the proteins are composed of two or more respectively.
polypeptide chains referred to as subunits. The spatial
arrangement of these subunits with respect to each other is Vitamins
known as quaternary structure.
Organic compounds required in the diet in the small
amounts to perform specific biological functions for
Denaturation of Proteins normal maintenance of optimum growth and health of the
Disturbance of hydrogen bonds either by acids or alcohols organism, are termed as vitamins.
or heat, results in unfolding of globules. Thus, helix get Types of Vitamins
uncoiled and protein loses its biological activity due to the
change in temperature or pH. This is called denaturation of ● Fat or oil soluble vitamins, e.g. A D, E and K.
proteins. During denaturation, secondary and tertiary ● Water soluble vitamins, e.g. B group vitamins and
structures of proteins are destroyed while primary vitamin C.
structures remain intact.

Some Important Vitamins, their Sources and their Deficiency Diseases


Name of Vitamin Sources Deficiency Diseases

Vitamin-A (Retinol) Fish liver oil, carrots, butter and milk Xerophthalmia (hardening of cornea of eye) night blindness,

Vitamin-B 1 (Thiamine) Yeast, milk, green vegetables Beri-beri (loss of appetite)

Vitamin-B 2 (Riboflavin) Milk, egg white, liver, kidney Cheilosis (fissuring at corners of mouth and lips)

Vitamin-B 6 (Pyredoxine) Yeast, milk, egg yolk, cereals Convulsions, nervousness

Vitamin-B 12 (Cyanocobalamine) Meat, fish, egg and curd Pernicious anaemia (RBC deficient in haemoglobin)

Vitamin-C (Ascorbic acid) Citrus fruit , amla and green leafy vegetables Scurvy (bleeding gums)

Vitamin-D Exposure to sunlight, fish and egg yolk Rickets and osteomalacia

Vitamin-E Wheat, germ oil, sunflower oil Increased fragility of RBC and muscular weakness

Vitamin-K Green leafy vegetables Increased blood clotting time.

Nucleic Acids ● Nucleoside has deoxyribose sugar and nitrogenous base.


A unit formed by the attachment of a base to 1¢ position
These are the biological polymers. They function as the
of sugar is known as nucleoside.
chemical carriers of cell’s genetic information.
● In nucleosides, the sugar carbons are numbered as
DNA (Deoxyribose nucleic acid) 1¢ , 2 ¢ , 3 ¢ etc., in order to distinguish these from the bases.
Nucleic
When nucleoside is linked to phosphoric acid at
acids RNA (Ribose nucleic acid)
5¢-position of sugar moiety, we get a nucleotide.
● Nucleotides are joined together by phosphodiester
linkage between 5¢ and 3¢ carbon.
DNA (Deoxyribose Nucleic Acid)
Adenine (A)
● It is the polymer of nucleotide. Purine
Nitrogenous Guanine (G)
● It is a genetic material.
base in DNA Thymine (T)
● It has double helical structure.
Pyrimidine
● Nucleotide has deoxyribose sugar, phosphate and Cytosine (C)
nitrogenous base. ● DNA has A == T, C ºº G

@iitjeehelps
POLYMERS, BIOMOLECULES AND CHEMISTRY IN ACTION 729

RNA (Ribose Nucleic Acid) (b) Chlorodiazepoxide and meprobamate, are relatively
mild tranquilisers, suitable for relieving tension.
● It is also a polymer of nucleotide units but in it, the Equanil is used in controlling depression and
nucleotide unit contains ribose sugar instead of hypertension.
deoxyribose sugar.
(c) Barbiturates such as seconal, luminal, veronal are
● RNA has uracil (U ) instead of thymine (T ). hypnotic (sleep producing) drugs while valium and
● RNA molecules are of three types. These are messenger serotonin, are non-hypnotic drugs.
RNA (mRNA), ribosomal RNA (r RNA) and transfer RNA
(ii) Analgesics These reduce or abolish pain.
(tRNA). They perform different functions.
These are classified as follows :
Biological Functions of Nucleic Acids (a) Non-narcotic (non-addictive) analgesics, e.g. aspirin
● DNA has an ability of self duplication during cell and paracetamol are analgesics as well as
division and identical DNA strand are transferred to antipyretics (fever reducing).
daughter cell. In this way, DNA is responsible for
NOTE Aspirin (acetyl salicylic acid) and paracetamol (N-acetyl-p-amino
maintaining the identity of different species of phenol) are non-narcotic (non-addictive) analgesics. Aspirin
organisms over million of years. inhibits the synthesis of chemicals, known as prostaglandins
● RNA molecules synthesised various types of proteins in which stimulate inflammation in the tissue and cause pain.
the cell but the message for the synthesis of a particular Because of its anti blood clotting action, aspirin is also used in
type of protein is present in DNA. the prevention of heart attacks.
(b) Narcotic analgesics, e.g. morphine, heroin
Chemicals in Medicines (morphine diacetate), codeine etc.
Drugs are chemicals of low molecular masses, which
interact with macromolecular targets (carbohydrates, Antimicrobials
proteins and nucleic acids) to produce a biological
(i) Antibiotics These are the chemicals synthesised from
response. When the biological response is therapeutic
microbes and have either cidal (killing) effect or a static
and useful, these chemicals are called medicines.
(inhibitory) effect on microbes.
Different Classes of Drugs
A few examples of the two types of antibiotics are as follows:
Some important classes of drugs are given below :
(a) Bactericidal, e.g. penicillin (a narrow spectrum
Antacids antibiotic), ampicillin and amoxicillin
(semisynthetic modification of pencillin), ofloxacin
Treatment for acidity was administration of antacids.
(broad spectrum), aminoglycosides (streptomycin)
Antacids raise the pH of stomach to some appropriate
(broad spectrum) etc.
level. Sodium hydrogen carbonate, a mixture of
aluminium and magnesium hydroxide etc., are the (b) Bacteriostatic, e.g. erythromycin, tetracycline,
commonly used antacids. chloramphenicol (a broad spectrum antibiotic) etc.
(ii) Antiseptics These are applied to the living tissues such
Antihistamines as wounds, cuts, ulcers and diseased skin surfaces,
Antihistamines or antiallergic drugs are used as e.g. furacine, soframycine, dettol (a mixture of
treatment for allergies. Some synthetic drugs, chloroxylenol and a-terpinol), 0.2 per cent solution of
brompheniramine (dimetapp) and terfenadine (seldane), phenol. Bithionol (also called bithional) is added to
also act as antihistamines. Histamine also stimulate the soaps, iodine, iodoform etc.
secretion of gastric juice (in stomach) but antihistamines
(iii) Disinfectants These are applied to inanimate objects
do not affect this secretion because antiallergic and
such as floors, drainage system, e.g. one per cent
antacid drugs work on different receptors.
solution of phenol, chlorine and SO2 (in very low
Neurologically Active Drugs concentrations), are disinfectants.
(i) Tranquiliser It is a class of chemical compounds
used for the treatment of stress, mild and severe
mental diseases. These are commonly called
Antifertility Drugs
psychothera-peutic drugs. These are the essential These have lead to the concept of family planning. Birth
component of sleeping pills. control pills essentially contain a mixture of synthetic
estrogen and progesterone derivatives.
Some examples according to the action of drugs are e.g. Norethindrone (synthetic progesterone) and the
(a) Antidepressant drugs, e.g. iproniazid, phenelzine estrogen derivative in combination with progesterone
etc., are used to reduce depression. derivative, (ethinylestradiol) (novestrol) etc.

@iitjeehelps
730 SELF STUDY GUIDE BITSAT

Chemicals in Food Cleansing Action of Soap


Substances which are added to food either to improve its On applying soap to a dirty wet cloth, the hydrocarbon part
taste and flavour or to preserve it, are called food additives. (non-polar part) of soap dissolves in grease or dust while
Main categories of food additives are as follows: the polar carboxylate part is directed towards water.
Thus, an emulsion is formed between grease particles and
Artificial Sweetening Agents water molecules, which appears in the form of foam.
Some examples of artificial sweetening agents are On washing the cloth with excessive water, these dirt or
saccharin (ortho-sulphobenzimide), aspartame (nutra dust or grease particles are washed away from the surface
sweet), sucralose, alitame etc. of cloth along with soap and the cloth becomes clean.
Alitame is high potency sweetener, although it is more
stable than aspartame. Sucralose is trichloro derivative of grease
sucrose. Its appearance and taste are sugar like.
It is stable at cooking temperature. Hence, its use is of great
value to diabetic persons who need to control in take of
Cleansing action of soap
calories. Micelle formation

Food Preservatives ● Advantages Soaps are biodegradable cleansing agents,


Food preservatives prevent spoilage of food due to therefore they do not cause pollution.
microbial growth, e.g. sodium benzoate, salts of sorbic acid ● Disadvantages Soaps do not work in hard water because
and propanoic acid etc. hard water contains calcium and magnesium ions.
Cleansing Agents ● These ions form insoluble calcium and magnesium soaps
respectively.
These are also known as surfactants or surface active agents.
In fact, those chemicals which concentrate at the surface of Synthetic detergents
the solution or interfaces or surface films, reduce surface Alkyl benzene sulphonates are the synthetic detergents.
tension of the solution and help in removing dirt and dust by These are also called syndets.
emulsifying grease are known as surfactants. Soaps and
The detergents are classified into following three types on the
detergents belong to this class.
basis of ionic charge present at the soluble end of their chain :
Soaps Anionic detergents
These are the sodium or potassium salts of higher fatty e.g. Sodium dobecyl benzene sulphonate
acids and are prepared by alkaline hydrolysis of fats or oils.
Anionic detergents are also used in toothpaste.
Fats or oils are esters of higher fatty acids.
O Cationic detergents
½½ é CH3 ù
+
CH2 —O — C — C17H35 ê | ú
| O êCH3 ¾ (CH 2 )15 ¾ N ¾ CH3 ú Br -
| ½½ ê ú
|
CH —O— C — C 17H35 + 3NaOH ¾® ê CH ú
| ë 3 û
Cetyl trimethyl ammonium bromide
|
CH 2 — O— C —C 17H35 These are used in hair conditioners and are expensive,
½½ therefore these are of limited use.
O
Glyceryl ester of stearic acid (fat) Non-ionic detergents
CH 2 — OH e.g. CH3 (CH 2 )16 COO(CH 2CH 2O)n CH 2CH 2OH
| They are used in liquid dishwashing detergents.
3C 17H35 COONa + CH — OH Advantages Detergents can be used both in soft and hard
Soap |
water as they give foam even in hard water.
CH2 — OH
G lycerol
Disadvantages In detergents, hydrocarbon chain is highly
branched, so bacteria cannot degrade this easily. In other
The reaction is known as saponification. words, we can say that these are non-biodegradable.

@iitjeehelps
POLYMERS, BIOMOLECULES AND CHEMISTRY IN ACTION 731

Practice Exercise
1. Polymers are also referred to as 10. The element present in teflon is
a. micromolecules b. small molecules a. fluorine b. chlorine
c. macromolecules d. huge molecules c. nitrogen d. oxygen
2. On the basis of structure of polymers, they can be 11. Glyptal is classified as a
classified as a. polyolefin b. polyester
a. linear, branched and network polymers c. polyamide d. polyether
b. natural, synthetic and network polymers
12. Bakelite is made from phenol and formaldehyde. The
c. natural, synthetic and semi synthetic polymers
d. natural, synthetic and linear polymers
initial reaction between them is the example of
a. electrophilic aromatic addition
3. The correct classification of polymers based on b. nucleophilic aromatic substitution
molecular forces is c. free radical reaction
a. elastomers, fibres and thermosetting polymers d. aldoI reaction
b. elastomers, fibres, thermoplastic and thermosetting
polymers 13. Artificial silk is a
c. homopolymers and copolymers a. polypeptide b. polysaccharide
d. None of the above c. polythene d. polyvinyl chloride

4. Repeating unit of polystyrene is 14. Which one of the following pairs is not correctly
matched?
a. ¾ CH2 ¾ CH¾ b. ¾ CH2 ¾ CH¾
½ ½ a. Terylene Condensation polymer of terephthalic acid
Cl C6H5 and ethylene glycol
b. Perspex A homopolymer of methyl methacrylate
c. ¾ CH2 ¾ CH¾ d. ¾ CF2 ¾ CF2 ¾ c. Teflon Thermally stable cross - linked polymer of
½ phenol and formaldehyde
CN d. Synthetic rubber A co-polymer of butadiene and
5. Consider the following polymers, styrene
A = Nylon-66, B = Buna-S, C = Polyethene 15. Identify the following polymer :
Arrange these in the increasing order of their
intermolecular forces (lower to higher).
a. A < B < C b. B < C < A c. B < A < C d. C < A < B
6. Synthetic polymer prepared by using caprolactam is a. Gutta percha b. Neoprene
known as c. Polypropylene d. Natural rubber
a. terylene b. teflon
16. Orlon is a polymer of
c. nylon-6 d. neoprene
a. styrene b. tetrafluoro ethylene
7. Relation between number of average molecular mass c. vinyl chloride d. acrylonitrile
(M n ) and weight of average molecular mass (M w ) of
17. Melamine formaldehyde resin is
synthetic polymers is a. an addition polymer
a. Mn < Mw b. Mn > Mw b. a copolymer
c. Mn = Mw d. Mn > Mw c. a fibre type polymer
d. dimer of melamine and formaldehyde
8. Bakelite is obtained from phenol by reacting it with
a. acetaldehyde b. acetal 18. Which one of the following compounds is different
c. formaldehyde d. chlorobenzene from the rest?
a. Sucrose b. Maltose
9. The structural formula of monomer of poly methyl c. Lactose d. Glucose
methacrylate (PMMA) is
CH3 19. A carbohydrate which cannot be hydrolysed to simpler
½ compounds, is called
a. CH2 == CHCOOCH3 b. CH2 == C — COOCH3 a. monosaccharide
c. CH3COOCH== CH2 d. CH3COO C == CH2 b. disaccharide
½ c. trisaccharide
CH3 d. polysaccharide

@iitjeehelps
732 SELF STUDY GUIDE BITSAT

20. Carbohydrate that yield.............monosaccharide unit H O


½ ½ ½½
on hydrolysis are called oligosaccharide. b. ¾ N¾ C ¾ N¾ C ¾ NH ¾ C ¾ NH ¾
a. 1 – 10 b. 2 – 10 ½½ ½ ½
c. 4 – 10 d. 5 – 10 O H
H H
21. Which of the following structures represent the ½ ½ ½ ½ ½ ½ ½ ½
L-glucose? c. — N— C— C— C— C— N— C— C— C—
½½ ½ ½ ½ ½ ½ ½
CHO CHO O
H OH HO H H H H H
HO H H OH ½ ½ ½ ½ ½ ½ ½ ½
a. b. d. — N— C— C— N— C— C— N— C— C— N— C—
H OH HO H ½ ½½ ½ ½½ ½ ½½ ½
H OH HO H O O O
CH2OH CH2OH 29. Match the following amino acids given in Column I
CHO CHO with their characteristic feature of side chain given in
H HO
the Column II and select the appropriate option from
OH H
the codes given below.
H OH HO H
c. d.
H OH HO H
Column I Column II
HO H H OH
CH2OH CH2OH COOH
A. Asparagine (Asn, N) 1. HN H
22. Each polypeptide in a protein has amino acids linked
each other in a specific sequence. This sequence of
amino acid is said to be
a. primary structure of proteins
B. Proline (Pro, P) 2. HOOC ¾ CH 2 ¾ CH 2 ¾
b. secondary structure of proteins
c. tertiary structure of proteins C. Glutamic acid (Glu, E) 3. HO ¾CH 2 ¾
d. quaternary structure of proteins
D Serine (Ser, S) 4. H 2 N ¾COCH 2 ¾
23. Which of the following pairs form the same osazone? .
a. Glucose and fructose b. Glucose and galactose
c. Glucose and arabinose d. Lactose and maltose
Codes
24. Adenosine is an example of A B C D
a. nucleotide b. nucleoside a. 4 1 2 3
c. purine base d. pyrimidine base b. 4 2 1 3
25. The base adenine occurs in c. 4 3 1 2
a. DNA only b. RNA only d. 4 1 3 2
c. Both DNA and RNA d. protein 30. Pyridoxin is also known as
26. Enzyme trypsin converts a. Vitamin B2 b. Vitamin B6
a. proteins into a-amino acids c. Vitamin B12 d. Vitamin B1
b. starch into sugar 31. Artificial sweetener used in soft drinks, is
c. glucose into glycogen a. aspartame b. cellulose
d. a-amino acids into proteins c. fructose d. glucose
27. Complementary bases present in DNA are 32. The one which is not present in DNA, is
a. uracil and adenine, cytosine and guanine a. uracil b. thiamine
b. adenine and guanine, thymine and cytosine c. adenine d. guanine
c. adenine and guanine, guanine and uracil
d. adenine and thymine, guanine and cytosine 33. Which of the following statement(s) is/are correct
regarding vitamins?
28. Which structure represent the peptide chain? a. They are designated by A, B, C and D.
H O H b. They are further named as B1, B2, B6 and B12 ....
½ ½ ½ ½ ½½ ½ ½ ½ ½ ½ c. Vitamin pills should not be taken without the advice
a. — C— N— C— C— C— N— C— C — N— C— C— C—
of doctor
½ ½ ½ ½ ½ ½ ½½ ½ ½
H O d. All of the above

@iitjeehelps
POLYMERS, BIOMOLECULES AND CHEMISTRY IN ACTION 733
34. Following amino acid has been found in protein 43. Consider the following structure,
prothrombin, but remaines undetected due to the O
H H
formation of another common acid (A) S CH3
C6H5  CH2  C  NH 

H3N—CH—COO CH3
COO A N COOH
CH2—CH O
COO H
Identify A. It is the structure of
⊕ ⊕ a. penicillin K b. penicillin V
a. H3N—CH—COO b. H3N—CH—COO c. penicillin G d. chloramphenicol
COO
CH2—CH2—COO CH 44. A drug that is antipyretic as well as analgesic is
COO a. chloropromazine hydrochloride
b. para-acetamidophenol
c. H2N—CH—COO d. H2N—CH—COOH c. chloroquine
COO d. penicillin
CH2COOH CH
COO 45. Consider the following reaction,
35. Which of the following hormones is responsible for the OH
growth of animals? CH3
CH3COCl, Py KMnO4
a. Auxin b. Insulin X Y
c. Adrenaline d. Somatotropin
36. Which of the following is present in DNA? The end product Y formed in the above reaction is a
a. Deoxyribose b. Starch well known medicine. Which of the following is
c. Riboflavin d. None of these incorrect regarding Y ?
37. Which of the following biomolecules contain a a. It has analgesic as well as antipyretic properties
non-transition metal ion? b. It helps to prevent heart attack
c. It has anti-blood clotting property
a. Haemoglobin b. Chlorophyll
d. It supresses the gastric anomaties
c. Insulin d. Vitamin B12
38. Molecular masses of drugs lie in the range of 46. Tranquillisers are the substances used for the
a. ~ 5 u - 10 u b. ~ 50 u - 60 u
treatment of
c. ~ 100 u - 500 u d. Above 2000 u a. cancer
b. AIDS
39. Difference in the antiseptics and disinfectants is c. mental diseases
a. Antiseptics are used against micro-organisms d. physical disorders
while disinfectants are used against
insects 47. Match the medicines given in Column I with their use
b. antiseptics are used only over skin while disinfectants given in Column II.
can be taken orally also Select the correct option from the codes given below:
c. antiseptics merely inhibit the growth and disinfectant
kill the micro-organisms Column I Column II
d. antiseptics are used over living tissues while
A. Ranitidine 1. Tranquiliser
disinfectants cannot be used over living tissues
40. Chloramine -T is a/an B. Furacine 2. Antibiotic
a. antiseptic b. disinfectant C. Phenelzine 3. Antihistamine
c. analgesic d. antipyretic
D. Chloramphenicol 4. Antiseptic
41. The chemical name of aspirin is
a. methyl salicylate
b. acetylsalicylic acid Codes
c. sodium salicylate A B C D
d. salicylic acid a. 3 4 1 2
b. 4 1 2 3
42. Which statement about aspirin is not true? c. 1 2 3 4
a. Aspirin belongs to narcotic analgesics d. 2 3 4 1
b. It is effective in relieving pain
c. It has antiblood clotting action 48. Chloramphenicol is used in the treatment of
d. It is a neurologically active drug a. typhoid b. malaria
c. acidity d. tuberculosis

@iitjeehelps
734 SELF STUDY GUIDE BITSAT

49. Arsenic drugs are mainly used in the treatment of 52. Which of the following is the structure of equanil?
a. jaundice b. typhoid O CH3 O
c. syphilis d. cholera ½½ ½ ½½
a. H2N ¾ C ¾ O ¾ CH2 ¾ C ¾ CH2 ¾ O ¾ C ¾ NH2
50. Compound which is added to soap to impart antiseptic ½
properties is CH3
a. sodium lauryl sulphate
b. sodium dodecylbenzene sulphonate H
N
c. rosin CH3
d. bithional N C
51. The pH value of gastric juice in human stomach is b. CH2
about 1.8 and in the small intestine, it is about 7.8. C N
+
Cl –
The pK a value of aspirin is 3.5. Aspirin will be O
a. ionised in the small intestine and almost unionised in C 6H 5
the stomach O CH3 O
b. unionised in the small intestine and in the stomach ½½ ½ ½½
c. completely ionised in the small intestine and in the c. H2N¾ C ¾ O ¾ CH2 ¾ C ¾ CH2 ¾ O ¾ C ¾ NH2
stomach ½
d. ionised in the stomach and almost unionised in the (CH2 )2 CH3
small intestine
d. None of the above

BITSAT Archives
1. The catalyst used for olefin polymerisation is [2014] 7. Which of the following antibiotics contain nitro group
a. Ziegler-Natta catalyst attached to aromatic nucleus in its structure? [2013]
b. Raney-nickel catalyst a. Tetracyclin b. Penicillin
c. Wilkinson catalyst c. Streptomycin d. Chloramphenicol
d. Merrified resin
8. Alizarin is an example of [2012]
2. A copolymer of ethene and vinyl chloride contains a. triaryl dye b. azo dye
alternate monomers of each type. What is the mass c. vat dye d. anthraquinone dye
percentage of vinyl chloride in this copolymer? [2014]
9. Which of the following hormones, is responsible for
a. 38% b. 69%
c. 72% d. 82%
the growth of animals? [2012]
a. Auxin b. Insulin
3. The number of disulphide linkages present in insulin c. Adrenaline d. Somatotropin
are [2014]
10. The polymer polyurethanes are formed by treating
a. 1 b. 2 c. 3 d. 4
diisocyanate with [2012]
4. Which of the following statements is not true about the a. butadiene b. isoprene
drug barbital? [2014] c. glycol d. acrylonitrile
a. It is used in sleeping pills
11. A synthetic rubber which is resistant to the action of
b. It is a non-hypnotic drug
oils, gasoline and other solvents is [2011]
c. It is tranquilliser
d. It causes addiction a. buna-S b. polyisoprene
c. neoprene d. polystyrene
5. The monosaccharide having anomeric carbon atoms
are [2013]
12. The number of polypeptide chains present in a
molecule of haemoglobin is/are [2011]
a. geometrical isomers
b. a and b-optical isomers a. four b. one c. two d. three
c. having symmetrical carbon atoms 13. The pentose sugar in DNA and RNA has the [2011]
d. None of the above a. open chain structure b. pyranose structure
6. In vulcanisation of rubber, [2013] c. furanose structure d. All of these
a. sulphur reacts to form a new compound 14. Which of the following is an artificial edible colour?
b. sulphur cross-links are introduced [2011]
c. sulphur forms a very thin protective layer on rubber a. Saffron b. Carotene
d. All of the above c. Tetrazine d. Melamine

@iitjeehelps
POLYMERS, BIOMOLECULES AND CHEMISTRY IN ACTION 735
15. The well known urinary antiseptic urotropine is formed 22. Alizarin belongs to the class of [2007]
when formaldehyde reacts with [2011] a. vat dyes b. mordant dyes
a. NH2OH b. NH3 c. basic dyes d. reactive dyes
c. NH2 × NH2 d. C6H5NH× NH2
23. 2,4-dichlorophenoxyacetic acid is used as [2007]
16. Correct statement is [2010] a. fungicide b. insecticide
a. keratin is fibrous protein c. herbicide d. moth repellant
b. androsterone is male sex hormone
24. Bakelite is a product of the reaction between [2007]
c. vitamin B1 is antineutritic factor
d. All of the above a. formaldehyde and NaOH
b. aniline and urea
17. A certain compound gives negative test with ninhydrin c. phenol and methanal
and positive test with Benedict’s solution. The d. phenol and chloroform
compound is [2009]
25. Cellulose is a polymer of [2007]
a. a protein b. a monosaccharide
a. glucose b. fructose
c. a lipid d. an amino acid
c. ribose d. sucrose
18. Natural rubber and gutta-percha respectively are 26. Raffinose is [2005]
[2009]
a. trisaccharide b. monosaccharide
a. cis-polyisoprene and trans-polyisoprene
c. disaccharide d. None of these
b. both are cis-polyisoprene
c. both are trans-polyisoprene 27. How many hydrogen bonds is/are present between
d. trans-polychloroprene and cis-polychloroprene pair of thymine and adenine in DNA? [2005]
19. Match the following. [2008] a. 1 b. 2
c. 3 d. No bond occurs
List I List II
28. Dacron is polymer of [2005]
A. Oxyhaemoglobin 1. Analgesic a. glycol and formaldehyde
b. glycol and phenol
B. Aspirin 2. Oxygen carrier
c. glycol and phthalic acid
C. Haemoglobin 3. Photosynthesis d. glycol and terephthalic acid
D. Chlorophyll 4. Oil of winter green 29. Natural rubber is a polymer of [2005]
a. styrene b. isoprene
5. Fe 2+ paramagnetic
c. ethylene d. butadiene
The correct match is 30. Milk of magnesia is used as [2005]
Codes a. antichlor b. antacid
A B C D A B C D c. antiseptic d. food preservative
a. 5 1 2 3 b. 4 2 1 3
c. 3 1 2 4 d. 5 2 3 1
31. Which destroy antigens? [2006]
a. Insulin b. Antibodies
20. If M w is the weight average molecular weight and M n c. Chromoprotein d. Phosphoprotein
is the number average molecular weight of a polymer, 32. Nylon-66 is an example of [2006]
the poly dispersity index (PDI) of the polymer is given a. poly propylene b. polyester
by [2008] c. polyamide d. polystyrene
Mn Mw 1
a. b. c. Mw ´ Mn d. 33. The purine base present in RNA is [2006]
Mw Mn Mw ´ Mn
a. guanine b. thymine c. cytosine d. uracil
21. Hydrolysis of sucrose with dilute aqueous sulphuric 34. Glyptal polymer is obtained by the reaction of glycerol
acid yields [2008] with [2006]
a. 1:1 D- (+)-glucose, D-(-)-fructose a. malonic acid
b. 1:2 D- (+)-glucose, D-(-)-fructose b. acetic acid
c. 1:1 D- (-)-glucose, D-(+)-fructose c. phthalic acid
d. 1:2 D- (-)-glucose, D-(+)-fructose d. maleic acid

@iitjeehelps
736 SELF STUDY GUIDE BITSAT

Answer with Solutions


Practice Exercise 17. (b) Melamine-formaldehyde resin is a copolymer.
18. (d) Glucose is a monosaccharide while the others are
1. (c) Macromolecules is another term used to define
disaccharides. Sucrose is a combination of glucose and
polymers.
fructose. Maltose is a combination of two glucose units.
2. (a) Lactose (or milk sugar) is a combination of glucose and
3. (b) Polymers can be classified on the basis of molecular galactose (a hexose sugar).
forces as elastomers, fibres, thermoplastic and 19. (a) Monosaccharide is the simplest carbohydrate which
thermosetting polymers. cannot be hydrolyse to simpler compounds.
4. (b) 20. (b) Carbohydrates that yield 2-10 monosaccharides units
5. (b) Nylon-6,6 is a fibre, thus it has strong intermolecular on hydrolysis are called oligosaccharides.
forces like H-bonding. Buna-S is an elastomer, thus it has 21. (b) L-glucose is the mirror image of D-glucose.
weakest intermolecular forces. Polythene is a
22. (a) Sequence of amino acids is called primary structure of
thermoplastic polymer, thus the intermolecular forces
proteins.
present in polythene are in between elastomers and
fibres. Thus, the order of intermolecular forces is 23. (a) All monosaccharides which differ in configuration at C1
B <C < A and C2 give the same osazone. Since, glucose and
fructose differ from each other only in configuration at C1
6. (c) Caprolactam is the monomer unit of nylon-6. and C2 therefore, they give the same osazone. All other
M options given in the questions do not satisfy this condition
7. (a) Polydispersity index (PDI) of polymer = w
Mn and hence, do not form the same osazone.
For natural polymers, PDI = 1, i.e. M w = M n 24. (b) Adenosine is a nucleoside while adenosine
triphosphate is a nucleotide.
For synthetic polymers PDI = 1, i.e. M w > M n
25. (c) Adenine is a purine base which is common in DNA
8. (c) and RNA.
9. (b) PMMA is the polymer of methyl methacrylate. PMMA 26. (a) Trypsin converts proteins into a-amino acids.
is also known as fiexiglass or perspex. It is used in 27. (d) In DNA, the complementary bases are adenine and
making lenses, transparent domes etc. thymine, guanina and cytosine.
CH3 é CH3 ù 28. (d) The peptide linkage (—NH—CO—) is formed by the
½ Polymerisation ê ½ ú
condensation of amino acid molecules.
nCH2 == C ¾ COOCH3 ¾¾¾¾¾® ê CH2 ¾ C ¾ ú
Methyl methacrylate ê ½ ú HNH ¾ CH ¾ C ¾ OH + H × NH ¾ CH ¾ C ¾ OH
ê COOCH ú | || | ||
ë 3û n
R O R O
10. (a) -H O
¾¾® 2
¾ HN ¾ CH ¾ C ¾ NH ¾ CH ¾ C ¾
11. (b) | || | ||
12. (a) The reaction of phenol and formaldehyde, is called R O R O
Lederer Mannase reaction. It follows electrophilic addition Hence, following structure represents the peptide chain
and rearrangement mechanism. H H H O
13. (b) Chemically treated cellulose (polysaccharide) is called | | | | | | ||
silk. ¾ N¾ C ¾ C ¾ N¾ C ¾ C ¾ N ¾ C ¾ C ¾
| || | || |
14. (c) Teflon is thermally stable polymer of O O
tetrafluoroethylene (F2C == CF2 ).
29. (a) A ® 4, B ® 1, C ® 2, D ® 3
Polymerisation
n F2C == CF2 ¾¾¾¾¾® —( F2C — CF2 —
)n 30. (b) Pyridoxin is also known as vitamin B6.
15. (d) Natural rubber is cis-polymer. 31. (a) Aspartame is the safest and most popular alternative
16. (d) OrIon or PAN is the polymer of acrylonitrile (or vinyl of sugar. Its chemical formula is
cyanide). It is used in making carpets etc. ƒCOOCH3
æ ö CH2 ¾ CH
ç ÷ ½ ‚ NHCOHNH ¾ CH COOH
Polymerisation 2 2

b
n CH2 == CH× CN ¾¾¾¾¾® —— ç CH2 — C H ¾— ÷
çç ½ ÷÷
è CN ø n
PAN or Orlon 32. (a)

@iitjeehelps
POLYMERS, BIOMOLECULES AND CHEMISTRY IN ACTION 737
33. (d) Vitamins are designated by alphabets A, B, C, D. 44. (b) Para-acetamidophenol is used as an antipyretic as
Some of them are further named as sub-groups, e.g. well as analgesic.
B1, B2 B6, and B12 etc. Excess of vitamins is also harmful
OH OCOCH3 OCOCH3
and vitamin pills should not be taken without the advice of
doctor. CH3 CH3 COOH
CH3COCl KMnO4
34. (a) 45. (d)
⊕ – X o-acetyl salicylic
H3N—CH—COO ⊕
– H3N—CH—COO
– acid (aspirin)
COO Y
CH2—CH _ CO
– 2 –
COO CH2—CH2COO
Aspirin increases the gastric anomaties, hence it is
35. (d) Somatotropin is the hormone, secreted by anterior banned in various countries.
lobe of pituitary gland. It is also called growth hormone as
it stimulates protein-synthesis, glycogenesis and some 46. (c) Tranquilisers reduce anxiety and are employed for
other biological activities. Its deficiency causes midgets or treatment of mental disease.
dwarfism. 47. (a) A ® 3, B ® 4, C ® 1, D ® 2
36. (a) The sugar present in DNA is 2-deoxyribose while that 48. (a) It is effective in treatment of typhoid.
present in RNA is a base.
49. (c) Arsenic drugs, also called arsenical (like atoxyl,
37. (b) Chlorophyll contains Mg. salvarson, neoarsphenamine etc) are mainly used in
38. (c) Molecular masses of drugs lie in the range of sleeping sickness, syphilis etc.
~ 100 u = 500 u. 50. (d) Bithional impart antiseptic property to soap.
39. (d) Antiseptics are used over living tissues while 51. (a) Aspirin is a moderate acid (pK a = 3.5), hence it is
disinfectants used for inanimate objects such as floors,
almost unionised in stomach due to its strong acidic
toilets, instruments etc.
medium because of HCl. It is due to common ion effect.
40. (a) Antiseptic drugs cause destruction of While in small intestine, the medium is alkaline, hence
micro-organisms, which produce septic diseases, e.g. aspirin will be sufficiently ionised in small intestine.
dettol, savlon, aeriflavin, iodoform and some dyes such
O CH3 O
as chloramine -T, methylene blue.
½½ ½ ½½
41. (b) 52. (a) H2N ¾ C ¾ O ¾ CH2 ¾ C ¾ CH2 ¾ O ¾ C ¾N H2
42. (a) Aspirin is a non-narcotic drug. ½
CH3
43. (c) The given structure is of penicillin G. Equanil

BITSAT Archives
1. (a) Ziegler-Natta catalyst [TiCl4 + Al(C2H5 )3] is used as a Mass% of vinyl chloride in the copolymer
catalyst in the polymerisation of olefins. 62.5 ´ 100
= = 69.06 » 69%
2. (b) The structure of copolymer of ethene and vinyl 90.5
chloride is shown below 3. (b) Insulin is composed of two peptide chains referred to
n(CH2 == CH2 ) + n(CH2 == CHCl) chains A and B. Chain A of 21 residues and chain B of 30
Ethene vinyl chloride residues are cross-linked by two disulphide bridges.
4. (b) Barbital is a sleep-producing drug, i.e. hypnotic
æ Cl ö
ç ½ ÷ tranquiliser. It causes addiction.
÷
¾® ¾ç CH2 ¾ CH2 ¾ CH2 ¾ CH¾¾ 5. (b) C1 carbon of monosaccharides is called anomeric
ç ÷
ç ÷ carbon. When —OH group is attached with C1 carbon
è Copolymer øn towards right, it is called a-form and when —OH group is
Molecular weight of ethene (CH2CH2 ) = 28 towards left, it is called b-form, such pair or optical
Molecular weight of vinyl chloride (CH2CHCl) isomers which differ in the configuration only around
= 62.5 anomeric carbon are called anomers.
Empirical formula weight of copolymer 6. (b) In vulcanisation of rubber, sulphur cross-links are
= 28 + 62.5 = 90.5 introduced at the reactive sites of double bonds.

@iitjeehelps
738 SELF STUDY GUIDE BITSAT

7. (d ) NHCOCHCl2 18. (a) Natural rubber is cis-polyisoprene while gutta-percha is


trans-polyisoprene.
O2N CH—CH—CH2OH
19. (a) Aspirin is used as analgesics as well as antipyretics,
OH i.e. it serve a dual purpose. Chlorophyll is used in
Chloramphenicol
photosynthesis. Oxyhaemoglobin contains Fe2+ ion, so it
8. (d) Alizarin is an anthraquinone dye. It gives a bright red is paramagnetic and haemoglobin works as oxygen carrier.
colour with aluminium and a blue colour with barium. Hence A ® 5, B ® 1, C ® 2, D ® 3
9. (d) Somatotropin is the harmone, secreted by anterior 20. (b)
lobe of pituitary gland. It is also called growth harmone 21. (a) On hydrolysis with dilute aqueous sulphuric acid,
as it stimulates protein synthesis, glycogensis and some sucrose gives a equimolar mixture of D-(+)-glucose and
other biological activities. Its deficiency causes midgets D-(-)-fructose.
or dwarfism. H2SO4
10. (c) O == C == N ¾ R ¾ N == C == O C12H22O11 + H2O ¾¾® C6H12O6 + C6H12O6
Diisocyanate Sucrose D-(+)-glucose D-( - )-fructose

+ HO ¾R ¢¾ OH ¾® Sucrose is dextrorotatory but after hydrolysis, it gives


Glycol
dextrorotatory glucose and laevorotatory fructose.
éO O ù Laevorotatory fructose is more, so the mixture is
ê ½½ ½½ ú laevorotatory.
¾ê C ¾ NH¾R ¾ NH ¾ C ¾ O ¾R ¢¾ Oú¾ 22. (b) Alizarin is mordant dye. Alizarin gives a bright red
ê ú
ê ú colour with aluminium and a blue colour with barium.
ë ûn 23. (c) 2, 4-D or 2, 4-dichlorophenoxyacetic acid is used as a
Polyurethanes
herbicides.
11. (c) Neoprene is synthetic rubber. It is a polymer of
chloroprene and is resistant to the action of oils, 24. (c)
gasoline and other solvents. 25. (a) Cellulose is a polymer of glucose. b-D-(+)-glucose units
are attached to each other by C1 to C4 bonds through b
12. (a) Haemoglobin molecule contains four polypeptide -glycosidic linkage in structure of cellulose.
chains.
26. (a) Carbohydrates that yield 2 to 0 monosaccharide units
13. (c) The structures of sugar of DNA (i.e. deoxyribose) and on hydrolysis. They are further classified as disaccharides,
that of RNA (i.e. ribose) are as
trisaccharides, etc. Riffinose is trisaccharide.
HOH2C OH HOH2C OH
O O 27. (b) Between pair of thymine and adenine in DNA,
two-hydrogen bonds are present.
H H H H
H
28. (d ) Dacron is polymer of glycol and terephthalic acid.
H H H
OH OH 29. (b) Natural rubber is considered as a linear polymer of
OH H
Ribose sugar Deoxyribose sugar isoprene
(2-methyl-1, 3-butadiene). It is also called as cis -1,
Thus, it is clear that these have furanose structure. 4-polyisoprene.
14. (c) Tetrazine is an artificial edible colour. 30. (b) It is a used as an antacid.
15. (b) When formaldehyde reacts with ammonia, a well 31. (c) A chromoprotein is a conjugated protein that contains a
known urinary antiseptic urotropine (also called pigmented prosthetic group which destroy antigens.
hexamethylene tetramine) is obtained.
32. (c) It is an example of polyamide. It is prepared by
16. (d)
condensation polymerisation of hexamethylene diamine
17. (b) Since, it gives a negative test with ninhydrin, it with adipic acid under high pressure and temperature.
cannot be a protein or an amino acid. Since, it gives a
positive test with Benedict’s solution, it must be a 33. (d) The purine base present in RNA is uracil
monosaccharide but not a lipid. 34. (c) Phthalic acid

@iitjeehelps
30
Qualitative Analysis

In inorganic qualitative analysis, an inorganic ionic compound or mixture of compounds, is analysed


for its cations and anions (i.e. basic radicals and acid radicals). There are various methods of
qualitative analysis like borax bead test, flame test, cobalt nitrate test etc., but the most important
method is wet test.

Qualitative Analysis for Acid Radicals


Acids radicals have been grouped in two groups on the basis of decomposing acid (group reagent) :

Group I
On heating the mixture with dil.HCl or dil.H 2SO4 , a gas is evolved which is identified by its properties.
Acid radicals Evolved gas
(i) Carbonate (CO2-
3 ) Gas (CO2 ) evolves with brisk effervescence which turns lime water to milky.

(ii) Sulphite (SO2-


3 ) Gas (SO2 ) with smell of buring sulphur, turns acidic dichromate paper to green
[Cr(SO4 ) 3 ] .

(iii) Sulphide (S 2- ) Gas (H 2 S) with smell of rotten eggs, turns lead acetate paper to black (PbS).

(iv) Nitrite (NO-2 ) Brown gas (NO2 ) with pungent odour, turns KI-starch paper to blue.

(v) Acetate (CH 3COO- ) Smell of vinegar, aqueous salt + FeCl 3 ¾® red colour

Group II
Mixture is heated with conc. H 2SO4 , when a gas is evolved which is identified by its properties.
Acid radicals Gas evolved
-
(i) Chloride (Cl ) Colourless, pungent gas (HCl), gives white fumes on bringing glass rod moistened with NH 4OH.
-
(ii) Bromide (Br ) Colourless, pungent gas (HBr)
D
Mixture + MnO2 + conc. H 2 SO4 ¾¾® red vapours (Br2 )

(iii) Iodide (I - ) Violet vapours (I 2 ) , turns starch paper to blue.

(iv) Nitrate (NO-3 ) Brown, pungent gas (NO2 ).

(v) Oxalate Colourless, odourless gas (CO) evolves, which burns with blue flame.
(C 2O42- )

@iitjeehelps
740 SELF STUDY GUIDE BITSAT

Some specific tests Qualitative Analysis for Basic Radicals


Chromyl Chloride Test Basic radicals have been grouped in 6 groups on the basis
Chloride on heating with solid K 2 Cr2 O7 and of solubility products of their salts with group reagent.
conc.H2 SO4 gives orange-yellow vapours of chromyl of Group Basic radical Group reagent Precipitate
chloride ( CrO2 Cl 2 ) which on passing over NaOH and then on + 2+
I Ag , Pb , Hg 2+
2 dil. HCl AgCl, PbCl 2 , Hg 2Cl 2
treating with lead acetate forms yellow precipitate of lead (All white)
chromate.
II Hg 2+ , Pb 2+ H 2 S (in acidic HgS, PbS, CuS
4NaCl + K 2Cr2O7 (s ) + 3H 2SO4 (l ) ¾¾® II A Bi 3 + , Cu2+ , Cd 2+
medium of dil. (all black), Bi 2 S 3
HCl) (brown), CdS
K 2SO4 + 2CrO2Cl 2 ( g ) ­ + 3H 2O + 2Na 2SO4 (insoluble in yellow
(yellow)
Chromyl chloride ammonium
(orange-yellow fumes) sulphide)

CrO2Cl 2 ( g ) + 4 NaOH ¾¾® II B As 3 + , Sb 3 + , Sn 2+ , SnS (black), As 2 S 3


Sn 4 + (soluble in (yellow), Sb 2 S 3
Na 2CrO4 (aq) + 2NaCl(aq ) + 2H 2O(l ) YAS) (orange)

Pb(CH3COO)2 (aq ) + Na 2CrO4 (aq ) ¾¾® III Fe 3 + , Al 3 + , Cr 3 + NH 4OH (in Fe(OH) 3 (brown),
presence of Cr(OH) 3 (green),
PbCrO4 (s ) ¯ + 2CH3COONa excess NH 4Cl) Al(OH) 3 (white
Lead chromate
(yellow ppt) gelatinuous)
IV Co2+ , Ni 2+ , Zn 2+ H 2 S (in alkaline NiS, CoS (black), ZnS
Chromyl chloride test is not given by chlorides of Hg, Sn, medium of (white)
Ag, Pb and Sb. Bromides and iodides do not give chromyl NH 4OH)

chloride test. V Ba 2+ , Ca 2+ , Sr 2+ (NH 4 ) 2CO3 (in BaCO3 , CaCO3 ,


presence of
SrCO3 (all white)
Ring Test NH 4OH)
This test is for the identification of nitrate ion (NO3- ) . To the VI Mg 2+ Na 2 HPO4 Mg(NH 4 )PO4 (white)
mixture, freshly prepared FeSO4 solution is added shaken.
Test for ammonium (NH +4 )
Now a drop of
conc. H 2SO4 is added by the side of the test tube, a brown Ammonium ion on heating with NaOH gives NH3 gas
ring is formed at the junction of two liquids due to the which is recognised by taking a glass rod dipped in HCl
formation of nitroso ferrous sulphate. above the mouth of test tube.
NaNO3 + H 2SO4 ¾® NaHSO4 + HNO3 NH3 ( g ) + HCl( g ) ¾¾® NH 4Cl(s )­
White fumes
6FeSO4 + 3H 2SO4 + 2HNO3 ¾® It gives yellowish-brown precipitate with Nessler’s reagent.
3Fe 2 (SO4 )3 + 4H 2O + 2NO K 2HgI 4 + 2NH3 ¾¾® NH 2 (Hg)I + NH 4I + 2KI
II
FeSO4 + NO + 5H 2O ¾® [Fe (H 2O)5 NO]SO4 Nessler's
reagent
Brown ring
2 NH 2 (Hg)I + H 2O ¾® NH 2 (Hg) ¾ O ¾ (Hg) ¾ I + NH 4 l
Test for Some Other Acid Radicals Iodide of Millon’ s base
(yellowish-brown ppt)
(a) Sulphate (SO2-
4 ) Solution of mixture in
Some Important Points
dil. HCl + BaCl 2 ¾® white ppt ( BaSO 4 )
(i) Before the test of group III, the filterate of group II is
is insoluble in mineral acids. boiled to evolve out H 2S gas so that sulphide of higher
(b) Phosphate (PO34- ) Mixture is boiled with conc. HNO3 group radicals could not precipitate in group III.
and then ammonium molybdate solution is added (ii) In group III analysis, NH 4Cl is added before NH 4OH to
when a canary yellow precipitate is obtained. decrease the ionisation of NH 4OH by common ion
effect so that only group III radicals could precipitate.
Interferring Acid Radicals
Borate, oxalate, fluoride and phosphate are called Borax Bead Test
interferring radicals as they interfere in the usual test of
In this test, a platinum wire loop is made and heated. Now it
basic radicals.
is dipped into borax powder.

@iitjeehelps
QUALITATIVE ANALYSIS 741
It is heated again when borax loses water and decomposes Different basic radicals give different colours, thus they can
to give a transparent glassy bead of sodium metaborate. be identified.
¾D ® Na 2B 4O7 + 10H 2O
Na 2B 4O7 × 10H 2O ¾ Basic Colour Colour
D
Radical (in oxidising flame) (in reducing flame)
Na 2B 4O7 ¾® 2NaBO2 + B 2O3
1442443 Co2+ Blue Blue
Transparent glassy bead 2+
Cu Light blue or green Colourless or red
Now mixture is taken over bead and it is again heated in
Fe 3 + Yellow brown Bottle green
oxidising (non-luminous) flame and then in reducing 3+
(luminous flame) and colour is observed in each case. Cr Green Green

Practice Exercise
1. A substance on treatment with dil. H 2SO4 liberates a 8. Which chloride turns black with NH4 OH ?
colourless gas which produces (i) turbidity with baryta a. Lead chloride b. Mercurous chloride
water and (ii) turns acidified dichromate solution c. Mercuric chloride d. Silver chloride
green. The reaction indicates the presence of
9. Which among the following is an interferring radical?
a. CO2-
3 b. S2- c. SO2-
3 d. NO-2 a. Sulphide b. Sulphate
2. In qualitative analysis when H2S is passed through an c. Phosphate d. Nitrite
aqueous solution of salt acidified with dilute HCl a 10. Which compound does not dissolve in hot
black precipitate is obtained. On boiling the precipitate dilute HNO3?
with dil. HNO3 , it forms a solution of blue colour. a. HgS b. CuS c. PbS d. CdS
Addition of excess of aqueous solution of NH3 to this
11. A metal nitrate reacts with KI to give a black
solution gives
precipitate which on addition of excess of KI convert
a. deep blue precipitate of Cu(OH)2
into orange colour solution. The cation of metal nitrate is
b. deep blue solution of [Cu(NH3 )4] 2+
a. Hg2+ b. Bi3+ c. Pb2+ d. Cu+
c. deep blue solution of Cu(NO3 )2
d. deep blue solution of Cu(OH)2 × Cu(NO3 )2 12. A brown precipitate obtained by reaction of alkaline
3. The aqueous solution of which of the following salt will solution of Nessler’s reagent and NH 3 are due to the
be coloured? formation of
a. Zn(NO3 )2 b. LiNO3 a. mercuric hydroxide b. HgI42-
c. CrCl3 d. Potash alum c. Hg + Hg (NH2 )I d. iodide of Millon’s base
4. Which does not give borax bead test? 13. Which of the following on reaction with NaOH and
a. Fe b. Cu Na 2O2 gives yellow colour?
c. Cr d. Pb a. Cr(OH)3 b. Al(OH)3
c. Zn(OH)3 d. None of these
5. A brown ring is formed in the ring test for NO-3 ion. It is
due to the formation of 14. Consider the following reactions,
a. [Fe(H2O)5(NO)] 2+ b. FeSO4 × NO2 [A]+ H 2SO4 ¾® [B ]
c. [Fe(H2O)4(NO2 )] 2+ d. FeSO4 × HNO3 a colourless gas having irritating smell
6. Among the following pairs, the pair of salts that gives [B ]+ K 2Cr 2O7 + H2SO4 ¾® Green solution
yellow precipitate separately with aqueous solution of A and B respectively are
BaBr2, is a. Cl– and HCl b. CO2–
3 and CO2

a. Na 2CO3, AgNO3 b. K 2SO4, K 2CrO4 c. SO2–


3 and SO2 d. S2– and H2S
c. K 2SO4, AgNO3 d. AgNO3, K 2CrO4 15. A solution of metal ion when treated with KI gives a red
7. Formation of purple colour on the addition of sodium precipitate which dissolves in excess KI to give a
nitroprusside to sodium carbonate extract indicates colourless solution. Moreover the solution of metal ion
on treatment with a solution of cobalt (II) thiocyanate
the presence of gives rise to a deep blue crystalline precipitate. The
a. PO3-
4 b. SO2-
3 metal ion is
c. S2- d. S2O2-
3 a. Pb2+ b. Hg2+ c. Cu2+ d. Co2+

@iitjeehelps
742 SELF STUDY GUIDE BITSAT

16. An aqueous solution contains the ions Hg2+ 2+


2 , Hg , 22. Which one of the following statements is correct?
2+ 2+
Pb and Cd . Which of these will precipitate by the a. Manganese salts give a violet borax bead test in the
addition of dil. HCl? reducing flame
b. From a mixed precipitate of AgCl and AgI, ammonia
a. Hg2Cl2 and PbCl2 b. Only Hg2Cl2
solution dissolves only AgCl
c. Only PbCl2 d. PbCl2 and Hg2Cl2
c. Ferric ions give a deep green precipitate on adding
17. Which one of the following does not impart colour to potassium ferrocyanide solution
the flame? d. On boiling a solution having K + , Ca 2+ and HCO-3
a. LiCl b. KI ions, we get a precipitate of K 2Ca(CO3 )2
c. MgCl2 d. CaCl2
23. Which is soluble in NH4 OH ?
18. Identify the incorrect statement about Al2(SO4 )3 . a. PbCl2 b. AgCl c. PbSO4 d. CuCO3
a. It is a white salt
b. On adding NaOH solution to its aqueous solution, a 24. Which of the following will not give positive chromyl
precipitate is formed which get dissolve in excess of chloride test?
NaOH solution a. CuCl2 b. HgCl2
+
c. With lead acetate solution also, it gives a precipitate c. ZnCl2 d. C6H5 N H3Cl-
d. With silver nitrate, its aqueous solution gives a
precipitate 25. Which of the following does not give flame test?
a. Ba 2+ b. Mg2+ c. Sr 2+ d. Ca 2+
19. On addition of conc. H 2SO4 to a chloride salt,
colourless fumes are evolved but in case of iodide 26. Which of the following will give blood red colour with
salt, violet fumes come out. This is because FeCl3 in sodium extract?
a. H2SO4 reduces HI to I2 b. HI is of violet colour a. NH2CONH2 b. NH2CSNH2
c. HI gets oxidised to I2 d. HI changes to HIO3 c. C2H5NHNH2 d. CH3C ºº N
20. When bismuth chloride is poured into a large excess 27. Which of the following gives blood red colour with
of water, then white precipitate produced is due to KCNS?
a. Bi(OH)3 b. Bi2O3 a. Cu2+ b. Fe3+
c. BiOCl d. Bi2OCl2 c. Al3+ d. Zn2+
21. A purple coloured compound is obtained, on treating 28. Which pair of ions gives precipitate when their dilute
sodium sulphide with sodium nitroprusside. In this aqueous solutions are mixed?
reaction, the oxidation state of Fe a. CO23- , NH4+ b. SO23- , Na +
a. changes from + 3 to + 2 c. PO34- , Fe3+ d. Na + , S2-
b. changes from + 2 to + 4
c. changes from + 2 to + 3 29. The ion that cannot be precipitated by both HCl and
d. No change in oxidation state H2S, is
a. Pb2+ b. Cu+ c. Ag+ d. Sn2+

BITSAT Archives
1. In Nessler’s reagent for the detection of ammonia, the 2. Which of the following silver salts is insoluble in
active species is [2007] water? [2005]
a. Hg2Cl2 b. Mg2+ a. AgClO4 b. Ag2SO4
c. Hg2l2 d. Hgl 2- c. AgF d. AgNO3
4

@iitjeehelps
Answer with Solutions
Practice Exercise 15. (b) Hg2+ + 2I- ¾¾® HgI2

1. (c) SO2- HgI2 + 2I- ¾¾® [HgI4] 2-


3 gives SO2 with dil. H2 SO4 which gives turbidity Excess
with Ba(OH) 2 and turns acidified dichromate solution 2+
Hg + Co(SCN) 2 ¾¾® Hg(SCN) 2 ¯
green due to its reduction to Cr 3+ ions.
Blue crystalline ppt
2. (b) 3CuS + 8HNO2 ¾® 3Cu(NO3 )2 + 2NO + 3S + 4H2O
16. (a) Hg2+ 2+
and Pb are present in I group, hence only
2
Cu2+ + 4NH4OH¾® [Cu(NH3 )4] 2+ + 4H2O
Deep blue complex
these will be precipitated. The solubility product of MgCl2
3+ and CdCl2 are higher, hence they will not precipitate.
3. (c) Cr is a green coloured ion.
17. (b)
4. (d) Borax bead test is given only by coloured salts or ions.
Hence, Pb does not give borax bead test. 18. (d) Aqueous solution of Al2(SO4 )3 does not give
5. (a) precipitate with silver nitrate (AgNO3 ) solution.
6. (d) Ba 2+ + K 2CrO4 ¾® BaCrO4 ¯ + 2K + 19. (c)
Yellow
20. (c) Bismuth and antimony chloride give white precipitate
-
Br - + AgNO3 ¾® AgBr ¯ + NO3 of oxychloride in water.
Yellow
BiCl3 + H2O ¾® BiOCl + 2HCl
7. (c) Na 2S + Na 2[Fe(CN)5(NO)] ¾® Na 4[Fe(NOS)CN)5] +3
Milky +3
Sodium nitroprusside Violet (purple)
21. (d) Na 2[Fe(CN)5 NO] + Na 2S ¾® Na 4 [Fe(CN)5 NOS]
8. (b) It form black precipitate of mercury with NH4OH.
22. (b) NH3 is an inorganic solvent. AgI has much higher
Hg2Cl2 + 2NH4OH® [NH2 ¾ Hg ¾ Hg ¾ Cl] + NH4Cl + H2O
Unstable covalent character in comparison to AgCl. Thus, AgCl
¯ dissolves readily in NH3.
NH2HgCl ¯ +Hg ¯ 23. (b) AgCl is completely soluble in NH4OH while AgBr is
White Black
sparingly soluble in it. Agl is insoluble in NH4OH.
9. (c) Phosphate causes the precipitation of group IV, V and 24. (b) HgCl2 does not give positive chromyl chloride test due
Mg2+ cation in group III as PO3-
4 and hence, creates
interference. to its covalent nature.

10. (a) HgS is soluble only in aqua-regia (i.e. 3HCl + HNO3). 25. (b) Mg2+ and Be2+ do not impart colour to the flame in
the flame test.
11. (b) Bi(NO3 ) 3(aq ) + 3KI(aq ) ¾¾® BiI3(s ) ¯ + 3KNO3 (aq )
Black ppt 26. (b) Compound (b) contains both N and S, therefore it will
BiI3(s ) + KI(aq ) ¾¾® K[BiI4] form NaSCN which reacts with FeCl3 to give ferric
Orange thiocyanate which is blood red in colour.
12. (d) NH3 form Millon’s iodide base [I ¾ HgOHg ¾ NH2] 27. (b) Fe3+ + 3KCNS ¾® Fe(CNS)3 + 3K +
with Nessler’s reagent. Ferric thiocyanate
(blood red)
13. (a) 2Cr(OH)3 + 4NaOH + 3[O] ¾® 2 Na 2CrO4 + 5H2O
Yellow 28. (c) Fe 3+ (aq ) + PO43–(aq ) ¾® FePO4 ¯
ppt
14. (c) Since, SO 3 has a smell of burning which is irritating, 2+
thus, [A] is SO32- and [B ] is SO2, which turns acidified 29. (d) Sn can be precipitated only by H2S and not by HCl.
K 2Cr2O7 solution green, as it get reduced to Cr 3+ ions. Rest can be precipitated by both reagents.

BITSAT Archives
1. (d) Nessler’s reagent gives brown ppt of iodide of Millon’s base with ammonium salt.
[Hgl4] 2- + NH4Cl + 4OH- ¾® NH2HgOHgI + I- + Cl- + 3H2O
,
lodide of Millon s
base (brown ppt)

2. (b) Ag2SO4 is insoluble in water.

@iitjeehelps
31
Stereochemistry

Isomerism
Compounds having the same molecular formula but differ in structural formulae, properties and
spatial arrangement, are called isomers and the phenomenon is called isomerism.

Types of Isomerism
Isomerism can be of the following two types:
(i) Structural isomerism (ii) Stereo isomerism

Structural Isomerism
When the isomerism is due to difference in the sequence (connectivity) of atoms within the
molecules (without any reference to space), the phenomenon is called structural isomerism. In other
words, the structural isomers are the compounds which have the same molecular formula but
different structural formulae, hence different properties. The structural isomerism is also called
constitutional isomerism and categorised as
(i) Chain isomerism (ii) Position isomerism
(iii) Functional isomerism (iv) Metamerism
(v) Tautomerism (vi) Ring chain isomerism
(vii) Isotopic isomerism
NOTE Tautomerism and ring chain isomerism are considered as the specialised sub-types of functional isomerism.

Stereo Isomerism
When the isomerism is caused by different arrangement of atoms or groups in space, i.e.
3D-arrangement, the phenomenon is called stereo isomerism. In other words, the stereo isomerism
is shown by those compounds which have the same structural formula (alongwith same molecular
formula) but differ in configuration (the term configuration refers to three dimensional arrangement
of atoms that characterise a particular compound).

@iitjeehelps
STEREOCHEMISTRY 745
This isomerism is called stereo isomerism or (iii) C== N, frozen rotation about carbon-nitrogen
configurational isomerism and can be categorised as double bond in oximes and imines.
(i) Geometrical or cis/trans-isomerism (iv) — N== N —, frozen rotation about nitrogen-nitrogen
(ii) Optical isomerism double bond in azo compounds.
Caution Point Neither resonance forms nor conformations 2. Both the atoms of above written bonds should contain
constitute isomers. different substituents.
Two molecules with
same molecular formula Geometrical Isomerism in Alkenes
The forms in which similar groups are present on the same
No Constitutional side of double bond are called cis forms [Latin cis = same
Is connectivity isomers side] and the forms, in which similar groups are present on
same
the different sides are called trans forms [Latin trans =
Yes across].
Stereo isomers These forms show usually similar chemical but different
physical properties. However, some forms may show
difference in chemical properties as well. Geometrical
Non-congurent No isomerism is not seen in those forms in which the double
Diastereomers
mirror images
bonded carbon atom have same groups attached to it. e.g.
a ¾C ¾a a ¾C ¾a
Yes ½½ or ½½
Enantiomers a1¾ C ¾ b 42444444
44444 b ¾ C ¾3a
Simple flow chart representing the classification of isomers (I)
Both the structures are identical

Geometrical Isomerism a ¾C ¾a a ¾C ¾a
½½ or ½½
When a p-bond is present between two atoms, it restricts
b1¾ C ¾ d d ¾ C ¾3
b
the free rotation of atoms around its axis. It gives rise to two 44444 42444444
(II)
different structural formulae (properties of these forms Both the structures are identical
differ due to different arrangement of atoms about double
bonds in alkenes or about single bonds in cyclic Geometrical figures in (I) and (II) will be identical, hence, no
compounds). geometrical isomers are possible.
This isomerism which arises due to difference in spatial The conversion of cis-isomer into trans-isomer or
arrangement of groups about the vice-versa is possible only if either of the isomer is heated to
1. doubly bonded C-atoms in alkenes, a high temperature or absorbs light. The heat supplies the
2. single bonded C-atoms in cyclic compounds, energy to break the p-bond. Some examples of geometrical
3. doubly bonded C and N-atoms in oximes, isomers are as
4. doubly bonded N-atoms in diazo compounds, is called C 2H 5 ¾ C ¾ H C 2H 5 ¾ C ¾ H
geometrical isomerism. ½½ ½½
C 2H 5 ¾ C ¾ H H ¾ C ¾ C 2H 5
Conditions for Exhibiting Geometrical cis -hex-3-ene trans -hex-3-ene

Isomerism H ¾ C ¾ COOH COOH ¾ C ¾ H


The conditions for exhibiting geometrical isomerism ½½ ½½
include H ¾ C ¾ COOH H ¾ C ¾ COOH
Maleic acid [cis ] f umaric acid [trans ]
1. There should be frozen rotation about two atoms in the
molecules, e.g. These isomers are
(i) C== C , frozen rotation about C¾ C double (i) differ in physical properties such as melting point, dipole
bond in alkenes. moment and acidities. The trans-isomer has no dipole
moment since it is symmetrical, unlike the cis-isomer.
(ii) ½¾¾½ , frozen rotation about carbon-carbon single The cis-isomer is stronger acid than the trans-isomer.
bond in cycloalkanes.

@iitjeehelps
746 SELF STUDY GUIDE BITSAT

(ii) differ even in chemical properties, e.g. cis acid undergo


dehydration at 100°C and form an anhydride. However, Path of propagation
the trans-acid does not form an anhydride until heated
to 240°C.
Unpolarised light Polarised light
Distinction Between cis and trans-Isomers (b) Optical activity The property of a compound by virtu
1. The trans-forms are more stable than the of which it can rotate the plane of polarised light, is
corresponding cis-isomer. This is because, in the called optical activity.
cis-isomer, the bulky groups are on the same side of
(c) Asymmetric C-atom C-atom having 4 different
double bond. The steric repulsion of the groups makes groups or atoms attached separately to its four
the cis-isomer less stable than the trans-isomer in valencies, is called asymmetric carbon atom or
which the bulky groups are far apart (they are on the stereocentre.
opposite sides of the double bond).
(d) Chiral molecule A molecule completely lacking plane
2. Melting point of trans-form is higher than the melting of symmetry is called chiral molecule. In other words,
point of cis-form because of symmetrical packing of it can also be defined as the one which is not
trans-compounds in its crystal lattice. superimposable on its mirror reflection or the one in
3. Boiling point of cis-form is higher than that of the which line of symmetrisation is lacking. An
asymmetric C-atom will generally be considered as
trans-form because of steric hindrance in cis-compound
chiral and we can say that
and high polarity of cis-compound.
(1) A chiral molecule mostly possess at least one chiral
4. Solubility of cis-isomer is higher as compared to
carbon atom.
trans-isomer, as molecules of trans-isomer are more
tightly held in the crystal lattice than the (2) A chiral molecule does not possess any element of
corresponding cis-isomer and hence, it is easier to symmetry like
break the crystal lattice of cis-isomer as compared to (i) Plane of symmetry
trans- isomer. a
5. The cis-isomer of a symmetrical alkene (alkene in
b c
which both the carbon atoms have similar group) has a Plane of symmetry
definite dipole moment, while the trans-isomer has b c
zero dipole moment.
a

Optical Isomerism (ii) Centre of symmetry


O O
Compounds with similar physical as well as chemical Me NH Me Me NH H
properties and with difference only in their behaviour Centre of
symmetry
towards plane polarised light are called optical isomers and H NH H H NH Me
this property is called optical isomerism. O O
No centre of symmetry Has centre of symmetry
Important Terms Related to Optical (cis-form) (trans-form)
Isomerism (iii) Axis of symmetry
Before going in detail, it is better to understand some terms Me H
related to this isomerism :
(a) Plane polarised light According to wave theory, an H Me
ordinary ray of light is considered to have particulate Axis of symmetry
vibrations in all possible planes perpendicular to its H Me
path of propagation. However in polarised light, the
particulate vibrations occur in one plane only. Me H

@iitjeehelps
STEREOCHEMISTRY 747
(e) Achiral molecule A molecule which is superimposable Mirror
on its mirror image is called achiral. COOH COOH COOH

Usually the compounds having atleast one asymmetric H—C—OH HO—C—H H—C—OH
C-atom (the C-atom all the four valencies of which are
satisfied by four different groups or atoms) are capable to HO—C—H H—C—OH H—C—OH
exhibit optical isomerism, e.g. lactic acid. COOH COOH COOH
H Optically active forms Line of symmetrisation Meso-tartaric acid
½
H3C ¾ C ¾ OH asymmetric C-atom 2. Racemic mixture The equimolar mixture of two
½ optically active forms (d and l forms) of a compound is
COOH called racemic mixture. The mixture is optically
Optically active inactive as both the forms neutralises the effect of one
The optical isomers can be categorised into the following two another. Such a neutralising effect is called external
types : compensation, thus racemic mixtures are optically
inactive due to the process of external compensation.
(a) Optically Active Compounds The formation of racemic mixture is called
Those compounds which have the ability to rotate the racemisation while separation of these forms from the
plane of polarised light are called optically active mixture is called resolution.
compounds. These compounds are infact chiral, i.e. have Resolution can be achieved by mechanical method,
absence of elements of symmetry. biological method using enzymes, chemical methods
Among the optically active compounds, following types are (by making their diastereomers) or by
included : chromatographic separation (by using special
1. Dextrorotatory or d or (+) forms which rotate the plane adsorbents).
of polarised light to the right (clockwise).
2. Laevorotatory or l or (–) forms which rotate the plane Enantiomers and Diastereomers
of polarised light to the left (anticlockwise). Optically active compounds which are
non-superimposable mirror images of each other are
(b) Optically Inactive Compounds
called enantiomers, e.g. in tartaric acid, first two figures
Those compounds which do not have such an ability, i.e. written are called enantiomers.
which do not rotate the plane of polarised light are called
optically inactive compounds. Optical isomers which are not the mirror images of each
other are called diastereomers, e.g. in the following
The reasons for the inactivity of these compounds are : representations, the structure of all the four figures is same
(i) Presence of element of symmetry but spatial arrangement of groups differ as fig. (1) and (2)
(ii) Presence of d and l-forms in equal amounts are enantiomeric, similarly fig. (3) and (4) are enantiomeric.
On the basis of above reasons, optically inactive compounds Diastereomers
can be of two types :
Mirror Mirror
1. Meso form It has the plane of symmetry within the Z Z Z Z
molecule, i.e. the one-half of the molecule is
dextrorotatory and rest half is laevorotatory. Thus, a′—C—a a—C—a′ a—C—a′ a′—C—a
within the molecule, both halves show equal and
opposite optical activity. b′—C—b b—C—b′ b′—C—b b—C—b′
As a result, the molecule becomes optically inactive. x′—C—x x—C—x′ x—C—x′ x′—C—x
This phenomenon of optical inactivation in which the
rotation produced by one-half of the molecule is Z′ Z′ Z′ Z′
cancelled by its another half is called internal Fig. 1 Fig. 2 Fig. 3 Fig. 4
compensation, i.e. meso forms are said to be optically
Enantiomeric pair Enantiomeric pair
inactive due to the phenomenon of internal
compensation, e.g. the optically active and inactive However, Fig. (1) is diastereomeric with fig. (3) and (4)
forms of tartaric acid are as follows : (same molecular formula, different optical behaviour and

@iitjeehelps
748 SELF STUDY GUIDE BITSAT

not the mirror images of each other). Similarly, fig. (2) is 4. The bonds coming towards you, are represented by
diastereomeric with fig. (3) and (4). Contrary to horizontal lines. In other words, group attached to the
enantiomers, two diastereomers have different melting horizontal lines are understood to be present above the
points, boiling points and solubilities. They will have plane of the paper.
different chemical reactivities towards most reagents, i.e. Using the above guidelines, the Fischer projection formulae
show difference in physical as well as in chemical
for lactic acid and tartaric acid are shown as
properties.
CH3 COOH
Projection Formulae HO H
H OH
H OH
Different projection formulae have been given to define the
COOH COOH
structure a molecule completely. These can be converted Lactic acid Tartaric acid
into each other.
These are as follows : Systems of Nomenclature of Optical
(a) Wedge-Dash Formula Isomers
The configuration of organic molecules can be visualised The nomenclature of optical isomers can be done in
by three dimensional (3D) structures, which may be accordance with
depicted by any of the following representations.
Group behind Groups in
(a) D, L-System of Nomenclature of Partial
H H H
the plane the plane Configuration
C C C The D, L system of nomenclature is used for optically active
Z X Z X Z X
Y Y Y polyhydroxy carbonyl compounds (carbohydrates) mainly.
This nomenclature was given by Emil Fischer who
Group in front of plane considered glyceraldehyde as the parental sugar.
(I) (II) (III) Position of these The portion of
 OH groups CHO CHO glyceraldehyde
Here the thick solid (solid wedge) line represents the bonds decides the which is supposed
lying above the plane of paper (projecting towards the orientation of H—C—OH HO—C—H to present in all
compounds sugars
viewer), thin lines indicate the bonds lying within the plane
of paper while the dotted line indicates the bonds lying CH2OH CH2OH
below the plane of paper. Such a representation is called D-series L-series
Glyceraldehyde
Wedge-Dash formula.

(b) Fischer Projection Formulae All sugars whose Fischer projection formula shows the
¾ OH group on the right hand side on chiral C-atom
A planar representation of the three dimensional structure adjacent to the terminal —CH2OH group (i.e. second last
is called Fischer projection formula. C-atom) are called the sugars of D-series. Similarly, if the
Following guidelines are used to draw it : ¾ OH group is positioned left then it looks like
1. The chain of carbon atoms in arranged vertically in ½
Position of this OH decides
such a way that the most oxidised carbon occupy the ¾® OH ¾ C ¾ H
top position.
the configuration ½
CH 2OH
2. The asymmetric carbon atom lies in the plane of L-series
paper and is represented at the interaction of crossed
Here, L stands for laevo, i.e. left hand side and not
lines.
laevorotatory. All such sugars having this configuration are
3. The bonds going away from you are represented suppose to belong to the L-series. It must be noted that
by vertical lines. In other words, vertical lines represent there is no relation between the sign of rotation (i.e. + , - or
the groups present behind the plane of paper. d , l ) and the configuration (D and L) of the enantiomer.

@iitjeehelps
STEREOCHEMISTRY 749
1 1
This nomenclature is also used for a-amino acids. Br Br
e.g. H has the
COOH COOH H C* lowest piority C* H
4 4
½ ½ Cl CH3 H3C Cl
H ¾ C ¾ NH 2 H 2 N ¾ C ¾ H 2 3 3 2
½ ½ 3D structure of the molecule is now
oriented such that the lowest priority
R R group is as far as possible from the
D-amino acid L-amino acid viewer, when the viewer views the
3D structure along C—H bond.
1 1
(b) R, S -System of Nomenclature Br Br
The actual three dimensional arrangement of groups in an H C* C* H
asymmetrical molecule is called its absolute configuration. 4 4
We can specify the configuration by using the R,S system Cl CH3 H3C Cl
2 3 3 2
i.e. rectus and sinister system [Here, rectus = right hand side,
sinister = left hand side].
This nomenclature was proposed by Cahn, Ingold and Prelog Br Br
and involves two steps in it:
2 3 3 2
Step 1 Following a set of sequence rules, a sequence of Cl CH3 CH3 Cl
priority is assigned to four different groups or atoms S R
attached to a chiral C-atom. The most acceptable Highest to lowest priority Highest to lowest
is counter clockwise, hence, priority is clockwise,
criterion of priority is based on atomic numbers, i.e.
S configuration hence, R configuration
atom with higher atomic number will get higher
priority, e.g. in the case of bromochloroiodomethane,
the sequence of priority is I, Br, Cl, and H. Measurement of Optical Activity
Step 2 The molecule is next visualised so oriented that the Optical activity of a compound can be detected and
group of lowest priority is directed away from you and measured by means of a polarimeter. When a solution of a
the arrangement of remaining groups observed. While known concentration of an optically active compound is
observing you have to proceed from the group of placed in a polarimeter, the beam of polarised light
highest priority to the group of second priority and is rotated through a certain number of degrees either to the
then to the third. right (clockwise) or to the left (anticlockwise). This degree
is measured and called rotation produced by the solution.
(i) If your eye travels in a clockwise direction, the
configuration is called R [Latin rectus = right]. The degree of rotation depends upon the number of
molecules of the compound encountered by light along its
(ii) If your eye travels anticlockwise, the configuration is path. This degree of rotation, if measured, is called specific
called S [Latin sinister = left]. rotation and can be defined as the rotation produced by a
Two positioning of the lowest priority group down and solution of 10 cm length and unit concentration (1 g/mL) for
away from the observer can be assumed easily in three the given wavelength (l) of light at the given temperature.
dimensions but difficult a bit in two dimensions. e.g.
Thus, for a system in which plane of polarised light is
Observer viewing rotated through a degrees at a temperature of t°C and using
—C—
—C— from this side the sodium-D lines as the source, the specific rotation[ a ]tD
This valency is away is given by
This valency is away
from the observer
from the observer [ a ]tD = 100 a / l ´ c
[three dimensional view of [two dimensional view
of the C-atoms]. Here, l = length of the fluid filled tube in decimeters
a C-atom].
For positioning the group of (10 cm = 1 decimetre)
You can easily position the lowest priority away from you, c = number of grams per 100 mL of solution
group of lowest priority away you have to perform pair inter
from you if you see the changes among the groups The specific rotation varies both with light wavelength and
C-atom three dimensionally. attached to asymmetric C-atom. temperature.

@iitjeehelps
750 SELF STUDY GUIDE BITSAT

(b) Newmann Projection


Conformations or It is a simple method to represent the conformations. In this
Conformers method, two carbon atoms forming the s-bond are
represented by two circles, one behind the other so that
The s electron cloud joining the carbon atoms in alkanes is
cylindrically symmetrical about a line joining the two only the front carbon is seen. The C¾ H bonds of the front
carbon nuclei, hence bond strength should be the same for carbon are depicted from the centre of the circle while
all possible arrangements. Different arrangements of atoms C¾ H bonds of the back carbon are drawn from the
that can be converted into one another by rotation about circumference of the circle.
single bonds are called conformations. Arrangement I is H Back
called the eclipsed conformation; arrangement II is called carbon
H H
the staggered conformation. The infinite intermediate
Front
conformations in between the above two are called skew
carbon
conformations.
H H
H H H H
H
H H H H Newmann projections of ethane
H
H H H
Arrangement I Arrangement II Out of all the conformations possible for ethane, only two
extreme conformations are important, which are given below :
Caution point Conformers are also termed as rotamers and
conformational isomerism as rotamerisms. (a) Staggered Conformation
In this arrangement, the hydrogens of the carbon atoms are
Conformations of Ethane staggered with respect to one another. As a result, they are
at maximum distance apart and have minimum repulsion
In ethane molecule (CH3 ¾ CH3 ), two carbon atoms are between them.
bonded by a single covalent bond. If one of the carbon
atoms (methyl group) is kept fixed and the other is (b) Eclipsed Conformation
rotated about the C¾ C bond, an infinite number of In this conformation, the hydrogens of one carbon atom are
arrangements of the hydrogen atoms attached to one directly behind those of the other. Consequently, the
carbon atom with respect to other are possible. These repulsion in these atoms is maximum. The saw horse
infinite numbers of different arrangements are called projections of these conformations are represented in the
figure given below.
conformations.
To represent these conformations, we can draw H H H H
three dimensional pictures. C C
However, chemists represent the conformations in two simple
H H H
ways as shown below:
H H
(a) Saw Horse Representation C C
In it, we view carbon-carbon bond from an oblique angle H H
and indicate spatial arrangements by showing all the C¾ H Staggered
conformation H
bonds as shown in figure. Eclipsed
H conformation
Saw Horse representation of staggered and
C eclipsed conformations of ethane
H H
The Newmann projections for staggered and eclipsed
H H conformations of ethane are shown in figure given below. It
C is clear that when the staggered conformation is rotated
through an angle of 60°, it changes to eclipsed conformation
H and similarly, when eclipsed conformation is rotated
Saw Horse representation showing rotation through the same angle, it gives back the staggered
around C¾ C single bond in ethane conformation.

@iitjeehelps
STEREOCHEMISTRY 751

60° H H The variation of energy versus rotation about the


H C¾ C bond has been shown in the following figure :
H H
H
60°
H H
H H H HH
H
60° H
Staggered conformation Eclipsed conformation H H
H

Energy
Newmann projections of staggered and eclipsed Eclipsed (less stable)
conformations of ethane
12.5 kJ mol–1

Relative Stabilities of the H H

Conformations of Ethane H H H H
Two conformations of ethane differ in their relative stabilities.
The staggered conformation has minimum repulsions Staggered
between the H-atoms attached tetrahedrally to the two H H (stable) H H
H H
carbon atoms. On the other hand, the eclipsed conformation Rotation
has maximum force of repulsion between H-atoms.
The variation of energy versus rotation about C¾ C bond
Therefore, the staggered conformation is more stable than
the eclipsed conformation. The difference in the energy of various conformations
The difference in the energy contents of the staggered and constitutes an energy barrier in rotation. For free rotation,
eclipsed conformations is 12.5 kJ mol -1 . This small barrier energy barrier is 0.6 kcal/mol. For restricted rotation,
to rotation is also called torsional barrier of the single bond. energy barrier is in between > 0.6 and < 16 kcal/mol.
However, this energy difference is not large enough to For frozen rotation, energy barrier is ³ 16 kcal/mol.
prevent rotation Even at ordinary temperatures, the
Caution Point The dihedral angle between the C¾ H bonds
molecules have thermal or kinetic energy to overcome this
on adjacent carbons in staggered and eclipsed conformations
energy barrier.
of ethane are 180° and 0° respectively whereas between two
Therefore, the two conformations of ethane go on changing methyl groups in the gauche and anti conformations of
from one form to another thus consequently, it is not n-butane, these are 60° and 180° respectively.
possible to isolate the different conformations of ethane.

Practice Exercise
1. Which class of compounds can exhibit geometrical 3. The number of isomers of the compound C2FClBrI is
isomerism? a. 3 b. 4 c. 5 d. 6
a. C6H5CH==NOH
4. Which will form geometrical isomers?
b. CH3CH ==CHCH3 Cl
c. HOOCCH— CH2 — CHCOOH a. b. CH3CH NOH

d. All of the above Cl

2. Which of the following shows geometrical isomerism? c. d. All of these


a. 1, 2-dichloroethene
b. 1,2-dimethylcyclopropane
CO— NH
c. CH3CH CHCH3 5. In , the double bonds are
NH—CO
a. cis, cis b. cis, trans
d. All of the above c. trans, cis d. trans, trans

@iitjeehelps
752 SELF STUDY GUIDE BITSAT

6. Number of geometrical isomers for the molecule 14. Which of the following statements is not correct ?
R H a. A meso compound has chiral centres but exhibits no
C C R optical activity
H C C are b. A meso compound has no chiral centre, thus it is
H H optically inactive
a. 2 b. 3 c. A meso compound has molecules in which one-half of
c. 6 d. 5 molecule is superimposable on the other even
through chiral centre is present in them
7. Racemic modification can be resolved by d. A meso compound is optically inactive because the
a. the use of enzymes rotation caused by one-half of molecule is cancelled
b. fractional crystallisation by the rotation produced by another half
c. fractional distillation
15. An enantiomerically pure acid is treated with racemic
d. None of the above
mixture of an alcohol having one chiral carbon. The
8. Racemic tartaric acid is optically inactive due to ester formed will be
a. external compensation a. optically active mixture b. pure enantiomer
b. internal compensation c. meso compound d. racemic mixture
c. presence of plane of symmetry
16. Which of the following will exhibit geometrical
d. All of the above
isomerism?
9. (+) and (-) forms of optically active compounds are a. Propene b. Butene-2
different in c. Butene-1 d. 1,1-dichloro butane
a. boiling points b. melting points
17. Which statement is true?
c. specific gravity d. specific rotation
a. A compound with R configuration is the
10. Which of the following Fischer projection formula is (+) enantiomer
same as D-glyceraldehyde? b. If configuration changes from + to –, that essentially
CH2OH CH2OH means inversion of configuration takes place
c. An achiral molecule reacts always with racemic
a. OH CHO b. H OH forms, to give a chiral molecule,
H CHO d. By breaking two bonds on the chiral centre,
CHO CHO configuration changes
c. H OH d. H CH2OH 18. The compound whose stereo chemical formula is
CH2OH OH written below, exhibits x-geometrical isomers and
y -optical isomers. The value of x and y respectively
11. How many carbon atoms in the molecule
are
HOOC—(CHOH)2 —COOH are asymmetric? H3C H
a. 1 b. 2
c. 3 d. None of these Br H
HO
12. Number of chiral centres in CH3
H3 C CH3
a. 4 and 4 b. 2 and 2
c. 2 and 4 d. 4 and 2
is/are
19. Incorrect statement is
a. ethane can have an infinite number of conformations
a. 1 b. 2 b. cyclopropane molecule has considerable angle strain
c. 3 d. 4 c. eclipsed form of ethane is less stable than staggered
13. The following compound can exhibits conformation
H d. staggered conformation possesses maximum energy
H3C
C C H 20. At room temperature, the eclipsed and the staggered
H3C C forms of ethane cannot be isolated because
H3C COOH
a. both the conformers are equally stable
a. tautomerism b. they interconvert rapidly
b. optical isomerism c. there is a large energy barrier of rotation about the
c. geometrical isomerism s-bond
d. geometrical and optical isomerism d. the energy difference between the conformers is large

@iitjeehelps
STEREOCHEMISTRY 753
25. Me
r Br2
21. The most stable conformation of ethane chlorohydrin Me H X
at room temperature is OH
a. fully eclipsed b. partially eclipsed Five compounds with formula C4 H 8Br2. How many
c. gauche d. staggered structures of X are possible?
22. The number of chiral carbon atoms present in the a. 2 b. 3
c. 4 d. 5
molecule
Cl H OH

is
26. Configuration of HN COOH and are
CH2 H
a. R, R b. R, S c. S, S d. S, R
a. 3 b. 4 c. 2 d. 1 27. The specific rotation of a pure enantiomer is + 16°. Its
23. Which of the following are diastereomers? observed rotation if it is isolated from a reaction with
Me Me 25% racemisation and 75% retention, is
a. –12°
H Br Br H b. +12°
I. II.
H Br H Br c. +16°
d. –16°
COOH COOH
COOH COOH 28. Out of the following, the alkene that exhibits optical
isomerism is
H Br Br H a. 3-methyl-2-pentene
III. IV.
H Br H Br b. 4-methyl-1-pentene
Me Me c. 3-methyl-1-pentene
d. 2-methyl-2-pentene
a. (I) and (III) b. (II) and (IV)
c. (I) and (II) d. None of these 29. The alkene that exhibits geometrical isomerism is
a. propene
24. The R-isomer among the following are b. 2-methyl propene
CHO H c. 2-butene
d. 2-methyl-2-butene
I. H OH II. D OH
30. The absolute configuration of
CH2OH CH3 CO2H
HO2C
CH3 COOH is

III. H OH IV.H3 C NH2 OH H H OH

CH2CH3 H a. S, S
b. R, R
a. I and II b. II and III c. R, S
c. III and IV d. I and III d. S, R

BITSAT Archives
1. Which among the following is likely to show 2. Which of the following compound (s) has
geometrical isomerism? [2012] Z-configuration? [2011]
a. CH3CH == NOH Cl Br Cl F
b. CH3CH == CH2 I. C == C II. C == C
H F H Br
c. CH2 == CH¾ CH == CCl2 Br CH3
III. C == C
d. CH3C( Cl) == C( CH3 )2 Cl H
a. Only I b. Only II c. Only III d. I and III

@iitjeehelps
Answer with Solutions
Practice Exercise 8. (a) Racemic tartaric acid is optically inactive due to
external compensation. Racemic tartaric acid is an
1. (d) All the compounds in which there should be restricted equimolar mixture of optically active d and l- forms.
rotation about a bond in the molecule, show geometrical 9. (d) If a substance rotates the plane polarised light in
isomerism. Oximes of the type R —CH == N — OH, clockwise direction, it is dextrorotatory (+). If it rotates the
C6H5 —C == N— OH and cyclic compound like plane polarised light in anticlockwise direction then it is
½ laevorotatory (–). Thus, + /- form differ in specific rotation.
CH3
10. (c) The configuration in the Fischer projection of which
HOOCCH — CH2 — CHCOOH show geometrical —OH group is on right hand side, H-atom is on left hand
side, —CHO group is on upper side and CH2OH is on
isomerism. lower side is known as D-configuration.
2. (d) Disubstituted cyclic compounds and disubstituted CHO
alkenes show geometrical isomerism. H OH
CHCl H H CH2OH
D-glyceraldehyde
CHCl H 3C CH3
(Non-cyclic, but have 11. (b) A carbon atom which is attached to four different group is
(Cyclic, two different groups at
bond so exhibits called an asymmetric carbon atom or chiral centre.
adjacent positions, so exhibits *
geometrical isomerism) geometrical isomerism) HOOC(CHOH)2 COOH has two asymmetric carbon ( C)
atoms.
CO—NH * *
HOOC — CH— C H— COOH
H3CCH CHCH3 ½ ½
NH—CO
OH OH
(Cyclic)
12. (b) CH3 CH3 Chiral
F Br F l centres
F Cl
3. (d) C C C C C
Cl C * *
l Cl Br Br
Z E Z I
F l F Cl F Br
C C C C C C (* is asymmetric C-atom)
Br Cl I Br l Cl
E Z E
H3 C H
4. (d) The isomerism which arises due to restricted rotation 13. (b) C C H
H3 C *
C
about a bond in a molecule is known as geometrical
isomerism. H3C COOH
Cl
The above compound has chiral centre. Hence, it can
, CH3—CH NOH and exhibit optical isomerism while geometrical isomerism is
not possible due to the presence of identical groups on
Cl
double bonded carbon atoms.
All of these form geometrical isomers. 14. (d) Compounds which do not show optical activity inspite
5. (c) In ,the double bonds are trans and cis. of the presence of chiral carbon atoms are called
meso -compounds.
15. (a) If an enantiomerically pure acid is treated with racemic
The first and third bonds are identical.
mixture of an alcohol having chiral carbon, the product
6. (b) When n = even number, then for two identical ends, formed will be optically active mixture.
number of geometrical isomers 16. (b) Butene-2 exhibits geometrical (cis, trans) isomerism.
= 2n - 1 + 2n/ 2 - 1 = 21 + 20 = 3 CH3 — CH == CH— CH3
Butene-2
7. (a) The separation of racemic mixture back into d and l
isomers is known as resolution. It can be done by H3C CH3 H3C H
(I) mechanical method. C == C C == C
(II) biochemical method using enzymes. H H H CH3
(III) chemical method (salt formation). cis-isomer trans -isomer

@iitjeehelps
STEREOCHEMISTRY 755
17. (d) We know that by breaking two bonds on the chiral, 25. (b)
centre, configuration changes. Me Me +
H+
Me Me
H 3C H OH

Me Me Me Me
18. (b) Compound H has one chiral carbon CH2 + Me +
Br HO *
CH3
X
atom and one double bond thus, it has two geometrical H
Me Br2
(cis and trans) and two optical isomers.
CH2 + Anti-addition Me * Br
19. (d) Staggered conformation is most stable due to its Br
minimum energy. Optically active
20. (b) Staggered and eclipsed conformers cannot be
physically separated because energy difference between + and –
Two products
them is so small that they most readily interconvert at
room temperature. Me Me
21. (c) Me Me Br2 H Br Br H
Cl H +
Anti-addition Br H H Br
H O Me Me
+ –
H H Two products
H Me
Me Br2 H Br
The gauche conformation is most stable due to the Me
presence of H-bonding between H-atom of OH and Cl. Syn-addition H Br
Me
22. (c) Carbon bonded with four different groups is known as
chiral carbon atom. In case of given compound, Meso
the number of chiral carbon atoms are two. (one product)
Cl
26. (b) Compounds in which the chiral centre is a part of the
* ring are handed in a analogous fashion.

* H
1 2
HN—C—COOH
23. (c) Superimposable compounds which are also not the
mirror images of each other, are called diastereomers. CH2 3
Thus, I and II are diastereomers.
R
2 4
CHO H 3
HO 4
H
4 1 3 1 HO H
24. (a) H OH D OH C 2
1
3 2 CH2 CH2
CH2OH CH3
(I) (II) C—H CH2
R R C C
C C
3 2 C
CH3 COOH ⇒ 3 — C— 2
1 3 1
H OH H3C NH2 1
4 S
2 4
CH2CH3 H 27. (b) % Enantiomeric excess
(III) (IV)
Observed specific rotation
S S = ´ 100
Specific rotation of pure enantiomer
Note If H is on vertical line, R represents clockwise and S
3/4
represents anticlockwise rotation. Opposite is true when Observed specific rotation = ´ ( +16° ) ´ 100 = + 12°
H or lower priority group is present on horizontal line. 100

@iitjeehelps
756 SELF STUDY GUIDE BITSAT

H
½
28. (c) CH3 ¾ CH2 ¾ C* ¾ CH == CH2
½
CH3
3 -methyl-1-pentene
It has one chiral centre.
29. (c) H 3C H H 3C H H 3C H 3C

H 3C H H H
H H
Propene 2-methyl propene cis-2-butene

H 3C H H3C CH3

H CH3 H 3C H
trans-2-butene 2-methyl-2-butene
30. (b)
2
2 COOH
COOH
HOOC
1 3 or H OHR
H OH HO HR
HOR H
1 COOH

BITSAT Archives
1. (a) Among the given, only CH3CH == NOH (oxime) satisfy the conditions essential for exhibiting geometrical isomerism. So, it
will exhibit syn-anti geometrical isomerism.
H3C H H 3C H
C C

N N
OH HO
syn anti

2. (d) When the groups with higher priority (i.e. with high atomic number) are present on same side of double bond, then the
configuration is Z but when present on opposite side of double bond, the configuration is E.
Cl Br Cl F Br CH3
(I) C == C (II) C == C (III) C == C
H F H Br Cl H
(Z ) (E ) (Z )

(Priority : Cl > H and Br > F) (Priority : Cl > H and Br > F) (Priority: Br > Cl and CH3 > H)
Hence, compounds (I) and (III) have Z-configuration.

@iitjeehelps
@iitjeehelps
@iitjeehelps
1
Complex Numbers

Complex Number and Its Representation


A number in the form of z = x + iy, where x , y ∈ R, i = −1 is called a Y
Imaginary axis P(x, y)
complex number. The real numbers x and y are respectively called
real and imaginary parts of complex number z. y
i.e. x = Re( z ) and y = Im ( z )
θ
The complex number z = x + iy is represented by a point P, whose X′ X
O x Real
coordinates are referred to rectangular axes XOX ′ andYOY ′ which axis
are called real and imaginary axes, respectively.
This plane is called argand plane or Gaussian plane. The magnitude
 y Y′
of the complex number z is | z| = x 2 + y 2 and θ = tan −1  
x
A complex number z is said to be purely real or imaginary, if y = 0 or x = 0, respectively.
NOTE Iota (i) is neither 0 nor greater than 0 nor less than 0.
Integral Powers of i We know that, i = −1, i 2 = − 1, i3 = − i , i 4 = 1, etc.
In general, i 4 n = 1, i 4 n + 1 = i , i 4 n + 2 = − 1, i 4 n + 3 = − i , ∀ any integer n.
The sum of four consecutive powers of i is zero. i.e. i n + i n + 1 + i n + 2 + i n + 3 = 0, n ∈ I

Algebra of Complex Numbers


If z 1 = x 1 + iy 1 and z 2 = x 2 + iy 2 are two complex numbers, then
(i) Addition of complex numbers is z 1 + z 2 = ( x 1 + x 2 ) + i( y 1 + y 2 )
Its additive identity is 0 + 0i.
(ii) Subtraction of complex numbers is z 1 − z 2 = ( x 1 − x 2 ) + i( y 1 − y 2 )
(iii) Multiplication of complex numbers is z 1 z 2 = ( x 1 x 2 − y 1 y 2 ) + i( x 1 y 2 + x 2 y 1 )
Its multiplicative identity is 1 + 0i.
z ( x x + y1 y2 ) + i ( x 2 y1 − x 1 y2 )
(iv) Division of complex numbers is 1 = 1 2
z2 x 22 + y 22

@iitjeehelps
760 SELF STUDY GUIDE BITSAT

Conjugate of a Complex Number The value of θ is found by solving these equations and θ is
called the argument or amplitude of z. If − π < θ ≤ π, then θ is
Complex numbers z = a + ib and z = a − ib are called
conjugate to each other. called the principal argument of z.
(i) If x < 0, y < 0, then principal arg ( z ) lies between −π
and − π /2.
Properties of Conjugates
(ii) If x = 0, y < 0, then principal arg ( z ) is − π /2.
If z , z 1 and z 2 are complex numbers, then
(i) ( z ) = z (ii) z 1 + z 2 = z 1 + z 2 (iii) If x > 0, y < 0, then principal arg ( z ) lies between
− π /2 and 0.
(iii) z 1 − z 2 = z 1 − z 2 (iv) z 1 z 2 = z 1 ⋅ z 2
(iv) If x > 0, y = 0, then principal arg ( z ) is 0.
z  z (v) If x > 0, y > 0, then principal arg ( z ) lies between 0
(v)  1  = 1
 z2  z2 and π /2.
(vi) z + z = 2Re( z ) = 2 Re ( z ) (vi) If x = 0, y > 0, then principal arg ( z ) is π /2.
(vii) z − z = 2 i Im ( z ) (vii) If x < 0, y > 0, then principal arg ( z ) is π.
(viii) z + z = 0 ⇒ z is purely imaginary. Properties of arg (z)
(ix) z = z ⇔ z is purely real.
(i) arg ( z 1 z 2 ) = arg( z 1 ) + arg ( z 2 )
(x) z 1 z 2 + z 1 z 2 = 2 Re( z 1 z 2 ) = 2 Re ( z 1 z 2 )
(xi) If z = f ( z 1 ), then z = f ( z 1 ) z 
(ii) arg  1  = arg( z 1 ) − arg ( z 2 )
 z2 
Modulus of a Complex Number (iii) arg ( z ) is not defined, if z = 0
Let z = a + ib, then the modulus of z is the positive real (iv) arg ( z n ) = n arg ( z ) (v) arg ( z ) + arg ( z ) = 2π
number a 2 + b 2 and is denoted by | z|. z   z1 
(vi) If arg  2  = θ, then arg   = 2kπ − θ , k ∈ I
i.e. | z| = a 2 + b 2 = |Re( z )|2 + |Im( z )|2 , | z| ≥ 0, ∀ z ∈ C  z1   z2 

Properties of Moduli Polar Form of a Complex Number


(i) | z| ≥ 0 ⇒ | z| = 0 iff z = 0, i.e. Re ( z ) = Im( z ) = 0 Expression r (cos θ + i sin θ ) is called the polar form of the
(ii) zz = | z |2 complex number x + iy.
(iii) | z 1 z 2 | = | z 1 | | z 2 | Here, r = x 2 + y2
z1 | z 1| x
(iv) = cos θ =
z 2 | z 2| x + y2
2

(v) | z n | = | z|n y
⇒ sin θ =
(vi) | z 1 + z 2 | ≤ | z 1 | + | z 2 | x + y2
2

(vii) | z 1 − z 2 | ≥ | z 1 | − | z 2 |
(viii) | z 1 | − | z 2 | ≤ | z 1 + z 2 | ≤ | z 1 | + | z 2 | Triangle Inequality
(ix) | z 1 + z 2 | = | z 1 | + | z 2 | + 2 Re ( z 1 z 2 )
2 2 2
In any triangle, sum of any two sides is greater than the
(x) | z| = | z | = |− z| = |− z | third side and difference of any two sides is less than the
(xi) z 1 z 2 + z 1 z 2 = 2 | z 1 || z 2 |cos (θ 1 − θ 2 ) third side.
where, θ 1 = arg ( z 1 ) and θ 2 = arg ( z 2 ) (i) | z 1 + z 2 | ≤ | z 1 | + | z 2 | (ii) | z 1 + z 2 | ≥ || z 1 | − | z 2 ||
(xii) | z 1 + z 2 |2 + | z 1 − z 2 |2 = 2 {| z 1 |2 + | z 2 |2 } (iii) | z 1 − z 2 | ≤ | z 1 | + | z 2 | (iv) | z 1 − z 2 | ≥ || z 1 | − | z 2 ||
NOTE Complex numbers do not possess the inequality, i.e.
Argument of a Complex Number 3 + 2 i > 1 + 2 i does not make any sense.

For a complex number z = x + iy , the argument or


 y
amplitude of z ( =/ 0) = tan −1  
x
De-Moivre’s Theorem
i.e. solution of the system of equations If n is an integer, positive, negative or a rational number,
x y then
cos θ = , sin θ = and is denoted byarg ( z ).
x 2 + y2 x 2 + y2 (cos θ + i sin θ )n = cos nθ + i sin nθ

@iitjeehelps
COMPLEX NUMBERS 761

Other Forms of De-Moivre’s Theorem 1 − (α n ) p


i.e. 1 + α p + α 2 p + … + α( n − 1 ) p = =0
−n 1 − αp
(i) (cos θ + i sin θ ) = cos nθ − i sin nθ
∴ α n = 1, p ≠ kn
(ii) (cos θ − i sin θ ) = cos nθ − i sin nθ
n
If p = kn , then sum of their pth powers is n.
(iii) (cos θ − i sin θ )− n = cos nθ + i sin nθ i.e. if p = kn , then α p = α kn = (α n )k = 1.
Q Each root is 1.
Square Roots of a Complex Number ∴ Sum = 1 + 1 + 1 + … + 1 = n
If z = a + ib is a complex number, such that
a + ib = x + iy, where x , y are real numbers, then Geometrical Applications of
a + ib = ( x + iy )2 ⇒ a + ib = ( x 2 − y 2 ) + i(2 xy ) and square Complex Numbers
root of z by equating real and imaginary part can be given 1. If z 1 and z 2 are two complex numbers, then
 1 1  (i) | z 1 − z 2 | is the distance between the points affixes
z =± ( a2 + b 2 + a) + i ( a 2 + b 2 − a ) , b > 0
2 2 z 1 and z 2 .
 
mz 2 + nz 1
 1 1  (ii) is the affix of the point dividing the line
=± ( a2 + b 2 + a) − i ( a 2 + b 2 − a ) , b < 0 m+n
 2 2  joining the points with affixes z 1 and z 2 in the ratio
m : n internally, where m ≠ n .
nth Roots of Unity
2 πk mz 2 − nz 1
 2kπ   2kπ  i (iii) is the affix of the point dividing the line
(cos θ + i sin θ )1/ n
= cos   + i sin   =e n m−n
 n   n 
joining the points with affixes z 1 and z 2 in the ratio
where, k = 0, 1, 2 ,... , (n − 1) m : n externally, where m =/ n .
Thus, nth roots of unity are i = 1, α , α 2 , α3 , …, α n − 1 (iv) If the affixes of the vertices of the triangle are z 1 , z 2
2π 2π z + z 2 + z3
where, α = ei 2 π / n = cos + i sin and z3 , then the affix of its centroid is 1 .
n n 3
2. Three points with affixes z 1 , z 2 , z3 are collinear, if
Cube Roots of Unity
z1 z1 1
Cube roots of unity are 1, ω , ω 2 , where
z2 z2 1 = 0
−1 + 3 i 2 −1 − 3i z3 z3 1
ω= ;ω = and ω3 = 1
2 2
3. (i) | z − z 1 | = r represents the circle with centre z 1 and
Some Important Results radius r .
● The sum of the cube root of unity i.e. 1 + ω + ω2 = 0 and (ii) | z − z 1 | < r represents the interior of the circle with
their product = 1⋅ ω ⋅ ω2 = ω3 = 1 centre z 1 and radius r .
In general, (ω)3 n = 1, (ω)3 n + 1 = ω and (ω)3 n + 2 = ω2 , where n z − z1
4. = k represents a circle, if k =/ 1 and if k = 1, then it
is an integer. z − z2
● Each complex root of unity is the square of the other. represent a straight line which is the perpendicular
● The three cube roots of unity are the vertices of an bisector of the line segment joining points with affixes
equilateral triangle inscribed in the circle| z| = 1. z 1 and z 2 .
● The four fourth roots of unity are
5. ( z − z 1 )( z − z 2 ) + ( z − z 1 ) ( z − z 2 ) = 0 represents the
1, − 1, i , − i
circle with line joining points with affixes z 1 and z 2 as a
diameter.
Properties of Roots of Unity
6. If z 1 , z 2 and z3 are the C(z3)
(i) Sum of the roots of unity in square roots, cube roots,
affixes of the points A, B , C
4th roots or nth roots is zero.
respectively, then the angle
(ii) They are in GP of common ratio d = e 2πi / r , where between AB and AC is given A(z1)
r = 2 , 3 , 4, …, (n − 1).  z − z1 
by arg  3 .
(iii) Sum of their pth powers is zero.  z2 − z1  B(z2)

@iitjeehelps
Practice Exercise
1. The multiplicative inverse of (6 + 5 i )2 is π π
10. If z = 1 + cos + i sin , then sin (arg z ) is equal to
11 60 11 60 9 60 5 5
a. − i b. + i c. − i d. None
61 61 61 61 61 61 10 − 2 5 5 −1 5+1 2 −1
a. b. c. d.
2. If 8 iz + 12 z − 18z + 27i = 0, then the value of | z | is
3 2 4 4 4 4
3 2 3  1
a. b. c. 1 d. 11. If Re   = 3, then z lies on
2 3 4 z 
3. If z1 and z 2 are two complex numbers such that a. circle with centre on Y-axis
| z1| = | z 2| and arg(z1) + arg(z 2 ) = π, then z1 is equal to b. circle with centre on X-axis not passing through origin
a. 2z 2 b. z 2 c. circle with centre on X-axis passing through origin
d. None of the above
c. −z 2 d. None of these
12. If a complex number z lies in the interior or on the
4. If | z − 1| = 1, then arg (z ) is equal to boundary of a circle of radius 3 and centre at ( −4, 0),
1 1 then the greatest and least values of | z + 1| are
a. arg( z ) b. arg( z + 1)
2 3 a. 5, 0 b. 6, 1
1 c. 6, 0 d. None of these
c. arg( z − 1) d. None of these
2
13. If arg(z1) = arg(z 2 ), then
 z − z1  π
5. If z1 = 8 + 4 i , z 2 = 6 + 4 i and arg   = , then z a. z 2 = kz1−1 (k > 0) b. z 2 = kz1 (k > 0)
z − z2 4 c. | z 2| = | z 1| d. None of these
satisfies
a. | z − 7 − 4 i | = 1 b. | z − 7 − 5 i | = 2
14. If z1 and z 2 are two complex numbers such that
z1 z 2
c. | z − 4 i | = 8 d. | z − 7 i | = 18 + = 1, then
z 2 z1
6. If z1, z 2 and z 3 are three complex numbers such that a. z1, z 2 are collinear
1 1 1 b. z1, z 2 and the origin form a right angled triangle
| z1| = | z 2| = | z 3 | = + + = 1, then | z1 + z 2 + z 3 | is
z1 z 2 z 3 c. z1, z 2 and the origin form an equilateral triangle
a. equal to 1 b. less than 1 d. None of the above
c. greater than 3 d. equal to 3 15. If z1, z 2 and z 3 , z 4 are two pairs of conjugate complex
7. If 1, ω and ω 2 are the three cube roots of unity and z  z 
α, β and γ are the cube roots of p , q (< 0), then for any numbers, then arg  1  + arg  2  is equal to
 z4  z3
xα + y β + z γ 
x , y and z , the expression   is equal to a. 0 b. π /2 c. 3π / 2 d. π
xβ + y γ + zα
a. 1 b. ω 16. If a = cos α + i sin α, b = cos β + i sin β, c = cos γ + i sin γ
c. ω 2 d. None of these b c a
and + + = 1, then cos (β − γ ) + cos( γ − α )
x +1 ω ω 2 c a b
+ cos (α − β ) is equal to
8. If x + ω ω2 1 = 3 is an equation of x, where 1,
a. 3/2 b. −3 / 2 c. 0 d. 1
x +ω 2
1 ω
17. The least positive integer n for which
ω and ω 2 are the complex cube roots of unity. Then, n
 1+ i  2 −1 1
the value of x is   = (sec + sin−1 x ), where x ≠ 0; −1≤ x ≤ 1, is
a. 0 b. 1  1− i  π x
c. −1 d. None of these a. 2 b. 4
c. 6 d. 8
9. If a , b and c are integers not all equal and ω is a cube
root of unity (where ω =/ 1), then minimum value of 18. If z = (5 + 12 i ) + (12 i − 5 ), then the principal value of
| a + bω + cω 2| 2 is equal to arg(z ) can be
3 1 π π 3π
a. 0 b. 1 c. d. a. b. − c. − d. All of these
2 2 4 4 4

@iitjeehelps
COMPLEX NUMBERS 763
2 − z π 29. The maximum value of | z |, when z satisfy the condition
19. If arg   = , then locus of z is
2 + z 6 | z + 3 / z | = 3, is
a. straight line b. circle 2 − 21 3 − 20 3 + 21 3 + 20
a. b. c. d.
c. parabola d. None of these 2 2 2 2
π 
334
20. If zr = cis , then z1 z 2 … is equal to 1 i 3
2 r 30. If i = −1, then 4 + 5  − + 
 2 2 
1
a. 1 b. −1 c. −2 d. −  1 i 3
335
2 + 3 − +  is equal to
21. If z1, z 2 and z 3 represent the vertices of an equilateral  2 2 
triangle such that | z1| = | z 2| = | z 3 |, then a. 1 − i 3 b. −1 + i 3 c. i 3 d. −i 3
a. z1 + z 2 = z 3 b. z1 + z 2 + z 3 = 0
Directions (Q. Nos. 31 and 32) Let z = a + ib =(a ,b ) be any
1
c. z1z 2 = d. z1 − z 2 = z 3 − z 2 complex number, ∀a ,b ∈R and i = −1. If (a , b ) ≠ ( 0, 0), then
z3
arg ( z ) = tan−1   , where arg ( z )≤ π
b
22. For any two complex numbers z1 and z 2, we have a 
| z1 + z 2|2 = | z1|2 + | z 2|2, then  π, if arg ( z ) < 0
and arg ( z ) + arg ( − z ) =  .
z  z  − π, if arg ( z )> 0
a. Re 1  = 0 b. Im  1  = 0
 z2  z2 31. If arg(z ) > 0, then arg( −z ) − arg(z ) = λ 1 and if
z  arg(z ) < 0, then arg(z ) − arg ( −z ) = λ 2, where
c. Re 1  ≠ 0 d. None of these a. λ 1 + λ 2 = 0 b. λ 1 − λ 2 = 0
 z2
c. 3λ 1 − 2λ 2 = 0 d. 2λ 1 − 3λ 2 = 0
23. If α, β are two different complex numbers such that
β −α 32. The value of {arg(z )+ arg( −z ) − 2π}{arg( −z ) + arg(z )},
| α| = 1, | β| = 1, then the expression equals
1− α β ∀z = x + iy, where i = −1; x , y > 0, is
1 a. π b. −π c. 0 d. not defined
a. b. 1 c. 2 d. None
2 33. The complex numbers z1, z 2 and z 3 satisfying
13 z1 − z 3 1 − i 3
=
24. The value of ∑ (i n + i n + 1), where i = −1, equals z2 − z3 2
, are the vertices of a triangle
n =1
which is
a. i b. i − 1 c. −i d. 0 a. of area zero b. right angled triangle
25. The continued product of the four values of c. equilateral d. obtuse angled triangle
3/ 4
 π π 34. Let z1 and z 2 be nth roots of unity which subtend a
 cos + i sin  is right angle at the origin, then n must be of the form
 3 3
3 1 a. 4k + 1 b. 4k + 2 c. 4k + 3 d. 4k
a. −1 b. 1 c. d. −
2 2 1+ ω ω2 1+ ω 2
 π  π 35. The value of −ω −(1+ ω ) 2
1+ ω , where ω is cube
26. If xn = cos   + i sin  n  , n ∈ N , then x1 x 2 x 3 … is
 2n  2  −1 −(1 + ω )2
1+ ω
equal to
root of unity, is equal to
a.1 b. −1 c. zero d. None
a. 2 ω b. 3 ω 2 c. − 3 ω 2 d. 3 ω
−1
27. If α + i β = cot (z ), where z = x + iy and α is a constant,
then the locus of z is 36. The square roots of 7 + 24 i are
a. x + y − x cot 2 α − 1= 0
2 2 a. ± ( 4 + 3 i ) b. ± ( 4 − 3 i ) c. ± ( 3 + 4 i ) d. ± ( 3 − 4 i )
b. x 2 + y 2 − 2x cot α − 1= 0  z1 
c. x 2 + y 2 − 2x cot 2 α + 1= 0 37. If | z1| = | z 2| and arg   = π, then z1 + z 2 is equal to
z2
d. x 2 + y 2 − 2x cot 2 α − 1= 0 a. 0 b. purely imaginary
28. If (a1 + ib1) (a 2 + ib 2 )...(an + ibn ) = A + iB , then c. purely real d. None of these
n
b   1− i 
100
∑ tan−1  ai  is equal to 38. If   = a + ib, then
i =1 i  1+ i 
B B B  A a. a = 2, b = − 1 b. a = 1, b = 0
a. b. tan   c. tan−1   d. tan−1  
A  A  A B c. a = 0, b = 1 d. a = − 1, b = 2

@iitjeehelps
764 SELF STUDY GUIDE BITSAT

6i −3i 1 40. Let z1 and z 2 be two complex numbers such that z1 ≠ z 2


39. If 4 3i −1 = x + iy , then and | z1| = | z 2|. If z1 has positive real part and z 2 has
20 3 i (z + z )
negative imaginary part, then 1 2 may be
(z1 − z 2 )
a. x = 3, y = 1
a. purely imaginary
b. x = 1, y = 3
b. real and positive
c. x = 0, y = 3 c. real and negative
d. x = 0, y = 0 d. None of the above

BITSAT Archives
1. The complex number z = x + iy which satisfies the 1+ a
8. If a = cos θ + i sin θ, then is equal to [2009]
z − 3i 1− a
equation = 1, lies on
z + 3i θ
[2014] a. cot b. cot θ
2
a. the X-axis θ θ
b. the straight line y = 3 c. i cot d. i tan
c. a circle passing through origin 2 2
d. None of the above 9. If ω is a complex cube root of unity, then
 sin 2πk i cos 2πk 
6  π
2. The value of ∑  − sin (ω10 + ω 23 ) π −  is equal to [2008]
 is [2013]
 4
k =1
 7 7 
1 1
a. −1 b. 0 c. − i d. i a. b.
2 2
3. If (x + iy )1/ 3 = 2 + 3i , then 3x + 2y is equal to [2013] c. 1 d.
3
a. −20 b. − 60 c. − 120 d. 60 2
10. If (cos θ + i sin θ )(cos 2 θ + i sin 2 θ ) …
4. Area of the triangle in the argand diagram formed by
the complex numbers z, iz, z + iz , where z = x + iy , is (cos nθ + i sin nθ ) = 1, then the value of θ is [2007]
2mπ
[2012] a. b. 4mπ
a. | z | b. | z |2 n(n + 1)
1 4mπ mπ
c. 2 | z |2 d. | z |2 c. d.
2 n(n + 1) n(n + 1)

1 11. For all complex numbers z1 and z 2 satisfying z1 = 12


5. If z = cos θ + i sin θ, then the value of z n + will be
z n
[2011] and z 2 − 3 − 4 i = 5, the minimum value of z1 − z 2 is
a. sin 2n θ b. 2 sin n θ c. 2 cosn θ d. cos 2n θ a. 4 b. 3 [2007]
n
c. 1 d. 2
 1+ i 
6. Find the least value of n for which   = 1. 1+ i 3
 1− i  12. The amplitude of is [2006]
[2010] 3 +i
a. 4 b. 3 c. −4 d. 1 π π
a. b.
1 ω ω2 6 3
c. 0 d. None of these
7. If ω is a cube root of unity, then ω ω 2 1 is equal
ω2 1 ω 13. Let x = α + β, y = αω + βω 2, z = αω 2 + βω, ω being an
imaginary cube root of unity. Product of xyz is [2005]
to [2010, 06]
a. α 2 + β 2 b. α 2 − β 2
a. 1 b. ω c. ω 2 d. 0
c. α 3 + β 3 d. α 3 − β 3

@iitjeehelps
Answer with Solutions
Practice Exercise Now,
1
+
1
+
1
=1
z1 z 2 z 3
1. (d) Let z = ( 6 + 5 i )2 = 36 + 2 × 6 × 5i + 25 i 2 = 11+ 60 i
⇒ | z1 + z 2 + z 3| = 1
Then, z = 11− 60 i
and | z1 + z 2 + z 3| = 1
and | z | = (11)2 + ( 60)2 = 121+ 3600 = 3721 = 61
⇒ | z1 + z 2 + z 3| = 1 ⇒ | z1 + z 2 + z 3| = 1
z
∴ Multiplicative inverse of z = 7. (c)Q p < 0, take p = − q 3 (q > 0)
| z |2
11− 60 i 11 60 ∴ p1/ 3 = q( −1)1/ 3 = − q, − q ω, − qω 3
= 2
= − i
( 61) ( 61) ( 61)2
2
Thus, α = − q, β = − q ω, γ = − q ω 2
2. (a) Given, 8iz 3 + 12z 2 − 18z + 27i = 0 x + yω + zω 2
∴ Given expression = = ω2
⇒ 4z ( 2 iz + 3) + 9 i ( 2 iz + 3) = 0
2
xω + yω 2 + z
⇒ ( 2 iz + 3)( 4z 2 + 9 i ) = 0 x +1 ω ω2
3 8. (d) x + ω ω 2 1 =3
⇒ 2 iz + 3 = 0 or 4z 2 + 9 i = 0 | z | =
2 x + ω2 1 ω
3. (c) Let z1 = r (cos θ1 + i sin θ1)
x + 1+ ω + ω 2 ω ω2
and z 2 = r (cos θ 2 + i sin θ 2 )
LHS = x + ω + ω + 1 ω 2 2
1 [ applying C1 → C1 + C2 + C3]
Since | z 2| = | z1|
x + ω 2 + 1+ ω 1 ω
Also, arg ( z1) + arg ( z 2 ) = π
∴ arg ( z 2 ) = π − arg ( z1) ⇒ arg ( z 2 ) = π − θ1 x + 0 ω ω2 1 ω ω2
∴ z 2 = r1 {cos( π − θ1) + i sin ( π − θ1)} = x+0 ω 2
1 = x 1 ω2 1 [Q1+ ω + ω 2 = 0]
= r1 ( − cos θ1 + i sin θ1) x+0 1 ω 1 1 ω
= − r1 (cos θ1 − i sin θ1) = − z1
⇒ z1 = − z 2 3 1+ ω + ω 2 1+ ω + ω 2
=x 1 ω2 1
4. (c) Given, | z − 1| = 1
1 1 ω
⇒ z − 1= ei θ , where arg( z − 1) = θ …(i)
iθ [applying R1 → R1 + R2 + R3]
⇒ z =e + 1 ⇒ z = 1+ cos θ + i sin θ
3 0 0
θ θ θ
= 2 cos2 + 2 i sin ⋅ cos [Qei θ = cos θ + i sin θ] = x 1 ω2 1 = 3x (ω 3 − 1)
2 2 2
θ 1 1 1 ω
⇒ arg ( z ) = = arg ( z − 1) [from Eq. (i)]
2 2 ⇒ 3x (ω − 1) = 3 ⇒ x (ω 3 − 1) = 1 ⇒ x × 0 = 1 [Qω 3 = 1]
3

z − z1 (x − 8 ) + i ( y − 4 ) ∴ x=
1
5. (b) Hint = [not defined]
z − z 2 (x − 6 ) + i ( y − 4 ) 0
 y − 4 9. (b) | a + bω + cω 2|2 = (a + bω + cω 2 )(a + b ω + c ω 2 )
−1  y − 4  π
⇒ tan−1   − tan  =
 x − 8  x − 6 4 = (a + bω + cω 2 )(a + bω 2 + cω ) [Q ω = ω 2 and ω 2 = ω]
2 = a 2 + b 2 + c 2 − ab − bc − ca
(y − 4)
(x − 8)(x − 6) π 1
⇒ = tan = 1 = [(a − b )2 + (b − c )2 + (c − a )2]
(x − 8)(x − 6) + (y − 4)2 4 2
(x − 8)(x − 6) So, it has minimum value 1 for a = b = 1and c = 2.
θ
⇒ (x − 7)2 + (y − 5)2 = ( 2 )2 10. (b) If z = 1+ cos θ + i sin θ, then arg ( z ) =
2
⇒ | z − 7 − 5i | = 2 π
6. (a) | z1| = | z 2| = | z 3| = 1 π
[given] ∴ arg ( z ) = 5 =
2 10
Now, | z1| = 1 ⇒ | z1| = 1
2
π 5 −1
⇒ z1z1 = 1 ⇒ sin (arg z ) = sin   = sin 18° =
 10 4
Similarly, z 2 z 2 = 1, z 3 z 3 = 1

@iitjeehelps
766 SELF STUDY GUIDE BITSAT

 1  z   1 z  18. (d) Here, (5 + 12 i ) = 5 + 2 ( 3)( 2 i ) = ( 3 + 2 i )2 = ± ( 3 + 2 i )


11. (c) Given, Re   = 3 ⇒ Re  2  = 3 Q = 2 
z | z|   z | z|  and (12 i − 5) = −5 + 2 ( 3)( 2 i )
x
⇒ = 3 ⇒ 3x 2 + 3y 2 − x = 0 = ( 2 + 3 i )2 = ± ( 2 + 3 i )
x2 +y2
So, it is a circle whose centre is on X-axis and passes ∴ z = (5 + 12 i ) + (12 i − 5)
through the origin. = ± (3 + 2 i ) ± (2 + 3 i )
12. (c) Given, | z + 4 | ≤ 3 ⇒ z = 5 + 5 i, −1+ i, −5 − 5 i, 1− i
Now, | z + 1| = | z + 4 − 3 | ≤ | z + 4 | + |3 | π 3π 3π π
Hence, the principal values of arg( z ) are , , − ,− .
=| z + 4| + 3 ≤ 3 + 3 = 6 4 4 4 4
Hence, the greatest value of | z + 1| = 6 π
19. (b) Hint arg ( 2 − z ) − arg ( 2 + z ) =
Since, least value of the modulus of a complex number 6
is 0. π
⇒ arg [( 2 − x ) − iy ] − arg [( 2 + x ) + iy ] =
∴ | z + 1| = 0 ⇒ z = − 1 6
Now, | z + 4 | = |−1+ 4 | = 3  −y  −1  y  π
⇒ tan−1   − tan  =
So, | z + 4 | ≤ 3 is satisfied by z = − 1.  2 −x  2+x 6
∴ Least value of | z + 1| = 0 4y π 1
zz ⇒ = tan =
13. (a) Here, z1 = 1 1 = | z1|2 z1−1 x −4+y
2 2
6 3
z1
20. (b) The argument of a product is the sum of the arguments
⇒ arg ( z1−1) = arg ( z1) = arg ( z 2 ) ⇒ z 2 = kz1−1 (k > 0) of the factors.
z1 z 2 π π π
14. (c) Given, + = 1 ⇒ z12 + z 22 = z1z 2 ∴ arg( z1 ⋅ z 2... ) = + 2 + 3 + ...
z 2 z1 2 2 2
⇒ z12 + z 22 + z 32 = z1z 2 + z1z 3 + z 2z 3, where z 3 = 0 π
 a 
So, z1, z 2 and the origin form an equilateral triangle. = 2 =π Q S ∞ = 
1−
1  1 −r 
15. (a) Since, z 2 = z1 and z 4 = z 3 2
∴ z1z 2 = z1z1 = | z1|2 and z 3z 4 = z 3z 3 = | z 3|2 ∴ z1z 2z 3… = cis π = − 1
z  z  zz   | z |2  21. (b) Since, | z1| = | z 2| = | z 3|
Now, arg  1  + arg  2  = arg  1 2  = arg  1 2 
 z4  z3  z 4z 3   | z 3|  ∴O is the circumcentre of an equilateral ∆ABC.
x1 + x 2 + x 3 y + y2 + y3
z 2
 z1  ∴ =0= 1
= arg  1  = 0 Q is a real number  3 3
 z3  z
   3  where, z1 = x1 + iy1
a x1 + x 2 + x 3 y + y2 + y3
16. (d) Here, = cos(α − β ) + i sin (α − β ), ⇒ +i 1 =0
b 3 3
b c ⇒ (x1 + iy1) + (x 2 + iy 2 ) + (x 3 + iy 3 ) = 0
= cos(β − γ ) + i sin (β − γ ) and = cos( γ − α ) + i sin ( γ − α )
c a ⇒ z1 + z 2 + z 3 = 0
a b c
Q + + =1 22. (a) We have,| z1 + z 2|2 = | z1|2 + | z 2|2
b c a
∴ cos(α − β ) + cos(β − γ ) + cos( γ − α ) + i [sin (α − β ) ⇒ | z1|2 + | z 2|2 + 2 | z1|| z 2| cos(θ1 − θ 2 ) = | z1|2 + | z 2|2
+ sin (β − γ ) + sin ( γ − α )] = 1+ 0i where, θ1 = arg( z1), θ 2 = arg( z 2 )
On equating real parts, we get π
⇒ cos(θ1 − θ 2 ) = 0 ⇒ θ1 − θ 2 =
cos(α − β ) + cos(β − γ ) + cos( γ − α ) = 1 2
 1 z  π  z  |z |  π
17. (b) For −1≤ x ≤ 1and x ≠ 0, sec−1   = cos−1 x ⇒ arg  1  = ⇒ Re  1  = 1 cos   = 0
x   z2 2  z 2  | z 2|  2
 1 π
∴ sec−1   + sin−1 x = cos−1 x + sin−1 x = Q | β| = 1
x  2 β −α β −α
23. (b) = ∴ | β|2 = 1
 1+ i 
n
2 π 1− α β β β − α β  
∴   = × =1 ∴ β β = 1
 1− i  π 2
β −α 1 | β − α| | β − α|
(1+ i )2 
n
= = = =1
⇒   =1 ⇒ i =1
n β( β − α ) | β| |β − α | | β − α |
 2 
[Q| β| = 1and | β − α | = | β − α |]
So, the least positive integral value of n is 4.

@iitjeehelps
COMPLEX NUMBERS 767
∴ (a1 + ib1)(a 2 + ib2 )… (an + ibn ) = A + iB
β −α β −α  1 ⇒ r1r2 … rn [cos(θ1 + θ 2 + …+ θn ) + i sin (θ1 + θ 2 + ...+ θn )]
or = Q α α = 1⇒ α = α 
1− α β 1− 1 β = A + iB ⇒ r1 r2 r3 … rn = A 2 + B 2
α
B
| β − α| and θ1 + θ 2 + … + θn = tan−1  
= | α| = 1⋅ 1= 1 [Q| β − α | = | α − β|]  A
| α − β| n
b  b  b 
13
24. (b) ∑ (i + in n+1
13
) = ∑ i (1+ i )
n ∑ tan−1  ai  = tan−1  a11 + … + tan−1  ann 
i =1 i
n =1 n =1
B
= (1+ i )[i + i 2 + i 3 + … + i13] = tan−1  
 A
= (1+ i )[(i + i 2 + i 3 + i 4 ) + (i 5 + i 6 + i 7 + i 8 ) 3
29. (c) | z | = | z + 3 / z − 3 / z | ≤ | z + 3 / z | + |−3 / z | = 3 +
+ (i 9 + i10 + i11 + i12 ) + i13] [Qi13 = i ] | z|
= (1+ i )[ 0 + 0 + i ] = i + i 2 = i − 1 3 t2 − 3t − 3  1 21
2
Let | z | = t , t ≤ 3 + ⇒ ≤ 0 ⇒ t −  ≤
3/ 4 3/ 4 t t  2 4
 π π   π  π 
25. (b)  cos + i sin  = cos 2kπ +  + i sin 2kπ +  
 3 3   3   3  3 − 21 3 + 21
So, <t ≤
3/ 4 2 2
 π π
= cos( 6k + 1) + i sin ( 6k + 1)  ; k = 0, 1, 2,3, ... 3 + 21
 3 3 Hence, the maximum value of | z | is .
2
 π 3  π 3
= cos ( 6k + 1) ⋅  + i sin ( 6k + 1) ⋅  30. (c) Hint The given expression becomes
 3 4  3 4
4 + 5ω 334 + 3ω 335 = 4 + 5ω + 3ω 2
[by De-Moivre’s theorem]
[Qω 334 = (ω 3 )111 ⋅ ω and ω 335 = (ω 3 )111 ⋅ ω 2]
 π  π
= cos( 6k + 1)  + i sin ( 6k + 1) 
 4  4 31. (b) Given, arg( z ) > 0 and arg( − z ) − arg( z ) = λ 1, then
π π (arg z ) + arg( − z ) = λ 1
∴ Roots are cos + i sin , (k = 0);
4 4 ⇒ −π = λ1 ⇒ λ1 = − π
7π 7π 13π 13π
cos + i sin , (k = 1); cos + i sin ,(k = 2) Again, given arg( z ) < 0
4 4 4 4 Then, arg( z ) − arg ( − z ) = λ 2
19 π 19 π
and cos + i sin ,(k = 3) ⇒ − [ − arg ( z ) + arg ( − z )] = λ 2
4 4
⇒ λ2 = − π
 π π 7π 7π 
Products of roots = cos + i sin  cos + i sin ∴ λ1 = λ 2
 4 4  4 4 
13π 13π   19 π 19 π  32. (a) {arg( z ) + arg ( − z ) − 2π } {arg( − z ) + arg ( z )}

cos 4 + i sin 4  cos 4 + i sin 4  = {arg( z ) + π − arg ( z ) − 2π } {arg( − z ) − arg( z )}
 π 7π 13π 19 π   π 7π 13π 19 π  = ( − π ) − {arg( z ) − arg( − z )} = ( − π )( − π ) = π
= cos  + + +  + i sin  + + + 
4 4 4 4  4 4 4 4 
z1 − z 3 1 − i 3 z3
= cos 10π + i sin 10π = 1 33. (c) =
z2 − z3 2
π/3
 π  π
i
π = cos  −  + i sin  − 
26. (b) We have, xn = e 2n  3  3
π
1  −i
iπ  +
1
+
 2 22 23
1
+… 

=e 3
  z2
∴ x1 x 2 x 3… = e = −1 z1 − z 3
z1
∴ = | e −i π / 3| = 1
27. (d) α + i β = cot −1 ( z ) or cot (α + i β) = x + iy z2 − z3
and cot (α − i β ) = x − iy π
and angle between z1 − z 3 and z 2 − z 3 is .
∴ cot 2 α = cot [(α + i β ) + (α − i β )] 3
cot (α + i β ) ⋅ cot(α − i β ) − 1 (x 2 + y 2 − 1) Therefore, triangle is equilateral.
= =
cot (α + i β ) + cot(α − i β ) 2x cos 2 r π sin 2 r π
34. (d) z = (1)1/n = (cos 0 + i sin 0)1/n = +i
∴ x 2 + y 2 − 2x cot 2 α − 1= 0 n n
i 2r π
28. (c) Let ak + ibk = rk (cos θk + i sin θk ) or z = e n , where r varies from 0 to n − 1and each root is
b
Then, rk = ak2 + bk2 and tan θk = k unimodular as | ei θ | = 1.
ak

@iitjeehelps
768 SELF STUDY GUIDE BITSAT

i 2kπ 100
 1− i 
Let z1 = 1 and z 2 = e n 38. (b) Given,   = a + ib
π  1+ i 
i
where, z 2 − 0 = ( z1 − 0)e 2 [by given condition]  1− i   1− i  
100

2k π π ⇒   ×  = a + ib
 1+ i   1− i  
i i
or e n = 1⋅ e 2
100
∴ n = 4k (1− i )2   −2 i 
100
⇒ a + ib =   = = ( −i )100
35. (c) Using 1+ ω + ω 2 = 0, then  2   2 
1+ ω ω2 −ω ⇒ a + ib = [(i )4] 25 = 1+ 0 i [Q( −1)100 = 1]
∆ = 1+ ω 2
ω −ω 2
Hence, a = 1, b = 0
ω2 + ω ω − ω2 6 i −3 i 1 6i 0 1
Applying C1 → C1 + C2 39. (d) We have, 4 3 i −1 = 4 0 −1
0 ω2 −ω 20 3 i 20 0 i
∆= 0 ω − ω 2 = (ω 2 + 2ω )( − ω + ω 2 ) = − 3ω 2 [applying C2 → C2 + 3iC3]
ω 2 + 2ω ω − ω2 ⇒ 0 = 0 + 0i
∴ x = 0, y = 0
36. (a) Let z = 7 + 24 i, then | z | = 25 , x = 7
40. (a) Let z1 = a + ib = (a, b ) and z 2 = c − id = (c , − d )
 25 + 7 1/ 2  25 − 7 
1/ 2
∴ Square root = ±   +i    [Q y > 0] where, b > 0 and d > 0
 2   2  
 Then, | z1| = | z 2|
= ± ( 4 + 3i ) ⇒ a + b2 =c 2 + d 2
2

z  z1 + z 2 (a + ib ) + (c − id )
37. (a) We have, arg 1  = π Now, = ..(i)
 z2 z1 − z 2 (a + ib ) − (c − id )
⇒ arg( z1) − arg( z 2 ) = π [(a + c ) + i(b − d )] [(a − c ) − i(b + d )]
=
⇒ arg( z1) = arg( z 2 ) + π [(a − c ) + i(b + d )] [(a − c ) − i(b + d )]
Let arg( z 2 ) = θ (a 2 + b 2 ) − (c 2 + d 2 ) − 2 (ad + bc ) i
=
Then, arg( z1) = π + θ a 2 + c 2 − 2ac + b 2 + d 2 + 2bd
∴ z1 = | z1|[cos( π + θ ) + i sin ( π + θ )] −(ad + bc ) i
= 2
= | z1|( − cos θ − i sin θ ) a + b 2 − ac + bd
and z 2 = | z 2 |(cos θ + i sin θ ) [Q| z1| = | z 2|, from Eq. (i)]
= | z1|(cos θ + i sin θ ) [Q | z1| = | z 2|] (z + z2 )
Hence, z1 + z 2 = 0 So, 1 is purely imaginary.
( z1 − z 2 )

BITSAT Archives
z − 3i  e i 2 π / 7 − e 2 πi 
1. (a) Given, = 1⇒| z − 3i| = | z + 3i| = −i 
z + 3i i 2π / 7 
 1− e 
[if | z − z1| = | z − z 2|, then it is a perpendicular e i 2 π / 7
− 1
bisector of z1 and z 2] = −i  i 2π / 7 
=i
 1 − e 
Hence, perpendicular bisector of (0,3) and (0, −3) is X-axis.
6 3. (c) (x + iy )1/ 3 = 2 + 3i
 sin 2πk i cos 2πk 
2. (d) ∑  −  On cubing both sides, we get
k =1
 7 7 
6 6
x + iy = ( 2 + 3i )3
 cos 2π k i sin 2πk 
= ∑ −i  −  = −i ∑ e
i 2 πk / 7
⇒ x + iy = ( 2)3 + ( 3i )3 + 3 × 2 × 3i ( 2 + 3i )
 7 7 
k =1 k =1 ⇒ x + iy = 8 − 27i + 18 i ( 2 + 3 i )
= − i[ei 2π / 7 + ei 4π / 7 + .....+ ei 12π / 7] ⇒ x + iy = 8 − 27i + 36 i − 54 ⇒ x + iy = − 46 + 9 i
 (1− ei 12π / 7 )  ei 2π / 7 − ei 14π / 7  On comparing real and imaginary parts both sides, we get
= − i ei 2π / 7  = − i   x = − 46, y = 9
 1− ei 2π / 7   1− e
i 2π / 7

Then, 3x + 2y = 3 ( −46) + 2(9 ) = − 138 + 18 = − 120

@iitjeehelps
COMPLEX NUMBERS 769
4. (d) Since, iz = zeiπ /2. This implies that Y ∴ cos(θ + 2θ + 3θ + K + n θ ) + i sin(θ + 2θ + 3θ + K + n θ ) = 1
iz is the vector obtained by rotating B  n(n + 1)   n(n + 1) 
⇒ cos  θ + i sin  θ = 1
vector z in anti-clockwise direction iz  2   2 
through 90° . A(z) On comparing the coefficients of real and imaginary parts
∴ OA ⊥ AB
both sides, we get
1
So, area of ∆ OAB = OA × OB  n (n + 1)   n (n + 1) 
2 cos  θ = 1 and sin  θ = 0
O
X  2   2 
1 1
= | z || iz | = | z |2 n (n + 1) 4mπ
2 2 ∴ θ = 2m π ⇒ θ =
2 n (n + 1)
5. (c) Given, z = cos θ + i sin θ
1 11. (d) The two circles whose centre and radius are C1 ( 0, 0),
∴ z n + n = (cos θ + i sin θ )n + (cos θ + i sin θ )−n
z r1 = 12, C2( 3, 4), r2 = 5 and it passes through origin, i.e. the
= cos nθ + i sin nθ + cos nθ − i sin nθ centre of C1.
= 2 cosnθ
A B
n 5
 1+ i 1 + i  5
6. (a) Hint  ×  =1 ⇒ i =1
n
 1− i 1 + i  C2(3, 4)
C1(0, 0)
7. (d) Applying C1 → C1 + C2 + C3, we get
1 ω ω 2 1+ ω + ω 2 ω ω 2 0 ω ω 2
ω ω2 1 = ω + ω2 + 1 ω2 1 = 0 ω2 1 = 0 Now, C1 C2 = 32 + 42 = 5
ω2 1 ω ω2 + ω + 1 1 ω 0 1 ω
and r1 − r2 = 12 − 5 = 7
8. (c) Since, a = cos θ + i sin θ ∴ C1C2 < r1 − r2
1+ a 1+ cos θ + i sin θ
∴ = Hence, circle C2 lies inside the circle C1.
1− a 1− cos θ − i sin θ
From figure, the minimum distance between them is
[(1+ cos θ ) + i sin θ] [(1− cos θ ) + i sin θ] AB = C1B − C1A = r1 − 2r2 = 12 − 10 = 2
=
[(1− cos θ ) − i sin θ] [(1− cos θ ) + i sin θ]
1+ i 3 1+ i 3 ( 3 − i )
θ θ 12. (a) Let z = = ×
i ⋅ 4 sin ⋅ cos 3 + i ( 3 + i) ( 3 −i)
2i sin θ 2 2 = i cot θ
= =
(1− cos θ ) + sin θ
2 2 θ 2 3 − i + 3i + 3 3 +i
4 sin2 = =
2 3+1 2
9. (a) Since, ω is a cube root of unity.  1 π
∴ amp(z ) = tan−1  =
 π  π  3 6
∴ sin (ω10 + ω 23 ) π −  = sin(ω + ω 2 )π − 
 4  4 13. (c) Given that, x = α + β, y = αω + βω 2, z = αω 2 + βω
 π 
= sin  − π −  [Q 1+ ω + ω 2 = 0] Now, xyz = (α + β )(αω + βω 2 )(αω 2 + βω )
 4
π π = (α + β )(α 2ω 3 + α β ω 2 + α β ω 4 + β 2ω 3 )
 1
= − sin π +  = sin = = (α + β )[α 2 + α β (ω 2 + ω) + β 2]
 4 4 2
10. (c) We have, (cos θ + i sin θ )(cos 2 θ + i sin 2 θ ) [Q1+ ω + ω 2 = 0 and ω 3 = 1]

…(cos nθ + i sin n θ ) = 1 = (α + β )(α 2 − α β + β 2 ) = α 3 + β 3

@iitjeehelps
2
Quadratic Equation

Quadratic Equation
An equation of the form ax 2 + bx + c = 0, where a ≠ 0 and a , b , c , x ∈ R, is called a real quadratic
equation. The numbers a , b and c are called the coefficients of the equation. The quantity D = b 2 – 4ac
−b ± D
is known as the discriminant of the equationax 2 + bx + c = 0 and its roots are given by x = .
2a
An equation of the form az 2 + bz + c = 0, where a ≠0 and a, b, c, z ∉ C (complex), is called a complex
−b ± D
quadratic equation and its roots are given by z = .
2a

Nature of Roots of Quadratic Equation


Nature of roots of a quadratic equation ax 2 + bx + c = 0 implies whether the roots are real or
imaginary by analysing the quantity D.
1. Let a , b , c ∈ R and a ≠ 0, then the equation ax 2 + bx + c = 0 has
(i) real and distinct roots if and only if D > 0.
(ii) real and equal roots if and only if D = 0.
(iii) complex roots with non-zero imaginary parts if and only if D < 0. If
p + iq ( where, p , q ∈ R, q ≠ 0) is one root of ax 2 + bx + c = 0, then second root will be ( p − iq ).
2. If a , b , c ∈Q and D is a perfect square, then ax 2 + bx + c = 0 has rational roots.
3. If a , b , c ∈ Q and p + q ( where, p , q ∈ Q ) is an irrational root of ax 2 + bx + c = 0, then other root
will be ( p − q ).
4. If a = 1, b , c ∈ I are roots of ax 2 + bx + c = 0, are rational numbers, then these roots must be
integers.
5. If ax 2 + bx + c = 0 is satisfied by more than two distinct complex numbers, then it becomes an
identity i.e. a = b = c = 0.

@iitjeehelps
QUADRATIC EQUATION 771
6. If the roots of ax 2 + bx + c = 0 are both positive, then (i) α 2 + β 2 = (α + β )2 − 2 αβ
the signs of a and c should be like and opposite to the (ii) (α − β )2 = (α + β )2 − 4 αβ
sign of b.
(iii) α3 + β3 = (α + β )3 − 3 αβ (α + β )
7. If the roots of ax 2 + bx + c = 0 are both negative, then
(iv) α3 − β3 = (α − β )3 + 3 αβ (α − β )
signs of a ,b and c should be like.
8. If the roots of ax 2 + bx + c = 0 are equal in magnitude
5. Common Roots (Conditions)
but opposite in sign, then b = 0 and c < 0.
Suppose that the quadratic equations are ax 2 + bx + c = 0 and
9. If the roots of ax 2 + bx + c = 0 are reciprocal to each
a ′ x 2 + b ′ x + c ′ = 0.
other, then c = a.
(i) When one root is common, then the condition is
10. In the equation ax 2 + bx + c = 0 ( where, a ,b ,c ∈ R), if
(a′ c − ac′ )2 = (bc′ − b′ c )(ab′ − a′ b ).
c
a + b + c = 0, then the roots are 1, and if a − b + c = 0, (ii) When both roots are common, then the condition is
a a b
c = .
then the roots are −1 and − . a′ b′
a

Relation between Roots Formation of an Equation


and Coefficients 1. Quadratic Equation
If α and β are the roots of a quadratic equation, then the
1. Quadratic Roots equation will be of the form
If α and β are the roots of quadratic equation x 2 − (α + β )x + αβ = 0
ax 2 + bx + c = 0, then
Sum of roots = α + β = −
b 2. Cubic Equation
a
If α , β and γ are the roots of the cubic equation, then the
c
and product of roots = αβ = equation will be of the form
a
x 3 − (α + β + γ )x 2 + (αβ + βγ + γα ) x − αβγ = 0
2. Cubic Roots
If α , β and γ are the roots of cubic equation
3. Biquadratic Equation
ax 3 + bx 2 + cx + d = 0; a ≠ 0, then If α , β , γ and δ are the roots of the biquadratic equation,
b then the equation will be of the form
α +β + γ = −
a x 4 − (α + β + γ + δ )x 3 + (αβ + βγ + γα + αδ + βδ + γδ )x 2
c d − (αβγ + βγδ + γδα + δαβ ) x + αβγδ = 0
βγ + γα + αβ = and αβγ = −
a a

3. Biquadratic Roots Sign of Quadratic Equation


If α , β , γ and δ are the roots of biquadratic equation Let f ( x ) = ax 2 + bx + c or y = ax 2 + bx + c, where a , b , c ∈ R
ax 4 + bx 3 + cx 2 + dx + e = 0 ; a ≠ 0, then and a ≠ 0.
b
α +β + γ +δ= − (i) If a > 0 and D < 0, then f ( x ) > 0,∀x ∈ R.
a
c (ii) If a < 0 and D < 0, then f ( x ) < 0,∀x ∈ R.
(α + β )( γ + δ ) + αβ + γδ =
a (iii) If a > 0 and D = 0, then f ( x ) ≥ 0,∀x ∈ R.
d e (iv) If a < 0 and D = 0, then f ( x ) ≤ 0,∀x ∈ R.
αβ ( γ + δ ) + γδ (α + β ) = − and αβγ δ =
a a (v) If a > 0, D > 0 and f ( x )=0 have two real roots α and
β ( where, α < β ), then f ( x ) > 0, ∀ x ∈( − ∞ , α ) ∪ (β , ∞ )
4. Symmetric Roots and f ( x ) < 0, ∀ x ∈(α , β ).
If α and β are the roots of quadratic equation (vi) If a < 0, D > 0 and f ( x ) = 0 have two real roots α and β
ax 2 + bx + c = 0, then to find the symmetric function of α (where, α < β ), then f ( x ) < 0, ∀ x ∈ ( −∞ , α ) ∪ (β , ∞ )
and β, we use the following results: and f ( x ) > 0, ∀ x ∈(α , β ).

@iitjeehelps
772 SELF STUDY GUIDE BITSAT

Greatest and Least Values of Quadratic


Expression y = ax 2 + bx + c
The maximum and minimum values are as follows:

Value of a Graph Maximum value Minimum value

a >0 Not defined −


D
at x = −
b
4a 2a

 b D
− ,− 
 2a 4a 

a <0  b D D b Not defined


− ,−  − at x = −
 2a 4a  4a 2a

Practice Exercise
1. If x = 1 + 1 + 1 + ... , then x equals 7. The condition that x 3 − px 2 + qx − r = 0 may have two
of its roots equal to each other but of opposite sign, is
1+ 5 2+ 5 −1 + 5 −1 − 5 a. r = pq b. r = 2p 3 + pq
a. b. c. d.
2 2 2 2 c. r = p q
2
d. None of these
2. If a + b + c = 0, then the roots of the equation 8. The number of values of k for which the equation
4ax 2 + 3bx + 2c = 0, where a , b , c ∈ R , are x 2 − 3x + k = 0 has two distinct roots lying in the
a. real and distinct b. imaginary interval (0, 1), is
c. real and equal d. infinite
a. three b. two
3. If the roots of the quadratic equation c. infinite d. no value of k will satisfy
x 2 − 4x − log3 a = 0 are real, then the least value of a 3c
9. If ax + 2bx − 3 c = 0 has no real root and
2
< a + b,
is 4
1 1 then the range of c is
a. 81 b. c. d. None
81 64 a. ( −1, 1) b. ( 0, 1) c. ( 0, ∞ ) d. ( − ∞, 0)
4. If α, β are the roots of ax 2 + bx + c = 0 and α + h, β + h 1 1 1
10. If the roots of the equation + = are equal
are the roots of px 2 + qx + r = 0, then the value of h is x + p x +q r
b q  1 b q  1 a p in magnitude but opposite in sign, then the product of
a.  −  b.  −  c. −  −  d. None the roots is
a p 2 a p 2 b q 
a. −2 ( p 2 + q 2 ) b. − ( p 2 + q 2 )
5. If the equation ( 3x )2 + ( 27 × 31/ p − 15 ) x + 4 = 0 has
− (p 2 + q 2 )
equal roots, then p is equal to c. d. −pq
1 2
a. zero b. 2 c. − d. None
2 11. The harmonic mean of the roots of the equation
6. If the equations x + ax + 12 = 0, x + bx + 15 = 0 and
2 2 (5 + 2 )x 2 − ( 4 + 5 )x + ( 8 + 2 5 ) = 0 is
x 2 + (a + b )x + 36 = 0 have a common positive root, then a. 2 b. 4 c. 7 d. 8
the ordered pair (a, b ) is 12. The roots of the equation | x 2 − x − 6 | = x + 2 are
a. ( − 6, − 7) b. ( − 7, − 8) c. ( − 6, − 8) d. ( − 8, − 7) a. −2, 1, 4 b. 0, 2, 4 c. 0, 1, 4 d. −2, 2, 4

@iitjeehelps
QUADRATIC EQUATION 773
π P Q 26. If b > a , then the equation (x − a ) (x − b ) − 1 = 0 has
13. In ∆PQR , R = . If tan and tan are the roots of
2 2 2 a. both roots in [a, b]
the equation ax 2 + bx + c = 0, then b. both roots in ( −∞, a )
a. a = b + c b. b = c + a c. c = a + b d. b = c c. one root in (b, ∞ ) and other in (b, − ∞ )
14. If the roots of the equation (a + 1)x 2 − 3 ax + 4a = 0 d. one root in ( −∞, a )
(a ≠ − 1) are greater than unity, then the values of a 27. If α and β(α < β ) are the roots of the equation
are
x 2 + bx + c = 0, where c < 0 < b , then
 16   16 
a. − , − 1 b. [ 0, − 1] c. − ,1 d. [0, 1]
 7   7  a. 0 < α < β b. α < 0 < β < | α |
c. α < β < 0 d. α < 0 < | α | < β
15. The number of real roots of the equation
(x − 1)2 + (x − 2)2 + (x − 3 )2 = 0 is 28. If the roots of the equation lx 2 + mx + m = 0 are in
a. 2 b. 1 c. 0 d. 3 a b m
x −1 − x −1
ratio a : b , then the value of + + is equal to
16. If y = 3 +3 (where, x is real), then the least b a l
value of y is a. 0 b. 1 c. 3 d. None
a. 2 b. 6 c. 2/3 d. None 29. For the equation 3x + px + 3 = 0, p > 0, if one of the
2

17. If the equation x 2 − ( 2 + m )x + (m 2 − 4m + 4 ) = 0 has roots is square of the other, then p is equal to
coincident roots, then 1 2
a. m = 0, m = 1 b. m = 0, m = 2 a. b. 1 c. 3 d.
2 3
2 2
c. m = , m = 6 d. m = , m = 1 30. If x 2 + 5 = 2x − 4 cos (a + b ), where a , b ∈ ( 0, 5 ), is
3 3
satisfied for atleast one real x, then the maximum
18. If α, β are the roots of x 2 + x + 2 = 0 and γ, δ are the value of a + b in [ 0, 2π ] is
roots of x 2 + 3x + 4 = 0, then a. 3 π b. 2 π
(α + γ )(α + δ )(β + γ )(β + δ ) is equal to c. π d. None of these
a. −18 b. 18 c. 24 d. 44
31. If a , b , c ∈ R and ax 2 + bx + c = 0 has no real roots,
19. If a + b + c = 1, then ab + bc + ca lies in the interval
2 2 2
then
 1  1  a. c (a + b + c ) > 0 b. c − c (a − b − c ) > 0
a. [1, 2] b. 0, c. − , 1 d. [0, 1]
 2   2 
c. c + c (a − b − c ) > 0 d. c (a − b − c ) > 0
1
20. If x 2 − 5x + 1 = 0, then x 5 + is equal to 32. The value of P such that the difference of the roots of
x5 the equation x 2 − Px + 8 = 0 is 2, is
a. 2424 b. 3232 c. 2525 d. None a. ± 3 b. ± 6 c. ± 2 d. ± 1
21. The value of k for which the number 3 lies between 33. If α and β are the roots of the equation
the roots of the equation
ax 2 + bx + c = 0, then the quadratic equation whose
x 2 + (1 − 2k )x + (k 2 − k − 2) = 0, is given by α β
a. k < 2 b. 2 < k < 5 c. 2 < k < 3 d. k > 5 roots are and , is
1+ α 1+ β
22. If α , β and γ are the roots of the equation, a. a x 2 − b (1− x ) + c (1− x )2 = 0
1+ α 1+ β 1+ γ b. a x 2 − b (x − 1) + c (x − 1)2 = 0
x 3 − x − 1 = 0, then + + has the value
1− α 1− β 1− γ c. a x 2 + b (1− x ) + c (1− x )2 = 0
a. 0 b. − 1 c. − 7 d. 1 d. a x 2 + b (x + 1) + c (1+ x )2 = 0

23. If one root of the equation x 2 − λx + 12 = 0 is even prime 9c


34. If a + b + c > and the equation
while x + λx + µ = 0 has equal roots, then µ is equal to
2 4
ax 2 + 2 bx − 5c = 0 has non-real complex roots, then
a. 8 b. 16 c. 24 d. 32
a. a > 0, c > 0 b. a > 0, c < 0
24. If α and β are the roots of the equation ax + bx + c = 0, 2
c. a < 0, c < 0 d. a < 0, c > 0
then (1 + α + α 2 ) (1 + β + β 2 ) is equal to
(x − b ) (x − c )
a. 0 b. positive c. negative d. None 35. For real values of x, the expression will
(x − a )
25. The number of real solutions of the equation assume all real values provided
2 2
(5 + 2 6 )x − 3 + (5 − 2 6 )x − 3 = 10 is a. a ≤ c ≤ b b. b ≥ a ≥ c
a. 2 b. 4 b. 6 d. None c. b ≤ c ≤ a d. a ≥ b ≥ c

@iitjeehelps
BITSAT Archives
1. If α and β are the roots of the equation x 2 − px + q = 0, 8. If the roots of the equation ax 2 + bx + c = 0 are real
then the value of and distinct, then [2010]
α2 + β2 2 α3 + β3  3 b
(α + β )x −  x +  x + ... ,x is [2014] a. both roots are greater than −
 2   3  2a
b
b. both roots are less than −
a. log(1 − px + qx 2 ) b. log (1 + px − qx 2 ) 2a
c. log(1 + px + qx 2 ) d. None of these b
c. one of the roots exceeds −
3 − |x | 2a
2. The set of all real x satisfying the inequality ≥ 0, d. None of the above
4 − |x |
9. The number of solutions of the equation z 2 + z = 0 is
is [2013]
[2010, 09]
a. [ − 3, 3] ∪ ( − ∞, − 4) ∪ ( 4, ∞ )
a. 1 b. 2 c. 3 d. 4
b. ( − ∞, − 4) ∪ ( 4, ∞ )
10. If the sum of the roots of the equation ax 2 + 2x + 3a = 0
c. ( − ∞, − 3) ∪ ( 4, ∞ )
is equal to their product, then the value of a is [2009]
d. ( − ∞, − 3) ∪ ( 3, ∞ )
a. −2 / 3 b. −3
3. If N is any four digit number say x1, x 2, x 3 , x 4 , then the c. 4 d. −1/ 2
N
maximum value of is equal to 11. If α + β = − 2 and α 3 + β 3 = − 56, then the quadratic
x1 + x 2 + x 3 + x 4 [2013] equation whose roots are α and β, is [2008]
1111
a. 1000 b. a. x 2 + 2x − 16 = 0 b. x 2 + 2x + 15 = 0
4
c. x 2 + 2x − 12 = 0 d. x 2 + 2x − 8 = 0
c. 800 d. None
4. If 4 − 5i is the root of the quadratic equation 12. The cubic equation whose roots are thrice to each of
x + ax + b = 0, then (a , b ) is equal to
2
[2012] the roots of x 3 + 2x 2 − 4x + 1 = 0, is [2008]
a. ( 8, 41) b. ( − 8, 41) c. ( 41, 8) d. ( − 41, 8) a. x − 6x + 36x + 27 = 0 b. x + 6x + 36x + 27 = 0
3 2 3 2

5. If α and β are the roots of the quadratic equation c. x 3 − 6x 2 − 36x + 27 = 0 d. x 3 + 6x 2 − 36x + 27 = 0


1 1 13. The number of solutions of the equation
4x 2 + 3x + 7 = 0, then the value of + is [2012]
α β x
1 + sin x sin2 = 0, in [ −π, π], is [2007]
3 3 2
a. − b. −
4 7 a. 0 b. 1 c. 2 d. 3
3 4 14. If one root of the quadratic equation ax + bx + c = 0 is 2
c. d.
7 7 equal to nth power of the other root, then the value of
6. If α, β are the roots of ax 2 + bx + c = 0 and α + k , β + k 1 1

b − 4ac
2 (ac n )n +1 + (a nc )n +1 is equal to [2007]
are the roots of px 2 + qx + r = 0, then is 1
q − 4 pr
2
a. b b. −b c. d. −bn+ 1
equal to [2012]
bn+ 1
a 15. If α, β, γ are the roots of the equation
a. b. 1
p 2x 3 − 3x 2 + 6x + 1 = 0, then α 2 + β 2 + γ 2 is equal to
a
2 [2006]
c.   d. 0 15 15 9
 p a. − b. c. d. 4
4 4 4
7. If α and β are the roots of the equation x − 2x + 4 = 0,
2
16. If sin A, sin B , cos A are in GP, then the roots of
then the value of αn + βn will be [2011] x 2 + 2x cot B + 1 = 0 are always [2005]
n+1 n+1
a. i 2 sin(nπ / 3) b. 2 cos(nπ / 3) a. real b. imaginary
n −1 n −1
c. i 2 sin(nπ / 3) d. 2 cos(nπ / 3) c. greater than 1 d. equal

@iitjeehelps
Answer with Solutions
Practice Exercise 8. (d) Given equation is x 2 − 3x + k = 0
∴ Discriminant, D = b 2 − 4ac
1. (a) Hint x = 1 + 1 + 1 + K
= ( −3)2 − 4 × 1 × k = 9 − 4k
⇒ x = 1+ x ⇒ x = 1+ x ⇒ x − x − 1 = 0
2 2
For distinct roots, D > 0
2. (a) Here, D = ( 3b )2 − 4 ⋅ ( 4a ) ⋅ ( 2c ) 9
⇒ 9 − 4k > 0 ⇒ 9 > 4k ⇒ k <
= 9b − 32ac = 9( −a − c ) − 32ac = 9a − 14ac + 9c
2 2 2 2 4
9
 a  2 14 a   a 7 2 49  Every value of k which is less than will give two distinct
= 9c 2   − ⋅ + 1 = 9c 2  −  − + 1 > 0 4
  9 c  
 c   c 9 81  roots. But these roots will not lie in the interval ( 0, 1).
Hence, the roots are real and distinct. 9. (d) Here, D = 4 b 2 + 12ca < 0
3. (b) Since, the roots of the given equation are real. ⇒ b 2 + 3 ca < 0 …(i)
∴ D > 0 ⇒ 16 + 4 log3 a ≥ 0 ⇒ log3 a ≥ − 4 ⇒ ca < 0
1 If c > 0, then a < 0
⇒ a ≥ 3 −4 ⇒ a ≥
81 3c
Also, < a + b ⇒ 3 ca > 4a 2 + 4ab
b c 4
4. (b) Hint α + β = − , αβ =
a a ⇒ b 2 + 3 ca > 4a 2 + 4ab + b 2 = ( 2a + b )2 ≥ 0 …(ii)

Also, α+h+β+h=−
q
and (α + h ) (β + h ) =
r From Eqs. (i) and (ii), c > 0, which is not true.
p p ∴ c <0

5. (c) The given equation will have equal roots iff 10. (c) Simplified form of given equation is
b 2 − 4ac = 0 ⇒ ( 27 × 31/ p − 15)2 − 144 = 0 ( 2x + p + q )r = (x + p )(x + q )
⇒ x + ( p + q − 2r )x − ( p + q )r + pq = 0
2
⇒ 27 × 31/p − 15 = ± 12 ⇒ 27 × 31/p = 27 or 3
1 Since, sum of roots = 0
Now, 31/p = 1 ⇒ =0 p+q
p ∴ − ( p + q − 2r ) = 0 ⇒ r =
2
1 and product of roots = − ( p + q )r + pq
Also, 31/p = 3−2 ⇒ =−2
p ( p + q )2 1
1 1 =− + pq = − ( p 2 + q 2 )
But cannot be zero, so p = − . 2 2
p 2 11. (b) Let α , β be the roots of the given equation.
6. (b) We have, x 2 + ax + 12 = 0 ...(i) 4+ 5 8+ 2 5
x 2 + bx + 15 = 0 ...(ii) Then, α + β = and αβ =
5+ 2 5+ 2
and x 2 + (a + b )x + 36 = 0 ...(iii)
Let H be the HM of α and β, then
On adding Eqs. (i) and (ii), we get 2αβ 16 + 4 5
2x 2 + (a + b )x + 27 = 0 ...(iv) H= = =4
α+β 4+ 5
On subtracting Eq. (iii) from Eq. (iv), we get
12. (d) We have, | x 2 − x − 6 |
x 2 − 9 = 0 ⇒ x = 3, − 3
 x 2 − x − 6, if x ≤ − 2 or x ≥ 3
Thus, common positive root is 3. =
∴ ( 3)2 + 3a + 12 = 0 ⇒ a = − 7  − (x 2
− x − 6 ), if − 2 < x < 3
and 9 + 3b + 15 = 0 ⇒ b = − 8 Case I When x ≤ − 2 or x ≥ 3, then
Hence, the ordered pair (a, b ) is ( − 7, − 8). |x2 −x − 6| =x + 2 ⇒ x2 −x − 6 =x + 2
7. (a) Let α , β, γ be the roots of the given equation such that ⇒ x 2 − 2x − 8 = 0 ⇒ (x − 4) (x + 2) = 0 ⇒ x = − 2, 4
α = − β, then Case II When −2 < x < 3, then
α+β+γ=p ⇒ γ=p
|x2 −x − 6| =x + 2
Since, γ is a root of the given equation, so it satisfies the
equation ⇒ − (x 2 − x − 6) = x + 2
γ 3 − pγ 2 + qγ − r = 0 ⇒ x2 − 4 = 0 ⇒ x = ± 2
⇒ p 3 − p 3 + pq − r = 0 ⇒ r = pq Hence, roots are −2, 2 and 4.

@iitjeehelps
776 SELF STUDY GUIDE BITSAT

π π P Q π 1 1
13. (c) We have, R = ⇒P + Q = ⇒ + = 20. (c) x + = 5 ⇒ x 2 + 2 = 52 − 2 = 23
2 2 2 2 4 x x
P Q 1
tan + tan and x 3 + 3 = 53 − 3 × 5 = 110
π 2 2 x
⇒ 1 = tan =
4 1 − tan P tan Q  1  1
Now, x 2 + 2  x 3 + 3  = 23 × 110 = 2530
2 2  x   x 
b
− 1  1
a ∴ x 5 + 5 = 2530 − x +  = 2525
⇒ 1= =
b x  x
c c −a
1− 21. (b) Let f (x ) = x 2 + (1 − 2k )x + k 2 − k − 2
a
⇒ a + b =c The number 3 lies between the roots of the given
equation, if f ( 3) < 0.
14. (a) For roots greater than unity,
Now, f ( 3) = 9 + (1 − 2k )3 + k 2 − k − 2
Discriminant ≥ 0, sum of roots > 2 and (a + 1)f (1) > 0
3a ⇒ f ( 3) = k 2 − 7k + 10
⇒ 9a 2 − 16a(a + 1) ≥ 0, >2
a+1 Hence, f ( 3) < 0
⇒ k 2 − 7k + 10 < 0 or (k − 2)(k − 5) < 0
and (a + 1)(a + 1 − 3a + 4a ) > 0
a−2 ∴ k > 2 and k < 5 or k < 2 and k > 5
⇒ a( 7a + 16) ≤ 0, >0
a+1 ∴ k lies in the interval (2, 5).
22. (c) Given, α , β and γ are the roots of the given equation.
and (a + 1)( 2a + 1) > 0
−16 Then, α + β + γ = 0, αβ + βγ + γα = − 1 and α ⋅ β ⋅ γ = 1
⇒ ≤ a ≤ 0, a < − 1 or a > 2 1+ x y −1
7 Let y = ⇒ x =
1 1− x y +1
and a < − 1 or a > −
2 1+ α
⇒ y 3 + 7y 2 − y + 1 = 0 ⇒ Σ = −7
16 1− α
⇒ − ≤a < −1
7 23. (b) We know that, only even prime is 2, then
15. (c) Since, (x − 1)2, (x − 2)2, (x − 3)2 cannot be zero ( 2)2 − λ ( 2) + 12 = 0
simultaneously for any real value of x. ⇒ λ=8 ...(i)
∴Number of roots = 0 and x 2 + λx + µ = 0 has equal roots.
1
16. (c) Since, 3 x − 1 + 3 – x − 1 = ( 3x + 3−x ) ∴ λ2 − 4 µ = 0 [Q D = 0]
3
⇒ ( 8 )2 − 4 µ = 0 [from Eq. (i)]
1
≥ ⋅ 2 3x ⋅ 3−x [Q AM ≥ GM] ⇒ µ = 16
3
2 24. (b) Given equation is
⇒ 3x −1 + 3−x −1 ≥
3 ax 2 + bx + c = 0
17. (c) Hint b 2 − 4ac = 0 Since, α , β are the roots of this equation.
18. (d) Since, α + β = − 1, αβ = 2 , γ + δ = − 3, γδ = 4 −b
∴ α+β= ...(i)
a
∴ (α + γ )(α + δ )(β + γ )(β + δ ) = (α 2 − 3 α + 4)(β 2 − 3 β + 4) c
and αβ = ...(ii)
= 4 − 3αβ 2 + 4β 2 − 3α 2β + 9 αβ − 12 β 2 + 4α 2 − 12 α + 16 a
= 4 − 3 ( 2) β + 4β 2 + 4α 2 − 3 ( 2) α + 9 ( 2) − 12 (β + α ) + 16 Now, (1 + α + α 2 )(1 + β + β 2 )
= 4 − 6 β + 4(α 2 + β 2 ) − 6 α + 18 + 12 + 16 = 1 + (α + β ) + α 2 + β 2 + αβ (1 + α + β ) + α 2β 2
= 50 + 6 + 4 [(α + β ) − 2 αβ]
2
b  −b 
2
2c c  b b2
= 1− +  − + 1 −  + 2
= 56 − 12 = 44 a a a a a a
1 [from Eqs. (i) and (ii)]
19. (c) Since, a 2 + b 2 + c 2 − ab − bc − ca = Σ (a − b )2 ≥ 0 a 2 − ab + b 2 − ac − bc + c 2
2 =
∴ 1 − Σ ab ≥ 0 ⇒ Σ ab ≤ 1 …(i) a2
(a + b − 2ab ) + (b 2 + c 2 − 2bc ) + (c 2 + a 2 − 2ac )
2 2
Now, (a + b + c ) =2
Σ a + 2 Σ ab
2 =
2a 2
1 (a − b ) + (b − c ) + (c − a )2
2 2
⇒ Σ ab ≥ − …(ii) =
2 2a 2
Σ ab lie in − , 1.
1
From Eqs. (i) and (ii), Here, all terms are in square, therefore it is always
 2  positive.

@iitjeehelps
QUADRATIC EQUATION 777
(5 − 2 6 )(5 + 2 6 ) ∴ α = 1is not possible.
25. (b) We have, 5 − 2 6 =
(5 + 2 6 ) If α = ω, then α + α 2 = ω + ω 2 = − 1
25 − 24 1 −p
= = ∴ −1 = ⇒ p=3
5+ 2 6 5+ 2 6 3
2
−3 Again, if α = ω 2, then
Let (5 + 2 6 )x =y
α + α 2 = ω2 + ω4 = ω2 + ω = − 1
Then, given equation becomes
−p
1 ∴ −1 = ⇒ p=3
y + = 10 ⇒ y 2 − 10y + 1 = 0 3
y
10 ± 4 6 30. (c) We have, x 2 + 5 = 2x − 4 cos(a + b )
⇒ y = =5 ± 2 6
2 ⇒ x 2 − 2x + 1 + 4 = − 4 cos (a + b )
When y = 2 6 + 5, then ⇒ (x − 1)2 + 4 [1 + cos (a + b )] = 0
x2 − 3
(5 + 2 6 ) = (5 + 2 6 ) ⇒ x − 3 = 12
⇒ x = 1 and 1 + cos(a + b ) = 0
⇒ x2 = 4 ⇒ x = ± 2 ⇒ cos(a + b ) = − 1 ⇒ a + b = π , 3π , K
When y = 5 − 2 6, then (5 + 2 6 )x
2
−3
= (5 − 2 6 ) 31. (a) Since, the roots of ax 2 + bx + c = 0 are non-real. Thus,
2
−3 2f (x ) = ax 2 + bx + c will have same sign for every value of x.
⇒ (5 + 2 6 )x = 5 + 2 6 )− 1 ⇒ x 2 − 3 = − 1
⇒ f ( 0) = c , f (1) = a + b + c , f ( − 1) = a − b + c
⇒ x =2 ⇒ x =±
2
2
and f ( 2) = 4a − 2b + c
∴ x = 2, − 2 − 2, 2
∴ c ⋅ (a + b + c ) > 0
26. (d) The given equation can be rewritten as c (a − b + c ) > 0 and c ( 4a − 2b + c ) > 0
x 2 − (a + b )x + ab − 1 = 0 32. (b) Hint Using α − β = ± (α + β )2 − 4αβ
Let α , β be its roots.
∴ α + β = a + b, αβ = ab − 1 33. (c) Since, roots of ax 2 + bx + c = 0 are α and β.
If one root is less than a, then the other root will be 1 1
Hence, roots of cx 2 + bx + a = 0 will be and .
greater than b. α β
So, we have one root lies in ( −∞, a ) and other in (b, ∞ ). Now, if we replace x by x − 1, then roots of
27. (b) Clearly, α + β = − b < 0 1 1
c (x − 1) + b (x − 1) + a = 0 will be 1 + and 1 +
2
.
α β
αβ = c < 0
Now, αβ < 0 and α < β 1
Again replace x by , we get
⇒ α is a negative root. x
α + β < 0 and α < 0 ⇒ β < | α | c (1 − x )2 + b (1 − x ) + ax 2 = 0
∴ α < 0 <β <|α | α β
whose roots are and .
28. (a) Let α , β be the roots of the given equation, then 1+ α 1+ β
−m m 34. (b) Given, 4a + 4b − 5c > 0
α+β= , αβ =
l l Let f (x ) = ax 2 + 2bx − 5c , then f ( 2) = 4a + 4b − 5c > 0
α a
Also = Since, f (x ) = 0 has imaginary roots, therefore f (x ) will
β b
have same sign as that of a for all x ∈ R.
a b m α β m
Now, + + = + + Since, f ( 2) > 0, therefore a > 0 and c < 0.
b a l β α l
(x − b )(x − c )
α+β m −m / l m m m 35. (b) Suppose m =
= + = + =− + =0 (x − a )
αβ l m /l l l l
⇒ x 2 − (b + c + m )x + (bc + am ) = 0
29. (c) Let α , α be the roots.
2
Since, x is real, so we must have
∴ Product of the roots, (b + c + m )2 − 4(bc + am ) ≥ 0
α ⋅α 2 = 1 ⇒ α 3 = 1
⇒ m 2 + 2(b + c − 20)m + (b − c )2 ≥ 0
⇒ α = 1, ω, ω 2
⇒ [m + (b + c − 2a )] 2 + (b − c )2 − (b + c − 2a )2 ≥ 0
p −p
Again, α + α 2 = − ⇒ 1+ 1= Since, m may take any real value, so we must have
3 3
⇒ p = − 6 (if α = 1) (b − c )2 − (b + c − 2a )2 ≥ 0 ⇒ (b − a )(a − c ) ≥ 0
But p > 0 ⇒ (a − b )(a − c ) ≤ 0 ⇒ b ≥ a ≥ c or b ≤ a ≤ c

@iitjeehelps
BITSAT Archives
1. (a) Given, αx − 
1 1 q 2 − 4pr b 2 − 4ac − q 2 − 4pr b 2 − 4ac
(αx )2 + (αx )3 − ... ⇒ − = +
 2 3 
2p 2a 2p 2a
 1 1 
+ βx − (βx )2 + (βx )3 − ... q 2 − 4pr b 2 − 4ac q 2 − 4pr b 2 − 4ac
 2 3  ⇒ = ⇒ =
p a p2 a2
= log (1 − αx ) + log (1 − βx ) = log [1 − (α + β )x + αβx ]
2
2
Now, α + β = p and αβ = q b 2 − 4ac  a 
∴ = 
∴Given series = log (1 − px + qx 2 ) q 2 − 4pr  p 
3 − |x| 7. (b) Since, α and β are the roots of x 2 − 2x + 4 = 0.
2. (a) Given, ≥0
4 − |x| ∴ α + β = 2 and αβ = 4
⇒ 3 − | x | ≤ 0 and 4 − | x | < 0 Now, (α − β ) = (α + β )2 − 4αβ
or 3 − | x | ≥ 0 and 4 − | x | > 0 = 4 − 16 = 2 3 i
⇒ | x | ≥ 3 and | x | > 4
On solving, we get
or | x | ≤ 3 and | x | < 4
2α = 2 + 2 3 i
⇒ | x | > 4 or | x | ≤ 3
⇒ x ∈( −∞, − 4) ∪ [ −3, 3] ∪ ( 4, ∞ ) 1 3   π π
⇒ α = 2 + i = 2  cos + i sin 
N 2 2   3 3
3. (a) We have,
x1 + x 2 + x 3 + x 4 2 − 2 3i  π π
and β= = 2  cos − i sin 
1000x1 + 100x 2 + 10x 3 + x 4 2  3 3
=
x1 + x 2 + x 3 + x 4   π π 
n
  π π 
n
∴ α n + βn = 2  cos + i sin   + 2  cos − i sin  
 900x 2 + 990x 3 + 999x 4    3 
3    3 3 
= 1000 −  
 x1 + x 2 + x 3 + x 4   nπ  nπ
= 2n 2 cos = 2n + 1 cos
∴Maximum value is 1000.  3  3

4. (b) If 4 − 5i is the root of x 2 + ax + b = 0, then 4 + 5i is also 8. (c) The roots of the given equation are
the root. − b − b 2 − 4ac
α=
∴ Sum of roots = − a = 8 ⇒ a = − 8 2a
and product of roots = b = 16 + 25 = 41 − b + b 2 − 4ac
and β=
∴ (a, b ) = ( −8, 41) 2a
5. (b) Since, α , β are real and distinct, therefore b 2 − 4ac > 0.
6. (c) Since, α, β are the roots of the equation ax 2 + bx + c = 0. If a > 0, then β > −
b
and if a < 0, then a > −
b
. Thus,
2a 2a
b 2 − 4ac 
b b
Then, α=− +  one of the roots exceeds − .
2a 2a  2a
 …(i)
b b − 4ac 
2
9. (d) Let z = x + iy , then
and β=− − 
2a 2a  z2 + z = 0
Also, α + k , β + k are the roots of the equation ⇒ (x 2 − y 2 + 2 ixy ) + (x − iy ) = 0
px 2 + qx + r = 0.
⇒ x 2 − y 2 + x = 0 and 2xy − y = 0
q q − 4pr
2
Then, α + k = − + Now, 2xy − y = 0 ⇒ y ( 2x − 1) = 0
2p 2p
⇒ y = 0 or x = 1/ 2
q q 2 − 4pr If y = 0, then
and β+k =− −
2p 2p x2 −y2 + x = 0
q q − 4pr
2
b b − 4ac 2
⇒ x2 + x = 0
⇒ k =− + + −
2p 2p 2a 2a ⇒ x = 0 or x = − 1
[from Eq. (i)] If x = 1/ 2, then x 2 − y 2 + x = 0
q q 2 − 4pr b b 2 − 4ac ⇒ y =±
3
and k =− − + +
2p 2p 2a 2a 2
[from Eq. (i)] Thus, the given equation has four solutions.

@iitjeehelps
QUADRATIC EQUATION 779
10. (a) 14. (b) Hint Let α and α n be the roots of the equation. Then,
11. (d) Given, α + β = − 2 and α 3 + β 3 = − 56 −b
α + αn = …(i)
a
⇒ (α + β )(α 2 + β 2 − αβ ) = − 56 ⇒ α 2 + β 2 − αβ = 28
c c
Now, (α + β )2 = ( −2)2 ⇒ α 2 + β 2 + 2αβ = 4 and α ⋅ αn = ⇒ αn + 1 =
a a
⇒ 28 + 3αβ = 4 ⇒ αβ = − 8 1
c  n+1
∴ Required equation is ⇒ α=  …(ii)
a
x 2 − ( −2)x + ( −8) = 0 ⇒ x 2 + 2x − 8 = 0
3
12. (d) Given equation is x 3 + 2x 2 − 4x + 1 = 0 15. (a) Hint Here, α + β + γ = …(i)
2
Let α , β and γ be the roots of the given equation. 9
⇒ α 2 + β 2 + γ 2 + 2 [αβ + βγ + γα ] =
Then, α+β+γ=−2 4
αβ + βγ + γα = − 4 and αβγ = − 1 16. (a) Since, sin A, sin B and cos A are in GP.
Let the required cubic equation has the roots 3α , 3β and
∴ sin2 B = sin A cos A …(i)
3γ.
⇒ 3α + 3β + 3γ = − 6 x + 2x cot B + 1 = 0
2
[given]
3α ⋅ 3β + 3β ⋅ 3γ + 3γ ⋅ 3α = − 36 Now, b − 4ac = 4 cot B − 4
2 2

and 3α ⋅ 3β ⋅ 3γ = − 27 4 cos2 B − 4 sin2 B


∴ Required equation is =
sin2 B
x 3 − ( −6)x 2 + ( −36)x − ( −27) = 0
4(1 − sin2 B ) − 4 sin2 B
⇒ x 3 + 6x 2 − 36x + 27 = 0 =
sin2 B
x = 4 [1− 2 sin2 B]
13. (a) Given that, 1 + sin x ⋅ sin2 =0
2
sin2 B
 1 − cos x  4 [1 − 2 sin A cos A]
∴ 1 + sin x   =0 =
 2  [from Eq. (i)]
sin2 B
⇒ 2 + sin x − sin x cos x = 0 2
 sin A − cos A 
⇒ sin 2x − 2 sin x = 4 =4  >0
 sin B 
Since, the maximum values of sin x and sin 2x are 1,
∴Roots are always real.
which is not possible for any x in [ −π , π ].

@iitjeehelps
3
Sequences and Series

Sequences and Series


Sequence is a function whose domain is a subset of N i.e. a set of natural numbers. It displays the
images of 1, 2 , 3 ,K,n ,K as f 1 , f 2 , f3 ,... , f n ,…, where f n = f (n ). If the terms of a sequence follows certain
pattern, then it is called a progression. If a1 , a2 , a3 ,... , an is a sequence, then the expression gives the
series a1 + a2 + a3 + ... + an .

Arithmetic Progression (AP)


It is a sequence in which the difference between two consecutive terms is same.
i.e. a , a + d , a + 2 d , a + 3 d , … where, a is the first term and d is the common difference.

nth Term of an AP
Let a be the first term, l be the last term and d be the common difference of a certain sequence in AP.
Then, nth term of AP is T n = a + (n − 1) d = l
where, d = T n − T n − 1, n ≥ 2, n ∈ N
nth term from last, T n′ = l − (n − 1) d

Sum of the First n Terms of an AP


If first term of an AP is a, common difference is d and number of terms is n, then
n
(i) Sum of n terms, S n = [2a + (n − 1) d ]
2
(ii) When first term and last term are given, then
n
S n = [a + l ]
2
(iii) When last term and the common difference are given, then
n
S n = [2 l − (n − 1) d ]
2

@iitjeehelps
SEQUENCES AND SERIES 781

Properties of an AP On subtracting Eq. (ii) from Eq. (i), we get


● If a1 , a2 , a3 , K are in AP, then S (1 − r ) = a − ar n , so
➣ a 1 ± k , a 2 ± k , a 3 ± k , ... are also in AP. a (1 − r n )
a a a (i) S n = , when r < 1
➣ a 1 k , a 2 k , a 3 k ,... and 1 , 2 , 3 ..., are also in AP. 1−r
k k k
If a1 , a2 , a3 , K and b1 , b2 , b3 , K are two AP’s, then a (r n − 1)

(ii) S n = , when r > 1
➣ a 1 ± b1 , a 2 ± b2 , a 3 ± b3 , ... are also in AP. r −1
a a a
➣ a 1 b1 , a 2 b2 , a 3 b3 , K and 1 , 2 , 3 K are not in AP. (iii) If last term ( l ) is known, then
b1 b2 b3
 a − lr
If a1 , a2 , a3 ,... are in AP, then , when r < 1

 1 −r
a 1 + a n = a 2 + a n − 1 = a 3 + a n − 2 = ... Sn = 

lr − a
ar − k + ar + k  , when r > 1
➣ ar = , ∀ k; 0 ≤ k ≤ n − r  r −1
2
● Three numbers in an AP can be taken as a − d , a , a + d . Sum of Infinite Terms of a GP
● Four numbers in an AP can be taken as When |r | < 1 and number of terms is infinite, then
a − 3d , a − d , a + d , a + 3d
lim r n = 0
● If nth term of any sequence is linear expression in n such n→ ∞
that t n = an + b, then sequence is an AP with common a t
∴ S∞ = , where r = n
difference a. 1−r tn − 1
● If sum of n terms of any sequence is quadratic expression
in n i.e. S n = an 2 + bn , then sequence is an AP.
Properties of a GP
● If a1 , a2 , a3 , K are in GP, then
Geometric Progression (GP) ➣ a 1k , a 2k , a 3k , ... and
a1 a2 a3
, , , K are also in GP.
k k k
It is a sequence in which the ratio of any two consecutive 1 1 1 1
terms is same i.e. a, ar , ar 2 , … ➣ , , , , K are also in GP.
a1 a2 a3 a4
where, a is the first term and r is the common ratio.
● If a1 , a2 , a3 , a4 , K and b1 , b2 , b3 ,b4 ,K are in GP, then
a1 a2 a3
➣ a 1b1 , a 2b2 , a 3b3 , K and , , , K are also in GP.
nth Term of a GP b1 b2 b3
Let the series be a , ar , ar 2 , ar 3 , K ➣ But a 1 ± b1 , a 2 ± b2 , a 3 ± b3 , K are not in GP.
1 −1
t 1 = a = ar ● If a1 , a2 , a3 , K are in GP (ai > 0, ∀ i ), then
t 2 = ar = ar 2 − 1 ➣ log a 1 , log a 2 , log a 3 , K are in AP. In this case, the converse
M M is also true.
t n = ar n − 1 ➣ a 1r , a 2r , a 3r , K are also in GP, where r ∈ Q.

(i) So, nth term of a GP,


➣ a 1 a n = a 2 a n − 1 = a 3 a n − 2 = ...

➣ a r = a r − k ⋅ a r + k , ∀k ; 0 ≤ k ≤ n − r
tn 
t n = ar n − 1 = l Qr =  ● If a1 , a2 , a3 , a4 , K, an − 1 , an are in GP, then
 t n − 1 
a2 a3 a4
(ii) nth term of a GP from the end, = = =K
a1 a2 a3
l
t n′ = n − 1 an
r = =r
an − 1
Sum of n Terms of a GP a
● Three numbers in GP can be taken as , a , ar .
Let the first term of a GP be a and common ratio be r , then r
series is a , ar , ar 2 , K, ar n − 1 . ●
a a
Four numbers in GP can be taken as 3 , , ar , ar 3 .
Let S = a + ar + ar 2 + K + ar n − 1 …(i) r r
S ⋅ r = ar + ar 2 + K + ar n − 1 + ar n …(ii) NOTE If a, b, c are in AP as well as in GP, then a = b = c.

@iitjeehelps
782 SELF STUDY GUIDE BITSAT

Arithmetic Mean (AM) Geometric Mean (GM)


Let a , x and b be in AP. Then, the arithmetic mean of AP is x If three numbers are in GP, then middle one is called the GM
and can be written as between the other two.
a+b Let a , x and b be in GP. Then,
x= =A
2 x b
=
Similarly, AM of n numbers a1 , a2 , K, an is a x
1
(a1 + a2 + ... + an ) ⇒ x 2 = ab
n
⇒ x = ab =G
Insertion of n Arithmetic Mean
between Two Given Numbers Insertion of Geometric Mean between
Let x 1 , x 2 , x3 , K, x n be n AM’s between a and b. Then, Two Given Numbers
a , x 1 , x 2 , x3 , K, x n ,b are in AP and total number of terms are Let a and b be the given numbers and x 1 , x 2 , x3 , ... , x n be n
n + 2. If d is the common difference of an AP, then geometric means inserted between them.
t n + 2 = a + (n + 2 − 1) d = b
Then, a , x 1 , x 2 , x3 , K, x n , b are in GP.
b −a
d= 1
n +1 n+1 b n + 1
∴ b = t n + 2 = ar ; r = 
 b −a  a
Then, x r = a + r   , where r = 1, 2 ,3 ,...
 n + 1 x 1 = t 2 = ar
(i) Sum of n arithmetic means x 2 = t3 = ar 2
n
= x 1 + x 2 + x3 + K + x n = ( x 1 + x n ) [sum of AP] M M M
2
x n = t n + 1 = ar n
n a + b
= [a + d + b − d ] = n   [AM of a and b]
2  2  r
b n + 1
(ii) Sum of n AM’s between a and b is nA. ∴ xr =a   , where r =1, 2 ,3 ,...
 a
i.e. A1 + A2 + A3 + K + An = nA
On putting the value of r , we shall find n geometric means.
(iii) Any three numbers in AP can be taken as
The product of n geometric means between a and b is equal
a − d , a , a + d.
to the nth power of geometric mean of a and b.
(iv) Any four numbers in AP can be taken as
a − 3d , a − d , a + d , a + 3d . i.e. G1 ⋅G2 K Gn = (ab )n / 2 = G n [QG = ab ]
(v) In arithmetic series containing even numbers, we 1
(i) The product of n geometric means between a and
assume, number of terms in such series as 2n, nth a
and (n + 1) th terms will be two middle terms. is 1.
(vi) In arithmetic series containing odd numbers, we a
(ii) Three numbers in GP can be taken as ,a ,ar .
assume, number of terms in such series as (2n + 1), r
then (n + 1) th term will be middle term. a a
(iii) Four numbers in GP can be taken as 3 , ,ar , ar 3 .
(vii) Sum of terms from beginning and end of arithmetic r r
series is constant. a a
(iv) Five numbers in GP can be taken as 2 , ,a ,ar , ar 2 .
NOTE ● If pth term of an AP is q and q th term is p, then Tp + q = 0. r r
● If S p = q and Sq = p for an AP, then S p + q = − ( p + q ). NOTE ● If A and G are the AM and GM between two numbers a and
➣ If S p = Sq for an AP, then S p + q = 0. b, then a, b are given by [ A ± ( A + G )( A − G )].
➣ If pth term is
1 1
and q th term is , then ● The product of first n terms of GP is
q p n( n − 1)
Tpq = 1. P = a ⋅ ar ⋅ ar 2 ... ar n −1 = an r 2

@iitjeehelps
SEQUENCES AND SERIES 783

Relation between AM Harmonic Progression (HP)


and GM A sequence is said to be a harmonic progression, if
reciprocal of terms of the sequence form an AP i.e.
If a and b are two numbers, then 1 1 1 1
a1 , a2 , a3 , K, an are in HP, if , , , K, are in AP.
a+b a1 a2 a3 an
AM =
2
nth term of HP from beginning,
and GM = ab 1 a1a2
Tn = =
a +b 1  1 1  a2 + (n − 1) (a1 − a2 )
Now, AM – GM = − ab + (n − 1)  − 
2 a1  a2 a1 
2
 a − b nth term of HP from end,
=  ≥0
 2  1
T n′ =
⇒ AM – GM ≥ 0 1  1 1
− (n − 1)  − 
an  a2 a1 
Hence, AM ≥ GM
a1 a2 an
=
a1 a2 − an (n − 1) (a1 − a2 )
NOTE ● If A and G are the AM and GM between two positive
numbers, then the numbers are A ± A 2 − G 2.

● If A and G are respectively AM and GM between two Harmonic Mean (HM)


positive numbers a and b, then the quadratic equation
having a and b as its roots, is x2 − 2 Ax + G 2 = 0. If two numbers a and b are in HP, then harmonic mean
2ab
(HM) between them is given by HM = .
a +b
Arithmetico-Geometric
Insertion of Harmonic Mean
Progression (AGP) If a , x 1 , x 2 ,K, x n ,b are in HP, then x 1 , x 2 ,.. , x n are n harmonic
A series in which each term is the product of the means between a and b.
corresponding terms of an AP and GP, is called These are the reciprocal of n arithmetic means between
arithmetico-geometric progression (AGP). 1 1
and .
e. g. 1 + 3 x + 5x 2 + 7x 3 + K a b
Here, 1, 3 , 5, ... are in AP and 1, x , x 2 , K are in GP. (n + 1)ab 2 (n + 1)
Hence, x 1 = ,x = , … and so on.
a + nb 2a + (n − 1)b

Sum of n Terms of AGP


Let a1 , a2 , K, an be in AP and b1 , b2 , K, bn be in GP. Relation between AM, GM
Then, a1b1 , a2b2 , ... , anbn will be in AGP. If the first term of an and HM
AP is a, common difference is d and first term of a GP is b,
Let a and b be two real positive unequal numbers, then AM,
common ratio is r , then GM and HM between them are respectively,
ab , (a + d ) br , (a + 2d ) br 2 , Kare in AGP. a+b 2ab
A= , G = ab , H = ⇒ G 2 = AH
ab dbr (1 − r n − 1 ) [a + (n − 1) d ] br n 2 a+b
Now, S n = + − ,r ≠ 1
1−r (1 − r )2 1−r Also, A> G > H
If − 1 < r < 1, then lim r n = 0 Hence, G lies between A and H.
n→ ∞
NOTE If A, G and H are arithmetic, geometric and harmonic means
and lim nr n = 0
n→ ∞ between three given numbers a, b and c, then the equation
having a, b,c as its roots, is
ab dbr
∴ Sum to infinity, S ∞ = + [Q − 1 < r < 1] 3G 3
1 − r (1 − r )2 x3 − 3 Ax2 + x − G3 = 0
H

@iitjeehelps
784 SELF STUDY GUIDE BITSAT

(v) (a) Sum of first even natural numbers


Sum upto n Terms of Special 2 + 4 + 6 + K+ 2n = n (n + 1)
Series ∑ n , ∑ n 2 , ∑ n 3 , etc. (b) Sum of first odd natural numbers
1 + 3 + 5 + K + (2n − 1) = n 2
(i) Sum of first n natural numbers
n
n (n + 1) (vi) Sum of n terms of series
∑ k = 1 + 2 + 3 +K+ n = 2 12 − 2 2 + 3 2 − 42 + 52 − 6 2 +K
k=1
 n (n + 1)
(ii) Sum of the squares of first n natural numbers , when n is odd
 2
n
1 =
−n (n + 1)
∑ k 2 = 12 + 2 2 + 3 2 + .... + n 2 = 6 n (n + 1) (2n + 1)  , when n is even
k=1  2
(iii) Sum of the cubes of first n natural numbers 1 1 1 1 n
NOTE ● + + + ... + =
2 1⋅ 2 2 ⋅ 3 3 ⋅ 4 n(n +1) n+1
n  n  1 
2

∑ k3 = 13 + 23 + K + n3 =  ∑ k =  (n ) (n + 1) 1
+
1
+ ... +
1
= −
1 1
k=1 k = 1  2  ●
1⋅ 2 ⋅ 3 2 ⋅ 3 ⋅ 4 n( n +1)(n+2) 4 2(n +1)(n + 2 )
(iv) Sum of product of first n natural numbers taken two n(n + 1) (n + 2 )
● 1⋅ 2 + 2 ⋅ 3 + K + n (n + 1) =
at a time is 3
1 n (n + 1)(n − 1)(3n + 2 ) ● 1⋅ 2 ⋅ 3 ⋅ 4 + 2 ⋅ 3 ⋅ 4 ⋅ 5 + ... + n(n + 1) (n + 2 )(n + 3)
[ Σ n3 − Σ n 2] = 1
2 24 = n (n + 1) (n + 2 )(n + 3)(n + 4)
5

Practice Exercise
1. The number of numbers lying between 100 and 500 6. The first, second and middle terms of an AP are a , b , c
that are divisible by 7 but not by 21, is respectively. Then, their sum is equal to
a. 57 b. 19 c. 38 d. 40 2 (c − a ) 2c (c − a )
a. b. +c
2. Let Sn denotes the sum of n terms of an AP, whose b −a b −a
2c (b − a )
first term is a. If the common difference c. d. None of these
d = Sn − kSn − 1 + Sn − 2, then k is equal c −a
a. 3 b. 2 c. 5 d.7 7. The number of common terms to the two sequences
3. The sum upto n terms of the sequence 17, 21, 25, … , 417 and 16, 21, 26, …, 466 is
log a , log ar , log ar 2, K is a. 21 b. 19 c. 20 d. 91
n n a
a. log a 2r n −1 b. log ar n − 1 8. The least value of a for which 51 + x + 51 − x ,
,
2 2 2
c.
3n
log ar n − 1 d.
5n
log a 2r n −1 25 x + 25 − x are three consecutive terms of an AP, is
2 2
a. 10 b. 5
4. If m times the mth term of an AP with non-zero c. 12 d. None of these
common difference equals n times the nth term of an 9. Let a1, a 2, a 3 , … be in AP with common difference not
AP, where m ≠ n, then (m + n )th term of this AP is
multiple of 3. Then, the maximum number of
a. mn b. zero
consecutive terms so that all are prime number, is
c. 2mn d. None
a. 2 b. 3
5. Let Tr be the r th term of an AP, for r = 1, 2, 3,K . If for c. 5 d. infinite
1 1
some positive integers m and n, tm = , tn = , then  7
n m 10. If log3 2, log3 ( 2x − 5 ) and log3  2 x −  are in AP,
 2
mnth term of an AP is equal to
1 1 1 then x is equal to
a. b. + a. 2 b. 3
m m n
c. 4 d. 2, 3
c. 1 d. None

@iitjeehelps
SEQUENCES AND SERIES 785
11. If the sum of n terms of an AP is given by 20. If one GM, g and two AM’s, p and q are inserted
Sn = a + bn + cn 2, where a , b and c are independent between two numbers a and b, then ( 2p − q )( p − 2 q )
of n, then is equal to
a. a ≠ 0 a. g 2 b. −g 2
b. d ≠ 3b c. 2g d. 3g 2
c. d = 2c 21. If a , b , c are in GP and x, y are the AM’s between a , b
d. first term of an AP is b − c and b , c respectively, then
a c a 2c c
12. A farmer buys a used tractor of ` 12000. He pays a. + =2 b. + =
x y x y 2a
` 6000 cash and agrees to pay the balance in annual
1 1 3 2 1 3
installment of ` 500 plus 12% interest on the unpaid c. + = d. + =
amount. How much will the tractor cost him? x y b x 2y ac
a. `16680 b. `16670 c. `16681 d. `16682 22. If x , y and z are positive integers, then the value of
13. In a GP of positive terms, if any term is equal to the (x + y ) ( y + z ) (z + x )
sum of the next two terms. Then, the common ratio of a. = 8xyz b. > 8xyz
the GP is c. < 8xyz d. = 4xyz
a. sin 18° b. 2 cos 18° c. cos18° d. 2 sin 18°
23. The minimum value of the expression
14. Let S be the sum, P be the product and R be the sum 3 x + 31 − x , x ∈ R , is
of the reciprocals of 3 terms of a GP. Then, P 2R 3 : S 3 1
a. 0 b.
is equal to 3
a. 1 : 1 c. 3 d. 2 3
b. (common ratio)n : 1 24. If A1, A2 are two AM’s and G1, G2 are two GM’s between
c. (first term) 2 : ( common ratio) 2
A1 + A2
d. None of the above a and b, then is equal to
G1G2
15. The sum of three numbers in GP is 56. If we subtract
a+b 2ab
1, 7 and 21 from these numbers in that order, we a. b.
obtain an AP. Then, the numbers are 2ab a+b
a+b a+b
a. 8, 16, 32 b. 8, 16, 30 c. 16, 0, 30 d. 32, 16, 9 c. d.
1/ 6 1/ 36
ab ab
16. The product of ( 32)( 32) ( 32) …, is equal to
25. If a , b , c are in GP, then the equations
a. 16 b. 64 c. 32 d. 0 ax 2 + 2bx + c = 0 and dx 2 + 2ex + f = 0 have a
17. If S denotes the sum of infinity and Sn denotes the common root, if d /a , e/b , f /c are in
1 1 1 a. GP b. AP
sum of n terms of the series 1 + + + + …, such c. HP d. None of these
2 4 8
1 26. If the sum to
infinity of the series
that S − Sn < , then the least value of n is
1000 35
1 + 4x + 7x + 10x + K is
2 3
. Then, the value of x is
a. 8 b. 9 c. 10 d. 11 16
18. Let an be the n th term of a GP of positive integers. If a. 2/5 b. 1/5
100 100 c. 3/5 d. None of these
∑ a 2n = α and ∑ a 2n + 1 = β such that α =/ β, then the 27. If cos(x − y ), cos x and cos(x + y ) are in HP, then
n =1 n =1
cos x sec ( y / 2) is equal to
common ratio is
α β a. ± 2
a. b. b. ±1/ 2
β α
1/ 2 1/ 2 c. ± 2
α  β
c.   d.   d. None of the above
 β α
28. If AM, GM and HM of first and last terms of the series
19. Let S1, S 2, … be squares such that for each n ≥ 1, the 25, 26, 27, … , N − 1, N are the terms of the series,
length of a side of Sn equals the length of a diagonal of then find the value of N.
Sn + 1. If the length of a side of S1 is 10 cm, then for a. 25
which of the following values of n is the area of Sn less b. 225
than 1 sq cm? c. 1225
a. 7 b. 8 c. 9 d. 10 d. None of the above

@iitjeehelps
BITSAT Archives
1. If p , q , r and s are positive real numbers such that 11. The HM of two numbers is 4. If their arithmetic mean A
p + q + r + s = 2, then M = ( p + q ) (r + s ) satisfies the and geometric mean G satisfy the relation
relation [2014] 2A + G 2 = 27, then the numbers are [2009]
a. 0 < M ≤ 1 b. 1 ≤ M ≤ 2 a. 2, 6 b. 3, 6
c. 2 ≤ M ≤ 3 d. 3 ≤ M ≤ 4 c. 1, 3 d. 1, 2
n
2. The sum of the series [2013] 12. For any integer n ≥ 1, ∑ k (k + 2) is equal to
k =1
1 + 2 ⋅ 2 + 3 ⋅ 22 + 4 ⋅ 23 + ... + 100 ⋅ 299 is [2008]
n(n + 1)(n + 2)
a. 100 ⋅ 2 + 1
100
b. 99 ⋅ 2 + 1
100
a.
6
c. 99 ⋅ 299 − 1 d. 100 ⋅ 2100 − 1 n(n + 1)( 2n + 1)
b.
3. The sum of the sequence 5 + 55 + 555 + …upto n 6
n(n + 1)( 2n + 7)
terms is [2013] c.
6

5 10 (10 − 1) + n
n  
5 10 (10 − 1)
n  n(n + 1)( 2n + 9 )
a.   b.  − n d.
9  9  9  9  6
5 10 (10n + 1 − 1)  5 10(10n − 1 − 1)  1 1 3
c.  − n d.  − n 13. In ∆ABC, if + = , then C is
9  9  9  9  b +c c +a a +b +c
equal to [2008]
4. If a , b , c are in GP and a 1/ x = b 1/ y = c 1/ z , then x , y , z
a. 90° b. 60°
are in [2013]
c. 45° d. 30°
a. AP b. GP
c. HP d. None of these 14. The sum of 24 terms of the series
2 + 8 + 18 + 32 + …, is [2007]
1 3 7 15
5. The sum of n terms of the series + + + +… is a. 300 b. 200 2
2 4 8 16
[2012] c. 300 2 d. 250 2
a. n − 1 + 2−n b. 1 15. The sum to n terms of the infinite series
c. n − 1 d. 1 + 2−n 1⋅ 3 2 + 2 ⋅ 5 2 + 3 ⋅ 72 + ... is [2007]
n
6. 0.2 + 0.22 + 0.222 + K upto n terms is equal to [2012] a. (n + 1)( 6n 2 + 14n + 7)
6
 2  2   1
a.   −   (1 − 10−n ) b. n −   (1 − 10−n ) n
b. (n + 1) ( 2n + 1)( 3n + 1)
 9   81 9 6
 
2   
1  2 c. 4n 3 + 4n 2 + n
c.   n −   (1 − 10−n ) d.
9  9  9 d. None of the above
7. If AM and HM between two numbers are 27 and 12 16. If a = log2 3, b = log2 5 and c = log7 2 , then log140 63 in
respectively, then their GM is [2011] terms of a , b , c is [2007]
a. 9 b. 18 c. 24 d. 36 2ac + 1 2ac + 1
a. b.
2 2+ 4 2+ 4 +6 2c + abc + 1 2a + c + a
8. The value of + + + K is 2ac + 1
1! 2! 3! [2011] c. d. None of these
a. e b. 2e 2c + ab + a
c. 3e d. None of these 17. When 2301 is divided by 5, then the least positive
9. Let a 1, a 2, …, a 10 be in AP and h1, h2,…, h10 be in HP. If remainder is [2006]
a1 = h1 = 2 and a10 = h10 = 3, then a 4h7 is [2010] a. 4 b. 8
a. 2 b. 3 c. 5 d. 6 c. 2 d. 6

10. The sum of the infinite series 18. Let α, β, γ and δ be four positive real numbers such
2 1 2 5 1 2 5 8 1 that their product is unity, then the least value of
1 + ⋅ + ⋅ ⋅ 2 + ⋅ ⋅ ⋅ 3 + K, is [2009]
3 2 3 6 2 3 6 9 2 (1 + α )(1 + β )(1 + γ )(1 + δ ) is [2005]
a. 21/ 3
b. 41/ 3 a. 6 b. 16
c. 81/ 3
d. 21/ 5 c. 0 d. 32

@iitjeehelps
Answer with Solutions
Practice Exercise 7. (c) Let m th term of the first sequence be equal to n th term
of the second sequence. Then,
1. (c) The numbers between 100 and 500 that are divisible by
17 + 4 (m − 1) = 16 + 5(n − 1) ⇒ 4m + 13 = 5n + 11
7, are 105, 112, 119, 126, …, 490, 497.
⇒ 4m + 2 = 5n ⇒ n = m − (1/ 5) (m − 2)
Let such numbers be n.
Since, n is an integer, so m − 2 must be a multiple of 5m
∴ tn = an + (n − 1) d ⇒ 497 = 105 + (n − 1) × 7
and must be of the form 5k + 2 with k ≥ 0.
⇒ n − 1 = 56 ∴ n = 57
Then, n = 4k + 2. The first sequence has 101 terms and
The numbers between 100 and 500 that are divisible by 21, the second has 91 terms.
are 105, 126, 147, …, 483.
∴ 0 ≤ 5k + 2 < 101 and 0 ≤ 4k + 2 ≤ 91
Let such numbers be m.
⇒ 0 ≤ k ≤ 19
∴ 483 = 105 + (m − 1) × 21 ⇒ 18 = m − 1 ⇒ m = 19
Hence, the given sequence have 20 common terms.
∴ Required number = n − m = 57 − 19 = 38 a
2. (b) We know that, tn = Sn − Sn − 1 …(i) 8. (c) 51 + x + 51 − x , , 25x + 25−x are in AP.
2
tn − 1 = Sn − 1 − Sn − 2 a
and …(ii) ⇒ 2 ⋅ = 51 + x + 51 − x + 25x + 25− x
Now, d = tn − tn − 1 ⇒d = (Sn − Sn − 1) − (Sn − 1 − Sn − 2 ) 2
⇒ d = Sn − 2Sn − 1 + Sn − 2 ⇒ a = 5 ⋅ 5x + 5 ⋅ 5− x + (52x + 5−2x )
Hence, k =2  1  1 
⇒ a = 5 5x + x  52x + 2x 
3. (a) The sequence can be rewritten as  5   5 
log a, (log a + log r ), (log a + 2 log r ) , K We know that the sum of a positive real number and its
which is an AP, where A = log a, D = log r reciprocal is always greater than or equal to 2.
Now, tn = A + (n − 1) D = log a + (n − 1) log r = l  1  1 
∴ 5x + x  ≥ 2 and 52x + 2x  ≥ 2 for all x
n n  5   5 
∴ Sn = ( A + l ) = [log a + log a + (n − 1) log r ]
2 2  1  1 
⇒ a = 5 5x + x  + 52x + 2x  ≥ 5( 2) + 2 ⇒ a ≥ 12, ∀ x
n 2 n −1  5   5 
= log a r
2
9. (d)Qa 2 − a1 = a 3 − a 2 = K = common difference
4. (b) Hint mam = nan ⇒ m[a + (m − 1)d ] = n[a + (n − 1) d ]
=/ multiple of 3 = d [say]
5. (c) Let first term be a and common difference be d of an Then, consecutive terms are
AP.
1 1 m −n a1, a1 + d , a1 + 2d , a1 + 3d , …
Then, Tm − Tn = − ⇒ (m − n ) d =
n m mn ∴ d is not a multiple of 3.
⇒ d =
1
, so a =
1 Let d = 3λ + 1 or 3λ + 2
mn mn ∴ Consecutive terms
1 1 1 a1, a1 + 1 + 3λ , a1 + 2 + 6λ , a1 + 3 + 9 λ ,… are all prime.
∴ Tmn = a + (mn − 1)d = + mn ⋅ − =1
mn mn mn Hence, number of terms is infinite.
6. (b) We have, first term = a, second term = b,  7
difference (d ) = b − a and middle term = c. 10. (b)Q 2 log3( 2x − 5) = log3 2 + log3  2x − 
 2
This means that there are an odd number of terms in AP.  x 7
Let there be ( 2n + 1) terms in AP. ∴ ( 2 − 5) = 2  2 −
x 2

 2
Then, (n + 1)th term is the middle term. ⇒ t 2 + 25 − 10 t = 2t − 7 [put 2x = t ]
c −a
Middle term, c = a + nd ⇒ c = a + n(b − a ) ⇒ n = ⇒ t 2 − 12t + 32 = 0 ⇒ (t − 8)(t − 4) = 0
b −a
2n + 1 ⇒ 2x = 8 or 2x = 4
∴ Sum = [ 2a + ( 2n + 1 − 1)d ]
2 ∴ x = 3 or x = 2
n At x = 2,log3( 2x − 5) is not defined.
Q Sn = [ 2a + (n − 1)d ]
2
Hence, x = 3 is the only solution.
1  c − a  c − a 
= 2   + 1 2a + 2  (b − a ) n
2  b − a  b − a  11. (c) Sn = [ 2a′ + (n − 1) d ] = a + bn + cn 2
2
1  2(c − a )  2c (c − a ) n (n − 1)
= + 1 2c = +c ⇒ na′ + d = a + bn + cn 2
2  b − a  b −a 2

@iitjeehelps
788 SELF STUDY GUIDE BITSAT

 d n 2d ⇒ a + ar 2 − 2ar = − 14 + 22
⇒ a′ −  n + = a + bn + cn 2
 2 2 ⇒ a + ar 2 − 2ar = 8 …(ii)
d d
⇒ a = 0, b = a′ − , c = ⇒ d = 2c On dividing Eq. (i) by Eq. (ii), we get
2 2
a + ar + ar 2 56
12. (a) Tractor cost = ` 12000, Down payment = ` 6000 =
a + ar 2 − 2ar 8
Balance amount = ` 6000
6000 × 12 × 1  P × R ×T  1+ r + r 2 7
Interest on Ist installment = QI = ⇒ =
100  100  1 + r 2 − 2r 1
= ` 720 ⇒ 1 + r + r 2 = 7 + 7r 2 − 14r
Now, unpaid amount = 6000 − 500 = ` 5500 ⇒ 6r 2 − 15 r + 6 = 0
5500 × 12 × 1
Interest on IInd installment = = ` 660 ⇒ 2r 2 − 5 r + 2 = 0 [dividing by 3]
100
On factorising it by splitting the middle term, we get
Again, unpaid amount = 5500 − 500 = ` 5000
5000 × 12 × 1 2r 2 − ( 4 + 1) r + 2 = 0
Interest on IIIrd installment = = ` 600
100 ⇒ 2r 2 − 4r − r + 2 = 0
Total interest paid by him ⇒ 2r (r − 2) − (r − 2) = 0
= 720 + 660 + 600 + ... + 12 terms ⇒ (r − 2) ( 2r − 1) = 0
which is an AP with 1
⇒ r = 2,
a = 720, d = 660 − 720 = − 60 2
Therefore, total interest If r = 2, then from Eq. (i), we get
12 a + 2a + 4a = 56
= [ 2 × 720 + (12 − 1) ( − 60)]
2 ⇒ 7a = 56 ⇒ a = 8
= 6 [1440 − 11 × 60] = 6 [1440 − 660] Then, numbers are a = 8, ar = 8 × 2 = 16
= 6 × 780 = ` 4680 and ar 2 = 8 × 4 = 32
Hence, total amount or actual cost i.e. 8, 16, 32
= 12000 + 4680 = ` 16680 1
If r = , then from Eq. (i), we get
13. (d)Qtn = tn + 1 + tn + 2 2
a a a 4a + 2a + a
∴ ar n − 1 = ar n + ar n + 1 ⇒ 1 = r + r 2 + + = 56 ⇒ = 56
1 2 4 4
−1 ± 5 7a
⇒ r = [r > 0] ⇒ = 56 ⇒ a = 32
2 4
5 −1 Then, numbers are
Therefore, r = 2 = 2 sin 18°
4 1
a = 32, ar = 32 × = 16
a 2
14. (a) Let us take a GP with three terms , a, ar . Then,
r 1
and ar 2 = 32 × = 8
a a (r 2 + r + 1 ) 4
S = + a + ar =
r r i.e. 32, 16, 8
r 1 1 1  r 2 + r + 1 Hence, required numbers are 8, 16, 32 or 32, 16, 8.
and P = a 3, R = + + =  
a a ar a  r  16. (b) We have, ( 32) ( 32)1/ 6 ( 32)1/ 36K
1 1
3 1+ + +K
1  r 2 + r + 1 = ( 32) 6 36 = ( 32)t
a ⋅ 3
6

2
P R 3
a  r  1 1 1  a 
∴ = =1 t = 1+ + + ... =
S3  r 2 + r + 1
3 where,
1 Q S ∞ = 1 − r 
a3  
6 36 1−  
 r  6
6
Therefore, the ratio is 1: 1. =
5
15. (a) Let three numbers in GP be a, ar , ar 2.
∴ Product = ( 25 )6/ 5 = 26 = 64
Then, a + ar + ar 2 = 56 …(i) 1 1 1
Again, a − 1, ar − 7, ar 2 − 21 are in AP. 17. (d) S = 1 + + + + ...
2 4 8
⇒ 2 (ar − 7) = (a − 1) + (ar 2 − 21) 1  a 
S = =2 Q S ∞ = 1 − r 
[if a, b and c are in AP, then 2b = a + c ] 1−
1  
⇒ 2ar − 14 = a + ar 2 − 22 2

@iitjeehelps
SEQUENCES AND SERIES 789
  1 n  20. (b) Since, g = ab . Also, a , p , q and b are in AP.
1 1 −   
 2 n −1
 a (1 − r ) 
n b −a
Sn =   = 2 −  1 So, common difference d is .
1/ 2
 
 2 Q Sn = 1− r  3
 

Potrebbero piacerti anche